You are on page 1of 960

This page

intentionally left
blank

Copyright 2004, New Age International (P) Ltd., Publishers


Published by New Age International (P) Ltd., Publishers
All rights reserved.
No part of this ebook may be reproduced in any form, by photostat, microfilm,
xerography, or any other means, or incorporated into any information retrieval
system, electronic or mechanical, without the written permission of the publisher.
All inquiries should be emailed to rights@newagepublishers.com

ISBN (13) : 978-81-224-2979-4

PUBLISHING FOR ONE WORLD

NEW AGE INTERNATIONAL (P) LIMITED, PUBLISHERS


4835/24, Ansari Road, Daryaganj, New Delhi - 110002
Visit us at www.newagepublishers.com

Preface

Preface

As the title of the book How to learn calculus of one variable Suggests we have tried to present the entire
book in a manner that can help the students to learn the methods of calculus all by themselves we have felt that
there are books written on this subject which deal with the theoretical aspects quite exhaustively but do not
take up sufficient examples necessary for the proper understanding of the subject matter thoroughly. The
books in which sufficient examples are solved often lack in rigorous mathematical reasonings and skip accurate
arguments some times to make the presentation look apparently easier.
We have, therefore, felt the need for writing a book which is free from these deficiencies and can be used as
a supplement to any standard book such as Analytic geometry and calculus by G.B. Thomas and Finny which
quite thoroughly deals with the proofs of the results used by us.
A student will easily understand the underlying principles of calculus while going through the worked-out
examples which are fairly large in number and sufficiently rigorous in their treatment. We have not hesitated to
work-out a number of examples of the similar type though these may seem to be an unnecessary repetition. This
has been done simply to make the students, trying to learn the subject on their own, feel at home with the
concepts they encounter for the first time. We have, therefore, started with very simple examples and gradually
have taken up harder types. We have in no case deviated from the completeness of proper reasonings.
For the convenience of the beginners we have stressed upon working rules in order to make the learning all
the more interesting and easy. A student thus acquainted with the basics of the subject through a wide range
of solved examples can easily go for further studies in advanced calculus and real analysis.
We would like to advise the student not to make any compromise with the accurate reasonings. They should
try to solve most of examples on their own and take help of the solutions provided in the book only when it is
necessary.
This book mainly caters to the needs of the intermediate students whereas it can also used with advantages
by students who want to appear in various competitive examinations. It has been our endeavour to incorporate
all the finer points without which such students continually feel themselves on unsafe ground.
We thank all our colleagues and friends who have always inspired and encouraged us to write this book
everlastingly fruitful to the students. We are specially thankful to Dr Simran Singh, Head of the Department of
Lal Bahadur Shastri Memorial College, Karandih, Jamshedpur, Jharkhand, who has given valuable suggestions
while preparing the manuscript of this book.
Suggestions for improvement of this book will be gratefully accepted.
DR JOY DEV GHOSH
MD ANWARUL HAQUE

This page
intentionally left
blank

Contents

vii

Contents

Preface
1. Function

v
1

2. Limit and Limit Points

118

3. Continuity of a Function

151

4. Practical Methods of Finding the Limits

159

5. Practical Methods of Continuity Test

271

6. Derivative of a Function

305

7. Differentiability at a Point

321

8. Rules of Differentiation

354

9. Chain Rule for the Derivative

382

10. Differentiation of Inverse Trigonometric Functions

424

11. Differential Coefficient of Mod Functions

478

12. Implicit Differentiation

499

13. Logarithmic Differentiation

543

14. Successive Differentiation

567

15. LHospitals Rule

597

16. Evaluation of Derivatives for Particular Arguments

615

17. Derivative as Rate Measurer

636

18. Approximations

666

viii

Contents

19. Tangent and Normal to a Curve

692

20. Rolles Theorem and Lagranges Mean Value Theorem

781

21. Monotonocity of a Function

840

22. Maxima and Minima

870

Bibliography

949

Index

950

Function

1
Function

To define a function, some fundamental concepts are


required.
Fundamental Concepts
Question: What is a quantity?
Answer: In fact, anything which can be measured or
which can be divided into parts is called a quantity.
But in the language of mathematics, its definition is
put in the following manner.
Definition: Anything to which operations of
mathematics (mathematical process) such as addition,
subtraction, multiplication, division or measurement
etc. are applicable is called a quantity.
Numbers of arithmetic, algebraic or analytic
expressions, distance, area, volume, angle, time,
weight, space, velocity and force etc. are all examples
of quantities.
Any quantity may be either a variable or a constant.
Note: Mathematics deals with quantities which have
values expressed in numbers. Number may be real or
imaginary. But in real analysis, only real numbers as
values such as 1, 0, 15,

2 , p etc. are considered.

Question: What is a variable?


Answer:
Definitions 1: (General): If in a mathematical discussion, a quantity can assume more than one value,
then the quantity is called a variable quantity or simply a variable and is denoted by a symbol.
Example: 1. The weight of men are different for different individuals and therefore height is a variable.

2. The position of a point moving in a circle is a


variable.
Definition: 2. (Set theoritic): In the language of set
theory, a variable is symbol used to represent an
unspecified (not fixed, i.e. arbitrary) member (element
or point) of a set, i.e., by a variable, we mean an element
which can be any one element of a set or which can
be in turn different elements of a set or which can be
a particular unknown element of a set or successively
different unknown elements of a set. We may think of
a variable as being a place-holder or a blank for
the name of an element of a set.
Further, any element of the set is called a value of
the variable and the set itself is called variables
domain or range.
If x be a symbol representing an unspecified
element of a set D, then x is said to vary over the set
D (i.e., x can stand for any element of the set D, i.e., x
can take any value of the set D) and is called a variable
on (over) the set D whereas the set D over which the
variable x varies is called domain or range of x.
Example: Let D be the set of positive integers and x
D = {1, 2, 3, 4, }, then x may be 1, 2, 3, 4, etc.
Note: A variable may be either (1) an independent
variable (2) dependent variable. These two terms have
been explained while defining a function.
Question: What is a constant?
Answer:
Definition 1. (General): If in a mathematical
discussion, a quantity cannot assume more than one

How to Learn Calculus of One Variable

vale, then the quantity is called a constant or a


constant quantity and is denoted by a symbol.
Examples: 1. The weights of men are different for
different individuals and therefore weight is a variable.
But the numbers of hands is the same for men of
different weights and is therefore a constant.
2. The position of a point moving in a circle is a
variable but the distance of the point from the centre
of the circle is a constant.
3. The expression x + a denotes the sum of two
quantities. The first of which is variable while the
second is a constant because it has the same value
whatever values are given to the first one.
Definition: 2. (Set theoritic): In the language of set
theory, a constant is a symbol used to represent a
member of the set which consists of only one member,
i.e. if there is a variable c which varies over a set
consisting of only one element, then the variable c
is called a constant, i.e., if c is a symbol used to
represent precisely one element of a set namely D,
then c is called a constant.
Example: Let the set D has only the number 3; then
c = 3 is a constant.
Note: Also, by a constant, we mean a fixed element
of a set whose proper name is given. We often refer to
the proper name of an element in a set as a constant.
Moreover by a relative constant, we mean a fixed
element of a set whose proper name is not given. We
often refer to the alias of an element in a set as a
relative constant.
Remark: The reader is warned to be very careful
about the use of the terms namely variable and
constant. These two terms apply to symbols only not
to numbers or quantities in the set theory. Thus it is
meaningless to speak of a variable number (or a
variable quantity) in the language of set theory for
the simple reason that no number is known to human
beings which is a variable in any sense of the term.
Hence the usual text book definition of a variable as
a quantity which varies or changes is completely
misleading in set theory.
Kinds of Constants
There are mainly two kinds of constants namely:

1. Absolute constants (or, numerical constants).


2. Arbitrary constants (or, symbolic constants).
Each one is defined in the following way:
1. Absolute constants: Absolute constants have the
same value forever, e.g.:
(i) All arithmetical numbers are absolute constants.
Since 1 = 1 always but 1 2 which means that the
value of 1 is fixed. Similarly 1 = 1 but 1 1 (Any
quantity is equal to itself. this is the basic axiom of
mathematics upon which foundation of equations
takes rest. This is why 1 = 1, 2 = 2, 3 = 3, x = x and
a = a and so on).
(ii) p and logarithm of positive numbers (as log2, log
3, log 4, etc) are also included in absolute constants.
2. Arbitrary constants: Any arbitrary constant is
one which may be given any fixed value in a problem
and retains that assigned value (fixed value)
throughout the discussion of the same problem but
may differ in different problems.
An arbitrary constant is also termed as a parameter.
Note: Also, the term parameter is used in speaking
of any letter, variable or constant, other than the
coordinate variables in an equation of a curve defined
by y = f (x) in its domain.
Examples: (i) In the equation of the circle x2 + y2 =
a2, x and y, the coordinates of a point moving along a
circle, are variables while a the radius of a circle may
have any constant value and is therefore an arbitrary
constant or parameter.
(ii) The general form of the equation of a straight line
put in the form y = mx + c contains two parameters
namely m and c representing the gradient and y-intercept of any specific line.
Symbolic Representation of Quantities,
Variables and Constants
In general, the quantities are denoted by the letters a,
b, c, x, y, z, of the English alphabet. The letters from
a to s of the English alphabet are taken to represent
constants while the letters from t to z of the English
alphabet are taken to represent variables.
Question: What is increment?
Answer: An increment is any change (increase or
growth) in (or, of) a variable (dependent or

Function

independent). It is the difference which is found by


subtracting the first value (or, critical value) of the
variable from the second value (changed value,
increased value or final value) of the variable.
That is, increment
= final value initial value = F.V I. V.
Notes: (i) Increased value/changed value/final value/
second value means a value obtained by making
addition, positive or negative, to a given value (initial
value) of a variable.
(ii) The increments may be positive or negative, in
both cases, the word increment is used so that a
negative increment is an algebraic decrease.
Examples on Increment in a Variable
1. Let x1 increase to x2 by the amount x. Then we
can set out the algebraic equation x1 + x = x2 which
x = x2 x1.
2. Let y1 decrease to y2 by the amount y. Then we
can set out the algebraic equation y1 + y = y2 which
y = y2 y1.
Examples on Increment in a Function
1. Let y = f (x) = 5x + 3 = given value
(i)
Now, if we give an increment x to x, then we also
require to give an increment y to y simultaneously.
Hence, y + y = f (x + x) = 5 (x + x) + 3 = 5x +
5 x + 3
(ii)
(ii) (i) y + y y = (5x + 5 x + 3) (5x + 3)
= 5x + 5 x + 3 5x 3 = 5 x
i.e., y = 5 x
2. Let y = f (x) = x2 + 2 = given value,
then y + y = f (x + x) = (x + x)2 + 2 = x2 +
x2 + 2x x + 2
y = x2 + x2 + 2x x + 2 x2 2 = 2x x +
x2
Hence, increment in y = f (x + x) f (x) where
f (x) = (x2 + 2) is y = x2 + x2 + 2x x + 2 x2 2
= 2x x + x2
3. Let y =

1
= given value.
x

1
Then, y + y = x + x

Hence, increment in y = f (x + x) f (x) where


f (x) =

1
x

1
x x + x
1
y = x + x
=
x x + x
x

x
x x + x

4. Let y = log x = given value.


Then, y + y = log (x + x)
and y = log (x + x) log x = log

FG
H

FG x + x IJ =
H x K

IJ
K

x
x
5. Let y = sin , given value
Then, y + y = sin ( + )

log 1 +

and y = sin ( + ) sin = 2cos


sin

FG 2 + IJ
H 2 K

FG IJ .
H2K

Question: What is the symbol used to represent (or,


denote) an increment?
Answer: The symbols we use to represent small
increment or, simply increment are Greak Letters
and (both read as delta) which signify an increment/
change/growth in the quantity written just after it as
it increases or, decreases from the initial value to
another value, i.e., the notation x is used to denote
a fixed non zero, number that is added to a given
number x0 to produce another number x = x0 + x. if
y = f (x) then y = f (x0 + x) f (x0).
Notes: If x, y, u. v are variables, then increments in
them are denoted by x, y, u, v respectively
signifying how much x, y, u, v increase or decrease,
i.e., an increment in a variable (dependent or
independent) tells how much that variable increases
or decreases.
Let us consider y = x2
When x = 2, y = 4
x = 3, y = 9

How to Learn Calculus of One Variable

x = 3 2 = 1 and y = 9 4 = 5
as x increases from 2 to 3, y increases from 4
to 9.
as x increases by 1, y increases by 5.
Question: What do you mean by the term function?
Answer: In the language of set theory, a function is
defined in the following style.
A function from a set D to a set R is a rule or, law
(or, rules, or, laws) according to which each element
of D is associated (or, related, or, paired) with a unique
(i.e., a single, or, one and only one, or, not more than
one) element of R. The set D is called the domain of
the function while the set R is called the range of the
function. Moreover, elements of the domain (or, the
set D) are called the independent variables and the
elements of the range (or, range set or, simply the set
R) are called the dependent variables. If x is the
element of D, then a unique element in R which the
rule (or, rules) symbolised as f assigns to x is termed
the value of f at x or the image of x under the rule
f which is generally read as the f-function of x or, f
of x. Further one should note that the range R is the
set of all values of the function f whereas the domain
D is the set of all elements (or, points) whose each
element is associated with a unique elements of the
range set R.
Functions are represented pictorially as in the
accompanying diagram.
D

y = f (x )

Highlight on the Term The Rule or the


Law.
1. The term rule means the procedure (or
procedures) or, method (or, methods) or, operation
(or, operations) that should be performed over the
independent variable (denoted by x) to obtain the
value the dependent variable (denoted by y).
Examples:
1. Let us consider quantities like
(i) y = log x
(iv) y = sin x
(v) y = sin1x
(ii) y x3
(iii) y = x
(vi) y = ex, etc.
In these log, cube, square root, sin, sin1, e, etc
are functions since the rule or, the law, or, the function
f = log, ( )3,

, sin, sin1 or, e, etc has been

performed separately over (or, on) the independent


variable x which produces the value for the dependent
variable represented by y with the assistance of the
rule or the functions log, ( )3,

, sin, sin1 or, e,

etc. (Note: An arbitrary element (or point) x in a set


signifies any specified member (or, element or point)
of that set).
2. The precise relationship between two sets of
corresponding values of dependent and independent
variables is usually called a law or rule. Often the rule
is a formula or an equation involving the variables
but it can be other things such as a table, a list of
ordered pairs or a set of instructions in the form of a
statement in words. The rule of a function gives the
value of the function at each point (or, element) of the
domain.
Examples:

One must think of x as an arbitrary element of the


domain D or, an independent variable because a value
f of x can be selected arbitrarily from the domain D as
well as y as the corresponding value of f at x, a
dependent variable because the value of y depends
upon the value of x selected. It is customary to write
y = f (x) which is read as y is a function of x or, y is
f of x although to be very correct one should say
that y is the value assigned by the function f
corresponding to the value of x.

af

(i) The formula f x =

1
1+ x

tells that one should

square the independent variable x, add unity and then


divide unity by the obtained result to get the value of
the function f at the point x, i.e., to square the
independent variable x, to add unity and lastly to
divide unity by the whole obtained result (i.e., square
of the independent variable x plus unity).

Function

(ii) f (x) = x2 + 2, where the rule f signifies to square


the number x and to add 2 to it.
(iii) f (x) = 3x 2, where the rule f signifies to multiply
x by 3 and to subtract 2 from 3x.
(iv) C = 2 r an equation involving the variables C
(the circumference of the circle) and r (the radius of
the circle) which means that C = 2 r = a function
of r.
(v) y =

64 s an equation involving y and s which

means that y =

64 s a functions of s.

3. A function or a rule may be regarded as a kind of


machine (or, a mathematical symbol like
sin1,

cos1,

, log, sin,
tan1,

cot1,
cos, tan, cot, sec, cosec,
1
1
sec , cosec , etc indicating what mathematical
operation is to be performed over (or, on) the elements
of the domain) which takes the elements of the domain
D, processes them and produces the elements of the
range R.
Example of a function of functions:
Integration of a continuous function defined on some
closed interval [a. b] is an example of a function of
functions, namely the rule (or, the correspondence)
that associates with each object f (x) in the given set

z af
b

of objects, the real number

f x dx .

Notes: (i) We shall study functions which are given


by simple formulas. One should think of a formula as
a rule for calculating f (x) when x is known (or, given),
i.e., of the rule of a function f is a formula giving y in
terms of x say y = f (x), to find the value of f at a
number a, we substitute that number a for x wherever
x occurs in the given formula and then simplify it.

af

(ii) For x D , f x R should be unique means


that f can not have two or more values at a given
point (or, number) x.
(iii) f (x) always signifies the effect or the result of
applying the rule f to x.

(iv) Image, functional value and value of the function


are synonymes.
Notations:
f

We write 1. " f : D R" or " D R" for f is a


function with domain D and range R or equivalently,
f is a function from D to R.

af

2. f : x y or, x y or, x f x

for a

function f from x to y or f maps (or, transforms) x


into y or f (x).
3. f : D R defined by y = f (x) or, f : D R by
y = f (x) for (a) the domain = D, (b) the range = R, (c)
the rule : y = f (x).
4. D (f) = The domain of the function f where D
signifies domain of.
5. R (f) = The range of the function f where R signifies
range of.
Remarks:
(i) When we do not specify the image of elements of
the domain, we use the notation (1).
(ii) When we want to indicate only the images of
elements of the domain, we use the notation (2).
(iii) When we want to indicate the range and the rule
of a function together with a functional value f (x), we
use the notation (3).
(iv) In the language of set theory, the domain of a
function is defined in the following style:

D (f): x: x D1 where, D 1 = the set of


independent variables (or, arguments) = the set of all
those members upon which the rule f is performed
to find the images (or, values or, functional values).
(v) In the language of set theory, the range of a
function is defined in the following way:

a f laf

af q

R f = f x : x D , f x R = the set of all


images.
(vi) The function f n is defined by f n (x) = f (x) f (x)
n. times
= [f (x)]n, where n being a positive integer.
(vii) For a real valued function of a real variable both
x and y are real numbers consisting of.
(a) Zero

How to Learn Calculus of One Variable

(b) Positive or negative integers, e.g.: 4, 11, 9, 17,


3, 17, etc.
(c) Rational numbers, e.g.:

9 17
,
, etc.
5 2

(d) Irrational numbers e.g.: 7 , 14 , etc.


(viii) Generally the rule/process/method/law is not
given in the form of verbal statements (like, find the
square root, find the log, exponential, etc.) but in
the form of a mathematical statement put in the form
of expression containing x (i.e. in the form of a formula)
which may be translated into words (or, verbal
statements).
(ix) If it is known that the range R is a subset of some
set C, then the following notation is used:
f : D C signifying that
(a) f is a function
(b) The domain of f is D
(c) The range of f is contained in C.
Nomenclature: The notation " f : D C" is read f
is a function on the set D into the set C.
C
D
x5
x6

f
x1
x2
x3
x4

y1
y2
y3
y4

y5
y6

N.B: To define some types of functions like into


function and on to function, it is a must to define a
function " f : D C" where C = codomain and
hence we are required to grasp the notion of codomain. Therefore, we can define a co-domain of a
function in the following way:
Definition of co-domain: A co-domain of a function
is a set which contains the range or range set (i.e., set
of all values of f) which means R C , where R = the
set of all images of f and C = a set containing images
of f.
Remember:
1. If R C (where R = the range set, C = co-domain)
i.e., if the range set is a proper subset of the co-domain,
then the function is said to be an into function.

2. If R = C, i.e., if the range set equals the co-domain,


then the function is said to be an onto function.
3. If one is given the domain D and the rule (or
formula,) then it is possible (theoretically at least) to
state explicitly a function as any ordered pair and one
should note that under such conditions, the range
need not be given. Further, it is notable that for each
specified element ' a' D , the functional value f (a)
is obtained under the function f.
4. If a D , then the image in C is represented by f
(a) which is called the functional value (corresponding
to a)and it is included in the range set R.
Question: Distinguish between the terms a function
and a function of x.
Answer: A function of x is a term used for an image
of x under the rule f or the value of the function f at
(or, for) x or the functional value of x symbolised
as y = f (x) which signifies that an operation (or,
operations) denoted by f has (or, have) been performed
on x to produce an other element f (x) whereas the
term function is used for the rule (or, rules) or
operation (or, operations) or law (or, laws) to be
performed upon x, x being an arbitrary element of a
set known as the domain of the function.
Remarks: 1. By an abuse of language, it has been
customary to call f (x) as function instead of f when a
particular (or, specifies) value of x is not given only
for convenience. Hence, wherever we say a function
f (x) what we actually mean to say is the function f
whose value at x is f (x). thus we say, functions x4, 3x2
+ 1, etc.
2. The function f also represents operator like

,
( )n, | |, log, e, sin, cos, tan, cot, sec, cosec, sin1, cos
1, tan1, cot1, sec1 or cosec1 etc.
3. Function, operator, mapping and transformation
are synonymes.
4. If domain and range of a function are not known, it
is customary to denote the function f by writing y = f
(x) which is read as y is a function of x.
Question: Explain the terms dependent and
independent variables.
Answer:
1. Independent variable: In general, an independent
variable is that variable whose value does not depend

Function

upon any other variable or variables, i.e., a variable in


a mathematical expression whose value determines
the value of the whole given expression is called an
independent variable: in y = f (x), x is the independent
variable.
In set theoretic language, an independent variable
is the symbol which is used to denote an unspecified
member of the domain of a function.
2. Dependent variable: In general a dependent
variable is that variable whose value depends upon
any other variable or variables, i.e., a variable (or, a
mathematical equation or statement) whose value is
determined by the value taken by the independent
variable is called a dependent variable: in y = f (x), y is
the dependent variable.
In set theoretic language, a dependent variable is
the symbol which is used to denote an unspecified
member of the range of a function.
e.g.: In A = f r = r 2, r is an independent variable
and A is a dependent variable.

bg

Question: Explain the term function or function of


x in terms of dependency and independency.
Answer: When the values of a variable y are
determined by the values given to another variable x,
y is called a function of (depending on) x or we say
that y depends on (or, upon) x. Thus, any expression
in x depends for its value on the value of x. This is
why an expression in x is called a function of x put in
the form: y = f (x).
Question: What are the symbols for representing the
terms a function and a function of a variable?
Answer: Symbols such as f, F, etc are used to
denote a function whereas a function of a variable is
denoted by the symbols f (x), x , f t , F t ,
t and can be put in the forms: y = f (x); y = x ;
y = f (t); y = F (t); y = t , that y is a function of
(depending on) the variable within the circular bracket
( ), i.e., y depends upon the variable within circular
bracket.
i.e., y = f (x) signifies that y depends upon x, i.e., y
is a function of x.
S = f (t) signifies that s depends upon t, i.e., s is a
function of t.

af

af

af

a f af af
af

C = r signifies that c depends upon r, i.e., c is


a function of r.

Notes:
1. Any other letter besides f , , F etc may be used
just for indicating the dependence of one physical
quantity on an other quantity.
2. The value of f /functional value of f corresponding
to x = a / the value of the dependent variable y for a
particular value of the independent variable is
symbolised as (f (x))x = a = f (a) or [f (x)]x = a = f (a) while
evaluating the value of the function f (x) at the point
x = a.
3. One should always note the difference between
a function and a function of.
4. Classification of values of a function at a point x
= a.
There are two kinds of the value of a function at a
point x = a namely
(i) The actual value of a function y = f (x) at x = a.
(ii) The approaching or limiting value of a function y
= f (x) at x = a, which are defined as:
(i) The actual value of a function y = f (x) at x = a:
when the value of a function y = f (x) at x = a is
obtained directly by putting in the given value of the
independent variable x = a wherever x occurs in a
given mathematical equation representing a function,
we say that the function f or f (x) has the actual value
f (a) at x = a.
(ii) The approaching value of a function y = f (x) at x
= a: The limit of a function f (x) as x approaches some
definite quantity is termed as the approaching (or,
limiting) value of the function y = f (x) at x = a. This
value may be calculated when the actual value of the
function f (x) becomes indeterminate at a particular
value a of x.
5. When the actual value of a function y = f (x) is
anyone of the following forms:

0
, 00 , 0 ,
0

any real number


, , 0 , 1 , imaginary,

0
for a particular value a of x, it is said that the function
f (x) is not defined or is indeterminate or is meaningless
at x = a.
6. To find the value of a function y = f (x) at x = a
means to find the actual value of the function y = f (x)
at x = a.

How to Learn Calculus of One Variable

Pictorial Representation of a Function, its


Domain and Range.
1. Domain: A domain is generally represented by
any closed curve regular (i.e., circle, ellipse, rectangle,
square etc) or irregular (i.e. not regular) whose
members are represented by numbers or alphabets or
dots.
2. Range: A range is generally represented by
another closed curve regular or irregular or the some
closed curve regular or irregular as the domain.
3. Rule: A rule is generally represented by an arrow
or arc (i.e., arc of the circle) drawn from each member
of the domain such that it reaches a single member or
more than one member of the codomain, the codomain
being a superset of the range (or, range set).
Remarks:
1. We should never draw two or more than two arrows
from a single member of the domain such that it reaches
more than one member of the codomain to show that
the venn-diagram represents a function. Logic behind
it is given as follows.
If the domain are chairs, then one student can not
sit on more than one chair at the same time (i.e., one
student can not sit on two or more than two chairs at
the same time)
R = range

D = domain
f = rule =
a function
x

Fig. 1.1 Represents a function


f = rule = a function

C = codomain

D = domain
x1
x2
x3

y1
y2
y3
y4
y5

Fig. 1.2 Represents a function

R = range

C = codomain

D = domain
x1
x2
x3
x4

y1
y2

y3
y4
y5

y6

Fig. 1.3 Does not represent a function


C = codomain

D = domain
x1
x2
x3
x4

y1
y2

R = range
y4
y5

y3

y6

Fig. 1.4 Represents a function


2. In the pictorial representation of a function the
word rule means.
(i) Every point/member/element in the domain D is

a f

af

joined by an arrow or arc to some point in


range R which means each element x D
corresponds to some element y R C .
(ii) Two or more points in the domain D may be joined
to the same point in R C (See Fig. 1.4 where the
points x2 and x3 in D are joined to the same point y2 in
R C.
(iii) A point in the domain D can not be joined to two
or more than two points in C, C being a co-domain.
(See Fig. 1.3)
(iv) There may be some points in C which are not
joined to any element in D (See Fig. 1.4 where the
points y4, y5 and y6 in C are not joined to any point in
D.
Precaution: It is not possible to represent any
function as an equation involving variables always.
At such circumstances, we define a function as a set
of ordered pairs with no two first elements alike e.g., f
= {(1, 2), (2, 4), (3, 6), (4, 8), (5, 10), (6, 12), (7, 14)}
whose D = domain = {1, 2, 3, 4, 5, 6, 7}, R = range = {2,
4, 6, 8, 10, 12, 14} and the rule is: each second element
is twice its corresponding first element.
But f = {(0, 1), (0, 2), (0, 3), (0, 4)} does not define a
function since its first element is repeated.

Function

Note: When the elements of the domain and the range


are represented by points or English alphabet with
subscripts as x 1 , x 2 , etc and y 1 , y 2 , etc
respectively, we generally represent a function as a
set of ordered pairs with no two first elements alike,
i.e., f: {x, f (x): no two first elements are same} or, {x, f
(x): no two first elements are same} or, {(x, y): x D

af

and y = f x R } provided it is not possible to


represent the function as an equation y = f (x).
Question: What is meant whenever one says a
function y = f (x) exist at x = a or y = f (x) is defined at
(or, f or) x = a?
Answer: A function y = f (x) is said to exist at x = a or,
y = f (x) is said to be defined at (or, f or) x = a provided
the value of the function f (x) at x = a (i.e. f (a)) is finite
which means that the value of the function f (x) at x =
a should not be anyone of the following forms

0 0
, 0 , 0 ,
, , 0 , 1 , imaginary

af

(ii) lim f x exists means lim f x has a finite


xa

xa

value.
(iii) f ' (a) exists means f ' (a) has a finite value.

z af
b

(iv)

f x dx exists means that

finite value.

Notes:
1. A function f: R R defined by f (x) = a0 xn + a1 xn
1+ + a
m 1 x + am where a0, a1, a2, am are
constants and n is a positive integer, is called a
polynomial in x or a polynomial function or simply a
polynomial. One should note that a polynomial is a
particular case of algebraic function as we see on
taking m = 1 and A0 = a constant in algebraic function.
2. The quotient of two polynomials termed as a
rational function of x put in the form:
n 1

+ ... + am 1 x + am

b0 + b1 x + ... + bm x

5 , 5 0 , log (3), 5 2 are


undefined or they are said not to exist.
(iii) Whenever we say that something exists, we mean
that it has a definite finite value.
e.g.:
(i) f (a) exists means f (a) has a finite value.

af

(i) Algebraic function: A function which satisfies


the equation put in the form:
Ao [f (x)] m + A1 [f (x)] m 1 + A2 [f (x)] m 2 + + Am
= 0, where A0, A1, Am are polynomials is called an
algebraic function.

Remarks:
(i) A symbol in mathematics is said to have been
defined when a meaning has been given to it.
(ii) A symbol in mathematics is said to be undefined
or non-existance when no meaning is attributed to
the symbol.
e.g.: The symbols 3/2, 8/15, sin1(1/2), log (1/2)
are defined or they are said to exist whereas the
9 , cos

We divide the function into two classes namely:


(i) Algebraic
(ii) Transcendental which are defined as:

a0 x + a1 x

a real number
value,
.
0

symbols

Classification of Functions

z af
b

f x dx has a

is also an algebraic function. It is defined in every


interval only in which denominator does not vanish.
If f1 (x) and f2 (x) are two polynomials, then general

af

rational functions may be denoted by R x =

af
af

f1 x
f2 x

where R signifies a rational function of. In case f2


(x) reduces itself to unity or any other constant (i.e., a
term not containing x or its power), R (x) reduces
itself to a polynomial.
3. Generally, there will be a certain number of values
of x for which the rational function is not defined and
these are values of x for which the polynomial in
denominator vanishes.

af

e.g.: R x =

2 x 5x + 1
2

x 5x + 6

is not defined when x

= 2 or x = 3.
4. Rational integral functions: If a polynomial in x is
in a rational form only and the indices of the powers
of x are positive integers, then it is termed as a rational
integral function.

10

How to Learn Calculus of One Variable

5. A combination of polynomials under one or more


radicals termed as an irrational functions is also an

af

x = f x ; y=

algebraic function. Hence, y =


x

53

af

= f x ; y=

serve as examples for

x +4

(iv) e

irrational algebraic functions.


6. A polynomial or any algebraic function raised to
any power termed as a power function is also an

f af

algebraic function. Hence, y = x , n R = f x ;

y= x +1

af

= f x serve as examples for power

functions which are algebraic.


Remarks:
1. All algebraic, transcendental, explicit or implicit
function or their combination raised to a fractional
power reduces to an irrational function. Hence,
y= x

53

af

= f x ; y = sin x + x

1
2

af

= f x

serve

as examples for irrational functions.


2. All algebraic, transcendental, explicit or implicit
function or their combination raised to any power is
always regarded as a power function. Hence, y = sin2
x = f (x); y = log2 | x | = f (x) serve as examples for power
functions.
Transcendental function: A function which is not
algebraic is called a transcendental function. Hence,
all trigonometric, inverse trigonometric, exponential
and logarithmic (symoblised as TILE) functions are
transcendental functions. hence, sin x, cos x, tan x,
cot x, sec x, cosec x, sin1 x, cos1 x, tan1 x, cot1 x,
sec1 x, cosec1 x, log |f (x) |, log | x |, log x2, log (a + x2),
ax (for any a > 0), ex, [f (x)]g (x) etc serve as examples
for transcendental functions.
Notes: (In the extended real number system)
(A)
(i) e = when x =
(ii) ex = 1 when x = 0
(iii) ex = 0 when x = .
x

(B) One should remember that exponential functions


obeys the laws of indices, i.e.,
(i) xe ey = ex + y
(ii) xe / ey = ex y
(iii) (ex)m = emx
x

1
e

(C)
(i) log 0 =
(ii) log 1 = 0
(iii) log =
Further Classification of Functions
The algebraic and the transcendental function are
further divided into two types namely (i) explicit
function (ii) implicit function, which are defined as:
(i) Explicit function: An explicit function is a
function put in the form y = f (x) which signifies that a
relation between the dependent variable y and the
independent variable x put in the form of an equation
can be solved for y and we say that y is an explicit
function of x or simply we say that y is a function of x.
hence, y = sin x + x = f (x); y = x2 7x + 12 = f (x) serve
as examples for explicit function of xs.
Remark: If in y = f (x), f signifies the operators (i.e.,
functions) sin, cos, tan, cot, sec, cosec, sin1, cos1,
tan1, cot1, sec1, cosec1, log or e, then y = f (x) is
called an explicit transcendental function otherwise it
is called an explicit algebraic function.
(ii) Implicit function: An implicit function is a
function put in the form: f (x, y) = c, c being a constant,
which signifies that a relation between the variables y
and x exists such that y and x are in seperable in an
equation and we say that y is an implicit function of x.
Hence, x3 + y2 = 4xy serves as an example for the
implicit function of x.
Remark: If in f (x, y) = c, f signifies the operators (i.e.,
functions) sin, cos, tan, cot, sec, cosec, sin1, cos1,
tan1, cot1, sec1, cosec1, log, e and the ordered
pain (x, y) signifies the combination of the variables x
and y, then f (x, y) = c is called an implicit algebraic
function of x, i.e., y is said to be an implicit algebraic
function of x, if a relation of the form:

Function

ym + R1 ym 1 + + Rm = 0 exists, where R1, R2,


Rm are rational function of x and m is a positive integer.
Note: Discussion on the explicit and the implicit
functions has been given in detail in the chapter
differentiation of implicit function.
On Some Important Functions
Some types of functions have been discussed in
previous sections such as algebraic, transcendental,
explicit and implicit functions. In this section definition
of some function used most frequently are given.
1. The constant function: A function f: R R
defined by f (x) = c is called the constant function.
Let y = f (x) = c
y = c which is the equation of a straight line
parallel to the x-axis, i.e., a constant function
represents straight lines parallel to the x-axis.
Also, domain of the constant function = D (f) =
{real numbers} = R and range of the constant function
= R (f) = {c} = a singleton set for examples, y = 2; y = 3
are constant functions.
Remarks:
(i) A polynomial a0 xn + a1 xn 1 + a m 1 x + am
(whose domain and range are sets of real numbers)
reduces to a constant function when degree of
polynomial is zero.
(ii) In particular, if c = 0, then f (x) is called the zero
function and its graph is the x-axis itself.
2. The identity function: A function f: R R
defined by f (x) = x is called the identity function
whose domain and range coincide with each other,
i.e., D (f) = R (f) in case of identity function.
Let y = f (x) = x
y = x which is the equation of a straight line
passing through the origin and making an angle of
45 with the x-axis, i.e., an identity function represents
straight lines passing through origin and making an
angle of 45 with the x-axis.
3. The reciprocal of identity function: A function

af

1
is called the
x
reciprocal function of the identity function f (x) = x or
simply reciprocal function.
f: R {0} R defined by f x =

11

af

1
x
xy = 1 which is the equation of a rectangular
hyperbola, i.e., the reciprocal of an identity function
represents a rectangular hyperbola.
Also, D (f) = {real number except zero} = R {0}
and R (f) = {real numbers}
Let y = f x =

4. The linear function: A function put in the form: f


(x) = mx + c is called a linear function due to the fact
that its graph is a straight line.
Also, D (f) = {real numbers except m = 0} and R (f)
= {real number except m = 0}
Question: What do you mean by the absolute value
function?
Answer: A function f: R R defined by f (x) = | x |

RS x , x 0 is called absolute value (or, modulus or,


x , x < 0
T
norm) function.
=

Notes: (A) A function put in the form | f (x) | is called


the modulus of a function or simply modulus of a
function which signifies that:

af

(i) | f (x) | = f (x), provided f x 0 , i.e., if f (x) is


positive or zero, then | f (x) | = f (x).
(ii) | f (x) | = f (x), provided f (x) < 0, i.e., if f (x) is
negative, then | f (x) | = f (x) which means that if f (x)
is negative, f (x) should be multiplied by 1 to make f
(x) positive.
(B) | f (x) | = sgn f (x) f (x) where sgn

af

f x =

a f , f axf 0
f a xf
f x

= 0, f (x) = 0
i.e., sgn f (x) = 1 when f (x) > 0
= 1 when f (x) < 0
= 0 when f (x) = 0
where sgn signifies sign of written briefly for the
word signum from the Latin. Also, domain of absolute value function = D (f) = {real numbers} and range
of absolute value function = R (f) = {non negative real
numbers} = R+ {0}.

(C) 1. (i) | x a | = (x a) when x a 0

| x a | = (x a) when x a < 0

12

How to Learn Calculus of One Variable

(ii) | 3| = 3 since 3 is positive.


| 3 | = (3) since 3 is negative. For this reason,
we have to multiply 3 by 1.
2. If the sign of a function f (x) is unknown (i.e., we
do not know whether f (x) is positive or negative),
then we generally use the following definition of the
absolute value of a function.

af

f x

af

f x

a xf

3. Absolute means to have a magnitude but no sign.


4. Absolute value, norm and modulus of a function
are synonymes.
5. Notation: The absolute value of a function is
denoted by writing two vertical bars (i.e. straight lines)
within which the function is placed. Thus the notation
to signify the absolute value of is | |.
6. | f 2 (x) | = f 2 (x) = | f (x) |2 = (f (x))2
7. In a compact form, the absolute value of a function

af

may be defined as f x

af

= f (x), when f x 0
= f (x), when f (x) < 0

f1 x

af

a xf

9.

af

f x

af

af
f a xf k

af

and f x k ,

k > 0.

a f k f axf k or f a xf k which
signifies the union of f a x f k and f a x f k ,
10.

17.
18.

a f = f a xf , f a xf 0
a f f a xf
f a x f + f a xf f a xf + f a x f
f a x f f a xf f a xf f a xf
f1 x
f2 x

19. | 0 | = 0, i.e. absolute value of zero is zero.


20. Modulus of modulus of a function (i.e. mod of | f
(x) | ) = | f (x) |

a f

x < 0 x = x,

x , 0 .
(b) | x | = | x | = x, for all real values of x
2

(c)

x =

(d)

x a a x a and x a x a

and x a .
Geometric Interpretation of Absolute Value
of a Real Number x, Denoted by | x |

k k f x k which signifies

the intersection of

16.

and | x | = x, when

= f 2 x f1 x = f 2 x

1
.
2

af

(a) | x | = x, when x 0 x = x , x 0 ,

x 2 = x 3 x + x = 2 3 2x = 1
x=

af

14. | f x | f x
15. | f 1 (x) f 2 (x) |
= | f 1 (x) | | f 2 (x) |

Remarks: When

af
e.g.: x 2 = x + 3 a x 2 f = a x + 3f
which is solved as under this line. a x 2 f =
a x + 3f 2 = 3 which is false which means this
equation has no solution and a x 2 f = a x + 3f
8.

af

af

12. | f x | 0 always means that the absolute value


of a functions is always non-negative (i.e., zero or
positive real numbers)
13. | f (x) = | f (x) |

f x

k > 0.
11. | f (x) |n = (f (x)n, where n is a real number.

The absolute value of a real number x, denoted by | x


| is undirected distance between the origin O and the
point corresponding to a (i.e. x = a) i.e, | x | signifies
the distance between the origin and the given point x
= a on the real line.
Explanation: Let OP = x
If x > o, P lies on the right side of origin O, then
the distance OP = | OP | = | x | = x
a

P (x)

a
0

P (x)

13

Function

If x = O, P coincides with origin, the distance OP =


|x|=|o|=o
If x > O, P lies on the left side of origin o, then the
distance OP = | OP | = | OP | = | x | = x
Hence, | x | =
x, provided x > o means that the absolute value
of a positive number is the positive number
itself.
o, provided x = o means the absolute value of
zero is taken to be equal to zero.
x, provided x < o means that the absolute value
of a negative number is the positive value of
that number.
Notes:
1. x is negative in | x | = x signifies x is positive in |
x | = x e.g.: | 7| = (7) = 7.
2. The graphs of two numbers namely a and a on
the number line are equidistant from the origin. We
call the distance of either from zero, the absolute value
of a and denote it by | a |.
3.

x = a x = a
2

4. x = a

x =

a x =

5.

x =

e.g.: 1 = 1

4 = 2
16 = 4
Remember:
1. In problems involving square root, the positive
square root is the one used generally, unless there is

100 = 10 ;

a remark to the contrary. Hence,


2
x = x .

169 = 13 ;
2

2. x + y = 1 x = 1 y
2

x =

1 y

x = 1 y x = 1 y

e.g.: cos = 1 sin cos =


a

x = a x=

x =

signifies that if x is any given number,

then the symbol

3. To indicate both positive square root and negative


square root of a quantity under the radical sign, we
write the symbol (read as plus or minus) before
the radical sign.

x represents the positive square


2
root of x and be denoted by | x | whose graph is
symmetrical about the y-axis having the shape of
English alphabet 'V '. which opens (i) upwards if y =
| x | (ii) downwards if y = | x | (iii) on the right side if
x = | y | (iv) on the left side if x = | y |.
An Important Remark

1 sin cos = 1 sin

one should note that the sign of cos is


determined by the value of the angle ' ' and the
value of the angle ' ' is determined by the quadrant
in which it lies. Similarly for other trigonometrical
functions of , such as, tan2 = sec2 1 tan
2

sec 1 tan =
2

cot = cosec 1 cot =

cosec 1 cot =
2

1. The radical sign " n " indicates the positive root


of the quantity (a number or a function) written under

25 = + 5 .
2. If we wish to indicate the negative square root of a
quantity under the radical sign, we write the negative
it (radical sign) e.g.:

sign () before the radical sign. e.g.: 4 = 2 .

sec 1

cosec 1

sec = 1 + tan sec =


2

1 + tan sec = 1 + tan , w h e r e

the sign of angle ' ' is determined by the quadrant in


which it lies.
3. The word modulus is also written as mod and
modulus function is written as mod function in
brief.

14

How to Learn Calculus of One Variable

On Greatest Integer Function


Firstly, we recall the definition of greatest integer
function.
Definition: A greatest integer function is the function
defined on the domain of all real numbers such that
with any x in the domain, the function associates
algebraically the greatest (largest or highest) integer
which is less than or equal to x (i.e., not greater than
x) designated by writing square brackets around x as
[x].
The greatest integer function has the property of
being less than or equal to x, while the next integer is
greater than x which means x x < x + 1 .
Examples:
(i) x =

LM OP
NQ

3
3
= 1 is the greatest integer in
x =
2
2

3
.
2
(ii) x = 5 [x] = [5] = 5 is the greatest integer in 5.
(iii) x = 50 x =

50 = 7 is the greatest

integer in 50 .
(iv) x = 2.5 [x] = [2.5] = 3 is the greatest integer
in 2.5.
(v) x = 4.7 [x] = [4.7] = 5 is the greatest integer
in 4.7.
(vi) x = 3 [x] = [3] = 3 is the greatest integer
in 3.
To Remember:
1. The greatest integer function is also termed as
the bracket, integral part or integer floor function.
2. The other notation for greatest integer function is
or [[ ]] in some books inspite of [ ].
3. The symbol [ ] denotes the process of finding the
greatest integer contained in a real number but not
greater than the real number put in [ ].
Thus, in general y = [f (x)] means that there is a
greatest integer in the value f (x) but not greater than
the value f (x) which it assumes for any x R .

NQ

This is why in particular y = [x] means that for a


particular value of x, y has a greatest integer which is
not greater than the value given to x.
4. The function y = [x], where [x] denotes integral
part of the real number x, which satisfies the equality
x = [x] + q, where 0 q < 1 is discontinuous at every
integer x = 0 , 1 , 2 , ... and at all other points, this
function is continuous.
5. If x and y are two arbitrary real numbers satisfying
the inequality n x < n + 1 and n y < n + 1 ,
where n is an integer, then [x] = [y] = n.
6. y = [x] is meaningless for a non-real value of x
because its domain is the set of all real numbers and
the range is the set of all integers, i.e. D [x] = R and R
[x] = {n: n is an integer} = The set of all integers,
3, 2, 1, 0, 1, 2, 3, , i.e., negative, zero or positive
integer.
7. f x = 0 0 f x < 1 . Further the solution

af

af

af

of 0 f x < 1 provides us one of the adjacent


intervals where x lies. The next of the a adjacent intervals
is determined by adding 1 to the left and right end
point of the solution of 0 f x < 1 . This process
of adding 1 to the left and right end point is continued
till we get a finite set of horizontal line segments
representing the graph of the function y = [f (x)]

af

More on Properties of Greatest Integer


Function.
(i) x + n = n + x , n I and x R
(ii) x = x , x I
(iii) x = x 1, x I
(iv) x n x n , n I
(v) x n x < n + 1, n I
(vi) x > n x n
(vii) x < n x < n , n I and x R
(viii) x + y x + y , x , y R
(ix)

LM x OP = L x O , n N and x R
N n Q MN n PQ

Function

(x) x = [x] + {x} where { } denotes the fractional part


of x , x R
(xi) x 1 < x x , x R

x x < x + 1 for all real values of x.

(xii)

Question: Define logarithmic function.


Answer: A function f : 0 , R defined by f (x) =
log a x is called logarithmic function, where
a 1 , a > 0 . Its domain and range are 0 , and R
respectively.

b g

b g

Question: Define Exponential function.


Answer: A function f: R R defined by f(x) = ax,
where a 1 , a > 0. Its domain and range are R and
0 , respectively.

b g

Question: Define the piece wise function.


Answer: A function y = f (x) is called the piece wise
function if the interval (open or closed) in which the
given function is defined can be divided into a finite
number of adjacent intervals (open or closed) over
each of which the given function is defined in different
forms. e.g.:

af

1. f x = 2 x + 3 , 0 x < 1
f (x) = 7, x = 1

af

f x = x ,1 < x 2

2. f (x) = x2 1, 0 < x < 2


3.

af
f a xf = 1 + x , 1 x < 0
f (x) = x 1, 0 < x < 2
f a xf = 2 x , x 2

15

af
f a xf = 4 x

1
2
x
3
3
2
2
1, x 1
3
2. A function y = f (x) may not be necessarily defined
by a single equation for all values of x but the function
y = f (x) may be defined in different forms in different
parts of its domain.
3. Piecewise function is termed also Piecewise defined
function because function is defined in each piece.
If every function defined in adjacent intervals is linear,
it is termed as Piecewise linerar function and if every
function defined in adjacent intervals is continuous,
it is called piecewise continuous function.
2

f x = x + 2,

Question: What do you mean by the real variables?


Answer: If the values assumed by the independent
variable x are real numbers, then the independent
variable x is called the real variable.
Question: What do you mean by the real function
(or, real values of function) of a real variable?
Answer: A function y = f (x) whose domain and range
are sets of a real numbers is said to be a real function
(or more clearly, a real function of a real variable) which
signifies that values assumed by the dependent
variable are real numbers for each real value assumed
by the independent variable x.

f x = x + 2, x 2

Note: The domain of a real function may not be


necessarily a subset of R which means that the domain
of a real function can be any set.

Examples:
1. Let A = , a , b , a , b and B = {1, 2, 3,
4, 5}
f = , 1 , a , b , 2 , a , 4 , b , 3 is
a real function since B is a subset of the set of real
numbers.
2. If f : R R such that f x = 2 x 1, x R ,
then f is a real function.

Notes:
1. Non-overlapping intervals: The intervals which
have no points in common except one of the end
points of adjacent intervals are called non overlapping
intervals whose union constitutes the domain of the

LM 1 OP , LM 1 , 2 OP and LM 2 , 1OP
N 3Q N3 3 Q N 3 Q

piece wise function. e.g.: 0 ,

serve as an example of non-overlapping intervals


whose union [0, 1] is the domain of the piece wise
function if it is defined as:
1
f x = 2x + 1, 0 x
3

af

l k p k p k pq
ma f bk p g bk p g bk p gr
af

Remarks:
1. In example (i) The domain of f is a class of sets and
in example (ii) The domain of f is R. But in both
examples, the ranges are necessarily subsets of R.
2. If the domain of a function f is any set other than
(i.e. different from) a subset of real numbers and the
range is necessarily a subset of the set of real

16

How to Learn Calculus of One Variable

numbers, the function must be called a real function


(or real valued function) but not a real function of a
real variable because a function of a real variable
signifies that it is a function y = f (x) whose domain
and range are subsets of the set of real numbers.
Question: What do you mean by a single valued
function?
Answer: When only one value of function y = f (x) is
achieved for a single value of the independent variable
x = a, we say that the given function y = f (x) is a
single valued function, i.e., when one value of the
independent variable x gives only one value of the
function y = f (x), then the function y = f (x) is said to
be single valued, e.g.:
1. y = 3x + 2
2. y = x2

3. y = sin 1 x, y
2
2
serves as examples for single valued functions because for each value of x, we get a single value for y.
Question: What do you mean by a multiple valued
function?
Answer: when two or more than two values of the
function y = f (x) are obtained for a single value of the
independent variable x = a, we say that the given
function y = f (x) is a multiple (or, many) valued
function, i.e. if a function y = f (x) has more than one
value for each value of the independent variable x,
then the function y = f (x) is said to be a multiple (or,
many) valued function, e.g.:
2

1. x + y = 9 x = 9 x y has two
real values, x < 3 .
2
2. y = x y = x is also a multiple valued
2

function since x = 9 y = 9 y =

9 y = 3 ( | y | =
=

= y for y > 0 and | y |

= y for y > 0).

Question: What do you mean by standard functions?


Answer: A form in which a function is usually written
is termed as a standard function.
e.g.: y = xn, sin x, cos x, tan x, cot x, sec x, cosec x,
1
sin x, cos1 x, tan1 x, cot1 x, sec1 x, cosec1 x, log
ax, log ex, ax, ex, etc. are standard functions.

Question: What do you mean by the inverse


function?
Answer: A function, usually written as f 1 whose
domain and range are respectively the range and
domain of a given function f and under which the
image f 1 (y) of an element y is the element of which y
was the image under the given function f, that is,
f

a yf = x f a xf = y .
D
f

y = f (x )

D
f

R
y

y=f

( x)

Remarks:
1. A function has its inverse it is one-one (or,
one to one) when the function is defined from its
domain to its range only.
2. Unless a function y = f (x) is one-one, its inverse
can not exist from its domain to its range.
3. If a function y = f (x) is such that for each value of
x, there is a unique values of y and conversely for
each value of y, there is a unique value of x, we say
that the given function y = f (x) is one-one or we say
that there exists a one to one (or, one-one) relation
between x and y.
4. In the notation f 1, (1) is a superscript written at
right hand side just above f. This is why we should
not consider it as an exponent of the base f which
means it can not be written as f

1
.
f

5. A function has its inverse it is both one-one


and onto when the function is defined from its domain
to its co-domain.

Function

Pictorial Representation of Inverse


Function
To have an arrow diagram, one must follow the
following steps.
1. Let f : D R be a function such that it is oneone (i.e. distinct point in D have distinct images in R
under f).
2. Inter change the sets such that original range of f
is the domain of f1 and original domain of f is the
range of f1.
3. Change f to f1.
Therefore, f : D R defined by y = f (x) s.t it is
1
one-one f : R D defined by f1 (y) = x is an
inverse function.
On Intervals
1. Values and range of an independent variable x: If
x is a variable in (on/over) a set C, then members
(elements or points) of the set C are called the values
of the independent variable x and the set C is called
the range of the independent variable x, whereas x
itself signifies any unspecified (i.e., an arbitrary)
member of the set C.
2. Interval: The subsets of a real line are called
intervals. There are two types (or, kinds) of an interval
namely (i) Finite and (ii) Infinite.
(i) Finite interval: The set containing all real numbers
(or, points) between two real numbers (or, points)
including or excluding one or both of these two real
numbers known as the left and right and points is
said to be a finite interval. A finite interval is classified
into two kinds namely (a) closed interval and (b) open
interval mainly.
(a) Closed interval: The set of all real numbers x
subject to the condition a x b is called closed
interval and is denoted by [a, b] where a and b are
real numbers such that a < b.

In set theoretic language, [a, b] = {x: a x b , x


is real}, denotes a closed interval.

17

Notes:
1. The notation [a, b] signifies the set of all real
numbers between a and b including the end points a
and b, i.e., the set of all real from a to b.
2. The pharase at the point x = a signifies that x
assumes (or, takes) the value a.
3. A neighbourhood of the point x = a is a closed
interval put in the form [a h, a + h] where h is a
positive number, i.e.,
[ a h, a + h] = { x : a h x a + h , h is a
small positive number}
4. All real numbers can be represented by points on
a directed straight line (i.e., on the x-axis of cartesian
coordinates) which is called the number axis. Hence,
every number (i.e. real number) represents a definite
point on the segment of the x-axis and conversely
every point on the segment (i.e., a part) of the x-axis
represents only one real number. Therefore, the
numbers and points are synonymes if they represent
the members of the interval concerned.(Notes 1. It is
a postulate that all the real numbers can be represents
by the points of a straight line. 2. Neigbourhood
roughly means all points near about any specified
point.)
(b) Open interval: The set of all real numbers x subject
to the condition a < x < b is called an open interval
and is denoted by (a, b), where a and b are two real
numbers such that a < b.
a

In the set theoretic language, (a, b) = {x: a < x < b,


x is real}
Notes:
1. The notation (a, b) signifies the set of all real
numbers between a and b excluding the end points a
and b.
2. The number a is called the left end point of the
interval (open or closed) if it is within the circular or
square brackets on the left hand side and the number
b is called the right end point of the interval if it is
within the circular or square brackets on the right h
and side.

18

How to Learn Calculus of One Variable

3. Open and closed intervals are represented by the


circular and square brackets (i.e., ( ) and [ ] )
respectively within which end points are written
separated by a comma.
(c) Half-open, half closed interval (or, semi-open, semi
closed interval): The set of all real numbers x such
that a < x b is called half open, half closed interval
(or, semi-open, semi closed interval), where a and b
are two real numbers such that a < b.

In set theoretic language,

aa , f = {x: x > a, x is real}


or, aa , f = {x: a < x < , x is real}
(c) The interval a , f : The set of all real numbers

x such that x a is an infinite interval and is denoted

by a , .
a

(a, b) = {x: a < x b , x is real}


Note: The notation (a, b] signifies the set of all real
numbers between a and b excluding the left end point
a and including the right end point b.
(d) Half closed, half open (or, semi closed, semi open
interval): The set of all real numbers x such that
a x < b is called half-closed, half open interval
(or, semi closed, semi open interval), where a and b be
two real numbers such that a < b.
a

In set theoretic language, [a, b) = {x: a x < b , x


is real}
Note: The notation [a, b) signifies the set of all real
numbers between a and b including the left end point
a and excluding the right end point b.
2. Infinite interval

In set theoretic language,

a , = {x: x a , x is real}

or, a , = {x: a x > , x is real}

(d) The interval , a : The set of all real numbers


x such that x < a is an infinite interval and is denoted

by , a .

a , af = {x: x < a, x is real}


or, a , a f = {x: < x < a , x is real}
(e) The interval a , a : The set of all real numbers

x such that x a is an infinite interval and is denoted

by , a .

(a) The interval , : The set of all real numbers

x is an infinite interval and is denoted by , or


R.

In set theoretic language,

R = , = {x: < x < , x is real}

a f

(b) The interval a , : The set of all real numbers x


such that x > a is an infinite interval and is denoted

a f

by a , .

In set theoretic language,

a , a
a , a

= {x: x a , x is real}
= {x: < x a , x is real}

Remember:
1. In any finite interval, if a and/b is (or, more) replaced
by and / , we get what is called an infinite
interval.

19

Function

2. a x b signifies the intersection of the two


sets of values given by x a and x b .
3. x a or x b signifies the union of the two
sets of values given by x a and x b .
4. The sign of equality with the sign of inequality
(i.e., or ) signifies the inclusion of the specified
number in the indicated interval finite or infinite. The
square bracket (i.e., [,)also (put before and/after any
specified number) signifies the inclusion of that
specified number in the indicated interval finite or
infinite.
5. The sign of inequality without the sign of equality
(i.e. > or <) signifies the exclusion of the specified
number in the indicated interval finite or infinite. The
circular bracket (i.e. ( , ) also (put before and/after any
specified number) signifies the exclusion of that
specified number indicated interval finite or infinite.
6. (i) : x a , b a x b and

f a f

x a,b

c
x a , b x , a b , where [a,
b] c = R [a, b] complement of [a, b] =

a , a f ab , f .
(ii) : x aa , b f a < x < b
x a , a b , f .

a f

and x a , b

7. Intervals expressed in terms of modulus: Many


intervals can be easily expressed in terms of absolute
values and conversely.
(i) | x | < a a < x < a x (a, a), where a is
any positive real number and x R .

(ii)

x a a x a x a , a

a is any positive real number and x R .

where

aa , f either x <a or x> a, a being any positive

real number and x R .


a

(iv)

x a either x a or x a x

a , a

a, .
a

Evaluation of a Function at a Given Point


Evaluation: To determine the value of a function y =
f (x) at a given point x = a, is known as evaluation (or,
more clearly evaluation of the function y = f (x) at the
given point x = a)
Notation: [f (x)]x = a = (f (x))x = a = f (a) is a notation to
signify the value of the function f at x = a.
Type 1: To evaluate a function f (x) at a point x = a
when the function f (x) is defined by a single expression,
equation or formula.
Working rule: The method of finding the value of a
function f (x) at the given point x = a when the given
function f (x) is defined by a single expression,
equation or formula containing x consists of following
steps.
Step 1: To substitute the given value of the
independent variable (or, argument) x wherever x
occurs in the given expression, equation, or formula
containg x for f (x)
Step 2: To simplify the given expression, equation or
formula containg x for f (x) after substitution of the
given value of the independent variable (or, argument)
x.
Solved Examples

(iii) | x | > a x [a, a] x , a

1. If f (x) = x2 x + 1, find f (0), f (1) and f


Solution: f (x) = x2 x + 1
f (0) = 02 0 + 1 = 1

F 1I .
H 2K

20

How to Learn Calculus of One Variable

f (1) = 12 1 + 1 = 1

Working rule: It consists of following steps:

F 1I = F 1I F 1I + 1
H 2K H 2K H 2K

Step 1: To consider the function f (x) = f1 (x) to find


the value f (a1), provided x = a1 > a and to and to put
x = a1 in f (x) = f1 (x) which will provide one the value
f (a1) after simplification.
Step 2: To consider the function f (x) = f2 (x) to find
the value f (a), provided x = a is the restriction against
f2 (x) and put x = a in f2 (x). If f (x) = f2 (x) when the
restrictions imposed against it are x a , x a ,

and f

3
1 1
+1=
4
4 2

af

2. If f x =

af

f 1+ h =

af

f 1 =

f af

1
x

...(1)

1
1+ h

...(2)

Solution: f x =

1
f 1+ h f 1
, find
.
x
h

1
= 1 ... (3)
1

a f a f 1 +1 h 1 = 1 +hh
a4f = f a1 + hf f a1f

f 1+ h f 1 =

...(3)

h / 1 + h
1
=
1+ h
h

Type 2: (To evaluate a piecewise function f (x) at a


point belonging to different intervals in which different
expression for f (x) is defined). In general, a piece wise
function is put in the form
f (x) = f1 (x), when x > a
= f2 (x), when x = a
f3 (x), when x < a, x R
and one is required to find the values (i) f (a1) (ii) f
(a) and (iii) f (a0), where a, a0 and a1 are specified (or,
given) values of x and belong to the interval x > a
which denote the domains of different function f1 (x),
f2 (x) and f3 (x) etc for f (x).
Note: The domains over which different expression
f1 (x), f2 (x) and f3 (x) etc for f (x) are defined are intervals
finite or infinite as x > a, x < a, x a , x a , a < x <
b, a x < b , a < x b and a x b etc and
represent the different parts of the domain of f (x).

a x < b , a < x b , a x b or any other


interval with the sign or equality indicating the
inclusion of the value a of x, we may consider f2 (x)
to find the value f (a). But if f (x) = f2 (x) = constant,
when x = a is given in the question, then f2 (x) = given
constant will be the required value of f (x) i.e. f (x) =
given constant when x = a signifies not to find the
value other than f (a) which is equal to the given
constant.
Step 3: To consider the function f (x) = f3 (x) to find
the value f (a2) provided x = a2 < a and x = a2 in f (x) =
f3 (x) which will provide one the value f (a2) after
simplification.
Remember:
1. f (x) = f1 (x), when (or, for, or, if) a x < a 2 signifies
that one has to consider the function f (x) = f1 (x) to
find the functional value f1 (x) for all values of x (given
or specified in the question) which lie in between a1
and a2 including x = a1.
2. f (x) = f2 (x), when (or, for, or, if) a 2 < x a 3 ,
signifies that one has to consider the function f (x) =
f2 (x),to find the functional value f2 (x) for all values of
x (given or specified in the question) which lie in
between a2 and a3 including x = a3.
3. f (x) = f3 (x), when (or, for, or, if) a4 < x < a5 signifies
that one has to consider the function f (x) = f3 (x) to
find the functional value f3 (x) for all values of x (given
or specified in the question) which lie in between a4
and a5 excluding a4 and a5.
Solved Examples
1. If f : R R is defined by
f (x) = x2 3x, when x > 2
= 5, when x = 2
= 2x + 1, when x < 2, x R

Function

find the values of (i) f (4) (ii) f (2) (iii) f (0) (iv) f (3)
(v) f (100) (vi) f (500).
Solution: 1. 3 4 > 2 , by definition, f (4) = (x2
3x) for x = 4 = 42 3 (4) = 16 12 = 4
(ii) 3 2 = 2, by definition, f (2) = 5
(iii) 0 < 2, by definition, f (0) = 2 (0) + 1 = 1
(iv) 3 < 2, by definition, f (3) = 2 (3) + 1 = 5
(v) 100 > 2, by definition, f (100) = (100)2 3
(100) = 10000 300 = 9700
(vi) 500 < 2, by definition f (500) = 2 (500) +
1 = 1000 + 1 = 999
2. If f (x) = 1 + x, when 1 x < 0
= x2 1, when 0 < x < 2
2x, when x 2

a f FH 12 IK , f FH 21 IK

find f 3 , f

Solution: 3 f (x) = 2x for x 2


f (3) = 2 3 = 6 (3 x = 3 2 )

af
F 1I = 1 + F 1I = 1
f
H 2K H 2 K 2
F3 x = 1 [1 , 0)I
H
K
2

3 f x = 1 + x , for 1 x < 0

3 f (x) = x2 1, for 0 < x < 2

F 1I = F 1I
H 2K H 2K
F3 x = 1 a0 , 2fI
H 2
K
f

1 = 1 +

1
3
=
4
4

Refresh your memory:


1. If a function f (x) is defined by various expressions
f1 (x), f2 (x), f3 (x) etc, then f (a0) denotes the value of
the function f (x) for x = a0 which belongs to the
domain of the function f (x) represented by various
restrictions x > a, x < a, x a , x a , a < x < b,

a x b , a x < b , and a < x b etc.


2. Supposing that we are required to find the value
of the function f (x) for a point x = a0 which does not

21

belong to the given domain of the function f (x), then


f (a0) is undefined, i.e., we cannot find f (a0), i.e., f (a0)
does not exist.
f (x) = x2 1, when 0 < x < 2
= x + 2, when x 2
find f (1)
Solutions: 1 domain of f (x) represented by the
union of the restrictions 0 < x < 2 and x 2 (i.e., 0 <
x < 2 or x 2 ). For this reason f (1) is undefined
(i.e., f (x) is undefined at x = 1).
3. Sometimes we are required to find the value of a
piecewise function f (x) for a0 h where h > 0, in
such cases, we may put h = 0.0001 for easiness to
guess in which domain (or, interval) the point
represented by x = a h lies.
e.g.: If a function is defined as under
f (x) = 1 + x, when 1 x < 0
= x2 1, when 0 < x < 2
2x, when x 2
find f (2 h) and f (1 + h)
(Footnotes: 1. f (a) exists or f (a) is defined a
lies in the domain of f. 2. f (a) does not exist or f (a) is
undefined a does not lie in the domain of f.)
Solution: 1. Putting h = 0.001, we get 2 h = 2 0.001
= 1.999 and 1.999 (0, 2) = 0 < x < 2
f (2 h) = (2 h)2 1 = 22 + h2 4h 1 = 4 + h2
4h 1
= 3 + h2 4h = h2 4h + 3
2. Putting h = 0.001, we get 1 + h = 1 + 0.001 =
0.999 and 0.999 [1, 0) = 1 x < 0
f (1 + h) = (1 + x)x =1 + h = 1 + h 1 = h
Domain of a Function
Sometimes a function of an independent variable x is
described by a formula or an equation or an expression
in x and the domain of a function is not explicitly
stated. In such circumstances, the domain of a function
is understood to be the largest possible set of real
numbers such that for each real number (of the largest
possible set), the rule (or, the function) gives a real
number or for each of which the formula is meaningful
or defined.

22

How to Learn Calculus of One Variable

Definition: If f : D R defined by y = f (x) be a


real valued function of a real variable, then the domain
of the function f represented by D (f) or dom (f) is
defined as the set consisting of all real numbers
representing the totality of the values of the
independent variable x such that for each real value
of x, the function or the equation or the expression in
x has a finite value but no imaginary or indeterminate
value.
Or, in set theoretic language, it is defined as:
If f : D R be real valued function of the real
variable x, then its domain is D or D (f) or dom (f)
= { x R : f x has finite values }

af
= { x R : f a x f has no imaginary or indeterminate

value.}
To remember:
1. Domain of sum or difference of two functions f (x)

af af

and g (x) = dom f x g x

= dom (f (x)) dom

(g (x)).
2. Domain of product of two functions f (x) and g (x)

af af

g (x) = dom f x g x

= dom (f (x)) dom (g (x)).


3. Domain of quotient of two functions f (x) and g (x)

LM f a xf OP = dom f a xf dom b ga xfg


N ga x f Q
lx: gaxf 0q
= dom b f a x fg dom b ga x fg lx: ga x f 0q i.e.,
= dom

the domain of a rational function or the quotient


function is the set of all real numbers with the exception
of those real numbers for which the function in
denominator becomes zero.
Notes: 1. The domain of a function defined by a
formula y = f (x) consists of all the values of x but no
value of y (i.e., f (x)).
2. (i) The statement f (x) is defined for all x signifies

that f (x) is defined in the interval , .


(ii) The statements f (x) is defined in an interval
finite or infinite signifies that f (x) exists and is real
for all real values of x belonging to the interval. Hence,

the statement f (x) is defined in the closed interval


[a, b] means that f (x) exists and is real for all real
values of x from a to b, a and b being real numbers
such that a < b. Similarly, the statement f (x) is defined
in the open interval (a, b) means that f (x) exists and
is real for all real values of x between a and b (excluding
a and b)

LM f a xf = 0 , or
N ga xf = 0
LMRS f a xf 0
ga x f 0
(ii) f a x f g a x f 0 MT
MMRS f a xf 0 , or
NT g a x f 0
LMRS f a xf 0
ga x f 0
(iii) f a x f g a x f 0 MT
MMRS f a xf 0 , or
NT g a x f 0
LMRS f a xf 0
f a xf
ga xf < 0
0 MT
(iv)
MMRS f a xf 0 , or
ga x f
NT g a x f > 0
LMRS f a xf 0
f a xf
ga xf < 0
0 MT
(v)
MMRS f a xf 0 , or
ga x f
NT g a x f > 0
4. (i) e x a j < 0 a < x < a
af af

3. (i) f x g x = 0

(ii)
(iii)

ex

ex
e

(iv) x a

j 0 a x a
j > 0 x < a or

x> a

j 0 x a or

xa

23

Function

a x a f a x b f < 0 a < x < b aa < b f


x aa , b f
(ii) a x a f a x b f 0 a x b aa < b f

5. (i)

x a 2 , b2
They mean the intersection of
(a) x > a1 and x < b1

(b) x a 2 and x b2

a x a f a x bf > 0 x < a or x > b aa < bf


x a , a f ab , f
(iv) a x a f a x b f 0 x a or x b aa < bf
x a , a b , f
(iii)

They mean the union of


(a) x < a and x > b
(b) x a and x b

Method of Representation of
Union and Intersection on Real Lines
If the set of the points on the line segment AB be the
set E and the set of the point on segment CD be the
set F, then the union of E and F is the segment AD =
AB + BD = sum or union and the intersection of E and
F is the segment CB = common segment.
common
segment

Type 1: Problems based on finding the domains of


polynomial functions.
Working rule: One must remember that a polynomial
in x has the domain R (i.e., the set of the real numbers)
because any function f of x which does not become
undefined or imaginary for any real value of x has the
domain R. Hence, the linear y = ax + b; the quadratic
y = ax2 + bx + c; and the square functions y = x2 have
the domain R.
Solved Examples
Find the domain of each of the following functions:
1. y = 11x 7
Solution: y = 11x 7 is a linear function and we know
that a linear function has the domain R.
Hence, domain of y (= 11x 7) = R = , +

Question: How to represent the union and


intersection on a number line?
Answer: Firstly, we recall the definitions of union and
intersection of two sets.
Union: The union of two sets E and F is the set of
elements belonging to either E or F.
Intersection: The set of all elements belonging to
both sets E and F is called intersection of E and F.

Finding the Domain of


Algebraic Functions

Now some rules to find the domain of real valued


functions are given. They are useful to find the
domain of any given real valued function.

2. y = x2 3x + 7
Solution: y = x2 3x + 7 is a quadratic function and
we know that a quadratic function has the domain R.
Hence, domain of y = (= x2 3x + 7) =
R = , +

y = x2

3.
Solution: y = x2 is a square function and we know
that a square function has the domain R.
Hence, domain of y = (= x2) = R = , +

Type 2: Problems based on finding the domain of a


function put in the form:
f x
,g x 0
(i) y =
g x
1
,g x 0
or, (ii) y =
g x
Working rule: It consists of following steps:
1. To put the function (or, expression in x) in the
denominator = 0, i.e., g (x) = 0
2. To find the values of x from the equation g (x) = 0
3. To delete the valued of x from R to get the required

af af
af
af af

domain, i.e., domain of

af
af

f x
1
= R {roots of
or
g x
g x

af

the equation g (x) = 0}, where f (x) and g (x) are


polynomials in x.

24

How to Learn Calculus of One Variable

Note: When the roots of the equation g (x) = 0 are


imaginary then the domain of the quotient function

af
af

f x
1
put in the form:
=R
or
g x
g x

af

Solved Examples
Find the domain of each of the following functions:

domain = R {5}
4. y =

2x 4
2x + 4

Solution: y =

2x 4
2x + 4

Now, putting, 2x + 4 = 0

1. y =

x 3x + 2

2x 4 x =

x + x6

domain R {2}

Solution: y =

x 3x + 2
2

x +x6

Now, putting x2 + x 6 = 0
x2 + 3x 2x 6= 0
x (x + 3) 2 (x + 3) = 0
(x + 3) (x 2) = 0
x = 2, 3
domain = R {2, 3}
2

2. y =

x 2x + 4

x 2x + 4

6. y =

x + 2x + 4

x +1 =

x = 1 3 imaginary or complex numbers.


domain = R
x
5 x

1
2

x 1

Solution: y =

Now, putting, x2 + 2x + 4 = 0
x2 + 2x + 4 = 0
(x + 1)2 + 3 = 0
(x + 1)2 = 3

3. y =

Now, putting (x 1) (x 2) = 0
x = 1, 2
domain = R {1, 2}

a f ax 1f1ax 2f
1
Solution: f a x f =
ax 1f ax 2f
5. f x =

x + 2x + 4

Solution: y =

4
= 2
2

1
2

x 1

Now, putting, x2 1 = 0
2

x = 1 x = 1
domain = R {1, 1}

7. y =

1
x

1
x
Now, putting x = 0 x = 0 i.e. y is undefined at x

Solution: y =
=0

domain = R {0}
2

x
Solution: y =
5 x
Now, putting, 5 x = 0
x=5

8. y =

x 3x + 2
2

x + x6
2

Solution: y =

x 3x + 2
2

x + x6

Function

Now, putting, x2 + x 6 = 0 x2 + 3x 2x 6 = 0
x (x + 3) 2(x + 3) = 0 (x 2) (x + 3) = 0 x =
2, 3
domain = R {2, 3}
9. y =

1
2x 6

Solution: y =

6
=3
2

kp a

f a

domain = R 3 = , 3 3 , +

10. y =

1
x 5x + 6
2

x 5x + 6

k p a

1. y =

f a f

af
f a xf
ga x f

f x

(iii)

(iv)

Step 2: To find the values of x for which a x + b 0


to get the required domain.
Step 3: To write the domain = [root of the inequation
a x + b 0, + )

Solved Examples
Find the domain of each of the following fucntions:

Type 3: Problems based on finding the domain of the


square root of a function put in the forms:

(ii)

af

f x , when f (x) = ax + b.

a f consists of the values of x for which


f a xf 0 .
2. x c x c , + f .

domain = R 2 , 3 = , 2 2 , 3

(i)

It consists of two types when:


(i) f (x) = ax + b = a linear in x.
(ii) f (x) = ax2 + bx + c = a quadratic in x.
(i) Problems based on finding the domain of a

f x

x2 5x + 6 = 0
x2 3x 2x + 6 = 0
x (x 3) 2 (x 3) = 0
(x 3) (x 2) = 0
x = 2, 3

a3, f

f x .

Notes: 1. The domain of a function put in the form

Solution: y =

af

put in the form:

Working rule: It consists of following steps:


Step 1: To put a x + b 0

Now, putting 2x 6 = 0

x=

1. Problems based on finding the domain of a function

function put in the form:

1
2x 6

25

af
f a xf
ga x f
g x

Now we tackle each type of problem one by one.

Solution: y =

Now, putting x 0 x 0
domain = 0 , +

2. y =

2x 4

Solution: y =

2x 4

Now, putting 2 x 4 0 x

domain = 2 , +
3. y =

x +

Solution: y =
Putting y1 =
y = y1 + y2

4
=2
2

x1
x +

x1

x and y2 =

x , we have

26

How to Learn Calculus of One Variable

a f
a f
Now, domain of y e= x j = D asay f = 0 , + f
domain of y = dom y1 dom y2
1

ax + bx + c = x

domain of

a < f .

[from example 1.] again, we require to find the domain

(ii) a = coefficient of x2 = + ve (and, ax2 + bx + c =

of y2 =

a x

x 1 .

x 1 0 x 1

Putting

e=

a f

domain y 2

x 1 = D2 say = 1 , + Hence, domain of

a f

y = D (say) = dom (y1) dom y2

= D1 D2

= 0 , + 1, +
= 1,

f a x f 0) x does not lie between


and domain of ax + bx + c = R a , f =
a , f a , + f .
(iii) a x + f a x + f should be written as
bx afg , bx afg while finding the domain of
the square root of ax + bx + c = a a x + f a x + f .
2

Solved Examples
Find the domain of each of the following functions:
1. y =

shaded portion = D1 D2

x 3x + 4

Solution: y =

x 3x + 4

(ii) Problems based on finding the domain of a


function put in the form:
y=

af

f x , when f (x) =

ax2

+ bx + c and ,

are the roots of ax2 + bx + c = 0 <

working rule: It consists of following steps:


2

Step 1: To put a x + b x + c 0
2

Step 2: To solve the in equation a x + b x + c 0


for x by factorization or by completing the square.
Step 3: To write the domain of

ax + bx + c =

x only when the coefficient of x2 = a = ve

f a x f and to write the


+ bx + c = R a , f only when

and ax2 + bx + c = a x
domain of

ax

the coefficient of x2 = a +ve and ax2 + bx + c =

a x

f a x f .

Notes: (i) a = coefficient of x2 = ve (and, ax2 + bx +

c= a x

f a x f 0) x lies between and

Now x 3x + 4 0

j
x a x 4f + a x 4f 0
a x + 1f a x 4f 0
a x 4f b x a1fg 0 x does not lie be2

x 4x + x 4 0

tween 1 and 4 x 1 or x 4 .
domain = R (1, 4)

a x 2f a x 5f
Solution: y = a x 2f a x 5f

2. y =

Now, (x 2) (x 5) 0 x does not lie between


2 and 3.
x 2 or x 5
domain = R (2, 5)
3. y =

x 5x + 6

Solution: y =

x 5x + 6

Function
2

Now, x 5x + 6 0

Now, 5 6 x x 0

x2

x + 6x + 5 0

f a x 3f 0

x 2 or x 3
domain = R (2, 3)

x + 5x + x + 5 0

x + 5x 6
2

Solution: y =

x + 5x 6

and 1 5 x 1 .
domain = [5, 1]

Now, x + 5x 6 0
2

x 5x + 6 0

x2

16 x 24 x

Solution: y =

a1 xf a x + 3f
Solution: y = a1 x f a x + 3f
Now, a1 x f a x + 3f 0
a x 1f a x + 3f 0
a x 1f a x + 3f 0
a x 1f b x a3fg 0
7. y =

f a x 3f 0

x lies between 2 and 3


2x3
domain = [2, 3]
5. y =

f a f
a x + 5f a x + 1f 0
b x a1fg b x a5fg 0 x lies between 5
x x+5 + x+5 0

4. y =

16 x 24 x

3 x 1
domain = [3, 1]

Now, 16 x 24 x 0
2

2 x 3x 0

8. y = 1 x

2 x + 3x 0

Solution: y = 1 x

a f
x b2 x a3fg 0

x 2x + 3 0

LM
N

5 6 x x

Solution: y =

j0

x 1 0

OP
Q

fa

0 x lies between 1 and +1


1 x 1
domain = [1, 1]

5 6 x x

1 x

f b a fg

x 1 x + 1 0 x 1 x 1

2
domain = 3 , 0
6. y =

Now, 1 x 2 0

3
x lies between and 0
2
3
x0
2

27

9. y = 4 x

Solution: y = 4 x

28

How to Learn Calculus of One Variable


2

Now, putting 4 x 0

a f a x + 2f 0
a x 2f b x a2fg 0

x2

x lies between 2 and 2 2 x 2


domain = [2, 2]
4x

Solution: y =

1
2

4x

a f a x + 2f 0
a x 2f b x a2fg 0

x2

x lies between 2 and 2 2 x 2


domain = [2, 2]
4 x

a x 2f a x 3f
Solution: y = a x 2f a x 3f
Now, a x 2 f a x 3f 0

13. y =

x 2 or x 3
x does not lie between 2 and 3
domain = R [2, 3] = , 2 3 , +

14. y =

x + 2x + 3

Solution: 3 y =

x + 2x + 3

1
Solution: y =
2

4 x

Now, putting 4 x 0
2

x 40

a f a x + 2f 0
a x 2f b x a2fg 0

x2

2 x 2
domain = [2, 2]
2

x 4x + 3

Solution: y =

x 40

12. y =

domain = R [1, 3] = , 1 3 , +

1
2

x does not lie between 1 and 3


x 1 or x 3

Now, putting 4 x 0

11. y =

a f a f
a x 1f a x 3f 0

x3 x3 0

x 40

1
10. y =
2

x 3x x + 3 0

x 4x + 3

Now, x 2 4 x + 3 0

Now, x + 2 x + 3 0

a f
a x + 1f
x+1

+ 2 0, x

2 , which is true for all x R

domain = R = ,

Type (ii): Problems based on finding the domain of


a function put in the form : y =

af
af

f x
g x

While finding the domain of the square root


of a quotient function (i.e; y =
remember the following facts:

af
af

f x
) one must
g x

29

Function

values of x for which

af
af

af
af

f x
) consists of those
g x

1. The domain of y ( =

af
af

f x
0
g x

af

af

af

f x
0 f x 0 , g x > 0 , or f x 0 ,
2.
g x
g (x) < 0.
3.

FG x IJ 0 x <
H x K

or < x ac-

cording as < or < .


4.

FG x IJ 0 x or x < if < and


H x K

LM
N

Hence, domain = R 3 ,

LM 2 , + I
N3 K

I a
K

2
= , 3
3

or, alternatively:

2
x
3x 2
3 0 x < 3 or
0
x+3
2x + 6

f LMN IK
2
Hence, domain = a , 3f L , + I
MN 3 K
x

2
2
x , 3
,+
3
3

x 1
x +1

x or x > if < .

2. y =

5. The function in the denominator 0 always.

Solution: y is defined for all those x for which

Solved Examples
Find the domain of each of the following functions:

3x 2
2x + 6

1. y =

3x 2
0
2x + 6

a
a

f UV
f W
or, (2) a3x 2f 0U
a2 x + 6f < 0VW , i.e; x < 3
2
3

x2
x+2

a f

x , 2 2 , +

Hence, domain = , 2 2 , +

2
(1) and (2) x , or x < 3 x
3

a , 3f LMN 23 , + IK

x2
x2
0
0 x < 2 or, x 2
x+2
x 2

Solution: y is defined for all those x for which

(1) 3x 2 0
2
, i.e; x
2x + 6 > 0
3

Hence, domain = , 1 1 , +
3. y =

a f

x , 1 1 , +

Solution: y is defined for those x for which

x 1
x 1
0
0 x < 1 or x 1
x +1
x 1

Type (iii): Problems on finding the domain of a


function put in the form: y =

af

g x

Working rule: It consists of following steps:


1. To put g (x) > 0
2. To find the values of x for which g (x) > 0

30

How to Learn Calculus of One Variable

3. To form the Domain with the help of the roots of


the in equation g (x) > 0.
Note: The domain of a function put in the form
y=

Type (iv): Problems on finding the domain of a


function put in the form: y =

a f consists of all those values of x for which

g x

Solved Examples
1. Find the domain of each of the following functions:

y=

a2 xf ax + 3f

Solution: y is defined for all those values of x for


which (2 x) (x + 3) > 0 (x 2) (x + 3) < 0 x lies

between 3 and 2 3 < x < 2 x 3 , 2 hence,


domain = (3, 2)
2. y =

a1 xf ax + 2f

Solution: y is defined for all those values of x for


which (1 x) (x + 2) > 0 (x 1) (x (2)) < 0 x

lies between 2 and 1 2 < x < 1 x 2 , 1


hence, domain = (2, 1).
3. y =

a f is the same as for


g axf

f x

the domain of a function of the form y =

1. y =

x
2

x 3x + 2

Solution: y is defined when x2 3x + 2 > 0 x2 2x


x + 2 > 0 x (x 2) (x 2) > 0 (x 1) (x 2)
> 0 x < 1 or x > 2.

f a

Hence, domain = R [1, 2] = , 1 2 , +

x 5x + 6

f a

Hence,

domain

3 x , 2 3 , +

a3, + f

af

g x

Solved Examples
Find the domain of each of the following functions:

which means.
1. To put g (x) > 0 and to find the values of x from the
in equality g (x) > 0.
2. To form the domain with the help of obtained values
of x.

Solution: y is defined for all those values of x for


which x2 5x + 6 > 0 x2 3x 2x + 6 > 0 (x
3) x 2 (x 3) > 0 (x 3) (x 2) > 0 x < 2 or x >

4. y =

Working rule: The rule to find the domain of a


function of the form y =

g (x) > 0.

af.
g axf

f x

= R 2 , 3 = , 2

2. y =

a1 xf ax 2f

Solution: y is defined when (1 x) (x 2) > 0 (x


1) (x 2) < 0 x lies between 1 and 2 1 < x < 2

a f

x 1, 2 .
Hence, domain = (1, 2)

Finding the Domain


of Logarithmic Functions

x
Solution: y is defined for all those values of x for

which x > 0 x < 0 x , 0

Hence, domain = , 0 .

There are following types of logarithmic functions


whose domains are required to be determined.
(i) y = log f (x)
(ii) y = log

af

f x

(iii) y = log | f (x) |

Function

FG f a xf IJ
H ga x f K
f a xf
y=
log g a x f

Solution: Method (1)


y is defined when (3x 2 4x + 5) > 0

(iv) y = log

(v)

F
H

Type 1: Problems based on finding the domain of a


function put in the form: y = log f (x).
Working rule: It consists of following steps:
Step 1: To put f (x) > 0 and to solve the in equality f
(x) > 0 for x.
Step 2: To form the domain with the help of obtained
values of x.
Notes: 1. The domain of the logarithmic function y =
log f (x) consists of all those values of x for which f (x)
> 0.
2. Log f (x) is defined only for positive f (x).
Solved Examples
Find the Domain D of each of the following functions:
1. y = log (4 x)
Solution: y is defined when 4 x > 0 x > 4
x < 4.

af a

af a

D y = , 4
2. y = log (8 2x)
Solution: y is defined when (8 2x) > 0 2x > 8
x < 4.
D y = , 4
3. y = log (2x + 6)
Solution: y is defined when (2x + 6) > 0 2x > 6

a
f
Hence, D a y f = a3 , + f

x > 3 x 3 , +

4. y = log {(x + 6) (6 x)}


Solution: y is defined when (x + 6) (6 x) > 0 (x +
6) (x 6) < 0 x lies between 6 and 6 6 < x <

3 x

(vi) y = log log log f (x)


Now we tackle each type of problem one by one.

31

6 x 6 , 6
Hence, D (y) = (6, 6)
5. y = log (3x2 4x + 5)

I
K

4
5
>0
x+
3
3

LMF x 4 x + F 4 I F 4 I + 5I OP > 0
MNGH 3 H 6 K H 6 K 3JK PQ
LF 2 I + F 5 4 I OP > 0
3M x
MNH 3 K H 3 9 K PQ
F 2 I + a3f F 11I > 0
3 x
H 3K
H 9K
F 2 I > 11 which is true x R
3 x
H 3K 9
D a y f = R = a , + f
3

Notes: 1. Imaginary or a complex numbers as the


2

roots of an equation a x + b x + c = 0 domain

af

of log f x = R = , + as in the above


example roots are complex.
2. The method adopted in the above example is called
if method.
3. A perfect square is always positive which is greater
than any negative number.
Method 2. This method consists of showing that
2

a x + b x + c > 0 , x if a > 0 and discriminant = b2


4ac < 0 here 3 > 0, and discriminant = 16 60 = 34
<0
y is defined x R

af

Therefore, D y = R = , +
6. y = log (x3 x)
Solution: y is defined when (x3 x) > 0 x (x2 1)
> 0 x (x + 1) (x 1) > 0 (x 0) (x + 1) (x 1) >
0 (x (1)) (x 0) (x 1) > 0
Now let f (x) = (x (1)) (x 0) (x 1)
If x < 1, then f (x) < 0 as all the three factors are
< 0.

32

How to Learn Calculus of One Variable

If 1 < x < 0, then f (x) > 0


If 0 < x < 1, then f (x) < 0 and if x > 1, then f (x) > 0

a f a f
D a y f = a 1 , 0f a1 , f

Hence, f(x) > 0 x 1 , 0 1 ,

Type 2: Problems based on finding the domain of a


function put in the form y = log

af

f x

Working rule: One must remember that the function


y = log

af

f x

is defined when y =

af

f x > 0,

i.e., f (x) > 0 which means the domain of the function


y = log

af

f x

consists of all those values of x for

which f (x) > 0.


Solved Examples
Find the domain of each of the following functions:
1. y = log

x4

Solution: y = log

a x 4f > 0 x > 4
D a y f = a4 , f

x4

is defined when

2. y = log 6 x
Solution: y = log 6 x is defined when (6 x) >
0 6>x x<6

af a f
3. y = log e x 4 + 6 x j
Solution: y = log e x 4 + 6 x j is defined
when e x 4 + 6 x j > 0 a x 4 f and
b6 xg 0 x 4 and x 6 4 x 6
D a yf = 4 , 6
D y = , 6

Type 3: Problems based on finding the domain of a


function put in the form: y = log | f (x) |
Working rule: It consists of following steps:
1. To put f (x) = 0 and to find all the values of x from
the equation f (x) = 0.

2. To form the domain of y which is the set of all real


values of x excluding those values of x at which y is
undefined , i.e.,
D (y) = R {values of x at which f (x) =0}
Note: While finding the domain of a function of the
form: y = log | f (x) |, one must find all those values of
x at which the function f (x) (i.e., the function under
the symbol of absolute value) becomes zero and then
those values of x should be deleted from R (i.e., the
set of all real numbers).
Solved Examples
Find the domain of each of the following functions:
1. y = log | x |
Solution: log | x | is undefined only when | x | = 0, i.e.,
x=0

af

kp

af

D y = R 0
2. y = log | 4 x2 |
Solution: log | 4 x2 | is undefined only when | 4 x2
| = 0; i.e., 4 x2 = 0 x2 4 = 0 (x 2) (x + 2) = 0
(x 2) (x (2)) = 0 x = 2 or x = 2

D y = R 2, 2

Type 4: Problems based on finding the domain of a


function put in the form: y = log

FG f a xfIJ .
H g a xf K

Working rule: It consists of following steps:


1. To put

af
af

f x
> 0 and to solve the in equalities f
g x

(x) > 0 and g (x) > 0 or f (x) < 0 and g (x) , 0 seperately.
2. To form the domain of y with the help of obtained
values of x for which

af
af

f x
> 0.
g x

Notes: 1. The domain of a logarithmic function of

FG f a xfIJ consists of all those


H g a xf K
f a xf
values of x for which
> 0.
g a xf
the form y = log

2. One must test whether g (x) is positive or negative


by the following scheme:

33

Function

(i) If a > 0 and the discriminant (i.e.; D = b2 4ac) of


g (x) = ax2 + bx + c is < 0 (i.e., D = ve), then g (x) = ax2
+ bx + c is positive x and in such case, one must
consider only the function in numerator = f (x) > 0 to
find the solution set of y =

af
af

f x
> 0.
g x

Solved Examples
Find the domain of each of the following functions:

F 5x x I
GH 4 JK
2

1. y = log

F 5x x I > 0 5x
GH 4 JK
2

Solution: y is defined when

x2 > 0 x (5 x) > 0 (x 0) (x 5) < 0 0 < x

a f
D a y f = a0 , 5f
F xI
2. y = log H K
10
< 5 x 0, 5

af a

D y = 0, +

N.B.: In the above two examples the functions in


denominator are positive. This is why considerable
function to be greater than zero is only the function
in numerator.
3. y = log

Fx
GH x

2
2

I
J
+ 4 x + 6K
5x + 6

2
2

I >0
J
+ 4x + 6K
5x + 6

between 2 and 3 x < 2 or x > 3 x , 2

a3, + f .
D a y f = a , 2f a3 , + f

N.B.: In the above example (3), the discriminant D =


16 4 1 6 = 16 24 = ve for the function x2 + 4x +
6 in denominator which x2 + 4x + 6 > 0. For this
reason, we considered only the function x2 5x + 6 in
numerator > 0.
4. y = log

F x I > 0x>0
H 10 K

Fx
GH x

x2 5x + 6 > 0 x2 3x 2x + 6 > 0 x (x 3) 2
(x 3) > 0 (x 2) (x 3) > 0 x does not lie

F
GH x

I
J
10 x + 24 K
x5

Solution: Method (1)

Solution: y is defined when


x 0, +

Solution: y is defined when

is

defined

a x 5f > 0
a x 4f a x 6f

when

F
GH x

I=
J
10 x + 24 K
x5

(x 4) (x 5) (x 6) > 0

(multiplying both sides by (x 4)2 (x 6)2)


But (x 4) (x 5) (x 6) > 0 when (a) all the above
factors > 0 (b) one of the three factors > 0 and each of
the other two factors < 0.
Hence, we have the following four cases:
Case (i): When (x 4) > 0 and (x 5) > 0 and (x 6)
>0

a f

x > 4 and x > 5 and x > 5 x 6 ,


Case (ii): When (x 4) > 0 and (x 5) < 0 and (x 6)
<0
x > 4 and x < 5 and x < 6 4 < x < 5

a f

x 4,5
Case (iii): When (x 4) < 0 and (x 5) > 0 and (x 6)
<0
x < 4 and x > 5 and x < 6 5 < x < 6 x

34

How to Learn Calculus of One Variable

Case (iv): When (x 4) < 0 and (x 5) < 0 and (x 6)


>0
x < 4 and x > 5 and x > 6 x

af a f a f

D y = 4, 5 6,
Method (2)
x5
2

x 10 x + 24

>0

a x 5f > 0
a x 4f a x 6f

a fa f
D a y f = a4 , 5f a6 , f
2

f a x 6f 0

x 4, x 6
(ii)

x 10x + 24

af
af

f x
having the form y = log g x is defined when g (x)

af

> 0 and g x 1 which means that domain of y


consists of all those values of x for which g (x) > 0

af

and g x 1 i.e., D (y) = D + (g (x)) {roots of

af

g x = 1 }, where D + (g (x)) signifies the solution


set of g (x) > 0.

af

af

Note: log f x log 1 f x 1

(i) x 10 x + 24 0 x 4

x5

af
af

f x
log g x

working rule: one must remember that a function

x5
> 0 4 < x < 5 or x > 6
x4 x6

Method (3)
For y to be defined

Type 5: Problems based on finding the domain of a


function put in the form:

y=

If x < 4, then (x 4) < 0, (x 5) < 0, (x 6) < 0


If 4 < x < 5, then (x 4) > 0, (x 5) < 0, (x 6) < 0
If 5 < x < 6, then (x 4) > 0, (x 5) > 0, (x 6) < 0
and if x > 6, then (x 4) > 0, (x 5) > 0, (x 6) > 0
Hence,

this reason both the functions (x 5) and (x2 10x +


24) simultaneously are considerable.

Solved Examples
Find the domain of each of the following functions:

(A1)
(i) y =

>0

Solution: y is defined when (1 + x) > 0 and log (1 +

(a) x 5 > 0 and x2 10x + 24 > 0 or


(b) x 5 and x2 10x + 24 < 0
from (a), x 5 > 0 and (x 4) (x 6) > 0
x > 5 and (x < 4 or x > 6)
(x > 5 and x < 4) or (x > 5 and x > 6)
But x > 5 and x < 4 is not possible
x > 5 and x > 6 x > 6
from (b), x < 5 and (x 4) (x 6) < 0
x < 5 and (x > 4 and x < 6)
x < 5 and (4 < x < 6)
4 < x < 5 (A3)
Now, combining (A2) and (A3), we get

x) 0 . 1 + x > 0 x > 1 x 1, + .

log 1 + x = 0 log 1 + x log 1 ( log 1= 0)

1+ x 1

x0
(A2)

a f a f

x > 6 or 4 < x < 5 which x 4 , 5 6 ,

af a f a f

x
log 1 + x

af a

Type 6: Problems based on finding the domain of a


function put in the form: y = log log log f (x)
To remember: One must remember the following facts:
1. Inequalities of the form log a x > c, log a x < c, where
a > 0 and a 1 are called simplest logarithmic
inequalities.

D y = 4, 5 6,

N.B.: On must note that in the above example


discriminant of the function x2 10x + 24 in the
denominator is 102 4 1 24 = 100.96 = 4 = + ve. For

f kp

D y = 1 , + 0

LM RSa > 1
x>a
x>cM T
MMRS0 < a < 1
NT 0 < x < a
c

2. log a

Function

LM RS a > 1
0< x<a
x<c MT
MMRS0 < a < 1
NT x > a
c

3. log a

LM RS g a xf > 0
MML Tga afx>f >1 0
ax f MM RSTg fa xxf > g axf
MMMM 0 < f axf < 1
MNMNRST g a xf < g axf
1
2

af

Working rule: There are following working rules to


find the domain of a function put in the forms:

(i) y = log a log b f (x) a > 0 , a 1 , b > 1


(ii) y = log a log b log c f (x)

aa > 0 , a 1 , b > 1 , c > 0 , c 1f

Rule 1: log a log b f (x) exists


log b f (x) > 0
log b f (x) > log 1
f (x) > 1 and solve for x
Rule 2: log a log b log c f (x) exists
log b log c f (x) > 0
log b log c f (x) > log 1
log c f (x) > 1
f (x) > c1 if c > 1; f (x) < c if c < 1 and solve for x.
Aid to memory: To find the domain of a given function
put in the above mentioned form.
1. One must remove first log operator from left hand
side of the functions of the forms: y = log a log b log c
f (x) or log a log b f (x) and the rest log of a function
(i.e., log b log c f (x) or log b f (x)) should be put > 0.
2. Use the rules:

LM RS a > 1
x>a
x>cM T
0
MMRS < a < 1
NT0 < x < a
c

(a) log a

LMRS a > 1
0< x<a
x < c MT
MM RS0 < a < 1
NT x>a

(b) log a

4. log f a x f g1 x > log f a x f g 2

35

Solved Examples
Find the domain of each of the following functions:
1. y = log2 log3 (x 4)
Solution: y = log2 log3 (x 4) exists only for log 3 (x
4) > 0
(x 4) > 3
x4>1
x>5
D y = 5, +
2. y = log2 log3 log4 (x)
Solution: y = log2 log3 log4 (x) is defined when log3
log4 (x) > 0
log3 log4 (x) > log3 1
log4 x > 1
x > 41 x > 4

af a

af a

D y = 4, +
3. y = log10 [1 log10 (x2 5x + 16)]
Solution: y = log10 [1 log10 (x2 5x + 16)] is defined
when [1 log10 (x2 5x + 16)] > 0
log10 (x2 5x + 16) > 1
log10 (x2 5x + 16) < 1
x2 5x + 16 < 101 (3 loga x < c x < ac when
a > 1)
x2 5x + 16 10 < 0
x2 5x + 6 < 0
(x 2) (x 3) < 0

a f

2<x<3 x 2, 3
D (y) = (2, 3)

Domain of Trigonometric Functions


Question: Find the domain and range of the following
functions:
1. y = sin x 2. y = cos x 3. y = tan x
4. y = cot x 5. y = cosec x 6. y = sec x
Answer: 1. y = sin x

36

How to Learn Calculus of One Variable

Since we know that the value of the function y =


sin x is undefined and imaginary for no real value of x.
Hence, the domain of y = sin x is the set of all real
numbers. Again we know that | sin x | 1 1

R (tan x) = R = , = the set of all real numbers.


4. y = cot x
since, we know that the value of the function y =

sin x 1 for any real value of x which means that the


range of y = sin x is the closed interval [1, 1].

cot x =

Aid to memory:

D (sin x) = , + = R = the set of all real numbers.


R ( sin x) = [1, 1]
2. y = cos x
since we know that the value of the function y =
cos x is undefined and imaginary for no real value of
x. Hence, the domain of y = cos x is the set of all real
numbers, again we know that | cos x | 1 1

cos x 1 for any real value of x which means that the


range of y = cos x is the closed interval [1, 1].
Aid to memory:

cos x
is undefined for all those values of x
sin x

for which the function sin x in denominator is zero


(i.e.; for sin x = 0 x = n = any integral multiple
of , n being an integer). Hence, the domain of the
function y = cot x is the set of all real numbers
excluding n , n being an integer. Again, we know
that cot x can assume any value however large or
small for real value of x which means that
< tan < is the range of the function y = cot x.
Aid to memory:
D (cot x) = R {x: x = n , n being an integer}

R ( cot x) = R = , = the set of all real


numbers.

D (cos x) = , + = R = the set of all real numbers.


R (cos x) = [1, 1]

5. y = cosec x
Since we know that the value of the function y =

3. y = tan x
since we know that the value of the function y =

cosec x =

sin x
is undefined for those values of x for
cos x

1
is undefined for those values of x for
sin x

which the function cos x in denominator is zero (i.e.,


for those values of x for which the function cos x in
denominator is zero (i.e.; for cos x = 0 x =

which the function sin x in denominator is zero (i.e.,


for sin x = 0 x = n = any integral multiple of ,
n being an integer). Hence, the domain of y = cosec x
is the set of all real numbers excluding n , n being an
integer. Again we know that | cosec x | 1 cosec

a2n + 1f 2 = odd multiple of 2 , n being an integer).

x 1 or cosec x 1 , x n . Thus, x n ,
we have cosec x 1 or cosec x 1 . Hence, the

Hence, the domain of y = tan x is the set of all real

range of y = cosec x is the set of all real numbers not


in the open interval (1, 1).

tan x =

f 2 , n being an integer.

numbers excluding 2n + 1

Again, we know that tan x can assume any value


however large or small for real value of x which means
that < tan x < is the range of y = tan x.
Aid to memory:

D (tan x) = R = { x: x = a 2n + 1f , n being an integer}


2

Aid to memory:
D (cosec x) = R {x: x = n , n being an integer}
R (cosec x) = R (1, 1)
6. y = sec x
Since we know that the value of the function y =
sec x =

1
is undefined for those values of x for
cos x

which the function cos x in the denominator is zero

Function

f 2 , n being an

(i.e., for cos x = 0 x = 2n + 1

integer). Hence, the domain of y = sec x is the set of all

f 2 , n being an

real numbers excluding 2n + 1

integer. Again we know that | sec x | 1 sec x 1

or sec x 1, , x 2n + 1 , n being a integer.


2

f 2 , we have sec x 1 or sec x

Thus x 2n + 1

1. Therefore, the range of y = sec x is the set of all


real numbers not in the open interval (1, 1).
Aid to memory:

f 2 , n I } I = the set

D (sec x) = R {x: x = 2n + 1

of integers = { 0 , 1 , 2 , 3 , ...}

2. cos x 1

Find the domain of the following:


1. y = sin2 x + cos4 x
Solution: Since, domain of sin2 x = R = {x: x R }
and domain of cos4 x = R = {x: x R }

af

D y =RR=R
Type 2: Problems based on finding the domain of a
function put in the form: y = a sin x b cos x or, y =
a cos x b sin x, where a, b, x R .
Working rule: One must remember that the domain
of the function of the form: y = a sin x b cos x or y
= a cos x b sin x is the set of all real numbers since
a sin x or b cos x is defined for all a, b, x R as well
as a cos x or b cos x is defined for all a, b x R
which means that this sum and/difference is (or, are)
defined for all a, b, x R .

D y = R = , + = , +

Refresh your memory:


1. sin x 1

Solved Examples

af

R (sec x ) = R (1, 1)

37

1
1 cosec x 1
sin x

Solved Examples
Find the domain of each of the following functions:
1. y = sin x cos x
Solution: y = sin x cos x is defined for x R since

1
1 sec x 1
cos x

Now we consider different types or problems


whose domains are required to be determined.
Type 1: Problems based on finding the domain of a
function put in the form: y = sinn x coxm x (n and m
being integers) = sum or difference of power of sin x
and cos x;
Working rule: One must remember that domain of

the function y = sinn x cosm x is R = , =

sin x and cos x are defined for x R

af

D y = R = , +

2. y = 3 cos x + 4 sin x
Solution: y = 3 cos x + 4 sin x is defined for all x R
since 3 cos x and 4 sin x are defined for all, x R

af

D y = R = ,

since sin x and cos x are real valued

Type 3: Problems based on finding the domain of


trigonometric rational functions:

functions of the real variables x sinn x and cosm x


are real valued functions of the real variable x the
sum of sinn x and cosm x are real valued functions of
the real variable x.

Working rule: It consists of following steps:


Step 1: To put the functional value (or, simply
function) in denominator = 0 and to find the valued of
the independent variable.

, +

38

How to Learn Calculus of One Variable

Step 2: To delete the values of the independent


variable from R to get the required domain, i.e., domain
of trigonometric rational functions = R { real values
of the argument for which functional value in
denominator = 0}.

1. y =

Solution: Putting 1 + cos x = 0

2. sin = 0 = n , n I

f 2 , n I
sin = sin = n + a 1f , n I
cos = 0 = 2n + 1

af

D y =R
2. y =

la2n + 1f : n I q

1
2 cos 3 x

Solution: Putting 2 cos 3 x = 0


cos 3x = 2 which is not true for any real x

af

D y = R = , +

3. y =

1
2 sin 3 x

Solution: Putting 2 sin 3x


sin 3x = 2 which is not true for any real x

af

D y = R = ,

cos = cos = 2 n , n I

Domain of Inverse Trigonometric Functions

U|
cos = cos V = n
tan = tan |
|W

Before studying the method of finding the domain of


inverse trigonometric (or, arc) functions, we discuss
the domain on which each trigonometric functions is
reversible.

sin = sin

3.

cos x = 1 x = 2 n + 1

Notes: 1. If no real solution is available after putting


the functional value in denominator = 0, then domain
of trigonometric rational functions is R = , + .
We face this circumstances generally when we obtain
cos m x or sin mx = k; | k | > 1, (after putting the
functional value in denominator = 0) from which it is
not possible to find out the values of x since maximum
and/minimum value of sin m x and/cos m x = +1 and
1 respectively.

1
1 + cos x

integer)

(n being an

a f

4. cosn = 1 n , n being an integer.

sinn = 0 , n being an integer.


5. The domain of the function put in the form:

y=

af

f x ,

af
af

f x
,
g x

af
,
g a xf
g a xf
1

f x

LM
N

OP
Q


, , y 1,1
2 2
which signifies that the function y = sin x defined on
1.

y = sin x sin

LM
N

y = x , x

OP
Q


,
has an inverse function defined
2 2

the interval

on the interval [1, 1].


where f

Notes:

y = 1 , 1 , R sin

j LMN

y =


,
2 2

OP
Q

(x) and g (x) denote trigonometric functions, is


obtained by the same working rule as the case when
f (x) and g (x) are algebraic functions.

(i) D sin

Solved Examples


,
.
2 2
(iii) The notation of the inverse of the sine function
is sin1 (or, arc sin).

Find the domain of each of the following functions:

(ii) sin sin

LM
N

= x , x

y = y , y 1 ,1 and sin (sin x)

OP
Q

Function

LM
N

(iv) y = sin x, x

OP
Q


,
, is one-one and onto
2 2

functions. this is why it is possible to define its inverse


on the interval [1, 1].
2. y = cos x cos1 y = x, x 0 , , y 1 , 1
which signifies that the functions y = cos x is defined

39

(iii) The notation of the inverse of the tangent


function is tan1 (or, arc tan).

F
H

(iv) y = tan x , x

I
K


,
is a one-one and on-to
2 2

function. This is why it is possible to define its inverse


on the set of al real numbers (i.e.; R).

a f

on the interval 0 , has an inverse function x =


cos1 y defined on the interval [1, 1].

4. y = cot x cot y = x , x 0 , , y R which


signifies that the function y = cot x defined on the

Notes:

interval 0 , has an inverse function x = cot1 y


defined on the set of all real numbers.

(i) D cos

y = 1 , 1 , R cos
1

y = 0, .

(ii) cos cos y = y , y 1 , 1 and cos (cos x)


= x , x 0, .
(iii) The notation of the inverse of the cosine function
is cos1 (or, arc cos).
(iv) y = cos x , x 0 , , y 1 , 1 is a one-one
and on-to function. this is why it is possible to define
its inverse on the interval [1, 1].
3. y = tan x tan

F
H

y = x , x

I
K


,
, y R
2 2

a f

Notes:

(i) D cot

f e

y = R = , + , R cot

e yj = y , y R and cot
x , x a0 , f

(ii) cot cot

ja f
acot xf =

y = 0,
1

(iii) The notation of the inverse of the contangent


function is cot1 (or, arc cot).

a f

(iv) y = cot x , x 0 , , y R is a one-one and onto function. this is why it is possible to define its

inverse on the interval , .

RS UV , y R
T2 W

which signifies that the function y = tan x defined on

5. y = sec x sec y = x , x 0 ,

the interval

a 1 , 1f which signifies that the function y = sec x is


R U
reversible on the interval 0 , S V , that is, it has
T2 W

F , I
H 2 2K

has an inverse function x =

tan1 y defined on the set of all real numbers (i.e.; R)


Notes:

f e

(i) D tan y = R = , + , R tan y =

F , I .
H 2 2K
(ii) tan etan y j = y , y R
F I .
x , x ,
H 2 2K
1

an inverse function x = sec1 y defined on the interval


R = (1, 1).
Notes:

and tan

a tan x f =

f e

(i) D sec y = R 1 , 1 , R sec y = 0 ,

RS UV .
T2 W

(ii) The notation of the inverse of the secant function


is sec1 (or, arc sec).

40

How to Learn Calculus of One Variable

a f
asec xf = x , x 0 , RST 2 UVW .
RU
(iv) y = sec x , x 0 , S V , y R a 1 , 1f is
T2 W
e

(iii) sec sec y = y , y R 1 ,1 ;sec

a one-one and on-to function, this is why it is possible


to define its inverse in the interval R (1, 1).
6.

y = cosec x x = cosec

LM
N

y, x

OP
Q

,
2 2

k0p , y R a1 , 1f which signifies that the function


L O
y = cosec x is reversible on the interval M , P
N 2 2Q
k0p that is, it has an inverse function x = cosec y
1

defined on the interval R (1, 1).


1

f e

(i) D cosec y = R 1 , 1 , R cosec y =

LM , OP k0p
N 2 2Q
(ii) cosec ecosec y j = y , y R a 1 , 1f
L O
cosec a cosec x f = x , x M , P k0p .
N 2 2Q
1

af
af
f a x f 1 and f a x f 1 to form the

inequalities f x 1 and f x 1 .
2. To find the intersection of the solution set of the

inequalities
domain of the function of the form: y = sin1 (f (x)) or,
y = cos1 (f (x)).
Notes:

(i) sin x 0 2 n x 2n + 1 , n being an


integer.
(ii) sin x > 0 2n < x < 2n + , n being an
integer x lies in the first or second quadrant.
Solved Examples
Find the domain of each of the following functions:
1

FG 2 IJ
H 2 + sin x K

Solution: y is defined when 1

2
1
2 + sin x

2 sin x 2 2 + sin x
and

(iii) The notation of the inverse of the cosecant


function is cosec1 (or, arc cosec).
(iv)

af

inequality 1 f x 1 and to solve two

1. y = cos

Notes:

Working rule: It consists of following steps:


1. To put f (x) in between 1 and 1, i.e., to form the

L O
y = cosec x , x M , P k0p , y R a1,1f is
N 2 2Q

a one-one and on-to function. This is why it is possible


to define its inverse on the interval R (1, 1).
Now we discuss the method of finding the domain
of different types of problems.
Type1: Problems based on finding the domain of a
function put in the form: y = sin1 (f (x)) or, y = cos1
(f (x)).

2 sin x 2

...(i)

and 2 2 + sin x

...(ii)

(i) 2 sin x 2 sin x 4 sin x 4


which is true x R D1 = < x < = R .
(ii) 2 2 + sin x 0 sin x sin x 0

2n x 2n + 1 D2 = 2n , 2n + 1

af

D y = D1 D2

= 2n , 2n + 1 , n being an integer.
2. y = cos1 (2x + 3)
Solution: y is defined when 1 2 x + 3 1

1 2x + 3

...(i)

and 2 x + 3 1

...(ii)

Function

Now,(i) 2 x + 3 1 2 x 1 3 x

4
x 2 D1 = 2 ,
2

af
L1 O
= M , 2P
N2 Q

D y = D1 D2

1 2x + 3 1 3 2x 2 2x

(ii)

2
x 1 x x 1 D2 = , 1
2

af

41

Remember:
c

1. log a x > c x < a , if 0 < a < 1 .

D y = D1 D2

2. log a x > c x < a c , if a > 1 .

= 2 , 1

Type 2: Problems based on finding the domain of a


function put in the form:

af

y = sin

f x

or y = cos
D2

af

1. To put f x 0 and to solve for x.

1 1 2x

...(i)

and 1 2 x 1

...(ii)

Now, (i) 1 1 2 x 1 1 2 x

2 2 x x 1 D1 = , 1

(ii) 1x 1 2 x 0 x 0 D2 =

af

4. y =

2. To

{sin sin

a f}

1 and to solve for x.

o f a xf t

af

1 f x

af

3. (1) and (2) 0 f x 1 , domain of y is the


intersection of the solution set of (1) and (2).

af

af

f x is defined for f x 0

af

f x or cos

af

af

af

f x is defined

af

f x 1 and f x 0 0 f x 1 ,

for 0

Solution: y is defined when 1 log 2 x 1

f x

y = sin

(log2 x)

sin y 1 sin y 1

consider

Note:

D y = D1 D2
= [0, 1]
sin1

af

f x

Working rule: It consists of following steps:

3. y = cos1 (1 2x)
Solution: y is defined when 1 1 2 x 1

0,

i.e.; domain of the function put in the form

af

af

1 log 2 x

...(ii)

y = sin

f x

consists of all

and log 2 x 1

...(i)

those values of x for which

f x 0 and

Now, (i) 1 log 2 x 2

LM
N

I
K

1
x
2

1
1
D1 =
, .
2
2

af

Solved Examples
Find the domain of the following:

(ii) log 2 x 1 2 x x 2 D2 =

or cos

f x 1.

, 2

f x

1. y = sin

af

42

How to Learn Calculus of One Variable

< 2x <

Solution: Method (1)

x is defined for x 0

<x<
2
2
< x <

x 1 and x 0 .

y is defined for 0

af

D y = 0,1
D (y) = [0, 1]
Method (2)

af a

D y = , + = R

(1) Putting x 0 D1 = 0 ,
2

2. sin y 1 sin y 1 sin sin

Type 4: Problems based on finding the domain of a


function put in the form:
y = sec1 (f (x)) or y = cosec1 (f (x))

af

1 f x < +.

D y = D1 D2

af

2. To solve the inequalities < f x 1 and

= 0,1

af

1. To form the inequalities < f x 1 and

1 x 1 D2 = , 1

af

Working rule:

af

1 f x < + for x to form the domain of the


function of the form y = sec1 (f (x)) or y = cosec1
(f (x)).
Domain of a Function Put in the Form

D2

Type 3: Problems based on finding the domain of a


function put in the form: y = tan1 (f (x)) or y = cot1 (f
(x)).
Working rule: It consists of following steps:
1. To put f (x) in between and + i.e., to form

af

af

af

y = f1 x f 2 x

Working rule: It tells to find the domains of two


functions, say f1 (x) and f2 (x) separately whose
intersection is the domain of this sum or difference.

the inequality < f x < + .


2. To find the solution set of the inequality

Notes: By considering the two domains (i.e., intervals)


on the scale, we find their intersection (i.e., the interval
of common points).

< f x < + to form the domain of the


function of the form y = tan1 (f (x)) or y = cot1 (f (x)).

Solved Examples

Note: y = tan1 (f (x)) or y = cot1 (f (x)) defined for all

Find the domain of each of the following functions:

af

af

those real values of x for which < f x < +


i.e.; the domain of the function of the form y = tan1 (f
(x)) or y = cot1 (f (x)) consists of all those real values

af

of x for which < f x < + .

1. y = 1 x +

x3
2x + 1

af

Solution: Let f 1 x = 1 x

2
2

Solved Examples
Find the domain of the following:
1. y = tan1 (2x + 1)
Solution: 3 y is defined when < 2 x + 1 < +

3 f1 (x) is defined when 1 x 0


2

x 1 x 1 1 x 1

b a fg =

D f1 x

1 , 1 = D1 (say)

Function

af

Again, let f 2 x =

FG x 2 IJ 0
H x + 2K
x 2 and b x 2gb x + 2g 0
x 2 and a x 2f b x a2fg

x3
2x + 1

af

3 f 1 x is defined when

3 f2 (x) is defined when 2 x + 1 0

1
2

R 1U
D b f a x fg = R S V = D (say)
T 2W
F 1 I F 1 , I
= ,
H 2K H 2 K
1
1
Thus, D a y f = L1 , I F , 1O = D
MN 2 K H 2 PQ
2

2. y =

1
2

4x +

b a fg = a , 2f 2 , + f = D asayf

D f1 x

af

and f 2 x =

D2
+

x5

Solution: Let f 1 x =

af

0 x 2 or x < 2

43

4x

FG 1 x IJ 0
H1 + xK
x 1 and a1 x fa1 + x f 0
x 1 and a x 1f a x + 1f 0
a x 1f b x a1fg 0 1 < x 1 a3 x 1f
D b f a x fg = a1 , 1 = D asayf
Thus, D a y f = D D =
f2 (x) is defined when

x5

f1 (x) is defined when 4 x 0 x 4

b a fg a

D f 1 x = , 4 = D1

f2 (x) is defined when x 5 0 x 5

f a f
b a fg
Thus, D a y f = D D =

4. y =

D f 2 x = 5 , = D2 say
1

3. y =

1 x
1+ x

af

Solution: Let f 1 x =

af

and f 2 x =

1 x
1+ x

x2
x+2

af

Solution: Let f 1 x =
+

D2

x2
+
x+2

3 x + cos

af

and f 2 x = cos

FG x 2 IJ
H 3 K
3 x

FG x 2 IJ
H 3 K

3 f1 (x) is defined when 3 x 0 x 3

b a fg = a , 3 = D asayf

D f1 x

f2 (x) is defined when 1

x2
1
3

3 x 2 3
3 + 2 x 2 + 2 3 + 2
1 x 5

44

How to Learn Calculus of One Variable

b a fg =

a f

D f2 x

1 , 5 = D2 say

af

Thus, D y = D1 D2

= , 3 1 , 5
= [1, 3]

Domain of a Function Put in the Forms


(i) y = | f (x) |

af
f axf
a x f f a xf

af

af

or ,

f2

af

af

is the sum or difference of

af

f1 x
f2 x f3 x

af

af

or ,

1
f2 x f3 x

af

af

is

a rational function. Hence, its domain is dom f1 (x)


dom f2 (x) dom | f3 (x) | {x: f2 (x) | f3 (x) | = 0}.
Notes:
1. When the sum of two non-negative numbers (or,
functions) is zero, then both the numbers (or,
functions) are separately zero.
2

e.g: x + x = 0 x = 0 and x = 0; | x | +
x

= 0 | x| = 0

and | x3

f x

af
f a xf

af

> a a < f x < a

af

af

> a f x > a or f x < a

LMx 4 < 5 LMx < 1


N x4>5 Nx>9
x a , 1f a 9 , f

x R = 1 , 9

Solved Examples
Find the domain of each of the following functions:
1. y = | sin x |
Solution: 3 y = | sin x | is defined for every real value
of x (i.e.; for any value of x R )

af k

p a

af k

p a

D y = R x : x R = , +
2. y = 1 | x |
Solution: y = 1 | x | is defined for every real value of
x (i.e.; for any value of x R ).
D y = R x : x R = , +
2

3. y =

x +3
2

x + | x|

| = 0.

2. (x 1) (x 2) (x 3) (x 4) is > 0 for x > 4, or 2 < x


< 3 or x+< 1; and < 0 for 3 <+ x < 4 or1 < x < 2.+
1

7.

e.g.: x 4 > 5

two functions. Hence, its domain is the intersection


of domains of the functions f (x) and | g (x) |.
(ii) y =

x > a x < a or x > a x R a , a

1
x f3 x

Working rule: To find the domain of a function


involving absolute value function, one must remember
that:

af

6.

b, +

(i) y = f x g x

x < a a < x < a x a , a

9. x < a or x > b x R a , b x , a

f2

5.

8.

(ii) y = f x g x
(iii) y =

(+ always at right end and then alternatively +


and )
3. | x | = x for x > 0; | x | = x for x < 0 (where x = ve
x = + ve , i.e; x < 0 x > 0).
4. | f (x) | = f (x) for f (x) > 0; | f (x) | = f (x) for f (x) < 0
(where f (x) = ve f (x) = + ve, i.e; f (x) < 0 0
f (x) > 0).

Solution: y =

x2 + 3
x 2 + | x|

a rational function.

Putting x2 + | x | = 0
x2 = 0 and | x | = 0 and from each equation, we
get
x = 0 ( 3 x = 0 x = 0 and x = 0 x = 0)
2

Function

af

kp

D y =R 0
4. y =

x 1

...(ii)
x 1 x 1 , 1
on finding the intersection of (i) and (ii), it is

1
x x

f a

obtained D (y) = 1 , 0 0 , 1 .

1
Solution: y =
a rational function
x x

Problems on the Range of a Function

Putting x | x | = 0
|x|=x

As discussed earlier, the range of a function defined


by y = f (x) in its domain is the set of values of f (x)
which it attains at points belonging to the domain.
For a real function, the codomain is always a subset
of R, so the range of a real function f is the set of all
points y such that y = f (x), where x D f = domain

x 0 ( x = x provided x > 0)

af

D y = R x: x 0

af

Domain of a Function
Containing Greatest Integer Function
Rule: Domains of functions involving greatest integer
function are obtained by using different properties of
greatest integer function.
Solved Examples
1. Find the domain of y = sin1 [x]
Solution: y = sin1 [x] is defined when 1 < [x] < 1
Now 1< [x] < 1

1 x < 2
x 1 , 2

D (y) = [1, 2]
2. Find the domain of y = sin1 [2 3x2].
Solution: y = sin 1 [2 3x 2 ] is defined when
1 2 3 x

k
,k0
x
4. y = x2n
5. y = x2n + 1

x > 0 x R

Next, 2 3x 1
3x 3

Funtion defined
by an expression

3. y =

1 2 3x < 2
Again, 2 3 x2 < 2
3x2 < 0
x2 < 0

of f.
In general, a function is described either by a single
expression in x in its domain or by various expressions
defined in adjacent intervals denoting different parts
of the domain of the function and neither its domain
nor range is mentioned. In such cases, it is required
to be found out the domain and the range of the given
function.
Already, how to find out the domains of different
types of functions has been discussed. Now the
methods of finding the range of a given function will
be explained.
Firstly, domains and range sets of standard
functions will be put in a tabular form.

1. y = kx, k 0
2. y = kx + l

Now 1 2 3 x

45

6. y =

7. y = ax2 + bx + c,

...(i)

Domain

Range

(, )
(, )

(, )
(, )

R {0}

R {0}

(, )
(, )

(0, )
(, )

[0, )

[0, )

(, )

LM D , + IJ ,
N 4a K

D = b2 4ac

a>0
8. y = ax2 + bx + c,
a<0

(, )

FG , D IJ ,
H 4a K
D = b2 4ac

46

How to Learn Calculus of One Variable

Now the methods to find the range of a given


function are provided, when its domain is an infinite
interval.
How to Find the Range of Function
Step 1: Put y = f (x)
Step 2: Solve the equation y = f (x) for x to obtain x =
g (y).
Step 3: Find the values of y for which the values of x
obtained from x = g (y) are in the domain of f, i.e. find
the domain of g (y) in the same way as the domain of
f (x) is obtained considering g (y) as inverse of f (x).
Step 4: The set of all values of y obtained in step (3)
is the required range, i.e. the domain of g (y) is the
required range of the given function y = f (x).
Remarks:
1. The method mentioned above is fruitful only when
the domain of a given function is infinite, i.e. the
domain of a given functions is not a closed interval
[a, b], a , b R = the set of reals.
2. When a function defined by a single formula y = f
(x) does not become imaginary or undefined for any
value of independent variable x, the domain of the
function y = f (x) is the set of all real numbers denoted
by R. To obtain its range, one should consider the
domain < x < using the axioms of inequality
in < x < .
3. When it is possible to put a function in the form of
Px2 + Qx + R, where P, Q and R are linear expressions
in y, one should use the rule of discriminat, i.e.,
2

D = b 4 ac 0 for real x.
4. In case the domain of a function y = f (x) is a finite
set D = {a1, a2, a3, , an}, then its range is obtained
by forming the set whose members are the values of [f
(x)] x = a1, a2, a3, , an.

Type 1: Functions put in the forms: (i) y = ax + b (ii) y


= ax2 + bx + c whose domains are not given.
Rule: When the domain of a function is not given
and the question says to determine the range of a
functions, one is required to find out its domain at
first in the following way:
It should be checked whether the given function
becomes imaginary or undefined for any value of

x R i.e. given function y = f (x) does not become


imaginary or undefined for any value of x R
domain of the given function is the set of all real
numbers denoted by R.
Lastly, one should find the range of the given
functions using the axioms of inequality in
< x < .
Example worked out:
1. Find the domain and range of each of the following
functions:
(i) y = x (ii) y = x + 2
Solution: (i) y = x does not become imaginary or
undefined for any x R y = x is defined for all

x R .

af

D y = R < x < < y <


(3 y = x is given)
R y = R = the set of reals.

af

(ii) y = x + 2 does not become imaginary or undefined


for and x R y = x + 2 is defined for all x R
D

(y)

Now,

af

D y = R < x <

< x + 2 < (on using the axiom of


inequality) < y < R ( y) = R = the set of
reals.
Note:
In case one is required to find out the range of linear
function y = ax + b whose domain is a given subset of
the set of reals namely R, the range of y = ax + b is
obtained with the help of given domain and the use
of various axioms of inequality.
Examples: (i) Find the range of f (x) = 4x 5 for
6 x 3 .
Solution: 6 x 3

24 4 x 12
24 5 4 x 5 12 5

29 4 x 5 7

af
R a f f = k y : 29 y 7p where y = f (x)

f x 29 , 7

Function

Similarly, the range of each of the following


functions:
(ii) f 1 (x) = 2x + 3 for 1 x 7 is

k y : 1 y 17p , where y = f (x).


1

k y : 19 y
2

3 x 4

is

23 , where y2 = g (x).

2 x 5

(iv) h (x) = 5x 6 for

k y : 16 y 19p , where y = h (x).


3

Solved Examples
1. Find the range of the following functions:
(i) y =

x (ii) y =

(iii) g (x) = 5 6x for

is

2. Find the domain and range of each of the following


functions:
(i) y = x2 (ii) y = x2 4.
Solution: (i) y = x2 does not become imaginary or

Solution: (i) y =
Again y =

0, .
(ii) y = x2 4 does not become imaginary or undefined
for any x R y is defined for all

af

x R D y = R .
Now, y = x2 4
x2 4 y = 0
x2 (y + 4) = 0

but x 0
2

y 4 ,

R y = 4 ,

la

fq = 4 a4 + yf 0

x ).

a f
R a yf = 0 , f
y 0,

(ii) y =

x3

Again y =

x3

y = x 3, 3 y 0
y +3= x

but x 3

(ii) y =

y +33 3x = y +3

Type 2: Functions put in the forms:

af

(since y =

y 4

f x

y 0 if y is non-negative which is given

x3

y+40

(i) y =

x 3 0

D = 0 4 1 y + 4

af

y = x, 3 y 0

y 0

a f
e
y 0 y 0 , f R a yf =

x0

x R D (y) = R Now, y = x 2 x 2 y = 0
D = 0 4 1 y = 4 y 0 3 D = b 4 ac

3 2x

x+2

y 0

x 3 (iii) y =

(iv) y =

undefined for any x R y = x is defined for all

y 0

f x

y 0

af

Rule: Find the domain of y and then express x in


terms of y. Lastly put g (y) in the domain of y and
solve it to find the range of y.

47

y 0 if y is non-negative which is given.

af

y 0, R y = 0,

48

How to Learn Calculus of One Variable

(iii) y =

3 2x

x=

3 2x 0

1
y

2x 3

But x > 2

2x 3

3
2

Again y =

3 2x

y2 = 3 2x, 3 y 0
y2 3 = 2x
3 y2 = 2x

2 > 2

>0

y > 0 | y| > 0

y > 0 if y is non-negative which is given

a f

af a f

y 0, R y = 0,

F3 y I = x
G
H 2 JK
2

but x

Type 3: Functions put in the forms:


(i) y =

3
2

C
ax + b
(ii) y =
Ax + B
Ax + B

Rule: Express x in terms of y by cross multiplication


and simplification. Lastly use the rule:
f y
R y = R roots of deminator of 1
=0
f2 y

F3 y I 3
G
H 2 JK 2
2

RS
T

af

3 y 3

a f UV
af W

Solved Examples

y 0

1. Find the range of the following functions:

y 0

(i) y =

1
x
1
(ii) y =
(iii) y =
x
x+2
x 1

(iv) y =

x 1
x
(v) y =
x+3
5 x

y 0 if y is non-negative which is given.

af

y 0, R y = 0,

(iv) y =

Solutions:
1
(i) y =
x

x+2

x + 2 > 0 x > 2
Again y =
2

y =

x+2

1
y

1
,y0
y
R (y) = R {0}
x=

1
, 3y0
x+2

x+2=

(ii) y =

1
x1

x 1=

1
,y0
y

Function

R (y) = R {0}
x
, x 2
(iii) y =
x+2

yx + 2y = x
2y = x xy = x (1 y)

y=

ax + bx + c
2

Ax + Bx + C

provided its numerator and

denominator do not have one common factor.


Solved Examples

2y
x=
,y 1
1 y

(1) Find the range of the following functions:

R (y) = R {1}

(i) y =

x
,x5
(iv) y =
5 x
5y xy = x
5y = x + xy = x (1 + y)

1
2

x 4
x

(ii) y =

1+ x

(iii) y =

R (y) = R {1}

(iv) y =

x 1
, x 3
x+3

xy + 3y = x 1
3y + 1 = x xy = x (1 y)
x=

3y + 1
,y 1
1 y

R (y) = R {1}
Type 4: Functions put in the forms:
(i) y =

D
2

Ax + Bx + C

(ii) y =

ax + b
2

Ax + Bx + C

(iii) y =

Ax + Bx + C

Rule: Cross multiply and obtain Px2 + Qx + R = 0


where P, Q and R are functions of Y (i.e. an expression
in y). Lastly use the rule D > 0, where D = b2 4ac.
Remark: The above rule is valid in

x 2x + 4
2

x + 2x + 4

1
2

x 3x + 2

Solutions:
(i) y =

1
2

x 4

, x 2

x2 y 4y = 1
x=

y 1 + 4y
y

f,y0

Now, D > 0
y (1 + 4y) > 0

1
or y 0
4

1
or y > 0 since y 0
4

a f FH

ax + bx + c

5y
x=
, y 1
1+ y

(v) y =

49

R y = ,
(ii) y =

x
1+ x

y + yx2 = x

OP a f
Q

1
0,
4

50

How to Learn Calculus of One Variable

yx2 x + y = 0
x=

b1g

1
y3
3
Also y = 1 x = 0

4 y y
2y

,y0

a f LMN 13 , 3OPQ

R y =

1 1 4y
,y0
2y
Now, D > 0
2

1 4y 0

a fa f
a2 y + 1f a2 y 1f 0

f,y0

y+2 2

x + 2x + 4

either y = 2 2 or y + 2 2

either y 4 or y 0 but y 0

x2 y x2 + 2xy + 2x + 4y 4 = 0
(y 1) x2 + 2 (y + 1) x + (4y 4) = 0
+ 4) = x2 2x + 4

a f a fa
2 a y 1f

af a

y 1
Now, D > 0

a f

R y = , 4 0 ,

f,

2 y + 1 4 y + 1 4 y 1 4 y 4

Type 5: Functions put in the forms:


(i) y =

ax + b
2

Ax + Bx + C
2

a f 4 a y 1fa4 y 4f 0
4 a y + 1f 16 a y 1f 0
a y + 1f 4 a y 1f 0
l y + 1 + 2 a y 1fq l y + 1 2 a y 1fq 0
a y + 1 + 2 y 2 f a y + 1 2 y + 2f 0
a3 y 1f a3 y f 0
a3 y 1f a y 3f 0
2

4 y 2 y 1

y + 4y 0

4 y+1

2y

x 2x + 4

a f

a f a3yf

3 y

9 y 8y + 4y 0

x =

, x 1, 2

Now, D > 0

L 1 1O
Also, y = 0 x = 0. Hence, R (y) = M , P
N 2 2Q
y (x2 + 2x

x 3x + 2

x=

1
1
y ,y0
2
2

(iii) y =

yx2 3yx + 2y 1 = 0
yx2 3yx + (2y 1) = 0

1 + 2y 1 2y 0

(iv) y =

(ii) y =

Ax + Bx + C
ax + b
2

(iii) y =

ax + bx + c
2

Ax + Bx + C
n

(iv) y =

x a
xa

whose numerator and denomina-

tor contain a common factor.


Rule: Cancell the common factor present in numerator
and denominator. After cancellation of common factor,

Function

(i) If y = mx + c (linear in x), then R (y) = R { value of


y = mx + c at the zero of common factor}, where R =
the set of reals.
(ii) If y = a1 x2 + b1 x + c1 (quadratic in x), then R (y)
= range of y = a1 x2 + b1 x + c {value of y = a1 x2 + b1
x + c1 at the zero of the common factor}

a1 x + b1
(linear rational in x), then R (y)
a2 x + b2

(iii) If y =

af
af

f1 y
= R { zero of the denominator of x =
and
f2 y
a x + b1
at the zero of common
the value of y = 1
a2 x + b2
factor}.

x2 + 2x (y 4) = 0

4+4 y4

2
Now, D > 0

l a

4 1+ y 4

fq =

fq 0

y 30
y3

af

R y = 3, since (x2 + 2x + 4) for x = 2 = 4 +


4 + 4 = 12 and

Since 12 4 =

1. Find the range of the following functions:


x 1
x1

af

(iii) y =

x 3x + 2
2

x + x6
2

fa

x 1 x+1
x 1
=
,x1
Solutions: (i) y =
1

x
x 1

y = x + 1, x 1
x=y1

kp

x is defined for all y R 2


R (y) = R {2}
since x + 1 = 2 for x = 1 and y = 2 x = 1
x 8 a x 2f e x + 2 x + 4j
y=
=
,x2
x2
a x 2f
3

+ 2x + 4

x2 + 2x y + 4 = 0
y = x2

+ 2x

x2

+ 2x 8 = 0 x =

a f = 2 6 x = 2 , 4
2

x 8
(ii) y =
x2

(iii) y =

x2

2 4 4 1 8

(ii)

la

441 y 4

y = 12 gives a point x = 4 D f

Solved Examples

(i) y =

x=

51

x 3x + 2

x + x6

a
a

x 2x x + 2

f a
f a

x + 3x 2 x 6

f a
f a

fa
fa

f
f

x x2 x2
x 1 x 2
=
x x + 3 2 x + 3
x2 x+3

yx + 3y = x 1, x 2
3y + 1 = x xy = x (1 y)
x=

Again,

3y + 1
,y 1
1 y

FG x 1 IJ
H x + 3K
af

=
x=2

1
1
and x = 2 for y = 1, or y =
5
5

RS 1 UV
T 5W

Hence, R y = R 1 ,

(iv) Find the domain and range of the function defined


as

e x + 3 x 4j e x 9 j
y=
e x + x 12j a x + 3f
2

52

How to Learn Calculus of One Variable

Solution: y =

a x + 4f a x 1f a x 3f a x + 3f
a x + 4f a x 3f a x + 3f

= (x 1) for x 4 , 3 , 3
One should note that denominator is zero for x =
4, 3 or +3. This means that y is undefined for these
three values of x. for values of x 4 , 3 or 3, one
may divide numerator and denominator by common
factors and obtain y = (x 1) if x 4 , 3 or 3.
Therefore, the domain of y is the set of all real numbers
except 4, 3 and +3, i.e. D (y) = R {4, 3, +3} and
the range of y is the set of all real numbers except
those values of y = (x 1) obtained by replacing x by
4, 3 or 3, i.e. all real numbers except 5, 4 and 2, i.e.
R (y) = R {5, 4, 2}.
Type 6: Functions put in the form: y = log f (x).

af

af

Rule: y = log a f x a = f x > 0


i.e. change the given logarithmic form into the
exponential form and then solve it using the in

equation: a > b a > 0 , a 1


(i) y > loga b for a > 1, b > 0
(ii) y < loga b for 0 < a < 1, b > 0
(iii) y R for a > 0, b < 0.

af

11
2
=3 x
3
3

11
3

y log

I
K

F 11I
H 3K

LM 11 , I
N 3 K
L 11 , I
R a y f = Mlog
N 3 K
y log

2. Find the range of the function

y = log 2

x4 +

Solution: y = log 2
y

2 =

2y

6 x

x4 +

x4 +

6 x

6 x

a x 4 f a6 x f

=x4+6x+2

2y

2 =2

a x 4f a 6 x f 0

y > g x ...(A)(say)
Remark: For, a > 1 a > a
i.e. when it is possible to change ay + f (x) > 0 into
the form ay > ag (x), one should use (A).

2y

20

2y

2 2y 1 y

Solved Examples

Again from (ii) 2 2 y 1 1 =

Find the range of the function y = log (3x2 4x + 5).

1.
Solution: y = log (3x2 4x + 5) where 3x2 4x + 5 > 0

F
H

5
3

I
K

4
5
e = 3x 4 x + 5 = 3 x x +
=
3
3
y

|RS
|T

3 x

|UV
|W
R|F 2 I + 11U|V = 3F x 2 I
= 3S x
|TH 3K 9 |W H 3 K
F I F 4I
H K H 6K

4
4
x+
3
6

...(i) (on squaring)

af

g x

F
H

2 y 1

j = e x

x 10 x + 24 + 2

1
2
2

x + 10 x 24

+ 10 x 24

2 y 1

= 0 , which is

a quadratic in x and whose D = b2 4ac is

a10f

11
3

RS
T

4 24 + 2
2 y 1

2 y 1

j UVW 0 which

...(ii)

Function

2y

Note: Range of a cos x + b sin x

LM
N

1 1

1 2

11

OP
Q

a = 1, b = 1

2 y 1

R y = 2 , 2 in the above question.

2 y 1

Solution: y = cos +

a f LMN 12 , 1OPQ

3 sin

Again it is known that

Type 7: Functions put in the form: y = a cos x + b sin


x.

F
H

1 sin

Rule: The range of y = a cos x + b sin x is


2

a +b , a +b

OP
Q

I
K

1
6

F
H

i.e. 2 2 sin

I
K

2
6

Hence, R (y) = [2, 2]

Solved Examples

Note: In this question, a = 1, b =

1. Find the range of y = sin x cos x


Solution: y = sin x cos x

hence, range of cos +

LM 1 sin x 1 cos xOP


2
N2
Q
L F I sin x sin F I cos xOP
2 Mcos
H 4K Q
N H 4K
F I
2 sin x
H 4K

= 2

Again, it is know that

F
H

1 sin x

3 sin

LM 1 cos + 3 sin OP
2
N2
Q
L F I cos + cos F I sin OP
= 2 Msin
H 6K Q
N H 6K

R y =

=2

1
y1
2

2. Find the range of y = cos +

1 2y 2

a +b

af

0 2y 1 2

1 1 (from (ii))

2 2

LM
N

= a +b , a +b

2 y 1

02
1 2

2 y 1

53

I
K

1
4

i.e. 2 2 sin F x I 2
H 4K
Hence, range of y = R a y f = 2 ,

3 1+ 3 =

3 sin = 2 , 2 on using,

range of a a cos + b sin =

LM
N

a +b , a +b

OP .
Q

Type 8: Functions put in the forms:


(i) y = a b sin x (ii) y = a b cos x
(iii) y =

C
C
(iv) y =
, where a,
a b sin x
a b cos x

b and c are constants.


Rule: Start from sin x 1 or cos x 1 and form:
(i) k a b sin x L

(ii) k a b cos x L

54

How to Learn Calculus of One Variable

(iii) k

C
L
a b sin x

Rule: Range of those functions containing greatest


integer function is obtained by using different
properties of greatest integer function.

(iv) k

C
L where k and L are cona b cos x

Solved Examples

stants, by using the axioms of inequality.

1. Find the range of y = [cos x]


Solution: 1 cos x 1

Solved Examples
1. Find the range of y = 2 + sin x.
Solution: 1 sin x 1

2 1 2 + sin x 1 + 2
1 2 sin x 3
1 y 3

af

R y = 1 , 0 , 1
2. Find the range of y = 1 + x [x 2]
Solution: On using the property:

t t < t + 1, it is seen that

y 1, 3

x2 x2< x 2 +1

af

R y = 1, 3

2. Find the range of y =

y = cos x

R|1, for 1 cos x < 0


= S 0 , for 0 cos x < 1
|T 1 , for cos x = 1

x2 x2 x2 x2 <

1
2 sin 3x

Solution: 1 sin 3x 1

1 sin 3x 1

x 2 x 2 +1
0 x 2 x 2 <1
3 x 2 + 3 x 2 < 1 + 3 (adding 3 to
each side)

2 1 2 sin 3x 1 + 2

3 x +1 x 2 < 4

1 2 sin 3x 3

3 f x <4
R (y) = [3, 4)

1
1

2 sin 3x 3

1
1

1
3 2 sin 3x

1
y 1
3

LM1 , 1OP
N3 Q
L1 O
R a y f = M , 1P
N3 Q
y

Type 9: Finding the range of a function containing


greatest integer function.

af

Type 10: Finding the domain and range of a piecewise


defined functions.
1. y = f1 (x), when x < a
= f2 (x), when x > a
i.e. two or more functions of an independent
variable namely x defined in adjacent intervals.
2. y = c1, when x < a
= c2, when a < x < b
= c3, when b < x
i.e. two or more different functions defined in
adjacent intervals.
3. y = f1 (x), when x a
= f2 (x), when x = a

Function

Now, it will be discussed in detail how to find the


domain and range of each type of piecewise function.
Rule: The domain of each type of a piecewise
function is the union of each given interval whereas
the range of each type of a piecewise function is the
union of different range of each given function
determined by considering each different given
intervals and applying the axioms of inequality to
obtain the different given functions in the form of
inequalities.
Note: The range of a piecewise function put in the
form:
y = c1, when x < a
= c2, when a < x < b
= c3, when b < x
i.e. the range of a piecewise function defined by
different constants in adjacent intervals is the set
whose members are given constants (constant
functions) defined in given adjacent intervals, i.e. R
(y) = {c1, c2, c3}, where c1, c2 and c3 are different
constants defined in adjacent intervals.

2. Find the domain and range of the function defined


by

RS x 1 , if x < 3
T2 x + 1 , if 3 x
Solution: x < 3 x a , 3f
x 3 x 3 , f
D a y f = a , 3f 3 , f = a , f
y=

Again, x < 3 x 1 < 3 1 x 1 < 2 ...(i)


Also, x 3 2 x 6 2 x + 1 7

y=

RSx + 3 , when x 3
T 2, when x = 3

Solution: x 3

1. Find the domain and range of the function defined


by

x , 3 3 , = R 3
Next, y = 2, when x = 3

Again, x < 1
3x < 3
3x 2 < 3 2
...(i)
3x 2 < 1
Also, x > 1
...(ii)
x2 > 1
Hence, from (i) and (ii), it is concluded that

f a

R (y) = , 1 1 , = ,

3. Find the domain and range of the function defined


as

x > 3 or x < 3

RS3x 2 , if x < 1
T x , if 1 x
Solution: x < 1 x a , 1f
x 1 x 1, f
D a y f = a , 1f 1 , f = a , f

...(ii)

Hence, (i) and (ii) R (y) = , 2 7 , =


R [2, 7) i.e. all real numbers not in [2, 7).

Solved Examples

y=

55

f a f

af

kp

kp kp

D y = R 3 3 = R

Now, x 3

x +36

lq

R y| x 3 = R 6
Also, y = 2, when x = 3

g lq
R a y f = R k6p k2p = R k6p
R y| x = 3 = 2

i.e. the range of the given function consists of all


real numbers except y = 6.
4. Let there be a function defined as

R|x , if x 2
S| 7 , if x = 2
T
2

y=

find its domain and range.


Solution: x 2

56

How to Learn Calculus of One Variable

x < 2 or x > 2

f a f

kp

af

kp kp

x , 2 2 , = R 2
Next, y = 7, for x = 2

D y = R 2 2 = R
Now, x 2
2

x 4

g
k4p k0p ,3 x

R y| x 2
+

=R
Also, y = 7, for x = 2

g lq
R a y f = R k4p k7p k0p
= R k4p k0p
R y| x = 2 = 7
+

i.e. the range consists of all non-negative real


numbers except y = 4.
Note: One should note that y = c, for x = a, where c
and a are constants, represents a point P (a, c), i.e. a
point whose abscissa is a and whose ordinate is c.
5. If the domain of a function is A = {x: x R , -1 < x
< 1} and the function is defined as

R| 1, when x > 0
f a x f = S 0 , when x = 0
|T1, when x < 0

Find the range of f (x).


Solution: The range of a piecewise function whose
each function is constant defined in its domain is the
union of different constants.
Therefore, R (y) = [1, 0, 1}
Type 11: A function y = f (x) whose domain s a finite
set.
Rule: If the domain D of y = f (x) is a finite set, i.e. D =
{ a1, a2, a3,, an}, then its range R (f) = {f (a1), f (a2),
f (a3), , f (an)}.
Solved Examples
1. If A = {1, 2, 3, 4, 5}, B = {a, b, c, d, e} and f = {(1, b),
(2, d), (3, a), (4, b), (5, c)} be a mapping from A to B,
find f (A).

Solution: f (A) = {f (1), f (2), f (3), f (4), f (5)}


= (b, d, a, c}
2. If A = {0, 1, 1, 2} and f : A R is defined by f
(x) = x2 + 1, find the range of f.
Solution: f (x) = x2 + 1
f (0) = 1
f (1) = 2
f (1) = 2
f (2) = 5
f (A) = {f (0), f (1), f (1), f (2)}
= (1, 2, 5}
3. If A = {0, 1, 2, 3} and f (x) = 3x 5 is a function
from A on to B, find B.
Solution: f (x) = 3x 5
f (0) = 5
f (1) = 2
f (2) = 1
f (3) = 14
f (A) = B = {1, 2, 5, 14}
4. If A = {1, 2, 3, 4} and f (x) = x2 + x 1 is a function
from A on to B, find B.
Solution: f (x) = x2 + x 1
f (1) = 1
f (2) = 5
f (3) = 11
f (4) = 19
f (A) = B = {1, 5, 11, 19}
Type 12: A function y = f (x) defined in an open
interval (a, b).
Rule 1: a < x < b
increasing in (a, b).
Rule 2: a < x < b
increasing in [a, b].
Rule 3: a < x < b
decreasing in (a, b).
Rule 4: a < x < b
decreasing in [a, b].

f (a) < f (x) < f (b) if f (x) is


f (a) < f (x) < f (b) if f (x) is
f (b) < f (x) < f (a) if f (x) is
f (b) < f (x) < f (a) if f (x) is

Notes:
1. When y = f (x) is an increasing function in the
open interval (a, b) or in the closed interval [a, b],
then f (a) = L (say) is least and f (b) = G (say) is
greatest value of the given function y = f (x) in (a, b)
or [a, b].

57

Function

2. When y = f (x) is a decreasing function in the open


interval (a, b) or in the closed interval [a, b], then f (a)
= G (say) is greatest and f (b) = L (say) is least value of
the function y = f (x) in (a, b) or [a, b].
3. Range of y = f (x) = R (f) = (least f (x), greatest f (x))
if the domain of f (x) is an open interval (a, b) where f
(x) is continuous and increasing or decreasing for
x D f = a,b
In the same fashion, R (f) = [least f (x), greatest f
(x)] if the domain of f (x) is a closed interval [a, b]
where f (x) is continuous, and increasing or decreasing

Solution: f (x) = cos x x (1 + x), is decreasing in

for x D f = a , b .
4. In case a function y = f (x) is defined in its domain
is neither increasing nor decreasing but it is
continuous in its domain then its range is also
determined by the rule of finding greatest and least
value of the given function f (x) which will be explained
in the chapter namely maxima and minima of a function.

LM0 , OP and so in LM , OP
N 2Q
N6 3Q

F I f a xf f F I
H 3K
H 6K
F I = cos F I F1 + I = 1
Now, f
H 3K H 3K 3 H 3K 2 3
f

af a f
af

Solved Examples
1. Find the range of the function f (x) = x3 whose

domain is D = x : x R , 2 < x < 2 .


Solution: f (x) = x3 is increasing in (2, 2)
f (2) = (2)3 = 8
and f (2) = (2)3 = 8
Therefore, R (y) = (f (2), f (2) = (8, 8)

F
H

I
K
F I
Solution: y = tan x is increasing in H , K <
2 2
2

2. If y = tan x defined in

x<


,
, find its range.
2 2

< tan x < since


2

L
x

tan x =

and L tan x = .
x

FI
H 2K

RS
T

3. If A = x :
x), find f (A).

x
6
3

UV and f (x) = cos x x (1 +


W

F I = cos F I F1+ I = 3
H 6 K H 6 K 6 H 6 K 2 6 36
L 1 , 3 OP
f b Ag = M
MN 2 3 9 2 6 36 PQ
2

Note: If y = f (x) is a continuous function whose


domain D = [a, b] = [a, c) [c, b] where a < c < b and
f (x) increasing in (a, c) and decreasing in [c, b), then
to find its range R (f), one is required to find out f (a),
f (b), f (c) and
R (y) = (greatest f (x), least f (x))
In the same way, R (y) = [ greatest f (x), least f (x)] if
y = f (x) is defined in the domain D = [a, b] = [a, c)
[c, b] such that f (x) is decreasing in [a, c) and
increasing in [c, b].
Solved Examples
1. Find the range of the function f (x) = x2 + 1 in the
domain (5, 2).
Solution: f (x) = x2 + 1 with its domain (5, 2) and f (x)
is decreasing in (5, 0) and increasing in [0, 2).
f (5) = 26
f (0) = 1
f (2) = 5
Therefore, it is clear that f (x) lies between 1 and 26.
R (f) = (1, 26)
2. Find the range of the function f (x) = x2 in (2, 2).

58

How to Learn Calculus of One Variable

Solution: f (x) = x2 with its domain (2, 2) f (x) is


decreasing in (2, 0) and increasing in [0, 2).
f (2) = 4
f (0) = 0
f (2) = 4
Therefore, it is seen that f (x) between 0 and 4.
R (f) = (0, 4)
Composite Function
Definition: If given functions are f : D1 C1 and

a f

g : D2 C2 such that f D1 D2 , then the


composite function gof is the function from D1 to D2

defined by (gof) (x) = gof (x) = g (f (x)), x D1 and

af

f x D2 .

a f

N.B.: (i) f D1 D2 means that range of f


domain of g.

af

(ii) Dgof = x : x D f and f x Dg

s where gof

represents the composite of f and g defined by (gof)


(x) = g (f (x)) having the domain Dgof = D (gof).
(iii) A function does not exist whenever its domain
is an empty set.
f (D1) < D2
C2

C2

Solved Examples
1. If f (x) = x2 and g (x) = x + 1, find (gof) (x).
Solution: On applying the definition,
(gof) (x) = gof (x)

e j
= g a yf ]
= a y + 1f
2

y= x

f (x)
gof (x) = g (f (x))

Note: Very often the law establishing the relationship


between the independent variable and dependent
variable is specified by means of a formula. This
method of representation of function is called
analytical. Further, the expression in x is called analytic
expression.

= g x

D2

D1

Working rule: The rule to compute gof (x) for two


analytic expressions in x for f (x) and g (x) says.
1. Firstly to replace f (x) by its given analytic
expression in x.
2. Secondly, in the given analytic expression in x for
g (x), to replace each x by the function f (x) and then
to put f (x) = analytic expression in x for f (x).
Thus, gof (x)
= value of g at f (x)
= [analytic expression for g (y)]y = f (x) ,which
signifies that the independent variable x in the analytic
expression for g (x) should be replaced by the analytic
expression in x for f (x) whereas the constant in the
analytic expression in x for g (x) remains unchanged.

gof

Remember: One must remember that g (f (x)) = gof (x)


means that g is a function of f (x) which is itself a
function of x. This is why gof (x) = g (f (x)) is called a
function of a function of the independent variable x.
Further, one should note that gof (x) signifies the
value of the function g at f (x) = given analytic
expression in x (or, simply given expression in x).
Type 1: Formation of composite of two functions of
x whose analytic expressions in x are given.

y=x

= x +1

af

2. If f (x) = x + 3 and g x =
Solution: (gof) (7) = gof (7)
= g (3 + 7)
= g (10)
=

e xj

10

= 10

af

3. If f (x) = x3 and g x =
Solution: g (f (x)
= g (x3)
=

x find (gof) (7).

e xj
3

x find g (f (x).

Function

N.B.: Whenever f (x) = an analytic expression in x


and we are required to find f [f {f (x) }], we adopt the
following procedure:
1. We replace x by f (x) in the given expression in x
which provides us f {f (x)}.
2. Again we replace x by f {f (x)} in the given
expression in x which provides us f [f {f (x)}].

=x
4. If f (x) = x + 1 and g (x) = x find g (f (x)).
Solution: g (f (x)) = gof (x)
= g ((x+1))

e x ja

af

x +1

7. If f x =

= x +1
5. If f (x) = sin x and g (x) = x2, find (gof) (x).
Solution: (gof) (x) = g (f (x)) = g (f (x)) = g (sin x)
= (sin x)2

b a fg and b f axfg .

Solution: Given

af

af

af

Again, replacing x by f (f (x)) in (i), we get

b b a fgg = 1 f bf fbafxafxg fg
FG x IJ
H 1 2x K
=
F x IJ
1 G
H 1 2x K
x
R 1 1U
=
, x S1 , , V
1 3x
T 2 3W

f f f x

af

x3
3x + 1
1
, x = 3 , x =
,x
x3
3x + 1
3

f x =

...(i)

af

f x =

On replacing x by f (x) in (i) , we have

FG x IJ
H1 xK =
f a xf
=
f b f a x fg =
1 f a xf
F x IJ
1G
H1 xK
FG x IJ
H 1 x K = x , x RS1 , 1 UV
FG 1 2 x IJ 1 2 x T 2 W
H 1 x K

af

x3
3x + 1
and x =
find
x3
3x + 1

f x

x
6. If f x =
find f (f (f (x))).
1 x
x
,x 1
Solution: Given f x =
1 x

59

af
af

3 x + 1
x 3

FG x 3 IJ + 1
H 3x + 1K = 6 x 8 = 4 3x , for
FG x 3 IJ 3 8 x 6 3 + 4 x
H 3x + 1K

3
=

1 3
x ,
,3
3 4

Now to find {f (x)}, we consider the given

af

function x =

x 1
whose independent
3x + 1

variable x is replaced by f (x).

af

f x =
=

3x + 1
3
x3

a f
f a f

FG 3x + 1IJ + 1
H x 3K

3x + 1 3 x 3
10
1
=
= , x 1, 3, 0
3 3x + 1 + x 3
10 x
x

N.B.: To find f { (x)}, we replace x by (x) in the


given function for f (x) and to find {f (x)}, we replace
x by f (x) in the given expression for (x). Further we
should note that f { (x)} means the value of f at
(x) and {f (x)} means the value of at f (x).

60

How to Learn Calculus of One Variable

Refresh your memory: If given functions are


f : D1 C1 and g : C1 C2 , then the composite
function gof is the function from D1 to C2 defined by
(gof) (x) = gof (x) for every x in D1 and f x C1

af

domain of g.
g

f
f (x)

g f (x)

x
gof

Note: The working rule to fund (gof) is the same


provided the given functions are:
(i) f : D1 C1 and g : D2 C2 such that

a f

f D1 D2 .

= 8x2 4 3
= 8x2 7
(gof) (2) = (8x2 7)2
= 8 (2)2 7
=847
= 32 7 = 25
Notes: (i) If we are required to find (fog) (x) and (fog)
(1) for the just above defined functions, then (fog)
(x) = f (g (x)) = f (4x 3) ( 3 g (x) = 4x 3)
= 2 (4x 3)2 1( 3 f (x) = 2x2 1 f (g (x)) = 2 (f
(x))2 1)
= 2 (16x2 24x + 9) 1
= 32x2 48x + 17
(fog) 1 = 32 (1)2 48 (1) + 17
= 32 + 48 + 17 = 97

a f a xf a fog f a x f

(ii) In general gof

(ii) f : D1 C1 and g : C1 C2

5. If the mapping f : R R be given by f (x) = x2 +


2 and the mapping g : R R be given by

Solved Examples
1. If the mapping f : D1 C1 is defined by f (x) =
log (1 + x) and the mapping g : C1 C2 is defined
by g (x) = ex find (gof) (x).
Solution: (gof) (x) = g (log (1 + x)); x > 1 ( 3 f (x) =
log (1 + x))
= elog (1 + x) ( 3 g (x) = ex)
= (1 + x); x > 1 ( 3 elog f (x) = f (x))

af

g x =1

a f a xf a fogf a xf .

show that gof

Solution: (gof) (x) = g (f (x))


= g (x2 + 2)
= 1

2. If f : D1 C1 is defined by f (x) = x + 1, x R
and g : C1 C2 is defined by g (x) = x2, find (gof) (x).
Solution: (gof) (x) = g (f (x)) = g (1 + x) (3 f (x) = 1 +x)

a f e3 g a xf = x

= 1+ x

b a fg = f a xf j

1
2

1 x + 2

= 1

1
2

x 1

(fog) (x) = f (g (x))

FG
H

= f 1

g f x

1
compute (gof) (x) and (fog) (x) and
1 x

IJ FG
K H

= 1+

1
2

x +1

IJ FG
K H

1 x 1
x
1
=f
=f
1 x
1 x
1 x

FG x IJ
H 1 x K

g : R R is defined by g (x) = 4x 3, x R ,
compute (gof) (x) and (gof) (3).
Solution: (gof) (x) = g (f (x)) = g (2x2 1) ( 3 f (x) = 2x2
1)

In the light of (i) and (ii), it is clear that


(gof) (x) (fog) (x)

af

b a fg =

= 4 (f (x)) 3 (3 g x = 4 x 3 g f x

af

4 f x 3)
= 4 (2x2 1) 3

IJ
K

3. If f : R R is defined by f (x) = 2x2 1 and

+2=

...(i)

a1 xf

+2

...(ii)

Note: The operation that forms a single function from


two given functions by substituting the second
function for the argument of the first fucntion (for the
independent variable of the first function) is also
termed as composition. It is only defined when the

Function

range of the first is contained in the domain of the


second. Repeated composition is denoted by a
superscript numeral as f (n); so far example f o f o f o f =
f (4).

af

af

1+ x

af

f x

fof x =

1+ f

1 + 2x

1 + 3x

1+

2
2

1+ x

1+ x

1+ x + x
1+ x

af

1+ x

the same as the domain of

1 + 2x

nx : x D

af s

and f x g

f , i.e Dgof

and D gof = Df when

R f Dg .

F
GG
H

x
1 + 2x

I
JJ
K

af

G x

= f o G (x) =

1+G
x
=

= G (x) (say)

and f o f o f (x) = f o

Question: Define the composite of two functions


namely f and g whose domains are D 1 and D2
respectively.
Answer: 1. If f and g are two functions whose domains
are D1 and D2 respectively, then gof is the composite
of two functions namely f and g defined by (gof) (x) =
g (f (x)).
Further, we should note that the domain of gof is

f x D2 (domain of g). But if the range of f is a


subset of the domain of g, then the domain of gof is

1 + 2x

the set of all those x D1 (domain of f) for which

x
1+ x

Type 2: Problems based on finding the composite of


two functions whenever the domains are mentioned
in the form of intervals:

x
1+ x

a xf

x
=

Solution: 3 f x =

1 + 2x

1 + 2x + x

1+ x

af

6. Find f o f o f if f x =

61

1 + 2x
1+

the set of all those x D2 (domain of g) for which

a xf

af

g x D1 (domain of f). But if the range of g is a


subset of the domain of f, then the domain of fog is
the same as the domain of i.e; D fog =

nx : x D

1 + 2x

2. If f and g are two functions whose domains are D1,


and D2 respectively, then fog is the composite of two
functions namely f and g defined by (fog) (x) = f (g
(x)).
Further, we should note that the domain of fog is

Rg D f .

af

and g x D f

and D fog = D g when

62

How to Learn Calculus of One Variable

Notes: 1. If f (x) = f1(x), x > a and g (x) = f2 (x), x > b


then f is defined for x > a and gof (x) is defined for f (x)
> b which is solved for x to find the domain of (gof) (x)
i.e; domain of (gof) (x) is the intersection of the solution
sets of the inequalities x > a and f (x) > b. Similarly, g
is defined for x > b and fog (x) is defined for g (x) > a
which is solved for x to find the domain of (fog) (x),
i.e; domain of (fog) (x) is the intersections of the
solution sets of the inequalities x > b and g (x) > a.
2. If f (x) = f1 (x), a < x < b and g (x) = f2 (x) , c < x < b
then f is defined for a < x < b and gof (x) is defined for
c < f (x) < d which is solved for x to find the domain of
gof (x), i.e; domain of (gof) (x) is the intersection of
the solution sets of the inequalities a < x < b and c <
f (x) < d. Similarly, g is defined for c < x < d and fog (x)
is defined for a < g (x) < b which is solved for x to find
the domain of (fog) (x), i.e; domain of (fog) (x) is the
intersection of the solution sets of the inequalities c
< x < d and a < g (x) < b.
3. In general fog gof which fog x

a fa f

a fa f

gof x .
4. It should be noted that the notations fog and fg
represent two different functions namely a function f
of a function g and the product of two functions f and
g respectively.
Now we explain the rules of finding the composition
of two functions of xs whenever their domains are
mentioned as intervals.
Working rule to find (gof) (x): It consists of following
steps:
Step 1: Finding gof (x) (i.e; value of g at f (x) as usual.
Step 2: (i) Considering the inequality obtained on
replacing x by f (x) in the domain of g which is given
in the form of an interval finite or infinite.
(ii) Considering the inequality which represents the
domain of f.
Step 3: Finding the intersection of the inequalities (i)
and (ii) to get the domain of (gof) (x).
Working rule to find (fog) (x): It consists of following
steps:
Step 1: Finding (fog) (x) (i.e; value of f at g (x) as
usual.
Step 2: (i) Solving the inequality obtained on
replacing x by g (x) in the domain of f which is given
in the form of an interval.

(ii) Solving the inequality which represents the


domain of g.
Steps 3: Finding the intersection of inequalities (i)
and (ii) to get the domain of (fog) (x).
Solved Examples
1. If f (x) =

x + 4 , x 4 and g (x)

x 4 find (gof) (x).


Solutions:
(i) (gof) (x) = gof (x)
=
=

af

x 4,

af

f x 4 , f x 4 and x 4
x +4 4, x+4 4

(ii) We solve the inequality

x + 4 4 for x:

x + 4 16
x 16 4
x 12

(iii) D (gof) = 4 , 12 , = 12 ,
(iv) (gof) (x) =
12 or, (gof) (x) =

RSx 4 and , i.e. x >


T x 12
x + 4 4 , x 12 , f

x+4 4 ,

2. If f (x) = 1 + x, 0 < x < 1 and g (x) = 2 x, 1 < x < 2 find


(gof) (x).
Solutions:
(i) (gof) (x) = gof (x)
= 2 f (x), 1 < f (x) < 2 and 0 < x < 1
= 2 (1 + x), 1 < 1 + x < 2 and 0 < x < 1
( 3 f (x) = 1 + x)
= 1 x, 1 < 1 + x < 2 and 0 < x < 1
(ii) We solve the inequality 1 < 1 + x < 2 for x:
11+ x 2

111+ x 1 2 1
0 x 1
(iii) D (gof) = [0, 1] [0, 1] = [0, 1]
(iv) (gof) (x) = 1 x, 0 < x <
3. If f (x) = x2, 0 < x < 1 and g (x) = 1 x, 0 < x < 1 find
(gof) (x).

63

Function

Solutions:
(i) (gof) (x) = gof (x)
= 1 f (x), 0 < f (x) < 1 and 0 < x < 1
= 1 x2, 0 < x2 < 1 and 0 < x < 1 ( 3 f (x) = x2)
(ii) We solve the inequality 0 < x2 < 1 for x

Again considering the inequality

0 x 1

x 1 x 1
Hence, (a) and (b)

1 x 1
(iii) D (gof) = [1, 1] [0, 1]
(iv) (gof) (x) = 1 x 2 , x 1 , 1 0 , 1 , i.e;
x 0,1

af

1 x
4. If f : 0 , 1 0 , 1 be defined by f x = 1 + x ,
0 x 1 and g: 0 , 1 0 , 1 be defined by g (x)
= 4x (1 x), 0 < x < 1, find (gof) (x).
Solutions:
(i) (gof) (x) = gof (x)

F 1 x IJ , 0 1 x 1 and 0 < x < 1


= go G
H1 + xK 1 + x
F 1 x IJ LM1 FG 1 x IJ OP , 0 1 x 1 and 0
=4G
H1 + xK N H1 + xKQ 1 + x
=4

FG 1 x IJ FG 2 x IJ , 0 1 x 1 and 0 < x <


H 1 + xK H1 + xK 1 + x

1
(ii) We solve the inequality 0

1 x
1
1+ x

...(b)

x 0 , 1 , i.e. 0 x 1 .
(iii) D (gof) = [0, 1] [0, 1] = [0,1]

(iv) (gof) (x) = 4

FG 1 x IJ FG 2 x IJ , 0 x 1
H1 + xK H1 + xK

Type 3: Problems based on finding the composite of


two piecewise functions.
Rule: For finding gof (x) defined as under:

a f RST ff aaxxff,, ab << xx << bc

f x =

and

a f RSTgg aaxxff,, << xx <<

g x =

one must put y = f (x) and hence to find g (y) when

< y < and < y < for which it is required to


be determined the intersection of each two intervals
given below:
(i) a < x < b and < y < where y = f1 (x) defined in
the given interval namely a < x < b.
(ii) b < x < c and < y < where y = f2 (x) defined
in the given interval namely b < x < c.
(iii) a < x < b and < y < where y = f1 (x) defined
in the given interval namely a < x < b.

1 x
1 x
1 x
1 0
1
and
1+ x
1+ x
1+ x

1 x
Now considering the inequality
1,
1+ x
1 x
1 1 x 1 + x 0 2x
1+ x

0 x

1 x
0 1 x 0 30 x 1
1+ x

<x<1

g b

1 x
0,
1+ x

...(a)

(iv) b < x < c and < y < where y = f2 (x) defined in


the given interval namely b < x < c.
If the intersection of any two intervals mentioned
from (i) to (iv) is finite, then g (y) = gof (x) is defined
and if their intersection is , then g (y) = gof (x) is not
defined.
The union of finite intersection of any two
intervals mentioned from (i) to (iv) is the required

64

How to Learn Calculus of One Variable

domain of the composite of two given piecewise


functions.
Similarly for finding fog (x), one must put y = g (x)
and so to find f (y) where a < y < b and b < y < c for
which it is required to be determined the intersection
of the intervals given below:
(i) < y < and a < y < b where y = g1 (x) defined
in the given interval < y < .
(ii) < y < and a < y < b where y = g2 (x) defined
in the given interval < y < .
(iii) < y < and b < y < c where y = g1 (x) defined
in the given interval < y < .
(iv) < y < and b < y < c where y = g2 (x) defined
in the given interval < y < .
If the intersection of any two intervals mentioned
from (i) to (iv) is finite, then f (y) = fog (x) is defined
and if their intersection is , then f (y) = fog (x) is not
defined.
Solved Examples

1 x < 2 and 1 x 1

1 , 2 1 , 1 = 1 , 1 = 1 x 1

b a fg a f

For 1 x 1 , f g x = f y = y + 1

af

=g x + 1
2

e af

= x + 1 3 g x = x in 1 x < 2

Case (ii): Considering the intervals in (a) and (d),


2 x 3 and y 1

af

2 x 3 and g x 1

2 x 3 and x + 2 1 (3 g (x) = x + 2 in
2 x 3)
2 x 3 and x 1

2 , 3 , 1 =

a f b a fg is not defined

f y = f g x

Case (iii): Considering the intervals in (b) and (c),


1 x < 2 and 1 < y 2
2
1 x < 2 and 1 < x 2 ( 3 g (x) = x2 in

1. Two functions are defined as under:

1 x < 2 )

a f RST2 x x++1 ,11, x< x 1 2 and


R| x , 1 x < 2
g a xf = S
T|x + 2 , 2 x 3 find fog and gof.

1 x < 2 and { 1 < x < 2 or

f x =

2 x < 1}

Solution: f (g (x)) = f (y) where y = g (x)


f (y) = y + 1, y < 1
= 2y + 1, 1 < y < 2
g (x) = x2, 1 < x < 2
= x + 2, 2 < x < 3

af

1 x < 2 and x 1
( 3 g (x) = x2 in 1 < x < 2)

In 1 < x <
...(a)
...(b)
...(c)
...(d)

Case (i): Considering the intervals in (a) and (c),


1 x < 2 and y < 1

1 x < 2 and g x 1

f e

j e
2 , f b g a x fg = f a y f = 2 y + 1 for

1 , 2 1 , 2 = 1 , 2 = 1 < x < 2

1< y 2

af

= 2 g x + 1 = 2x + 1
Case (iv): Considering the intervals in (b) and (d),
2 x 3 and 1 < y 2

af

2 x 3 and 1 < g x 2
2 x 3 and 1 < x + 2 2 ( 3 g (x) = x + 2
in 2 x 3 )

2 x 3 and 1 < x 0

Function

1 x 2 and 1 2 x + 1 2 (3 f (x) = 2x +

2 , 3 1 , 0 =

f (g (x)) = f (y) is not defined.


Hence fog (x) = x2 + 1, for 1 x 1 , = 2x2 + 1 for
1< x 2 .
Next, g (f (x)) = g (y) where y = f (x)
f (x) = x + 1, x < 1
= 2 x + 1, 1 < x 2

af

...(a)
...(b)
...(c)

= y + 2,2 y 3

...(d)

Case (i): Considering the intervals in (a) and (c), x 1

1 x 2 and 2 2 x 1

LM
N

bg

x 1 and 1 x + 1 < 2
x 1 and 2 x < 1

a
In b 2 x < 1g , g (f (x)) = g (y) for 1 y < 2
, 1 2 , 1 = [2, 1] = 2 x < 1

3 y = f (x) = x + 1 in x 1 ).
Case (ii): Considering the intervals in (a) and (d),
x 1 and 2 y 3

= (x + 1)2 (

OP
Q
g b f a x fg = g a y f is not defined.
1 , 2 1 ,

1
=
2

Case (iv): Considering the intervals in (b) and (d),


1 < x 2 and 2 y 3

af

af
x 1 and 2 f a x f 3 ( 3 f (x) = x + 1 in

1 < x 2 and 2 2 x + 1 3 (3 f (x) = 2x + 1


in 1 x 2 )

1 < x 2 and

1, 2

x 1)

1
x 1
2

LM 1 , 1OP =
N2 Q

g (f (x)) = g (y) is not defined.


Hence gof (x) = (x + 1)2 for 2 x < 1 and = 4 for
x = 1.
2. If f (x) = x3 + 1 , x < 0
2

x 1 and 2 f x 3

= x + 1, x 0

af a f
= a x 1f , x 1

and g x = x 1 3 , x < 1
1
2

x 1 and 1 x 2

1
2

1 < x 2 and 2 f x 3

x 1 and 1 f x < 2

= y

1 in 1 x 2 )

1 x 2 and 1 x

g y = y , 1 y < 2

and 1 y < 2

65

kp

, 1 1 , 2 = 1
when x = 1, g (f (x) = g (y)
= y + 2 for 2 y 3

= x = 1 + 2 ( y = x + 1 in x 1 )
= x + 3 = 4 for x = 1
Case (iii): Considering the intervals in (b) and (c),
1 x 2 and 1 y 2

af

1 x 2 and 1 f x 2

compute gof (x).


Solution: g (f (x)) = g (y) where y = f (x)

g (y) = y 1 3 , y < 1

a f
f a x f = x + 1, x < 0

...(a)

= y 1 2,y 1

and

= x 2 + 1, x 1

...(b)
...(c)
...(d)

66

How to Learn Calculus of One Variable

Case (i): Considering the intervals in (a) and (c),


x < 0 and y < 1
x < 0 and f (x) < 1
x < 0 and x3 + 1 < 1 (3 f (x) = x3 + 1 in x < 0)
x < 0 and x3 < 0
common region = , 0 = x < 0

in (x < 0), g (f (x)) = g (y)

a f for y < 1
= b f a x f 1g
= y1

1
3

af

=x
Case (ii): Considering the intervals in (b) and (c),
x < 0 and y 1

af

x < 0 and f x 1
3

x < 0 and x + 1 1 ( 3 f (x) = x3 + 1) in x < 0)


3

x < 0 and x 0

, 0 0 , =

g (y) = g (f (x)) is not defined.


Case (iii): Considering the intervals in (a) and (d),
x 0 and y < 1

x 0 and f (x) < 1


x 0 and x2 + 1 < 1 ( 3 f (x) = x2 + 1 in x 0 )
x 0 and x2 < 0

x 0 and x < 0

f a

x 0 and x 0

x 0 and x 0

common region = 0 , x 0
in (x > 0), (g (f (x)) = g (y)

a f for x 0
= b f a x f 1g
1
2

1
2

= x + 1 1 3 , ( 3 f x = x 3 + 1 in x < 0)

x < 0 and x 0

x 0)

= y 1

1
3

x 0 and x + 1 1 (3 f (x) = x2 + 1 in

= x +11

1
2

( 3 f (x) = x2 + 1 in x 0 )

= x, Hence gof (x) = x for all x.


3. If f (x) = x2 4x + 3, x < 3
= x 4, x > 3
and g (x) = x 3, x < 4
= x2 + 2x + 2, x > 4
describe the function fog.
Solution: fog (x) = f (y), where y = g (x)
f (y) = y2 4y + 3, y < 3
= y 4, y > 3
g (x) = x 3, x < 4
= x2 + 2x + 2, x > 4

Case (i): Considering the intervals in (a) and (c), x <


4 and y < 3
x < 4 and g (x) < 3
x < 4 and x 3 < 3 ( 3 g (x) = x 3 in x < 4)
x < 4 and x < 6

f a

f a

= y2 4y + 3 for y < 3

g (y) = g (f (x)) is not defined.

= (x 3)2 4 (x 3) + 3

af

x 0 and f x 1

, 4 , 6 = , 4 x < 4
in (x < 4), fog (x) = f (y)

0 , , 0 =

Case (iv): Considering the intervals in (b) and (d),


x 0 and y 1

...(a)
...(b)
...(c)
...(d)

(3 y = g (x) = x 3 in x < 4)
2

= x 10 x + 24

67

Function

Case (ii): Considering the intervals in (a) and (d),


x 4 and y < 3

x 4 and g (x) < 3


x 4 and x2 + 2x + 2 < 3 ( 3 g (x) = x2 + 2x + 2
in x 4 )

= y 4 for y 3
= x2 + 2x + 2 4 (3 y = x2 + 2x + 2 in x 4 )
= x2 + 2x + 2
Case (iv): Considering the intervals in (b) and (c), x
< 4 and y 3

af

x 4 and (x2 + 2x 1) < 0

x < 4 and g x 3

x 4 and (x + 1)2 2 < 0

x < 4 and x 3 3 ( 3 g (x) = x 3 in x < 4)

x 4 and (x

x < 4 and x 6

+ 1)2 < 2

x 4 and x + 1 <

x 4 and 2 < x + 1 < 2

x 4 and 2 1 < x < 2 1

f e

4 , 2 1, 2 1 =

f (g (x)) = f (y) is not defined


Hence fog (x) = x2 10x + 24 for x < 4 and x2 + 2x
+ 2 for x > 4.

a f RST13 + xx ,, 02 < xx 23 .

4. Find f (f (x)) if f x =

f (g (x)) = f (y) is not defined.


Case (iii): Considering the intervals in (b) and (d),
x 4 and y 3

af

x 4 and g x 3
2

x 4 and x + 2 x + 2 3

e
x 4 and a x + 1f
x 4 and a x + 1f

x 4 and x + 2 x 1 0
2

20

x 4 and x + 1 2
x 4 and { x + 1 2 or x + 1 2 }
x 4 and { x 2 1 or x 2 1 }

x 4 and x 2 1
or x 4 and x 2 1

f 2 1 , j = 4 , f
or 4 , f e , 2 1 =
in a x 4f , f b g a x fg = f a y f
4,

, 4 6 , =

Solution: Given:

a f RST13 + xx ,, 02 < xx 23

f x =

... (a)
... (b)

To find: f (f (x)).

a f RST13 + xx ,, 02 < xx 23
R1 + y , 0 y 2
f a yf = S
T3 y , 2 < y 3

Let y = f x =

... (c)
... (d)

Case (i): Considering the intervals in (a) and (c),


0 x 2 and 0 y 2

0 x 2 and 0 1 + x 2
0 x 2 and 1 x 1
0 , 2 1 , 1 = 0 , 1
in [0, 1], f (y) = f (f (x)) = 1 + y
=1+1+x
=2+x

Case (ii): Considering the intervals in (a) and (d),


0 x 2 and 2 < y 3
0 x 2 and 2 < 1 + x 3
0 x 2 and 1 < x 2
0 , 2 1 , 2 = 1, 2

68

How to Learn Calculus of One Variable

f (y) = f (f (x) = 3 y = 3 1 x = 2 x for


1< x 2
Case (iii): Considering the intervals in (b) and (c),
2 < x 3 and 0 3 x 2

af

a f b a fg a f

a f x b g a g f x g b if g (x)
is increasing in [a, b] = D (gof)
D (f )

D (g )

R ( f)

R (gof)

2 < x 3 and 3 x 1
2 < x 3 and 1 x 3

f (x)

2 , 3 1, 3 = 2 , 3
in (2, 3], f (y) = f (f (x)) = 1 + y = 1 + 3 x = 4 x

Case (iv): Considering the intervals in (b) and (d),


2 x 3 and 2 y 3

2 x 3 and 2 3 x 3
2 x 3 and 1 x 0

2 x 3 and 0 x 1
2, 3 0,1 =

f (y) = f (f (x)) is not defined.


Hence,

R|2 + x , 0 x 1
f b f a x fg = S 2 x , 1 < x 2
|T4 x , 2 < x 3

Type 4: Determination of domain and range of


composite function
Rule: Determination of domain of composite function
namely gof consists of following steps:
1. Determination of the interval namely
S = domain of outer function namely g (x)
range of inner function namely f (x).
2. Determination of all the elements present in the
domain of f (x) whose images form the interval S is
the determination of all the elements forming the set
which is the required, domain of the composite
function namely gof i.e. the solution set of the
interval S where f (x) lies (i.e. the solution set of f (x)
S ) is the required domain of the composite function
namely gof.
Next to determine the range of the composite
function gof one should use the rule:

gof (x)

gof

Note: One should note that the rule mentioned in


type (4) holds true to determine the domain and range
of the composite function if the given composite
function is defined by a single formula y = gof (x).
Solved Examples
Find the domain and range of each of the following
functions:
(i) y = sin cos x (ii) y = tan cos x (iii) y = cos tan x
(iv) y = tan cos x (v) y = log sin x
Solutions: (i) y = sin cos x
R (cos x) = [1, 1]
D (sin x) = R
S = R [1, 1] = [1, 1]
and so cos x S 1 cos x 1

x R
Again, x R 1 cos x 1

a f

af

sin 1 y sin 1 since sin x is continuous


and increasing in [1, 1].
(ii) y = tan cos x
R (cos x) = [1, 1]
D (tan x) = R
S = R [1, 1] = [1, 1]
and so cos x S 1 cos x 1
x R
Again, x R 1 cos x 1
tan 1 tan cos x tan 1
(iii) y = cos tan x
R (tan x) = ,

a f

a f
D (cos x) = a , f

Function

tan x S < tan x <


x R

1 cos tan x 1

R cos tan x = 1 , 1

f a

0+

b g

R log sin x = , 0

Even and Odd Functions


Firstly, the definitions of even and odd functions are
provided.

(iv) y = tan cos x

< log sin x 0 as log x is continuous and


increasing in (0, 1] and lim log =

Again, x R < tan x < tan x R

69

cos x = 0 , 1

af af a f

(i) Even function: x D f : f x = f x

af

S = R 0,1 = 0,1

f x is even, i.e. for x = a = any real number


belonging to the domain of definition of f (x), f (x) is

cos x S 0

defined at x = a f x is also defined at x = a and

D (tan x) = R

cos x 1

0 cos x 1
2n

LM
N

x 2n +
2
2

OP
Q

x 2n

, 2 n +
2
2

Again, 2n

x 2n +
2
2

0 cos x 1
0 cos x 1
tan 0 tan cos x tan 1 as tan x is continuous and increasing in [0, 1].

R tan cos x = 0 , tan 1


(v) y = log sin x
R ( sin x) = [1, 1]

a f a f
S = a0 , f 1 , 1 = a 0 , 1
sin x S 0 < sin x 1

2 n < x 2n + 1

x 2n , 2n + 1

Again 2 n < x 2 n + 1

0 < sin x 1

f (a) = f (a) f x is even or more simply, it is the


function of an independent variable, changing neither
the sign nor absolute value when the sign of the
independent variable is changed.
sin t
e.g: x4 + 2x2 + 1; cos x ;
etc.
t
Notes:
1. Any algebraic function (or, expression) which
contains only even power of x is even.
2. n is even xn is even.
3. y = cosn x is even whether n is odd or even.
4. y = sinn x is even only when n is even.
5. y = | x | is an even function.

a f af

a f

(ii) Odd function x D f : f x = f x

af

f x is odd, i.e. for x = a = any real number


belonging to the domain of definition of f (x), f (x) is

af
f (a) = f (a) f a x f is odd, or more simply, it is the
defined at x = a f x is also defined at x = a and

D log x = 0 ,

af

af

function of an independent variable changing the sign


but not absolute value when the sign of the
independent variable is changed.
Notes:
1. Any algebraic function (or, expression) which
contains only odd power of x is an odd function.
(Note: The constant function f (x) = c is even and
when c = 0, i.e. y = 0 which is also termed as zero
function representing the x-axis is an even function).

70

How to Learn Calculus of One Variable

2. x2n + 1 = an odd function.


3. y = sin(2n + 1) x is an odd function.

(iii) f is even but g is odd fog is an even function.


(iv) f is odd but g is even fog is an even function.

(iii) Properties of even functions: The sum difference,


product and quotient of two even functions is again
an even function, i.e. f (x) and g (x) are even

Solved Examples

af
af

af af

f x
f x g x , f (x) g (x) and
are even.
g x
(iv) Properties of odd functions: 1. The sum and
difference of two odd functions is again an odd

af af

function, i.e. f (x) and g (x) are odd f x g x


2. The product and quotient of two odd functions is
again an even function; i.e. f (x) and g (x) are odd

af af

f x g x and

af
af

f x
are even.
g x

How to test whether a given function y = f (x) is


odd or even.
Working rule: The rule to examine (or, test) a given
function y = f (x) to be odd and even is (i) to replace x
by (x) in the given function f (x) and (ii) to inspect
whether f (x) changes its sign or not. It f (x) changes
its sign, one must declare it to be odd and if f (x) does
not change its sign, one must declare it to be even.
Notes:
1. f x f x f x is neither even nor odd.
e.g: If the oddness and eveness of the function f
(x) = e2x sin x is examined, it is seen that f (x) = e2x +
sin (x) f x which means that f (x) is neither

af

af

af

af

odd nor even.


2. Any function y = f (x) can be uniquely expressed
as the sum of an even and odd function as follows:

af

af a f

af a f

1
1
f x + f x +
f x f x
2
2
3. A piecewise function is even if each function
defined in its domain is even and a piecewise function
is odd if each function defined in its domain is odd.
4. f and g are two functions such that
(i) f is even and g is also even fog is an even
function.
(ii) f is odd and g is also odd fog is an odd
function.
f x =

Examine the oddness and eveness of the following


functions.
1. f (x) = x4 + 2x2 + 7
Solution: f (x) = x4 + 2x2 + 7
f (x) = (x)4 + (x)2 + 7
= x4 + 2x2 + 7
= f (x)
Therefore, f (x) is an even function.

af

af

2. g x = x 16x + 11x

92
x

Solution: g x = x 16x + 11x

a f a f
F
= Gx
H

g x = x

a f

92
x

a f a92xf
92 I
+ 11x
J
xK

16 x

16 x 3

+ 11 x

= g (x)
Therefore, g (x) is an odd function.
3. f (x) = x2 | x |
Solution: f (x) = x2 | x |
f (x) = (x)2 | x |
= x2 | x |
= f (x)
Therefore, f (x) is an even function.

FG x + 1IJ
H
K
F
I
Solution: f a x f = log G x + x + 1J
H
K
f a x f = log F x + a x f + 1I
H
K
af

4. f x = log x +

= log

F x +
GG
H x+

x +1
2

x +1

x+

I
JJ
K

x +1

Function

F x + x + 1I
= log G
GH x + x + 1 JJK
F 1 I
= log G
GH x + x + 1 JJK
F
I
= log 1 log G x + x + 1J
H
K
F
I
= log G x + x + 1J
H
K
2

Rx x , x 1
a f |S 1 + x + 1 x , 1 < x < 1
|T x x , x 1

7. f x =

Solution: The given function can be rewritten as


under:

R| x , x 1
f a x f = S2 + x + x , 1 < x < 1
|| x , x 1
T

= f (x)
So, f (x) is an odd function.

Fa
GH a

I
J
+ 1K
F a 1I
Solution: f a x f = x G
H a + 1JK
F a 1I
f a x f = a xf G
H a + 1JK
F
I F I
1 a
a 1J
G
= x G
= x G
J
J
GH a + 1JK H 1 + a K
F a 1I = f a x f
= xG
H a + 1JK
af

5. f x = x

x
x

1
x

1
x

71

since [1 + x] = 1 + [x] and [1 x] = 1 + [x]


Also, it is known that [x] + [x] = 0 when x is an
integer and [x] + [x] = 1 when x is not an integer.
Hence, again in the light of above facts, the given
function can be rewritten as:

R| x , x 1
||1 , 1 < x < 0
|
f a x f = S2 , x = 0
||1 , 0 < x < 1
||
T x , x 1
2

which is clearly an even function.

x
x

x
x

So, f (x) is an even function.


6. f (x) = sin x + cos x
Solution: f (x) = sin x + cos x
f (x) = sin (x) + cos (x)
= sin x + cos x which is clearly neither equal to f
(x) nor equal to f (x).
Therefore, f (x) is neither even not odd.

Periodic Functions
Definition: When f (x) = f (x + P) = f (x + 2P) = = f (x
+ nP), then f (x) is said to be periodic function of x, for
its values repeat, its period being P (where
P 0 , x domain of definition of f) which is the
smallest positive number satisfying the above
property f (x) = f (x + P) = f (x + 2P) = = f (x + nP)
where n Z , n 0 .
Notes:
1. The numbers of the form nP, n Z , n 0 are also
called period of the function. But generally the smallest
positive number P is called the period of the function
(or, fundamental period of the function) unless

72

How to Learn Calculus of One Variable

nothing is mentioned about the period of the function.


2. The smallest positive period for sine and cosine is
equal to 2 and for the tangent and cotangent, it is

equal to . Since, sin x = sin x + 2 =

sin x + 4 = ..., is periodic , its period being 2 .

Similarly, cos x is periodic, its period being 2 . Hence,


the periodicity property of the trigonometric functions
can be expressed by the following identities.

a
f
cos x = cos a x + 2 n f , n Z
tan x = tan b x + n g , n Z
cot x = cot b x + n g , n Z
sin x = sin x + 2 n , n Z

3. The following formulas of the trigonometric


functions to find out fundamental period of the
trigonometric functions are very useful.
If we have an equation of the form y = T (Kx), aT
(Kx), aT (Kx + b), where a = amplitude of trigonometric
function, K = any constant, b = any other constant, T
= any trigonometric function sin, cos, tan, cot, sec,
cosec, then the fundamental period P of the
trigonometric function having the form:
(i) y = sin (Kx), a sin (Kx) or a sin (Kx + b) is given by

2
, where K is any constant.
the formula P =
K
(ii) y = cos (Kx), a cos (Kx) or a cos (Kx + b) is given
2
, where K is any constant.
K
(iii) y = tan (Kx), a tan (Kx) or a tan (Kx + b) is given
by the formula P =

, where K is any constant.


K
(iv) y = cot (Kx), a cot (Kx) or a cot (Kx + b) is given
by the formula P =

by the formula P = , where K is any constant.


K
(v) y = sec (Kx), a sec (Kx) or a sec (Kx + b) is given
by the formula P =

2
, where K is any constant.
K

(vi) y = cosec (Kx), a cosec (Kx) or a cosec (Kx + b) is


given by the formula P =

2
, where K is any
K

constant.
Remember:
1. Period of any trigonometric function, its cofunction and its reciprocal is the same. Thus,
(i) The period of sin x and cosec x = 2 .
(ii) The period of cos x and sec x = 2 .
(iii) The period of tan x and cot x = .
2. Only those trigonometric functions are periodic
whose angles are linear expressions in x (i.e. angle =
ax + b). For examples, sin3 x, cos4 x, tan (4x + 5), etc.
3. Those trigonometric functions are not periodic
whose angles are not linear expressions in x (i.e., angle

ax + b ). For examples, sin

F 1 I , cos
H xK

x , etc.

4. No periodic function other than a constant can be


algebraic which means that algebraic functions can
not be periodic excepting a constant function.
5. If the function f1 (x) has the period P1, and the
function f2 (x) has the period P2, then the function

af

af

y = a f 1 x b f 2 x a and b being given numbers,


has the period equal to least common multiple (i.e.;
l.c.m) of numbers of the set {P1, P2}
e.g.: y = 2 sin x 3 tan x has the period 2 since
period of sin x = 2
period of tan x =

L.c.m of 2 , = 2
6. A function of trigonometric periodic function is
also periodic provided the angle = ax + b; i.e.; f (T (ax
, (...)n, |...|, log,
etc and T signifies sin, cos, tan, cot, sec and cosec.
+ b)) is periodic where f signifies

a f

e.g.: tan x , cos x , sin 2 x + b , etc are functions


of sin x, cos x and tan x and are periodic.
7. The sum and difference of periodic and non
periodic function is non-periodic. Moreover, the

73

Function

product of a periodic and non-periodic function is


non-periodic. e.g.,
(i) f (x) = sin x + cos x is non-periodic since sin x is
periodic and cos x is non-periodic.
(ii) f (x) = x cos x is non-periodic since x being an
algebraic function is non-periodic and cos x is
periodic.
8. Whenever, we use the term period, we always mean
the fundamental period which is the smallest positive
value of P for which the relation:
f (x) = f (x + P) holds true for all values of x, P being
a constant, i.e.; the values of f (x) at the points x and
(x + P) are same.(Note: If f (x) is a periodic function
with period T and g (x) is any function such that
domain of f is a proper subset of domain g, then gof is
period with period T).
e.g.: y = sin (x [x]) is periodic with period 1,
because (x [x]) is periodic with period 1.
Working rule to fund the period: It consists of
following steps: (Trigonometric functions)
1. To denote the desired period by P and to replace x
by (P + x).
2. To put T [a (x + P) + b] = T (ax)
3. To put aP = a constant multiple of P = 2 or
according as the given function is sin, cos, tan, cot
sec or cosec.
4. To solve a P = 2 or to get the required (or,
desired) period of the trigonometric periodic function.
Solved Examples
Find the period of each of the following functions:
1. y = sin 3x
Solution: Method (1)
On denoting the period of the function y = sin 3x
by P, we get y = sin 3 (x + P) = sin 3x
sin (3x + 3P) = sin 3x
3P = 2
2
P=
3
Method (2)
On using the formula of period of trigonometric
2
function, we get P =
where K = multiple of x.
K

2
3

2. y = cos

F xI
H 2K

Solution: Method (1)


On, denoting the period of the function

F x I we get y = cos F x I
H 2K
H 2K
F x + P IJ = cos F x I
cos G
H 2 K H 2K

y = cos

P
= 2 P = 4
2
Method (2)
Using the formula of period of trigonometric

function, we get P = 2 / K , where K =

1
=
2

1
= 4 .
2
3. y = sin 2x + cos 3x
Solution: Method (1)
We are required to find out the period of each
addend of the given sum function y = sin 2x + cos 3x
Now, sin 2 (x + P1) = sin 2x
multiple of x = 2 /

sin 2 x + 2 P1 = sin 2 x 2 P1 = 2 P1 =
similarly, cos 3 (x + P2) = cos 3x

cos 3x + 3P2 = cos 3x 3 P2 = 2 P2 =

RS
T

L.c.m of ,

UV
W

2
3

2
2
=
= 2
3
1

Hence, period of y = sin 2x + cos 3x = 2


Method (2)
On using the formula of period of trigonometric
function sin (Kx), P1 =

2
, where K = 2 = a constant
K

multiple of x.

2
=
2

74

How to Learn Calculus of One Variable

Again using the formula of period of trigonometric

2
function cos (Kx), P2 =
, where K = 3 = a constant
K
multiple of x.
=

2
3

RS 2 UV = 2
T 3W
Hence, the required period of the given sum
L.c.m of ,

4. y = sin

F 3x I + sin F 2 x I
H2K H 3K

LM 3 ax + P fOP = sin F 3x I
N2
Q H2K
F 3 3 I F 3x I 3 P
sin x + P = sin
H2 2 K H 2 K 2
1

= 2

4
3

LM 2 ax + P fOP = sin F 2 x I
N3
Q H 3K
F 2 x + 2 P I = sin F 2 x I P = 3
sin
H 3 3 K H 3K
R 4 U 12 = 12
L.c.m of S , 3 V =
T3 W 1

Similarly, sin

(Note: f (x) = x [x] f (x + 1) = x + 1 [x + 1] =


x + 1 ([x] + 1) = x [x] = f (x) f (x) is periodic with
period 1).
Method (2)
On using the formula of period of trigonometric

2
3
, where K =
function sin (Kx), P1 =
K
2
=

2
3
= 2 = 3
2
2
3

L.c.m of

Solution: Method (1)


We find the period of each addend of the given
sum function.

sin

2
2
, where K =
K
3

RS 4 , 3UV = 12
T3 W

Remember:
L.c.m of two or more fractions

function = 2

P1 =

and P2 =

2
2 4
= 2 =
3
3 3
2

l.c. m of numerators
h.c. f of denominators

Question: How to show the following function to be


not periodic:
Answer: To show that a given trigonometric function
f (x) is not periodic, we adopt the rule consisting of
following steps.
Step 1: Replacing x by (T + x) in the given function
and equating it to f (x) which means one should write
f (T + x) = f (x).
Step 2: Considering the general solution of

af

(i) sin [f (x + T)] = sin [f (x)], where f x ax + b and

a f , , etc, is
f a x + T f = n + a 1f f a x f (3 sin x = sin
x = n + a1f an = 1 , 2 ,...f )
(ii) cos [f (x + T)] = cos [ f (x)], where f a x f ax + b
and f signifies the operators a f ,
, etc, is
f a x + T f = 2n f a x f ( 3 cos x = cos
x = 2 n an = 1 , 2 ,...f )
(iii) tan [f (x + T)] = tan [ f (x)], where f a x f ax + b
and f signifies the operators a f ,
, etc, is
f a x + T f = n + f a x f ( 3 tan x = tan
x = n + an = 1 , 2 ,...f )
f signifies the operators

n n

n n

n n

Function

af

Step 3: If T = F x f (x) is not periodic which means


that if there is a least positive value of T not
independent of x, then f (x) will not be a periodic
function with period of T.

af

af

Or, T = F x f x periodic. (Note: A


monotonic function can never be periodic and a
periodic function can never be monotonic. That is,
monotoncity and periodicity are two properties of
functions which can not coexist).
Solved Examples

argument belonging to their common domain, i.e.; two


functions f and g are equal the following two
conditions are satisfied.
(i) f and g have the same domain D.
(ii) f and g assume the same (or, equal) value at each
point of their common domain, i.e.; f (x) = g (x),
x D .

Notes:
1. The above two conditions are criteria to show that
given two functions namely f and g are equal.

Show that each of the following functions is not


periodic.

2.

x
= 1, provided x 0
x

1. f x = sin

3.

f x
= 1, provided f x 0
f x

af

x
af
f a x + T f = f a x f sin x + T = sin x
T + x = n + a1f
x which does not provide

Solution: f x = sin

us the value of T independent of x. Hence, f (x) is not


a periodic function.
2. f (x) = cos x2
Solution: f (x) = cos x2

af

= 2 n x

f x + T = f x cos x + T

x+T

x+T =
T =

2n x

2 n x

= cos x

75

af
af

af

Working rule to show two functions to be equal: It


consists of following steps:
1. To find the domain of each function f and g.
2. To inspect whether dom (f) = dom (g) as well as f
(x) = g (x), x D , D being the common (or, same)
domain of each function f and g, i.e.; if dom (f) = dom
(g) and f (x) = g (x), x D , then we say that f = g
and if any one of the two conditions namely dom (f) =
dom (g) and f (x) = g (x), x D is not satisfied, we
say that f g .
Solved Examples
Examine whether the following functions are equal or
not:

af

which does not provide us the value of T


independent of x. Hence, f (x) is not periodic.

1. f x =

Question: Explain what you mean when we say two


functions are equal.
Answer: Two functions f : D C and g: D C
are said to be equal (written as f = g) if f (x) = g (x) for
all x D which signifies that two functions are equal
provided their domains are equal as well as their
functional values are equal for all values of the

Solution: f x =

af

af

x ,g x = x
2

af

x ,g x = x

x is defined for all real values of x


dom (f) = R = the set of all real numbers ...(i)
Again, 3 g (x) = | x |
| x | is defined for all real values of x
dom (g) = R = the set of all real numbers ...(ii)
Also, we inspect that f (x) = g (x) = | x | for every
real values of x
...(iii)

76

How to Learn Calculus of One Variable

(i), (ii) and (iii)

RSdom a f f = dom a g f = R
T f a x f = g a x f , x R , R

being the common domain


f=g

af

2. f x =

x +1

af

Solution: f x =

,g x =1
2

x +1
2

af

,g x =1

x +1
Now, we find the domain of f:
2

af

af

Putting x + 1 = 0 x = 1 x = i = imaginary

f (x) is defined for all real values of x


dom (f) = R = the set of all real numbers ...(i)
Again, g (x) = 1 is defined for all real values of x
dom (g) = R = the set of all real numbers ...(ii)
Also, we inspect that f (x) = g (x), x R ...(iii)

R dom a f f = dom a gf = R
(i), (ii) and (iii) S
T f a x f = g a x f , x R , R
being the common domain
f=g

Note: To find the values of the argument x for which


two given functions f and g may be equal (i.e.;
whenever f (x) and g (x) may be equal by performing
any operation or rule like cancellation, extracting the
root, etc on any one of the given functional value)
means to find the common domain of each function f
and g over which they are defined.
Working rule: We have the rule to find for what
values of the argument given functions are identical.
It says to find the common domain of each function f
and g to examine whether their functional values are
equal for all values of the argument belonging to the
common domain of f and g, i.e.; whether f (x) = g (x),
x D , D being the common domain of each
function f and g should be examined.

Solved Examples
Find for what values of x following functions are
identical.

af

x
,g x = x
x
(iii) f (x) = log10 x2, g (x) = 2 log10 x
Solution: (i) f (x) = x dom (f) = R

(ii) f x =

af

x +1

af

(i) f x = x , g x =

af

...(a)

2
x = x for x 0 and = x for x < 0

g x =

g (x) is defined for all real values of x


...(b)
dom (g) = R
Hence, (a) and (b) f (x) = g (x), x 0 ,

af

x
= x , provided x 0
x
dom (f) = R {0}
g (x) = x
dom (g) = R

(ii) f x =

...(a)
...(b)

kp

Hence, (a) and (b) f (x) = g (x), x R 0


(iii) f (x) = log10 x2, g (x) = 2 log10 x, provided x > 0
and 2 log (x) for x < 0
f (x) is defined for all real values of x
...(a)
dom (f) = R
g (x) = 2 log10 x, provided x > 0
g (x) is defined for all positive values of x

af

a f

dom g = R = 0 ,

...(b)

a f

Hence, (a) and (b) f (x) = g (x), x 0 ,


Problems on one-to-one Function

Firstly, we recall the definition of one-to-one (or,


simply one-one) function.
Definition 1: It the given function f : D R
defined by y = f (x) is such that there is unique y in the
range R for each x in the domain D and conversely
there is a unique x in the domain D for each y in the
range R, then it is said that given function y = f (x) is
one-to-one.
Definition 2: (Set theoretic): If different elements of
domain of the function have different images (or,
values) in the range (or, codomain), then it is said that
the function is one-to-one.

78

How to Learn Calculus of One Variable

Remark: It the domain of f (x) = sin2 x is extended to


0,
x is not one-one, because for two different points

then its range is same, i.e. [0, 1], but f (x) = sin2

5
, f (x) = sin2 x has the same value,
and
6
6
as it is clear from the following:
namely

F I = sin F I = F 1 I = 1
H 6K
H 6K H 2K 4
F 5 I = sin F 5 I = sin F I
f
H 6K
H 6K
H 6K
F I 1
= sin H K =
6
4
2

(ii) 3 2 , 2 3 , 3

f (2) = (2)2 = 4
and f (2) = (2) = 4

af

a f

f 2 = f 2

Hence, f (x) = x2 is not one-one because for two


different values of x in [3, 3], f (x) = x2 has the same
value 4.
+

(iii) For x1 , x2 R ,

a f

Problems on on-to Functions


Before doing the problems on on-to functions, we
recall its definition. On-to function: If
f : D R C is a function such that every element
in C occurs as the image of at least one element of D,
then f : D R C is called an on-to function from
D to C.
In other words, f : D R C is called on-to
function when the range (or, the range set) of f equals
the co-domain, i.e., range set = co-domain (i.e., R = C)
signifies that a function f : D R C is on-to
function. Hence, f : D R C is on-to function
f D = C , where f (D) is called the image of D
signifying the set of images of all elements in D and is
defined as:
f D = f x : x D = range set = R (f) = R
(simply)

a f

a f l af

Notes:
1. If a function is not on-to, it is called into
function, i.e., if the function f : D R C is
such that certain elements of the set C are left out,
which are not the images of an element of the set D,
then f is called into function.
In other words f : D R C is said to be into,

af

if f D C , i.e; when the range set co-domain,


then the function is said to be into function.

a f

f x1 = f x 2
x1 = x 2

x1 = x2
Hence f is one-one in R+.

F , I ; ,
H 2 2K
f ax f = f ax f

R=C

x1
x2

y1
y2

on-to function

x1
x2
x3

y1
y2
y3

into function

(iv) For x1 , x2
1

tan x1 = tan x 2
x1 = x2
Hence f is one-one in

F , I .
H 2 2K

2. A one-to-one function may be into or onto. i.e.,


there are two types of one-one function namely (i)
one-one onto (ii) one-one into, which are defined as:
(i) one-one (symbolised as 11) function is called
onto provided there is no-element in the range set R
which does not appear as an image of a certain element

Function

79

of the domain D or, simply, a function which is both


one-one and onto is called one-one onto. Shortly, we
write one-one onto one-one + onto. Further it is
notable that one-one onto function satisfies the
following properties.

Remark: When we say that a function is one-one


and onto, it is assumed that number of elements of
domain and range are equal such that each member of
the domain has a different image in the range set
whether the domain is a finite set or an infinite set.

(a) No two element of the domain have the same


image.
(b) Every element of the range (or, co-domain) is the
image of some element of the domain which means
alternatively there is no element on the range (or, codomain) which is not the image of any element of the
domain.

Question: How would you show that a given function


defined by a single formula y = f (x) in its domain is
onto.
Answer: There are mainly two methods to examine
whether a given function defined by a single formula
y = f (x) in its domain is onto or not.
Method 1: If the range of the function = codomain of
the function, then given function f is onto. If the range
is proper subset of codomain of the function f, then f
is into.
Note: Method (1) is fruitful to examine whether a
given function is onto or not only when the domain
of the given function is finite and contains a very few
elements.

D
x1
x2
x3

R
one-one on-to
function which is
symbolized as
one-one
f: D
R
on-to

y1
y2
y3

one-one on-to function which is symbolised as


one one

R.
f : D on
to
(ii) one-one into function: A one-one function is
called into provided the range set is contained in the
co-domain (i.e., R C ) such that co-domain
contains at least one element which is not an image of
any one element in the domain D, then the function is
called one-one into, or simply a one-one function is
called into provided it is not onto. Shortly we write
one-one into one-one + into. It is notable that oneone into function satisfies the following properties.
(a) No two elements of domain have the same image.
(b) There is at least one element in co-domain which
is not the image of any element of the domain.
f

C ( R C)
R

x1
x2

y1
y2
y3

one-one on-to
function which is
symbolized as
one-one
f: D
R
on-to

One-one in-to function which is symbolised as:


one one

R C .
f : D in
to

Method 2: To show that f is onto, it is required to be


shown that y B , x A such that y = f (x), where
A = domain of f and B = codomain of f.
i.e. one should assume y B and should show
that x A such that y = f (x).
Step 1: Choose an arbitrary elements y in B
(codomain).
Step 2: Put f (x) = y.
Step 3: Solve the equation y = f (x) for x, say x = g (y).
Step 4: If x = g (y) is defined for each y codomain
of f and x = g (y) domain of f for all y codomain of
f, then f is declared to be on to.
If this requirement is not fulfilled by at least one
value of y in the codomain of f, then f is decalred to be
into (i.e. not onto).
i.e. to show that f is not onto (i.e. in to), one should
point out a single element in the codomain of f which
is not the image of any element in the domain.
Notes:
1. Method (2) is fruitful to examine whether a given
function is onto or not only when the domain of the

80

How to Learn Calculus of One Variable

given function is infinite or finite but containing very


large number of elements.
2. When either domain or codomain or both is not
mentioned, it is always understood to be R while
examining a given function to be onto or into.
3. An into function can be made onto by redefining
the codomain as the range of the original function.
4. Any polynomial function f : R R is onto if its

Again, y R and y 0 ,
Also,

5
5
3 3
0, y
+
2y
2y 2 2

degree is odd and any polynomial function


f : R R is into if its degree is even.

x =

5
3
3
+ R
2y 2
2

Solved Examples

Thus, y R 0 , x =

1. A function f : R R is defined by f (x) = 2x + 3


examine whether f is onto.
Solution: In this question,
domain of f = R
codomain of f = R
Let y R

af

y3
Now y = f x = 2 x + 3 2 x = y 3 x =
2
x =

y3
R
2

2x =

5
5
3
+3 x =
+
y
2y 2

RS UV
TW

kp

that y = f (x).
3. Show that the function f : R 3 R 1

kp

af

defined by f x =

kp

Solution: Let y R 1 , y R and y 1

af

Now y = f x =

x2
x3

y3
R
2

a f

x 1 y = 2 3y x =
Now y R and y 1

2. A function f : R

x =

RS 3 UV R k0p is defined
T2 W

5
. Show that f is onto.
2x 3

Solution: In this question,


domain of f = R

kp

x2
is onto.
x3

(domain) such that y = f (x).


Hence, f is onto.

af

RS UV such
TW

5 3
3
+ R
2y 2
2

y x 3 y = x 2 x y x = 2 3y

Thus, y R (codomain), x =

by f x =

5
3
+ R
2y 2

RS 3 UV
T2 W

codomain of f = R {0}

lq
5
5
2x 3 =
Now y = f a x f =
y
2x 3
Let y R 0 , y R and y 0

af

2 3y
= g y (say)
1 y

kp

2 3y
is defined y R 1
1 y

Also, x =

kp

kp

2 3y
R 3 , y R 1
1 y

Hence, f is onto.
4. A function f : Z N is defined by f (x) = x2 + 3.
Test whether f is onto.
Solution: In this question,
Domain of f = the set of integers = Z
Codomain of f = the set of natural numbers = Z
Let y N
Now, y = x2 + 3

Function
2

x = y 3 x =

y 3 which is not

defined for y < 3, i.e. x =

af

y 3 = g y is not

defined y N
Hence, x =

y 3 Z if y < 3

For example I N is not the f-image of any x Z


and hence f is not onto.

5. Let A = x : 1 x 2 = B and a function

f : A B is defined by f (x) = x2. Examine whether


f is onto.
Solution: In this question,
Domain of f = codomain of f = 1 x 2
Let x 1 , 2 = B
2

Now y = x x = y which is not defined


for y < 0.
Clearly, x 1 , 2 = A for all y 1 , 2 = B

1
B is not the f-image of any
2
x A and hence f is not onto.
For example

6. If f : R R be defined f (x) = cos (5x +2), show


that f (x) = cos (5x + 2) is not onto.
Solution: Since it is known that

1 cos 5x + 2 1 which range of cos (5x


+ 2) = [1, 1]
range of f = 1 , 1 R = codomain of f

f : R R defined by f (x) = cos (5x + 2) is not


onto.

7. Let A = x : 1 x 1 = B and a function

f : A B is defined by (i) f (x) = | x | (ii) f (x) = x | x |.


Examine whether it is onto or not.
Solution: (i) In this question,
Domain of f = codomain of f = 1 x 1
Let y 1 , 1 = B and y = f (x) = | x |
Now y = x y is always non-negative

81

range of f = [0, 1] which is B


f is not onto.
(ii) 3 y = x | x |
y = x x for x 0

y = x2 for x 0 , 1 , according to question.


x =

y
y for x 0

x=

x is defined for y 0
x is defined for y 0 , 1 according to
question.
...(a)
Again, 3 y = x | x |
y = x (x) for x < 0
x2 = y for x < 0
x =

x = y for x < 0
x = y for x 1 , 0
x is defined for y < 0

x is defined for y 1 , 0 according to


question.
...(b)
From (a) and (b), it is concluded that range of f =
[1, 0] [0, 1] = [1, 1] = B = codomain of f which
f is onto.
8. Check whether the function f : R R defined

af

by f x =

x
is onto or into. Also find R (f).
1+ x

Solution: Let y R = codomain of f

af

and y = f x =
Now, y =

y=

x
1+ x

x
1+ x

x
for x > 0
1+ x

82

How to Learn Calculus of One Variable

y + yx = x
y = x yx = x (1 y)

Solution: Onto test:

y
0
1 y

a f
y a y 1f 0
f

y 0,1

(negative) and x =

x
1+ x

Hence, for y < 0 , x 1 , 0 such that f (x) = y.


Case 2: y = 0 for x = 0.

a f

Case 3: To show that for any y > 0, there is x 0 , 1


such that f (x) = y

y
, 3 y 1
1+ y

Now, y =

x
1 x

y yx = x
x (1 + y) = y

y
<0
1+ y

y ( y + 1) < 0
y (y (1)) < 0

x=

...(b)
1 < y < 0 y 1 , 0
From (a) and (b), it is concluded that range of f =

f a f

(1, 0) 0 , 1 = 1 , 1 R = codomain of f
Hence, f is not onto

a f

9. Prove that the function f : 1 , 1 R defined


by
x
, 1 < x < 0
f x =
1+ x
= 0, x = 0

af

z
1+ z

0 < x < 1 1 < x < 0

x
for x < 0
1 x

Now, x < 0

y
, 3y 1
1 y

...(a)

y yx = x
y = x + yx = x (1 + y)
x=

z
1+ z

x=

0 y 1 but y 1

y=

Now if y is negative (i.e. y = z), then x =

x
1+ x

y + xy = x
x (1 y) = y

y 1 y 0

Again, y =

Now, y =

Now, x > 0

Case 1: To show that for any y < 0, there is x 1 , 0


such that f (x) = y.

y
x=
, 3 y 1
1 y

x
, 0 < x < 1 is onto.
1 x

y
, 3 y 1
1+ y

for y > 0, 0 < x < 1


for any y > 0, x 0 , 1 such that f (x) = y

a f
Hence, for any y a , f , x a 1 , 1f such that f
(x) = y.
Hence, the given function is onto.
Notes:
1. In fact this function is bijective whose oneoneness can be shown in the following way.

Function

To test whether f is one-one, different possible


cases are as follows:

Case 1: When x1 , x2 1 , 0
f (x1) = f (x2)

x1
x2
=
1 + x1 1 + x 2
x1 + x1 x2 = x2 + x1 x2
x1 = x2

af

9. Given the function f x =

a f

Case 2: When x1 , x2 0 , 1
f (x1) = f (x2)

+ 1), f

a f a f

Case 3: When x1 = 0 and x2 0 then f x1 f x2

a f

a f
this case clearly x x f a x f f a x f, as
x
x
> 0.
< 0 and f a x f =
f ax f =
1 x
1+ x
f

Case 4: When x1 1 , 0 and x2 0 , 1 then in


1

12. If f (x) = cos x, find f

af
F I
sin x cos x
13. If f a x f =
, find f H K .
3
sin x + cos x

Hence, the given function is one-one.


2. A piece-wise function is one-one each
function defined in its respective sub domains is oneone.
3. A piece-wise function is on-to each function
defined in its respective sub domains is on-to.
1. Finding the Value of a Given Function
Type 1: When the given function is not a piecewise
function.
Exercise 1.1

af

x +1
, find f (x2) and (f (x))2.
x 1

15. If f x =

x + x+1

af

16. When f x =

9
3+ x

, find f (1 + h).

determine f (0), f (3) and

F 1I .
H xK
find f (1), f (1) and f

F1 x I
GH 1 + x JK .
2

4. If f x = 1 x , find f (sin x) and f

x x+1

17. A function f is defined on R as follows


f (x) = c, c being a constant, x R

1. If f (x) = 3x 2, find f (1).


2. If f (x) = 3x2 5x + 7, find f (2).
3. If f (x) = x4 3x2 + 7, find f (1) and f (2).
2

af

14. If f x =

F I , f a0f , f F I , f F I
H 2K
H 3K H 6K

and f .

af

F 1 I and f (5).
H aK

3x 5
, find f (3) and
x+7

f (2).
10. If f (x) = 2x2 4x + 1, find f (1), f (0), f (2), f (2), f (a)
and f (x + 8).
11. If f (x) = (x 1) (x + 5), compute f (2), f (1), f (0) f (a

as f (x1) = 0, f x 2 0

F I .
H 4K

6. If f (x) = x3 2x2 + x 1, find f (0), f (1), f (1) and f (2).


7. If f (x) = x4 x3 + 2x2 + 4, find f (0), f (1) and f (2).
8. Given the function s (t) = t2 6t + 8, find s (0), s (2)
and s (1).

x1
x2
=
1 x1 1 x2
x1 x1 x2 = x2 x1 x2
x1 = x2

5. If f (x) = sin x + cos x, find f

83

Answers:
1. f (1) = 5
2. f (2) = 29

F 3I .
H 2K

84

How to Learn Calculus of One Variable

3. f (1) = 5, f (2) = 11

F 1 x I = 2x
f asin x f = cos x , f G
H 1 + x JK 1 + x
F I = 2
f
H 4K

Exercise 1.2

4.

5.

1. If f (x) = 1 + x, when 1 x < 0


= x2 1, when 0 < x < 2
= 2x, when x > 0

find f (3), f (2), f (0), f

6. f (0) = 1, f (1) = 1, f () = 5, f (2) = 1


7. f (0) = 4, f (1) = 8, f (2) = 20
8. s (0) = 8, s (2) = 0, s (1) = 15

a f

af

1
1
9. f 3 = 3 , f 2 =
2
9
10. f (1) = 1, f (0) = 1, f (2) = 1, f (2) = 17, f (a)
= 2a2 4a + 1, f (x + 8) = 2x2 + 28x + 97
11. f (2) = 7, f (1) = 0, f (0) = 5, f (a + 1) = a2 + 6a,

F 1 I = 1 + 4a 5a , f a5f = 0
f
H aK
a
F I = 0, f (0) = 1, f FGH IJK = 1 , f FGH IJK
12. f
3
2
6
H 2K
3
=
, f a f = 1
2
F I = 2 3
13. f
H 3K
2

F 1 I , f (2 h), f (1 + h),
H 2K

FG f F 1 I IJ , where h > 0 is sufficiently small.


H H 2KK

2. If f (x) = 2 + 3x, when 1 < x < 1


= 3 2x, when 1 < x < 2
find f (0), f (1), f (1 +h), f (2 h), f (2), f (f (1.5)
3. If f (x) = 3x, when 1 < x < 0
= 4, when 0 < x < 1
= 3x 1, when 1 < x < 3

x
, when x 0
x
= 1, when x = 0
find f (0), f (2), f (2)
5. If f (x) = x, when x < 0
= x2, when x < x < 2
= 2x, when 2 < x

e j= x

15. f x

af

4. If f x =

h +h+1

14. f 1 + h =

find f (2), f (0), f (0.5), f (5), f (3) and f

find f (3), f (2) and f

h + 3h + 3
2

F x + 1IJ
and b f a x fg = G
H x 1K
1

x +1
2

F I
af
af
H K
F 3I = 2
f a 1f = c , f a1f = c , f
H 2K

3
1
9x
16. f 0 = 3 , f 3 = , f
=
2
4
x
3x + 1

17.

Type 2: When the given function is a piecewise


function.

af

F 1I
H 2K

sin x
, when x 0
x
= 0, when x = 0

6. If f x =

find f

F I , f (0) and f F I
H 3K
H 6K

7. If f : R R where
f (x) = 2x + 5, when x > 4
= x2 1, when x 9 , 9
= x 4, when x < 9
find f (15) and f (f (5))

FG f F 1 IIJ
H H 2 KK

85

Function

8. A function f is defined on R is follows:


f (x) = 0 if x is rational
= 1 if x is irrational

F 1I , f e 2 j and f (0)
H 3K

find f

9. If f (x) = x2 + 2, when 0 < x < 2


= 5, when x = 2
= x 1, when 2 < x < 5
= x + 1, when x > 5
find f (1), f (0), f (2), f (3), f (5), f (7), f (2 + h), f (5 + h)
and f (2 h).
10. A function f is defined as follows:
f (x) = 2x + 6 for 3 < x < 0
= 6 for 0 < x < 2
= 2x 6 for 2 < x < 5
find f (1), f

F 1 I and f (4)
H 2K

Answers:
1. f (3) = 6, f (2) = not defined, f (0) = not defined

F 1 I = 3 , f FG f F 1 I IJ = 1 , f (1 + h), f (2 h) =
H 2K 4 H H 2KK 4

h2 4h + 3.
2. f (0) = 2, f (1) = not defined, f (1 + h) = 1 2h,
f (2 h) = 2h 1, f (2) = 1, f (f (1.5)) = 2.
3. f (2) = 5, f (0) = 4, f (0.5) = 4, f (5) = not defined, f (3)

F F 1I I F
= 8, f G f J = f G 3
H H 2 K K GH

1
2

I F 1 IF 1 I
JJ = f GH 3 JK GH as 0 < 3 <1JK
K

4. f (0) = 1, f (2) = 1, f (2) = 1.

F 1I = 1 .
H 2K 4
F I = 2 , f a0f = 0 , f F I = 3 .
f
H 2K
H 6K

5. f (3) = 6, f (2) = 2, f

6.

7. f (15) = 19, f (f (5)) = 53.


8. f

F 1I = 0 , f e 2 j = 1 , f a0f = 0 .
H 3K

9. f (1) not defined, f (0) = 2, f (1) = 3, f (2) = 5, f (3) =


2, f (5) = not defined, f (7) = 8, f (2 + h) = (2 + h) 1, f (5
+ h) = (5 + h) 1, f (2 h) = (2 h)2 + 2.

F 1 I = 6, f (4) = 2
H 2K

10. f (1) = 4, f

Type 3: Problems on showing f (a) = f (b).


Exercise 1.3
1. Given the function f (x) = x4 x2 + 1, show that f (1)
= f (1).
2. Given the function f (x) = x4 + x2 + 5, show that f (2)
= f (2).
3. Given the function f (x) = x3 + x, show that f (1) =
f (1).
4. Given the function f (x) = x5 + x3, show that f (2) =
f (2).
5. If f (x) = x4 x2 + 1, show that f (x) = f (x).
6. If f (x) = sin x + tan x, show that f (x) = f (x).
7. Given that f (t) = at, show that f (x) f (y) = f (x + y).

af

8. If f x =

af

9. If f x =

1+ x
, show that f
x
x

1 x

af

10. If f x =

11. If

tan

2
2

F 1 I = f a xf .
H xK
a f

, show that f sin = tan .

a f

FI
HK

1 x
2
, show that f cos = tan
.
1+ x
2

af

1+ x
, show that
1 x

f x =

a f

f tan =

F + I .
H4 K
af

12. Given that f x =

x 1
show that
x +1

FG x 1IJ = 1
H x + 1K x
f aa f f abf
ab
=
(ii)
1 + f aa f f abf a + b
t
1
+ t , show that f (t) f (t).
13. If f at f =
2
(i) f

a 1

86

How to Learn Calculus of One Variable

14. If f (x) = log x, show that f (x y) = f (x) + f (y) and


f (xm) = mf (x).

af

15. If f x =

a a
a +a

, show that f x + y

af af
af af
a2 x 1f a x 2f , show that f F 1I =
16. If f a x f =
H cK
x 2x + 1
f ac f .
=

f x + f y
.
1+ f x f y

17. If f (x) = sec x + cos x, show that f (x) = f (x).


18. If f (x) = x4 + x2 2 cos x, show that f (x) = f (x).

af

2x 1
, show that f (y) = x.
x2

af

x +1
, show that
2x + 3

19. If y = f x =
20. If y = f x =

af

ax + b
, show that f (y) = x.
cx a

af

4x 3
, show that f (y) = x.
3x 4

af

23. If f x = b

FG x a IJ + a FG x b IJ , show that
H b a K H a bK

f (a) + f (b) = f (a + b).


24. If f (x) = log x, x > 0, show that
(i) f (x y) = f (x) + f (y)
(ii)

x + 2x + 3

af

x + 5x + 9
is not defined
x +1

F xI
f G J = f a x f f a yf
H yK

x + 4x 5

2. Show that f x =
for x = 1.

af

3. Show that f x =

x=

1
.
2

af

x + 3x + 1
2

4x 4x + 1

is undefined for

4. Show that f x =

x 3x + 5
2

2 x + 5x 3

is indeterminate

at x = .

af

1
2

x 5x + 6

is not defined for

a f a x 2f a x 3f

6. Show that f x =

is non-

existent for any value of x lying between 2 and 3.

a f a1 xf ax 1 2f ax 3f

7. Prove that f x =

is

not defined for 1 < x < 2.

a f a1 xf a x 2f a x 3f

8. Prove that f x =

is

not defined for any value of x lying between 1 and 2


but defined for any value of x lying between 2 and 3.

af

sin x cos x
is not defined
1 tan x

af

sin x
is not defined for
1 cos x

9. Prove that f x =

.
4

(iii) f (e x) = f (x) + 1
(iv) f (xn) = nf (x)

for x =

25. If f (x) = cos x, g (x) = sin x, show that


(i) f (x + y) = f (x) f (y) g (x) g (y)
(ii) g (x + y) = g (x) f (y) + g (y) f (x)

10. Show that f x =

2. Examining the Existance of a Function.

is non-existent

for x = 1.

x = 2 and for x = 3.

af

22. If y = f x =

af

1. Show that f x =

5. Show that f x =

3x + 4
f y = 8 x + 11 .
21. If y = f x =

Exercise 1.4

x = 0.

Function

af

11. Show that f x =

for x =

.
4

af

12. Show that f x =


for x =

sin x cos x
does not exist
1
cos x
2

sin x cos x
is non-existent
1 tan x

.
4

af

1
tan x is undefined at
cos x

af

14. Show that f x =

1
is not defined at x = 0.
x

af

15. Examine whether f x =

cos x sin x
1+

2 cos x

exists at

3
and x =
.
4
4
16. Find x for which the following functions are not
defined.
x=

x+5
(i) y =
3x 4
2
x

(iii) y =

x1
2x 5

(iv) y =

x +1
x +1

Hint: To find the points at which rational functions


of x are not defined, one should put denominator of
rational function of x equal to zero and solve for x.

15. f (x) exists at x =

and f (x) does not exist at


4

3
.
4

4
5
(ii) x = 0 (iii) x =
(iv) x = 1
3
2

(v) x = 2 (vi) x = 3 (vii) x = 1, 2 (viii) x = or


2

3
x=
and in general x = any odd multiple of .
2
2

(ix) x = 2n + 1 , n Z (x) x = n , n Z
2
3. Finding the Domain of a Given Function
16. (i) x =

3.1. Finding the domain of a given algebraic


function
Type 1: When the given function is a polynomial
in x.
Exercise 1.5

x + 3x + 2
x2
2

(vi) y =

x 3x + 2

(viii) y = tan x
(ix) y = sec x
(x) y = cosec x

x=

x= .
2

(v) y =

x +2

(vii) y =

Answers:

13. Show that f x =

(ii) y =

87

x + 4x + 3
x+3

Find the domain of each of the following functions:


1. y = x2
2. y = x2 1
3. y = x3 + 1
Answers:
1.

a , f 2. a , f 3. a , f

Type 2: When the given function is a rational


function of x.

88

How to Learn Calculus of One Variable

Exercise 1.6

14. y =

x +1
x+1

Find the domain of each of the following functions:

1. y =

1
4x 2

15. y =

x + 3x + 2
x2

2. y =

x+2
2x 8

16. y =

x + 4x + 3
x+3

3. y =

x 4
x+2

4. y =

1
1 x

5. y =

6. y =

17. y =

1.
2.

4x 1

3.

3 x 5x 2

4.

x 1

5.

7. y =
8. y =

x 9 x + 20

6.

x
2

x 3x + 2

7.
8.

9. y =

x 3x + 2
2

x + x6
2

10. y =

x 4x + 9

9.

F , 1 I F 1 , + I
H 2K H 2 K
a , 4f a4 , + f
a , 2f a2 , + f
a , 1f a1, 1f a1 , + f
a , 3f a3 , 4f a4 , + f
F , 1I F 1 , 2I a2 , + f
H 3K H 3 K
a , 4f a4 , 5f a5 , + f
R k1 , 2p
a , 3f a3 , 2f a2 , + f

10. R
11. R

x + 4x + 9

11. y =

x+5
3x 4

12. y =

2
x

13. y =

x 1
2x 5

x 3x + 2

Answers:

x x 12

x +2

RS 4 UV
T3W

12. R {0}
13. R
14.
15.
16.
17.

RS 5 UV
T2 W

R {1)
R {2}
R {3}
R {1, 2}.

Function

Type 3: When the given function is put in the form:


y=

af

f x , where f (x) = ax + b.

Exercise 1.7

89

8. y =

x 3x + 2

9. y =

2x x

Answers:
1.

LM 3 , 3 OP
N 2 2Q

2. y = 18 6x

2.

x 1

3. y =

3x 12

3.

4. y =

x3

4.
5. [5, 2]

5. y =

3 2x

6.

6. y =

2x 3

7.

Find the domain of each of the following functions:


1. y = 1 x

Answers:
1.
5.

a , 1 2. a , 3 3.
F , 3 OP 6. LM 3 , + I .
H 2Q N2 K

4, +

4.

3, +

8.

a , 2 4 , + f
a , 5 3 , + f

a , 1 3 , + f = R 1 , 3
a , f
a , 1 2 , + f .

Type 5: When the function is put in the form:


y=

a f , where f (x) = ax + b or ax + bx +c.


2

f x

Type 4: When the given function is put in the form:


y=

af

Exercise 1.9

f x , where f (x) ax2 + bx + c.

Find the domain of each of the following functions:


Exercise 1.8
1. y =
Find the domain of each of the following functions:
1. y =
2. y =

9 4x

1
x+2
1

2. y =

16 9 x

x 1
2

3. y =

x 2x 8

4. y =

x + 8 x + 15

5. y =

a2 xf a5 + xf

6. y =

x 4x + 3

7. y =

3x 4 x + 5

3. y =

1
2

x 3x + 2

4. y =

5. y =

1
2

4 x + 8 x 3
1
3 + 2x x

90

How to Learn Calculus of One Variable

gb

g b x 3g 0 , x 2
D a y f = 1 , 2 f 3 , f
x +1 x2

6. y =

a1 xf ax 2f
1

7. y =

6x

Answers:
1.
2.
3.
4.

a2 , + f
F 4 , 4I
H 3 3K
a , 1f a2 , + f
F 1 , 3I
H 2 2K

Hint:

D y = , 3 2 , 5

af a

a , 6f .

Type 6: When the given function is put in the form:

x 1
x +1

2. y =

x8
12 x

3. y =

4x 8
3 6x

a x + 1f a x 3f
a x 2f
a x + 1f a x 3f 0 , x 2

1. , 1 1 ,
2. [8, 12)

x2

F 1 , 2OP
H2 Q
4. 1 , 2f 3 , + f
5. a , 1 a3 , f
6. b , 3 b2 , 5g
3.

Type 7: When the given function is put in the form:

y=

af
f a xf

f1 x
2

4. y =

Hint:

Answers:

Find the domain of each of the following functions:


1. y =

f a f

Exercise 1.10

a x + 3f
a2 x f a x 5f
a x + 3f a2 xf a x 5f 0

7.

y=

6. y =

x 3 , 2 , 5 but x 2 , 5

af
af

x 1
x3

5. y =

5. (1, 3)
6. (1, 2)

f x
g x

Exercise 1.11
Find the domain of the following:
1. y =

x
2

x 3x + 2

Function

91

Type 3: When the given function is put in the form: y


= loga logb logc f (x).

Answer:
1. R [1, 2]
3.2. Problems based on finding the domain of a given
logarithmic function:
Type 1: When the given function is put in the form: y
= log f (x).
Exercise 1.12

Exercise 1.14
1. y = log2 log3 log 4 x
2. y = log log

x
2

Answers:
Find the domain of each of the following functions:
1. y = log (6 4x)
2. y = log (4x 5)
3. y = log (x + 8) + log (4 x)
4. y = log (x 2)
5. y = log (3 x)
6. y = log (3x2 4x + 5)
7. y = log (x2 x 6)
8. y = log (5x x2 6)

a f 2. a2 , f

1. 4 ,

3.3. Finding the domain of inverse circular functions


Type 1: When the given function is put in the form: y
= sin1 f (x) or cos1 f (x).
Exercise 1.15

Answers:

Find the domain of each of the following functions:


1. y = sin1 (1 2x)

1.

2. y = sin

2.
3.
4.
5.
6.
7.

F , 3I
H 2K
F 5 , + I
H4 K
a8 , 4f
a2 , f
a , 3f
a , f
a , 2f a3 , f

Exercise 1.13

F 2x I
H 3K

3. y = cos1 4x
4. y = cos

F xI
H 2K

5. y = cos1 (3x 1)
6. y = cos1 2x
7. y = sin1 (x 2)
Answers:
1. [0, 1]

8. (2, 3).
Type 2: When the given function is put in the form; y
= log | f (x) |.

2.

3.

LM 3 , 3 OP
N 2 2Q
LM 1 , 1 OP
N 4 4Q

4. [2, 2]
Find the domain of each of the following functions:
1. y = log | x |
2. y = log | x 2 |
3. y = log | 4 x2 |
Answers:
1. R {0} 2. R {2} 3. R {2, 2}

5.

LM0 , 2 OP 6. Find 7. [1, 3].


N 3Q

3.4. Finding the domain of a sum or difference of


two functions:

92

How to Learn Calculus of One Variable

Exercise 1.16

9.

Find the domain of each of the following functions:


1. y =

x 3 + 1 x

2. y =

4x +

x5

3. y =

x 1+

6 x

x 1+

5. y =

2x x +

af

3. y =

f2

Exercise 1.17

Find the domain of each of the following functions:


1. y = | x 2 |

8x 4 x 3
2x x

8x 4 x 3

af

D y = D1 D2 =

= [0, 2] = D1 (say) and

F 1 , 3 I = D (say)
H 2 2K
2

F 1 , 3I
H 2 2K
2

7. y =

x 2 3x + 2 +

8. y =

x2
+
x+2

4x

10. y =

x +

Find the range of the following functions:


1. y = x | x |
2. y = 11 7 sin x
3. y = 3 sin x + 4 cos x

3 + 2x x2

R|
S|
T

1 x
1+ x

4. y = sin log
3

+ log10 x x

5. y =

Answers:
1. 2. 3. [1, 6] 4. , 1 2 ,

I
K

1 3
,
5.
6. {1} 7. 1 , 1 2 , 3
2 2
8. Defined no where

F
GG
H

4x
1 x

x
x

1
3 cos 2 x
1
7. y =
2 cos 3x
6. y =

1
log10 2 x 3
x2

1. R 2. R {0} 3. [1, 2)

Exercise 1.17.1

x +1

Answers:
3.6. Finding the range of a function:

x 1 + 2 1 x +

x
x
3. y = cos1 [x]

2. y =

6. y =

F
H

af
f a xf
a xf f a xf

domain of

a2 , f .

2. y = f x g x

x 3x + 2

Hint: Domain of

10.

3.5. Finding the domain of a function put in the


forms:
1. y = | f (x) |

4. y =

9. y =

a1 , 0f a1 , 2f a2 , f

8. y = log3 (5 + 4x x2)
9. y = x [x]
10. y = [x] x

I U|
JJ V
K |W

Function

Answers:

a , f 2. [4. 18] 3. [5, 5] 4. [1, 1] 5. {1,


L 1 1 O L1 O
1} 6. M , P 7. M , 1P 8. a , log 9 f 9. [0, 1)
N 4 2 Q N3 Q
1.

10. (1, 0].

Hint for (2): 1 sin x 1


7 7 sin x 7
11 7 11 7 sin x 7 + 11
4 11 sin x 18

Hint for (8): y = log 3 5 + 4 x x

4x x

j3

=5+

=9 x2
y

Answers:
1. D (y) = R;
R (y) = {2, 2}
2. D (y) = R
R (y) = {4, 1, 3}
3. D (y) = R
R (y) = R {3}
4. D (y) = R

af

R y = 0,
5. D (y) = R

af

af a

R y = 4 ,
6. D (y) = R
R y = , 6

>0

0 < 3 9
< y log 3 9
Exercise 1.17.2

93

4. Finding the composite of two function


Exercise 1.18

F
H

1. If f (x) = tan x, x

I
K

af

2

and g x = 1 x ,
,
2 2

Find the domain and range of each of the following


functions:

find (gof) (x).

1.

3. If f (x) = e2x and g x = log x , x > 0 find (gof) (x).

2.

3.

R2 , if x 3
y=S
T 2 , if x > 3
R|4 , if x < 2
y = S1 , if 2 x 2
|T3 , if x > 2
R2 x 1, if x 3
y=S
T 0 , if x = 3
R|
y = S 25 x , if x 5
|T x 5 , if x 5
R|x 4 , if x < 3
y=S
T| 2 x 1 , if x 3
R6 x + 7 , if x 2
y=S
T 4 x , if x > 2
2

4.

5.

6.

af

2. If f x =

af

4. If f x =

x and g (x) = | x |, find (gof) (x).

af

x +1
, x 2 x being real and g (x) =
x+2

x2, find (gof) (x).

af

5. If f x =

af

x
1
and g x = find (gof) (x).
x
x

6. If f : R R is defined by f (x) = sin x, x R and

g : R R is defined by g (x) = x2, compute (gof) (x)


and (fog) (x).
7. If f : R R is defined by f (x) = 2x2 1 and

g : R R by g (x) = 4x 3, x R compute (gof) (x)


and (fog) (x). Also find (gof) (2) and (fog) (1).
8. If f : R R is defined by f (x) = x2 3x + 2 and
g : R R by g (x) = 4x + 3, x R compute (gof) (x)
and (fog) (x). Hence find the values of (gof) (3) and
(fog) (3).

94

How to Learn Calculus of One Variable

9. If f : A B is defined by f (x) = x + 1, x R and

g : B C by g (x) = x2, find (gof) (x).


10. If f (x) = cos x, g (x) = x3, x R , find (gof) (x) and
(fog) (x).
11. If f (x) = x2 + 2 and g (x) = x 1, x R , find (fog)

11. (fog) (x) = x2 2x + 3; (gof) (x) = x2 + 1; (fog) (2)


= 11 and (gof) (2) = 5
12. (gof) (x) = 12x2 + 36x + 25; (fog) (x) = 6x2 1; (gof)
(2) = 145 and (fog) (2) = 23
13. (gof) (x) = (1 x)2
16. | x |.

(x) and (gof) (x). Hence, find (fog) (2) and (gof) (2).
12. If f (x) = 2x + 3 and g (x) = 3x2 2, x R , find (gof)

Exercise 1.19

(x) and (fog) (x). Hence, find the values of (gof) (2)
and (fog) (2).
13. If the mapping f : A B is define by f (x) = log

Find the domain and range of each of the following


ones:
1. sin (1 + cos x)

(1 x) and the mapping g : B C is defined by g (x)


= e2x, find (gof) (x).
14. If the mapping f : R R be given by

2.

3. cos sin x
2

af

1
f x =1+
and the mapping g : R R be
x 1

a faxf (fog)

given by g (x) = x2 + 1, show that gof


(x).

af

15. If f is defined as f x =

1
, show that f [f {f
1 x

(x)}] = x, x 0 , 1 .
16. If f (x) = | x |, find f [f (x)].

4. tan sin x
5. log 1 x
6.

2.

1 tan x
x =

3. log e
4.
5.

2x

FG x + 1 IJ
H x + 2K

=x

x
, x0
x

6. (gof) (x) = sin2 x and (fog) (x) = sin x2


7. (gof) (x) = 8x2 7, (fog) (x) = 32x2 48x + 17, (gof)
(2) = 25 and (fog) (1) = 97
8. (gof) (x) = 4x2 12x + 11, (fog) (x) = 16x2 + 12x + 2,
(gof) (3) = 11 and (fog) (3) = 182
9. (1 + x)2
10. (gof) (x) = cos3 x (fog) (x) = cos x3

log 1 x

FG 1 x IJ
H1 + xK
F 1 x IJ
sin log G
H1 + xK

7. log

8.

Answers:
1.

a f

sin cos x

9. tan (sin x + cos x)


10. tan (sin x + 2cos x)
11. cos | sin1 x |
12. loge loge x
13. loge loge loge x
14. loge loge loge loge x
15. sec1 sin x
16. sin sec1 sin x
17. log (1 + sin sec1 sin x)
18. cos sin1 x
Answers:

a f
O
L
D = M2n , 2n + P n Z ; R =
2Q
N 2
D = 2n , a2n + 1f ; R = cos1 , 1

1. D = , ; R = 0 , 1
2.
3.

0 , sin 1

Function

a f
D = a 1 , 1f ; R = a , 0f

(iii) Find h (h (x)) if

4. D = , ; R = 0 , tan 1
5.
6. D = {0}, R = {0}

a f

7. D = 1 , 1 ; R = ,
8. D = (1, 1); R = [1, 1]

a f RST5x+x1, ,xx>11

h x =

(iv) Find i (i (x)) if

R| x , x < 0
i axf = Sx , 0 x 1
|T2 x , x > 1

9. D = , ; R = tan 2 , tan 2
10. D = , ; R = ,

Answers:

11. D = 1 , 1 ; R = 0 , 1

a f a f
D = ae , f ; R = a , f
D = ee , j ; R = a , f

12. D = 1, ; R = ,
13.
14.

17.

1. Hint: fog (x) = f (g (x)) = f

a f 2 ; R = k0 , p

D = a2n + 1f ; R = k0p
2

D = a2n + 1f ; R = k0p
2

15. D = 2n + 1
16.

95

FG
H

x +1

IJ
K

FG
H

x +1

1 = x + 1 1 = x

IJ
K

hofog (x) = h (x2) = 0

b a fg |RS|xx ,,xx <00


T
R4 + x , when x 0
g b g a x fg = S
T4 x , when x < 0
4

2. (i) f f x =

(ii)

18. D = 1 , 1 ; R = 0 , 1

R|x + 2 , x 0
|5 ax + 1f , 0 , x 1
h bh a x fg = S
||5 e5 x j , 1 < x < 2
T6 x , x 2
2

Exercise 1.20

(iii)

1. If the functions f, g and h are defined from the set


of real numbers R to R such that f (x) = x2 1,

a f x +1
R0 , when x 0
h a xf = S
T x , when x < 0
g x =

then find composite function hofog.


2. (i) Find f (f (x)) if

af

f x =

R|x , x 0
S| x , x < 0
T
2

2 2

(iv)

R|2 + x , x < 1
| x , 1 x < 0
i bi a x fg = S x , 0 x 1
||2 x , 1 < x 2
|Ta2 xf , x > 2

5. Eveness and oddness of y = f (x).


Exercise 1.21

(ii) Find g (g (x)) if

a f RST22 + xx,, xx < 00

g x =

(A) By considering f (x), discover which of the


following are even functions, which are odd functions
and which are neither even nor odd.

96

How to Learn Calculus of One Variable

1. f (x) = x4 + 7x2 + 9

af

12. Odd 13. Neither even nor odd 14. Odd

2. f x = x + x +

af
f a xf =

3. f x =
4.

(B) 1. e =

x + 16

1
4x + 9

5. f (x) = sin x
6. f (x) = cos x
7. f (x) = tan x
8. f (x) = sin x + cosec x
9. f (x) = sin x + tan x
10. f (x) = cosec x + tan x
11. f (x) = tan2 x
12. f (x) = tan3 x
13. f (x) = sin x + cos x

af

cos x
x

(B) Express the following functions as the sum of an


even and odd functions.
1. f (x) = ex
2. f (x) = (1 + x)100

af

3. f x = sin 2 x + cos

F x I + tan x
H 2K

4. f (x) = x2 + 3x + 2
5. f (x) = 1 x3 x4 2x5
(C) Let f : 2 , 2 R be a function. If for
x 0, 2 ,

R|x sin x , 0 x
2
f a xf = S
|T 2 a xf , 2 < x 2
define f for x 2 , 0 when
(i) f is an odd function.
(ii) f is an even function.
Answers:
(A) 1. Even 2. Odd 3. Even. 4. Even 5. Odd
6. Even 7. Odd 8. Odd 9. Odd 10. Odd 11. Even

100

1
1+ x
2

100

+ 1 x

100

a f
F x I + tan x
3. sin 2 x + cos
H 2K
1
F x I + tan x + sin a2 xf +
= [sin 2 x + cos
H 2K
2
F x I + tan a xf]
cos
H 2K
1
F x I + tan x {sin a2 xf +
+ [sin 2 x + cos
H 2K
2
F x I + tan a xf}]
+ cos
H 2K
1
1+ x
2

14. f x = x +

2. 1 + x

x
x
1 x
1 x
+
e +e
e e
2
2

100

1 x

4. x + 3x + 2 =

100

1 2
2
x + 3x + 2 + x 3 x + 2 +
2

1 2
2
x + 3x + 2 x + 3 x 2
2
3

5. 1 x x 2 x =

e1 + x

x + 2x ] +

(1 + x x + 2 x )]
(C) For oddness,

1
3
4
5
[1 x x 2 x +
2

1
3
4
[1 x x 2 x
2

R| axf , 2 x
2
f a xf = S 2

|T x sin x , 2 x 0

for eveness,

Function

R| axf , 2 x <
2
f axf = S 2

|T x sin x , 2 x 0

Exercise 1.22
(A) Find the periods of the following functions:
1. f (x) = cos 3x

af

2. f x = cos

F xI
H 4K

3.

6.

F
H

12.
13.
14.
15.
16.
17.
18.
19.

F x I + 2 sin F x I
H 2K
H 3K

(c) 2 (d) none of these


2

Answers:

2
2. 8 3.
4.
5. 5 6. 2
2
3
3

2
2
2
7.
8.
9.
10.
11. 12.
5
3
3
3
2
13. 14. 15. 16. 17. 2 18.
3

19.
3
2
(B) 1. 2 2. 24 3. 6 4. 16 5. 2 6.

7. 12 8. 2 9. 10. 2
(A) 1.

I
K
F I
f a x f = 4sin 3x +
H 4K
f a x f = tan x

9. f x = 2sin 3x +

11.

7. y = 3 cos

(a) (b)

7. f (x) = sin 10x


8. f (x) = 10 sin 3x

10.

5. y = 2 sin x + 3 cos 2x
6. y = a sin x + b cos x

(C) The periods of f (x) = | sin x | + | cos x | will be

F xI
f a x f = cot
H 5K
F xI
f a x f = cot
H 2K
af

4.

8. y = 3 sin x 4 cos 2x
9. y = 1 + tan x
10. y = tan 2x cos 3x

3. f (x) = tan 2x
4. f (x) = tan 3x
5.

F x I 3 sin F x I
H 4K
H 3K
F 2 x I 4 cot F 3x I 2
y = tan
H 3K
H2K
F 3x I 3cos F 5x I + cos 5x
y = sin
H4K
H 8K

2. y = sin

5. Periodic Functions
Type 1: Finding the periods of trigonometric
functions.

97

10

f (x) = | cos x |
f (x) = | sin x |
f (x) = tan1 (tan x)
f (x) = cos2 x
f (x) = sin2 x
f (x) = sin3 x
f (x) = log (2 + cos 3x)
f (x) = ecot3x

(B) Find the periods of the following functions:


1. y = sin 5x cos 4x + 1

2
Type 2: Problems on showing that a given function
is not periodic.
(C) (b)

Exercise 1.23
1. Show that the following functions are not periodic:

af

(i) f x = sin

98

How to Learn Calculus of One Variable

F 1 I , x 0 , f a0f = 0
H xK

af

(ii) f x = sin

(iii) f (x) = sin x2

af

(iv) f x = cos
(v) f (x) = cos x2

2. Show that the following functions are not periodic:


(i) f (x) = x cos x

af

(ii) f x = x sin
(iii) f (x) = cos

af

x2

F 1I
H xK

+ sin

(iv) f x = sin x + cos x


(v) f (x) = x + sin x
(vi) f (x) = x + cos x

Answers:
3. (i) Non-periodic (ii) Periodic but has no
fundamental period (iii) Not periodic.
7. Examining a one-one and on-to function
Exercise 1.24

x 8 x + 18
2

x + 4 x + 30

a one-one

function?

af

2. Verify whether the functions f x =

af

Is f one-one-onto?
11. Show that the mapping
(i) f : R R defined by f (x) = x, x R is oneone-onto.
(ii) f : R R defined by f (x) = x3, x R is oneone-onto.
(iii) f : Q Q defined by f (x) = 2x + 3, x Q is
one-one-onto.
(iv) f : R R defined by f (x) = 4x + 3, x R is
one-one-onto.

x + x +1

af

1+ x

1+ x

12. Is

is one-one.
3. Let f : R R be defined by f (x) = ax + b, a, b
being fixed real numbers and a 0 , show that f is
one-one and on-to.
4. If f x =

a f 2x (ii) h (x) = x | x | (iii) k (x) = x


(iv) a x f = sin x (v) g (x) = | x |
x
.
10. Let f : R R be defined by f a x f =
(i) f x =

3. Examine which of the following functions are


periodic:
(i) f (x) = [x] (ii) f (x) = 5 (iii) f (x) = x [x]

af

a f xx 23 is f bijective? Give reasons.


= kx : 1 x 1p = D for each of the

given by f x =

9. Let D1
2
following functions from D1 to D2, find whether it is
surjective, injective or bijective:

x2

1. Is the function f x =

6. If f : 0 , 2 1 , 1 be defined by f (x) = sin x,


show that f is on-to but not one-one.
7. Show that the following functions f : R R are
both one-one and on-to:
(i) f (x) = x3 (ii) f (x) = 3x + 4
8. Let D1 = R {3}, D2 = R {1} and f : D1 D2 be

f x =

the

f:RR

map

defined

by

x + 4 x + 30

one-one?
2
x 8 x + 18

, D2 = 1 , 1 , show that the
13. Let D1 = ,
2 2

LM
N

OP
Q

map f : D1 D2 defined by f (x) = sin x is bijective.


Answers:

is the function one-to-one?

5. If f : 0 , 2 1 , 1 be given by f (x) = sin x,


show that f is onto but not one-one.

1. No 2. No 4. No
8. Yes, since f x1 = f x 2 x1 = x 2 and
3y 2
x=
= f y which is true, y R 1
y 1
which means f is onto.

af

a f a f

kp

Function

9. (i) Injective but not surjective (ii) bijective (iii)


Neither injective nor surjective (iv) surjective but not
injective (v) Neither injective nor surjective
10. Not 12. Not
On the Graph of a Function
First of all we would like to define domain and range
of a function in terms of projection of the graph of y =
f (x) on axes.
1. Domain of a function: The projection of the graph
of y = f (x) on the x-axis is called the domain of the
function y = f (x).
2. Range of a function: The projection of the graph
of y = f (x) on the y-axis is called the range of the
function y = f (x).
Y
B
f (x )
y=

Y
D

D (f) = CD

y = f (x )

If the domain of a function f is a finite interval, the


graph of f can be explicitly plotted in a plane, but if the
domain of f is an infinite set, it is not possible to plot
all these points. In such case, we plot enough point to
get an idea of the general shape of the graph of y = f (x).
The following characteristics of a graph of a
function are worth noting.
1. The graph of a function is a subset of the plane
and it is uniquely determined by the function.
2. For each a in the domain of f, there exists exactly
one point (a, f (a)) on the graph of the function f,
since by our definition of the function, the value of f
at a is uniquely determined. Geometrically, it means
that a D f the vertical line x = a meets the
graph of f in one point only. again, a D f the
vertical line x = a does not meet the graph anywhere
at all, i.e. the point (a, f (a)) is missing (absent) on the
graph of f, i.e. there is a hole in the graph of f at a, i.e.
the graph of f is broken (not unbroken) at x = a since
there is a point on the graph of f whose abscissa is a
but there is no ordinate corresponding to the abscissa
x = a and such points with no ordinate at an abscissa
x = a can not be located on the graph of a function.

af

af

On the Method of Graphing a Function

R (f ) = CD

99

B
X

We speak of the graph of a point P (x, y) meaning the


point representing the ordered pair (x, y).We also
speak of graphing a point P (x, y), meaning to construct
and locate the point P on a coordinate plane (a plane
with axes).
Now we define what is the graph of a function.
Definition: The graph of a function f defined on its
domain in a coordinate plane is the graph of the set
G {(x, f (x): x is in the domain of f}
or, equivalently, the graph of a function f (x) defined
on its domain is the graph of the equation y = f (x).
That is, the point P (a, b) is on the graph of y = f (x)
b = f (a).

Mainly there are two methods of graphing a function


which are:
1. The method of plotting a graph point by point.
2. The method of plotting a graph by derivative.
Hence, we use point by point method to draw
the graph of a function defined by the formula y = f (x)
in its domain unless we learn how to find the
derivative of a function.
How to Draw the Graph Using
Point by Point Method
1. Find the domain of the given function y = f (x)
2. Find the zeros of the given function, i.e. solve the
equation f (x) = o whose solutions divide its domain
into intervals where the function has the constant sign.
3. Examine whether the given function passes
through the origin, i.e. check whether (o, o) satisfied
the equation y = f (x).
4. Find the intercepts on the axes, i.e. the points where
the given curve cuts the x-axis and/y-axis.

100

How to Learn Calculus of One Variable

5. Find whether the given function is odd or even.


6. Find whether the given function is periodic.
7. Plot a few additional points to get an idea of the
general shape of the graph of y = f (x).
Notes:
1. If the function is periodic, it is sufficient to investigate
the behavior of the function on any closed interval
whose length is equal to the period of the function
and then constructing the graph on that interval,
extend it to the whole of the domain of the function.
2. The position of a straight line is determined if any
two points on it are known. Consequently in the case
of straight lines, it is advised to find two points only
instead of many to economise time, only with this
care that the points are not very close to each other.
3. One should give such values to x as will enable
him to get integral values of y since it is easier to plot
integral units than the fractional units.
4. The graph of an even function is symmetric with
respect to the axis of ordinates (i.e. y-axis) and the
graph of an odd function is symmetric with respect to
the origin.
On What is Symmetry of a
Curve with Respect to a Line
A curve is said to be symmetric with respect to a line
or symmetrical about a line) when all chords of the
curve drawn perpendicular to the line are bisected by
it. The lines of most importance in our discussion
of an algebraic curve are the x-axis, the y-axis the
lines bisecting the first and second quadrants which
are y = x and y = x.
If the curve is symmetric with respect to x-axis,
y = o, two points on the curve with the same abscissa,
x, will have the ordinates y and y. That means that
substitution of y for +y in the equation y = f (x) will
result in an equation that is same as or can be reduced
to the original equation. Thus, in an algebraic equation,
if only even powers of y are present, or every term of
the equation contains an odd power of y, the locus of
the equation is symmetric with respect to the x-axis.
Examples: The loci of the following equations are
symmetric with respect to the x-axis:
x2 + y2 = 25 (circle)
y2 = x (parabola)
y2 = x3 (semi-cubical parabola)

xy y3 = 0
A similar discussion will bring us to the conclusion
that if only even powers of x are present in the equation
y = f (x), or every term of the equation y = f (x) contains
an odd power of x, its locus is symmetric with respect
to the y -axis.
Examples: The loci of the following equations are
symmetric with respect to the y-axis:
x2 + y2 = 25 (circle)
x2 = y (parabola)
y3 x2 y = 0
If the curve is symmetric with respect to the line y
= x, a pair of symmetric points, lying on opposite
sides of this line will have their coordinates reversed.
This can be shown geometrically. It means that we
should be able to interchange the x and y in the
equation y = f (x) and obtain an equation that can be
reduced to the original one. Hence, if the x and y in an
algebraic equation can be interchanged without
producing an essentially different equation, the locus
of the equation will be symmetric with respect to the
line y = x.
Examples: The loci of x2 + y2 = 25 and xy = 1 are
symmetric with respect to the line y = x.
Similarly, if the subsitution of y for x and x for y
gives an equation that is essentially the same as the
original one, the locus of the equation y = f (x) is
symmetric with respect to the line y = x.
On What is Symmetry with
Respect to the Origin
A curve is said to be symmetric with respect to a
point (about a point) when the point bisects all chords
of the curve drawn through it. Such a point of
symmetry is called the centre of the curve. The most
important point for our study is, perhaps, the origin.
If the curve is symmetric with respect to the origin
and the point (x, y) is on the curve, then the point
(x, y) will be on the curve, a fact that is made
apparent by a figure. Hence, if substitution in an
algebraic equation of x for x and y for y results in
an equation that can be reduced to the original, the
locus of the equation is symmetric with respect to the
origin.
Examples: The loci of the following equations are
symmetric with respect to the origin.

Function

x2 + y2 = 25 (circle)
xy = 5 (hyperbola)
x2 y2 = 25 (hyperbola)
y = x3 (cubical parabola)
N.B.: A parabola is a U-shaped curve which cups
(curves into the shape of a cup) up or cups down.
x

1
0
1
2

k
k
k
k
k
k
k

The Graphs of y = f (x)


y

1. y = k

4
3
2
1

K
4

y=K

N.B.: A graph of the identity function y = x is a line


which
(i) Passes through the origin.
(ii) Bisects the angle between first and third quadrant
or it is a line with slope 1.
x
y
3. y = x
y
4
3

y = x

3
4

N.B.: A graph of a constant functions y = k is a line.


(i) Parallel to the x-axis.
(ii) Above the x-axis at a distance k from it if k > o.
(iii) Below the x-axis at a distance | k | from it if k > o.
(iv) On the x-axis if k = o.
x
y
2. y = x

N.B.: The graph of the function y = x is a line


(i) Passing through the origin and
(ii) bisecting the angle between the second and the
fourth quadrants, i.e. it is a line with slope 1.
x
y
4. y = mx + c.
y
m<0
C<0

y=mx+C
y=x

3
2

y
1

y
m=0
C>0

y=C

m<0
C=0

x
4

1
0
1
2

1
2
1
2

1
0
1
2

1
2
1
2

101

0
2
3
4

y=mx
x

102

How to Learn Calculus of One Variable

5. y =

8. y = x2

y
2

y = x
y
5

y =

x
y = x

y = x2

1
2
3

1
4
9

2
1

x
2

y=

x
2

0
1
4
9
16

0
1
2
3
4

N.B.:
1. The curve y = x2 is symmetric with respect to the
y-axis since f (x) = f (x), x R . Also, it lies on and
above the x-axis passing through the origin.
2. The curve y = x2 is symmetric with respect to the
y-axis since f (x) = f (x), x R . Also, it lies on and
below the x-axis passing through the origin.
On the Graphs of y = | f (x) |
The graphs of y = | f (x) | is the graph of the union
of two functions defined by
y = f (x), when f (x) > 0
or y = f (x), when f (x) < 0

6. y = sin2 x + cos2 x

Note: The conditions f (x) > 0 and f (x) < 0 imposed on


y = f (x) and y = f (x) determine the intervals where f
(x) is positive or negative whereas the condition f (x)
= 0 on f (x) > 0 determines the points where two curves
y = f (x) and y = f (x) intersect the x-axis.
How to Draw the Graph of y = | f (x) |

7. y = x2
x

y = x2

1
2
3

1
4
9

The method of procedure is to determine firstly the


intervals where f (x) is positive and negative and
secondly the points where y = f (x) and y = f (x)
intersect the x-axis.
The graph of a function y = | f (x) | is always obtained from the graph of the function y = f (x) whose
portion lying above the x-axis remains unchanged for
positive part of y = f (x) and the portion lying below
the x-axis as a plane mirror is taken as the image of
negative part of y = f (x) on and above the x-axis in the
required interval.

Function

How to Determine the Intervals


Where f (x) is Positive or Negative
1. Find the zeros of f (x), i.e. the roots of f (x) = o.
2. Partition the real line by zeros of f (x).
3. Consider the intervals:
, x1 , (x1, x2), (x2, x3) ... (xn1, xn), x n , , if
x1, x2, x3, , xn are the zeros of f (x) such that x1 = the
smallest number among all the zeros of f (x). xn = the
greatest number among all the zeros of f (x).
4. Check the sign (i.e. positivity and negativity) of
f (x) in each interval determined by the zeros of f (x).

5. The same method of procedure is applicable to


draw the graph of those functions containing absolute
value function.
The Graph of y = | f (x) |
1. y = | x |
The graphs of y = | x | is the graphs of the union of
two functions defined by
y = x and x > 0
or y = x and x < 0
on using the slope-intercept method, we graph y =
x where the domain of y is {x: x > 0}.

How to Check the Sign of


f (x) in Different Intervals

y
4

1. Take one particular point c belonging to each of


the adjacent intervals.
2. Put the particular point c in f (x).
3. Use the facts:
(i) : f c > 0 f x i.e. y is positive for every x
in that interval where c belongs.
(ii) : f c < 0 f x i.e. y is negative for every x
in that interval where c belongs.
For example, let us consider
y = x2 4x and y = 4x x2

af
af

af
af

In , 0

x
1

The equation y = x where x < 0 defines a function


whose domain is {x: x < 0}. Again using the slopeintercept method, we graph y = x.
y

a
a

f
f

4
3

a f
a f
a f
a f

f (x) = x2 4x > 0 for every x 4 ,


since f (5) = 5 > 0 where 5 4 ,
In (0, 4):
f (x) = x2 + 4x > 0 for every x 0 , 4
since f (1) = 3 > 0 where 1 0 , 4

Notes:
1. y = f (x) is positive in an interval
is positive in its subinterval.
2. y = f (x) is negative in an interval
is negative in its subinterval.
3. The graph of y = | f (x) | always lies on and above
the x-axis.
4. The graph of y = | f (x) | always lies on and below
the x-axis.

a f

f (x) = x2 4x > 0 for every x , 0


since f (1) = 5 > 0 where 1 , 0
In 4 , :

103

x
4 3

2 1

0 1
1

2
3
4

Also, {(x, y): y = | x |} = [(x, y): y = x and x > 0] [(x,


y): y = x and x < 0]
Now, we combine the last two graph to obtain the
graph as under:

104

How to Learn Calculus of One Variable

af a f
R a yf = 0 , f

135

1
45
0

6 5 4 3 2 1

f a

D y = , 2 2 , = , and

y=

x
2

2x
2

x2
2

y=

2
1

Note: A table of ordered pairs also easily can be


constructed and the absolute value function y = | x |
can be graphed as given below:
x

y=|x|

4
2
0
1
3

4
2
0
1
3

x
x
y = 1 for x > 0
or y = 1 for x < 0

4. y =

af a f
R a yf = 0 , f

f a

f a

f
y = x

y=

6 5 4 3 2 1

x2
for x > 2
2
2x
or y =
for x < 2
2

0 1

x
2

x
1

5. y = | x2 4x |
2

y=

x
3

y=1

x2
2

y = 1

D y = , 0 0 , = ,

2 x x

x
for x > 0
3
and y = x for x < 0

D y = , 0 0 , = ,
R (y) = {1, 1}

3. y =

y=

2. y =

af a

0 1

6 5 4 3 2 1

y = x 4 x for x 4 x 0 , i.e. x (x 4) > 0


i.e. x < 0 or x > 4
and y = (x2 4x) for x2 4x < 0, i.e. x (x 4) < 0,
i.e. 0 < x < 4

af a

a f

fa

D y = , 0 0 , 4 4 , = ,
and

af

R y = 0,

106

How to Learn Calculus of One Variable

The Graph of y = | f1 (x) | + | f2 (x) |

in 1 , 1

1. y = | x | + | x 1 |
x = 0 and x = 1 are the zeros of x and (x 1)

a f

, 0 , [0, 1] and 1, are required intervals


whose union is the domain of the given function y =
| x | + | x 1 |.

y= x+1 + x 1

y = (x + 1) (x 1) = x + 1 x + 1 = 2
since (x + 1) > 0 and ( x 1) < 0 in (1, 1)

a f

in 1 ,
y= x+1 + x 1

in , 0
y = x + x 1

y = x + 1 + x 1 = 2x

y = x (x 1) = x x + 1 = 2x + 1

since (x + 1) > 0 and (x 1) > 0 in 1,

a f

in 0 , 1
y= x + x1

y = 2x

y = x (x 1) = x x + 1 = 1

y = 2x

a f

in 1 ,
y= x + x1

2
1

y = x + x 1 = 2x 1
y

y = 2x + 1

y = 2x 1
y=1

Notes:
1. x + 1 > 0 and x 1 > 0 x > 1 y > 2 since x >
1 2x > 2 y > 2.
2. x + 1 < 0 and x 1 < 0 x < 1 y > 2 since x
< 1 2x > 2 y > 2.
3. The range of y = | x + 1 | + | x 1 | is the set of all real
numbers greater than or equal to 2, i.e. the semi-closed

interval 2 , .
2. y = | x + 1 | + | x 1 |
x = 1 and x = 1 are the zeros of (x + 1) and (x 1).

a f

, 1 , [1, 1] and 1, are the required


intervals whose union is the domain of the given
function y = | x + 1 | + | x 1 |.

in , 1
y= x+1 + x 1

y = (x + 1) (x 1) = x 1 x + 1 = 2x

since (x + 1) < 0 and (x 1) < 0 in , 1

3. y = 2 | x 2 | | x + 1 | + x
x = 1 and x = 2 are the zeros of (x 2) and (x + 1)

a f

, 1 , [1, 2] and 2 , are the intervals


whose union is the domain of the given function y =
2 | x 2 | | x + 1 | + x.

in , 1 :
y = 2 x 2 x +1 + x

y = 2 (x 2) + (x + 1) + x = 5
in 1 , 2 :
y = 2 x 2 x +1 + x

y = 2 (x 2) (x + 1) + x = 2x + 3

107

Function

a f

in 2 , :
y = 2 x 2 x +1 + x

y = 2 (x 2) (x + 1) + x = 2x 5

3
2

y
1

y=5

0 1
1

4 3 2 1

4
3

y = 2 x + 3

y = 2x 5

x
1

0 1
1

Hence, the given function can be rewritten as


under:
x < 1
5

R|
S|
T

y = 2 x + 3 1 x 2
2x 5 x > 2
Therefore, the graph of y = 2 | x 2 | | x + 1 | + x is
a polygonal line as above.
On the Graph of y = [f (x)]
The graph of y = [f (x)] is a set of horizontal line
segments (i.e. a set of line segments, each being
parallel to x-axis), each of which includes the left end
point but excludes its right end point. A small shaded
circle is put at the left end point of a horizontal line to
show the inclusion of that point and a small unshaded
circle o is put at the right end point of a horizontal line
to show the exclusion of that point as n
n+1
Moreover one should note that each horizontal
line representing the graph of y = [f (x)] always lies on
and below a straight line.
The graph of a function y = [f (x)] is always
obtained from the graph of y = f (x) where y = [f (x)] I
is marked on the y-axis of unit length such as [2, 1),
[1, 0), [0, 1), [1, 2) etc. for which horizontal lines are
drawn through integers till they intersect the graph.
Further, one should note that on y-axis for the form
[n, n + 1), y = n if y increases in its domain.
The graph of y = [f (x)]
1. y = [x]
y = x =0 x x <1

[x] = y

0 x<1

1 x < 2

2x<3

1 x < 0

2 x < 1

N.B.: The graph of y = [x] lies on and below the line y


= x.
2. y = [x]
y = x = 0 0 x < 1 0 x > 1

1 < x 0
y

4
3
2
1
0 1
4 3 2 1
1
2
3
4

108

How to Learn Calculus of One Variable

[-x] = y

2 < x 1

the line y = x + 1.
4. y = 2 [x] 1

1
0

1 < x 0
0< x1

1< x 2

LM x OP + 1 lies on and below


N2 Q

N.B.: The graph of y =

x+1

3
2
1

N.B.: The graph of y = [x] lies on and below the line


y = x.
x
3. y =
+1
2
x
x
= 0 0 <1 0 x < 2
y=
2
2

LM
N
LM
N

OP
Q
OP
Q

x
4 3 2 1

0 1
1

2
3
4

2 [x]

[2x] 1 = y

0 x<1
1 x < 2

0
2

1
1

2x<3

4
3
2
1
0 1
1

4 3 2 1

x
2

LM x OP LM x OP + 1 = y
N2 Q N2 Q

0 x<2

2x<4

2 x < 0

4 x < 2

N.B.: The graph of y = [2x] 1 lies on and below the


line y = x 1.
On the Graph of y = | [x] |
The graph of y = | [x] | consists of all parallel line
segments each being parallel to x-axis which lie on
and above the x-axis such that all parallel line segment
on the right side are on and below the line y = x and all
the parallel line segments on the left are on and above
the line y = x.
1. y = | [x] |

y =

x =0 x =00 x<1

109

Function
y

a f LMN x +2 4 OPQ
1
f a xf = x
x

(iv) f x =
4
3

(v)

2
1

x
4 3 2 1

0 1
1

Answers:
1. (i)

2
3
4

f (x) =

x + |x |
2

| [x] | = y
x

2 x < 1

1 x < 0

0 x <1

2x<3

(ii)
y

2
1

0 1
1

Exercise on Graphing the Functions

af

x+ x
2
(ii) f (x) = | x 3 | 4
(iii) f (x) = 2x [x], where [x] = greatest integer
function.
(iv) f (x) = [x 2] + 2, where [x 2] = greatest integer
function.
(v) f (x) = 2x2 12x + 20
(vi) f (x) = x2 + 8x 16

2. Construct graphs for the following functions.


(i) y =

f (x) = |x 3| 4

1. Graph the following functions:


(i) f x =

4
(3, 4)

(iii)
y

f (x) = 2 x [x]

3
2
1

x
4 3 2 1

0 1
1
2

(ii) f (x) = | 2x 1 |

(iii) f (x) = 2x | x 1|

110

How to Learn Calculus of One Variable


y

(iv)

(ii)

4
4

f (x) = [ x 2] + 2

3
2

f (x) = |2x 1|

2
1
1

x
0 1
1

4 3 2 1

0 1

(iii)

4
2

(v)
f (x ) = 2x |x 1|

y
1
4

f (x) = 2(x 3) + 2

3
2

(3, 2)

(iv)

x
0 1

4
4

(vi)
y

x
0 1

f (x) =

x + 4
2

PQ
x

MN

f (x) = (x 4)

(v)
f (x) = |x| 1
[x ]

f (x) is undefined for 0 x < 1

2. (i)
y

y =

x
2

x
1

Function

On the Inverse of a Function


Before defining the inverse of a function with respect
to different aspects, one must know the following
facts:

af

1. A function f : A f A is always on-to


function.
2. f : A B is one-one function f : A

af

f A B is a bijection, where
A = domain of f
B = codomain of f
f (A) = range of f, also denoted by R (f).
3. If A and B are finite sets and f : A B is a
bijection, then n (A) = n (B), i.e. number of elements in
domain = number of elements in co-domain.
4. When there is only one value of the function y = f
(x) for every value of x = a in its domain, then the
function y = f (x) is said to be single valued function
in its domain.
The polynomial, the rational fraction, exponential
and logarithmic functions are important functions
which are single valued.
5. When there are two values of the function y = f (x)
for each value x = a in its domain, the function y = f (x)
is said to be two valued (or, double valued) function
in its domain.
Examples of double valued functions are:
n

(i) y = a o x + a1 x

n 1

+ ... + a n 1 x + a n n 1

N
(ii) y =
where N and D are polynomials in x.
D
2

6. When both y = f (x) and x = f1 (y) obtained by


solving y = f (x) for x in terms of y, are single valued
functions, then the function f establishes a bijection
(or, a one-to-one correspondence) between its domain
and range.
Now, the definition of the inverse of a function is
provided.
Definition 1: (In terms of one-one function): The
inverse of a one-one function f, denoted by f1 is the
function which is defined for every y = f (x) in the
range of f by f 1 (y) = x, i.e. if f is a function which is
one-one in a part D of its domain and R is the set of

111

values taken by f at points of D, then the function f1


with domain R and range D, denoted by f1 (y) = x if y
= f (x) for every y R is said to be the inverse of the
function f on D.
Notes:
1. In some cases when the given function f is not
one-one function in the entire domain, a part D of its
domain is selected where the function f is one-one
and the inverse of a function will exist over f (D) for
the new domain of f.
2. A function has an inverse it is a one-one
function in its domain and the equation y = f (x) can
be solved for x in terms of y, i.e. x = f 1 (y) which must
be single valued. For an example, y = 3x + 2 is a 1-1
function.
y2
is a single valued function.
3
3. It may or may not be possible to find the inverse of
a given function in the following cases:
If the given equation y = f (x) gives x = f1 (y) which is
double valued function or the given equation is
double valued function, then in both cases, the given
function (or, equation) y = f (x) has no inverse, as for
an example,
x=

y= x +9 x=

y 9 which determines

two different values of x for each value of y 9 in


the range of f.
4. When it is said that the inverse x = f1 (y) of the
function y = f (x) is single valued for y = b in range of
the original function y = f (x), it is meant that for each
member y = b of the range of the original function y =
f (x), there is exactly only one element in the domain
of the original function y =f (x).
Definition 2: In terms of one-one and on-to
function: If f is a one-one and onto from A to B, then
1
there exists a unique function f : B A such
that for each y B there exists exactly only one
element x A such that f (x) = y, then f1 (y) = x. The
function f1 so defined is called the inverse of f.
Further, if f 1 is the inverse of f, then f is the inverse
of f1 and the two functions f and f 1 are said to be the
inverse of each other.

112

How to Learn Calculus of One Variable

Notes:
1. A function
f

f:AB

has an inverse

f : A B is one-one and on-to.

2. If a function f is continuous, monotonic and


defined on a real interval working as a domain of the
given function f, then a continuous monotonic inverse
f1 exists. For example, f (x) = y = 2x + 3 where 0 < x <
1, has an inverse f

a yf = x = 21 a y 3f where 3 <

y < 5.
The variables x and y are often interchanged in the
inverse function, so that in this example f (x) = y = 2x
+ 3 is said to have the inverse.

axf = y = 12 ax 3f

This can be written


f : x 2 x + 3 on 0 , 1

:x

1
x 3 on 3 , 5
2

How to Find f1 as a Function of x


Step 1: The equation y = f (x) should be solved for x
in terms of y.
Step 2: x and y should be interchanged. The resulting
equation will be y = f1 (x).
Solved Examples
1. Find the inverse of y =

1
x + 1.
2

Solution:
Step 1: On solving for x in terms of y:

y=

1
x +1
2

2y = x + 2
x = 2y 2
Step 2: On interchanging x and y:
y = 2x 2

af

Hence, the inverse of the function f x =

1
x+1
2

is the function f1 (x) = 2x 2.


2. Find the inverse of the function y = x2, for x > 0.
Solution:
Step 1: On solving for x in terms of y:
y = x2

y =

= x = x ( 3 | x | = x because x >

0)
Step 2: On interchanging x and y:

y= x
The inverse of a function y = x2, x > 0 is the
function y =

x.

One should note that, unlike the restricted function


y = x2, x > 0 the unrestricted function y = x2 is not oneone and on-to and therefore has no inverse.
On the Criteria to Test Whether
a Given Function f has its Inverse
There are following criteria to test whether a given
function y = f (x) from its domain D to its co-domain C
has an inverse.
Criterion 1: One-oneness and ontoness of the
function, i.e. one should show that the given function
y = f (x) from its domain D to its codomain C is a oneone and on-to function.
Note:
Criterion 1: Is fruitful to test the existance of an
inverse of the function y = f (x) whose domain D and
whose codomain C are known.
Criterion 2: Single valuedness of both the function
f and f1, i.e. one should show that both the given
function y = f (x) defined on its domain and x = f1 (y)
defined on the range of y = f (x) are single valued.
Criterion 3: One-oneness and single valuedness of
a function, i.e., one should show that the function y =
f (x) from its domain D to its range R (in case range is
known but its codomain is not known) is one-one
and then show that the function x = f1 (y) obtained

Function

by solving the given function y = f (x) for x in terms of


y is a single valued function.
Notes:
1. In case of function whose domain and range are
known but whose codomain is not known, one should
suppose that range is coincident with codomain and
then use any one of the criteria to show whether the
given function has the inverse.
2. In case of a function whose domain and range are
known, one can use either the criterion (2) or the
criterion (3) which is easy.
Criterion 4: One should see whether a given function
y = f (x) is continuous, monotonic and defined on a
real interval working as a domain of the given function
f.
Note: Criterion (4) is fruitful to test the existance of
an inverse of the function y = f (x) whose domain is
known and its range can be determined by using any
mathematical manipulation.
To remember:
1. The domain of the inverse of a function f1 is the
set of all values of y for which x = f1 (y), i.e. the range
of the function f, i.e. the domain of f1 is the range of
f.
2. The range of the inverse of a function f1 is the set
of all values of x for which y = f (x), i.e. the domain of
f, i.e. the range of f1 is the domain of f.
3. A function which has an inverse is said to be
invertible.
4. The symbol denoted by f1 is read as eff inverse.

(x1 1) (x2 3) = (x1 3) (x2 1)


3x1 x2 + 3 = 3x2 x1 + 3
2x2 = 2x1
x1 = x2
f is 1 1
Also, any y Y , y =

x 1
x3

y (x 3) = x 1
x (y 1) = 1 + 3y
x=

3y 1
,y 1
y 1

y Y x =

3y 1
x such that y = f (x)
y 1

all the elements of y are f-images of an element in


x, i.e., f is on-to.
Hence, f is one-one and on-to f has an inverse.
Let the inverse of f be g, i.e. g = f1

af

Then g y = x =

Thus f

3y 1
x= f
y 1

a yf = 3yy 11

the inverse function of f.

LM
N

defined by f (x) = sin x, x

1. Test whether the function f : x y defined by

if so, find f1.

af

x 1
where.
x3

kp

Solution: For x1 , x2 R 3 ,
f (x1) = f (x2)

x1 1 x2 1
=
x1 3 x3 3

a yf = 3yy11 .

a xf = 3xx 11

is

OP
Q


,
1 , 1
2 2

OP
Q


has its inverse
,
2 2

LM , OP
N 2 2Q
sin a x f sin a x f f a x f f a x f

Solution: x1 x2 and x1 , x 2

x = R {3} and Y = R {1}, R being the set of reals,


has its inverse.

LM
N

2. Test whether the function f :

Solved Examples

f x =

113

f is one-one.

LM , OP
N 2 2Q
that f (x) = y, i.e. y = sin x f is on-to.
Again any y 1 , 1 x

such

114

How to Learn Calculus of One Variable

Hence, f is one-one and on-to f has its inverse


which is given by

LM
N


: 1 , 1 ,
2 2

OP
Q

is the required

inverse of the given function.


3. Does the function y = x2 have an inverse in the
interval [1, 1]?
Solution: y = f (x) = x2 is the given function whose

x = y is not a single valued function.


Hence, f has no inverse.
4. Show that y = | x | has no inverse. Restrict its domain
suitably so that f1 may exist and find f1.
Solution: y = | x |

f
now, for every x D a f f , a unique value of y R a y f
+

kp

Clearly, D (y) = R and R y = R 0 = 0 ,

is determined y = x is a single valued function

af

Now, y = | x | for y 0 ,
2

y = x

af

af

af

af

a yf = x = y
D e f j = R a f f = 0 , f
For ever value of y R a f f , a unique value of
x D a f f is determined.
1

for each value of y in [0, 1], the range of f, x


does not have a unique value.

for every x D f .

Now, for every value of x D f , a unique value

x= y f

y for y 0 , 1

af

af

y = x , D f = 0 , and R f = 0 ,

y = x is a single valued function x D f .


Again, y = f (x) = x

x = y

R x, x 0
y=S
T x , x < 0

Case 1: When the restricted domain is 0 , then y


= | x | = x, x > 0

of y R f is determined.

domain is R and 1 , 1 R
1 < x < 1
1 < x < 0 and 0 < x < 1
1 > x2 > 0 and 0 < x2 < 1
0 < x2 < 1
0 < f (x) < 1
Range of f is [0, 1]
Now y = x x =

or , 0 , f will have the inverse.

kp

However, if the domain of f is restricted to 0 ,

a yf = sin a yf , y 1, 1
f a x f = sin a x f , x 1 , 1
1

every y R 0 .

y = | x | has no inverse.

defined by

x = y which is a double valued function for

x = y is a single valued function for every

af

y R f .
Hence, y = f (x) = x is single valued function for

a f as well as x = f (y) = y is a single


valued function for every y R a f f .
every x D f

y = f (x) has its inverse in its restricted domain

0, .
Now, since x = f1 (y) = y
y = f1 (x) = x is the required inverse for y = f (x)
= x.

Case 2: When the restricted domain is , 0 , then


y = x, x < 0

afa

af a f
Now, it is observed that for every x D a f f , a
unique value of y R a f f is determined.
y = x , D f = , 0 and R f = 0 ,

Function

y = f (x) = x is a single valued function for

af a

every value of x D f = , 0 .

af a

Again, y = f (x) = x, D f = , 0 and

af a f

R f = 0 , which
x = y x = y

a yf = x = y
f a y f = y which is a single valued
function for every y R a f f .
f

Both y = x in its domain and x = y in the


range of f are single valued functions.
y = x has its inverse which is x = f1 (y) = y,

e j = R a f f = a0 , f .

i.e., f1 (x) = x, D f

5. Does the function f (x) = 1 2x have an inverse?


Solution: f (x) = 1 2x is defined for every value of
x R
Further, f (x) = 1 2x
y = 1 2x where y = f (x)
y 1 = 2x
1 y = 2x
x = log2 (1 y) which is defined for y < 1

af a

R y = , 1

af

Now, for every value of x D f = R , a unique

af a

value of y R f = , 1 is determined.

f (x) = 1 2x is single valued in its domain.

af

Also, for every value of y R f , a unique value of

af

x D f is determined from the equation x = f1 (y) =


log2 (1 y) is a single valued function in the range of
the given function y = 1 2x.
Hence, y = 1 2x exhibits a one-one correspondence between its domain and range.
y = 1 2x has an inverse.
Now, f1 is found in the following way:
x = f1 (y) = log (1 y)
f1 (x) = log (1 x) which is the required
inverse of the given function f (x) = 1 2x.

115

6. Find the inverse of the function defined as:

R|x , x < 1
f a xf = Sx , 1 x 4
|T8 x , x > 4
2

Solution: The given function is piecewise function


defined as:

R|x , x < 1
f a xf = S x , 1 x 4
|T8 x , x > 4
2

af

Now, y = f x x = f
y = x, x < 1
x = y, y < 1

a yf

y = x ,1 x 4

x=

y , 1 y 16

e3 1 x 4 1 x

16

y=8 x,x>4
2

y
x=
, y > 16
64

e3 x > 4

x > 2 8 x > 16

Hence,

R|
y, y < 1
|
x = S y , 1 y 16
|| y
T 64 , y > 16
R|
y, y < 1
|
f a y f = S y , 1 y 16
|| y
T 64 , y > 16
2

116

How to Learn Calculus of One Variable

R|
x, x < 1
|
axf = S x , 1 x 16
|| x
T 64 , x > 16
2

is the required inverse of the given piecewise


function.
Note: While finding the inverse of a piecewise
function, one should find the inverse of each function
defined in its respective sub domain.
Exercises on finding the inverse of a function
Exercise 1.25

2. No inverse
3. No inverse

a xf = x 2
domain: a , f
1
5. f a x f =
a2 xf
4. f

domain: R {2}

a xf = FGH 13x+2 x4 IJK


domain: R R
ST 21 UVW
8x
7. f a x f =
x
6. f

Find the inverse of the given function, if there is one


and determine its domain.
1. f (x) = x3
2. f (x) = x2 + 5

af

af

2x 1
x

af

x+4
2x 3

3. f x =

x
4. f (x) = (x + 2)3
5. f x =
6. f x =

af

7. f x =

af

8. f x =

8
3

x +1
2
2

8x 1

9. f (x) = 2 | x | + x

af

10. f x =

3
1+ x

Answers:

axf = x
domain: a , f

1. f

domain: R {0}

a xf =

2+ x
8x
domain: x < 2 or x > 0
9. No inverse
10. No inverse
8. f

Exercise 1.26
Perform each of the following steps on the given
function:
(a) Solve the equation for y in terms of x and express
y as one or more functions of x:
(b) For each of the functions obtained in (a), determine
if the function has an inverse, and if it does, determine
the domain of the inverse function.
1. x2 + y2 = 9
2. x2 y2 = 16
3. xy = 4
4. y2 x3 = 0
5. 2x2 3xy + 1 = 0
6. 2x2 + 2y + 1 = 0
Answers:

af

af

1. (a) f 1 x = 9 x , f 2 x = 9 x ;
(b) Neither has an inverse.

117

Function

af

2. (a) f 1 x =

af

x 16 , f 2 x = x 16 ;

3. (a) f x =

af

4. f x =

(b) f

4. (a)

(b) f 1

1
f2

1
,x>0
x

1
2

x +4

,x0

5. f (x) = x5 + x3
3

af

3. f x = x

(b) Neither has an inverse.

a f 4x
axf = 4x , domain: R {0}.
f a xf = x , f a xf = x
a xf = x ; domain: R
a xf = x ; domain: R.
2x + 1
f axf =
3x

2. f (x) = (x + 3)3

6. f (x) = x3 + x
Answer:

1. domain of f
2. domain of f

1
1

: 0 , range of f

(b) No inverse.

af

2x + 1
6. (a) f x =
2
(b) No inverse.

3. domain of f1: R; range of f


4. domain of f

Determine if the given function has an inverse, and if


it does, determine the domain and range of the given
function.

af

1. f x =

x4

a f

: 0,

F 1 OP ; range of f : a , 0
H 4Q

: 0,

5. domain of f1: R; range of f1: R


6. domain of f1: R; range of f1: R

Exercise 1.27

: 4,

: R range of f1: R

5. (a)

118

How to Learn Calculus of One Variable

2
Limit and Limit Points

These two concepts are defined based on the


following concepts.
1. Open -neighbourhood of a Given Point
The set of all points (on the number line, or in a plane,
or in an n-dimensional space, or in any space where
the distance between any two points can be measured)
whose distances from a given point are less than a given
positive number is called an open -neighbourhood
of a given point.
In notation, we express an open -neighbourhood
of a given point x0 on the number line as:
N (x0 ) = {x R : | x x0 | < , > 0}
Notes: 1. x N (x0 ) | x x0 | <
x (x0 , x0 + )
2. (i) on the real line, an open e-neighbourhood of a
given point is a line segment (a part of the real line)
without (not counting) the end points whereas the given
point whose open e-neighbourhood is sought is the
midpoint of the line segment, i.e., an open interval with
a midpoint x0 and without left end point x0 e and
right end point x0 + e represented as (x0 e, x0 + e ) is
an e-neighbourhood of a given point x0.

x0

x0

x0 +

(ii) In two dimensional space (in the real plane or


complex plane), an open -neighbourhood of a given
point is the set of all those points excluding the

circumference which is also termed as boundary of


the circle in real analysis. In other words, a circle
centred at a given point (whose open neighbourhood is sought) and whose radius is the
given number is an open -neighbourhood of the
given point if we exclude all those points whose
distances from the center equal the radius. In real
analysis, a circle in a plane is termed as circular
neighbourhood of the given point.

x0

(iii) In three dimensional space, it is the set of all


those points inside the sphere. A sphere is also
termed as spherical neighbourhood of a given point.
3. An open -neighbourhood of a given point is also
termed as:
(i) Open sphere (centred at the given point with a given
radius ) and it is symbolized as S (x0 ) or S (, x0 ).
(ii) Open ball (centred at the given point with a given
radius ) and it is symbolized as B (x0 ) or B (, x0 ).
2. A Closed -neighbourhood of
a Given Point
The set of all points (in any space) whose distances
from a given point are less than or equal to a given
positive number is called a closed -neighbourhood
of a given point.

Limit and Limit Points

2. The world every, each or all used before neighbourhood or neighbourhood emphasizes
firstly that the said properly must hold even if the
-neighbourhood or neighbourhood of the given point
P is arbitrarily small and secondly that if there exists
even one -neighbourhood or neighbourhood of the
point P which does not contain at least one point of
the set which is not P, P can not be said to be a limit
point of the set A.
Notes: 1. The limit points of a set may or may not
belong to the set.
2. The limit point of a set is also termed as:
(i) Limiting point
(ii) Accumulating point and
(iii) Cluster point.
3. The set of all the limit points of a set A is called the
derived set which is denoted by A or D (A), i.e.,
(i) A = D (A) = {x0 R : N (x0) {x0} A ,
N (x0)} = {x0 R : x0 is the limit point of A} on the
number line R.
(ii) A = D (A) = {x0 X : S (x0) {x0} A ,
S (x0)} in a metric space X.
(iii) A = D (A) = {x0 X : N (x0) {x0} A ,
N (x0)} in a topological space X.
4. x0 D (A) x0 is a limit point of A.
S (x0) {x0} A in a metric space
((x0 , x0) (x0, x0 + )) A on the real line
N (x0) {x0} A in a topological space
5. We may divide a limit point of a set into two halves
namely.
(a) Left limit point and
(b) Right limit on the number line R.
Now we define each one on the number line R in
the following way:
(a) Left limit point of a set: A point x0 R is a limit
point of the set A R > 0, there is at least one
point x A such that 0 < x0 x < (x0 < x < x0).
i.e. (i) A point x0 R is right limit point (or right hand)
limit point of the set A x0 is a limit point of the
subset of A lying to the right of the point x0.
(b) Right limit point of a set: A point x0 R is a right
limit point of a set A R > 0, there is at least
one point x A such that 0 < x x0 < (x0 < x < x0 +

123

) Further, one should keep in mind that x0 R is a


limit point of a set A R x0 R is a left limit point
or a right limit point of the set A R.
A

x0

N (x 0)
x0

x0 +

i.e. (ii) A point x0 R is left limit point (or left hand)


limit point of he set A x0 is a limt point of the subset
of A lying to the left of the point x0.
6. A set A is closed each limit point of the set A is
a member (point) of the set A A point x is a limit
point of a set A and x A the set A contains all its
limit points D (A) A.
7. x0 D (A) x0 closure of A {x0} x0 is not
a limit point of the set A.
8. More on closure point and closure of a set: A
closure point or closure of a set is also defined in
terms of the limit point of a set.
(i) Closure point of the set (in terms of the limit
point): A point x0 in a space X is called a closure
point of the set A contained in the space X (A X)
x0 A or x0 is a limit point of A.
(ii) Closure of a set: Closure of a set denoted by is
the set of all points of A together with all those points
(in space) which are arbitrarily close to A. That is, the
closure of A, denoted by , is the union of the set A
and the set of all its limits points, i.e., = A D (A).
Now, we give the definition of an isolated point of
a set.
2. Isolated Point of a Set
It is also defined with respect to different aspects.
Definition (i): (In terms of neighbourhood): A point
of a set is called an isolated point of the set there
exists a neighbourhood of that point in which there is
no other point of the set. That is, a point of the set
whose one neighbourhood includes in itself
(contains) no other point of the set, i.e., N x0 = {x0},
is called an isolated point of the set.
Definition (ii): (In terms of deleted neighbourhood):
A point of a set whose one deleted neighbourhood
does not intersect the given set is called an isolated
point of the set. That is, a point belonging to the set

124

How to Learn Calculus of One Variable

which is not the limit point of the set is called an


isolated point of the set.
Hence, in notation we can express the definition
of an isolated point of the set in different spaces in
the following ways:
(a) A point x0 A R is called an isolated point of
the set A N (x0) {x0} A = N (x0) A =
{x0} for some N (x0).
(b) A point x0 A X, where X is a metric space, is
called an isolated point of the set A S (x0) {x0}
A = S (x0) A = {x0} for some S (x0).
(c) A point x0 A X, where X is a topological
space, is called an isolated point of the set A N x0
{x0} A = N x0 A = {x0}, for some Nx0
Notes: 1. An isolated point of a set is a point of the
set.
2. x0 is an isolated point of the set A x0 is not a
limit point of the set A x0 D (A).
3. Roughly speaking, an isolated point of a set is a
point x0 of the A around which there is no point of the
set A which is different from (distinct from, or other
than) the point itself namely x0.
Kinds of the Limit Point of a Set
There are two types of the limit point of a set namely:
(a) Interior point of a set.
(b) Boundary point of a set.
Each one is defined with respect to different aspects:
(a) Interior point:
Definition (i): (In terms of the limit point): An interior
point of the set is a point of the set which is not the
limit point of the complement of the set. That is, a
limit point x0 of the set A is an interior point of the set
A there are only the points of the set A in some
neighbourhood of the point x0.
Definition (ii): (In terms of neighbourhood): A point
x0 in a space X is called the interior point of the set A
contained in the space X (A X) There is a
neighbourhood of the point x0 which is a subset of
the set A whenever the point x0 is in the set A x0
A and a N x0 such that N x0 A.
Hence, in notation, we can express the definition
of the interior point of a set in different spaces in the
following ways:

(i) On the number line, a point x0 in a set A R is


called an interior point of the set A x0 can be enclosed in an interval (x0 , x0 + ), > 0, such that all
the point of this interval are the points of the set A,
e.g: every point of a line segment other than (not
counting, or without) the end points of the line segment is the interior point of the set composed of all
points of the line segment.
(ii) In a metric space X, a point x0 in a set A X is
called an interior point of the set A x0 can be enclosed in an -neighbourhood N (x0) such that all the
points of this -neighbourhood N (x0) are the points
of the set A.

N (x)
0

A
X

(b) Boundary point:


Definition (i): (Intuitive concept): A point x0 in a
space X is called a boundary point of the set A
contained in a space X (A X) x0 is arbitrarily
close to both the set A and its complement AC. That
is, the points which are arbitrarily close to both the
set and the complement of the set are called boundary
points of the set.
Definition (ii): (In terms of limit point): A point x0 in
a space X is called a boundary point of the set A
contained in a space X (A X) x0 is the limit point
of both the set A and its complement AC.
Definition (iii): (In terms of neighbourhood): A point
x0 in a space X is called boundary point of the set A
contained in a space X (A X) every
neighbourhood of the point x0 intersects both the set
A and the complement of the set A (AC ) at some points
x0 is a point of closure of both the set A and its
complement AC.
A

x0
N ( x)

Limit and Limit Points

Hence, in notation we can express the definition


of the boundary point of a set in different spaces in
the following ways:
(i) A point x0 on the number line R is a boundary
point of the set A contained in a space R (A R)
N (x0), N (x0) A , N (x0) AC .
(ii) A point x0 in a metric space X is a boundary point
of the set A contained in the space X (A X)
S (x0), S (x0) A , S (x0) AC .
(iii) A point x0 in a topological space X is a boundary
point of the set A contained in a space X (A X)
Nx, Nx A , Nx AC .
Besides these, in connection with the interior points
of the set and the boundary points of the set, there
are two more sets namely:
1. Interior of a set and
2. Boundary of a set
Which are defined in the following ways.
1. Interior of a set: A set whose members are all the
interior points of the set is called interior of the set,
that is, the set A = {x: x A and some Nx A} is
called the interior of the set A.
The symbol to denote the interior of a set A is int
(A), Ai or A0.
2. Boundary of a set
Definition (i): (intuitive concept): The boundary of
a set A is the set of all those points which are arbitrarily
close to both the set A and its complement AC.
Definition (ii): A set of all those points which are
boundary points of a set A is called the boundary of
a set A.
The boundary of a set A is symbolized as:
b (A), Ab or (A)
Hence, in terms of interior point and boundary
point of a set, we can say,
A set is closed all the limit points (interior and
boundary points both) of a set belong to the set.
Similarly, in terms of the interior and the boundary
of a set, we can express the closure of a set A as:
A = int (A) b (A)
Now we define one more important concept know
as open set in the following way:
Open set: A set A of points (on the number line, or in
the plane, or in ordinary space, or in n-space or in any
space) is called open if, whenever it contains a point

125

P, it also contains all points (on the number line, or in


the plane, and so forth) near P, that is, all points of
some interval with midpoint P. That is, a set A
contained in a space X (A X) is called open
every point of the set A is an interior point of the set
A (i.e., A = int (A)), or in other words, A set A contained
in a space X (A X) is called open given any point x
in A, a neighbourhood of the point x (Nx) such that
Nx A.
Notes: 1. Roughly speaking, the boundary of a
region (a set of points which is either a non empty
open set or such a set together with some or all of the
points forming its boundary), if it exists, is the set of
points in the region from those not in (Simply to avoid
clumsiness of language we often say points are in a
region instead of the points are points of region.
All the regions considered by us will be what are known
as open region; so that in the cases in which we use
it, our definition of boundary agrees with the usual
definition, in which points of the region may also be
the points of the boundary).
2. In 3-pace, the closure of a set A is the set A together
with all its skin, whether the skin is part of the set or
not. The interior of the set A is the set A minus any
part of the skin which it contains. The boundary of a
set A is its skin.

b ( A)

3. End points of any interval on the number line are


the boundary points of the interval.
4. A boundary point may belong to either the set A
or its complement AC but the boundary point is the
limit point of both of them.
5. Every point of an open set is an interior point lying
in the set itself.
6. x0 is a limit point of A x0 A and x0 A .
7. x0 A x 0 is a closure point of the set A
trivially.
8. Every limit point of the set A is also a point of
closure of the set A but not conversely.

126

How to Learn Calculus of One Variable

9. Every interior point of the set A is also a limit point.


Dense Set
We consider three kinds of sets:
1. Dense set or dense in itself set.
2. Everywhere dense set in a space.
3. Non-dense set or nowhere dense set.
Each one is defined in the following ways:
1. Dense set (dense in itself set)
Definition (i): (Intuitive concept): A set in a space is
said to be dense in itself (or simply dense set) For
any two distinct arbitrary points in the set, there is at
least one distinct point between the two given points.
Definition (ii): (In terms of neighbourhood): A set A
in a space X is said to be dense in itself Every
neighbourhood of every point x of the set contains at
least one point of the set which is not x.
Definition (iii): (In terms of the limit point): A set A
in a space X is said to be dense in itself Every point
of the set A is the limit point of the set A.
2. Everywhere dense set in a space
Definition (i): (Intuitive concept): A dense set A in a
space X means that the points of the set A are distributed thickly throughout the space X. In other words,
the set A contains points as near as we like to each
point of the space X No neighbourhood of any point
in the space X is free from the points of the set A
The set A is dense (or, every where dense) in a space X.
Definition (ii): (In terms of distance): A set A in a
space X A X is said to be dense (or, everywhere dense) in the space X Given any point x in
the space X x X and any small number > 0 ,
there is at least one an other number x0 in the set
A x0 A such that the distance between the point
x in the space X and the point x0 in the set A is less
than the given small positive number , i.e.,
(a) x x 0 < on the number line
(b) d x , x0 < in a metric space
(c) x x0 < in a normed space

Definition (iii): (In terms of neighbourhood): A set


A in a space X is dense (or, everywhere dense) in the
space X Every neighbourhood of every point
P N P in a space X contains at least one point of
the set A.

c b gh

Definition (iv): (in terms of closure point): A set A in


a space X is dense (or, everywhere dense) in the space
X Every point of the space X is a point of closure
of the set A Every point of the space X is a point
of the set A or a limit point of the set A (i.e.
X = A = A D A ).

bg
Definition (v): (In terms of limit point): A set A in a
space X b A X g is said to be dense (or everywhere
dense) in the space X Every point of the space
X A is a limit point of the set A).
3. Nowhere dense (or, non-dense) set in a space
Definition (i): (In terms of neighbourhood): A set A
in a space X A X is said to be nowhere dense
(non-dense) in the space X Every neighbourhood
of every point in a space X contains a certain
neighbourhood of a point in a space such that this
certain neighbourhood is free from the points of the
set A For every point x in the space X x X
and each neighbourhood of x (N (x)), there is a
neighbourhood of an other point y (N (y)) in the
space X y X such that N y N x and
N y A = interior of the closure of the set
o
A is empty (i.e., A = int A = ).

bg

bg

c h

bg

c h

Definition (ii): (In terms of dense exterior): A set with


dense exterior is said to be a non-dense set, i.e., A set
A in a space X A X is said to be nowhere dense
(or, non-dense) in the space X The complement
of the closure of the set A is dense in the space X.

Definition (iii): (In terms of closure and boundary):


A set whose closure is a boundary set is a nondense set.
Notes: 1. The statement :The interior of the closure
of the set A is empty means that the closure of the
set A has no interior point.
2. The complement of the closure of a set is its
exterior.
3. One should note that nowhere dense sets are
closed sets with no interior points, i.e., nowhere dense
sets are closed sets with only boundary points
whereas more generally closed sets are sets with interior points and boundary points. A nowhere dense
set is thought of as a set which does not cover very
much of the space.

Limit and Limit Points

4. A set A is nowhere dense in R (real line)


For each x and each neighbourhood N x
= x , x + , there is another neighbourhood
N y = y , y + , y X
such
that
N y N x and N y A = .

b
g
bg b
bg bg

bg

g
bg

Perfect set: A set A in a space X is perfect The


set A is dense in itself and closed Every point of
the set A is a limit point of the set A and every limit
point of the set A belongs to the set A D A = A .

bg

Sequence and Its Related Terms


A sequence is nothing but a special kind of the
function whose domain is the set of all natural numbers
and the range is a set contained in a space, i.e., A
function f : N S is called a sequence, where
N = the set of natural numbers,
S = a set contained in a space X, i.e.,
S X , where
X = any space, namely, a real line, a metric space, a
normed spaced or a topological space, etc.
S = R1 R the sequence is real sequence, i.e.,
A real sequence is a function from the set N of natural
numbers into the set R1 of real numbers where
R1 R .
Hence, 1. S X where X = a metric space
the sequence is said to be in a metric space.
2. S X , where X = a normed space the
sequence is said to be in a normed space.
3. S X , where X = a topological space the
sequence is said to be in a topological space.
One should understand real sequence wherever
the term sequence is used (as we will consider them
only).
Notation: A sequence with general term xn is written
as: {xn}, (xn) or <xn>
Nomenclature of Terms of the Sequence
The term written on the extreme left is called first
term next to it second term and so on and a term
whose subscript is n, i.e., xn = nth term which is a
function of n always.
Different Ways of Describing a Sequence

127

Generally, there are two ways to describe a sequence.


1. A sequence is described by listing its first few
terms till we get a rule for writing down the other
different terms of the sequence, e.g.

RS1, 1 , 1 , 1 , L UV is the sequence whose nth term is


T 2 34 W

1
.
n
2. An other way of representing the terms of the
sequence is to specify the rule for its nth term, e.g: the

RS
T

sequence 1,

1 1 1
, , ,L
2 3 4

UV can be written as {x }
W
n

1
, n N gives a rule for the nth term
n
of the sequence.
where xn =

Different Types of Sequence


1. Constant sequence: The sequence {xn} where
xn = c R , n N is called a constant sequence.
In this case x n = c , c , c , c , L .
2. Cauchy sequence: A sequence {xn} is called
cauchy sequence After a certain term of the
sequence, the numerical difference between any two
terms of the sequence is less than any given small
positive number .
In notation, a sequence {xn} is a cauchy sequence
given any small number > 0 , there is an integer
N depending on i.e. N
such that
xi x j < , i > N , j > N, i.e. after a certain term
namely the Nth term, the difference or the distance
between any two terms of the sequence is less than .

l q l

x1

x2

x3

x4

xi

xj

Notes: (i) It is not necessary that all the terms of a


sequence should be different from each other.
(ii) Care must be taken to distinguish between the
range of the sequence and the sequence itself, e.g. the
n
sequence {xn}, where xn = 1 , n N is given

b g

128

How to Learn Calculus of One Variable

l q l

l q

by xn = 1, 1, 1, 1, L whose range is 1, 1 ,
i.e. the range of this sequence {xn} is a finite set
whereas the sequence is an infinite set.
(iii) A sequence, by definition, is always infinite while
the range of the sequence need not be infinite, e.g:
The sequence {xn} for which xn = 1, n N , i.e.
xn = 1, 1, 1, 1, L is an infinite sequence whose
range is {1} which is a finite set.
(iv) One should always remember that whenever it is
written a term (or, terms) of a sequence, it always
means a member (or, members) of the sequence.

l q l

3. Bounded set
Definition: A set D is bounded it is bounded
above and below two numbers k and m such
that all the members of the set are contained in the
closed interval [k, m], i.e., k x m, x D and
k m ; or, in other words: A set D is bounded
an m > 0 such that x m, x D .

x=K

x x

x=m

Where x , x , x , ... are points of the set D.


Boundedness and Unboundedness
In the light of definitions of a bounded set and an
unbounded set, boundedness and unboundedness
of a sequence and a function are defined. This is why
firstly the definition of a bounded set is presented.
Boundedness of a Set
1. Bounded below set (or a set bounded on the left)
Definition: A set D is said to be bounded below or
bounded on the left a number m such that no
member of the set is less than the number m an
m R : x m , x D a point m such that no
point of the set lies to the left of m.

x m

x x

Where x , x , x , ... are points of the set D.


2. Bounded above set (or a set bounded on the right)
Definition: A set D is said to be bounded above or
bounded on the right a number k such that all
the members are less than or equal to the number
a k R : x k , x D a point k such
that no points of the set lie to the right of k.

x x

xr

xk

Where x , x , x , ... are points of the set D.

Unboundedness of a Set
1. Unbounded above set (or, a set unbounded on the
right)
Definition: A set D is said to be unbounded above
or unbounded on the right whatever the number
k is chosen (or taken) however large, some member of
the set D is > k.
2. Unbounded below set (or, a set unbounded on the
left)
Definition: A set D is said to be unbounded below
or unbounded on the left however large a number
m is chosen (or taken), there is some member of the
set D which is < m.
3. Unbounded Set
Definition: A set D is said to be unbounded (not
bounded) if it is not a bounded set, i.e., for any m > 0,
x D such that | x | > m.
Boundedness of a Sequence
1. Bounded above sequence (or a sequence bounded
on the right)
Definition: A sequence {xn} is said to be bounded
above or bounded on the right the range of a
sequence is a set bounded above a
k R : x n k , n N a point k such that
no terms of the sequence lie to the right of k.

Limit and Limit Points

x3

x2

x1

129

2. Unbounded below sequence (or a sequence


unbounded on the left)
xn k

Where x1, x2, x3, are terms of the sequence.


2. Bounded below sequence (or a sequence bounded
on the left)
Definition: A sequence {xn} is said to be bounded
below or bounded on the left the range of a
sequence is a set bounded below an
m R : xn m, m N a point m such that
no terms of the sequence lie to the left of m.

Definition: A sequence {xn} is said to be unbounded


below or unbounded on the left how large a number
m is taken, there is some member of the sequence
< m the range of the sequence is unbounded
below or unbounded on the left.
3. Unbounded sequence
Definition: A sequence {xn} is unbounded the
range of the sequence is unbounded m > 0,
an n : x n > m .
Boundedness of a Function

xn k

x1

x2

x3

1. Bounded above function (or a function bounded


on the right)

Where x1, x2, x3, are terms of the sequence.


3. Bounded sequence
Definition: A sequence {xn} is said to be bounded
the range of the sequence is bounded the
range of a sequence is a set bounded above and below
both at the same time a positive number m such
that xn m, n N in other words, the
sequence is bounded by two number m and m, i.e.
xn m, m , n N geometrically, all the
terms of the sequence lie in a certain neighbourhood
(m-neighbourhood) of the point x = 0

x1

x2

x3

x4

xn

Where x1 , x 2, x 3, x n, are points of the


sequence.
Unboundedness of a Sequence
1. Unbounded above sequence (or a sequence
unbounded on the right)
Definition: A sequence {xn} is said to be unbounded
above or unbounded on the right whatever the
number k is chosen, however large, there is some
member of the sequence > k the range of the
sequence is unbounded above or unbounded on the
right.

Definition: A function y = f (x) defined on its domain


D is said to be bounded above or bounded on the
right the range of the function f is bounded above
or bounded on the right a real number m such
that f x m for all x D , where the number m
itself is termed as an upper bound of the function f.
2. Bounded below function (or a function bounded
on the left)

bg

Definition: A function y = f (x) defined on its domain


D is said to be bounded below or bounded on the left
the range of the function f is bounded below or
bounded on the left a real number k such that
f x k for all x D , where the number k itself is
termed as a lower bound of the function f.
3. Bounded function

bg

Definition: A function y = f (x) defined on its domain


D is bounded the range of the function f is
bounded the range of the function f is bounded
above and bounded below two real numbers k
and m such that k f x m for all x D and
k m . In other words there exists M > 0 such that
f x M , x D.
In the language of geometry, a function y = f (x)
whose domain is D is bounded the curve (the
graph of the function) y = f (x) defined on its domain
D is situated between two horizontal lines.

af

bg

130

How to Learn Calculus of One Variable

Unbounded Function
1. Unbounded above function (or a function
unbounded on the right)
Definition: A function y = f (x) defined on its domain
D is said to be unbounded above or unbounded on
the right whatever the number k is chosen,
however large, f (x) > k for some x D the range
of the function is unbounded above or unbounded
on the right.
2. Unbounded below function (or a function unbounded on the left):
Definition: A function y = f (x) defined on its domain
D is said to be unbounded below or unbounded on
the left however large a number m is taken, f (x) <
m for some x D the range of the function f is
unbounded below or unbounded on the left.
3. Unbounded function (The function f (x) is said to
be unbounded one or both of the upper and lower
bounds of the function are infinite)
Definition: A function y = f (x) defined on its domain
D is unbounded the range of the function is
unbounded the range of the function is
unbounded above or unbounded below or both at
the same time.
i.e. M > 0, f x > M , for some x D .

af

Notes: 1. One should note that the set, the variable,


the sequence or the function is said to be bounded
above, bounded below or bounded whereas the
constant or the number which bounds (keeps on the
left or on the right side of itself) the set, the variable,
the sequence or the function is termed as lower bound,
upper bound or simply bound (plural bounds) of
these.
2. The fact that a sequence, a function, a variable or
a set is bounded by two numbers k and m (k m)
geometrically means that all the terms of the sequence,
all the values of a function or a variable or all the
members of the set are contained in a closed interval
[k, m].
3. If the domain of a bounded function is restricted,
the function remains bounded.
4. The restriction of an unbounded function may or
may not be bounded.

For example, f (x) = x2 is unbounded on R but if the


function f is restricted to the closed interval [0, 1], it
becomes bounded. But when the function f is
restricted to the positive real numbers, it remains
unbounded.
Illustrations:

bg

1. f x =

bg

f x =

,x0
x
= 1 when x > 0
x

bg

x
= 1 when x < 0
x
The range of the function f is [1, 1] which is a
finite set containing only two members 1 and 1.
Therefore f is a bounded function.
and f x =

bg

2. f x =

1
,2 < x 5
x2

Q 2< x5

0< x 25
Now, x 2 > 0

1
+ as x 2
x2

Also, x 2 3

1
1

x2 3

LMQ
N

x 2 > 0 and a b > 0

2 < x 5

1 1

a b

bg

OP
Q

1
1
1
f x
x2 3
3

The range of the function f is bounded below


1
and
is its greatest lower bound and f is not
3
bounded above.

bg

1
, 0 < x < is unbounded because by
x
1
choosing x sufficiently small, the function f x =
x
can be made as large as required, i.e., infinitely great

3. f x =

bg

Limit and Limit Points

a f

1
> 0 in 0 , . Hence this function is bounded
x
below but not bounded above.
4. f (x) = x sinx defined on domain 0 < x < takes
positive and negative values and is unbounded below
and above, because by choosing sufficiently large
values of x, f (x) can be made sufficiently large and
positive or large and negative.
also

Note: One must remember that a mathematical


quantity or entity is called bounded its absolute
value does not exceed some constant positive number
M. For example, cosx is bounded for all real values of
the variable x because cos x 1 .
Limit of a Sequence
It is defined in various ways:
Definition (i): (In terms of -neighbourhood): A
fixed number l is the limit of a sequence {xn}
Any -neighbourhood of that fixed number l
denoted by N l contains all the terms of the
sequence {xn} after a certain term namely the Nth
term (xN) of the sequence {xn} N depending on .

bg

Definition (ii): ( N definition): A fixed number L


is the limit of a sequence {xn} Given any small
positive number , it is possible to find out a term
namely xN such that all the terms after the Nth term
(xN) of the sequence differ from the fixed number l
by a number which is less than Given a small
number > 0 , an integer N such that xr l <
for every value of r which is greater than N (i.e.
r > N ).
1

x1

x2

x3

x4

r
xr

Notes: (i) when a variable takes on the values of a


sequence which has a limit l, it is said that the variable
has the limit l. If x is a variable and L is the limit of
the sequence {xn} defined by x = xn, n = 1, 2, 3, , one
must indicate that x has the limit l by the notation
x l instead of xn l as n , where the

131

notation n means that the numbers of the terms


of the sequence {xn} becomes very great. Hence, a
constant l is the limit of the variable x defined by x =
xn, n = 1, 2, 3, Any given small number > 0 ,
there exists a positive integer N (N depends on )
so that for all values of n greater than N , x differs
from L in absolute value by a number less than .
Further, the definition is symbolized as follows:
Given x = xn, n = 1, 2, 3, , then l is the limit of the
variable x Given any small number > 0 , an
integer N exists such that x l < for all
n > N .
(ii) To indicate that N is a function of or N depends
on , it is usual to write N instead of merely N.
(iii) When a sequence has a limit, it is said to be
convergent.
(iv) Already the terms limit point of a set, and limit of
a sequence have been discussed. But there is a little
difference among them. One should note that a set
has a limit point whereas a convergent sequence has
a limit which is unique. There is another term limit
point of a sequence which is used and discussed in
real analysis. By limit point (limit points) of a sequence,
one means the limit (limits) of a convergent
subsequence (convergent subsequences) of a
sequence. In case the sequence itself is convergent,
to the limit l, any sub-sequence also converges to the
same limit l.
Note: The limit of a convergent sequence may or may
not by a member (or, term) of the sequence itself. For
1
example, if there is a sequence {xn} where xn = ,
n
then lim xn = 0 but it can be seen that there is no
n
member (or, the term) of the sequence whose value is
zero.

Use of N Definition

The N definition of the limit of a sequence does


give us a criterion to check whether a given fixed
number obtained by any mathematical manipulation
or method is the limit of the sequence or not.
How to show that a given number is the limit of a
sequence?

132

How to Learn Calculus of One Variable

Solve the inequality xn L < , where L = a


given number which is required to be shown the limit
of the sequence and xn = nth term of the sequence,
and obtain the inequality n > f using if method.
Examples: 1. Show that the sequence {xn} where

bg

1
, n N converges to 0.
n
Solution: In order to show that {xn} converges to 0,
it is required to be shown that for an > 0 , it is
possible to obtain a positive integer N such that
xn 0 < for all n > N .

Now xn =

and xn

xn =

If

1
0 <
n

or, if
or, if

, n N

1
3
=
2
2 2n 2 + 5

2
or, if, 4n + 10 >

3
10

2
or, if, 4n >

3 10

LM 1 OP + 1 , i.e., whatever
N Q

is chosen, the absolute difference between the nth


term and 0 can be made as small as one likes after a

certain term namely x N , i.e., by definition

lim xn = 0 , i.e. {xn} converges to 0.

2
or, if, n >

2. Show that the sequence {xn} where x n =

Solution: To show that {xn} converges to

n +1
2

2n + 5

1
, one is
2

required to show that for an > 0 , it is possible to


obtain an N such that xn

<

or, if, n >

3 10
4
3 10
= f (say)
4

or if n > N where N = f + 1

1
.
2

Hence, for any > 0 , a positive integer N

n N converges to

2 2n 2 + 5

2
or, if, 4n >

or if n > N where N =

4n 2 + 10 1
>
3

1
<
n

or, if n >

3
<
4n 2 + 10

or, if,

1
<
n

2n 2 + 5

3
n2 + 1
1
1
=
=
2 2n 2 + 5 2 2 2n 2 + 5

xn

if,

Now xn 0 <

n2 + 1

1
<n> N.
2

such that xn

1
< , n > N .
2

by definition lim x n =
n

{xn} converges to

1
.
2

1
2

Limit and Limit Points

How to Find N Algebraically


for a Given Epsilon

3.

Solve the inequality xn l < for n after


substituting the given expression in n for xn using if
method.
1
Examples: 1. Find N if xn = 1 n and
2
1
=
.
128
Solution: lim x n = 1

l q RST

UV
W

xn 1 < where xn = 1

if 1

1
2

1
and =
128

1
1
1 <
n
128
2

or, if

1
2n

<

lim

FG cos n IJ = 0
H n K

The following theorems on limits of sequences are


also stated without proofs and can be made use of
them in working out examples while finding the limits
of given sequences.

yn = m , then
If lim x n = l and nlim

xn + yn
Theorem 1: nlim

= lim xn + lim yn = l + m
n

xn yn
Theorem 2: nlim

b g
= F lim x I F lim y I = l m
H
K H
K
Fx I
Theorem 4: lim G J
Hy K
F lim x I
H
K = l , if m 0
=
F lim y I m
H
K
Theorem 5: lim bc x g
F
I
= c G lim x J = c l , where c is a constant.
H
K
xn y n
Theorem 3: nlim

1
or, if n <
128
2

or, if 2 n > 2 7

or, if n > 7 = N (or, simply N = 7)

1
, an integer N = 7 was
128

a f

found such that xn 1 < for n > N = 7 N


depends on the choice of and so N is a function
of .
Theorems of the Limit of a Sequence
The following results can be proved by making use
of the definition of the limit of a sequence
and use of these can be made in working out problems
of finding the limits of the given sequences.

FG sin n IJ = 0 ;
H nK

= lim xn lim yn = l m

1
128

Hence, for a given =

lim

133

1
=0
1. nlim
n
a
2. lim p = 0 , where a is any real number and p is
n n
positive.

Note: To find the limit of a sequence whose nth term


is given means that one is required to find out the
limit of its nth term as n which can be determined
by using the same method of evaluation of lim f x
x
replacing x by n which will be explained in methods of
finding the limit of a function y = f (x) at a point x = c.

af

134

How to Learn Calculus of One Variable

Geometrical Meaning of the


Limit of a Sequence
Geometrically lim xn = l means that however close
n
the horizontal lines y = l + and y = l are taken,
there exists a vertical line at x = N such that all the
points (n, xn) to the right of the vertical line x = N lie
within the horizontal lines Y = l .
l +

y = l+

y=l

1
, n N has 0 as a
n
limit point which is also a limit point of its range

RS1, 1 , 1 , 1 , LUV .
T 234 W

N=x

2. The sequence xn =

(n, x n)

Examples: 1. The constant sequence xn = c ,


n N has only one limit point namely c in the
sense that c c , c + , i.e. any neighbourhood of C contains at least one term of the
sequence which is not necessarily different from C.
Further one should note that the constant sequence
xn = c , n N has its range {c} which is a finite
set and so the range of the constant sequence has no
limit point.

On the Relation Between the Limit of


a Sequence and Limit Point of the Range
Set (or, Simply Range) of the Sequence
One should note that there is no term limit point of a
sequence because in fact only an infinite set which is
dense has a limit point whereas a finite set has no limit
point. But there is a theorem which described the
relationship in between the terms limit of a sequence
and the limit point of the range of the sequence in a
space.
Statement of the theorem: Let {xn} be a sequence in
a space such that lim xn = x . Let A be the range
n

of the sequence {xn}. Then


(a) If A is a finite set, then xn = x for infinitely many
n.
(b) If A is an infinite set, then x is the limit point
of A.
Remarks: 1. the limit points of a sequences {xn} are
either the points of the range of the sequence or the
limit points of the range of the sequence.
2. If a point is a limit point of the range of a sequence,
then it is also a limit point of the sequence but the
converse may not always be true.
3. If a sequence has a limit l, then it is the limit point
of the sequence but converse is not usually true.

3. The sequence xn = 1 + (1)n, n N has only two


limit points namely 0 and 2 whereas its range {0, 2}
has no limit point.

b g FGH1 + n1 IJK , n N has


only two limit points namely 1 and 1 which is also the
3 3
4 4
R
U
limit points of its range S 2, 2 , , , , , LV .
2 2
3 3
T
W

4. The sequence xn = 1

Definition of Limit Point of a Sequence


In the above, the meaning of the term limit points of
a sequence has been explained. Now the definitions
available in connection with limit points of a
sequence are provided.
The concept of the limit points of a sequence is
defined in two ways.
Definition 1: (In terms of neighbourhood): A point p
is the limit point of the sequence For any given
small number > 0 and any given integer N, an
other integer n N such that xn p < .
Or, in words, a limit point of a sequence is a point
p such that for any given integer N, each
neighbourhood of p contains at least one term of the
sequence after the Nth term.
Or, it can be said that a limit point of a sequence is
a point p such that for any given integer N, each
neighbourhood of p contains infinite number of terms

135

Limit and Limit Points

of the sequence after the Nth term in the sense that


the terms having the same value are counted as often
as they occur as terms of the sequence.
Remarks: 1. One should note that elements of the
sequence need not be distinct appearing in definite
order as various distinct terms of the sequence like
first term, second term, a third term and so on. As a
consequence, all the infinitely many terms xn of this
definition may be the same number and so any number
that occurs an infinite number of times in a given
sequence or in a subsequence (subsequences) of a
sequence is a limit point of the sequence according
to the definition 1 since what the definition 1 requires
is that there should be at least one term, i.e., an infinite
number of terms of the sequence in the sense that the
same element is counted as often as it occurs as a
term of the sequence.
2. A constant sequence xn = c , n N has only
one limit point namely the constant c.
Example: Let there be a sequence defined by
n
xn = 1 , n N , i.e.

b g
lx q = l 1, 1, 1, 1, Lq . This sequence has 1 and
n

1 as limit points. The reason for which is that every


neighbourhood of 1 contains an infinite number of
terms x2, x4, x6, and every neighbourhood of 1
contains an infinite number of terms x1, x3, x5, in
the sense that the same element is counted as often
as it occurs as terms (first term, second term, third
term and so on) of the sequence.
Moreover one should note that each of the terms
x2 = second term, x4 = fourth term, x6 = sixth term,
is 1 and each of the terms x1 = first term, x3 = third
term, x5 = fifth term, is 1.
Lastly one should note that the sequence

b g

n
xn = 1 , n N is itself not convergent.

Definition 2: (in terms of limit of a subsequence):


The limit (limits) of a convergent subsequence
(convergent subsequences) of a sequence is (are)
called the limit point (limit points) of the sequence.
Remarks: 1. There is a convergent sequence
The limit and the limit point of the sequence both are
same.

2. A sequence may contain one or more convergent


subsequences.
Examples: 1. The sequence {xn} where xn =

1
,
n

n N has only one limit point namely the real

number 0 since xn =

1
is a convergent sequence.
n

b g

2. The sequence {x n } for which xn = 1 ,

n N , i.e. {xn} = {1, 1, 1, 1, 1, } has got two


limit points namely 1 and 1 since there are two

b g

subsequences x2 n = 1

2n

b g

and xb2 n + 1g = 1

2n + 1

n N , whose limits are 1 and 1 respectively.


1
,
n
n N , has got only one limit point namely 1 since
it is a convergent sequence.
4. The sequence {xn} where xn = 1, n N has only
one limit point namely 1 since it is a convergent
sequence.
5. The sequence {xn} where xn = n, n N has no
limit point since it is not a convergent sequence.
3. The sequence {x n } for which xn = 1 +

Difference Between Limit and


Limit Point of a Sequence
There are some distinctions between a limit point and
the limit of a sequence.
1. If all the members of the sequence {xn} from a
certain term x N b g onwards (i.e. x N + 2 , x N + 3 , L )
lie within the interval l , l + , then l is the limit
of the sequence. But when l is the limit point of the
sequence {xn}, then it is sufficient that at least one
term of the sequence (not necessarily different from
l) lie within the interval l , l + . e.g.: For any
> 0 , xn = 1 1 , 1 + , n N . This is
why 1 is the limit point of the constant sequence.
Further it should be noted that 1 is also limit of the
constant sequence xn = 1.

136

How to Learn Calculus of One Variable

2. Limit of a sequence is unique if it exists whereas a


limit point of a sequence is not unique since a limit
point (limit points) of a sequence is the limit (limits)
of a convergent subsequence (convergent
subsequences) of a sequence which means that there
may be one or more than one limits according as there
is one or more than one convergent subsequences of a
sequence whereas the sequence whose convergent
subsequence (convergent subsequences) is (are)
considered, need not be convergent and in case the
sequence is convergent, its limit and limit point both
n
are same, e.g.: The sequence defined as xn = 1 ,
n N is divergent, i.e. it has no limit. But it has
2n
2n + 1
and xn = 1
two subsequences xn = 1
whose limits are 1 and 1 respectively which are
termed as the limit points of the sequence
n
xn = 1 , n N noting that the sequence
n
( xn = 1 , n N ) itself has no limit but it has
got two limit points namely 1 and 1 respectively
accordingly as n is even or odd.

b g
b g

b g

b g
b g

b
bg

The concept of the limit of a function is defined in


various ways:
Definition (i): (In terms of neighbourhood): we say
that a fixed point p is a limit of the function f at a limit
point a of the domain of the function f if there is a
fixed point p such that if we choose any neighbourhood of the point p denoted by N p , it
is possible to find a -neighbourhood of the limit
point a of the domain of the function f denoted by
N a such that the values of the function lie in
N p for every value of the independent variable x
which lies in -deleted neighbourhood of the limit
point a of domain of the function denoted by
N a = 0 < x a < , i.e. There is a fixed point p
such that for every N p , a N a such that
f x N p for all x N a The fixed point
p is the limit of the function f at the limit point a of
the domain of the function f.

bg

bg
bg

bg

bg

bg

bg

Note: It is common to say that y = f (x) has a limit p at


a point x = a instead of saying that y = f (x) has a limit
p at the limit point x = a of its domain.

Explanation of definition

The following example as an explanation of


definition of the limit of a function is presented.

b g eb x 2g j is not defined at x = 2
5b x 2gb x + 2g
Now f b x g =
b x 2g = 5b x + 2g when
f x =

Limit of a Function

bg
bg bg

Definition (ii): ( - -definition): A fixed point p is


the limit of a function f at a limit point a of the domain
of the function f > 0 , a > 0 ( depends
on ) such that for every value of x in the deleted
neighbourhood 0 < x a < , the value of the
function f (x) lies in the neighbourhood p , p + i.e. > 0 , > 0 (
depends on ) such that x 0 < x a <
f x = p . Which
f x p < xlim
a
means p is the limit of the function f at x = a, where a
is a limit point of the domain of the function.

5 x2 4

x2

bg b g bg
f b x g 20 = 5 b x + 2g 20 ,

[ x = 2 f 2 = 5 2 + 2 , i.e. f 2 20 ]
In fact,

x2
= 5x 10 = 5 x 2 , x 2

bg

1
1
f x 20 <
50
10
1
Similarly, 0 < x 2 <
500
1
f x 20 <
100
1
1
f x 20 <
0< x2 <
1000
5000
0< x2 <

bg

bg

for

137

Limit and Limit Points

Hence, we see that for every small positive


number ,

0< x2 <

bg

1
f x 20 <
5

In other words, for any small > 0, a


(=

1
in this example) such that
5

Step 3: Thirdly, out of the two expressions f1(k) and


f2(k) obtained after simplification, one should choose
that one which is greater.
Step 4: Fourthly, one should form the inequality
f 1 k < , where f 1 k > f 2 k and lastly solve
f 1 k < for k.

bg
bg

bg

bg

Note: When f1 (k) = f2 (k), anyone can be chosen to


form the in equality f 1 k < .

bg

f x 20 < whenever 0 < x 2 < .

bg

Solved Examples

This fact is expressed by saying that 20 is the limit

e j as x tends to 2. it is written as
of f b x g =
b x 2g
5 e x 4j
lim
b x 2g = 20
Note: The b g definition does not provide a
technique to calculate the limit which is a fixed number
l. What the b g definition does is supply a
5 x2 4

x2

criterion which one uses to test a number l to see


whether it is actually the limit of a function f as x tends
to a, where a is the limit point of the domain of the
function.
How to find a > 0 for a given f, l, a
and > 0 algebraically
The process of finding a > 0 such that for all x

af

0< xa < f x l <


Consists of the following steps:
Step 1: Firstly, one should suppose that
f x l = u and simplify it.
Step 2: Secondly, on supposing that k is a small
positive number and letting x a = k , i.e.,
x = a k , one should make the substitution x = a +
k and x = a k respectively in f x l = u and
simplify it which gives two expressions in K namely
f1(k) and f2(k) (say).

af

bg

1. Show that lim 3x + 2 = 8


x2

Proof:
Method 1: Let 3x + 2 8 = u

(i)

and x 2 = k , i.e., x = 2 k

x = 2 + k u = 3 2 + k + 2 8 , from

(i).

= 6 + 3k + 2 8
= 3k

= 3k + 2 2

bQ k > 0g

= 3k

and x = 2 k u = 3 2 k + 2 8 , from (i)

= 6 3k + 2 8
= 3k + 2 2

Now, 3k < k <

x2 <=

= 3k Q k > 0

= 3k

= (say)
3

3 x + 2 8 < lim 3x + 2 = 8 ,
x2

Hence proved.

3x + 2 = 8
Method 2: xlim
2

if 3x + 2 8 <
or, if 3x 6 <

138

How to Learn Calculus of One Variable

= (say)
3

or, if x 2 <

Thus 0 < x 2 < =

k <

g
F 2 x 3IJ = 1
lim G
H 3x + 4 K 10

lim 3 x + 2 = 8 . Hence, proved.


x2

2. Show that

Proof: Let

FG 2 x 3IJ = 1 Hence, proved.


H 3x + 4 K 10
F x 4I = 4
3. Show that lim G
H x 2 JK
2

x2

Proof:

Method 1: Let

and x 2 = k , i.e., x = 2 k

17 x 34
3x + 4 10

17 x 2

b3x + 4g 10

... (i)

17 k
10 3k 10

17 k
17k
>
But
10 3k 10
10 + 3k 10

17 k
< , for sufficiently small k.
10 3k 10

g
17k < 10 b10 3k g

Now,

17k < 100 30 k


17k + 30 k < 100

F x 4 I 4 , from (i).
GH x 2 JK
b x 2gbx + 2g 4 = b x + 2g 4
b x 2g

Now u =

17k
x = 2 + k u =
10 + 3k 10
and x = 2 k u =

F x 4I 4 = u
GH x 2 JK
2

2x 3
1

3x + 4 10

2x 3
1

<
3x + 4 10

x2

2x 3
1

=u
3x + 4 10

20 x 30 3x 4
3x + 4 10

100
=
17 + 30

lim

x2

and x 2 = k x = 2 k
Now, u =

100
= (say)
17 + 30

0< x2 <

3x + 2 8 <

17 + 30 k < 100

or, if 3 x 2 <

= x2
x=2+ku= 2+k 2 =k
and x = 2 k u = 2 k 2 = k

k < = (say)
Hence, 0 < x 2 < =

F x 4I 4
GH x 2 JK
2

<

(i)

Limit and Limit Points

F x 4 I = 4 . Hence, proved.
lim G
H x 2 JK
F x 4I = 4
Method 2: lim G
H x 2 JK
F x 4I 4 <
If G
H x 2 JK
b x 2gb x + 2g 4 <
or, if
b x 2g
or, if b x + 2 g 4 <

But k 2 + 4 k > 4 k k 2

x2

Now, k 2 + 4 k <

k 2 + 4k < 0

x2

k <

4 + 16 + 4
2

= 2 +

x2

(* Q x 2 4 = x 2

= x2

b x 2g

x2 = 4
4. Show that xlim
2

+ 4x 4 4

+ 4x 8

x2 4 =

x2

b x 2g

+ 4x 8

+4 x2 )

x2 4 <

Proof:

if

Method 1: Let u = x 4
2

(i)

b2 + k g

= 4 + 4k + k 2 4 = k 2 + 4k

and x = 2 k u = 2 k

b x 2g

= 4 4k + k 2 4 = k 2 4k

+4 x2 <
2

+4 x2 <0

or, if x 2 <
or, if x 2 <

= 2 +
2

or, if x 2

and x 2 = k , i.e., x = 2 k

= 4k k2 for small k,

4+ =

Method 2: lim x 2 = 4 *

4 + = (say)

x2

F x 4I 4 <
GH x 2 JK
F x 4 I = 4 . Hence, proved.
lim G
H x 2 JK

lim x 2 = 4 . Hence, proved.

= k 2 + 4k Q k > 0

4 + 2 4 +

x2 4 <

0 < x 2 < =

x=2+k u=

Thus, 0 < x 2 < 2 +

or, if x 2 < = (say)

139

4 + 16 + 4
2
4 + 2 4 +
2

4 + = (say)

Thus, x 2 4 <
if 0 < x 2 < = 2 + 4 +

140

How to Learn Calculus of One Variable

lim x 2 = 4 . Hence proved.

5. Show that lim 2 x 2 3x + 4 = 13 .


x3

lim

x3

Proof:
Method 1: Let 2 x 3 x + 4 13 = u

(i)

e 2x

3x + 4 = 13 . Hence, proved.

bg

a=3

bg
= 2b x 3 g

and x 3 = k , i.e., x = 3 k

f x l = 2 x 2 3x + 4 13 = 2 x 2 3x 9

Now u = 2 x 3x + 4 13
2

= 2 x 2 3x 9 , from (i)

+2 x3

bg
2 b x 3g + 9 b x 3g <

for any > 0, f x l <

x =3+ k u

3x + 4 9 <

Method 2: Here f x = 2 x 2 3x + 4 , l = 13 and

if

b g 3b3 + k g 9
2 e9 + 6k + k j 9 3k 9

= 2 3+ k

e2 x

x2

or, if 2 x 3

+9 x3 <

or, if 2 x 3

+9 x3 <0

= 18 + 12 k + 2 k 3k 18
2

= 2k 2 + 9k = 2k 2 + 9k Q k > 0

and x = 3 k

or, if x 3 <

0< x 3 <=

u= 2 3 k

= 2 9 6k + k

3 3 k 9

j 9 + 3k 9

= 2 k 2 9 k = 9 k 2 k 2 , for small k.

Now 2 k + 9 k <
2k 2 + 9k < 0

0< x3 <=

lim 2x 2 3x + 4 = 13
x2

The use of Definition


to Prove Theorems

The definition does not only give a criterion


to check the value obtained whether it is limit or not
but also it enables us to prove many theorems and
many useful results.

9 + 81 + 8

9 + 81 + 8

2
2
But 2 k + 9 k > 2 k 9 k

= (say)

2 x 2 3x + 4 9 <

= 18 12 k + 2 k 2 + 3k 18

k<

9 + 81 + 8

= (say)

Theorem

bg

bg bg

bg

f x = f a lim f x
1: xlim
xa
a

= f x

9 + 81 + 8
4

Proof:

bg

bg

f x f x

bg

bg

f x f a (i)

Limit and Limit Points

bg

bg

LM f a xf OP =
N g a xf Q

Also, lim f x = f a
xa

for any given

(iv) lim

bg

bg

> 0, a > 0 s.t f x f a

xa

< , x for

which 0 < x a <

(ii)

(i) and (ii) > 0, a > 0 s.t

bg

bg

f a

f x

< ,

for

xa

bg

f x

=l .

xa

LM
N

[If follows that lim x n = lim x


xa

bg

= f a

provided

xa

a f = LMN lim x OPQ

f x

xa

0< xa <
lim

lim f x

xa

m0
(v) For any positive integer n = 1, 2, 3,
lim

which

af l
= ,
lim g a x f m

xa

OP
Q

= an ]

(vi) When m and n are positive integers, then


n

Theorem 2: Show that lim c = c

(a) if m is even lim x m = a m for 0 < a <


xa

Proof: Here, f (x) = c and l = c

(b) if m is odd lim x m = a m for < a <


xa

xa

bg

f x l = cc =0

bg

> 0, f x l < for

0 < x a < , where > 0 is any number.


Theorem 3: lim x = a

af

f x l < when 0 < x a <


Hence, the result.
Now, we state (without proof) some results on limits

af

bg

Let lim f x = l and lim g x = m


xa

xa

And let C be any constant. Then

bg bg
= lim f b x g + lim g b x g = l + m
L
O
(ii) lim C f b x g = C M lim f a x fP = C l
N
Q
(iii) lim f b x g g b x g
LM lim f a xfOP LM lim g a xfOP = l m
N
QN
Q
(i) lim

xa

xa

f x + g x

xa

xa

xa

xa

xa

xa

bg

bg

(vii) If f 1 x f 2 x f 3 x

for all x in an

open interval containing a except possibly at

af

af

lim f 1 x = lim f 3 x = l , then

x = a and if

xa

af

xa

Proof: = f x l = x a

af

bg

xa

lim f 2 x = l

xa

141

(This theorem is called Sandwitch Theorem or


Pinching Theorem)
Note: The expression except possibly at x = a means
that f (x) may or may not be defined at x = a.
Now, we can make use of these results in evaluating
limits of polynomials, rational functions and powers
of such functions.
Examples: Evaluate

1. lim 2 x 3 3x 2 + 6 x + 5
x2

j
= lim e2 x j lim e3x j + lim b6 x g + lim b5g
= 2 lim e x j 3 lim e x j + 6 lim a x f + lim b5g

3
2
Solution: lim 2 x 3x + 6 x + 5
x2
3

x2

x2

x2

x2

x2

x2

= 2 23 3 22 + 6 2 + 5

x2

x2

142

How to Learn Calculus of One Variable

= 16 12 + 12 + 5
= 21
Note: We see in the above example that if

bg

bg

f x = 2 x 3 3x 2 + 6 x + 5 , then, lim f x = 21
x2

Also, f (2) = 21

bg

e2.3

3.3 + 4

17

j = 13
17

x2 + 2x 5
3. xlim
4

Hence, lim f x = value of f (x) at x = 2 = f (2).

x2 + 2x 5
Solution: xlim
4

bg bg

= lim x 2 + 2 x 5

x2

LM e
jOPQ
N
= LM lim e x j + lim b2 x g + lim b 5gOP
Q
N
= e4 + 2.4 5j

However, lim f x = f a is not in general


xa

true. We

have

already

b g eb x 2g j , then
5 x2 4

f x =

seen

that

bg

x4

bg bg

x3

F 2x
GH x

2
2

I
J
+ x+5 K
3x + 4

x4

= 19 3

lim f x = f a , then f (x) is said to be continuous

2. lim

x4

at x = a.
As seen above, f (x) = 2x 3 3x2 + 6x + 5 is
continuous at x = 2.
In fact, any polynomial function is continuous for
each value of x R .

4.

lim

x 64

e5

Solution:

x + 3x 2 2 x
lim

x 64

e5

5
4

x + 3x 2 2 x

b g

x3

x3

e j

x3

bg

x3

bg

1
2

= 1576

= 32 + 3 + 5 = 17 which is not zero.

F 2 x 3x + 4 I
lim G
H x + x + 5 JK
lim e2 x 3x + 4j
=
lim e x + x + 5j
lim e2 x j lim b3x g + lim b4g
2

x3

b g

+ 3 64

3
2

x3

x3

17

x3

b64g

5
4

1576 1024 2
2
=
1024
1024

1613822
1024

5. Prove that lim sin x = 0


x0

Proof: From the figure, for

x3

x 64

1
2
= 40 + 1536
5
1024
4

x3

j
5

x 64

= 5 8 + 3 83 2

= lim x 2 + lim x + lim 5

5
4

x 64

Solution: Limit of the denominator is

= 5 lim x 2 + 3 lim x 2 2 lim x 4


= 5 64

lim x 2 + x + 5

But f (2) is undefined. We will see in the


definition of the concept of continuity that if
xa

x4

if

lim f x = 20

x2

sin x =

length of MP
OP

<x<
2
2

Limit and Limit Points

143

By Pinching theorem and (i), we get

MP
1

lim x sin

x0

= MP

FG 1 IJ = 0
H xK

Note: The above result (results) can be shown in


various ways which is (are) shown in this book.

length of AP
length of arc AP

7. Prove that lim cos x = 1


x0

B
P

Proof: For

<x< ,
2
2

cos x = 1 sin 2 x 1 sin 2 x


x

M
1

(Q

sin x x

< x < 0 < cos x < 1 and


2
2

0 a 1 a a )

sin x 0 = sin x x 0
For any > 0, a > 0 s.t sin x 0 <

Hence, we have for

1 sin 2 x cos x 1

for 0 < x 0 <


Hence, lim sin x = 0
x0

Since,

FG 1 IJ = 0
H xK
F 1I
F 1I
x sin G J x sin G J
H xK
H xK

x0

FG 1 IJ
H xK

Now, since lim x = 0


x0

By the Pinching theorem


lim x sin

x0

Again,
x sin

FG 1 IJ
H xK

FG 1 IJ
H xK

=0

x sin

(i)

FG 1 IJ
H xK

x sin

FG 1 IJ
H xK

x0

F lim sin xI
H
K

x0

that lim cos x = 1 .


x0

x 1 = x
0 x sin

lim 1 sin 2 x = 1

... (i)

= 1 0 = 1 , It follows from Pinching theorem and (i)

6. Prove that lim x sin


Proof: Q 0

<x< ,
2
2

8. lim

x0

FG sin x IJ = 1
H xK

Note 1: This limit plays a significant role in


mathematical analysis. It is often called first remarkable
limit.
Note 2: A strict proof of this limit depends upon
defining sin x as a power series in x and upon certain
properties of power series. Therefore, its proof by
geometrical argument will be presented in the chapter
to find the limits of trigonometrical function using
practical methods.
One Sided Limits
It is recalled that in the definition of the limit of f (x) as
x tends to a, it was required that the function f (x)
should be defined in some deleted neighbourhood of

144

How to Learn Calculus of One Variable

a (i.e., a is an interior point of an open interval where


f (x) is defined and f (x) may or may not be defined at
x = a).
Now, let us consider the function f (x) =

x2

clearly, f (x) is not defined for x < 2. Hence, f (x) is not


defined in any deleted neighbourhood of 2.
Hence, lim

x2

x0

x and lim

x3

x 2 9 do not

exist.

F I
sin G J
H x K , then lim f b xg does not
Note: If f b x g =
F I
sin G J
H xK
x0

1
exist. In fact for x =
(where n = any nonzero
n
integer) sin

FG IJ = 0 and so f (x) is not defined.


H xK

Hence, in any deleted neighbourhood of x = 0, we


have points where f (x) is undefined.
Definition (i): (Right hand limit): (In terms of
neighbourhood):
(a) A fixed point p is a right hand limit of the function
f at the right limit point a of the domain of the function
f if there is a fixed point p such that if we choose any
-neighbourhood of p denoted by N p , it is
possible to find a -neighbourhood of the right limit
point a of the domain of the function f denoted by
N p such that the values of the function f (x) lie in
N p for every value of independent variable x
which lies in a right hand -deleted neighbourhood
of the right limit point a of the domain of the function
f denoted by N a = 0 < x a < .
(b) (In terms of p definition): A fixed point p
is the right hand limit of a function f at the right limit
point a of the domain of the function f > 0 ,
a > 0 ( depends on ) such that for every
value of x in the right hand -deleted neighbourhood

bg

bg
bg

bg
b g

bg

f x p <

bg

that

a< x<a+

is

p < f x < p + .

bg e j

lim f x , f a + and f a + 0 are

Notation:

x 2 does not exist.

Similarly, lim

0 < x < , the value of the function f (x) lies in the neighbourhood p , p + i.e. given any > 0 ,
it is possible to find a > 0 such that 0 < x a <

x a+

available notations for the right hand limit of a


function f at x = a, where a is right limit point of the
domain of the function f.
Definition (ii): (left hand limit): (In terms of
neighbourhood)
(a) A fixed point p is the left hand limit of the function f at the left limit point a of the domain of the
function f if there is a fixed point p such that if we
choose any -neighbourhood of p denoted by N p ,
it is possible to find a -neighbourhood of the left
limit point a of the domain of the function f denoted
by N p such that the values of the function f (x) lie
in N p for every value of the independent variable
x which lies in the left hand -deleted neighbourhood
of the left limit point a of the domain of the function

bg

bg
bg

bg
b g

f denoted by N a = 0 < a x < i.e., a < x < a .


(b) (In terms of definition): A fixed point p
is the left hand limit of a function f at the left limit
point a of the domain of the function f > 0 ,
a > 0 depends on such that for every value
of x in the left deleted -neighbourhood 0 < a x <
, the value of the function f (x) lies in the neighbourhood p , p + .

That is, given > 0 , it is possible to find > 0


such that

bg

0<a x< f x p <

bg

or, a < x < a p < f x < p +


Notation:

bg e j

lim f x , f a

x a

and f (a 0) are

available notations for the left hand limit of a function


f at x = a, where a is the left limit point of the domain
of a function.

Limit and Limit Points

Notes: 1. Limit of a function f is said to exist at an


interior point of its domain or at the limit point not in
its domain left hand limit and right hand limit of
the function are finite and equal at the interior point
of the domain of the function.
2. Limit of a function f is said to exist at the right limit
point of its domain right hand limit of the function
f is finite at the right limit point of the domain of the
function f.
3. Limit of a function f is said to exist at the left limit
point of its domain left hand limit of the function
f is finite at the left limit point of the domain of the
function f.
4. The -neighbourhood of a point a excluding
the point a, is divided into two parts by the point a
on the real line. These are intervals a , a and
a , a + where it is required to restrict x to lie to
find the left hand limit and right hand limit respectively.

Limit of a Function as x

Limit of a function f as x : Let the function f

bg

that a number L = lim

number M > 0 M > K

bg

such that

x > M

f x L < .

It is sometimes of interest to consider two separate


cases of seeking the limits of a function f, viz., when x
tends to + and when x tends to . We define
each one in the following way.
Limit of a Function f as x + : Let the function

b g
f b xg > 0 , a

f be defined on the interval a, + , then it is said


that a number L = lim

x +

number
M > 0
f x L < .

bg

such

that

x> M

bg

> 0, a

f x

number M > 0 such that

af

x<M f x L <

af

Notes: 1. One must note that if lim f x = L , then

bg

lim f x

= lim

x +

bg

f x

2. (i) One must use lim


+
x0

= lim

bg

f x = L

F KI = +
H xK

(K is a

positive number)

F K I = (K is a positive number)
H xK
FKI
lim G J = (K is a positive number)
HxK
F K I = 0 (K is a positive number)
lim
H xK
F K I = 0 (K is a positive number)
lim
H xK
FKI
lim G J = 0 (K is a positive number),
HxK

(ii)

lim

x0

x0

(iv)

x +

(v)

(vi)

A number L = lim f x > 0, a


x

be defined on the interval , a , then it is said

(iii)

Cauchy definition: (Also, termed as ( M)


definition): To define the limit of a function f as x ,
first of all the domain of a function f is fixed.
Let a function f be defined out side of some interval
[c, d], or it can be said that the function f is defined in
the neighbourhood of infinity (symbolized as ), i.e.
the function f is defined for all x satisfying the
inequality | x | > K, (i.e. x > K or x < K, where K > 0),
i.e. the domain of the function f is not bounded.

145

K
=0
x x
(vii) The notation
lim

bg

bg

lim f x = lim f x
x a

xa

bg

(viii) A function f for which lim f x = 0 is called


xa

infinitesimal as x a i.e. a function whose limit is


zero at the limit point a of the domain of a function f
is called infinitesimal.

146

How to Learn Calculus of One Variable

Solved Examples

3
<
| x| 2

or, if
x

1
=0
x

bg

1
and it is given L = 0
x

1. Prove that lim

Proof: Let f x

bg

f x L < ,

i.e.,
if

or, if

1
0 <
x

1
<
x

or, if | x | > 3 + 2
or, if | x | >
Hence

3 + 2
= M (say)

3 + 2
x+5
1 < for x >

x+2

FG x + 5 IJ = 1
H x + 2K
Remark: 1. lim f b x g lim f b x g
lim f b x g does not exist.
F 2x + 3 I
Question: Prove that lim G
GH 4 x + 2 JJK
lim

1
<
x

or, if x >

or, if 3 < | x | 2

x +

1
= M (say)

1
1
Hence,
0 < for x >

x
lim

1
=0
x

2. Prove that lim

bg

Proof: Let f x =

exist.

FG x + 5 IJ = 1
H x + 2K
x+5
and it is given L = 1
x+2

bg

f x L < ,

i.e.,

x+5
1 <
x+2

Proof:

lim

x +

2x + 3
4x + 2

= lim

x +

and lim

= lim

F
GG
H

I
JJ
K

3
2
x2
=
2
4
x 4+
x

x 2+

x + 5 x 2
if
<
x+2
3
or, if
<
x+2

F
GG
H
FG
H

IJ
K

2x2 + 3
4x + 2

does not

(i)

I
JJ
K

LM x 2 + 3 OP
x P
MM
=
2I P
F
4
+
x
G
J
MN H x K PQ
2

2
4

(ii)

Limit and Limit Points

Hence, (i) and (ii)


exist.

F
lim G
GH

2x2 + 3
4x + 2

I
JJ does not
K

bg

2. The notation lim f x = means that


xa

bg

lim

Examples: (i) lim

x0

(ii) lim

x2

3. lim

FG 1 IJ = since lim
H xK

x0

1
=+
x

FG 1 IJ = since lim 1 = +
x2
H x 2K
f b x g = for every positive number M,

bg

> M whenever x > P .

that f x

In other words lim

lim

bg

f x

= + , i.e.

b g = +.
F 2x I =
GH x + 1JK

f x

x +

Example: lim

(x1, x2, x3, xn, belonging to the domain of


definition of the function and being different from a,
i.e., xn X , xn a ).
The corresponding sequence of values of y

b g y = f bx g , y = f b x g , ...,
= f b x g , ... has a limit, which is the number L.

y1 = f x1 ,
yn

Notes: (i) It is emphasized that the concept of the


limit of a function at a point a is possible only for a
limit point a of the domain of the function.
(ii) Heins definition of the limit of a function is
conveniently applied when it is required to show
that a function f (x) has no limit. For this it is
sufficient to show that there exist two sequences

x n

xn such that lim xn = lim x n = a

and

but the corresponding sequences

F 2x I
x G
H x + 1JK
F
I
2 J
G
x G
+ as x +
GH 1 + x1 JJK
lim f b x g = is possible for the function y =
2

N.B:

Definition: (Heins): A number l is the limit of the


function f at a limit point a of the domain of the
function f, i.e., lim f x = L For any sequence
xa
of values of x converging to the number a

2x3
Since 2
x +1

Let f : X R , X R , and supposing that a is a


limit point of the set X which is the domain of the
function f.

x2

however large, there exists a positive number P such

Heins Definition of the Limit of a Function

bg

= + .

f x

xa

147

f (x) whose domain is a subset of R and the range is


unbounded.

m f bx g r
n

and

m f b x g r do not have identical limits, i.e. if


lim m f b x g r lim m f b x g r , then lim f b x g
does not exist and if lim f b x g = L , then
lim f b x g = L for every sequence x 0 .
F 1I
Example: 1. Show that lim sin G J does not exist.
H xK
n

x n 0

x n 0

x0

x0

xn 0

x0

Solution: On choosing two sequences xn =


and xn =

1
2n +

(n = 1, 2, 3, )

1
2n

148

How to Learn Calculus of One Variable

Solved Examples

We get

xn 0 n

Evaluate the following ones:

xn 0 n
Now,
lim sin

x0

x0

FG 1 IJ
H xK

(ii)

FG 1 IJ
a
f H x K
= lim c sin b2 n gh = 0
F 1I
Also, lim sin G J
H xK
F 1I
= lim sin G J
a
f H x K
F F
II
= lim G sin G 2n + J J = 1
2 KK
H H
Hence, lim f b x g = 0 and lim f b x g = 1
F 1I
lim sin G J does not exist.
H xK
= lim sin
x n 0

x0

x n 0

x n 0

Limit of The Product of an Infinitesimal


and a Bounded Function
Definition (i): A function f is bounded in a neighbourhood of a point x = a a real number

bg

m such that f x m , x in x a < .


Definition (ii): A function f is bounded in a deleted
-neighbourhood of a point x = a a real
number m such that f x m , x in
0 < x a < .
Now an important theorem which provides us a
relation between an infinitesimal and a bounded
function.

bg

bg

Theorem: lim f x = 0 and g (x) is bounded


in a deleted

neighbourhood

bg bg

lim f x g x = 0 .
xa

of

x0

bg

Solution: (i) Let f (x) = x and g x = sin

the

point

FG 1 IJ
H xK

bg
F 1I
and g b x g = sin G J 1, x , x 0
H xK
g b x g is bounded in a deleted neighbourhood
Now, lim f x = lim x = 0
x0

of the point 0.
Hence, lim x sin
x0

x0

FG 1 IJ = 0
H xK

x n 0

x0

xa

FG 1 IJ
H xK
F 1I
lim x cos G J
H xK

(i) lim x sin

(ii) Let f (x) = x and g (x) = cos

FG 1 IJ
H xK

bg
F 1I
and g b x g = cos G J 1 , x , x 0 .
H xK
gb x g is bounded in a deleted neighbourhood
Now lim f x = lim x = 0
x0

of the point 0.
Hence, L x cos
x0

x0

FG 1 IJ = 0
H xK

Geometrical Meaning of the


Limit of a Function
On the Limit: In the language of geometry, the limit of
a function y = f (x) at a point x = c, where c is a limit
point of the domain of the function f, can be stated as:
If given any > 0 , a > 0 (i.e. a positive
real number depending upon ): such that a part
(i.e. a portion) of the graph of the function y = f (x) lies
in the rectangle bounded by the lines

149

Limit and Limit Points


Y

x = c , x = c + (vertical lines)

y = l , y = l + (horizontal lines)
then L is the limit of the function y = f (x) at the point
x = c.
That is, a portion of the graph of the function y = f
(x) can be obtained in a rectangle whose height can
be taken as small as one pleases by choosing the
width as small as one requires.
Y

c+

N (c)

y=l
y=l

x=c

y=l+

x=c+

N(l)

N(l)

y = f( x )

x=c

l+

l+

y = f( x)

N (c)

Left (or left hand) Limit


Geometrically, the left limit of the function y = f (x) at a
point x = c, where c is the left limit point of the
domain of the function f, can be stated as:
Given an > 0 , a > 0 : a portion of the
graph of the function y = f (x) lies in the rectangle
bounded by the lines

Right (or right hand) Limit


In the point of the view of geometry, the right limit of
the function y = f (x) at a point x = c, where c is the
right limit point of the domain of the function f, is
narrated as:
Given any > 0, a > 0 : a portion of the
graph of the function y = f (x) lies in the rectangle
bounded by the lines

x = c, x = c +

y = L , y = L +
then f (x) is said to have the right limit at x = c.
That is, a portion of the graph of the function y = f
(x) can be obtained in a rectangle whose height is
arbitrarily small by choosing the width (on the right of
the point x = c) as small as one requires.

x = c , x = c (vertical lines)
Y

l
l

y=l +

y=l
y=l

x=c +

l +
N(l )

y = f( x)

x=c

y = l , y = l + (horizontal lines)
then it is said that f (x) has the left limit at the point
x = c.
That is, a portion of the graph of the function y = f
(x) can be obtained in a rectangle whose height is
arbitrarily small by choosing the width (on the left of
the point x = c) as small as one requires.

c +
N(c )

150

How to Learn Calculus of One Variable

A Short Review on the Limit and the


Value of a Function y = f (x) at a Point x = c

Difference Between the Limit and the


Value of a Function y = f (x) at a Point x = c

In connection with the limit and the value of a function


y = f (x) at a point x = C, the following points should
be noted.

The main difference between the limit and the value


of function y = f (x) at a point x = c is the following:
The limit of a function y = f (x) at a point x = c is a fixed
number L in whose -neighbourhood lie the values
of the function f (at each value of the independent
variable x situated in the -deleted neighbourhood
of the limit point of the domain of the functions f)
which are arbitrarily close to (i.e. at little distance from
l, i.e. little less or little more in absolute value) to the
fixed number l while the value of the function y = f (x)
at a point x = c represented by f (c) is a number obtained
by use of the substitution x = c in the given function

1. While finding the limit of a function y = f (x) at a


point x = c, one is required to consider the values of
the function at values of x in the deleted
neighbourhood of the point x = C which are arbitrarily
close to a fixed number l named as the limit of the
function y = f (x) at the point x = c.
2. The value of the function y = f (x) at the point x = c
is left out of the discussion. This (i.e. f (a)) may or
may not exist. Even if f (a) exists, f (a) need not be
equal to or even close to the limit l of the function f
at c.
For example,

bg

Let f x =

x2 4
,x2
x2

= 6, x = 2
In this example f (2) = 6 is given but the limit of f (x)
at x = 2 is 4.

bg

y = f (x), i.e. (f (x))x = c = f (c) or in brief, lim f x is a


xc

fixed number close (near) to which there are values of


the function f for each value of x lying in the deleted
neighbourhood of the limit c of the domain of the
function f whereas f (c) is the value of the function f
(x) at x = c which is a number obtained from the rules
defining the function by making the use of
substitution x = c in it.

Continuity of a Function

151

3
Continuity of a Function

In general, the Limit of a function y = f (x) at a limit


point of its domain namely x = a need not be equal to
the value of the given function y = f (x) at the limit
point x = a which means that the limit of the given
function may or may not be equal to the value of the
given function, i.e. lim f ( x ) is not necessarily equal
xa

to f (a).
However, there is an important class of functions
for which the limit and the value are same. Such
functions are called continuous functions.
Definition: If x = a is a limit point in the domain of a
given function y = f (x) and the limit y = f (x) at x = a is
f (a), then the function y = f (x) is said to be continuous
at the given limit point x = a and a is termed as the
point of continuity of the given functions y = f (x), i.e.
f (x) is continuous at the limit point x = a of its domain

lim f (x) = f (a). Hence, in words, the continuity


xa
of a given function at a given limit point in the domain
of the given function limit of the same function at
the given limit point = value of the given function at
the given limit point.
Notes: 1. The definition of continuity says that given
function should be defined both for the limit point x =
a D f and for all other points near x = a (near x = a
means in the open interval (a h, a + h), where h is
a small positive number) in D (f).
2. In more abstract form, the definition of continuity
of a function at the limit point in its domain tells us
that a function f is continuous at the limit point
x=a

bg

(i) f (a) is defined, i.e., the limit point a lies in the


domain of f.
(ii) lim f ( x ) exists
xa

(iii) xlim
a f (x) = f (a)
3. One should note that

bg

(i) The definition of lim f x = p requires that a


xa

is the limit point of D (f) where a is not necessarily in


D (f) while the definition of continuity of a function at
the limit point a requires that the limit point a must
be in D (f) which means that it is a must for a to be an
interior point of D (f).
(ii) By definition of continuity given above it is possible for a function to be continuous at a limit point in
its domain D (f) but not to have a limit as x a .
Situation arises in the case of a function that is continuous at an isolated point of its domain D (f).
( ) Definition of Continuity of a
Function at the Limit Point of its Domain

It says that a function y = f (x) is continuous at the


limit point x = a given > 0 , ( ) such
that x a < f ( x ) f (a ) < .
The definition of continuity of a functions y =
f (x) at the limit point x = a of its domain, given
above, implies that, if any neighbourhood
( f (a ) , f (a) + ) is chosen of the number f (a),
of arbitrary length, 2 where is any positive

152

How to Learn Calculus of One Variable

number, however small, then for this -neighbourhood


of f (x), there is always a -neighbourhood
a , a + of the limit point a such that for
every value of x in the -neighbourhood, the
-neighbourhood
value of f (x) lies in
( f (a ) , f (a) + ) .

Remarks: In connection with definition of


limit and continuity of a function at the limit point of
(and in) its domain, one should note that
(i) definition of continuity is same as
definition of limit of a function in which l = limit of f
(x) at the limit point x = a is replaced by f (a) = value
of the function f (x) at x = a.
(ii) > 0, however small, means that one may take
= 01
. , 0.01, 0.001, 0.0001 and so on according to
degree of accuracy which one proposes to adopt.
The key point is that is an arbitrary (not fixed)
number of our own selection, and that it may be taken
as small as we please.
(iii) It is common to say that y = f (x) is continuous at
a point x = a D f instead of saying that y = f (x)
is continuous at a limit point x = a in the domain of the
given function y = f (x).

bg

Continuity of a Function at a Limit Point in


its Domain in the Language of a Sequence
In the language of sequences, the definition of
continuity of a function at a point may be stated in
the following way:
A function y = f (x) is continuous at the point x =
a if for any sequence of the values of the independent
variable x = a1, a2, a3, , an, which converges to
a, the sequences of the corresponding values of
the function f (x) = f (a1), f (a2), f (a3), , f (an),
converges to f (a).
Theorems on continuous functions at a point: Some
theorems on continuous functions at a point of its
domain are direct results of definition of continuity of
a function and theorems on limits at a point.
A. The sum, difference, product and quotient of two
continuous functions at any point x = a in their
domain is continuous at the same point x =a, provided
in the case of quotient, the divisor (the function in
denominator) at the same point x = a is not zero, i.e. if

f (x) and g (x) are two functions continuous at any


point x = a, then the functions (i) y = f (x) + g (x) (ii) y =

f ( x)
,
g ( x)
g (a ) 0 are also continuous at the same point x = a.
B. The scalar multiple, modulus and reciprocal of a
function continuous at a point in their domain are
continuous at the same point, provided in case of
reciprocal of a functions, the function in denominator
is not zero at the point where continuity of y is required
to be tested, i.e., if f (x) is a function continuous at a
point x = a and k is any constant, then the functions

f (x) g (x) (iii) y = f (x) g (x) (iv) y =

1
, f (a ) 0
f ( x)
are also continuous at the same point x = a.
C. Continuity of a composite function at a point: If
the inner function of a composite function is
continuous at a point a in its domain and the outer
function is continuous at the point representing the
value of the inner function at the point a, then the
composite function is continuous at the point a of
continuity of the inner function, i.e. if y = f (x) is
continuous at the point x = a and the function u = g
(y) is continuous a the point f (a) = b (s a y), then the
composite function u = g (f (x)) = F (x)
(say) is continuous at the point x = a. i.e.,
(i) y = k f (x), (ii) y = 1 f (x)1, (iii) y =

c b gh

lim g f x = g {f (a)}.

xa

Remember: 1. Every point at which the given function


is continuous is called a point of continuity of the
function.
2. Every point at which the condition of continuity of
the given function is not satisfied is called a point of
discontinuity of the function.
Question: When a given function y = f (x) is
continuous or discontinuous at a point x = a? Mention
the commonest functions continuous or
discontinuous at a point (points).
Answer: 1. On continuity: A function y = f (x) is
continuous at x = a lim f ( x ) = f (a )
xa

Continuity of a Function

153

The Commonest Functions


Continuous at a Point(s)

(viii) lim cos 1 x = cos 1 a where | < a < | which

(i) All standard functions (algebraic polynomial, rational, irrational, trigonometric, inverse trigonometric, exponential, logarithmic and constant functions
(symbolised as APRL-CIT E functions) at each point

1 to +1.

sin x = sin a for all real


of their domain Hence, xlim
a

for all real values of x.

xa

means cos1 x is continuous for every value of x from

numbers in the domain of sin x which means sin x is


continuous at every point which lies in its domain.
(ii) lim cos x = cos a for all real numbers in the
xa

domain of cos x which means cos x is continuous at


every point which lies in its domain.
(iii) lim tan x = tan a where a is a real number
xa
odd multiple of
other than
2n + 1
2

n = 0 , 1 , 2 , ... which means tan x is con2


tinuous for all real values of x excepting

x = 2 n + 1 , n being an integer.
2
(iv) lim cot x = cot a where a is a real number other

xa

than

(different

from)

n ,

multiple

of

n = 0 , 1 , 2 , ... , which means cot x is


continuous for all real values of x excepting x = n ,
n being an integer.

(ix) lim tan 1 x = tan 1 a for all real number in the


xa

domain of tan1 x which means tan1 x is continuous


(x) lim cot 1 x = cot 1 a for all real numbers which
xa

means cot1 x is continuous for all real values of x.


(xi) lim sec 1 x = sec 1 a where a is a real number
xa

> 1 or < 1, which means sec1 x is continuous for all


values of x which do not belong to the open interval
(1, 1).
(xii) lim cosec 1 x = cosec 1 a where a is a real >1
xa

or < 1 which means cosec1 x is continuous for all


real values of x which do not belong to the open
interval (1, 1).
n

(xiii) lim x = a
xa

for all real number, provided

n 0 and for all real numbers other than zero,


provided n < 0 which means xn is continuous for all
real values of x when n (i.e., index or exponent of the
base of power function xn) is non-negative and xn is
continuous for all real values of x excepting zero when
n is negative.

(v) lim sec x = sec a where a is real number


xa

other than 2 n + 1
= odd multiple of
2

n = 0 , 1 , 2 , ... which means sec x is


2
continuous for all real values of x excepting

x = 2 n + 1 , n being an integer.
2
(vi) lim cosec x = cosec a where a is a real number

(xiv) lim e x = e a , for all real numbers which means

other than n , multiple of n = 0 , 1 , 2 , ... ,


which means cosec x is continuous for all values of x

(xvii) lim log x = log a provided a 0 which

xa

xa

ex is continuous for all real values of x.


x

(xv) lim a = a
xc

aa > 0f for all real number which

means a x (a > 0) is continuous for all real values of x.


(xvi) lim log x = log a provided a > 0 which means
xa

log x (x > 0) is continuous for all positive values of x.


xa

excepting x = n , n being an integer.

means log |x| (x > 0 or x < 0) is continuous for all


positive and negative values of x but not at x = 0.

(vii) lim sin 1 x = sin 1 a where | < a < | which

(xviii) lim = , for all real numbers which means

means sin1 x is continuous for every value of x from

a constant function is continuous for all real values


of x.

xa

1 to +1.

xa

154

How to Learn Calculus of One Variable

2. Discontinuity: A function y = f (x) is discontinuous


at a point x = a lim f ( x ) f (a ) , i.e., a
xa
function y = f (x) is discontinuous at a point x = a (or,
a function y = f (x) has a point of discontinuity namely
a) if and only if limit and value of the function are
not equal at the same point x = a.
The Commonest Functions
Discontinuous at a Point(s)
1. One of the commonest cases of discontinuity which
occurs in practice is when a function is defined by a
single formula y =

af

af
af

f x
g x

with a restriction

g x 0 y assumes the form a fraction with a zero


denominator for a value (or, a set of values) of x
provided by the equation g (x) = 0, e.g.:

a f 1x , x 0 i.e., it is discontinuous at x = 0.
sin x
(ii) f a x f =
, x 0 i.e., it is discontinuous at x
x
(i) f x =

= 0.

af

(iii) f x = sin

F 1 I , x 0 i.e., it is discontinuous
H xK

at x = 0.
2. All standard and non-standard functions defined
by a single formula y = f (x) are continuous at each
point of their domain excepting a finite set of points
at which they are undefined which means all standard
and non-standard functions are discontinuous at a
point (or, a set of points) at which they are not defined,
e.g.:

af

x 1
(i) f x =
, x 1 is discontinuous at x = 1.
x1
(ii) f (x) = tan x and sec x are discontinuous at

f 2 , and continuous at all other values

x = 2n 1

of x.
(iii) f (x) = cot x and cosec x are discontinuous at x =
n , n I and continuous at all other values of x.
3. When a function is a piecewise function, then it
has a chance of having both a point of continuity and

a point of discontinuity at a common point (points) of


adjacent intervals, e.g.:

R|2 x , for x 3 is discontinuous at x = 3.


S| 3 x , for x > 3
T
R| e2 x + 3j for < x 1
| 6 5 x for 1 < x < 3
(ii) f a x f = S
|| x 3 for 3 x <
T
is continuous at x = 1 and discontinuous at x = 3.
af

(i) f x =

1
5

4. Whenever a function is defined by

af

af

f x = f 1 x , if x a
= a constant, if x = a (i.e. f (a) = constant)
then there is a chance of having, a point of continuity
or a point of discontinuity at x = a, e.g.:
(i) f x = x + 2 , if x 2

af

= 3, if x = 2
has a point of discontinuity at x = 2.
(ii) f x = x + 2 , x 2

af

= 4, x = 2
has a point of continuity at x = 2.
Remember: 1. A function of undefined quantity is
always undefined. For this reason sin
and tan

F 1 I are undefined at x = 0.
H xK

F 1 I , cos F 1 I
H xK H xK

On Limits of a Continuous Function


1. The limit of a continuous function of variable =
that function of the limit of the variable, i.e.

b g FGH

IJ
K

lim f x = f lim x where f (x) is a continuous

xa

xa

functions at x = a.
2. The limit of a continuous function of a continuous
function of an independent variable = outer
continuous function of the limit of the inner
continuous function of the independent variable, i.e.,

c b gh = f FH lim g b xgIK provided y = g (x)

lim f g x

xa

xa

is continuous at the point x = a and the function u =


f (y) is continuous at the point g (a) = b (say).

Continuity of a Function

Remember: 1. Finding the limit of a continuous


function may be replaced by finding the value of the
function of the limit of the independent variable, i.e.,

bg

f x =
if f (x) is continuous function, then xlim
a
f

F lim xI . This is sometimes expressed briefly


H K
xa

thus: the limit sign of a continuous function can be


put before the independent variable, e.g.:
(i) lim x = a
xa

(ii) lim sin x = sin


xa

F lim xI = sin a , etc.


H K
xa

2. The concept of continuity of a function can be


used to find its limit, i.e., if the function f (x) is continuous at x = a, then in order to find its limit

bg

Geometrical Meaning of Continuity


of a Function y = f (x) at a given
Point x = a in its Domain
From the point of view of geometry, a function y = f (x)
is continuous at a given point x = a in its domain
means that the graph of the functions y = f (x) is
unbroken at P (a, f (a))
1. The point P (a, f (a)) lies on the graph of the function
y = f (x), i.e. f (x) is defined at x = a.
2. If Q (x, y) is a point on the graph of y = f (x) and
nearer to the point P (a, f (a)), then on which ever side
of the point P, the point Q may be, it must be possible
to make the distance between P and Q as small as one
wants along the graph of the function by making the
distance between x and a small accordingly.
y
P (a, f( a))

lim f x , it is sufficient to calculate its value at the

xa

bg bg
F 1 + 2 sin x IJ
lim G
H 3 x + cos x K

F 1 + 2 sin x IJ is also conthe quotient function y = G


H 3 x + cos x K

tinuous for all positive values of x which means it is

lim
x

f ( a)

Solution: The functions in numerator and denominator are continuous for all positive values of x. so

continuous at x =

Q (x, y )

Q (x, y )

f x = f a .
point x = a since xlim
a

Example: Evaluate:

and hence,
2

F I
FG 1 + 2 sin x IJ = 1+ 2 sin GH 2 JK = 2
H 3 x + cos x K 3FG IJ + cos FG IJ
H 2K H 2K

3. A function y = f (x) defined in an interval is said to


be piecewise continuous if the interval (in which given
function f (x) is defined) can be divided into a finite
number of non overlaping open subintervals over
each of which the functions f (x) is continuous.

155

x=a
P (a, 0)

If a function y = f (x) is continuous throughout an


interval (a, b), the graph of the function in this interval
is without any gap, break or jump, i.e. the graph of the
function is unbroken in this interval, i.e. the graph of
the function has no point missing corresponding to
each value of the independent variable in this interval.
In rough language, if the point of a pencil is placed at
one end of the graph, we can move the pencil on the
graph to the other end of the graph without ever
having to lift the pencil off the paper. Further, if a line
is drawn across the graph, it will pass through at least
one point on the graph.
Note: It is better to say that f is continuous at a point
x = a the graph of the function y = f (x) is unbroken
at and in the neighbourhood of the point (a, f (a)).
If a function is discontinuous at a point x = a, then
it is a must that the graph of the function has a gap, a
break, or hole at the point whose abscissa is x = a, i.e.
the point (a, f (a)) will be missing on the graph of y =

156

How to Learn Calculus of One Variable

f (x). If the function y = f (x) is not defined at x = a, then


there is a gap, a break or a hole in the graph as there is
no point on the graph whose abscissa is x = a.
Moreover, if a function is defined by different formulas
(different expression in x) in different intervals whose
left and right end points are same known as adjacent
intervals in succession, there is usually a possibility
of discontinuity at the common points of the adjacent
intervals. Thus, if we have one functional formula y =
f1 (x) for x a and another y = f2 (x) for x < a we have
the possibility of a discontinuity at x = a.

The function f (x) has a discontinuity at x = 0.

af

2. f x =

x a
,xa
xa
y

2a

Here the function f (x) is discontinuous at x = a.

y = f1 (x )
y = f2 (x)

( a, 0)

In case the point of the pencil is made to move on


the graph of the function, then at the point of the
discontinuity, the point of pencil will have to be lifted
off the paper and will jump from one part of the curve
to the other. i.e., while drawing a graph when the point
of the pencil leaves contact with the paper, the function
becomes discontinuous at the point where contact is
left.
Illustrations on Discontinuity
of a Function at a Given Point
1.

Remember: If a function y = f (x) is not defined (has


no finite value) for any particular value of its
independent variable, then the corresponding point on
the graph of the function will be missing and the graph
will have a hole (a break) at that point.
Two Sided Continuity
of a Function at a Point
In general, a function y = f (x) defined on its domain is
right continuous (continuous from the right) at the right

af

bg

limit point x = c in its domain lim+ f x = f c


x c

and it is left continuous (continuous from the left) at

bg

the left limit point x = c, in its domain lim f x


xc

bg

= f c .
Hence, a function y = f (x) is continuous at the limit
point x = c in its domain . It is both right continuous
and left continuous at x = c.

R| a , when x < 0
f a x f = S2 a , when x = 0
|T3a , when x > 0

f(x) = 3 a
( c , f( c ))

y = f( x )

2a

f (x ) = a
0

c+

x
c+

Continuity of a Function

Continuity of a Function
in an Open Interval

functions, i.e., these are functions continuous at every


point on its domain.

A function y = f (x) is said to be continuous in an


open interval (a, b) it is continuous at every point
of the interval (a, b) Geometrically, the graph of
the function y = f (x) is unbroken between the points
(a, f (a)) and (b, f (b)).
Continuity of a Function
in a Closed Interval
A function y = f (x) is said to be continuous in a
closed interval [a, b] it is continuous in the open
interval (a, b) and is continuous at the left end point
x = a from the right and is continuous at the right end
point x = b from the left.
Geometrically, a function y = f (x) is continuous in
a closed interval (a, b) The graph of the function
y = f (x) is an unbroken line (curved or straight) from
the point (a, f (a)) to the point (b, f (b)).
Furthers, one should note that a function y = f (x)
defined over [a, b] is continuous at the left end point

bg bg

x = a from the right lim + f x = f a and the


xa

function y = f (x) defined over (a, b) is continuous at


the right end point x = b from the left

bg bg

lim f x = f b .
xb

157

Classification of Points of
Discontinuity of a Function
Let x = a be the limit point of the domain of the
function y = f (x). The point a is a point of
discontinuity of the function f (x) if at this point, f (x)
is not continuous.
Let f (x) be defined in a deleted neighbourhood of
the point a then a is
1. A point of removal discontinuity of the function f

af

(x) if there is a limit lim f x = b , but either f (x) is


xa

not defined at the point a or f (a) b. If we set f (a)


= b, then the function f (x) becomes continuous at the
point a, i.e., the discontinuity will be removed.
2. A point a of discontinuity is of the first kind of the

bg

af

function f (x) if there are lim + f x and lim f x

bg

xa

xa

bg

but lim + f x lim f x .


xa

xa

3. A point a of discontinuity is of the second kind of


the function f (x) if at least one of the one sided limits
of the function f (x) does not exist at the point x = a,

bg

bg

i.e., either or both of lim + f x and lim f x do


xa

xa

not exist.
y = f( x )

4. A point a is of mixed discontinuity if one of the one


sided limits exists, i.e., if one of the limits namely

bg

bg

lim f x or lim f x exists.

x a+
f( b )

5. A point is of infinite discontinuity of the function


f (x) if either or both of the one sided limits are infinite,

f (a )

x=a x=a x=b x=b

xa

bg

bg

i.e., if either or both of lim + f x and lim f x


xa

xa

Continuity of a Function

are infinite.

A function y = f (x) is called continuous It is


continuous at every point on its domain, e.g.: ex, sin
x, cos x, any polynomial function in x are continuous

Notes: One should note that a is


(i) A point of discontinuity of the first kind from the

bg

bg

left at a if a lim f x exists but lim f x f (a).


xa

xa

158

How to Learn Calculus of One Variable

(ii) A point of discontinuity is of the first kind


from the right at a if

bg bg

bg

lim + f x

xa

exists but

lim f x f a .

x a+

af

af

1
, when x 0 , f 0 = 1 . Then f (x) is
x
defined in (1, 1) and continuous at every point in
this interval except at x = 0. Evidently no two fixed
f x =

1
< M for all x
x

(iii) A point of discontinuity is of the second kind

numbers can be found such that m <

from the left at a if lim f x does not exist.

in (1, 1)

(iv) A point of discontinuity is of the second kind

2. If f (x) is continuous and positive at x = c in its


domain, then for all sufficiently small values of h, f (c
+ h) and f (c h) are both > 0. In other words, if f (x) is
continuous and positive at x = c, then a hneighbourhood of the point c can be found
throughout which f (x) is positive.
Similarly, if f (x) is continuous and negative at x = c
in its domain, then f (c + h) and f (c h) are both,
negative for all sufficiently small values of h.

bg

xa

bg

from the right at a if lim + f x does not exist.


xa

Properties of Continuous Functions


Now we state without proof some important properties of continuous functions.
1. If f (x) is continuous in a closed interval [a, b], then
range of f (x) is bounded. In other words, if f (x) is
continuous in [a, b], then we can find two numbers m

af

and M such that m < f x < M , x a , b


y = f(x)

y = f( x)

f( c h )

f(c)

f( c + h )

M
ch

m
0

af

1
, then f (x) is continuous in the open
x
interval (0, 1), its range consists of all real numbers
> 1 and evidently no number M can be found such
f x =

1
that < M for all x in 0 < x < 1. Again consider the
x
function f (x) defined in (1, 1) as follows:

c+h

(c + h )

Note: This property may not be true if the domain of


f (x) is not a closed interval or if f (x) discontinuous
even at a single point in its domain. For example, if

(c h )
0

f(c h)

f(c)

f( c + h )

y = f ( x)

Note: If h > 0, the symbol f (c + h) indicates the value


of the function f (x) for a value of x greater than c,
whereas the symbol f (c h) indicates the value of the
functions f (x) for a value of x less than c.

Practical Methods of Finding the Limits

159

4
Practical Methods of
Finding the Limits

Practical Methods on Limits of a Function


f (x) as x a, where f (x) is Expressed in a
Closed Form

In practice, the classical definition or


definition of the limit of a function at x = a (or, as
x a ) is not used in finding out the limit of a
function at x = a (or, as x a ) whenever a single
function whose limit is required to be found out is
provided to us. This is why for most practical
purposes, we can adopt the following method to find
the limit of the function f (x) at x = a (or, as x a ), in
many examples.

Similarly, the above method (2) tells us that if


f (a) assumes any meaningless form, then various
mathematical techniques are applied to simplify f (x)
such that when we put x = a in the simplified form of
the given function, it does not assume any
meaningless form and so we get the required result
having a finite value obtained by putting x = a in the
simplified form of the given function f (x).

1. lim f x = f a if f (a) is finite (where f (a) =

N.B.: Simplified form g (x) of f (x) is obtained by using


any mathematical manipulation (or, technique) like
factorization, rationalization, substitution, changing
all trigonometric functions in terms of sine and cosine
of an angle or using any formula of trigonometry or
algebra, etc. whichever we need.

value of the given function f (x) at x = a). But when f


(a) = any indeterminate form, then

Problems based on the limit when the value of the


function is not indeterminate:

2. lim f x = lim g x = g a if g (a) is finite

Working rule: To evaluate lim f x , firstly, we

(where g (a) = value of the function g (x) at x = a)


where f (x) is simplified to avoid its in determinate
form and g (x) is the simplified form of the given
function f (x).
The above method (1) tells us in words that

check on putting x = a in the given function whether


it assumes a meaningless form or not. If f (a) does not
assume indeterminate form, this will be required limit,

whenever we are required to find out lim f x , we

Examples:

first put x = a in the given function f (x) and find f (a).


If f (a) does not assume meaningless form, then this is
the required limit.

1. lim (3x2 + 4x2 5x + 6) = 3.0 + 4.0 5.0 + 6 = 6

xa

xa

bg bg

bg

xa

bg bg

xa

bg

xa

bg

i.e., lim f (x) = f (a) if f (a) is finite.


x a

x 0

2. lim

x2

3 + 2x x2
3 + 2 2 22
3
=
=
2
2
x + 2x 3 2 + 2 2 3 5

160

How to Learn Calculus of One Variable

3. lim

x2

x 3 8 23 8 8 8 0
=
=
= =0
2+2
4
4
x+2

x 2 + 5x + 6 0 + 5 0 + 6 6
=
= =
4. lim
x0
0+0
0
| x 3| + | x|

1 + sin 2 x
=
5. lim
x 2 1 cos 4 x
=

FG IJ
H 2K
F I
1 cos G 4J
H4 K
1 + sin 2

1+ 0
1
1 + sin
=
=
1 1
2
1 cos

To evaluate lim

xa

b g
f bxg
where both f (x) and g (x)
f bxg
1

are zero when x = a is put in the given function:

Working rule:
1. Reduce the indeterminate form to a determinate
form by using various mathematical techniques
namely:
(i) Method of factorization or method of division.
(ii) Method of substitution or differential method.
(iii) Method of rationalization.
(iv) Method of expansion.
(v) Method of simplification by using any
mathematical manipulation which are generally use
by trigonometrical formulas, algebraic formulas or
changing all trigonometrical ratios in terms of sine
and cosine of an angle.
2. Put x = a in the determinate form of the function

b g.
bxg

f1 x
f2

Remember:
1. For most practical purposes, we can obtain the
limit in case of an indeterminate form as the value
which is obtained by reducing the indeterminate form
into a determinate form by some algebraic operations
like removing common factor or using expansion such
as binomial, exponential, logarithmic or trigonometric
substitution, etc.

2. All the indeterminate form can be reduced to

0
0

0
3.
or any indeterminate form may contain a
0
common factor which makes the given function
indeterminate whose removal by various techniques
provides us a determinate form.
or

Question: When to use which method?


Answer: 1. Method of factorization is generally used
when given function is in the quotient form whose
numerator and denominator contains algebraic or
trigonometric functions which can be factorized.
2. Method of rationalization is generally used when
the given function is in the quotient form whose
numerator and denominator contains algebraic or
trigonometric expression under the square root
symbol
.
3. Method of substitution or differential method is
generally used when given function can not be
factorized easily or factorization of the given function
is difficult or not possible. The given function may be
algebraic or trigonometric expression in the quotient
form.
4. Method of simplification is generally used when
the given function contains trigonometric functions.
This method tells us to modify the given trigonometric
function by simplification in such a fashion that when
we put x = given limit of the independent variable, we
must get a finite value. This method is also used when
given function is in the difference form like

LM 1 1 OP
MN f b xg f b xg PQ
1

providing us form at

x = a.
5. Expansion method is applied when given function
contains trigonometric functions like sin x, cos x, tan
x, exponential function ex, logarithmic functions like
log x, log (1 + x) or binomial expression like (x + a)n,
etc., whose expansion is known to us and it is quite
possible to remove the common factor from numerator
and denominator after expansion.

161

Practical Methods of Finding the Limits

Problems based on algebraic functions


Form:

b g where f (x) and f (x) are polynomials


bxg

f1 x
f2

in x and x a .
The above form is evaluated in the following way.
Rule 1: If g (a) = value of the function appearing in
denominator 0, then lim

xa

bg
bg

bg
bg

f x
f a
which
=
g x
g a

means we put x = a = limit of given independent


variable in the numerator and denominator of the given
fraction (or, rational function) provided the value of
the denominator at x = a is not equal to zero.
Rule 2: If f (a) = g (a) = 0 value of the function at
x = a in Nr = value of the function at x = a in Dr = 0,
then we adopt the following working rule.
Working rule to evaluate the limit if

LM f b xg OP
MN f b xg PQ

0
, then (x a) is a common factor appearing in
0
numerator and denominator of the given function (or,
rational algebraic function) under consideration.
3. If we put x = a and the given function becomes
0
, then (x + a) is a common factor appearing in both
0
numerator and denominator of the given function
under consideration.
4. The vanishing factor (x a) 0 because x a

x a x a 0.

1
2

In order to find the limit of f (x) in such a case, our first


aim is to remove the vanishing factor from the
numerator and denominator of f (x) with understanding that (x a) is not zero, however small it may
be and then in the resulting expression, which is
determinate, we put x = a = limit of the independent
variable. Thus the required limit is obtained.
2. If we put x = + a and the given function becomes

x=a

0
.
0
1. Numerator and denominator are divided by the
common factor appearing in numerator and
denominator of the given fraction.
2. Remove the common factor from numerator and
denominator by the rule of cancellation.
3. Put x = a = given limit of the given independent
variable in the simplified form of the function (i.e., the
function free from common factor appearing in Nr
and Dr directly or indirectly) which gives us the
required limit of the function as x a .
=

Facts to know:

0
for
0
x = a, it does so on account of (x a) or power of
(x a) occurring as a factor in both numerator and
denominator of the given function f (x) under
consideration. Such a common factor which produces
1. When f (x) assumes the indeterminate form

0
form or any indeterminate form is called The
0
vanishing factor because this factor always vanishes.

5. The phrase at the point x = a or for the value


x = a means when x assumes or takes the value a
Remember:

bg

1. Let f x =

bg
b xg

f1 x
f2

and we require lim

xa

b g.
bxg

f1 x
f2

f1 (a) = f2 (a) = 0 (x a) is a common factor of the


given function appearing in numerator and
denominator of the given function f (x).
2. Simplification of the expression

b g is done
bxg

f1 x
f2

0
or
on putting

0
a for x while finding the limit of f (x) as x a .
whenever it assumes the form

Problems Based on Method of Factorization


Examples worked out:
Find (or, evaluate)
1. lim

x 1

x2 4x + 3
x 2 5x + 4

162

How to Learn Calculus of One Variable

LM x
MN x

Solution: 3

2
2

OP
5 x + 4 PQ

4x + 3

=
x =1

1 4 + 3
15+4

(x 1) is a common factor in Nr and Dr of the


given fraction. Dividing Nr and Dr by the common
factor (x 1), we get

b x 1gb x 3g = x 3 ; b x 1g
x 5x + 4 b x 1gb x 4g x 4
L x 4 x + 3 OP = lim L x 3 O = 1 3
lim M
MN x 5x + 4 PQ MN x 4 PQ 1 4
2

x 1

2. lim

x4

2 2
= .
3 3

x2 2x 8

OP
PQ

x3 2x2 9x + 4
x2 2x 8

x2 + 2x 1
; x4 .
x+2

lim

LM
N

1
x+2

x4

OP
Q

1
x+2

1
1 24 1 23
=4 =
=
4+2
6
6
6 .

LM x 5x + 7 x 3OP
MN x x 5x 3 PQ
L x 5x + 7 x 3 OP
Solution: M
MN x x 5x 3 PQ
3. lim

x3

x 2x 9x + 4
x + 2x 1
= lim
2
x

4
x+2
x 2x 8
2

0
0

x=3

Nr has (x 3) as a factor and Dr has (x 3) as


a factor.
Now, dividing Nr and Dr by the common factor
(x 3), we get
x 3 5x 2 + 7 x 3

Now taking the limits on both sides as x 4


since both sides are equal (i.e., if two functions are
equal, their limits are equal), we get,
x4

x3 2x2 9x + 4
x4
x2 2x 8
lim

b x 4g e x + 2 x 1j
=
b x 4gb x + 2g

x4
x2 2x 8

Now, taking the limits on both sides as x 4 , we


get

=4
x=4

=x

b x 4g = x 1 b3 x 4g .
b x 4gb x + 2g
b x + 2g

lim x lim

2x2 9x + 4

=x

x4

64 32 36 + 4 68 68 0
=
=
16 8 8
16 16 0

x2 2x 8

x4

(x 4) is a common factor in Nr and Dr of the


given fraction. Dividing Nr and Dr by the common
factor (x 4), we get

x3 2x2 9x + 4

= lim x

x3 2x2 9 x + 4
x2 2x 8

Lx
Solution: 3 M
MN
=

x 1

16 + 8 1 24 1 23
=
=
.
4+2
6
6

Alternative method: By direct division

44
0
=
= .
55
0

x2 4x + 3

x x 5x 3
3

x 2x + 1
2

x + 2x + 1

b x 3g ex
b x 3g ex
f

; x3

2
2

j
+ 2 x + 1j
2x + 1

163

Practical Methods of Finding the Limits

LM x 5x + 7 x 3 OP
MN x x 5x 3 PQ
L x 2 x + 1OP
= lim M
MN x + 2 x + 1PQ
3

lim

x3

5. lim

x 1

4
1
= 2
=
=
3 + 2 3 + 1 16 4

7
3

OP
PQ

=
x =1

0
0

(x 1) is a common factor in Nr and Dr of the


given fraction.
Now, dividing Nr and Dr by (x 1) we get
x 3 3x 2 + 2

b x 1g ex

2x 2

x + x x x x x1
6

+ x5 x 4 x 3 x 2 x 1
2

x2 2x 2

x 1

x 1

lim

1+111111
12 2

25
=1
14

x2

LM
N

x2 1
x +1
= lim
2
x

1
x2
x 3x + 2

LM x
MN

; x 1

x1

OP
PQ

OP
Q

1+1
2
=
= 2
1 2 1

6. lim

Now taking the limits on both sides as x 1

L x 2 x + 1 OP
lim M
MN x 3x + 2 PQ
Lx + x x x x
= lim M
x 2x 2
MN

x =1

0
0

Now taking the limits on both sides as x 1 , we


get
x 1

x 7 2 x5 + 1

b x + 1gb x 1g
x 3x + 2 b x 1gb x 2g
x+1
=
; b x 1g .
x2
2

2x5 + 1
3x 2 + 2

b x 1g ex
=

x2 1

x 7 2x5 + 1
4. lim 3
x 1 x 3x 2 + 2

LM x
MN x

OP
3 x + 2 PQ

x2 1

(x 1) is a common factor in Nr and Dr of the


given fraction.
Now, dividing Nr and Dr by (x 1), we get

32 2 3 + 1

LM
MN x

Solution:

x3

Solution: 3

x2 1
x 3x + 2
2

Solution:

3x + 2
x2

LM x
MN

OP
PQ

3x + 2
x2

OP
PQ

=
x=2

0
0

Nr has (x 2) as a factor and Dr has (x 2) as


a factor
Now dividing Nr and Dr by the common factor
(x 2), we have

gb

g b
g

x 1 x 2
x 2 3x + 2
=
= x 1 ;
x2
x2

b x 2g

Now, taking the limits on both sides as x 2 , we


have

lim

x2

x 2 3x + 2
x2

164

How to Learn Calculus of One Variable

b g

Now dividing Nr and Dr by the common factor


(x 2), we get,

= lim x 1 = 2 1 = 1
x2

7. lim

x3

x 3 3x 2 + 5x 15
x 2 x 12

Solution:

LM x
MN

3x 2 + 5 x 15
x + x 12
2

OP
PQ

x=3

x 2 x 12

b x 3g e x + 5j
=
b x 3gb x + 4g
2

lim

LM x 3x + 5x 15OP
MN x x 12 PQ
L x + 5 OP
lim M
MN x + 4 PQ
lim e x + 5j
=
lim b x + 4g
3

lim

x3

x2

9. xlim
1

LM x + 1OP
MN x + 1 PQ

=
x = 1

0
0

Nr has (x + 1) as a factor and Dr has (x + 1) as


a factor: (x + 1) is a common factor in Nr and Dr of the
given fraction.
Now, dividing Nr and Dr by the common factor
(x + 1), we get

ge

x + 1 x2 x + 1
x3 + 1
=
x +1
x +1

= x 2 x + 1 for x 1

x3

x3

lim

x 1

32 + 5 9 + 5 14
=
=
=
=2
3+4
7
7

x3 + 1
= lim x 2 x + 1
x + 1 x 1

= 1 ( 1) + 1 = 1 + 1 + 1 = 3.
Exercise 4.1

x2 4
x2

LM x 4 OP
MN x 2 PQ

x2 4
= lim x + 2
x2
x2

x 3

Problems set on method of factorization or division

Solution:

x3 + 1
x +1

8. xlim
2

gb

= 2 + 2 = 4.

Solution:

x2 + 5
; 3x 3
x+4

= x + 2 ; 3x 2

0
0

Nr has (x 3) = a factor and Dr has (x 3) = a


factor: (x 3) is a common factor in Nr and Dr.
Now, dividing Nr and Dr by the common factor
(x 3), we get
x 3 3x 2 + 5 x 15

gb

x2 x+2
x2 4
=
x2
x2

=
x=2

0
0

Nr has (x 2) as a factor and Dr has (x 2) as


a factor: (x 2) is a common factor of Nr and Dr of
the given fraction.

Find the limits of the following functions:


Answers

x 7 2x5 + 1
(Bhag65A)
x 1 x 3 3x 2 + 2
x 1
2. x L
(I.I.T.1976)
3
1 x 3x 2 + 2
1.

(1)

FG 1IJ
H 3K

165

Practical Methods of Finding the Limits

3.
4.

x2

2 x2 7x + 6
5x 2 11x + 2

x 2 7 x + 12
(Bombay69, 70)
x 3 64

x a
(R.U.65)
xa

x4

5.

(Bombay65)

xa

FG 1 IJ
H 9K
FG 1 IJ
H 48K

(4a3)

x 4x + 3

(2)

x3

2 x 11x + 15
x 3 + 27
7. L
x 3 x 5 + 243
x+3
8. L
x 3 x 3 + 27
9.

L1

FG 1 IJ
H 15K
FG 1 IJ
H 27 K
FG 5IJ
H 3K

1 32 x 5

1 8x 3
Type 2: Limits of irrational functions as x a .
x 2

Form 1:
Form 2:

bg bg
f b x g f b x g occurring in the Nr or

f x or f x
1

m
n

2. lim

xa

of each other.
(b)

a +

j and e

j are rationalizing

factors of each other.

ep

a +q b

j and e p

a q b

bg

f x =

N.B.:
1. Any

one

ep

of

ea + b j , e

bg

f b x g = LM lim f b x gOP
Q
N
m
n

m
n

xa

Working rule for from 2:


1. If only Nr contains the radical of the above form,
rationalize Nr by multiplying Nr and Dr by
rationalizing factor of Nr.
2. If only Dr contains the radical of the form
mentioned above, rationalize Dr by multiplying Nr
and Dr by rationalizing factor of Dr.
3. If Nr and Dr both contain radicals of the above
type, rationalize Nr and Dr both separately by
multiplying and dividing Nr and Dr by rationalizing
factor of Nr and Dr.

a +

or

a + q b may be provided in the question


whose limit is required to find out where a and b
indicate a function of x.
2. Our main aim is to remove the common factor by
rationalization of Nr or Dr or both Nr and Dr.
3. After removing the common factor, we put x = a =
limit of the independent variable x in the irrational
function free from the common factor known as
simplified form.

bg

becomes

Problems based on the form 1

f2 x

bg

bg

f x or f x

Solved Examples
Evaluate:
1. lim

x 1

8x + 1
x+3

Solution: lim

x 1

bg

f1 x

0
at x = a.
0

lim f x

xa

j are rationa-

lizing factors of each other.

4. The above rule is valid when

Working rule for form 1:


If the given function has the form 1 mentioned above,
we adopt the following working rule:
Use the following formulas:
xa

ea + b j and ea b j are rationalizing factors

in Dr or in both Nr and Dr.

1. lim

(a)

(c)

6.

Facts to know about rationalizing factor:

8x + 1
x+3

F 8x + 1I
lim G
H x + 3 JK

x 1

a f
lim a x + 3f

lim 8 x + 1

x 1

x 1

m
n

166

How to Learn Calculus of One Variable

lim 8 x + lim 1

x 1

x 1

lim x + lim 3

x 1

bg

81 +1
1+3

x 1

3
9
= .
2
4

2.
3.

bg

bg

e 3j

16 x

e 15 j

b x 4g

1 3x

bg

f1 x

Problems based on the form 2

1. xlim
1

Solution:

Now,

x +1

bg

f2 x

x 1

x2 1 +

x 1

x2 1

x2 1 +

OP
PQ

x =1

x1

x2 1

b x 1gb x + 1g + x 1
b x 1gb x + 1g
b x 1g x + 1 + 1
x 1 e x + 1j

0
=
0

=
=

x 1

LM
MN

g OP
PQ

x3 1 x 1
x 1

=
x=1

0
0

x3 1 x 1
x 1

b x 1g ex

j b

+ x +1 x 1
x 1

x2 1

LM
MN

Solution:

Now,

2 +1

x3 1 x 1

x 1

Examples worked out:


Evaluate:

x2 1 +

x 1

1+1

2. lim

(10)

x 1

1+1 +1

2x + 1

x 1

x2 1 +

x 1

m
n

x 1

; (for x 1 )

x +1+1

= lim

Answers
x 1

x 1

f x or f x
Find the limit of the following:

1.

x +1

Hence, lim

Exercise 4.2

On the form

x +1+1

x 1

LM
N

x2 + x + 1
x 1

x 1

= lim

x 1

LM
N

x3

OP
Q

b
1 b x 1g

x2 + x + 1

Hence, lim

x1

x 1 ; for x 1

x 1

x2 + x + 1

= 1+1+1 11 =

x 1

OP
Q

30=

Practical Methods of Finding the Limits

Note: The above examples gives as a hint that we


should not rationalize the given irrational function
blindly but we must check whether it is possible to
remove the common factor (or, not) by factorization
which is present in Nr and Dr of the given irrational
function, i.e., if it is possible to remove the common
factor from the numerator and denominator of
irrational function after factorization and then using
cancellation, we must remove it.

x a

b x ag
L x a OP
Solution: M
MN b x ag PQ
3. lim

Hence,

xa

x a

b x ag

Now,

x5

Solution:

e xj e aj
2

=
5. lim

25

OP
PQ

x4

=
x=5

0
and Nr contains
0

as irrational expression which means rationalizing of


Nr is required.

2+

x 1

b x + 5g e2 +
2

1
x 1

x1

x1

x1

x1

x 25
2

a x + 5f e2 +

b5 + 5g e
10

x1

x 1

4 x +1

x 25

2+

b g
b x 5gb x + 5g 2

x5

2 a

22 x 1

= lim

x 1

x1

x 25

x5

LM 2
MN x

25

lim

x + a

a + a
2

x a

b x + 5gb x 5g e2 +
b x 5g
=
b x + 5gb x 5g e2 +

xa

4. lim

x=a

0
0

xa

x a
xa

Hence, lim

= lim

x +

x1

j e x aj
; b for x ag
a

x +

e2
ex

167

x 1

1
51+2

4 +2

1
10 2 + 2

1
1
=
10 4
40
3 5+ x
1 5 x

Solution:

LM 3
MN 1

OP
5 x PQ
5+ x

=
x=4

0
0

Since Nr and Dr both contain radicals whose


factorization is not possible which means we are

168

How to Learn Calculus of One Variable

required to rationalize Nr and Dr both separately on


multiplying and dividing by the rationalizing factor.

Nr = 3

5+ x

e3

je

5+ x 3+ 5+ x

e3 +

5+ x

b3g e 5 + x j
=
e3 + 5 + x j
2

95 x

e3 +

5+ x

e3 +

6. lim

x 1

x 2 x2
2x 2 + 2x2

Solution: lim

x 1

2x 2 + 2x2

{x e2 x j}{2 x + 2 + 2 x }
2 + 2x I F x + 2 x I F2x + 2 + 2x I
KH
KH
K
2

= lim

x 1

FH 2 x

{x e2 x j}{2 x + 2 + 2 x }
= lim
{4 x e2 + 2 x j}{ x + 2 x }
x 1

Nr
Dr

1+ 5 x

Now, given expression =

x 2 x2

Now on rationalizing the Nr and Dr with the help


of rationalizing factor of Nr and Dr.

x 4

e3 + 5 + x j
=
b x 4g
e1 + 5 x j

j
5 + xj

F
I
b g GH 13 ++ 55 + 44 JK
b 1gb1 + 1g = 2 = 1
=
b3 + 3g 6 3

5 x

b4 x g

1+ 5 x

= 1

1+

j
5 + xj

5 x

x4

e3 + 5 + x j
Dr = e1 5 x j
e1 5 x je1 + 5 x j
=
e1 + 5 x j
b1g e 5 x j
e1 + 5 x j
1 b5 x g
=

e1 +
e3 +

Required limit = lim

4x

b4 xg e1 + 5 x j for
x4
b4 x g e3 + 5 + x j

e2 x 2jFH 2 x +
= lim
e2 x 2j FH x +
2

x 1

2 + 2x2
2 x2

IK

IK
; (for x 1 )

Practical Methods of Finding the Limits

= lim

2x + 2 + 2x2

x 1

x+

2 x

[on removing the common

factor]
=2
7. xlim
2

4.

(3)

x 1

x 1

5.

x + 8 10 x

x 1

x +3

4 x2
3

x2 + 5

6.

e4 x j
2

Solution: lim

x2

x2 + 5

7.

Now on rationalizing the Dr only since radical


appears in Dr only

e4 x jFH 3 + x + 5IK
= lim
FH 3 x + 5IK FH 3 + x + 5IK
e4 x j FH 3 + x + 5IK (for x 2 )
= lim
;
e4 x j
= lim F 3 + x + 5I
H
K
2

x2

x2

x 1

x 1

9.

10.

x3

x 1

x2

= 6.
Problems based on method of rationalization

FG 1IJ
H 3K

x 11
x 2

FG 1 IJ
H 10 K

FG 1IJ
H 3K

8.

5 x

FG 2 IJ
H 3K

f
a2 x 3f e x 1j
e2 x + x 3j

x2

11.

x4

Type 3:

x +31
x+2

2x + 3 3

b x 6g

(0)

F 1I
GH 2 5 JK
FG 1IJ
H 3K

x+4 5
x 1

3 5+ x
1

5 x

bg

bg

Form: f 1 x f 2 x = as x a

Exercise 4.3

Working rule: To evaluate


Find the limit of the following
1.

2.

3.

x 1

x 1

x4

2 x 1
(M.U. 68)
1 x

Answers

FG 1 IJ
H 2K

x 1+
2 x
x4

bg

x 1

(0)

FG 1 IJ
H 4K

bg

f1 x f 2 x =

as x a our main aim is to reduce the form

b g to the form 00

which can be explained in

the notational form in the following way:

x 1

169

bg

bg

1. Write f 1 x f 2 x =

1
1

bg

f1 x

1
1

bg

f2 x

170

How to Learn Calculus of One Variable

1
1

f2 x
f1 x
0
=
2. Write 1 =
whose limit is
1
1
0

f1 x
f2 x

bg
bg

bg
bg

found by the method already explained.

Note: 1. In practice, to reduce the form to

0
, we generally simplify the given
0
expression by using any mathematical operation (like
taking l.c.m or changing all trigonometric functions in
to sin and cosine of an angle, etc.). After obtaining
the form

0
, we are able to find its limit, e.g.:
0
(i) lim tan x sec x , = form

the form
x 2

gb

sin x
1
tan x sec x =

cos x cos x
=

sin x 1 0

= as
cos x
0
2

1 cos
2

1 1 + 2 sin 2

2 sin
2 = 0 as 0 .
=
0
2
2

LM 2
MN1 x

1
1 x

OP b= formg as x 1
PQ
1

b1 + xgb1 xg b1 xg
2 b1 + x g
b1 xg = 0 as x 1
=
=
e1 x j b1 xgb1 + x g 0
=

LM 1 1 OP = , we take
MN f b xg f b xg PQ
1

first l.c.m and then we subtract. Taking l.c.m and


0
subtracting reduces the form to the form
0
for which we adopt the usual method of removing
common factor of Nr and Dr as factorization of
cancellation, etc.

Problems based on the form:

LM 1 1 OP b= as x ag
MN f b xg f b xg PQ
1

Examples worked out:


Evaluate

LM 2 + 1 OP
MN1 x x 1PQ
L 2 + 1 OP
Solution: M
MN1 x x 1PQ
1. lim

x 1

(iii)

xa

F 1 cos IJ , b= formg
lim G
H K

(ii)

Note 2: If lim

Now,

LM 2
MN1 x

OP b
PQ

2
1

2
1 x
1 x

b g
b gb g
b1 xg = 1
=
b1 xgb1 + xg b1 + xg
L 2 + 1 OP = 1
Thus, M
MN1 x x 1PQ 1 + x
=

1
, = as x 1
1 x

2 1+ x

1 x 1+ x

(1)

Now taking the limit on both sides of (i) as x 1 ,


we get

lim

x 1

LM 2
MN1 x

1
x 1

OP
PQ

171

Practical Methods of Finding the Limits

= lim

x 1

1
1
1
=
=
1+ x 1+1 2

lim

LM 1 2 OP
MN x 1 x 1PQ
L
O
Solution: M 1 2 P ; b= as x 1g
MN x 1 x 1PQ
2. xlim
1

1
2
x2 + 1 2
4
=
x 1 x 1
x2 1 x2 + 1

Now,

ex

je

=
(1)

LM 1 2 OP = lim LM 1 OP
MN x 1 x 1PQ MN x + 1PQ

lim

x 1

3.

xa

Solution:

2a
1

2
2

x
a
a

LM 2a
MN x a
2

L
Now, M
MN x

2a
2
a2

a x

x2 a2

x2
x 4
2

OP
PQ

4
4

OP
PQ

OP b= as x 2g
PQ

x+24
4
=
4
x2 4

b x 2g
b x + 2gb x 2g

1
x + 2 ; (for x 2 )

LM 1
MN x 2 x

lim

1
x+a

LM 1
MN x 2 x

x2

OP b
PQ
1 O 2a x a

P= x a
x a PQ
e
j

OP
Q

1
1
1
=
=
x+a
a + a 2a

LM 1
MN x 2 x
b

LM
N

OP = 1
4 PQ x + 2

4
2

(1)

Now taking the limits on both sides of (1) as


x 2 , we have

OP
PQ

1
, = as x a

xa

Thus,

1
1
=
1+1 2

L
lim M
MN x

x 1

LM
MN

OP
PQ

2a
1
1
= lim

2
x
a

xa
x+a
a

1
4
2
4. xlim
2 x 2
x 4

Now taking the limits on both sides of (1) as


x 1 , we have

x a

Now,

LM
MN x

= lim

Solution:

e x 1je x + 1j x + 1
L 1 2 OP = 1
Thus, M
MN x 1 x 1PQ x + 1
2

xa

x2

Now taking the limits on both sides of (1) as x a

OP
PQ

4
1
= lim
x

2
x+2
4

1
1
=
2+2 4

(1)

LM 1 1 OP
MN x 2 x 3x + 2 PQ
L 1 1 OP b= formg
Solution: M
MN x 2 x 3x + 2 PQ
5. lim

LM 1
MN x 2 x

as x 2

172

How to Learn Calculus of One Variable

Now,

=
=

LM 1
MN x 2 x

1
3x + 2

OP
PQ

1
1

x2
x 2 x 1

gb

x1

gb g
1
=
; b for x 2g
x 1

5.

x2

6.

OP
PQ

1
1
=
x
1
3x + 2

LM 1 1 OP
MN x 2 x 3x + 2 PQ
L 1 OP = 1 = 1
= lim M
N x 1Q 2 1
lim

x2

2.

FG 1IJ
H 3K
(0)

FG 3IJ
H 2K
FG 1IJ
H 8K
FG 1 IJ
H aK

xn a n
= n a n 1 ; (where n is an integer > 1)
xa

xa

xm am
m mn
; (where m and n are
=
a
n
n
n
x a

integers > 1)

LM 1 3 OP
MN1 x 1 x PQ
1 L 1
2 O
L

M
x 1 N x + 3 3 x + 5 PQ

x 1

xa

(ii) lim

1 1
or, as x a
0 0

Find the limit of the following

x 1

8.

xa

(i) To show: lim

xa

xn an
= n a n 1 provided n is
xa

an integer > 1
Proof: 3 we know that

Exercise 4.4

1.

(i) lim

Problems based on the form:

x8

FG 7 IJ
H 2K

Problems based on the formulas:

x2

x2

LM 1 1 OP
MN f b xg f b xg PQ

7.
... (1)

Now, taking the limits on both sides of (1) as


x 2 , we have

1
x
2

x3

LM 1
MN x 2 x

4.
1

b x 2gb x 1g b x 2gb x 1g

x2
=
x2 x1

Thus,

LM8x 3 4 x + 1OP
MN 2 x 1 4 x 1PQ
L 1 3 OP
L M
MN x 3 x 3x PQ
L 1 + 6x OP
L M
MN x 2 8 x PQ
OP
L 1
2
L M
N x 2 x a x 1fa x 2f Q
L 1 8 OP
L M
MN x 8 x 8x PQ
L 1 a OP
L M
MN x a x a x PQ
2

3.

Answers

xn an

ge

= x a x n 1 + a x x 2 + a 2 x n 3 + ... + a n 1

(1)

FG 1 IJ
H 32 K

(1)
on dividing both sides of (1) by (x a), we get

xn an
xa

173

Practical Methods of Finding the Limits

bx agex
=

n 1

+ a x n 2 + a 2 x n 3 + ... + a n 1

bx ag

On dividing both sides of (1) by h, we get

xn an
h

= x n 1 + a x n 2 + a 2 x n 3 + ... + a n 1 (2)

Now, on taking the limits on both sides of (2) as


x a , we get

lim

xa

xn an
xa

= lim x n 1 + a x n 2 + a 2 x n 3 + ... + a n 1
xa

an n
=

xa

lim a 2 x n 3 + ... + lim a n 1

xa

xa

=a

n 1

+ aa

=a

n 1

n 1

+a

n2

+a

+ a a n 3 + ... + a n 1
n 1

+ ... + a

n 1

(up to n terms)

b g a
R|F h I U|
R|F h I U|
a SG 1 + J V a = a SG 1 + J 1V
T|H a K W|
T|H a K W|
LR| n F h I n bn 1g F h I U| OP
= a MS1 + G J +
MN|T N1 H a K N2 GH a JK + ... V|W 1PQ
h L
n1 h
= a n M1 +
a N
N2 a +
O
terms containing high powers of hP (1)
Q
n

xn an
xa
a n n

LM
N

h
n 1 h
1+
+ terms containing higher powers of h
a
a
2
h

OP
Q

(3)
Lastly on taking the limits on both sides of (3) as
h0

xn an
h0 x a
lim

an n
= lim

h 0

LM
N

h
n1 h
+ terms containing higher powers of h
1+
a
2 a
h

= an n
lim

xa

OP
Q

1
= n a n 1
a

xn an
= n a n 1 3 x a h 0
xa

x m a m m m n
= a
where m and
n
xn an

(ii) To show: xlim


a
n are integers > 1

OP
Q

= n a n 1
N.B.: This relation is true for n = any rational number
whose proof is provided with the help of binomial
expansion.
x = a + h x a = h and as x a , h 0
a+h

(2)
On putting x a = h on the l.h.s of (2), we get

= lim x n 1 + lim a x n 2 +
xa

LM
N

h
n1 h
1+
+ terms containing higher powers of h
a
a
2
h

Proof:

x a

m
n

ex a j
a x af
=
ex a j
a x af

b x ag ex
=
b x ag e x

m 1
n1

j
j

+ a x m 2 + ... + a m 1
+ ax

n2

+ ... + a

n 1

174

How to Learn Calculus of One Variable

x m 1 + a x m 2 + ... + a m 1

(1)

x n 1 + a x n 2 + ... + a n 1

Now, on taking the limits on both sides of (1) as


x a , we get
x a

= lim

xa

m1

n 1

1. lim

xa

LM x
MN x

5/ 2
1/ 2

a 5/ 2
a 1/ 2

OP
PQ

x 5 / 2 a 5/ 2
x a x 1/ 2 a 1/ 2

xm am
xn an

lim

Examples worked out:


Evaluate:

Solution: lim

+ ax
+ ax

m 2
n2

+ ... + a
+ ... + a

m 1
n 1

5 / 2 ab
1/ 2

5/ 2 1/ 2

ma

na

m 1

n 1

= 5a 2

m m n
a
n

= 5a 2

N.B.: This relation holds true even if m and n are


rational numbers.

x a
Aid to memory: 1. lim
x a x a
n

an

= index of power

a
2. xlim
a x n a n
x

Problems based on the formulas


n

(ii)

x a

xa

x a
xa

Solution: lim

x 64

m
n

= na

1
x3

n 1

1
x6

= lim

x 64

1
x3

m mn
a
n

Working rule: We should use the formulas (i) and


(ii) directly provided that given function has the form

xm am
xn an
either (i)
, or (ii) n
and we are
xa
x an
required to find the limit of these functions as x a .

b g
b64g
64

1 GH 6 3 JK
a
(using formula)
= 6
1
3

F 1 1I

F 1 2 IJ
6 K

G J 1 G
1 3
= a H 6 3K = a H
6 1
2
1

x a

1
x6

F 1 1I

constant raised to the power m minus n.

x3 4

m = index of the power of the contant appearing in Nr


the
n = index of the power of the contant appearing in Dr

(i)

2. lim

x 64

times base a raised to the power n minus 1.


m

x6 2

1 6 1 1
a = 1
2
2
a6

1 1
2 6a

1
6

64

bas a = 64g

e j

1
26 6

1 1 1
=
2 2 4

1
6
1
3

Practical Methods of Finding the Limits

3. lim

xa

or alternatively,

x4 a4
xa

lim

x 1

x a
= 4a 4 1 (using the formula)
xa
4

Solution: lim

xa

4.

LM x
lim M
MN x

xa

3
2
2

OP
PP = 4a
Q

a2
a2

Fx
lim G
GG x
H

Solution:

xa

3
2

I
JJ = 2
JK

1
4

(i)

3
1
3

=
4
a
4 a

bg

1
3
1 d 4 i ( 3 Here,
1
4

1
) (according to formula)
4
1

b1g

3
4

xm am
xn an

(ii)

xn an
xa

Find the limit of the following:


1.

xa

x a
x a

= lim

x 1

1
1

bg

x4 1
x 1

x2

1
4

x 32
3

x 8
2

3.

x 2

Answers

FG 3 a IJ
H5 K
2

2.

bg

1 1
1
1 d4 i
4
1
=
1
3
1 c 4h
4
1
=
1
1
4
4
=1
1
=4

af

=4

1
4

x 1

1
4

Exercise 4.5

L x 1 OP
5. lim M
MN x 1PQ
L x 1 OP
Solution: lim M
MN x 1PQ
x 1

Problems based on the form

F 3 4 IJ
2 K

F 1I
3 GH 2 JK
a

x 1

b gd

1
1
1
4

4
1

F3 I
3 GH 2 2JK
a

1
4

=4

a2
a2

x1

m = 1, n =

G
3 1
= aH
2 2
=

175

x 4
x x 2 2

FG 20IJ
H 3K
4 2
3

4.

x2

x 32
x2
5

5.

6.

7.

x2

x 32
4

x 16

x 2
x2

x 3
x3

x2

x3

(80)

FG 5 IJ
H 2K
F 1I
GH 2 2 JK
F 1I
GH 2 3 JK

176

How to Learn Calculus of One Variable

LM x + 5x OP = 0 form as x 0 (x 0)
MN 4 x PQ 0
2

8.

1 x
x 1 1 x

(7)

x +8
9. L
x 2 x + 2

(12)

Solution:

is a factor of Nr and Dr on dividing Nr and Dr by


(x 0) = x, we get

Problems based on types of functions mentioned


earlier but x 0 instead of x a
Working rule: When x 0 (zero) and types of the
functions are same as mentioned earlier when x a
(any constant)
Rule: When x 0 , the same rule is applied to find
out the limit of a function as x a which means
rule to find out the limit of a function as x a
(constant) = rule to find out the limit of a function as
x 0 (zero).
Theorem: If f (x)
= a polynomial in x = a0 x n + a1 x n 1 + ... + an
(where a0, a1, a2, an are constants and n is a +ve

bg

f x = a n = the constant present


integer) then xlim
0

in the given polynomial in x which is free from the


variable raised to any +ve index.

LM x + 5x OP = lim L 0 + 5 O = 0 + 5 = 5
MN 4 x PQ MN 4 PQ 4 4
2

lim

x0

x0

x +11

3. lim

x0

LM
MN

Solution:

x0

x0

x0

= lim

x0

x 0

= 3 lim x 2 + 4 lim x 2 5 lim x + 6


x0

x0

= 3.0 + 4.0 5.0 + 6


= 6.

LM x + 5x OP
MN 4 x PQ
2

2. lim

x0

LM
MN

x+11
x

x0

= lim

x0

4. lim

x0

OP
x + 1 1 PQ
x+1+1

b x + 1g 1 = lim
x e x + 1 + 1j
xe
x0

x0

lim 3x + 4 x 5x + 6
2

OP = 0 form as x 0
on ratiPQ 0

x +11

lim

= lim 3 x 2 + lim 4 x 2 lim 5x + lim 6


x0

x +11

x0

x0

2
2
Solution: lim 3x + 4 x 5x + 6

onalizing the Nr, we get

= lim

3x + 4 x 5x + 6
Evaluate: 1. xlim
0
2

... (1)

On taking the limits on both sides of (i) as x 0 ,


we get

Examples worked out:


2

x/ x + 5
x + 5x
x+5
=
=
4x
4 x/
4

x+1+1

x+1+1

1
0+1+1

1
1
=
1+1 2

x+a a
x

LM x + a a OP = 0 form as x 0 on,
MN x PQ 0
rationalizing the Nr, we get
Solution:

Practical Methods of Finding the Limits

x0

= lim

x0

LM
MN

x+a

x+a

x0

5. xlim
0

OP
a PQ

x+a +

x0

x+a +

x0

= lim

x0

lim

bx + ag a

= lim

x+a +

b3 log 1 = 0g on rationalizing the Nr, we get


L x + 1 1OP
lim M
MN log b1 + xg PQ
L x + 1 1 x + 1 + 1OP
= lim M
MN log b1 + xg x + 1 + 1PQ

x+a a

lim

0+a +

x0

x0

2 a
= lim

x0

1 1 x

OP = 0 form as x 0
x
,
1 x PQ 0

Solution:

on

rationalizing the Dr, we get

L x OP
lim M
MN1 1 x PQ
L x 1+
= lim M
MN1 1 x 1 +
x e1 + 1 x j
= lim
x0

lim

x0

F
GH 1

x0

Solution:

g e

x +1+1

LM x + 1 1OP
MN log b1 + xg PQ
1

log 1 + x
1

0+1+1

1
x

x+1+1

1
log e e

LM x + 1 1OP = 1 1
MN log b1 + x g PQ 2
F3 lim b1 + xg = eI
H
K
L x + 1 1OP = 1
lim M
MN log b1 + xg PQ 2
L 5 + 3x OP
7. lim M
N 7 2x Q
L 5 + 3x OP 0 which given function
Solution: M
N 7 2x Q 0
F b5 + 3xg I does not assume FG 0 IJ form at x = 0
GH b7 2 xg JK
H 0K
L 5 + 3x OP = lim b5 + 3xg
lim M
N 7 2 x Q lim b7 2 xg
1
x

OP
1 x PQ

x0

1 x

x0

I = lim 1 +
e
J
1 xK

x0

1 x

x+11
log 1 + x

ge

x0

=1+ 1 0
= 1 + 1 = 2.
6. lim

lim

x0

x0

log 1 + x

lim

LM
MN1

177

LM x + 1 1OP = 0 form as x 0
MN log b1 + xg PQ 0

x0

x0

x0

x 0

178

How to Learn Calculus of One Variable

lim 5 + lim 3x

x0

x0

lim 7 lim 2 x

x0

x0

5 + 3.0
5
=
7 2 .0 7

b1 + xg
lim

First important form:

ba xg

1
5

x ( = independent variable)

x0

as x 0

1
5

zn a n
as z a
za

za

1.

x0

1
n

LM b1 + xg
MN x

1
n

x0

2.

b1 + xg
lim

x0

1
5

1 1
n

1 1
5

1
5

1+ x 1

x
Solution: We put 1 + x = z
Now, 1 + x = z z 1 as x 0

zn an
= n a n 1
za

lim

x0

LM
MN

1+ x 1

OP = lim 1 + x 1
b1 + xg 1
PQ
x0

x
1

b1 + xg b1g
b1 + xg 1

bg

z 2 12
z 1 z 1

= lim

1
n

1
1
n

1
5

x0

OP 0
PQ = 0 form as x 0

We put 1 + x = z
Now, 1 + x = z z 1 as x 0

lim

af

1
1
5

3. lim

Solution:

b1 + xg 1
= lim
b1 + xg 1

z 5 15
z 1

z 1

Examples worked out:


Evaluate:

b1 + xg
lim

OP 0
PQ = 0 form as x 0

x0

= lim

Working rule:
1. Put a x = z
2. Change the limit as x 0 z a
3. The above substitution transforms the given
problem in to the form:

4. Use the formula: lim

We put 1 + x = z
Now, 1 + x = z z 1 as x 0

Some Important Forms

LM b1 + xg
MN x

Solution:

1
n

1
n

= lim

z 1

LM z b1g
MN z 1
1
n

1
4

OP
PQ

4.

bg

1
1
2

1
2

1 1
2

b x + hg
lim

1
2

x2

h 0

h
Solution: We put x + h = z
Now, x + h = z z x as h 0

b x + hg
lim
h 0

1
=

1
2

x2

z2 x2
z x
zx
( 3 h = x + h x = z x)

= lim

1 12
1
1
=
x =
1
2
2
2 x
2x

Practical Methods of Finding the Limits

b x + hg
lim

5.

Problems based on limits of a function as x 0

x2

h 0

h
Solution: We put x + h = z
Now, x + h = z z x as h 0

lim

h 0

b x + hg

Exercise 4.6

b x + hg x
b x + hg x
2

= lim

h 0

Find the following limits:


2

x0

Second important form: A rational function in x


whose Nr and Dr consist of a polynomial of fractional
indices and the independent variable x 0 .
Working rule: Divide Nr and Dr (each term of Nr
and Dr) by the lowest power of x occurring in the
given function

3. lim

5x 3
3x 3

4. lim

x2
x

x0

x 0

x0

LM x
MN x

Solution:

1
3

4
5

+ 3x + 2 x
2
3

+ 4x + 2x

1
5

OP = 0 form as x 0 .
PQ 0

Now, on dividing Nr and Dr by the lowest power

(4)

(0)
(0)

1
x

does not exist

does not exist

x3

8. lim

3x + 4
5x + 6

FG 2 IJ
H 3K

9. lim

2x2 + x 2
3x 2 x + 1

(2)

x 3 + 4x 3 + 2x 5
7
10

FG 5IJ
H 3K

x 0

Answers

(1)

5. lim

7. lim

x
x

x 0

x 10 + 3x 5 + 2 x

x0

2. xlim
0

6. lim

Examples worked out:


Evaluate
1. lim

4
3
1. lim x + 9 x 7 x + 4

z2 x2
= 2x
= lim
z x z x

179

x0

x0

x 5 , we get

x 10 + 3x 5 + 2 x
1

x 3 + 4x 3 + 2x 5

10. lim

x 2 + 3x 5 + 2 x 5
7

x 15 + 4 x 15 + 2

x0

(1)

Lastly, on taking the limits on both sides of (1), as


x 0 , we get
7

lim

x0

x 10 + 3x 5 + 2 x
1

x 3 + x 3 + 2x 5

0+0+0 0
=
= =0
0+0+2 2

= lim

x0

x 15 + 4 x 15 + 2

x + x

11. lim

b x 1gb2 x + 3g
b x + 5gb3x 2g

12. xlim
0

x 2 3x + 2
x2 + x 6

x0

x 2 + 3x 5 + 2 x 5

6x 2 x + 5

13. xlim
0
14. lim

x 0

2 x 2 + 3x
3x 2 5x
x 2 4x 5
x2 + x 2

b g
FG 3 IJ
H 10 K
FG 1 IJ
H 3K
FG 3IJ
H 5K
FG 5 IJ
H 2K

180

How to Learn Calculus of One Variable

x 2 + 5x
15. lim 2
x0 x + x
1+ x 1 x

16. lim

x0

x 1+ x

x0

9x 3
x
x2
x2
x5

26. lim

h 0

does not exist

1 1 x
x
3

h 0

(2)

1 1 x2

x0

25. lim

FG 3 IJ
H 2K
FG 1 IJ
H 6K

1+ x +1

x0

24. lim

(a + |a| )

1+ x 1

21. lim

x 0

a a2 x2

0 3

23. lim

(1)

1 x4 1 x

20. xlim

x0

1 + x 1 + x4

18. lim

22. lim

FG 5 IJ
H 2K

1 + 2 x 1 3x

x0

x0

(1)

17. lim

19. lim

(5)

x+h

h
x+h
h

To evaluate lim

polynomials in x.

FG 1IJ
H 8K
F 1 I
GG JJ
H3 x K
F 1I
GH 2 x JK

af
af

f1 x
, where f1 (x) and f2 (x) are
f2 x

Working rule: One should:


1. Divide f1 (x) and f2 (x) by the highest power of x
occurring in the given fraction, i.e., divide each term

of the numerator and denominator of the given


fraction by the highest power of x occurring either in
numerator or in denominator. After division by
highest power of x present either in numerator or in
denominator of the given fraction, each term of the
numerator and denominator of the given fraction will
be reduced to the forms:

b c d
, , etc. where a, b, c, d, , are
,
,
x x2 x3
constants.
2. Take the limits of each term of numerator and
denominator both as x noting that
a,

b c d
,
,
, , etc. (appearing in numerator and
c x2 x3
denominator) all 0 excepting a constant a1 in
numerator and another constant a2 in denominator

whose quotient

FG a IJ will give us the required limit


Ha K
1

of the given fraction, if a2 0 .

Notes: 1. Highest power of x may be present in either


numerator or in denominator.
2. Highest power of x of numerator and denominator
may be the same.
3. The determination of limit of a function y = f (x) as
x + and x are also sometimes used to
find out the range of y = f (x) when x , +
and f (x) is continuous in R.

Explanation: 1. f (x) = x

a f L a xf =
f a xf = L a x f = +
R b f g = b , + g , since

lim f x =
x

and

lim

x +

Hence,

f (x) is

continuous.
2. f (x) = x2

ex j = + and
2

x +

ex j = +
2

Further x 2 0 and x 2 = 0 for x = 0

bg

Hence, R f = 0, + , since f (x) is continuous.

Practical Methods of Finding the Limits

3. f (x) = x3

ex j = and L ex j = +
Hence, R b f g = b , g , since f (x) is continuous.
Fe e I
4. f b x g = G
H 2 JK
L f a x f = and L f a x f = +
Hence, R b f g = b , g , since f (x) is continuous.

= lim

= lim

LM 2 x OP lim LM x OP
N x 1Q
N x + 1Q
LM OP
LM OP
2
MM 1 PP lim MM 1 1 PP
Nx x Q
N1 + x Q
x

=21=1

3. xlim

3 e2 x + 2 e 2 x
4 e2 x e 2 x

Problems based on the form:

lim

b g , where f (x) and f (x) are polynomials


b xg

f1 x
f2

181

Solution: xlim

3 e2 x + 2 e 2 x
4 e2 x e 2 x

in x.
Examples worked out:
Evaluate:

= lim

3 e2 x +
4 e2 x +

2
e2 x
1

2 x 3x + 5

e2 x

1. xlim

3x 2 + 27 x 29

Solution: xlim

2 x 2 3x + 5

= lim

3x 2 + 27 x 29

3
5
+ 2
x
x
(on dividing Nr and Dr
= lim
27 29
x
2
3+
x
x
2

by the highest power of x, i.e., x2 )

2
=
3

3+0
4+0

3
4

3+
4+

2
e4 x
1
e2x

[N.B.: 3 as x , e 4 x

2
e4x

0]

Problems based on the form:

N.B.: As x ,

1
1
0, 2 0
x
x

LM 2 x x OP
N x 1 x + 1Q
L 2 x x OP
Solution: lim M
N x 1 x + 1Q

lim

a0 x n + a1 x n 1 + a2 x n 2 + ... + a
b0 x n + b1 x n 1 + b2 x n 2 + ... + b

2. xlim

Exercise 4.7
Find the following limits:

1. xlim

6 x 3 5x 2 + 4
4 x 4x + 7
3

Answers

FG 3IJ
H 2K

182

How to Learn Calculus of One Variable

2. lim

ax + bx + c
2

l x + mx + n

3. xlim

x4 x2 + 3
x 2 + 5x + 13

4. lim

x + x + x +1
x5 + 1
3

5. xlim

b g

makes the coefficient of highest power free from


highest power of x in the following examples.

5x 3
7x + 8
ax + b
cx

7. xlim

5x 2 + x + 1
6 x 2 3x 5

8. xlim

2 x 2 x + 100
4x3 + 2

b x + 1gb2 x + 3g
b x + 2gb3x + 4g

a
= 0 if n > 0
xn

5. Remember that lim

Examples worked out:


Evaluate:

6. xlim

9. xlim

FG a IJ
H lK

(0)

FG 5 IJ
H 7K
FG a IJ
H cK

x4 + 1

1. lim

= lim

FH

x4 + 1 x4 1
x4 + 1 +

IK ;

x4 1

(on rationalizing Nr)

FG 5 IJ
H 6K

= lim

(0)

= lim

FG 2 IJ
H 3K

Problems based on rationalization when x


Remember:
1. If only Nr contains radical, rationalize Nr by its
rationalizing factor (rationalizing factor is also known
as conjugate).
2. If only Dr contains radical, rationalize Dr by its
conjugate.
3. If Nr and Dr both contain radicals, rationalize Nr
and Dr both by its conjugate.
4. After rationalization, we have
Nr = an expression in x
Dr = an expression in x with radical or without
radical, then we divide the rationalized function by
the square root of highest power of x (i.e.
highest power of x ) seeing the power of x under
the radical sign of the radical signs in Nr and Dr
contain an expression in x, i.e. the process of division
of rationalized function by highest power of x

x4 1

=0
2. lim

LM
N

Solution:

FH

FH

x4 + 1 x4 + 1
x4 + 1 +
2
x4 + 1 +

x2 + 1

LM
N

x2 1

x2 + 1

x2 1

x +1+

x 1

IK

x4 1

OP
Q

x2 1

x2 + 1

OP
Q

x2 +1 +

x2 + 1 x2 1
x2 + 1 +

x2 1

2
x +1+
2

lim

FH

x2 1

x2 + 1

IK

x4 1

IK

x2 1

x2 1

183

Practical Methods of Finding the Limits

= lim

x +1+

Solution: lim x
x

x 1

2
x

= lim

1+

= lim

1
1
+ 1 2
2
x
x

(dividing by
=0

FH

3. lim x x
x

x2 + 1

FG
H

IK

= lim

x +1

x2 + 1

j
LMx + x + 1OP
N
Q
x b 1g
L x 1 OP
x M1 +
MN x + x PQ

LM1 +
MN
b 1g

b 1g

FH

x +1
2

x x2 + 1 x2 + 1
x2 + 1 +

x2 + 1 +

IK

IK
IK

x2 1

x2 = x )

2
2

x2 = x
This is why we divide each term of Nr and Dr by x
Highest power of x in Dr =

x2 = x )

16x 2 9 x + 5

9 x 2 + 5x 7

b2 x 3g

Solution: lim

16x 2 9 x + 5

16
1I
K

x2 1

2 x*

x2 1

IK OP
PP
Q

1
1
1+ 2 + 1 2
x
x

1
=
=
2
1+ 1+0

4. lim x

x2 + 1 +

x2 1

1+ 1

5. lim

OP
PQ

1
1+ 2
x

FH

x2 + 1 +

=1
Note: * Highest power of x in Nr = x

(dividing by

IK FH

x2 1

(dividing by

2
2

x2 1

2x
x

= lim

= lim

= lim

IK OP
PP
Q

IK

x2 + 1

x2 + 1

FH

x x2 x2 + 1

= lim

= lim

IJ
K
x + 1IK FH x +
FH x + x + 1IK

LM x FH x
MM
N

FH

= lim

Solution: lim x x
x

x2 = x )

LM x FH
MM
N

FH

lim

9 x 2 + 5x 7

b2 x 3g

9
5
+ 2
x
x

9+

FG 2 3 IJ
H xK

5
7
2
x
x

184

How to Learn Calculus of One Variable

x2 = x )

(dividing by

16 0 + 0 9 + 0 0

=
=

20

Some Miscellaneous Problems


Evaluate:

43
2

LM
N

N.B.: 3

e3n

1. lim

1
=
2

j e2n

1 1
,
0 as x
x x2
3x + 4 x + 5

6. lim

b3x + 7g

Solution: lim

x 0 as n
Making this substitution and after simplification,
we get

2x + 3

3x 2 + 4 x + 5

2x2 + 3

3x 2 + 4 x + 5

F 3x
GH x

= lim

I
JK

4x
5
+ 2 + 2 x2
x
x

x 3+
= lim

3+
= lim

IJ
K

4
5
+ 2
x
x
7
3+
x

FG
H

3+00 2+0
3

F 2x
GH x

4
5
3
+ 2 x 2+ 2
x x
x
7
x 3+
x

FG
H

IJ
K

2+

3
x2

lim

e3 x j e2 + x j

j e2n

I
JK

3
+ 2 x2
x

3 2
4

An important form:
Form:

b g or f b xg
b xg f b x g

f1 x
n

f2

Examples Worked Out:


Evaluate:
x2 3

1. lim

ex

+1

x2 3

Solution: lim

+1

4 + 3x

1
4

4n + 3
2

F 3x + 7 IJ
xG
H x xK

e3n

x0

2x2 + 3

b3x + 7g

lim

x 2 = x and

Note that highest power of x in Nr =


highest power of x in Dr = x.

lim

+1

1
x

Solution: Let us put n =

b3x + 7g

OP
Q

4n + 3

3 2
3

ex

+1

185

Practical Methods of Finding the Limits

x2
= lim

x3

F x 3I
GH x JK
F x + 1I
GH x JK
2

=
3

x 31+

FG1 3 IJ
H xK
F 1 IJ
lim G 1 +
H xK
lim

Problems on irrational functions when x


Exercise 4.8
Find the following limits:

1+0

e 1 + x 1 xj
lim F x + 8 x 7 x
H

1. lim
2.

4. lim

2x2 + 1
x4

= lim

2+

x2

1
2+ 2
x

bg

lim 4

x2 + 1

x 2 + 16

x2 + 9

2+

1
x

6. lim

7. lim

lim

5. lim

FH

x+c

x2 + 4x
x

FH

b g
(3)

Find

x2 + 4

4x

Solution: xlim

IK

1 + x2 1

4x
2x + 1

+ 2x + 5

3. lim

= lim

Answers

1
1
=1

=2 2

10
3

IJ
K

2+0

x3

1
x2

x2
1

2. xlim

= lim

FG
H

lim 2 +

x 1
x

FG 3IJ
H 7K
FG c IJ
H 2K

x (P.U. 66)

IK

x 2 4 x (L.N. 86)

x3 + 4x

(4)

IK

x 3 4 x (M.U. 86) (4)

FG 1 IJ
H 2K
F I
Problems based on summation of series G formJ
H K
8. lim

FH x

x2 + x

IK (I.I.T 75)

Working rule:
1. Use the formulas for the sum of n-natural number,
square of n-natural numbers or cube of n-natural

186

How to Learn Calculus of One Variable

FG1 + 1 IJ FG 2 + 1 IJ
H nK H nK = 2 = 1
= lim

numbers for which the following formulas are very


fruitful.
(i) 1 + 2 + 3 + + n = sum of n-natural numbers

n
n+1
2
(ii) 12 + 23 + 32 + + n2 = sum of square of n-natural

gb

g.

n n + 1 2n + 1

numbers =

6
(iii) 13 + 23 + 33 + + n3 = sum of cube of n-natural

L n an + 1f OP
numbers = M
N 2 Q

22

n3

F1
GH n

Solution: lim

32

+ ... +

n3
+

22
n

32
n

n2
n3

e j
b1 r g

= lim

n
n
n+1
2

6n 2

2
2
n
= lim
= lim
n
n n + 1
1
1+
n

FG
H

LM n

M
3. Show that lim M
MM n
N

n =1
4

n3 =

n =1

n2
n3

I
JK

n =1
4

n2 n + 1
4
=
n4

LM n OP
P
M
lim M
MM n PPP
MN PQ
R|n bn + 1g
lim S
|T 4
=

n =1
4

lim n 4
n

IJ
K

0
=0
1+0

OP
PP = 1
PP 4
Q

Solution: We know that

+ ... +

n
1 + 2 + 3 + ... + n

I
JK

F 1 + 2 + 3 + ... + n I
= lim G
JK
n
H
F n bn + 1g b2n + 1g I
= lim G
JK
H 6n
F 1I F 1I
n G1 + J G 2 + J n
H nK H n K
2

Examples worked out:


Evaluate
2

Solution: lim

a 1 rn

n
1 + 2 + 3 + ... + n

1 1
,
, all 0 as n
n n2

F1
GH n

= lim

Refresh your memory: Method of finding the limit of


f (n) as n .
We divide the numerator and denominator by the
highest power of n-occurring in f (n) an then use the

1. lim

2. lim

(iv) a r n 1 = a + ar + ar2 + + arn 1 =

idea that

U|
V|
W

n2 n + 1
4

(i)

Practical Methods of Finding the Limits

R| n bn + 1g U|
S| 4 n V|
W
T
R| 1 F 1 I U|
= lim S G 1 + J V
|T 4 H n K |W
1
F 1I
=
lim G1 + J
H nK
4
2

= lim

xa

1
.
4

Problems based on the limits of f (n) as n


Exercise 4.9
Find the limits of the following functions:
Answers

FG 1 IJ
H 4K
FG 1 IJ
H 2K

n3

1. lim

1 + 2 + 3 + ... + n

2. lim

n2

gb

gb

n n1 n2 n3
n4 n5 n6 n7

3. lim

4. nlim

13 + 2 3 + 33 + ... + n 3
n4 + 1

gb

gb

gb

n 4 + 5n 2 + 7n 3
n n 2 ( n 2 + 2n 7)
n
6. lim
n 1 + 2 + 3 + ... + n
5. lim

7. lim

6n5 + n 4 7n 3 + 5
n 5 + 7n 3 + 6

Limits of trigonometric functions as x a


Form:

0
at x = a
or
0

xa

bg
t bxg
t1 x
2

Where f (x) = a trigonometric function whose


Numerator = Nr = t1 (x) = a trigonometric function or
trigonometric expression and denominator = Dr = t2
(x) = a trigonometric function or trigonometric
expression.
Moreover, x = the angle of trigonometric function
which tends to a finite number.

bg

Type 1: To find lim f x = lim

187

(1)

FG 1 IJ
H 4K
(1)
(0)

(6)

Working rule: When numerator and denominator


both are trigonometric functions or trigonometric
expressions which can be expressed in terms of sin
and cos and cancellation of common factor from
Nr and Dr is possible, we adopt the following
procedure.
1. Express all trigonometrical terms into sin and
cos and cancel the common factor from numerator
and denominator.
2. Put x = a = given limit of the independent variable
in the expression free from common factor (i.e., a factor
which makes f (a) meaningless) which gives us the
required limit of the given function f (x) as x a ,


, , , , , or 1, etc., for example.
2 6 4 3
Facts to know:
1. We may face the circumstances where changing
given function in terms of sin x and cos x to remove
the common factor does not provide us a common
factor which means further modification is required
which is done by using formulas of trigonometrical
ratios of submultiple angle. Thus firstly changing of
t-ratios in terms of sin x and cos x and secondly using
the formulas of t-ratios of submultiple angles, we are
able to find out the common factor which is cancelled
from numerator and denominator.
2. After cancellation of common factor from numerator
and denominator, we get the determinate value of the
simplified function at a given value x = a = limit of the
independent variable.
3. Modification of the given function is not stopped
unless we get a determinate value of the simplified
function which provides a finite value = required limit
of the given function as x a .
where a =

188

How to Learn Calculus of One Variable

4. Various mathematical manipulations can be


adopted to remove the common factor which is not
apparent directly in the given function.
Remember: When the methods discussed above fail
to give the limit of a function, a general method of
evaluation known as method of substitution or hmethod or differential method is adopted. All
questions solved by various methods can be solved
with the help of this method too.
Method of substitution:
1. Put x = a h (when x a ) where h 0
through +ve values.

2. Find lim f a h when y = f (x) = algebraic or


h0

trigonometric or mixed transcendental functions or


any type of function.
N.B.: 1. We always put x = a h so that when
x a , h 0 through +ve values [but for simplicity
of calculation we put x = a + h (or x = a h)] because
the limit of f (x) is said to exist at x = a if right hand

bg

limit (or, right limit) lim f x and left hand limit


xa
x > a

bg

f x exist and are equal, e.g.:


(or, left limit) xlim
a
x < a

FG hIJ
H 2 K = lim 1 cos h
= lim
F I
b sin hg
cos G hJ
H2 K
F hI
h
tan G J
2 sin
H 2 K FG h IJ
2
= lim
= lim
h
h
FG h IJ H 2 K
2 sin cos
2
2
H 2K
1 sin

h0

h0

bg

f2

of the independent variable x = a in the simplified

bg

form of the function f x =

h0

(ii) Evaluate lim (sec x tan x)


x

Solution: Put x =

h , then for h > 0, we have


2

lim sec x tan x = lim

FG 1 sin x IJ
H cos x K

whose limit is

1 + sin 2 x
1 cos4 x

Solution:

x0

f2 x

Problems based on type 1


Examples worked out:
Evaluate:

Solution: Put x = 0 h , then for h > 0, we have

g
g

bg
bg

f1 x

required as x a , when f1 (a) = f2 (a) = 0.

b
b

b g as x a but we always put the limit


b xg

f1 x

f x =

1. lim

lim 0 h + 2
x + 2 h0
=
=2
lim 0 h + 1
x +1

h0

= 1 0 = 0.
2. We never put x = a while finding the limit of

x+2
(i) Evaluate lim
x0 x + 1

lim

h0

FG 1 + sin 2 x IJ for x 0 (i.e., given


4 0
H 1 cos4 x K

function does not assume meaningless form as


x

)
4

FG 2IJ
F 1 + sin 2 x IJ =
H4 K
Hence, lim G
H 1 cos4 x K 1 cos FG 4IJ
H4 K
F I
1 + sin G J
H 2K = 1 + 1 = 2 = 1
=
1 cos
1 b 1g 2
1 + sin

Practical Methods of Finding the Limits

2. lim

x
2

tan x + cot x
tan x cot x

Solution:

FG 1 sin x IJ tan x
H cos2 x K
L b1 sin xg tan xOP
Solution: M
MN cos 2 x
PQ
3. lim

FG tan x + cot x IJ
H tan x cot x K

x
2

=
x 2

form is

meaning-less

FG
IJ
FG
IJ
H
K
H
K
FG sin x + cos x 2 sin x cos x IJ sin x
H 2
2
2
2K
=
FG cos x + sin x IJ FG cos x sin x IJ cos 2 x
H 2 2K H 2 2K

sin x + cos x
sin x cos x
2

sin 2 x cos 2 x
sin x cos x
2

esin x cos xj
2

Thus, we get,

1
cos2 x

tan x + cot x
1
for
=
tan x cot x
cos 2 x

n
x
(i)
4
Now, on taking the limits on both sides of (1) as

; n z
2
Thus, we get,

or n +

F x xI
FG 1 sin x IJ tan x = GH cos 2 sin 2 JK sin x
H cos2 x K
FG cos x + sin x IJ cos 2 x
H 2 2K

, we get
2

lim (given function) = lim (simplified form


x

of the given function)

F tan x + cot x IJ
lim G
H tan x cot x K
F 1 IJ = 1 = 1 = 1
= lim G
H cos 2 x K cos 1
x

FG cos x sin x IJ sin x


H 2 2K
=
FG cos x + sin x IJ FG cos x sin x IJ cos 2 x
H 2 2K H 2 2K
FG cos x sin x IJ sin x
H 2 2K
n
=
FG cos x + sin x IJ cos 2 x for x 2 + 4
H 2 2K

esin x + cos xj bsin x cos xg


=
bsin x cos xg esin x cos xj

Now, 1 sin x tan x = 1 sin x sin x


cos2 x
cos2 x
cos x
2

=0
x

form is meaningless.

sin x
cos x
+
tan x + cot x
cos x
sin x
n
Now,
; n z
=
, x
sin
x
cos
x
tan x cot x
4

cos x
sin x

189

which does not give us meaningless form at

This is required simplified form


(i)
2
Now, on taking the limits on both sides of (1) as

x , we get lim (given function) = lim


x 2
x 2
2
(simplified form of the given function)
x=

190

How to Learn Calculus of One Variable

FG 1 sin x IJ tan x
H cos2 x K
FG cos x sin x IJ sin x
H 2 2K
= lim
FG cos x sin x IJ cos 2 x
H 2 2K
FG cos sin IJ sin
H 2 4K 2
=
F I
cos G cos + sin J
H 4 4K
F 1 sin x IJ tan x
lim G
H cos2 x K
FG 1 1 IJ 1
H 2 2K = 0
=
b 1g FGH 12 + 12 IJK
F cosec x 1I
4. lim G
H cot x JK
F cosec x 1I = 0 form
Solution: G
0
H cot x JK

Now, taking the limits on both sides of (i) as

lim
x

cosec x 1
2

cot x

1
1
1
=
=

1
+
1
2
cosec + 1
2

Now,

1 sin 3 x
1 sin 2 x

0
form
0

b1 sin xg e1 + sin x + sin xj


b1 sin xgb1 + sin x g
2

1 + sin x + sin 2 x

for x 2n + 1 , n z
1 + sin x
2

Now, on taking the limits on both sides of (i), we


get
lim (given function) = lim (simplified form

of the given function)

lim

cosec x 1
2

F 1 sin x I
GH 1 sin x JK
3

F 1 + sin x + sin x I
GH 1 + sin x JK
F 1 + sin + F sin I I
G 2 JK J
G
2 H
=G
GG 1 + sin JJJ
2
H
K
= lim

1
n
=
; n z
for x
cosec x + 1
2

this is the required simplified form

cosec x 1

which does not give us a meaningless form at

bcosec x 1g
bcosec x 1gbcosec x + 1g
b

F 1 sin x I
GH 1 sin x JK
F 1 sin x I
Solution: G
H 1 - sin x JK

x
2

F cosec x 1I = lim F 1 I
GH cosec x + 1JK
GH cot x JK

5. xlim

Now,

lim

, we get lim (given function) = lim


2
x 2
x 2
(simplified form of the given function).
x

2
(i)

191

Practical Methods of Finding the Limits

1+1+1 3
=
1+1
2

FG sin 2 IJ
H cos K
F I
Solution: G sin 2 J
H cos K
6. lim

Thus,

0
form
0

for

n +
(i)
2
Now, on taking the limits on both sides of (i), we
get
lim (given function) = lim (simplified form

of the given function)

lim

FG sin 2 IJ =
H cos K

lim 2 sin

F 1 + cos x I
GH tan x JK
F 1 + cos x I
Solution: G
H tan x JK
x

tan x

=
x

0
0

1 + cos x
sec x 1
2

for

F 1 + cos x I = lim F cos x I


GH tan x JK
GH 1 cos x JK
b 1g = 1
cos
=
=
1 cos 1 b 1g 2
F sin x cos x IJ
8. lim G
H cos2 x K
F sin x cos x IJ = 0
Solution: G
0
H cos 2 x K
2

i.e.,

lim

x
2

b1 + cos xg ;
F 1 1I
GH cos x JK

b1 + cos xg = b1 + cos xg cos x


=
e1 cos xj e1 cos xj
2

n
sin x cos x
sin x cos x
+
=
; x
2
2
cos 2 x
2
4
cos x sin x

bsin x cos xg
1
bsin x cos xgbsin x + cos xg = bsin x + cos xg

Thus, we get,

sin x cos x
1
=
(i)
cos 2 x
sin x + cos x

Now, on taking the limits on both sides of (i) as

cos2 x

cos2 x
1 cos x

of the given function)

n
2

tan x

lim (given function) = lim (simplified form

Now,

1 + cos x

1 + cos x

Now,

get

n
, n z
(i)
2
Now on taking the limits on both sides of (i) as
x , we get

= 2 sin = 2.1 = 2
2

7. lim

we

sin 2 2 sin cos


=
= 2 sin
cos
cos

Now,

b1 + cos x g cos x = cos x


1
b cos x gb1 + cos xg 1 cos x
2

, we get
4
lim (given function) = lim (simplified form

of the given function)

192

How to Learn Calculus of One Variable

i.e.,

lim

F sin x cos x I = lim F 1 I


GH cos 2 x JK
GH sin x + cos x JK
x

1
1
=

1
1
+
sin + cos
4
4
2
2
1

=
x

0
0

1 sin x
=
Now,
cos x

g = 2 cos x
= 2 cos x

(i)
Now, on taking the limits on both sides of (i) as

, we get
4
lim (given function) = lim (simplified form

of the given function)


i.e.,

LM sin 2 x b1 + cos 2 xg OP
MN cos x sin x PQ

lim b 2 cos x g = 2 cos


4

lim

x
x
x
x
+ cos2 2 sin cos
2
2
2
2
2 x
2 x
cos sin
2
2

FG cos x sin x IJ
H 2 2K
=
FG cos x sin x IJ
H 2
2K
FG cos x sin x IJ FG cos x sin x IJ
H 2 2K H 2 2K
=
FG cos x sin x IJ FG cos x + sin x IJ
H 2 2K H 2 2K
FG cos x sin x IJ
H 2 2K
=
FG cos x + sin x IJ
H 2 2K
2

cos x sin x

sin 2

x=

bcos x sin xg
sin 2 x b1 + cos 2 x g
Thus, we get,
x

x=

2 sin x cos x 2 cos x


n
, n z
; x
cos x sin x
4

F 1 sin x I = F 1 sin x I x n +
GH cos x cos x JK GH cos x JK ;
2
F 1 sin x IJ = 0
bsec x tan x g
=G
0
H cos x K

2 cos x cos x sin x

= 2

cos x sin x

Solution: (sec x tan x)

10. lim (sec x tan x)

2
1
=
=
2
2

FG 2 IJ
H 2K
L sin 2 x b1 + cos 2 xg OP
9. lim M
MN cos x sin x PQ
L sin 2 x b1 + cos 2 xg OP
Solution: M
MN cos x sin x PQ
sin 2 x b1 + cos 2 x g
Now,
=

= 2

Thus, we get, (sec x tan x) =

1 sin x
cos x

x
x
sin
2
2
=
x
x
(i)
cos + sin
2
2
Now, on taking the limits on both sides of (i) as
cos

, we get
2

Practical Methods of Finding the Limits

lim (given function) = lim (simplified form

of the given function), i.e. lim (sec x tan x)


x

F cos x sin x I cos sin


G 2 2 JJ = 4 4
lim G
GH cos 2x + sin 2x JK cos 4 + sin 4

6.

cos2

bcot x cos xg

1
1

2
2
=
=
1
1
+
2
2

0
2
2

FG IJ
H K

7. lim
x

=0

at the given value x =

cos3 x

1 sin 3 x
cos2 x

8. lim
x

Notes: We observe from the just above solution that


(i) Firstly sec x and tan x are changed into sin x and
cos x to remove the common factor but on changing
sec x and tan x in terms of sin x and cos x, no common
factor is cancelled which means further modification
is required.
(ii) Secondly, using formulas of t-ratios of submultiple
angles, we are able to find out the common factor and
after cancellation of common factor, we get a finite
value 0 for the simplified form (of the given function)

. (i.e. at the limit of the


2

independent variable x).

cos2 x
1 sin 2 x

9. lim
x

10. lim
x

F sin x cos x I
lim G
H sin x cos x JK
F tan x cot x I
lim G
H sec x cosec x JK
2

1.

2.

F 2 - cosec x I
GH 1 cot x JK
F 2 cos x sin 2 x I
lim G
H cos 2 x JK

(3)

12. lim

2 sec 2 x
1 tan x

(2)

1 + cos3 x
sin 2 x

FG 3IJ
H 2K

13. lim

15. lim

e 2j

16. lim

(2)

4.

Answers

3. lim

cot 2 x

F 3 sin x cos x 2 I
GH 1 2 cos x JK
2

e2 2 j

(1)

FG 1 IJ
H 2K

cosec x 1

1 = cos3 x
1 + cos x

14. lim

Evaluate

(1)

11. lim

Problems based on type 1


Exercise 4.10

F 1I
GH 2 2 JK
FG 1 IJ
H 2K
FG 1 IJ
H 2K
FG 3IJ
H 2K

F sin x cos x I
GH tan x cot x JK
b1 sin g
lim

5. lim
x

193

x1

1 + cos x
tan x
2

FG cos x cos IJ
H cot x cot K

(2)

FG 1 IJ
H 2K
esin j
3

bg

Type 2: To find lim f x where f (x) = a


xa
trigonometrical function (or expression) mixed with
an algebraic function in any way (generally algebraic
function appears as addend, subtrahend, minuend,
multiplicand or divisor of trigonometric function or
expression) i.e.; to find

194

How to Learn Calculus of One Variable

bg

(i) lim f x = lim


xa

xa

bg

f x = lim
(ii) xlim
xa
a

bg

bg bg
bg bg
a b xg t b xg
t b xg

3. A trigonometric function is provided which does


not contain a common factor.

a1 x t1 x
a2 x t 2 x
1

Facts to Know:
1. If there exists a factor of t-ratios of angle as
sin , cos , tan or cot , etc, we are required to
write it as

t1 x
or xlim
a a x t x
1
2

bg bg
bg

bg
bg

tan
so that standard results of

limits of t-ratios may be used.


2. Standard results of limits of t-ratios are following

bg
bg

(i) lim

tan =

t x
f x = lim 1
(iii) xlim
a
xa a x
1

bg bg

a1 x t1 x
or xlim
a

bg

x0

a x
f x = lim 1
(iv) xlim
a
xa t x
1

Where a1 (x) and a2 (x) = algebraic functions (or,


expression)
t1 (x) and t2 (x) = trigonometric functions (or,
expression)
a = a finite value


, , , , or 1 for instance
2 6 4 3
We adopt the following working rule:
=

h0

since

sin m x
=m
x

(ii) lim

x0

tan m x
=m
x

(iii) lim cos m x = 1


x0

(iv) xlim
0

sin x
=1
x

(v) 1 = xlim
0

Working rule:
1. Put x = a + h (or, a h) (where h 0 ) in the given
function.
2. Modify the given function obtained after
substitution x = a + h or x = a h and remove the
common factor if it occurs.
3. Take the limit of the modified form which is a
function of h as

sin

sin =

bg

lim f x

xa

= lim f a + h (where h 0 ).
h0
Question: When to use method of change of limit,
method of substitution, h-method or differential
method?
Answer: Method of change of limit is used when
1. The given trigonometric function can not be
simplified easily.
2. The factorization of the given trigonometric
function is not possible or difficult.

tan x
x

(vi) lim cos x = 1


x0

Remember:
1. 0 2 0

0
2

In general, 0 m 0

0 where
m

m = any integer
2. Modification of the function obtained after
substitution x = a + h in the given function is done by
simplification using various trigonometrical formulas
and mathematical manipulations so that standard
formulas of limits of trigonometrical function
mentioned above may be applied.
3. f (a + h) = a function obtained by putting the
independent variable = x = a + h in the given function
where h 0 .

Practical Methods of Finding the Limits

4. The method of substitution is sometimes termed


as substitution and modification method since firstly
we substitute x = a h in the given function and
then we modify the function containing a h .

Problems based on type 2


Examples worked out:
Evaluate

1. lim sec x tan x


x 2

g
F 1 sin x IJ
= lim G
H cos x cos x K
F 1 sin x IJ
= lim G
H cos x K
F I
1 sin G + hJ
H2 K
= lim
F I
cos G + hJ
H2 K
x

LM h tan FG + h IJ OP
N 2 H 2 2a K Q
L h F h IJ OP
= lim Msin G cot
N 2 H 2a K Q
OP
LM
MM F h I PP
G sin J
M
H 2 K PP
= lim M
MM tan FG h IJ FG h IJ PP
MM H 2FaKh IH 2a K PP
NM GH 2a JK QP
= lim sin
h0

h0

Solution: lim sec x tan x


x

h0

h0

h0

= lim

h0

2 sin 2

h0

h
2

h
h
2 sin cos
2
2

FG h IJ
H 2 K FG h IJ
h
FG IJ H 2 K
H 2K

tan
= lim

h0

=10=0
2. lim sin
xa

FG x a IJ tan FG x IJ
H 2 K H 2a K

Solution: Putting x = a + h where h 0 as x a

lim given function


xa

1 cos h
- sin h

FG h IJ
H 2K
lim
h
FG IJ
H 2K
hI
F
tan G J
H 2a K F I
lim
FG h IJ GH a JK
H 2a K
sin

h0

= lim

195

3. lim
x

1
1 a
a
=
=

FG IJ
H K
F
H

cos x

x
2

I
K

Solution: Putting x =

lim
x

FG
H

cos x

x
2

IJ
K

+ h h 0 as x
2
2

FG + hIJ
H2 K
F
I
G + hJ
K
2 H2
cos

= lim

h0

196

How to Learn Calculus of One Variable

= lim

h0

FG sin h IJ
H h K

= lim

h0

sin h
=1
h

h0

h0

= 2 1 = 2 .
N.B.: Here we observe that Dr = a trigonometric
function while Nr = an algebraic function. Hence, they
can not have any factor in common. This is why we
must make use of method of substitution.
5. lim
x

b1 sin xg
FG xIJ
H2 K

Solution: Putting x =

+ h h 0 as x
2
2

b1 sin xg
FG xIJ
H2 K
LM1 sin FG + hIJ OP
H2 KP
= lim M
MM b hg PP
N
Q
x

h0

6.

Solution: Putting x =

LM 3 cos x + cos 3x OP
MN a2 xf PQ
L 3cos x + 4 cos x 3cos x OP
lim M
MN
PQ
b2 xg

lim
x

+ h h 0 as x
2
2
2

= lim
x

4 cos3 x

b2 x g
F I
4 cos G + hJ
H2 K
LM2 FG + hIJ OP
N H2 K Q
2

= lim

h0

lim

h0

h0

h0

= lim

h0

FG 1 cos h IJ
H h K
2

+ h 2 x = 2h
2

F
I
G
JJ
2h
= lim G
GG cos FGH + hIJK JJ
H 2 K
F h IJ
= 2 lim G
H sin h K

h0

h0

F 2 x IJ
lim G
H cos x K
x

FG 2 x IJ
H cos x K

Solution: Putting x =

N.B.: Here we observe that Nr = a trigonometric


function while Dr = an algebraic function. Hence, they
can not have any factor in common. This is why we
must make use of method of substitution.
4. lim

F hI
F 1 1 + 2 sin h I
2 sin G J
H 2K
G
2J
= lim G
= lim
J
h
h
GH
JK
FG sin h IJ
H 2K
= 2 lim
F hI
4G J
H 2K
L bsin h / 2g OP
2
lim M
=
4
MN h / 2 PQ
L sin h OP
1 M
1
1
2
= M lim
= b1g =
P
h
2 M
2
2
N 2 PQ
L 3 cos x + cos 3x OP
lim M
MN b2 - xg PQ

= 4 lim

h0

sin 3 h
4 h3

Practical Methods of Finding the Limits

L F sin h IJ OP
= M lim G
N H h KQ

lim h

h0

h0

8.

= 1 0 = 0

2 cos x sin x

7. lim
x

4x

Solution: Putting x =

+ h where h 0 as
4

LM F + hI + sin F + hI OP
N H 4 K H 4 KQ
LM4 F + hI OP
N H4 K Q

= lim

sin
=

= lim

h0

LM
N

2 cos cos h sin sin h + sin cos h + cos sin h


4
4
4
4
16 h 2

1
cos h sin h + cos h + sin h
2
16 h 2

2 cos h

= lim

h0

= lim

h0

h0

16 h 2

LM
N

2 2 sin 2
= lim

h0

2
= lim

h0 8

16 h

sin 2

= lim

FG h IJ OP
H 2K Q

FG h IJ
H 2K 1

h
4

2 1 cos h
16 h 2

OP
Q

+h
4
4

sin
=

LM
N

cos h + cos sin h cos cos h sin sin h


4
4
4
4
h

2 cos h +

2 sin h 2 cos h +
2h

OP
Q

2 sin h

2 2 sin h
= simplified form of the given
2h
function.
=

Now, lim (given function = lim (simplified


x

h0

form of the given function)

F
I
sin x cos x J
G
lim G
GH x 4 JJK
x

2 lim

h0

LMb
N

sin h
=
h

Solution: Method 1

= lim

2 1=

g FGH 2x IJK OPQ

1 x tan
9. lim
1
x

2
1
2
1 =
8
4
32

sin x cos x

x
4

FG + hIJ cos FG + hIJ


H4 K H4 K

h0

+ h where h 0 as
4

2 cos

Now, given function =

lim given function


x

F
I
sin x cos x J
G
lim G
GH x 4 JJK

Solution: Putting x =

197

h0

2 sin h
h

198

How to Learn Calculus of One Variable

Putting x = 1 + h h 0 as x 1

g FGH IJK

x
given function = 1 x tan
2

b g 2 b1 + hg
c3 1 x = 1 b1 + hg = 1 1 h = hh
h
L O
= b hg M cot hP = h cot
2
N 2Q
= h tan

F hI
tan G J
H2K
h
2
h
tan
2

x 1

= lim

h0

FG 2 IJ lim
H K

h0

FG h IJ
H 2 K = 2 1= 2
F hI
tan G J
H2K

or, alternatively:
Putting x = 1 h in the given function, we have

b1 xg tan FGH 2x IJK = 1 b1 hg LMNtan 2 b1 hgOPQ


F h IJ
= h tan G
H2 2 K
LM 2 h OP

h
h
F I
2
= h cot G J =
H 2 K tan FG h IJ = MM tan FG h IJ PP
H 2 K MN H 2 K PQ
L
F x IJ OP
lim Mb1 x g tan G
H 2 KQ
N
h1

h0

h 0

N.B.: 1. This example gives us light that we may put


x = a h in the given function while adopting hmethod, the result is same. But when (a x) appears
in the question, we prefer to put x = a h for easiness.
2. The above function = (1 x) tan

FG x IJ
H2K

whose

limit is required can be done by expressing it in sin x


and cos x.
Method 2:

L 2 O
FG IJ MN3 h = 2 hPQ
H K
L
F xIO
lim Mb1 x g tan G J P
H 2 KQ
N
2
=

= lim

F h I
G
JJ = 2 1 = 2
2
lim G 2

GG tan FG h IJ JJ
H H 2 KK

LMb1 xg tan FG x IJ OP
H 2 KQ
N
LM
F xI O
sin G J P
H 2 KP
= lim Mb1 x g
MM
F x IJ PP
cos G
H 2 KQ
N
lim

x 1

x 1

We get,

LMb1 xg tan FG x IJ OP
H 2 KQ
N
b hg sin 2 b1 + hg
= lim

cos b1 + hg
2
b hg cos FGH 2h IJK
= lim
b 1g sin FGH 2h IJK
c3 cos b90 + g = sin h
F hI
h cos G J
H2K
= lim
F hI
sin G J
H2K
lim

x 1

h0

h0

h0

Practical Methods of Finding the Limits

FG h IJ lim h
H2K
F hI
sin G J
H2K
2 F hI
G J

H 2 K
F hI
= lim cos G J lim
H2K
F hI
sin G J
H2K
FG h IJ
H2K
F hI 2
= lim cos G J lim
H2K
F hI
sin G J
H2K
= lim cos
h0

h 0

h0

12.

Exercise 4.11

Answers

(0)

3. lim 1 + cos x
x x 2

FG 1 IJ
H 2K

b g

cos
4. lim

x 1

5. lim

x
2

6. lim
x

FG x IJ
H2K

sec x tan x

x
2

FG xIJ tan x
H2 K

lim

FG
H

FG 1 IJ
H 2K

15.

17. lim

x1

sin cos

FG
H

IJ
K

3 tan x
3x

b1 xg

b1 xg

FG 1 IJ
H 2K
FG 1 IJ
H 4K
e 2j
FG 4 IJ
H 3K
(0)

1 + cos x

sin x
x 1

FG 1 IJ
H 36K

sin x

x1

18. lim
(1)

b xg

x
3

x 1

b6 x g

2 + cos x 1

lim

16. lim

3 cos x sin x

b2 x g

lim

e 3j

IJ
K

3 cos x + cos 3x

x
2

(0)

1 + cos x
2. lim
x
x

13. lim

14.

FG 1IJ
H 8K

1 2 cos x

sin x
3

x
6

Problems based on type 2

1 + cos x
1 sin x

lim

11. lim

2
2
=1 1=

1. lim

b - 2 x g

x
2

(2)

1 sin x

9. lim

10.

(4)

2 x sin x
cos x

x
2

h0

Evaluate

bx g

8. lim

h0

h0

sin 3x 3 sin x

7. xlim

199

F I
GH 2 JK
2

b g

200

How to Learn Calculus of One Variable

cos x + sin

bx 1g

19. lim

x1

F 3 I
GH 8 JK
2

x 2 3x + 2

20. lim

FG 1 IJ
H 2K

x 2 x + sin x 1

x 1

21. lim

FG x IJ
H2K

b x 1g

(2)

Special types of functions:

bg

bg

b g whose limit as x a

xf a af x

1. F x =

xa

is required

bg

bg

bg

f x f y
whose limit as x y is
x y

2. F x =

bg

required or F x =

bg

bg

f x f y
x y

bg

bg

f y f x
y x

whose limit as y x is required.


Remember:
Definition:
1. If lim

xa

bg

b g = L = a fixed value then L is

f x f a
xa

called the differential coefficient of f (x) and it is


denoted as f x .
2. If lim

h0

bg

bg

bg

x f a a f x
xa

xa

bg

b g , = L = a fixed value, then

f x+h f x

h
L is called the differential coefficient of f (x) and it is
denoted as f x .
Note:
1. In the above definition of f a , we denote a
particular value of the independent variable x by a
while in the definition of f x , we denote a particular
value of x by itself x instead of a. Thus, we observe x
has to play two roles at a time, one of which is of the
independent variable and the second is of a particular
value of the independent variable, i.e.; the first role is
of a variable while the other role is of a constant.

bg

bg

bg

(i)

bg

bg bg

x f a a f a a f x f a

L = lim

xa

xa

b g b x ag a lim f b xg f bag
xa
bx ag
= f ba g a f bag
= lim

f a

xa

sin x 2 1

x1

2. L = lim

xa

= value of the function f (x) at x = a a times d.c of


f (x) at x = a, e.g.:
1. lim

xa

x sin a a sin x
= sin a a cos a
xa

bg

Type 1: F x =

bg

xa

required as x a .
Working rule:

bg

x f a af x

whose limit is

1. Put x = a + h x a = h where h 0
(h > 0 or h < 0).
2. If f = sin or cos, use C D formulas to convert it
into product form and if f = tan, cot, sec or cosec, then
we are required to transform tan, cot, sec or cosec
into sin and cos and then use C D formulas to
convert it into product form.
Examples working out:
Evaluate:
1. lim

xa

x sin a a sin x
xa

Solution: Putting x = a + h (x a) = h
(x a) 0 as x a.
Now, xlim
a

x sin a a sin x
xa

= lim

ba + hg sin a a sin ba + hg

= lim

ba + hg sin a a sin ba + hg

h0

h0

a+ha

Practical Methods of Finding the Limits

= lim

h0

LM a msin a sin ba + hgr + h sin a OP


h
MN
PQ
FG
H

FG h IJ
IJ H 2 K FG h IJ + h sin a
K FG h IJ H 2 K
H 2K

2a + h
a 2 cos

2
= lim

h0

sin

using C D formula

F hI
sin G J
H 2K F h I
2a + h I
F
= lim a 2 cos G
H 2 JK FG h IJ GH 2 JK + sin a
H 2K
h0

= a cos a + sin a = sin a a cos a.


Notes: 1. When we put x = a h , then h 0
through positive values which means h > 0 and
h < 0 and when we put x = a + h, then h 0 means
h > 0 or h < 0 (both possibilities remain).
2. In questions, in case a function is given defined
by a single formula:
(i) y = f (x)

bg bg
f b xg
, g b x g 0 and one is required to find
y=
g bxg

(ii) y =

(iii)

1
,f x 0
f x

its limit at a given point, there is no need to calculate


the right hand and left hand limit separately, i.e. it is
sufficient to use the substitution either x = a + h or x
= a h in the given function to obtain a function h
and put h = 0 after simplification.
3. In case a function is defined by a single formula
into its domain, it is turmed as uniform function.
4. When a given function is a non uniform function
or a piecewise function and the question says to
examine the existence of the limit of the function, then
it is a must to calculate the right hand and left hand
limit separately, i.e. it is necessary to use the
substitution x = a + h and x = a h both in the given
function to obtain a function of h and lastly put h = 0
in the function of h after simplification.

bg

Type 2: F x =
as x y or,

b g b g whose limit is required

f x f y
x y

bg

bg

201

bg

bg

bg

f y f x
f x f x
=
yx
x y

F x =

whose limit is required as y x .


Working rule:
First method:
1. If f = sin or cos, we use C D formula to convert
the sum or difference into the product form and if
f = tan, cot, sec or cosec, we are required to transform
tan, cot, sec or cosec into sin and cos and then use
C D formula to convert it into product form.
N.B.: as x a , x a 0
2
Second method:
1. Put x = a + h (x a) = h where h 0 .
2. If f = sin or cos, we use C D formula to convert
the sum or difference into the product form and if
f = tan, cot, sec or cosec, we are required to transform
tan, cot, sec or cosec into sin and cos and then use
C D formula to convert it into product form.
Examples worked out:
Evaluate:
1. lim

xa

sin x sin a
xa

Solution: First method:


sin x sin a = 2 cos

x+a
xa
sin
2
2

sin x sin a
xa
2 cos

lim

FG x + a IJ sin FG x a IJ
H 2 K H 2 K1
2
FG x a IJ
H 2 K

xa

sin x sin a
xa

202

How to Learn Calculus of One Variable

FG x + a IJ sin FG x a IJ
H 2 K H 2 K
= lim
FG x a IJ
H 2 K
F x a IJ
sin G
+
x
a
F IJ lim H 2 K
= lim cos G
H 2 K
FG x a IJ
H 2 K
F a + a IJ 1 FG3 as x a , x a 0IJ
= cos G
H 2 K H
K
2
F 2a I
= cos G J
H2K
cos

xa

xa

xa

= cos a.
Second method:
Putting x = a + h x a = h 0
xa x a0h0
Now,

sin x sin a
xa

FG h IJ
F hI H 2K
= cos G a + J
H 2 K FG h IJ
H 2K
sin

(i)

Now taking the limit on both sides of (i) as h 0 ,


we get

lim

xa

sin x sin a
xa

F hI
= lim cos G a + J lim
H 2K
h0

h0

FG h IJ
H 2K
FG h IJ
H 2K

sin

= cos a 1 = cos a .

sin a + h sin a
h

2 cos

FG h IJ
F h I H 2 K bhg
cos G a + J
H 2 K FG h IJ
H 2K
=
sin

FG a + h + a IJ sin FG a + h a IJ
H 2 K H 2 K
h

F hI F hI
2 cos G a + J sin G J
H 2K H 2K
=
h

F hI
sin G J
H 2 K F hI
hI
F
2 cos G a + J
H 2 K FG h IJ GH 2 JK
H 2K
=
h

2. xlim
a

cos x cos a
xa

Solution: xlim
a

cos x cos a
xa

FG x + a IJ sin FG x a IJ
H 2 K H 2 K
= lim
b x ag
F x a IJ
sin G
H 2 K1
x + aI
F
= lim G 2 sin
lim
J
H
K
2
FG x a IJ 2
H 2 K
1F
xa
I
= 2 sin a 1 G3 as x a ,
0J
K
2H
2
2 sin

xa

xa

xa

Practical Methods of Finding the Limits

b g

1
2 sina
2
= sin a
=

3. xlim
y

tan x tan y
x y

2. xlim
a

b sin ag

3.

xa

esec aj

4.

xa

bsec a tan ag

5.

xa

b cosec a cot ag

6.

xa

e cosec aj

Solution: Putting x = y + h h = (x y)

x y given x y 0 h 0
Now, xlim
y

= lim

h0

tan x tan y
x y

tan y + h tan y
y+h y

LM b g
OP
MN b g
PQ
1 L sin b y + hg cos y cos b y + hg sin y O
= lim
M
PPQ
cos b y + hg cos y
h MN
1 L sin b y + h y g O
= lim
M
P
h MN cos b y + hg cos y PQ
L1
1
1 O
= lim M sin h

P
cos b y + hg cos y PQ
MN h
F sin h IJ lim 1 lim 1
= lim G
H hK
cos b y + hg
cos y
= lim

h0

sin y
1 sin y + h

cos y
h cos y + h

h0

h0

h0

h0

= 1

h0

1
cos2 y

Working rule:
1. Rationalize the Nr or Dr or both whose square
root appears.
2. Put x = a + h where h 0
3. Simplify the function of (a + h) and put h = 0 in the
simplified function of h.
Examples worked out:
Evaluate:
1. xlim

2 + cos x 1

b xg

FG sin x sin a IJ
H xa K

2 + cos x 1

b x g

Or rationalizing the Nr, we get

Exercise 4.12

1. xlim
a

Whenever a square root of a trigonometrical function


appears in the given function whose limit is required
as x a , we adopt the following working rule:

Solution: Given function =

= sec y .

Method of Rationalization

h 0

Evaluate

FG cos x cos a IJ
H xa K
F tan x tan a IJ
lim G
H xa K
F sec x sec a IJ
lim G
H xa K
F cosec x cosec a IJ
lim G
H xa K
F cot x cot a IJ
lim G
H xa K

2 + cos x 1

Answers

bcosag

203

b xg
e 2 + cos x 1je 2 + cos x + 1j
=
b xg e 2 + cos x + 1j
2

204

How to Learn Calculus of One Variable

1 + cos x

jb

2 + cos x + 1 x

Now, letting POA = > 0


As 0 , then P A
PN 0 (zero)
ON OA

Now, we put x = + h , where h 0 as x


then the above expression becomes

1 + cos + h

g j

2 + cos + h + 1 h 2

But length OP = r = radius remains constant.


Now,

2 + cos x 1

Required limit = lim

b x g

a1 cos hf
e a2 cos hf + 1j h
3 a x f = a hf = h
a1 cos hf lim
1
= lim
h
e a2 cos hf + 1j
F hI
2 sin
H 2 K lim
1
= lim
h
e a2 cos hf + 1j
F sin F h I I
G H 2 K JJ 1 1
= 2 lim G
GH 2h JK 4 2 1 + 1
= lim

h0

lim cos = lim

b
ON OA
= lim
=
= 1 (3 OA
h 0 OP OA

= OP = r)

lim tan = lim

h0

p
PN
O
= lim
=
= 0 (zero)
h 0 OP OP

lim sin = lim

h 0

h0

1 cos h

e b2 cos hg + 1j h

h0

p
PN
O
= lim
=
= 0.

0
b
ON
OA

Notes:
1. When < 0 , let = POA =

(i.e;

= )
0 0

As 0 , , then P A
PN 0 (zero)
ON OA

h0

1 1 1
= .
4 2 4
To find the limits of trigonometric functions of an
angle as 0
= 2 1

N
O

We have already derived the lim sin x = 0 and


x0

But length OP = r = radius remains constant.

lim cos x = 1 on pages 142 and 143 but here we are

x0

going to provide the same results with different


methods and some more results on limits.
Derivation: Let us consider a circle OAP and
PN drawn form P to the radius OA.

a f

p
PN
= lim
0 h
0 OP

lim sin = lim sin = lim


0

O
= 0 (zero)
OP

Practical Methods of Finding the Limits

a f

lim cos = lim cos = lim

= lim

b
ON
= lim
h 0 OP

lim

xa

af
af

af

af

f1 x
= 1 f 1 x f 2 x as x a
f2 x

2. As 0 , we have

OA
=1
OA

a f

PN
p
= lim
lim tan = lim tan = lim
0
0
0 b
0 ON
O
= 0 (zero)
OA
2. The limits of trigonometric functions of an angle
as 0 can also be found by noting that (i) sin
x, cos x and tan x are continuous functions at x = 0 (ii)
=

lim x = C (iii) the limit sign of a continuous function

x c

can be referred to the independent variable (or,


argument)

(i) sin (ii) tan (iii) log 1 + x x


sin
, where is measured in

circular measure (or, radian measure)


Proof: Let us consider a circular arc AB of radius r
which subtends a positive acute angle at the
center C.
Now we draw
1. the chord AB
2. the tangent AD at A and extend it until it meets
CB at D. Then AD is perpendicular to the radius CA = r

2. To show that lim

F I
i.e; lim f a x f = f lim x = f acf provided f (x) is a
H K
xc

205

D
B

xc

continuous function of x at x = c.

F I
Hence, lim sin = sin lim = sin 0 = 0
H K
F I
lim cos = cos lim = cos 0 = 1
H K
F I
lim tan = tan lim = tan 0 = 0
H K
0

3. lim cos = lim 1 sin = lim 1 sin


0

We have
1
1 2
A1 = area of CAB = CA CB sin = r sin (i)
2
2

A2 = area of the sector CAB =

lim 1 lim sin = 1 0 = 1

4. When is very small, vertical segment drawn


from one end point of the radius of the circle = arc of
the circle opposite to the central angle.
5. When 0 , vertical segment 0
Remember:

af
af

f1 x
= 1 , then f1 (x) and f2 (x) are called
xa f2 x
equivalent functions as x a which means
1. If lim

A3 = area of the CAD =

tan CA =

1 2
r tan
2

1 2
r
2

(ii)

1
1
AD CA = CA
2
2
(iii)

AD
in ACD which
AC
is a right angled triangle since AD being a tangent is
to the radius OA = r)
Again we have, area of CAB < area of the sector
CAB < area of CAD
(iv)
On putting the values of (i), (ii) and (iii) in (iv), we
get A1 < A2 < A3
( 3 ACD = , tan =

206

How to Learn Calculus of One Variable

1 2
1 2
1 2
r sin < r < r tan
2
2
2

On dividing (v) by

1<

(v)

1 lim

4. lim
0

sin
> lim cos
0

If is negative, let = , , i.e. =

lim

lim

a f
a f

sin
sin
sin
= lim
=
= lim

0
0

a f

sin
= 1 (by previous result)

lim

sin
sin
sin
= lim
= lim
=1
0
0

>0

<0

Remember:
1. When is small, sin = (approximately)

=1
2. lim
0 sin
Proof: lim

1
1
1
= lim
=
= =1
sin
sin

sin 0
1
lim
0

F
H

3. Prove that lim

Proof: lim

I
K

tan
=1

FG
H

tan
sin
1
= lim

0 cos

= lim
tan 0

F
H

1
1
1
=
= =1
tan
tan
1
lim
0

I
K

F
H

I
K

N.B.: Limit of the reciprocal of a function = reciprocal


of its limit provided that the limit of the function is not
equal to zero.
Limits of trigonometric functions as x 0

sin
1

0 0

tan

Proof: lim

sin
= 1, ( 0 < < i.e.; is positive)

lim

IJ
K

sin
1
1
lim
=1 =1

0
cos
1

sin
> cos

lim 1 > lim

FG lim sin = 1 and lim cos = 1IJ


K
H

1 2
r sin , we get
2

1
<
sin cos

1>

FG
H

= lim

F
H

I
K

af

Form 1: To find lim f x = xlim


0

af
af

t1 x
where f (x)
t2 x

= a trigonometric function whose numerator = t1 (x) =


trigonometric function or trigonometric expression.
And denominator = t2 (x) = a trigonometric function
or trigonometric expression.
Working rule: To find the limit of a trigonometric
function (whose both Nr and Dr are trigonometric
functions or trigonometric expressions) as the
independent variable tends to zero, we adopt the
following working rule:
1. We express all trigonometric functions in Nr and
Dr in terms of sin and cos or in terms of product
of sin and cos by using C D formulas of trigonometry or we may use any formula which is required
for simplification and cancel the common factor
(which makes f (0) meaningless) from Nr and Dr.
2. Lastly, we use the results gives below:
sin
=1
(i) lim
0
tan
=1
(ii) lim
0

(iii) lim cos = 1


0

IJ
K

Note:
1. Never forget to write:
sin

(a) sin =

Practical Methods of Finding the Limits

tan
so that we may use the

standard formulas of limits of trigonometrical ratios


mentioned above.

(b) tan =

Problems based on the form 1


Examples worked out:
Evaluate:
1. lim

x0

x0

LM sin 3x 5x 3 OP
N 3x sin 5x 5 Q
F sin 3x IJ FG lim 5x IJ lim F 3I
= G lim
H 3x K H sin 5x K H 5K
x0

x0

sin 3x
tan 4 x

LM
MM
N

OP
PP
Q
L lim F sin 3x I OP
H 3x K P
3 M
= M
4 M
F tan 4 x I
MN lim H 4 x K PPQ

=
5. lim

x 0

x0

x0

3. lim

x0

sin ax
x

x0

x0

sin 5x
x cos 5x

= lim

sin 5x
1
5 lim
x

0
cos 5x
5x

x0

FG
H

tan 5 x
x

= lim

x0

IJ
K

sin m
sin 2 x
=m
= 2 3 lim
x
0
m
2x

L sin ax OP
Solution: lim M
N x Q
L sin ax a OP
= lim M
N ax Q
L sin ax OP = a 1 = a
= a lim M
N ax Q
x0

tan 5 x
x
x 0

x0

3
5

Solution: lim

x 0

Solution: lim

x0

3
5

= 11

sin 3 x
3
3
= lim
tan 4 x
x0 4
4

3 1

4 1
3
=
4
sin 2 x
2. lim
x0
2x

sin 3x
Solution: xlim
0 sin5 x
= lim

sin 3x
tan 4 x

Solution: lim

sin 3x
4. xlim
0 sin5 x

=151
=5
n

6. lim

x0

sin 6 x
x

n
n

Solution: lim

x0

sin 6 x
x

F sin 6 x I a6 xf
H 6x K
= lim
n

x0

= lim

x0

FG sin 6 x IJ
H 6x K

F6 x I
GH x JK
n

lim

x0

207

208

How to Learn Calculus of One Variable

FG
H

= lim

x0

sin 6 x
6x

= 1 6 = 6

IJ
K

FG tan x sin x IJ
H 1 cos x K
F sin x sin x I
F sin x sin x cos x I
G
J
G cos x JJ
cos x
= lim G
= lim G
J

1
cos
x J
GH
GH 1 cos x JK
K

Solution: xlim
0

tan x
7. xlim
0 tan x

x 0

tan x
Solution: xlim
0 tan x

LM sin x cos x OP
N cos x sin x Q
L sin x 1 cos x x x OP
= lim M
sin x x Q
N x cos x
L sin x 1 cos x x OP
= lim M
sin x Q
N x cos x
x0

x0

x0

LM FG
N H

IJ
K

FG
H

LM sin x FG 1 cos x IJ OP = lim sin x


cos x
N cos x H 1 cos x K Q
sin x
F sin x I lim x
lim
x lim
H xK
x
=
= lim

= lim

IJ
K

FG
H

IJ
K

sin x

1
x
lim
lim
lim
lim cos x
x 0 cos x
x 0 sin x
x0
x 0 x

1111 = .

sin x
8. xlim
0 sin x

fOP
Q

x0

x0

lim cos x

1
=1
=
1

tan x sin x
9. xlim
0
1 cos x

x0

F
H

I
K

1 0
= 0 3 lim x = 0
x 0
1

cos 7 x cos 9 x
10. xlim
0 cos 3 x cos 5x
cos 7 x cos 9 x
Solution: xlim
0 cos 3x cos 5 x
2 sin 8 x sin x
2 sin 8x
= lim
2 sin 4 x sin x x 0 2 sin 4 x

F sin 8 x 8 x/ I
G
JJ = 8 = 2
= lim G 8 x
GH sin4 x4 x 4 x/ JK 4
x0

x0

x0

x0

lim cos x

x 0

x0

L sin x x OP
= lim M
N x sin x Q
1 I
O
LM
F
F
sin x I G
sin x J P
= M lim G
MM H x JK GG lim x JJ PPP
K Q
H
N

x0

x 0

= lim

sin x
Solution: xlim
0 sin x

x0

x0

11. xlim
0

sin 7 x sin x
sin 6 x

Solution: xlim
0

sin 7 x sin x
sin 6 x

= lim

2 cos 4 x sin 3x
2 sin 3x cos 3x

= lim

cos 4 x
cos 3x

x0

x0

209

Practical Methods of Finding the Limits

1
1
=1
=

Problems based on the form 1

12.

sin 7 x sin x
sin 6 x

(1)

13.

sin 5x sin x
sin 4 x

(1)

14.

sin + x sin x
cos + x cos x

15.

tan x sin x
1 cos x

16.

1 cos 2
1 cos 5

Exercise 4.13
Find the limits of the following functions as x 0 .
Answers

sec 4 x sec 2 x
1.
sec 3x sec x

2.

F 3I
H 2K
Fa I
GH b JK
2

1 cos ax
1 cos bx

3.

sin 2 x sin 4 x
sin 4 x sin 6 x

Find

4.

cos 2 x cos 8 x
sin 3x

Find

tan x sin x
5.

sin x
sin x tan x

6.

sin x

7. sin x cos

1
x

1 cos x
8.
1 + cos x
1 cos 2 x
9.
cos 2 x cos 8 x
10.

tan x sin x
sin 3x 3 sin x
1 cos x

11.

a f

sin 2 x

F 1I
H 2K
F 1I
H 2K
(0)

a
a

f
f

F 1I
H 15K
F 1I
H 8K
F 1I
H 8K

f
f

bcot g
(0)

F 4I
H 25K
af

Form 2: To find lim f x where f (x) = a trigonox0

metrical expression mixed with an algebraic function


in any way (generally algebraic function appears as
addend, subtrahend, minuend, multiplicand or divisor
of trigonometric function or expression), i.e. To find

af af
af af
t a xf
a a xf t a xf
(ii) lim f a xf = lim
or lim
a axf t axf
t a xf
(iii) lim f a x f = lim t a x f
a a xf
a a xf
(iv) lim f a x f = lim
t a xf
af

(i) lim f x = lim


x0

x 0

a1 x t1 x
a2 x t 2 x

x0

x0

x0

x0

x0

x0

(0)

a
a

x0

where a1 (x) and a2 (x) = algebraic functions


t1 (x) and t2 (x) = trigonometric function
We adopt the following working rule:
Working rule: Modify the given function by using
trigonometric formulas if required so that standard
results of limits of trigonometric functions may be
used.
N.B.: Standard results of limits of trigonometric
functions
1. lim

x0

sin x
=1
x

210

How to Learn Calculus of One Variable

2. lim

x 0

tan x
=1
x

Problems based on the form 2


Examples worked out
Evaluate:

FG cos x + cos 2 x IJ
H sin x K
F cos x + cos 2 x IJ
Solution: lim x G
H sin x K
LF x IJ acos x + cos 2 xfOP
= lim MG
Q
NH sin x K
LM
O
cos x + cos 2 x f P
a
= lim M
MM FH sin x IK PPP
x
N
Q
lim acos x + cos 2 x f
=
F sin x I
lim
H xK
1. lim x
x0

x0

x0

x0

x0

x0

=
2. lim

1+1
=2
1

Solution: lim

2 sin x sin 2 x

x0

= lim

2 sin x 2 sin x cos x

x0

= lim

x0

2 sin x 1 cos x

= lim 2 sin x
x0

2 sin
x

x
2

x0

x0

OP
PP
PQ

x0

= 1 (1)2
=1
3. lim

tan x sin x

x0

Solution: xlim
0

tan x sin x
x

LM tan x a1 cos xf OP
Q
N x
3 tan x a1 cos x f = tan x sin x
LM
F x IO
tan x I G 2 sin 2 J P
F
= lim MG
J
MNH x K GGH x JJK PPQ
= lim

x0

x0

2 sin x sin 2 x

x0

LM
F sin x I
sin
x
F I G 2J
= 4 lim MG
MMH x JK GGH x JJK
2
N
F sin x I
sin
x
F IJ GG lim 2 JJ
= lim G
H xK G x J
H 2 K

F
F IJ GG
= lim G
H K GG
H
LM F sin x I OP
G 2 JJ P 1
= 1 2 M lim G
MN GH 2x JK PQ 4
x0

2 x
sin
tan x
lim 2 2 2
x0
x
x
4
4

x0

= 1 2
=

1
2

1
4

I
JJ
JJ
K

Practical Methods of Finding the Limits

4. xlim
0

tan 2 x sin 2 x
3

x0

x0

tan 2 x sin 2 x

Solution: xlim
0

= lim

= lim
x

= lim

x0

FG tan 2 x IJ 2 lim FG sin x IJ


H 2x K
H xK
x0

= 2 1 2 (1)2
= 4.
5. lim

x0

x0

2 sin

= lim

x0

x0

cos 5 x 1
x

Solution: lim

x0

LM sin x 1 1OP
N x cos x Q
F sin x IJ lim FG 1 IJ lim a1f
= lim G
H x K H cos x K
x0

x0

=111=0

k2
4

8. xlim
0

f
f

cos 5 x 1
x

x 0

cos x cos 3 x
x sin 3x sin x

f
f

cos x cos 3 x
Solution: xlim
0 x sin 3x sin x

F x + 3x I sin F 3x x I
H 2 K H 2 K
= lim
F x + 3x I sin F 3x x I
x 2 cos
H 2 K H 2 K
2 sin

x0

k .
2

tan x x
x

x0

LMa2f F 5 I xOP
MN H 2 K PQ

= lim

x0

OP
xP
PP
Q

tan x x
x
x0

F k xI
H2K

x0

lim

=10=0

Solution: lim

LM sin FG k x IJ OP
H2K
= 2 lim M
MM FG k x IJ PPP
N H2K Q
Fk I
= 2 a1f G J
H4K
6. lim

x0

1 cos k x

x0

Solution: lim

LM sin F 5x I OP
H 2 KP
= lim M
5
F
MM H x IK PP
N 2 Q
7. lim

1 cos k x

x0

tan 2 x 2 sin x
x

5x
2

x0

LM sin 5x a2f
F 5I
2
= lim M
MM F 5x I H 2 K
N H2K

LM tan 2 x b1 cos 2 xg OP
x
MN
PQ

= lim

2 sin

= lim

2 sin 2 x sin x
2 x cos 2 x sin x

= lim

sin 2 x
x cos 2 x

x0

x0

211

212

How to Learn Calculus of One Variable

FG sin 2 x IJ lim FG 1 IJ
H x K H cos 2 x K
F sin 2 x IJ 2 lim FG 1 IJ
= lim G
H 2x K
H cos 2 x K
= lim

x0

LM 1 FG 1 cos x IJ OP
N x H sin x sin x K Q
L 1 F 1 cos x IJ OP
= lim M G
N x H sin x K Q
xO
LM
2 sin P
x
2
= lim M

PP
sin
x
x
MN
Q
= lim

x0

x0

x0

x0

x0

=121=2

tan 2 x x
9. xlim
0 3 x sin x

x0

tan 2 x x
Solution: 3x sin x

F tan 2 x I 2 x x x F tan 2 x 2 1I
H 2x K
H 2x
K
=
=
F sin x I x x F 3 sin x I
3x
H xK
H xK
FG tan 2 x 2 1IJ
H 2x
K for x 0
=
FG 3 sin x IJ
H xK
L tan 2 x x OP
Now, lim M
N 3x sin x Q
LM F tan 2 x 2 1I OP
H 2x
KP
= lim M
MM FH 3 sin x IK PP
x
N
Q
F tan 2 x IJ lim 1
2 G lim
H 2x K
21 1
=
=
=

LM
F sin x I
2 G
x
2J
= lim M
MM sin x 4 GGH x JJK
2
N
2

x0

=1
=

OP
PP
PQ

2
1
4

1
2

Problems based on form 2


Exercise 4.14

x0

x0

x0

x0

sin x
x

FG cosec x cot x IJ
H x K
F cosec x cot x IJ
Solution: lim GH
K
x
10. lim

1.

31

1 cos x
x

F 1I
H 2K

2.

1 cos x
x

3.

cosec x cot x
x

4.

1 cos 2 x
x

(0)

5.

sin x tan x
x

(0)

x0

lim 3 lim

x 0

Find the limits of the following functions as x 0


Answers

x0

x0

6.

cos x sec x
x

(0)

F 1I
H 2K

(1)

Practical Methods of Finding the Limits

Note: When given irrational trigonometric functions

x tan x
7. 1 cos x

(2)

1 cos 2 x
8.

9.

(2)

x 1 + cos m x
sin m x

F 2I
H mK
F 2 I
GH a b JK
F 3I
H 2K

x
10.
cos bx cos ax

11.

cos x cos 2 x
x

sin 2 x cos 2 x cos 3x


12.

13.

8
x

LM
MN

15.

3
2

1 cos
3
2

14.

213

2 x cos x 5 x
2 cos x 2 + x
x

(5)
2

OP F 1 I
PQ H 32 K

(1)

F 1I
H 12 K

Form 3: Limits of irrational trigonometric functions


as x 0 .
To find the limits of irrational trigonometrical
functions when it assumes an indeterminate form

F 0I
H 0K

at x = 0, we adopt the following working rule:


Working rule: Method of rationalization is adopted
to find the limit of irrational trigonometric functions
which means removal of radical sign (
) from
numerator or denominator or both which may be done
by using trigonometrical substitution or multiplying
and dividing by the conjugate of irrational trigonometric expressions.

F 0 I form at x = 0, we find the limit of


H 0K

irrational trigonometric functions by directly putting


x = 0 in the given function, e.g.:
1. xlim
0

IJ
K

FG
H

1 cos x
1 cos x
0
=
=0
= lim
x 0 1 + cos x
1 + cos x
2

x
0 mx 0
m
3. After rationalization, we put x = 0 in the rationalized
form of the given irrational function provided
independent variable x tends to zero.
2. x 0

4. If we have

x
x
x
x
cos + cos cos
2
4
2
4

x + 3 x + 2 x sin x

doe not assume

axf = f axf , we should find l.h.l

and r.h.l and we should remove mod symbol by using


the definition

af

f x
and

af

af
f a x f = f a x f if f a x f < 0
= f x if f x 0

and lastly if l.h.l = r.h.l, we say limit of the given


irrational trigonometric function exists and if l.h.l
r.h.l, we say that limit of irrational trigonometric
function does not exist.
Examples worked out:
Evaluate:
1. lim

x0

x
1 cos x
x

Solution: lim

x0

= lim

x0

= lim

x0

1 1 + 2 sin
x
2 sin

x
2

x
2 sin

x
2

x
2

214

How to Learn Calculus of One Variable

l.h.l = lim

x0
x<0

2 sin

2
2

1 cos x
sin x

2 sin

2
2

1= 2

x0
x<0

x0

e j

Note:

x
does not exist.
1 cos x

af

FG sin x IJ
H sin x K

= 2
2 sin x
sin x

r.h.l = lim

x0
x>0

2 sin x
sin x

= lim

x 0

Hence, l.h.l r.h.l

x 0

= 2 1

x0

x0

= 2 lim

x
2
2

2 sin x
sin x

2 sin x
sin x

e j

1 1 + 2 sin x
sin x

2 sin x
sin x

l.h.l = lim

x0

x
2

F x I = lim 2 F sin x I
2 sin
H 2K
H 2K
F x I
G J
lim G 2 J
GH sin 2x JK

Which lim

2 sin x
=
sin x

= lim

1=

1 cos x
=
sin x

x 0
x<0

restricted only to negative values of x.

Solution: Let y =

x0

r.h.l = lim

x0
x<0

x0

F x I
G J
lim G 2 J =
2
GH sin 2x JK

I
K

af

(ii) l.h.l = lim f x means the variable x in f (x) is

2. lim

x0

x
2

x0

= lim

2 sin

F x 2I
G J
lim G 2 J
GH sin 2x JK

F
H

x0

x0

F I
G x JJ
lim G
GH sin 2x JK

x
2

= lim

= 2 lim

x0

2 1

sin x
sin x

(i) r.h.l = lim f x means the variable x in f (x) is


x0
x>0

restricted only to positive value of x.

Hence, l.h.l r.h.l which lim

x0

does not exist.

1 cos x
sin x

Practical Methods of Finding the Limits

3. lim

Problems based on form 3

2 1 + cos x
2

x0

Exercise 4.15

sin x
2 1 + cos x

Solution: Let y =

Find the limits of the following functions as x 0


Answers

sin x

e
y=

2 1 + cos x
2

sin x

je

2 + 1 + cos x

2 + 1 + cos x

2 1 + cos x

b1 + cos x g e

Thus, we get y =

2 + 1 + cos x

a1+ cos xfe

1.

2.

a1 cos xfa1 + cos xfe 2 +


a1 cos xf
=
a1 cos xfa1 + cos xfe 2 +
=

1 + cos x

j
3.

1 + cos x

lim y = lim

=
=

x0

a1 + cos xf e

a1 + 1f e
2

1
2 + 1+1

1
2 + 1 + cos x

2 + 2
1

22 2
1

2 + 1 + cos x

e 2j

x
1 cos x

F 1I
H 2K

1 1 + tan x
tan x
sin x

(2)

1 + sin x 1
sin x

(1)

x
5.

4.

for x 0

for x 0
(i)
Now, on taking the limits on both sides of (i), as
x 0 , we get
x0

215

2 + cos z 1

b zg

;z=x

F 1I
H 4K

Limits of trigonometric functions as x


To find the limits a function involving a trigonometric
function as the independent variable tends to infinity,
we adopt the following working rule:
Working Rule:

1. Put x =

1
1
or t =
where t 0 when x
t
x

N.B.: 1. The rule of putting x =

1
1
or t =
is
t
x

known as reciprocal substitution.


2. Reciprocal substitution is useful when
independent variable x appears as a factor either in
numerator or denominator or when the angle of
trigonometrical function (or, ratio) is the reciprocal
of x.
Examples worked out:
Evaluate:

4 2
1. lim x sin
x

F 1I
H xK

216

How to Learn Calculus of One Variable

F 1I
H xK

Solution: Let y = x sin


On putting x =

1
1
t = , we have x
t
x

t 0
sin t
1
(i)
sin t =
t
t
Now, on taking the limits on both sides of (i) as
x , we get
y =

lim y = lim

t 0

sin t
= 1 which
t

FI
HK
L F I sin F I OP
2. lim M x cos
N H 4x K H 4x K Q
L F I sin F I OP
Solution: Let y = M x cos
N H 4x K H 4x K Q
LM
F I OP
sin
F I H 4x K P
= Mcos
MM H 4 x K 4 PP
4x
N
Q

y = cos t

FG sin t IJ OP
H t K 4Q
F sin t IJ lim F I
= lim cos t lim G
H t K H 4K

t0

t0

=11

x0


which
=
4 4

t 0

xa

af

N.B.: The result of this theorem remains true of either


or both of the given strict inequalities are replaced
by <.
Many important results of limits can be easily
obtained with the help of above theorem.

1 sin x 1

for x > 0
x
x
x

F 1 I lim FG sin x IJ lim F 1 I


H xK
H xK
H xK
F sin x IJ 0 FG3 lim F 1 I = 0IJ
0 lim G
H x K H H xK K
F sin x IJ = 0
lim G
H xK
1
2. 1 sin F I 1
H xK
F 1 I x for x > 0,
x x sin
H xK
F 1I
and x x sin G J x for x < 0
H xK
F 1I F
I
0 lim G x sin J 0 G3 lim b x g = 0J
K
H xK H
F 1I = 0
lim x sin
H xK
x

Now, on taking the limits on both sides of (i) as


x , we get

LM
N

af

xa

lim h x = L, then lim g x = L.

xa

(i)

lim y = lim cos t

af

Theorem: If f (x) < g (x) < h (x) and lim f x = L and

lim

where t 0 as x ,
4x

LM sin t OP
Nt Q 4

An important fact to know:

we have

F I sin F I OP =
H 4x K H 4x K Q 4

Examples worked out:


1. 1 sin x 1

1
lim x sin
=1
x
x

Now, on putting t =

LM
N

lim x cos

x0

x 0

3. 1 cos x 1

1 1
1
cos x for x > 0
x x
x

x0

Practical Methods of Finding the Limits

F 1 I lim F 1 cos xI lim F 1 I


H xK
Hx K
H xK

lim
x

0 lim

Problems based on finding the limits of trigonometric


functions as n
Exercise 4.16

cos x
0
x

cos x
lim
= 0.
x
x

F 1I
H xK
F 1I
lim x tan
H xK

Answers

1. lim x sin

(1)

Examples:
Evaluate:

2.

1. lim n sin , being measured in radian.


n
n

4. lim

(i)

0
n

FG3 n 1 0 0IJ
H
K
n
n

F I
= which lim n sin G J = .
H nK
n

5. lim

Now, on taking the limits on both sides of (i) as


n , we get

= 1

F sin x I
GH x JK
2

since we require
n
the same angle in denominator, we get

(does not exist)

Now multiplying Nr and Dr by

F sin I
G n JJ
lim y = lim G
GH n JK
F sin I
G n JJ
= lim G
GH n JK

(Find)

3. lim sin x

Solution: Let y = n sin


n

F sin I F sin I
G
nJ = G
y = n G
GG n JJJ
n G JJ
H n K H n K

217

(0)

sin 3 x

(0)

x2

FG tan x IJ
H x K

(does not exist)

7. lim 3x sin

(3)

6. lim

F 1I
H xK
F I
lim 2 x sin
H xK

8.

e2 j
2

cos x
(0)
x
Form 2: When the independent variable or its power
appears in the given function as an addend or
minuend, we adopt the following working rule.

9. lim

Working rule: It consists of following steps.


Step 1: To divide Nr and Dr by the highest power of
x appearing in Nr and Dr.
Step 2: To take the limit as x noting that

a1 a2 a3
,
,
, , etc. all 0 as x , provided
x x2 x3
a1, a2, a3, , etc. all are constants.

218

How to Learn Calculus of One Variable

The product of a bounded quantity and an


infinitesimal is an infinitesimal.

Examples worked out


Evaluate:
1. lim

e.g. Let f x =

x + cos x

Solution: y =

1
=

af

x + sin x
3

1
x

1
x

x + cos x

1+

lim

x + sin x

(i)

1+

sin x

cos x

F 1 + 1 sin x I
G x JJ
lim G x
GH 1 + 1 cos x JK
x
3

sin x = 0

F 1I = 0
H xK
1
(ii) lim x cos F I = 0
H xK
F 1 IJ = 0
(iii) lim a x a f sin G
H x aK
F 1 IJ = 0
(iv) lim a x cf cos G
H x cK
x0

Problems based on the form 2


Exercise 4.17

FG 0 + 0 IJ =
H 1 + 0K

Evaluate:
Answers

Note: Now we state a theorem which has a wide use.

2. lim

af
f a x f g a xf = 0 .

Theorem: If lim f x = 0 and g (x) is bounded,

x c

x c

x0

0 =0

then lim

(i) lim x sin

af af

Remember: The above theorem is also true even if


g (x) is bounded in a deleted neighbourhood of c,
e.g.:

1. lim

=0

xc

F 1I
F sin x IJ
lim G J + lim G
Hx K
Hx K
F cos x IJ
lim a1f + lim G
Hx K

FG 1 IJ = F lim 1 I
H x K H xK

Hence, lim f x g x = lim

cos x

lim y = lim

and let g (x) = sin x which is bounded since


1 sin x 1

Now, on taking the limits on both sides of (i) as


x , we get

sin x

Which is expressed in the following way also.

3. lim

x + sin x
x + cos x
x sin x
2

x + cos x
x sin x
2

9 x + cos x

(1)

(1)

(0)

Practical Methods of Finding the Limits

On limits of a function containing are function


Type 1: When a single inverse circular function of
an independent variable (i.e. t1 x, where t1 stands
for sin1, cos1, tan1, cot1, sec1, cosec1) appears
in a given function whose limit is required as x 0
or x a , then we adopt the following working rule:
Working rule:
1. Put the inverse circular function of an independent
variable which appears in the given function = and
change the inverse circular function into circular
function, i.e.; put t1 (x) = , where t1 = sin1, cos1,
tan1, etc. and write t = x where t = sin, cos, tan,
cot, sec, cosec.
2. Change the limit of independent variable x in terms
of .

af

Note: 1. The general method of finding the limits of


a given function containing the inverse circular
function of an independent variable x consists of
changing the inverse circular function into the circular
function (direct trigonometric function) for which it is
better to substitute for an arc function (or, inverse
trigonometric function) or a relation a number .
2. Trigonometric function, trigonometrical functions
and circular functions are synonymus.
3. After changing inverse circular function into
circular function, we use the rule of trigonometric
function as x a if required, method of substitution
is adopted.
Examples worked out:
Evaluate:
1

sin x
1. lim
x0
x

We put sin

x0

sin

lim y = 1
x0

2. lim

x 1

LM 1 x OP
MN 2 sin x PQ
1

Solution: Let y =

1 x

2 sin

1
We put sin x = x = sin ,

x 1

2
2

1 sin
(i)
2
Now, taking the limits on both sides of (i) as
y =

lim y = lim
x 1

LM1 sin OP
N 2 Q

Again putting =

z z 0 as
2
2

FG zIJ
H2 K
F I
2 G zJ
H2 K

1 sin
lim

z 0

F 2 sin z I
1 cos z O
L
G 2 JJ
= lim M
= lim G
P
N 2z Q GH +2z JK
2

Solution: y =

lim y = lim

sin x
x

x = , x = sin

x0 0

y =
(i)
sin
Now taking the limits on both sides of (i) as 0

219

z0

z0

LM F sin z I
OP
1 G
2 J F zI
= lim M G
MM z GH z JJK H 2 K PPP
N 2
Q
2

z0

=10=0

220

How to Learn Calculus of One Variable

3. lim

x 1

1 x

ecos x j
1

y =

Solution: Let y =

3 cos

sin

= tan 2

x 1 0

F sin I
tan
G 2 JJ
=
2G

GH JK

1 cos

F 2 sin
G
= lim G
GH

I
JJ
JK

x0

LM x FG1 1 x IJ OP
H
KP
4. lim M
MM 1 x esin xj PP
N
Q
F
I
x G1 1 x J
H
K
Solution: Let y =
1 x esin x j

=1

5.

We put sin

(i)

x = x = sin ,
2
2

x 0 0

OP
PP
PQ
F sin I

tan
1
L OP lim F I lim GG 2 JJ
= lim M
H 2K
NQ
GH 2 JK

F sin I F sin I OP
GG 2 JJ lim GG 2 JJ P = 1 1 1= 1
2
GH 2 JK GH 2 JK PQ 2

x0

LM
F sin I
tan 1 G
2J
lim y = lim M
MM 2 GGH JJK
2
N

LM
MM
N

Now taking the limits on both sides of (i) as x 0

LM sin sin OP
2
2 1
= lim M2

MN 2 2 4 PPQ
1
= lim
0
2

1 x 2 = cos =

We put cos 1 x = x = cos , 0

given limit = lim

as

cos in
2
2

1 x

ecos xj

sin 1 cos

1
1
2

1
2

O
LM
1 x P
lim M
MNecos xj PPQ

x 1

Solution: y =

ecos xj
1

We put cos 1 x = x = cos , 0

x 1 0

Practical Methods of Finding the Limits

y =

1 cos

e1

cos
2

lim y = lim

x0

j e1 +

1+

cos

cos

y=

2 sin
1 cos
2
= 2
= 2
1 + cos
1 + cos

cos

= cot =
=
cos (i)
tan
sin sin

Now on taking the limits on both sides of (i) as


x 0 , we get

(i)

lim y = lim

x0

Now, on taking the limits on both sides of (i) as


x 1 , we get

LM 2 sin OP
2
lim y = lim M
cos

1
MN e + j PPQ

= lim

x 1

LM 2 sin OP
2 P lim L
= lim M
MM1 +
MM 4 F I PP
N
N H 2K Q
2

x0

OP
cos PQ

x0

tan

x
x
1

We put tan 1 x = x = tan ,

2
2

x0 0

tan x
x

tan
(i)

Now on taking the limits on both sides of (i) as


x 0 , we get
y =

lim y = lim

x0

tan x
Solution: Let y =
x

tan
=1

Problems based on type 1

We put tan 1 x = x = tan ,


2
2
x0 0

y =
tan

OP
Q

x0

tan

6. lim

FG IJ lim cos = 1 1 = 1
H sin K

Solution: Let y =

2
1
2
1 1
lim y = 1
= 1 =
x 1
4
4
2 4
1+ 1

LM
N

= lim
cos
tan 0 sin

7. lim

=1
tan

Or, alternatively

221

Exercise 4.18
Find the limits of the following functions
Answers

(i)

Now, taking the limits on both sides of (i) as


x 0 , we get

1. lim

x0

sin x
x

(1)

tan x
2. lim
x0
x

(1)

222

How to Learn Calculus of One Variable


1

3. lim

x0

sin x
sin x

1+ x 1 x

4. lim

x0

sin

FH

(1)

IK
xj

F 1I
H 2K

x 1 1 x2
5. lim

x0

1 x 2 sin 1
sin

x0

x
x
2

FG 1 IJ
H 2K

7.

1
2

F 2I
H 3K

8. lim

x0

tan 2 x
sin 3 x

ecos xj
lim
1

9.

10. xlim
0

(2)

1 x

x 1

esin

sin

11. lim

x 1

12. lim

j
1
R
F
I
F 1 IU
x + 2 sin
H 2 sin xK ST3 4 sin H 2 sin xK VW

x 2x

F 1I
H 4K
F 1I
H 4K

ecos xj

x 1

cos

cos

1 + cos

x+1

F
GH

Hint: Put cos 1 x = x = cos , 0

x 1

I
J
2 K
1

as

= cos in 0
2
2

z
1
2 cos + z
+ +z
2

lim

z 0

= lim

z0

fe

F zI
2 sin G J e
H 2K

13.

lim1

+z ]

e xj
1 tan esin x j
x cos sin

FG 1 IJ
H 2K

Type 2: If the given function contains a function of


the type t1 [f (x)], where t1 stands for sin1, cos1,
tan1, cot1, sec1, cosec1 and f (x) = an algebraic
function of x.
Remember: We should remember the following
formulas which gives the idea where to use which
substitution:
2

1. (a) 1 sin = cos (b) 1 cos = sin


2

2. (a) 1 + tan = sec (b) sec 1 = tan


2

3. (a) 1 + cot = cosec (b)


4. (a) 1 + cos = 2 cos

cosec 1 = cot

2
(b) 1 cos = 2 sin
2
2

2 cos +
2

Again putting = + z z 0 as

FG 2 IJ
H K

F I
tan
H K
x cos esin x j
lim
1 cot esin x j

6. lim

y =

(1)

5. (a) 1 2 sin = cos 2 (b) 2 cos 1 = cos 2


2

6. cos 2 = cos sin = 1 2 sin = 2 cos 1


7. sin 2 = 2 sin cos
8. sin 2 =

2 tan
2

1 + tan

Practical Methods of Finding the Limits

9. cos 2 =

1 tan
2

1 + tan

10. tan 2 =

2 tan
2

1 tan
3

11. sin 3 = 3 sin 4 sin

F
I
GH
JK
asin 2 f

1
2 tan
1
sin
2
tan
1 + tan

1
1
sin
tan

2 as 2
tan
2
2

2
tan

12. cos 3 = 4 cos 3 cos


3

13. tan 3 =

3 tan tan

14. (a) tan + =

tan + tan
1 tan tan

tan tan
(b) tan =
1 + tan tan

FG
H

IJ
K

1 + tan

15. (a) tan


+ =
4
1 tan
(b) tan

F
GH

1 3 tan

FG IJ = 1 tan
H 4 K 1 + tan

lim

x0

2. lim

x0

1
2x
1
sin
2
x
1+ x

I = lim 2 = 2
JK
tan
0

1
1
2x
tan
2
x
1 x

Solution: We put x = tan , < <


4
4

x00
1
1
2x
tan
2
x
1 x

F
I
GH
JK
atan 2 f

2. (i) sin1 [sin x] = x for x


2
2
(ii) cos1 [cos x] = x for 0 x

1
2 tan
1
tan
2
tan
1 tan

(iii) tan1 [tan x] = x for < x <


2
2

1
1
tan
tan

Problems based on type 2


Examples worked out:
Evaluate:

1
2 as < 2 <
tan
2
2

2
tan

1. lim

x0

F
GH

1
2x
1
sin
2
x
1+ x

I
JK

Solution: We put x = tan ,


4
4
x00

F
GH

1
2x
1
sin
2
x
1+ x

I
JK

lim

x 0

2
1
1
2x
= lim
tan
2

0
tan
x
1 x

= 2 lim

= 2 1= 2
tan

223

224

How to Learn Calculus of One Variable

3. lim

x0

F
GH

1
1 3x x
tan
2
x
1 3x

I
JK

4. lim

x0

Solution: we put x = tan , < <


6
6

x00

F
GH

1
1 3 x x
tan
2
x
1 3x

I
JK

F
GH
atan 3f

1
1 3 tan
tan
2
tan
1 3 tan

1
1
tan
tan

1
3 as < 3 <
tan
2
2

= 3 lim

Remember:
1. If f (x) is an infinitesimal as x a , then
(i) sin1 [f (x)] ~ f (x) as x a
(ii) tan1 [f (x) ~ f (x) as x a

xa

lim

xa 0

= 3 1 = 3.
tan

F
GH
F1 x
GH 1 + x

1. lim

1
2x
1
sin
2
x
1+ x

2. lim

1
1
cos
x

3. lim

1
2x
1
tan
2
x
1+ x

F
GH

2
2

I
JK
I
JK

I
JK

lim

xa +0

af

f x =0=

i.e. r.h.l of f (x) at x = a is equal to the l.h.l of f (x) at


x = a.
From the definition of an infinitesimal, it follows

af

that if lim f x = b , we may write f (x) = b + f1 (x)


xa

af

where f1 (x) is an infinitesimal (i.e. lim f x = 0 )

Exercise 4.19
Evaluate

af
f ax f .

if lim f x = 0 which means

F 3x x I = lim 3
GH 1 3x JK
tan

Problems based on type 2

x0

(3)

2. The function f (x) is called an infinitesimal as x a

1
1
tan
lim
x0 x

x0

I
JK

Working rule:
1. Replace sin1 [f (x)] by f (x) and tan1 [f (x)] by f (x)
in the given function provided f (x) 0 as x a .
2. Find the limit of the modified form of the given
function (i.e.; the function obtained by substitution
sin1 [f (x)] = f (x) or tan1 [f (x)] in the given function)
as x a , where a = any constant = given limit of
the independent variable x.

3
tan

x0

Type 3: If the given function contains the function


sin1 [f (x)] or tan1 [f (x)], where f (x) = an expression
in x s.t f (x) 0 as x a for a being a constant.

I
JK

F
GH

1
1 3 x x
tan
2
x
1 3x

xa

Answers
(2)

(2)

(2)

Now, we shall state some basic results in the form


of theorems.
Theorem 1: The algebraic sum or difference of two
or more infinitesimals is an infinitesimal function.
Theorem 2: The product of an infinitesimal function
(or simply infinitesimal) and a bounded function is an
infinitesimal. This is most important theorem on
infinitesimal which is widely used to find the limit of
bounded function times a function tending to zero as
the independent variable x a .

Practical Methods of Finding the Limits

Theorem 3: The product of a finite number of


infinitesimal function as x a are also infinitesimals
as x a .
Theorem 4: The quotient of an infinitesimal divided
by a variable quantity tending to a non-zero limit is an
infinitesimal.
N.B.: In particular, the product of a constant quantity
by an infinitesimal is an infinitesimal.

1+ x

= lim

x0

x0

1. lim

tan

x 1

a1 xf

Solution: lim

x 1

= lim

x 1

a1 xf

a1 x f , bx 1g
2x + 1

af a f

x 2x +1

lim

x0

x0

3 f x = 1 x 0 as x 1
2

lim

= lim

x 1

x 2x + 1
2

x 2x + 1

1
= xlim
1

=1

2.

x2

x 2x + 1
tan

FG
H
lim

etan

x0

Solution:

IJ
K
xj

1 + x 1 sin

FG
H
lim

are ~x as x 0 ]

3. lim

x 1

IJ
K
xj

1 + x 1 sin

etan
I
1J x
K [3 sin
1

1+ x

x0

x0

FG
H
= lim

FG
H

FH

FG
H

IK

IJ
K

1+ x +1
1

IJ
K

1+ x +1

1+1
1
2
tan

ax 1f

1 sin

Solution: lim

x 1

x 1

x
2

tan

ax 1f

1 sin

a x 1f
1 sin

x
2

x
2

We put x = 1 + h, x 1 h 0
Hence, lim

x 1

a x 1f
1 sin

IJ
K

1+ x +1

1 + x2 + 1
x

x
2

IJ
K

1+ x +1

= lim
1
x and tan x

FG
H

1/ + x 2 1/

= xlim
0

IJ FG
KH

1+ x 1

Examples worked out


Evaluate:
1

FG
H
= lim

225

226

How to Learn Calculus of One Variable

a1 + h 1f
F + hI
1 sin
H2 2 K
2

= lim

h 0

= lim

h 0

h 0

= lim

h 0

2 sin

1
2

1
2

= lim

F hI
H2K

h 0

LM 2 sin F h I OP
MM H 2 K PP
MN h PQ
2

x 1

a 1 x as

af e

lim

sin

xa

= lim

a 1 x as x a

x 1 a a 1 x 0 as x a
x sin
xa

x 1a

a 1 x

xa

xa

F hI
sin
H2K
F hI
H2K

R| sin F h I U|
H2K
lim S
h V
|| 2 ||
T
W

je

x 1 a a 1 x

ax af e

x 1 a a 1 x
xa

Now,

x 1a

3f x =

x sin

x 1a

LM 2 sin F h I OP
H 2 KP
MM 4
PP
MM h 4
PQ
N
lim

x >0, a >0

h0

= sin

x sin
xa

Solution: sin

xa

h
1 cos
2

= lim

sin

4. lim

x 1 a

j e
2

x 1 a + a 1 x

x 1 a + a 1 x

a 1 x

a x af e x 1 a + a 1 x j
x a1 a f a a1 x f
=
ax af e x 1 a + a 1 x j
=

x ax a + ax

ax af e

x 1 a + a 1 x

ax af e

x 1 a + a 1 x

ax af

1
x 1 a + a 1 x

Practical Methods of Finding the Limits

x 1 a a 1 x
xa

Hence, lim

xa

= lim

xa

5. lim

x 1 a +

a 1 a +

3. lim

a 1 a
4.

2 a 1 a
tan

x tan
xa

Solution: tan

a x > 1

lim

xa

= lim

xa

tan

FG
H
lim

(1)

x tan
xa

IJ
K
xj

F 1 I
GH 1 + a JK

1 + x 1 sin

etan

x0

F 1I
H 2K

F 2I
H 3K

tan 2 x
sin 3 x

x0

x tan

x tan
xa

x 2x + 1

5. lim
1

a = tan

LM x a OP
N1 + ax Q

FG a f
H

tan

a x 1f

for

sin 2 x + sin
6. lim

x0

xa
xa
0 as x a
3f x =
1 + ax
1 + ax
1

x 1

xa

a 1 x

tan

x 1

=
=

2. lim

IJ
K

j etan xj
2

3x
1

sin 3x
7. lim
x 0
2x
1

8. lim

sin 2 x
7x

9. lim

tan 5x
3x

x 0

FG x a 1 IJ
H 1 + ax x a K

x 0

10. lim

1
= lim
x a 1 + ax

x0

sin
tan

1
1

227

2x
3x

F 2I
H 3K
F 3I
H 2K
F 2I
H 7K
F 5I
H 3K
F 2I
H 3K

On limits of exponential function

1
1
=
=
1 + a a 1 + a2

xa

Problems based on type 3


Exercise 4.20
Find the limits of the following functions:
Answers
1. lim

x0

2 x sin
2 x + tan

1
1

x
x

b a fg a f , where a = 0/any

Evaluation of lim f x

F 1I
H 3K

g x

constant/
One may adopt any one of the two methods to
find the limit of the exponential function (f (x))g (x) as
x a , a being either a constant or .
Method 1: It consists of following steps.
Step 1: Put y = (f (x))g (x)
Step 2: Use log y = log (f (x))g (x) = g (x) log f (x)
Step 3: Write lim log y = lim g (x) log f (x) = b
x a

(say)

x a

228

How to Learn Calculus of One Variable

Step 4: Required limit of the given exponential function

b a fg a f = e

= lim f x
xa

g x

(iv) e

Notes: 1. The limit of logarithm of a function is the


logarithm of the limit of the function as logarithm is a
continuous

function,

b a fg

x
(v) a = 1 +

lim log f x

i.e.,

xa

R
U
= log S lim f a x fV as logarithmic function is a
T
W
continuous function.
xa

2. a = m x = log a m , for a positive real number


m and a positive real number " a 1 ".
3. The method of expansion for evaluating limits is applicable to the functions which can be expanded in series.
4. Method (1) is applicable commonly to find the limit
of an exponential function (f (x))g (x) as x a or
x .
5. One must note that if f (x) is not throughout positive

c b gh

in the neighbourhood of x = a, then lim f x


xa

bg

g x

x
x
x
+

+ ...
1
2
3

N N

=1

x log a
x 2 log 2 a x 3 log 3 a
+
+
+ ...
1
2
3

2. Method (2) is applicable commonly to find the limit


of the exponential function (f (x))g (x) as x any
constant other than (different from) zero.
Examples worked out:
Evaluate the following:

1. lim 1 + x
x0

1
x

Solution: Let y = 1 + x x
...(i)
On taking logarithm on both sides of (i), we get

log y = log 1 + x

F
GH

1
x

1
log 1 + x
x

I
JK

does not exist because in this case the function is not


defined in the neighbourhood of x = a.

1
x
x
x
=
+

+ ...
x
x
2
3
4

Method 2: It consists of following steps.


Step 1: Put x = a + h in y = (f (x))g (x)
Step 2: Use log y = log (f (a + h))g (a + h) = g (a + h) log
(f (a + h))

F x + x x + ...I
= G1
H N2 N3 N4 JK

Step 3: Write lim log y = lim (g (a + h) log f (a +


h0

h0

h)) = b (say)
Step 4: Required limit of the given exponential function

b a fg a f = e

= lim f x
xa

g x

Notes: 1. One can use the following expansion if


required in step (2) in any method mentioned above.

(i) log 1 + x = x

x
x
x
+

+ ...
2
3
4

for

1 < x 1

x
x
x
(ii) log 1 x = x

+ ... for
2
3
4
1 x < 1
2

x
x
x
+
+
+ ...
(iii) e = 1 +
1
2
3
x

N N

...(ii)

On taking limit on both sides of (ii) as x 0 , we


get

F x + x x + ...I =
GH N2 N3 N4 JK
2

lim log y = lim 1

x0

x0

F1 0 + 0 + ...I
GH N2 N3 JK
F I
log lim y = 1 = log e a3 log e = log
H K
2

x0

lim y = e
x0

lim 1 + x
x0

1
x

=e

e=1

Practical Methods of Finding the Limits

F
H

2. lim 1 +
x

1
x

I
K

F
H

I
K
F 1 I F 1 1 + 1 + ...IJ
log y = x log 1 + = x G
H x K H x 2x 2x K
F 1 + 1 + ...IJ
= G1
H 2 x 3x K
F 1 + 1 + ...IJ = 1
lim alog y f = lim G1
H 2 x 3x K
F I
log lim y = 1 = log e
H K

Solution: Letting y = 1 +

1
x

lim y = e
x

F 1I
lim 1 +
H xK

=e

Remark: One should note that

(ii) lim 1 x
x0

1
x

F I
H K
F 1I
lim 1
H xK

(iii) lim 1 + 1
x
x
(iv)

1
e
= e and
=

x 2

a f

+h
2

FG
H

h 0

+h

a f
= cot h log acos hf =
= cos h

cot h

b gIJK

1
log cos h
tan h

F log F1 h + I I
GG GH N2 ...JK JJ
= lim G
GG h + h + 2 h + ...JJJ
H 3 15 K
2

h 0

F h + ... I
G
JJ
2
= lim G
GG h + h + 2 h + ...JJ
H 3 15 K
F3 log a1 xf = x x x ...I
GH
N2 N3 JK
F1 F h I I
= lim G h G 1 +
GH 2 H 3 JK JJK
2

=0

1
; Further we must note that
e

a f

tan x

af

4. lim x
x 1

1
x 1

af

Solution: Putting x = 1+ h in x

af

lim x

x 1

tan x

Solution: Letting y = sin x

x=

h 0

a function in x appearing as an index is always


reciprocal of the function in x within the bracket.
3. lim sin x

x 2

=e
1
x

b g

lim log y = lim

h 0

x0

tan

cot h

(i) lim 1 + x

FG F + hI IJ
H H 2 KK
log y = log acos hf
1

log acos hf
tan h
y = sin

229

1
x 1

= lim 1 + h
h0

1
h

1
x 1

Evaluate the following:

1. lim 1 + x
x0

1
x

= e 3 x 1 h 0

Exercise 4.21
and putting

, we have

230

How to Learn Calculus of One Variable

2. lim 1 + x
x0

a f FG a
H

1
x

F 3x I
H 2K
lim a1 + sin x f
4
x

3. lim 1

e3 a

cot x

x 1

xI
FG
J
H
2 K
lim a1 + cos x f

tan

5. lim 1 cos

x
x

3sec x

x0

1. e 2. e

m
x

2
1
1
1
m
4.
5.
6. e3 7.
8. e
e
e
e

3. e6

Problems reducible to log (f (x))g (x)


Evaluate the following.

log 1 + x
1. lim
x0
x

Solution: Let y =

log 1 + h

1
h

F a I = log a
b a fg = lim G log log
H a1 + hf JK log e
F3 lim a1 + hf = eI
H
K
= log a a3 log e = log e = 1f
F e 1I
3. lim G
H x JK
lim f x
x0

1
h

h0

1
h

h 0

log 1 + x

f
lim y = lim elog a1 + x f j
F
I
= log lim a1 + x f
H
K
F
I
= log e 3 lim a1 + x f = e
H
K
= 1 a3 log e = log e = 1f
F a 1I
lim G
H x JK for a > 0

y = log 1 + x

1
x

1
x

x0

log 1 + h
log a

a
log a
=
=
a f F log1+ah1+1hfI = logh log
1 + hf 1 log a1 + hf
a
GH log a JK
h

log a

(ii)

f x =

Answers:

x0

1
x

x0

1
x

x0

x0

Solution: Putting a = e in the above solution of (2),


we have
lim

x0

Fe
GH

I
JK

1
= log e = 1
x

F 1 m x IJ
Problems put in the form: y = G
H1 m xK

x0

m
x

and

x0

2.

...(i)

=1

1
x

8. lim 1 + m x
x0

x=

x 2

7. lim 1 x

I
JK

On taking logarithm on both sides of (ii) with base


e, we get
log e (ax) = log e (1 + h)
x log ea = log e (1 + h)

x 1

6.

1
x

And ax = 1 + h, where h 0 as x 0

x0

4.

Solution: let f x =

Working rule: To evaluate

F 1 m x IJ
lim G
H1 m xK
1

x0

m
x

, one

may adopt the rule consisting of following steps.

231

Practical Methods of Finding the Limits

m
1
=
m1 m in
x m1 x

Step 1: To put the given index

m
1
=
m2 m in Dr.
Nr and
x m2 x

RS lim a1 m xf UV
W
T

Step 2: To evaluate

and

RS lim a1 m xf UV
W
T
1
m2 x

x0

1
m1 x

x0

x0

1
x

m1 m

(A1) (say)
m2 m

(A2) (say)

FG a1 m xf IJ
H a1 m xfK

3.

1
x

1
x

1
x

x0

RS lim a1 xfc h UVa1f

T
F 1 + 2 x IJ
lim G
H 1 2x K

1x

x0

2.

x 0

1
x

=e

1
x

x0

1
x

F1 + x I c hc h
H 2K
=
F1 x I c hc h
H 2K

1
2

2x 12

R| lim F1 + x I U|
S| H 2 K V|
F
2 + xI
W
T
lim G
H 2 x JK = R| F x I c h U|c
S| lim H1 2 K V|
W
T
2x

x0

e2

F
H

12

4. lim 1 +
x0

=e

5
x
7

I
K

2
x

1
2

x 0

x 0

F 2 + x IJ
lim G
H 2 xK

1
x

x0

=e

2
x

1
x

1x

2
x

1
x

1x

1
x

1
x

x0

F1 + x I
F
2 + xI
G 2J
Solution: 3 G
H 2 x JK = GG 1 x JJ
H 2K

x0

x 0

e
e

F 1 + x IJ
1. lim G
H1 xK
F 1 + x IJ = a1 + xf
Solution: 3 G
H 1 x K a1 xf
a1 + xf
=
a f
{a1 xfc h }
F 1 + x IJ = lim a1 + xf
lim G
a
H1 xK
{a1 x fc h}
lim a1 + x f
=
a f
lim {a1 x fc h }

x0

Evaluate the following:

21x

21x

1
x

21x

x 0

Examples worked out:

1x

1
x

x0

1
2x

1
x

1
2x

1
2x

x 0

h
ha f

1
x

1
x

Step 3: To find the quotient of (A1) and (A2) to obtain


the required limit, lim

F 1+ 2 x IJ = a1+ 2 xf = a1+ 2 xfc


Solution: 3 G
H 1 2 x K a1 2 xf a1 2 xfc
F 1 + 2 x IJ = lim a1 + 2 xfc h
lim G
H 1 2x K
a1 2 xfc ha f
RS lim a1 + 2 xf UV
W
= T
RS lim a1 2 x fc h UVa f
W
T

21

232

How to Learn Calculus of One Variable

F 5 I F 5x I c h c h
Solution: 3 1 + x = 1 +
H 7 K H 7K
R|F 5x I c h c h U|
F 5I
lim 1 + x = lim S 1 +
H 7K
|TH 7 K V|W
c h
U
R
5x I c h |
|
F
= S lim 1 +
|T H 7 K V|W = e
F 3x I
5. lim 1
H 5K
F 3x I = F1 3x I c ha f
Solution: 3 1
H 5K H 5K
R|F 3x I d ib g U|
F 3x I
lim G1 J = lim SG 1 J
V|
H 5K
|TH 5 K
W
a f
R
3x I c h U|
|
F
= S lim 1
V =e
T| H 5 K W|
2
x

7
5x

2
x

10
7

7
5x

x0

7
5x

10
7

10
7

x0

5
x

x0

35x

5
x

35x

5
x

x0

x0

35x

x0

Working rule: To evaluate

x0

2.
3.

Step 1: To put the given index m x =

FG x IJ am m f
Hm K

FG x IJ am m f in Dr.
Hm K
R| F m I e j U|a
J
Step 2: To evaluate S lim G1
|T H x K V|W
in Nr and m x =

x
m1

R| F m I e
and S lim G1
J
|T H x K
2

x
m2

j U|amm f

x0

Answers:
4
1. e2 2. e6 3. e
Problems put in the form:

F1 m I
G x JJ
y=G
GH 1 mx JK

mx

and x

(A1)

V|
W

(A2) (say)

F1 m I
G x JJ
the required limit, lim G
GH 1 mx JK

mx

FG 2 x 1IJ
H 2 x + 1K

F1 1 I
H 2x K
F
2 x 1I
Solution: 3 G
H 2 x + 1JK = F1 + 1 I
H 2x K

2x

4
x

Step 3: To find the quotient of (A1) and (A2) to obtain

x0

mm1

(say)

1. lim
1
x

1
x

F 1 + 3x IJ
lim G
H 1 3x K
lim a1 x f

, one

Examples worked out:


Evaluate the following:

Evaluate the following;


1. lim 1 + 2 x

may adopt the rule consisting of following steps.

Exercise 4.22

mx

10
7

x0

F1 m I
G x JJ
lim G
GH 1 mx JK

F1 1 I a
H 2x K
=
F1 + 1 I a
H 2x K

fc h

2 x 12

2x

fc h
1
2

2x

233

Practical Methods of Finding the Limits

R| lim F1 1 I a
S| H 2 x K
F
2 x 1I
lim G
= T
J
+
2
1
H x K R| F 1 I a
S| lim H1 + 2 x K
T

2 x

12

2. lim

=e

1
2

21 12

FG 2 x + 3IJ
H 2 x + 1K

=e

2x

f U|c h
1
2

V|
W
f U|c h
V|
W
1
2

F1 + 3 I
H 2x K
F
2 x + 3I
=
Solution: G
J
H 2 x + 1 K F1 + 1 I
H 2x K
F 1 + 3 I F1 + 3 I
H 2x K H 2x K
=
F 1 + 1 I F1 + 1 I
H 2x K H 2x K
F1 + 3 I c hc h F1 + 3 I
H 2x K
H 2xK
=
a
f
c
h
F1 + 1 I F1 + 1 I
H 2x K
H 2x K
F 2 x + 3IJ
lim G
H 2 x + 1K
R| lim F1 + 3 I c h U|c h lim F1 + 3 I
S| H 2 x K V|
H 2x K
W
=T
R|S lim F1 + 1 I U|Vc h lim F1 + 1 I
H 2x K
|T H 2 x K |W
x +1

x +1

3
2

1
2

x +1

2x
3

3
2

2x

3
2

e 1

=e

F1 + 4 I
H xK
F
x + 4I
Solution: G
=
J
H x 3 K F1 3 I
H xK
F1 + 4 I c ha f
H xK
=
F1 3 I c ha f
H xK

1
2

R| lim F1 + 4 I c h U|a f
S| H x K V|
F
x + 4I
T
W
lim G
H x 3 JK = R| F 3 I c h U|a f
S| lim H1 x K V|
T
W
4

x
4

3x

e
e

4. lim

=e

FG x + 3 IJ
H x + 2K

F1 + 3 I
H xK
F
x + 3I
Solution: 3 G
=
J
H x + 2 K F1 + 2 I
H xK

F1+ 3 I c ha f
H xK
=
F1 + 2 I c h a f
H xK

R| lim F1 + 3 I c h U|a f
S H x K V|
F
x + 3I |
T
W
lim G
H x + 2 JK = R| F 2 I c h U|a f
S| lim H1 + x K V|
T
W
x
3

x
2

e2

3x 3

x +1

2x

FG x + 4 IJ
H x 3K

x
4

x +1

2x
3

3. lim

e
e

3
2

=e

x
3

x
2

234

How to Learn Calculus of One Variable

F
H

5. lim 1 +
x

a
x

I
K

I = F1 + a I c h a f
K H xK
af
R
aI
a I c h U|
|
F
F
lim 1 +
H x K = S|T lim H1 + x K V|W = e
F 5I
6. lim H1 K
x
F 5 I = F1 5 I c ha f
Solution: 3 1
H xK H xK
a f
R
U
5I
5I c h |
|
F
F
lim 1
H x K = S|T lim H1 x K V|W = e
F x IJ
7. lim G
H1 + xK
F I
F
1
x I
G 1 J
Solution: 3 G
H 1 + x JK = GGH 1 + 1 JJK = F1 + 1 I
x
H xK
F
1
1
x I
lim G
=
= =e
J
H 1 + x K lim F1 + 1 I e
H xK
F
H

Solution: 3 1 +

a
x

x
a

x
a

5.

6.

7.

x5

x5

Exercise 4.23
Evaluate the following:

F 1 I , a R f
H xK
F 1 I , a Rf
lim 1 +
H xK
F I , a Rf
lim 1 +
H xK
x+

2.

3.

8.

Answers:
5

1. e 2. e

3. e

4. e1 5. e 6. e2 7. e1 8. e

Problems reducible to the form:

a 1
e 1
y=
, a > 0 and /
x
x
Remember:

F a 1I = log a , aa > 0f and


GH x JK
F e 1I = log e = 1
(ii) lim G
H x JK
aany positive constant f
which mean lim
x

(i) lim

x0

x0

x0

x
= loge (the same positive constant)

Examples worked out:


Evaluate the following ones.
1. lim

x0

Fe
GH

ax

1
x

I
JK

2x +5

x +5

x +2

1. lim 1 +

F 1I
H xK
F 3x + 2 IJ
lim G
H 3x 1 K
F 2 x + 3IJ
lim G
H 2 x + 1K
F x IJ
lim G
H1 + xK
F 1I
lim 1 +
H xK

4. lim 1

Solution: 3

1 ee j
=

a x

ax

235

Practical Methods of Finding the Limits

Fe
lim G
H

ax

x0

ee j 1
1I
= lim
J
x K
x
a x

x 0

x0

= log e e = a log e e = a 1 = a

2.

1I
J
x K

x0

Solution: 3

Fe
lim G
H

1
x

1
x

ee j

1 x

1 x

x 0

F3
lim G
H

I
JK

Solution: 3

2x

Fa
GH

2x

ea j

2 x

I = lim ea j 1
JK
x
2 x

= log e a = 2 log e a = 2 log a

x 0

mx

Solution: 3

Fa
lim G
H
x0

e j

5 x

= log e 3 = 5 log e 3 = 5 log 3

6. lim

x0

Fe
GH

ax

e
x

Solution: 3

I
JK for a > 0

mx

x
mx

Fe
lim G
H

ax

I
JK

m x

I = lim ea j 1
JK
x
m x

x 0

I = lim e ee 1j
JK
x
a

x 0

e 1
a
=e
x

x0

7. lim

x0

Fa
GH

lim

x0

a
a
x
a
e e 1
e
e e e
=
=
x
x
x

e
x

= e lim
m+ x

Fa
GH

=a

a
x

m+ x

I
JK for a > 0

a
a
a a a
=
=
x
x

Solution:

1 ea j
=

ax

x 0

x 0

x
2

Fa
GH

1
3
1
= lim
x 0
x
x

5x

x0

4. lim

2x

x0

5 x

x0

I
JK

1 e3 j
=

Solution: 3

ee j 1
1I
= lim
J
x K
x
= log e = a1f log e = 1
F a 1I
lim G
H x JK

lim

= m log e a = m log a

5x

x0

3.

F3
GH

5x

5. lim

Fe
lim G
H

= log e a

m+ x

a
x

I = lim a ea 1j = a
JK
x
m

x 0

log e a = a

log a

ea 1j
x

F a 1I
GH x JK
x

lim

x0

236

How to Learn Calculus of One Variable

ea 1j eb 1j
=

Exercise 4.24

Evaluate the following:

F 5 1I
lim G
H x JK
F e 1I
lim G
H x JK
F 7 1I
lim G
H x JK
F 2 1I
lim G
H x JK
F e 1I
lim G
H x JK
F 2 1I
lim G
H x JK

Notes: (i)

x
1

+ a2

x0

a2 a3

I = log F a I , aa , b > 0f
JK
H bK
+ a + ... + a n I
JK
x
... a f , aa , ..., a > 0f
x

b
x

x
n

Examples worked out:

x0

1.

Fa
lim G
H

x0

Ia
JK

b 2
, a,b > 0
x

Fa
lim G
H

Answers:
1. log 5
2. m
3. log 7
4. log 2
5. 3
6. log 2
Problems put in the form:

x0

a b
g x

af

a b
x

x
g x

af

I
JK

+b 2
a 1
b 1
= lim
+ lim
x
x

0
0
x
x
x

I
JK

x0

write

2.

to

F I F
GH JK GH
= log a + log b = log babg .
F a + b + c 3I
lim G
JK , (a, b, c > 0)
x
H
x

x0

Fa b I
y=G
H g a xf JK and x 0 , where a > 0, b > 0.
Fa b I
Working rule: The rule to evaluate lim G
H g a xf JK
says

j e

a 1 b 1
=
+
x
x

x0

x
x
a 1 + b 1
a +b 2
Solution: 3
=
x
x

x0

6.

3x

5.

x
3

x0

4.

x0

Fa
(ii) lim G
H
= log aa

x0

3.

Fa
lim G
H

x 0

mx

2.

x
g x

whose limit as x 0 is the required limit.

1.

F a 1 b 1I F x I
a f GH x x JK GH g axfJK

Solution: 3

je

je

a 1 b 1 c 1
=
+
+
x
x
x

F a + b + c 3I
GH
JK
x
F a 1I + lim F b 1I + lim F c 1I
= lim G
GH x JK
GH x JK
H x JK
x

lim

x0

x0

x
x
x
a + b + c 3 a 1 + b 1 + c 1
=
x
x

x0

x 0

237

Practical Methods of Finding the Limits

a f

= log a + log b + log c = log a b c

3.

Fa
lim G
H

b
x

x0

I
JK

= lim

x0

j eb

1
x

j e

x0

Fa
GH

b
x

4.

6.

x0

x0

a b
Solution: 3
sin x

x0

x0

2x

=
lim

x 0

je

2x

2x

x sin x

I
JK

1 e 1

je

2x

1 e 1
x tan x

1 e 1
x

tan x
x
x
2x

je

= lim

x0

Fe
GH

x tan x

I
JK

2x

x0

Fa
GH

sin x

lim

x0

Fa
GH

I
JK

FG
H

IJ
K

I
JK for a > 0

Solution: 3

F
GH

1
e 1
x
lim
lim
x0
x 0 tan x
x
x

= log e2 1 1
= 2 log e = 2
7. lim

1 e 1

x 0

sin x

x
sin x

sin x

1 sin x

sin x
x

I
JK

F a 1I lim F sin x I
GH sin x JK GH x JK
sin x

= lim

x0

1 a 1
x

sin x
x
x
2x

x 0

x tan x

ee
lim

1 a 1

2x

Fa
GH

x0

x sin x

x0

je

2x

x0

x0

Solution: 3

ee
lim

a b
x
=

sin x
x

F a b I = lim F a b I lim F x I
lim G
H x JK GH x JK GH sin x JK
F a I 1 = log F a I
= log
H bK
H bK
ea 1j ea 1j for a > 0
5. lim
x

I lim F a 1I lim F x I
JK GH x JK GH sin x JK

x 0

ee
Solution: 3

I = lim F a 1I lim F b 1I
GH x JK
GH x JK
JK
F aI
= log a log b = log
H bK
Fa b I
lim G
H sin x JK

lim

2x

= log a2 log a 1
= 2 log a log a = 2 log2 a

x
x
a 1 b 1
a b
=
Solution: 3
x
x

ea
=

Fa
GH

x 0

(i)

Now putting sin x = h so that as x 0 , h 0 ,


we have from (i)
lim

x0

Fa
GH

sin x

I = lim F a 1I 1
JK
GH h JK
h

x0

238

How to Learn Calculus of One Variable

FG
H

FG sin x IJ = 1IJ
H xK K

= log a 1 = log a 3 lim

8. lim

x0

Fe
GH

ax

e
x

Solution: 3

I
JK

bx

ax

e
x

bx

ee
=

e ax 1 e bx 1

x
x

lim

x 0

Fe
GH

ax

x0

I
JK

ax

j e

1 e

bx

9.

x0

lim

x0

F5
GH 4

4 2

I
JK

F
GH

F
GH

I
JK

e = log e = 1

x0

x0

x0

= log a + log b 2 log 2 = log a + log b log 22

x0

F ab I
H4K

x sin x
x

e +e

Solution: 3

I
JK
2 I
JK
x

3
x

x sin x
x

e +e

F 5I log F 5I
H 3 K = H 3K
=
F 4 I log 2
log
H 2K
Fa + b 2 I
10. lim G
JK for a > 0, b > 0.
x
H
x0

x +1

x0

x +1

Fa + b 2 I
GH
JK
x
F a 1I + lim F b 1I 2 lim F 2 1I
= lim G
GH x JK
H x JK GH x JK

11. lim

3 I
J
x K
2 I
JK
x

F5
lim G
I= H
JK
F4
lim G
H
x0

x0

F5
GH
=
F4
GH

log

j e

= log a + log b log 4 = log

1 + bx 1 2 2x 1

I
JK

5 3

lim

a3 log

Solution: 3

j e

a +b 2 2
=
x

x
x
a 1 b 1 2 2 1
=
+

x
x
x

ebx
e ax 1
e bx 1
= lim
lim
x 0
x0
x
x
x

ea

x +1

= loge ea loge eb = a loge e b loge e = a b

F5
lim G
H4

a +b 2
Solution: 3
x

2x

+ 1 2e

ex

x e sin x

ee

I F sin x I
=e
J G J
1K H x K
F x sin x I
lim G
H e + e 2 JK
F x I
= lim e lim G
H e 1JK
= e a1f 1 = 1
x

F
G
He

x sin x
1
e + x 2
e
x

x e sin x

x0

x 0

x0

lim

x0

FG sin x IJ
H xK

Practical Methods of Finding the Limits

Exercise 4.25

14. lim

e 1 x

x0

Evaluate the following ones:


1. lim e

15. lim

x0

x0

x0

3. lim

e
log x

tx

x 0

4. lim

16. lim

x 0

3 1
x

2 1
x

7 3
17. lim
x0
x

e
x

x 0

5. lim

1
x

ax

a b
, a > 0, b > 0
x

2. lim

bx

18. lim

x0

sin x

1
,a>0
sin x

ax

1
sin x

x0

19. lim

sin x

1 sin x

x0

6. lim

sin x

x 0

7. lim

eb

x +h

20. lim

x0

h 0

ex

x 2

tan x

+1

22. lim

h 0

9. lim

x0

10. lim

x0

11. lim

x
e

1
2

sin x
e

3x

x 0

1
x

12. lim

x0

2 1
x
x

e e
13. lim
x0
x

2 1

a1 + xf

1
2

tan x

sin x

x2 x
21. lim
x 0 1 cos x

h
e

x
x

8. lim

,a>0

ex +h ex
h

Answers:
1. 1
2. 0
3. t
4. (a b)
5. a
6. 1
2
x
7. 2 x e
8. Does not exist
9. log a
10. 1
11. 3
12. log 2
13. 2

239

240

How to Learn Calculus of One Variable

1
2

14.

15. log

F aI
H bK

16. log

3
2

17. log

F 7I
H 3K

1
2

1
2
20. Hint: Divide Nr and Dr by x. Answer: 2 log 2
21. 2 log 2
22. ex
Problems on R (ex)
Working rule: The rule we may adopt to evaluate

e j

log e + log e
2 log e + 2 log e

F 3e
GH 4e

2x

19.

lim R e x

log e 2 log e 2

2. lim

18. log a

x0

log e log e 1

is (1) to change R (e x) into the

combination (sum, difference, product and/quotient)


mx
e 1
of (i) xn (ii) emx (iii)
by using the method of
x
rationalization or any mathematical manipulation and
(2) to find the limit of the combination of (i), (ii) and
(iii) respectively.
Notes: (a) one should write

1
mx

+ 2e

2x

2 x

2 x

I
JK

F 3e + 2e I
Solution: lim G
H 4e e JK
F 3e + 2 I
G
e J
= lim G
GH 4e 1 JJK
e
F3 + 2 I
G e JJ = 3 + 0 = 3
= lim G
GH 4 1 JK 4 + 0 4
e
2 x

2x

2 x

2x

2x

2x

2x

2x

4x

4x

N.B.: As x , e

mx

if m > 0.

Exercise 4.26

for emx whenever

e
it occurs in the given rational functions of ex.
(b) R (ex) is the notation for rational function of ex.

Evaluate the following ones:

Examples worked out:


Evaluate the following:

1. lim

1. lim

x0

Fe
GH e

2x

e x
+ e 2 x

Solution: lim

x0

ee

x0

I
JK

2. lim

j e
ee 1j ee
x

2x

2 x

e 1 e 1

x
= lim 2 x x
x0 e
1 e 2 x 1

x
x
x

1 ex 1

ae +be
e +e

ae + be
x

e +e

3. lim

4. lim

ae + be
x

e +e

R|F e
S|GH e
T

+e

I + tan F 1 I U|
JK H x K V|
W

241

Practical Methods of Finding the Limits

bg
lim g a x f = lim cos x = cos 0 = 1
Hence, lim cos asin x f = cos lim asin x f

Answers:
1.
2.
3.
4.

lim f x = lim sin x = sin 0 = 0 and


x0

a+b
2
a
b
2

x0

x0

x0

x0

x 0

= cos 0 = 1
On Limits of function of a function

b a fg

Evaluation of lim g f x , where a = 0, any


xa

constant other than zero and/


Theorem: (Calculus by Burkey)

(i) lim g b f a x fg = g F lim f a x fI = g a L f


H
K
F
I
(ii) lim g b f a x fg = g G lim f a x fJ = g a Lf
H
K
F
I
(iii) lim g b f a x fg = g G lim f a x fJ = g a L f
H
K
xa

xa

x0

x0

x 1

x 1

b a fg

Step 1: Put the inner function = f (x) and find lim

b a fg

x a

Step 2: Put the outer function = g (x) and find lim

x L

(outer function) = lim g x = g (L) which will be the


required limit of the given composition of two
functions, say g (f (x)) as x a .
Examples worked out:
Evaluate the following ones:

a f

1. lim cos sin x


x0

Solution: Let f (x) = sin x and g (x) = cos x

x0

j = sin 1

Solution: Let f (x) = ex and g (x) = sin x

af

lim f x = lim e = e
x 1

af

x 1

and lim1 g x = lim1 sin x = sin


x e

(inner function) = lim f x = L (say)

af

x0

e j

3. lim sin e

procedure to evaluate lim g f x , a being zero,


xa
any constant different from zero and/infinity.

x L

x 1

x0

Working rule: One may adopt the following

xa

I = a1f = 1
K

Hence, lim g f x = lim sin cos x

af

means the functional value of g (x) for x = L and the


limit of g (x) as x tends to L are equal.
3

xa

Solution: Let f (x) = cos3 x and g (x) = sin x

lim f x = lim cos x = lim cos x

xa

F
af
H
and lim g a x f = lim sin x = sin 1

L, then

xa

x L

af

xa

x 0

Note: g is continuous at L lim g (x) = g (L) which

Let lim f x = L exists and g be continuous at


x a

2. lim sin cos x

x e

b a fg

1
e

F 1I
H eK

e j = sin FH 1e IK

Hence, lim g f x = lim sin e


x 1

x 1

F 1I , a 0
H xK
1
Solution: Let f a x f =
and g (x) = sin x
x
F 1I = 1
lim f a x f = lim
H xK a
F 1I
and lim g a x f = lim sin x = sin
H aK
4. lim sin
xa

xa

x a1

xa

x a1

242

How to Learn Calculus of One Variable

b a fg
IJ
K

Hence, lim g f x = lim sin


xa

FG
H

5. lim e
x2

1+ x + 2 x

xa

F 1 I = sin F 1 I
H xK H aK

Method of Expansion

Solution: Let f (x) = 1 + x + 2x2 and g (x) = ex

af
g a x f = lim e

lim f x = lim (1 + x + 2x2) = 1 + 2 + 2 4 = 11


x 2

and lim

x 11

x2

x 11

11

b a fg = lim FGH e

Hence, lim g f x
x2

=e

1+ x + 2 x

x2

IJ = e
K

11

Exercise 4. 27
Evaluate the following ones:

F xI
H 2K
lim sin aa x + b f

1. lim sin

x0

2.

sin (ac + b)

xc

3. lim e

2x

x 0

4. lim e

Answers

cos x

x 0

2
5. lim x , c I
xc

[c]2

Remember: Following expansions are widely used


for evaluating limits by method of expansion.

cos x
7. xlim

e j, a > 0
lim f b x g , when lim f b x g = l
x

9.

xc

xc

10. lim cos x


x0

11. lim e
x0

x = a, or lim x = a) to find the


x a means xlim
a
value of the limit of the given function in the quotient
form (or, in the product form).

xc

Working rule:
Step 1: We write the expanded form of the function
(or, the functions present in the given whose limit is
required) whose expansion is known to us.
Step 2: After expansion in series, we simplify and
cancel the common factor (or, factors) present in the
numerator and denominator of the given quotient
function if any one common factor exists. If there is
no common factor in the expanded form of the given
quotient function (or, functions), we leave the given
quotient function in the expanded form.
Step 3: Lastly, i.e., after expansion and simplification,
we put the limit of the independent variable (since

sin x
6. xlim

8. lim a

Question: Where to use expansion method for


evaluating limits?
Answer: The method of expansion for evaluating
limits of a given function at a given point is applicable
to the function (or, functions) which can be expanded
in series, i.e., if the given function (whose limit is
required to be found out) contains some function (or,
functions) whose expansion in series is known to us,
then firstly we make proper expansion for those
functions which are capable of being expanded.

ac

1. sin x = x

x
x

+ ...
5
7

N3 N N
+

x
x
x
+

+ ...
2. cos x = 1
2
4
6

|l|

3. tan x = x +

a f

2 5
x
+
x + ...
3
15
2

4. log 1 + x = x

5. log 1 x = x

a f

x
x
+
+ ... , 1 < x 1
2
3

x
x

... , 1 x < 1
2
3

Practical Methods of Finding the Limits

x0

x
x
x
=1
+

+ ...
1
2
3

N
a1 + xf = 1 + n x + n anN2 1f x

7. e

8.

N N

N N

+ ... , (1 < x

< 1), where n is a negative integer, a fraction and/any


real number.
x

9. a = e

a x log a f + ...
N2
b1+ xg = 1+ n c x +
2

x log a

= 1 + x log a +

10. For positive integer,

1 1
,
power function x n (i.e.
,..., etc.) and a
x x2
trigonometric, logarithmic or exponential function of
x which can be expanded in a series of power of x.

(b) the integrand contains (a + x)n or 1 x and a


function of x.
(c) all the expansions are valid even if x is replaced
by any other variable or a function of x, e.g.

sin x sin x

... ,
log 1 sin x = sin x
2
3
(1 < sin x < 1)

Examples worked out:


Evaluate the following ones:

a1 + xf

1. lim

x0

3x + 5x

x0

1
n

x0

Solution:

a1 + xf
lim

x0

1
n

F1 + 1 x + terms having higher powers of xI 1


H n
K
= lim
x0

F 1 x + terms having higher powers of x I


G
JJ
= lim G n
x
GH
JK
x0

F 1 + terms having x and its powerI = 1


Hn
K n
F tan x IJ
3. lim G
H xK
F tan x IJ
Solution: lim G
H xK
= lim

x0

3
2

(i) log 1 + sin x = sin x sin x + sin x + ... , (1


2
3
< sin x < 1)
(ii)

x0

n c2 x 2 + ... + n cn x n
N.B.: 1. We put (x) in (1 + x)n to obtain the expansion
of (1 x)n.
2. The method of expansion is also widely used when
(a) the given function is the product of reciprocal of

a1 + xf 1
Solution: lim
x a3 + 5x f
F1 + 5 x + 5 4 x + ...+ x I 1
GH N1
JK
N2
= lim
x b3 + 5x g
x e5 + 10 x + 10 x + ... + x j 5
= lim
=
3
x b3 + 5x g
a1 + xf 1
2. lim
5

x
x
x
+

+ ...
6. e = 1 +
1
2
3
x

243

x0

3
2

x0

Fx + x
G 3
= lim G
GG
H

x0

F xI
= lim G1 +
H 3 JK
2

x0

2 5
+
x + ...
15
x

3
x2

= e3

I
JJ
JJ
K

3
2

244

How to Learn Calculus of One Variable

FG 1 1 IJ
H sin x x K
F 1 1I
F x sin x I
Solution: lim G sin x x J = lim G x sin x J
H
K
H
K
R| x F x x + x ...I U|
GH N3 N5 JK |
|
= lim S
|| x FG x x + ...IJ V||
|T H N3 K |W
R| x x + ... U|
| N5 |V
= lim S N3
|| x x + ... ||
W
T N3
R| x F 1 x + ...I U|
| GH N3 N5 JK |V
= lim S
|| x FG1 x + ...IJ ||
|T H N3 K |W
R| x F 1 x + ...I U|
GH N3 N5 JK |
|
= lim S
=0
|| FG1 x + ...IJ V||
|T H N3 K |W
F 3 sin x sin 3x IJ
5. lim G
H x sin x K
F 3 sin x sin 3x IJ
Solution: lim G
H x sin x K
F x + x ...I FG 3x b3xg + ...IJ
3G x
H N3 N5 JK H N3
K
= lim
F
I
x
x Gx
H N3 + ...JK

N e

x0

x0

x0

x0

6. lim

x0

x0

x0

x0

x0

7.

R|
|| x FH 16 13IK + ...
= lim S
|| x F1 x + x ...I
|T GH N3 N4 JK
F tan x sin x I
lim G
H sin x JK

U|
|| 1
V| = 6
||
W

x0

Solution: lim

x0

F tan x sin x I
GH sin x JK
3

F x + x + 2 x + ...I F x x + x ...I
GH 3 15
JK GH N3 N5 JK
= lim
F x x + x + ...I
GH N3 N5 JK
3

x3
= lim

x0

x0

x0

x 0

1 + log 1 x

fIJ
JK
sin x
F e 1 + log a1 xfI
lim G
JJK
GH
sin x

R|F1+ x + x + x + ...I 1 U| + F x x x ...I


S|GH N2 N3 JK V| GH 2 3 JK
W
= lim T
F x x + x ...I
GH N3 N5 JK

x0

Fe
GGH

Solution:

x0

x0

2
x
3 3 + ...
3
= lim
= 24
3
x0
x
+ ...
3

4. lim

FG 1 + 1 IJ + x FG 2 1 + ...IJ
H 3 6 K H 15 N5 K
F x x + x + ...I
GH N3 N5 JK
5

Practical Methods of Finding the Limits

x
= lim

x0

F 1 + 1 I + x F 2 1 + ...I
H 3 6 K H 15 N5 K
F x + x ...I
x G1
H N3 N5 JK
5

F 1 + 1 I + x F 2 1 I ...
H 3 6 K H 15 N5K
= lim
F1 x + x ...I
GH N3 N5 JK
x0

F1+ x + x + ...I + F1 x + x ...I 2


GH N2 JK GH N2 JK
= lim
2

x0

= lim

F x + x + ...I
GH N2 N4 JK
2

245

x0

F 1 + x + x + ...I
GH N2 N4 N6 JK
F 1 + 1I = 1
=
F 1 + 0 + 0IJ = 2 = 1
H 3 6K 2
=2G
H N2
K N2
Fe e I
F a 1I
8. lim G
10. lim G
for a > 0.
H x JK
H x JK
Fe e I
F a 1I
Solution: lim G
Solution: lim G
H x JK
H x JK
F1+ x + x + x + ...I F1 x + x x + ...I
F e 1I
GH N2 N5 JK GH N2 N3 JK
= lim G
= lim
H x JK
x
F1+ x log a + x alog a f + x alog af + ... 1I
F x + x + ...I
GH
JK
2 Gx +
JK
N2
N3
N
N
3
6
H
= lim
= lim
x
x

= lim 2
x0

x0

x0

x0

x0

x log a

x0

x0

x0

F x + x + ...I = 2
= lim 2 G1 +
H N3 N5 JK
F e + e 2I
9. lim G
H x JK
F e + e 2I
Solution: lim G
H x JK
2

x0

x
2

x0

x0

x 0

x
2

FG
H

= lim log a +
x0

= log a

F
H

11. lim x e x e
x

F
H

x
N blog ag
2

1x

I
K

x ex e
Solution: xlim

1x

I
K

+ ...

IJ
K

246

How to Learn Calculus of One Variable

R|F F 1 I F 1 I F 1 I
I
G
| H x K + H x K + H x K + ...JJ
= lim x SG1 +
JJ
||GG N1 N2 N3
HT
K
F F 1 I F 1I F 1I
IU
GG1 H x K + H x K H x K + ...JJ ||
JJ V|
GG N1 N2 N3
K |W
H
F 1 F 1I F 1I
I
G
J
H
K
H
K
x
x
= lim x 2 G x +
+
+ ... J
GG N1 N3 N5
JJ
H
K
I
1 F
1 F 1 I 1 F 1I
= lim x 2 G1 + G J +
+ ...J = 2
x H N3 H x K N5 H x K
K
Fe 1 xI
12. lim G
H x JK
Fe 1 xI
Solution: lim G
H x JK
F1 + x + x + x + ...I 1 x
GH N1 N2 N3 JK
= lim
2

x0

x0

x
x

= lim

x0

13.

Fe
lim G
H

F 1 + x + ...I
H N2 N3 K =
x

sin x

x0

Solution: xlim
0

sin x
= lim

x0

x0

14. lim

x 1

sin x 1
x

Fe
GH

sin x

I
JK

sin x 1
x

FG sin x IJ FG 1 + sin x + ...IJ = 1 = 1


K N2 2
H x K H N2 N3
log x
x 1

l a fq
a f
R|a x 1f a x 1f + a x 1f ... U|
|
|V
2
3
= lim S

1
a
f
x
||
||
T
W
R| a x 1f + a x 1f ... U|
= lim S1
V|
3
|T 2
W
lim

x 1

log 1 + x 1
log x
= lim
x 1 x 1
x 1
2

x 1

x 1

=10+0=1
Method 2
Let x = 1 + h where h 0 as x 1
lim

x 1

a f

= lim

F
GH

h 0

h
h
+
...
2
3
h

h 1
= lim

h h
+
...
2
3
h

F h h
= lim G 1 +
H 2 3
F log x 1IJ
15. lim G
H xe K
h 0

a f

log 1 + h
log 1 + h
log x
= lim
= lim
x 1 h 0 1 + h 1 h 0
h

I
JK

Solution: Method 1

h 0

N2 = 2

x
2

= lim

x0

F 1 + sin x + ...I
H N2 N3
K

I
JK

I
JK

... = 1

xe

Solution: Let x = e + h, where h 0 as x e

Practical Methods of Finding the Limits

FG log x 1IJ = lim log ae + hf 1


h
H xe K
log ae + h f log e
= lim
a3 log e = 1f
h
F e + hI
log
H e K = lim log F1 + h I
= lim
H eK
h
|RF h I |UV
= lim log S 1 +
|TH e K |W
h
= lim log {a1 + h f } , where h =
e
1
1F
I
= log e = log e = 3 lim a1 + x f = e
H
K
e
e

lim

xe

7.

h 0

h 0

1
h

h 0

1
e

1
h

Evaluate the following ones:

log 1 + x t
t

ee
lim

1 log 1 + x
sin x

5. xlim
0

(0)

x2 42
lim
4.
x 4 log x 3
x

af

1
sin x
7

Answers
(x)

j a

x0

9. lim

log 1 + sin x
x

(1)

(1)

(112)

(1)

sin x

1
6. lim log 1 + 2 sin
0

(1)

F 1I
H 2K

On existence of the limit of a function at a given point

Exercise 4.28

x0

log x
x 1

x0

8. xlim
1

1 cos x
11. xlim
0 x log 1 + x

h 0

3. lim

1
e

1
h

2.

sin x
10. xlim
0 log 1 + x

1
e

h 0

t 0

x e

x0

FG 2 IJ
H eK

2 log x 1

h0

e
h

1. lim

lim

247

(2)

Before one knows how to examine the existence of


the limit of a function y = f (x) defined on its domain at
an indicated point, it is required to be known some
more concepts given below.
1. Adjacent intervals: Two intervals are said to be
adjacent The left end point of one is the same as
the right end point of the other.
Hence, the intervals
(i) x < 0 and x > 0
(ii) 0 < x < 1 and 1 < x < 2
(iii) 1 < x < 2 and 2 < x < 3
(iv) 1 < x < 3 and x > 3, etc.
are examples of adjacent intervals because their left
and right end points are same.
2. Nonadjacent intervals: Two intervals are said to
be nonadjacent The left end point of one is
different from the right end point of the other.
Hence, the intervals
(i) x < 0 and x > 1
(ii) 0 < x < 1 and 2 < x < 3
(iii) 0 < x < 2 and x > 3
(iv) x < 3 and x > 4, etc.
are examples of nonadjacent intervals because their
left and right end points are different (not same)
How to examine the existence of the limit of a function
at a given point
To examine the existence of the limit of a function f at
a given point in its domain, one is required to use the
following rule:

248

How to Learn Calculus of One Variable

Left hand limit (L.H.L or l.h.l) at a given point =


Right hand limit (R.H.L or r.h.l) at a given point
the limit of a given function at a given (indicated in
the question) point exists.
How to examine the nonexistence of the limit of a
function at a given point
To examine the nonexistence of the limit of a function
f at a given point in its domain, one is required to use
the following rule:
Left hand limit at a given point Right hand limit
at a given point The limit of a given function at a
given point does not exist.
Notes: 1. In case, a function f is defined by a single
formula y = f (x) in a -neighbourhood of a point x =
c, there is no need to find out the left hand and right
hand limits separately for the given function f at a
given point c where the given function f may or may
not be defined while evaluating the limit of a function
at a given point c but while examining the existence
of the limit in the case of functions denoted by a
single formula y = f (x) defined in a -neighbourhood
of a given point c, it is a must to calculate both the
limits at a given point c where the given function
may or may not be defined.
2. In case, a function f is defined by two or more than
two formulas (different forms or expressions in x) in
adjacent intervals, it is necessary to find out both the
left hand limit (for the expression in x which is one
form of the given function f defined in an interval
indicating the left end point) and the right hand limit
(for the expression in x which is an other form of the
given function f defined in an other interval indicating
the right end point) noting that the left and right end
points of adjacent intervals are same (common) where
the left and right hand limits are calculated whether
the question says to evaluate the limit or examine the
existence of the limit of a given function at the
common point of adjacent intervals.
3. In questions, where f (x) contains modulus or
greatest integer functions, one is required to find out
both the left and right hand limits at a given point or
at the point where |f (x)| = 0 or greatest integer function
is zero, i.e. [f (x)] = 0.
4. A piecewise function is always defined in adjacent
intervals with different forms (expressions in x).

5. General models of a piecewise function:


A:
1. f (x) = f1 (x), when x < a
f (x) = f2 (x), when x > a
f (x) = f3 (x), when x = a
2. f (x) = f1 (x), when x < a
f (x) = f2 (x), when x > a
3. f (x) = f1 (x), when x < a
f (x) = f2 (x), when x > a
B:
1. f (x) = f1 (x), when c < x < a, c < x < a or c < x < a
f (x) = f2 (x), when a < x < d, a < x < c or c < x < a
2. f (x) = f1 (x), when c < x < a
f (x) = f2 (x), when a < x < d
f (x) = f3 (x), when x = a
3. f (x) = f1 (x), when x a
f (x) = f2 (x), when x = a
Remarks: 1. f (x) = f1 (x), when x < a or c < x < a
The function f1 (x) is to be selected to find out left
hand limit as well as the value of the function f (x) at
x = a.
That is, in the case of piecewise function,
The left hand limit of a function at the right end
point of an interval = lim (the function f1 defined in
xa

an interval whose left end point is a) and the value of


the function f at x = a is (f1 (x))x = a.
2. f (x) = f2 (x), when x > a or a < x < d The
function f2 (x) is to be selected to find out right hand
limit as well as the value of the function f (x) at x = a.
That is, in the case of piecewise function, the right
hand limit of a function f at the left end point of an
interval = lim+ (the function f2 defined in an interval
xa

whose right end point is a) and the value of the


function f at x = a is (f2 (x)) x = a.
3. f (x) = f3 (x), when x = a The function f3 (x) is to
be selected to calculate the value of the function f (x)
at x = a.
4. f (x) = f2 (x) when x a The same function
f2 (x) is to be selected to find out the left and right
hand limits both since x < a and x > a.

Practical Methods of Finding the Limits

That is, (the left hand limit at x = a) = (the right


hand limit at x = a) = lim (the function of x opposite
xa

af

to which x a is written) = lim f 3 x .


xa

5. f (x) = a constant c when x = a The function


f (x) has the value c at x = a, i.e., one is given the
value of a function f at x = a which is c and for this
reason, there is no need to calculate the value of the
function.
6. The value of a function f (x) at a point x = a is not
required to be calculated while examining the existence
of the limit of a function at a given point.
7. The value of a function f (x) at a point x = a is
required to be calculated while testing or examining
the continuity of a function f at a given point x = a.

249

Hence, the value of a function f at a point x = a


exists the value of a function f at x = a is a finite
number.
How to find left hand limit of a piecewise function f at
a point x = a
Step I: Replace x by (a h) in the given form of a
function f and also in an interval whose right end
point is a, where h 0 through positive values (i.e.
h is a small positive number).
Step II: Simplify the function f (a h), i.e. a function
in h and cancel out the common factor h (if any).
Step III: Substitute h = 0 in the simplified function
and its further simplification provides us the required
left hand limit of the given function f at the right end
point of adjacent intervals.

On existence and nonexistence of the value and the


limit of a function at a given point

How to find right hand limit of a piecewise function f


at a point x = a

The following possibility may arise.


1. The value of a function at a given point exists but
the limit of a function at a given point does not exist.
2. The limit of a given function at a given point exists
but the value of a given function at a given point
does not exist.
3. The value and the limit both of a function at a
given point exist but are not equal.
4. The value and the limit both of a function at a
given point exist and are equal.
5. The value and the limit both of a function at a
given point do not exist.

Step I: Replace x by (a + h) in the given form of a


function f and also in an interval whose left end point
is also a, where h 0 through positive values (i.e.

Notes: 1. When the value and the limit both of a


function at a given point exist and are equal, then the
function is said to be continuous at a given point and
in the rest cases, the function is said to be
discontinuous at a given point.
2. A function y = f (x) defined on its domain may not
always have a value for each value given to the
independent variable x. It is quite possible that for a
particular value or a set of values of x, there is no
value of the function y = f (x), (i.e. there is a value of

0
or imaginary). In such a case, it is
0
said that y = f (x) is undefined, undetermined,
interminate, meaningless or does not exist for those
values of x.
y = f (x) like ,

h is a small positive number).


Step II: Simplify the function f (a + h), i.e. a function
in h and cancel out the common factor h (if any).
Step III: Substitute h = 0 in the simplified function
and its further simplification provides us the required
right hand limit of the given function f at the left end
point of adjacent intervals.
Notes: 1. In the case of modulus and greatest integer
functions, the above method of procedure is
applicable since indeed, it is these functions which
are piecewise functions.
2. In the case of functions defined by a single formula
in a neighbourhood of the given point, there is a need
to calculate left and right hand limit by making the
substitution x = a h in the given single formula
(expression in x) defined in the neighbourhood of a
given point x = a.
Problems based on functions containing no modulus
function
1. Show that lim

x 0

FG cos x IJ does not exist.


H x K

Solution: Let h 0 through positive values.

250

How to Learn Calculus of One Variable

l.h.l = lim
h

g = lim cos bhg


b0 hg
b h g

cos 0 h

h0

b g
F 1I
= lim G J lim bcos hg = 1 =
H hK
cos b0 + hg
cos b +hg
= lim
r.h.l = lim
b0 + hg
b+hg
= lim

h 0

= lim

l 0

x 0

af

3. If f x =

Solution:

cos h
h

FG 1 IJ lim bcos hg = 1 =
H hK

F cos x IJ does not exist.


lim G
H xK

x0

h 0

b hg e d i
= lim

h 0
1 + ed i
1
h

1
h

af

1
0h

1+ e

e j
1
0+ h

= lim

h 0

,h

h 0

Dr by e

h 0

+1

1 + eh

eh e
1 e

1h

h1

+e

h1

eh

(multiplying Nr and

)
1
e

h1

+1

=1

Hence, l.h.l r.h.l

,h > 0

(multiplying Nr and Dr by e

af

lim f x does not exist.


x 0

af

5. If f x =

1
h

1h

h0

1
h

h1

1
0+ h

eh

= lim

b+hg e d i
= lim
h 0
1 + ed i
= lim

=0

h1

= lim

0
=
=0
1+ 0
1
0+h

1+ e

e j
1+ e

e
0 + hg e
b
r.h.l = lim
h 0

1
0 h

1h

e j
Let h > 0 r.h.l = lim
h 0
e j
1+ e

, show that lim f x exists.

e
0 hg e
b
Solution: l.h.l = lim

1
0h

1
0+h

1+ e

1
0 h

x0

1
x

x0

e j
l.h.l = lim
h 0
e j
1+ e

h 0

Hence, neither l.h.l nor r.h.l exists

af

af

, show that lim f x does not

1 + ex

= lim

l0

2. If f x =

ex

exist.

h0

x ex

af

lim f x exists.

la

h 0

OP
Q

Hence, l.h.l = r.h.l = 0

cos h
= lim
h 0 h

h 0

LM
N

1
0
= 0 e h 0 as h 0
0+1

ex 1
1

ex + 1

af

, show that lim f x does not


x0

exist.
1h

Solution: l.h.l = lim =


h 0

e
e

h1

h1

+1

01
,h>0
0+1

Practical Methods of Finding the Limits

LM
N

= 1 3 lim e

h1

h0

OP
Q

That is,

=0

y=

r.h.l = lim

h 0

= lim

h0

1h

eh 1
1

1 + eh

1 e

1h

1+ e

1h

,h>0

(multiplying Nr and Dr by

RS x , when x < 0
T x , when x 0

Now let h > 0;


x = 0 h y = (0 h) = + h, and x = 0 + h
y = (0 + h) = h,
Further, x = 0 h h 0 when x 0 and x
= 0 + h h 0 when x 0+
lim

x = lim x = lim h = 0

x 0+

).

x 0+

h 0

lim x = lim x = lim h = 0

10
=1
1+0

x0

h 0

x0

l.h.l r .h.l

Hence, lim+ x = lim x

lim f x does not exist.

lim x does exist.

x0

af

x 0

af

x 0

x0

1
x

af

6. If f x = e , x 0 then show that lim f x


x0

e j
1
0 h

h 0

F3 e
H

h1

1
0+h

h 0
1
h

1
h

=0

= lim e

y=

ch

y=

1
h

h 0

when h 0I
K

l.h.l r .h.l

af

lim f x does not exist.


x 0

Solution: y =

I
K

0 as h 0

r.h.l = lim e

F3 e
H

= lim e

c h

h0

e j

x
x

2. Show that lim

x0

does not exist.


Solution: Let h > 0;
l.h.l = lim e

251

x 0

Solution: y = | x |
y = x , when x > 0 and
y = x, when x < 0

does not exist.

x
x

x
= 1 , when x > 0 and
x

x
= 1 , when x < 0
x

That is, y =

RS1, when x < 0


T 1, when x > 0

Now let h > 0;


x = 0 h y = 1, and x = 0 + h y = 1
Further, x = 0 h h 0 when x 0 and x =
0 + h h 0 when x 0+

Problems based on modulus function


1. Examine the existence of lim x .

x
x

lim+
x0

lim

x0

af

x
= lim+ 1 = lim 1 = 1
h0
x
x0

a f

a f

x
= lim 1 = lim 1 = 1
h 0
x
x0

252

How to Learn Calculus of One Variable

Hence, lim+
x0

x0

ex
4. Examine the existence of lim

lim

3. Examine whether lim

x 0

Solution: y =

Solution:

x
=
x

a x f
=
x

y=

, when x < 0

x
= x2, when x < 0
x

and y =

Hence y =

can be rewritten as

|RS x , when x < 0


|T x , when x > 0
2

Now let h > 0;


x = 0 h y = (0 h)2, = h2, and x = 0 + h
y = (0 + h)2, = h2, when h > 0
Further x = 0 h h 0 when x 0 and x =
0 + h h 0 when x 0+

lim+

lim

x
x

x0

x0

x0

x
x

That is, y =

R|S b x + 2g , when x > 2


|Tb x + 2g , when x < 2

Now, x = 2 + h, h > 0
y = (2 + h + 2),
= (4 + h),
and x = 2 h, h > 0
y = (2 h + 2),
= (4 h),
Further, x = 2 + h h 0, when x 2+ and x
= 2 h h 0, when x 2

ex 4j =
2

lim+
x2

e j = lim eh j = 0

= lim x
x0

x
x

and

h 0

x0

x0

lim

= lim+ x = lim h = 0

Hence, lim+

a x 2f a x + 2f , when x 2 < 0
a x 2 f

a f

a f

lim+ x + 2 = lim 4 + h = 4

x2

x2

h 0

ex 4j = lim a x + 2f = lim a4 hf = 4
lim
2

x
x

a x 2f a x + 2f , when x 2 > 0
a x 2f

x
x
=
=
Also, y =
, when x > 0
x
x
x
= x2, when x > 0

y=

= (x 2), when x < 2

= (x + 2), when x > 2

x2

exists.

ex
y=

x2

x 2

x
lim
does not exist.
x0 x

= lim
x0

h0

x
x

x 2

x2

x 2

h0

e x 4j
2

Hence, lim+
x2

x2

e x 4j
2

lim

x 2

x 2

e x 4j does not exist.


lim
2

x2

x2

does exist.

5. Show that lim

x0

sin x
x

does not exist.

Practical Methods of Finding the Limits

sin x
x

Solution: y =

y=

y=

sin x
, when sin x < 0
x

R| sin x , when sin x 0 , x 0


x
That is, y = S sin
|T x x , when sin x < 0
Now x = 0 + h, h > 0 and sin h > 0
y=

sin 0 + h
sin h
=
,
0+h
h

and x = 0 h, h > 0 and sin h > 0


y=

sin 0 h
sin h
=
0h
h

Again x = 0 + h h 0 when x 0+ and


x = 0 h h 0 when x 0

F sin x IJ = lim FG sin h IJ = 1


Hence, lim G
H xK
H hK
F sin x IJ = lim FG sin h IJ = 1
lim G
H xK
H hK
F sin x IJ lim FG sin x IJ = 1
lim G
H xK
H xK
F sin x I does not exist.
lim G
H x JK
x0

x0

h 0

h 0

x0

x0

x0

1 cos 2 x

6. Does lim

x0

2 sin x
x

exist?

y=

R| 2 FG sin x IJ , when sin x 0 , x 0


| H xK
That is, y = S
|| 2 FGH sinx x IJK , when sin x < 0
T
Now, x = 0 + h, h > 0 and sin h > 0

FG sin h IJ , and x = 0 h, h > 0 and sin


H hK

y=+ 2
h>0

FG sin h IJ
H hK

y= 2

Further, x = 0 + h h 0 when x 0+ and


x = 0 h h 0 when x 0

FG sin x IJ = lim 2 FG sin h IJ = 2


H xK
H hK
F sin x IJ = lim 2 FG sin h IJ = 2
2G
H xK
H hK
F sin x IJ lim 2 FG sin x IJ
2G
H xK
H xK

Hence, lim+ 2

h 0

x0

and lim
x0

lim+
x0

lim

h0

x0

1 cos 2 x

x0

Solution: y = e

, when x > 0 and y = ex, when x < 0

R|e
S| e
T

does not exist.

x0

That is, y =

sin x

7. Examine the existence of lim e

y=e

1 cos 2 x
x

Solution: y =

FG sin x IJ when sin x > 0 and


H xK
F sin x IJ when sin x < 0
2 G
H xK

y=

sin x
, when sin x > 0 and
x

253

, when x 0
, when x < 0

Now, x = 0 + h, h > 0

254

How to Learn Calculus of One Variable

y=e

y=e

, and x = 0 h, h > 0

Further, x = 0 + h h 0 when x 0+ and


x = 0 h h 0 when x 0
lim+ e

= lim e

=e =1

h 0

x0

lim e = lim e = e = 1
x

x 0

h 0

Hence, lim+ e

x0

lim e

x0

= lim e = 1

Solution: Let h > 0;


x = 2 + h f (x)
= f (2 + h) = 5, and x = 0 h f (x)
= f (2 h) = (2 h)2 + 1
= 4 4h + h2 + 1
= h2 4h + 5, when 2 < h < 0
Further x = 2 h h 0 when x 2 and x =
2 + h h 0 when x 2 +

lim+ 5 = lim 5 = 5
h 0

x2

x0

x2

does exist and = 1

x2

h 0

x 0

a f

lim x = lim h = 0
h 0
2

x0

Hence, lim+ x = lim x = 0


x0

x0

af

lim f x does exist.


x 0

N.B.: The value of the function f (x) = 1 at x = 0 is not


required to be considered to show the existence of
the limit of a given function at the point x = 0.
2. Examine the existence of the limit of the function
f (x) as x 2 if it exists where

R|x + 1 , when 0 < x < 2


f bxg = S
|T5 , when x 0
2

x2

af

lim f x does exist.


x2

3. Examine the existence of the limit of f (x) at x = a,


2

where f (x) =

x a
, when 0 < x < a
xa

= 2a, when x > a


Solution: Let h > 0;
x=ah

b g b g babahghg aa
2

f x = f ah =

lim+ x = lim h = 0

h0

Hence, lim+ 5 = lim x + 1

Problems based on piecewise function


1. Examine the existence of the limit of the function
f (x) as x 0 if it exists where
f (x) = x, when x < 0
= 1, when x = 0
= x2, when x > 0
Solution: Let h > 0;
x = 0 + h f (x)
= f (0 + h) = (0 + h)2 = h2 = h2, and x = 0 h f (x)
= f (0 h) = (0 h) = h,
Further, x = 0 h h 0 when x 0 and
x = 0 + h h 0 when x 0 +

lim x + 1 = lim h 4h + 5 = 5

a 2ah + h a
h

2ah + h
and x = a + h
h
f (x) = f (a + h) = 2a
Further, x = a h h 0 when x a and
x = a + h h 0 when x a +
=

lim+
xa

Fx
GH x

xa

I = 2a
JK

F x a I = lim F 2ah + h I = lim h F 2a + h I


GH h JK
GH x a JK GH h JK
2

lim

a2
2
+ a2
2

h0

h 0

Practical Methods of Finding the Limits

a
f
F x a I = 2a
= lim G
H x a JK
lim f a x f exists.

R|x , when x < 1


f a x f = S x , when x > 1
|| 2 , when x = 1
T

= lim 2a + h = 2a

h 0

xa

xa

Problems based on redefined function


1. Examine the existence of the limit of f (x) at x = 0, if
it exists where f (x) = 1, when x 0
= 2, when x = 0
Solution: The given function
x0
a f RST12 ,, when
when x = 0 can be rewritten as under:
R|1, when x < 0
f a x f = S1 , when x > 0
|T2 , when x = 0

f x =

Let h > 0;
x=1h
f (x) = f (1 h) = (1 h)2
= (1 2h + h2)
and x = 1 + h
f (x) = f (1 + h) = (1 + h)2, when 1 + h > 1
= (1 + 2h + h2)
Further x = 1 h h 0 when x 0 and x =
1 + h h 0 when x 0 +
2

h 0

x 1

h 0

x 1

x 1

j=1

af

x 1

lim f x does exist.

bg
bg
and lim f b x g = lim f bhg = 1
lim f a x f does exist.

lim f x = lim f h = 1
h0

h 0

x 0

Note: x a x > a or x < a, where a R .


2. Examine the existence of the limit of f (x) at x = 1,
where f (x) = x2, when x 1
= 2, when x = 1
Solution: The given function

R|x , when x 1
S| 2 , when x = 1 can be rewritten as under:
T
2

j=1

x 1

= 0 + h h 0 when x 0 +

af

lim+ x = lim x = 1

Further, x = 0 h h 0 when x 0 and x

f x =

and lim x = lim 1 2 h + h

x 0+

Now lim+ x = lim 1 + 2 h + h

Let h > 0;
x=0h
f (x) = f (0 h) = 1 and x = 0 + h
f (x) = f (0 + h) = 1

x0

255

Exercise 4.29.1
Problems on functions defined by a single formula
with no modulus function
Do the limits of the following functions exist?

F 1I
H xK
F 1I
lim cos
H xK

1. lim sin

[Ans: Does not exist.]

2.

[Ans: Does not exist.]

x0

x0

3. lim e x

[Ans: Does not exist.]

x0

4. lim tan
x0

F 1I
H xK

[Ans: Does not exist.]

256

How to Learn Calculus of One Variable

5. lim

x0

x
x

[Ans: Does exist.]

1.

Exercise 4.29.2
Problems on functions defined by a single formula
with the modulus function
Examine the existence of the limit of each given
function.
1. lim

x 1

2. lim

x0

3. lim

x1
x1

[Ans: Does not exist.]

x
1 cos x

[Ans: Does not exist.]

4. lim

x0

[Ans: Does exist.]

2 cos x

1 cos 2 x
sin x

4x + 3
2

x 1

, when x 1

[Ans. Exists]

2 , when x = 1

Rx
a f |S x , x 0 [Ans. Does not exists]
|T 0 , x = 0
F 1 I , when x 0 [Ans. Exists]
f a x f = x sin
H xK

2. f x =

3.

= 0, when x = 0

af

sin x
,x0
x
= 0, x = 0

4. f x =

[Ans. Exists]

On existence of limit of greatest integer function

1 + cos 2 x

x0

R| x
f axf = S
|T

[Ans: Does not exist.]

Exercise 4.29.3
Problems on piecewise functions
Examine the existence of the limit of each function
defined as given below:
1. f (x) = x2 + x + 2, when x < 1
f (x) = x4 + 3, when x > 1
[Ans: Exists at x = 1]
2. f (x) = x3, when x < 1
f (x) = x5, when x > 1
f (x) = 1, when x = 1
[Ans: Exists at x = 1]
3. f (x) = x2 2x + 3, when x < 1
= x + 1, when x > 1
[Ans: Exists at x = 1]

when x < 4
a f RST5x x3 ,, when
[Ans. Exists at x = 4]
x4

4. f x =

Exercise 4.29.4
Problems on redefined functions
Examine the existence of the limit of each function
defined by y = f (x) at the indicated point x = a , a R

Firstly one should note the following key points:


1. [x] denotes the first integer less than or equal to x
(given real number) lying on the number line to the
left side of x (given real number) The integer on
the number line which is nearest to x on the left side
of the given real number. Hence, to find [x] numerically
at any given real number for x, we always consider
the first integer (or, nearest integer) which is on the
left side of the given real number lying on the number
line. In the light of this explanation, we are able to
provide the following useful rules:
(a1) [x] = x provided x = an integer +ve, ve or zero
i.e., 0, 1, 2, , e.g.:
(i) [0] = 0
(ii) [5] = 5
(iii) [3] = 3
(a2) [x] = an integer immediately to the left of x
provided x is positive or negative fraction (i.e., x is
not an integer), e.g.:
(i) [5/2] = 2
(ii) [5/2] = 3
(iii) [0.1] = 1
(iv) [(0.75)] = 1
(v) [6.35] = 7

Practical Methods of Finding the Limits

257

h is sufficiently small positive real number > 0 then


we have
5/2 2 3/2 1 1/2 0

1/2

3/2

5/2

(a3) [x] = 0 for all positive real values of x just less


than unity (0 < x < 1), e.g.:

LM OP
NQ
3
L3O
(ii) x =
x =M P=0
12
N12 Q
2
L2 O
(iii) x = x = M P = 0
3
N 3Q

(i) x =

1
1
=0
x =
2
2

(a4) [x] = 1 for all positive real values of x just greater


than unity (1 < x < 2), e.g.:
(i) x =

LM OP
NQ

3
3
=1
x =
2
2

(ii) x = 1 1 x = 1 1 = 1
(a5) [x] = 1 for all negative values of x whose absolute
value is first greater than unity (1 < x < 0), e.g.:
(i) [0.1] = 1
(ii) [0.0001] = 1
(iii) [0.0009] = 1
(iv) [0.23.49] = 1
(v) [0.9999] = 1
(a6) If n be any integer and m be any real number, then
for 0 < h < 1 (i.e. [h] does not exceed zero) and
0 < m h < 1 (i.e. [m h] does not exceed zero) where h is
sufficiently small positive number greater than zero,
we have
(i) [n + h] = n
(ii) [n + m h] = n as [8 + 12 h] = 8
(iii) [n 1 + h] = n 1
(iv) [n 1 + m h] = n 1
(v) [0 + h] = 0
(vi) [n h] = n 1
(vii) [n m h] = n 1 as [8 12 h] = 7
(viii) [n 1 h] = n 1 1 = n 2
(ix) [0 h] = 0 1= 1
(a7) If n = an improper positive fraction and m be any
real number, then for 0 < h < 1, and 0 < m h < 1, where

n h / n m h = integral part of the improper


fraction
= the whole number before the decimal
e.g.:
(i) [1.5 h] = 1
(ii) [1.5 + h] = 1
(iii) [2.4 + h] = 2
(iv) [2.4 h] = 2
(v) [2.000001 h] = 2

(a8) If n = a negative improper fraction and m be any


real number, then for 0 < h < 1 and 0 < m h < 1, where
h is sufficiently small positive number > 0, then we
have [n h]/[n m h] = integral part of the proper
fraction 1
= the whole number before the decimal 1
e.g.:
(i) [2.3 h] = 2 1 = 3
(ii) [5.0006 h] = 5 1 = 6
(iii) [7.00001 12 h] = 7 1 = 8
(iv) [1.1234 2.3 h] = 1 1 = 2
Some important facts about the function y = [x].
(i) x = x x I
(ii)
(iii)

x < x x I

a f

x = k k I k x < k + 1 , i.e. x = k

a f a f

k < x < k + 1, k N and x = k + I k + 1


x < k, k N

2. To calculate arithmetically n h / n m h
where n and m are any two arbitrary numbers and h
lies between 0 and 1 (i.e.; 0 < h < 1), we may adopt the
following working rule, in the problems on limits.
(i) Put h = any one of the numbers 0.1, 0.01, 0.001,
0.0001, 0.00001, , etc. whose number of zeros =
number of digits after decimal in the given value of n
provided we calculate [x] = n h where x = n h .
(ii) Put h = any one of the numbers 0.1, 0.01, 0.0001,
0.001, 0.00001, , etc. whose number of zeros = the
sum of number of digits in the integral part and decimal
part of m provided we calculate [x] = n mh where
x = n mh .

258

How to Learn Calculus of One Variable

(iii) Calculate ( n mh ) arithmetically when n and m


are known particular numbers.

Limit method to examine the existence of greatest


integer function

(iv) Use the fact: the integer on the number line which
is to the left side of n m h / n h will represent

To evaluate (or, to find or to examine the existence of


lim f x = lim (greatest integer/bracket

n mh / n h .

fa

Examples
(i) [1 + h] = [1 + 0.1] = [1.1] = 1
(ii) [1 h] = [1 0.1] = [0.9] = 0
(iii) [1.5 + h] = [1.5 + 0.01] = [1.51] = 1
(iv) [1.5 h] = [1.5 0.01] = [1.49] = 1
(v) [2.000001 h] = [2.000001 0.0000001] = [2.000009]
=2
(vi) [2.201 h] = [2.201 0.0001] = [2.2009] = 3
(vii) [2 h] = [2 0.1] = [2.1] = 3
(viii) [8 12 h] = [8 12 0.001] = [8 0.012] = [7.988]
=7
(ix) [8 + 12 h] = [8 + 12 0.001] = [8 + 0.012] = [8.012]
=8
(x) [0 h] = [0 0.1] = [0.1] = 1
(xi) [0 + h] = [0 + 0.1] = [0.1] = 0
(xii) [1 + h/2] = [1 + 0.5 h] = [1 + 0.5 0.01] = [1 + 0.005]
= [1.005] = 1
(xiii) [2.3 233.6 h] = [2.3 233.6 0.00001] = [2.3
0.002336] = [2.2977] = 2
Precaution
1. While calculating [ n h ] arithmetically, h should
be chosen so small that it does not increase or
decrease [n] by unity, n is not integer.
e.g.: If we choose h = 0.1 to calculate [2.000001 h],
we get [2.000001 0.1] = [1.900001] = 1 as a greatest
integer contained in [2.000001 h] = which is false
because [2.000001 h] = 2.
2. While calculating [ n mh ], h should be chosen
so small that m h must be slightly greater than zero.
e.g.: If we choose h = 0.1 to calculate [2.3 299 h], we
get [2.3 299 h] = [2.3 299 0.1] = [2.3 29.1] = [
26.8] = 27 which is false because [2.3 299 h] = 2.
Similarly, [2.3 + 299 h] = [2.3 + 299 0.1] = [2.3 +
29.1] = [31.4] = 31 which is false because [2.3 + 299 h]
= 2.
3. The above working rule is valid to find the limit of
n m h / n h as h 0 .

xn

af

xn

function), where n = any real number, we adopt the


following working rules.
Working rule 1
1. Find the left hand limit using the following scheme:
(a) put x = n 1 + h (where 0 < h < 1) in f (x)
(b) find the greatest integer in f (n 1 + h) where
0 < h < 1 and remove the symbol of square bracket
from [f (n 1 + h)]
(c) use lim (greatest integer) = greatest integer
h0

2. Find the right hand limit using the following


scheme:
(a) put x = n + h' (where 0 < h' < 1) in f (x)
(b) find the greatest integer in f ( n + h') where
0 < h' < 1 and the symbol of square bracket be removed
from [f (n + h')]
(c) use lim (greatest integer) = greatest integer
h0

Working rule 2
1. Find the left hand limit using the following scheme:
(a) Put x = n h in f (x) where 0 < h < 1
(b) Find the greatest integer in f (n h) and
remove the symbol of square bracket from
[f (n h)]
(c) Use lim (greatest integer) = greatest integer.
h0

2. Find the right hand limit using the following


scheme:
(a) Put x = n + h in f (x) where 0 < h < 1
(b) Find the greatest integer in f (n + h) and the
symbol of square bracket be removed from
[f (n + h)]
(c) Use lim (greatest integer) = greatest integer.
h0

Note:
1. If we are asked to examine the existence of

af

lim f x , we have to examine l.h.l and r.h.l. If they

h n

af

are equal, it is declared that lim f x exists and of


h n

Practical Methods of Finding the Limits

af

they are not equal, it is declared that lim f x


h n
does not exist.
2. We recall that
(i) [n 1 + h] = n 1, provided 0 < h < 1 and n is
an integer.
(ii) [n + h] = n, provided 0 < h < 1 and n is an
integer
(iii) [n h] = (n 1), provided 0 < h < 1 and n is
an integer

3 [n + h'] = n for 0 < h < 1 and n = an integer


l . h. l r . h .l lim x does not exist
x 1

Method 2
l.h.l = lim x
x 1

a f

= lim 1 h = lim 1 1 = lim 0 = 0


h 0

h0

h 0

3 [n h] = n 1 provided 0 < h < 1 and n = an


integer

Examples
(i) [2 1 + h] = 2 1 = 1
(ii) [2 + h] = 2
(iii) [2 h] = 2 1 = 1

r.h.l = lim+ x = lim 1 + h = lim 1 = 1

Note: Existence of a function in limit Existence of


a function in the sense of limiting value Existence
of limit of a function for the limit of an independent
variable.
3. Method (1) is applicable only when we have
greatest integer function [f (x)] only whose limit is
required whereas method (2) is applicable in every
case and particularly when the given function is the
combination (sum, difference, product and /quotient)
of f1 (x) and [f2 (x)].
Type 1: To evaluate (or, to find or, to examine the
existence of)

af

lim f x , where n = an integer

h n

259

h 0

x 1

h0

3 [n + h] = n provided 0 < h < 1 and n = an


integer
l . h. l r . h .l lim x does not exist
x 1

2. lim x
x2

Solution: Method 1

a f

l.h.l = lim x = lim 2 1 + h = lim 2 1 = 1


x2

h 0

h 0

3 [n 1 + h] = n 1 for any integer n

r.h.l = lim+ x = lim 2 + h = lim 2 = 2


x2

h 0

h 0

Examples worked out:

3 [n + h'] = n for any integer n

Examine the existence of the following limits


1. lim x
x 1

l . h. l r .h . l lim x does not exist.


x2
Method 2

Solution: Method 1
l.h.l = lim x

2 h = lim 1 = 1
l.h.l = lim x = hlim
0
h 0

x 1

= lim 1 1 + h = lim h = lim 1 1 = lim 0 = 0


h 0

h 0

h 0

bg

r.h.l = lim x = lim 1 + h = lim 1 = 1


h 0

[n h] = n 1 for any integer n

h 0

3 [n 1 + h] = n 1 for 0 < h < 1 and n = an


integer

x 1

x2

h 0

2 + h = lim 2 = 2
r.h.l = lim+ x = hlim
0
h 0
x2

3 [n + h] = n for any integer n


l . h. l r .h . l lim x does not exist.
x2

260

How to Learn Calculus of One Variable

h0

xn

a f

l.h.l = lim x = lim n 1 + h = lim n 1 = n 1


h0

xn

a f

a f

= lim 1 1 + h = lim h = lim 1 = 1 3 h = 1

3. lim x for any integer n.


h0

h 0

h 0

Thus, we observe, l.h.l r.h.l which means


lim 1 x doe not exist.

x 1

3 [n 1 + h] = n 1 for 0 < n < 1 and n being


an integer
h 0

x 1

h 0

3 [n + h'] = n for 0 < h' < 1 and n being an


integer
Thus, we observe that lim x lim x
x n

x 1

Solution: l.h.l = lim x 1

r.h.l = lim x = lim n + h = lim n = n


x n=

5. lim x 1

x n+

h 0

h 0

h 0

r.h.l = lim x 1
x 1+

= lim 1 + h 1 = lim h = lim 0 = 0 3 h = 0

l.h.l r.h.l.

h 0

lim x does not exist for any integer n.

h 0

h 0

x 1 does not
l.h.l r.h.l which means xlim
1

xn

Method 2

exist.

l.h.l = lim

x n

6. lim x

h 0

h0

Solution:

3 [n h] = n 1 for 0 < h < 1 and n being an


integer

h 0

x lim

x l.h.l

x n+

= lim 8 12h + 6h h

h 0

= lim 7 3 7 < 8 12h < 8


h0

=7
Type 2: To evaluate (or, to find or to examine the

af

r.h.l.

f x when n an integer (i.e.,


existence of) xlim
n
n = a fraction +ve or ve), we adopt the working rules
mentioned earlier in type 1.

lim x does not exist.


xn

4. lim 1 x

Examples worked out:

x 1

Evaluate if the following limit exists.

Solution: l.h.l = lim 1 x


x 1

a f

lim 1 1 h = lim h = lim 0 = 0 3 h = 0


h 0

r.h.l = lim 1 x
x 1+

= lim 8 12 h

[n + h] = n for 0 < h < 1 and n being an


integer
x n

lim x

x 2

= lim 2 h
h0

x = lim n + h = lim n = n
r.h.l = xlim
h 0
h 0
n+

Thus, we observe, lim

x 2

= lim n h = lim n 1 = n 1

h 0

a f

= lim 1 h 1 = lim h = lim 1 = 1 3 h = 1

h 0

1. lim x
x 15

Solution: l.h.l = lim

x 15

= lim 1 5 h = lim 1 = 1 3 1 < 1 5 h < 2


h0

h0

261

Practical Methods of Finding the Limits

r.h.l = lim

x 15+

LM F 1 + hI + F 1 + hI OP
MN H 2 K H 2 K PQ
1 1
L 1
O
= lim M1 + + h + + 2 h + h P
4 2
N 2
Q
L3 1
L7 O
O
= lim M + + 2h + h P = lim M + 2hP = lim 1
N2 4
N4 Q
Q
F3 1 < 7 + 2h < 2I
H 4
K

= lim 1 5 + h = lim 1 = 1 3 1 < 1 5 + h < 2


h 0

h0

= lim 1 +

h 0

Thus, we observe, l.h.l = r.h.l limit exists and

h 0

lim x = 1.

x 15

2.

lim

x 24

h 0

Solution: l.h.l = lim

x 24

= lim 2 4 h = lim 2 = 2 3 2 < 2 4 h < 3


h 0

h0

r.h.l = lim

x 24 +

= lim 2 4 + h = lim 2 = 2 3 2 < 2 4 + h < 3


h 0

h 0

l.h.l = r.h.l limit exists and lim

x 24

x = 2.

3. lim x , where n < x0 < n + 1 for some integer n.


x x0

Solution: l.h.l =
r.h.l =

lim

x x0 + 0

lim

x x0 0

x = lim x0 h = n
h 0

x = lim x0 + h = n

Solution: Method 1
Let f (x) = x2 = z

3 x 2 x2 2 z 2
lim z = lim
Z 2

x 2

x2

Now, let h > 0,


l.h.l = lim z
Z 2

small h > 0)
r.h.l. = lim+ z
= lim 2 + h = 2 (3 2 < 2 + h < 3 for sufficiently
h0

lim1 2 x

x 2

small h > 0)
lim 2 x

Thus, we observe, l.h.l r.h.l lim z

x 21

LM F 1 hI OP = lim
N H 2 KQ
a3 0 < 1 2h < 1f

= lim 2
h 0

lim 1 + x + x

h 0

x 21 +

Solution:

Z 2

x x0

5.

x 2

h 0

lim x exists and = n.

Solution:

lim

h 0

= lim 2 h = 1 ( 3 1 < 2 h < 2 for sufficiently

h 0

( 3 for sufficiently small h n < x0 h < n + 1 and n


< x0 + h < n + 1)

4.

6.

h 0

Z2

1 2h = lim 0 = 0
h 0

= lim

Z 2

Method 2
l.h.l =
= lim

lim 1 + x + x

x 21 +

x 2 does not exist.

h 0

lim x

x 2

LMe
N

2 h

j OQP = lim
2

= lim 2 2 2 h
h 0

h 0

2 + h 2 2h

262

How to Learn Calculus of One Variable

= lim 0

( 3 2 + h 2 2 h 2 2 2 h for sufficiently
small h > 0)

h 0

=0

af

= lim 1 ( 3 for sufficiently small h > 0,


h 0

LMsin F + hI OP = 0, as 0 < h 0 < + h


2
N H 2 KQ
F + hI 0 < sin F + hI
sin 0 < sin
H2 K
H2 K
F + hI < 1
0 < sin
H2 K

2 2 2h = 1 )

=1
r.h.l = lim + x

x 2

= lim

h0

LMe
N

2 +h

j OQP = lim
2

h 0

2 + h + 2 2h

= lim 2 + 2 2h

lim sin x = 0

h 0

x 2

( 3 2 + h + 2 2 h 2 + 2 2h
neglecting
higher powers of h for h being sufficiently small > 0)

= lim 2 ( 3 for small h > 0, 2 + 2 2 h = 2 )


h 0

=2
l.h.l r.h.l which means lim

x 2

8.

lim sin x

x 4

Solution:

lim sin x

x 4

LM F hI OP
N H 4 KQ

= lim sin
does not

exist.

h 0

= lim 0
h 0

=0

7. lim sin x
x 2

LM F hI OP
N H 2 KQ

Solution: l.h.l = lim sin x = lim sin


x 2

h 0

= lim 0
h 0

=0

LM F hI OP = 0 as h 0 and h > 0 h < 0


N H 2 KQ

3 sin

0< h<
2
2

F hI < 1
0 < sin
H2 K
LM FG + hIJ OP
N H 2 KQ

r.h.l = lim sin x = lim sin


x 2 +

h 0

LM F hI OP = 0 , as 0 < h <
4
4
N H 4 KQ
F hI < sin
sin 0 < sin
H4 K 4
F hI < 1 < 1
0 < sin
H4 K 2

3 sin

Note: The real number is approximately equal to


3.14.
Type 3: To evaluate (or, to find or to examine the
existence of) the function which is the combination
(i.e.; sum, difference, product or quotient) or
composition (i.e. a function of a function) of f1 (x)
and [f2 (x)] (i.e.; a function and greatest integer
function) as x a a real number.

Practical Methods of Finding the Limits

Examples worked out:


x x0

exist when x0 is an integer (ii) lim x x exists


x x0
when x0 is not an integer.
Solution: Let f (x) = x [x]
(i) If x0 = n (an integer), then for h > 0 (h being
sufficiently small) f (x0 h) = f (n h) = n h [n h]
= n h (n 1) = 1 h

lim f x0 h = lim 1 h = 1
h 0

h 0

lim f x0 + h = lim h = 0

x x0

Now we will solve such problems which are all


particular cases of Q. no. (1) for the beginners
independently.
2. Evaluate (if it exists) the following

l.h.l = lim x x
x 0

g
h0

h 0

h0

Solution: l.h.l = lim x x

g does not exist.

b
g
= lim a2 h 1f ( 3 [n h] = n 1 when n is an
= lim 2 h 2 h
h0

f
r.h.l = lim b x x g = lim b2 + h 2 + h g
= lim a2 + h 2 f ( 3 [n + h] = n when n is an
x2

h 0

h 0

integer)

= lim h = 0
h 0

l.h.l r.h.l which means xlim x x


2

not exist.

(iii) lim x x
3

g does

FG 3 h L 3 hOIJ
MN 2 PQK
H2
F 3 h 1I
= lim
H2
K
L3 O
( 3 for sufficiently small h > 0, M hP = 1 )
N2 Q
F 1 hI = 1
= lim
H2 K 2
r.h.l = lim b x x g
F 3 L 3 OI
lim G + h M + hPJ
H2
N 2 QK
h 0

h 0

h 0

= lim 0 + h 0 + h = lim h h
h0

x0

h 0

b
g
b
= lim b h a1fg b3 h = 1g
= lim a h f + lim 1 = 0 + 1 = 1
r.h.l = lim b x x g
x 0

x2

Solution: l.h.l = lim

= lim 0 h 0 h = lim h h

h 0

(ii) lim x x

x 2

h 0

r.h.l l.h.l lim x x

h 0

(ii) If n < x0 < n + 1 for some integer n, then for


some sufficiently small h > 0, f (x0 h) = x0 h [x0
h] = x0 h n x0 n as h 0 and f (x0 + h) = x0
+ h [x0 + h] = x0 + h n x0 n as h 0
r.h.l = l.h.l = x0 [x0]
the lim (x [x] ) exists and = x0 [x0]

Solution: Let h > 0

h 0

= lim 1 h = 1

x x0

=0

= lim h = 0

integer)

lim (x [x]) does not exist

x 0

f b3 h

h 0

h0

r.h.l. l.h.l.

(i) lim x x

h 0

x 2

and f (x0 + h) = f (n + h) = n + h [n + h] = n + h n = h
h0

= lim h 0

Question 1: Prove that (i) lim (x [x]) does not

263

x 23 +

h 0

264

How to Learn Calculus of One Variable

F 3 + h 1I ( 3 for sufficiently small


H2
K
3
L O
h > 0, M + hP = 1 )
N2 Q
F 1 + hI = 1
= lim
H2 K 2

(3 for sufficiently small h > 0, [3 h] = 2)

= lim

h 0

h 0

x 3+

x
x

3+h
3+h
( 3 for small h > 0,
= lim
h 0
3+h
3

= lim

h 0

[3 + h] = 3)

1
l.h.l = r.h.l =
2

lim3 x x

Thus, we observe, l.h.l r.h.l lim

x 2

3.

r.h.l = lim

bx + x g

lim

x 2 3

g exists and = 12 .
b

Solution: l.h.l = lim x + x


x 23

lim 2 3 h + 2 3 h

h 0

x3

h > 0, [2.3 h] = 2)

(i)

r.h.l = lim + x + x
x 23

b
g
= lim a2 3 + h + 2 f ( 3 for sufficiently small h
h0

h 0

> 0, [2.3 + h] = 2)

= lim 4 3 + h = 4 3
h 0

x 23

Solution: l.h.l = lim


x3

h0

x
x

lim

x1
x1

x 1 + 0

bx 2 + x + 2 g
F x 1 + 2IJ
(v) lim G x +
H x 1 K
1
(vi) lim x L O
MN x PQ
1 L1O
(vii) lim
x MN x PQ
F 1 L 1 OI
(viii)
lim G x + M x + PJ , where K is an
H 4 N 4 QK
lim

x2 0

x 1+ 0

x 13 +

integer and [t] denotes the greatest integer less than


or equal to t.

x
x

= lim

lim

x 00

x k + 43 0

[x]) exists and = 4.3.


x 3

LM x OP
N2 Q

x 13

Thus, we observe, l.r.l = r.h.l = 4.3 lim (x +

4. lim

(ii)

(iv)

= lim 2 3 + h + 2 3 + h

lim

x 2 + 0

(iii)

= lim 4 3 h = 4.3
h 0

x
does
x

not exist.
5. Evaluate each of the following one sided limits

= lim 2 3 h + 2 ( 3 for sufficiently small


h 0

3
=1
3

x
x

3h
3h 3
= lim
=
h 0
3h
2
2

LM x OP
N2Q
L 2 + h OP = lim LM1 + h OP , h > 0
= lim M
N 2Q
N 2 Q

Solutions: (i)

h 0

lim

x 2 + 0

h 0

Practical Methods of Finding the Limits

=0+2=2

= lim 1
h 0

3 [3 h] = 2 as [n h] = n 1 when n = an
integer

=1

h
h
< 2 1 < 1 + < 2 from
2
2
definition of greatest integer function
31 +

(ii)

lim

x 0 0

= lim

h 0

the

x
x
0h
h
,h>0
= lim
h 0 h
0h

F h + 2I
H h K
= lim a1 + 1 + 2f
= lim 1 +
h 0

h0

= lim

h 0

x1
x1

3 | h | = h as h > 0 and
h 0

0
h

3 h = 0 as h 0 and h > 0 0 < h < 1

bx 2 + x + 1 g
= lim b2 h 2 + 2 h + 1 g , h > 0
= lim b h + 3 h g = lim a hf + lim a3 h f

lim

x20

h0

h 0

h 0

LM 1 OP
NxQ

LM OP
1
1
F
I
= lim
H 3 + hK MM 1 + h PP , h > 0
N3 Q
F 1 + hI LM 3 OP
= lim
H 3 K N1 + 3h Q
F 1 + hI lim LM 3 OP
= lim
H 3 K N1 + 3h Q
h0

h 0

h 0

= 0 + lim 2

h 0

x 3 +

1+ h1
h
,h>0
= lim
h 0 h
1+ h1

h 0

= lim 4 = 4

(vi) lim1 x

= lim 0 = 0

(iv)

IJ
K

+ 2 ,h>0

h 0

3 [h] = 1 as [n h] = n 1 when n is an integer

= lim

x 1+ 0

3 [1 + h] = 1 as [n + h] = n when n = an
integer

h 0

lim

FG x + x 1 + 2IJ
H x 1 K
1+ h 1
F
= lim G 1 + h +
1+ h 1
H
F h + 2IJ
= lim G1 +
H h K

lim

h 0

h 0

x 1 + 0

(v)

h 0

F 1 I
= lim
H h K
F 1I
= lim
H hK

(iii)

265

h 0

h 0

h0

af

1
lim 2
3 h0

1
2
3

2
3

266

How to Learn Calculus of One Variable

L 3 OP = 2 as h 0 and h > 0 1 + 3h > 1


3M
N1+ 3h Q
1
3
< 1
<3
1 + 3h
1 + 3h

lim1

(vii)

x 3

LM OP
NQ

(i) lim cos x

1 1
x x

x0

h0

RF 3 IJ LM 3 OPUV
= lim SG
TH 1 + 3h K N1 + 3h QW
F 3 IJ lim LM 3 OP
= lim G
H 1 + 3h K N1 + 3h Q
F 3 IJ lim a3f
= lim G
H 1 + 3h K
h 0

h 0

h 0

h 0

h 0

= (3) (3)
=9

1 + 3h

3 <

< 1

1 + 3h

> 3,

x k + 4 0

= lim k h + 1 k
h 0
h 0

lim cos x

x 4 +

Solutions: (i) r.h.l = lim+ cos x


x0

= lim cos 0 + h = lim cos h


h0

h0

= lim cos 0
h 0

= lim 1
h 0

=1

3 [h] = 0 as [n + h] = n when n = an
integer
x 0

= lim cos 0 h = lim cos h


h 0

h 0

a f

= lim cos 1
h0

= cos 1

lim3

k
= lim k h + 1p

(iii)

h0

h 0

h>0

x 0+

= lim cos1

3
<2
1 + 3h

RSx + 1 Lx + 1 OUV
T 4 MN 4 PQW
RF 3 I 1 LF 3 I 1 OU
lim S k + h + M k + h + P V,
TH 4 K 4 NH 4 K 4 QW

(viii)

(ii) lim e

l.h.l = lim cos x

LM 3 OP = 3, as h 0 and h > 0 1+ 3h > 1


N1+1 3h Q 3

3 [k + 1 h] = k as [n h] = n 1 when n = an
integer

6. Evaluate each of the following if it exists.

R|
LM
OU
1
1 P|
= lim S
|T 13 h MMN 13 h PPQV|W , h > 0

=1

3 [h] = 1 as [n h] = n 1 when n
= an integer

r .h.l l. h. l and so the limit does not exist.


(ii) lim+ e

x0

= lim e

0+ h

h 0

= lim e

h 0

h0

= lim 1 = 1
h 0

= lim e

Practical Methods of Finding the Limits

(iii)

lim cos x = lim cos

x 4 +

h 0

LM + hOP
N4 Q

= lim cos 0
h0

= lim 1
h 0

LM + hOP = 0 as 0 < + h < 1


N4 Q
4

=1
Note: In those problems which are the combination
or composition of functions whose one function is
the greatest integer function, we should consider two
necessary facts while finding their limits as h 0
(i) find the greatest integer contained in [the function
of h] before putting h = 0
(ii) cancel the highest common power of h from the
numerator and denominator in the quotient of two
functions of h before putting h = 0
How to find the limit of a piecewise functions
containing greatest integer function
When the limit of a piecewise function containing at
any integral point x = a is sought, it must be redefined
in adjacent intervals whose left and right end points
are the same namely the integral point x = a.
How to find the adjacent intervals containing the
integral point x = a
Step 1: Start towards left from the integral point x = a
and stop at the first integer you arrive at (say b), i.e.,
obtain b < x < a where b = the integer just on the left
of the integral point x = a.
Step 2: Start towards right from the integral point
x = a and stop at the first integer you arrive at (say c),
i.e. obtain a < x < c, where c = the integer just on the
right of the integral point x = a.

x=b

x=a

x=c

Rule: To find the limit of a piecewise function


containing greatest integer function and redefined in
adjacent intervals [b, a) [a, c) is determined by
removing the symbol [ ] with the help of the definition.
[f (x)] = n, when n < f (x) < n + 1
[f (x)] = n 1, when n 1 < f (x) < n where n I

267

and lastly put x = a in each different forms of the


expression free from greatest integer function.
Examples worked out:
1. If f (x) =

sin x
, x 0
x

= 0, [x] = 0

af

then lim f x =
x0

(a) 1 (b) 0 (c) 1 (d) none of them.


Solution: On redefining f, it is as under

af

f x =

sin x
, 1 x < 0
x

= 0, 0 x < 1
Again on using the definition for [x],
[x] = 1 when 1 < x < 0
[x] = 0 when 0 < x < 1
the function f becomes

af

f x =

a f
a f

sin 1
, when 1 < x < 0, i.e.
1

af

sin 1
= sin 1 for 1 < x < 0 and f (x) = 0,
1
when 0 < x < 1
f x =

bg

Hence, lim f x = lim sin 1 = sin 1


x0

bg

x 0

and lim+ f x = lim+ 0 = 0


x0

x0

af

af

Since lim f x lim+ f x


x0

x 0

the limit of f (x) at x = 0 does not exist


Hence, (d) is true.
2. Examine the existence of limit of a function defined

af

af

by f x =

x 1
x = 1.
Solution: It is given
f x =

2
2

x 1

, for x 1 = 0, for x2 = 1; at

, for x 1

268

How to Learn Calculus of One Variable

= 0, for x2 = 1
On redefining the given function f,

af

f x =

x 1

, when 0 < x2 < 1

e z j = z 11 , for 0 z < 1

11
= 0 , for 1 < z < 2
z 1
= 0, for z = 1

x 1
2

, when 1 < x 2 < 2

= 0, when x2 = 1
Again on using the definition for [x2],
[x2] = 0 for 0 < x2 < 1
[x2] = 1 for 1 < x2 < 2
the function f becomes

af

f x =

1
2

x 1

11
2

x 1

, for 0 < x2 < 1

= 0, for 1 < x2 < 2

bg

z 1

z 1

Type 4: Problems based on finding the value of a


constant whenever the limit of a given function is
finite value/a finite number.
Examples worked out:

af

x 1

and lim f x = lim


x 1

z 1

x 1

Now, lim+ f x = lim+ 0 = 0


x 1

z 1+

z 1

z 1

= 0, for x2 = 1

e z j = lim 0 = 0
F 1 I =
but lim f e z j = lim G
H z 1JK
lim f e z j does not exist
lim f e z j does not exist
lim f a x f does not exist
Now, lim+ f

x 1

F 1 I =
GH x 1JK
2

af
lim f a x f does not exist.

i.e. lim f x does not exist


x 1

x 1

alternative method

1. Find the value of k if

lim kx 5 = 3

x2

Solution: lim (kx 5) = k 2 5 = 2k 5

(i)

and lim (kx 5) = 3

(ii)

x 2

x 2

Equating (i) and (ii), we have, 2k 5 = 3

2k = 8 k =

8
=4
2

x 2 = z x 2 = z x = z x = z for x 0

2. Find the value of K if lim (3x 2) = 7

Also, x 1 z 1
Hence the given function becomes

Solution: lim 3x 2 = 3k 2

e zj =

z 1
z 1

, for z 1

= 0 for z = 1
Which can be redefined as under:

x k

and lim 3 x 2 = 7
xk

xk

(i)
(ii)

(1) and (2) 3k 2 = 7 3k = 7 + 2 = 9 k =


9/3 = 3

269

Practical Methods of Finding the Limits

Problems based on existence of the limits of greatest


integer function/combination of a function and the
greatest integer function

2. (i) exists and lim f (x) = 1 (ii) does not exist since
x 1

l.h.l = 1 and r.h.l = 0 (iii) exists and lim f (x) =


x 1

Exercise 4.30.1
1. Examine the existence of the limits of the following
functions as x 0
(i) f (x) = [x] + [1 x], real x
(ii) f (x) = x + [x], real x
(iii) f (x) = [x] + [x], when x 0
f (0) = 1
(iv) f (x) = x [x]
2. Examine the existence of the limits of the following
functions at the indicated points.
(i) f (x) = [1 x] + [x 1] at x = 1
(ii) f (x) = [x + 2] | 2 + x | at x = 2

LM 1 + x OP 1
N 2 Q 2 at x = 1
(iii) f (x) =
2

Exercise 4.30.2
Find the value of k if

Answers

1. lim kx 5 = 3

(4)

x2

sin x
, x 0 ; = 0, [x] = 0 then find if
x

x2 k 2
= 4
x2 x k

(2)

4. lim

sin kx
=3
x

(3)

5. lim

kx 2 + 4 x 8
=3
2 x 2 3x + 5

(6)

b
g
lim a3 x 2 f = 7

6. lim k + 2 cos x = 5

(5)

7.

(3)

x 2

xk

af

8. If f x =

sin 3 x
tan b x
, when x < 0 =
, when
x
x

af

x > 0 and lim f x exists, find the value of b.


x0

lim f (x) exists where [x] denotes the greatest integer

x 0

Solution: lim

x0

less than or equal to x.


Answers:

af

1. (i) exists and lim f x = 0 (ii) does not exist since


x 0

l.h.l = 1 and r.h.l = 0 (iii) exists and lim f (x) = 1


x 0

(iv) exists and lim f (x) = 0


x 0

(6)

3. lim kx 2 + 5x 3 = 4

3. Show that the function f defined by


f (x) = [x 1] + | x 1 | for x 1
f (1) = 0 has no limit at x = 1.
4. Show that the function f defined by f (x) = [x 3] +
[3 x], where [t] denotes the largest integer < t exists
at x = 3 and is equal to 0.

2. lim

af

Problems based on finding the value of a constant

x 0

LM x + 1 OP
1
2Q
(iv) f (x) = N
at x =

5. If f x =

does not exist since l.h.l = 1 and r.h.l = 0.


5. As l.h.l r.h.l, so the given limit does not exist.
Hint: l.h.l = sin 1 and r.h.l = 0

x 1

1
(iv)
2

l.h.l = lim

x0

lim

x0

sin 3x
x

sin 3 x
3=3
3x

(i)

tan b x
tan b x
= lim
b = b = r.h.l (ii)
x

0
x
bx

270

How to Learn Calculus of One Variable

af

lim f x exists l . h. l = r . h .l b = 3 Ans.

x0

(vi) The value of lim

Objective problems

(A) 0 (B) (C) 3 (D) 5

Exercise 4.30.3

2x
is
(i) The value of lim
x 0 5 + 3x
2
2
(A)
(B)
(C) 0 (D) 2
3
5

(i) lim x = 0

(iii) The value of lim

x 0

(iv) The value of lim

x0

(v) The value of lim

x0

(A) 1 (B) 2 (C)

[Ans. B]

[Ans. T]

(iii)

[Ans. T]

x 2

(iv) lim

2 x 3x
=1
3x

[Ans. F]

(v) lim

x
=1
tan x

[Ans. T]

x0

x 0

[Ans. D]

(vi) lim

x 2

x 4
=4
x+2

[Ans. F]

[Ans. A]

x x 1
=
(vii) lim
x
2x
2

[Ans. F]

(viii) lim

2x
is
sin x
1
(D) not possible
2

f
lim a1 sin x f = 0

sin 5 x sin 3x
is
x

2
(A) 2 (B) 0 (C) 1 (D)
15

(ii) lim 1 + cos x = 0


[Ans. C]

sin 5x
is
x

1
(D) 5
5

[Ans. T]

x0

sin x
is
(ii) The value of lim
x 2 1 + cos x
1
(A) 0 (B) 1 (C)
(D)
2

[Ans. B]

(b) State whether the following statements are true


or false

(a) Choose the correct answer.

(A) 1 (B) 0 (C)

3x 5
is
x

[Ans. B]

2 x + 5x
=2
x x+1

[Ans. T]

Practical Methods on Continuity test

271

5
Practical Methods on
Continuity Test

A little more on how to test the continuity of a


function at x = a. To test whether f (x) is continuous at
x = the following procedure may be adopted:
1. Find f (a). If f (a) is undefined, the function f (x) is
discontinuous at x = a.
2. If the value of the function represented by f (a) at
x = a has a finite value, find the l.h.l and r.h.l

af
a f and
f a x f = lim f aa + hf respectively where

represented by lim f x = lim f a h


xa

lim+

xa

h 0

h 0

h 0 through positive values (i.e; h > 0 and h < 0).


3. If both l.h.l and r.h.l are equal to f (a), f (x) is
continuous at x = a.
4. If either of the l.h.l and r.h.l is different from f (a),
f (x) is discontinuous at x = a.
Explanation
1. Replace x by a f (x) and in the given expression
in x and find the value of the given function f (x) at
x = a. Now if the value of the function f (x) at x = a is
undefined the given function is declared to be
discontinuous.
2. If the value of the function f (x) at x = a is a
finite value, we are required to find l.h.l and r.h.l
respectively by the method already explained.
3. If all the three (a) value of the function at x = a,
i.e; f (a) (b) l.h.l and (c) r.h.l of the given function
obtained are equal, then f (x) is continuous at x = a

which means f (a) = l.h.l = r.h.l f (x) is continuous


at x = a.
4. If f a l .h.l r .h.l or l .h.l r .h.l =
f (a) or r .h.l l .h.l = f (a), we declare that the given
function f (x) whose test of continuity is required is
discontinuous at the given point x = a.

af

N.B.: The above method of testing a given function


to be continuous at x = a is applied when the given
function is defined by different equations on imposing
the conditions on the independent variables x by < or
< or > or > or = etc.
Aid to Memory

af

1. f a = lim [the function f1 (x) opposite to which


xa

x < a / x < a / c < x < a /]


= lim [the function f2 (x) opposite to which
xa

x > a / x > a / c > x > a / c > x > a]


f (x) is continuous at x = a.
2. f (x) = f3 (x), when x = a means we should consider
the function f3 (x) (i.e; an expression in x) to find out
the value of the function x = a.
3. f (x) = a constant, when x = a means we are
provided the value of the function which is the given
constant for the independent variable x = a and thus
further we are not required to find out the value of the
function f (x) at x = a.

272

How to Learn Calculus of One Variable

4. f (x) = f1 (x), when (or, if or, provided) x a or


c > x a means we should consider f1 (x) for finding
r.h.l and the value of the function at x = a.
5. f (x) = f2 (x), when x a or c < x a means we
should consider f2 (x) for finding l.h.l and the value of
the function at x = a.
6. f (x) = f1 (x) when x < a or c < x < a we should
consider f1 (x) only for finding the l.h.l and not for the
value of the function f (x) at x = a as the sign of
equality does not appear in the given restrictions
against the given function f (x) = f1 (x).
7. f (x) = f2 (x) when x > a or c > x > a means we should
consider f2 (x) only for finding r.h.l and not for the
value of the function f (x) at x = a as the sign of
equality does not appear in the given restriction
against the given function f (x) = f2 (x).
8. f (x) = f4 (x), when x a means the same function
f4 (x) should be considered for finding the l.h.l and
r.h.l.
Note:
1. In the light of above explanation, we may declare
that the sign of equality = with the sign of
inequalities > or < retains the possibility to consider
the same function for the l.h.l as well as the value of
the function both or r.h.l as well as the value of the
function both whereas only the sign of inequality >
or < excludes the possibility to consider the same
function for the value of the given function opposite
to which > or < is written.
2. f (x) = f4 (x), when x a means there is no need to
find out l.h.l and r.h.l separately but only to find out
the limit of f4 (x) at x = a and use the definition limit =
value of the function at the given point x = a to test
the continuity (or, to find the l.h.l and r.h.l by putting
x = a h as h 0 through positive values in the
given function f (x) = f4 (x) and then use the definition
l.h.l = r.h.l = value of the function (which is given at x
= a by imposing the condition x = a against the given
function f (x) as f (x) = a constant, when x = a)
continuity of the given function at x = a.
3. A function f (x) defined in an interval is called a
piecewise continuous function when the interval can
be divided into a finite number of non-overlapping
open sub intervals over each of which the function is
continuous.

4. All the points at which the function is continuous


are called points of continuities and all those points
at which the function is discontinuous are called
points of discontinuities (or, simply discontinuities
only).
5. f (x) is continuous at x = a x = a is the point of
continuity of f (x) f (x) has a point of continuity
namely x = a.
6. f (x) is discontinuous at x = a x = a is the point
of discontinuity of f (x) f (x) has a point of
discontinuity namely x = a.
A Highlight on Removal Discontinuity
Question: When a function is not defined for x = a, is
it possible to give the function such a value for x = a
as to satisfy the condition of continuity?
Answer: When a function is not defined for the
independent variable x = a as to satisfy the condition
of continuity if we arbitrarily suppose that value of
the function which must be the limit of the given
function If the value of the function at a point =
limit of the function at the same point is supposed,
then the given function becomes continuous at that
point.
2

Example: y =

x 9
is not defined at x = 3 but for
x3

any other value of x,

a x + 3f a x 3f = (x + 3)
a x 3f
and lim a x + 3f = 6
F x 9I = 6
lim G
H x 3 JK
y=

x3

x3

Now, if we suppose f (3) = 6, i.e; the value of the


function to be 6 for x = 3, the function becomes
continuous.
Removal Discontinuity
If

bg

bg bg

lim f x = lim f x f a ,

xa+

x a

then the

function f (x) as said to have removal discontinuity at

Practical Methods on Continuity test

x = a because the discontinuity can be removed by


making the value of the function f (x) at x = a equal to

y =

af

lim f x .

xa

x3 x+3
x2 9
=
, x3
x3
x3

= (x + 3)

There are two types of discontinuities which can


be removed by assuming the value of the function
f (x) at a point (or, number) x = a = limit of the function
f (x) at a point (or, number) x = a.

gb

273

af

af

af

Thus, we see that f 3 lim f x f x is


x3

discontinuous at x = a.
2

1. If the function is not defined at x = a, then the


function is discontinuous at x = a.

af
function f aa f lim f a x f , then the function f (x) is

2. If lim f x exists finitely but the value of the


xa

xa

discontinuous at x = a. Thus, there are two types of


discontinuities which can be removed by assuming
value = limit for the given point x = a.
Remember:
(A) A function is discontinuous at x = a if

af

x 9
is continuous at x = 2
x3

F x 9 I = 5 and f (2) = 5
GH x 3 JK
f a2f = lim f a x f = 5 .
lim b5x g
F
5x I
=
lim
2.
GH 6 2 x JK lim b6 2 xg =
2

Since lim

x2

x2

x3

x3

1. f (x) is not defined at x = a f (x) = meaningless


at x = a f a =

N.B.: But y =

x3

bg

0
/ / 0 ... etc.
0

f x =

af
f a x f f aa f

5x
is discontinuous at x = 3.
6 2x

2. When lim f x =

Now, we consider the continuity of the following


functions at the point x = a.

3. When lim

1.
2.
3.
4.
5.

x a
xa

4.
5.
6.
7.

When l .h.l r .h.l = f (a)


When l .h. l = r . h. l f a
When r . h. l l . h. l f a
Limit value of the function at the given point.

af
af

(B) A function f (x) is said not to exist at a point x = a

af

if f x

x=a

= /meaningless/imaginary.

Continuity of rational functions.


Continuity of absolute value functions.
Continuity of exponential functions.
Continuity of logarithmic functions.
Continuity of trigonometric functions.

Type 1: When no condition is imposed on the


independent variable against the defined function,
i.e; when given function is not piecewise.
Highlight on the Working Rule

Examples:
2

1. Show that y =

x 9
is discontinuous at x = a.
x3

Solution: lim y = lim x + 3 = 6


x3

bg

3y= f x =

x3

...(1)

x 9 0
= at x = 3 (undefined)
0
x3
2

... (2)

Limit of the given function as x a = value of the


given function at x = a, in the examples to follow.
Solved Examples
Test the continuity of the following functions at the
indicated points.
1. y = x2 + 3x at x = 2
Solution: y = x2 + 3

274

How to Learn Calculus of One Variable

lim y = lim x + 3x
x2

x2

If l1 = l2 = c then f (x) is said to be continuous at


x = a.

= 22 + 3 2 = 4 + 6 = 10
f (2) = 10 when f (x) = x2 + 3x

Solved Examples

f 2 = lim f x

1. If f (x) = 5x 4, when 0 < x 1 = 4x3 3x, when


1 < x < 2 Show that f (x) is continuous at x = 1

af

x2

af

Hence, f (x) is continuous at x = 2.


x 9
2. y =
for x = 2
x3

x 1

x 1

x2

fa

x+3 x3
x 9
= lim
x2
x3
x3

x2

= lim x + 3 = 2 + 3 = 5
x2

a f 42 93 = 51 = 5
f a2f = lim f a x f
f 2 =

Type 2: In case, f (x) = f1 (x), when x < a


= f2 (x), when x > a
= c, when x = a
i.e; when different functions are provided with
different restrictions imposed on the independent
variable x as x > a / x < a / x = a / x a / x a / etc
.against each function or in otherwords, when the
given function is a piecewise function, i.e. the given
function is defined adjacent intervals.
Highlight on the Working Rule

bg

lim+ f x = lim f 1 x = l1 , where f1 (x) is a form

xa

xa

of the given function defined in an interval whose


right end point is a.

bg

bg

lim f x = lim f 2 x = l2 , where f2 (x) is a

xa+

xa

af

x 1

form of the given function defined in an interval whose


left and point is also, a.

x 1

=43=1

=54=1
f (1) = 5 4 for f (x) = 5x 4 when 0 < x 1

af

af

af

af

Hence, f 1 = lim f x = lim f x f x is


x 1+

x 1

continuous at x = 1.

af

x
, when 0 x 1 = 2 x 2 3x + 3 ,
2
2
when 1 x 2 test the continuity of f (x) at the
point x = 1.

2. If f x =

x2

Hence, f (x) is continuous at x = 2

bg

lim f x = lim 5x 4

x 9
Solution: y =
x3
lim y = lim

af

Solution: lim+ f x = lim 4 x 3 x

af
a f FH

1
x
Solution: lim f x = lim
=
x 1 0
x 1 2
2
3
2
lim f x = lim 2 x 3x +
x 1+ 0
x 1
2
3
3 1
= 2 3 + = 1 + =
2
2 2

af

f x =

I
K

af

x
for 0 x 1 f 1 = 2
2

af

af

af

af

Hence, f 1 = lim f x = lim f x f x


x 1+ 0

x 1 0

is continuous at x = 1.

af

x
, when 0 x 1
Note: If f x =
2

3
, when 1 x 2
2
then to find the value of the function f (x) at x = 1, we
may consider any one of the two pieces. Hence, in
the above example if we consider
2

= 2 x 3x +

Practical Methods on Continuity test

af

af

f a f x is continuous at x = a
5. Show that the function for f (x) defined by
1
1
f (x) = x + , when 0 < x <
2
2
1
1
= , when x =
2
2

3. Test the continuity of the function f (x) at x = 2


f (x) = 2x + 1 for x 2
= x2 1 for x > 2

at x =

Solution: l .h .l = lim

Solution: f

x20

r . h. l = lim

x 2 + 0

ex

af

F
H

1 = 4 1= 3

af

lim

, x > a is continuous at x = a.
2
x
Solution: f (a) = 0 [ 3 f (x) = 0 when x = a is given in
the problem]
(1)

af

F
GH

lim+ f x = lim a

xa

=a

a
a

af

xa

I
JK

=aa=0

Fx
GH a

xa

1
2

(2)

1
2

1 1
+ =1
2 2

(3)
1
x +
2

af

1
x
2

af

F 1 I f (x) is discontinuous at x = 1 .
H 2K
2

N.B.: In fact, the in-equality of any two of (1), (2) and


(3) ensures discontinuity of the given function at
(2)

I
JK

x=

1
.
2

6. Test the continuity of the function f (x) at x = 2

af

f x =

a
=
a=aa=0
a

(1)

(1), (2) and (3) lim f x = lim f x

lim f x = lim

x a

F 1I = 1
H 2K 2
F 1I
f a x f = lim x
H 2K

1
x
2

I
K

1
1
, when < x < 1 is discontinuous
2
2

F 1 1I = 0
H 2 2K
F 1I
lim f a x f = lim x +
H 2K

x
a,0<x<a
a
= 0, x = a

=a

x a

f x =

1
2

1
x +
2

f (x) is discontinuous at x = 2.
4. Show that the function defined as

af

x a+

af

= x

l .h . l l . h. l lim f x does not exist


x2

af

(1), (2) and (3) lim f x = lim f x =

3
for 0 x 1 , then f (1) =
2
3
3 2 + 3 1 so, in general,
2 3 + = 1 + =
=
2
2
2
2
when the condition (or, restriction) imposed on an
independent variable x contains x a / x a /
c > x a / c < x a , then we may consider any one
of both functions to find the value of the function at
x = a because both functions provided us the same
value.
2

f x = 2 x 3x +

a2 x + 1f = 5

275

(3)

x 4
, when 0 < x < 2
x2

f (x) = x + 2, when 2 x 5

276

How to Learn Calculus of One Variable

x 4
af
x 2
= lim a x + 2 f = 2 + 2 = 4
f a x f = lim a x + 2f = 2 + 2 = 4
2

Solution: l . h . l = lim f x = lim

x 2

x 2

x2

r . h. l = lim+

x2

x2

f (2) = (x + 2)x = 2 = 2 + 2 = 4
Hence, l.h.l = r.h.l = f (2) f (x) is continuous at
x = 2.
7. Test the continuity of the given function at x = 1
f (x) = 2x + 3, when x 1
(1)
= 8 3x, when 1 < x < 2
(2)
Solution: f (1) = 2 1 + 3 = 2 + 3 = 5

af a f
f a x f = lim a2 x + 3f = 2 + 3 = 5

(1)
(2)

lim

(3)

x 1

x 1
x 1

(1), (2) and (3) f (x) is continuous at x = 1.


8. Test the continuity of the given function f (x) at
x=2

af

f x =

x 4
,
x 2 when 0 < x < 2

af

x2+

x2

bg
F 9 x IJ
f a1f = G
H x + 2K

(1)

r . h. l = lim+ f x = lim

x 1

x 1

x + 2 1+ 2
=
=3
x
1

91 9
= =3
1+ 2 3

x 4
x2

f
f a x f = lim a x + 1f

(1)

(3)

af

x 1

af

x 1

(2)

=1+2=3
(3)
(1), (2) and (3) f (x) is continuous at x = 1
Now, we test the continuity at x = 3
f (3) = [f (x)]x = 3 = (4x 1)x = 3
= 4 3 1 = 12 1 = 11
(1)

l .h .l = lim 4 x 1
x 3

x 2

(2)
x 2

exist f (x) is discontinuous at x = 2.


9. Test the continuity of the given function f (x) at
x = 1.

9x
,
x + 2 when 0 < x 1

(2)

(1), (2) and (3) l.h.l = r.h.l = f (1) f (x) is


continuous at x = 1.
10. Test the continuity of the given function f (x) at
x = 1 and x = 3 f (x) = x + 2, when x < 1
= 4x 1, when 1 x 3
= x2 + 5, when x > 3
Solution: f (x) = [f (x)]x = 1 = (4x 1)x = 1
=411=3
(1)

x 1

(1) and (2) l . h. l r . h. l lim f (x) does not

af

9x
x+2

9
9
= =3
1+ 2 3

lim f x = lim x + 2

=2+1=3

f x =

=411=3

= lim x + 2 = 2 + 2 = 4
r . h. l = lim

x 1

x 1

lim f x = lim 4 x 1

Solution: l .h . l = lim f x = lim

x2

af

Solution: l .h. l = lim f x = lim

x 1+

f (x) = x + 1, when 2 x 5

x 2

x+2
, when 1 < x 2
x

x =1

lim f x = lim 8 3 x = 8 3 = 5

x 1+

= 4 3 1 = 12 1 = 11

r . h. l = lim

x 3+

ex

(2)

+5

= 32 + 5 = 9 + 5 = 14
(3)
(2) and (3) r .h.l l. h.l f (x) is discontinuous at x = 3.
11. Test the continuity of the given function f (x) at
x = 0, 1 defined as f (x) = 2, when x 0 .

Practical Methods on Continuity test

= 3x + 2, when 0 < x 1

Or, in the notational form,


x a+

bg
b g
f b x g = lim b2g = 2

r . h. l = lim f x = lim 3 x + 2 = 2
x 0

x0

x 0

f (0) = [f (x)]x = 0 = 2
(1), (2) and (3)

af

(1)
(2)
(3)

af

af

f 0 = lim f x = lim f x
x 0+

x 0

f (x) is continuous at x = 1
(b) Test of continuity at x = 1
lim

x 1+ 0

af

f x = lim f 1 + h , h > 0
h 0

= lim

h 0

1+ h
1+h
= lim
= +
1 + h 1 h 0 h

f
lim f a x f does not exist
f 1+ 0 = +
x 1

Hence, f (x) is not continuous at x = 1


f (x) has a discontinuity of second kind at x = 1.
Type 3: When a function is defined as
f (x) = f1 (x), when x a
= a constant c (say), when x a
Remember:
1. A point of removable discontinuity/removable
discontinuity: If the limits of a function from the right
and left exists and are equal but is not equal to the
value of the function at a point, then the function is
said to have (or, contain) a point of removable
discontinuity (or, simply a removable discontinuity)
at the considered point.
Or, more explicitly,
A point of discontinuity (or, simply a discontinuity)
namely x = a is called removable discontinuity if the
limit of the function exists but the function either is
not defined at x = a or has a value different from the

af af

limit x = a (i.e; lim f x f a )


xa

xa

af

f (x) is said to have (or, to contain) a point of removable


discontinuity (or, simply a removable discontinuity)
namely x = a (or, at x = a), e.g:

Solution: (a) Continuity test at x = 0

l .h . l = lim

af

If at x = a, lim f x = lim f x f a , then

x
,
x 1 when x > 1

x 0+

af

277

af

x + x
for x 0
x
= undefined, for x = 0
(ii) f (x) = | x |
f (0) = 3 are the functions having removable
discontinuity at x = 0.
2. A function having removable discontinuity at a
point x = a can be made continuous by giving the
function a new value c equal to the limit of the
function at the point x = a.
3. A discontinuity is called a removable discontinuity
because it can be removed from the function and the
function becomes continuous whereas a nonremovable discontinuity is any discontinuity which
is not removable.
4. A function f (x) having (or, containing) removable
discontinuity is also said to be redefined at x = a if the
function f (x) is made continuous by assuming (or,
setting) the value of the function f (x) at x = a to be
equal to the limit of the function at x = a.
5. Jump discontinuity: A function f (x) is said to have
a jump discontinuity at x = a if the left hand limit and
right hand limit of the function f (x) at x = a exist and
are finite but are not equal.
Or, in the notational form,

(i) f x =

If

af

lim f x = L1 and

x a

af

lim f x = L2 but

xa+

L1 L2 , then we say that the function f (x) has a


jump discontinuity namely x = a or we say that f (x)
has a jump discontinuity at x = a or a discontinuity of
the first kind (or, a point of jump discontinuity or, a
point of discontinuity of the first kind) at x = a. e.g:,

(i) f (x) = 1, when x > 0


= 1, when x < 0
is a function having a jump discontinuity (or,
discontinuity of the first kind) at x = 0 or we can say
x = 0 is a point of jump discontinuity or a point of
discontinuity of the first kind of f (x).

278

How to Learn Calculus of One Variable

6. A function f (x) is said to have a discontinuity of


the second kind at a point x = a if at least one of one
sided limit fails to exist at the considered point x = a.
Or, in the notational form,

af

af

If at least one of the limits lim f x / lim f x


xa

x a+

does not exist, we say that the function f (x) has a


discontinuity of the second kind at x = a. e.g.,

af

(i) f x = sin

F 1I , x 0
H xK

(l.h.l = r.h.l at x = a) = lim [the function opposite


xa

which x a is written] and see whether l.h.l = r.h.l =


lim [the function written before x a ]

af af

N.B.:
1. f (a + 0) and f (a 0) are the notations used for l.h.l
and r.h.l at x = a.
2. x a either x > a or x < a x a

1. Show that the function f (x) is discontinuous at


x=1
f (x) = x2, when x 1
= 2, when x = 1
Solution: (l.h.l at x = 1)
2

= lim f x = lim x = 1

(r.h.l at x = 1)

af

x 1

= lim f x = lim x = 1
x 1+

f (1) = 2 (given)

x 1

,x 1

F x 4 x + 3I
GH x 1 JK
e x 3x x + 3j
= lim
ax + 1f ax 1f
x a x 3f a x 3f
= lim
a x + 1f a x 1f
a x 3f a x 1f
= lim
a x + 1f a x 1f
a x 3f
= lim
a x + 1f
2

= lim

x 1

x 1

x 1

1 3 2
=
= 1
1+1
2

f (1) = 2 (given)
Hence, (l.h.l = r.h.l at x = 1) f 1 f x is
discontinuous at x = 1.

af

Solved Examples

af

x 4x + 3

x 1

= lim f 1 x = f a = given value = c

x 1

af

f x =

x 1

Highlight on the Working Rule

xa

x 1

af

= 2, x = 1
Solution: (l.h.l = r.h.l at x = 1)

f (0) = 0
is the function having a discontinuity of the second
kind at x = 0 or we can say x = 0 is a point of
discontinuity of the second king of f (x).

x a

af

(i), (ii) and (iii) l . h.l = r . h. l f 1 f x is


discontinuous at x = 1.
2. Test the continuity of the function f (x) at x = 1
defined by

(i)

af

3. If f x =

(iii)

x 3x + 2
2

x 4x + 3

,x 1

= 2, x = 1 show that given function is discontinuous


at x = 1.
Solution: (l.h.l = r.h.l at x = 1)

Fx
GH x
Fx
= lim G
Hx
= lim

(ii)

af

x 1

x 1

2
2

2
2

I
J
4 x + 3K
3x + 2

I
J
x 3x + 3 K

x 2x + 2

279

Practical Methods on Continuity test

a x 1f a x 2f
a x 1f a x 3f
a x 2f
= lim
a x 3f

5. Test the continuity of the given function at x = 1

= lim

af

x 1

1 2 1 1
=
=
1 3 2 2

af

bg

4. If f x =

x0

= lim

x0

= lim

x0

= lim

x0

= lim

x0

F
GH
x

4+x
x

1
, x 0 and f 0 = ,
2

4x

a4 + x 4 + x f
4+ x +

40

4+ x+

x 1

a x 1f e x

= lim

ax 1f e x

= lim

ex

je

1
1
=
32+2
12

2
1
=
2+2 2

Thus, l.h.l = r.h.l = f (0) =

je

+1+1

x+3+2

1+ 3 + 2

f (1) = 2 (given)

af

Thus, l.h.l = r.h.l f 1 discontinuity of the


function at x = 1.
6. Test the continuity of the function f (x) at x = 1
2

f x =

1
continuity of the
2

x+3+2

af

af

je

+ x +1

+ x +1

x+3+2

je

+ x +1

x1

4 x

I
J
x + 3 + 2K
x+3 +2

x + 3 4

= lim

x 1

e1

1
f 0 = (given)
2

function f (x) at x = 0.

x+32

4+x + 4x

4+0 +

I
J
4 xK

F
GH

x 1

x 1

4 x

2x
4+ x +

= lim

x 1

4+ x+ 4 x

x 1

x 1

4+ x 4 x

af

bg

4x

x
test the continuity at x = 0.
Solution: l.h.l = r.h.l at x = 0
= lim

x+32

= lim

f (1) = 2 (given)
Hence, (l.h.l = r.h.l at x = 1) f 1 f x is
discontinuous at x = 1.

4+ x

,x 1

x 1

=2, x = 1
Solution: f (1 0) = f (1 + 0)

x 1

x+32

f x =

x +1
x1

,x1

,x=1
2
Solution: f (1 0) = f (1 + 0)

280

How to Learn Calculus of One Variable

x +1
x 1

= lim

x 1

F
= lim G
GH

x0

x +1

x1

x 1

x +1+
2

x +1+

I
JJ
2K
2

x +1 2

= lim

x 1

ax 1f FGH

F
GH

= lim

x +1+

IJ
K

= lim

x0

= lim

x0

= lim

x0

x+3
x

x +33

x+3+

x+3+
1
x+3+ 3

x+3+
x+3+

I
J
3K
3

x 1

ax 1f FGH x + 1 + 2 IJK
ax + 1f
= lim
FG x + 1 + 2 IJ
H
K
= lim

x 1

x 1

af

2 +

1
2 3

(b) The discontinuity is removable if we define the


function as follows

x+3
x

f x =

1+1

3+ 3

,x0

,x=0

2 3

8. A function is defined as under

2
2 2

af

f 1 =

af

1
2

(given)

af

af

Hence, f (1 0) = f (1 + 0) = f 1 f x is
continuous at x = 1
7. Show that the function f defined as

af

x+3 3
is discontinuous at x = 0, and
x
then determine if the discontinuity is removable.
Solution: (a) (i) f or f (x) is not defined at x = 0 as
f x =

bg

0
(undefined)
0
f (x) has discontinuity at x = 0.

f x

x =0

af

(ii) lim f x = lim


x0

x 0

f x =

F
GH

x+3
x

x x6
2

x 2x 3

, at x 3

5
, at x = 3
3
show that f (x) is discontinuous at x = 3
Solution: lim f x = lim f x
=

af

x 3+

x 3 0

F x x6I
= lim G
H x 2 x 3JK
a x 3f a x + 2f
= lim
a x 3f a x + 1f
2

x3

x3

I
JK

= lim

x3

x +2 3+2 5
=
=
x +1 3+1 4

af

Practical Methods on Continuity test

af

f 3 =

5
(given)
3

bg

3.

bg

bg

at x = 3.
Type 4: Problems based on continuity of an absolute
value function (or, modulus function)
Firstly, we recall the definition of absolute value
function.
Definition: An absolute value function is a function
defined on the domain of all real numbers such that
with any number x in the domain, the function
associates algebraically the non-negative number
2
x , which is designated by writing two vertical
lines around x as | x |. Therefore the value of the
function at x is

x =

= x, when x < 0
2

4.

x =

5.

xa =

ax af

x = x = x , when x < 0
which means an absolute value function (or, modulus
function or, simply mod function) maps every positive
real number onto its positive number, zero onto zero
and every negative real number onto its negative,
which is the corresponding positive number. Thus | x
| is never negative. Hence, an absolute value function
represented by the symbol | | designated by writing
two vertical lines is like an electric current rectifier
that converts either positive or negative current into
positive. Further we should note that the domain of
an absolute value function is the set of all real numbers
and the range is the set of all positive real number
including zero which can be written in the following
notational form.

Function

Image

Domain

Range

||

|x|

R 0

af

kp

1. | f (x) | = f (x), when f x 0


= f (x), when f (x) < 0
2. | x a | = (x a), when (x a) 0 x a ;
= a x, when (x a) < 0 x < a

6. | x | = | x | = x, when x is positive

bg

7. lim x = c , for any real c lim f x


xc

xc

bg

lim f x

xc

8.

af

f x

b f a xfg

9. x 0 x > 0 and x < 0


10. | ( | f (x) | ) | = mod of a mod function f (x) = | f (x)| =

= x , when x 0

Remember:

x = x , when x 0

x =

lim f x f 3 f x has discontinuity


x 3

281

mod of f (x) since

x = x .

Method to test the continuity of an absolute value


function
To test the continuity of an absolute value function,
we may adopt the following working rule.
(Note: (i) | x | is read as modulus of x / mod of x/simply
mod. x. (ii) R + 0 means a set of all positive real

kp

numbers including zero).


Working rule:
1. Convert the given mod problem in f (x) and f (x)
on imposing the condition (or, restriction) which
becomes the problem for finding the l.h.l and r.h.l by
using the mod function definition when we are
required to find l.h.l, we should put | f (x) | = f (x)
which involves no mod symbol as well as we should
put | f (x) | = +f (x) which involves no mod symbol | |.
2. Find the l.h.l and r.h.l.
3. See whether l.h.l and r.h.l are equal or not.
4. If l.h.l = r.h.l, then we declare the existance of limits
for the given mod function at the given point x = a.
5. Find the value of the function at the given point
x = a.
6. If l.h.l = r.h.l = value of the given function at a
given point x = a then we declare that continuity of
the given mod function | f (x) | holds at the given point

282

How to Learn Calculus of One Variable

x = a and if l .h.l r . h. l , we declare that


discontinuity of the given mod function | f (x) | holds
at the given point x = a.

x0

r . h. l = lim e

1. Discuss the continuity of the function f (x) at x = 8

af

x 8

a f

x8
= lim 1 = 1
x 8
x8

a x 8f = lim 1 = 1
r . h. l = lim
x8

x8

Thus, l .h.l r .h.l given function is


discontinuity at x = 8 or, alternatively, the function

af

x8
is not defined at x = 8 and so
x8
discontinuity at x = 8.
2. Discuss the continuity of the function f (x) at x = 0
f x =

x x+1
for x 0 and f (0) = 1.
f x =
x

Solution: l .h.l = lim

x0

x < 0)

x x +1
( 3 | x | = x when
x

la

fq a f
x b x + 1g
( 3 | x | = x when x > 0)
r . h. l = lim
= lim x + 1 = 0 + 1 = 1
x0

x0

= lim x + 1 = 1
x0

l . h. l r . h.l given function is discontinuous at x = 0.


3. Discuss the continuity of the function f (x) at x = 0

af

f x =e

f 0 =e

Solution: l .h . l = lim

af a

=e

af

x8
x8 .

x 8

x0

Solved Examples

f x =

= lim e = e = 1

Solution: l .h. l = lim e


x0

a f

=e =1
0

=1

af

l .h . l = r . h. l = f 0 given function has a


continuity at the origin (x = 0).
4. Discuss the continuity of the function f (x) at x = 0.

af

x
x

af

(i) f x =
(ii) f x =

af

Solution: (i) The function f x =

x
x

is not de-

fined at x = 0 and so it is discontinuous at x = 0.

af

(ii) The function f x =

is not defined at x = 0
x
and so it is discontinuous at x = 0.
5. Discuss the continuity of the function f (x) at

af

x = 2 f x = x +

x+2
.
x+2

af

Solution: The function f x = x +

x+2
is not
x+2

defined at x = 2 and so discontinuous at x = 2.


6. Show that f (x) = | x | is continuous at the origin.
Solution: f (x) = | x |

a f
r .h.l = lim a x f = 0
f a0f = 0 = 0
l .h . l = r . h. l = f a0f given function f (x) is
l .h. l = lim x = 0
x0

x0

continuous at x = 0.

Practical Methods on Continuity test

7. Discuss the continuity of the function f (x) at x = 8

b g b xx 88g

f x =

3
3

, x 8 ; and f (8) = 1.

b g b xx 88g , x 8
b x 8g
l .h .l = lim
b x 8g
= lim a1f = 1
3

Solution: f x =

2. A function is defined as
f (x) = | x | + | x 1 | + | x 2 |
Solution: Since we know that | x |, | x 1 | and | x 2 |
are continuous everywhere x + | x 1 | + | x 2 |
is continuous everywhere
x + | x 1 | + | x 2 | is continuous at x = 0, 1

Examples on redefined functions which is the


combination of f (x) and | f (x) |
1. Discuss the continuity of the function f (x) at x = 2

x8

x 8

F x 8IJ
r . h. l = lim G
H x 8K

283

af

f x =

, when x 0 = 0, when x = 0.

Solution: l .h . l = lim

x 0

x 8

= lim 1 = 1

x 0

F x I = lim x = 0
e j
GH x JK
3

r . h. l = lim

x8

Hence, l .h.l r .h.l given function is discontinuous at x = 8.


Note
1. Any polynomial function, ex, sin x, cos x, | x | are
functions which are continuous everywhere.
2. Log x is continuous everywhere except x 0
where it is not defined.

af
af

p x
is continuous
q x
everywhere except at those points (or, those values
of x) which make q (x) = 0.
4. | f (x) | is continuous everywhere provided f (x) is
continuous everywhere.
3. Ratio of two polynomials

F x I = lim x = 0
e j
GH x JK
3

Problems on | f (x) |
1. Discuss the continuity of | x | + | x 1 | at x = 0, 1.
Solution: Since x is continuous everywhere x
is continuous everywhere similarly, (x 1) being a
polynomial is continuous everywhere x 1 is
continuous everywhere.
Hence, | x | + | x 1 | is continuous everywhere
x + | x 1 | is continuous at x = 0, 1

x0

x 0

f (0) = 0

af

af

l . h. l = r .h . l = f 0 = 0 f x is continuous at x = 0.
2. Discuss the continuity of the function f (x) at x = 0

af

f x =

x
x

, when x 0 = 0, when x = 0.

Solution: l . h. l = lim

x0

r . h. l = lim

x0

F x I = 1
HxK

F xI = 1
H xK

l .h.l r .h.l f (x) is discontinuous at x = 0.


3. If f (x) = | x | + 1, x 0 = 1, at x = 0 test the continuity
of f (x) at x = 0.
Solution: f (x) = | x | + 1

a f a f
r . h. l = lim a x + 1f = a0 + 1f = 1

l .h . l = lim x + 1 = 0 + 1 = 1
x0

x0

f (0) = | 0 | + 1 = 0 + 1 = 1

284

How to Learn Calculus of One Variable

af

l . h. l = r . h.l = f 0 given function f (x) is


continuous at x = 0.
4. Discuss the continuity of f (x) at x = 0

af

x x
, when x 0 = 2, when x = 0.
x

f x =

Solution: l .h . l = lim

x0

= lim

x0

FG x a xf IJ =
H x K

Examples on trigonometrical and inverse trigonometrical functions redefined


Remember:

x0

2. lim

x
tan

x0

x0

3
2

bg

l .h.l r.h.l f (x) is discontinuous at x = 0.

x0

f 0 =

x0

sin

x0

2x
=2
x

F x x IJ = 0
r . h. l = lim G
H x K

1. lim

FG sin 3x 3x 1 IJ
H 3x
2x K
F sin 3x 3IJ
= lim G
H 3x 2 K
3
F sin 3x IJ = 3 1
= lim G
H 3x K 2
2
= lim

2
(given)
3

af

f x =

sin x
,x0
x

= 1, x = 0

a f
a f
F sin x IJ = 1
= lim G
H x K
F sin x IJ = 1
r .h. l = lim G
H x K

=1

Solution: l .h. l = lim

x0

af

l .h . l = r . h. l f 0 the given function f (x)


is discontinuous at x = 0
2. Test the continuity of f (x) at x = 0, if

=1

sin x
x

x0

sin x
3. lim
=1
x0
x

x0

tan x
4. lim
=1
x0
x

f (0) = 1

5. lim cos x = 1
x 0

Now we come to the problems.


1. Test the continuity of the function f (x) at x = 0
where

af

1
R
a f |Ssin x cos x , when x 0
|T0 , when x = 0
R|sin x cos 1 , when x > 0
x
Solution: f a x f = S
|Tsin x cos 1x , when x < 0
f x =

sin 3 x
f x =
,x0
2x

2
, x=0
3

af

Hence, l .h. l = r . h. l = f 0 f (x) is continuous


at x = 0.
3. Test the continuity of the function f (x) at x = 0
defined as under
1

Solution: l .h. l = r . h. l = lim

x0

FG sin 3x IJ
H 2x K

e3 sin a xf = sin xj
1

Practical Methods on Continuity test

F
H

l .h. l = lim sin


x0

LM
N

= 0 3 lim sin

x0

F
H

r . h. l = lim sin
x0

x cos

1
x

I
K

x = 0 and cos

x cos

1
1 for x 0
x

I
K

OP
Q

af

f x =

tan

, x 0 and f (0) = 0.

x
3

x + x + 2x

x0

tan

tan
tan
3
2
lim tan + lim
lim tan
0
0
0

af

af

tan
lim 2 tan
0
0

=10+10+10=0
f (0) = 0 (given)

Hence, lim f x = f 0 the given function

af
f bxg g bxg = 0 .

f (x) is continuous at x = 0. (Note: (i) If lim f x = 0


xa

g x m , , then

af

lim

xa

(ii) If lim f x = 0 , then f (x) is said to be an


xa

1
h

=0

h 0

h 0

h0

af

h 0

a f

f 0 0 = lim f 0 h = lim f h = lim

h 0

eh = +

x0

af

r . h. l = lim e

= lim

and

x 0

F tan I + lim F tan I + lim 2 tan


j
GH JK GH JK e

x0

h 0

Solution: l .h. l = lim e

tan + tan + 2 tan

+ lim

x = 0).

= lim

Solution: For h > 0,

2. Test the continuity of y = e

[on putting x = tan x 0 , 0 ]

, x 0 f (0) = 0.

f (x) has infinite discontinuity at x = 0.


4

= lim

1
x

Solution: l.h.l = r.h.l = lim


4

af

f x =e

f 0 + 0 = lim f 0 + h = lim f h = lim

f (0) = 0 (given)
Thus, l.h.l = r.h.l = f (0) given function is
continuous at x = 0.
4. Examine whether the following function is
continuous at x = 0
x + x + 2x

Examples on redefined exponential function of x or


defined function of x
1. Test the continuity at x = 0 of the following function

1
=0
x

285

infinitesimal (or, infinitely small)).

af

f 0 =

1
0

a f=

= lim

x 0

1
e

at the origin (i.e.


x

lim e = e = 1

x 0

1
e

1
=1
1

1
=1
1

e
Thus, l.h.l = r.h.l = f (0) given function f (x) is
continuous at x = 0 (or, origin).
Problems based on greatest integer function/
combination of two greatest integer function/
combination of a function and a greatest integer
function defined or redefined
1. Discuss the continuity of the following functions
at x = 0.
(i) f (x) = [x] + [1 x] for all real x
(ii) f (x) = x + [x] for all real x
(iii) f (x) = [x] + [x] if x 0 , f (0) = 1
(iv) f (x) = x [x]
where [t] denotes the largest integer less than or equal
to t.
Solution: (i) f (x) = [x] + [1 x] for all real x

286

How to Learn Calculus of One Variable

af

b x + 1+ x g
= lim c 0 h + 1 a0 hf h
= lim b h + 1 + h g 3 1 > h > 0
1<1+h<2
= lim a 1 + 1f = 0
and 1 < h < 0

l .h . l = lim

x 0 0

f x = lim

x 0 0

h0
h0

h 0

[h] = 1
[1 + h] = 1 and [1 h] = 0

r . h. l = lim

x 0+0

af

f x = lim

x0 +0

bx

+ 1 x

b 0 + h + 1 0 h g
= lim b h + 1 h g
= lim a0 + 0f = 0
lim f a x f = 0 and f (0) = [0] + [1 0] = 0 + 1 = 1
Hence, lim f a x f = 0 f a0f = 1 f (0) = 1 which
= lim

h0

f (0) = (0 + [0] ) = 0
l.h.l r .h.l which means f (x) is discontinuous
at x = 0
(iii) f (x) = [x] + [x] if x = 0
f (0) = 1

g
= lim c 0 h + a0 hf h , h > 0
= lim b 0 h + 0 + h g
= lim b h + h g
= lim a1 + 0f = 1

l .h . l = lim

x 00

bx

+ x

h0
h0

h 0

h 0

h0
h0

x 0

x0

means f (x) is discontinuous at x = 0.


(ii) f (x) = x + [x] for all real x

l .h. l = lim

x0 0

= lim

x0 0

af

f x

bx + x g

b
g
= lim b0 h 1g = 0 1 = 1
r . h. l = lim f a x f
= lim b x + x g
= lim b0 + h + 0 + h g
= lim bh + h g
= lim ah + 0f = 0
= lim 0 h + 0 h
h 0
h0

x 0+ 0

x0 + 0

h 0
h 0
h0

3 [h] = 1 and [h] = 0


r .h . l = lim

x 0+0

bx

+ x

a
c
= lim b h + h g
= lim a0 1f = 1

= lim 0 + h + 0 + h
h0

fh

h 0

h 0

f (0) = 1 (given)

af

l . h. l = r . h.l = f 0 = 1 f (x) is continuous at x = 0


(iv) f (x) = x [x]

bx x g
= lim ah f h , h > 0
= lim lahf 0q b3 h

r . h. l = lim

x 0+0
h 0
h0

=0

= lim 0 = 0
h 0

b g
= lim la0 h f 0 h q , h > 0

l .h. l = lim x x
x0

h 0

Practical Methods on Continuity test

la hf h q
= lim la h fa 1fq b3 h

b a fg
= lim a1f

= lim

= lim 0 + 1

h 0
h 0

h 0

= 1

h 0

= 1
f (1) = [1 1] + [1 1] = [0] + [0] = 0

= lim h = 0
h 0

f (0) = 0 [0] = 0
l.h.l = r.h.l = f (0) = 0 which means the function
f (x) is continuous at x = 0.
2. Discuss the continuity of the following functions
at the indicated points.
(i) f (x) = [1 x] + [x 1] at x = 1
(ii) f (x) = [x + 2] + [2 x] at x = 2

LM 1 + x OP 1
2
Q 2
f axf = N

af

l . h. l = r . h. l f 1 f (x) is discontinuous
at x = 1
(ii) f (x) = [x + 2] + [2 x]
l . h. l = lim

x20

(iv)

m2 h + 2 +
= lim m 4 h + h r
h0

=3+0=3

LM x + 1 OP
2 Q at x = 1
f a xf = N

g,h>0
= lim m 2 + h + 2 + 2 a2 + hf r
= lim m 4 + h + h r

r . h. l = lim

x2+0

af

l .h. l = lim f x
= lim

x 1 0

b 1 x

+ x1

b 1 1 + h + 1 h 1 g, h > 0
= lim b h + h g
= lim b0 + a1fg
= lim a1f
h 0

= 4 + (1) = 3
f (2) = [2 + 2] + [2 2] = [4] + [0] = 4 + 0 = 4

bg

h 0

tinuous at x = 2

LM 1 + x OP 1
2
Q 2
f axf = N
2

(iii)

x 1+ 0

b 1 x

+ x 1

c 1 1 h + 1 + h 1 h, h > 0
= lim c h + h h
= lim

h0

h0

LM 1 + x OP 1
N2 Q 2
2

h 0

0 < h < 1 and -1 < h < 0 [h] = 0


and [h] = 1

bg

Hence, l .h. l = r . h. l f 2 f x is discon-

h0

r . h. l = lim

+ 2 x

h0

= lim

= 1

b x+2

h0

Solution: f (x) = [1 x] + [x 1]
x 1 0

g
2 a2 hf r , h > 0

+ 2 x

h0

at x = 1

b x+2

= lim

(iii)

287

r . h. l = lim

x 1+ 0

x2

LM 1 + a1 + hf OP 1
N2
Q 2 ,h>0
= lim
a1 + hf
LM 1 + 1 + h + 2hOP 1
N2
Q 2
= lim
2
2

h0

h 0

1 + h + 2h

288

How to Learn Calculus of One Variable

LM 1 + 1OP 1
2
Q 2 =1 1 = 1
f a1f = N
1
2 2
l . h. l = r . h.l = f a1f f a x f is continuous at

LM 1 + 1 + 2hOP 1
2
Q 2
= lim N
1 + 2h

h0

LM 3 + 2hOP 1
2
Q 2
= lim N

x =1.

1 + 2h

h 0

1
2
= lim
h 0 1 + 2h
1

LM 3 + 2hOP = 1
N2 Q

(iv)

as

3
+ 2h < 2 for
2
1
2 = 1 h being sufficinetly
= lim
h 0 1 + 2h
2 small > 0
1<

LM x + 1 OP
2Q
f a xf = N

l .h . l =

x 1 0

LM 1 + x OP 1
N2 Q 2

h 0

x2

LM 1 + a1 hf OP 1
2
Q 2 ,h>0
= lim N
a1 hf
LM 3 2hOP 1
2
Q 2 [on neglecting higher
= lim N
a1 2hf

= lim

h0

2
2

h0

h 0

power of h].

1
2
= lim
h 0 1 2h
1
2 = 1
= lim
h 0 1 2h
2
3

= lim

h0

LM 3 2hOP = 1 as 1 < 3 2h < 2 for h


N2 Q
2

being suffuciently small > 0

LM
N

h
1
h
2

1
1

OP
Q

= lim 1
h 0

=1

LM
N

3
1

LM 1 h + 1 OP
2
2Q
= lim N
1
LM hOP , h > 0
N 2 Q

l .h. l = lim

lim

1
x 0
2

LM x + 1 OP
N 2Q

OP
Q

1
h = 1 and [h] = 1 as 0 < h < 1
2

1 < h < 0 1 <

1
h<0
2

LM 1 + h + 1 OP
2
2Q
r . h. l = lim N
LM 1 + hOP , h > 0
N 2 Q
h0

= lim

h0

LM
N

h
1
+h
2

OP
Q

Practical Methods on Continuity test

= lim

h0

= lim

0
1

x3+ 0

OP
Q

1
+ h = 1 and [h] = 0 as 0 < h < 1
2

LM 1 + 1 OP
F I N 2 2Q = 0
H K L 1 O L 1 O
MN 2 PQ MN 2 PQ
0
LM 1 OP = 1 as 1 < 1 < 0
=
=0
1
2
N 2Q
l .h. l r . h. l = f a0f f (x) is discontinuous
1
=
f
2

1
.
2
3. Show that the function f defined by f (x) = [x 3] +
[3 x], where [t] denotes the largest integer < t, is
discontinuous at x = 3. Modify the definition of f so
as to make it continuous there.
Solution: f (x) = [x 3] + [3 x]
at x =

x 3 0

= lim

x 3 0

af

f x

b x3 +

3 x

b 3h3 +
= lim b h + h g
= lim a1 + 0f = 1
= lim

h 0

33+h

g,h>0

3 h = 0 and [h] = 1 as 0 < h < 1


1 < h < 0

af

f x

f (3) = [3 3] + [3 3] = [0] + [0] 0 + 0 = 0

af

l . h. l = r . h .l f 3 f (x) is discontinuous
at x = 3

af

Further, l .h . l = r . h. l f 3 f (x) has a removal discontinuity at x = 3. This is why to remove


the discontinuity at x = 3, are must modify the definition of f as follows.
f (x) = [x 3] + [3 x], when x 3
= 1, when x = 3
4. Show that the function f defined by f (x) = [x 1] +
| x 1| for x 1 , and f (1) = 0 is discontinuous at
x = 1. Can the definition of f at x = 1 be modified so as
to make it continuous there?
Solution: f (x) = [x 1] + | x 1| for x 1
f (1) = 0 (given)
x 1 0

h0

x 3+ 0

3 h = 0 and [h] = 1 as 0 < h < 1


1 < h < 0

l . h. l = lim

h 0

r . h. l = lim

h0
h 0

1
1 < h < 0 1 < + h < 0
2

l .h. l = lim

c 3+h 3 + 3 3 h h
= lim = c h + h h
= lim a1 + 0f = 1
h0

LM
N

3 x

= lim

= lim 0 = 0

b x3 +

289

= lim

x 1 0

af

f x

lx1 +

x1

l 1 h 1 + 1 h 1 q ,h>0
= lim l h + h q
= lim a 1 + hf
= lim

h 0
h0

h 0

= 1 + 0 = 1
3 | h | = | h | = h as h > 0 and 0 < h < 1
1 < h < 0 [h] = 1

290

How to Learn Calculus of One Variable

r . h. l = lim

x 1+ 0

= lim

x 1+ 0

af

l.h.l = r.h.l = f (5)


But f (5) = k
(i) and (ii) 10 = k k = 10

f x

lx1 +

x1

l1+ h 1 +
= lim l h + h q
= lim a0 + hf

q
q

= lim

1 + h 1 ,h>0

h 0

=0+0=0

l .h .l r .h .l = f 1 = 0 f (x) is discontinu-

af

ous at x = 1. Further, l .h . l r . h. l lim f x does


x 1

not exist which we can not modify the definition


of f at x = 1 in any way to make it continuous at x = 1.
Type 5: Problems based on finding the value of a
constant k if the given function it continuous at a
given point x = a.
Working rule: To find the value of a constant k if
the given function is continuous at a given point
x = a, we adopt the following procedure.
1. Find the l.h.l and r.h.l
2. Equate l.h.l = r.h.l = f (a) = a constant given in the
question and solve the equation for k.
Solved Examples
1. A function f is defined as

af

a x 2f

x 25
, x 5 and f (5) = k, x = 5 if f (x)
x+5

is continuous at x = 5, find K.

x 5

= lim

x 5

= lim

x 5

,x2

F x 25I
GH x + 5 JK

a x + 5f a x + 5f
a x + 5f
a x 5f = a5 5f = 10

Now, since, f (x) is continuous at x = 5 (given in


the problem)

x + x 16 x + 20

a x 2f
a x 2f e x + 3x 10j
= lim
a x 2f a x 2f
a x 2f a x + 5f
= lim
a x 2f
= lim a x + 5f = 2 + 5 = 7
x2

x2

x2

x2

Now, since f (x) is continuous at x = 2 (given in the


problem)
(i)
l.h.l = r.h.l = f (2)
But f (2) = k (given in the problem)
(ii)
Hence, (i) and (ii) 7 = k , i.e; k = 7
Ans.
3. Find the value of k if the following function is
continuous at x = 0

bg

1 cos kx
, x 0 and f (0) = 2.
x sin x

Solution: l .h. l = r . h. l = lim

x 0

Solution: l .h . l = r . h. l = lim

Solution: l .h. l = r . h. l = lim

f x =

f x =

3 [h] = 0 as 0 < h < 1

af

x + x 16 x + 20

= k, when x = 2 and f (x) is continuous at x = 2, find the


value of K.

h 0
h 0

af

2. If f x =

(i)
(ii)
Ans.

2 sin 2
= lim

x0

FG kx IJ
H2K

x sin x

LM F kx I
sin G J
H 2K k
M
= lim M
MM FG k x IJ 4
NH 4 K
2

x0

1 cos kx
x sin x

OP
x P

2
sin x P
PP
Q

291

Practical Methods on Continuity test

bg

=2 1

Find the values of a, b, c if f (x) is continuous at


x=0
Solution:

k2
4

k2
2
Now, that given function f (x) is continuous at
...(i)
x = 0 l.h.l = r.h.l = f (0)
But f (0) = 2 (given)
(ii)
(i) and (ii) l.h.l = r.h.l = f (0)
=

k2
=2
2

k= 2
4. Find the value of k if the following function f (x) is
continuous at x = 0

bg

sin kx
,x0
f x =
x
f (x) = 4 + x, x = 0
x0

FG sin kx k IJ
H kx K

FG sin kx IJ
H x K

sin kx
k
x0
kx
=k1=k
Also, f (0) = 4 + 0 = 4
Now, f (x) is continuous at x = 0 l.h.l = r.h.l =
f (0)
(i)
But f (0) = 4
(ii)
(i) and (ii) k = 4 Ans.
5. A function f (x) is defined as follows

af

sin a + 1 x + sin x

, for x < 0

x
= c, for x = 0
2

x + bx
bx

x0

x0

= (a + 1) 1 + 1 = a + 1 + 1 = a + 2

af

x + bx

r . h. l = lim f x = lim

x 0

bx

, for x > 0

L x + bx x x + bx +
= lim M
MN bx x x + bx +
LM
OP
x + bx x
= lim M
MM bx x FG x + bx + x IJ PPP
KQ
N H
LM
OP
bx
= lim M
MM bx x FG x + bx + x IJ PPP
KQ
N H
OP
LM
x
= lim M
F
MM x G x + bx + x IJ PPP
KQ
N H
LM
OP
x x
= lim M
MM x FG x + bx + x IJ PPP
KQ
N H
2

x0

= lim

f x =

x 0

x0

x0

Solution: l .h. l = r . h. l = lim

x0

x 0

x0

k2 = 4

= lim

LM sin aa + 1f x + sin x OP
x
N
Q
L sin aa + 1f x + sin x OP
= lim M
x Q
N x
L
sin aa + 1f x sin x O
= lim Maa + 1f
+
P
x aa + 1f
x Q
N
sin aa + 1f x
sin x
= lim aa + 1f
aa + 1f x + lim x
af

l .h . l = lim f x = lim

x0

x0

x0

x0

...(1)

x
,x>0

OP
P
xQ
x

292

How to Learn Calculus of One Variable

LM
OP
x
= lim M
MM FG x + bx + x IJ PPP
KQ
NH
L 1 OP = 1
= lim M
MN 1 + bx + 1PQ 2
x0

x0

...(2)

f (0) = c (given)
f (x) is continuous at x = 0 l.h.l = r.h.l = f (0)

a=

3
2

R|a = 3
|c = 1 2
S| 2
||b = any non - zero finite value
T
af

log 1 + ax log 1 bx
x

LM
N

af af

lim f x = f 0

af

f (x) if there is a limit lim f x = b but either f (x) is


xa

not defined at the point x = a or (f (x) at x = a) b and


if we set f (a) = b, then the function f (x) becomes
continuous at the point x = a, i.e; the discontinuity is
removed.

bg b

7. If f x = 1 + x

1
x

, when x 0 = k, when x = 0
find the value of k if f (x) is continuous at x = 0.

f (where y = f (x))
log y = log a1 + x f
1
log y = log a1 + x f
x
1
x

1
x

f is not do find at

lim log y = lim


x0

x0

1
bx

= a (1) + b (1) = a + b
Now, for f (x) to be continuous at x = 0,

Solution: y = 1 + x

x = 0.
Find the value which should be assigned to f at
x = 0 so that it is continuous at x = 0 is
(a) a b (b) a + b (c) log a + log b (d) none of
these.
Solution:

af

Note: Problem (6) is a problem of a point of removable


discontinuity of the function f (x) which tells x = a is
a point of removable discontinuity of the function

1
c=
2
Again, no restriction is imposed on b it can
have any non-zero finite value ( 3 b = 0 makes f (x)
undefined)
Thus, we conclude

f x =

1
bx

f (0) = a + b
Thus, the correct answer is (b).

1
3
1
2 = and c =
2
4
2

6. The function

1
ax

x0

x0

1
=c
2

Thus, we get a =

1
ax

x0

x0

a+2=

LM 1 log a1+ axf + b 1 log a1 bxfOP


bx
N ax
Q
L
O
= lim Ma log a1 + ax f + b log a1 bx f P
N
Q
= a lim log a1 + ax f + b lim log a1 bx f
= lim a

1
1
lim f x = lim
log 1 + ax log 1 bx
x0
x0 x
x

fOPQ

x0

1
log 1 + x = 1
x

lim y = e = e
x0

Now, l . h. l = r .h . l = lim y = e
x0

(3 x 0 x > 0 and x < 0)

(i)

Practical Methods on Continuity test

Now, f (x) is given to be continuous at x = 0


l.h.l = r.h.l = f (0)
e = k, i.e; k = e. Ans.
Continuity in an Interval
1. A function f or f (x) defined in the open interval
(a, b) is said to be continuous in the open interval
(a, b) f (x) is continuous at any arbitrary point
x = c where a < c < b. Hence, to test the continuity in
an open interval (or, for all x), we simply consider an
arbitrary point x = c s.t a < c < b = (a, b) and we show

af

af

that lim f x = f c
h 0

h 0

through +ve values

af

af

af

or , lim f x = lim f x = f c
xc
x <c

x c
x >c

a 7a + 3a 1

(i)

a 3a

bg

a 3 7a 2 + 3a 1
(ii)
a 2 3a

f x =
Similarly, xlim
a
x<a

and f (a) =

a 7a + 3a 1

(iii)

a 3a

Hence, (i), (ii) and (iii)

af

af

af

lim f x = lim f x = f a the given

or, lim f c + h = lim f (c h) = f (c), where h 0


h 0

293

Note: We must remember that it is not possible to


test the continuity of the function at every point of
an interval, however small it may be. This is why to
test the continuity of a function f (x) in an interval
(a, b), we always consider an arbitrary point

af af

xa
x >a

x a
x <a

function f (x) is continuous at x = a but a is any


arbitrary point in (0,3), so the given function f (x) is
continuous in 0 < x < 3.
Or, alternatively,

af

f a =

af

lim f x =

xa

a 7 a 3a 1
2

a 3a
3

a 7a 3a 1
2

a 3a

af

x = c s.t a < c < b and we show that lim f x = f c .

Hence, f (a) = lim f x continuity of f (x) at

This rule is applicable when the given function is not


a piecewise function or when the given function is
not redefined.

any arbitrary point x = a continuity of f (x) in the


given interval (0, 3).
2. Some-times taking some points in the given interval
(open or closed), we test the continuity of the function
at each such point separately belonging to the given
interval (open or closed). This is the case when the
given function is a piecewise function, i.e; if a function
is defined by different formulas for different ranges
of values of x, then there is a possibility of
discontinuity at the values where the two ranges of x
meet. Thus if we have one expression f1 (x) for x > a
and another expression f2 (x) for x < a, then the
probable point of discontinuity of the function given
to us is x = a. Similarly, if we have one expression f3 (x)
for x a' and another expression f4 (x) for x > a' ,
then the probable point of the discontinuity of the
function f (x) given to us is x = a'.

xc

Explanation
1. Test the continuity of the function

af

f x =

x 7 x + 3x 1
2

x 3x

in the interval

a f

0 < x < 3, i.e; x 0 , 3


Solution: Let x = a be any arbitrary point in (0, 3) s.t
0<a<3
Now,

af

xa
x >a

a a + h f 7 a a + h f + 3 a a + hf 1
aa + h f 3 aa + h f
3

lim f x = lim

h 0

(putting x = a + h)

xa

294

How to Learn Calculus of One Variable

Explanation
1. Consider the function f (x) defined as under
f (x) = 0 for all values of x > 1
f (x) = 1 for all values of x < 1

af

1
for x = 1
2
where, we can see that only probable point of the
function f (x) at which it may be discontinuous is
x = a since f (x) (or, the value of the function f )
changes its expression in the neighbourhood of x = 1.
2. Consider the function f (x) defined as under
f (x) = (2x 1), when x < 0
f (x) = 2x, when x 0
where by inspecting the behaviour of the given
function f (x), we can see that only probable point of
f (x) at which it may be discontinuous is x = 0 since
the function f (x) (or, the value of the function f )
changes its expression in the neighbourhood of x = 0.
3. The functions which we face in elementary
applications of the calculus are usually either
continuous for values of x or have discontinuities
only for a number of values which makes the function
undefined/imaginary/infinite.
f x =

Explanation

1
this function is discontinuous at x = 0.
x
(ii) y = tan x this function is discontinuous at

(i) y =

x=

.
2

(iii) y =

x
2

x 4

this function is discontinuous

at x = 2 .
4. If the continuity fails to exist for some value of x
between a and b in case the interval is open, we say
that the function is discontinuous in (a, b).
5. If the continuity fails to exist for some value of x
between a and b (including a and b) in case the interval
is closed, we say that the function is discontinuous
in [a, b].

Solved Examples
1. At what points of the interval, shown against each
function are the following functions discontinuous?
(i) f (x) = x if x 0
f (0) =1, in the interval [1, 1]
(ii) f (x) = 4x + 7 for x 2
f (2) = 3, in the interval [4, 4]

af

(iii) f x =

9 x 16
3

27 x 64

, when x

4
3

F 4 I = 2 , in the interval [1, 3]


H 3K 3

Solutions: (i) By inspecting the behaviour of the


given function, we can see that the only probable
point of the interval [1, 1] at which the given function
may be discontinuous is x = 0 because the function
f (x) changes its expression in the neighbourhood of

LMi.e; when x 0, the value of OP


bg
PP
MM
b
g
Q
N
Thus, l .h. l = r . h.l = lim f a x f = lim x = 0
x = 0 the function f x = x and
f 0 =1
x0

f (0) = 1 (given)

x0

af

(i) and (ii) l . h. l = r . h. l f 0 f (or, f (x)) is


discontinuous at x = 0.
(ii) we can see that only probable point of the interval
[4, 4] at which the function may be discontinuous is
x = 2 since the value of the function f (or, f (x)) changes
its expression in the neighbourhood of x = 2

af

b a fg = 3 ]
Thus, l .h . l = r . h. l = lim a4 x + 7 f = 8 + 7 = 15

[3 f x

x2

= 4 x + 3 and f x

x =2

x2

[f (x)]x = 2 = f (2) = 3

af

l .h .l = r . h. l f 2 f (or, f (x)) is
discontinuous at x = 2.
(iii) The only probable point of discontinuity of the

function is x =

4
since the function f or f (x) changes
3

Practical Methods on Continuity test

its expression in the neighbourhood of x =

bg

f x

4
x
3

af

f x

x=

4
3

9 x 2 16
27 x 3 64
=

f x =

4
3

= lim

4
x
3

4
3

9 x 2 16

is discontinuous in the open interval (3, 1).

27 x 3 64

a3x 4f a3x + 4f
a3x 4f e9 x + 12 x + 16j

x 2
x < 2

r . h. l = lim

x 2
x > 2

3x + 4
9 x + 12 x + 16

F 4I + 4
H 3K
8
1
=
=
=
F 4 I + 12 F 4 I + 16 48 6
9
H 3K H 3K
4
2
But f F I =
H 3K 3
F 4I
Hence, l . h. l = r .h. l f
H 3K
F 4I
lim f a x f f
H 3K

Note: There is no need to test the continuity at x = 0


where

x 0
x <0

x 0
x >0

r . h. l = lim

x 1+

=2+1=3

x 0

Continuity at the End


Points of a Closed Interval

4
3

a f
f a x f = lim a2 x + 1f = 2 1 + 1
x 1

af

Hence, l .h. l r . h.l at x = 0 which means f (x) is


discontinuous at x = 0 and so discontinuous in the
given open interval (3, 1).

Solution: l .h . l = lim f x = lim x + 1 = 1 + 1 = 2


x 1

x0

r . h. l = lim f x = lim x + 2 = 0 + 2 = 2

2. Discuss the continuity of f in [0, 2] if


f (x) = x + 1, 0 x < 1

af

af

l .h. l = lim f x = lim x 1 = 1

4
3

x 1

ax 1f = 2 1 = 3

Thus, ( l .h.l r .h.l at x = 2) f (x) is


discontinuous is [3, 1]

= 2x + 1, 1 x 2

= 4 + 3 = 1

f (or, f (x)) is not continuous at x =

Solution: l .h. l = lim 2 x + 3 = 2 (2) + 3

f (1) = 2 + 1 = 3
f (x) is discontinuous at x = 1.
Therefore, f (x) is discontinuous in the given closed
interval [0, 2].
3. Show that the function

R2 x + 3 , when 3 < x < 2


a f |S x 1, when 2 x < 0
|T x + 2 , when 0 < x < 1

2
3

Thus l .h . l = r . h. l = lim

= lim

4
, i.e.
3

295

Definition: Set a function f (x) be defined on (or, over


or, in) the closed interval a x b = a , b where
a = left end point and b = right end point.
1. The function f (x) is said to be continuous at the
left end point a of a closed interval a x b iff

af

af

r.h.l of f (x) at x = a is = f (a) , i.e.; lim f x = f a

af

af

xa
x >a

/ lim+ f x = f a , where f (a) = value of the


xa

function at x = a.

296

How to Learn Calculus of One Variable

2. The function f (x) is said to be continuous at the


right end point b of a closed interval a x b iff

af

af

l.h.l of f (x) at x = b is f (b), i.e; lim f x = f b /

af af

xa
x <a

lim f x = f b , where f (b) = value of the

x b

function at x = b.
Note:
1. Continuity at the left end point of closed interval
is required only when a given function is not defined
for x < a, where a = left end point If we are not
provided f (x) = x for x < a and we are provided
f x = 1 x for x a we are required to test
the continuity at x = a = l.e.p for which only r.h.l is
required to find out.
2. Continuity at the right end point of a closed interval
is required only when a given function is not defined
for x > b, where b = right end point if we are not
provided f (x) = x for x > b and we are provided
f x = 1 x for x b we are required to test
the continuity at x = b = r.e.p for which only l.h.l is
required to find out. This is why we do not need to
find out the right hand limit of the given function.

af

af

af

af

af

af

Aid to memory

l . h. l
r . e. p needs
l . e. p
r . h. l
Now, we come to the model of the questions which
are provided to us. Generally the model of the
question is the following:
a xb
a f RST ff aaxxff,, when
when b x c

1. f x =

and the interval is [a, c]

a<x<b
a f RST ff aaxxff,, when
when b x c

2. f x =

and the interval is (a, c]

a x<b
a f RST ff aaxxff,, when
when b x c

3. f x =

and the interval is [a, c]

a x<b
a f RST ff aaxxff,, when
when b x < c

4. f x =

and the interval is [a, c)


In all above types of problems b may be regarded
as the common point where the two ranges of values
of the independent variable x meet when a given
function f (x) is defined by various formulas for
different ranges of x as well as a and c may be regarded
as the end point of a closed interval or semi (or, half)
open and semi closed or semi closed and semi open
interval as (a, c] or [a, c).
Now, a question arises how to test the continuity
at x = a, b, c when the given function is a piecewise
function.
Question: How to test the continuity at x = b =
common point where the two ranges of x meet?
Answer: Continuity at the common point where the
two ranges of x meet requires to find out l.h.l, r.h.l
and the value of the function at x = b for which we
should consider both given functions f1 (x) and f2 (x)
against which the restriction.
x > b and x b
or, x b and b
or, x > b, x < b and x = b
or, x b and x < b is imposed.
Question 2: How to test the continuity at x = l.e.p of
[a, c] = a x c ?
Answer: Continuity at x = l.e.p of [a, c] or, [a, c)
requires to find the r.h.l of the function f (x) = f1 (x)
against which x a is written and the value of the
function f (x) at x = a and the other function f (x)
= f2 (x) against which the restriction x < a is imposed
is not defined (or, given). This is why l.h.l of f (x) = f2
(x) as x a is not required which means that

af

lim f 2 x is not required to find out. While testing

xa

the continuity at l.e.p of the closed interval [a, c] or


semi closed and semi open interval [a, c).
Question 3: How to test the continuity at x = r.e.p of
[a, c] = a x c ?
Answer: Continuity at x = r.e.p of [a, c] or, (a, c]
requires to find out l.h.l of the function f (x) = f2 (x)
against which x < c is written and the value of the

297

Practical Methods on Continuity test

function f (x) = f2 (x) at x = c and the other function f


(x) = f1 (x) against which x > c is imposed is not defined
(or, given). This is why r.h.l of f (x) = f1 (x) as x c

af

is not required which means lim+ f 1 x


xc

is not

required to find out while testing the continuity at


r.e.p of the closed interval [a, c] or semi open and
semi closed (a, c] interval.
Solved Examples

x0

x 1

af

= 8x , 2 x 4
test the continuity at x = 0, 2, 4
Solution: (1) At x = 2

af

= x+2

x=2

r . h. l = lim

(i)

(i)

(i)

(i)

[f (x)]x = 2 = (x3 + 2) = 8 + 2 = 10
(ii)
(i) and (ii) l.h.l = f (2) continuity of the
given function at the right end point of the closed
interval [0, 2].
Note: f (x) is continuous at x = 0 as r.h.l at x = 0 is
= f (0).

x =2

8x

x =2

8x = 8 2 = 4

x2

= x2 + 2, 1 x 2
Solution: Here f (x) is defined on the closed interval
[0, 2] = 0 x 2 Hence, to test the continuity of
f (x) at x = 2 only l.h.l is required since x = 2 is a right
end point of the closed interval [0. 2].
x2

af

f x = f 1

x 1 + 0

f (x) is continuous at x = 0.
Hence f (x) is continuous in [0, 2]
3. A function f (x) is defined by
f (x) = (x + 2), 0 x 2

f x

[f (x)]x = 1 = f (1) = [2x]x = 1 = 2 1 = 2


(ii)
(i) and (ii) l.h.l = value of the function
continuity of the given function f (x) at the right end
point of the closed interval [0, 1] = 0 x 1 .
2. Test the continuity of f (x) at x = 2, when
f (x) = x2 + x + 1, 0 x 1

l .h. l = lim x + 2 = 8 + 2 = 10

af

f x = lim

x2

[f (x)]x = 0 = f (0) = 0
(ii)
(i) and (ii) r .h. l value of the function at x = 0
discontinuity of the given function at the left end
point of the closed interval [0, 1] at x = 1
l . h. l = lim 3 x 1 = 2

x 1 0

l .h. l = lim (x + 2) = 4

1. A function f (x) is defined as follows


f (x) = 0, when x = 0
= 3x 1, when 0 < x < 1
= 2x, when x = 1
Test the continuity at x = 0 and x = 1.
Solution: At x = 0
r . h. l = lim 3 x 1 = 3 0 1 = 1

Also lim

= 4 (ii)
(iii)

(i), (ii) and (iii) l.h.l = r.h.l = value of the function


at x = 2.
(2) At x = 0 = left end point of the closed interval
[0, 4]

r . h. l = lim x + 2 = 2

af

f x

x0

x =0

= x+2

x=0

(i)
=2

(ii)

(i) and (ii) the function f (x) is continuous at


x = 0 = left end point of the closed interval [0, 4].
(3) At x = 4 = right end point of the closed interval
[0, 4]

l .h. l = lim

x4

af

f x

x =4

e 8x j =
8x

x=4

84 =4 2
= 8 4 = 32 = 4 2

(1) and (2) continuity at x = 4 since l.h.l = value


of the function at x = 4
4. A function f is defined in the following way:
f (x) = 0, when x = 0
= 2x 1, when 0 < x < 1
= 2 x, when x 1
is f (x) continuous at x = 0 and x = 1?
Solution: At x = 0
We inspect that f (x) is not defined for x < 0 since
f (x) = x = an expression in x when x < 0 is not
provided.

af

298

How to Learn Calculus of One Variable

af

r.h.l = f (0 + 0) = lim f x = lim 2 x 1 = 1

af

f x

x =0

x0

x0

af

af

af

interval (a, b) provided lim f x = f c where

= 0 given = f 0

xc

(i) and (ii) f (x) is not continuous at x = 0 = left


end point (l.e.p) of the closed interval [0, 1] at x = 1

a f a f
r . h. l = f a1 + 0f = lim a2 x f = 2 1 = 1
= a2 x f
= 2 1=1
f a xf

l .h . l = f 1 0 = lim 2 x 1 = 2 1 = 1
x 1

x 1

x =1

Or, alternatively,
The function y = f (x) is continuous in an open

x =1

Hence, f (1 0) = f (1 + 0) = f (1) continuity at


x = 1.
Facts to Know

x = c is an arbitrary point (or, number) s.t.


4. Criterion of continuity in a closed interval: The
function y = f (x) is continuous in a closed interval [a,
b] (i) f (x) is continuous at each point of (a, b)

af

af

(ii) lim+ f x = f a
xa

af

(iii) lim f x = b hold


x b

good .
Or, alternatively,
If the domain of a real valued function (or, real
function) f or f (x) is a closed interval [a, b], then f or
f (x) is continuous in (or, over/on/) closed interval

af

af

(i) lim f x = f c where x = c is any arbitrary

We end this chapter by mentioning some facts about


continuity which the students must remember.
1. Criterion of continuity at a real number: The
function y = f (x) is continuous at the point x = a
provided (i) it is defined at this point (ii) there is a limit

af

xc

af

point s.t a < c < b (ii) lim f x = r . h .l at the l.e.p =


xa
x >a

f (a) = value of the function at the l.e.p (i.e; left end

af

namely lim f x = L (iii) this limit is equal to the

point) (iii) lim f x = l.h.l at the r.e.p = f (b) = value

value of the function at the point (or, real number)


x = a.
Or, alternatively,
A function y = f (x) is continuous at the real number
x = a / x = a is a point of continuity of f (x) (i) f (a)

of the function y = f (x) at r.e.p (i.e; right end point)


5. Criterion at the left end point of a closed interval
[a, b]: f1 (x) is continuous at x = a = left end point of
the closed interval

x a

af

af

af

exists (ii) lim f x exists (iii) lim f x = f a


xa

xa

If one or more of these three conditions fail to hold


good at x = a, we say that f (x) is discontinuous at
x = a or x = a is a point of discontinuity of the function
y = f (x).
2. Criterion of continuity in an interval: The
function which has continuity at every point of an
interval is continuous throughout that interval.
3. Criterion of continuity in an open interval: The
function y = f (x) is continuous in an open interval (a,
b) provided f (x) is continuous at each point of (a, b)
and the function y = f (x) is discontinuous in (a, b) if
f (x) is discontinuous atleast at one point of (a, b).

xa
x <b

af

af af

a , b lim+ f 1 x = lim f 1 x = f a
xa

xa
x >a

(r.h.l at x = a) = value of the function at x = a.


6. Criterion at the right end point of a closed interval
[a, b]: f1 (x) is continuous at x = b = right end point of
the closed interval

af

af af

a , b lim f 1 x = lim f 1 x = f b
x b

xb
x <b

(l.h.l at x = b) = value of the function at x = b.


N.B.: (i) To test the continuity at right end point of a

af

closed interval, l.h.l = lim f 2 x is not required


xa
x <a

since the function f2 (x) is not given (or, defined) for


x < a where y = f (x) is a piecewise function defined in

Practical Methods on Continuity test

a closed interval which is divided into a finite number


of non-overlapping sub intervals over each of which
different functions are defined.
(ii) To test the continuity at the left end point of a

af

closed interval, r . h. l = lim f 2 x is not required to


xb
x >b

find out since the function f2 (x) is not given (or,


defined) for x > b where y = f (x) is a piecewise function
defined in a closed interval which is divided into a
finite number of non-overlapping sub intervals over
each of which different functions are defined.
(iii) If the interval is closed one, the limit at the left
end will mean right hand limit and the limit at the right
end will mean the left hand limit.
6. If functions f1 (x) and f2 (x) are continuous in an
interval (a, b) or [a, b], then the function
(i) c1 f1 (x) + c2 f2 (x)
(ii) f1 (x) f2 (x)
(iii)

af
af

f1 x
, provided f2 (x) 0 for any value of x
f2 x

belonging to the interval (open or closed), are also


continuous in the open interval (a, b) or in the closed
interval [a, b].

x = a, we consider another function defined in an


interval (or sub interval) containing all those values
of x which are greater than (or, greater than or equal
to) a.
Problems
Type1: When the function is defined.
Exercise 5.1
Examine each function for the continuity defined by

af

1. f x =

af

2. f x =

bg b

4. f x = 1 + x

xa

x = a.
8. While testing the continuity of a redefined function

af

we use (l.h.l = r.h.l at x = a) = lim f 1 x , where f (x)


xa

= f1 (x) for x a provided f1 (x) is not a mod function


(or a combination or composition of mod function) or
a greatest integer function.
9. When a given function is a piecewise function,
then to find the l.h.l at a point x = a, we consider a
function defined in an interval (or, sub interval)
containg all those values of x which are less than (or,
less than or equal to) a and to find the r.h.l at a point

[Ans: discont. at x = 0]

1
x

[Ans: cont. at x = 4]

, when x 0

f (x) = e, when x = 0

f x =

af af

x + 2x 9
, when x 1
x1

3. f (x) = 4x + 3, when x 4
f (x) = 3x + 7, when x = 4

value of the function at x = a because

lim f x = f a when y = f (x) is continuous at

[Ans: cont. at x = 3]

f (x) = 4, when x = 1

5. f x =

af

x 9
, when x 3
x3

f (x) = 6, when x = 3

7. The continuity of the function can be used to


calculate its limits which means if the function y = f (x)
is continuous at the point x = a, then, in order to find
out its limit lim f x , it is sufficient to calculate the
xa

299

af

cos ax cos bx

af

b a
2

af

6. f x =

x
2

[Ans: cont. at x = 0]

, when x 0

when x = 0 [Ans: cont. at x = 0]

, when x 0
x
f (x) = 0, when x = 0

af

7. f x = x sin

F 1 I , when x 0
H xK

f (0) = 0

[Ans: cont. at x = 0]

af

1 cos x

af

1
when x = 0
2

8. f x =

f x =

[Ans: cont. at x = 0]

, when x 0
[Ans: cont. at x = 0]

300

How to Learn Calculus of One Variable

R|1 cos mx , when x 0


| 1 cos nx
f a xf = S
m
||
, when x = 0
T
n

9.

R| sin
f axf = S x
|T

10.

[Ans: cont. at x = 0]
x

, when x 0

1 , when x = 0

[Ans: cont. at x = 0]

R|x sin F 1 I , when x 0


f axf = S
HK
|T x 0 , when x = 0

11.

19. Examine the continuity of the function f (x) at


t =

af

f t =

cos t

,t

2
t
2

af

[Ans: continuous at = ]
2
2
20. Discuss the continuity of the function
f t = 1, t =

af

f x =

U|
V|
|W

tan 7 x
, for x 0
sin 4 x
at x = 0
7
= , for x = 0
4

[Ans: cont. at x = 0]

12.

R| tan x sin x , when x 0


x
f axf = S
1
||T
, when x = 0

[Ans: cont. at x = 0]
Type 2: Problems on piecewise function:

Exercise 5.2.1

[Ans: cont. at x = 0]

af

13. f x =

sin 3x 3 sin x

a x f

f (x) = 4, when x =

af

14. f x =

, when x
[Ans: cont. at x = ]

x tan x
, when x 0
1 cos x

f (x) = 2, when x = 0

af

15. f x =

1+ x 1 x

[Ans: cont. at x = 0]
2

when x 0
2
x
f (x) = 2, when x = 0
[Ans: discont. at x = 0]
16. Show that f (x) = | x 5 | is continuous at x = 5.
17. Show that

R xa
a f |S x a , x a
|T 1 , x = a

f x =

is discontinuous at x = 1.
18. Examine the continuity of f (x) = | x b | at x = b.
[Ans: cont. at x = b]

Examine the continuity of the function f (x) defined


by
1. f (x) = 2x 1, when x < 2
f (x) = x2 4x + 5, when x 2
[Ans: discont. at x = 2]
2. f (x) = x 3. when x < 4
f (x) = 5 x, when x 4
[Ans: cont. at x = 4]
3. f (x) = 3 2x, when x 2
f (x) = x 1, when x > 2
[Ans: discont. at x = 2]

af

4. f x =

x
, when x < 4
16

f (x) = 1 2x x2, when x 4


[Ans: discont. at x = 4]
5. f (x) = x2 2x + 3, when x 1
f (x) = x + 1, when x > 1
[Ans: cont. at x = 1]
6. f (x) = x2, when 0 < x < 1
f (x) = x, when 1 x < 2

Practical Methods on Continuity test

f (x) = 6 + x3, when 2 x < 3


[Ans: cont. at x = 1, 2]
7. f (x) = 1, when x < 0
f (x) = 0, when x = 0
f (x) = 1, when x > 0
[Ans: discont. at x = 0]

af

8. f x =

af

x
, when 0 x 1
2

3
, when 1 x 2
2
[Ans: cont. at x = 1]
9. f (x) = 5x 4, when 0 x 1
2

f x = 2 x 3x +

f (x) = 4x3 3x, when 1 < x 2


[Ans: cont. at x = 1]
10. f (x) = x, when 0 x <
f (x) = 1 x, when

[Ans: cont. at x =

1
]
2

11. f (x) = x3 + 1, when 0 x < 1


f (x) = 3x2 1, when 1 x 2
[Ans: cont. at x = 1]
Exercise 5.2.2
1. If f (x) = 0, when x = 0

f x = 1 x , when 0 < x <

1
2

1
.
2
1
[Ans. cont. at x = 0 and x = ]
2

then test the continuity of f (x) at x = 0 and x =

af

x +1
, when 5 < x 7
x 1

then test the continuity of f (x) at x = 5.


[Ans: discont. at x = 5]
3. If f (x) = x3 + 1, when 0 x < 1
f (x) = 3x2 1, when 1 x 2
then test the continuity of f (x) at x = 0, 1 and 2.
[Ans: cont. at x = 0, 1, 2]
4. The function f (x) is defined as under

R| x , for x 0
if f a x f = S x , for 0 < x < 1
|T2 x , for x 1

is the function f (x) continuous at x = 0 and x = 1


[Ans: cont. at x = 0 and x = 1]
5. Examine the continuity of f (x) at x = 0 and x = 1 if

1
2

1
x <1
2

bg
1
1
f a x f = , when x =
2
2

af

f x =

301

x 4x + 3
2. If f x =
, when 0 x 5
x4

R|1, when x = 0
f a x f = S x 1 , when 0 < x < 1
|T0 , when x = 1

[Ans: discont. at x = 0 and cont. at x = 1]


6. Examine the continuity of f (x) at x = 1 and x = 2 if

R|x , when 0 x < 1


f b x g = S2 x 1 , when 1 x < 2
|Tx + 1, when 2 x
2

[Ans: cont. at x = 1 and x = 2]


7. The function f is defined in the following way

R|3 + 2 x , when 3 x < 0


2
|
3
f a x f = S3 2 x , when 0 x <
2
||
3
|T3 2 x , when x 2
Prove that f (x) is continuous at x = 0 and
discontinuous at x =

3
.
2

302

How to Learn Calculus of One Variable

Problems on greatest integer function


Exercise 5.2.3
1. Test the continuity of the function f (x) at

2
if f (x) = [x], where [x] is the greatest integer
3
function.
2. If f (x) = (x [x]), where [x] is the greatest integer
x=

3
.
2
3. Test the continuity of the function f (x) at x = 4
function, test the continuity of f (x) at x =

af

Answers:

2
3

FG IJ
HK

3
1
2. Limit = , find f
and examine whether limit
2
2
3I
F
= value G cont. at J .
H
2K

af

f x =

Type 3: Problems based on finding the value of a


function at a point where the function is continuous.
Exercise 5.3

xa
x

,xa

2
3

x = a. Find f (a).
[Ans: 3a ]
x tan x
, x 0 is continuous at x = 0.
2. f x =
1 cos x
Find f (0).
[Ans: 2]

1 cos x

7. If f (x) is continuous at x = 3 and is defined as

af

1
]
2

x 9
2

x 4x + 3

+ a , when x < 3

= 2, when x = 3
2

x 3x
2

x 2x 3
Find a and b

+ b , when x > 3

8. f (x) is continuous at x =

af

is continuous at

[Ans: 3a ]
2

[Ans: A =

f x =

af

, when x 0
2
x
f (x) = A, when x = 0
Find A so that f (x) is continuous at x = 0.

f x =

3
; (discont. at x = 4)
2
5. (l.h.l = r.h.l at x = 2.5) = 2; find f (2.5) and examine
whether limit = value.

x a

3. l.h.l = 2 and r.h.l =

bg

6. A function f (x) is defined by

4. Show that f (x) = x [x] is discontinuous at x = 1.


5. Test for continuity of f (x) = [x] at x = 2.5.

1. f x =

Find f (a).

x
f x =
x2.

1. Cont. at x =

a f a1 sin2 xfx , x 2 is continuous at

F I .
[Ans: 1 ]
x = . Find f
H
K
2
2
8
sin x 1

,x
4. f a x f =
is continuous at
2
cos x

F I .
[Ans: 1 ]
x = . Find f
H
K
2
2
2
x a
5. f a x f =
, x a is continuous at x = a.
3. f x =

[Ans: a = 1 and b =

and is defined as
4

cos x sin x
+ a , when 0 < x <

4
x
4

= 2, when x =

= x2 + b, when x >

5
]
4

Practical Methods on Continuity test

Find a and b.
2

[Ans: a = 2 +

2 and b = 2

]
16

9. f (x) is continuous at x = 0, where

bg

sin kx
, for x 0
x
= 4 + x, for x = 0
Find the value of k.
f x =

303

f (1) = k
If f (x) is continuous at x = 1, find k.
[Ans: k = 5]
5. A function f (x) is defined as under
f (x) = x2 + A, when x 0
f (x) = x2 A, when x < 0
what should be A so that f (x) is continuous at
x = 0.
[Ans: A = 0]

[Ans: k = 4]

Type 4: Problems based on finding the value of a


constant when the given function is continuous at a
point.

Type 5: Problems based on test for continuity at one/


both end points (l.e.p / r.e.p / l.e.p and r.e.p both) of a
closed interval [a, b] or semi open-semi closed (a, b]
or semi closed and semi open [a, b).

Exercise 5.4

Exercise 5.5

1. Find the value of k for which the functions defined


below is continuous at x = 1.

af

x 1
(a) f x =
, for x 1
x 1

f (1) = k
(b) f (x) = 5x 3k, if x 1
= 3x2 kx, if x > 1
(c) f (x) = 3x 4k, when x 1
= 2kx2 3, when x < 1

af

[Ans: k = 2]
[Ans: k = 1]
[Ans: k = 1]

x 3x + 2
, when x 2
2. If f x =
x2

= p, when x = 2
and f (x) is continuous at x = 2, find the value of p.
[Ans: p = 1]

af

a sin 2 x
, when x > 0
x
= 2, when x = 0

3. If f x =

2b

j , when x < 0

1+ x 1

x
and f (x) is continuous at x = 0, find the values of a
and b.
[Ans: a = 1 and b = 2]

af

4. f x =

3x x 2
, for x 1
x 1

1. If f (x) = 0, when x = 0

1
1
x , when 0 < x <
2
2

1
1
, when x =
2
2

then test the continuity of f (x) at x = 0 and x =

1
.
2

[Ans: discont. at x = 0 and x =

1
]
2

2. If f (x) = x2 + 1, when 0 x < 1


= 3x2 1, when 1 x 2
then test the continuity of f (x) at x = 0, 1 and 2
[Ans: cont. at x = 0, 1 and 2]
3. A function f (x) is defined as follows
f (x) = 0, when x = 0
= 3x 1, when 0 < x < 1
= 2x, when x = 1
Is f (x) continuous at x = 0 and x = 1?
[Ans: discont. at x = 0 and cont. at x = 1]
4. Test the continuity of f (x) at x = 0, 1, 2 where
(i) f (x) = 1 + x, when 0 x < 1
= 2 x, when 1 x 2
[Ans: cont. at x = 0 and 2;
discont. at x = 1]

304

How to Learn Calculus of One Variable

(ii) f (x) = x2, when 0 x < 1


= 2x 1, when 1 x < 2
= x + 3, when x 2
[Ans: cont. at x = 0 and x = 1
and discont. at x = 2]
5. A function f (x) is defined as follows:
f (x) = 0, when x = 0

1
1
x, when 0 < x <
2
2

1
1
, when x =
2
2

3
1
x , when < x < 1
2
2
= 1, when x = 1, discuss the continuity of the
=

function for x = 0,

1
and 1.
2

[Ans: discont. at all points x = 0,

1
, 1]
2

Derivative of a Function

305

6
Derivative of a Function

Question: What is differential Calculus?


Answer: Differential Calculus provides us rules and
methods for computing the limit:

f af
a f

f x + x f x
y
= lim
, for a large
x

0
x
x
class of functions where, y = f x + x f x is
the increment in the value of the function
y = f x (or in dependent variable y),
x = x + x x is the increment in independent
(or in the valueof independent) variable and
lim

x 0

af
a f

af

y
= increment (or incremental) ratio
x
=

The Domain of a Derivative

increment in functional value dependent variable


increment in independent variable

Question: What is the derivative of a function?


Answer: The derivative of a function f is that function,
denoted by f , whose function (functional) value at
any limit point x of the domain of the function f which
is in the domain of the function f, denoted by f x ,
is given by the limit of the incremental ratio as the
increment in the independent variable tends to zero.
That is in notation,

af

af

f x = lim

x 0

Notes:
1. The derivative f of a function f is also termed as:
(i) slope function (ii) derived function (iii) differential
coefficient of the function f.
2. Instead of saying that there is a derivative f of
the function f at a point x = a in the domain of the
function f which is also a limit point in the domain of
the function f it is common to say that there is a
derivative f of the function f at a point x = a in the
domain of the function f defined by the equation
y = f (x).

f af

f x+ x f x
y
= lim
,
x 0 x
x

provided that this limit exists at each limit point x of


the domain of the function f which is in the domain of
the function f defined by the equation y = f (x).

The domain of a derivative is defined with respect to


different aspects:
Definition 1. (In terms of limit points):
f : D R is a function defined by y = f (x), where D
is a subset of reals domain of the derivative f
of the function f = D D D , where D is the set
of all limit points of the domain D of the function f.
Definition 2. (In terms of existance of the limit of
incremental ratio):
D f = x D f : f x exists , i.e. the domain
of a derived function y = f x is a subset of the
domain of the function f because it (domain of f )
contains all points x in the domain of the function f

a f l

where lim

x 0

a f af

af

y
exists but does not contain those
x

exceptional points where lim

x 0

y
does not exist.
x

306

How to Learn Calculus of One Variable

The Range of a Derivative


The range of a derivative is the set of all values of the
derivative f (of a function f ) at values of x in the
domain of the function.

b a fg l a f

a fq .

Thus, range of f = f D f = f x : x D f

Notes:
1. Derivatives at isolated points are not defined.
2. A function is always continuous at an isolated
point of the domain of the function.
3. In fact, if the domain of a function has an isolated
point, then the function would be continuous there
without being differentiable.
4. If the limit point of the domain D of the function f
lying in D at which derivative is sought is not
mentioned, it is understood that it is required at any
limit point x of the domain of the function f which is in
the domain of f.
5. Instead of saying to determine the functional value
f (x) of the derivative f of a function f at
x D f , it is common to say to determine the
derivative y = f x of the function y = f (x)

af

af

Question: What is differentiation of a function?


Answer: It is the process of finding the derivative

af

f x = lim

x 0

y
x .

Question: How is the derivative of a function


generally determined?
Answer: The derivative of a function of the
independent variable x, say f (x), is determined by the
general process indicated in the limit of increment
ratio as the increment in the independent variable
tends to zero, i.e. indicated in:

af

af

f x + x f x
y
dy
d
or
= lim
f x = lim
.
0
0

x
x
x
x
dx
dx

Question: What are the symbols to express the


derivative of a function with respect to an independent
variable?
Answer: D is the most common notation for
indicating the operation of obtaining the derivative
of a function with respect to an independent variable
with the attentions: (i). Functional letter or functional
value of the function is written on the right side of the

symbol D. (ii) the independent variable with respect


to which differentiation is to be carried out is written
at the bottom and on the right side of the symbol D.
Thus Dx y, Dx f, Dx f (x) or Dx (f (x)) if f (x) is the
expression containing more than one term or f (x) is
the sum of a finite number of function.
Note: The phrase with respect to x is shortly written
as w.r.t x.
An other notation

d
is also in frequent use when
dx

either it is mentioned or obvious in the problem that x


is the independent variable of the function y = f (x).

d
is used to indicate the
dx
derivative of a function having x as its independent
variable, the function or the functional value is written
d
on the right side of the symbol
. Thus:
dx
d f d
dy
d
,
f x,
or
f x if f (x) is the
dx dx
dx
dx
expression containing more than on term or f (x) is the
sum of a finite number of function.
Hence in the light of above explanation, Dx and

When the symbol

af

b a fg

d
are the most common notations for indicating
dx
the operation of obtaining the derivative of a function
with respect to x. These are notations which are
prefixed to the functional letter or functional value as:

af

(i) Dx y , Dx y ,

af

dy
d
and
y
dx
dx

af

(ii) Dx f , Dx f x ,

af

d f
d
and
f x Lastly, the
dx
dx

third notation employed for indicating the operation


of obtaining the derivative is to put a dash or prime at
the top and on the right of the functional letter as
f x , f or y , but it is noteable that the notation
f or y has the disadvantage of not indicating the
variable with respect to which the differentiation is to
be carried out. This is why f x or y x is also
written if the independent variable is x.

af

af

af

Derivative of a Function

af

d
, Dx ( ) or D ( ) is termed as operator
dx
which tells what operation should be carried on with
the function put in the bracket to get an other function
named as derivative but D ( ) alone is ambiguous
when the expression in x representing the functional
value contains constant as well as an independent
variable both put after the operator D as D (x2 + x +
K) which has a possibility or ambiguity of being which
one of the letters namely x and K is constant. This is
why D (f (x)) is also sometimes used instead of Dx
(f (x)) when there is no ambiguity of the letter used as
an independent variable.
Remark:

d
, Dx or simply D is also called differential
Note:
dx
operatir.
Therefore, there are following identical symbols
frequently used:
(i) y =

af

af

b a fg

dy
d
d
d
=
y =
f x =
f x =
dx dx
dx
dx

af

af

Dx f x = f x , when y = f (x) or

(ii) f =

d f
to denote the derivative as an other
dx

function f of a function f.
The Nomenclature of the Symbols
Notation
lim

xa

af

lim

x0

Dx

af

d
or
dx
f

af

Read as
limit as eKs tends to a, of ( )
or limit of ( ) as eKs tends to a.
limit as delta-eKs tends to zero,
of ( ) or limit of ( ) as delta-eKs
tends to zero.
x-derivative of ( ) or derivative

af

Notation

Read as

y prime or y dash

af

307

f x

f prime of x or f dash of eKs.

Dx f

dee - eKs of f

Dx f (x)

dee - eKs of f of eKs

d f
dx

dee - dee - eKs of f or d f over dx

af

d
f x
dx

deedeeeKs of f of x or dee f of
eKs over dee - eKs

dy
dx

dee - y over dee - eKs

bg

Note: The notation f x

af

x = a or (f (x)) x = a or

f a signifies the value of the derivative f of the


function f at x = a D f .

af

Question: What do you mean by ab-initio


differentiation?
Answer: Differentiation ab-initio (or ab-initio
differentiation), differentiation from the first principle
or differentiation from the delta method means that
the theorems on differentiation or the results on
differentiation of standard forms of the function are
not to be applied for obtaining the derivatives of given
functions.
Or, alternatively, finding the derivative of a function
of an independent variable using the definition in
terms of limit, theorems on limits and standard results
on limits is called differentiation from the definition.
This process has no practical utility but a knowledge
of it is required.
Now there is illustration of ab-initio differentiation
consisting of five steps before working out problems
on different typed of functions.
Question: Explain the general method of finding
differential coefficient of a function.
Answer: The general method of finding the
differential coefficient of a function is indicated in

of ( ) with respect to x.
f prime or f dash
(Contd.)

308

How to Learn Calculus of One Variable

af

f af

f x+ x f x
d
f x = lim
, x D f
0
x
dx
x

af

af

y
d
or, d x y = lim
which means
x0 x
Step 1: To put y = f (x) = given function
Step 2: To add y to y and x to x wherever it is
present in the given function, i.e., to obtain
y + y = f x + x .
Step 3: To find y by subtracting the first value (y)
from the second value y + y , i.e., to obtain

a
f
a y + yf y = f a x + x f f a x f
y = f a x + xf f a xf
Step 4: To divide y = f a x + x f f a x f by x,
f a x + xf f a xf
y
=
i.e., to obtain
x

Step 5: To take the limit as x 0 on both sides of

af

af

d
f x + x f x
y
=
to find d x y , i.e., to
x
x
obtain

lim

x 0

af

af

f x+x f x
d
y
y
= lim
=
dx
x x 0
x

respectively.
Remember: 1. The above method (or, process) to
find the derivative of a function is what the delta
method says to one to find the derivative of a function.

y
y
means that the ratio
tends to a
x 0 x
x
definite value which is unique as x 0 and hence
it is to be carefully noticed that on the above definition,
2.

lim

y
and
x
not of the ratio of the limiting values of y and x
which is indeterminate put in the form
we speak of limiting value of a certain ratio

F 0 I ( i.e. lim y = 0
H 0 K lim x 0

y
3. x being regarded as fixed, the ratio x will be a
function of x .
Question: How to simplify the last step in delta
method?
Answer: (a) In case of algebraic power, logarithmic
or exponential function, we go on simplifying till x,
some power of x and/a term containing x comes
as a common factor if we use expansion method.
(b) On using binomial expansion for any index for a

means lim y 0 when lim x = 0 )


(Note: The differential coefficient is shortly written
as d.c.)

= 1+ n +

a fa f
N

N.B.: (i) The common factor being x, some power


of x and/a term containing x must be separated
out of the bracket provided we use expansion method
to find the limit .
(ii) If we do not use expansion method to find the
limit of power (algebraic) logarithmic and/exponential
function, there is no need of taking x , some power
of x and/the terms containing x as a common
factor.
(iii)

b f a xfg = b f a x + xfg
n

when x 0

(Note: If y be a function of x (i.e. y = f (x)) and the


value of x changes, then the value of y will also
change.)
(c) In case of trigonometric functions, we go on
x
except for tan x and
2
cot x where we get sin x and then we divide both
sides of the equation defining y by x and lastly
we take the limit as x 0 on both sides of the

simplifying till we get 2 sin

y
.
x
(d) In case of inverse circular functions, we find x
instead of y , i.e. in case of inverse circular functions,
first of all we remove inverse operators (or, notations)
and we proceed as in case of trigonometrical functions
equation defining

since y 0 as x 0

n n +1 x
2
+ ... , we consider only two terms of the series viz
1 + n x that will serve our purpose while finding
limit.
power function 1 + x

Derivative of a Function

and lastly we take limit as y 0 (instead of limit


as x 0 in case of direct functions or
trigonometrical functions) on both sides of the
equation defining

y
since x 0 y 0 .
x

lim

sin r
= r and

tan r
= r which mean limit of sin (or, tan)

of any constant multiple of an angle over the same


angle when that angel tends to zero, is the same
constant which is the multiple of the angle or, we use

(b) lim

tan
sin
= 1 which mean
= 1 (b1) lim

limit of sin (or, tan) of any angle over the same angle
when that angle tends to zero, is unity. e.g.,

(a1) lim

aa f
a b f

2 sin
lim

sin
lim

x0

F xI
H2K=

x 0

sin
lim

F xI
H 2 K =1

x
2

x 0

F xI
H 2 K = 1 or

sin
lim

x 0

F xI
H2K

F xI
F xI
sin
sin
H
K
H 2 K = 1 1 = 1
1
2
lim
= lim
=
F xI 2
F xI 2 2
2
H2K
H2K
acf lim sin a3 xf = 3 or, lim sin a3 xf
x 0

x 0

x 0

x 0

a f
a f
a f
a f
ad f lim tan anx x f = n, n R , or

3sin 3 x
sin 3 x
= lim
= 3 lim
= 3 1 = 3
x 0
3 x
3 x
x0

x 0

x 0

a f

a f
a f

tan n x
n tan n x
= lim
x0
x
n x

a f
a f

tan n x
=n
x 0
n x

= n lim

af

e x and y are increments in x and y and they


are regarded as single quantity like x, y, z, ... etc. This
is why they may be added, subtracted, multiplied and
divided like the numbers. e.g.,

N.B.: (i) we use


(a)

lim

309

(i)

x
1
= x y = x
y
y

(ii)

x y

=1
y x

(iii)

x
=
y

1
y
x

FG IJ
H K

Derivatives of Elementary Functions


Algebraic Functions
1. Find the derivative of a constant function y = c by
method. (or by first principles)
Solution: Let y = f (x) = c
(i)

y + y = f x + x = c

(ii)

Hence, (i) (ii) y = c c = 0

y
=0
x

y
d
= lim 0 = 0 3 lim c = c
y = lim
x 0 x
x 0
xa
dx

af

F
H

I
K

derivative of a constant is zero.

af af
constant, D a f f = R .

d
d
y =
c = 0 , provided y = c = any
dx
dx

2. Find the derivative of an independent variable w.r.t


itself by delta method (or ab-initio).
Solution: Let y = f (x) = x
(i)

310

How to Learn Calculus of One Variable

y + y = f x + x = x + x

...(ii)

Hence, (i) (ii) y = x + x x = x

y
=1
x

y
d
= lim 1 = 1 3 lim c = c
y = lim
x 0 x
x 0
xa
dx

F
H

af

bg

d
x = 1, provided y = x = identity
dx
function, D f = R .
3. Find the derivative of a constant multiple of an
independent variable w.r.t the same independent
variable.
Solution: Let y = f (x) = ax
(i)

is unity

a f

f a

y + y = f x + x = a x + x
Hence, (i) (ii)

(ii)

y = a x+ x ax = a x + a x a x = a x

=a
x

F
H

af

I
K

y
d
y = lim
= lim a = a 3 lim c = c
x 0 x
x 0
xa
dx

derivative of a constant mulitple of the


independent variable w.r.t the same independent
variable is constant times d.c. of the independent
d
d
y =
ax
variable w.r.t itself
dx
dx

af

a f

af

d
x = a 1 = a , provided y = a x, where a is
dx
any constant and x is the independent variable,
=a

a f

D f = R.
4. Find the derivative of a power function w.r.t its
base.
Solution: Method 1
Let y = xn, n Q
(i)

y + y = x + x

(ii)

Hence, (i) (ii) y = x + x

a3 x = x + x xf
a

af

x + x x
y
d

y = lim
= lim
x0 x
x0
x + x x
dx

I
K

derivative of an independent variable w.r.t itself

x + x x
y

=
x
x + x x

Now substituting z = x + x , we have z x as

x 0
n

= lim

zx

z x
z x

af

x + x x
d
y = lim
x 0
dx
x + x x

and
n

= nx

n 1

(By limit theorem)

derivative of a power function w.r.t its base is


the power function whose index is decreased by unity
times the original (given) index of the given base

af

e j

n
n 1
d
d
y =
x = n x , provided y = x n ,
dx
dx

n Q .
Method 2
Let y = xn, (n is any rational number)

y + y = x + x
Hence, (i) (ii)

(i)

y = x + x x = x

(ii)

R|SFG1 + xIJ
T|H x K

U|V
W|

1 (iii)

Now on using binomial theorem because

x
< 1 and n is a +ve or ve integer or fractions,
x
we have
(iii) = y = x n

LMF
x n bn 1g F x I
MNGH1 + n x + N2 GH x JK

LM
N

a f FG IJ
N H K
O
having higher powers of x P
Q
which

I
JK

OP
PQ

+ ... 1

n n 1
y
x
n n
+ terms
=x
+

2
x
x
2
x

af

y
d
y = lim
x 0 x
dx

311

Derivative of a Function

LM n + n an 1f FG x IJ + ...OP
N x N2 H x K Q
LM1 + an 1f FG x IJ + ...OP
nx
N N2 H x K Q

= lim x
x 0

= lim

x 0

FG3 lim a1 + xf
H

x0

e j

d n
x = nx n 1 for x > 0, n R also holds
dx
true noting that for x < 0, xn may not be real as

b3g

1
2

3 R .

Logarithmic and Exponential Functions


1. Find the derivative of logarithm of an independent
variable w.r.t the same independent variable, the
independent variable being positive (d.c. of
logarithmic function) by delta method.
Solution: Method 1.
Let y = log x, x > 0
(i)

y + y = log x + x

(ii)

Hence, (i) (ii) y = log x + x log x

F x + x IJ = log FG x + x IJ = log FG1+ x IJ


= log G
H x K Hx x K H x K

FG
H

FG
H

IJ
K

y
1
x
1
1
x
=
= x
log 1 +
log 1 +
x
x
x
x
x
x

FG
H

x
1
= log 1 +
x
x

IJ
K

x
x

x 0

x
x

x 0

= lim

x 0

IJ
K

OP
PPQ

x 0

F 1 I log
H xK

lim

x 0

FG1 + x IJ
H xK

composit function rule on limits)

1
1
1
= log e = log e e = 1
x
x
x

x
x

1
which derivative of logarithm of an
x
independent variable w.r.t the same independent
variable, the independent variable being positive, is
reciprocal of the independent variable, the
independent variable being positive

b g
D a f f = R

Remember:
(i) n log m = log mn
(ii) x 0

x
0
x

lim
(iii) xlim
0 log f (x) = log x 0 f (x)

lim

x
0
x

FG1 + x IJ
H xK

x
x

=e

(v) log e = log e e = 1, where e lies between 2 and 3.


These are facts which have been used to find the
d.c. of y = log x, x > 0, w.r.t x.
Method 2
Let y = log x , x > 0
(i)

y + y = log x + x

(ii)

Hence, (i) (ii) y = log x + x log x

FG x + x IJ = log FG1 + x IJ
H x K H xK
x 1 F xI
y =
G
J + 13 FGH xx IJK ...
x
2H x K
y 1 1 F xI

= G
J
x
x 2 H x K + terms having higher
= log

(on using

bg

d
y =
dx

1
d
log x = , x > 0 provided y = log x, x > 0 ,
x
x

(iv)

x
x

L 1 F xI
d

y f = lim M log G1 +
a
MMN x H x JK
dx
F 1 I lim log FG1 + x IJ
= lim
H xK
H xK

= e and log e = log e e = 1

n 1

= n xn 1 , n Q
Remark:

IJ
K

1
x

powers of x .

312

How to Learn Calculus of One Variable

LM
N

af

FG IJ OP
H K Q

y
d
1 1 x
= lim

+ ...
y = lim
x 0 x
x 0 x
dx
2 x2

af

y 1
d

=
y = lim
x 0 x
dx
x

af

a f

Cor: Derivative of log a x (a > 0, 1), x > 0

d
=
dx

d
log x
log a x
, we have
dx
log a

FG log x IJ = 1
H log a K log a
af

a f

af

log f x
where as
log a

e =1+

x x
x
+
+ + ... where
1 2
3

N N N

1
1
1
+
+
+ ... and x is a real number;
1
2
3

N N N

F
H

e is also defined as lim 1 +


n

1
n

I
K

Remark: e is a transcendental irrational number.


2 < e < 3 and e = 2.718 nearly.
2. Find the derivative of the function y = log |x|,
(x 0) w.r.t x abinitio.
2

Solution: y = log x = log x =

y + y =

log x

x + x
1
log
x
2

d
1
1
log x =

dx
log a x

log10 f (x) = log10 e loge f (x) and log10 e = 0.4344


2. If the base of the logarithm of a function is e, then
the logarithm of the function is called natural
logarithmic function and is denoted as lnx without
the base e. Hence, loge x = lnx.
3. Meaning of e : ex means the infinite series (or,
infinite power series) 1 +

1
=
, (x > 0) ( 3 log a is a constant)
x log a
Notes: 1. log a f x =

1
log x + x
2

IJ
FG
K
H
FG3 log f a xf = log f a xf log g axfIJ
H g axf
K
y 1 1
F x + x IJ

=
log G
H x K
x 2 x
1 x 1
F x + x IJ
=
log G
H x K
2 x x
R|F x + x I U|
1 1
y
= log SG
lim
J
V
and
H
K
2 x
x
|W
|T x
LM 1 1 R| x + x U| OP
F
IJ V P
= lim M log SG
H
MN 2 x |T x K |W PQ
=

d
d
1
y =
log x = , x > 0
dx
dx
x

Using log a x =

y =

1
2
log x
2

1
log x + x
2

x
x

x 0

x
x

x 0

R|F x + x I
S|GH x JK
T
R|F x + x I U|
1 1
= log lim SG
J
2 x
|TH x K V|W
R| F x + x I U|
1 1
= log S lim G
J
2 x
|T H x K V|W
R| F x I U|
1 1
= log S lim G1 +
J
2 x
|T H x K V|W
F
I
1 1
= log aef G3 lim a1 + x f = eJ
H
K
2 x
F I
H K

1
1
= lim
lim log
2 x 0 x x 0

x
x

x
x

x 0

x0

x
x

x 0

x0

1
x

x
x

U|
V|
W

313

Derivative of a Function

1 1
1 1
1 1
2
log e = 2 log e = 2 1
2 x
2 x
2 x

1
, x 0, D f = R 0
x

a f

kp

Notes: A.

bg

(i) log f x

= log

b xg

af

1
2
log f x
2

loga

y + y = e
Hence, (i) (ii)

1
log a m .
n
Remark: log x2 2 log x for all x 0, because x 0
either x > 0 or x < 0 whereas log x2 = 2 log x only
when x > 0 as (B) (iii) Says. This is why log e2 = 2 log
e since e>0.
log a n m =

Cor: Derivative of loga |x|, (a > 0, 1), x 0 on using

d
log x
log a x
, we have
dx
log a

FG log x IJ
H log a K
1
d
1
1
=

log x g =

b
log a d x
log a x
=

d
dx

x + x

x + x

F
GH

(ii)

e =e e
x

e =e

I
JK

ee

x e
1
y
=e
x
x

x e
y
d
1
= lim e
y = lim
x 0 x
x 0
x
dx

x
d
e 1
y = lim e lim
x 0
x0
x
dx

(iii) The logarithm of the power of a positive number


is equal to the exponent times the logarithm of the
base:
loga mn = n loga m, n R
(iv) The logarithm of a root of a positive number is
equal to the logarithm of the number divided by the
index of the root:

log a x =

y = e

F mI = log m - log n
H nK
a

fa

2. Find the derivative of an exponential function w.r.t


the index, the index being an independent variable
(d.c. of exponential function) by delta method.
Solution: (a) Method 1
Let y = ex
(i)

1
2
log x
2
B.: (i) The logarithm of the product of positive
numbers is equal to the sum of the logarithms of the
factors:
loga (mn) = loga m + loga n
(ii) The logarithm of the quotient of two positive
numbers is equal to the logarithm of the dividend
minus the logarithm of the divisor:

(ii) log x = log

1
, x 0 3 log a is a constant
x log a

F
GH

af

F
GH

I
JK
F3 lim F e 1I = log e = 1I
GH
JK
GH x JK

af

= ex 1

I
JK

x0

= ex which derivative of an exponential function


with base e w.r.t the index, the index being an
independent variable, is itself the exponential function
without any change in the given form

af

e j

x
x
d
d
y =
= e = e , provided y = ex .
dx
dx

Notes to Remember: One must remember that terms


having no x must be regarded as constants when

x 0 . This is why we write


lim ex = ex

(i)

x0

(ii)

lim xn-1 = xn-1, etc

x 0

Method 2
Let y = ex

(i)

314

How to Learn Calculus of One Variable

y + y = e
Hence, (i) (ii)

x + x

(ii)

j
a xf + a xf + ... 1IJ
e F
y

=
1+ x +
G
x
x H
N2
N3
K
y = e

x + x

e = e e

e = e

F
GH

x
y
e
=
x +
x
x
2

y
x

a f + a xf
N
N3

ee

I
JK

+ ...

FG1 + x + terms having higher powers of x.IJ


H N2
K
d

a yf = lim yx = lim e FGH1+ N2x + ...IJK = e


dx
=e

x 0

x 0

(b) Let y = ax, x R , a > 0


y + y = a

(i)

x + x

(ii)

Hence, (i) (ii)


x + x

y = a

x
y
=a
x

a = a a

a =a

F a 1I
GH x JK

ea

af

F
GH

x a
y
d
1

= lim a
y = lim
x 0 x
x 0
x
dx

x 0

= a log e

F a 1I
GH x JK

F
a G3
H

lim

x0

a 1
= log e
x

= a l n a 3 log e a = l n a
x

I
JK

= lim a lim
x 0

I
aJ
K

Which derivative of an exponential function


with base a > 0 w.r.t the index, the index being an

independent variable, is the exponential function with


d
the same base a > 0 times ell en a d x (y) =
d
x
x
x
d x (a ) = a ln a, provided y = a , x R , a > 0 and
Remember: (i) The function f (x) = ax, where a is any
positive real number and x is any real number (i.e.
a > 0, x R ) is called the general exponential function
with base a.
(ii) The function f (x) = ex for all x R is called
natural exponential function or simply exponential
function.
(iii) f (x) = ln x is called ell en (,y ,u) function or
natural logarithmic function which is alternation to
log x.
(iv) We should note that d.c. of
(a constant) independent variable
= (the constant) the independent variable
times log (the constant), where constant stands for
any positive real number (i.e.; e or a)

d
Hence, d x (ex) = ex loge e = ex log e = ex 1 = ex and
d
x
x
x
d x (a ) = a loge a = a log a
(Note: In calculus, when no base of logarithmic
function is mentioned, it is always understood to be
"e". Hence, loge x = log x = lnx)
Trigonometrical Functions
1. Find the derivatives of all elementary trigonometrical functions w.r.t their independent variables by delta
method.
Solution: 1. Differential coefficient of sin x w.r.t x
Let y = sin x
(i)

y + y = sin x + x

(ii)

Hence, (i) (ii) y = sin x + x sin x

FG x + x + x IJ sin FG x + x + x IJ
H 2 K H 2 K
F x IJ sin FG x IJ
y = 2 cos G x +
H 2K H2K
= 2 cos

315

Derivative of a Function

FG
H

IJ
K

(ii) Differential coefficient of cos x w.r.t x


Let y = cos x

y
x
x 1
= 2 cos x +
sin

x
2
2 x

y + y = cos x + x

F xI
sin
H2K
xI
F
= cos G x +
H 2 JK x

FG x + x + x IJ sin FG x x x IJ
H 2 K H 2 K
F x IJ sin FG x IJ
y = 2 sin G x +
H 2K H 2K
F x IJ sin FG x IJ
= 2 sin G x +
H 2K H2K
F xI
sin
y

x
F
IJ H 2 K

= 2 sin G x +
H 2 K x
x
F xI
sin
F x IJ H 2 K
= sin G x +
H 2 K F xI
H2K

af

x 0

FG
H

x 0

IJ
K

y
x
= lim cos x +

2
x x 0

FG
H

= lim cos x +

IJ
K

x
lim
x 0
2

sin

sin

F xI
H2K

x
2

F xI
H2K

x
2

= cos x 1 = cos x which derivative of sine of


an angle w.r.t the same angle is cosine of the same
d
d
y =
sin x = cos x .
angle
dx
dx
Notes to Remember: 1. While finding the d.c. of
trigonometrical functions using delta method, one
must remember that trigonometrical function which
becomes zero on putting x = 0 is modified by
writing

af

a f

af

y
d
y = lim
x0 x
dx

FG
H

lim sin x +
x 0

sin
tan
(ii) tan =
so that

standard formulas of limits of trigonometrical


functions.
tan
(a) lim = sin = 1 (b) lim =
= 1 may be

used

F C + D IJ cos FG C D IJ
sin C sin D = 2 cos G
H 2 K H 2 K

= sin x 1 = sin x

used to find y in simplified form while finding d.c.


of trigonometrical functions (particularly sin of an
angle and co sine of an angle).

F
GG3
GH

sin

F xI
H2K

x
2

F xI I
H 2 K = 1JJ
F xI J
H2K K

sin
lim

x 0

which derivative of cosine of an angle w.r.t the


same angle is negative of sine of the same angle

bg

b g

d
d
y =
cos x = sin x .
dx
dx
Notes: One must remember that

is

IJ
K

x
lim
x 0
2

(i) sin =

2.

= 2 sin

d
y
dx

= lim

(ii)

Hence, (i) (ii), y = cos x + x cos x

(i)

(i) cos C cos D = 2 sin

FG C DIJ sin FG C + DIJ


H 2 K H 2 K

316

How to Learn Calculus of One Variable

a f

(ii) sin = sin are used to find y in


simplified form while finding d.c. of cos x w.r.t x.
(iii) Differential coefficient of tan x w.r.t x
Let

y = tan x , x n +

y = tan x + x
Hence, (i) (ii)

(i)
(ii)

a
a

f
f

sin x + x
sin x

cos x + x
cos x

y =

a
a

sin x + x x
cos x + x cos x

y =

a
a

y =

(ii)

y
sin x
=
x x cos x + x cos x

y
d
y = lim
x 0 x
dx

af

f
f

cos x + x sin x sin x + x cos x


sin x + x sin x

a
a

sin x x x
sin x + x sin x

y =

(i)

cos x + x
cos x

sin x + x
sin x

sin x
cos x + x cos x

y + y = cot x + x
Hence, (i) (ii)

cos x sin x + x sin x cos x + x


cos x + x cos x

(iv) Differential coefficient of cot x w.r.t. x


Let y = cot x, x n

y = cot x + x cot x

y = tan x + x tan x
=

Notes to Remember: One must remember that (i) sin


(A B) = sin A . cos B cos A sin B is used to find y
in simplified form while finding d.c. of tan x w.r.t. x

a f
a f

sin x
sin x
=
sin x + x sin x
sin x + x sin x

y
sin x
=
x
x sin x + x sin x

y
d
y = lim
x

0
x
dx

af

LM sin x 1 1 OP
N x sin x sin a x + xf Q
F sin x IJ lim FG 1 IJ lim
F sin x IJ lim FG 1 IJ lim FG 1 IJ
= a 1f lim G
= lim G
H x K H cos x K H cosa x + xfK
H x K
H sin x K
sin
1
1 F
I
FG 1 IJ
= 1

= 1J
3 lim
G
H
K
cos x cos x

H sin a x + xf K
1
F 1 IJ FG 1 IJ
=
= sec x which derivative of
= a 1f a 1f G
cos x
H sin x K H sin x K
= lim

x 0

x 0

LM sin x 1 1 OP
N x cos x cos a x + xf Q
x 0

a f

= 1

lim

x 0

x 0

x 0

x 0

x 0

tangent of an angle w.r.t the same angle is square of


secant of the same angle

sec 2 x provided y = tan x

bg

b g

d
d
y =
tan x = ,
dx
dx

a f FG 1 IJ = cosec
H sin x K

= 1

Derivative of a Function

which derivative of cotangent of an angle w.r.t


the same angle is minus one times square of

af

d
d
y =
cot x
cosecant of the same angle
dx
dx
= cosec2 x, provided y = cot x.

Remember: (i) sin (A B) = sin A cos B cos A sin


B is used to find y in simplified form while finding
d.c. of cot x w.r.t. x.
(v) Differential coefficient of sec x w.r.t. x
Let

y = sec x, x n +
2

y + y = sec x + x
Hence, (i) (ii)

(i)

1
1

cos x + x
cos x

f
cos x cos a x + x f
y =
cos a x + x f cos x
F x I sin F x + x x I
2 sin x +
H 2K H 2 K
=
cos a x + x f cos x
F x I sin F x I
2 sin x +
H 2K H2K
y

=
x
x cos a x + x f cos x
F x I sin F x I
2 sin x +
H 2K H2K
=
F xI 2
cos a x + x f cos x
H2K
F sin F x I I F sin F x + x I I
H 2 K JJ GG H 2 K JJ
y G

=G
x G F x I J G cos a x + x f cos x J
H H2K K H
K

af

y
d
y = lim
x 0 x
dx

x0

FG 1 IJ
H cos x K

lim

x0

lim

x0

F sin F x + x I I
GG H 2 K JJ
GH cosa x + xf JK

sin x I
a f FGH cos1 x IJK FGH cos
J = sec x tan x
xK

= 1

which

derivative of secant of an angle w.r.t the same


angle is secant of the same angle times tangent of the

af

d
y
dx
provided y = sec x.

same angle

d
sec x sec x tan x,
dx

(ii)

y = sec x + x sec x

= lim

F sin x I
GG 2 JJ
GH 2x JK

317

Remember: 1. cos C cos D = 2 sin

FG D C IJ sin
H 2 K

FG C + D IJ is used to find y in simplified form while


H 2 K

finding d.c. of sec x w.r.t x.

(vi) Differential coefficient of cosec x w.r.t. x


Let y = cosec x, x n

y + y = cosec x + x
Hence, (i) (ii)

(i)
(ii)

y = cosec x + x cosec x

1
1

sin x + x
sin x

f
sin x sin a x + x f
y =
sin a x + x f sin x
F x I sin F x x x I
2 cos x +
H 2K H 2 K
y =
sin a x + x f sin x
F x I F sin x I
2 cos x +
H 2K H
2 K
y =
sin a x + x f sin x
F x I sin F x I
2 cos x +
H 2K H2K
y

=
x
F xI 2
sin a x + x f sin x
H2K
=

318

How to Learn Calculus of One Variable

F x I sin F x I
H 2K H2K
y

=
x
F xI
sin a x + x f sin x
H2K

for finding their derivatives have been recapitulated


in the chapter Chain rule for the derivatives. This is
why it is advised to consult those chapters.

cos x +

Remarks: 1. In particular, if x = a is a point in the


domain of a function defined by a single formula

af

= lim

x 0

y = f (x), then lim

F sin x I
GG 2 JJ
GH 2x JK

lim

x 0

FG 1 IJ
H sin x K

af

af

lim

by f a . Hence, if f x =

x 0

F cos F x + x I I
GG H 2 K JJ
GH sin a x + xf JK
xI
a f a f FGH sin1 x IJK FGH cos
J
sin x K

2x

af

, then f 2

F 1
G 2 a2 + hf
f a 2 + hf f a 2 f
= lim
= lim G
h
GG h
H
8 a2 + hf
= lim
16 a2 + hf h
h 0

h 0

1
16

I
JJ
JJ
K

= 1 1

= cosec x cot x which derivative of


cosecant of an angle w.r.t the same angle is minus one
times cosecant of the same angle times cotangent of

af

af

f a+h f a
, h is a small
h0
h
positive number, is called the differential coefficient
of f (x) at x = a provided this limit exists and is denoted

d
y
dx

d
d
cosec x
the same angle
y =
dx
dx
cosec x cot x, provided y = cosec x.

h0

= lim

8 8 + 12 h + 6 h + h

16 h 2 + h

h0

Recapitulation: The derivative of every co-function


(i.e., cos, cot, cosec) can be obtained from the
derivative of the corresponding function (i.e., sin, tan,
cos) by (i) introducing a minus sing, and (ii) Replacing
each function by its co-function. Hence applying this

d
d
rule to =
(sec x) = sec x tan x, we get =
dx
dx
(cosec x) = cosec x cot x, and so on.
Inverse Circular Functions
Question: Find the derivatives of inverse circular
functions with respect to their independent variables.
Answer: The derivatives of inverse circular functions
with respect to their independent variables have been
derived and discussed in the chapter Derivatives of
Inverse Circular Functions in detail and the formulas

= lim

h 12 + 6 h + h

= lim

h0

16 h 2 + h

h0

16 2 + h

12 + 6 h + h
3

12
3
=
16 8
32

2. In the case of functions defined by a single formula


(rule or expression in x) in a neighbourhood of a point
belonging to their domains, there is no need to
calculate left hand derivative (symbolised as L f a
or f a ) and right hand derivative (symbolised as

af
af

af

af

R f a or f + a ) separately about which, there


is discussion in the chapter differentiability at a point
in the domain of the function. Further, in the case of
piecewise functions defined in adjacent intervals
(Intervals whose left end point and right end point

Derivative of a Function

are same), it is necessary to calculate both the left


hand and the right hand derivatives at the common
point of the adjacent intervals.

12.

3. The domain of f x is a subset of points in the

Answers: 1. (i) 2, (ii) 2 x, (iii) 15 x2,

af

af

f x+h f x
,
h0
h
( h > 0 ) exists excluding all those points where this
limit fails to exist, i.e. excluding all those points where
f ' (x) is undefined.
domain of f (x) where the = lim

, (x > 0) (v) 3 x 2, (x > 0)


3

3 52
1 2
, (x > 0)
x , (x > 0) (vii) x
2
2
2. 21 x2 10 x + 4 3. 12 x 5

Problems on algebraic functions

3
x (v) x

(vi)

4.

1. (i) 2 x (ii) x2 (iii) 5 x3 (iv)

x x

1
(iv)

Exercise 6.1

Differentiate by method (read as delta method)


the following functions w.r.t their independent
variables.

6.

8.

x2 2 x 1

b x 1g

b g

, x 1 5.

(vii) x 2
x
2. 7 x3 5 x2 + 4 x +13
3. ( 2 x + 3 ) ( 3 x 7 )

, x>

x
10.

x +a
3
5

2x 2

11.

a x+b
2

x +a

1
x+a

for x + a > 0

Find the derivatives of the following functions w.r.t x


ab - initio.

x+5
7. x 3

9.

32

Problems on trigonometric functions

2 x 1

2x3

Exercise 6.2

8.

1
a+x
2

b x > 0g

1
5. x + 1

11.

a x + 1f , b x 1g

FG 1 IJ 7. 8 , b x 3g
b2 x 1g H 2 K b x 3g
1
a
F 3I
F b IJ
, G x > J 9.
, Gx >
2K
a K
2x3 H
2 a x+b H
1

12.

x +1
4.
x 1

10.

(vi)

6.

319

FG IJ
H K

x
1. sin x 2. cos x 3. sin 2x 4. sin 2

5.
2

cos x 7. cos2 x 8. sin2 x

sin x 6.

9. sin x2 10. cos


13. cot

FG x IJ
H2 K

FG x IJ
H2 K

14.

11. tan x 12. tan 2 x

sec x 15. cosec2 x

16. tan 4x 17. cosec 3x 18. cot 2x 19. tan ax

320

How to Learn Calculus of One Variable

20. sec (2x + 5) 21. sin (x2 + 1) 22. cos (ax2 +bx +c)
23. x2 cos x 24. tan2 ax
Answers:
1. cos x 2. sin x 3. 2 cos 2x
4.

5.

6.

F I
H K
cos x
c2n < x < b2n + 1g h
2 sin x

1
x
cos
2
2

FG 2n < x < 2n + IJ
2
2K
cos x H

21. 2x cos (x2 + 1)


22. (2ax + b) sin (ax2 + bx + c)
23. 2x cos x x2 sin x

FG
H

24. 2a tan ax sec2 ax; ax n +

IJ
K

IJ
K

Exercise 6.3

sin x
2

7. sin 2x
8. sin 2x
9. 2x cos x2
10.

FG
H

20. a sec (2x + 5) tan (2x + 5); 2 x + 5 n +

FG IJ
HK
FG
H

12. sec2 2 x x

IJ
K

n
+
2
4

1.

IJ
K

FG x IJ for x 2n
H 2K
I
1
F
sec x tan x G 2n < x < 2n + J
14.
H 2
2
2K
15. 2 cosec x cot x, b x n g
F n + IJ
16. sec 4 x ; G x
H 4 8K
F n IJ
17. 3 cosec 3x cot 3x; G x
H 3K
F n IJ
18. 2 cosec 2x; G x
H 2K
I
F
19. a sec ax; G ax n + J
H
2K

FG
H

3
2
x>
3x+2
3

3.

4.

FG
H

a
b
x>
a x+b
a

FG
H

IJ
K

1
3
x>
5 x + 3 log a
5

IJ
K

Exercise 6.4

IJ
K

1
2. x log a (x > 0)

1
cosec 2
2

3. log (ax + b) 4. loga (5x + 3)


Answers:

11. sec2 x x n +

13.

Find from the first principle the derivatives w.r.t x of:


1. log (3x + 2) 2. loga x a > 0 , a 1

1
x
sin
2
2

FG
H

Problems on logarithmic functions

Problems on exponential functions


Find from the definition the derivatives of the
following:
1. e2x 2. emx 3. ex 4. e x 5. e4x 6. a5 x (a > 0)
7. asin x 8. atan x (a > 0) 9. esin2x
Answers: 1. 2e2x 2. m em x 3. ex
e x

, (x > 0) 5. 4 e4x 6. 5a5x log a


2 x
7. a sin x cos x log a 8. a tan x sec2 x log a
9. 2esin2x cos 2x.
4.

Differentiability at a Point

321

7
Differentiability at a Point

To clear the concepts of differentiability of a function


at a limit point in the domain of a function. The
definitions of some other concepts connected with it
are required to be provided.
to
1. Difference function f : D on

R defined by
y = f (x) is a function and there is an attention on a
neighourhood of a limit point x = a D such that
for each value of x in neighbourhood of the limit
point x = a D there is a number f (x) f (a)
which is called the value of the difference function
for the given function f and a given limit point namely
a, in the domain of the function f. The difference f (x)
f (a) is the algebraic increment in f (a) for the
increment (x a) in the value of x at the limit point
x = a. It is customary to denote the difference function
by the symbol f = f + f f .
Further, domain of the difference function f is
the same as the domain of the function f, i.e.
D f = D f .
2. Difference quotient (or increment ratio) function:
to

R defined by y = f (x) and there is


f : D on
an attention on a neighbourhood of a limit point
x = a D such that for each x in deleted
neighbourhood of the limit point x = a D there

af

a f

af

af

f x f a
is a number
which is called the value
xa
of a difference quotient function or the value of an
increment ratio function for a given function f.
Moreover, the domain of increment ratio function is
of course the domain D of the function f with the

exception of the limit point x = a at which the


increment ratio function is not defined. Now, it can
be readily guessed that if f (x) is continuous at the
limit point x = a, then limit of the difference function
at the limit point x = a is the number zero whereas the
increment ratio function is not defined at the limit
point x = a. We are to find out whether the left-hand
and the right-hand limits of the increment-ratio
function at the limit point x = a exist or not.
The concept of the derivative of a function at a
limit point x = a in the domain of a function is defined
in various ways:
Definition 1: (In terms of neighbourhood): we say
that a fixed number f a is the derivative of the
function y = f (x) at a limit point x = a in the domain
of the function f There is a fixed number f a
such that if we choose any - neighbourhood of the
fixed number f a denoted by N f a , it is
possible to find out a -neighbourhood of the limit
point a in the domain of the function f denoted by
N a such that the values of the increment-ratio
f x f a
function g (x) =
lies in N f a
xa
for every value of x which lies in the -deleted
neighbourhood of the limit point a in the domain of f
denoted by N a = 0 < x a < .
That is a number denoted by f a is called the
derivative of a function f at a limit point x = a in the
domain of the function f There is a number
f a such that for every - neighbourhood N of
a number f a denoted by N f a , a neighbourhood of the limit point a in the domain of

af

b a fg

af

af

af

af

af

af

af

af

b a fg

af

b a fg

322

How to Learn Calculus of One Variable

af
b a fg
a f a

the function f denoted by N a such that g (x)


f x f a
=
N f a
for
every
xa
x N a = 0 < x a < = a , a a , a +

af af
af

= -deleted neighbour-N of the limit point a in the


domain of the function f.
Notes: 1. The definition of the derivative of a
function can be redefined in short in terms of deleted neighbourhood N of a limit point a in the
domain of the function f.
Definition 2: (In terms of deleted neighbourhood):
A number denoted by f a is called the derivative
of function f at a limit point x = a D f neighbourhood N of a number f a denoted by
N f a , a deleted neighbourhood N of
the limit point x = a D f denoted by N a
such that
g x
= a , a a, a +

af

af
af

b a fg
af
af
af
a
f a
f
f a x f f aa f
=
N b f aa fg , x N aa f .
xa

2. One should keep in mind that the definitions of


the limit of a function y = f (x) at a limit point
x = a D f and the derivative of function y = f
(x) at a limit point x = a D f are similar in the
following sense.
(a) The derivative of a function f at a limit point x =
a D f is the limit of the increment ratio function

af

af

bg
f a x f f aa f
g a xf =
at a limit point x = a D a f f .
xa

(b) In the definition of the limit of a function f at a limit


point x = a D f , the functional values lies in
N L , x N a whereas in the definition of
the derivative of a function f at a limit point
x = a D f , the values of an increment ratio

af

function

af

af
af

af

g x =

af

af af

b a fg

f x f a
N f a ,
xa

x N a
Definition 3: ( definition): A number f a is
the derivative of the function f at a limit point
x = a D f > 0 , > 0 ( depends on
) such that for all points x,

af

af

bg

bg bg

f x f a
f a
xa

0< xa <

<

On one Sided Derivatives


There are two types of derivatives namely (i) right
hand derivative and (ii) left hand derivative.
1. Right hand derivative:
Definition (a): In terms of neighbourhood: A function
f is said to have the right hand derivative at a point
x = a D f , which is also the right hand limit point
of D (f), denoted by f + a -neighbourhood
N of
f + a denoted by N f + a , a neighbourhood N of the right hand limit point a of
the domain of the function which is in the domain of
the function f denoted by N a such that the

af
af

af

b a fg

af

af

af af

f x f a
xa
N f + a , x which lies in a right deleted
neighbourhood N of the right hand limit point a
of the domain of the function f denoted by
N a = 0 < x a < = a < x < a +
Definition (b): (In terms of definition):
f + a is called right hand derivative of function f at
the right hand limit point x = a of the domain of the
function f which is in the domain of the function
f > 0 , > 0 ( depends on ) such that
functional values g x =

b a fg

af
af

af

af

af

af

af

af
f aa f is a

0< xa<

f x f a
f + a <
xa

i.e. a < x < a +

f x f a
f + a <
xa

Note: One should keep in mind that


notation for a finite number.

(ii) Left hand derivative:


Definition (a): In terms of neighbourhood: A function
f is said to have the left hand derivative at a point
x = a D f which is also the left hand limit point
D (f) denoted by f a - neighbourhood N
f a denoted by N f a , a of
neighbourhood N of the left hand limit point a of

af
af

af

b a fg

323

Differentiability at a Point

the domain of function f which is in the domain of the


function f denoted by N a , such that the

af
f a x f f aa f
functional values g a x f =
xa
N b f aa fg , x which lies in a left deleted

neighbourhood N of the left hand limit point a of


the domain of the function f denoted by
N a = 0 < a x < = a < x < a .
Definition (b): In terms of definition: A fixed
number f a is called left hand derivative of a
function f at the left hand limit point x = a of the
domain of the function f which is in the domain of the
function f > 0 , a > 0 ( depends on
) such that

af

af

af

af

af

af

af

af

0< xa<

f x f a
f a <
xa

i.e. a < x < a

f x f a
f a <
xa

On other forms: To find the derivative of a function f at


a limit point x = a in its domain, one should avoid making
use of neighbourhood definition or ( ) definition
for the reason that these definitions have no practical
utility. This is why other forms are used in terms of
which the derivative of a function f at a limit point x =
a in its domain is defined in the following ways.
1. Right hand derivative: If the function f is defined
at a right hand limit point x = a, then the right hand
derivative of the function f at x = a, denoted by

af

af

af

af

f x f a
f + a , is defined by f + a = lim+
xa
x a

or equivalently if x = a + h , then

bg

f + a = lim

h 0

b g , bh > 0g

f a +h f a

h
2. Left hand derivative: If the function f is defined
at a left hand limit point x = a, then the left
hand derivative of the function f at x = a, denoted by

af

or

equivalently

bg

bg

x a

if

x=ah,

f a , is defined by f a = lim

bg

f x f a
xa

then

bg

f a = lim

b g , a h > 0f .

f ah f a

h
3. Derivative of a function at a limit point in its domain:
If a function f is defined at a limit point x = a, then the
derivative of f at x = a, denoted by f a , is defined
h 0

af

af

af af

f x f a
or equivalently if x =
xa

by f a = lim

x a

bg

a + h , then f a = lim

b g b g . Hence, to

f a+h f a

h 0

h
find the derivative of f at x = a, one should first of all
find the difference quotient

af

f a+h f a
h

and

f a f ,(h > 0) and then proceed to find

f ah f a

h
its limit as h 0 . In finding out this limit, one can
make use of all the theorems on limits.
Note: The symbols L f a for f a and R
f a for f + a are also in use.

af

af

af

af

Question: When is a function y = f (x) is said to be


differentiable at a limit point x = a D f ?
Answer: The function y = f (x) is differentiable at a
limit point x = a D f a lies in the domain
of the derived function f x , i.e. f a exists, i.e.

af

af

af

f + a = lim

af

h 0

af
af
f aa + hf f aa f
, (h > 0) = f ' (a); and
a

f afa

af

f ah f a
, h>0 = f a
h
= a finite number.

f a = lim

h 0

Question: When is a function differentiable in an


interval?
Answer: A function y = f (x) is said to be differentiable
in an open interval (finite or infinite) The function
y = f (x) has a derivative at each limit point in between
the left and right end points of the open interval.
Also, a function y = f (x) is said to be differentiable
in a closed interval The function y = f (x) is
differentiable at and in between the left and right end

324

How to Learn Calculus of One Variable

limit points of the closed interval, i.e. a function f (x) is


differentiable on a closed interval [a, b] it is
differentiable on the interior (a,b) and if the limits

af

afa

f a+h f a
, h > 0 = right
h 0
h
hand derivative at the left end point of the closed
f + a = lim

af

f af a

f bh f b
, h>0 =
interval, f b = lim
h 0
h

left hand derivative at the right end point of the closed


interval, exist.
Lastly, a function is said to be differentiable if it is
differentiable at each limit point of its domain.*
Notes: 1. If a function y = f (x) is defined on a closed
interval [a, b], then the value of its left hand derivative
on the right end point and the value of its right hand
derivative on the left end point are taken, respectively,
as the values of its derivatives at the end points of
the closed interval.
2. A function which has a continuous derivative is
called continuously differentiable.
How to test differentiability of a function at a point?
A simple rule to test the differentiability of a function
f at a point x = a in its domain is to find its derived
function f x of the function f (x) using the
definition of the derivative f of a function f at any
limit point x in the domain of the function, i.e.

af

af

f x = lim

x 0

f af

f x+ x f x
x

and then to

af

check whether f x is defined (continuous) or


undefined (discontinuous) at the given limit point x =
a, i.e. (i) f x is defined at limit x = a f x is
differentiable at limit point x = a (ii) f x is
undefined at limit point x = a f x may or may
not be differentiable at x = a for which one must make
use the definition of the left hand and right hand
derivative at the given point x = a whose equal finite
value confirms the differentiability of f (x) at the limit
point x = a and whose unequal finite (or, infinite)
value confirms the non-differentiability of f (x) at the
limit point x = a D f .

af

af

af
af

af

* All standard functions are differentiable in their domains.

Note: If one is asked to test the differentiability of a


function f at a limit point x = a in its domain with the
use of the definition of the derivative of a function f at
a limit point a D f the above method should
not be adopted.

bg

Non-differentiability of a function at a point


Question: When a function f (x) is said to be nondifferentiable at a limit point x = a D f ?
Answer: A function f (x) is said to be non-differentiable
at a limit point x = a D f f ' a does not
exist. For instance,

af
bg

af

af af
f ba g = f ba g = or
f aa f = f aa f = or
f aa f = + and f aa f = or
f aa f = or
f aa f = or

1. f + a f a or
2.
3.
4.
5.

6.
7. f (a) is undefined or imaginary.
That is, in words, a function f (x) is nondifferentiable (not differentiable) at a limit point
x = a D f 'a' does not lie in the domain of
f x , i.e. f a does not exist, i.e.,
1. Either left hand derivative or right hand derivative
or both left hand derivative and right hand derivative
do not exist at the limit point x = a D f .
2. right hand derivative and left hand derivative exist
but are unequal at the limit point x = a D f .
3. f (x) is undefined or imaginary at the limit point
x D f .
4. In most cases a modulus function y = | f (x) | or a

af

af

af

af

bg

radical function y =

af

af

axf is non-differentiable

at a point x = a D f x = a makes f (x) zero,

b a fg

a f f axf is nondifferentiable at x = a D a f f , am , n R f .
i.e. f x

x=a

= 0 f x or

Notes: 1. Points of discontinuity of a function f (x)


are also points of non-differentiability of the function.
2. Points at which f (x) is non-differentiable are called
points of non-differentiability of f (x).

Differentiability at a Point

3. The function y = f (x) defined for real x, by

af

f x =x

F 1I , x 0
sin
H xK

Step 3. Simplify the function

af
(ii) f a0f exists but f a x f is not continuous at
x = 0, if p = 2
(iii) f a x f is continuous at x = 0 if p > 2.
4. The set of all those points where f (x) is differentiable is called domain of differentiability.
5. Derivatives at isolated points are not defined whereas
a function is always continuous at an isolated point.
Question: Explain the cases where to use the concepts
of left and right hand derivative to test the
differentiability of a function at a point on its domain?
Answer: There are mainly four cases where the
concepts of one sided derivative (left hand derivative
and right hand derivative ) are used.
1. When a function is defined as under:

b g RS f cb x, g ,
T
1

LM f aa hf f aaf OP
N h Q

and cancel out the common factor h (if any).

f (0) = 0
has the following properties:
(i) f 0 does not exist if p = 1

f x =

325

Step 4. Find hlim


0

LM f aa hf f aaf OP which is the


N h Q

required left side derivative at the right end point of


the adjacent intervals where a given function f (x) is
defined.
How to find right hand ( or right side) derivative of a
piecewise function f (x)at a common point x = a in the
adjacent intervals.
Step 1. Find f (a) from a form ( expression in x) of the
function f (x) in a semiclosed or closed interval whose
left or right end point is a or from a form of the
function f (x) with a restriction x = a.
Step 2. Replace x by (a + h ) in the given form of a
function f (x) and also in an interval whose left end
point is a. This is f (a + h), for h > 0.
Step 3. Simplify the function

xa
x=a

LM f aa + hf f aaf OP
h
N
Q

and cancel out the common factor h ( if any).

2. When a function f (x) is a piecewise function, i.e.


when a function f (x) is defined by more than two
formulas (different expression in x) in adjacent
intervals.
3. When a function contains modulus, radical or
greatest integer function.
4. When the question says to examine the
differentiability of a function f (x) at a point x = a in its
domain or to examine whether f a exists.

af

On methods of finding one sided derivatives


Method 1:
How to find left hand (or left side) derivative of
piecewise function f (x) at a common point x = a in the
adjacent intervals.
Step 1. Find f (a) from a form (expression in x) of the
function f (x) defined in a semiclosed or closed interval
whose left or right end point is 'a' or from a form of the
function f (x) with a restriction x = a.
Step 2. Replace x by (a h) in a given form of the
function f (x) and also in an interval whose right end
point is a. This is f (a h), for h < 0.

Step 4. Find hlim


0

LM f aa + hf f aaf OP which is the


h
N
Q

required right side derivative at the left end point of


the adjacent intervals where a given function f (x) is
defined.
Notes: 1. In the case of functions containing modulus
or greatest integer function, the above method of
procedure is applicable since indeed, it is these
functions which are piecewise functions.
2. When a function f (x) is redefined, one should put
x = a h in the same expression and in a restriction
x a to find f ( a + h ) and f ( a h ) and then it
remains to find f (a) from the expression with a
restriction x = a and lastly f + a and f a are
determined, i.e.

af
af
R f a xf , when x a
f axf = S
T f a xf , when x = a
1
2

should be changed into the form:

326

How to Learn Calculus of One Variable

h0
f RST ff aahhff ,, when
when h = 0 and finally it
requires the use of the definitions:
f aa + hf f aa f
f aa f = lim
, ah > 0f
h
f aa h f f aa f
f aa f = lim
, ah > 0f .

f ah =

h0

h0

Method 2:
1. Find f (a) by putting x = a in one of the given
function f1 (x), f2 (x), f3 (x) or f4 (x) against which the
sign of equality with the sign of inequality in the
given restrictions in the form of different intervals
whose union provides us the domain of the given
function f (x). i.e;
Find f (a) by putting x = a in that function (one of
f1 (x), f2 (x), f3 (x) or f4 (x) etc) against which any one
of the restrictions x a , x a , x = a , a x < c ,
c < x a , ... etc is imposed.
2. Find f (a + h) by putting x = a + h in the different
given functions f1 (x), f2 (x), f3 (x), etc. and in the
different intervals ( i.e.; x a , x a , x = a ,
a x < c , c < x a , ... etc.) as the restrictions ( or,
the conditions) imposed against each different
functions f1 (x), f2 (x), f3 (x) etc i.e;
Put x = a + h in all the different functions f1 (x), f2
(x), f3 (x) etc. and in all different given intervals
which are imposed as a restriction against each
different function f1 (x), f2 (x), f3 (x) etc as well as in
f (x) to find f (a + h).
3. Solve the restrictions only to have a function
[f1 (a + h), f2 (a + h), f3 (a + h), etc] for h > 0 and
a function [f1 (a + h), f2 (a + h), f3 (a + h), ... etc for
h < 0.
4. Use the definition:
f a+h f a
L f a = f a = lim
,
h0
h
for h < 0

bg

bg

bg

bg

R f a = f + a = lim+

bg

b g,

f a+h f a

h0
h
for h > 0 which
(i) Put f1 (a + h), f2 (a + h), or f3 (a + h), which is an
expression in h for h < 0 (already determined) by

af

replacing f (a + h) in the definition of L f a i.e.;


put f (a + h) = proper one of f1 (a + h), f2 (a + h), or f3
(a + h) which is an expression in h for h < 0 in L f 0 .
(ii) Put f1 (a + h), f2 (a + h), or f3 (a + h), which is an
expression in h for h > 0 (already determined) by
replacing f (a + h) in the definition or R f a i.e.;
put f (a + h) = proper one of f1 (a + h), f2 (a + h), f3 (a
+ h) or f4 (a + h) etc. already determined which is an
expression for h > 0 on R f 0 .
(iii) Put f (a) already determined in the definition of

af

af

af

af

af

L f 0 and R f 0 .

(iv) Simplify

a f a f for h < 0

f a +h f a
h

(v) Find the limit of

and h > 0

af

f a+h f a
for h < 0 and
h

h > 0 as h 0 after simplifying

af

f a+h f a
h

for h < 0 and h > 0


Note: 1. Method 2 (or, the second method) is
applicable when we can obtain various functions
f1 (h), f2 (h), f3 (h) etc. (i.e. functions or expressions
in h) against which h > 0 and h < 0 are imposed as the
restrictions if we put x = a + h in various functions
f1 (x), f 2 (x), f3 (x), etc. and in the intervals
x a , x a , x .> a , x < a , a x < b , etc.
which if f (a + h) = f1 (h), h 0
= f2 (h) , h < 0
are obtained by putting x = a + h in the given functions
and in the given intervals.

b g

2. Inequalities x > a a ,

x < a , a
x a a,
x a , a

N.B.: (i) An inequality in x only means an interval


(ii) We put x = (a + h) every where in the given
function i.e., in
(a) f (x)
(b) f1 (x), f2 (x), f3 (x), etc.
(c) The restrictions x > a , x a , x < a , x a ,
c < x < a , c x < a , c < x a , c x a , ... etc.
Whenever we follow the method 2.

Differentiability at a Point

A Theorem
Theorem: If a function possesses a finite derivative
at a point, then it is continuous at that point.
Or, if a function f (x) is derivable ( or, differentiable)
at x = a, then f (x) is continuous at x = a.
Or, the function f (x) is continuous at x = a if its
derivative f x at x = a i.e.; f a is finite.

af

a fg

af

Or, f a is finite f (x) is continuous at x = a .


Proof: Hypothesis is lim

xa

bg bg

f x f a
xa

af

= f a

We claim that f (x) is continuous at the origin (i.e.;


x = 0) but not differentiable at the origin.
Now, because,
f (0) = | 0 | = 0,

af af

f x f a =

af

af

x a

xa

Now, taking the limit as x a , we get

RS f axf f aaf ax afUV


T ax af
W
R f a xf f aaf UV
lim f a x f f aa f = S lim
ax af W
T
R
U
S lim a x a fV
T
W
lim f a x f f aa f = f aa f 0 L3 lim a x a f
MN
f a x f f aa f O
= 0 and from (1) , f aa f = lim
PQ
xa
lim f a x f f aa f = 0
lim f a x f lim f aa f = 0
lim f a x f = lim f aa f = f aa f
f a x f is continuous at x = a
af af

lim f x f a = lim

x a

x a

xa

xa

xa

xa

xa

xa

xa
xa

xa

xa

xa

Hence, the theorem is proved.


Remark: But the converse of the theorem is not true.
We cite the following example.
Let f (x) = | x |

a f

f 0 0 = lim f 0 h = lim f h
h0

h0

af

= lim h = lim h = 0 ,

h0

h0

f 0 + 0 = lim f 0 + h = lim f
h0

h 0

h0

h0

af

Thus, from (i) and (ii), we get

...(i)

a hf

= lim h = lim h = 0

which is finite [from the definition] (i) Now,


f x f a

327

...(ii)

lim f 0 h = lim f 0 + h = 0 which

h0

h 0

f (x) is continuous at x = 0
Now, test of differentiability:

af

L f 0 = lim

h0

f af

f 0h f 0
h

a f af
a f
a hf 0 = lim FG h IJ = 1
L f a0f = lim
H hK
h
= lim

h0

f h f 0
h

h0

af

R f 0 = lim

h 0

af

h 0

...(iii)

af af

f h f 0
h

R f 0 = lim

h 0

ahf 0 = lim

h 0

h
h
= lim

h
0
h
h

=1
...(iv)
Thus, from (iii) and (iv), we get, L f ' (0) = 1
1 = R f 0 which f 0 does not exist.

af

af

Conclusion:
1. Differentiability at a point continuity at the
same point.
2. Continuity at a point
/ differentiability at the
same point.
Note: 1. We say that f (x) is differentiable at the left
end point a of an interval [a, b] or [a, b), we mean
that f + a exists.

af

328

How to Learn Calculus of One Variable

Similarly, we say that f (x) is differentiable at the


right end point b of an interval (a, b] or [a, b] we
mean that f b exists.
2. By a closed interval [a,b] or a x b , we mean
that the end points a and b of the interval are also to
be considered while by an open interval (a,b) (or, ] a,
b [ or, a < x < b) we mean that end points a and b are
not to be considered in any problem.

af

Important deductions based on the definition of


derivative of a function f (x) at a point x = a

xa

af

af

af

af

af

af

af

xa

af

x a

af af

a fa f
a f
= f aa f a f aa f
xa

af

af

f a xa
f x f a
a lim
x
a

x a
x a

= value of f (x) at x = a a times d.c. of f (x) at x = a

af

= f x

af

e.g: a lim

2. If

x=a

y=

then = lim

y x

(a) lim

yx

LM d y OP
Nd x Q

af

af

af

f y
= f y
x y

tan x tan y
d tan y
2
= sec y =
x y
dy

sin x sin y
d sin y
= cos y =
x y
dy

af

N.B.: (i) Remember that when x y we get f y


and when y x , we get f x as in (2) and (3) i.e.;
after f , converging point is written.
(ii) On the left side of the symbol " " independent
variable of he function is mentioned and the right
side of the symbol " " the constant quantity (i.e.;
the limit of independent variable) is mentioned. i.e.;

af

bg

lim f a , a = independent variable, b = lim a

[adding and subtracting a f (a) in Nr]

= lim

xy

af

f x

(b) lim

xf a af a a f x f a

= lim

lim

xy

af

xf a af x
x a

xa

then

af

f x f y
,
x y

af

= value of f (x) at x = a a times d.c of f (x) at x = a.


Proof: lim

3. If y =

tan x tan y
d tan x
2
= sec x =
x y
dx

xy

xf a af x
= f a a f a
x a

then lim

yx

e.g.: (a) lim

xf a af x
,
x a

(i) If y =

(b) lim

x=a

x sin sin x
= sin cos
x

a f a f FG or, = f a yf f a xf IJ ,
y x
H
K
f a xf f a yf
= f a x f e.g.,
x y

f x f y
x y

sin x sin y
d sin x
= cos x =

x y
dx

a b

Types of the problem


Type A
1. To find l.h.d and r.h.d or f a by using the
definition of d.c
2. Existence and non-existence of f x at a point
x=a.
3. To show whether a given function is differentiable
or not at a given point.
4. Examination of differentiability at a given point or,
discussion of differentiability at a given point.
All above types of problems have the following
working rule.
1. Find L.H.D or R.H.D
2. Observe whether L.H.D and R.H.D are equal or
not.
3. Conclude f x at a point to be differentiable or
not according to the observation.

af

af

af

Type B
1. To find the value of constants.
Type 1: Problems based on piecewise function.

Differentiability at a Point

Examples worked out:


1. If f (x) = 3x 4, x 2

Now,

af

f (x) = 2 (2 x 3 ), x > 2 examine f 2 .


Solution: Method 1
f (x) = (3 x 4), for x < 2
f (2 h) = 3 (2 h) 4, for h > 0 [3 2 h < 2]
=63h4=23h
(1)
(2)
f (2) = 3 2 4 = 6 4 = 2
(1) (2) = f (2 h) f (2) = 2 3 h 2 = 3 h
(3)
Again, f (x) = 4 x 6 for x > 2
f (2 + h) = 4 (2 + h ) 6 = 8 + 4 h 6 = 2 + 4 h
(4)
[2 + h > 2]
(4) (2) f (2 + h) f (2) = 2 + 4 h 2 = 4 h
(5)
Now, R.H.D = lim

h0

= lim

h0

L. H.D = lim

h0

= lim

h0

f 2+h

h
4h
=4
h

a f ,h>0

f 2

af

f 2h f 2
,h>0
h

h0

= lim

h0

af

af

f 2

for h < 0

2 + 3h 2
3h
= lim
=3
h0 h
h

f 2+h

R.H. D = lim

h0

a f , for

f 2

(4)

h>0

2 + 4h 2
4h
= lim
=4
(5)
h0
h0 h
h
Hence, from (4) and (5), we see that l.h.d. r.h.d
= lim

af

which f 2 does not exist.


2. Test the differentiability of the function

a f RST2 xx 1

f x =

, 0 x1
1< x

at x = 1.

when h < 0
when h > 0

f (1) = 1 (considering f (x) = x for the value of the


function f (x) at x = 1)
(7)

af

f RST2 a11++hhf , 1 ,

f 1+ h =

af

af

L f 1 = lim

From (6) and (7), we see that f 2 f + 2


f 2 does not exist.
Method 2: Given is
f (x) = 3x 4, x 2
f (x) = 2 (2 x 4), x > 2
By definition of function f (x),
f (2 + h) = [3 (2 + h) 4], 2 + h 2
= 2 [2 (2 + h) 3], 2 + h > 2
f (2 + h) = [ 6 + 3 h 4], h 0
= 2 [4 + 2h 3], h > 0
f (2 + h) = 2 + 4h when h > 0
= 2 + 3 h when h 0
and f (2) = 2

f 2+h

Solution: By using method (2),


from the definition of the function f (x),
(6)

3 h
=3
h

L. H.D = lim

329

h0

= lim

h0

af

R f 1 = lim

h0

= lim

af

(1)
(2)
(3)

h0

f 1+ h

a f ,h<0

f 1

1+h 1
= 1 and
h

af

f 1+ h f 1
,h>0
h
2h + 1 1
=2
h

af

af

thus, L f 1 R f 1 f 1 does not exist


f (x) is not differentiable at x = 1
3. A function f (or, f (x)) is defined by f (x) = x + 2,
when 0 x < 2

= 8 x , when 2 x 4 ,
find the left hand derivative and right hand derivative
of f at x = 2.

330

How to Learn Calculus of One Variable

Solution:

af

af

f 2h f 2
l.h.d = f 2 = lim
,h>0
h0
2 h 2

af

f 2h f 2
= lim
h0
h

(A)

Now, f (x) = x + 2 for x < 2


f (2 h) = 2 h + 2 = 4 h

(1)

32 h < 2

bg

f 2 =

8x

x =2

= 16 =

44 =4

(2)

Putting (1) and (2) in (A), we get

af

f 2 = lim

h0

h
4 h 4
= lim
=1
h 0 h
h

Again,

af

r. h.d = f + 2 = lim

h0

= lim

h0

f af

(3)

af

f 2+h f 2
,h>0
2 + h 2

f 2+h f 2
h

af
f a2 + hf = 8 a2 + h f =

(B)

... (4)

4. If

= lim

h0

= lim

h0

= lim

h0

j e

h/

16 + 8 h + 4

8 h/
16 + 8 h + 4
8

16 + 8 h + 4

(4)

af

af

x<0

0 x 1

x ,
3

x +1,

x >1

af

= lim

af

a f

(1)
(2)
(3)

f af

f 0h f 0
,h>0
h

af

f h f 0
h0
= lim
= 1 (4)
h 0 h
h

f + 0 = lim

h0

= lim

f af

f 0+h f 0
,h>0
h

af af

f h f 0
h
2

h 0
h
= lim
= lim h = 0
h0
h0 h
h0
h

= lim
16 + 8 h + 4

16 + 8 h + 4

16 + 8 h 16

af

x ,

Now, f 0 = hlim
0

h0

16 + 8 h 4

h0

= lim

8
8
= =1
4+4 8

test the differentiability at x = 0 and x = 1


Solution: At x = 0
f (x) = x2
f (0) = 0 when x > 0,
f (x) = x2 when x < 0,
f (x) = x
f (0 + h) = (0 + h)2 = h2, h > 0
f (0 h) = (0 h) = h

16 + 8 h 4

h0

16 + 0 + 4

R|
f axf = S
|Tx

Putting (2) and (4) in (B),

af

thus, (3) and (4) f 2 = f + 2 = 1 f 2


exists and = 1.

32 + h > 2

f + 2 = lim

h0

and f x = 8 x for x > 2

16 + 8 h

af

af

af

Thus, f 0 f + 0 f 0 does not exist

af

f x is not differentiable at x = 0
Similarly, we can test the differentiability at x = 1

a f RST216x + x7 ,,

5. If f x =

entiability at x = 3.
Solution: Let h > 0.

af

when x 3
when x 3

R.H. D = f + 3 = lim

h0

test the differ-

f af

f 3+h f 3
,h>0
h

Differentiability at a Point

= lim

h0

LM OP
N Q
(1)

h0

bg

h0

h0

g b g ,h>0

f 3h f 3

L. H.D = f 3 = lim
= lim

then we may consider any one of f1 (x) and f2 (x) for


considering the value of f (x) at x = a because both
give the same value.

h
16 3 + h 13
= lim
h

0
h
h

= lim 1 = 1

LM
N

7 + 2 3 h 13
2h
= lim
h0
h
h

= lim 2 = 2

OP
Q

(2)

h0

Thus, (1) and (2) L.H.D R.H.D f (x) is not


differentiable at x = 3
Second method:
f (3) = [2x + 7]x = 3 = 2 3 + 7 = 13
(1)
Now, using the definition of the function,
f (3 + h) = [2 (3 + h) + 7], 3 + h 3
(2)
f (3 + h) = 6 + 2 h + 7, h 0
f (3 + h) = [16 (3 + h)], 3 + h 3
... (3)
f (3 + h) = 13 h, h 0
Now, using the definition,

af

f af

L f 3 = L

f a+h f a
, h<0
h

= L

13 + 2 h 13
2h
= L
h0 h
h

h0

h0

= L 2=2
h0

af

R f 3 = L

h0

= L

h0

= L

h0

f af

f 0+h f 0
,h>0
h

h0

sin h
1
sin h h
= lim h
= lim
0

h0
h
h
h2

LMh h + h ...OP h
MN N3 N5 PQ
3

= lim

h0

LMform is = 0 OP
0Q
N

LM h + h + ...OP = 0
MN N3 N5 PQ
a1 + h cos hf 1
Again, f a0f = lim
h
3

= lim

...(1)

h0

h0

af

af

af

(2)

Thus, we see that f + 0 f 0 f 0 does


not exist f x is not differentiable at x = 0.
Type 2: Problems based on the function which are
redefined.

af

1. If
(5)

af

Thus, (4) and (5) f 3 f + 3 f 3


does not exist f x is not differentiable at x = 3
Remember: If f (x) = f1 (x), when x a
f (x) = f2 (x), when x a ,

af

Solution: f + 0 = lim

Examples worked out:

13 h 13
h
= L
h 0 h
h

af

= 1 x cos x , when x 0 ,
show that the function f (x) is not differentiable at x = 0.

h0

f af

af

sin x
when x > 0
x

= lim cos h = 1

f a+h f a
, h>0
h

af

af

6. If f x =

(4)

a 1f = 1

331

af

f x =

x 1
2

2x 7x + 5

, when

af

1
= , when x = 1 find f 1 .
3

af

Solution: f x =

af

f 1 = lim

h0

x 1
2

2x 7x + 5

f af

f 1+ h f 1
h

x 1 and

332

How to Learn Calculus of One Variable

a1 + hf 1
1
F I
H
3K
2 a1 + hf 7 a1 + hf + 5
= lim

LM a2 + hf 2
O
1P
a2 + hf 3a2 + hf + 2 P
= lim M
MM
PP
h
MN
PQ
h b2 + hg + 3 b2 + hg 2
= lim
h {b2 + hg 3 b2 + hg + 2}

h0

h
2

= lim

h0

2 h 3h
h

h0

1
3

h0

1
1
+
2h 3 3
= lim
h0
h
= lim

h0

= lim

2
2
= lim
=
h 0 3 2h 3
9

= lim

af
f aa + hf f aa f
f aa f = lim
h0

= lim

h
not required to find l.h.d. and r.h.d. separately but
when we have to examine f a or to test the
differentiability of the given function f (x) at x = a
which is redefined, it is a must to find l.h.d and r.h.d.
for the same function f (x) = f1 (x), when x by
putting (a + h) and (a h) separately since x a
means x > a and x < a which we have to consider
the same function f (x) whose independent variable is
replaced by (a h) while finding l.h.d. and the
independent variable x in f (x) is replaced by (a + h)
while finding r.h.d.

af

x2
2

x 3x + 2

, x 2 and = 1, x = 2

examine the differentiability at x = 2.

af

f af

f 2+h f 2
Solution: f + 2 = lim
h0
h

= lim

h 4 + h + 4 h 6 3h + 2

h 6+h +h6

h 4 h 4 h + 6 + 3h 2

h h +h

h6h h+6

h0

h h +h
h

= lim

h0

ah + 1f

ah + 1f = 1
f a2 hf f a2 f
f a2 f = lim
= lim

h0

h0

a2 h f 2 1
a2 hf 3a2 hf + 2
2

= lim

h0

h 4 + h + 4 h + 6 + 3h 2

h0

= lim

h 4 + h + 4 h + 6 + 3h 2

af

2. If f x =

h0

which we are

h 4 + h + 4 h 6 3h + 2

h0

Note: Problems where f (x) = f1 (x), when x a =


constant, when x = a are provided and we are required
to find f a , we use the definition,

h 4 + h + 4 h + 6 + 3h 2

h0

3 + 2h 3
3h 2 h 3

Differentiability at a Point

a2 hf 2 a2 hf + 3a2 hf 2
= lim
h {a2 hf 3a2 hf + 2}
2

h0

2 h 2 4 + h 4 h + 6 3h 2

= lim

h0

h0

h0

h h h
h

0h

af

af

h0

af

, x0

test the differen-

= lim

h0

1+ e
h

af

1
h

af

x
1+ e

f 0 = lim

h0

1x

x
1+ e

1x

, when x > 0

1+ e
= lim
h0
h

a f

h0

, when x < 0

af

f (0) = 0

(1)

x
, when x > 0
1 + e1 x

1+ e

1
h

(4)

1
1+e

1
=1
1+0

1
h

(6)

af

af

1+ e

af

h
1+ e

1
h

af

Thus, we see that f + 0 f 0 f 0 does


not exist f x is not differentiable at x = 0

h0

1
0+ h

(2)

(5)

1
h

Remember: For h > 0 (i) lim e

0+h

1
=0

For h > 0,

1
h

f af

a h f
= lim

Now, 3 f x = 0 when x = 0

f 0+h =

f 0h f 0
,h>0
h

f (x) = 0 , when x = 0

f x =

1+ e

h0

x=0

Solution: Given that f x =

af

f af

= lim

tiability at x = 0.

f x =

1
0h

1
1
= lim
=
= 1
h0 h 1
1
0
h1

1x

f 0+h f 0
; h>0
h

f + 0 = lim

R| x
f a x f = S1 + e
|T 0 ,

bg

Now, using the definition,

4h 6 h + 4h + 6
2

(3)

1+ e h

1+ ex

1+ e

f +1 2 = f 2 = 1 which gives function


f (x) is differentiable at x = 2.

3. If

1+ e

0 =

1
h

bg

h 6 + h h 6

h0

= lim

Again, 3 f x (when x < 0) =

= lim

b g bg

f 0+h f 0 =

f 0h =

= lim

(2) (1)

h 6 + h h 6

h 4 h + 4 h + 6 3h 2

333

1
h

(ii) lim e = e
h0

1
h

=e

=0

334

How to Learn Calculus of One Variable

R
a f |Sx
|T

1
, when x 0
x
0,
when x = 0

h0

af

f 0 = lim

is differentiable at x = 0.

h0

1
Solution: Given that f (x) = x2 sin , when x > 0
x

a3 x 0 x > 0 and

x<0

F 1I = 0
H hK
f a0 h f f a0f
,h>0

= lim h sin

sin

f x =

4. Show that

F 1I 0
H hK

h sin
= lim

h0

1
, when x < 0
x
f (x) = 0 , when x = 0
Now, f (x) = 0, when x = 0
f (0) = 0
f (x) = x2 sin

f (x) (when x > 0) = x2 sin

h 2 sin
= lim
(1)

1
x

f a

For h > 0, f 0 + h = 0 + h

F 1I = 0
(6)
H hK
(5) and (6) f a0f = f a0f = 0 which means
that f a0f exists. i.e.; f a x f exists at x = 0 f (x) is

sin

FG 1 IJ
H 0 + hK

F 1I
(2)
H hK
F 1I
b2g b1g f b0 + hg f b0g = h sin G J 0
H hK
F 1I
= h sin
(3)
H hK

h0

g b

g sin FGH 0 1 h IJK


F 1I
sin
H hK

f 0h = 0h
=h

af

h0

f af

f 0+h f 0
,h>0
h
2

h sin
= lim

h0

F 1 I , when x 0 = 0 , when x = 0.
H xK

Show that the function f (x) does not have the


derivative at the point x = 0.

FI
HK

1
, when x 0
Solution: 3 f (x) = x sin
x

af

f 0 = lim

h0

Now, using the definition,

f + 0 = lim

5. If f (x) = x sin

f (0) = 0

1
x

F 1I
H hK

differentiable at x = 0.

For h > 0

h0

FG 1 IJ
H hK

= lim h sin

= h sin

Again, f (x) (when x < 0) =x2 sin

(5)

= lim

f af

f 0+h f 0
h

a0 + hf sin FGH 0 +1 h IJK 0

h0

(4)

F 1I
H hK

h sin
= lim

h0

F 1 I which has no finite value


H hK
which means that f a x f does not exist at x = 0.
= lim sin
h0

Differentiability at a Point

6. If f (x) = x2, when x 1 , and f (x) = x, when


x < 1, show that the function f (x) does not have
derivative x = 1.
Solution: f (x) = x2, when x > 1
f (1 + h) = (1 + h)2, when 1 + h > 1, i.e. h > 0
= (1 + h)2, when h > 0 and f (x) = x, when x < 1
f (1 + h) = (1 + h), when 1 + h < 1, i.e.; h < 0
= (1 + h), when h < 0, f (1) = 1

af

= lim

h0

h0

a1 + hf

= lim

h0

2h + h
= lim
h0
h

f af

f 1+ h f 1
,h>0
h

f + 1 = lim

1 + 2h + h 1
h

h h+2
= lim
h0
h

h0

h0

=0+2=2

af

h0

f af

Note: 1. Generally, we are asked to test the


differentiability of a mod function | f (x) | at a point
x = a at which f (x) = 0.
2. In most cases of | f (x) |, point x = a at which
f (x) = 0, we have l.h.d r.h.d which in most cases,
the mod function | f (x) | is not differentiable at a point
x = a at which f (x) = 0.
3. To find the derivative or existence of a derivative
of a mod function at a point x = a where x = a is a point
at which f (x) = 0, we are required to find the l.h.d and
r.h.d separately,
4. If x = a be a point at which f (x) 0 in | f (x) |, then
we use the following formula to find the value of the
derivative of | f (x) | at x = a.

LM d f axf OP
Nd x Q

LM f axf f a xfOP
N f a xf
Q

=
x=a

5. | h | = | h | = h and |

= lim h + 2 = lim h + lim 2

h2

x=a

= h2 = | h |2

Examples worked out:


(1)

f 1+ h f 1
f 1 = lim
,h<0
h0
h

1. If f (x) = | x |, show that the function f (x) does not


have derivative at x = 0
Solution: 3 f (x) = | x |

af

f af

1+ h 1
1 h 1
= lim
h0
h
h

R f 0 = lim

f 0+h f 0
,h>0
h

2
h
2 h
= lim
lim
= lim
h
h

0
0
h0
h
h
h

= lim

0+h 0
h
= lim
h0 h
h

= lim

h0

F I
H K

FI
HK

= 1 =
(2)
(1) and (2) f + 1 f 1 which f 1
does not exist which means that f (x) has no finite
derivative at x = 1.

af

af

335

af

h0

h0

= lim

h0

af

L f 0 = lim

h0

Type 3: Problems based on a mod function | f (x) |.


Whenever we want to test the differentiability of a
mod function | f (x) | at a point x = a at which f (x) = 0 or
f (x) 0, we are required to find the l.h.d. and r.h.d at
a given point x = a because a mod function | f (x) | can
be expressed as a piecewise function in the following
way.
| f (x) | = f (x), when f (x) 0
= f (x) , when f (x) < 0
This is why a mod function | f (x) | also is called the
piecewise function.

= lim

h0

h
= lim 1 = 1
h h0

(i)

af

f 0h f 0
h

0h 0
h
= lim
h0 h
h

a f
...(ii)
(i) and (ii) R f a0f L f a0f which f a0f
= lim

h0

h
= lim 1 = 1
h h0

does not exist i.e.; f (x) has no finite derivative at x = 0.


2. Examine the differentiability of f (x) at the indicated
points.

336

How to Learn Calculus of One Variable

(i) f (x) = | cos x | at x =

= lim

h0

(ii) f (x) = | x3 | at x = 0
Solution: (i) f (x) = | cos x |

R f

= lim

FG IJ = lim
H 2K

F I
cos G + hJ
H2 K

h0

cos
2 ,h>0

h0

af

cos

L f 0 = lim

,h>0

h0

= lim

1 h

= lim

h 0

= lim

F hI
H2 K

cos

F hI
H2 K

h0

h0

a f

Thus from (i) and (ii), we see that

af

af

af

(ii)

h0

h0

h
3

= lim

h0

log 1 + h
=1
h

L f 1 = f 1 = lim

log 1 + h 0

= lim

af

h0

af

b g ,h>0

log 1 + h 0

= lim

h0

,h>0

h0

= lim
0

b g

f 1+ h f 1

h 0

h0

af

log 1 + h log 1

= lim

.
2

bg

R f 1 = f + 1 = lim

F I L f F I
From (i) and (ii), we see that R f
H 2K
H 2K
F I does not exist which f axf is not
f
H 2K

R f 0 = lim

af

3. If f (x) = | log x | , find f + 1 and f 1 .


Solution: 3 f (x) = | log x |

sin h
h

a0 + hf

a f

= 1 lim h = 1 0 = 0 (ii)
h0

bg

a f
= a 1f a1f = 1

= 1 lim

h0

R f 0 = L f 0 which f 0 exists.
f (x) is differentiable at x = 0.

sin h 0
sin h
= lim
h

0
h
h

h0

h
h
= 1 lim
h0 h
h

= lim

2 ,h>0

= lim

a f

cos

= lim

a0 hf

(i)

h0

af

(i)

h0

h0

F I = lim
L f
H 2K

(ii) f (x) = | x3 |

h0

h
h
= lim
h0 h
h

= lim h = 0

sin h
= lim
=1
h0
h

differentiable at x =

= lim

1 sin h 0

= lim

sin h 0

h0

h0

af

f 1 h f 1
,h>0
h

log 1 h log 1
h

337

Differentiability at a Point

5. f (x) = 1 + | sin x |. Examine differentiability at x = 0.

log 1 h 0

= lim

h0

log 1 h 0
h

= lim

h0

log 1 h

= lim

h0

a f

bg

= 1 lim

h0

af

4. If f x = 1 +

a x 2f

af

= lim

f = a 1f a1f = 1
af

af

a x 2f
f b2 + h g f b2 g
R f b2g = f b2g = lim
,h>0
+

= lim

1+

h 0

a2 + h 2 f

h0

= lim

h0

= lim h

h
h

= lim

h0

FG 2 3IJ
H 3 K

h0

h
h

= lim h
h0

= lim h

F 1I
H 3K

= lim

= lim

h0

= lim

h0

a f

a hf

= 1 lim h

a f

2h2
h

1
h

af

1
h

a f

= 1 lim h

a f

(i)

f l
h

h 0

q,

= lim

1 + 1 sin h 1
sin h
= lim
h0
h
h

= lim

sin h
sin h
= 1 lim
h0 h
h

a f

a f af
af
af

(ii)
= 1 1 = 1
Thus, from (i) and (ii), we see that
R f 0 L f 0 which f 0 does not exist.
f (x) is not differentiable at x = 0.

af

6. f (x) = | x 2 |. Examine differentiability at x = 2.

= lim

h0

af

= lim

h0

2+h2 0

h0

h
h

= lim

L f 0 = lim

h0

= 1 =

a f l

bg

1
3

sin h

1 + sin 0 h 1 + sin 0

R f 0 = lim

= lim

1 + sin h 1 + 0

h0

h0 3

h3
= lim
h0 h

FG 2 3IJ
H 3 K

= 1 lim

sin h
=1
h

h>0

= lim

h0

af

h0

h0 3

af

1 + sin h 1

L f 0 = lim

h0

F 2 1I
H3 K

f 2h f 2
L f 2 = f 2 = lim
,h>0
h0
h

1+

h0

h0

af

= lim

r,

h0

h
3

1 + sin h 1 + 0

h0

, find f + 2 and f 2 .

h0

= lim

g m

h0

h>0

log 1 h
= lim
h0
h

log 1 h
h

Solution: 3 f x = 1 +

1 + sin 0 + h 1 + sin 0

R f 0 = lim

h0

h0

,h>0

h
= lim 1 = 1
h h0

2h2 0
,h>0
h

h 0
1 h
= lim
h0
h
h

(i)

338

How to Learn Calculus of One Variable

a f

h
= lim 1 = 1
h h0

= lim

h0

(ii)

Thus, from (i) and (ii) we see that f (x) is not


differentiable at x = 2 caused by R f 2 L f 2 .

af

af

Type 4: To find the values of the constants so that a


given function f (x) becomes differentiable at a given
point x = a.
To find the values of the constants so that a given
function f (x) becomes differentiable at a given point
x = a, the following two conditions must be satisfied.
1. f (x)should be continuous at a given point x = a for
which we must have L.H.L = R.H.L at athe same point
x = a = value of the function at x = a = f (a).
L.H.D at x = a = R.H.D at x = a
Hence, we adopt the following working rule to find
the values of the constants so that a given function
f (x) becomes differentiable at a given point x = a
Working rule:
1. Find

xa

af

f x =

xa

af af

af

f + a = f a .

Examples worked out:


1. If f (x) = 3 x2 + 5 x, when x 0 and f (x) = a x + b,
when x > 0 find a and b so that f (x) becomes
differentiable at x = 0.
Solution: For continuity of f (x) at x = 0

af af

Lim+ f x = Lim f x = f 0

x0

x0

Lim 3 x 2 + 5 x = Lim a x + b
x0

= lim

h0

30h

h0

h0

af

+5 0h 0
h

= lim

3h 5 h
= lim 3 h + 5 = 5
h 0
h

af

R.H. D f + 0 = lim

h0

(B)

f af

f 0+h f 0
, (h > 0)
h

= lim

a 0+h +b0
ah + b 0
= lim
h0
h
h

= lim

ah
h

h0

h0

a3 b = 0 from (A)f
(C)

h0

Now equating (B) and (C), we get a = 5


(3 differentiability at x = a L.H.D = R.H.D) and
b = 0 (from (A))

UV
W

a=5
Thus, b = 0 is the required answer.

2. For what values of a and b is the function

Remember: Differentiability at a point x = a


continuity at the same point x = a which is used to
find the values of the constants so that a given
function f (x) becomes differentiable at a given point
x = a.

af

f 0h f 0
, (h > 0)
h

= lim a = a

f x = f a from the

definition of continuity of a function at a point x = a


2. Find left hand derivative and right hand derivative
at the same point x = a by definition and then put
L.H.D = R.H.D since differentiability at a point means

af

af

L. H.D f 0 = lim

0=bb=0

x0

a f LMM2 a xx + b
N
2

f x =

when x 1
when x > 1

differentiable at

x = 1.
Solution: For continuity of f (x) at x = 1
f (1) = 12 = 1 ( 3 f (x) = x2 when x = 1)

af
f a x f = lim a2 a x + bf = 2 a + b
2

L. H.L = lim f x = lim x = 1


x1

R.H.L = lim+
x 1

x 1

x 1

Now, since f (x) should be continuous at x = 1


L.H.L = R.H.L = f (1)
(A)
2 a + b = 1 which 2 a + b = 1

g
(A)

L. H.D = lim

h0

f af

f 1h f 1
, (h > 0)
h

Differentiability at a Point

a1 hf
= lim

h0

h 2h
= lim
h0
h

= lim 2 h = 2
h 0

R.H.D = lim

h0

(B)

f af

f 1+ h f 1
, (h > 0)
h

= lim

2 a 1 + h + b 1
h

= lim

2 a h + 2 a + b 1
h

= lim

2 a h + 1 1
(3 2a + b = 1 from (A))
h

h0

h0

h0

2ah
= lim 2 a = 2 a
(C)
h0 h
h0
Now, equating (B) and (C), we get
2a = 2 ( 3 differentiability at x = 1 L.H.D =
R.H.D at the same point x = a)
a=1
Now, putting this value of a = 1 in (A), we have
= lim

b = 1 2 1 = 1 ( 3 2a + b = 1)
Hence,

UV
W

a =1
f (x) would be differentiable
b = 1

at x = 1 only if a = 1 and b = 1.
On continuity and differentiability of a function at a
point x = a.
Question: Explain the cases where l.h.l at a point
x = a = r.h.l at the same point x = a is used to test the
continuity of a given function f (x) at a given point
x = a.
Answer: There are following cases where the concept
of l.h.l at a point x = a = r.h.l at the same point x = a is
used to test the continuity of a given function f (x) at
a given point x = a.
(i) When a function f (x) is redefined.
(ii) When a function f (x) is a piecewise function.
(iii) When a function f (x) is a mod function.

339

Note: 1. Whenever we want to find the limit of a


mod function | f (x) | at a point x = a, we are required to
find the l.h.l and r.h.l at the same given point x = a
because a mod function is also a piecewise function.
2. To examine the continuity and differentiability in
an interval, we are required to check the given
function at the following points.
(a) The point on both sides of which two different
functions f1 (x) and f2 (x) are defined in two different
subinterval (or, part of the domain of the given
function). These points are termed as turning points
of the definition or interior points of the interval if the
given function f (x)is a piecewise function.
(b) Points (including the end points of the closed
interval) at which the given function becomes infinite,
imaginary, or indeterminate.
(c) The end points of the closed interval where it has
one sided limiting values or derivatives. i.e.;

af af

lim + f x = f a

(i)

xa

and

af af

lim f x = f b

xb

where a and b are the end points of the closed interval


[ a, b]
(ii)

xa

lim

xb

lim +

af

af

af

f x f a
=a
x a

finite value and

af

f x f b
= a finite value. Where a and b
x b

are the end points of the closed interval [a, b].


working rule to test the continuity and differentiability
of the given function at the same point x = a.
To test the continuity and differentiability of the
given function at the same point x = a, we should test
the continuity at the point x = a and differentiability
at the same point x = a separately. i.e.;
(i) To test the continuity at x = a, we are required to
show that l.h.l of the given function at the point x = a
= r.h.l of the given function at the point x = a = value
of the function at x = a.
(ii) To test the differentiability at x = a, we are required
to show that l.h.d of the given function at a point
x = a = r.h.d of the given function at a point x = a.
Remember: Whenever a function is redefined in an
interval (open or closed) i.e.;

340

How to Learn Calculus of One Variable

bg

bg

f x = f1 x , x c
= d, x = c

UV in the interval (a,b) or


W

[a, b] where a, b, c are constants.


Then only probable point of the interval (a, b) or
[a, b], at which the given redefined function may be
discontinuous or non-differentiable is x = c. For this
reason, we test the continuity and differentiability at
x = c, e.g.,
(i) f (x) = x if x 0
f (0) = 1 in the interval [ 1, 1]
The only probable point of the interval [ 1, 1], at
which the function may be discontinuous or nondifferentiable is x = 0.
(ii) f (x) = 4 x + 7 if x 2
f (2) = 3 in the interval at [ 4, 4]
The only probable point of the interval [ 4, 4], at
which the function may be discontinuous or nondifferentiable is x = 2.

af

(iii) f x =

9 x 16
3

27 x 64

for x

4
3

F 4 I = 2 in the interval [ 1, 3]
H 3K 3

The only probable point of the interval [ 1, 3], at


which the redefined function may be discontinuous
or non-differentiable is x =

af

(iv) f x =

1 sin x

F xI
H2 K

4
.
3

for x

Questions: 1. A function f is defined as follows:


f (x) = x for x 0
f (x) = x for x 0 .
Test the continuity and differentiability of the
given function at x = 0.
Solution: (i) Continuity test at x = 0.

af

l.h.l at x = 0 = lim f x
x0

a f a f

= lim x = 1 lim x = 0 1
x0

=0

x0

af

(a1)

af

r.h.l at x = 0 = lim+ f x = lim x


x0

x0

=0
(a2)
f (0) = 0
(a3)
(a1), (a2) and (a3) l.h.l at x = 0 = r.h.l at x = 0 =
value of the function at x = 0 = 0.
f (x) is continuous at x = 0.
(ii) Differentiability test at x = 0.

bg

0h 0

l.h.d at x = 0 = L f 0 = lim

h0

FG h IJ = lim 1 = 1 (b )
H hK
a0 + h f 0
r. h.d at x = 0 = R f a0f = lim
h
h0

h0

h0

= lim

h0

LM
N

The only probable point of the interval 0 ,

OP
Q

3
,
2

at which the redefined function may be discontinuous

.
2

Type 1: Problems based on piecewise functions:

= lim

F I = 1 in the interval LM0 , 3 OP .


f
H 2K 2
N 2Q

or non-differentiable is x =

Worked out examples on continuity and


differentiability at a given point x = a.

h
=1
h

bg

(b2)

bg

(b1) and (b2) L f 0 R f 0

f (x) is not differentiable at x = 1.


2. Examine the following function for continuity and
differentiability:
f (x) = x2 for x 0
f (x) = 1 for 0 < x 1
f (x) =

1
for x > 1
x

Differentiability at a Point

Solution: Considerable points at which we are


required to test the continuity and differentiability
are x = 0 and x = 1.
(i) Continuity and differentiability test at x = 0
f (0) = 0
(a1)

bg

l.h.l at x = 0 = lim f x
x0

x0

(a2)

x0

af

(a3)

bg

l.h.l at x = 1 = lim f x

bg

= lim 1 = 1
x 1

bg

(b2)

r.h.l at x = 1 = lim+ f x
x1

F 1I = 1
= lim
H xK

(b3)

(b1), (b2) and (b3) l.h.l at x = 1 = r.h.l at x = 1 =


value of the function at x = 1 which f (x) is
continuous at x = 1
Again,
l.h.d at x = 1 = L f 1 = lim

h0

a f af

f 1 h f 1
,
h

h>0

11
= lim
=0
h0 h

af

given piecewise function is non-differentiable at


x = 1.

3. Discuss the continuity and differentiability of the


following function:
f (x) = x2 for x < 2
f (x) = 4 for 2 x 2
f (x) = x2 for x > 2
Solution: Considerable points at which we are
required to test the continuity and differentiability of
the given piecewise function f (x) are x = 2 and 2.
(i) Continuity and differentiability test at x = 2
f ( 2) = 4
(a1)

l.h.l at x = 2 =
= lim

x1

af

F I
a f a f GH JK
F 1 IJ = 1
= a 1f G
...(b )
H 1 + 0K
(b ) and (b ) L f a1f R f a1f which
h
1
= 1 lim
h 0 1 + h
h 1+ h

(a2) and (a3) l.h.l at x = 0 r.h.l at x = 0 which


given piecewise function is discontinuous at
x = 0. Caused by this, given piecewise function is
non-differentiable at x = 0.
(ii) Continuity and differentiability test at x = 1
Now, f (1) = 1
(b1)
x1

bg

= lim 1 = 1

h0

r. h.l at x = 0 = lim + f x
x0

1
1
1+h
1/ 1/ h
= lim
= lim
h0
h0 h 1 + h
h
= lim

= lim x = 0

341

x 2

r. h.l at x = 2 =

a f af

f 1+ h f 1
r.h.d at x = 1 = R f 1 = lim
,
h0
h
h>0

ex j = 4
2

(a2)

af

lim + f x

x 2

= lim 4 = 4

(a3)

x 2

(a1), (a2) and (a3) l.h.l at x = 2 = r.h.l at x = 2


= value of the function at x = 2 which f (x) is
continuous at x = 2
Again,

a f

L f 2 = lim

(b1)

af

lim f x

x 2

h0

a 2 hf

= lim

h0

= lim 4 h = 4
h0

a f

f 2 h f 2
, (h > 0)
h
2

= lim

h0

4 + 4h + h 4
h

(b1)

342

How to Learn Calculus of One Variable

a f

R f 2 = lim

h0

44
0
= lim
=0
h

0
h
h

a f

(b2)

a f

(b1) and (b2) L f 2 R f 2 .


Caused by this, the given function is nondifferentiable at x = 2.
(ii) Continuity and differentiability test at x = 2

af

l.h.l at x = 2 = lim f x
x2

= lim 4 = lim 4 = 4

(c1)

x2

x2

af

r. h.l at x = 2 = lim + f x
x2

e j

= lim x 2 = 4
x2

(c2)

f (2) = 4
(c3)
(c1), (c2) and (c3) l.h.l at x = 2 = r.h.l at x = 2
= f (2) which given piecewise function is
continuous at x = 2.
Again,

af

h0

= lim

h0

af

R f 2 = lim

h0

b2 + hg
= lim
h0

= lim

h0

af

f 2h f 2
, (h > 0)
h

L f 2 = lim

FG IJ
H K
f a2 + hf f a2f
, (h > 0)

4/ 4/
0
= lim
h

0
h
h = 0 (d1)

= lim

h0

af

(a2)

f (0) = 1 + sin 0 = 1

(a3)

x0

x0

(d2)

4. Examine the continuity and differentiability of the


following function in the interval < x < .

...(a1)

h0

(a1), (a2) and (a3)

af

af

af

lim f x = lim + f x

x0

x0

= f 0 which given piecewise function f (x) is


continuous at x = 0
...(A)

= lim

h0

af

(d1) and (d2) R f 2 L f 2 which


f (x) is not differentiable at x = 2.

f (x) = 1 in < x < 0

af
r. h.l at x = 0 = lim f a x f
= lim l 1 + sin a0 + h fq = 1
x0

R f 0 = lim

af

l.h.l at x = 0 = lim f x = lim 1 = 1

af

= lim 4 + lim h = 4 + 0 = 4
h0
h 0

I
K

in
x<
2
2
Note: In this question we are required to examine the
continuity and differentiability in an interval
< x < .
Hence, we adopt the rule to examine the continuity
and differentiability in an interval.
Solution: Considerable points at which it is required
to test the continuity and differentiability of the given

piecewise function f (x) are x = 0, .


2
(i) Continuity and differentiability at x = 0.

4/ + 4 h + h 2 4/
h

h 4+h
= lim 4 + h
h0
h

af

F
H

af

f x =2+ x

Again, L f 0 = lim

f (x) = 1 + sin x in 0 x <

h0

= lim

h0

h0

af

f 0h f 0
, (h > 0)
h

11
0
= lim
=0
h 0 h
h

...(b1)

f af

f 0 + h f 0 ,h>0
h

1 + sin 0 + h 1
h

sin h
=1
...(b2)
h0
h
(b1) and (b2) L f 0 R f 0 which
given piecewise function f (x) is not differentiable
at x = 0.
...(B)
= lim

af

af

Differentiability at a Point

(ii) Continuity and differentiability test at x =

F I = 2 + F I
f
H 2K
H 2 2K

=2

(c1)

= 1+1=2
2

(c2)

af

R| F
S| GH
T

= lim 2 + x

IJ
K

(c1), (c2) and (c3)

af

af

which f x is continuous at x =

...(c3)

F I
H 2K

af

lim f x = lim + f x = f

x
2

U| = 2
V|
W

L f

e j = lim

h f
h

h0

1 + sin

= lim

h0

2 sin
= lim

h0

h 2

(C)

= lim

1 cos h
h

F hI
F hI
H 2 K = lim sin H 2 K sin F h I
h
F hI H 2 K
H 2K
h 0

(d1)

h0

j e j , (h > 0)

+h f
h

(d2)

e j = R f e j

which

a
f
f
x
, 0 x < 1,
f axf =
2
2

5. If

3
, 1 x 2 discuss the continuity
2

af

x
, 0 x <1
2

= 2 x 3x +

af

...(a1)

3
, 1 x 2
2

f x = x, 0 x < 1

e j = lim

h
= lim h = 0
h0
h

the given piecewise function f (x) is differentiable

...(D)
at x =
2
In the light of the results (A), (B), (C) and (D), we
observe and declare that f (x) is continuous in the
interval , but it is not differentiable in
, .

f x =

e j , (h > 0)

=1 0= 0
R f

of f, f , and f on [0, 2].


Solution: (i) Given is

h 0

2
2

= 2 x2 3 x +

Again,

+h

(d 1 ) and (d 2 ) L f

x
2

h0

= lim f x
2 x +

= lim

h0

af

r. h.l at x =

= lim

l.h.l at x = = lim f x = lim 1 + sin x

2 x
x
= 1 + sin

2+

343

af

= 4 x 3, 1 x 2

f x = 1 , 0 x < 1

a f LMN

3
2

= 2 1 31+

OP
Q

...(a4)
...(a6)

x =1

3
3 1
= 1 + =
...(b1)
2
2 2

af

l.h.l at x = 1 = lim f x
x 1

...(a3)
...(a5)

= 4, 1 < x 2
(ii) Continuity test at x = 1 for f (x),
f 1 = 2 x 3x +

...(a2)

344

How to Learn Calculus of One Variable

Fx I = 1
GH 2 JK 2
r. h.l at x = 1 = lim f a x f

= lim x = 1

x 1

= lim

...(b2)

x 1

x1

F
H

x 1

x 1

I
K

3
3 1
= 2 3 + = ...(b )
3
2
2 2

af

continuous at x = 1 for f x ,

af

x =1

= 4 1 3= 4 3=1

...(c1)

r. h.l at x = 1 = lim+

af
a f ...(c )
f a x f = lim a4 x 3f

= (4 3) = 1

...(c3)

l.h.l at x = 1 = lim f x = lim x = 1


x 1

x1

x1

af

x 1

af
r. h.l at x = 1 = lim f a x f = lim 4 = 4 (d )
(d ) and (d ) f a x f is discontinuous at

af

L f 1 = lim

h0

af

R f 1 = lim

Note: f (x) does not exist at x = 1.


6. A function f is defined as follows:
f (x) = x for 0 x 1 and f (x) = 2 x for x 1 .
Test the character of the function at x = 1 as regards
its continuity and differentiability.
Solution: (i) Continuity test at x = 1
f (1) = 1
(a1)

af

l.h.l at x = 1 = lim f x
x 1

h0

= lim

h0

af

f 1 h f 1
, (h > 0)
h

a1 hf 1 = lim
h

h0

h
h

h0

(b1)

f af

f 1+ h f 1
, (h > 0)
h

2 1+ h 1

a f

= lim

h 0

= lim 1 = 1

x = 1.
Hence, in the light of above observation and the
results, we declare that the function f and f are
continuous in [0, 2] but f is discontinuous in [0, 2]
since f is discontinuous at x = 1 0 , 2 .

which

= lim 1 = 1

x 1

x 1

h0

= lim

l.h.l at x = 1 = lim f x = lim 1 = 1 (d1)


+

a f af

x 1

given piecewise function is continuous at x = 1


(ii) Differentiability test at x = 1

h0

for f x

x 1

af

(a3)

lim f x = lim+ f x = f 1

x 1

x 1

(a1), (a2) and (a3)

(c1), (c2) and (c3) f x is continuous at x = 1

af

= lim 2 x = 1

(b1), (b2) and (b3) l.h.l at x = 1 = r.h.l at x = 1 =


value of the function f (x) at x = 1 which f (x) is
f 1 = 4x 3

af

r. h.l at x = 1 = lim+ f x

x 1

= lim 2 x 3 x +

(a2)

af

af

h
h
(b2)

(b1) and (b2) L f 1 R f 1 which


given piecewise function is not differentiable at x = 1
Type 2: Problems based on redefined functions:

af

Questions: (i) If f x =

x
1+ e

1
x

x 0 f (x) = 0 ,

af

x = 0 show that f is continuous at x = 0 but f 0


does not exist.
Solution: (i) Continuity test at x = 0

af

r.h.l at x = 0 = lim + f x
x0

= lim

h 0

b0 + hg

e j
1+ e
1
0+ h

, h > 0 = lim

h 0

h
1

1+ e h

=0

(a1)

345

Differentiability at a Point

af

Solution: 1. Continuity test at x = 0


f (0 ) = 0

l.h.l at x = 0 = lim f x
x0

= lim

h 0

b0 hg , h > 0 = lim

e j
1+ e

1 + ed

h 0

1
0h

1
h

= 0 (a2)

f (0) = 0
(a3)
(a1), (a2) and (a3) l.h.l at x = 0 = r.h.l at x = 0 =
value of the function at x = 0 which given redefined
function f (x) is continuous at x = 0.
(ii) Differentiability test at x = 0

af

R f 0 = lim

h0

f af

= lim

1+ e

h0

af

h0

h
= lim

h0

1+

h0

1
1+

e h

h0

F
h G1 + e
GH

1
h

I
JJ
K

(b1)

f af

1
=1
1+ 0

af

I
JJ
K

af

which

f 0 does not exist.

2. If

b g
b0 h g

sin 0 h

, h>0

= lim

h 0

F sin h
GH h
2

h 0

, x 0 f (x) = 0 , x = 0 discuss

the continuity and differentiability of f (x) at x = 0.

(a3)

(a1), (a2) and (a3) f (x) is continuous at x = 0


(ii) Differentiability test at x = 0

R| FG sin a0 + hf 0IJ U|
| H a0 + hf K |V , h > 0
R f a0f = lim S
|| a0 + hf ||
W
T
h0

sin h
h

=1

(b1)

R| F sin b0 hg
|| GGH b0 hg
L f b0g = lim S
|| b hg
|T

h0

(b2)

(b1) and (b2) R f 0 L f 0

af
1
f a x f = sin x
x

1
h

x0

(a2)

R| sin h a hfU|
V|
S| h
W
T
I lim a hf = 1 0 = 0
JK

h0

F
h G1 + e
GH

bg

x0

0
h0

h=1 0= 0

l.h.l at x = 0 = lim f x = lim

= lim

= lim

, h>0

h 0

1
eh

b g
b0 + h g

sin 0 + h

h0

f 0h f 0
,h>0
h

1+ e
h

= lim

h 0

0
= lim

sin h

= lim

= lim

=0
L f 0 = lim

1
h

= lim

h0

x0

f 0+h f 0
,h>0
h
h

bg

r.h.l at x = 0 = lim+ f x = lim

(a1)
2

= lim

sin h

I U|
JJK |
|V , h > 0
||
|W

=1
(b2)
2
h
(b 1 ) and (b 2 ) R f 0 = L f 0 which
redefined function f (x) is differentiable at x = 0
and f 0 = 1 .
h0

af

af

af

346

How to Learn Calculus of One Variable

3. If f (x) = x2 sin

F 1I
H xK

= lim

af
af

= lim

h0

af

RSa 1f ahf a 1f sin F 1 I UV


H hK W
T

= lim h sin

l.h.l at x = 0 = lim f x

h0

F 1 IJ , h > 0
= lim a0 h f sin G
H 0 hK
F 1 I = 0 (a )
= lim a 1f eh j sin
H hK
r. h.l at x = 0 = lim f a x f
F 1 IJ , h > 0
= lim a0 + h f sin G
H 0 + hK
F 1I = 0
= lim eh j sin
(a )
H hK
2

h0

h0

x0

h0

show that
(i) f (x) is continuous at x = 0
(ii) f 0 = 0
(iii) f x is discontinuous at x = 0
Solution: (i) Continuity test at x = 0
x0

h0

F 1I
H hK

h sin

for x 0 f (x) = 0 for x = 0

1
=0
h

af

...(b2)

af

(b1) and (b2) L f 0 = R f 0 = 0 which

af

f 0 =0

af
F 1 I cos F 1 I at
3 f a x f = 2 x sin
H xK H xK
f a0f = 0 (already determined)
Now, r.h.l at x = 0 = lim f a x f

(iii) To show that f x is discontinuous at x = 0.

x0
(c1)

x0

R| b g F 1 I cos F 1 I U|
S|T
GH 0 + h JK GH 0 + h JK V|W , h > 0
f (0) = 0
(a )
R F 1 I cos F 1 I UV
= lim S2 h sin
(a ), (a ) and (a ) lim f a x f = lim f a x f
T H hK H hKW
F 1 I lim cos F 1 I
= f a0f which redefined function f (x) is
= 2 lim h sin
H hK
H hK
continuous at x = 0
(ii) To find f a0f :
F 1I
= 0 lim cos
H hK
F
1 I
b0 + hg sin GH 0 + h JK 0
F 1 I which does not exist r.h.l at
,h>0
= lim cos
R f b0g = lim
H hK
h
x = 0 of the function f a x f does not exist. (c )
F 1I = 0
= lim h sin
...(b
)
H hK
Again, l.h.l at x = 0 = lim f a x f
b0 hg sin FGH 0 1 h IJK 0
R|
F 1 IJ cos FG 1 IJ U|V
= lim S2 b0 hg sin G
,
h
>
0
L f b0g = lim
H 0 h K H 0 h K W| , h > 0
T|
h
R
F 1 I cos F 1 I UV
= lim Sa 2f ahf sin
H h K H hK W
T
2

h0

= lim 2 0 + h sin
h0

x0

x0

h0

h0

h0

h0

h0

h0

h0

x0

h 0

h0

h0

347

Differentiability at a Point

a f a f FH h1 IK lim cos FH h1 IK
b3 cos af = cos g
F 1 I lim cos F 1 I
= a 2f lim ahf sin
H hK
H hK
F 1I
= a 2f 0 lim cos
H hK
F 1 I which does not exist l.h.l at
= lim cos
H hK
= lim 2 h sin
h0

h0

h0

x = 0 of the derived function f ' (x) does not exist


(c3)

af

Hence, (c2) or (c3) the derived function f x


is discontinuous at x = 0, (of second kind).

RS
T

af

UV
W
R 1
U
= lim Sh + h sin log h V
3
T
W
h0

jUVW

1
2
sin log x
, x0
3
f (x) = 0 , x = 0. Test the continuity and differentiability
at x = 0
Solution: (i) Continuity test at x = 0
f ( 0) = 0
(a1)
4. If f x = x 1 +

= lim h + lim
h0

a f RST1 + 13 sinloga0 - hf UVW , h > 0


L 1
O
= lim M h h sinlog h P
N 3
Q
1
= lim a hf lim h sin log h
3
2

h0

a0 hf RST1 + 13 sin log a0 hf UVW 0


2

af

L f 0 = lim

h0

h>0

RS
T

= lim 1 +
h0

= lim 1 +
h0

af

= lim 0 h

h0

=1+

h0

h0

1
2
1
lim h sinlog h = 0 0
h
3 0
3
( 3 Product of an infinitesimal and a bounded
function = 0, in the limit)
=00=0
(a2)
=0

af

r.h.l at x = 0 = lim + f x
x0

= lim 0 + h
h0

f RST1 + 13 sin loga0 + hf UVW , h > 0


2

1
2
sin log h
3

UV
W

b3 f a0f = 0g

1
2
lim sin log h
3 h0

1
2
lim sin log h
h

0
3
2

but lim sin log h oscillates between +1 and 1.


h0

af

L f 0 does not exist.

h0

2
1
h sin log h = 0 + 0
3

=0
(a3)
(a1), (a2) and (a3) l.h.l at x = 0 = r.h.l. at x = 0
= value of the function at x = 0 which f (x) is
continuous at x = 0.
(ii) Differentiability test at x = 0

l.h.l at x = 0 = lim f x
x0

1
2
sin log h
3

h0

h0

h0

RS
T

= lim h 1 +

h0

af

h0

RS
T

= lim 1 +
h0

a0 + hf RST1 + 13 sin loga0 + hf UVW 0


2

R f 0 = lim
h>0

(b1)

1
2
sin log h
3

UV
W

2
2
1
lim sin log h but lim sin log h
h

0
h0
3 h0
oscillates between +1 and 1.

= lim 1 +

af

R f 0 does not exist.

(b2)

(b1) or (b2) f (x) is not differentiable at x = 0.

348

How to Learn Calculus of One Variable

On Derivative of y = [x]
Let y = f (x) = [x], where D ( f ) = R, i.e.; x R
there are two cases:

Case 1. When x is an integer.


considering f (x) = [x] at x = N, it is seen that for
h>0
h0

h0

= N + 0 = N 3 x + n = n + x ,n I
h0

h0

a f
x Ig

= lim N + -1 lim h 1
h0

b3 x

= x 1 if

a f

h0

f (N) = N
Hence, the function y = [x] is discontinuous at
x = N; through it is right continuous at this point
y = [x] can not have derivative at any integral value of x.
Case 2: When x is non-integer (or, non-integral).
Considering f (x) = [x] at x = N + h, where 0 < h < 1, it
is seen that for x > 0
N + h + x

= lim

N + lim

x 0
x 0

x 0

lim

= lim

N + lim

x 0

N +0 N 0
x

FG N N IJ = 0
H x K
f a N + h x f f a N + hf
f a N hf = lim
= lim

x 0

x 0

= lim

N + h x N + h
x

= lim

N + h x N h
x

= lim

N +0 N 0
x

x 0

x 0

FG N N IJ = 0
H x K
f a N + h f = f a N hf = 0 , x I
= lim

h + x

x 0

(i)

N + h x

x 0

= lim

x 0

= N + 0 = N ( 3 [x + n] = n + [x], n I )
x 0

N + h + x N h
x

= N + 1 0 1= N 1

= lim

= lim

x 0

lim N h = lim N + lim h

h0

f a

N + h + x N + h
x

x 0

x 0

f N + h + x f N + h
x

= lim

x 0

lim N + h = lim N + lim h

h0

f + N + h = lim

h x

= N + 0 = N as h x < 1
(ii)
f (N + h) = [N + h] = N
(iii)
Hence, (i), (ii) and (iii) the function y = [x] is
continuous at a non-integer point x = N + h.
Again, we know that continuity of a function y =
f (x) at any point x = a
/ differentiability of the
function y = f (x) at the same point x = a.
Hence, the derivative test of y = [x] at a non-integer
point namely x = N + h, (0 < h < 1), is required.

y = [x] has the derivative zero at any nonintegral point namely x = N + h, where N = an integer
and h is small positive number such that 0 < h < 1.
On Differentiability
Type 1. Problems based on piecewise functions.
Exercise 7.1
1. Test the differentiability of the following functions
at the indicated points.

Differentiability at a Point

(i)

R|
|| 1 ,
f a x f = S 1 + sin x ,
|| F I
|T2 + H x 2 K

at x =

when x < 0
when 0 x <
2

when

x
2

and non-

differentiable at x = 0.

R|
|| 1,
f b x g = S 1 + sin x ,
|| F I
|T2 GH x 2 JK ,

when x < 0
when 0 x <

when x
2

Answer: Non-differentiable at both points x =

(iii)

and
2

when 0 < x < 1


when 1 x 2
when x > 2

(iv)

x,

when x < 0

when 0 x 1

x ,
2

x + 1,

when 0 x < 2
when 2 x < 3

at x = 1 and x = 2. Where [x] is the greatest integer not


greater than x.
Answer: Not differentiable at both points x = 1 and
x = 2.
2. If the function f (x) is defined by

R|3 + 2 x ,
f axf = S
|T3 2 x ,

when x > 1

at x = 0 and x = 1.
Answer: Non-differentiable at both point x = 0 and
x=1

3
<x0
2
3
when 0 < x <
2

when

show that D f (0) does not exist.


3. If the function f (x) is defined by
2

at x = 1 and x = 2.
Answer: Non-differentiable at x = 1 and differentiable
at x = 2

R|
f axf = S
|Tx

a f RSax x 1xf ,x ,
T

a f |RS|xx ,,
T

x = 0.

R| x ,
|
f bxg = S 2 x ,
||x 1 x ,
T 2

when x > 1

at x = 1.
Answer: Non-differentiable at x = 1

f x =

.
2

at x = 0 and x =

when 0 x 1

(vi) f x =

and x = 0.
2

Answer: Differentiable at x =

(ii)

a f RST2 xx, 1 ,

(v) f x =

349

when x 0
when x < 0

then find left hand derivative and right hand derivative


at x = 0. Is f (x) differentiable at x = 0?
Answer: Non-differentiable at x = 0
4. Does the differential coefficient of the following
function exist at x = 0 and x = 1

R|1 x ,
f b x g = S1 + x ,
|Tx x + 2 ,
2

when x < 0
when 0 x 1
when x > 1

Answer: Non-differentiable at both points x = 0 and


x = 1.
Type 2: Problems based on redefined functions:
Exercise 7.2

R F 1I
a f |Sx sin H x K ,
|T 0 ,
differentiability at x = 0.
1. If f x =

when x 0
when x = 0

Answer: Non-differentiable at x = 0

test the

350

How to Learn Calculus of One Variable

2. Show that

R|x
f a xf = S
|T

is differentiable at x = 0.

R
a f |Sax 2f
|T

3. If f x =

sin
0,

sin

F 1I ,
H xK

0,

FG 1 IJ ,
H x 2K

for x 0
for x = 0

when x 2
when x = 2

test the differentiability at x = 2.


Answer: Differentiable at x = 2.

R
a f |Sx
|T

43

4. If f x =

cos
0,

F 1I ,
H xK

5. If

x 1
2

7x + 5
1
,
3

when x 0
when x = 0

, when x 1
when x = 1

test the differentiability at x = 1


Answer: Differentiable at x = 1.

R|sin FG 1 IJ ,
b g S H xK
|T 0 ,

6. If f x =

x0
x=0

test the differentiable at x = 0


Answer: Non-differentiable at x = 0

FG 1 IJ , when x 1 ; f a1f =
2
H x 1K
g a x f = a x 1f f a x f for all x, test the

af

7. If f x = tan
and if

1
x

I
JJ
K

for x 0

f (0) = 0 at the point x = 0.


Answer: Not differentiable at x = 0.
11. Examine the differentiability of the function

a f a x 1 af cosec FGH x 1 a IJK ,

xa

f (a) = 0 at the point x = a.


Answer: Find.

af

1
sin a x for x 0
x
f (0) = 1 for differentiability at x = 0 where a is a
positive constant such that a 1 .
Answer: Find.
13. Discuss the differentiability of the function
12. Test the function f x =

af

f x =e

sin

F 1 I , when x 0 f (0) = 0 at
H xK

the point x = 0.
Answer: Differentiable at x = 0
14. The function f is defined as follows

af

f x =e

1
x

, x 0 f (0) = 0 test the differen-

tiability at x = 0
Answer: Differentiable at x = 0.
Type 3: Problems based on mod functions:
Exercise 7.3

differentiability of g (x) at x = 1.
Answer: g (x) is differentiable at x = 1

af

1
2
sin x , when x 0 = 0, when x = 0
x
test the differentiability at x = 0.
Answer: Differentiable at x = 0.
9. Discuss the differentiability of the function

8. If f x =

bg

F
f a x f = G1 e
GH
f x =

test the differentiability at x = 0


Answer: Differentiable at x = 0.

R|
f b xg = S 2 x
||
T

Answer: Not differentiable at x = 0.


10. Test the differentiability of the function

f x = x cos

FG 1 IJ , for x 0 f (0) = 0 at the point x = 0.


H xK

1. Is | x | differentiable at x = 0?
Answer: Not differentiable at x = 0.
2. Test the differentiability of the function f (x) =
| x 1 | at x = 0 and x = 1.
Answer: Differentiable at x = 0 and non-differentiable
at x = 1.
3. Show that f (x) = | x 1 | + | x + 1 | is differentiable
at all points except x = 1 and x = 1.

Differentiability at a Point

af

4. Test the differentiability of the function f x =

x
x

at x = 1.
Answer: Differentiable at x = 1.
5. Is | x 1 | differentiable at x = 1?
Answer: Non-differentiable at x = 1.

af

af

(ii) f x =

Type 4: To find the values of the constants so that a


given function becomes differentiable at a given point
x = a.

1
, when x 1
x

= a x2 + b , when | x | < 1
Answer: a =

differentiable at x = 0?
6. Is f x = e
Answer: Non- differentiable at x = 0.

351

1
3
and b = .
2
2
1

(iii)

a x 1f , when x < 1
f a xf = 2

= a x2 + b x , when x 1 .
Answer: a = b = 0.
On Continuity and Differentiability

Exercise 7.4

Type 1: Problems based on piecewise functions:

1. For what choices of a, b, c if any does the function

R|x ,
f b x g = Sa x + b ,
|Tc ,
2

x0
0< x 1
x >1

becomes differentiable at x = 0 and x = 1.


Answer: a = b = c = 0.
2. For what choice of a, b, c if any does the function

R|a x b x + c ,
f b x g = Sb x c ,
|Tc ,
2

when 0 x 1
when 1< x 2
when x > 2

becomes differentiable at x = 1 and x = 2.


Answer: a = b = c = 0.
3. If f (x) = x2 + 2x, when x < 0, = a x + b, when x >
0 find a and b so as to make the function f (x)
continuous and differentiable at x = 0.
Answer: a = 2 and b = 0.
4. Find the constants a and b so as to make the
following function f (x) continuous and differentiable
for all x where
(i) f (x) = x2 , when x K
= a x + b , when x > K
Answer: a = 2K and b = K2

Exercise 7.5
1. Test the differentiability of the function

a f RST2 xx, 1 ,

f x =

when 0 x 1
when 1 < x

at x = 1. Is the function continuous at x = 1.


Answer: Non-differentiable at x = 1 but continuous
at x = 1.
2. Prove that the function f (x) defined by

R|3 + 2 x ,
f a xf = S
|T3 2 x ,

3
<x0
2
3
when 0 < x <
2

when

is continuous at x = 0 but D f (0) does not exist.


Answer: Continuous but non-differentiable at
x = 0.
3. If f (x) = 1 + x, when x 2
= 5 x, when x > 2
test the continuity and differentiability of the function
f (x) at x = 2.
Answer: f (x) is continuous every where and
differentiable every where except x = 2.
4. Discuss the continuity and differentiability of the
function where f (x) = x2, when x < 2
= 4, when 2 x 2
= x2, when x > 2.

352

How to Learn Calculus of One Variable

Answer: f (x) is continuous every where and


differentiable everywhere except x = 2 and x = 2.
5. A function f (x) is defined as follows
f (x) = 1 for < x < 0

= 1 + sin x for 0 x <


2

F
H

=2+ x

I
K

for

x < .
2

Discuss the continuity and differentiability of f (x)

.
2
Answer: Non-differentiable at x = 0 and differentiable

but it is continuous at both points x = 0 and


at x =
2

x= .
2
6. Prove that the function f defined by f (x) = 3 2x ,
when x < 2 = 3 x 7, when x 2 is continuous but
not differentiable at x = 2.
7. A function f is defined by

at x = 0 and x =

a f RST35xx 42,,

f x =

when x 1
when x > 1

is the function f (x) continuous and differentiable at


x = 1?

8. If

R| 0 ,
|
f a x f = S sin x ,
||
|Tcos x ,

when x < 0

when 0 x
4

when x >
4

Answer: Continuous but not differentiable at x = 0.


11. A function f (x) is defined as follows
f (x) = x for 0 x 1 , f (x) = 2 x for x 1 . Test
the character of the function at x =1 regarding its
continuity and differentiability.
Answer: Continuous but non-differentiable at
x = 1.
12. Show that the function f (x) defined by
f (x) = x2 1, when x 1 = 1 x , when x < 1 is
continuous but not differentiable at x = 1.
Type 2: Problems based on redefined functions:
Exercise 7.6

bg

1
sin x 2 , when x 0
x
= 0 , when x = 0
discuss the continuity and differentiability of f (x) at
x = 0.
Answer: Continuity and differentiable at x = 0.
1. If f x =

af

2. If f x = x tan

F 1 I , when x 0
H xK

= 0 , when x = 0
examine the continuity and differentiability at
x = 0.
Answers: Continuous but bot differentiable at
x = 0.

af

3. If f x = e

1
x

sin

F 1 I , when x 0
H xK

= 0 , when x = 0
show that f (x) is differentiable at x = 0.

af

show that f (x) is continuous but not differentiable at


x = 0.
9. A function f (x) is defined as follows:
f (x) = 1 + x, when x 2 , = 5 x , when x > 2. Test

4. If f x =

the character of the function at x = 2 as regards its


continuity and differentiability.
Answer: Continuous but non-differentiable at
x =2.
10. Examine whether f (x) is continuous and has a
derivative at the origin when f (x) = 2 + x , when x 0
f (x) = 2 x , when x < 0.

show that f (x) is continuous at x = 0 but f 0 does


not exist.

1+ e

1
x

, when x 0

f (0) = 0

af

af

5. Show that f x = x sin

F 1I ,
H xK

f (0) = 0 is differentiable at x = 0.

for x 0

Differentiability at a Point

F
x Ge
H
6. If f b x g =
Fe
GH

1
x

1
x

e
+e

I
JK
I,
JK

(i) f (x) = | cos x | at x =

1
x

1
x

x0

=0, x=0
show that f (x) is continuous but not differentiable at
x = 0.
Type 3: Problems based on mod functions:
Exercise 7.7
1. Let f (x) = | x 2 |. Is the function continuous and
differentiable at x = 2?
2. Examine the continuity and differentiability of the
following functions at the indicated points:

Answer: Continuous at x =

353

but non-differentiable
2

.
2
(ii) f (x) = | x3 | at x = 0
Answer: Continuous at x = 0 and also differentiable
at x = 0.
(iii) f (x) = 1 + | sin x | at x = 0
Answer: Continuous at x = 0 but non-differentiable
at x = 0.
(iv) f (x) = | x 2 | at x = 2
Answer: Continuous at x = 2 but non-differentiable
at x = 2.
x
(v) f x =
at x = 0
x
Answer: Discontinuous and non-differentiable at
x = 0.
at x =

bg

354

How to Learn Calculus of One Variable

8
Rules of Differentiation

Before we gather the rules of differentiation, let us


recapitulate some phrases and concepts frequently
used in calculus.
1. Differentiation at any limit point (or, simply
differentiable): A function f is said to be differentiable
at any limit point x (or, simply differentiable), where x
is assumed to have any finite value in D(f)

lim

h0

f af

f x+h f x
h

bg b

is

finite

when

g bg

x D f and x + h D f is the same whether


h 0 through positive values or through negative
values. The value of this limit for every finite value of
x is called derivative of the function f at x and is

af
f a x + hf f a x f
,

denoted as f x , i.e.,

af

f x = lim

where x is suph
posed to have any finite value in D (f), whether h 0
through positive values or through negative values.
2. Differentiability at a point: A function f is said to
be differentiable at a point x = a, a being a particular
h 0

bg

f af

f a+h f a
is
h
finite for the finite given value of x D (f) and is the
same whether h 0 through positive values or
through negative values. The value of this limit is
called derivative of the function f at x = a and is
finite value D f

af

lim

denoted as f a , i.e.,

h0

af

f af

f a+h f a
, where a is a
h
particular finite value given for x D f and h 0
through positive values or through negative values.
f a = lim

h 0

bg

Remark:
1. The result obtained after the evaluation of the limit,

f af

f af

f x+h f x
, for the function f at the limit
h
point x is called derivative of f at any limit point x (or,
simply at x or at the point x) or derived function of f at
any limit point x because it is derived from the function
f at the limit point x and is symbolised as f (x) for
every finite value of x in its domain (whereas the
derivative or derived function is symbolised as f for
the function f) while the function f at the limit point x
is said to be derivable or differentiable at any point x
(or, simply, at x) provided the limit,
lim

h 0

f x+h f x
is finite for every finite value
h 0
h
of x and is the same whether h 0 through positive
values or through negative values.
2. Instead of saying that the function f is differentiable
at any limit point x (or, at a point, or, at a particular
point x = a), we also say that the function f has a
derivative at any limit point x (or, at a point, or, at a
particular point x = a) in D (f).
3. If the function f is written as y = f (x), then its
derived function is also symbolised as
lim

y =

af

b a fg

af

d
d
d
y =
f x =
f x .
dx
dx
dx

Rules of Differentiation

4. By an abuse of language, it has been customary to


call f (x) as function instead of f.
5. It is also said that a function y = f (x) is differentiable
at some point x (or, at a particular point x = a, or, at
any point x) instead of the function f is differentiable
at some point x (or, at a particular point x = a, or, at
any point x) or f (x) is a differentiable function of the
independent variable x.
Now we explain the general rules of differentiation
which are actually the fundamental theorems on
differentiation.
1. Show that the function which is a constant times a
differentiable function is differentiable and the
derivative of a constant times a differentiable function
is the constant times the derivative of the function.
Solution: Let y = a f (x), where a is a constant and f
(x) is a differentiable function of the independent
variable x. It is required to show that y = a f (x) is
differentiable and

af

af

af

d
y =af x .
dx

3y =af x

y + y = a f x + x

af

x0

af

af

a f x + x a f x
x

= a lim

f af

f x + x f x
x

af

d
y =af x
dx
Hence, y = a f (x) is a differentiable function and
d
d
y =
a f x = a f x which means while
dx
dx
differentiating a constant times a differentiable

af

b a fg

y
d
y = lim
x 0 x
dx

= x0

y
d

y = lim
x0 x
dx

x0

a y f = f a x f + g a x f
3 y = f a xf + g a xf
y + y = f a x + xf + g a x + x f
y = f a x + x f + g a x + x f b f a x f + g a x fg
y f a x + x f + g a x + x f b f a x f + g a x fg

d
dx

= lim

af

bg
f a x + x f + g a x + x f b f a x f + g a x fg
lim

y = a f x + x a f x
x

function, the constant (or, numerical factor) may be


d
.
taken out of the differentiation symbol
dx
2. Show that the sum of two separate functions
differentiable on a common domain is differentiable
on the same common domain and the derivative of
the sum of two differentiable functions is the sum of
the derivatives of the separate functions.
Solution: let y = f (x) + g (x) be the sum of two
functions, say f (x) and g (x) differentiable on a
common domain (or, interval) I. It is required to show
that the sum function y = f (x) + g (x) is also
differentiable on the same common domain I and

a
f af
y a f a x + x f a f a xf

af

355

= lim

x0

b g

x0

= lim

x0

x0

f x + x
g x + x
+ lim
x0
x
x

f x

lim

lim

lim

bg

g x

x0

g bg

f x + x f x
+
x

f af

af

af

g x + x g x
= f x + g x
x

Hence, y = f (x) + g (x) is differentiable on the


interval I and

356

How to Learn Calculus of One Variable

af

b a f a fg = f axf + g axf

d
d
f x +g x
y =
dx
dx

Cor: The sum of a finite number of functions


differentiable on a common domain is differentiable
on the same domain and the derivative of the sum of
a finite number of differentiable functions is the sum
of their derivatives, i.e.
If f1, f2, f3, .. fn be differentiable functions at the
same point x, then y = f1 (x) + f2 (x) + + fn (x) is also
differentiable at the same point x and
d
y = f 1 x + f 2 x + ... + f n x which is
dx
known as extension rule (or, formula) for the derivative
of the sum of a finite number of a differentiable
functions.

af

af

af

af

3. Show that the product of two functions


differentiable on a common domain is differentiable
on the same common domain and the derivative of
the product of two differentiable functions is the first
function times the derivative of the second function
plus the second function times the derivative of the
first function.
Solution: Let y = f (x) g (x), where f (x) and g (x) are
functions differentiable on a common interval I. It is
required to show that the product function y =
f (x) g (x) is differentiable on the same common
interval I and

b a f a fg a f a f a f a f
3 y = f a x f g a xf
y + y = f a x + xf g a x + xf
y = f a x + xf g a x + x f f a x f g a x f
y f a x + xf g a x + xf f a xf g a xf

=
d
f x g x = f x g x + g x f x
dx

af a

Now adding and subtracting f x g x + x


in numerator, we have

y
=
x

f a

f af a

f af a

f af af

f x+ x g x+ x f x g x+ x + f x g x+ x f x g x
x

lim

x0

lim

x0

LM b
MN

bg

g bg b

f x+ x f x
d
y = lim
g x+ x
x0
x
dx

gOPP +
Q

LM g b x + xg g b xg f b xgOP
MN x
PQ
f b x + xg f b xg
lim g b x + x g +
lim
x
g a x + xf g a xf
lim f a x f

x0

x0

bg bg
g a x f f a x f

x0

bg bg bg bg

= f x g x + g x f x = f x g x +

Hence, y = f (x) g (x) is differentiable on the


common interval I and

b g c b g b gh = f b xg g b xg +
g a x f f a x f
d
d
y =
f x g x
dx
dx

Note: It is immaterial to consider any function of the


two given functions as the first function and the
second function, i.e., any one of the two given
functions may be considered as the first function and
the other one as the second function. This is why we
are at liberty to consider any one of the two given
functions as the first function and the other one as
the second function.
Cor: The product of a finite number of functions
differentiable on a common domain is differentiable
on the same domain and the derivative of the product
of a finite number of differentiable functions is the
sum of the n terms obtained by multiplying the
derivative of each one of the factors by the other (n
1) factors undifferentiated, i.e. if f1, f2, , fn be separate
functions differentiable on a common interval I (or, at
any point x), then y = f1 f2 f3 fn is also differentiable
on the same common interval I (or, at the same point
x) and

Rules of Differentiation

af d

i a f
d
d
f i d x i + ... + a f f ... f f
dx
dx

d
d
fn +
y = f 1 f 2 f 3 ... f n 1
dx
dx

df f
1

... f n 1

n 1

Solution: Let y =

g (x) are functions differentiable on a common domain

FG f a xf IJ = g a xf f axf f axf g axf


H g axf K
b g a xfg
f a xf
, g a xf 0
3y =
g a xf
f a x + xf
y + y =
g a x + xf
f ax + xf
f a xf
y =

g a x + xf
g a xf
f a x + xf g a x f g a x + xf f a xf
=
g a x + xf g a xf
f a x + xf g a x f g a x + x f f a xf
y

=
x
x g a x + x f g a xf
d
dx

e j

af

af

x
d
d
=1
y =
x = lim
x0 x
dx
dx

Now, x n = x x x ... x (upto n factors) is


differentiable and using the extension rule for the
derivative of product of a finite number of
differentiable functions, we have

e j b g bg b g bg
d
... + a x x ... x f
axf [each product within the
dx
d
d
d n
x +
x + x x ... x
x = x x ... x
dx
dx
dx

bracket contains (n 1) factors undifferentiated.]

=x
= x

n 1

n 1

af

af af
af

f x
, g x 0 is
g x
differentiable on the same common domain I and
I. It is required to show that y =

n
d
x using the extension rule for
Question: Find
dx
the derivative of the product of a finite number of
differentiable functions, n being a positive integer.
Solution: Let y = x be a differentiable function

af af
af

f x
, g x 0 , where f (x) and
g x

(f1), where each product within the bracket contains


(n 1) factors undifferentiated. This is known as the
extension rule for the derivative of the product of a
finite number of differentiable functions.

af

n 1

F3 d a xf = 1I
H dx K
4. Show that the quotient of two functions
differentiable on a common domain is differentiable
on the same common domain excepting the points (or
common domain) where the function in denominator
is zero and the derivative of the quotient of two
differentiable functions is the function in denominator
times the derivative of the function in denominator
minus the function in numerator times the derivative
of the function in denominator, all divided by the
square of the function in denominator.

Now adding and subtracting f (x) g (x) in


numerator, we have

n 1 d
d
x (upto n-times)
x + ... + x
dx
dx

(1 + 1 + upto n-times) = n x

357

y
=
x

f af

af af
a

af af
f af

af a

f x + x g x f x g x + f x g x f x g x + x
xg x + x g x

af

y
d
1
= lim
y = lim
x 0 x
x0 g x + x g x
dx

f af
LM f a x + xf f a xf g a xf g a x + xf g a xf f a xfOP
x
x
N
Q
1
=
b f a x f g a x f g a x f f a x fg
g a xf g a xf
g a x f f a x f f a x f g a x f
=
b g a xfg

358

How to Learn Calculus of One Variable

af
af

f x
is
g x
differentiable at every point of the common domain I
excepting only those points (of common domain) at
which the function in denominator is zero and
Hence, the quotient function y =

af

b a f a fg
g a x f f a x f f a x f g a x f
, g a xf 0
=
b g a xfg

d
d
y =
f x g x
dx
dx

Notes:
1. We should note that in the formula of the derivative
of quotient of the two functions differentiable on a
common domain (or, at any point x), the derivative of
the function in denominator occuring in numerator is
always with negative sign.
2.

d
dx

FG 1 IJ = f a xf
H f a xfK b f a xfg

af
af

f x
of two functions
g x
differentiable at the point x = a, is a function which is
differentiable at the same point x = a, provided
g a 0.
3. The quotient y =

af

af
af

f x
of two functions
g x
differentiable at every value of x is a function which
is differentiable at every value of x, provided g x 0
at those values of x at which g (x) is differentiable.
5. One can differentiate a function at only those
points at which it is defined. e.g.
4. The quotient y =

af

a f g a1xf dxd b f a xfg = dxd FGH g 1axfIJK


g a x f
,
=
b g a xfg but this is not valid when g (x) = 0 for

(i) f x =

1
is undefined when g (x)
g x
= 0 for any value of x. So it is better to write
any value of x since

af

b a fg = dxd FGH g a1xfIJK = b gga' axxfgf , gaxf

d
f x
dx

for

any x.

af

af

b a fg

a fg

d
d
log g x =
f x =
dx
dx

(ii) f x = log g x

af
bg
af
af
g a x f
d
d
log g a x fg =
, g a xf > 0 .
f a x fg =
b
b
dx
dx
g axf

g x
, but this is not valid when g x 0 for any
g x
value of x because log g (x) is not defined when
g x 0 . So, it is proper to write

bg

c b gh

b g
a f
a f
a f
a

d
d
tan x = sec 2 x ,
f x =
dx
dx

but this is not valid when x = 2n + 1


because
2

tan x is not defined for x = 2n + 1 . Hence, it is


2

d
2
tan x = sec x , x 2n + 1 .
proper to write
2
dx
6. Even if a function is defined at a point, its
derivative need not exist at that point. e.g.
(iii) f x = tan x

af

(i) f x =

x exists (i.e., it is defined) for x > 0 but


1
d
x =
its derivative
does not exists at
dx
2 x

e j

e j

1
d
, x > 0.
x =
dx
2 x
1
(ii) f x = sin x exists (i.e., it is defined) for

x = 0. So, it is proper to write

af

| x | < 1 but its derivative

1
d
sin x =
dx

1
1 x

and does not exist for x = 1 . So, it is proper to write

1
1
d
, x < 1.
sin x =
dx
1 x2
7. One function can not have two different derivatives
at a point while two different functions can have the
same derivative. e.g.,
(i)

1
d
sin x =
dx

1
1 x

and
2

1
d
cos x
dx

Rules of Differentiation

1 x
8. A function or its derived function (or, derivative)
may not exist at all points of an interval. e.g.,

a f 1x does not exist at x = 0 in [1, 1].


f a x f = x has no derivative (or, derived

(i) f x =
(ii)

function) at x = 0 because lim

h 0

0+h
h

does

not exist at x = 0 in [1, 2].


9. When a function or its derived function (or,
derivative) does not exist atleast at one point
belonging to the interval being open or closed, we
say that the function or its derived function does not
exist over (or, in, or on) that interval open or closed.
e.g.,

af

(i) f x =

does not exist in [1, 1] because

x 1
f (x) does not exist at x = 1 1 , 1 .

e j

d
x does not exist in [1, 1] because
dx
0+h 0
h
h
= lim
= lim
lim
(3 | h | =
h 0
h0 h
h0 h
h

(ii)

h for h > 0)

= lim

af

x = 0 1 , 1 or [1, 1].

FG IJ
H K

af

10. The domain of the derived function f x is a


subset (of the domain of f) which contains all elements
x (in the domain of f (x)) at which the

f af

f x+h f x
exists, whether h 0
h 0
h
through positive values or negative values, but does
not contain those exceptional points x where the
derivative (or, derived function) fails to exist or is
undefined. e.g.,
lim

bg

sin x

,0< x 1
2 x
which means domain of the derived function f x
in the set of all 0 < x < 1.

af

bg

bg

2
11. f x = cos ax + bx + c f x

2ax + b sin ax 2 + bx + c

, which means the


2 ax 2 + bx + c
domain of the derived function f x is the set of all
real numbers, for which 0 < ax2 + bx + c < 1.
=

af

Important Facts to Know


In connection with the function, differentiation of the
function or integration of the function at any finite
value of the independent variable, the following key
points must be kept in ones mind.
1. One must consider the restriction that the function
in denominator 0 for any finite value of the
independent variable against the fractional form of
the function, the derivative or integral even if this
restriction is not stated (or, written) explicitly because
one must never divide by zero. e.g.,

a f a x x 2f means f a xf = a x x 2f , x 2
x
f a x f = x f a x f =
means f a x f
x

(i) f x =
(ii)

1
h
= lim
= which
2
h 0
h 0 h h
h0
h
h
means f x = x has no derivative at the point
h

= lim

bg

(i) f x = cos x f x =

, for | x | < 1.

359

x
,x 0
x

za

sec x tan x dx = sec x means sec x tan x dx

= sec x , x 2n + 1
2
2. We say that a function, its derivative or its integral
does not exist (or, we say that the function, its
derivative or its integral is discontinuous) at any finite
value of the independent variable when the function,
its derivative or its integral assumes the form of a
fractional with a zero denominators at any
considered finite value of the independent variable.
e.g.,
(iii)

360

How to Learn Calculus of One Variable

a f 1x , x 0 1x is discontinuous at x = 0.
1
1
, x 0
f a x f = x , x 0 f a x f =
2 x
2 x

(i) f x =
(ii)

is discontinuous at x = 0.

g 2

integral is discontinuous at x = a2n + 1f .


2
(iii)

sec 2 x dx = tan x , x 2n + 1

af

f x = x , for x < 0, is non-differentiable at x = 0.


Solution: For differentiability, we have

af

f + 0 = lim

h 0

f af a

= lim

= lim

h 0

h 0

= lim

h 0

af

f 0 = lim
= lim

h 0

h0

f 0+h f 0
, h>0
h
h 0
h
= lim
h 0 h
h

b3 h

f af a

h 0
h
= lim
h0
h
h

h 0

IJ = ...(ii)
hK

Hence, (i) and (ii) non-differentiability of the

af

af

x , for x 0 , f x
= x , for x < 0, at the point x = 0.
4. When the denominator of a function, derived
function or the integral of a function does not vanish
for any real value of the independent variable, we
state (or, write) that the function, derived function
(or, simply derivative) or integral of the function exists
for all real values of the independent variable. e.g.,

a f

af

(i) f x =

(ii) tan

(iii)

(i)

...(i)

h h

FG
H

= lim

ex

1
2

4x + 7x + 9
x

+1

1
1+ x

, x

, x
1

dx =

x +1

, x

(A) One must remember that power, exponential,


logarithmic, trigonometric and inverse trigonometric
functions are differentiable on any interval on which
they are defined and their derivatives can be found
from the formulas.

f 0h f 0
, h>0
h

= lim

= h for h > 0

Recapitulation

= h for h > 0

1
h
= lim
=
h

0
h h
h

function defined by f x =

b g bg
b g bg

af

h 0

3. By using the relations


f a+h f a
(i) f + a = lim
, (h > 0)
h0
h
f ah f a
(ii) f a = lim
, (h > 0),
h 0
h
One can show the non-differentiability of the
function y = f (x) at a finite value of the independent
variable x = a when its derivative assumes the form of
a fraction with a zero denominator at the same
considered finite value a of the independent variables
x. e.g.,
(i) Show that the function f x = x , for x 0 ,

bg
bg

h 0

the

b3 h

= lim

af

d
c = 0 , c being any constant.
dx

e j

(ii) d x n = nx n 1 , n Q
dx
(iii)
(iv)
(v)

e j

x
d x
a = a log a , a > 0
dx

e j
d
alog xf = 1x , x > 0
dx
x
d x
e =e
dx

Rules of Differentiation

a f

Solved Examples

a f

Differentiate the following w.r.t. x.


1. 1995

(vi) d sin x = cos x


dx

Form 1: y = any constant

(vii) d cos x = sin x


dx

2. 3
7
3. a2, a being a constant.

a f

(viii) d tan x = sec 2 x , x n +


2
dx

a f

(ix) d cot x = cosec 2 x , x n


dx

a f

a f

(x) d sec x = sec x tan x , x n +


2
dx

(xi) d cosec x = cosec x cot x , x n


dx
(B) The following rules of finding the derivatives are
valid for differentiable functions.
(i)

b a fg

b a fg

d
d
af x =a
f x , a being any constant.
dx
dx

b a f a fg

b a fg

b a fg

(ii) d f x g x = d f x + d g x
dx
dx
dx

b a f a fg a f b a fg a f b a fg

(iii) d f x g x = g x d f x f x d g x
dx
dx
dx

FG a f IJ = g a xf dx b f axfg f a xf dx b g axfg ,
H a fK
g a xf
b g axf 0g
d

(iv) d f x
dx g x

Notes:
1. If the question does not say to find the limits,
derivatives and integrals by using their definitions,
one can use their formulas derived by using their
definitions.
2. If a constant appears as a constant multiple of the
differentiable function, the constant can be taken out

af

d
.
dx
3. If a constant appears as an additive quantity in a
differentiable function, it vanishes while
differentiating that function since d.c. of a constant
is zero.

of the symbol

361

Solutions: (i) d 1995 = 0


dx
(ii)
(iii)

FI
HK
d
ea j = 0
dx

d 3
=0
dx 7
2

Form 2: y = a f (x), where f (x) = an algebraic


expression in x and the value of the function sin, cos,
tan, cot, sec, cosec, sin1, cos1, tan1, cot1, sec1,
cosec1, log, e, | |, etc. at any point x.
Differentiate the following w.r.t. x.
3

1. 28 x 4

F
H

I
K

F I
H K

3
3
d
d
28 x 4 = 28
x4
dx
dx
3 1
1
3
21
= 28 x 4
= 21x 4 = 1 ; x > 0
4
x4
2. 3x2

Solution:

Solution:

F I
H K

e j

e j

d
d 2
2
3x = 3
x = 3 2 x = 6x
dx
dx

3. 6 sin x

a f

d
d
6 sin x = 6
sin x = 6 cos x
dx
dx
4. a sin1 x
Solution:

Solution:

d
1
1
d
sin x
a sin x = a
dx
dx
1

=a

1 x

a
1 x

;|x|<1

5. m log x
Solution:

a f

d
d
m
m log x = m
log x = ; x > 0.
dx
dx
x

362

How to Learn Calculus of One Variable


2

6. x
m

F I= 1 d x
GH JK m dx e j

af

Solution:

d x
dx m

af

1
2x
2x =
m
m

Differentiate the following w.r.t. x.


1. (lx2 + mx + c)
Solution:

d
2
lx + mx + c
dx

e j

a f

af

e j

af

2x 2

Solution:

F
H

I
K

3
d
3
5x 2 x 2
dx

af

I
K

c 3 1h
3
x 2
2

3. y =

x +

Solution: y =

af

12

I
JK

e j

j
a f

d
d
d 2
5 sin x +
tan x
x +
dx
dx
dx

= 2x + 5 cos x + sec2 x; x n +
6. y =

a f

x +

a f

d 2
d
x + 5sin x + tan x
y =
dx
dx

x b
+
a x

Solution: y =

= sec x (sec x + tan x); x n +

= 15 x 3 x 2

+ x

d
d
sec x +
tan x
dx
dx
= sec x tan x + sec2 x

3
2

d
d
y =
sec x + tan x
dx
dx
=

e j

= 5 3x 2

1 21
1 3
x x 2 ; for x > 0.
2
2
4. y = sec x + tan x
Solution: y = sec x + tan x

3
d
d
3
5x
2x 2
dx
dx

1
2

5. y = x2 + 5 sin x + tan x
Solution: y = x2 + 5 sin x + tan x

F I
H K
d
d
= 5 ex j 2 F x I
dx
dx H K
=

32

d 2
d
=l
x +m
x +0
dx
dx
= l 2x + m 1 + 0 = 2lx + m

F5x
H

d
d
2
d
c
mx +
=
lx +
dx
dx
dx

2.

x +

21

af

Form 3: y = a f 1 x b f 2 x , where a and b are


constants and f1 (x) and f2 (x) are algebraic expressions
in x or the values of the functions sin, cos, tan, cot,
sec, cosec, sin1, cos1, tan1, cot1, sec1, cosec1,
log, e, | |, etc. at the same point x.

FG
H

IJ
xK
d
d F 1 I
=
xj +
G J
e
dx
dx H x K
1
F 1I F x I
= x +
H 2K H K
2
F3 1 = e x j + F x I
GH x
H K

d
d
y =
dx
dx

af

x b
+
a x

F
H

I
K

FI
HK
d
1 d
=
a
xf + b e x j
dx
a dx

F I
H K

d
d x b
d x
d b
+
=
+
y =
dx
dx a x
dx a
dx x
1

Rules of Differentiation

a f

2
1
1
2
+ b x = b x ; for x 0
a
a

Form 4: y = f1 (x) f2 (x), where f1 (x) and f2 (x) are


algebraic expressions in x or the values of the
functions sin, cos, tan, cot, sec, cosec, sin1, cos1,
tan1, cot1, sec1, cosec1, log, e, | | etc. at the same
point x.
Differentiate the following w.r.t. x:
1. y = (2x + 1) (x 1)2
Solution: y = (2x + 1) (x 1)2 = (2x + 1) (x2 2x + 1)

LMa
N

af

jOQP

fe

d
d
2
y =

2x + 1 x 2x + 1
dx
dx

f dxd ex

= 2x + 1

je

j dxd a2x +1f

2 x +1 + x 2x +1

= (2x +1) (2x 2)+ (x2 2x + 1) 2


= 4x2 2x 2 + 2x2 4x + 2 = 6x2 6x = 6x (x 1)
Note: On multiplying term by term, the product of
two or more than two algebraic polynomial functions
of xs can be always put in the form of the sum of a
finite number of terms, each term having the form xn
and / axn and then it can be differentiated using the
rule of the derivative of the sum of a finite number of
differentiable functions of xs. Hence, applying this
rule to the above given function, one can have its
differential coefficient given below:
y = (2x + 1) (x 1)2 = (2x + 1) (x2 2x + 1) = 2x3
4x2 + 2x + x2 2x + 1 = 2x3 3x2 + 1

af

j
d
d
d
d
=
2x j
3x j +
1g = 2
b
e
e
ex j
dx
dx
dx
dx

d
d
3
2

y =
2 x 3x + 1
dx
dx
3

= (x2 + 1) (3x2 + 0) + (x3 + 2) (2x + 0)


= 3x2 (x2 + 1) + (x3 + 2) 2x
= 3x4 + 3x2 + 2x4 + 4x
= 5x4 + 3x2 + 4x
3. y = (1 x)2 (1 3x2 + 5x3)
Solution: y = (1 x)2 (1 3x2 + 5x3) = (x2 2x + 1)
(5x3 3x2 + 1)

e5x

bg e

jOQP
d
d
x + 2j + e x + 2j
= e x + 1j
e
ex
dx
dx
af

LMex
N
3

je

2x + 1

4. y = x4 log x
Solution: y = x4 log x

af

j
d
d
log x f + log x
a
ex j
dx
dx
F 1 I + log x e4 x j

H xK

d
d 4
y =
x log x
dx
dx
=x

=x

= x3 + 4x3 log x
= x3 (1 + 4 log x); for x > 0.
5. y = sin x log x
Solution: y = sin x log x

af

f
d
alog xf + log x dxd asin xf
= sin x
dx

d
d
y =
sin x log x
dx
dx

= sin x

1
+ log x cos x
x

sin x
+ cos x log x ; for x > 0
x
6. y = cot x cos1 x
Solution: y = cot x cos1 x
=

+1 x +2
3

= (x2 2x + 1) (15x2 6x) + (5x3 3x2 + 1) (2x2 + 0)


= (x2 2x + 1) (15x2 6x ) + 2 (x 1) (5x3 3x2 + 1)
= 3x (1 x)2 (5x 2) 2 (1 x) (5x3 3x2 + 1)
= (1 x) {3x (1 x) (5x 2) 2 (5x3 3x2 + 1)}

e j

d
d
y =
dx
dx

j dxd e x

d 2
x +0
dx
= 2 3x2 3 2x = 6x2 6x = 6x (x 1)

j e

d
d
5x 3 3x 2 + 1 +
y = x2 2x + 1
dx
dx

3x + 1

2. y = (x2 + 1) (x3 +2)


Solution: y = (x2 + 1) (x3 +2)

+1

363

af

d
1
d
y =
cot x cos x
dx
dx

364

How to Learn Calculus of One Variable

a f

= cos

d
d
1
cot x + cot x
cos x
dx
dx

= cos x

x cosec x + cot x
2

= cosec x cos

cot x

1 x

7. y = x3 tan x
Solution: y = x3 tan x

bg

F
GG
H

1
1 x

= sin x

I
JJ
K

; for 0 < | x | < 1

d
d 3
d
tan x +
y =
x tan x = x 3
dx
dx
dx

af

j
d
asin xf + sin x dxd ee j
dx
x

af

j
e j

F
H

af

I
K

d
d
y = sin x
log x
dx
dx

a f

a f

13. y = sin2 x
Solution: y = sin2 x = sin x sin x

9. y = ex log | x |
Solution: y = ex log | x |

d
d x
y =
e log x
dx
dx
x d
d x
e +e
= log x
log x
dx
dx
x
x 1
= log x e + e
x
x
1
= e log x +
; x0
x
10. y = sin x log | x |
Solution: y = sin x log | x |

a f

d
d
cos x + cos x
sin x
dx
dx
= sin x (sin x) + cos x cos x
= sin2 x + cos2 x
= cos2 x sin2 x = cos 2x

= ex cos x + sin x ex
= ex (sin x + cos x)

= sin x

a f

d
d
y =
sin x cos x
dx
dx

d x
d
e sin x

y =
dx
dx
=e

12. y = sin x cos x


Solution: y = sin x cos x

8. y = ex sin x
Solution: y = ex sin x

af

d
d
cot x + cot x
cos x
dx
dx
= cos x ( cosec2 x) + cot x (sin x)
= cosec2 x cos x cot x sin x; x n

e j

= x2 (3 tan x + x sec2 x); x n +

af

d
d
y =
cos x cot x
dx
dx
= cos x

d 3
x
dx
= x3 sec2 x + tan x (3x2) = x3 sec2 x + 3x2 tan x

tan x

sin x
+ cos x log x ; x 0
x

11. y = cos x cot x


Solution: y = cos x cot x

b g

1
+ log x cos x
x

g + log x dxd asin xf

af

a f

a f

d
d
d
y = sin x
sin x + sin x
sin x
dx
dx
dx
= sin x (cos x) + sin x (cos x)
= sin x cos x + sin x cos x = 2 sin x cos x

14. y = sec x tan x


Solution: y = sec x tan x

af

d
d
y =
sec x tan x
dx
dx

a f

f
a f

d
d
tan x + tan x
sec x
dx
dx
= sec x (sec2 x) + tan x (sec x tan x)
= sec3 x + tan2 x sec x
= sec x (sec2 x + tan2 x)
= sec x (1 + tan2 x + tan2 x)
= sec x

Rules of Differentiation

= sec x (1 + 2 tan2 x); x n +

e2 x 5j cot x + 4 x cot x log x 2 x 5 log x


=
e j
x

15. y = xn cot x
Solution: y = xn cot x

af

d
d n
y =
x cot x
dx
dx

cosec2 x; for x n
2. y = ex log | x | sec x
Solution: y = ex log | x | sec x

a f

Form 5: y = f1 (x) f2 (x) f3 (x), where f1 (x), f2 (x) and


f3 (x) are algebraic expressions in x or the values of
the functions sin, cos, tan, cot, sec, cosec, sin1,
cos1, tan1, cot1, sec1, cosec1, log, e, | | etc. at the
same point x.

d
d
(1 2 3) = (2 3)
(1)
dx
dx
d
d
+ (1 3)
(2) + (1 2)
(3) which means one
dx
dx
should differentiate separately each function of x
marked as 1, 2, 3 (standing for the first, second and
third function considered at our liberty) and multiply
each differentiated function of x by two remaining
functions of xs undifferentiated and lastly add each
product to get the differential coefficient of the product
of three differentiable functions of xs.
Refresh your memory:

Find the differential coefficient if


1. y = (2x2 5) cot x log | x |
Solution: y = (2x2 5) cot x log | x |

bg e

d
d

y = 2 x 2 5 cot x
log x
dx
dx
d
2
2
cot x log x
2x 5 + 2x 5
dx
d
cot x
log | x |
dx

e
a f

e j

d
d
x
cot x + cot x
dx
dx
= xn (cosec2 x) + cot x ( n xn 1)
= n xn 1 cot x xn cosec2 x = xn 1 (n cot x
x cosec2 x); x n
=x

j e

365

h+

F 1I
= e2 x 5j cot x G J + cot x log x b4 x g +
H xK
e2 x 5j log x ecosec x j

bg

e j

d
d x
d
y = log x sec x
e + sec x e x
log x
dx
dx
dx

log x e

a f

d
sec x
dx

= log x sec x e + sec x e

1
+ log x
x

e sec x tan x

F
H

= e sec x log x +

I
K

1
+ log x tan x ;
x

for

2
3. y = 2x3 sin x log x
Solution: y = 2x3 sin x log x

x 0 , n +

af

e j
d
sin x
alog xf
dx

d
d
3
3
y = sin x log x
2 x + 2 x log x
dx
dx

a f

d
3
sin x + 2 x
dx

e j + 2x

1
x
2
3
2
= 6x sin x log x + 2x log x cos x + 2x sin x; for
x>0
= sin x log x 6 x

4. y =

log x cos x + 2 x sin x

x e tan x
x

Solution: y =

x e tan x

af

a f

x d
x
d
d
y = x e
tan x + e tan x
dx
dx
dx

x tan x

e xj+

e j

d x
e
dx

h+

= x e sec x + e tan x

1
2 x

+ x tan x e

366

How to Learn Calculus of One Variable

=e

F
GH

x sec x +

tan x
2 x

I
JK

x tan x ;

for

x > 0 and n +
2

af
af

f1 x
, where f (x) and f (x) are
Form 6: y =
1
2
f2 x
algebraic expressions in x or the values of the
functions sin, cos, tan, cot, sec, cosec, sin1, cos1,
tan1, cot1, sec1, cosec1, log, e, | | etc. at the same
point x.

FG a f IJ
H a fK

ex

eb g j b g b g

(ii)

c b gh = df c g b xgh d g b xg (proved later)


dx
dg b x g
dx

Differentiate the following w.r.t. x:


5
bx + c
5
2
= bx + c x + a
1. y = 2
x +a

je j
d
d F

y g = ebx + cj
b
e x + aj IK +
dx
dx H
e x + aj dxd ebx + cj
5

d 2
x +a +
dx

j e5b x j
1

j e

j b2 xg + e x
2

+a

e
j e x + aj
2 x ebx + cj
5b x
=

e x + a j e x + aj
5 b x e x + a j 2 x ebx + cj e x + a j
=
ex + aj
= 5b x

ex

+a

5b x 6 + 5ab x 4 2b x 6 2c x

ex

2 x bx + c

+a

3b x + 5ab x 2c x

ex

+a

Or, alternatively,
Directly using the rule of the derivative of the quotient
of two differentiable functions of xs , the d.c. of the
given function of x w.r.t. x is

cb gh

df g x

+a

e
e5 b x j

ea f a f j

d
d
1
2
= 1 2
2
2 which
dx
dx
has been explained in the chapter chain rule for the
derivative.

Note: (i)

= bx 5 + c x 2 + a

d 1
d

1 2 1
=
Refresh your memory:
dx 2
dx

2 1 1 2
=
which means one should

2
express the function in denominator in the form of a
power function with negative index before
differentiating the quotient of two differentiable
functions xs by using the rule of the derivative of the
product of two differentiable functions of xs noting
that (1) and (2) represent functions of xs in Nr and Dr
respectively.

a f af af a f
af

j b g e

= bx 5 + c 1 x 2 + a

af
j e

d
y =
dx
2
5
5
d
d 2
x +a
bx + c bx + c
x +a
dx
dx

j e

j e

e x + aj
e x + a j e5b x + 0j eb x
=
ex + aj
2

+ c 2x

Rules of Differentiation

ex

5b x + 5ab x 2b x 2c x
+a

4. y =

3b x + 5ab x 2c x

ex

+a

af

y =
dx

x
log x

x
Solution: y =
log x

e j
a f

af

n 1

log x x

alog xf

=x

n 1

a f

1
x

af

y =
dx

; for x > 0 and 1

af

y =
dx

sin x

a f
ex j

a f

e j

x cos x sin x 3x

x cos x 3 sin x
x

1 x

sin

x cos x

sin x

e j

d
d 3
sin x sin x
x
dx
dx
3 2

a f

1
1
d
d
sin x sin x
sin x
dx
dx
2
sin x

sin x

a f

; for x 0 , and 1.

sin x
sin x

x
=

sin x
Solution: y =
sin x

1
x

sin x
x

x cos x log x sin x


x log

n log x 1
2

Solution: y =

log x cos x sin x

sin x
x

5. y =

log x
3. y =

a f
b g

d
d
sin x sin x
log x
dx
dx
2
log x

log x

log

n d
d n
x x
log x
log x
d
dx
dx

y =
2
dx
log x

nx

sin x
log x

2. y =

sin x
log x

Solution: y =

367

sin x sin 1 x cos x 1 x 2


1 x 2 sin 2 x

and x 0
6. y =

1+ x

; for x 0 .
Solution: y =

1+ x

; for | x | < 1

368

How to Learn Calculus of One Variable

e1 + x j dxd ee j e dxd e1 + x j
af
e1 + x j
x

y =
dx

2 2

e1 + x j e e a2 xf
e1 + x j
x

2 2

a1 xf
e1 + x j

dz
dz
and 1 in (4)
dx
dx

1. y =

e + log x
sin x 5x

2 2
x

af

af

or, z1 = g1 x g 2 x = sum of two algebraic or


transcendental function of xs.
or, z1 = g1 (x) g2 (x) = product of two algebraic or
transcendental functions of xs.
Working rule: One can find the d.c. of the function
of x put in the form (7) using the rule which consists
of following steps.
Step 1: Put z = f 1 x f 1 x / f 1 x f 2 x and

af af af af
= g a xf g a xf / g a xf g a x f
1

sin x 5x

Putting z = ex + log x, we have

af

af

e + log x

Solution: y =

z = f1 (x) = any single algebraic or transcendental


function of x
or, z = f 1 x f 1 x = sum of two algebraic or
transcendental functions of xs.
or, z = f1 (x) f2 (x) = product of two algebraic or
transcendental functions of xs.
similarly, z1 = g1 (x) = any single algebraic or
transcendental function of x.

z1

a f

Differentiate the following w.r.t. x:

z
, where
z1

Form 7: y =

dz
dz
z 1
dx
dx
2
z1

z1

Note: One should do the problems directly without


making substitutions z and z1 for the functions of xs
(in Nr and Dr) put in the form of the sum, difference or
product function after practising the above working
rule.

e1 + x 2 xj
e1 + x j

d z
Step 3: Use the formula:
=
dx z1

Step 4: Put

2 2

FG IJ
H K

dz
dz
Step 2: Find
and 1 using the rules for the
dx
dx
derivative of the sum, difference or product of two
differentiable functions of xs.

af

a f

e j

d
d x
d x
d
log x
e +
z =
e + log x =
dx
dx
dx
dx
1
; for x > 0
x
Again, putting z1 = sin x 5x3, we have
x

=e +

af

a f

(1)

e j

d 3
d
d
3
d
sin x 5x =
sin x 5
x
z1 =
dx
dx
dx
dx
= cos x 15x

(2)

FG IJ
H K

d z
Now using the formula
=
dx z1

z1

dz
dz
z 1
dx
dx ,
2
z1

a f

we have

FG IJ
H K
esin x 5x j FGH ex + 1x IJK ecos x 15x j ee
esin x 5x j
af

d
d z
=
y =
dx
dx z1
3

3 2

x > 0 and 5x 3 sin x

+ log x

, for

Rules of Differentiation

FG3 y = z IJ
H zK

2. y =

x log x

F
I
GH
JK
a1 + log xf dxd e x e j x e
=
a1 + log xf
af

d
d
xe
y =
dx
dx 1 + log x
x

e + sec x

369

d
1 + log x
dx

x log x

Solution: y =

e + sec x

a1 + log xf a x + 1f e x e 1x
=
a1 + log xf
e a1 + log x f a1 + x f 1
, for x > 0.
=
a1 + log xf
x

Putting z = x log x, we have

af

d
d
x log x
z =
dx
dx

a f

af

d
d
1
log x + log x
x = x + log x 1
dx
dx
x

=x

= 1 + log x
Again, putting z1 = ex + sec x, we have

a f

(1)

a f

e j

d
d x
d x
d
sec x
e +
e + sec x =
z1 =
dx
dx
dx
dx
x

= e + sec x tan x

FG IJ
H K

z1

a f

d
dz
z1
z
dx
dx
,
2
z1

a f

je

ee

+ sec x

3. y =

F
GH

I
JK
x

a1 + log x f e1+ e j e x + e j 1x
=
a1 + log xf
a1+ log xf e1+ e j FG1 + ex IJ
H K
=
a1+ log xf
x

xe
1 + log x
x

Solution: y =

af

d
d
x+e
y =
dx
dx 1 + log x

+ sec x e + sec x tan x x log x

for x > 0 and n +

d
d z
y =
dx
dx z1

a1 + log xf ee

x+e
Solution: y =
1 + log x

a1 + log xf dxd e x + e j e x + e j dxd a1+ log xf


=
a1 + log x f

FG IJ
H K

af

x+e
4. y =
1 + log x
x

...(2)

d z
Now, using the formula
=
dx z1

we have

xe
1 + log x

1/ + log x + e x + e x log x 1/

b1 + log xg

ex
x

370

How to Learn Calculus of One Variable

g
x b1 + log x g

e x x + x log x 1 + x log x

5. y =

; for x > 0.

1 cos x
1 + cos x

FG
H

af

d
d 1 cos x
y =
dx
dx 1 + cos x

af

d
d 1 sin x
y =
dx
dx 1 + cos x

x
1 + cos x

a1 + cos xf
asin x cos xf ecos x + sin xj
=
a1 + cos xf
2

sin x cos x 1

b1 + cos xg

8. y =

af

x
1 + cos x

F
GH

d
d
x
y =
dx
dx 1 + cos x

I
JK

a1 + cos xf dxd e x j x dxd a1 + cos xf


=
a1 + cos xf
x b2 + 2 cos x + x sin x g
=
; x b2n + 1g .
b1 + cos xg
2

Solution: y =

cos x cos x + sin x sin x

6. y =

1 sin x
1 + cos x

FG
IJ
H
K
a1+ cos xf dxd a1 sin xf a1 sin xf dxd a1+ cos xf
=
a1 + cos xf
b1 + cos xg b cos xg b1 sin xg b sin xg
=
b1 + cos xg

IJ
K
a1+ cos xf dxd a1 cos xf a1 cos xf dxd a1+ cos xf
=
a1+ cos xf
a1 + cos xf sin x + sin x a1 cos xf
=
a1 cos xf
sin x a1 + cos x + 1 cos x f
=
a1 + cos x f
2 sin x
=
; x 2n + 1
b1 + cos xg b g .

1 sin x
1 + cos x

Solution: y =

1 cos x
1 + cos x

Solution: y =

7. y =

; x 2n + 1

x cos x
1+ x

Solution: y =

af

x cos x
1+ x

F
GH

d
d x cos x
y =
dx
dx 1 + x 2

I
JK

d
d
1+ x j
x cos x f a x cos x f
a
e
e1 + x j
dx
dx
=
e1 + x j
2

2 2

e1+ x j FH x dxd acos xf + cos x dxd axfIK a x cos xf a2 xf


=
e1 + x j
2

2 2

Rules of Differentiation

e1 + x j a x sin x + cos xf 2 x
=
e1 + x j
2

Differentiate the following w.r.t. x.


cos x

e1 + x j

2 2

e1 x j cos x x e1 + x j sin x
=
e1 + x j
1
Form 8: y = , a z 0f where z = a single function
z
2

of x, sum, difference or product of two or more than


two differentiable functions of x 0 for any finite
value of the independent variable x.
1
Working rule: To find the d.c. of y = , z 0 one
z
can adopt the rule which consists of following steps,
provided dz f 1 x = a single function of x.
Step 1: Put z = f 1 x f 2 x , or f 1 (x) f2 (x)
whichever is given.

x sec x
Putting z =

sec x, we have

af

d
d 4
z =
x sec x
dx
dx

e j

= x sec x tan x + 4 x sec x

FG IJ
HK

bg

af

d
d
d 1
z
y =
=
dx
dx
dx z
2
z

x sec x tan x + 4 x sec x

ex

af

sec x

a x sec x tan x + 4 sec xf


8

x sec x

af

d
z
d
d 1
dx
Step 3: Use
y =
= 2
dx
dx z
z
Notes:
1. When z = f1 (x) = a single function of x, one should
use directly the formula:

FI
HK

a f

d 4
d
sec x + sec x
x
dx
dx

=x

af
FI
HK

FG IJ = f a xf which means the


H a f K f a xf

d
1
d 1
=
dx
f
dx z
1 x

x4

d
z
Step 2: Find
dx

af

Solution: y =

2 2

af
af

x sec x

x sin x + cos x x sin x + x cos x 2 x cos x

1. y =

2 2

371

1
2
1

d.c. of a reciprocal of a function of x is negative of the


d.c. of the function of x in numerator divided by the
square of the function in denominator.
2. One should do the problems without making a
substitution z for the given sum, difference and
product functions of x in denominator after practising
the above given working rule.

sec x x tan x + 4
5

x sec x
=

x tan x + 4
5

x sec x

x sin x + 4 cos x
x

g , b x 0g

and

2
or, alternatively, it can be done in the following way:
x n +

y=

1
4

x sec x

cos x
x

372

How to Learn Calculus of One Variable

FG
H

af

IJ
K

d
d cos x
y =
dx
dx x 4
x

a f
ex j

x sin x 4 x cos x

Solution: y =

a x sin x + 4 cos xf
x

af

d
d
(x a) + (x a) (x c)
z = (x b) (x c)
dx
dx

d
d
(x b) + (x b) (x a)
(x c)
dx
dx
= (x b) (x c) + (x a) (x c) + (x b) ) (x a)

af
FI
HK
a x bfa x cf + a x a fa x cf + a x bfa x af ;
=
a x a fa x bfa x cf

d
z
d
d 1
y =

= dx 2
dx
dx z
z

1
sin x cos x

FG
H

af

1
d
d
y =
dx
dx sin x cos x

d
sin x cos x
dx
=
2
sin x cos x

af

IJ
K

x a , b , c.

a
f
F sin x d acos xf + cos x d asin xfI
H dx
K
dx
=
asin x cos xf

4. y =

j = ecos

x sin

sin x cos x

F cos x sin x I
GH sin cos x sin x cos x JK
2

sin x cos x

1
cos x

Solution: y =

sin x + cos x

a x af a x bfa x cf

Putting z = (x a) (x b) (x c), we have

1
sin x cos x

Solution: y =

a x af a x bfa x cf

x sin x + 4 cos x

.
2

3. y =

4 2

= (cosec2 x sec2 x) = sec2 x cosec2 x;

d
d 4
cos x cos x
x
dx
dx

2. y =

e j

F 1 1 I
GH sin x cos x JK

af

1
cos x

FG
H

1
d
d
y =
dx
dx cos x

IJ
K

a f
a f
asin xf
=
d
cos x
dx
=
2
cos x

cos x

Rules of Differentiation

sin x
2

cos x

sin x
1

cos x cos x

= tan x sec x = sec x tan x, x n +

.
2

= sin x

Form 9: y = sum of a finite number of terms such that


each term is the product and/quotient function, i.e.

af af af af af
g a xf
f a xf
g a x f or y = f a x f g a x f g a x f /
g a xf
f a xf
g axf

FG
H

af

cos x
d
d
2
y =
x sin x +
dx
dx
x

a f FGH IJK
x a sin x f cos x
cos x +

e j

= 2 x sin x + x

3. y =

x +

cot x
x

Working rule: To find the d.c. of the sum of a finite


number of terms such that each term is the product
and/quotient function, one may adopt the rule
consisting of following steps:
Step 1: Use the rule for the derivative of the sum of a
finite number of differentiable functions of xs
regarding each product and/quotient function as a
single function of x.
Step 2: Find the d.c. of the product and quotient
function.
Differentiate the following w.r.t. x:
1. y = x sin x + sin x cos 3x
Solution: y = x sin x + sin x cos 3x

af

d
d
y =
x sin x + sin x cos3x
dx
dx

f
b g

d
d
sin x cos 3x
x sin x +
dx
dx
d
d
d
=x
sin x + sin x
x + sin x
cos 3x +
dx
dx
dx
d
cos 3x
sin x
dx
= x cos x + sin x 3 sin x sin 3x + cos 3x cos x.
cos x
2
2. y = x sin x +
x
=

Solution: y = x sin x +

bg
a f

cos x
x

Solution: y =

x +

cot x
x

FG x + cot x IJ
H
x K
d
=
e x j + dxd FGH cotx x IJK
dx

sin x cos x
2 ; x 0.
x
x

= 2 x sin x + x cos x

IJ
K

d 2
d
d cos x
2
x +x
sin x +
dx
dx
dx
x

y = f 1 x f 2 x g1 x g 2 x / f 1 x
1

af

d
d
y =
dx
dx

1
2 x
1
2 x

x cosec x cot x
x

e x cosec

x + cot x
2

j ; x n .

4. y = sin x + x cos x
Solution: y = sin x + x cos x

373

af

d
d
y =
sin x + x cos x
dx
dx

a f

d
d
sin x +
x cos x
dx
dx

a f
= cos x + x a sin x f + cos x
= cos x + x

f
af

d
d
cos x + cos x
x
dx
dx

= cos x x sin x + cos x


5. y = 2x sec x 2 tan x sec x
Solution: y = 2x sec x 2 tan x sec x

374

How to Learn Calculus of One Variable

af

d
d
y =
2 x sec x 2 tan x sec x
dx
dx

= 2 sec x

bg

b g

5. y =

b g

6. y =

7. y = 6

d
d
d
x + 2x
sec x 2 sec x
tan x
dx
dx
dx

a f

d
tan x
sec x
dx
= 2 sec x + 2x sec x tan x 2 sec x sec2 x tan x
sec x tan x
= sec x (2 + 2x tan x 2 sec2 x tan2 x)
Form 1: Problems on constant functions, i.e., y = c, c
being a constant.
Exercise 8.1
Differentiate w.r.t. x if
1. y = e
2. y =
3. y = log e
4. y = 1993
Note: Since -method is not mentioned by which
one has to do the problems which means one is free
to do them by any other method also (i.e. one can use
the method of applying directly the formulas obtained
from -method.
Answers:
1. 0, 2. 0, 3. 0, 4. 0.
Form 2: Problems on power function and/a constant
multiple of power function, i.e., y = xn and y = cxn, c
being a constant.

8 a 2 b2

8. y =

7 3 27 x

F 7x I
y=G
H 5x JK
3

9.

1
2

10. y =

x4
7

Answers:
1. 9x8
2.
3.

72
x 10
15

2 x

4.
5.

; x 0.
; x > 0.

14 52
x ; x 0.
3

3
x ; x 0.
2

6.

3 52
x ; x > 0.
2

Exercise 8.2

7.

9
12
x 7 ; x 0.
7

Differentiate the following w.r.t. x


1. y = x9

8.

4
8 a 2 b 2
x 3 ; x 0.
63

2. y =

8
x

9. 35 x x 7 , x 0 .

9
1

2
3. y = 15 x

4. y = 7 x

23

10.

4
3

x
.
7

Form 3: Problems on a constant multiple of


transcendental functions of xs, i.e., y = c any one of
the functions of xs say sin x, cos x, tan x, cot x, sec x,

Rules of Differentiation

cosec x, sin1 x, cos1 x, tan1 x, cot1 x, sec1 x,


cosec1 x, log x and ex, c being a constant.

3.

5x

Exercise 8.3
Find the differential coefficients of the following
functions w.r.t. x:
1. 7 ex

3.
4.

alog xf

7.

9.

FG 1 + x IJ
H x K

2.

3.

2 cos x

4.

sec x tan x , x n + , n Z
2
2

x 7

;x>0
7

17 x 168 x 33 + 14 x

4. 45 5x

af af

differentiable functions of xs, i.e., y = f 1 x f 2 x +

af

2x 2

5x

I
JK

af

2 x

3.

Form 4: Problems on the sum of a finite number of

af

... f n x and/ a1 f 1 x a2 f 2 x ... an fn (x),


a1, a2, , an being constants.

2 1

76 8 x

38

x +

1
x

39
4

;x 0

;x 0

1 34
x + 5 sin x ; x n + and 0
2
4
8. 2x + a ; x a
7.

9.

x 1
x

Differentiate the following functions w.r.t. x:


1. ax2 + bx + c

2x
2
5. 12x + 3 cos x 5 sin x 2ex
6. 2 cos x 5 sin x

Exercise 8.4

2.

13

Answers:
1. 2ax + b

8
; x > 0, 1
x

F
GH

x a
xa

10. 6log x

1
1
4 1 + x2

4 8

5
+3
sec x

8.

3
sec x
2
1

5.

16 x

2 sin x

tan x
5.
4
Answers:
1. 7 ex

2.

31

+ 87
3
2x
5. 4x3 + 3 sin x + 5 cos x 2ex
6. 2 sin x 5 cos x
4. 15x

2.

17 x 42 x + 14 x + 11

10.

;x > 0

6
1

;x > 0
x 2 x

375

376

How to Learn Calculus of One Variable

Form 5: Problems on the product of two differentiable


functions of xs, i.e., y = f1 (x) f2 (x).
Exercise 8.5
Differentiate the following w.r.t. x:
1. (x + 4) (5x 6)
2.

F x + 1 I FG
H xK H

x +

IJ
xK

sin x
+ cos x log x ; x > 0
x
9. 2 cos x 2x sin x
10. 3ex + (3x + 1) ex
8.

11.

cos x
sin x log x ; x > 0
x

12. 3x2 tan x + x3 sec2 x ; x n +

, n Z
2

, n Z
2

e log x
7
x
4. e log x

13. sin x sec2 x + tan x cos x ; x n +

5. 9 x log x
6. 4x5 ex

Form 6, 7 and 8: Problems on quotient function such


that the function in the numerator and denominator is

3.

14. Find
15. Find

either the sum or the product function, i.e., y =

e
9
8. sin x log x
9. 2x cos x
10. (3x + 1) ex
11. cos x log x
12. x3 tan x
13. sin x tan x
14. ex tan1 x
15. ex sin1 x
7. 11 sin x

af af af af af
unity and z = g a x f / g a x f g a x f / g a x f g a x f
where z = f 1 x / f 1 x f 2 x / f 1 x f 2 x /
1

2.

2x
x

3.

Find

j; x > 0

FG
H

IJ
K

ex
+ (log x ) e x ; x > 0
4.
x
5.

9
2 x

log x ; x > 0

6.

(4x5 + 20x4) ex

7.

11
(sin x + cos x) ex
9

af

d
y if
dx

1. y =

+ 4x + 3

1
e
log x +
;x>0
7
x

Exercise 8.6

Answers:
1. 10x + 14

b x 1g e3x

z
,
z1

2. y =

ax + b
cx + d
x3 + 3
x3 5
2

3. y =

4. y =
5. y =

x + 9 x + 10
2

x 7 x + 12

a +

log x
sin x

; a > 0.

Rules of Differentiation

6. y =

cos x
log x
n

x
7. y =
log x

8. y =

cos x
x

9. y =

x
tan x

23. y =

1 tan x
1 + tan x
2

10. y =

3x + 4
sin x + cos x

11. y =

5 + tan x
8x + 9

12. y =

tan x + cot x
log x

13. y =

e
1+ x

14. y =

tan x

x+e
x
e + tan x

x
sin x

19. y =

cot x
x
1

log x

29. y =

log x
x

e
log x
28. y =
cos x

Answers:

2.

18. y =

27. y =

1.

cot x x
1
16. y =
sin x
1
17. y =
log x

20. y =

1 cos x
1 + cos x

x + sec x
24. y =
1 + tan x
x cos x
25. y = 2
x +4
sin x + cos x
26. y =
sin x cos x

15. y =

22. y =

3.

ad bc

; x3 5

24 x 2

ex

16 x 2 + 4 x + 178

ex

4.

7 x + 12

a
a

; x 3, 4

; x > 0, a

1
sin x cos x log x
; x n , x > 0
5. x
sin 2 x
sin x log x
6.

1
cos x
x
;x>0

log x

e tan x
sec x + tan x
21. y =
sec x tan x

;x

bcx + d g
3

d
c

7.

nx

n 1

log x x
2

log x

n 1

;x>0

377

378

8.

9.

How to Learn Calculus of One Variable

x sin x + cos x
x

;x 0

2 x tan x x 2 sec 2 x
2

tan x

e x tan x sec 2 x e x

n
, n Z
2

;x

f a

fe

j,

b8x + 9g sec x 8 tan x 40 ; x 9


8
b8x + 9g
1
sec x cosec x j log x atan x + cot x f
e
x
12.
;

27.

6 x sin x + cos x cos x sin x 3x + 4

asin x + cos xf

x n

.
4
2

11.

Find
Find
Find
Find
Find
Find

13.

x ex

x cos x

14.

15.

x
16.
17.

ee

je

j + ecosec x + n x jee
ecot x x j
2

n 2

n
and cot x x n
2
cos x
2

sin x
1

x log 2 x

sin 2 x

+ tan x

af af af af af
g a xf
f axf
g a x f or, y = f a x f g a x f g a x f /
g a xf f a xf
g axf
/

f a xf
g a xf f a xf g a x f

j;

Exercise 8.6

2. x sin x +

;x>0

2 x sin x x cos x

n 1

;x>0

Differentiate the following functions of xs w.r.t. x:


1. x sin x + sin x cos x

; x n

18.

+ sec x cot x x

; x > 0 , n +

y = f 1 x f 2 x g1 x g2 x / f 1 x

x2

n
2

;x>0

1 log x

;x

Form 9: Problems on the sum of a finite number of


terms such that term is the product and/quotient
function, i.e.,

; x 1

b1 + xg
sec x e x + e j e1 + e j tan x

; x n + , e
2
ex + e j

cos x + x sin x log x

28.
29.

n
2

x > 0, x

tan x

x ex

log x

1 x log x

ee

20.
21.
22.
23.
24.
25.
26.

10.

x cosec 2 x cot x
; x n
x2

19.

3.

; x n
4.

c+

cos x
x

x + x +1 +

x 1 2x + 1
+
2x 1
x 1

Rules of Differentiation

x 1
2
= x + x +1
Hint:
x1

2x + 1
2
=1+
2x 1
2x 1
sin x
5. 1 + cos x
6.

1+ x

fa

1 tan x
21. 1 + tan x
2

23.

x e +5

26.

x sin x
12. sin x + cos x

14.

29.

1+ x
1

1+ x

sin x + cos x
x

1 + sin x
30. 1 sin x

x
2

x + cosec x
1 + cot x

x sin x
27.
1 + tan x
cot x
28. 2
x + sin x

x tan x
11. sec x + tan x

tan x
25. x + sin x

e sec x tan x
9.
1 + tan x
x sin x
10. cos x sin x

13.

x + sec x
1 + tan x
x cos x

x +4
sin x + cos x
24. sin x cos x

x
7. 2 + cos x
8.

1 cos x
20. 1 + cos x

22.

x x2
x+1 x +4

sin x

sec x + tan x
19. sec x tan x

+ e sec x

1 + sin x
31. 1 + cos x

15. x sec x +

x
1 + sin x

32.

x log x
16.

e tan x
x

17.

e + sin x
1 + log x
x

e + log x

18.

e log x

33.

log x e

x
x log x
x

e tan x
x

34. x tan x +

e
1 + cos x

Answers:
1. sin x + x cos x + cos2 x sin2 x

379

380

How to Learn Calculus of One Variable

sin x cos x
2 ;x0
x
x

2. 2 x sin x + x 2 cos x

16.

LMtan x a1 + log xf x log x etan x + sec


N
e

3. Find
4.

a2 x + 1f + a2 x + 1f a2 x 4 1f

1
2

; x 1,

1
5.
; x 2n + 1
1 + cos x

x n +

17.

6.

7.

a x + 1f a x + 4f
2

18.

8.

2 + cos x + x sin x

a2 + cos xf
e1 + x j cos x 2 x sin x e b2 x + 1g
2 x
e1 + x j
2

2e sec x tan x sec x


9.
, x n
2
4
1 + tan x

11.

12.

x + sin x cos x sin x

f , x n +

22.

2 2

2 2

2 sec x

, x n

a1 + tan xf
a1+ tan xfa2 x + sec x tan xf e x
a1+ tan xf
2

and x n +
4
2

23.

24.

; x n +
1 sin 2 x
4

+ 4 sin x

a x + sin xf

; x0

a2 x cosec x cot xfa1 + cot xf cosec x e x


a1+ cot xf
2

26.

x sec x + tan x sec x 1 sin x


25.

+ sec x sec x

x n +
2

e4 x j cos x x e x
e x + 4j

15.

x n

14.

a1 + log xf x ee + cos xj e sin x


;x>0
x a1 + log x f
F 1 log xI
2e
Hx
K ;x>0
ee log xj
2 sec x asec x + tan x f

asec x tan x f ; x n + 2
2 sin x
a1 + cos xf ; x b2n + 1g
2

21.

2
acos x sin xf

x cos x + sin x a1 + sin x f


, x n +
2
a1 + sin xf
x + sin x asin x + cos x f

asin x + cos xf ; x n 4
e a1 x f
e1 + x j
2 x

+ e sec x a1 + tan x f , x n +
2
e1 + x j
2 x a1+ sin xf x cos x
x sec x tan x + 2 x sec x +
,
a1+ sin xf
x

13.

20.

10.

19.

and x > 0.
2

jOPQ
,

; x 1, 4

tan x

7 x + 8x 8

2x

x n and n

+ cosec x

j;

381

Rules of Differentiation

x
2 x sin x
+
27.
1 + tan x

ecos x sin x tan xj


;
a1 + tan xf
2

31.

a1 + cos xf ; x b2n + 1g
e a x 2 f + 1 2 log x ; x > 0

1 + sin x + cos x
2

x n + and n
2
4

32.

cosec x x cosec + 1 cot x 2 x + cos x

28.

x + sin x

x n + and x 2 sin x
2
29.

30.

a2 x 1f cos x a2 x + 1f sin x , x > 0


2x

2 cos x

a1 sin xf

x
x
2
e tan x 1 + log x e x log x sec x + tan x

33.
;

tan x

x > 0 , x n and x n +

34. x

e x sec

; x 2n +

f
ee

x n +

x + 3 tan x +

j;

2
x

e 1 + sin x + cos x

and x 2n + 1
2

a1+ cos xf

f;

382

How to Learn Calculus of One Variable

9
Chain Rule for the Derivative

Firstly, we recall the basic ideas of composition of


differentiable functions.
1. Composition of two differentiable functions:
If y = f1 (z1) is a differentiable function of z1 and z1
= f2 (x) is a differentiable function of x, then y = f1 f2 (x)
is a differentiable function of a differentiable function
of x or composition (or, composite) of two
differentiable functions f1 and f2 whose composition
in the arrow diagram can be expressed as
f

af

af

2 f
1

x
2 x f 1 f 2 x
In practice, a differentiable function of a
differentiable function of x is obtained by replacing
the independent variable x in the differentiable
function by another differentiable function of x.
Explanation
(i) y = ex is a differentiable function of x replacing x by
another differentiable function log x (x > 0), we have
(ii) y = e log x which is a composition of two
differentiable functions e and log.

2. Composition of a finite number of differentiable


functions:
If y = f1 (z1) is a differentiable function of z1;
z1 = f2 (z2) is a differentiable function of z2;
z2 = f3 (z3) is a differentiable function of z3;
z3 = f4 (z4) is a differentiable function of z4;

zn 2 = fn 1 (zn 1) is a differentiable function of


zn 1;
zn 1 = fn (zn) is a differentiable function of zn;

zn = f (x) is a differentiable function of x;


then y = f1 f2 f3 for f (x) is a differentiable function
of a differentiable function of a differentiable function
of a differentiable function of x (or, composite/
composition of (n + 1) number of differentiable
functions f1, f2, fn and f) whose composition in the
arrow diagram is expressed as
x

af

f x

f1

f1

fn

af

fn f x

af

f n 1

af

f n 1 f x ...

f 2 ... f n f x . But if zn = x = identity function


being differentiable, we get the composition of nnumber of differentiable functions f1, f2, f3, , fn
represented by the notation:
y = f1 f2 f3 fn (x) whose arrow diagram of the

composition

af

fn x

is

f1
...

fn

af

fn x

af

fn 1

f n 1

f 1 f 2 ... f n x .

In practice, if the process of replacing the


independent variable x in the preceeding differentiable
function by another differentiable function of x is
continued upto (n + 1) or n- number of times, we get
what is called the composition of (n + 1)or n-number
of differentiable functions or composite of (n + 1) or
n-number of differentiable functions of a differentiable
function of a differentiable function of x.
Explanation
1. y = sin x is a differentiable function of x, replacing
x by a differentiable function of x, say ex, we get
2. y = sin (ex) which is a differentiable function of x
being the composition of two differentiable functions

Chain Rule for the Derivative

namely sin and e. Replacing x by a differentiable


function x x > 0 , , we get
3. y = sin e

which is a differentiable function of x

being the composition of three differentiable functions


namely sin, e and
. Replacing x by a differentiable
function of x, say sec1 x, we get

F
GH

4. y = sin e

sec

I
JK

1
sec

sec

which is a differentiable

sec

sin

sin

e
e

sec

sec 1 x

Remember: Composition of two or more than two


differentiable functions is a differentiable function.
Question: What is the chain rule for the derivatives?
Answer: It is a rule (or, a theorem) which is used to
find the derivatives of composite (or, composition) of
two or more than two differentiable functions.
1. Chain rule for the derivatives of composite of two
differentiable functions:
The chain rule for the derivatives of composite of
two differentiable functions states that if y is a
differentiable function of z and z is a differentiable
function of x, then the derivative of a differentiable
function y is the product of the derivative of y with
respect to z and the derivative of z with respect to x.
Or, more explicitly,
If y = f (z) is a differentiable function of z and
z = f (x) is a differentiable function of x, then
y = F (f (x)) = G (x) (say) is a differentiable function of
x and

dy dy dz
=

.
dx dz dx

This is known as the chain rule of differentiation


since the derivative with respect to x of y = F (f (x))
involves the following chain of steps. Firstly
differentiation with respect to z of the whole differentiable function y = F (f (x)). Secondly differentiation
with respect to x of inner differentiable function z = f

dy
.
dx
2. The chain rule for the derivative of a finite number
of differentiable functions:
The chain rule for the derivative of a finite number
of differentiable functions is the generalised form of
chain rule for the derivatives of the composite of two
differentiable functions. It states that
If y = f1 (z1) is a differentiable function of z1;
z1 = f2 (z2) is a differentiable function of z2;
z2 = f3 (z3) is a differentiable function of z3;

zn 1 = fn (zn) is a differentiable function of zn and zn


= f (x) is a differentiable function of x, then, y = f1 f2 f3
fn f (x) is a differentiable function of x, and
(x) lastly, the product of these gives

function of x being the * or, alternatively, y = f1 (z1) is


a differentiable function of z 1; z 1 = f 2 (z 2) is a
differentiable function of z2 and so on; zn1 = fn (x) is
a differentiable function of x y = f1 f2 f3 ... fn (x) is
a differentiable function of x composition of four
differentiable functions namely sin, e,
and sec1
and this composition of four differentiable functions
can be expressed in the following way.

383

dzn 1 dzn
dy
dy dz1 dz2
=

...
and if y = f1
dx dz1 dz2 dz3
dzn
dx

(z1) is a differentiable function of z1; z1 = f2 (z2) is a


differentiable function of z2, and so on; zn 1 = fn (x) is
a differentiable function of x; then y = f1 f2 f3 fn (x) is

dy
dy
a differentiable function of x and dx = dz
1
dzn 1
dz1 dz2 dz3

...
.
dz2 dz3 dz4
dx

af

Theorem: Show that y = F (z) and z = f x y =


F z f x provided F (z) and f (x) are
differentiable functions.

af af

Proof: y = F (z)

y + y = F z + z

a
f af
F a z + zf F a zf
y

=
y = F z + z F z
x

384

How to Learn Calculus of One Variable

af

z
F z + z F z z
y
3
=1

z
x
x
z

y = F (f (x)), where F and f represent ( )n, e, log, sin,


cos, tan, cot, sec, cosec, sin1, cos1, tan1, cot1,
sec1, cosec1, etc.
1. (c)' = 0, where c is any constant
n
n 1
=n f x
f x
2. f x

3. sin f x = cos f x f x

4. cos f x = sin f x f x
2

5. tan f x = sec f x f x
2

6. cot f x = cosec f x f x

7. sec f x = sec f x cosec f x

8. cosec f x = cosec f x cot f x

af
b a fg
b a fg
af af
b a fg
af af
b a fg
af af
b a fg
shifting the place of z and x
a
f
af af
b
g
in the denominator
af
af
b a fg
a
f
a
f
af
y
b
g
lim
x

1
f
x
sin
a
f
=
f a xf
9.
e
j
F az + zf F a zf z I
F
= lim G

J
1 f a xf
z
xK
H

1
F F a z + zf F a zf IJ lim FG z IJ
f a xf
= lim G
10. ecos f a x fj =
z
H
K
H xK
1 f a xf
using the product rule of limits

1
11. e tan f a x fj =
f a xf
d F a z f dz
a
1
f
x
+
f
=

b3 y = F a zfg
dz
dx

1
12. ecot f a x fj =
= F a zf f a x f
f a xf
1 + f a xf
Remember:

(A): The following rules of finding derivatives are


1
f
x
=
f a xf
sec
a
f
13.
e
j
valid for the differentiable functions.
f
x
f
x
1

a
f
a
f

1. b f a x f + f a x fg = f a x f + f a x f

f a xf
14. ecosec f a x fj =
2. c k f b x gh = k f b x g , k being a constant
f
x
f
x
1
a
f
a
f

3. b f a x fg f a x f = f a x f f a x f + f a x f f a x f
f a xf

15. blog f a x fg =
; f a xf > 0

F
f a xf I
f a xf f a x f f a xf f a x f
f a xf
;
4. G
H f a xf JK =

f a xf
16. F e a f I = e a f f a x f
H K
b f a xf 0g

(C): If y is a function of x then


5. b f f a x fg = f a z f f a x f , where z = f (x)

af

F z + z F z z
y
=

x
z
x

x0

x0

x0

x0

2
2

f x

f x

(B): Power, exponential, logarithmic, trigonometric and


inverse trigonometric functions are differentiable on
any interval on which they are defined and their
derivatives with respect to the inner differentiable
function f (x) are found from the following formulas
provided their composition is represented as

dy n
dy
= n y n 1
, n Q
dx
dx
dy
d y
e =ey
2.
dx
dx
1.

385

Chain Rule for the Derivative

3.

1 dy
d
log y =
,y>0
dx
y dx

4.

d sin y
dy
= cos y
dx
dx

d
(base), where the base must be a differentiable
dx
function of x.

d
(t-function having another function of x at
dx
the place of the angle)
2.

5.

d cos y
dy
= sin y
dx
dx

6.

d tan y
dy
= sec y tan y
dx
dx

7.

d cot y
dy
= cosec 2 y
dx
dx

8.

d sec y
dy
= sec y tan y
dx
dx

9.

d cosec y
dy
= cosec y cot y
dx
dx

10.

d sin 1 y
=
dx

1
1 y

d cos y
=
dx

11.

1
1 y

dy
dx

dy
dx

FH

1
d cot 1 y
dy
=

2
dx
dx
1+ y

14.

d sec 1 y
=
dx
y

IK

d
(the function of the independent variable x after
dx
removing t-operator) where t or t-operator, means all
the sex trigonometric functions namely sin, cos, tan,
cot, sec, and cosec.
d
(t1-function having another function as an
dx
inner function of x)
3.

e - inner function of xj
=
F inner function of x afterI
dG
H removing t - operator JK
d t

d
dx

(inner

function of the independent variable x after removing


t1-opertor)

dy
2
y 1 dx

4.

1
d cosec 1 y
dy
=

15.
2
dx
y y 1 dx
Note: All the formulas for the derivatives of the
differentiable power, exponential, logarithmic,
trigonometric and inverse trigonometric of a
differentiable function f (x) can be expressed in words
in the following way.
1.

IK

d tan 1 y
1
dy
12.
=

2
dx
dx
1+ y
13.

FH

t - function having another function


d of
x at the place of the angle
=
function of the independent variable
d the
x after removing t - operator

d
(any function of x)n , n Q
dx
= index (that function of x used as a base)n 1 =
given index minus one

d
(ea function of x = index of the base e)
dx
= e index without any change

d
(index given as a
dx

function of x)
5.

d
log (any function of x)
dx
=

1
d

(given function of x
given function of x dx

after removing log-operator)


(Note: (any function of x)n is read as n power of any
function of x/any function of x to the power n/any
function (or, any function of x) raised to the power
n.)

386

How to Learn Calculus of One Variable

On method of finding the derivative of composite (or,


composition) of two differentiable functions
A differentiable function f1 of a differentiable function
f 2 of the independent variable x is symbolically
represented as y = f1 f2 (x), where
f 1 = outer differentiable function (or, outside
differentiable function) which means sin, cos, tan,
cot, sec, cosec, sin 1, cos1 , tan1, cot1, sec 1,
cosec1, ( )n, log,
, or, e, etc.
f 2 = inner differentiable function (or, inside
differentiable function) which means also sin, cos,
tan, cot, sec, cosec, sin1, cos1, tan1, cot1, sec1,
, or, e, etc
cosec1, ( )n, log,
f 1 f 2 (x) = given function/whole function/
dependent variable f 2 (x) = inner differentiable
function of x/insider differentiable function of x.
There are two methods of finding the derivative of
the composite of two differentiable functions.
(A): Method of substitution
(B): Method of making no substitution
(A): On method of substitution: This method
consists of following steps:
Step 1: Put a new variable z for the function of x
whose differential coefficient can be found from the
formulas.
d
c =0
1.
dx
n 1
d n
x = nx
2.
dx
d
sinx = cos x
3.
dx
d
cos x = sin x
4.
dx
d
2
tan x = sec x
5.
dx
d
2
cot x = cosec x
6.
dx
d
sec x = sec x tan x
7.
dx
d
cosec x = cosec x cot x
8.
dx

af
e j
a f
a f
a f
a f
a f
a f

9.

10.

1
d
sin x =
dx

1
1 x

1
d
cos x =
dx

1
1 x
1

11.

1
d
tan x =
2
dx
1+ x

12.

1
1
d
cot x =
2
dx
1+ x

13.

1
d
sec x =
dx

1
2

x 1
1

1
d
14. dx cosec x =

15.

x 1

a f

1
d
log x = , x > 0
dx
x

e j

x
d x
e =e
dx
N.B: The formulas for the derivatives of inverse
trigonometric functions have been derived in the
chapter containing inverse circular functions but we
have mentioned here their derivatives because their
application is required here.

16.

Note: All these results remains valid even if x is


replaced by any other variable z, u, v, w or t etc.
Explanation

e j
e j
d
et j = n t
dt
d
asin zf = cos z , dud asin uf = cos u ,
2.
dz
d
asin t f = cos t
dt
d
acos zf = sin z , dud acos uf = sin u ,
3.
dz
d
acos t f = sin t
dt

1.

n 1 d
n
n 1
d n
u = nu ,
z = nz ,
du
dz
n

n 1

387

Chain Rule for the Derivative

a f
a f
d
atan t f = sec t
dt
d
acot zf = cosec z , dud acot uf = cosec u ,
5.
dz
d
acot t f = cosec t
dt
d
asec zf = sec z tan z , dud asec uf = sec u tan u ,
6.
dz
d
asec t f = sec t tan t
dt
d
acosec zf = cosec z cot z , dud acosec uf
7.
dz
d
= cosec u tan u , acosec t f = cosec t cot t
dt
4.

d
d
2
2
tan z = sec z ,
tan u = sec u ,
du
dz

1
d
sec z =
dz

12.

z 1

1
d
sec u
du

1
d
sin z =
dz

8.

1
d
sin t =
dt

10.

e
e

1 z

1 t

1 u

1 z

1
1 u

1
d
tan u =
du

1
1 u

u 1

z 1

t 1

1
d
cosec u
du

a f

1+ z
2

1
d
cot u =
du

1
1+ u

t 1

1
1
d
d
log z = , z > 0 ,
log u = , u > 0 ,
dz
z
du
u

14.

a f

1
d
log t = , t > 0
dt
t

e j

e j

e j

z d
u
u d
t
t
d z
e =e
e =e ,
e =e ,
dt
du
dz

c b
b
d f a z f dz
=

gh d bnew variable zg
dx

Problems on Composition of Two Differentiable


Functions

Method of substitution

1
d
cosec t =
dt

Solved Examples

1
1+ t

dz
dx
Step 4: Express the result in terms of x by using the
relation z = f2 (x) established in step (1).

1
1 t

Step 2: Put y = f1 (a new variable z) = f1 (z)


Step 3: Use the formula:

dy d f 1 new variable z
=
dx
d new variable z

1
d
,
cos u =
du

u 1

1
d
sec t =
dt

1
d
cosec z =
dz

13.

1
d
cot z =
dz

1
d
cot t =
dt

1
d
tan z =
dz

1
d
tan t =
dt

11 .

1 z

1
d
sin u =
du

15.

1
d
cos z =
9.
dz
1
d
cos t =
dt

1 t

Find the derivative of the following


1. y = tan 3x2
Solution: y = tan 3x2

multiply by 3

3x

= z1

2
tan

tan 3 x

=y

388

How to Learn Calculus of One Variable

dy
dy dz1
=

, where z1 = 3x2
dx dz1 dx
=

d tan 3x

e j

d 3x
2. y = e

j d e3x j = 6x sec

dx

a f

divide by 1 x

FG 1 + x IJ
H1 xK
B
= z1

dy
dy dz1
=

, where z1
dx dz1 dx

e j

FG
H

1+ x
1 x

e j
1+ x
1 x

B
=y
F 1 + x IJ
=G
H1 xK

=e

=e

1+ x
1 x

IJ
K

IJ
K

a f a f a f
a f
j b1 xg 1 b1 + xgb1g
b1 xg
j F1 x + 1 + xI
G
H a1 x f JK
2

1+ x
1 x

a1 xf

e j
1+ x
1 x

square

= z1

dx

ex j

2 2

=
x

x
2

a f

2x
1

FH3 x
1

= x

=x

IK

x 1

a f

A composite (or, composition) of two differentiable


functions f1 and f2 is symbolically represented as
y = f1 f2 (x) and its derivative making no substitution
is found using the following working rule.
Working rule: The differential coefficient of the
differentiable function f1 of the differentiable function
f2 of the independent variable x is equal to the product
of the differential coefficient of the whole function y
= f1 f2 (x) with respect to the inner differentiable
function f2 (x) and the differential coefficient of the
inner differentiable function f2 (x) with respect to the
independent variable x; i.e. if y = f1 f2 (x) , then

af
af

af

af

af

dy d f 1 f 2 x d f 2 x
=

= f 1 f 2 x f 2 x
dx
d f2 x
dx

which tells us to apply the formulas as discussed in


(B) after the theorem for the chain rule.
Note:
1. y = f 1 f 2 x (any differentiable function of
x)n, provided f2 (x) = any differentiable function of x
means algebraic function and/transcendental function
of x and f 1 means to raise the base (being any
differentiable function of x) to the n th power.

y = f 1 f 2 x f 1 f 2 x = a f 1 ax + b ,
2.
provided f2 (x) = a linear algebraic function of x. e.g.,

(i) sin ax + b = a cos ax + b

(ii) cos ax + b = a sin ax + b

af

3. y = sec1 x2
Solution: y = sec1 x2

j d ex j

(B) On method of making no substitution while


finding the derivative of composite (or, composition)
of two differentiable functions

d e
d 1+ x
=

1+ x
dx 1 x
d
1 x
d
d
e 11+ xx j 1 x dx 1 + x 1 + x dx 1 x
=e

2
1 x

FG
H

2x

e j

1+ x
1 x

e j

3x

1+ x
1 x

a1 + xf

d x

e j

Solution: y = e

d sec

sec

sec

=y

dy
dy dz1
2
=

, where z1 = x
dx dz1 dx

af b

b a
b a

fg
fg

a fg

Chain Rule for the Derivative

btan aax + bfg = a sec aax + bf

(iv) bcot aax + b fg = a cosec a ax + b f

(v) bsec a ax + bfg = a sec aax + b f tan a ax + bf

(vi) bcosec a ax + bfg = a cosec aax + b f cot a ax + bf

(vii) eaax + bf j = n a aax + bf


and so on.
2

(iii)

5. y = sec (log x)
Solution: y = sec (log x)

n 1

Problems on composition of two differentiable


functions

a f

a f
a f
sec a log x f tan a log x f
=

d sec log x d log x


dy d sec log x
=
=

dx
dx
d log x
dx

x
6. y = tan (sin1 x)
Solution: y = tan (sin1 x)

Solved Examples

FH

j e

d sin x 3 dx 3
dy
d
=
sin x 3 =

dx dx
dx
dx 3

= cos x 3x = 3x cos x
2. y =

dy d
=
dx

sec

tan x
dx

j=d

1 x

dy d cos e

=
dx
dx

= sin e

4. y = tan 4x
Solution: y = tan 4x

b g

j = d ecos e j de
dx
d ee j
x

dy

=
dx

FH e

d sec tan

jIK

dx

FH e xjIK d etan xj
=

dx
d e tan x j
F 1 I
= sec etan x j FH tan e tan x jIK G
H 1 + x JK
x sec e tan x j
FH3 tan etan xj = xIK
=
d sec tan

e = e sin x

f a f

d tan 4 x d 4 x
dy d tan 4 x
=
=

dx
dx
d 4x
dx
2

Solution: y = sec tan

tan x d tan x sec 2 x

=
d tan x
dx
2 tan x

tan x

esin xj

3. y = cos ex
Solution: y = cos ex

7. y = sec tan

Solution: y =

jIK

FH e xjIK d sin

dx
d esin x j

d tan sin

tan x

1
dy
d
=
tan sin x
dx dx

Making no substitution
Find the differential coefficient of the following.
1. y = sin x3
Solution: y = sin x3

a f
2

= sec x 4 = 4 sec x

389

1+ x
8. y = tan1 (sin x)
Solution: y = tan1 (sin x)

a fj

dy d tan sin x

=
dx
dx

390

How to Learn Calculus of One Variable

a f a f
a f
1

d tan sin x d sin x


cos x

=
2
d sin x
dx
1 + sin x

9. y = sin (cos1 x)
Solution: y = sin (cos1 x)

FH e

d sin cos

dy

=
dx

jIK

FH e jIK d ecos xj
=

dx
d ecos x j
cos ecos x j
=
1

10. y = tan

FH

j IK

x =x

d x
1

1+

e xj

dy d sin sin

=
dx
dx
=

d sin sin

d sin

x
dx

2 x

2 x 1+ x

1 x

1 x

I
K

=1

fj

cos x
2

cos x

cos x
; cos x 0
cos x

13. y = cosec (4 3x)


Solution: y = cosec (4 3x)

2
2

asin xfj d asin xf

=
d a sin x f
dx

d sin

11. y = sin sin1 x


Solution: y = sin sin1 x

j
jd

F
H

x = x = sin ,

d tan

dy d sin sin x

=
dx
dx
=

dy d tan

=
dx
dx

Solution: y = tan

; x x 1 3 cos cos

may also simplified in the

dy cos sin x
Hence,
=
=
2
dx
1 x
12. y = sin1 (sin x)
Solution: y = sin1 (sin x)

1 x

cos = cos = 1 sin = 1 x

1 x

3 sin

d sin cos x

Note:

following way.

dx

1 x

cos sin

dy d cosec 4 3x
=
dx
dx

d cosec 4 3x

d 4 3x

fg

fg d a4 3xf
dx

= cosec (4 3x) cot (4 3x) (3) = 3cosec (4


3x) cot (4 3x)

j d esin xj = cos esin xj


1

dx

1 x

On method of substitution for the differential


coefficient of composition (or, composite) of a finite
number of differentiable functions
A differentiable function f1 of a differentiable function
f2 of of a differentiable function fn on a differentiable
function f of the independent variable x (or,
composite/composition of a finite number of
differentiable functions namely f1, f2, fn and f) is

Chain Rule for the Derivative

symbolically represented as y = f1 f2 f3 fn f (x), where


f1 f2 f3 fn f (x) = given function/whole function/
dependent variable.
f2 f3 fn f (x) = inner differentiable function/ inside
differentiable function/inner function/inside function/
independent variable.
f1, f2, f3, , fn or f, each = constituent differentiable
function/constituent function.
f (x) = inner most differentiable function/inner most
function.
There are two methods of substitution for finding
the derivatives of composition of a finite number of
differentiable functions.
Method 1: The first method of substitution for finding
the derivatives of composition of a finite number of
differentiable functions consists of the following
steps.
Step 1: Start from the left hand side introducing a
new variable whenever a new function occurs untill
we get a simple function of x (or, simply an identity
function x) whose derivatives can be found by using
the rules for the differential coefficient of power,
trigonometric, inverse trigonometric, logarithmic,
exponential, sum, difference, product or quotient of
two (or, more than two) differentiable functions of x;
i.e.;
Separate the first function and put the rest function
= z1
Separate the second function and put the rest
function = z2

We continue the process of separating the function


and putting the rest = a new variable unless we get a
simple function of x having the standard form which
can be differentiated by using the rules for the
derivatives of power, trigonometric, inverse
trigonometric, logarithmic, exponential, sum,
difference, product or quotient of two (or, more than
two) differentiable functions of x.
Step 2: Differentiate separately each function
supposed as z1, z, z3, , z n with respect to the
independent variable x of the given function and
multiply all these derivatives thus obtained to have
dzn 1 dzn
dy
dy dz2
=

...
dx dz1 dz3
dzn
dx

391

Step 3: Express the result in terms of x using the


relations
f2 f3 f4 fn f (x) = z1,
f3 f4 fn f (x) = z2,
f4 fn f (x) = z3

fn f (x) = zn 1
f (x) = zn which are stablished in step (1)
Note: Generally in practice, we put z1 for the inner
most function which is simply an identity function x
or a simple function of x, say f (x) having the standard
form xn, sin x, cos x, tan x, cot x, sec x, cosec x, sin1
x, cos1 x, tan1 x, cot1 x, sec1 x, cosec1 x, log x, ex
etc. starting from the right hand side which means we
may start introducing a new variable from the right
hand side whenever a new function occurs linked in a
chain unless we get the first function in the left hand
side.
Explanation
(A) If the given function y = f1 f2 f3 fn f (x), then
z1 = f (x)
z2 = fn (z1)
z3 = fn 1 (z2)

zn = f2 (zn 1)
y = f1 (zn) = given function

dzn 1 dz2 dz1


dzn
dy dy
=

...
, provided
dx dzn dzn 1 dzn 2 dz1 dx
we start making substitutions from right hand side.
(B): If the given function y = f1 f2 f3 fn f (x), then
z1 = f2 (z2) = f2 f3 f4 fn f (x)
z2 = f3 (z3) = f3 f4 fn f (x)
z3 = f4 (z4) = f4 fn f (x)

zn 2 = fn 1 (zn 1)
zn 1 = fn (zn)
zn = f (x)
dy dy dz1 dzn 1 dzn
=

...
, provided we start
dx dz1 dz2
dzn
dx

making substitutions from left hand side.

392

How to Learn Calculus of One Variable

Remember:
1. Total number of function in y = f1 f2 f3 fn f (x)
= n + 1 n N , including f whereas total number of
substitutions made = n.
2. Total number of the derivatives of all the
constituent differentiable functions f1, f2, f3 ,, fn and

f = n + 1 n n.
3. We make no substitution for the given function
y = f1 f2 f3 fn f (x), since it is already given equal to y.
4. We must make a substitution for the expression
(or, the function) in x under the radical whenever we

have

an expression in x .
5. The reader should mind that the method of
substitution becomes lengthy when the chain of the
intermediate variable is more than one. This is why
the reader is advised to derive the differential
coefficient of the composition of a finite number of
differentiable functions without making substitutions
since method of substitution is easy to handle only
in elementary cases.
Method 2: To make the substitution z1, z2, , zn etc
while finding the derivative of composite of a finite
number of differentiable functions, we may adopt the
following procedure also.
Step 1: Start with a function of x (or, simply x) which
can be differentiated by using the rules for the
differential coefficient of power, trigonometric, inverse
trigonometric, logarithmic, exponential, sum,
difference, product or quotient of two (or, more than
two) differentiable functions.
Step 2: Seek the way how to reach y = f1 f2 fn f (x)
= the whole given function from x (or, the power,
trigonometric, inverse trigonometric, logarithmic,
exponential, sum, difference, product or quotient of
two (or, more than two) two differentiable function
(or, functions) of x.
Step 3: Name each of these steps as the variable z1,
z2, z3 , zn etc. and differentiate these all with respect
to x.
Explanation:

af
B
B
= a xf = a z f
x f x
f

af
B
= az f

fn

f n

f x
2

f n 1

af

f n 1 f n x ...

B
a f

= z3

f1

f1

af

B
= a yf

f 2 ... f n f x

Solved Examples
1. Let y = [sin (log x)]2
Here, starting from x and reaching y in successive
steps can be explained in the following way.

a f
B
= a yf

log
take sin
square

log x
sin log x sin log
x take

B
a f

af

= x

B
a f

= z1

= z2

Remark: We put the variables for the function of x


as last one = given composite differentiable function
= y before y = z2
before sin log x = z1
before log x = x
and in this way we reach x. Further we should note
that total number of functions = 3, (log, sin, ()2)
whereas the substitution made = 2 in number.
2. Let y = e

sin tan 2 x

Here, again starting from x and reaching y in


successive steps can be explained as below
1

a2 xf
B
= az f

take tan
2 x tan
x doulble

B
B
a f = az f
sin e tan 2 x j
B
= az f
= x

take sin

sin tan
take exponential

e

B
af

a2 xf

= y

Remember: In each successive step while finding


the derivative of a differentiable function of a
differentiable function of a differentiable function
of x, we differentiate each constituent differentiable
function regarding the rest as an inner differentiable
function using the rule of composite of two
differentiable functions; i.e.,

a fg = d df f fa xafxf d fdxa xf dxdx

d
fm fn x
dx

Chain Rule for the Derivative

On method for the derivative of composition of a finite


number of differentiable functions without making
any substitution
To find the derivative of the composition of a finite
number of differentiable functions namely f1, f2, f3, ,
fn represented as y = f1 f2 f3 fn f (x), we have a rule
known as the chain rule for the derivative of composite
(or, composition) of (n + 1) number of differentiable
functions.
Rule: The derivative of a differentiable function f1 of
a differentiable function f 2 of a differentiable
function f n of a differentiable function f of the
independent variable x

LMoperand = all the remaining successiveOP


differentiable functions excepting
PP
= f M the preceeding differentiable
MMfunction aor, functionsf which
PQ
Nhas a havef been differentiated.
= f b f f ... f f a x fg f b f f ... f f a x fg
f b f f ... f f a x fg f b f f ... f f a x fg ...
f b f a x fg f a x f
n

i =1

The above working rule may be expressed in the


following way.
1. Start from left to find the derivative of each
successive (following in order/coming one after
another) functions coming near and near to x with
respect to the remaining all other function (or,
functions) (excepting the differentiated function or
functions) till we arrive at x or an expression in x
being power, trigonometric, inverse trigonometric,
logarithmic, exponential, sum, difference, product or
quotient of two (or, more than two) differentiable
function (or, functions).
2. Find the product by multiplying all the results being
the derivative of each constituent differentiable
function. We may express in words the facts in (1)
and (2) in the following way.

F operand = the restI F operand = the restI


GG function upto x JJ GG function up to x JJ

f G without any
f G without any

J
GG change exceptingJJ GG change exceptingJJJ
GH f
JK GH f and f
JK
1

393

F operand = the rest I


GG function up to x JJ

f G without any change J ... f blast operand g


GG excepting f , f andJJ
GH f
JK
blast operand g .
n

Where f n (last operand) means the derivative of


the last function fn whose last operand is simply an
identity function x or a simple function of x being
differentiable whose derivative can be found by using
the rule for the derivatives of sum, difference, product
or quotient of two or more than two differentiable
functions.
Notes: 1. If the last operand = identity function = x,

af

dx

= x = 1 which is generally ignored while


dx
finding the derivative.
then

d
may be thought as an operator which changes
dx
the form of only that function (or, operator like sin,
cos, tan, cot, sec, cosec, sin1, cos1, tan1, cot1,
sec1, cosec1, log, e, etc) before which it is put
excepting the exponential operator e which does
not change its form before and after the differentiation.
e.g.,
d
1

=

1.
dx
2
d

n
n 1
2.
= n

dx
d

3.
sin = cos
dx
d

4.
cos = sin
dx
d
2

5.
tan = sec
dx
d
2

6.
cot = cosec
dx
d

7.
sec = sec tan '
dx
2.

e
a
b
b
b
b
b

af
f af af
a fg a f a f
a fg
afaf
a fg
afaf
a fg
afaf
a fg a f a f a f
j

394

How to Learn Calculus of One Variable

a fg
a fj

b
e

af

af af

cosec = cosec cot


dx
1
1
d

sin =

9.
2
dx
1

1
1
d

cos =

10.
2
dx
1
8.

a fj

a fj

a fj

a fj

1
d
tan =
11.
dx

af

1+

13.

1
d
sec =
dx

a fj

1
d
cosec =
14.
dx

a f

f1 +

(iv) y =

...

f3
f2 +

af

...

f x

f 3 + ...

af

fn + x

af

f x

af af af af
g a xf
y = f a xf f a xf
g a xf
y = f a xf g a xf g a xf
g a xf
y = f a xf
g a xf
y = f a xf f a xf

(v) (a) y = f 1 x f 2 x g1 x g2 x
1

(c)

a f

1
2

f2

(iii) y =

(b)

a f
d
blog a fg = a1f a f
dx

FH IK

af

a
f
af

1
d
1
cot =
dx
1+

15.

af

af

12.

af

af

(ii) y = f 1

a f

16. d e a f = e a f
dx
where dots within circular brackets/absolute value
sign denote the operand being the same in the l.h.s
and r.h.s. Further, we should note that it is only the
operator e which remains unaltered before and after
the differentiation.

af

Remember: The above explanation throws light


upon the fact that only function (or, operator) is
differentiated successively till we get x when the given
function is y = f1 f2 fn f (x), where each constituent
function f1, f2, , fn or f being differentiable represents
sin, cos, tan, cot, sec, cosec, sin1, cos1, tan1, cot1,
sec1, cosec1, log, e, ( )n, n , | |, etc.
Problems on the composition of a finite number of
differentiable functions
Different types of problems in the composition of a
finite number of differentiable functions may be
guided only by the problems appearing in different
forms which are
(i) y = f1 f2 f3 fn f (x) or, y = (f1 f2 f3 fn f (x))n

(d)

(e)

Now we consider each one by one


Form: (i) y = f1 f2 f3 fn f (x)
(ii) y = (f1 f2 f3 fn f (x))n
whenever we are given the problems in the above
forms, we find their derivatives using the following
working rule.
Working rule: To find the derivative of differentiable
function having the form (1), we use directly the rule
for the derivative of the composition of a finite number
of differentiable functions and the derivative of a
differentiable function having the form (2) is found
using the following rule:
n
n 1
=n F x

(a) Use the formula: F x

b a fg

af

b a fg

F x where F (x) = f1 f2 fn f (x).

af

(b) Find the derivative F x using the rule for the


derivative of the composition of a finite number of a
differentiable functions.
Do not forget

af

b a fg x f a x f b f a x fg f b f a x fg
af
2. b f b f a xfgg x f axf f b f a xfg b f b f a xfgg
1. f 1 f x

f1

f1

where f (x) = f1 f2 f3 fn f (x)/sum/difference/


product or quotient of two or more than two
differentiable functions.

395

Chain Rule for the Derivative

Solved Examples

cos x

Without substitution

; sin x > 0

4 x sin x

Find the differential coefficients of the following.


1. y =

Solution: y =

4. y = tan 1 + x + x

tan 2 x

dy d
=
dx
d
=
d
=

tan 2 x , defined for tan 2x > 0

tan 2 x
dx
tan 2 x d tan 2 x d 2 x

tan 2 x
d 2x
dx

a f

f a f

sec 2 x

Solution: y = tan 1 + x + x

FG
H

dx
1

, tan 2x changes
2
into sec2 2x, 2x changes into 2 while differentiating.

2. y =

changes into

sin x

sin x

dy d

=
dx

sin x

x cos x
sin x

1+ x + x

af

d sin x

e j

d x

j d ex j
2

dx

; sin x2 > 0

d sin x
d sin x

tan tan x

Solution: y =

d sin x
d

e xj

jd

x
dx

, 1+ x + x 2

tan tan x

a fj
d tan a tan x f d tan a tan x f d tan x
=

d tan a tan x f
d tan x
dx
sec x sec a tan x f
=
; tan (tan x) > 0
2 tan a tan x f

dy
d
=
dx dx

sin x

changes into 1 + 2 x
5. y =

changes into

sin

a1 + 2 xf sec
=

f 1 f 2 f 3 x = tan 1 + x + x

dy
d
=
dx dx

j
2 1+ x + x

af

Remember: sin changes into cos, while


differentiating.

Solution: y =

Remember: tan changes into sec2

d sin x

3. y = sin

f3 x = 1 + x + x

dx

d sin x
=

IJ
K e

Note: That f1 = tan, f 2 =

Solution: y =

d 1+ x + x
dy
d
2
=

tan 1 + x + x
2
dx dx
d 1+ x + x

d 1+ x + x

tan 2 x

Remember:

tan tan x

396

How to Learn Calculus of One Variable

Note: That f 1 =
=
f2 = tan, f3 = tan, f4 = x,

f1 f2 f3 f4 =

tan tan x

af

1
2

8. y =

FH

etan x jIK d etan x j d e x j


dx
d e tan x j
d ex j
2

1 tan x

2 x sec x

tan x 2 < 1 .

7. y = tan1 (a ex x2)
Solution: y = tan1 (a ex x2)

dy
=
dx

FH

d tan

ea e

FH

ea e

d ae x

dx

d tan

j
a e e2 x + x j
=
1 + ea e x j
x

1+ a e x
x

2 2

Note: Here, f1 = tan1,


f2 (x) = a ex x2

2 2

a e x + 2x e

=
x

Note: f1 = cos1
f2 = tan
f3 = ( )2
f3 (x) = (x)2
f1 f2 f3 (x) = cos1 tan x2

jIK

1
sec

FG sec
H
FH3 x

FG
H

IJ
K
IJ d esec
K

dx

IJ
K
x

sec

IJ FG
KH

j dx

F x
GH e j

2 2

x 1
2

=x

dx

IK

1
1

IJ FG
KH

= x

x 1

2x
I
1J
K

IJ
K

IJ , | x | > 1
K

Form 2:

jIK d ea e
j

=
x

f1
x

dx

sec

F
2G
H

sec 2 x 2 2 x

1 tan 2 x 2

FG
H

FG
H

d sec

dx

d FH cos etan x jIK


dy

d cos

Solution: y = sec x
1 2
dy
d

=
sec x
dx dx

6. y = cos1 (tan x2)


Solution: y = cos1 (tan x2)

dx

sec

f2

af

f 3 ... f x

i.e. the form of the composite function being


differentiable obtained by the operation of applying
a differentiable function (sin, cos, tan, cot, sec, cosec,
sin1, cos1, tan1, cot1, sec1, cosec1, log, e, | |, etc.)
and the operation of taking the square root is
successively performed more than once upon a
differentiable function of x.
Working rules: 1. Method of substitution:
(a) Put z =

af

f x which may be regarded as the

inner most function


(b) Use

e z j = 2 1 z azf dxdz

(c) Lastly express z in terms of x.

397

Chain Rule for the Derivative

2. Method of making no substitution:


The above type of problems may be differentiated by
using directly the formula for the derivative of a finite
number of differentiable functions regarding firstly
as f1 and secondly f2 as any function being
differentiable like trigonometric, inverse trigonometric,
power, logarithm i.e., mod or exponential in every
successive step i.e. we have to use the formula:

a f
f a x fg f b f

f 1 f 2 f 3 ... f n f x

b
f b f a x fg f a x f ,

= f 1 f 2 f 3 ... f n

a fg

differentiable function as
and trigonometric,
inverse trigonometric, logarithmic, mod or exponential
etc. in every step.

d 1+ x 2

cos

2 sin 1 + x
x cos
=

F2
GH

FG
H

FG
H

1+ x

sin 1 + x

I FG
JK H

Solution: y = cos sin x


dy
=
dx

Method 1:
Solution: y =

sin 1 + x

dy
d
=
sin z
dx dx
1
dy
d sin z dz

dx 2 sin z
dx
dx

1
2 sin 1 + x
x

2 sin 1 + x

Method 2:

af

d
d
y =
dx
dy

F
GH

cos

FH

1 + x2

1 + x2

sin 1 + x

I
JK

1+ x

a f

2x

IJ
K

IK

dx

= sin sin x

cos x

2 sin x

2 1+ x

dx

2x

cos z

IJ
K2

d F cos sin x I d F sin x I d esin x j


H
K H
K
=

d esin x j
de xj
d F sin x I
H
K
d e xj

Putting z = 1 + x ,

FH

d cos sin x

IJ
K
1+ x

Find the differential coefficient of the following.


2

1+ x

2. y = cos sin x

sin 1 + x

dx

IK

Solved Examples

1. y =

1+ x 2

dx

f 4 ... f n f x ...

regarding each successive

FG sin 1 + x IJ d F sin 1+ x I d F
K H
H
K H
F
I
F
I
d sin 1 + x
H
K d H 1+ x K

IK
; sin e1 + x j > 0

FH sin
=
4F
H

IK e j
, sin
x I e xj
K

sin x cos x
sin

3. y = sin x
Solution: y = sin x

1
2 x

x >0

398

How to Learn Calculus of One Variable

dy

=
dx

FH

sin x

FH

dx

=
4

IK d esin x j d e x j

dx
xj
d e xj

FH

sin x
cos

FH

sin

IK

cos

e j

2 x

j de

dy d sin cos x
=
dx
dx
=

d sin cos x
d

cos x

= cos cos x

cos x

b g

d cos x

af

j d bcos xg

2 cos x

sin x cos cos x


2 cos x

sin x

1 + x + x2

af

af

f 1 x + f 2 x + ... f n x +

x >0

f a xf
af
a xf + f a xf

f x

dx

af

f x

where each function of x is either a constant or a


differentiable function of x excepting the inner most
differentiable function of x which can never be a
constant; the form of the composite function obtained
by performing the operation of taking square root
and addition of a differentiable function of x or a
constant successively more than once upon a
differentiable function of the independent variable x
which can be expressed in the arrow diagram as

fn
1

Form 3:

4. y = sin cos x
Solution: y = sin cos x

sec 2

e x j , sin
x I e xj
K

b g
F
2 H 1 + x + x IK
FH 1 + x + x IK 2 x + 1

sec 2 1 + x + x 2 2 x + 1

sin x

d sin

IK

af

+ fn x

f n

af

a xf + f a x f

+ fn 1 x

...

The method of finding the derivative of the


composite differentiable function of the above form
is explained by the examples done below using the
method of substitution.

, for cos x > 0

Solved Examples
Find the differential coefficient of the following.

5. y = tan 1 + x + x

1. y =

a+ a+ a+ x

Solution: y = tan 1 + x + x
dy
d
2

=
tan 1 + x + x
dx dx

FG
H

FG
H

Solution: y =

IJ d 1+ x + x
j
K e
I d e1+ x + x j dx
F
d G 1+ x + x J
K
H

d tan 1+ x + x
2

IJ d FG
K H

IJ
K

1+ x + x

a+ a+ a+ x
2

Putting a + x = u , we have y =
and

du
d
=
dx dx

FG
H

a+x

IJ
K

a+ a+u

399

Chain Rule for the Derivative

d
=

FG
H

a+ x

d a+ x

IJ d ea + x j
K
2

dx

a+x

...(1)

3y =

a+ a+u

Again

putting

a + u = v,

we

have

y = a + v and
dv d
=
du
=
=

a+u

du

a+u

d a+u

j d aa + u f

3y = a + v

a + v = w , we have

...(3)

2 a+v

af a f af

FH 2

e3 v =

IK F 2
GH
a + uj

FH

F
a + u IK G
H

=
4

a + a +u

a+

I FG
JK H

a+ a+x

af

x f x

...

dy
dw dv du
= 1 2 3 =

dx
dv du dx
x
1
1
=

2
2 a+v 2 a+u
a+ x
=

IK

i.e. the form of the composition of a finite number of


differentiable functions obtained by performing the
operation of taking the square root more than once
upon a differentiable function of the independent
variable x which can be expressed in the arrow diagram
as

IJ
K

a + x2

af

...(2)

IJ F
KH

a + a + x2

... f x

2 a+u

d a+v
dw
1
=

dv
dv
2 a+v

a+x

I F
JK GH

Note: The above procedure of finding the derivative


of the composite differentiable function having the
form mentioned above is fruitful only when f1 (x), f2
(x), , fn (x) are all constants excepting the inner most
differentiable function f of x but when f1 (x), f2 (x),
, fn (x) are all constants besides the inner most
differentiable function f of x, we directly use the
chain rule for the derivative without making any
substitution.
Form 4:

du

Lastly, putting

a + a + a + x2

FG3 u =
H

F
GH

a+ a+x

I FG
JK H

af

f x

af

f x

af

... f x . The method of finding the

derivative of the composite differentiable function


having the above form is explained by the examples
done below using the method of substitution.
Solved Examples
Find the differential coefficient of the following.

x
a+x

IJ
K

1. y =

x+1

Solution: y =

a+x

IJ
K

dz d
=
dx

x+1

x + 1 = z , we have y =

Putting

x +1
dx

z and

400

How to Learn Calculus of One Variable

x +1

d x +1

j d a x + 1f

differentiable functions of xs for finding

dx

dw
.
dx

dy
1
dz
=

dz 2 z dx
=
=

1
2 z

Note: The given problem may be the combination


(sum, difference, product and/quotient) of composite
differentiable functions of xs or it may be the
combination of a differentiable x and a differentiable
function of a differentiable function of x.

1
2 x +1

2 FH

x + 1 IK 2 e

1
x +1

Solved Examples
Find the differential coefficient of the following

, x > 1
x + 1 I e x + 1j
K
Form 5: (a) y = f a x f f a x f g a x f g a x f
g a xf
(b) y = f a x f f a x f
g a xf
(c) y = f a x f g a x f g a x f
g a xf
(d) y = f a x f
g a xf
(e) y = f a x f f a x f
=

4F
H

where anyone or all of f1 (x), f2 (x) , g1 (x) and /g2 (x)


may be a differentiable function of a differentiable
function of the independent variable x.
Working rule: The working rule consists of following
steps.
Step 1: Put each addend, subtrahend and minuend
equal to u, v and w respectively and then y becomes
equal to u v w ; i.e. y = u v w .

F I
H K

d
Step 2: Take the differential operator
on both
dx
sides of the equation defining y as a function of the
independent variable x. i.e.,

du dv
,
and
dx dx

dy
d
du dv dw
=
uvw =

dx dx
dx dx
dx
Step 3: Use the rules for the derivative of the product,
quotient and/composite of two or more than two

FG cot x + 1 x IJ
H x
K
F cot x + 1 x IJ
Solution: y = GH
K
x
2

1. y =

cot x
2
and v = 1 x , we have y
x
dy du dv
=
+
= u + v and
which means we have to
dx dx dx
du
dv
and
separately and then their sum is to
find
dx
dx
be found out.
Putting u =

du x cosec x + cot x
Now,
=
2
dx
x

2 x
and dv =
dx 2 1 x 2

(i)

(ii)

af a f

dy du dv
=
+
= 1 + 2
dx dx dx

x cosec 2 x + cot x

2. y =

x
1 x2

tan x
2
+ x 1 x
x

Solution: y =

tan x
2
+ x 1 x
x

,0 < x <1

401

Chain Rule for the Derivative


2

= 2 x cos 4 x 4 x sin 4 x

tan x
2
and v = x 1 x , we have
x

Putting u =

dy du dv
=
+
y = u + v and
dx dx dx

du
x sec x tan x
=
Now,
2
dx
x

dv
=
and
dx

LM 1 x
N

LM
= 1 x
MN
F 1 2x
=G
GH 1 x

1 x
2
2

OP
PQ

I
JJ
K

have y = u + v and

2
2

I
JJ , 0 <
K

and

ax + bx + c

IJ
K

dy du dv
=
+
dx dx dx

= v, we have y = u + v and

j e

j b2 x g
12

FG
H

...(i)

IK
+ bx + c IK

ax 2

ax + bx + c

FH

d sin ax 2 + bx + c
d

FH

ax 2 + bx + c

IK

IK

IJ
K

dx
= 3 sin 2 ax 2 + bx + c cos ax 2 + bx + c

...(i)

dv
2
= 2 x cos 4 x + x sin 4 x 4
dx

dv
=
dx

FH
d FH sin

FG
H

d sin ax 2 + bx + c

= u and x cos 4 x = v , we

1 x

IJ
K

= sin (ax2 + bx + c) (2ax + b)


= (2ax + b) sin (ax2 + bx + c)

IK

IJ
K

+ x cos 4 x

x cos 1 x

ax + bx + c

dy du dv
=
+
dx dx dx

and

1
du
= cos 1 x 2 1 x 2
Now,
2
dx

ax + bx + c

x <1

FG
H

du d cos ax + bx + c d ax + bx + c
=

Now,
2
dx
dx
d ax + bx + c

+ x cos 4 x

Putting sin 1 x

FH

Solution: y = cos ax + bx + c + sin

FG sin
H

Solution: y = sin 1 x

4. y = cos ax + bx + c + sin

...(ii)

1 x

+ 2 x cos 4 x 4 x 2 sin 4 x , x < 1

Putting cos (ax 2 + bx + c) = u and

af a f
F 1 2x
x tan x
G
GH 1 x
x

3. y = sin 1 x

x cos 1 x 2

j a2 xfOPQ
21

dy du dv
=
+
= 1 + 2
dx dx dx
x sec 2

af a f

dy du dv
=
+
= 1 + 2
dx dx dx

(i)

2
x
+ 1 x
2

...(ii)

1
2

2 ax + bx + c

2ax + b

402

How to Learn Calculus of One Variable

f FGH

3 2ax + b sin
=

IJ FG
KH

ax + bx + c cos ax + bx + c

IJ
K

2 ax +bx + c

FG
H

...(ii)

af a f

3 b2ax + bg F sin
H

ax

+ bx + c I F cos
KH

ax

2 ax + bx + c

+ bx + c I
K,

for ax2 + bx + c > 0.

2. y = e

Exponential Functions
We recall that exponential functions are differentiable
on any interval on which they are defined and their
derivatives are obtained by using the formula
f a xf
d f axf
=e
f x
e
dx

x x

af

Solved Examples
Find the differential coefficient of the following.

Solution: y = e

x x

=e

x x

x x

x x

x x

3. y = e

a cot x f

x +1

j
2

d
dx

x x

F d x dx I
GH dx dx JK
j F 1
I
G
2 x J for x > 0.
H2 x K
j

a cot x f

dy
d a cot x f2
=
e
dx dx

cot x
d cot x
dy d e b g d cot x
=

dx d cot x 2
d cot x
dx
2

dy
d

=
e
dx dx

x x

Solution: y = e
x +1

2x

d e

=e

x2 + 1

dy
d e
=
e
dx dx

x
d x
e =e
dx

1. y = e

Note: If f (x) = x = an identity function,

x2 + 1

xe

Solution: y = e

where the derivative f x is obtained by using the


rule for differential coefficient or power, exponential,
logarithmic, inverse trigonometric, sum, difference,
product, quotient or composite of two or more than
two differentiable functions.

x +1

IJ
K

2 x +1

af

x +1

=e

dy

= i + ii
dx
= (2ax + b) sin (ax2 + bx + c) +

2
d x +1
x +1
dy
de
d 2

x +1
2
2
dx
dx
d x +1
d x +1

b g

=e

acot x f

b g
e

2 cot x cosec x

403

Chain Rule for the Derivative

= 2 cot x e b

esin x j
1

4. y = e

Solution: y = e

esin x j

dy
d esin

=
e
dx dx

d e

esin x j

d sin

=e

sin

2 sin

a fg = ff aaxxff , f a xf > 0

d
log f x
dx

cosec 2 x , x n

and if some differentiable function | f (x) |, provided


f x 0 at any point belonging to any interval on
which f (x) is defined is under the sign of logarithm,
its derivative found using the chain rule for the
derivative of the composite differentiable function is

af

dy
=
dx

cot x

d
log f x
dx

a f g = ff aaxxff , f a xf 0 .
d
blog f a xfg for
Remember: 1. The derivative
dx
d
blog f a xf g for f a xf 0 is called
f (x) > 0 and/
dx
b

d sin

d sin

2 sin

esin x j
xe
1

1 x2

d sin x
dx

1
1 x

, x <1

logarithmic derivative of the function f (x). Further we

should note that either the derivative log f x

for f (x) > 0or the derivative log f x


for

af
af

a fg

a fg

f x 0 is equal to the ratio of the derivative


f x to the value of the function f (x); i.e. if y = f (x)
is a positive differentiable function of x and / y = f (x)
is a differentiable function of x such that f (x) may be

af
af

f x
is
f x

Logarithmic Functions

positive and negative both, then the ratio

We recall that logarithmic functions are differentiable


on any interval on which they are defined and their
derivatives are

dy
and then
dx
dividing it by the given value of the differentiable
function represented at f (x).
2. If some differentiable function of x is under the
sign of logarithm, it is pre-assumed (or, understood)
that f (x) is positive (i.e. f (x) > 0 is pre-assumed or
understood) or we have to mention that f (x) > 0 while
finding the logarithmic derivative, i.e. to differentiable
the function of x that can be put in the form:
log f (x) = a logarithm of the function of the
independent variable x, it is pre-assumed (or,
understood) that f (x) is a positive differentiable
function of x whenever no restriction (or, condition)
which makes f (x) positive is imposed on the
independent variable x or we have to mention that
f (x) > 0.
3. Logarithm of a function of x (or, logarithmic
function of x) put in the form log f (x) is not defined

a f

1.

1
d
log x = , x > 0
dx
x

2.

d
log x
dx

3.

4.

g = 1x , x 0
f a xf
d
, f a xf > 0
log f a x fg =
b
dx
f a xf
f a xf
d
, f a xf 0
log f a x f g =
b
dx
f a xf

i.e. of some positive differentiable function of x, say f


(x) is under the sign of logarithm, its derivative found
using the chain rule for the derivative of the composite
differentiable function is

obtained on finding the first derivative

404

How to Learn Calculus of One Variable

for f (x) < 0 at any point x belonging to any interval on


which it is defined. This is why we take the modulus
of the value of the function f (x) if the value of the
function concerned f (x) is negative at any point x
while finding the logarithmic derivative of the
differentiable function of x.
4. Sometimes we are given a differentiable function
of x under the sign of logarithm along with the interval
(or, the quadrant) in (or, on, or, over) which the value
of the differentiable function f (x) > 0 or the condition
imposed on the independent variable x (like x > a, x <
a, x a etc.) which makes f (x) positive is given.
Notes: 1. If the value of the function f (x) is both
positive and negative in the interval (or, the quadrant)
in which it is defined, we take the modulus (or,
absolute value) of the value of the function f (x) only
when the function f concerned at x is to be positive as
in case of logarithmic differentiation.
2. By using the rules for differentiating the power,
exponential, logarithmic, trigonometric, inverse
trigonometric, sum, difference, product, quotient or
composite of two or more than two differentiable
functions, we are able to find the differential coefficient
of the value of the differentiable function f (x) written
under the sign of logarithm.

af

af

3. f x 0 f x > 0
On Types of Problems
In general there are two types of logarithmic functions
whose derivative is required to find out.
1. The value of a differentiable function f (x) written
under the sign of logarithm with or without an interval
(or, quadrant) in which f (x) is positive.
2. A differentiable function | f (x) | written under the
sign of logarithm.
Further we should note that type (1) has the
following forms:
(i) y = log f (x) and / y = log log log f (x)
(ii) Power of logarithmic function, i.e. y = [log f (x)]n
and / y = [log log log log f (x)]n
(iii) A logarithmic function with base other than e,

af

i.e. y = log a x f f x .

On Language
1. It is common to say the function f (x) and / the
value of the function f (x) for the function f at (or, of)
x, the value of the function f at (or, of) x and / the
function of x namely (or, say) f (x).

af
af

f x
is the ratio of the derived
f x
function f at (or, of) x to the function f at x instead
Hence, we say

af
af

f x
is the ratio of the derived function
f x
(or, derivative) f x to the function (or, the value of
the function) f (x).
2. Plural of a function of x is functions of xs. This
is why whenever we want to mention more than one
function of x, we write (or, say) functions of (or, at )
xs. however, if f1 (x) is a differentiable function of x
and f2 (x) is also a differentiable function of x, we
write f1 (x) and f2 (x) are differentiable functions of xs.
of saying

af

Problems based on first type


Form 1: Problems on the form
y = log f (x)
or, y = log log log log f (x); f (x) > 0
Solved Examples
Find the differential coefficient of the following.

FG 1 + tan x IJ , 0 < x <


4
H 1 tan x K
F 1 + tan x IJ
Solution: y = log G
H 1 tan x K
= log a1 + tan x f log a1 tan x f
d
dy
d
log a1 + tan x f
log a1 tan x f

=
dx
dx dx
1. y = log

sec x
sec x
+
1 + tan x 1 tan x

= sec

LM 1 tan x + 1 + tan x OP
N a1 + tan xf a1 tan xf Q

405

Chain Rule for the Derivative

F 2 I
GH cos x JK
2

2 sec x
2

1 tan x

FG
H

dy
d
=
log x +
dx dx

F cos x sin x I
GH cos x JK

2
cos 2 x

2
2

cos x sin x

FG 2 x 6 IJ , 3 < x < 5
H 5 x K
F 2 x 6 IJ = log b2 x 6g log b5 xg
Solution: y = log G
H 5 x K
dy
d
d

=
log a 2 x 6f
log a5 x f
dx dx
dx
1
2
1
1
=

2x 6 5 x x 3 x 5
=

FG
H

FG
H

x +a

1
2

x+ x +a

LM
N

1+

1
2

x +a

1
2

x+

x +a

LM
MN
L
M
MN

IJ
K

1 2
2
x +a
2

2 x +a
2

x +a

x 1

IJ
K

x +a

dy
1
1
2
=

cosec x
dx log cot x cot x
2

2x

cosec x
; cot x > 1
cot x log cot x

+ x

6. y = log [log (cot x)]


Solution: y = log [log (cot x)]

12

2x

1+

1
2

x +a

IJ
K

OP
Q

FG
H

x+

x 1 ,x >1

Solution: y = log x +

1
sin x
cos x

4. y = log x +

dy
=
dx

21
1 2
x 1 2x
2

1
x

IJ
K

F
I
GG1 + x JJ
x 1K
1 H
F x 1 + xI
GG
JJ
1 H
x 1 K
x +a I
K

Solution: y = log x +

dy
d

=
log cos x
dx dx

= tan x

FH

Solution: y = log cos x

5. y = log x +

3. y = log cos x , 0 < x <

x+

2
x3 x5

fa

x 1

2. y = log

1+

x+

= 2 sec 2 x , 0 < x <

LM
N

1
x+

x 1

OP
PQ

OP
PQ

OP
Q

406

How to Learn Calculus of One Variable

3 cot x > 1 log cot x > 0


log log cot x is defined
7. y = log [log (log x)]
Solution: y = log [log (log x)]

dy
dx

a f
d log a log x f

d log log log x

1
1
1

log log x log x x

1
1
1

;
x log x log log x for x > 0 i.e. when y is

d log log x d log x

d log x
dx

Find the differential coefficient of the following.


1. y = (log x)3; x > 0
Solution: y = (log x)3

a f

dy
d
=
log x
dx dx

dy
= 3 log x
dx

a f

a f

d log x
d log x

d log x
dx

1 3 log x
=
x
x

a f

a f

3 log x
.
x
2. y = [log (cos x)]4
Solution: y = [log (cos x)]4
=

a f

defined.
8. y = log log log log log x5
Solution: y = log log log log log x5

Solved Examples

dy d log log log log log x


=
dx
dx

a f
d log acos x f
d log a cos x f d cos x
=

d log acos x f
d cos x
dx
1
= 4 log a cos x f
a sin x f
cos x

dy
d
=
log cos x
dx dx

d log log log log log x

d log log log log x 5


d log log log log x 5
d log log log x
d log log x 5

d log x 5

d log log log x 5


d log log x 5

d log x 5
d x5

je

je

5x

je

5
5

je

Form 2: Problems on the form


y = [log f (x)]n
or, y = [log log log log f (x)]n

sin x
cos x

< x<
2
2
Solution: y = [log (cos3 x)]2
5

x loglogloglog x logloglog x loglog x log x

for all those values of x at which y is defined.

3
3. y = log cos x

x loglogloglog x logloglog x loglog x log x


5

= 4 log cos x tan x ; where cos x > 0.

d x5
dx

= 4 log cos x

dy
d
=
log cos3 x
dx dx

e j
d log ecos x j

d log cos3 x

= 2 log cos3 x

= 6log cos3 x

d log cos3 x
3

d cos x

3 cos2 x 2
cos3 x
sin x
cos x

j d cos

cos x

sin x

d cos x
dx

Chain Rule for the Derivative

FG
H

= 6 log cos 3 x tan x ;

<x<
2
2

IJ
K

2. y = log log | x |, | x | > 1


Solution: y = log log | x |

Remember:
f (x) f (x) f (x) up to n times = [f (x)]n = f n (x).

Problems on Second Type


Form 1: Problems on the form
y = log | f (x) | and y = log log log log | f (x) |;

bg

f x 0

dy
d
=
log log x
dx dx
=

1
1
1
=
log x x x log x

FH

dy
d
3
=
log x + 1
dx dx

Find the differential coefficient of the following

a + b tan x
a b tan x

Solution: y = log

FG
H

a + b tan x
dy
d
=
log
dx dx
a b tan x

a + b tan x
a
; tan x
a b tan x
b

IJ
K

aa b tan xf
aa + b tan xf
aa b tan xf eb sec xj aa + b tan xf eb sec xj
aa b tan xf
aa b tan xf
=
aa + b tan xf
aa b tan xf eb sec xj + aa + b tan xf eb sec xj
aa b tan xf
aa b tan xf b sec x aa b tan x + a + b tan xf
=
aa + b tan xf
aa b tan xf
2

2 a b sec x
2

a b tan x

ex

3x 2
3

tan 1 x

+1

j IK

sec 1 x

2 x sec 2 1 x 2

for x 1

j a 2 x f

e j
= 2 x sec e x 1j cosec e x 1j
tan 1 x 2
2

5. y = log | sin x|, x a multiple of


Solution: y = log | sin x |, x n , n Z

dy
d
=
log sin x
dx dx

1
cos x
sin x

cos x
= cot x , x n , n Z
sin x

fa

+1

FH

3x 2 =

dy
d
2
=
log tan 1 x
dx dx
=

ex

IK

4. y = log | tan (1 x2) |, | x | < 1


Solution: y = log | tan (1 x2) |

2 a b sec x
=
a + b tan x a b tan x

3. y = log | (x3 + 1) |
Solution: y = log | (x3 + 1 ) |

Solved Examples

1. y = log

407

Form 3: Problems on the form:

af

y = log a x f f x

408

How to Learn Calculus of One Variable

Working rule: To differentiate a logarithm of function


of x whose base is another function of x, we adopt
the following working rule.
Step 1: Write y =

af
af

y=

af
af

log f x
Step 2: Differentiate y =
w.r.t. x by using
log x

D N N D
D

alog xf
L log x log a x + 1f OP
1
=
M
x
alog xf N x + 1
Q
2

Exercise Set on

Composition of two differentiable functions


Form: y = (any differentiable function of x)n
or, y = f1 f2 (x)

log e a log a
=
Remember: log b a =
log e b log b

Exercise 9.1.1

Solved Examples
Find the differential coefficient of the following.
1. y = log sin x

1 + tan x ; 0 < x <


2

Solution: y = log sin x 1 + tan x


y=

log 1 + tan x
log sin x

dy
of each of the following functions.
dx
1. y = (3x + 1)4
2. y = (7x2 + x)2
Find

3. y = 1 x
4. y =

dy
dx

log sin x

d
d
log 1+ tan x log 1 + tan x log sin x
dx
dx
2
log sin x

a bx

8. y =

a p qxf

9. y =

log sin x

log sin x
2. y = logx (1 + x), x > 0 , 1
Solution: y = logx (1 + x)

7. y =

x 2 + ax + 1 ; a 2 < 4

6. y = 7 x + 11x + 39

sec x log sin x cos x log 1 + tan x


+
1 + tan x
sin x

5. y = (x3 + 1)5

F 1 IJ sec x loga1+ tan xf 1 acosxf


logsin x G
sin x
H1+ tan xK
=
2

log x log x + 1

x+1
x

dy
=
dx

the rule for differentiating the quotient of two


differentiable functions.
i.e. y =

log e f x
log f x
=
log e x
log x

af
af

log 1 + x
log x

10.
11.
12.
13.

bl mxg

3
4

, n Z

y = (3x 7)12
y = (5x5 3x)24
y = (4x + 3)5
y = (3x2 + 2x + 1)8

3
2

Chain Rule for the Derivative

F
H

1
x

14. y = x +

I
K

15. y = 3 x + 4

5
2

3x + 5

FG
H

17. y = (1 x)6

1
3 4 x + 5x 2 7 x + 6

;x>0

8.

9.

2 1 x2

a2 x + a f

IK

a x + 1fa x + 2fa x + 3f

3
2
7 x + 11x + 39
2
b
2 a bx

;x<

a
b

74

3
q p qx
4

dy
of each of the following.
dx
y = sin x3
y = tan x2
y = sin (cot x)
y= sin (sec x)
y = sin 5x
y= cos 2x
y = sin x2

Find

4 x 3 + 5x 2 7 x + 6

; x <1

5. 5 (x3 + 1)4 3x2

7.

FH

3x + 12 x + 11

19.

2 x + ax + 1

6.

IJ
K

Exercise 9.1.2

2 x

4.

18.

bx + 1gbx + 2gbx + 3g ; x > 1

Answers
1. 12 (3x + 1)3
2. 2 (7x2 + x) 14 + 1
3.

10
7
x
3
3

4x2 +
3

19. y =

3
2

17. 6 (1 x)5

3x + 5

18. y =

3x

16.

16. y =

15. 15 x 3 x + 4

j a14 x + 11f
1
2

1.
2.
3.
4.
5.
6.
7.

8. y = sin x
;x<

p
q

mn

l
;x
if n > 0
n +1
m
l mx

9. y = sin

F 2I
H xK

10. y = cot 2x
11. y = cot (1 2x2)
12. y = cos (1 x2)
1

10. 36 (3x 7)11


11. 24 (10x 3) (5x2 3x)23
4
12. 20 (4x + 3)6, x
3

13. y =

sin

14. y =

cos

13. 16 (3x + 1) (3x2 + 2x + 1)7


14. (2x 2x3) x 0

15. y =

tan

1
1

16. y = cosec

x
x
x

FG 1 x IJ , 0 x < 1
H1 + xK

409

410

How to Learn Calculus of One Variable

FG 2 x + 7 IJ
H 1 2x K
F 1 IJ
y = sin G
H xK

18.

17.

18.

Answers
1. 3x2 cos x2

4. cos (sec x) (sec x tan x); x n +

Composition of more than two differentiable functions


Form 1: y = f1 f2 f3 fn f (x)
or,
y = (f1 f2 f3 fn f (x))n

5. 5 cos 5x
6. 2 sin 2x
7. 2x cos x2

9.

cos x
2 x

15.

16.

3x + 4

2. y = cot sin x
,x>0

3.
4.
5.
6.
7.

F 2 cos 2 I
G x JJ , x 0
G
GH x JK
2

n
2
11. 4x cosec2 (1 2x2)
12. 2x sin (1 x2)

14.

Exercise 9.2.1
1. y = cosec

10. 2 cosec2 2x, x

13.

cos

Exercise Set on

2. 2x sec2 x2, x n + , n Z
2
3. cos cot x cosec2 x, x n
2

8.

F 2 x + 7I , x 1
b1 2 xg GH 1 2 x JK 2
F 1 IJ , x > 0
1

cos G
H xK
2x x
16

17. y = sin

1
2 sin 1 x

1
1 x2

,0< x <1

F 1 I
J; x < 1
G
2 cos x GH 1 x JK
F 1 IF 1 I
;x>0
J
GG
H 2 tan x JK GH 1 + x JK
F 1 x IJ cot FG 1 x IJ
2
cosec G
H1 + xK H 1 + xK
a x + 1f
1

y = cot (tan x)
y = sin2 x2
y = cos2 x2
y = tan3 x2
y = (log x2)2
1

8. y =

sec

9. y =

cosec

10. y =

sin

11. y = (tan1 x3)2


12. y = (sec1 xm)n
13. y = (cos1 xp)q

14. y = cot

1
3

15. y = cot e
16. y = secn (ax2 + bx + c)

17. y =

cos 3x + 4

18. y = secn (m x)
19. y = tan1 (4ex + 3)2
20. y = sec3 (m sin1 x)

411

Chain Rule for the Derivative

21. y = cos5 (log tan2 x3)4

e j
tan a tan x f

22. y =

15.

sin sin x

23. y =

IJ
K

3x + 4 cot 3x + 4

2. 2 x cosec sin x cos x


3.
4.
5.
6.

9.

3x

6 x tan

1+ x

2x

e
1 + e4e

j e

1 x
2

23.

sec

m n 1

24.

x 2m 1

p q x p 1 cos1 x p
1 x
x 2 cot 1 x 3
1 + x6

j
+ 3j

21. 120 x cos log tan x

22.

m 1

3 4

sin log tan x

elog etan x jj FGH sectan xx IJK


cos x cos esin x j
4 x sin esin x j
sec x sec a tan x f
2 tan a tan x f
b2 sin alog xfg cos alog xf 1x
3

1 x

xm

2x

8e x 4e x + 3

mn x
12.

14.

x 1

2
1

IJ e1 + e j
K

18. m n secn (m x) tan m x

x cosec 1 x 2 x 4 1

2 sin

13.

cos 3x + 4

3x + 4

10.

11.

20.

3m sec 3 m sin 1 x tan m sin 1 x

4
2
log x
7.
x

2 sec

17.

19.

cosec2 (tan x) sec2 x


4x sin x2 cos x2
4x cos x2 sin x2
6x tan2 x2 sec2 x2

8.

cot

3x sin 3x + 4

Answers (under suitable restrictions on x)

FG 3 cosec
H 2

FG
H

16. n (2ax + b) secn (ax2 + bx + c) tan (ax2 + bx + c)

24. y = (sin (log x))2

1.

2p

23

Exercise 9.2.2

q 1

Find

dy
of each of the following functions.
dx

1. y = sin cos tan x


2. y = sin cos tan cot x

412

How to Learn Calculus of One Variable

3. y = cos tan x + 1
4. y = cos sin log x
5.
6.
7.
8.

y=e

esin tan

2x

11.

tan x

11. y = e

x2

esec
x

je

Form 2:
f1

af

f2

f 3 ... f x

Exercise 9.3

12. y = sin cos tan mx , m > 0


13. y = sin cos tan sec x
Answers (under proper restrictions on x)
1.

e cos cos tan x jesin tan x j sec

2. (cos cos tan cot x) (sin tan cot x)


cosec2 x

3. sin tan x + 1 sec

x+1

fa

5. e

j cos

etan

2x

8.

2 x cos x
sin x

9.
10.

1
2 x

cot x)

2 x +1

dy
of the following functions.
dx

1. y = sin cos ax
2. y = sin cos tan mx
3. y = sin x
4. y = cos x
5. y =

2
1 + 4x2

6. (cosec2 sin cos x) (cos cos x) sin x

1
1
1
7. log log x log x x

Find

(sec2

1
4. sin sin log x cos log x
x
sin tan 1 2 x

je

mx sin tan mx cos cos tan mx

tan x

6. y = log x
Answers (under suitable restrictions on x)

FH

IK e
j
ax IK e ax j

a cos cos ax sin ax

1.

FH

cos

FH

IK e
tan mx IK e

m cos cos tan mx sin tan mx sec mx

1
2
sec x cot x e
2

tan x

2.

2x

e1 + x j tan
4

13. sec x tan x (sec2 sec x) (sin tan sec x) (cos


cos tan sec x)

9. y = e
10. y = log (tan1 x2)
1

1 + x4

m
12.

y = cot sin cos x


y = log log log x (x > 0)
y = log sin x2

e tan

2 x e tan

3.

FH

cos

cos x
4 x

cos x

tan mx

Chain Rule for the Derivative

Required Answer

sin x

4.

4 x

cos x

sec

5.

4 x

1
1.
cos x
2

F
GG1+
GH 2

tan x
1

6.

4x

log x

Form 3:

y=

af

af

f 1 x + f 2 x + ... +

af

F
GG
GH

1
sin x + sin x +

1
sin x + sin x

+
4

FH

I
JJ
sin x JK
I
JJ
I
sin x K sin x J
K

1
sin x +

2. Find
3. Find

f x

4.

Exercise 9.4

2 2x 2 + 2x

dy
Find
of each of the following functions
dx

5.

1. y =

sin x + sin x + sin x

Form 4:

2. y =

cos x + cos x + cos x

3. y =

x+ x+ x

4 ax a +

ax

af

... f x , f (x) being a differentiable function

of x.
Exercise 9.5

4. y =

2 + 2x
dy
of each of the following functions.
dx

5. y = a + ax

Find

Answers
Hint: In the above problems, we may use directly the
chain rule without making any substitution as

1. y =

sin x

2. y =

3. y =

1.

F
I
d G sin x + sin x + sin x J
H
K
d FH sin x + sin x + sin x IK
R| d
I
F
x
+
S| dx asin xf + GG dd esinsinxx++ sinsinxxj JJ FGH d sin
dx
K
H
T
d sin x d sin x I U

JV
d sin x
dx K W

413

Answers (under suitable restrictions on x)


1.

2.

FH

4F
H

cos x
sin x
1

IK e

sin x

IK e x j

414

3.

How to Learn Calculus of One Variable

F
2G
H

x
x

FG3
H

I FG x IJ
JK H K

or
2x

= x and x

x
x

j
IJ
K

1
2

af af af af
g a xf
y = f a xf f a xf
g a xf
g a xf
y = f a xf
g a xf
y = f a xf g a xf g a xf
1

Answers
1. 2 sin log x, x > 0

(d)

2. sin

3. sec

Exercise 9.6

4. tan

dy
of each of the following functions.
dx
1. y = x sin log x x cos log x, x > 0
Find

1
2. y = x sin x

3.

4.

5.

6.

F xI , x
H 2K

x + 1 1I
x
+
J
1+ x
x + 1 + 1K
5x
+ cos a 2 x + 1f

LM 1 + 1 OP
MN x 2 b x + 1g x PQ

x +1

7.

e j
3 e1 x j
5 3 x2

4
3

F
y = log G
H

8.

a + x

9.

x a

cos 2 x
10.

1 x

FG
H

x
a
2
2
a +x +
log x +
2
2

11.
2

x +a

IJ
K

12.

sin x
1
sin 3 x cos x
1
3

sin x

2 sin 4 x + 2

7. y =

8. y =

, x <1

x, x > 1

6.

+ 2 1 x 2 sin 1 x 2 x , x < 1

F
x log G x +
H

5. 2 x + 1 , x

I
y = x sec
x 1J , x > 1
K
F xI , x
y = log e x + 4j x tan
H 2K
y = x x + 1 + log F x + x + 1I , x
H
K
1

+ log tan x

1
x 1 cos x
log tan
2
2 2 sin 2 x

12. y =

(c)

+ cot x

2 sin x

(a) y = f 1 x f 2 x g1 x g 2 x
1

3 sin 3 x

11. y =

Form 5:

(b)

cos 3 x

10. y =

x
=
,x 0
x

FG
H

x
a
2
2
x a
log x +
2
2

9. y =

x a

IJ
K

Chain Rule for the Derivative

Form 5: continued
Problems based on combination composite
differentiable functions of xs.

2. y = sin x + cos
3.

F
y = cos eax + bx + cj + sin G
H
2

4. y = log log x e

ax

I
+ bx + c J
K

5x

5. y = cot x tan x
Answers

2.

1
cos log x cot x
x
1
2 x

cos x 1 2 sin x

f e
2

cos ax + bx + c

j
a2ax + bf

9. y = e

log cos x
x

1
2 x

2 x

log cos x e

sec

1
2 x

tan x

sin x log sin x

m 1
x cos n 1 x m cos x cos x
7. sin

e x
2 x

10.

elog cos

2 x

dy
of each of the following functions
dx
1. y = sin2 3x cos3 2x
2. y = x2 cot 2x

x tan x

ecos

x + sin x

Exercise 9.6.3

Exercise 9.6.2

Find

10. y = e sin x
Note: This type of problems can be done using the
rules of logarithmic differentiation.

9.

n sin x sin x
8. 2 cos (3x + 4) cos (2x + 3) 3sin (2x + 3) sin (3x + 4)

1
5x
5e
x log x

5. 3 cot x cosec x

6. cot x cos x

ax + bx + c

4.

log cos x

7. y = sin x cos x
8. y = sin (2x + 3) cos2 (3x + 4)

5.

3 2
2
3. 2ax + b sin ax + bx + c + sin ax + bx + c
2
2

Answers (under proper restrictions on x)


1. 6 sin 3x cos2 2x cos 5x
2. 2x (cot 2x x cosec2 2x)
3. 2x sec2 x (1 + x tan x)
4. 3x2 cosec3 x (1 x cot x)

1.

5. y = e

6. y = cos x log sin x

Find dy of each of the following functions.


dx
1. y = sin log x log sin x
2

3. y = x2 sec2 x
4. y = x3 cosec3 x

Exercise 9.6.1

415

Find

dy
of each of the following functions
dx

1. y =

2 x sin x
x

3
2

416

How to Learn Calculus of One Variable

2. y =

tan

1+ x

4. 6 cosec 6x (cosec x cot 6x)


x
2

sin

5.

b x + 1g x 1
y=
b x + 4g e
2

4. y =

5. y =

sin

6. y =

sin m x
cos n x

7. y =

sin x

10. 2 sec2 2x cos (x + 1) tan 2x sin (x + 1)


Exercise 9.6.4

Find

cos x

9. y =

cos
sin

10. y =

baxg
bbxg

3. y = x

x
e

2.

a2 x 3x cos x + sin xf
x
1

x1
2

ex

+ 3x + 2

2 x

5. y = x 3 2 x

6. y = x 1
2

x 2x + 2

7. y = x x + c

1.

2 2

a x + 1f
a x + 4f

Answers

3.

x sin x

a1 2 xf
e1 + x j

tan

4. y = 2 x

Answers

2. y = x 1 + 2 x

Note: This type of problem can also be done using


the rules of logarithmic differentiation.

1.

g b

1. y = x 1 x + 1

tan 2 x
sec x + 1

dy
of each of the following functions.
dx

f
f

cos 3 2 x
8. y =
sin 3 + 2 x
m

m 1

9. ma sin a x sin b x + bn cosb x cos a x

a
a

f FG cos
H sin

1 x

3
2

6. m cos mx sec nx + n sin mx sec nx tan nx


7. (4 sin3 x sec4 x + 5 tan5 x sec x)
8. [2 sin (3 2x) 2cos (3 2x) cot (2x + 3)] cosec
(2x + 3)

sin 6 x
1 + cos 6 x
e

3.

FG x + 1 x IJ
H
K
e1 x j

2.

LM 5x 3 x + 6 OP
MM 2 e x 1j x + 4 PP
Q
N
2

a x + 1fa5x + 1fa x 1f

1 + 3x
1 + 2x

3. x 4 x 2 + 3x + 2

j e11x
2

+ 24 x + 10

n +1

I
J
bx K
ax

Chain Rule for the Derivative

4.

x 8 5x
2x

3 4x

5.

3 2x

10. y =

1 x

x 2x + 2

x2

x 2 + e2

x +c
2

Exercise 9.6.5

Find

x 1
x +1

9. y =

2x 4x + 3

7.

x 2x + 3

6.

2x 4x + 3

8. y =

dy
of each of the following functions.
dx

1. y =
2. y =
3. y =

4. y =
5. y =
6. y =

7. y =

11. y =

12. y =

1 4x

b1 + 3xg
1+ x
2+ x

14. y =

1 x
1 x

15. y =

a x 1fa x 2f
a x 3fa x 4f

ea

1 x
1+ x

3
2

1 + 3x

4.

x 8 5x
2 x

3 4x

3 2x

Answer (under proper restrictions on x)


1. Find
2. Find
3. Find

1 + 2x

1
2

13. y =

x 11
x +1+1

3x

x
x +1

5.
6.
7.
8.
9.

1
2

LM
MN

1+ x
1 x

a1 + xf

Find
Find
Find
Find

b x + 1g

1
x2 1

OP
1 + x PQ
1 x

417

418

10.

How to Learn Calculus of One Variable

e1 x j
2

11.

12.

13.

14.

6. y = e

e1 4 x j
3 b2 3 x g
2 b1 + 3x g
3
2

3
2

2 2+x

7. y = e

tan x

8. y = e

cos x

9. y = e

cot

10. y = e

x + 3x + 2

11. y = e

1 x

sin

sec

esin x j
1

12. y = e

j
a x 3fa x 4f a x 1fa x 2f
2

3
2

1
2

Form 6: Problems based on exponential functions


of xs having the form y = ef (x), where f (x) stands for
any one of the elementary functions of xs (like sin x,
cos x, tan x, cot x, sec x, cosec x, sin1 x, cos1 x,
tan1 x, cot1 x, sec1 x, cosec1 x, xn log x, ex or their
combination) or composite of a differentiable function
of a differentiable function of a differentiable
function of x.
Exercise 9.7

cot x

13. y = e

2 x 10 x + 11

15.

e3x 6 x + 2j

a1 xf

1
2

1+ x

5. y = e

3
2

14. y = e

cosec

15. y = e

16. y = e

17. y = e

18. y = x e

2x

Answers (Under proper restrictions on x)


2x

Find

dy
of each of the following functions.
dx

1. y = e

2x

2. y = e

3. y = e

sin x

4. y = e

2. 3x e

3. cos x e

3 x

e
1.
2

4. 6 x e
5.

sin x

3 x

x
1+ x

1 + x2

Chain Rule for the Derivative

6. 6 x 1 e
2

7. sec x e

8.

9.

cos

cot

10.

sin

11.

Exercise 9.7.1

Find

sec

1 x

x
2

cot x

FG e
H

sin

IJ
K

5. y = log tan

cosec x

2 cot x

cosec 2 x cot x e cosec


14.

15.

16.

17.

x
e

FG
H

6. y = log 4x
7. y = log x2
8. y = log (5x 14)
9. y = log (xn + a)
10. y = log (ex + 1)
11. y = log (ex + ex)
12. y = log (cos x + 3)
13. y = log sin x
14. y = log tan x
15. y = log tan1 x
16. y = log log tan1 x
17. y = log tan

2 x
2x e
2 x

18. e

2x

18. y = log
x
2

xe
or
x

a2 x + 1f

IJ
K

F xI
H 2K

2 x
e

2. y = log (x + 3)2
3. y = log sin2 x
4. y = log sin 3x

dy
if
dx

1. y = log x + 1 + x

x 1

2 sin

13.

tan x

12.

Form 7: Problems based on logarithmic functions of


xs having the form:
y = log f (x), where f (x) stands for any one of the
elementary functions of xs (like sin x, cos x, tan x,
cot x, sec x, cosec x, sin1 x, cos1 x, tan1 x, cot1 x,
sec1 x, cosec1 x, xn, log x, ex or their combination) or
composition of a differentiable function of a
differentiable function of of a differentiable function
of x.

1 x
e

6x + 2

1+ x
e

1 x

e3x

419

,x0
19. y = log

F
GH

F + xI
H 4 2K

I
J
x + 4 + 9K
x+4 9

1+ x + x

1 x + x

420

How to Learn Calculus of One Variable

20. y = log

a cos x b sin x
a cos x + b sin x

21. y = log

1 4x
1 + 4x

FG
H
F
y = log G x
H

9.

IJ
K
I
1J
K

22. y = log x 1 + x

23.

Fx +
GG
Hx

26. y = log

F
y = log G
H

27.

28. y = log

I
JJ
1K

x 1
x

1 + ax
1 ax

I
J
1 xK

x log x

12.

2
2.
x+3
3. 2 cot x
4. 3 cot 3x

7.

2
x

x +a

e +1
x

e e
e +e

sin x
cos x + 3

13. cot x
14.

15.

16.

1
sin x cos x
1

e1 + x j tan
2

e1 + x j tan
2

1
1

x log tan

17. sec x
18.

19.

20.

x
x
1
sec , cosec = cosec x
5.
2
2
2
1
6.
x

n 1

nx

1
1+ x

11.

Answers (under proper restrictions on x)


1.

10.

1+ x 1 x
1+ x +

5
5x 14

1 cos x
1 + cos x

24. y = log

25. y = log

8.

a x 77f
1 x
4

x + x +1

ea

ab
2

22.

cos x b sin x

4
21.

x +4

16 x 1
1
1 x

Chain Rule for the Derivative

23.

1
x 1

24. cosec x

19.

2
25.

x 1

20. y = log

a
26.

27.

FG 1 + sin x IJ
H 1 sin x K
F a + b tan x IJ
y = log G
H a b tan x K

18. y = log

1 a x

LM a4 x + 1f a3x + 2f OP
MNa2 x + 3f a6x 4f PQ
L a x 1f a2 x 3f OP
y = log M
MN a2 xf PQ
F e 1I
y = log G
H e + 1JK
F x x + 1I
GG
JJ
log
y=
H x +2 K
F 1 + x xI
y = log G
GH 1 + x + x JJK

21. y = log

1
x 1 x

1+ x
1 x

1
4

1
3

1
2

1
6

22.

2 log x
28.
x

Exercise 9.7.2

dy
if
dx
1. y = log ( 3x 2)
2. y = log sec x
3. y = log cosec 2x

23.

Find

24.

4. y = log x + 1
5. y = log log x
6. y = log (sec x + tan x)
7. y = log sin x
8. y = log (cosec x cot x)
9. y = log sin1 x
10. y = log cos x2
11. y = log cos1 x4
12. y = log sin 2x
13. y = log log log x3

25.

Answers (under proper restrictions on x)


1.

3
3x 2

2. tan x
3. 2 cot 2x
4.

x
2

x +1

x2

14. y = log sin e


15. y = log sin (x2 + 1)

FG
H
F
y = log G x +
H

IJ
K
I
1J
K

16. y = log x +

x +1

17.

5.

1
x log x

6. sec x
7. cot x
8. cosec x

421

422

How to Learn Calculus of One Variable

9.

sin

x 1 x

10. 2 x tan x
11.

2
x
x
+ 2
3
x
x +1 x +2

24.

25.

1+ x

4 x 3
cos 1 x 4

1 x8

Exercise 9.7.3

12. 2 cot 2x
13.

14.

3
x log x log log x 3
2x e

cos e

sin e

1.
2.
3.
4.

Answers

2 x cos x + 1

15.

1
1+ x

2.

2x
,x 6
x 6

3.

sec 2 x
,xn
2
tan x

4.

1
,x 0
x

x 1

18. 2 sec x

Exercise 9.7.4

2ab
19.

a cos x b sin x
1

20.

1 x

1
1
1
1
+

21.
4 x + 1 3x + 2 2 x + 3 6 x 4
22.

23.

2
6
5
+
+
x 1 2x 3 2 x
2e
e

2x

17.

1. a tan ax + b , ax + b n +

sin x + 1

16.

dy
if
dx
y = log | cos (ax + b) |
y = log | x2 6 |
y = log | tan x |
y = log | x |

Find

Differentiate the following w.r.t. x

2.
3.
4.

5.
1

I
F
H
2K
y = tan log x , F 0 < x < e I
H
K
y = sin log x , a x > 0f
log x
y=
, a x > 0f
1 + x log x
x sin x
y=
, a x > 0f
1 + log x

1. y = log cos x , 0 < x <

Chain Rule for the Derivative

a f , FH 2 < x < 2 IK
y = log F x + x + a I
H
K

6. y = log cos x
7.

Answers
1. tan x
1
2
sec log x
2.
x
1
cos log x
3.
x

a f
a f

4.

5.
6.

a f
a
f
x cos x a1 + log x f + sin x log x
a1 + log xf
2 tan x log acos x f
1
2
log x
x
2
1 + x log x

1
7.

x +a

423

424

How to Learn Calculus of One Variable

10
Differentiation of Inverse
Trigonometric Functions
Differentiation of Inverse Circular Functions

Note:

Before explaining the techniques of finding the


differential coefficient of inverse circular functions,
we recall the definition of an inverse of a function
which tells us that the mapping must be one-one and
onto in order that an inverse of a function should
exist. Hence, to make the trigonometric functions oneone and onto, we restrict the domain of each
trigonometric function by the principal values of the
angle (i.e., the smallest positive value of the angle or
the smallest numerical value of the angle) because
each trigonometric function is a many valued function
and a many valued function has no inverse.

(i) sin

Definitions:
1. We define sin

F
H

x as an angle measured from

I
K

i.e.,
to
whose sin is x. The
2
2
2
2

angle satisfying the inequality is


2
2
1
called the principal value of sin x . Hence, =

(ii)

2. We define cos

x as an angle measured from

0 to i.e. 0 whose cosine is x. The angle


satisfying the inequality 0 is called the
1

principal value of cos x . Hence, = cos x is an


angle representing an inverse circular function whose
domain is [ 1, 1] and whose range is 0, . In the
notational form, the inverse of a cosine function is
defined as = f

a x f = cos

x x = cos and

0, , x 1, 1 .
Note:

acos f = for 0,
cos e cos x j = x for x [ 1, 1]

(i) cos
(ii)

sin x is an angle representing an inverse circular


function whose domain is 1 x 1 and whose range

asin f = for LMN 2 , 2 OPQ


sin esin x j = x for x 1, 1
1

3. We define tan

and
2
2

x as an angle lying between

F i.e; < < I whose tan gent is x.


H 2 2K

. In the notational form, the inverse of


2
2
sin function is defined as
= f 1 x

The angle satisfying the inequality

= sin 1 x x = sin and

called the principal value of

is

bg

LM , OP , x 1, 1
N 2 2Q

tan

< < is
2
2

x. Hence

Differentiation of Inverse Trigonometric Functions


1

= tan x is an angle representing an inverse circular


function whose domain is entire number line

(i.e. < x < ) and whose range is

F i.e; < < I .


H 2
2K

F
H


,
2 2

I
K

= cot

x because in this interval 0 < < ,

= cot 1 x is defined at all points. For this reason, it is


usual to take the interval 0 < < as the range for
= cot

x for practical purpose instead of

considering the range

In the notational form, the inverse of trigonometrical tangent is defined as = f

F
H

= tan and
Note:
(i) tan

a tan f =

I
K

a xf =

tan 1 x x


,
, x R .
2 2

b g

b g

(i) cot 1 cot x = for 0,

< < , 0 ) whose cotangent is x. The


2
2

angle satisfying the inequality < ,


2
2
0 is called the principal value of cot

x . Hence,

= cot x is an angle representing an inverse


circular function whose domain is entire number
line (i.e; < x < ) and whose range is

F , I k0p = F , 0I F 0 , I .
H 2 2K
H 2 K H 2K

In the notational form, the inverse of cotangent

I kp
K

af

b g

4. Some writers define cot x as an angle

lying between and


excluding = 0 (i.e.,
2
2

F
H

x is undefined at = 0 .

0, , x R . We shall follow this definition.


Note:

a xf =

bg

cot 1 x x


,
0 , x R .
2 2
An important remark: It is customary to take the
= cot and

follows = f 1 x = cot 1 x x = cot and

(ii) tan tan x = x for x R

= cot

F , 0I F 0 , I in which
H 2 K H 2K

Hence, an alternative definition of cot 1 x is also


available which is expressed in the notational form as


for F , I
H 2 2K

function is defined as = f

425

b g

open interval (as a domain) 0, in (or, on, or over)


which the function x = cot has an inverse

(ii) cot cot 1 x = x for x R

5. We define sec

x as angle measured from 0

FG
H

RS UVIJ
T WK

i.e; 0 ,
whose
2
2
secant is x. The angle belonging to the interval

1
0,
is called the principal value of sec x .
2
to excluding =

RS UV
TW

Hence = sec x is an angle representing an


inverse circular function whose domain is

b , 1g b1, g and the range is


FG i.e; L0, IJ FG , OIJ .
H MN 2 K H 2 PQK

0,

RS UV
T2 W

In the notational form, the inverse of


trigonometrical secant function is defined as

a xf = sec x x = sec and 0 ,


RU
S V , x a , 1f a1, f .
T2 W
6. (i) f f a x f = x for all x in the domain of f .
= f

426

How to Learn Calculus of One Variable

af

(ii) f 1 f x = x for all x in the domain of f.


Therefore these relations (i) and (ii) hold for the
restricted trigonometric functions and their inverse.
Note:
(i) An angle denoted by sec

= sec

x = cos

F I
H K

1
x

x given by the relation


x 1 is also accepted

as a definition of inverse trigonometrical secant


function of an angle .

g
L I F , OP
(iii) sec asec f = for M0 ,
N 2K H 2 Q
(iv) sec e sec x j = x for x 1
(ii) sec

(ii) cosec

Functions

sin

cos
tan

cot

g b g
LM , OP k0p FG i.e; LM , 0I F 0 , OPIJ .
H N 2 K H 2 QK
N 2 2Q

x
x

a xf = cosec x x = cosec x and


L I F O
M , 0 0 , P , x b , 1g b1, g
N 2 K H 2Q
1

Note:

(i) An angle denoted by cosec x given by the


relation = cosec

x = sin

F 1I ,
H xK

x 1 is also

accepted as a definition of inverse of cosecant


function of

or 0 < <
(we adopt)
sec

lx

< x 1;

1 x <
cosec

lx

< x 1 ;

RS
T

0 ,

RS
T

, 0
2
2

UV
W

UV
W

1 x

In the notational form, the inverse of the


trigonometrical secant function is defined as
1

r
1 x 1r
1 x 1

(some writers)

domain is , 1 1, and whose range is

= f

Range( may be a real


number or an angle)

RS UV
2W
T 2
m 0 r
mx < x < r RST 2 < < 2 UVW
mx < x < r RST 2 < < 2 , 0UVW

x = x for x 1

Domain

mx
mx

FG , IJ l0q , is called the principal value of


H 2 2K

cosec x . Hence, = cosec x is an angle


representing an inverse circular function whose

Remember:
1. The restricted domains and the ranges over which
each respective inverse circular function (defined
earlier) can be summarised in the chart.

6. We define cosec x as an angle measured

to
excluding = 0 whose cosecant
from
2
2
is x. The angle belonging to the interval

acosec f = for LMN 2 , 0IK FH 0 , 2 OPQ

(iii) cosec cosec

x is not defined for x 1, 1

2. The principal value of sin


1

tan x and cot


of the angle.

x ,

cosec

x,

x is the smallest numerical value


1

3. The principal value of cos x and sec x is the


smallest positive value of the angle.
4. A circular function of an angle = a number.
5. An inverse circular function of a number = an angle.
6. General value of sin

a f

x = n + 1 whose

stands for the principal value of sin

x.

Differentiation of Inverse Trigonometric Functions


1

x = 2 n whose
1
stands for the principal value of cos x .

7. General value of cos

Important Notes:
1. Unless otherwise stated by the value of each
inverse circular function, we mean its principal value.
2. If y = f x , then the derivative of x with respect
to y is given by the formula:

af

dx
=
dy

F dy I , where f is assumed to be one-one


H dx K dy

and onto as well as

F
H

a f IK
continuous, and x = f a y f = g a yf be the inverse
function, then if f a x f be finite and non zero,
1
g a yf =
f a x f [ E.G. Philips ( p q 4 ) ]
(1)
Proof: y = f a x f
(2)
x = f a yf
1. y + y = f b x + x g
(3)
2. x + x = f b y + yg
(4)
1

Also, as x 0 , y 0 and as y 0 ,

af

x 0 (Q x = f 1 y is continuous)
Now, using the definition of derivative of a function,
we have,

b yg = g b y g
f b y + yg f b y g
lim

y0

y0

a y + yf f a yf
a y + yf y
1

[adding and subtracting y in denominator ]

a x + xf x
f a x + xf f a xf
[Q y + y = f b x + x g from (3) and x + x
= f b y + y g from (4)]
= lim

y0

x
f x + x f x

= lim

x 0

when y 0 ]

=
lim

x 0

is assumed to be finite and

dx
dy
= f x 0 or, alternatively,
non-zero i.e ;
dx
let y = f (x) be a single valued monotonic and

= lim

x = n + where
1
stands for the principal value of tan x .
N.B.:
1. The principal value of each circular function is
obtained by putting n = o in its general value.
8. General value of tan

427

af

[since x 0

1
f x+x f x
x

FG a
H

f a f IJ [Taking reciprocal]
K

1
f x

af

af
dy
Then f a x f =
dx

af af
dx
g a yf =
dy

1
y =g y
N.B.: If y = f x and x = f

af

1
f x

af

g y =

dx
=
dy

and

F dy I
H dx K

which provides us a formula to

be very useful in the differentiation of inverse


functions.
Differential coefficients of inverse circular functions

af

af

If y = f 1 x x = f y , then y is called an
inverse function of x and x is called the direct function
of y.
Question: What is the rule for the derivative of the
inverse function ?
Answer: The rule for the derivative of the inverse
function is the reciprocal of the derivative of the
original function (i.e. direct function) expressed as

428

How to Learn Calculus of One Variable

bg

bg

bg

1
d
d
d

y =
y
x =1
d
dy
dx
dx
(x)
dy

af

Step 7. Lastly find

af

af

d
d
y =1
x
dx
dy

The rule to find d.c of inverse functions by - method


consists of following steps.
Step 1. Change inverse functions into direct functions
which means y = f 1 x should be expressed as

af

x = f y , where f 1 stands for sin , cos , tan ,


1

2 sin

F yI
H2K

af

except for tan

and cot

x . In

order to find its limit as y 0 , we can use any one


of the following methods.

cot , sec , cosec or any other inverse function


and f stands for sin, cos, tan, cot, sec, cosec or any
other direct function.
Step 2. Add x to x and y to y whereever these
are present in the direct function which means forming

using the rule

d
d
y
x = 1 which means taking the
dx
dx
d
d
x to find
y .
reciprocal of
dy
dx
Notes: 1. In the process of finding the d.c of inverse
trigonometrical functions, we generally get

af

Recapitulation of working rule to find d.c. of inverse


functions by - method

af

af

af

d
y
dx

2 sin

(i)

F yI 2 F yI
H 2 K = H 2 K = 1 y2 = 1

(Q when is small, sin = )

FG sin r IJ = lim FG r sin r IJ


H K
H r K
Step 3. Find x by subtracting the first value x
= f ( y ) from the second value x + x = f b y + yg
F sin r IJ = r 1 = r , r being a constant.
= r lim G
which means forming the equation x
H r K
= f b y + yg f b yg .
F yI
F yI
Step 4. Divide x = f b y + y g f a yf by y
2 sin
2 sin
H
H2K
2 K
which means forming the equation
= lim
(iii) lim
y
F yI
2
f b y + yg f a y f
x
H2K
=
.
y
y
F yI
sin
Step 5. Take the limit as y 0 on both sides of
H 2 K =1
= lim

F yI
x f b y + y g f b yg
=
H2K
the equation
which means
y
y
b

the equation x + x = f y + y .

(ii) lim
0

y 0

y0

y 0

forming the equation

bg

g bg

f y+ y f y
d
.
y = lim
y0
y
dx
Step 6. Express the direct function ( like sin, cos, tan,
cot, sec, cosec or any other direct function ) of y in
terms of x using the relation x = f ( y).

2. x 0 y 0
3. Finding d.c. by - method means finding d.c. of
the given function by making no use of rules of
differentiation and d.c. of standard functions but one
can use only fundamental theorems on limits and
standard results on limits.

Differentiation of Inverse Trigonometric Functions

Practical methods of finding d.c. of inverse circular


functions
In practice, of the question says simply to find d.c. of
an inverse circular function, one can use the following
methods.
Method (A): It consists of following steps.
(i) Change the given inverse circular function in to
direct circular function ( called commonly circular

af

function), i.e. of y = f 1 x is given, it should be


changed into x = f ( y).
(ii) While differentiating, we must choose the variable
y as an independent variable (instead of usual variable
x as independent variable) w.r.t. which the direct
circular function should be differentiated, i.e.,
d
d
x= f y
x =
f y = f y
dy
dy
d
y which is
(iii) Now required derivative is
dx
d
x ,
obtained by simply taking the reciprocal of
dy

af

af

c a fh

af
af

af

af

d
1
d
1
( y) =

x
i.e;
d
d
dx
dy
x
y
dy
dx

af

af

Method B: It may be explained in the following way.

b xg x = f b yg
x = f f b yg Q y = f a xf
(1)
x = f a yf y = f a x f y = f f a yf

Let y = f

or,

d a fi = F d 1 I
GH dy a xfJK
d
1

a
yf =
dx
FG d a xfIJ dQ f a x f = yi
H dy K

d
f 1 x
dx

1
1
=
f y
f f 1 x

af

af

Hence, whenever, we have the form y = f f y ,


it can be differentiated by using the chain rule, i.e,
1
d
d
y =
f
f y
dy
dy
1
d
d
=
f f y
f y
d f y
dy

af

a fe

a fj
a fj b a fg

(using chain rule)

1=

d a fi a f
FG Q d a yf = 1, f a yf = xIJ
H dy
K

d
d
f 1 x
x
dx
dy

.
d a fi

Standard formulas or d.c of inverse circular functions



1
, show that
1. If y = sin x , y ,
2 2
dy
1
=
dx
1 x 2

FG
H

IJ
K

Proof : First method: (Derivation of d.c. Using the


definition)

c x 1h
I
F
x = sin y , H y K
...(1)
2
2
Step 2. x + x = sin a y + y f
(2)
Step 3. x = sin a y + y f sin y (subtracting (1)
Step 1. Let y = sin

from (2))

FG
H

= 2 cos y +

(2)
1

429

1
=
x

Step 4:

IJ
K

x ,

FG IJ
H K

y
y
sin
2
2

1
y
y
sin
2 cos y +
2
2

y
=
x

FG
H

IJ
K

FG IJ
H K

(3)

y
y
y
sin
2 cos y +
2
2

F
H

I
K

F I
H K

(Multiplying both sides of (3) by y )

F yI
yI
H2K
F
cos y +
y
H 2 K sin
2

(4)

430

How to Learn Calculus of One Variable

Step 5:

y
1
1
=
=
x cos y 1 cos y

lim

x 0

(Taking the

limits on both sides of (4) as x 0 )

F
I
Step 6: Q cos y = cos y GHQ f a x f = f a x f JK
aQ sin y = x f
= 1 sin y = 1 x
F I
Now cos y is positive for y G , J
H 2 2K
F I
cos y = cos y for y G , J
H 2 2K
e Q f b x g = f b xg for f b x g 0j
2

dy
=
dx

1
1 x 2

FG Q
H

IJ
K

lim

x 0

af h

cos y = cos y = 1 x

...(3)

... (4)

Putting (4) in (1), we have

dy
=
dx

1
1 x 2

, for x < 1

d sin x
=
dx

1
1 x

for x < 1

Cor 1.: On replacing x by f (x) in the L.H.S. and R.H.S.


of the above formula, we get
1

a f=

f x

1
1 f

af

f x

af

. f x

a xf

af

d sin | f x |
=
Cor 2.:
dx

FG IJ
H K

1
1 f

af

a xf

af

d| f x |
dx

= f2 x

b g

2. If y = cos 1 x , y 0 , , show that

1. y = sin x

x = sin y for

dy
1
=
.
dx
1 x 2

y
2
2
1

(from the definition of sin x )

dx
= cos y (differentiating both sides w.r.t. y)
dy

dy
=
dx

FG IJ
H K

af

= f x for f x > 0

Equating (2) and (3), we have

dx

1
dx
dy

1
dx
dy

IJ
K


,
cos y > 0 cos y = cos y
2 2

cQ f a x f

d sin

y dy
=
, x < 1.
x dx
Second method using the formula:
dy
=
dx

FG
H

and y

1
cos y

...(1)

b x 1g
x = cos y , b0 y g
Step 2.: x + x = cos a y + y f
Step 3.: x = cos a y + y f cos y
Step 1.: Let y = cos

(1) from (2))

Now, cos y = cos 2 y


= 1 sin 2 y = 1 x 2

Proof: First method (Derivation of D.C using the


definition)

...(2)

FG
H

= 2 sin y +

x,

IJ
K

FG IJ
H K

y
y
, sin
2
2

...(1)
...(2)
(subtracting

431

Differentiation of Inverse Trigonometric Functions

=
x

1
y
y
sin
2 sin y +
2
2

F
H

y
Step 4.:
=
x

I
K

F I
H K

...(3)

Using the formula:

y
y
y
2 sin y +
. sin
2
2

F
H

I
K

F I
H K

Step 5.:

F yI
H2K
yI
F
sin y +
H 2 K . sin F y I
H2K

... (4)

y
1
=
x
sin y + 0 .1

a f

lim

x 0

FH

af

f x

a f IK

f2 x

= 1 cos y

b g

dx
= sin y (differentiating both sides w.r.t. y)
dy

dy
=
dx

aQ cos y = x f

1 x

...(1)

sin y = 1 cos

2y

= 1 x

bQ f a xf

af

af g

= f x for f x > 0

sin y = sin y = 1 x

...(a)

lim

x0

...(3)

... (4)

Putting (4) in (1), we have

Equating (a) and (b), we have

FGQ
H

1
sin y

b g

b g
bQ f a xf = f a xf for f a xf > 0g

FG IJ
H K

...(2)
and y 0, sin y > 0 for y being measured from
0 to sin y = sin y

dy
1
=
, for x < 1
dx 1 x 2

sin y = sin y for y 0,

1
dx
dy

Equating (2) and (3), we have

Now sin y > 0 in 0,

dy
=
dx

F dy I
H dx K

x)

Now, sin y =

= 1 x 2

(Taking the

1
=
sin y
sin y Q

2. y = cos x
x = cos y for 0 y (from the definition of
cos

limit of both sides of (4) as x 0 )

Step 6.: sin y =

dy
=
dx

(multiplying both sides of (3) by y )


=

3. The domains of the derived functions of sin


1
and cos x are the open interval ( 1, 1).
Second method

IJ
K

...(b)

y dy
=
x dx , x < 1

Remark:
1
1
1. The derivatives of sin x and cos x become
undefined at x = 1 . For this reason we leave these
points out of our consideration.
2. The derivatives of the inverse trigonometric
functions are not transcendental but algebraic.

d cos x
=
dx

Note: (i) sin

1
1 x

x + cos

x=

, for x < 1

, x 1
2

d
1
1
d
sin x +
cos x = 0
dx
dx

d
d
sin 1 x =
cos 1 x
dx
dx

432

How to Learn Calculus of One Variable

1 x

; x < 1.

Step 5.:

af
f b xg
=

d
1
sin f x
dx

(ii)

d cos 1
=
dx

(iii)

af

1
d
cos f x
dx

3. If y = tan

= lim

x 0

1
1 f

bg

af

dy
dx

b x R g
F < y < I ...(1)
x = tan y , H
2
2K
Step 2.: x + x = tan b y + yg
...(2)
Step 3.: x = tan b y + y g tan y (subtracting (1)
1

from (2))

a f
a f
sin a y + y f cos y sin y cos a y + y f
=
cos a y + y f cos y
sin a y + y y f
sin y
=
=
cos y cos a y + y f cos y cos a y + y f
cos y a y + y f
1

=
sin y + y
sin y

cos y + y
cos y

1 + tan y

aQ tan y = x f

1
1+ x 2

dy
1
=
.
dx
1 + x2

sin y

dy
=
dx

...(a)

1
dx
dy

FG IJ
H K

1
3. y = tan x

x = tan y for

<y<
2
2

(from the definition of tan

x ).

dy
2
= sec y
dx
(differentiating both sides of (1) w.r.t. y)

dy
=
dx

1
dx
dy

FG IJ
H K
aQ x = tan y f

1 + tan y

sec y

1+ x

Cor 1.: On replacing x by f (x) in the L.H.S. and R.H.S


of the above formula, we get

d tan
...(3)

y
y
= cos y cos y + y
(Multiplyx
sin y
ing both sides of (3) by y )
...(4)
Step 4.:

Second method (using the formula):

Proof: First method: (Derivation of d.c using the


definition)

sec y

1
=
.
1 + x2

Step 1.: y = tan

= cos y =

d| f x |
dx

a xf
F I , show that
x, y ,
H 2 2K
1 f

RScos y cos a y + y f y UV
sin y W
T

(on taking the limit of both sides of (4)) as x 0

d f x

dx
x

bg

y
x

lim

x 0

af

f x

dx

Cor 2.:

d tan

dx

1
1+ f

af

f x

a xf

af

f x

1
1+ f

a xf

af

d f x
dx

Differentiation of Inverse Trigonometric Functions

4. If y = cot

b g

x , y 0, , show that

dy
1
=
.
dx
1+ x 2
Proof: First method (Derivation of d.c using
definition)
Step 1.: Let y = cot

x , x R

b g
x + x = cot a y + y f
x = cot a y + y f cot y

x = cot y , y 0,
Step 2.:
Step 3.:
from (2))

a
a

f
f

...(1)

sin y sin y + y
1
=
x
sin y

Step 5.:

lim

x 0

= lim

x 0

= sin y sin y = sin y


2

cosec y

...(1)

dy
=
dx

1
2

1 + cot y

aQ cot y = x f

1
1+ x

...(a)

(ii)

FG dx IJ
H dy K

1
cosec 2 y

Note: (i) tan

RS sin y sin a y + yf y UV
sin y W
T
=

dx
2
= cosec y (differentiating both sides of
dy

...(4)

y
x

(1) w.r.t. y)

(On taking the limit of both sides of (4) as x 0 )

FG IJ
H K

of cot 1 x )

...(3)

y
y
= sin y sin y + y
x
sin y
(Multiplying both sides of (3) by y )

Step 4.:

1
dx
dy

x = cot y for 0 < y < (from the definition

sin y y y
sin y
=
=
sin y sin y + y
sin y sin y + y

dy
=
dx

(subtracting (1)

dy
1
because dx =
is defined for all real values
1+ x 2
of x.

5. y = cot

tan 1 x and cot 1 x are the open interval ,

Second method using the formula:

sin y cos y + y cos y sin y + y


=
sin y + y sin y

dy
1
=
dx
1 + x2

Note: The domains of the derived functions of

...(2)

cos y + y
cos y

sin y + y
sin y

433

1
1 + cot 2 y

x + cot

x=

1
1+ x 2

aQ x = cot y f

, x
2

d
d
1
1
tan x +
cot x = 0
dx
dx

1
1
d
d
1
tan x =
cot x =
2
dx
dx
1+ x

af

1
d
tan f x
dx

af

af

d f x
1
d
1
cot f x =

2
dx
dx
1+ f x

af

434

(iii)

How to Learn Calculus of One Variable

d cot

af=

f x

dx

6. If y = sec
dy
=
dx
x

1 x 2

af

d f x

a xf dx
R U
y b0 , g S V , show that
T2 W

x,

1+ f

Step 5.: lim

x 0

of both sides of (4) as x 0 )


1
=
sec y tan y
Step 6.: sec y tan y = sec y . tan y

F
H

x , x >1

x = sec y , 0 < y < , y

Step 2.: x + x = sec y + y

I
K

...(1)

...(2)

a
a

f
f

cos y cos y + y
1
1

=
cos y + y
cos y
cos y cos y + y

F
H

I
K
a

F I
H K
f
cos y cos a y + y f
1

=
x
F y I sin F y I ... (3)
2 sin y +
H 2K H2K
y cos y cos a y + y f
y

=
Step 4.:
yI
x
F
F yI
2 sin y +
H 2 K sin H 2 K
y
y
sin
2
2
cos y cos y + y

2 sin y +

( Multiplying both sies of (3) by y )

cos y cos y + y
.
y
sin y +
2

F
H

I
K

1
y
2
sin
y
2

F
H
F
H

I
K
I
K

IJ FQ f a xf
KH
I
y 1J
K

tan y

sec

a f IK

= f2 x

= x x 2 1

Step 3.: x = sec y + y sec y (subtracting (1)


from (2))
=

FG
H
F
sec y G
H

= sec y

Proof: First method (derivation of the d.c. using the


definition)
Step 1.: Let y = sec

y cos y cos y
=
(Taking the limits
x
sin y . 1

...(4)

aQ sec y = xf
RU
Now, y b0 , g S V
T2 W
F I FG , IJ
y 0,
H 2K H 2 K
F I or y FG , IJ
y 0,
H 2K H 2 K

...(a)

Again,

FG IJ sec y and tan y both are positive


H 2K

y 0,

The product sec y . tan y is positive


and y

...(b)

F , I sec y and tan y both are negative


H2 K

...(c)
the product sec y tan y is positive
Hence, from (b) and (c), we observe that the
product of sec y and tan y is positive in the first
quadrant and in the second quadrant for

b g RST 2 UVW the product sec y tan y is


R U
positive for y b0 , g S V . Which means that
T2 W
R U
sec y tan y = sec y tan y for y b0 , g S V
T2 W
y 0,

435

Differentiation of Inverse Trigonometric Functions

af

af

af

= f x for f x > 0 )
Equating (a) and (d), we have
(Q f x

...(d)

sec y tan y = sec y tan y = x . x 2 1


dy
dx =
x

1
x 1
2

FGQ
H

lim

x 0

IJ
K

y dy
=
,
x dx

for x > 1.

FG Q tan y
H

= x

tan y

= x

sec x 1

tan y

IJ
K

= x x 2 1
Equating (3) and (4), we have

...(4)

sec y tan y = sec y tan y = x

x2 1

Second method using the formula:

dy
=
dx

FG IJ
H K

7. y = sec

x = sec y for 0 y , y
(from the
2

x)

of (1) w.r.t. y)
Now,

x2 1

and tan y and sec y both are negative for


2

< y<
2
the product sec y . tan y is always positive for,

and sec x tan y = sec x tan y

=
=

F 1I ,
H xK

x 1

FI
HK

1
1 2
x

FG 1 IJ ,
H xK

d
dx

x >1

F 1I
dx 1 x i H x K , x
1
F 1 I , x >1

Hx K
I
x 1
J
x JK
1

F
GG
H

, x 1

>1

x
x 1
1
2

cxh

(Q x = x

and x

=x )

, x >1
x x 2 1
Cor: On replacing x by f (x) in the L.H.S and R.H.S of
the above formula, we get

Hence, sec y tan y = sec y tan y

bQ f a xf = f a xfg
(for f a x f > 0 )

x = cos

tan y and sec y both are positive for

, x > 1.

1
1 1
dy d
d
=
sec x =
cos
dx dx
dx
x

...(2)

L I F , OP
y M 0,
N 2K H 2 Q

0 < y < , y

y = sec

...(1)

dx
= sec y tan y (differentiating both sides
dy

0< y<

FG IJ
H K

Third method

definition of sec

dy
1
=
dx
dx
dy

1
dx
dy

...(3)

d sec

dx

af

f x

af

f x

1
f

a xf 1

af

f x

436

How to Learn Calculus of One Variable

8. If y = cosec
that

dy
=
dx
x

F , I k0p , show
H 2 2K
, c x > 1h .
1

x , y

1
x2

x = cosec y ,

x ,

c x > 1h

= x x 2 1
Now,

F
H

...(2)

1
1

sin y + y
sin y

f
sin y sin a y + y f
=
sin a y + y f sin y
F yI
2 cos a y + y f sin
H2K
=
sin a y + y f sin y
sin a y + y f sin y
1

=
y
F y I y ...(3)
2 cos a y + y f sin
H2K
sin y sin a y + y f
1
=

...(4)

y
F
I
F yI
cos y +
H 2 K sin H 2 K
F yI
H2K

1
cosec y cot y

Step 5.: cosec y cot y = cosec y cot y

aQ cosec y = xf

...(a)


,
0
2 2

cosec y and cot y both are negative


The product cosec y . cot y is positive

F
H

I
K

...(b)
2
cosec y and cot y both are positive
the product cosec y cot y is positive
From (b) and (c), we observe that the product of
y 0,

and

F
H

I kp
K


,
0
2 2
The product cosec y cot y is positive for

cosec y and cot y is positive for y

F
H

I kp
K


,
0 which means that
2 2
cosec y cot y = cosec y cot y

af

af

af

(Q f x = f x for f x > 0
Equating (a) and (d), we have

...(d)

cosec y cot y = cosec y cot y

sin y sin y
y
=
Step 4.: lim
x0 x
cos y 1 (Taking the
limits of both sides of (4) as x 0 )

a f IK

f2 x

I kp
K
F I F I
y , 0 0,
H 2 K H 2K
F I
F I
y , 0 or y 0 ,
H 2 K
H 2K
F I
Again, y G , 0J
H 2 K
y

...(1)

Step 3.: x = cosec ( y + y ) cosec y (subtracting


(1) from (2) )

< y< ,y0


2
2

Step 2.: x + x = cosec y + y

cot y

= cosec y cosec y 1

Proof: First method (derivation of d.c. using the


definition)
Step 1.: Let y = cosec

FH Q f a xf

= cosec y

= x x 2 1
y dy
=
, so (5) becomes equal
x dx

and since, lim

x 0

...(5)
to

dy
=
dx
x

1
x2 1

, x >1

437

Differentiation of Inverse Trigonometric Functions

Second method using the formula:

1
dy
1
=
=
, x > 1.
dx
dx
x . x2 1
dy

dy
1
=
dx
dx
dy
9. y = cosec

Third method
x

x = cosec y , for

FG 1 IJ , x 1
H xK
dy
d
d F
1I

=
cosec x =
sin
,
dx dx
dx H
xK
dy
d F 1I
1

=
.
, x >1
dx
dx H x K
1
1

10. y = cosec 1 x = sin 1

y ,y0
2
2

... (1)

dx

= cosec y cot y
dy

(differentiating (1) w.r.t. y)

LM
N

...(2)

OP k p
Q
L I F O
y M , 0 0, P
N 2 K H 2Q

Now, y


,
0
2 2

x2

x 1 x
2

F
H

I,
K

1
x2

x 1

x >1

cosec y and cot y both are negative for

< y < 0 and cosec y and cot y both are positive


2

for 0 < y <


2
the product cosec y cot y is always positive

af

af

FGQ tan y
H

x 1

1
x

x2 1

eQ x

FHQ

IJ
K

cosec y cot y = cosec y cot y

IK

, x 1 or x 1
2

d
1
1
d
sec x +
cosec x = 0
dx
dx

d sec x
d cosec x
=
=
dx
dx

= x x2 1
Equating (3) and (4), we have

= x2

, x >1

tan y

x2 = x

Note: (i) sec x + cosec x =

= x cosec y 1

= x x 2 1
(2) becomes equal to

I
JJ
K

...(3)

and cosec y cot y = cosec y cot y


2

= f x for f x > 0 )

= x cot y

x 1

Hence, cosec y cot y = cosec y cot y

af

F
GG
H

< y< ,y0


for
2
2
(Q f x

...(4)

af

1
2

x 1

af

d
d
1
1
sec f x =
cosec f x
dx
dx

(ii)
=

af

f x

1
f

axf 1

af

d
f x
dx

438

How to Learn Calculus of One Variable

An important remark:
The domain of the derived function of sec

x and

x 1

cosec x are defined for x > 1. According to the


convention we have adopted
d sec x
=
dx

(i)

d cosec
dx

(ii)

x 1

x and / coses

x 1

1
2

x 1

c x > 1h

x 1
2

and

d cosec
dx

1
2

x 1

d cosec

dx

1
2

x 1

x
1

=
x

x 1

af

x2 1

1
1
1 2
x

d
dx

x
x 1
2

x 1
2

d
x
dx

b ge

a xf 1
f a xf

x2 1

FG 1 IJ
HxK

1
x

d x
dx

j.

dx

eQ x

x 2 1 . x

x
x

x
x

= x2

af

a xf 1

=
2

x 1

FG 1 IJ
HxK

F1I Qx
GH x JK e

, for x > 1

1
1
d
d
(i) dx sec x = dx cos

f x

d 1
dx x

1
1 2
x

, for x < 1

Remember:

c x > 1h

1
x

af

1
1
d
d
(ii) dx cosec x = dx sin

, for x > 1

,for x < 1
x

1
d
sec f x =
dx

1
x2 1

x2 1

f x

j dxd

Cor:

x because

are inaccurate unless

d sec x

=
dx

x 1

d sec x
=
dx

we consider the restriction x > 1 against the derived


functions of inverse circular functions namely
sec

+ x

a fe

x
x

= x2

af

d
f x
dx

Differentiation of Inverse Trigonometric Functions

Cor:

d cosec
dx

af

af

f x =

f x

af

a xf 1
f a xf

f x

a xf 1

af

d
f x
dx

Let y = f
dy

=
dx

af

2.

FG IJ
H K

af

as f = x is put in the last stage to get an algebraic


expression in x.
(ii) f = d.c. of f = prime of f
(iii) f = sin / cos / tan / cot / sec / cosec
(iv) f = d.c. of sin / cos / tan / cot / sec / cosec
2

d tan x
1
=
1
2
dx
sec tan x

4.

1 sin

sec

1
2

1 + tan

1
2

1 + cot

1 x

1+ x

1
1+ x

1
1
=
sec tan
sec tan

1
=
sec tan
1

x 1

1
x

sec 1

; x >1

d cosec x
dx
=

; x <1

Q0<<

6.

d sin 1 x
1
1
1
=
=
=
1.
1
cos
cos
dx

cos sin x

d sec x
1
=
5.

1
1
dx
sec sec x tan sec x

of the direct trigonometric function (of the given t 1


function ) whose operand is the given function which
is put equal to to get an algebric expression in x.
Thus:

; x <1

af

OP
Q

d cot 1 x
1
=
2
dx
cosec x cot 1 x

cot whose operand is f 1 x = given inverse


trigonometric function. Thus the d.c. of inverse
trigonometric function = reciprocal of the derivative

2
2

1 x

3.

= cos / sin / sec / cosec / sec tan / cosec

LM
N

1 cos

1
1
=
=
, where
1
f y
f f
x

af

b xg x = f b yg dxdy = f b yg

1
dx
dy

Q0

(i) f 1 x = is required to put and simplify as well

af

d cos x
1
1
1
=
=
=

1
dx
sin
sin
sin cos x

dx

3. The differential coefficients of those trigonometric


and inverse trigonometric function which begin with
c are negative.
4. Method of remembering the d.c. of standard
inverse circular functions

439

1
1

cosec cosec x cot (cosec x )

1
cosec cot

440

How to Learn Calculus of One Variable

af

LMQ OP
2Q
N 2

1
cosec cot

1
=
cosec cot

4. On replacing f (x) by f x in the L.H.S and R.H.S


and retaining | f (x) | where it is in the above formulas,
we get

1
2

cosec 1

(i)

; x >1

x 1

Note: 1. Insertion and removal of mod operator is


performed by considering the principal values of given
inverse circular function.
2. x < 1 means domains of derivatives of inverse
sine and cosine function are the same namely x < 1
whereas x > 1 means domains if derivatives of
inverse secant and cosecant function are the same
namely x > 1
3. On replacing x by f (x) in the L.H.S and R.H.S of the
above furmulas, we have

d sin
(i)

d cos

(iv)

bg

d tan

f x

a f=

f x

dx

a f=

f x

dx

1 f
1
1+ f

1
1+ f

af

d cosec
dx

a xf =

af

f x

bg

f x

bxg

f x
1

(vi)

a f=

1 f

f x

dx
d cot

dx

d sec
(v)

f x

dx

(ii)

(iii)

bg

b xg

bg

a xf
a xf

f x

a xf 1

1
f

f x

dx

a xf
a xfj

1 f

eQ f a xf
d cos f a x f

= f2

dx

d tan

(iii)

(iv)

af

f x

f x

d sec

dx

1+ f

af=

f x

dx
d cosec

af

af

f x

dx

a xf

d f x

f x

af

af

a xf

dx

af

dx

a xf 1

af

dx

d f x

f x

af

d f x

1+ f

dx

a xf

af

1 f

dx
d cot

af

d f x

af

d f x

a xf 1

dx

af

d f x
dx

af

5. a 2 T 2 can be written equal to a T , if


it is pre-assumed that a is positive and the angle
of any trigonometric function sin, cos, tan, cot, sec or
cosec is an acute angle.

af

(ii)

(vi)

af

f x

af

(v)

f x

d sin

af

f x

a xf 1

N.B.: T in T ( ) means the operator sin, cos,


tan, cot, sec or cosec indicating trigonometric
functions.

af

Type 1.: y = t 1 f x
Where t

af

f x

Which are known as derivatives of inverse


trigonometric function of a function of x formulas.

= sin / cos / tan / cot / sec / cosec

a f a f f a xf + f a xf
f a xf + f a xf
f a x f + f a xf
f a xf f a xf f a xf f a xf

af

f x =
1

f1 x + f 2 x
3

441

Differentiation of Inverse Trigonometric Functions

af af af

af
bg bg

Where f1 x , f2 x , f3 x and f4 x are trigonometric functions of x / algebraic expressions in x /


one or two of the four functions f 1 x , f 2 x ,
f 3 x and f 4 x may be constant.

bg

bg

Working rule:
1. Change the inverse trigonometric function into
direct function

y=t

af

af

af

f x t y = f x

af

af

dt y
df x
=
dx
dx

a sin xfa25 + 15 cos x 9 15 cos xf


a5 + 3 cos xf

dy
=
dx

eQ

2. Differentiate both sides w.r.t. x, i.e

dy
dx

sin y

16

b5 + 3 cos xg
a xf = f a xf j

f2

siny = sin y

a f dy = f a x f
dx

af

dy f x
=
dx

FG 3 + 5 cos x IJ
H 5 + 3 cos x K

= 1

FG d t a yfIJ
H dy K

4 sin x
5 + 3 cos x

Which gives for x n

af

the type : y = t 1 f x which can be used to


differentiate any type of inverse circular function
provided given function can be put in the form

af af

y = t 1 f x t y = f x .
Examples worked out on type (1) find the d.c. of the
following

FG 3 + 5 cos x IJ
H 5 + 3 cos x K
F 3 + 5 cos x IJ
Solution: y = cos G
H 5 + 3 cos x K
F 3 + 5 cos x IJ
cos y = G
H 5 + 3 cos x K
1. y = cos

aQ 0 y sin y 0f

3. Express the d.c. of t (y) in terms of f (x) and simplify.


Note: The above method is known as general method
to differentiate an inverse trigonometric function of

af

...(iii)

Now, from (ii),

dt y
dy

sin x
, sin y 0 .
sin y

...(iv)

4 sin x

dy
=
dx
=

...(i)

...(ii)

a5 + 3 cos xf a 5 sin xf a3 + 5cos xfa 3 sin xf


=
a5 + 3 cos xf
2

sin x

4 sin x

From (iii) and (iv), we get for x n

2. y = sin

Now differentiating both sides w.r.t. x

dy
sin y
dx

a5 + 3 cos xf

a5 + 3 cos xf

sin x

4 sin x
5 + 3 cos x sin x

Note: A direct method has been mentioned later.

sin x
=
sin y

Solution:

F 2x I
GH 1 + x JK
F 2x I
y = sin G
H 1 + x JK

sin y =

2x
1+x

...(1)

1+ x
d sin y

=
dx

j 2 2 x a2 x f
e1 + x j

2 2

442

How to Learn Calculus of One Variable

Which gives

2 1 + x2 4 x2

1 + x2

2
2 1 x
2 2x
dy
cos y
=
=
2 2
2 2
dx
1+ x
1+ x

e j
LMQ d sin y = d sin y dy OP
dy
dx Q
N dx
dy 2 e1 x j
1

; cos y 0
dx
e1 + x j cos y

...(ii)

Now, cos y = cos y = 1 sin y

= 1

d1 + x i

2 2

2 2

1+ x + 2x 4x
2

2
dy
=
dx
1 x

dy
2
=
dx
1 x

d1 + x i
2

b1 xg + b1 + xg
=
b1 xg
2

=
=

2
2

a xf = f a xf IK

...(ii)

sec y

1 + x 4 2x 2

d1 x i
d1 + x i
d1 x i F
d1 + x i HQ

= 1+ x

f a 1f

Now, sec y = 1 + tan y = 1 +

FG 1 + x IJ
H1 x K

[ from (1) ]

1 + x2 2x + 1 + x2 + 2x

2 2

d1 + x i
Q e1 + x j is always + ve e1 + x j

a1 xf + a1 + xf
a1 xf

sec y

...(i)

1 x 1 1+ x
d tan y
=
2
dx
1 x

2 2

1+ x
1 x

[from (1)]

e1 + x j 4 x
e1 + x j

tan y =

FQ y cos y 0I
H 2
K
2
4 x2

i d i
d i d i
2 e1 x j
, x 1.
=
e1 x j e1 + x j
F 1 + x IJ
3. y = tan G
H1 xK
F 1 + x IJ , x 1
Solution: y = tan G
H1 xK
2 1 x 2
1 + x2
dy
=

2
dx
1 x2
1 + x2

2 + 2x2

b1 xg
2 d1 + x i
=
b1 xg

b1 xg

...(iii)

443

Differentiation of Inverse Trigonometric Functions

Which gives

b1 xg = 1 ; x 1
b g 2 e1 + x j e1 + x j
4. y = sin a cos x f
Solution: y = sin a cos x f
dy
2
=
dx
1 x

cos y

sin y = cos x

d sin y d cos x
=
dx
dx

y cos y 0
2
2

cos y = cos y

dy
=
dx

cos y

sin x
1 cos2 x

; x n

sin x
, x n, n z
=
2
sin x
sin x

asin xf

Solution: y = sin

cos x
1

asin xf

sin y = sin x
d sin y d sin x
=
dx
dx

tan y =

cos x
cos x

; x n +

FG cos x sin x IJ
H cos x + sin x K

First method:

sin x

,y

1 sin 2 y

5. y = sin

sin x

Solution: y = tan

sin x

cos x

6. y = tan

I
K

Q sin y = sin x

1 sin x

sin x
=
cos y

= cos y

cos x

dy sin x

=
y
dx
cos y for
2

cos x
cos y

F y cos y 0 cos
H 2
2

dy
= sin x
cos y
dx

Fy I
H 2K

dy cos x
=
dx cos y
=

dy
= cos x
dx

FG cos x sin x IJ
H cos x + sin x K

cos x sin x
cos x + sin x

...(i)

d tan y
dx

bcos x + sin xgbsin x cos xg bcos x sin xgbsin x + cos xg


bcos x + sin xg
2

Nr =

LMecos x + sin x j + acos x sin x f OP


N
Q
2

= 2 cos x + sin x = 2
2

sec y

2
dy
=
dx
sin x + cos x

dy
2
=
dx
sin x + cos x

1
2

sec y

...(ii)

444

How to Learn Calculus of One Variable

Now, from (i),


2

RS cos x sin x UV
T cos x + sin x W
acos x + sin xf + acos x sin xf
=
acos x + sin xf

sec y

sec y = 1 + tan y = 1 +
2

sec y

=
=

acos x + sin xf

acos x + sin x f

Which gives

dy
2
=
dx
sin x + cos x

...(iii)

acos x + sin xf
2

= 1 Ans.

F
H

...(ii)

F cos x IJ
Now, sec y = 1 + tan y = 1 + G
H 1 + sin x K
a1 + sin xf + cos x
=
a1 + sin xf

1 + sin x + 2 sin + cos x


2

n + x
2
4
2

2
1 + sin x

...(iii)

Which gives

1 + sin x
1
1
dy
=

=
Ans.
1 + sin x
2
2
dx

FG cos x IJ
H 1 + sin x K

Method (2)

x
x
cos2 sin 2
cos x
2
2
=
2
1 + sin x
x
x
cos + sin
2
2

FG cos x IJ
H 1 + sin x K

tan y =

a1 + sin xf
2 a1 + sin x f
2 + 2 sin x
=
=
a1 + sin xf a1 + sin xf

I
K

Solution: Method (1)


y = tan

dy

= 1
dx
7. y = tan

LMtan F xI OP = n + x
4
N H 4 KQ

where n is such that

cos x sin x 1 tan x

= tan
x
=
cos x + sin x 1 + tan x
4
1

1
dy 1 + sin x
=
=
2
dx
1 + sin x
1 + sin x

Second method:

y = tan

dy
1
1
=

dx
1 + sin x sec 2 y

cos x + sin x + cos x + sin x

dy sin x sin y cos y


=
2
dx
1 + sin x

FG
H

cos x
1 + sin x

...(i)

d tan y

dx

a1 + sin x f dxd acos x f cos x dxd a1 + sin xf


=
a1 + sin xf
2

IJ
K

x
x
sin
2
2
=
x
x
cos + sin
2
2

FG
H

cos

y = n +

IJ
K

x
2 = tan x
=
x
4 2
1 + tan
2

4 2

1 tan

FG
H

IJ
K

445

Differentiation of Inverse Trigonometric Functions

bb + a sin xg dxd ba + b sin xg ba + b sin xg dxd bb + a sin xg


=
bb + a sin xg

dy
1
=
dx
2

8. y = tan

1
x

Solution: y = tan

1
tan y =
x
d tan y
d

=
dx
dx
2

sec y

1
x

F 1I = 1
H xK x

1
x

1
x2

dy
1
= 2
dx
x

eb a j cos x 1
=
ab + a sin xf 1 cos y
bQ sin y = sin y as 0 y g
2

d cos y
dx

FG a + b sin x IJ
H b + a sin x K

eb a j cos x
bb + a sin xg
2

| b + a sin x |

b + a sin x + 2 a b sin x a + b 2 sin 2 x + 2 a b sin x


2

eQ f a xf
=

a + b sin x
b + a sin x

eb a j cos x
=
ab + a sin xf

cos y =

1
dy b cos x a cos x
=

2
sin
y
dx

b + a sin x
2

F 1 I
GH 1 + tan y JK
F
I
G G 1 JJ LQ tan y = 1 O
x PQ
GH 1 + x1 JK MN

R|
U|
1 |
1
|V = 1
=
S
x | F x + 1I |
1+ x
|T GH x JK |W
F a + b sin x IJ
9. y = cos G
H b + a sin x K
F a + b sin x IJ
Solution: y = cos G
H b + a sin x K

ab + a sin xf b cos x aa + b sin xf a cos x


ab + a sin xf

dy
1
1

= 2
2
dx
sec y
x
=

dy
dx

sin y

LMb
N

eb
2

a fj

= f2 x

j acos xf 1
ab + a sin xf
+ e a b j sin x O
QP

b ge
eb a j eb

cos x b 2 a 2

a sin x
2

bb + a sin xg

446

How to Learn Calculus of One Variable

a fe
j
1
=
FG b a IJ FG 1 sin x IJ ab + a sin xf
H
KH
K
I
F
G b a J acos x f
K
H
=
2

cos x b a
2

b + a sin x

Note:

cos x

F a + b cos x IJ
10. y = sin G
H b + a cos x K
F a + b cos x IJ
Solution: y = sin G
H b + a cos x K
1

ea
=

b sin x

b + a cos x

dy
exists only when b 2 > a 2 .
dx

sin y =

ea b j sin x
=
ab + a cos xf

F a + b cos x IJ
1 G
H b + a cos x K
b + a cos x

eb + a cos x j aa + b cos xf
2

LMQ f a xf = f a xf
N
ea b j sin x
=
ab + a cos xf
2

af

f2 x

and

a fOQP

= f2 x

a + b cos x
b + a cos x

b + a cos x + 2 a b cos x a + b cos x + 2 a b cos x

d sin y
dx

ab + a cos xf dxd aa + b cos xf aa + b cos xf dxd ab + a cos xf


=
ab + a cos xf

ea
=

b sin x

b + a cos x

eb

j eb

a cos x

cos y

dy
dx

ab + a cos xf (b sin x) aa + b cos xf a a sin xf


=
ab + a cos xf

ea
=

j sin x

b + a cos x

eb

j e1 cos xj
2

1
dy b 2 sin x + a 2 sin x

2
cos
dx
y
b + a cos x

b sin x + a sin x

1
cos y

ab + a cos xf
FQ cos y = cos y as y I
H
2
2K
ea b j sin x 1
=
ab + a cos xf 1 sin y
2

b a

j sin x

b + a cos x
2

b a

1 cos x

b a
sin x

sin x
b + a cos x

Type 2: Method of substitution and use of chain


rule.
Form A: y = a constant t 1 f x
1

b a fg

or, y = a constant t [ a constant times f (x)]


Where, f (x) = a function of x / sin x / cos x/.... /
an expression in x etc.

Differentiation of Inverse Trigonometric Functions

Working rule:
1

* or alternatively,

b a fg

1. Put t
f x or t
differentiate w.r.t. x].

[ constant f (x) = u and

u
or, (a constant u) and differconstant
dy dy du
=

entiate it by using chain rule, i.e.


, where
dx du dx
dy
1
=
or, constant.
du constant
2. Put y =

Solved Examples:

ab
dy dy du
1
=

=
2
2
dx du dx ab a cos x + b 2 sin 2 x

dy
1
=
dx a 2 cos 2 x + b 2 sin 2 x

Form B:

where,

F
H

1 b
1
tan
tan x
ab
a

Solution: Let u = tan

tan u =

I
K

F b tan xI
Ha K

...(i)

b
tan x
a

1+

2
2

tan x

du b sec x
a
=
2
2
2
dx
a
a + b tan x

ab
du
= 2
2
dx a cos x + b 2 sin 2 x

Now again, let y =

u
ab

1
2

a cos x + b sin x

a quotient of two

1. Put

t au f
a f U|V y = a constant
1 .
du
v = f a xf |
W u = v dv = 2 v

u=

f x

Note: 1. The constant may be unity. Similarly any


one of f 1 x and f 2 x may be unity.
2. The above method may be termed as u v method
just for easiness.

af

af

Solved Examples on form (B)


Find the d.c. of the following
...(ii)

FG 1 + sin x IJ
H 1 sin x K
F 1 + sin x IJ
Solution: y = tan G
H 1 sin x K
F 1 + sin x IJ
Let u = G
H 1 sin x K
1

1. y = tan
...(iii)

ab
dy
1 du
1

=
2
2
dx ab dx ab a cos x + b 2 sin 2 x
[from (ii)]

du dv
dy
,
and
.
dv dx
du
3. Use the chain rule:
dy dy du dv
=

dx du dv dx

1
a

2. Find

du b
b sec x
1
2
= sec x
=

2
dx a
a
sec u

Working rule:

du b
2
sec u
= sec x
dx a

functions/an expression in x.

e f a xf j
f a xf f a xf
f a xf =
f a xf m f a xf

y = a constant t 1

Find the d.c. of the following.


1. y =

447

...(i)

448

How to Learn Calculus of One Variable

v=

1 + sin x
1 sin x

...(ii)

y = tan 1 v tan 1 u

...(iii)
where v =

Now, from (1),

u= v

FG 1 cos x IJ =
H 1 + cos x K

Let u =

1 cos x
1 + cos x

...(ii)

y = tan u
From (i),

FG 1 + sin x IJ
H 1 sin x K

u=

...(iii)

du
1
=
=
dv 2 v

and from (ii),


y = tan 1 u

1
1 + sin x
1+
1 sin x

FG
H

v=

IJ FG
K H
=

1
1 sin x + 1 + sin x
1 sin x

1 sin x
2

IJ
K
...(iv)

dy dy du dv
=

Hence,
dx du dv dx
=

1 sin x 1

2
2
1

FG 1 sin x IJ 2 cos x
H 1 + sin x K a1 sin xf

cos x

b1 + sin xg b1 sin x g

FG 1 cos x IJ
H 1 + cos x K

1 cos x
1 + cos x

dv sin x (1 + cos x ) + sin x (1 cos x )


=
2
dx
1 + cos x

dv sin x + sin x cos x + sin x sin x cos x


=
2
dx
1 + cos x

Now, y = tan u

dy
1
1
=
=
du 1 + u 2
1 cos x
1+
1 + cos x

1
1 + cos x + 1 cos x
1 + cos x

FG
H

1 cos x

,
2 cos x

FG
H

IJ FG
K H
=

IJ
K

1
2
1 + cos x

IJ
K

1 + cos x
2
Now,
dy dy du dv
=

dx du dv dx

FG 1 cos x IJ
H 1 + cos x K
FG 1 cos x IJ
y = tan
H 1 + cos x K
1

Solution:

x n +
2
2. y = tan

and from (ii),

dy
1

=
du 1 + u 2

...(i)

du
1

=
=
dv 2 v

a1 + cos xf 1
2

...(iv)

1 + cos x
1 cos x

2 sin

a1 + cos xf

sin x
1
1 sin x
, x n.

=
2
2
2 sin x
1 cos x

Differentiation of Inverse Trigonometric Functions

On Method of Trigonometric Substitution


Type 1: y = t
1

af

f x

where t =
/ cos1 / tan1 / cot1 / sec1 /
cosec1
f (x) = a trigonometrical function of x / algebraic
function of x / an algebric expression in x.
Note: 1. In such types of problems mentioned above,
our main aim is to remove inverse trigonometric
sin1

operator sin / cos / tan / cot / sec / cosec

by

a trigonometric substitution x = sin / cos / tan /


cot /sec / cosec or by any other means.

af

2. When y = t 1 f x is provided, where f (x) = an


expression in x, we substitute x = a trigonometrical
function of and t given expression in x becomes
the direct trigonometrical function (or, trigonometrical
function / circular function) of multiple angle of

b g

f f m = m which is differentiated w.r.t.


x and lastly is expressed in terms of x, where m = 1,
2, 3, .... etc.

(a) In sin

af

f x , we put = sin 1 x,

, then x = sin .
2
2
(b) In cos

af

(c) In

tan

(c2)

cosec 1 = cot

af

f x ,

a f

(ii) (a1) 1 + cos = 2 cos


(a2) 1 cos = 2 sin

1 cos = sin
2

(b1)

1 + tan = sec

(b2)

sec 1 = tan

(b2) 2 cos 1 = cos


(iii) sin 2 = 2 sin cos
2

(iv) cos 2 = cos sin = 1 2 sin


2

= 2 cos 1

(v) sin 2 =

2 tan
2

1 + tan
2

put

= tan

x,

1 tan
2

1 + tan

2 tan
2

1 tan
3

(viii) sin 3 = 3 sin 4 sin


3

(ix) cos 3 = 4 cos 3 cos


3

3 tan tan

(x) tan 3 =

1 sin = cos

(b1) 1 2 sin = cos

(vii) tan 2 =
we

, ... etc.

Where mod operator .... can be removed


from the trigonometric function by considering the
prinicipal values of = sin1 x, cos1 x, tan1 x, cot1
x, sec1 x, cosec1 x.

(vi) cos 2 =

< <
, then x = tan etc.
2
2
3. Remember the following formulas which give us
idea of trigonometric substitution

(a2)

1 + cot = cosec

(i) (a1)

(c1)

f x , we put = cos x, 0 ,

then x = cos

449

1 3 tan

(xi) (a1) tan + =

(a2) tan =
(xii) (a1) tan

tan + tan
1 tan tan

tan tan
1 + tan tan

A+B
1
1
= tan A + tan B
1 AB

450

How to Learn Calculus of One Variable

(b2) tan

Examples:

A B
1
1
= tan A tan B
1 + A B

FG + = 1 + tan IJ
1 tan K
H4
F
1 tan I
tan G =
J
1 + tan K
H4

(xiii) tan
(xiv)

(ii)

a f
sin sin e150 j 150 ( = 30 )
(ii) cos acos x f = x , when 0 x ,
F 2 IJ = 2
cos G cos
H 3K 3

(iii) tan atan x f = x , when < x < ,


2
2
I
F
tan tan
H 6 K = 6 , ... etc.
o

where we should note that sin1 x, tan 1 x, and

1
cosec x are angles which lie between and +
2
2
denoting their principal values and cos

1
x , cot x

and sec x are angles lying between 0 and denoting their principal values.
3. (A) If y = sin

x , cosec

x , or tan

x , then x

and 0. In this
2
x, y = cosec1 (x)

is negative means y is between


case y = sin1 (x) = sin1

(B) If y = cos

N.B.: 1. In the given question, we are given an


expression in x which are obtained by replacing
x = sin / cos / tan / cot / cosec and sec in
the above formulas in r.h.s. This is why the form of
the expression in x gives us the idea of proper
substitution x = sin / cos / tan / cot / sec /
cosec .
2. Remember:

1
(i) sin sin x = x , when x ,
2
2
1

a1f = sin a1f = 2

tan a1f = tan a1f = etc.


4

(i) sin

= cosec1 x, y = tan1 (x) = tan1 x

x , or cot x , then x is

and . In this case


negative means y is between
2
1
1
y = cos x = cos x
y = sec

y = cot

x , sec

a f
x = sec a x f
x = cot a x f
1

Examples:

F 1 I = cos F 1 I = = 2
H 2K
H 2K
3
3
F 2 I = sec a2f = = 2 etc.
(ii) sec
H 3K
3
3
(4) (i) sin e sin x j = x for 1 x 1
(ii) cos ecos x j = x for 1 x 1
(iii) tan e tan x j = x for x R
(iv) cot ecot x j = x for x R
(v) sec e sec x j = x for x 1
(vi) cosec e cosec x j = x for x 1
F 1 I or sin
5. If x 1 , then cosec x = sin
H xK
1I
F
x = cosec H K .
x
F 1 I or cot
(ii) If x > 0 , then tan x = cot
H xK
1
FI
x = tan H K .
x
F 1 I or cos
(iii) If x 1, then sec x = cos
H xK
1
F I
x = sec H K .
x
(i) cos

Differentiation of Inverse Trigonometric Functions

af

= sin

x , where

3 , i.e;
2
2

1
1

i.e; x
6
6
2
2

Solved Examples put in the form: y = t 1 f x


Find the d.c. of the follwoing:
1. y = cos

e1 2 x j

Solution: on putting x = sin = sin

, 1 2 sin 2 = cos 2 suggests to


2
2

put x = sin in 1 2 x

y = cos

e1 2 sin

= 2 = 2 sin

i to get
j = cos acos 2 f

= 2 = 2 sin

2
1 x

1 x2

if

; i.e; 0 x 1
2

0 ; i.e;
2

if 0 < x < 1

if 1 < x < 0

2x

1 x2

x
2. y = sin

dy
=
dx

1 x

for x <

1
2

6
2

y = 3 for

dy
=
dx

1 x

for

1
< x <1
2

Note: By chain rule

b 4 x g

3 1 4x2

1 x2 1 4x2
1

y = cos
= cos

, x 0 , 1

e4 x

3x

,x

e3 x 4 x j

x = sin = sin

e3 sin 4 sin j
3

1
, 1.
2

e4 cos 3 cos j
3

acos 3f = 3 = 3 cos

= 3 2 for 1 < x <

and
2
6

0 , 4 cos3 3 cos = cos 3 suggests to


put x = cos to get,

x,

3
where , 3 sin 4 sin = sin 3
2
2
3
suggests to put x = sin in 3 x 4 x to get,
1

x if

Solution: on putting x = cos = cos1 x , where

1 1 2x

Solution: on putting

y = sin

dy
=
dx

3. y = cos

dy
does not exist for x = 0 , 1, 1
dx

(iii) (Chain rule):

dy
=
dx

Note: (i) y is defined for x 1


(ii)

= 3 = 3 sin

Now, y = 3 for

1 x 0
dy

=
dx

asin 3f

x if 0
1

451

dy
=
dx

3
1 x2

for

dy
=
dx

3
1 x

x for 0

1
2

1
< x <1
2

Also y = 2 3 for x <

for x <

1
2
1
2

452

How to Learn Calculus of One Variable

4. y = tan

2x
1 x

= 2 for

Solution: on putting x = tan = tan

x for

2 tan
<< ,
= tan 2 suggests to put
2
2 1 tan 2

x = tan .
2 tan
1 tan 2

= tan 1 tan 2

= n + 2 = n + 2 tan 1 x

dy
2
=
; x 1.
dx 1 + x 2
1

1 + tan

= sin 2 suggests to put

y = sin 1

= sin

1 + x2

1 + tan 2

F 2 sin cos I = sin a2 sin cos f


GH cos + sin JK
a sin 2f

x 1 + x2

i.e; x 1
4
4

< < , i.e. x > 1


= 2 for
4
2
x , if

Solution:

F
GH

d 1 x2
dx 1 + x 2

I
JK

4x

d1 + x i

2 2

x 0.

(very easy method)

F1 x I
GH 1 + x JK
F1 x I
y = sin G
H 1 + x JK

1
7. y = sin

F1 x I
GH 1 + x JK

2x

= 2 = 2 tan

dy
=
dx

2 tan

2
2

4x

2x
x = tan in
1 + x2

= sin

<<
2
2
2

2x
2

2 tan

F1 x I
GH 1 + x JK
F1 x I
y = cos G
H 1 + x JK
1

Solution:

1+ x
1
Solution: On putting x = tan = tan x ,

dy
does not exist at x = 1
dx
(ii) To avoid different cases, we may differentiate
directly using chain rule as shown below in example
(6) and others
6. y = cos

< y< .
2
2

5. y = sin

2
for x > 1
1 + x2

Note: (i)

y = tan 1

where

dy
2
for x < 1
=
dx 1 + x 2

<<
i.e; x < 1
2
4

dy
=
dx

2
2

F1 x I
1G
H 1 + x JK
2
2

F
GH

d 1 x2
dx 1 + x 2

I
JK

Differentiation of Inverse Trigonometric Functions

d1 + x i
2

4x
=

d1 + x i

x 1 + x2

F 2x I
GH 1 + x JK
F 2x I , 0 y
y = cos G
H 1 + x JK

j 4x
e1 + x j

2 1+ x

b g

j e

e1 x j e1 + x j
2

2
2

, x 1.

F1 x I
GH 1 + x JK
F1 x I
y = cos G
H 1 + x JK
2

1+ x + 2 x 4 x

e1 + x j

2 2

1 x

1+ x

d1 + x i

2 2

1+ x
d cos y

=
dx

2 2

2 2

i d1 + x i
i d1 x i

cos y =

4 x2

e1 + x j 4 x
e1 + x j

Solution:

2
Now, sin y = 1 cos y

= 1

2 1 x2

9. y = cos

2
dy 2 1 x
1
=

, sin y 0 .
2
sin y
dx
1 + x2

dy
2
=
dx 1 + x 2 which is only possible when x < 1

2 1 x2
2 2x2
dy
=
=
2
2
dx
1 + x2
1 + x2

Note: If we do the above problem by the method


of trigonometric substitution, we get complete
result provided different cases are properly
considered otherwise (i.e; if different cases are not
properly considered) , we get an incomplete result

2 2

sin y

1 + x2 2 2x 2 x
2

1 + x2

j
e1 + x j

2
dy 2 1 x
=

Hence,
2
dx
1 + x2

2x

positive]

d cos y
=
dx

cos y =

2 2

2 2

,x0

1
8. y = cos

2 2

1 + x4 2 x2

2 2

2x

Solution:

e1 x j
=
=
e1 + x j
e1 + x j
e1 x j = e1 x j
=
e1 + x j e1 + x j
[ Q e1 + x j = 1 + x because e1 + x j is always

4x

453

j a2 xf e1 x j a2 xf
e1 + x j

2 x 2 x 3 2x + 2 x3

d1 + x i

2 2

2 2

454

How to Learn Calculus of One Variable

dy
=
dx

sin y

4 x

1+ x

4 x

dy

=
dx

1+ x

2 2

2 2

=
sin y

4x

1+ x

2 2

Now, sin y = 1 cos y

sin y as 0 y

d1 + x i d1 x i
d1 + x i
1 + x + 2 x d1 + x
d1 + x i

2 x2

1 + x4 + 2 x2 1 x4 + 2 x2

d1 + x i

1 + x2
dy
4x
Hence,
=
.
dx
2 x
1 + x2

d i
e1 + x j

4x

e1 + x j 2 x
e Q d1 + x i is always positivej
2

4 x

e1 + x j

2 2

FQ cos y =
H
F1 x I
1G
H 1 + x JK

1
cos y

cos y as

d1 + x i d1 x i
d1 + x i
1 + x + 2 x d1 + x
d1 + x i
2 2

2 2

y
2
2

2 2

2 2

dy
=
dx

4 x
dy 2 x 2 x 2 x + 2 x
=
=
2 2
2
dx
1+ x
1+ x

Now, cos y = 1 sin y

e1 + x j
1 + x2

cos y

2 2

2 x

2 2

2 2

2 2

2 2

4x

2 2

;x0

e1 + x j x
F1 x I
10. y = sin G
H 1 + x JK
F1 x I
Solution: y = sin G
H 1 + x JK
F1 x I
sin y = G
H 1 + x JK
d sin y e1 + x j a2 x f e1 x j a2 x f

=
dx
e1 + x j
1

c Q sin y =
F1 x I
1G
H 1 + x JK

sin y

2x

2 2

2 x2

I
K

Differentiation of Inverse Trigonometric Functions

4x

d1 + x i

2 x

(Q 1 + x

2 x

F 3x x I
GH 1 3x JK
F 3x x I
y = tan G
H 1 3x JK

12. y = tan

1 + x2

= 1 + x for 1 + x

j being always

4x
dy

=
2
dx
1+ x

11. y = sin

2 x

1 x

Solution: y = sin

d sin y
=
dx

cos y

dy
=
dx

1 x

1 x

y = tan 1

b g

2 x =

x
1 x2

x
1 x2

1
cos y

= tan
2

x2 = x

Now on putting ,

x
1 x2

1
x

F 4x I
GH 4 x JK
F 4x I ; x 2
y = tan G
H 4 x JK
F
I
F
GG x JJ = tan GG 22x
GG 1 F x I JJ
GG 1 F x I
H H 2K K
H H 2K

13. y = tan

Solution:

dy
3
1
=
;x
.
2
dx 1 + x
3

FQ cos y = cos y as y I
H
2
2K
F
I
1 G 1 x J = 1 e1 x j = 1 1 + x
H
K
dy
=
dx

F 3x x I ; x 1
GH 1 3x JK
3
F 3 tan I = tan btan 3g
GH 1 3 tan JK

= 3 + n = 3 tan 1 x + n

1 x2

3 tan tan
<< ,
= tan 3 suggests
2
2
2
1 3 tan

to put x = tan in

Q cos y = 1 sin y

x for

dy
=
dx

sin y = 1 x

on putting x = tan = tan

2 x
x 1+ x

Solution:

positive)

e1+ x j =

, x 0.

1 x2

d1 + x i

2 2

455

1
x
= tan = tan
2

< < , we have


2
2

I
JJ
JJ
K
F x I for
H 2K

456

How to Learn Calculus of One Variable

y = tan 1

F 2 tan I = tan btan 2 g = 2 + n


GH 1 tan JK
1

where < y <


2
2
y = 2 tan 1

dy
= 2
dx

14. y = sin

FG x IJ
H 2K

FG 2 x
H

Solution: y = sin

1 x
1

FG 2 x
H

,
2
2

1
2

IJ
K

1 x

FH

= sin

FQ
H

b2 sin cos g

y = sin

IJ
K

2
1 x

1 x

I
K

sin y = 2 x 1 x

for 2 x 2 < 1

for 2 > 2 x 2 > 1

...(v)

b g

2x
2 1 x

dy
= 2 1 x2
dx

a f

2 x

2x2
1 x2

2 1 x 2 2x 2
1 x2

R|1 x x U| 2 d1 2 x i
S| 1 x V| = 1 x
W
T
dy 2 e1 2 x j
1

dx

...(ii)

IJ
K

d sin y
2
= 2 1 x +
dx

=2

...(i)

FG 2 x
H

y = sin 1 sin 2 = 2 ,
1

when <
i.e; 1 x <
2
2
4

cos y

asin 2 f = 2 for 4 4

1
1
x
i.e;
or 2 x 2 1
2
2

1 x

y = sin

IK

cos = cos as
2
2
1

j=

1
dy
2
does not exist for x = 1,
2
dx
or, alternatively,

y = sin 1 2 sin 1 sin 2


1

...(iv)

2d
dy
=
=
dx
dx
2

from (ii) and (iii),

1 sin 2 = cos suggests to put

x = sin in 1 x

...(iii)

dy 2 d 2 d sin
=
=
dx dx
dx

on putting, x = sin = sin 1 x for

1
< x 1
2
Hence, from (i),

4
=
, x 2
4 + x2
1

<
4
2

i.e;

x
+ n
2

1+

a f

and y = sin sin 2 = 2 , when

1 x

cos y

Now, cos y = 1 sin y = 1 4 x

e1 x j
2

457

Differentiation of Inverse Trigonometric Functions

LMQ
N

cos y = cos y as

y
2
2

OP
Q

and this will be true in particular when a is a positive


constant and represents the principal values of
1

sin x

e1 2 x j = e1 2 x j
LM Q f a xf = f a xf OP
N
Q
dy 2 d1 2 x i
1

=
dx
1 x
d1 2 x i
2 d1 2 x i
=
; 2 x 1, x 1
d1 2 x i 1 x
2

2 2

= 1 4 x + 4 x =

tan x

2
2
= a sec = a sec = a sec .

More about substitution


1. (a1) If a x

cos , i.e; = sin

(iii) If x = a sec , then

occurs , we put x = a sin or

FI
HK

x
, when and
2
2
a

a is positive
(a2) If

x 2 a2

occurs , we put x = a sec or

cosec , i.e; = sec

F xI ,
H aK

when 0

FG IJ and a is positive.
H 2K
(a ) If a + x
dor , a + x i occurs , we put
F x I , when
x = a tan or a cot , i.e; = tan
HaK
2

< < and a is positive.


2
2

(a 4 ) If

a x or

a+ x

(ii) If x = a tan , then a 2 + x 2 = a 2 + a 2 tan 2

= a cos = a cos .

(i) If x = a sin , then a 2 x 2 = a 2 a 2 sin 2

F or , sin x I , cos x F or , cos x I ,


H
H
aK
aK
F or , tan x I , cot x F or , cot x I
H
H
aK
aK

or

ax
a+x

a+x
occurs, we put x = a cos2 .
ax
2. The preceeding square roots are a cos , a sec
and a tan when these quantities are not negative

2
2
= a tan = a tan .

3. If we are not given the expression which can be


transformed into the trigonometric function of
multiple angle of , we should avoid this substitution
rule since it becomes complicated and gives no fruitful
result to find d.c.. For this reason we should use chain
rule to find d.c. when substitution method fails, i.e;
we should use the formula.

dy
1
=
dx
dx
dy

FG IJ
H K

or chain rule (i.e; function of a

function rule).
Remember: Generally, we are provided the following

LM F
N H

form t 1 f

constant
or

x a = a sec a

1 x2

a2 x2

a 2 + x 2 a constant

1 x2 a

IK OP Whose differQ

ential coefficient is required to find out, where


t1 = sin1, cos1, tan1, cot1, sec1, and cosec1.
Solved Examples:
Find the d.c. of the following inverse trigonometric
functions.

458

How to Learn Calculus of One Variable

1. y = sin

LM
MN

x
1+ x

Solution: y = sin

LM
MN

OP
PQ

FQ for
H
2
2
y = tan a tan f = = sin
1

x
1+ x

OP
PQ

Firstly, on simplification after putting

1
x = tan = tan x , where
2
2

OP
1 + tan PQ
L tan OP
y = sin M
MN sec PQ
L tan OP
y = sin M
N sec Q
FQ sec = sec for I
H
2
2K
L sin cos OP
y = sin M
N cos
Q
y = sin

LM
MN

tan

y = sin

sin

y = = tan

x
2

a x

and a > 0 i.e; a = a


1

LM
MN

F xI
H aK

dy
=
dx

1
x

a
2

a x

1
=
a

F
GG
H

1 sin

= tan

FG sin IJ
H cos K

dy
=
dx

1
a

4. y = tan

1
a x2
2

LM
MN

, x a.

1+ x 1
x

OP
PQ

a x
a

sin

2
2

for

a sin

1 x

OP
PQ

OP
L a sin OP
= tan M
P
N a cos Q
a a sin Q
LMQ cos = cos for OP
2
2Q
N
F xI
y = tan tan = = sin
H aK

y = tan

LM
MN

, x 1.

1 x2

x = a sin = sin

Solution: Firstly, on simplification after putting

1
x = sin = sin x for
2
2
y = tan

Solution: Firstly, on simplification by putting

dy
1

=
dx 1 + x 2

2. y = tan

dy d sin 1 x
=
=
dx
dx

3. y = tan

I
K

cos = cos

I
JJ
K

1
a

459

Differentiation of Inverse Trigonometric Functions

Solution: y is defined for all x 0 . Firstly on


simplification by putting

1
x = tan = tan x for < <
2
2
y = tan

= tan 1

LM
MN

1+ x 1
x

= tan

LM
MN

1 + tan 1
tan

LM sec 1OP ; 0 , i.e; x 0


N tan Q

FGQ sec
H
1

OP
PQ = tan

= sec for

LM1 cos OP = tan


N sin Q

<<
2
2

OP
PQ

IJ
K

LM 2 sin OP
MM 2 sin 2 cos PP = tan FH tan 2 IK
MN 2 2 PQ

LM
MN

1+ x

Solution: y = cot

= cot

i b

LM
MN

d
O
1P
PQ

F cot I
H 2K

and y = +

, if < < 0
2
2

=+

1
1

tan x , if < < 0


2
2

Hence,

1
dy
=
; x0 .
dx 2 1 + x 2

ib

e j
1

Form: A function having the form t t x

LMt et
N

jOPQ

or

where t = sin / cos / tan / cot / sec / cosec

and t1 = sin1 / cos1 / tan1 / cot1 / sec1 / cosec1


is differentiated using the following working rule.

1+ x 1
x

1 + tan + 1O
PP
y = cot
tan
Q
L sec + 1OP
= cot M
N tan Q
1

F 1 + cos I = cot
GH sin JK

1
1

= tan x if 0 < <


2 2
2

OP
PQ

Working rule:
(1) Put x = sin in sin

x = cos in cos

The function is defined for all x 0 . Firstly, on


simplification by putting

1
x = tan = tan x for < < , 0
2
2

LM
MN

I
K
LM1+ 2 cos 1OP
MM 2 PP
N 2 sin 2 cos 2 Q

<<
2
2

= sec for

Type 2:

dy 1
1
1
, x0

=
=
2
dx 2 1 + x
2 1 + x2
1

= cot

1
1
= tan x
2 2

5. y = cot

FQ sec
H

x = tan in tan

x = cot in cot

x = sec in sec

x
x

x
x

x = cosec in cosec

So that we may obtain t

af

t = where

represents the principal values of sin


tan

x , cot

x , sec

x and cosec

x , cos
x.

x,

460

How to Learn Calculus of One Variable

af

LMQ sec =
N

af

dt
dt
dt d
d
2. Differentiate
=

=
dx
dx
d
dx
d

af

3. Express the required result in terms of x (or, in terms


of x and y both if required).

d i d i
d i

d i
d i

d t t 1 x
dy
or,
=
dx
d t t 1 x

2. y = tan sin

d i . d dt x i
d i
dx
d dt x i
Solved Examples on the form: y = t d t x i
d t t 1 x

y = tan sin

F I
H 2 2K
sin j = tan

x = for

x = tan sin

Solution: Put x = tan tan

x = for

<<
2
2

FG
H

F I
H 2K

FG IJ
H K

= sec =

dy
d sec sec sec
2
= sec
=
=
=
d sin cos
dx
dx
dx
d
d

Find the d.c. of the following.

Solution: Put x = sin sin

1. y = sec tan

cos

1
2

1 sin

IJ
K

as cos > 0 for

d1 x i

2 3/ 2

for x < 1.

d i

1
Solved Examples on the form: y = t t x

d sec
d
dy d sec d sec d

=
=

=
dx
dx
dx
d
dx
d

Find the d.c. of the following

dy sec tan
=
Q x = tan
d tan
dx
d
sec tan sin
=
=
cos = sin
2
cos
sec
tan
=
=
sec

x
2

1 + tan

<<
2
2

y = sec tan 1 tan = sec

OP
Q

1 + x2

n 1

= n t t 1 x

<<
2
2

Note: The above types of problems also can be done


1
1
dy d t t x d t x
by using the chain rule
=

dx
dx
d t 1 x

sec for

j
Solution: y = sec e cos x j , x 0
d LMsec ecos x jOP
dy d sec ecos x j
Q

=
= N
2

1. y = sec cos

dx
dx
Now, using chain rule, we have

LM e
N
LM e
N

jOPQ
jOQP

d sec cos x
dy
=
dy d sec cos 1 x

dx

d sec cos

d cos x

j d cos

dx

461

Differentiation of Inverse Trigonometric Functions

j e

j e

j 1b1xg

= 2 sec cos x sec cos x tan cos x

x < 1.
=

2. y = tan cos

LM e
N

2 sec cos x

jOQP

Solution: y = tan cos

tan cos

1 x
=

Put x = cos cos

y = sec cos x

j LNM e

x = tan cos

jOQP

x = , where 0

dy
d tan
=
d
dx
2

= 2 tan sec
1

Put x = cos cos

, i.e. x 0 .
2

y = tan 2 ,

for x < 1 , x 0 .
x3
or, alternatively,

x = , where 0

d
dx

y = sec

d sec
d
dy
= 2 sec
= 2 sec sec tan
dx
dx
dx
2

= 2 sec tan
2 sin

d cos x
dx
, x < 1, x 0 .

cos 1 x 2
3

2 1 cos
x

1 x

Q sin = sin for 0


=

x3 1 x2

2
x

for x 0 , 1.

y = sec 2 cos 1 x = sec cos 1 x

1
x2

1
x

2 tan sec
d cos
d

2 tan sec 2
, x < 1, x 0 .
sin
2
3

cos

2
x

aQ x = cosf for

x < 1, x 0

3. y = cot tan

Solution: Put x = tan = tan

Note:

e
L F
= Msec G sec
N H

=
=

2 1 x2

2 tan sec
=
dx
d

IJ OP
KQ

x,

<<
2
2

y = cot tan 1 tan = cot , 0 .


d cot
d
dy d cot d cot d

=
=

=
dx
dx
dx
d
dx
d

462

How to Learn Calculus of One Variable

cosec 2
cosec 2
cos2
=
=
=

= cot 2
2
2
d tan
sec
sin
d
=

1
2

tan

4. y = sin m sin

x
1

2 n + m sin x
2
2

x for

x = sin m sin

2
2

sin = sin m

if

dy m cos m m 1 sin m
=
=
2
dx
dx
1 sin
d

if cos m 0 and
2
dy + m 1 y
=
dx
1 x2

m 1 y2
dy
=+
dx
1 x2
=

Q sin m = y , for

1 x2

for

in case (2), x < 1.

3
1
< m sin x < 2n +
2
2

F
H

x = ,

2
2

I
K

sin = cos 2

d
dy d cos 2 d cos 2 d 2
=
=

= sin 2 2
dx
dx
d 2
dx
dx

dy 2 sin 2 2 sin 2
=
=
dx
d sin
dx
d
d
=

x < 1.

Q sin = x and sin m = y ,

if 2n +

y = cos 2 sin

m sin x 2n +
2
2

m 1 y2

1 x2

5. y = cos 2 sin

dy m 1 sin m
if cosm < 0
and
=
2
dx
1 sin
=

m 1 y2

in case (1), x < 1.

Solution: Put x = sin sin

x < 1.
if 2n

2 n + 1 m sin x
2
2
Hence,

1
1
2. sin y = 2 n + 1 m sin x

d
dy d sin m d sin m d m
=
=

= cos m m
dx
dx
d m
dx
dx

Now

if

Solution: Put x = sin = sin


y = sin m sin

y = sin m sin 1 x

1. sin 1 y = 2 n + m sin 1 x

b x 0g

or, alternatively,

sin 2

,
cos
2

2 2 sin cos
dy
=
= 4 sin = 4 x
dx
cos

for x < 1.
Q x = sin

463

Differentiation of Inverse Trigonometric Functions

6. y = cos 2 cos

=t

Solution: using chain rule, we have

d cos 2 cos1 x

dy
=
dx
d 2 cos 1 x

j d e2 cos xj d cos
1

d cos

= sin 2 cos 1 x 2

dx

1
d cos 1 x

,
d cos 1 x
1 x2

= sin 2 cos x

2 sin 2 cos x
1 x

1 x

1 x
= 4x

ax + yf
y = tan a x + y f
1

2 2 sin cos
1 x

4 x sin
=
sin

t1

b g

f x, y

= sin1 / cos1 / tan1 / cot1 / sec1 /

b g
and y or f b x , yg = an algebraic expression in x and
cosec1 f x , y = an algebraic implicit function of x
y mixed together.
Working rule:
1. Change the given inverse trigonometric function
into direct trigonometric function.
2. Differentiate both sides implicitly w.r.t. x remembering that y is a function of x.
dy
3. Finally solve for
.
dx

d tan y d x + y
=
dx
dx

d tan y dy dx dy

=
+
dy
dx dx dx
2

sec y
2

Form 1: y = t

b g

sec y
1

dy
.
dx

tan y = x + y

x < 1.

Where,

3. Finally solve for

Solution:

Q sin = sin for 0 for

Type 3:

= t 1 (an algebraic implicit function)

Working rule:
1. Change the given inverse trigonometric function
into direct trigonometric function.
2. Differentiate both sides implicitly w.r.t. x remembering that y is a function of x.

1. y = tan

Let cos x = , 0

a f

f2 x , y

Find the d.c. of the following.

2 sin 2

1
Solved Examples on the form: y = t f x , y

x <1

a f

Form 2: An algebraic implicit function f 1 x , y

dy
dy
=1+
dx
dx

dy dy

=1
dx dx

j dydx = 1

sec y 1

1
1
1
dy
=
=
=
2
2
dx
x+ y
tan y
sec y 1

e
j
2. y = sin a x + y f
Solution: y = sin a x + y f
1

sin y = x + y

d sin y d x + y
=
dx
dx

464

How to Learn Calculus of One Variable

cos y

dy dx dy
=
+
dx dx dx

cos y

dy dy

=1
dx dx

f d a xdx+ yf = 1 dydx
L dy OP = 1 dy
sec a x + y f tan a x + y f M1 +
N dx Q dx
dy
sec a x + y f tan a x + y f + sec a x + y f tan a x + y f
dx
a

f dydx = 1

cos y 1

dy
1
=
dx
cos y 1 .

=1

a f

f2 x , y

Find the d.c. from the following.

a x + yf
Solution: x + y = sin a x + y f
sin a x + y f = x + y
d sin a x + y f d a x + y f

=
1. x + y = sin

f d a xdx+ yf = 1 + dydx
L dy OP = 1 + dy
cos a x + y f M 1 +
N dx Q dx
dy dy
cos a x + y f + cos a x + y f

=1
dx dx
a

cos x + y

dy
= 1 cos x + y
dx

a f
a f
2. x + y = sec a x y f
Solution: x + y = sec a x y f
d sec a x + y f d a x y f

dy cos x + y 1
=
= 1
dx
cos x + y 1
1

dx

dx

a f a f dydx + dydx
= 1 sec a x + y f tan a x + y f
dy 1 sec a x + y f tan a x + y f

=
dx 1 + sec a x + y f tan a x + y f
3. xy = sin a x + y f
Solution: xy = sin a x + y f
sin a xy f = x + y
d sin a xy f d a x + y f

=
1

dx

cos x + y 1

dy
dx

dx

sec x + y tan x + y

Solved Examples out on the form:


f1 (x, y) = t

sec x + y tan x + y

dx

dx

a f d dxaxyf = 1 + dydx
L dy + y dx OP = 1 + dy
cos b xy g M x
N dx dx Q dx
LMQ d sin a xyf = d sin axyf d axyf OP
d a xy f
dx Q
N dx
dy
dy
x cos a xy f
+ y cos a xy f 1 = 1 +
dx
dx

cos xy

a f dydx dydx = 1 y cos axyf

x cos xy

a f

a f
1 y cos a xy f
dy 1 y cos a xy f

=
=
dx x cos a xy f 1
1 x cos a xy f
x cos xy 1

dy
= 1 y cos xy
dx

465

Differentiation of Inverse Trigonometric Functions

Type 4: Differentiation of inverse trigonometric


function of a function of x w.r.t. an other inverse
trigonometric function of a function of x / an other

af

function of x differentiate t 1 f x

af

af

w.r.t.

t 1 g x or, differentiate t 1 f x w.r.t. g (x)


where f (x) = a function of x
g (x) = an other function of x / sin x / cos x / tan x / cot
x / sec x / cosec x

= sin / cos / tan / cot / sec / cosec

dy
=1
dx
dy
1

=
dx cos y
cos y

af

1. Put t 1 f x
w.r.t.]

and t

af

= y [ The function put before


...(1)

dy
=
dx

cos y = cos y as

dy
=
dx

g x = z [the function put after w.r.t.] ...(2)

sin z

dy
dx
which is the required d.c.
function of x
dz
dx
Remember:

2.

af
af

af

Now,

af

Find the d.c. of the following.


x w.r.t. cos

Solution: Let sin 1 x = y

sin y = x , y
2
2

1
and cos x = z cos z = x , 0 z

1
2

1 x

LM
dy M
(3)

=
dz M
( 4)
MM
N

for x < 1.
Or, alternatively,

1
1
sin x = cos x
2

Solved Examples:
1

dz
1
=
dx
sin z

Q sin z = sin z as 0 z

df z
dz
= f z
dx
dx

1. sin

dz
=1
dx

1 cos z

df y
dy
= f y
dx
dx

e xj = 1

d ecos x j
d sin

... (1)
... (2)

OP
Q
...(3)

1 x2

2. Find the d.c. of (1) w.r.t. x using the rule of finding


d.c. of inverse trigonometric function. Similarly, find x
the d.c. of (2) w.r.t. using the rule of finding the inverse
trigonometric function.
3. Divide the d.c of y by the d.c. of z regarding z as a

1.

y
2
2

d cos z dx
=
dx
dx

(2)

for x < 1.

1 sin y

LMQ
N

t11 = another inverse trigonometric operator


excepting t 1 .
Working rule:

d sin y dx
=
dx
dx

(1)

1
1 x
1

1 x2

OP
PP = 1
PP
Q

...(4)

466

How to Learn Calculus of One Variable

F
I
JJ w.r.t. cosec GG
2. tan
H
1 x K
F x I
Solution: Let tan G
GH 1 x JJK = y
1

F
GG
H

1
1 x

I
JJ .
K

and cosec 1

1 x

F
GG
H

1 x

... (1)

1
2

I
JJ = z
K
... (2)

FG
H

ex

1 x

LM
MN

OP
PQ

x
1 x

dy
sec y
=
dx

FG
H
x

1 x +
dy

=
2
dx
1 x

IK +

IJ
K

a f

2 x

e1 x j

IJ e1 x j
K

...(3)

(2) cosec z =

d cosec z
d
=
dx
dx

1
2 1 x

dz
=
dx

1 x

cosec z 1

x2

1+

F
GG
H

1
1 x

1
x
1 x2

I
JJ
K

I
JJ
K
1

e1 x j
2

FG
H

1 x

e1 x j
2

IJ e1 x j
K

1 1 x
cot z

e1 x j aQ cot z > 0f
2

d1 x i F
GG
H
2

a 2 x f

dz
=
dx

cosec z cot z

F
GG
H

j e1 + tan yj

d1 x i
2

2 1 x

1 x

1 x

1 x2

OP
PP
PQ

1 x2

= 1
x 1

d tan y
d
=
dx
dx

x +1

1 x

1 x 1

Again,

1 x2

Now,

1 x2

F0 < z I
H
2K

FH

cosec z =

(1)

tan y =

d1 x i + x
1
=
FH 1 x IK d1 x i LM 1 x + x
MMdF i I
MN H 1 x K

1
1 x2

I
JJ
K

Differentiation of Inverse Trigonometric Functions

d1 x i
2

d1 x i
2

1 x2

2 x
2 x
sin
1 cos x 2 sin 2
2
Solution:
=
=
2 x
1 + cos x 2 2 x
cos
cos
2
2

1 1 x2

d1 x i

y = tan

x 1 x2

d1 x i
2

FH

1 x

= tan

x2

1 x2
2

IK

1
x
x
=

x
x
1 x2

1
1 x

1 x2

x
x

dy x
=
, x < 1.
dz
x

On Method of Transformation

2. t

1 sin x
1
or t
1 m sin x

1 sin x
1
or t
1 cos x

Working rule: Whenever 1 sin x and / 1 cos x


appear ( or, appears) under the radical sign
, we
express the function within the radical as a square of
some function.
Solved Examples:
Find d.c. of the following
1. y = tan

1 cos x
1 + cos x

RS tan x UV
T 2W

U|
V|
|W

x
2

2 x
x
tan sec
dy 1
2
2

=
dx 2 tan x sec 2 y
2

1 cos x
1 m cos x

1 cos x
1 sin x

x
x
tan
sec 2
dy
2
2
sec 2 y
=

x
dx
2
tan
2

1
=
2

Type 1:
Form: 1. t

R|
S|
|T

2 x
x
sin

1
2
2 = tan
2 x
2 x
cos
cos
2
2

sin

tan y = tan

Hence,

dy
(3)
dx

=
dz
( 4)
dx

467

1
=
2

tan
tan

F
GH

2 x
x
sec
2
2

x
x
1 + tan
2
2

I
JK

2 x
2 x
x
x
tan
sec
sec
1
2
2
2
2
=
2 x
x
2 tan x sec 2 x
tan 1 + tan
2
2
2
2

tan

F
H

I
K

x
tan
1
2
=
, x n
2 tan x
2
Or, alternatively,
tan y =
entiation,

1 cos x
, Now using logarithmic differ1 + cos x

468

How to Learn Calculus of One Variable

FG
H

1 cos x 1
1 cos x
= log
1 + cos x 2
1 + cos x

log tan y = log

l a

1
=
log 1 cos x log 1 + cos x
2

d tan y
1

tan y
dx

RS
T

RS
T

fUVW

sin x
sin x
2
1
dy 1
sec y
=
+
tan y
dx 2 1 cos x 1 + cos x

UV
W

1 + sin x
=
1 sin x

F cos x + sin x I
H 2 2K
F cos x sin x I
H 2 2K

1 + sin x
=
1 sin x

F1 + tan x I
H
2K
F1 tan x I
H
2K

1 + tan y dy

tan y
dx

R|1 + cos x + 1 cos x U| = 1 sin x 2


1
= sin x S
V
2
T| 1 cos x W| 2 sin x
2

1+

1 cos x
1 + cos x

1 cos x
1 + cos x

1 cos x
1 + cos x

dy
=
dx

tan

2. y = tan

2 sin x

1 + sin x
1 sin x

1 cos x
1 + cos x
=
2 sin x
1 + cos x

2 sin x

1 cos x
1 + cos x

x
x
cos + sin
2
2
=
x
x
cos sin
2
2

tan A + tan B
1 tan A tan B

F
H

1
1 + sin x
x
= tan
+
tan
1 sin x
4 2

y = tan

dy
1
=
dx sin x

F 2 IJ
sin x G
H 1 + cos x K
1 cos x a1 + cos xf a1 + cos x f

1 + cos x

dy
1
=
dx sin x

1 cos x
1 + cos x

Q tan A + B =

2
1 + cos x

F cos x + sin x I
1 + sin x H
2
2K
=
1 sin x
F cos x sin x I
H 2 2K

Solution:

fq

1
1
1
=
sin x
sin x
2 1 cos x
1 + cos x

IJ
K

tan

1
dy
d
=
tan
dx dx

F + xI
H 4 2K
F + xI
tan
H 4 2K

R|
1 + S + tan F + I
|T H 4 2 K

F + xI
H 4 2K
F + xI
tan
H 4 2K
tan

U|
V|
W

sec

I
K

F + xI
H 4 2K
2

F + x I sec F + x I
H 4 2K H 4 2K
=

x
2
F + I tan F + x I
1 + tan
H 4 2K H 4 2K
F + x I sec F + x I
tan
H 4 2K H 4 2K
1
=

xI
xI
2
F
F
sec
H 4 + 2 K tan H 4 + 2 K
1

tan

Differentiation of Inverse Trigonometric Functions

Note:

FG + x IJ
H 4 2K
F xI
tan G + J
H 4 2K
af

1
d
tan
f x
dx

{1 + f a xf}
2

af

d f x
dx

Working rule:
1. Transform given trigonometrical expression in sin
x and cos x in such a way that it becomes equal to

F x I = x for which are required to


H 4 2K 4 2
F xI
(i) Use sin x = cos
H 2 2K
t 1 t

(ii) Use multiple and / submultiple angle formulas of


trigonometric functions.
Or alternatively,
Put y = given function and change it into direct

dy
regarding y as a function
function and then find
dx
dy
of x, i.e; find
implicitly.
dx
Example: 1. y = tan

FG cos x IJ
H 1 + sin x K

cos x
dy
tan y =
1 + sin x , Now find dx implicitly
2. y = tan

FG 1 + sin x IJ
H cos x K

tan y =

Type 2:
Form: t 1 (an expression in direct t function
involving sin x and cos x only)

FG cos x IJ
H 1 + sin x K
F cos x IJ
Solution: y = tan G
H 1 + sin x K
F sin F xI OP
G H2 K P
y = tan G
GH 1 + cos FH 2 xIK PPQ
LM 2 sin F x I cos F x I OP
H 4 2K H 4 2K P
y = tan M
MM 1 + 2 cos FH x IK 1 PP
4 2
N
Q
L F x I OP
y = tan Mtan
N H 4 2KQ

1. y = tan

tan

469

such that < y <


2
2
4 2
Now, differentiating both sides w.r.t. x
dy
1
1

=0 =
dx
2
2
y =n +

FG 1 + sin x IJ
H cos x K
F 1 + sin x IJ
Solution: y = tan G
H cos x K
LM cos x + sin x + 2 sin x + cos x OP
2
2
2
2
y = tan M
PP
x
x
MN
cos sin
Q
2
2
2. y = tan

y = tan

function rule:
Find the d.c. of the following.

LM F cos x + sin x I
OP
H
K
MM x 2x 2x x PP
MN FH cos 2 + sin 2 IK FH cos 2 sin 2 IK PQ
LM cos x + sin x OP
MM 2x 2x PP
N cos 2 + sin 2 Q
2

1 + sin x
dy
cos x . Now find dx implicit

Solved Examples:

y = tan

470

How to Learn Calculus of One Variable

y = tan

y = tan

y = tan

LM1 + tan x OP
2
MM
xP
1 tan P
N
2Q

LM tan + tan x OP
MM 4 2x PP
N1 tan 4 tan 2 Q

LMtan F + x I OP = n + + x
4 2
N H 4 2K Q

<y<
2
2
dy
1 1

=0+ =
dx
2 2

such that

Type 3:
Form: y = tan

LM a b OP
N a m ab Q

The following points should be noted about the


bracketed expressions in the above.
(i) The signs connecting the two terms in the
numerator and denominator are opposite.
(ii) One of the two term in the denominator is 1 and
the other terms is the product of two terms in the
numerator.
(iii) Somtimes at the place of 1, another constant is
provided, then that constant should be changed into
1 by using the mathematical manipulation of dividing
numerator and denominator by that constant.

F 2 + 3 tan x IJ
Example: y = tan G
H 3 2 tan x K
F 2 + tan x I
G
JJ
= tan G 3
2
GH 1 3 tan x JK
F tan + tan x IJ Where tan = 2
y = tan G
H 1 tan tan K
3
1

y = tan

tan + x = n + + x

where

<y<
2
2

n is such that

dy
= 1 ( being a constant )
dx

Remember:
1. If y = tan

FG K x IJ , find dy
H 1 + K x K dx
1

Solution: Let y = n + tan K tan x


dy
1

=
2
dx
1+ x
dy
Similarly,
can be obtained if
dx

y = tan

FG K + x IJ
H 1 Kx K

Solved Examples:
Find the d.c. of the following.

FG x a IJ
H x + aK
F x a IJ
Solution: y = tan G
H x + aK
F xI
y = tan G J tan b1g + n
H aK
1. y = tan

dy

=
dx

2. y = tan

1
a
1+

x
a2

a
a + x2
2

FG a + bx IJ
H b ax K

Solution: y = tan

L a O
FG a + bx IJ = tan MM b + x PP
H b ax K
MM1 FH a IK x PP
N b Q
1

471

Differentiation of Inverse Trigonometric Functions

Let

a
= tan and x = tan B
b

LM tan + tan B OP
N1 tan tan B Q
tan a + B f = n + + B

y = tan

= tan

y = n + tan

y = tan

1
a
+ tan x
b

dy
1
=
dx 1 + x 12

e j

Note: tan x = cot


tan

tan + B

y = n + + B
y = n + tan

1
5
+ tan x Where < y <
2
2
2

dy
1

=
dx 1 + x 2

L x + a OP
4. y = tan M
N1 x a Q
L x + a OP
Solution: y = tan M
N1 x a Q
1

Now,

x + a

x a

tan A + B

x + tan

gives us an idea of the formula

e j

1
d
x2 + 0
dx

1
only for x > 0
x

FG 5 + 2 x IJ cot FG 2 5x IJ
H 2 5x K
H 5 + 2x K
1

5
2
or x > .
2
5

for x <

5
= tan and x = tan B
2

F 1I

LM tan + tan B OP = tan


N1 tan tan B Q

LM tan A + tan B OP = tan


N1 tan A tan B Q

dy
1
1 H 2K
1

=
x
=
dx 1 + x 2
2 x 1+ x

y = n + A + B = n + tan

y = tan

x = tan A

a = tan B , we get

FG 5 + 2 x IJ
H 2 5x K
F 5 + 2 x IJ
Solution: y = tan G
H 2 5x K
F 5+x I
JJ
G
y = tan G 2
GG 1 FGH 5 IJK x JJ
H 2 K
Let

tan A + tan B
1 tan A tan B

Hence, on putting ,

dy
1
1

=0+0+
=
dx
1 + x2 1 + x2

3. y = tan

tan A + B =

Problems on inverse circular functions


Exercise 10.1
1. Prove the following results by _ method.

1
d
(i) dx cos x =

(ii)

1
1 x

c x < 1h

1
d
1
, x
cot x =
2
dx
1+ x

1
d
(iii) dx cosec x =

1
x

x 1

c x > 1h

2. Find the differential coefficients of the following


functions w.r.t. their independent variables using
_ method.

472

How to Learn Calculus of One Variable


1

(iv) Log sin x

15. y = sin

Problems based on type 1:

af

F o r m : t 1 f x
1

l a fq

t 1 f x

l a fq

f t 1 f x
1

where t = sin / cos / tan / cot / sec / cosec


f = sin / cos / tan / cot / sec / cosec / log / e / .....

l a fq

1
f x
etc. in f t

14. y = tan

(i) sin x (ii) tan x (iii) sec x

17. y = sin

ea sin xj

16. y = tan

18. y = tan

1. y = sin x
1

2. y = tan

3. y = sin
4. y = cos

6. y = cos

9. y = tan

asec x + tan xf
e tan x j

23. y = tan

1
1

+ sin

e1 + x j

24. y = tan

25. y = sec

x + cos

j
y = tan asin x f
y = tan e sin x j
1

27.
28.

x
1
2

e
j
y = e tan 5x j
y = Lcos e x 2jO
MN
PQ

29. y = cot
30. y = tan

1 x

FG1 1 1 IJ
H x xK
e1 + e + 2 log xj
F x I
GG
JJ
a

x
H
K

1 2 2

1+x

26. y = sin x

F
GG
H

11. y = cos sin x

13.

22. y = cos

bg

tan x

tan

10. y = sin cos x

12.

10 x

7. y = tan 2 x
8. y = sec

21. y = sin

5. y = tan

20. y = e

ex e j
FG e IJ
H K
F x I
J
GG
H 1 x JK

19. y = tan 5 x

3x

Exercise 10.2

dy
Find
of the following functions.
dx

1
2 3

I
JJ
K

Differentiation of Inverse Trigonometric Functions

31. y = a cot

abx f
F 1 I OP
H xKQ

m tan

LM
N

32. y = cos a sin

9. y = sin

LM2 tan
N

10. y = cos

Type 2:
Problems based on substitution and change of form

af

11. y = cos

Form: t 1 f x
1

Where t = sin / cos / tan / cot / sec / cosec


and f ( x ) = algebraic function of x

dy
of the following functions.
dx

1. y = sin
2. y = sin

3. y = sin

4. y = sin

5. y = sin

6. y = sin

7. y = sin

8. y = sin

1 x

12. y = cos
13. y = cos

Exercise 10.3
Find

14. y = cos

15. y = cos

16. y = tan

18. y = tan

2
2

20. y = tan

I
JJ
K

1+ x
2

F 1 x I
GG
J
H 1 + x JK
F xx I
GG
J
H x + x JK
F 2x I
GH 1 x JK
F x I
GG
JJ
H a x K
LM 1 + x 1OP
MN x PQ
LM a + x + x OP
MN a + x x PQ
FG a x IJ
H 1 + ax K
2

19. y = tan

F a x
GG a
H
e2 x 1j

OP
Q

17. y = tan

1 x

F a x I
GG a JJ
H
K
e3 x 4 x j
F 2x a x I
GG a JJ
H
K
F a + x a xI
GH 2 a JK
FG x 1 x x 1 x IJ
H
K
F 2x I
GH 1 + x JK
F1 x I
GH 1 + x JK

1 x
1+ x

473

474

How to Learn Calculus of One Variable

21. y = tan

22. y = tan

23. y = tan

24. y = tan
25. y = tan

26. y = tan

F I
GG x x JJ
GH 1 + 23 JK

FG 1 + x IJ
H1 xK
F 3x x I
GH 1 + 3x JK
FG a + bx IJ
H b ax K

31.

32.

33.

xa
bx

34.

x + x

1 x2
35.
27. y = tan

x x

1+ x

28. y = tan

3
2

LM 3ax
MN a 2 x
2

36.

OP
PQ

37.

Hint: dividing Nr and Dr by a 2 , we have

OP
PP = tan
y = tan
PP
Q
LM 2 x + x OP
a
y = tan M a
= tan
2x x P
MN1 a a PQ
1

LM x
MM 3 a
MM1 2 x
N
a

29. y = tan

2
2

F 5 ax I
GH a x JK
2

LM 3a x x OP
MM a ea 3x j PP
Q
N
L 1 OP
y = tan M
MN x 1 PQ
F x I
y = tan G
GH 1 x JJK
F 4x I
y = tan G
H 4 x JK
L 1 + x 1 x OP
y = tan M
MN 1 + x + 1 x PQ
F 2 5x IJ
y = cot G
H 5 + 2x K
F 1 + x IJ
y = cot G
H1 xK
F x + 1I
y = sec G
H x 1JK
F 1 I
y = sec G
H 2 x 1JK
F1 + x I
y = cosec G
H 2 x JK

30. y = tan

LM x
MM 3 a
MM1 2 x
N
a

2
2

OP
PP
PP
Q

2x
1 x
+ tan
= ...etc.
a
a

38.

39.

Answers (under suitable restrictions on x):


1.

4.

1
1 x

2
2

a x

2.

5.

3.

a2 x 2
1

2 1 x

6.

3
1 x
1
1 x2

Differentiation of Inverse Trigonometric Functions

7.

2
2
8.
9.
1 + x2
1 + x2

1
10.

2
17.
1 + x2
1

19.

a x2
2

a x
2

1
a x
1
20.
1 + x2
2

1
1
1

21. 1 + x
2 x
1 + x2

23.
26.

31.

1
2 x

da

a 2 2 + 13 x 2
2

1 x2

+ 9x

i da

+ 4x

2. y = tan

2
1 + x2

3. y = tan

1 x

5. y = cot

6. y = tan

b x ag b x bg

1 x

30.

3a
a + x2

= tan

1+ x

7. y = tan
8. y = tan

9. y = tan

10. y = tan

Form: t 1 t x

11. y = tan

a
x
b

LM cos x sin x OP
N cos x + sin x Q
LM a b tan x OP
N a + b tan x Q
tan x + sec x

Where t = sin / cos / tan / cot / sec / cosec


t x = a trigonometrical function of x / a combination
of trigonometrical functions of x.

af

= x = tan

4
4 + x2

2
39.

Problems based on substitution and change of form


1

asec x + tan xf
acosec x cot xf
LM a cos x b sin x OP
N b cos x + a sin x Q

LM r asin cos x cos sin x f OP


N r acos cos x + sin sin xf Q
LM sin a xf OP = tan tan a xf
N cos a xf Q

y = tan

28. 1

Type 2 continued

af

1 cos x
1 + cos x

a
Hint: Put a = r sin and b = r cos = tan
b

1 x2
1
1
35.
2 36.
1 + x2
1+ x

FG a + b cos x IJ
H b + a cos x K
FG sin x IJ
H 1 cos x K

1
22.
1 + x2

33.

32.

2
38.
1 + x2

4. y = tan

x2 1
1

dy
of the following functions.
dx

1. y = cos

1
1
+
1 + x 1 + x2

x
1
34.
2
37.

18.

3
1
25.
2 24.
1+ x
1 + x2
2

1
1
1

27. 1 + x
2 x
1 + x2

29.

12.

2
2
15.
1 + x2
1 + x2

14.

1
x2 2

2 1
16.

11.

Find

1 x

13.

1 x

Exercise 10.4
2

1 sin x
1 + sin x

LM cos x OP
N1 + sin x Q

475

476

How to Learn Calculus of One Variable

12. y = tan

LM b tan xOP
Na Q

13. y = cot

14. y = tan

15. y = cot

16. y = sec

17. y = sec

18. y = tan

Exercise 10.5
Differentiate:

cosec x + cot x

asec x f
acosec xf
atan x f
LM a + b cos x OP
N b + a cos x Q

3.

4.

Answers (under suitable restrictions on x):


2

1.

5.

6. tan

sin x
1
2
12. 2
13.
14.
2
2
1 + cos x
a + b tan x
2
ab sec x

16.

1 + sin x
2

sin x

b a
17.
a + b cos x

18.

1 x

1+ x

1 2

w.r.t cos x

cos x

1
2

Differentiation of a function w.r.t. an other function

l a fq

1
Form: t f1 f2 x

l a fq

t t 1 f x

n b a fgs

1
g1 g2 x
w.r.t. t t

af

1
w.r.t. t t x

where t = sin / cos / tan / cot / sec / cosec


1

sin x cos x

Type 3:

F 2 x I w.r.t sin F 2 x I
GH 1 x JK
GH 1 + x JK
F 1+ x I
1+ x +1
J
w. r.t cos
8. tan G
GH x JK
2 1+ x
F 1 I w. r.t 1 x
9. sec G
H 2 x 1JK
F 2 x I w. r.t cos F 1 x I
10. tan G
GH 1 + x JK
H 1 x JK
F 2 x I w.r.t sin F 2 x I
11. tan G
GH 1 + x JK
H 1 x JK
F 1 x IJ w. r.t x
12. sin G
H1 + xK
7. tan

b a
1
1
1
1
2.
3.
4.
5.
2
2
2
2
b + a cos x

1
1
1
6. 1 7. 1 8. 1 9.
10.
11.
2
2
2

15.

2.

1 + cos x
1 cos x

e tan xj w.r. t e1 x j
F
I
tan G 1 + x xJ w. r.t tan x
H
K
L x + 1 1OP
cot M
MN x PQ w.r.t cot x
F 2 x I w.r. t sin F 1 + x I
2 sin G
GH 1 x JK
H 1 + x JK
cosec e cot x j w. r.t sec e tan x j

1. sec

t = sin / cos / tan / cot1 / sec1 / cosec1


f (x) = a function of x.

or,

477

Differentiation of Inverse Trigonometric Functions

13. tan

14. sin

15. tan

F 1 + x 1I
GG x JJ w.r. t tan x
H
K
F 2 x I w. r.t cos F 1 x I
GH 1 + x JK
GH 1 + x JK
F 1 x I
GG x JJ w.r.t cos x
H
K
2

16. tan

F
GG
H

1+ x 1
x

I
JJ
K

w.r.t sin

1
3.
2

F
GG
H

x
1+ x

I
JJ
K

1
1
4. 2 5. 1 6.
2
2
2
1
7. 1 8. 1 9. 4 10. 1 11. 1 12.
13.
1+ x
2
1
1
14.
15. 1 16.
.
2
2
Answers: 1. 1 2.

478

How to Learn Calculus of One Variable

11
Differential Coefficient
of Mod Functions

Differentiation of Mod Functions

af

f x

Definition:

b f a xfg

Formulas:
1.

a f f a xf ; for f a xf 0
af
n f b xg
f b xg =
f b x g ; f b x g 0 , n Q
f b xg
af

d
f x
dx

f x
f x

d
2.
dx

3.

4.

d x
x
=
;x0
dx
x
d x
dx

n x

a f f a x f which is the general formula


af
of (or, for) d.c. of mod function f (x) (at the point x
where f a x f 0 ).
f x

f x

Note: At the points where f (x) = 0, the value of

af

d f x

is to be found out by first principles. It may


dx
or may not exist at such a point because generally
mod functions are not differentiable at their roots
i.e. at the roots of | f (x) | = 0.
2.

bg

; x 0, n Q

af

f x

Proof:

af

d
f x
1.
dx

d
=
dx

af

f x

af

=z f x

=z

Now

f a x f LMQ f a x f =
f a x f OP
Q
N
d
f
x
a f ; when f a xf 0
1
=

dx
2 f a xf
d f a xf
f a xf
2 f a xf
=

=
f axf
dx
f a xf
2 f a xf
d
=
dx

, n Q

dx

d f x

n 1
dz
dz
= nz

dx
dx

af

d f x

af

af
f a xf

n f x

n 1

a f d fdxa x f
f axf
f a xf
f axf

=n f x

dx

=n f x

af

n 1

af

f x when f x 0

Differential Coefficient of Mod Functions

3. d x = d x
dx
dx

dx

for x 0
dx

2x

Find the d.c. of


1. y = | x3 |
Solution: y = | x3 |

2 x

2 x
=

Solved Examples

Problems based on algebraic functions

dy d x

=
dx
dx

2
2

2x x
2x

dx

a x 0f
Further, f b0g = 1 , f b0g = 1 where f (x) = | x |
f b0g does not exist.
x
=
x

4.

d x

d x
dx

=z

which means on differen-

n 1

=n x

n 1

d x
dx

bg

dx

af

d f x

3x

3x 2 x 3

; x 0 = 3x x , x 0

x3

af

dy
= 0 for x = 0
dx
2. y = | x |2 4 | x | + 2
Solution: y = | x |2 4 | x | + 2

x
x

af
f axf
f axf
f axf
f x

af

bg

d
x
dx

4 x +2

=2 x

.
2x
x

af

= 2x

d x

d x

af

f x for f x 0

=2 x

f x for f x 0

af

dy
d
=
dx dy

n x
x0
x
Further, f 0 = 0 for n > 1 where f x = x
Remember:
=

af

=n x

d f x

dx
dx

f + 0 = f 0 = 0

dx

af

, n Q

n 1 dz
dz
= nz
dx
dx

or,

Further,

x =z x
tiating

dx

d x

dx

4 x

dx
x

x
x

4 x
x

4 x

; x0
x
Further y' (0) does not exist.

bg

d 2
dx

,x0

479

480

How to Learn Calculus of One Variable

3. y = x | x |
Solution: Q y = x | x |

dy
d
=
x x
dx dx
= x

d x

= x

3. y = log | x |
Solution: y = log | x | which is defined for x 0
dx
dx

+ x

dx

ax 1f

For x 0;

x
+ x , x0
x

= x + x
=2 x ,x0

af

d log x d x

d x
dx
x
1

x
x

af

Also, f + 0 = f 0 = 0

Problems based on transcendental functions


Solved Examples
Find d.c. of
1. y = log | f (x) |
Solution: y = log |f (x) | which is defined for f (x) 0

bg

bg

bg
f b xg

1
f x

For x > 0,

dy
d
=
log f x
dx dx

d log f x
d

1
; x 0x
x
4. y = | log x |
Solution: y = | log x |, which is defined for x > 0
=

dy
= 2 x for all x.
dx

For f x 0 ,

bg

bg
f bxg

bg
f b xg
=
; f b xg 0
f bxg

Also f + 1 = lim

bg

=1

f x

bg

f 1 = lim

dy d log x 1 d x 1
=

For x 1 ,
dx
d x 1
dx

x 1 d
1

x 1
x 1
x 1 dx

h0

= lim

h 0

2. y = log | x 1 |
Solution: y = log | x 1 |; which is defined for x 1

log x 1
; for x 1
log x x

af

dx

dy d log x
=
dx
dx

d log x d log x

d log x
dx

d f x

f x

dy d log x
=
dx
dx

h0

log 1 + h log 1
;h>0
h

log 1 h log 1

log 1 h
h

= 1

dy
does not exist at x = 1
dx
5. y = | sin x |
Solution: y = | sin x |

dy
d
=
sin x
dx dx

;h>0

481

Differential Coefficient of Mod Functions

=
=

sin x d sin x
; sin x 0 , i.e.; x n

sin x
dx

h0

sin n + h sin n

h0

h>0

= lim

h0

= lim

h0

RSa2n + 1f UV = lim sin h = 1


h
2W
T
dy

does not exist for x = a2n + 1f


dx
2
f

= sin x cot x

a f

sin h

h 0

7. y = sin | x |
Solution: y = sin | x |

dy d sin x
=
dx
dx
=

sin h
h

=1

b g

f n = lim

h 0

FG sin h IJ , h > 0
H h K

dy
does not exist for x = n
dx
6. y = | cos x |
Solution: y = | cos x |

bg

cos x d cos x
; cos x 0

cos x
dx

cos x
sin x
cos x

h 0

8. y = cos

f 2 IK

(h > 0)

sin h
, h>0
h

F
H

cos n +

I
K

x
2

Solution: y = cos

x
2

d cos

x
2

dy

=
dx

Also, f +

h 0

af a

dy
does not exist for x = 0
dx

= cos x sin x ; x 2n + 1

LMa2n + 1f OP
2Q
N
F I
cos n + + h
H 2 K

h0

af

dx

x
x
2 d
cos
=

x dx
2
cos
2
cos

sin h sin 0
; h>0
h

= 1

f FH a

for x 0

f 0 = lim

=1

dy d cos x

=
dx
dx

cos x x

af

d sin x d x

; x0
d x
dx

Also, f + 0 = lim

= 1

= lim

=1

sin x
cos x
sin x

Also, f + n = lim

sin h

= lim

F
H

I
K

482

How to Learn Calculus of One Variable

FI
I HK b a
K

x
x
d
x
2
2
=
sin

; x 2n + 1
x
2
dx
cos
2

F
H

cos

sin
=

h 0

fa

1
dy
d
sin x
=
dx dx
1

d sin x d x
=

d x
dx
=

bxg

x
x 1 x

bg

f + 0 = lim

h 0

x
x

eQ x

= x2 = x ; x 0 , x < 1

=1

af

sin h
,h>0
h 0
h
= 1

dy
does not exist at x = 0
dx
The derivative is + at x = 1; at x = 1
12. y = | sin1 x |
Solution: y = | sin1 x |

dx
x

sin 1 h
;h>0
h

d x

; x 0, x 1

and f 0 = lim

dy d tan x

=
dx
dx
x

tan h
, h>0
h

Solution: y = sin1 | x | which is defined for x 1

x2
1; x 2
x2

dy
does not exist at x = 2
dx
10. y = tan | x |
Solution: y = tan | x |

dy
does not exist at x = 0
dx
11. y = sin1 | x |

f a f
x 2 cos x 2
a x 2f ; a x 2f

= sec

tan h
; h>0
h

d
x2
dx
x2 d x2
= cos x 2

; x2
x2
dx

fg

= cos x 2

= sec

h 0

; x0

= 1

dy d sin x 2
=
dx
dx

bg

Also, f + 0 = lim

bg

dy
Also,
does not exist for x = 2n + 1 .
dx
9. y = sin | x 2 |
Solution: y = sin | x 2 |

f 0 = lim

b a

= cos x 2

=1

x
x
sin
cos
2
2
=
; x 2n + 1
x
2 cos
2

fg

x
x
cos
2
2 1

x
2
cos
2

x sec

; x0

dy d sin x

=
dx
dx

Differential Coefficient of Mod Functions

sin

ja

sin 1 x

sin 1 x

1 x2

1
d
=

sin x ; x 0
1
dx
sin x

x
2

tan

; x 0, x < 1

dy
does not exist at x = 0 and the derivative is
dx
+ and at x = 1, 1 respectively.

1+

bxg

d
dx

=
2 sin

FH

x
2
=

;x0 Q x = x
2
x
1+ x

x 1+ x

= x

IK

FG
H

x
2

FG
H

IJ
K

Solution: y = log tan

x
2

IJ
K

F
H

dy
dx

which is defined for

dy d log log x
=
dx
dx

1
d

log x
log x dx

1
1

log x x

I LM
KN

af

a f OP
a fQ

f x
=
f x

af
af

d log x
f x
1
and
=
f x
dx
x

OP
Q

1
x log x

a + b tan x
16. y = log a b tan x
a + b tan x
which is defined for
a b tan x

Solution: y = log

d log f x
d
x
=

tan
Q
x dx
2
dx
tan
2
1

For x > 1,

x n

For x n ,

1
sin x

LMQ d log f axf


N dx

dy
does not exist for x 0
dx
14. y = log tan

x
x
cos
2
2

= cosec x
15. y = log (log | x |)
Solution: y = log (log | x |) is defined for | x | > 1

bxg

F xI d F xI
H 2 K dx H 2 K

x
2 1 1
=
2 x
x
2
sin
cos
2
2

1
dy
d
tan x

=
dx dx

cos

tan1

|x|
13. y =
Solution: y = tan1 | x |

sec

483

a b tan x i.e. a b tan x


2

For a b tan x ,
a + b tan x
dy
d
=
log
dx dx
a b tan x

484

How to Learn Calculus of One Variable

LMb sec
MM
N

aa b tan xf
aa + b tan xf
x aa b tan x f eb sec x j aa + b tan x f O
PP
aa b tan xf
PQ
2

ab sec x b sec x tan x + ab sec x +b sec x tan x


2

a b tan x
2 a b sec x
2

=e

c h

d d x
dx

a f

d x
dx

a f

x
x

af

d f x
dx

af
f a xf
f x

2
1

cos x

cos 45 cos x cos 45 sin x

cos x + 45

sin x

F
H

2 cos x +

I
K

Now differentiating both sides w.r.t. x, we get

x e
=
=
; x0
x
x
Remember: Rule to differentiate mod function
a cos + b sin . To differentiate mod of a function
a cos + b sin , we may adopt the following working
rule:
1. Express a cos + b sin (or, a sin + b cos ) as
a single cosine (or, single sine).
2. Use

2 cos x sin x

e
d x

=e

(ii) Use the A + B or A B formula as the case may


require.

dy
d
=
e
dx dx
=

1. y = | cos x sin x |
Solution: y = | cos x sin x |

Solution: y = e

acoefficient of cos f + acoefficient of sin f

Find the d.c. of

a b tan x

17. y = e

2
2
a + b which means to multiply and to divide the
given
expression
( a cos + b sin )
or
( a sin + b cos ) by

Solved Examples

Answer: (i) Multiply and divide the expression


( a cos + b sin ) or ( a sin + b cos ) by

afb af g

f x ; f x 0

Question: How to express a cos + b sin or


a sin + b cos as a single cosine (or, single cosine)

dy
d
=
dx dx

= 2

RS
T

F
H

2 cos x +

F
H

cos x +
dx
4

F I
H 4K
F I
d cos x +
H 4K

d cos x +
= 2

FG
H

= 2 cos x +

I
K

I UV
KW

F
H

d cos x +

F
H

d x+

I
K

I d Fx + I
K H 4K
dx

IJ RSsin FG x + IJ UV 1 1
K T H 4 K W cos FG x + IJ
H 4K

485

Differential Coefficient of Mod Functions

F
H

= 2

F
H

Solution: Q y = | sec x tan x | which is defined for

I sin F x + I
K H 4K
F I
cos x +
H 4K

cos x +

x n +

For x n =

I
K

af

0
4

for cos x +

Further, y is not differentiable at x when

F
H

dy d f x
=
dx
dx

I
K

=0
4
Or, alternatively,
by using general method
cos x +

af
f a xf

af

af

cos x sin x
d cos x sin x

cos x sin x
dx

when cos x sin x

f
f a
cos x sin x
=
acos x sin xf a1f asin x + cos xf
=

cos x sin x
sin x cos x
cos x sin x

i.e. x n =

af

d f x
dx

af

f x

sec x tan x
d

sec x tan x
sec x tan x dx

axf

a xf , this should be replaced by


d f axf
| f (x) | and then we should find
.
2

dx

Further y is not differentiable at x = n +

Refresh your memory: In calculus while differentiating a given function under the square root symbol,
when we simplify a given function under the square
root symbol and after simplification, we get

af

2. y = | sec x tan x |

Problems based on the substitution | f ( x) | =

f x

cos x sin x sin x + cos x


for tan x 1
cos x sin x

af
af

d f x
d f x

= (sec x) | sec x tan x |

f x , for f x 0

sec x tan x
asec x tan xf esec x tan x sec xj
sec x tan x a sec x f asec x tan x f
=
asec x tan xf

af

f x

,
2

dy d f x
=
dx
dx
=

af
f a xf
f x

dy d cos x sin x
=
dx
dx
Now, putting (cos x sin x) = f (x)

Solved Examples
Find the d.c. of
2

1. y = 1 cos x

Solution: y = 1 cos x =
y = sin x

dy d sin x
=
dx
dx

sin x

486

How to Learn Calculus of One Variable

sin x d sin x

sin x
dx

sin x

= sec x tan x

cos x , x n , n Z

sin x

y' (x) does not exist for x = n .


Precaution: It is a common mistake to write down
2

y = 1 cos x =

Solution: y = 1 sin x = cos x = cos x

, n Z
2

1 sin x
1 + sin x

Solution: y =

1 sin x
, defined for
1 + sin x

a1 sin xf a1 sin xf = a1 sin xf


a1 + sin xf a1 sin xf 1 sin x
a1 sin xf
2

cos x

2 cos

x
2 =

cos

.
2

x
x
= cos
2
2

dy
d
x
=
cos
dx dx
2
x
d
x
2
cos
=

x dx
2
cos
2

sec x tan x
2
sec x tan x sec x
sec x tan x

cos

x 2n + 1 +
2
Rationalizing the denominator, we get
=

sec x tan x
d

sec x tan x
sec x tan x dx

FG 1 + cos x IJ
H 2 K
F 1 + cos x IJ
Solution: y = G
H 2 K

cos x
cos x

= tan x cos x , x n +

4. y =

cos x d cos x

cos x
dx

= sin x

asec x tan xf

= sec x sec x tan x , x n +

dy d cos x

=
dx
dx
=

j
f e
sec x tan x
=
asec x tan xf asec xf asec x tan xf
=

2. y = 1 sin x
2

dy
d
=
sec x tan x
dx dx
=

sin x = sin x

dy
= cos x which is completely wrong.
dx

3. y =

FG 1 sin x IJ
H cos x K

x
2
=
x
cos
2
cos

F
H

I
K

F sin x I d F x I
H 2 K dx H 2 K

Differential Coefficient of Mod Functions

sin
=

6. | x | + cos x
7. sin x | x |

x
x
cos
2
2 1

x
2
cos
2

x
x
cos
2
2
, x 2n + 1
x
2 cos
2

sin
=

8.

sin x
x

9.

x
sin x

10.

5. y = 1 + sin 2 x 1 sin 2 x
11.

Solution: y = 1 + sin 2 x 1 sin 2 x


y=

acos x + sin xf

acos x sin x f

12.

y = cos x + sin x cos x sin x


d cos x sin x
dy d cos x + sin x

dx
dx
dx

13.

bcos x + sin xg bcos x sin xg + bcos x sin xg


asin x + cos xf
bcos x sin xg cos x + sin x +
=
bcos x + sin xg
bsin x + cos xg cos x sin x ,
bcos x sin xg

14.

cos x + sin x

cos x sin x

, n Z
4
Problems based on differentiation of mod of a function
for x n

Exercise 11.1
Find the differential coefficients of the following
functions.
1. | 5x + 3 |
2. | x2 a2 |
3. | x + 1 |
4. | x 1 |
5. x + | x |

x1
x1
x
x

a x 2f

x2

x x1
x1

1
x

15. x | x |
16. log | x |
17. | log x |
1 + cos 2 x

18.

2 cos x

x
19.

1 cos x

20.

1 + sin x 1 sin x

21.

1 + cos 2 x
1 cos 2 x

22.
23.
24.
25.
26.
27.
28.
29.
30.

log | sin x |
log | cos (ax + b) |
| sin x |
| cos x |
| tan x |
| cot x |
| sec x |
| cosec x |
sin | x |

487

488
31.
32.
33.
34.
35.

How to Learn Calculus of One Variable

cos | x |
tan | x |
cot | x |
sec | x |
cosec | x |

15. 2 x

5 5x + 3
5x + 3

2x x a

ex

2.

3.
4.

x+1
x+1

sin x
cos x
sin x

25.

cos x
sin x
cos x

26.

tan x
2
sec x
tan x

x1
x1

27.

cot x
2
cosec x
cot x

28.

sec x
sec x tan x
sec x

29.

cosec x
cosec x cot x
cosec x

f
f

5. 1 +
6.

x
sin x
x

7. cos x

9.

31. sin x

x
2

x x

32. sec

sin x x x x cos x

a f

x sin x

la x 2f x

33. cosec

+ x x

x2 x x

x2

x 1 2x 1 x x 1

x
x

x
x
x
x
x

x
x

34. sec x tan x

14.

11. 0

13.

30. cos x

10. 0

12.

x x cos x sin x x
8.

24.

Answers (with suitable restrictions on x).


1.

23. a tan ax + b

x 1

x2

35. cosec x cot x

x
x

FG x IJ
H xK

Change of Form before Differentiation


In certain cases the given function can be reduced
into a simple form before it is differentiated so that
process of differentiation becomes simple and easier.
Notable cases are those of fractions whose

489

Differential Coefficient of Mod Functions

denominator is a surdic quantity, which are simplified


by rationalizing the denominator.
Type 1
Form:

a f g axf
f a xf m g a x f

Solution: y =

2.

3.

4.

5.

j
ax + af ax af
x+a +

Working rule: To find the d.c. of the forms mentioned


above, we rationalize the denominator.

f x
1

Where t = sin / cos / tan / cot / sec / cosec


f (x) = a quotient of trigonometrical functions of x/
algebraic function of x which can be reduced into a
simple and easy form by simplification or substitution
before differentiation.

x a

x+a

xa

IJ
K

2. y =

x a

x a

x +a

x a

x a

x +a
x +a

x +a + x a
2

x +a

1
2

2
2
dy 1
d
=
1+
x a
dx a
dx

x 2 > a2 > 0

xa

RS e
UV
j
T
W
U|
dy 1 R
1
|

= S1 +
2xV ,
dx a |
T 2 x a |W
U|
R
dy 1 |
x

= S1 +
V for x > | a | > 0.
dx a |
T x a |W

Solved Examples on rationalization

x+a +

Solution: y =

1. y =

2a

N.B.: The second type has been explained in the


chapter of d.c. of inverse circular functions.

Find the d.c. of the following

xa

x+a + xa+2 x a

FG x +
H
=

N.B.: Irrational function is always rationalized by


substitution or rationalization while finding limit/d.c./
integral/value of the function at a particular point/
etc.

af

x+a + xa

, defined for

x+a + xa

x+a xa

e
=

If numerator and denominator are irrational


functions of x, rationalization helps to get the d.c. in
easy way.

Type 2: t

xa

[Rationalizing the denominator]

a f g axf
f a xf g a xf
f a xf m g a xf
f a xf
f a xf + a f a x f a
f a xf
f a x f f a xf
f x

x+a

x+a + xa

xa

x>|a|>0

f x
1.

x+a +

x +a x a

2
2

, defined for

x +a + x a
x +a + x a

490

How to Learn Calculus of One Variable

ex
=

je
ex

2x + 2 x a

x a
a

a2

x a

a2

ex

1
2a 2

dy 2 x
=
+
dx a 2
2

3. y =

x+
x

x +a
2

x +a

d 1i
1
2

4x

FG
H

dy 2 x

=
+
2
dx a 2
a

x a

2x

FG
H

d 4
x a4
dx

IJ a
K

Solution: y =

x +a

x +a

IJ
K
IJ
K

IJ
K
ex a j
2

x +a
2

x2 + x2 + a2 + 2 x x2 + a2
x2 x2 a2
2x2 + a2 + 2x x2 + a 2

RS FG 2 x
T Ha

dy 2
d
= 2 2x + 0 +
dx a
dx

dy 4 x 2
=

dx a 2 a 2

F
G 4x + 2
= G
GH a a
2

x2 + a2

x2 + a2 +

FH

x2 + a2

1 2x
2 x2 + a2

2x
x2 + a2

IJ UV
KW

2x
a2

I
JJ
IK a J
K
2

x a

IJ
K

for x2 > a2.

4. y =

1
2

Solution: y =

2
2

x +a
x +a

x +a +

x+

IJ FG x +
K H
I F
+a J Gx +
K H
2

a 2
Now, differentiating both sides w.r.t. x.

dy

dx
=

a
a
Now, differentiating both sides w.r.t. x.

2x

x +a

FG x +
H
=

x +

FG x +
H
=
FG x
H

x +a x +a

2a
=

2x + 2 x a

j
+ a j ex

+ a2 + x2 a2 + 2 x2 + a 2 x2 a 2

1
2

x +a +

x +a
2

x +b

a b

x +b

x +b

[Rationalizing the

denominator]
Now, on differentiating both sides w.r.t. x.

Differential Coefficient of Mod Functions

LM e
j N

dy
1
1 2
2
=

x +a
dx a 2 b 2
2

1 2 2
x +b
2

c 1h
1
2

c 1h

1
2

d 2 2
x +b
dx

LM
e j MN
L 1
x
=
M
ea b j MN x + a

jOPQ

dy
1
2x

= 2 2

2
2
dx
a b
2 x +a

5. y =

a+x

ax

a+x +

ax

Solution: y =

2x
2

x +b
1

a+x

x +b

OP
PQ

OP
PQ

ax

a+x +

ax

a x

x 1

x +1

x 1

x+1+
x +1

x1

j e
2

x1

FG
H

2
2

= x+

x 1

dy
=1+
dx

2 x 1
2x

=1+

IJ
K

IJ
K

x 1

7. y =

a 2 x f
a x

1
x

a x

OP
PQ

=1+

x 1

, x > 1.

x
x+2 +

x2
x

Solution: y =

d 2
x 1
dx

2 x 1

[Rationalizing the

LM
MN

2x + 2 x 1
x +1 x +1

2 x+

dy
a
1
= 2

dx
2x
x

, defined for x > 1

, defined for | x | < a.

2x
a

x +1 +

x 1

2a 2 a x
2a 2 a x
=
a+ xa+ x
2x
2

x +1

x2

OP
PQ , x 0 .

x + 1 + x 1 + 2 x 1
=
x +1 x 1

FG
H

x 1

a2 x2

x +1 +

a2 x2

denominator]

ax

2 a a x

x2

Solution: y =

, a > 0.

a+x

L
+M
MN

6. y =

j e a + x a xj , x 0
e a + xj e a xj
a + x + aa x f 2 a + x a x
=
a + x aa x f
=

d 2 2
x +a
dx

491

x+2 +
x

x2

x+2

x+2

j e
2

, defined for x > 2

x2
x2

492

How to Learn Calculus of One Variable

x2

LM
N

d
dx

...(i)*

LM F
MN x GH 2

1
4

FH a a
F
x Ga +
H
=

je

x+ 2 x 2 +

x+2 x2

1
x +2

I+e
J
x2 K
1

FG
H

x+2 x2
2 x

x + 2x x

jOP ,
PQ

I
2x J
K

LM a f a f OP
MN
PQ
L
O
2 x 2f P
a
dy 1 1 M a2 x + 2f

P
dx 4 2 M
+
x
2
x
x 2x Q
N
L
a2 x 2f OP
1 a2 x + 2 f
= M

8 M
N x + 2 x x 2 x PQ
L
bx 1g OP for x > 2 .
1 b x + 1g
= M

4 M x + 2x
x 2 x PQ
N
1 2x 2
dy 1 1 2 x + 2
=

2
2
dx 4
2 x + 2x
2 x 2x

x
a2 x2

, defined for x2 < a2

dy
dx

a2 x2

IK FH a +
I
x J
K

x2
a

a x
x

IJ FG a +
K H
x

x
2

LM x
N
=

a a x

a x

FH

a a + a 2 x 2
x

,a 0
9. y =

a x
2

+ x

x a
x +a

IJ
K

OP
PQ

a a x a + x
x

x
2

IJ dx
K dx

a x

2 x a + a x
x

LM
MN
=

a x

a f FGH

21

IK , x 0

FG
H

a2 x2

2
2
d
a + a x
dx

1 2
2
a x
2

IK

a+

1
* Or, alternatively (1) y =
4

Note:

8. y =

FH

a a + x

for x > 2

a a x

x a+

y=

OP
Q

Now rationalizing the denominator, we have

dy
dx

1
4

Solution: y =

d
x
dx
=

x+2

ax + 2f ax 2f
x e x +2 x2j

IK
; x 0, x

OP
Q

< a2

Differential Coefficient of Mod Functions

x a

Solution: y =

FG
H

FG
H
2

x +a
2

IJ FG
KH

2
a

I
xJ
K
2

IJ
K

x
x
2
=
= tan
x
2
cos
2

2x

tan

x
tan
1
x
2
= sec 2
; x n, n Z
2
2 tan x
2

Note: (i) This problem can be done by substitution


method also but that method becomes lengthy.

Type 1: Whenever 1 cos x appears under the


radical sign
, we always express the function
within the radical as a square of some function.
Type 2: Wherever 1 sin x appears under the
, we always express the function
radical sign
within the radical as a square of some function.
N.B.: The above method may be remembered as
expressing the function within the radical as a square
of some function.
Solved Examples
Find the d.c. of the following.

1 cos x
1 + cos x

Solution: y =

x
2 x 1
2
=
sec

x
2 2
tan
2

OP
PQ

Problems based on Trigonometical Transformation

1. y =

dy
d
x
=
tan
dx dx
2

sin

x
2
2 x
cos
2
sin

2
2

LM
1
MN4 x 2 x + a 2 x 2 x + a
LM 2 ex + a j + 2 x OP
MM4 x x + a PP
Q
N
LM 2 x + a OP
MN2 x x + a PQ for x > a .
2

x +a x
2

, defined for x2 > a2

2x + a 2x x + a

dy
dx
1

+ x

x +a + x
2

x +a

493

1 cos x
defined for x 2n + 1
1 + cos x

y=

1 cos x
1 cos x
= u where u =
1 + cos x
1 + cos x

We are required to find


1 c 12 1h du
1
du du 2
du
=
= u

2
dx
dx
dx 2 u dx
1

(ii) This problem also can be done by logarithmic


differentiation.

F 1 cos x IJ = 1 log FG 1 cos x IJ


log y = log G
H 1 + cos x K 2 H 1 + cos x K
F 1 cos x IJ
2 log y = log G
H 1 + cos x K
= log a1 cos x f log a1 + cos x f
1
2

494

How to Learn Calculus of One Variable

d log y d
d
= log 1 cos x log 1 + cos x
dx
dx
dx

F
H

2 cos

I
K

x
2

x
x
cos
2
2
x
2 cos
2

x
1
1
2 x
2
sec

x
2 2
tan 1
2

F
H

I
K

x
tan 1
x
2
sec
=

;
x
2
2 2
tan 1
2

1 sin x
, defined for x 2n + 1
1 + cos x

x
x
sin cos
2
2

sin

y=

x
x
cos
2
2
x
cos
2

tan

x
1
2

dy
1 d
x
=
tan 1
dx
2
2 dx

F
H

I
K

FG x a2n + 1f and x F 2n + I IJ
H 2 KK
H
3. y = 1 sin x
Solution: y = 1 sin x

F sin x cos x I
H 2 2K

= sin

dy

=
dx

x
x
cos
2
2

sin
=

IJ
K

tan

1 sin x
1 + cos x

Solution: y =

FG
H

IJ
K

1 cos x
; x n
1 + cos x

sin x

x 2n +

FG
H

dy
2
y
y
=
=
dx sin x 2 sin x

2. y =

sin x
sin x
1 dy
=
+
y dx 1 cos x 1 + cos x

F 1 + cos x + 1 cos x I = sin x 2


= sin x G
H 1 cos x JK
sin x

x
1
d
x
2

tan 1 ,
x
dx
2
2
tan 1
2
tan

d sin

x
x
cos
2
2
dx

x
x
cos
1
x
x
2
2
=
sin + cos
x
x
2
2
2
sin cos
2
2
sin

F
H

1
=
2

x 2n +

I
K

F
H

F sin x + cos x I sin x cos x


H 2 2K 2 2 ;
F sin x cos x I
H 2 2K

.
2

I
K
for

495

Differential Coefficient of Mod Functions

4. y = 1 + cos x

tan y = tan

Solution: y = 1 + cos x

x
2 cos
=
2
2

d cos

dy

=
dx

x
2 cos
2

d tan y

=
dx

x
2

F
H

I
K

x
1
x
2
= 2 sin
2
2 cos x
2

N.B.: The above problem also can be done by using


the chain rule for u , where u = given function
x = f (x) or by using logarithmic differentiation.

= tan

y = tan

x
2 x
sec
2
2
=

x
2 x
tan
2 1 + tan
2
2
tan

x
cos
1
x
2
=
sin
, x 2n + 1
x
2
2
cos
2

F
H

= tan

x
1
2
=
, x n
2 tan x
2
or, alternatively,

x
2 ; x 2n + 1
2 x
2 cos
2
2 sin

tan

1 cos x
1 + cos x

R|SQ tan x
I T| 2
K

x
2 x
tan
sec
dy
2
2

2 x
x
dx
tan
2 sec
2
2

1 cos x
1 + cos x

Solution: y = tan

dx

x
2 x
tan
sec
dy
2
2

x 2 sec 2 y
dx
tan
2

cos

x
2

x
x
tan
sec 2
dy
2
2
, x n
sec y
=

x
dx
2
tan
2

x
x
1
2
= 2
sin

x
2
2
cos
2

5. y = tan

d tan

dx

cos

x
2

R| 1sin x U|
S| x2 V| = tan RST tan 2x UVW
T| cos 2 W|
1

tan y =

1 cos x
1 + cos x

log tan y = log

1 cos x
, x n
1 + cos x

FG
IJ
H
K
1
= log a1 cos x f log a1 + cos x f
2
=

1 cos x
1
log
2
1 + cos x

x
2

U|V
W|

496

How to Learn Calculus of One Variable

LM
N

d tan y 1 sin x
sin x
1

=
+
tan y
dx
2 1 cos x 1 + cos x
=

LM
MN

sin x 1 + cos n x + 1 cos e x


2
2
1 cos x

OP
PQ

dy sin x
1
2
2
sec y
=

2
dx
tan y
2
sin x

1 + tan y dy
1

=
tan y
dx sin x

OP
Q

dx

y = log

1 cos x
1 + cos x
sin x

1 cos x 1 + cos x

1 + cos x
2 sin x

af
af
g a xf
, where f
g axf

g1 x
g2 x
1

1 + sin x
,
1 sin x

1 + cos x
,
1 cos x

1 cos x
,
1 sin x
is

logarithmic

1 + sin x
,
1 sin x

cot1, sec1, and cosec1.

1 + sin x

, defined for x n +
1 sin x
2

1
log 1 + sin x log 1 sin x
2

1 cos x
, x n
dx
2 sin x
1 + cos x
Note: The first method is more simple than the second
method. But a general method to find differential
coefficient of

2. y = f

1 cos x
1 sin x

f a
fq
l a
acos xf UV
dy 1 R cos x

= S

dx 2 T1 + sin x 1 sin x W
cos x
cos x U
1R
= S
+
V
2 T 1 + sin x 1 sin x W
R|1 sin x + 1 + sin x U|
1
= cos x S
V
2
T| 1 sin x W|
=

FG 2 IJ
H 1 + cos x K

1 cos x
1 + cos x

1. y = f

1 cos x
,
1 m cos x

Solution: First method:

FG 2 sin x IJ
H 1 + cos x K
dy b1 + cos x g

1 sin x
,
1 + sin x

6. y = log

dy
1

=
1 cos x dx sin x
1 + cos x

dy
=
dx

1 sin x
, f
1 cos x

5. y = 1 sin x , 1 cos x
differentiation.

1
sin x

2
1 + cos x

1 sin x
, f
1 m sin x

1 sin x
,
1 cos x

FG 1 cos x IJ
H 1 + cos x K dy =
1 cos x
1 + cos x

4. y =

1+

3. y = f

1
2
1

cos x
=
, x n +
2
2
2
cos x cos x

Second method
1 + sin x
=
1 sin x

1=

sin1, cos1, tan1,

x
x
+ cos
2
2
x
x
sin cos
2
2
sin

Differential Coefficient of Mod Functions

497

Problems based on Change of Form before


Differentiation

x
x
+ cos
1 + sin x
2
2
y = log
= log
x
x
1 sin x
sin cos
2
2
sin

Exercise 11.2
Find the differential coefficients of

x
x
x
x
= log sin + cos log sin cos
2
2
2
2

1
1.

x+a +

x+b

F 1 cos x 1 sin x I
F sin x + cos x I H 2 2 2 2 K 2. a1 + x f + a1 x f
H 2 2K
a1 + xf a1 x f
1
F 1 cos x + 1 sin x I

ea + x j + ea x j
x
x
F sin cos I H 2 2 2 2 K
H 2 2K
3.
ea + x j ea x j
R F cos x sin x I F cos x + sin x I U|
H 2 2K V
x
1 |H
2
2K
4.
= S

x
xI
x
x
2 |F
F
I
1 a x
|T H sin 2 + cos 2 K H sin 2 cos 2 K ||W
x
R F cos x sin x I F cos x + sin x I U
5.
x+2 + x2
H 2 2 K |V
1 |H
2
2K
= S
+
x
xI
2 |F
F x xI |
x
T| H sin 2 + cos 2 K H cos 2 sin 2 K W|
6.
x +2 1
R| F cos x sin x I + F cos x + sin x I U|
G 2 2 JK GH 2 2 JK |
aa + bxf + aa bxf
1 |H
= S
7.
V
2 |
F x cos x IJ
aa + bxf aa bxf
||
G sin
H 2
|T
2K
W
2+ x + x
R|1 2 sin x cos x + 1 + 2 sin x cos x U|
8.
1
2
2
2
2
2+ x x
= S
V|
2 |
cos x
W
T
LM F 2 + x + xI
1
2
1

=
=
, x n +
MMHint: H 2 + x x K = 2 + x + 2 x 2 2 + x
2 cos x cos x
2
MN
Remark: The above problem has not been solved
by logarithmic differentiation.

dy
=
dx

2
2

Answers (with proper restrictions on x)


1.

LM
f MN

2 ab

1
x+a

OP
x + b PQ
1

+ x2

OP
PP
PQ

498

How to Learn Calculus of One Variable

2.

3.

1 x

2a
x

R|
S|1 +
T

x
a

2. y = tan

a x

U|
V|
W

3. y =

6.

7.

1.

1 + sin x
1 sin x

asin x + cos xf acos x sin xf asin x cos xf


asin x + cos xf
asin x cos xf
acos x + sin xf
x
x
x
if tan > 0 , then tan
2
2
2
x dy 1
= tan
=
2 dx 2

2. y =

1 cos x
1 + cos x

Answers (with proper restrictions on x)

4. Find
5. Find

U|
R|
1

V|
S1 +
x |
+
1
x
|T e j |W
R
a |
a
1+

S
bx |
T a b x

U|
V|
W

Exercise 11.3
Differentiate the following functions.
1. y = 1 + sin 2 x 1 sin 2 x

x
dy 1
2
or,
, x n
=
dx 2 tan x
2
x
tan + 1
1
2 x
2
sec

3.
x
2
2 2
tan + 1
2
tan

F
H

f
I
K

Implicit Differentiation

499

12
Implicit Differentiation

Firstly, we recall the basic definitions in connection


with implicit differentiation.

Notation: The notation for the implicit function is


either

1. Explicit function: Whenever it is possible to


equate the dependent variable y directly to a function
of the independent variable x as y = f (x) , we say that
the dependent variable y is an explicit function of the
independent variable x. Here f (in the equation y =
f (x)) stands for all elementary functions (sin, cos, tan,
cot, sec, cosec, sin1 , cos1 , tan1, cot1, sec 1,
cosec1, log, e, ( )n, n , | |, etc.). examples,

1. F (x, y) = c, c being a constant, or, (Note: Implicit


function is also defined in the following way: If the
dependence of the dependent variable y on the
independent variable x is expressed by the equation:
F (x, y) = c or F (x, y) = 0 not solvable for y, then y is
called an implicit function of x.)
2. F (x, y) = 0 where F (x, y) denotes
(i) A rational integral function of x and y, i.e. it is the
sum of a finite series of the terms Cm,n xm, yn, where m,
n may have the values 0, 1, 2, 3, .
(ii) A function (sin, cos, tan, cot, sec, cosec, sin1,
cos1, tan1, cot1, sec1, cosec1, log, e, ( )n, n , | |,
etc.) of a rational integral function of x and y.
(iii) A combination of (i) and (ii) and the equation F
(x, y) = c or F (x, y) = 0 denotes the dependence of the
dependent variable y on the independent variable x
not solvable for y.

(i) y = x2 1
(ii) y = sin x3, etc.
N.B.: All algebraic, trigonometric, inverse
trigonometric, logarithmic and exponential functions
of xs are explicit functions of xs.
2. Implicit function: Whenever it is not possible to
equate the dependent variable y directly to a function
of the independent variable x as y = f (x), we say that
the dependent variable y is an implicit function of the
independent variable x.
Examples:
(i) x3 y4 = (x + y)7
(ii) x y = c
(iii) xy + x2 y2 = c
(iv) xm ym = (x + y)m + n
(v) y = cos (x y)
(vi) y = tan (x + y)
(vii) cot xy + xy = 3
(viii) sin (x + y) + sin (x y) = 1

Remember:
1. The equation F (x, y) = c or, F (x, y) = 0 determines
one or more values of y to be associated with the given
value of the independent variable x provided a definite
number from some domain is substituted for x.
2. If the set {(x, y) | F (x, y) = c} or, {(x, y) | F (x, y) = 0}
is the graph of a function (or, union of graphs or more
than one function), we say that the equation F (x, y)
= c or F (x, y) = 0 defined implicitly y as a function x.
Further we should note that if y = f (x) is a function of
x, then F (x, y) = F (x, f (x)) = c or F (x, y) = F (x, f (x))
= 0 is an identity, i.e. F (x, f (x)) is a constant function.

500

How to Learn Calculus of One Variable

Kinds of Implicit Function


1. Implicit algebraic function: y is said to be an implicit
algebraic function of x if a relation of the form:
Ym + R1 Ym 1 + + Rm = 0 exists, where R1, R2, ,
Rm are rational functions of xs and m is a positive
integer. e.g.,
(i) y2 2xy + x = 0
2

(ii) y

x
y +1= 0
x 1

2. Implicit transcendental function: y is said to be an


implicit transcendental function of x if a relation of
the form: T (x, y) = 0 exists, where T denotes
trigonometric, inverse trigonometric, logarithmic and
exponential functions and the ordered pair (x, y)
denotes a rational integral function of x and y. e.g.,
(i) tan (x, y) = 0
(ii) cos (x + 2y) = 0
(iii) sin (a + y) = 0
(iv) ey x = 0
(v) tan1 (x + y) = 0
(vi) log (xy) = 0
Question: What is implicit differentiation?
Answer: Differentiation of an implicit function is called
implicit differentiation, or more explicitly it is defined
dy
of an implicit function
as Finding the derivative
dx
put in the form F (x, y) = 0 or F (x, y) = c without
explicitly determining the function y = f (x) is called
the implicit differentiation.
N.B.: Whenever we differentiate an implicit function
put in the form F (x, y) = c or, F (x, y) = 0, we assume
that the given implicit function is differentiable and y
is a differentiable function of x.
Facts to Know:
1. Whenever we differentiate the algebraic implicit
function of x (or, the rational integral function of x
and y), we simply need do is to differentiate each term
of it with respect to x remembering that y is itself a
differentiate function of x, say g (x) and the derivative
of any function of y, say y with respect to x is
equal to its derivative with respect to y multiplied by

af

af

af

dy
d y
d
dy
=
y
, i.e.
dx
dx
dx
dx

dy
dy
dy
, provided y is a
=

dx
dx dx
differentiable function of x.
2. The derivative of a function (sin, cos, tan, cot, sec,
cosec, sin1, cos1, tan1, cot1, sec1, cosec1, log, e,
( )n, | |, etc.) of a rational integral function of x and y
with respect to the independent variable x is equal to
the derivative of the whole given implicit function
with respect to the rational integral function of x and
y times the derivative of the rational integral function
of x and y with respect to the independent variable x
Hence,

a f

a f a f
a f

dT x, y d x, y
d

T x, y =
dx
d x, y
dx

only, i.e.

pro-

vided denotes sin, cos, tan, cot, sec, cosec, sin1,


cos1, tan1, cot1, sec1, cosec1, log, e, ( )n, | |, etc
and the ordered pair (x, y) denotes the rational integral function of x and y. e.g.,
(i)

(ii)

a f a
a f
F dy I
= cos a x + y f 1 +
H dx K
d log a xy f d a xy f
d

log a xy f =
dx
d a xy f
dx
1 F dy
I
=
+ y , xy > 0
x
K
xy H dx
a

d sin x + y d x + y
d
sin x + y +

dx
d x+ y
dx

3. Question: When would you differentiate as an


implicit function?
Answer: (i) When it is neither convenient nor
possible to find y (dependent variable) in terms of x
only, i.e. if it is convenient or impossible to write y =
an expression in x only or y = f (x), where f stands for
sin, cos, tan, cot, sec, cosec, sin1, cos1, tan1, cot1,
sec1, cosec1, log, e, etc. then we differentiate the
given function as an implicit function of x. e.g.,
x3 + y3 + 3axy = 0
sin (x2 + y2) = y
sin

FG x IJ = y
H yK

log (x + y) = y

Implicit Differentiation
2

x + xy

= y

Step 1: Take

x log y = y, etc.
(ii) Whenever we would like to find out the derivatives
of an implicit function of x without solving its given
equation for y, we differentiate the given relationship
between the variables x and y as an implicit function
of x. e.g.,
x 2 + y 2 = a2
tan (x + y) = 0

F xI
log G J = 0
H yK
x
y

e = 0 , etc.

4. In the implicit function, y is said to be defined


implicitly as the function of x or x is said to be defined
implicitly as a function of y.
5. An implicit function expresses always an unsolved
relationship between the variables. e.g.,

OP
P are functions expressing unsolved
x + y =a P
P relationship between variables.
Z + y tan x = aP
PQ
o
x
= tan 60
y
2

6. An explicit function expresses always a solved


relationship between the variables. One variable is
solved in terms of the other. e.g.,
o

y = tan 60 y is an explicit function of x.

x=

y
tan 60

x is an explicit function of y.

d
dx

af

af

dF y
d F y dy
=

dx
dy
dx
which means wherever with respect to the
independent variable x, we differentiate a differentiable
algebraic, trigonometric, inverse trigonometric,
logarithmic and exponential function of y being
preassumed (or, understood) to be a differentiable
function of x, we should multiply the differential
coefficient of the differentiable function of y with
dy
.
respect to y by
dx
dy
Step 3: Collect the terms involving
on the left
dx
dy
hand side and the terms without
on the right
dx
hand side.
dy
Step 4: Finally, solve the equation for
.
dx
dy
Notes: 1. The final result for
is an expression in
dx
terms of both x and y.
2. While finding the derivatives of an implicit
algebraic function of x, often required derivatives

af

Problems on Implicit Algebraic Functions


To find the derivative of an implicit algebraic function
of x without solving its given equation for y, we adopt
the rule which consists of following steps.

following.
(i)

(ii)

af

dF y
d F y dy
are the
=

dx
dy
dx

obtained from

Z = a y tan x Z is an explicit function of


y and x.
The dependent variable is therefore the value of
the explicit function.

on both sides of the given

equation.
Step 2: Differentiation each term of the given equation
with respect to the independent variable x using the
rules for the derivatives of sum, difference, product,
quotient, composite of differentiable functions and a
constant multiple of the differentiable function
remembering that

x = a y x is an explicit function of y.

af

501

n 1 dy
d n
y ny
dx
dx

n
n
d
dx
dy
+
x y =
dx
dx
dx

= nx

n 1

+ ny

n 1

dy
dx

502

How to Learn Calculus of One Variable

n dy
n dx
d n n
(iii)
+ y
x y = x
dx
dx
dx

= nx y

d
dx

(iv)

n 1

Fx I =
GH y JK

n 1 n
dy
y
+ nx
dx
n

ny

n 2

nx

a y nf

n 1

n 1

e
j af
d
d

xy f +
a
ex y j = 0
dx
dx
F dy + y dx I + FG x dy
x
H dx dx K H dx

dy
if
dx
1. y8 5x2 y6 + x8 = 11
Solution: y8 5x2 y6 + x8 = 11

b g

d
d
11
x 8 5x 2 y 6 + y 8 =
dx
dx

d 8
d
d 8
5x 2 y 6 +
x
y =0
dx
dx
dx

8 x 10 x y 30x y

e j

e
e

2. x2 + y2 = a2
Solution: x2 + y2 = a2

e j

d 2
2
d 2
a
x + y =
dx
dx

dx
dy
+
=0
dx
dx

8x 7

+ y

dx
dx

I=0
JK

dy
dy
+ y + 2x2 y
+ y2 2x = 0
dx
dx

FG dy + 2 x y dy IJ = e y + 2 x y j
H dx
dx K
dy
e x + 2 x yj
= ey + 2x y j
dx
x

j
j

a
a

f
f

y 1+ 2 x y
dy y + 2 xy
y

=
=
=
2
dx
x
1
+
2
x
y
x
x + 2x y

4. x5 + x4 y2 y = 4
Solution: x5 + x4 y2 y = 4

dy
7 dy
+ 8y
=0
dx
dx

j dydx = 10x y

x 5y 6 4 x 6
dy

=
dx
y 5 4 y 2 15x 2

8 y 7 30 x 2 y 5

d
d
2 2
xy + x y =
c
dx
dx
2

Find

dy 2 x x
=
=
2y
dx
y

3. xy + x2 y2 = c
Solution: xy + x2 y2 = c

dy
dx

Solved Examples

e j

dy
=0
dx

dy
= 2x
dx

2y

ey j

n 1

dy 2 dy

=0
dy dx

2x + 2 y

n dy
dx
x
dx
dx

n 1

2x +

af

d 5
d
4 2
4
x + x y y =
dx
dx

e j e j
F dy + y
5x + x 2 y
H dx

d 5
d 4 2
dy
x +
x y
=0
dx
dx
dx
4

4x

I dy = 0
K dx

Implicit Differentiation

j e

dy
4
4
3 2
2 x y 1 = 5x + 4 x y
dx
4

dy 5 x + 4 x y
=
4
dx
2x y 1

j
d

eax j + dxd eby j + dxd a2hxyf + dxd a2 gxf +


dx
d
2
2
ax + by + 2hxy + 2 gx + 2 f y + c = 0
dx
2

2 1+ y

1+ x

F
GH 2

F
GH 2
F
G
H

af

dy
dy
dy
ax + by + hx + hy + g + f
=0
dx
dx
dx

f dydx = aax + hy + gf
dy bax + hy + g g

dy
=
dx

x +

6. x 1 + y + y 1 + x = 0

7.

Solution: x 1 + y + y 1 + x = 0

Solution:

d
dx

1+ y + 1+ y

1+ x

dy
=0
dx

y =1

x +

y =1

x +

y =

d
dx

dx
+
dx

d
d
x +
dx
dx

e 1 + x j + 1 + x dydx = 0
1
F dy I
G J + e 1+ y j 1 +
1 + y H dx K
2

F 1+ y + y I
GH
J
2 1+ xK
F x + 1 + xI
GH 2 1 + y
JK

d
x 1+ y + y 1+ x = 0
dx
d
dx

I dy
JK dx
I
y
J
1+ xK

+ 1+ x

1+ y +

hx + by + f

dx

1+ y

=0

by + hx + f

dy
dy
dy
+ 2hx + 2hy + 2 g + 2 f
=0
dx
dx
dx

I
JK

x
dy
dy
+ 1+ x
+ 1+ y +
dx
1 + y dx

2 1+ x

d
d
2f y +
c =0
dx
dx
2ax + 2by

dy
=0
dx

a f

dy
y
+ 1+ y +
+
dx
2 1+ x

5. ax2 + by2 + 2hxy + 2gx + 2fy + c = 0


Solution: ax2 + by2 + 2hxy + 2gx + 2fy + c = 0

y
1+ x

1
2 x

af

d
1
dx

y =0

1 dy
=0
2 y dx

1 dy
1
=
2 y dx
2 x

y
dy 2 y
=
=
dx
2 x
x

503

504

How to Learn Calculus of One Variable

Problems based on the combination of implicit


algebraic and transcendental functions
To find the derivatives of problems being the
combination of implicit algebraic and transcendental
functions of xs, we adopt the rule consisting of
following steps.
d
on both sides of the given
Step 1: Take
dx
equation.
Step 2: Differentiate the transcendental functions of
rational integral functions of xs and ys and implicit
algebraic functions of xs with respect to xs using
the rules for the derivatives of sum, difference,
product, quotient, composite of differentiable
functions and a constant multiple of the differentiable
function remembering that

af

(i)

af

af

d
(ii)
F (a r i f o x and y)
dx

a
a

(i)

af

af

dF y
d F y dy
=

dx
dy
dx
d
F (a r i f o x and y)
dx

(ii)

a
a

f a

d F a r o f o x and y d a r i f o x and y

d a r i f o x and y
dx

Where F stands for power, trigonometric,


inverse trigonometric, logarithmic, exponential, etc,
function.
a r e f o x and y stands for a rational
integral function of x and y; and y is pre-assumed (or,
understood) to be a differentiable function of x, say g
(x).

dy
on the left
dx
dy
hand side and the terms without
on the right
dx
hand side.
dy
Step 4: Solve the equation for
on the right hand
dx
side.

d
x+ y
dx

=n x+ y

e j

e jn y

= F y

dy
1. The final result for
is an expression either (i) in
dx
terms of both x and y only or (ii) interms of both x and
y with one (or, more than one) transcendental function

f FH1 + dydx IK
n 1

e j dy
e j dx

dF y
n
d
F y =
(c)
n
dx
d y
n

n 1

dy
dx

dF x y
n
n
d
F x y =
(d)
n
n
dx
d x y

Step 3: Collect the terms involving

Notes:

are the following ones.

(b)

f a

d F a r o f o x and y d a r i f o x and y

d a r i f o x and y
dx

dy
n 1 dy
(a)
= ny
dx
dx

dF x
d F x dy
=

dx
dy
dx

(or, functions) of the rational integral function (or,


functions) of x and y (or, xs and ys).
2. While finding the derivatives of an implicit function
being the combination of implicit algebraic and
transcendental functions of xs, often required
derivatives from:

e
e

dF x y
n n
d
F x y =
(e)
n n
dx
d x y

Solved Examples

dy
if
dx
1. y = sin (x + y)
Solution: y = sin (x + y)

Find

j d ex

dx

j d ex y j
n

dx

Implicit Differentiation

a f a f
a f
dy
F dy I

= cos a x + y f 1 +
H dx K
dx
dy
= cos a x + y f + cos a x + y f
dx

a f

dy d sin x + y d sin x + y d x + y
=
=

dx
dx
d x+ y
dx

dy
dy
cos x + y
= cos x + y
dx
dx

3x y

3 2

3x y

e
=
x e 3x y

2 2

a f a f
a f
dy
dx
F dx + dy I
x
+ y
= cos a x + y f
H dx dx K
dx
dx
dy
F dy I
x
+ y = cos a x + y f 1 +
H dx K
dx
dy
dy
x
+ y = cos a x + y f + cos a x + y f
dx
dx
a f

dy
dy
x
cos x + y
= cos x + y y
dx
dx

gh dydx = cos bx + yg y
dy cos b x + y g y

=
dx x cos b x + y g
x cos x + y

3. x3 y3 = cos (xy)
Solution: x3 y3 = cos (xy)

a f

d
d
3 3
cos xy
x y =
dx
dx
3

a f a f
a f

d cos xy d xy
3 dx
dy
x
+ y
=

dx
dx
d xy
dx
3

j=y
x
+ sin xy j

y 3x y + sin xy

d sin x + y d x + y
dy
d

xy = sin x + y =
dx
dx
d x+y
dx

2 2

2 2

2 3

y 3x y + sin xy
dy

=
3 2
dx
3x y + x sin xy

2. xy = sin (x + y)
Solution: xy = sin (x + y)

j dydx = 3x y y sin xy
= y e3x y + sin xy j
2 2

fg dydx = cosax + yf
cos a x + y f
dy

=
dx 1 cos a x + y f
a f

3 2

1 cos x + y

I
K

dy
dy
2 3
+ 3x y = x sin xy
y sin xy
dx
dx

3x y + x sin xy

F
H

dy
dy
2 3
+ 3x y = sin xy x
+ y
dx
dx

505

4.

x
= cosec xy
y
x
= cosec xy
y

Solution:

x = y cosec xy

dy
d
=
y cosec xy
dx dx

b g
b g
d cosec xy d a xy f
dy
1= y

+ cosec xy
d a xy f
dx
dx
F dy + y dx I +
1 = y acosec xy cot xy f x
H dx dx K
= y

dy
d
cosec xy + cosec xy
dx
dx

cosec xy

dy
dx

1 = yx cosec xy cot xy
2

y cosec xy cot xy

f dydx + cosec xy dydx

506

How to Learn Calculus of One Variable

yx cosec xy cot xy + cosec xy

f dydx

1 + y cosec xy cot xy
dy
=
cosec xy yx cosec xy cot xy
dx

5. y = tan1 (x + y)
Solution: y = tan1 (x + y)

a f a f
a f
dy
1
F dy I

=
1+
dx 1 + a x + y f H
dx K
1

a f

dy d tan x + y d tan x + y d x + y

=
=

dx
dx
d x+ y
dx

1+ x + y

1+ x + y

dy
dx

FG dy IJ = 1
b g H dx K 1 + b x + yg
F
I dy = 1
1
G1
H 1 + a x + y f JK dx 1 + a x + yf
1 + a x + y f 1 dy
1

=
dx
1 + a x + yf
1 + a x + yf
a x + yf dy = 1

1 + a x + y f dx 1 + a x + y f
1 + a x + yf
dy
1
1

=
dx 1 + a x + y f
a x + yf a x + yf

dy
1

dx 1 + x + y

dy
1
dy
1

=
dx
x + y dx
x+ y

x + y 1 dy
1

=
x+ y
dx
x+ y

dy
1
1
=

=
dx
x+ y
x + y 1
x+ y1

f a f
F 1 I dy = 1
G1
H a x + yf JK dx a x + yf

= 1 + y cosec xy cot xy

dy xy
=e
dx

xy dy
xy
dy
= xe
+ ye
dx
dx

xy dy
xy
dy
xe
= ye
dx
dx

a f

a f a f
a f

d log x + y d x + y
dy
d
=

log x + y =
dx dx
d x+y
dx

dy
1
dy
1
1
dy
=
1+
=
+

dx x + y
dx
x+ y
x + y dx

I
K a f a f

j dydx = y e

xy

dy
ye
=
dx 1 x e xy

8. x2 + y2 = log (xy)
Solution: x2 + y2 = log (xy)

F
a f H

xy

xy

xy

e j
e j a f a f
F dy dx I = e F x dy + y 1I
x +y
H dx dx K
H dx K

6. y = log (x + y)
Solution: y = log (x + y)

f a

xy

f a

d e
d xy
dy
d xy

=
e
=

dx dx
d xy
dx

1 xe

a f
a x + yf

7.
Solution: y = exy

y = exy

e
j b a fg
d blog a xy fg d a xy f
=

d a xy f
dx
1 F dy
dx
dy
I

+
=
Gx
+ y 1J
H
K
dx
dx
xy
dx

d 2
d
2
log xy
x + y =
dx
dx

2x + 2y

1 dy
1
dy
=
+
dx
y dx
x

507

Implicit Differentiation

2y

1 dy 1
dy

= 2x
dx
y dx
x

F 1 I dy = e1 2 x j
G2 y J
H y K dx
x
e2 y 1j dy = e1 2 x j

= x

dx

FG IJ FG
j H K H

9. exy = log (xy)


Solution: exy = log (xy)

b a fg

e j

d xy
d
e
=
log xy
dx
dx

e j d a xyf = d blog a xyfg d axyf

d a xy f
dx
d a xy f
dx
F dy + y 1I e = 1 dy + y
x
H dx
K
xy dx xy
d e

xy

xy

xe

FG
H

xy

xe

xy
1 dy
1
dy

= ye
dx
y dx
x

xy

IJ
K

1
dy 1 x y e

=
y
dx
x

1 x ye
dy

=
dx
x

xy

xy

ex y e 1j
F y I F 1 x y e IJ = FG y IJ
=G J G
H x K H1 x y e K H xK
xy

xy
xy

10. y = x log y
Solution: y = x log y

1 dy

+ log y
y dx

dy
x dy

= log y
dx
y dx

FG x IJ dy = log y
H y K dx
F y x IJ dy = log y
G
H y K dx
dy F y log y I

=G
J
dx H y x K
1

2
y
dy 1 2 x
y
1 2 x 2

dx
x
x
1 2 y 2
2 y2 1

dy
d log y dy
=x

+ log y 1
dx
dy
dx

d log y
dy
d
dx
=
+ log y
x log y = x
dx dx
dx
dx

I
JK

11. exy = cos (x2 + y2)


Solution: exy = cos (x2 + y2)

FH e
jIK
d ee j d a xy f d FH cos e x + y jIK d e x + y j

d a xy f dx
dx
d ex + y j
F dy + yIJ
e Gx
H dx K
dy I
F
= sin e x + y j 2 x + 2 y
H
dx K
F dy + y e I
xe
H dx
K
dy
= 2 x sin e x + y j 2 y sin e x + y j
dx

e j

d xy
d
2
2
=
cos x + y
e
dx
dx
2

xy

xy

xy

xy

xe

xy

FH

xy

dy
dy
2
2
+ 2 y sin x + y
dx
dx

j ye
dy
+ 2 y sin e x + y jIK
dx
2

= 2 x sin x + y

xe

xy

508

How to Learn Calculus of One Variable

= 2 x sin x + y

F
GG
H

e
e

j ye
2

xy

j
j

xy

2 x sin x + y + y e
dy

=
xy
2
2
dx
2 y sin x + y + x e

I
JJ
K

Problems based on implicit algebraic functions


Exercise 12.1

dy
if
dx
y2 = 5x2 + 1
y2 = 4ax
x2 + y2 = 9
2x2 + 3y3 = a2

25.
26.
27.
28.
29.
30.
31.
32.
33.

Answers
1.

5x
y

2.

2a
y

Find
1.
2.
3.
4.
5.

x
a
x

y
b

=1

15.

x +
3

y =
3

x
y

4.

2x
3y

5.

b x

x y + y x =1
x3 + y3 = xy
x + y = xy2
xy + x2 y2 = c
x2 y + xy2 = c3
x2 y + xy2 = 3x3 + 4y3
y = (x + y)2
x2 y = (x + 2y)3

a y
2

6.

b x
2

a y

7.

FG x IJ
H yK

n 1

1 2ax
8. 2by 1

16. x 2 + y 2 = a 2
17.
18.
19.
20.
21.
22.
23.
24.

3.

2 =1
2
a
b
7. xn + yn = an
8. ax2 + by2 = (x + y)
9. xy = a
10. 3x2 y = 16
11. x2 + y2 xy = a
12. ax2 + 2hxy + by2 = 1
13. x2 + y2 + 2hxy + 2gx + 2fy + c = 0
14. x3 + y3 = 3axy

6.

x3 y = (2x + 3y)2
(x3+ y3) xy = x5 y5
x5 + y5 5x2 y3 5x3 y2 = 0
x2 y = (2x + 3y)2
x2 y3 = (2x + y)5
x3 y4 = (x + y)7
x5 y4 = (x + y)9
x4 y5 = (x y)9
xa yb = (x y)a + b

9.

y
x

10.

2y
x

y 2x
11. 2 y x

FG
H

ax + hy
12. hx + by

IJ
K

Implicit Differentiation

13.

14.

FG g + x + hy IJ
H f + hx + y K

ay x

y ax

y
x

29.

x
y

F
GH

I
J
xK

y 2 x +
17. x
2 y +
y 3x

21.

y 1
1 2 xy

+ 10 xy + 12 y

+ 24 xy + 6 y
2

25.

8 x + 12 y 3 x y
3

x 12 x 18 y
4

26.

5x 4 xy y
4

f 2 xy
5 a2 x + y f

10 2 x + y
2 2

3x y

30.

y
x

31.

y
x

32.

y
x

33.

y
x

x + 2 xy 12 y

2 x+ y
23. 1 2 x + y

24.

3
4

dy
if
dx
y = sin (x + y)
y = tan (x + y)
y = cot (x + y)
y = sec (x + y)
y = cos (x y)
sin (x + y) + sin (x y) = 1
x = 2 cos y + 3 sin y

Find

9 x 2 xy y

F 3x
G
H 5x

Exercise 12.2

y F y + 2x I
G
J
x H x + 2yK

2 2

Problems based on the combination of implicit


algebraic and transcendental functions

y
x

3x y

3y x

22.

35 x + 81 y + 108 xy

20.

3x y 2 xy

28 x + 54 y + 70 xy

19.

2 2

I
J
2x y K

28.

16.

Fx
GH y

15.

18.

27.

509

x + 4 xy 5 y

4
4

I
JK

1.
2.
3.
4.
5.
6.
7.

8. x =

sin y + cos y

9. x = 2 3 sin y 4 cos y
10. cos (x + y) = y sin x
11. tan (x + y) + tan (x y) = 1
12. x = (2 cos1 y)2

510

How to Learn Calculus of One Variable

Answers
1.

cos x + y
1 cos x + y

a x + yf
1 sec a x + y f
cosec a x + y f

1 + cosec a x + y f
sec a x + y f tan a x + y f
1 sec a x + y f tan a x + y f
sin a x y f
sin a x y f 1

2.

sec

3.

4.

5.

6. 2 cot x cot y

1
7. 3 cos y 2 sin y
8.

2 sin x + cos y
cos y sin y

9.

3 sin y 4 cos y
3 cos y + 4 sin y

y cos x + sin x + y
10. sin x + y + sin x

11.

12.

sec
sec

2
2

a x + yf + sec a x yf
a x yf sec a x + yf
2

1 y2

16.

xy = tan 2 y x
y2 = tan (2y + x)
2

17. y = x + y sin
18.
19.
20.
21.
22.
23.
24.
25.
26.
27.
28.
29.
30.
31.
32.

F xI
H 2K

x cos y + y sin x = 0
x sin y + y cos x = 0
x cos y + y cos x = tan (x + y)
y3 = (x + sin x) (x cos x)
log (xy) = x2 + y2
exy = log (xy)
log y = exy
exy + xy = 0
x = y log (xy)
exy + log (xy) + xy = 0
log | xy | = x2 + y2
y log x = x y
x + y = tan1 (xy)
exy = cos (x2 + y2)
x = y log | xy |

FG IJ
H K
log a x + y f + sin ee j

y
33. y = x log a + bx

4 cos1 y

dy
if
dx
1. x + y = sin (xy)

15.

34. y =
Exercise 12.3

Find

2. x + y = sin (x + y)
3. x + y = tan (xy)
4. xy = tan (xy)
5. xy = sin (x + y)
6. xy = cos (x + y)
7. xy = sin (2x + 3y)
8. x y = sec (x + y)
9. xy = sec (x + y)
10. x2 y = sin y
11. x2 y2 = sin (xy)
12. x3 y3 = cos (xy)
13. x3 + y3 = sin (x + y)
14. xy = sin2 (x + y)

x
35. xy = log (x2 + y2)
36. x2 + y2 = log (x + y)
Answers

a f
a f

y cos xy 1
1. 1 x cos xy

Implicit Differentiation

2. 1

a xy f
a xy f 1

1 y sec

3.

x sec

4.

7.

8.

9.

17.

y
x

b g
x cos b x + y g
y + sin a x + y f

x + sin a x + y f
2 cos a2 x + 3 y f y
x 3 cos a2 x + 3 y f
1 sec a x + y f tan a x + y f
1 + sec a x + y f tan a x + y f
y sec b x + y g tan b x + y g

x sec b x + y g tan b x + y g

5.

6.

1+

y cos x + y

2 xy
10.

cos y x

a f

y cos xy 2 xy

11.

12.

13.

14.

20.

21.

3y

F
I
GH
JK
y F1 x ye I
G
J
x H x y e 1K
xy

xy

y e

24.

26.

xy

1 x ye

25.

xy

y
x

b x yg y or x y
x b x + yg
x b1 + log xy g

27.

y
x

e
j
x e1 2 y j
2

y 2x 1

28.

log x

16.

2x 1
y
22. x
2
1 2y

15.

a x + yf cos y + y sin x
cos x x sin y sec a x + y f
x + a x + sin x cos x f a1 + sin x + cos x f

sec

a f
3 x y y sin b xy g
3x y + x sin b xy g
cos a x + y f 3x
3 y cos a x + y f
sin 2 a x + y f y
x sin 2 a x + y f
F y + 2 tan a2 y xf sec a2 y xf I
G
H x 4 tan a2 y xf sec a2 y xfJK
sec b2 y + x g
2 y 2 sec b2 y + x g
2

y sin x sin y
19. x cos y + cos x

2 x y x cos xy

IJ
K

y cos x + cos y
18. x sin y sin x

23.

FG
H

x
x
3
sin 2
cos
y2
2
2
2
x
1 2 y sin 3
2

29.

a1 + log xf

511

512

How to Learn Calculus of One Variable

a x + yf
ax + yf x

y sec

30.

31.

32.

sec

e
jIJ
+ 2 y sin e x + y j JK
y a x yf
x ax + yf
y eay + bxy bx j
x a x y f aa + bx f
1 + {e cos ee j 3x y } b x + y g
2x y bx + yg 1

F ye
G
GH x e

xy

+ 2 x sin x + y

xy

33.

34.

2 2

2 x yx 2 y 3
35.

x + xy 2 y
3

1 2 x 2 xy

36.

2 xy + 2 y 1

Conditional identities based on a given explicit


function of x
Whenever we have to form a differential equation
with the help of a given explicit function of x, we
adopt the following working rule provided the
required differential equation contains only first
derivative.
dy
Working rule: Find the first derivative (i.e.
) of
dx
the given explicit function of x and use mathematical
manipulations to put the first derivative into the
required differential equation.
Notes:
1. In successive differentiation, we have discussed
in detail the methods of procedure of forming
differential equation whenever a function of x is given.
2. A given function x or an equation defining y as a
function of x can put into different forms according
to our need.

3. When a function of x under the radical sign appears


defining y as a function of x, we should try to remove
the radical sign by squaring or raising the same power
both sides of the equation defining y as a function of
x under the radical sign.
4. Whenever we are given a function of x put in the
forms:

af af
a f f axf
f a xf
a f f a xf
f a xf

(i) y = f 1 x
(ii) y =

f2 x

g1 x

a f f a x f g a x f ...
3
g a x f
g a x f
...
f a x f

g x

g2 x

af af

the bases f 1 (x), f 2 (x), f 3 (x) , f x , f x ,


f x , ... are assumed to be positive before taking
the logarithm of both sides of the equation defining y
as a function of x provided it is not mentioned in the
problems that the bases are positive (i.e. > 0).
5. If we are given a function of x put in the form:

af

b f a xfg b f a xfg b f a xfg


y=
b f a x fg b f a xfg b f a x fg
m1

m2

m3

where the bases f

af af

(x), f

af

(x), f

...

...

(x), ;

f x , f x , f x , ... are functions of xs where


the indices m1 , m2 , m3, ; m , m , m , ... are
constants, it is differentiated taking the logarithm of
both sides of the given equation defining y as a
function of x.
Solved Examples
1. y =

x +

Solution: y =

1
x

, show that 2 x

x +

1
x

,= x2 + x

1 3
dy 1 12
= x x 2
2
dx 2

2x

dy
+ y=2 x.
dx

F
GH

I
JK

12

1
1
dy
1 21 1 2
= 2x
= x2 x 2
x
dx
2
2
3

Adding (i) and (ii), we have

y + 2x

1
dy
= 2x 2 = 2 x
dx

(i)

(ii)

Implicit Differentiation

2. If y = x +

dy
1
+ y = 2x .
, show that x
dx
x

1
Solution: y = x +
x
x +1
x

j e

j af

F
GH

I
JK

2
dy 2 x x + 1
x +1

=
=

2
2
2
dx
x
x

dy F x + 1I

+G
J=2
dx H x K
F x + 1I = 2 x
dy
x
+ xG
dx
H x JK
dy F x + 1I
x
+G
J = 2x
dx H x K
1I
dy F x
x
+G
+ J = 2x
dx H x
xK
dy F
1I
x
+ x+
= 2x
H
dx
xK
2

dy
+ y = 2 x (From (1))
dx
6

3. If y = 1 + x , show that y
Solution: y = 1 + x

dy
5
= 3x .
dx

dy
1+ x

dx

1+ y

= 0.

1
x

1+ y

1+ x

1+ y

1+ x

dx
1+ x

1
, show that
x

dy
1
= 2
dx
x

Now,

dy
5
= 3x
dx

Solution: y =

dy
5
= 6x
dx

4. y =

x 2x x + 1
x

2y
y

2
d 2
d
x
x +1 x +1
x
dy
dx

= dx
2
dx
x

(i)

y=

y =1+ x

513

dx
1+ x

1+
=

x +1

1
4

x =
4
1+ x

x +1

1
x

dy
dx

dy
dx

dy

1+ y
dy

1+ y

=0

Remark: Since the derivative is a limit of the quotient


as x 0 (i.e. lim

x0

y
) which is symbolised as
x

dy
which does not indicate a quotient of dy and dx
dx
dy
but dx and dy are so defined as to consider
as a
dx
quotient of dy and dx which may be seperated as dy
1
dx or dy
. The above example (4) provides us a
dx

514

How to Learn Calculus of One Variable

Conditional identities based on a given implicit


function of x
Now, we will learn how to form a differential equation
with the help of a given implicit function of x. The rule
to form a differential equation with the help of a given
implicit function of x consists of following steps
provided the required differential equation contains
only first derivative.
dy
Step 1: Find the first derivative (i.e.
) of the given
dx
implicit function of x.
Step 2: Use mathematical manipulations to put the
first derivative into the required differential equation.

af

(iv)

f1 y

f2 y

f2 x

f2 x

a fa f a f
a x y fa x + y + xy f = 0

x + y + xy = 0 ( Q x y 0 since x and y
have opposite signs from (i))
Now differentiating both sides w.r.t. x, we have

f dydx = a1 + yf
a1 + yf
dy

=
dx
a1 + xf
1+ x

= x y =
1+ y =

x
x
1+ y =1
1+ x
1+ x

1
1+ x

(iii)

Putting (iii) in (ii), we have

dy
1
=
dx
1+ x

= 1+ x

FG
H

IJ
K

2. If y x + 1 = log x + x + 1 , show that

ex

j dydx + xy 1 = 0 .

+1

FG
H

IJ
K

Now differentiating both sides w.r.t. x, we have

Solution: x 1 + y + y 1 + x = 0

x 1+ y = y 1+ x

Solution: y x + 1 = log x + x + 1

1. x 1 + y + y 1 + x = 0 , show that

(ii)

Again, Q x + y + xy = 0 y 1 + x

f2 x

Solved Examples

dy
= 1+ x
dx

dy
dy
+x
+ y=0
dx
dx

1+

f2 y

bases to be positive to use logarithmic differentiation.


2. Whenever we are given product or quotient of
functions put in the form f1 (x) and f2 (y), we should
take firstly modulus to use logarithmic differentiation.
3. Logarithmic differentiation is only possible when
the given function is positive. This is why we take
the modulus of the given function or equation if it
has the possibility of being negative.
4. log f (x) always means f (x) is preassumed to be
positive similarly log f (y) means f (y) is preassumed to
be positive.

x + y x y + xy x y = 0

a f a f (iii) f a xf a f = f a yf a f
a f = f a y f a f , we should assume the
f a xf
a f (ii)

f2 y

x2 y2 + x2 y y2 x = 0

Refresh your memory:


1. Whenever we have functions put in the forms (i)

f1 x

x2 1 + y = y2 1 + x

dy
as a quotient of dy
dx
and dx which are known as differential of y and
differential of x respectively.
fruitful example for regarding

d
dx

FG
H

=
(i)

IJ
K

x +1 +

x+

x +1

x +1

FG
H

d
x+
dx

dy
dx
2

IJ
K

x +1

Implicit Differentiation

2x +

x +1

2 x +1
1

=
x+

xy

F
G1 +
G 2
+1 H
2

x +1

x +1

1
x

dy
dx

I
2 xJ
JK
+1

dy
xy log y y
=
2
dx
xy log x x

4. If sin y = x sin (a + y), show that


2

dy sin a + y
=
dx
sin a

Solution: sin y = x sin (a + y)

dy
dx

sin y = x sin a + y

log sin y = log x + log sin a + y


2

x+

x +1

x +1+ x
2

x +1

x +1

j dydx = 1 (multiplying both sides

xy + x + 1
by

x +1)

j dydx + xy 1 = 0

x +1

3. xy = yx, show that

dy xy log y y 2
=
.
dx
xy log x x 2

Solution: Firstly supposing that the bases x and y


both are positive and then taking the log of both
sides of the given equation.
xy = yx we have
y log x = x log y

b g

sin a
dy 1

=
sin y sin a + y dx x

dy 1 sin y sin a + y
=
dx
x
sin a

FG IJ
H K

dy
2
= xy log x y
dx

1 x sin a + y sin a + y

( Q sin y = x
x
sin a

5. If

sin

aa + y f

sin a
2

1 x + 1 y

y 2 + xy log x

xy log x x

sin (a + y) is given)

y
dy
x dy
+ log x
=
+ log y
x
dx
y dx

dy
dy
x2
= xy log y
dx
dx
(multiplying both sides by xy)

f
f

1
1 cos a + y dy
dy
cos y
= +

sin y
sin a + y dx
dx
x

FG cos y cos aa + yf IJ dy = 1
H sin y sin aa + yf K dx x
F cos y sin aa + yf sin y cos aa + yfIJ dy = 1
G
sin y sin aa + y f
H
K dx x
sin aa + y y f dy 1

=
sin y sin aa + y f dx
x

a f

d
log y + log y
dx

a
a

d
dy
log x + log x
dx
dx

= x

515

dy
=
dx

1 y

1 x

= a x y , show that

516

How to Learn Calculus of One Variable

a f

Solution: Given is 1 x + 1 y = a x y

F
H

C
2
2

Putting x = sin C ,

y = sin D ,

FG D IJ
H2
2K

I
K

1 x

cos C + cos D = a sin C sin D

cos C + cos D = a sin C sin D

C D
1

= cot a
2

x sin

y = 2 cot

2x + 2 y
2y

1 x

1
1
d
d

sin x
sin y = 0 ( Q 2 cot1 a =
dx
dx
constant)

dy dx

= 1.
dx dy

F
H

I
K

dy
dy
3x
= 3y 2 x
dx
dx

a f

dy
dy
=3 x
+ y 1
dx
dx

2 y 3x

f dydx = 3y 2 x

dy 3y 2 x
=
dx 2 y 3x

(i)

Again x2 + y2 = 3xy

b g

d 2
d
3xy
x + y2 =
dy
dy

FG
H

2x

dx
dx
+ 2y = 3 x + y
dy
dy

2x

dx
dx
3y
= 3x 2 y
dy
dy

1
1
1
d
d

2 cot a
sin x sin y =
dx
dx

d
d
2
2
3xy
x + y =
dx
dx

2 x 3y

dy
=
dx

dy
=0
dx

Solution: x2 + y2 = 3xy

F C + D IJ cos FG C D IJ
2 cos G
H 2 K H 2 K
F F C + D IJ sin FG C D IJ IJ
= a G 2 cos G
H H 2 K H 2 KK
F C D IJ = a sin FG C D IJ
cos G
H 2 K
H 2 K
F C D IJ
cos G
H 2 K = a bQ x y C D or Dg

F C D IJ
sin G
H 2 K
F C D IJ = a
cot G
H 2 K

sin

6. Show that x2 + y2 = 3xy

1 sin 2 C + 1 sin 2 D = a sin C sin D

1 y

1 y

in the hypothesis, we

have

C D = 2 cot

f dxdy = 3x 2 y

dx 3x 2 y
=
dy 2 x 3y

Hence, (i) (ii)


=

IJ
K

...(ii)
dy dx

dx dy

FG 3 y 2 x IJ FG 3x 2 y IJ = 1
H 2 y 3x K H 2 x 3 y K

Implicit Differentiation

7. Show that y = x +

+e

1
y

(i)

1
y

Again y = x +

=1

xy + 1
y

x y
dy
e
= y = e e
dx
e

Conditional identities based on a given implicit


function of x

y 1 = xy

Exercise 12.4

2 y 2 = 2 xy
2

2 2 y = 2 xy

Now, y = x +

1 dy
dy
+ 2
=1
dx
y dx
1
y

j dydx = 1 (using (i))


2

dy
= 1 (using (ii))
dx

j dydx = 1
y

dy
1
+ y = 2x .
, show that x
dx
x
1
dy
x + , show that 2 x
+ y=2 x.
x
dx

4. If y = x +

x y +3

8. Show that e + e = e

dy sin a + y
.
=
sin a
dx

1 + y + x + 2 2y

dy
y
=
.
dx 1 cos y

dy cos a + y
.
=
sin a
dx
3. If sin y = x sin (a + y), show that

I dy = 1
JK dx

1 + y + x 2 xy

1. If y = x sin y, show that x

2. If cos y = x cos (a + y), show that

dy
1 dy
=1 2
dx
dx
y

F
GH

(ii)

1
y

1+

bg

d
d
1
e y + e x =
dx
dx
y dy
x
e
e = 0
dx
y dy
x
e
=e
dx

y + x 2 xy =

x+ y

Solution: e + e = e
Dividing both sides of the given equation by ex + y,
we have
e

1
y

y x=

y=

1
y

Solution: y = x +

1
dy
2
2
x y +3
= 1.
y
dx

517

x+ y

yx
dx
= e
dy

5. If y =
6. If y =

1
, show that
x
2

7. If

1+ x

dy
=
dx

1+ y

1+ x

dy
1+ y

dx

1+ x

= 0.

2
+ 1 + y = a x y , show that

518

How to Learn Calculus of One Variable

e
a1 xfa1 + xf ,

y=

9. If

1 x
2 dy
+ y = 0.
, show that 1 x
1+ x
dx

8. If y =

show

that

e1 x j dydx + xy = 0 .
2

dy
10. If y = x , show that
vanishes when x =e.
dx
1
x

dy
2
2
1
= 1.
11. If y = x + , show that x y + 3
dx
y
1 x

12.

dy
=
dx

21. If x = b cos

F y I + eby y j
GH b JK

1 x

x 1+ y + y 1+ x = 0,

dy
= 1+ x
dx

23. If y = tan

13. If xm yn = (x + y)m + n, show that

1 x

1+x

15.

If exy 4xy = 2, show that

ex

19.

ex y, show that

20. If y = tan

, and t = cos1, x2, show

FG
H

2 log x
dy
=
2 .
dx
1 log x

IJ
K

x + 1 x , show that

j dydx + xy + 1 = 0 .

+1

26. If sin y = x sin (x + m), show that

dy
y x y
=
.
dx
x x+ y

dy sin m + y
=
.
dx
sin m

dy
y
= .
18. If xy log (xy) = log 2, show that
dx
x
If xy

dy
y
= .
dx
x

FG IJ
H K
dy
y F y + xI
= G
J.
17. If y = x log (xy), show that
dx
x H y xK
16. If x = y log (xy), show that

that

dy 1
= .
dx 2

25. If y x + 1 = log

dy
y
14. If y = ex y, show that dx = 1 + y .

show

24. If xy = ex + y, show that

dy
y
= .
dx
x

, show that

22. If

that

1
2

by y
.
y

dy
=
dx

+ 1 y = a x y , show that

1 y

log x
dy
=
dx
1 + log x

27. If u = sin1 (x y), x = 3t, y = 4t3, show that

du
2
= 31 t
dt

sin1

FG sin x IJ , show that dy = 1 .


H 1 + cos x K
dx 2

.
2

28. If y = sin (2
2

12

29. If p = cos

x)

dp
= 4.
dx

1 y
dy
=2
x), show that
2 .
dx
1 x

FG 3 + 5 cos x IJ , show that (5 + 3 cos


H 5 + 3 cos x K

519

Implicit Differentiation

30. If y = tan
a + x2)

FG x sin a IJ , show that (1 2x cos


H 1 x cos a K

dy
=sin a.
dx

F xI F yI
31. If H K + H K
a
b
m

= 1 , show that

dy
b
= at
dx
a

(a, b).
32. If x = a sin 2t (1 + cos 2t), y = a cos 2t (1 cos 2t),
show that

dy

= 1 , when t = .
dx
4

FG
H

IJ
K

33. If y = log x 3 + x 6 x + 1 , show that

dy
2
= x 6x + 1
dx

21

(vi) x = a t sin t , y = a 1 cos t , t 0 , 2


Notes:
1. Actually parametric equations x = f1 (t), y = f2 (t);
represents the x and y co-ordinates of a variable point
p on a given curve.
2. When the parameter t is eliminated by any process
from the parametric equations x = f1 (t), y = f2 (t); the
cartesian equation of the curve put in the form
F (x, y) = c or F (x, y) = 0 is obtained. e.g.,
1. If we have the equations x = r cos , y = r sin ;
where r is a constant and is a parameter, these
equations can be expressed in cartesian equation by
the eliminating ' ' using the mathematical
manipulation of squaring and adding the separate
equations for x and y, i.e. (Note: x = f1 (t) and y = f2 (t)
must be differentiable on a common domain.)
2

(1)

(2)

x = r cos x = r cos

Parametric Differentiation

y = r sin y = r sin

Firstly, we recall the basic definitions in connection


with parametric differentiation.
1. Parametric differentiation: The equations put in
any one of the following forms:

1 + 2 x + y = r sin + cos = r

af
af
(ii) x = f at f , y = f at f ; t T , T
(iii) x = f at f , y = f at f
(i) x = f 1 t , y = f 2 t ; t T1 , T2
1

which tells x and y are seperately differentiable


functions of (depending upon) the same variable t
(called the parameter) are said to be parametric
equations of the curve or simply parametric
equations. e.g.,
(i) x = t, y = t2; t being a parameter.
(ii) x = ct2, y = ct; where c is a constant and t is a
parameter.
(iii) x = r cos , y = r sin ; r being a constant and

being a parameter.

F I , a >b >0
H 2K
F I
x = e asin t + cos t f, y = e asin t cos t f, t ,
H 4 4K

(iv) x = a sec , y = b tan , 0 ,


(v)

af a f

which is the cartesian equation of the circle.


2. The point of the curve x = f1 (t), y = f2 (t); found by
giving a special value, say t1, to the parameter t is
called shortly the point t1 whereas it is not unusual
for the letter t itself to be used instead of t1 for a
special point.
3. Any other letter , s, u, etc can be used to
represent the parameter instead of t in parametric
equations.
Question: What is parametric differentiation?
Answer: Finding the derivative of the parametric
equations is called parametric differentiation.
Or, more explicitly,

dy
of the given parametric
dx
equations put in the form (i) x = f 1 t , y = f 2 t ;
(ii) x = f 2 t , y = f 2 t , t T1 , T2 (iii) x = f1 (t), y
= f2 (t); t T1 , T2 without eliminating the parameter
t (or, any other parameter , s, y, etc given the
parametric equations) by any process is called
parametric differentiation.
Finding the derivative

af

af a

af
f

af

520

How to Learn Calculus of One Variable

Question: Using the definition, find the differential


coefficient of parametric equations
x = f (t)
y = g (t);
where x and y are differentiable functions (depending
upon) a single variable t (called parameter)

af

Solution: x = f t x + x = f t + t

af

a f
(1) x = f at + t f f at f
(2) y = g at + t f g at f
y = g t y = y = g t + t

(1)
(2)
(3)
(4)

Now dividing (3) by t , we obtain

f af

f t + t f t
x
=
t
t

and dividing (4) by t , we obtain

f af

f af
f af

(7)

lastly, taking the limit as x 0 , t 0 ,

y 0 , we have form (7),


y
t t
y
=
lim
x
x 0 x
lim
t0 t

af
af

g t
dy
=
dx
f t
being valid for all values of t such that f t 0
which can be stated in words in the following way:
Derivative of y with respect x is equal to the
quotient of the derivative of y with to t and the
derivative of x with respect to t.
coefficient of g (t) with respect to f (t) is

af

To find the differential coefficient of the given


parametric equations put in any one of the forms:

af
af a f
x = f a t f , y = f at f : t a T , T f

(ii)

af

af

(iii) x = f1 t , y = f 2 t we have the rule which


consists of the following steps.

Again dividing (6) by (5), we obtain

a
a

Hence, the general rule for the differential

(i) x = f 1 t , y = f 2 t ; t T1 , T2
(6)

g t + t g t
y
y
t
t
=
=
x
f t + t f t
x
t
t

af af
af

g t
,f t 0
f t

Working rule of find the differential coefficient of


parametric equations
(5)

g t + t g t
y
=
t
t

Step 1: Find the derivative of y with respect to t.


Step 2: Find the derivative of x with respect to the
same parameter t.
Step 3: Divide the derivative of y with respect to t
by the derivative of x with respect to t.
Step 4: The quotient obtained in step (3) is the
required derivative of y with respect to x.
Solved Examples

lim

a
a

Find

f af
f af

g t + t g t
lim
t0
t
=
f t + t f t
lim
t 0
t

b a fg
b a fg

dy
d
f t
dy

= dt = dt
dx
d
dx
f t
dt
dt

dy
from the following equations.
dx

1. x = 3cos

y = 4 sin
Solution: y = 4 sin

dy
= 4 cos
d

...(i)

dx
= 3sin
d

...(ii)

521

Implicit Differentiation

af
af
f af = 3 cos and f af = 4 sin .

Note: Here x = f 1 and y = f 2


1

where

a1f dy = e sin t
a2f dx 1 + cos t
t

a1f dy = 4 cos = 4 cot


a2f dx 3 sin 3

2. x = 2 3sin

y = 3 + 2 cos

F
H

e sin t
1 + t cos t
t

I = ee
K

j a1 + ttcos t f

sin t

e
j
=
a1 + t cos t f
t

t e sin t

dy
Solution: y = 3 + 2 cos
d

= 0 + 2 sin = 2 sin

x = 2 3sin

...(i)
5. x =

dy
d

= 0 3 cos = 3 cos

...(ii)

a1f dy = 2 sin = 2 tan

a2f dx 3cos 3

1+ t

Solution: y =

1+ t

1 t

1+ t

dy
dt

...(i)
b a fg = a sin t
dx
x = a at + sin t f
= a a1 + cos t f
...(ii)
dt
a1f dy a sin t
a sin t

a2f dx = a a1+ cos t f = a F1+ 2 cos t 1I


H
2 K

= a 0 sin t

t
t
cos
t
2
2
=
= tan
t
t
2
a 2 cos cos
2
2

2 2

2t 2t 3 2t + 2t 3

e1 + t j
2

t
dy
= e sin t
dx

dx 1
= + cos t
dt
t

...(i)
...(ii)

4t

e1 + t j

...(i)

e1 + t j dtd b2t g 2t dtd e1 + t j


bg
e1 + t j
e1 + t j 2 2t a2t f = 2 + 2t 4t
=
e1 + t j
e1 + t j
2 e1 t j
2 2t
=
=
...(ii)
e1 + t j e1 + t j
2

2 2

2 2

Solution: y = e + cos t

4. x = log t + sin t
y = et + cos t

2 2

d
x =
dt

a 2 sin

e1 + t j a2t f e1 t j 2t
=
e1 + t j

x = log t + sin t

1 t

y =
dt

Solution: y = a 1 cos t

,y=

e1+ t j LMN dtd e1 t jOPQ e1 t j LMN dtd e1+ t jOPQ


bg
e1+ t j

3. x = a (t + sin t)
y = a (1 cos t)

2t

2 2

2 2

2 2

522

How to Learn Calculus of One Variable

b1g dy = 4t e1 + t j

b2g dx e1 + t j 2 e1 t j
2 2

2 2

4t

2 1t

6. x = sin

2t
1+t

Solution: y = cos

cos y =

1+ t

1+ t

1 1

F1 t I
GH 1 + t JK
2

1+ t

=
2
sin y

e1 + t j

4t

e1+ t j

2 2

F1 t I
1G
H 1 + t JK
2
2

1
2

1 cos y

4t

e1+ t j =

4t

e1+ t j

2 t

2t

e1+ t j t
2

, t 0 ...(i)

2t
1+ t

2t
1+t

d
d
1 + t j a2t f 2t e1 + t j
e
dt
dt
a f
e1 + t j
2

sin x =
dt

dy 2t 2t 2t + 2t
4t
sin y =
=
2
2
2
dt
1+ t
1+ t

2 2

sin x =

2 2

4t

e1 + t j

Again x = sin

4t

dy e1 + t j a2t f e1 t j 2 t
sin y
=
dt
e1 + t j

2 2

e1 + t j

e1 + t j e1 t j
e1 + t j
2 2

4t

e1 + t j

e1 t j

d
d
1+ t j e1 t j e1 t j e1+ t j
e
d
dt
dt
acos yf =
dt
e1+ t j

dy

=
dt

2 2

2 2

1 t

4t

2t

, y = cos

2 2

j 2 t 2t
e1 + t j

2 1+t

2 2

2
dx 2 + 2t 2 4t 2 2 2t 2 2 1 t
cos x
=
=
=
2
2
2
dt
1+ t 2
1+ t 2
1+ t 2

e j e j e j
2 e1 t j
1
dx 2 e1 t j 1
=

cos x
dt
e1 + t j
e1 + t j 1 sin x
2 e1 t j
1
=

e1 + t j 1 FG 2t IJ
H1 + t K
2

2 2

2 2

2 2

523

Implicit Differentiation

2 1 t 2

e1+ t j

2 2

1 + t 4 + 2t 2 4t 2

e1+ t j

j e1+ t j
j e1 t j
2

2 2

e1 + t

Solution: y = a sin t

2 2

...(ii)

F
H

j , t 0, 1

I
K

dy
d t
=
(e (sin t cos t)) = et (cos t + sin t) +
dx dt
(sin t cos t) et
= et (cot t + sin t + sin t cos t) = 2 sin t et (1)
and x = et (sin t + cos t)

e a

fj

dx
d t
=
e sin t + cos t
dt
dt
= et (cos t sin t) + (sin t + cos t) et
= et (cos t sin t + sin t + cos t)
= 2 cos t et

a1f dy 2 sin t e
a2f dx = 2 cos t e

IJ
K

a
sin t

F 1 sin tIJ = a e1 sin t j = a cos t ...(2)


= aG
H sin t K sin t sin t
a1f dy = a cos t sin t = sin t = tan t , t F 0 , I

H 2K
a2f dx a cos t cost
x = a at sin t f
t 0 , 2
9.
y = a a1 cos t f
dy
= a sin t
Solution: y = a a1 cos t f
...(1)
dt
2


,
4 4

I
K

...(2)

F
H

FG
H

x = a t sin t

= tan t , t

I
K

dy
a
1
t
t 1
= a sin t +
2 tan sec 2
dt
2 tan 2 t
2
2 2
2

= a sin t +

Solution: y = et (sin t cos t)

...(1)

t
a
2
= a sin t +
= a sin t +
t
t
t
2 tan
2 sin cos
2
2
2

7. x = e t (sin t + cos t), y = e t (sin t cos t),


t ,
4 4

F
H

a
2 t
log tan
2
2

a sec

e1 t j t
2

t 1 t

x = a cos t +

b1g dy = 2t e1+ t j e1 t j

b2g dx e1+ t j t 2 e1 t j
=

FG IJ
H 2K

t 0,

dy
dt

d
a sin t = a cos t
dt

, t2 1

j e1 + t j = 2 e1 t j
j e1 t j e1 + t j e1 t j
2

2 1 t

IJ
K

2 2

2 1 t 2
1+ t 2

FG
H

a
t
log tan 2
2
2
y = a sin t

8. x = a cos t +

dx
= a 1 cos t
dt

a1f dy a sin t
a2f dx = a a1 cos t f

...(2)

524

How to Learn Calculus of One Variable

FG t IJ cos FG t IJ cos FG t IJ
H 2 K H 2 K = H 2 K = cot FG t IJ ,
=
FG1 1 + 2 sin FG t IJ IJ sin FG t IJ H 2 K
H 2KK H 2K
H
2 sin

F1 t I
GH 1+ t JK
F 2t I
y = bG
H 1 + t JK
2

14. x = a

for t 0 , 2
Problems based on parametric equations

15. x =

Exercise 12.5

Find

dy
from the following equations.
dx

F
H

1. x = a cos + log tan


y = a sin
2. x = 2 cos cos 2
y = 2 sin sin 2
3. x = a cos
y = b sin
4. x = a sec
y = b tan
5. x = a cos3
y = b sin3
6. x = a sec2
y = a tan3
7. x = a (cos t + t sin t)
y = a (sin t t cos t)
8. x = a ( sin )
y = a (1 + cos )
9. x = a ( sin )
y = a (1 cos )
10. x = 2 cos2
y = 3 sin2
11. x = a log t
y = b sin t
12. x = tan1 t
y = t sin 2t
13. x = at2
y = 2at

I
K

y=

1+ t
3at

1+ t

16. x =

y=

3at

2t
1+ t

1 t

1+ t

17. x = 1 t

y = 1+ t
18. x = u2
3

y=

u 3u + 5
1+ u

19. x =

a
m

y=

2a
m

20. x = a

1 t
1+ t

y=at

1 t
1+ t

21. x = 2t | t |
y = t2 + t | t |

a
f
y = a asin cos f

22. x = a sin + cos

Implicit Differentiation

t 1

23. x = a

t +1

y = at

t 1
2

t +1

4.

3
2

18.

2t + 2 t

c 2t t h t

21.

b
cosec
a

cos + sin
22. cos sin
4

3
tan
6.
2
7. tan t

t + 2t 1
2t

23.

Exercise 12.6

8. cot
2

9. cot
2

Find

3
2

b
t cos t
a
12. (2t cos 2t + sin 2t) (1 + t2)
11.

dy
from the following equations.
dx

b
b

g
g

1.

x = a t sin t
y = a 1 cos t

2.

x = a cos t
t 0, , a 0
y = a sin t

t 0 , 2 , a 0

b g

F I
H K
x = e bsin t + cos t g
L O
t M , P
y = e bsin t cos t g
N 4 4Q

x = a sec

3. y = b tan 0 , 2 , a > b > 0

1
t

14.

F
GH

b 1 t2

a
2t

j
e1 2t j

t 2t
15.

20. t + t 1

b
5. tan
a

13.

2u u + 1

19. m

b
cot
a

10.

u + 6u 10u 3

Answers (with proper restrictions on the parameters)


1. tan

3.

1 t
1+ t

17.

2. tan

2t
1 t2

16.

I = F b
JK GH a

2
2

x
y

I
JK

4.

x = + sin
5. y = 1 cos ,

525

526

How to Learn Calculus of One Variable

LM IJ
N K

To find the differential coefficient of a function with


respect to an other function

x = a cos3 t
,a 0
t 0,
6.
3
2
y = a sin t

Question: Using the definition, find the differential


coefficient of a function f (x) with respect to another
function g (x), where f (x) and g (x) both are
differentiable functions having the same independent
variable x.
Solution: Let y = f (x) be a differentiable function of x
(1)
and z = g (x) be another differentiable function of x
(2)

x = at 2
t > 0, a 0
7.
y = 2at

x=
8.

y=

1 t

1+ t
2t

1+t

t>0

b
b

g
g

FG
H

IJ
K

x = a cos + sin

, ,a 0
9. y = a sin cos
2

Answers
1.
2.
3.

4.

5.

6.
7.

FtI
cot G J , t b0 , 2 g
H 2K
cot t , t b0 , g
b
F I
cosec , 0 ,
H 2K
a
L O
tan t , t M , P
N 4 4Q
F I
tan G J , b , g
H 2K
F I
tan t , t G 0 , J
H 2K
2

1t
,t >0
2t

9. tan ,

and z + z = g x + x

FG , OP
H2 Q

(3)
(4)

f af
and from (4), we have z = g a x + x f g a x f (6)
From (3), we have y = f x + x f x ...(5)

Dividing (5) by x , we obtain

f af

f x + x f x
y
=
x
x

(7)

Dividing (6) by x , we obtain

f af

g x + x g x
z
=
x
x

(8)

Dividing (7) by (8), we obtain

a
a

f af
f af

f x + x f x
y
y
x
x
=
=
z
g x + x g x
z
x
x

lastly, taking the limit as x 0 , y 0 ,

z 0 , we obtain from (9),

1
,t > 0
t

8.

Now, y + y = f x + x

y
y x 0 x
=
lim
z
x 0 z
lim
x0 x
lim

a
a

f af
f af

f x + x f x
x0
x
=
g x + x g x
lim
x 0
x
lim

Implicit Differentiation

af
af

Remember:

af
af

d f x
dy
f x
dy
dy dx
dx
=

=
= dx =
dz
dg x
dz
g x
dx dz
dx
dx

dy
dy
is not a quotient we may interpret
dx
dx
as a quotient which means we are able to write.
Although

Hence, the general rule for the differentiable


coefficient of a differentiable function f (x) with respect
to another differentiable function g (x) is

af
af

af
af

d f x
d f x
= dx
dg x
dg x
dx

dy
= dy dx
dx

(ii)

dy
1
dy
= dy
dy =
dx
dx
dx
dx
dy dy dz dy dx
=

dx dx dx dx dz

Solved Examples

which can be stated in words in the following way.

(i)

(iii)

d the function given to be differentiated


d the function w.r. t which we have to find the d.c

527

differential coefficient of the given function w.r.t x


differential coefficient w.r.t x of the function w. r.t which

1. cos2 x w.r.t. (log x)3


Solution: Putting y = cos2 x
and z = (log x), x > 0
We have

Remark: We must note that the function to be


differentiated and the other function with respect to
which we have to differentiate, both have the same
independent variable.
Working rule to differentiate f (x) w.r.t. g (x)
To find the differential coefficient of a function f (x)
w.r.t. another function g (x), where f (x) and g (x) both
are differentiable functions having the same
independent variable x, we adopt the rule consisting
of following steps.
Step 1: Find the differential coefficient of the given
function with respect to the independent variable x
which the function to be differentiated has.
Step 2: Find the differential coefficient with respect
to the same independent variable x of the other
function w.r.t. which d.c. of the given function is
required.
Step 3: Divide the differential coefficient obtained in
step (1) by the differential coefficient obtained in step
(2). And obtain the required differential coefficient of
f (x) w.r.t. g (x).

j a f

we have to find the d.c. of the given function

(1)
(2)
2

d cos x
d cos x
2
dy
d
=
cos x =

dx dx
d cos x
dx
= cos (sin x) = 2 sin x cos x = sin 2x
and

a f

(3)

d log x
dz d log x
=

dx
d log x
dx

(4)
a f 1x = 3alogx xf
b3g dy / dz = dy = x sin 2 x , x > 0
Hence,
b4g dx dx dz 3blog x g
2

= 3 log x

Or, alternatively,

j a f

d cos x d cos x
2
d

= sin 2 x (i)
cos x =
dx
d cos x
dx

a f

d log x
dx

a f

a f

d log x d log x 3 log x

=
d log x
dx
x

e j = a1f = x sin 2 x , x > 0


a2f 3alog xf
d alog x f
2

Hence,

d cos x

(ii)

528

How to Learn Calculus of One Variable

2. log x w.r.t. x3
Solution: Putting y = log x(1)
and z = x3
We have

(2)

a f

1
dy d log x
=
= ,x>0
dx
dx
x

e j = 3x

e j d e1 + x j
dx
d e1 + x j

d 1+ x

(3)

Hence,

2 sin 1 + x
2

b3g dy = dz = dy = 1 1
b4g dx dx dz x 3x

x
3. e w.r.t. x
Solution: Putting y = ex

1
3x 3

(1)

and z = x
We have

e xj = 2 1x , x > 0
x

sin 1 + x

2 2

w. r.t 1 + x

Solution: Putting y =

= 2e

sin 1 + x

(4)

j cos e1 + x j

2 2

2x
2 2

5. e sin

2 2

I
j JK

Solution: Putting y = e

2 2

2 2

sin

(1)

sin1

and z =
x
We have

2 2

dy
=
dx

,x 0

w.r. t. sin 1 x

(1)

F
I
d G sin e1 + x j J d FG sin e1 + x j IJ
H
K H
K
=
F
I
d G sin e1 + x j J
H
K d e1 + x j
2 2

2x 1 + x

e1 + x j cos e1 + x j
=
sin e1 + x j

sin 1 + x

2 2

dx

2 2

dy
=
dz

x , x >0

(2)

We have

dy
=
dx

j = 2x

b3g 2 x e1 + x j cos e1 + x j / 2 x , x 0
b 4g
sin e1 + x j

(4)

and z = (1 + x2)

F
GH

(3)

b3g dy / dz = 2e
Thus,
b4g dx dx 1

(3)

2 2

dz
d
=
dx dx

2 2

e j

j 2 e1+ x j 2 x

2 2

j cos e1 + x j
sin e1 + x j

2x 1 + x

dz d 1 + x
=
and
dx
dx

(2)

x
dy
d x
e =e
=
dx dx

4.

2 2

cos 1 + x

(4)

x > 0.

and

dz d x
=
and
dx
dx

2 2

FG
H

d e

(2)
1

sin

IJ
K

dx

FG
H
=
d esin
d e

sin

IJ
K d sin
dx
xj

Implicit Differentiation

e sin

and

1 x2

1
1 x

b3g dy dx =
b4g dx dz

, x 1

dz d sin 1 x
=
=
dx
dx

e sin

; x 1

1 x

FH

1 x 2

...(3)

21.

(4)

22.

IK

1
dy
= e sin x ; x 1
dz

Problems on differentiation of a function with respect


to another function
Exercise 12.7
Differentiable:
1. x5 w.r.t. x3
2. x2 + x + 3 w.r.t. x3
3. sin x w.r.t. cos x
4. tan1 x w.r.t. x2
5. sin1 x w.r.t. cos1 x
sin
6. x

w.r.t. tan1 x

x
7. e w.r.t. x

1 tan x

19.

20.

x
1+ x

w.r.t. tan x

1 + cos 2 x
w.r.t. x
1 cos 2 x

w.r.t. x

1 + tan x

x
w.r.t. sin x
sin x

23. ex w.r.t. log x


24. log x2 w.r.t. ex
25. e

sin

w.r.t. log x

sin 1 + x

26.

sin

2 2

w.r.t. (1 + x2)

27. e
w.r.t. cos1 x
1
28. tan x w.r.t. sin1 x
29. log sin x w.r.t. sin1 x

1 cos x
1 + sin x
w.r.t.
1 + cos x
1 sin x

30.

Answers
1.

5 2
x
3
2x + 1

,x0
3x 2
3. cot x , x n

2.

sin x
w.r.t. sin 1 x
8. e
9. sin x w.r.t. x3
10. sin x w.r.t. tan x
11. tan x w.r.t. sin x
12. cosec x w.r.t. cot x
13. cot x w.r.t. cosec x
14. tan x w.r.t. sec x
15. tan x w.r.t. cot x
16. cosec x w.r.t. sec x
17. sin x w.r.t. cos2 x
18. cos2 x w.r.t. sin x

529

4.

2x 1 + x2

,x0

5. 1 for x 1

6. 1 + x 2 x sin

7. 2 x e x , x 0
sin
8. e

9.

cos x
3x 2

F
GG
H

log x
1 x2

, x 1

,x0

10. cos3 x , x n +

I
JJ
K

sin 1 x
, 0 < x <1
x

530

How to Learn Calculus of One Variable

11. sec3 x , x n +

28.

12. cos x , x n
13. sec x , x n
14. cosec x , x

n
2

19.

e1 + x j

3
2

n
2

22.

1 tan x
4

, x n

n
,x
2
2
sin x cos x

sin x x cos x

23. x ex, x > 0


24.

25.

2
,x>0
x ex
x e sin

1 x2

,0 < x <1

e1 + x j cos e1 + x j
sin e1 + x j

2 2

26.

27. e sin

2 2

Before we find the derivatives of the problems on in


finite series or continued fraction which tends to
infinity, we must know the definitions of the following
terms.
1. Infinite series: A series having the number of terms
infinite (i.e. not finite) is called an infinite series. An
infinite series is written in any one of the following
forms.

, x n +
2

2 tan x

b g 1 x ,0< x 1
F 1 sin x IJ
tan x G
H 1 + cos x K

Differentiation of infinite series (or, continued


fraction which )

cot x
2
n

,x
20.
cos 2 x 1 cot x
2
21.

, x 1

, x 1

30.

1
17. cosec x , x n
2
18. 2 sin x for all x
cos2 x

1 + x2

29. cot x

15. tan2 x , x n +
16. cot3 x , x

1 x2

(i) x1 + x2 + x3 + + xn + , where dots denote the


existance of similar terms obeying the same rule as its
previous terms.
(ii)

(iii)

i =0

or, more briefly

x , where x
i

and xi

are the general term or nth term of the infinite series.


2. Continued fraction: A continued fraction is a
fraction expressed as a number plus a fraction whose
denominator is a number plus a fraction, i.e.
a continued fraction
a number + a fraction whose denominator = a
number + a fraction
b2
a1
b3
a2 +
b4
a3 +
b5
a4 +
a5 + ...
Notes:
1. A continued fraction may have either a finite or in
finite number or terms. A continued fraction having
finite number of terms is called terminating continued

Implicit Differentiation

fraction. A continued fraction having infinite number


of terms is called non-terminating continued fraction.
If a certain sequence of the as and bs occurs
periodically, the continued fraction is recurring or
periodic. The terminating continued fraction

a1 , a1 +

b2
a2 +

b3
a3

, etc are convergents of the

continued fraction whereas the quotients

b2 b3
,
a2 a3

etc are partial quotients. The continued fraction is a


simple continued fraction if bi = 1, i = 2, 3, ... .
2. For the sake of saving space, it is usual to write a
simple continued fraction in the more compact forms
as:

1 1
1 1 1
a+
... or ,
... instead of
b+ c+
a+ b+ c+
writing
a+

1
1
b+
c + ...

or ,
a+

bg

bg

y= f x +

1. If

bg

1
1
+
+ ...
f x
f x

bg

bg

then

bg

1
y= f x + .
y

dy
using the method of differentiating
dx
implicit functions.
2. Find

Note: We inspect that

x + 2 is the expression

which terminates.
retain the first and put the first = y

y = x + 2y
2. If we are given y = a non-terminating continued
fraction put in the form:

af

af

f x +

1
1

af

f x +

1
b+
c + ...

1
1
1
+
+
+ ...
f x +
f x
f x
f x

bg

x + 2 x + 2 x + ...

it is to be noticed that the letters a, b, c, all denote


integral numbers, the signs are all positive and each
of the numerator is unity.
We are now prepared to find the ways of
differentiating the non-terminating continued fraction
of the form

bg

y=

, where

1
a+

Remember:
1. In the given series being infinite, we firstly inspect
what expression in x terminates or converges and
then we retain only one terminating (or, converging)
expression in x as it is and the rest same terminating
expression is put equal to y (= l.h.s). e.g.,

y= f x +

531

a f f a x f 1+ ... ,

f x +

we retain f (x) as it is and the rest is put equal to

1
,
y

i.e. reciprocal of l.h.s.

af

y = f x +

1
y

3. Differentiation of a non terminating continued


fraction is also termed as differentiation of explicit
functions in an ad infinitum form and is
replaced by ad in f.
e.g.: y =

x +

1
x +

1
x +

1
x + ad in f

4. Differentiation of explicit functions in any ad


infinitum form becomes differentiable after
transformation to finite implicit form which is possible
as under.

532

How to Learn Calculus of One Variable

(i) y = x

(ii) y = e

(iii) y =

...

y=x
x ...

x y

...

y=

cos x + cos x + cos x + ...

Solution: y =

e j

y= e

2. y =

cos x + cos x + cos x + ...

y = cos x + y

e j
x

y = cos x + y

etc.

e j

d
d 2
cos x + y
y =
dx
dx

Problems based on "ad infinitum" form.

Solved Examples

2y

dy
dy
= sin x +
dx
dx

2y

dy dy

= sin x
dx dx

Find

dy
if
dx

1. y = e

x+e

x +e

x +e

Solution: y = e
y=e

x ...

x+e

f dydx = sin x

2y 1

x + e

x + e

x ...

3. y =

x+ y

e jb g
aQ log e = log e = 1f
d
d
log y f =
a
a x + yf

dx
dx

dy
1
sin x
sin x
1
=
=
; y , i.e. cos x
dx
2y 1
1 2y
2
4

b g

log y = log e x + y = x + y log e e = x + y


e

sin x + sin x + sin x + ...

Solution: y =

y=

sin x + sin x + sin x + ...

sin x + y

y = sin x + y

e j

d 2
d
sin x + y
y =
dx
dx

1 dy
dy
=1+
y dx
dx

1 dy dy

=1
y dx dx

2y

dy
dy
= cos x +
dx
dx

2y

dy dy

= cos x
dx dx

F 1 I dy = 1
G 1J
H y K dx
F 1 y IJ dy = 1
G
H y K dx
dy F y I

=G
J for y 1 , i.e. x 1
dx H 1 y K

f dydx = cos x

2y 1

1
1
dy
cos x
=
; y , i.e. sin x
2y 1
2
4
dx

4. y =

x + 2 x + 2 x + ...

Implicit Differentiation

Solution: y =

y=

x + 2 x + 2 x + ...
Solution: y = e

x + 2y

y = x + 2y

y=e

e j

d 2
d
y =
x + 2y
dx
dx

2y

1
dy
1
=
=
; y 1 , i.e. x 1
2y 2
2 y 1
dx

5. y = x

xx

g b

...

x
Solution: y = x

xx

,x>0

FH e y IK
=e

Note: y = e

d
d
log log y =
y log x
dx
dx

1
1 dy
d
dy

= y
log x log x
dx
log y y dx
dx

b g

dy
y
+ log x
dx
x

1
dy
dy
y

log x
=
y log y dx
dx
x

FG 1 y log x log y IJ dy = y
H y log y K dx x
IJ = y log y
dy y F
y log y

= G
dx x H 1 y log x log y K x b1 y log x log y g

,x>0

log y = y log x

b g

Remark:

d
d
log y =
y log x
dx
dx

1 dy
y
dy
= + log x
y dx
x
dx

FG 1 log xIJ dy = y
Hy
K dx x

1 y log x dy
y
=
y
dx x
2

dy
y
=
dx x 1 y log x

6. y = e

ex

ex

...

3
3

FH 33 IK
means 3

y=e

7. x = y

FH x y IK

y =e

means
y

x= y

ee j

...

Solution: x = y

...

/ log y = y log e

y= x

log log y = y log x

dy
2y 1
=1
dx

...

dy
dy
2y
2
=1
dx
dx

log y = x log e = x

dy
dy
=1+ 2
dx
dx

e
ex

533

...

,y>0

log x = log x x x log y

b g

d
d
log x =
x log y
dx
dx

x y

=e

xy

534

How to Learn Calculus of One Variable

log y = log x b

b g

dx
d
1
log y + log y
= x
dx
dx
x
= x

1 dy

+ log y
y dx

1
x dy
log y =
x
y dx

1 x log y x dy
=
x
y dx

FG
H

x + x + x + ...

x+ y

y = x+ y

e j

d
d 2
x+ y
y =
dx
dx

2y

x+

x+

IJ
K

x+ y

x+

,x>0

b x + yg + x log x
x

y x + y + x log x

x
1 y log x
x

e j
x

...

ej
y
2

e j = FG y IJ log x
y
2

b g

d
d
log y =
dx
dx
=

...

,x>0

= x

log y = log x

...

dx

FG 1 y log x IJ dy = bx + yg + x log x
x
H y K dx
F
I
+
+
log
x
y
x
x
F
IJ
b
g
dy
y

=G
G
J
dx H
x
K H 1 y log x K

x+

...

Solution: y = x
y= x

y=

dy
1
=
dx
2y 1

9. y = x

x+ y
1 dy
dy

log x
=
+ log x
y dx
dx
x

Solution: y =

dy
2y 1
=1
dx

FG
H

10. y =

dy dy
2y

=1
dx dx

dy
dy
=1+
dx
dx

cb

d
d
log y =
x + y log x
dx
dx

x + x + x + ...

y=

g dxd blog xg + log x dxd bx + yg


1 dy
1
F dy I

= b x + y g + log x G1 + J
H dx K
y dx
x
b x + yg + log x + log x dy
=

IJ
K

Solution: y =

= x + y log x

y 1 x log y
1 x log y
dy
y

=
=
dx
x
x
x2

8. y =

b g

= x+ y

1
x dy
=
+ log y
x
y dx

x+ y

b g

H 2K

FG y log xIJ
H2 K

y d
d

log x + log x
2 dx
dx

FG y IJ
H 2K

Implicit Differentiation

1 dy
y 1 1
dy

= + log x
2 x 2
y dx
dx

1
dy
y
+ log x
2x 2
dx

FG 1 1 log xIJ dy = y
H y 2 K dx 2 x
F 2 y log x IJ dy = y
G
H 2 y K dx 2 x
IJ
dy F y I F
2y

=G J G
dx H 2 x K H 2 y log x K

y
x 2 y log x

11. y =

cos x

Solution: y =

dy d
=
dx dx

cos x

cos x ...

cos x

cos x

fecos x j

d cos x y
j debcos x ygj dxd bcos x yg

...

; cos x > 0

IJ
K

dy
2 cos x y dx

dy
y 2 tan x
=
1 y log cos x
dx

13. y =

x + 2 + x + 2 + x + ...

Solution: y =

y=

x + 2 + x + 2 + x + ...

x+ 2+ y

y = x+ 2+ y
y x=

F 2 cos x y + 1I dy =
GH 2 cos x y JK dx 2

y x
cos x y

dy
dx

FG 1 log cos xIJ dy = y tan x


Hy
K dx

1
dy
sin x
dy
+

=
dx 2 cos x y dx
2 cos x y
sin x

1 dy
1
dy

=y
sin x + log cos x
y dx
cos x
dx

cos x ...

cos x y =

FG
H

cos x + y

...
cos x j

cos x

= y tan x + log cos x

cos x

2 cos x y

sin x
Qy =
2 y +1

log y = y log cos x

sin x

sin x
1 + 2y

a f a fe
a
Solution: y = acos x f

dy
1
dy
=
sin x
dx 2 cos x y
dx
=

2 cos x y + 1

12. y = cos x

y = cos x y

I F
JK GH

2 cos x y
sin x
dy
=

dx
2 cos x y
2 cos x y + 1

sin x

1 dy 1
dy
y

log x
=
y dx 2
dx 2 x

F
GH

535

2+ y

j = a2 + y f
d F
I d a2 + yf

y xj J =
G
e
K dx
dx H
e

I
JK

536

How to Learn Calculus of One Variable

2 y x

j FH 2 y dydx 1IK = dydx

4y y x

LM e
N

j OPQ dydx = 2 e y xj
2 e y xj
2 2+ y
dy

=
=
dx 4 y e y x j 1 4 y 2 + y 1
2

x 2 x 2 x 2 x 2 ...

Solution: y =

y=

x 2 x 2 x 2 x 2 ...

y + 2y = x
dy
dy
2y
+2
=1
dx
dx
dy
2 y +1
=1
dx

2y

dy
dy
= x
+ y
dx
dx

2y

dy
dy
x
= y
dx
dx

2y x

f dydx = y

dy
y
=
dx
2y x

f
1

y+

1
y+

Solution: x = y +

1
y + ...
1

y+

g
x= y+

Solved Examples

x = xy + 1

1. y = x +

2x = x
1
x+

1
x+

1
x

dy
if
dx

1
x + ...

1
x + ...

x+

dy
1
=
dx 2 y + 1

1
y

Problems based on non-terminating continued


fraction

Find

x+

y = xy + 1

2. x = y +

y= x+

x 2y

y2 = x 2y

dy
dy
2
2 y x =
dx
dx

4y y x 1

14. y =

Solution: y = x +

dy
+ y
dx

2x y = x

FG
H

dy
dx

2x y
dy
=
dx
x

IJ
K

1
y+

1
y + ...

Implicit Differentiation

3. y =

or, y =

x
a+

y=

x
b+

x
x
b + ...

a+

y=

x x x x x x
...
a + b+ a + b+ a + b+

y=

y=

y=

a+

x
b+

x
x
a+
b + ...

x
b+ y

a+

a
a

f
f

2y

b+ y x
a b+ y + x

4. y =

sin x
cos x
1+
sin x
1+
cos x
1+
1 + ...

Solution: y =

ba

sin x
cos x
1+
sin x
1+
cos x
1+
1 + ...

dy
dy dy
+
+ cos x
y sin x
dx
dx dx

dy
d
sin x + 1 + y
sin x
dx
dx

f dydx = y sin x + a1+ yf cos x


y sin x + a1 + y f cos x
dy

=
dx a1 + 2 y + cos x sin x f

5. y =

tan x
cot x
1+
tan x
1+
cot x
1+
tan x
1+
cot x
1+
1 + ...

or, y =

tan cot x tan cot x tan


...
1+ 1 + 1 + 1+ 1 +

dy
dy
+ 2y
=b
dx
dx

dy
b
=
dx
ab + 2 y

d 2
d
1 + y sin x
y + y + y cos x =
dx
dx

aby + y + xy = bx + yx

ab

2 y + 1 + cos x sin x

x
a b+ y + x
b+ y

1 + y sin x
1 + y + cos x

sin x
cos x
1+
1+ y

y + y + y cos x = 1 + y sin x

Solution: y =

537

Solution: y =

tan x
cot x
1+
tan x
1+
cot x
1+
tan x
1+
cot x
1+
1 + ...

538

How to Learn Calculus of One Variable

y=

tan x
tan x
=
cot x
1 + y + cot x
1+
1+ y
1+ y

FG
H

IJ
K

a1 + yf tan x
a1 + y + cot xf
y b1 + y + cot x g = b1 + y g tan x
y + y + y cot x = a1 + y f tan x
d
d

y + y + y cot x j =
ba1 + yf tan xg
e
dx
dx
y=

1
1 dy
dy
=
2
dx 2 x
y dx

1 dy
1
dy
+ 2
=
dx
y dx 2 x

F
GH

I dy = 1
JK dx 2 x

1+

2y

F
H

dy dy
dy
+
+ cot x
y cosec 2 x
dx dx
dx

= 0+
2y

I
K

f a f

dy
d
tan x + 1 + y
tan x
dx
dx

dy
dy
dy dy
+
+ cot x
tan x
dx
dx
dx dx

= y cosec x + 1 + y sec x

1 + 2y + cot x tan x

f dydx

6. y =

x +

Solution: y =

Conditional identities on non-terminating continued


fraction or 'ad infinitum' form.
To form a differential equation with the help of a given
equation, we adopt the rule which consists of
following steps.
Step 1: Take the given and express it as an implicit
function of x.
Step 2: Find the derivative of the implicit function of x.
Step 3: Use mathematical manipulations to put the
first derivatives (or, the derivative involved in the
required differential equation) in to the required form
of the differential equations.
Solved Examples

y cosec x + 1 + y sec x
dy
=
1 + 2 y + cot x tan x
dx

= y cosec x + 1 + y sec x
2

dy
y

=
dx 2 1 + y 2

1. If y = a

ax

ax

...

show that

1
x +

x +

1
x +

dy
y log y
=
dx
x 1 y log x log y

1
x + ...
1

x +

Solution: y = a

1
x +

1
x + ...

y=a

a
ax

...

, a > 0, x > 0

y=

1
x +
y

log y = x log a

log log y = y log x + log log a

Implicit Differentiation

dy dy

= cos x
dx dx

1
1 dy dy
y

=
log x +
log y y dx dx
x

2y

FG 1 log xIJ dy = y
H y log y
K dx x

2y 1

y 2 log y
dy

=
dx x 1 y log x log y

2. If y = x

...

Solution: y = x
y= x

f dydx = cos x

cos x
dy
=
dx
2y 1

4. If y =

dy
y
=
, show that x
dx 1 y log x

log x + log x + log x + ... , show

f dydx = 1x .

that 2 y 1

...

Solution: y =

,x>0

...(1)

log y = y log x

y=

log x + log x + log x + ... , x > 0

log x + y

1 dy
dy
y

= log x
+
y dx
dx
x

FG 1 log xIJ dy = y
Hy
K dx x

y = log x + y

2y

dy 1 dy
= +
dx x dx

f dydx = 1x

2y 1

dy
y

=
dx
x 1 y log x

dy
y2
x
=
1 y log x
dx

3. If y =

sin x + sin x + sin x + etc ... to ,

Solution: y =

y=

...(2)

5. If y =

x
1+

cos x
dy
show that dx = 2 y 1 .

1+

Solution: y =

1+

x
1 + ...

x
1+

y=

x
1+ y

y y = sin x

dy
1
=
dx 2 y + 1

sin x + y

, show that

sin x + sin x + sin x + ...

y = sin x + y

539

y + y= x

x
1+

x
1+

x
1 + ...

540

How to Learn Calculus of One Variable

dy dy
+
=1
dx dx
dy
2y + 1
=1
dx
dy
1

=
dx 2 y + 1
2y

y=

y=

y+

y+

, show that

1
1
y+
...

x= y+
1=

1
y+

1
2. If and

1
y+
...

(from (1))

= 1 + x + y 2 xy
2
2
2
= x xy + x + y 2 xy ( Q x y =

5. If y = x3, show that x

1
x2
x

6. If y = x3, show that x

= 2 x + y 3 xy

7. If y = x +

dy
2
2
= 2 x + y 3xy
dx

8. If y = sin

Remark: Special care must be taken when implicit


function or a function defined by an infinite process
are differentiated for the function y = F (x) or
F (x, y) = 0 at which it is undefined. e.g.,

FG 1 + x IJ , | x | < 1, show that f a xf = g a xf .


H1 xK
f axf
= sec x .
3. If f (x) = sec x + tan x, show that
f a xf
x
dy
= y a1 y f .
4. If y =
, show that x
x+5
dx

xy = 1 from (1))
2

1+ x

...(1)

FG 2 3x IJ , show dy + 3 = 0 .
H 1 + 6x K
dx 1 + 9 x
x
f a x f = sin
and g (x) = tan

1. If y = tan

1
dy
2
dx x

Conditional identities based on explicit function of x

1
x

dy
1
=1+ 2
dx
x
=1+ x y

cos x
dy
=
which means that we can find
dx
2y 1

dy
1
when ever y .
dx
2

2
dy
2
= 2 x + y 3xy .
dx

y+

f dydx = cos x

2y 1
1

Solution: x = y +

sin x + y

y = sin x + y

6. If x = y +

sin x + sin x + sin x + ...

dy
+ 3y = 0 .
dx

dy
= 3y , x .
dx

dy
1
+ y = 2x .
, show that x
dx
x
x + sin

(Hint: Put x = sin ,

1 x , show that

)
2
2

dy
= 0.
dx

Implicit Differentiation

9. If x = sin
that

2u
1+ u

and y = tan

2u
1u

dy
= 1.
dx

10. If y =

11. If

, show

F
H

I
K

dy
2
cos x + sin x
= sec x +
, show that
.
4
dx
cos x sin x
y = cos

cos 3x
3

show

that

cos x

dy
=
dx

3. y =

sin x + sin x + sin x + ...

4. y =

cos x + cos x + cos x + ...

5. y =

x + x + x + ...

6. y =

7. x = y +

3
.
cos x cos 3x
sin

12. If y =

1 x

13. If y = cos

,m show that (1 x2)

dy
xy = 1
dx

FG 3 + 5 cos x IJ , show that cos x =


H 5 + 3 cos x K

dy
4 5 y1
, where y1 =
.
dx
3 y1
14. If y = log

1 + tan x
dy
= sec 2 x .
, show that
1 tan x
dx

dy
+ y=0
dx
16. If y = eax sin (bx + c), show that

15. If y = ex, show that

LMa
N

2
2 ax
1 b
dy
= a + b e sin bx + c + tan
dx
a

Problems based on infinite series


Exercise 12.8

Find

2. y = x

1
y + ...

y+

1
x+

9. x = y

1
x+

x+e

...

x + ...

1
x + ...

...

Answers
y
1. 1 y

F
GH

1
y
2.
x 1 y log x

cos x
3. 2 y 1
sin x
4. 1 2 y
1
5. 2 y 1
2

x +e

y
6.
x 2 y log x

dy
if
dx

1. y = e

OP
Q

8. y =

...
x

7. 1 +
8.

1
x2
y

1+ y

I
JK

541

542

9.

How to Learn Calculus of One Variable

y 1 x log y
x

2. If y =

a2 y 1f dydx = cos x .

Conditional problems

(Hint: y =

Exercise 12.9

...

dy
1. If y = x
, show that y = x 1 y log x
.
dx
y
(Hint: Here y = x )
x

sin x + sin x + sin x + ... , show that

sin x + y y = sin x + y )

3. Show that

sin x
dy
=
, when
dx 1 2 y

cos x + cos x + cos x + ... .

Logarithmic Differentiation

543

13
Logarithmic Differentiation

Question: What is logarithmic differentiation?


Answer: Taking logarithm of both sides of an identity
before differentiation is known as logarithmic
differentiation.

implicit function or composite of two (or, more)

Question: Where to use logarithmic differentiation?


Answer: Logarithmic differentiation is used when
1. The given function defining y as a function of x is
the product of two differentiation functions of xs,
one (or both) of which may be quotient, power (or,
under radical), implicit function or composite of two
(or, more) functions; i.e. when y = f1 (x) f2 (x) where f1
(x) and f2 (x) are differentiable functions of xs.
2. The given function defining y as a function of x is
the product of a finite number of differentiable
functions of xs, some of which may be quotients,
powers (or, under radicals), implicit functions or
composite of two (or, more) functions; i.e. when y = f1
(x) f2 (x) f3 (x) fn (x), where f1 (x), f2 (x) fn (x) are
differentiable functions of xs.
3. The given function defining y as a function of x is
the quotient of two differentiable functions of xs
whose numerator and denominator may be a power
function (or, a function under radical), implicit function
or composite of two (or, more) functions; i.e. when

where f1 (x), f2 (x), , fn (x) and g1 (x) , g2 (x), , gn (x)


are differentiable functions of xs.
5. The given function defining y as a function of x is
a power of the function of x whose base is an implicit
or explicit function of x whereas the index is a real
number; i.e. when y = [f1 (x)]n, where f1 (x) is a
differentiable functions of x whereas the index is a
real number.
6. The given function defining y as a function of x is
composite exponential function which is a function
whose both the base and the exponent are
differentiable functions of xs; i.e. when

y=

af
af

f1 x
, where f1 (x) and f2 (x) are differentiable
f2 x

functions of xs.
4. The given function defining y as a function of x is
the quotient whose numerator and denominator
contain a finite number of differentiable functions of
xs some of which may be powers or, under radicals),

functions; i.e. when y =

af

y = f1 x

af af af af
af af af af

f 1 x f 2 x f 3 x ... f n x
,
g1 x g2 x g3 x ... gn x

a f,

f2 x

where f 1 (x) and f 2 (x) are


differentiable functions of xs. (Note: Logarithmic
differentiation may be defined as differentiation after
taking logarithm of both sides of an identity.
Remember:
1. Logarithmic differentiation is practically useful
when the given function defining y as a function of x
is a complicated one consisting of products, quotients
and powers (or, radicals) of differentiable functions
whose derivatives can not be found easily by using
the rule of differentiating the products, quotient or
power of the functions being differentiable.
2. We take logs throughout to avoide repeated
applications of the rules for the differentiation of
products and quotients of functions of xs.

544

How to Learn Calculus of One Variable

3. We can use logarithmic differentiation only when


the function concerned is positive in its domain.
Example
y=

sinn

x log y = sin x

log y = n log sin x

1 dy n cos x
=
y dx
sin x

dy
= n sin n 1 x cos x
dx
This result is valid when n is an integer > 1, whether
sin x is positive or negative.

Question: What are the rules of logarithmic


differentiation?
Answer: There are two rules of logarithmic
differentiation.
Rule 1: When y = f (x), f (x) being differentiable
positive function (i.e. f (x) > 0),

a f

a fg

d
d
log y =
log f x
dx
dx

af af

1 dy
1
d

f x , f x >0
y dx
f x dx

af

Rule 2: When y = f (x), f (x) being differentiable and

af

f x 0

bg

d
d
log y =
log f x
dx
dx

af af

1 dy
1
d

=

f x ,f x 0
y dx
f x dx

af

Notes: (A)
(i) y =

af

f x

y = ef (x)

(ii)
(iii) y = | f (x) |
(iv) y = log f (x)
(v) log a x f f x = y

af

(vi) y = log log log log f (x) are differentiable


functions where f (x) > 0 is pre assumed but in y = | f (x) |,
f (x) may be greater than zero or less than zero
remaining the possibility of being zero also whereas
| f (x) | means always > 0. For this reason wherever

f (x) < 0, we multiply f (x) by minus one (i.e. 1) to make


f (x) > 0.
(B)

af

d
log f x
dx

af
af

f x
f x

which means the

derivative of the function log | f (x) | is a logarithmic


derivative of the function f (x). To simplify the notation
in logarithmic differentiation, the sign of absolute
value of the function f (x) is usually omitted only
when the function f (x) concerned is positive. Hence,

af

af
af

f x
d
.
log f x =
dx
f x
(C) Logarithmic differentiation simplifies finding the
derivative of the given function.

this is why we write

On working rule of problems belonging to first type


When we are given the interval or the quadrant in
which f (x) = given function of x is positive or the
condition imposed on the independent variable x
makes f (x) > 0, we adopt the following rule to find the
differential coefficient of the given function by using
logarithmic differentiation.
Step 1: Take logarithm on both sides of the equation
defining y as a function of x, i.e. log y = log f (x).
Step 2: Differentiate the equation log y = log f (x)
using the rule:
(i)

1 dy
d
log y =
,y>0
dx
y dx

(ii)

d f x
1
d

, f (x) > 0
log f x =
dx
f x
dx

(iii)

af

af
dy
d
= y
log f a x f
dx
dx

af

Remember: (a) After taking logarithm, we are


required to use the following formulas.
(i) log u v = log u + log v, u > 0, v > 0.

u
= log u log v , u > 0, v > 0.
v
(iii) log uv = v log u, u > 0, v 0.
(ii) log

(iv) log u = n log u , n R , u > 0.


where u and v are functions of x.

545

Logarithmic Differentiation

(b) The above working rule may be remembered as


GLAD, the letters being in order which means
G = given function
L = take the logarithm
A = apply the logarithmic formulas
D = differentiate.
(c) We take logarithm on both sides of the equation
defining y as a function of x only when the given
function is positive because logarithmic differentiation is applicable only when the function concerned
is positive.
(d) The above working rule is applicable to the

af

a f provided

f2 x

composite exponential function f1 x


the function in the base is positive and for this reason
the above working rule is applicable to ef (x) since
e = 2.718 > 0.
Remarks: To take the logarithm of any quantity, we
have to be sure that it is positive.
Problems based on first type

Find the differential coefficient of the following.


1. y =

af

Solution: y =

x = x

1
2

1
log x
2
1 dy 1 1

=
y dx 2 x

1
2
dy
x
= y
+ 2

dx
x
x +1 3 x +1

F1 + x 2 I
b x + 1g GH x x + 1 3b x + 1gJK
I
F
3. y = cos x , H < x < K
2
2
Solution: y = acos x f
=

2
3

1
2

log y =

1
log cos x
2

1 dy 1
1

=
sin x
y dx 2 cos x

dy
1
= y tan x
dx
2
=
3

1
2

cos x tan x

log x
1
3

, defined for x > 0

1
log | log x |
3
1 dy 1
1
1
1

=
=
y dx 3 log x x 3x log x
log | y | =

dy
x
1
=
,x>0=
, x > 0.
dx
2x
2 x

x x +1
2

Solution:

x x2 +1

a f

| y| =

I
a fJK

Solution: y = log x

dy
y
=
dx 2 x

b x + 1g

F
GH

4. y =

2. y =

b g

1 dy 1 1
1
2
1

= + 2
2x
1
y dx x 2 x + 1
3 x +1

, defined for x > 0

log y =

Form 1: Problems based on irrational functions.


Solved Examples

1
2
log x 2 + 1 log x + 1
2
3

log y = log x +

dy
y
=
,x>0
dx 3x log x

Form 2: Problems based on product of two or more


than two differentiable functions.

2
3

| x| x 2 +1

b x + 1g

2
3

Solved Examples
Find the differential coefficient of the following.

546

How to Learn Calculus of One Variable

1. y = x 2 e 2 x sin 3x
2

Solution: y = x e

2x

= x 3 log x
sin 3x

FG 1 + 1 IJ , x > 0
H x 3x log x K

| y | = x 2 e 2 x | sin 3x |

Form 3: Problems based on quotient of two or more


than two differentiable functions

log| y | = log x 2 + log e 2 x + log | sin 3x |

Solved Examples

log | y | = 2 log| x | + 2 x + log | sin 3x | 3 log e = 1

3 cos 3x
1 dy 2

= +2+
y dx x
sin 3x

dy
2
= y 2 + 3 cot 3x +
dx
x

F
H

FG
H

Find the differential coefficient of the following.


1. y =

I
K

Solution: y =

= x 2 e 2 x sin 3x 2 + 3 cot 3x +

2
x

IJ
K

1 dy
1

=
y dx
x log x

dy
y
4
4
=
=
=
dx
x log x
x log x log x
x log x

Solution: y = e log x , defined for x > 0

log| y | = log e x + log| log x | = x + log | log x |

FG
H

dy
1

= y 1+
dx
x log x

2. y =

FG
H

= e log x 1 +

IJ
K

a f

Solution: y =

1
x log x

a f

1
3

,x>0

1 dy 1 1
1
1 1
1

= +
= +
y dx x 3 log x x x 3x log x

FG
H

IJ
K

log x
x

3. y =

FG
H

1 dy
1
1
1
1

=
= y

y dx
x log x
x
x log x
x
=

1
log | log x |
3

dy
1
1
= y
+
dx
x 3x log x

,x>0

log y = log log x log x

IJ
K

Solution: y = x 3 log x = x log x

x log x

log x
,x>0
x

3. y = x 3 log x

log | y | = log x +

b3 log e =1g

1 dy
1

=1+
y dx
x log x

4
,x>0
log x

log | y | = log 4 log | log x |

2. y = e log x

4
log x

e +e

Solution: y =

IJ
K

log x 1 log x
1 log x
=
2
x
x log x
x

e e

FG
H

IJ
K

e e
e +e

log| y | = log e x e x

log e x + e x

547

Logarithmic Differentiation
x

ee
=

j ee e j
ee + e j ee e j

x 2

+e

x 2

F
GH

e e
e +e

e e

je

e +e

I
JK

3. y = x x

Solution: y = x x , defined for x > 0

log y =

ee

+e

x 2

Form 4: Problems based on exponential composite


functions. y = f (x)g (x) which is defined only when the
base f (x) > 0.
Note: Whether the questions says or does not say
about the base f (x) of the exponential composite
function y = f (x)g (x) to be positive, it is understood
always that the base f (x) > 0 since y = f (x)g (x) is
defined only when f (x) > 0.
Solved Examples
Find the differential coefficient of the following.
1. y = (sin x)sin x, sin x > 0
Solution: y = (sin x)sin x

a f

log y = log sin x

sin x

1 dy
x

=2
+ log 1 + x
y dx
x+1

2x

e 2 x + e 2 x + 2 e 2 x e 2 x + 2
dy
= y
dx
e 2 x e 2 x
=

LM
OP
a
f
N
Q
L x + log a1 + x fOP
dy

=2yM
dx
Nx + 1
Q
L x + log a1 + xfOP , x > 1
= 2 a1 + x f M
Nx + 1
Q

1 dy e + e
e e

= x
x
x
x
y dx e e
e +e

1 dy
1
1 1 1 log x

= 2 log x + =
2
y dx
x x
x
x

LM
N
y = b1 + x g

4.

dy
1 log x
= y
2
dx
x

log y = 2 x log 1 + x

Solution: y = 1 + x

log x

, defined for x > 1

log y = log x log 1 + x

log 1 + x
log x
1 dy

=
+
y dx 1 + x
x

log 1 + x
log x
dy
= y
+
dx
1+ x
x

LM
N

= 1+ x

1 dy
1

= sin x
cos x + cos x log sin x
y dx
sin x

OP
Q

log x

= sin x log sin x

dy

= y cos x + cos x log sin x


dx
2. y = (1 + x)2x
Solution: y = (1 + x)2x, defined for x > 1

1
log x
x

ex j

log x

f OP
Q

LM log x + log a1 + x f OP
x
N1 + x
Q

5. y = x

,x>0

ex j
x

Solution: y = x

...(i)

e j log x

log y = x

log y = x x log x

...(ii)

548

How to Learn Calculus of One Variable

Again taking logarithm on both sides of the


equation (ii), we have log | log y| = log [(xx) | log x |] =
x log x + log | log x |

1
1 dy
1
1
1

= x + log x +

log y y dx
x
log x x
= 1 + log x +

1
x log x

LM
N

dy
1
= y log y 1 + log x +
dx
x log x

OP
Q

e
dy
= x
dx

Remember: The operation of taking logarithm on


both sides of the equation defining y as a function of
x (i.e. y = f (x) may be performed more than once if we
go on having an exponential composite function just
before the differentiation and we differentiate the
logarithmic function only when the occurrence of
exponential composite function in any intermediate
step is over as a factor of logarithmic function or,
alternatively we make the substitution for the
exponential composite function occuring in any
intermediate step after differentiation which is
explained in the following way.

ex j
x

y=x

a fe

log y = log x

e j log x

= x

...(i)

e j

x
x
1 dy
1
d

= x + log x
x
y dx
x
dx

F
H

e jIK

x
x
dy
1
d

= y x + log x
x
dx
x
dx

e j + x log x

1 dz
1

= x + log x 1
z dx
x

...(iii)

Now on putting (iii) in (ii) , we have

e j FGH
F1
x G + log x + log
Hx

IJ
K
I
xJ
K

= xx

Form 5: Problems based on the sum of two more


than two exponential composite (or, other) functions.
One should note that logarithmic differentiation of
the sum of two or more than two exponential
composite functions are performed by using the
theorem of differential coefficient of the sum of two
or more than two differentiable functions and the
differential coefficient of each addend being the
function of x is obtained seperately by using the rule
of logarithmic differentiation and lastly adding the
differential coefficient of each addend, we get the
differential coefficient of the whole function which is
given as the sum of two or more than two exponential
composite functions.
Note: We should remember that while finding the
differential coefficient of each addend being the
function of x, we must make the substitution u, v, w, ...
for each addend and then operation of taking logarithm
should be performed.
Solved Examples
Find the differential coefficient of the following.

b g

1. y = x tan x + tan x
...(ii)

Now on supposing that z = xx, x > 0, we have


log z = log x

e j a1 + log xf

= x

...(iii)

e j log x LMN1 + log x + x log1 x OPQ

dz
= z 1 + log x
dx

dy
1
= y xx
+ log x + log 2 x
dx
x

Now, putting (i) and (ii) in (iii), we have


x

cot x

FG
H

, 0<x<

IJ
K

Solution: y = xtan x + (tan x)cot x, defined for x > 0, tan


x>0

e j

ea f j

d
tan x
dy
d
cot x
tan
=
x
+
dx
dx dx
Now, on letting u = xtan x, we have
log u = log (xtan x) = tan x log x

...(i)

549

Logarithmic Differentiation

log v = sin x log tan x

1 du
1
2

= tan x + log x sec x


u dx
x

FG
H

du
tan x
2

=u
+ sec x log x
dx
x
=x

tan x

FG tan x + sec
H x

IJ
K

x log x

IJ
K

...(ii)

1 dv cot x
2
2

=
sec x cosec x log tan x
v dx tan x

cot x
2
2
dv
= v
sec x cosec x log tan x
dx
tan x

FG
H

I
x
sec x cosec x log tan xJ
a f FGH cot
tan x
K
2

IJ
K

Putting (ii) and (iii) in (i), we have

IJ a f
K
FG cot x sec x cosec x log tan xIJ
H tan x
K
I
F
2. y = (sin x)
+ (tan x) , H 0 < x < K
2
FG
H

dy
tan x tan x
2
=x
+ sec x log x + tan x
dx
x

tan x

Solution: y = (sin

a f

...(i)

1 du
1

= tan x
cos x + log sin x sec 2 x
u dx
sin x

e
= asin x f

du
2
= u 1 + sec x log sin x
dx
tan x

tan x

sin x

3
2

3
2

sin x

3
2

Now on letting u = xsin x, we have


log u = sin x log x

b g

1 du
1

= sin x + log x cos x


u dx
x

du
sin x
=u
+ cos x log x
dx
x

FG
H

IJ
K

FG sin x + cos x log xIJ


K
H x
Again, on letting v = asin x f
dv 3

= asin x f cos x
dx 2
sin x

...(ii)

3
2

dv 3
= cos x sin x
dx 2
Putting (ii) and (iii) in (i), we have

FG
H

1 + sec x log sin x

1
2

= tan x cot x + sec2 x log sin x

sin x

=x

(tan x)sin x

a f

a f e1+ sec x log sin xj + atan xf


asec x + cos x log tan xf

+ asin x f , 0 < x <


3. y = x
2
Solution: y = x
+ asin x f
dy
d

=
...(i)
e x j + dxd asin xf
dx dx
dy
= sin x
dx

dy d
d
tan x
sin x
=
+
sin x
tan x
dx dx
dx
Now, on letting u = (sin x)tan x, we have
log u = tan x log sin x

cot x

sin x

x)tan x

a f

sin x

...(iii)

dv
= v sec x + cos x log tan x
dx

sin x
= tan x
sec x + cos x log tan x ...(iii)
Putting (ii) and (iii) in (i), we have

cot x

= sec x + cos x log tan x

Again, on letting v = (tan x)cot x, we have


log v = log (tan x)cot x = cot x log tan x

= tan x

1 dv
1

= sin x
sec 2 x + log tan x cos x
v dx
tan x

IJ
K

...(iii)

3
dy
sin x sin x
=x
+ cos x log x + cos x sin x
2
dx
x

Again, on letting v = (tan x)sin x, we have

...(ii)

4. y = (sin x)x + x sin1 x, 0 < x < 1


Solution: y = (sin x)x + x sin1 x

550

How to Learn Calculus of One Variable

a f

d
1
dy
d
x
x sin x
=
sin x +
...(i)
dx
dx dx
Now on letting u = (sin x)x, we have
log u = x log sin x
1 du
1

= x
cos x + log sin x 1
u dx
sin x

du

= u x cot x + log sin x


dx
= (sin x)x (x cot x + log sin x)
Again on letting v = x sin1 x, we have
log v = log x + log sin1 x
1 dv 1
1
1

= +

2
v dx x sin 1 x
1 x

F
GG
H

dv
1
=v
+
dx
x

= x sin

= sin

1
sin

F1
GG x +
H sin

x 1 x

I
JJ
K

1
1

x 1 x

1 x

...(ii)

du
1
=
dx
x

I
JJ
K

FG
H

Now on letting u =

...(ii)

a1 log xf
x

dv v 1 log x
=
2
dx
x

x x 1 log x
2

x
Putting (ii) and (iii) in (i), we have

F I a1 + log xf + x a1 log xf
H K
x

dy
1
=
dx
x

1
x

6. y = x2x + (2x)x, x > 0


Solution: y = x2x + (2x)x

1
x

F I
H K

1
d
xx
dx

F 1I
H xK

1 dv 1 1
1
1
1

=
log x = 2 2 log x
v dx x x x 2
x
x

...(i)

, we have

F 1 I = x log e x j = x log x
H xK
1

a f

e j

dy
d 2x
d
x
2x
=
x
+
...(i)
dx dx
dx
Now on putting u = x2x, we have
log u = log (x2x) = 2x log x
1 du
2x

= 2 1 log x +
= 2 log x + 2
x
u dx
du
2x

= u 2 log x + 2 = 2 x 1 + log x
...(ii)
dx
Again, on putting v = (2x)x, we have
log v = log (2x)x = x log 2x

+ x

IJ blog x + 1g
K

1
log x
x

1
x

Again on letting v = x x , we have

F 1I
Solution: y = H K
x
dy
d F 1I

=
dx dx H x K

+ x ,x>0

log u = x log

...(iii)

b g b

F I
H K

du
= u log x + 1
dx

log v =

dy
x
x
= sin x x cot x + log sin x + sin 1 x +
dx
1 x 2
x

Putting (ii) and (iii) in (i), we have

1
5. y =
x

1 du
1

= x + log x 1 = 1 + log x
u dx
x

x+

a f a

1 dv
1

= x
2 + log 2 x = 1 + log 2 x
v dx
2x

Logarithmic Differentiation

fa fa

dv
x
= v 1 + log 2 x = 2 x 1 + log 2 x
dx
Putting (ii) and (iii) in (i), we have

f a f a1 + log 2 xf

dy
2x
= 2 x 1 + log x + 2 x
dx
7. y = xlog x + (log x)x, x > 1
Solution: y = xlog x + (log x)x

a f

e j = log ee j
y log x = a x y f log e = a x y f a3 log e = 1f
log x

log x
1 2x
du
= 2u log x =
dx
x
x
Again on putting v = (log x)x, we have
log v = x log log x

=
...(ii)

1 dv
x 1
1
=
+ log log x =
+ log log x
v dx log x x
log x

dv
1
=v
+ log log x
dx
log x

IJ
K

...(iii)

Putting (ii) and (iii) in (i), we have

a f FGH log1 x + log log xIJK

+ log x

Form 6: Miscellaneous problems


Solved Examples

xy = ex y , x > 0, x

1. If

log x
dy
=
dx
1 + log x

1
,
e

Solution: xy = ex y

show

that

2. If xm yn = (x + y)m + n, x > 0, y > 0, show that

dy
y
= , nx my .
dx
x
Solution: xm yn = (x + y)m + n

j = log a x + yf
m log x + n log y = am + nf log a x + y f
m n dy F m + n I F

+
=G
J 1 + dydx IK
x
y dx H x + y K H
F m + n n IJ dy = m m + n = mx + my mx nx
G
x a x + yf
H x + y y K dx x x + y
m

m+ n

my + ny nx ny dy
my nx

=
y x+ y
dx x x + y

my nx dy
my nx

=
y x + y dx x x + y

dy
y
= , since , nx my
dx x
Note: The result holds for x 0 , y 0 (shown later)

a1 + log xf

dy
log x
=
dx
1 + log x

F 1 + log log xIJ


= alog x f G
H log x
K

1 + log x 1

log x y

log x

F 1I
IJ a1+ log xf 1 x H 0 + x K
K
a1+ log xf

FG
H

log x

x
1 + log x

dy d
x

=
=
dx dx 1 + log x

log x

dy 2 x
=
dx

y log x + 1 = x

...(i)

1 du
1

= 2 log x
u dx
x

FG
H

x y

y log x + y = x

y=

dy
d log x
d
x
log x
=
x
+
dx dx
dx
Now, on putting u = xlog x, we have
log u = log x log x = (log x)2

...(iii)

551

552

How to Learn Calculus of One Variable

FG x IJ , x 1 , 0 , find its derivative.


H1 + xK
F x IJ log x log (1 + x) for
y = log G
H1 + xK

3. If y = log

Solution:
x 1 , 0

When x > 0 , x + 1> 1> 0

x
> 0 which means
1+ x

y is defined for x > 0.


Again when x < 1 , x + 1< 0
means y is defined for x < 1.

x
> 0 which
1+ x

y is defined for x R 1 , 0 = , 1

a f

0,

i.e. y is defined for x 1 , 0

Hence,

IJ IJ
KK

LMb x + 1g 1 x 1OP b x + 1g L x + 1 x O
MM e1+ x j PP = x MMN b1+ xg PPQ
Q
N
2

Remark:
1. In the above problem log

af

af

y= f x y = f x

Step 2: Take the logarithm on both sides of the


equation defining modulus of y as modulus of f (x),
i.e. log | y | = log | f (x) |
Step 3: Differentiate the equation log | y | = log | f (x) |
using the rule:
1 dy
d
log y =
; y 0.
(i)
dx
y dx
d f x
1
d

log f x =
; f x 0.
(ii)
dx
f x
dx

bg bg
bg
f b xg ; f b x g 0 .

bg

2. log

u
= log u log v ; u 0, v 0.
v

3. log | uv | = log | u |v = v log | u | ; u > 0, v R .

1
, x 1 , 0
x x +1

Step 1: Take the modulus on both sides of the


equation defining y as a function of x, i.e.

dy
d
= y
log
dx
dx
Remember: (a) After taking logarithm, we are
required to use the following formulas.
1. log | u v | = log | u | + log | v | ; u 0, v 0 .

dy
d
x
log
=
1+ x
dx dx

x +1

af

(iii)

FG FG
H H

condition imposed on the independent variable x


which makes f x 0 is given (or, not given), we
adopt the following working rule to find the differential
coefficient of the given problems by using logarithmic
differentiation.

FG x IJ log x log
H 1+ x K

(1 + x) since log x is undefined for x < 0 and also log


(x + 1) is undefined for x 1 .
2. x 1 , 0 x R 1 , 0

On working rule of problems belonging to second


type
When the interval or the quadrant in which f (x) =
given function of x is positive is not given or the

4. log u n = log u

= n log u , n R , u > 0.

where u and v are functions of x.


(b) The above working rule may be remembered as
MLAD the letters being in order which means
M = take mod
L = take log
A = apply logarithmic formulas
D = differentiate.
(c) We take the modulus of given function only when
the function may be negative because we can use
logarithmic differentiation only when the function
concerned is positive in its domain; i.e. the sign of
absolute value is written only when the expression
standing under the sign of logarithm may have a
negative value in its domain.

Logarithmic Differentiation

(d) It should be noted that the derivative of any

af

d
dy
function of y, say F (y), w.r.t. x is
i.e.
F y
dy
dx
dy
F y
obtained by the rule of differentiating a
dx
function of a function.

af

Problems based on second type

a f

1 dy
1
2

=
= 2
2
y dx 2 x 1
x 1

dy
y
= 2
dx
x 1

Solved Examples

Find the differential coefficient of the following.

Solution: y = 3 1 x

y = 1 x2

= 1 x

1
3

1 dy
2 x

=
y dx 3 1 x 2

dy
2 y x
=
dx 3 1 x 2

dy 2 x 1 x
=
dx
3 1 x2

= 1 x2

1
log 1 x 2
3

log y =

3. y =
1
3

FG x + 1IJ
H x 1K
F x + 1IJ
Solution: y = G
H x 1K
2. y =

2x

e1 x j
2

2
3

1
3

1
3

1
log cos x
3

1 dy 1 sin x
1

=
= tan x
y dx 3 cos x
3

dy
1
1
= y tan x = 3 cos x tan x
dx
3
3
1 cos x
1 + cos x

, x2 1

Solution:

F 1 cos x IJ
y=G
H 1 cos x K
1

1
2

x >1

4. y =

cos x

log y =

1
2

1 cos x 2
2 x
y =
= tan
1 + cos x
2

log y = log tan

x +12
y =
x 1

e j
FG x + 1IJ 1 ,
H x 1K e x 1j

y = cos x

j
=

b g

Solution: y = 3 cos x = cos x


1
3

a f

1
1
x +1 1
= log x + 1 log x 1
log y = log
2
2
x 1 2

Form 1: Problems based on irrational functions.

2
1. y = 3 1 x

553

1
2

= tan

x
2

F
H

I
K

1 dy
1
1
2 x

=
sec

x
2 2
y dx tan
2

x
2

2 21

= tan

x
2

554

How to Learn Calculus of One Variable

1
=
2

x
1
2 1 =
2 x
x
x
x
2 sin cos
sin
cos
2
2
2
2

dy
1
= y
=
dx
sin x

Solution:

1 cos x
cosec x , x n
1 + cos x

log y =

1
2

Note: Differentiation of mod of a function can be


performed easily by logarithmic differentiation.

a
a

f a
f a

f
f

2 cos 2 x

2
2 cos 2 x

= sec 2x

dy
= y sec 2 x
dx
cos x sin x
=
sec 2 x
cos x + sin x

a x + yf
= x+ y

m+n

m+ n

nx my dy
nx my

=
y x + y dx x x + y

dy
y
= ; x 0, y 0
dx
x
2
2. y = x (x + 1)
Solution: y = x (x2 + 1)

6. y = | cos x sin x |
Solution: y = | cos x sin x |

f a
f a

a f
m n dy m + n F
dy I

+
=
1+
x
y dx
x+ y H
dx K
F n m + n IJ dy = m + n m
G
H y x + y K dx x + y x
anx + nyf amy + nyf dy

y a x + yf
dx
amx + nyf amx + my f
=
x a x + yf

log y = log cos x sin x

m log x + n log y = m + n log x + y

cos x + sin x
cos x sin x
1 dy

2 cos x + sin x
y dx 2 cos x sin x

a
a

x y

g bcos x sin xg
=
2 ecos x sin x j
a1 + 2 sin x cos x f a1 2 sin x cos x f
=
=

fa

cos x sin x sin x + cos x


, cos x sin x 0
cos x sin x

Find d.c. of y from the following.


1. xm yn = (x + y)m + n , nx my
Solution: xm yn = (x + y)m + n

1
1
log cos x sin x log cos x + sin x
2
2

f
f

Solved Examples

1
2

cos x sin x
1
log
cos x + sin x
2

sin x + cos x

a
a

sin x + cos x
dy
= y
dx
cos x sin x

Form 2: Problems based on product of two or more


than two differentiable functions.

F cos x sin x IJ
y=G
H cos x + sin x K

cos x sin x
y =
cos x + sin x

cos x sin x
cos x + sin x

5. y =

cos

f
f

sin x cos x
sin x + cos x
1 dy

=
=
y dx
cos x sin x
cos x sin x

y = x x +1 = x

y = x x +1

ex

+1

Logarithmic Differentiation

been used only to show that the procedure of


logarithmic differentiation is also applicable in that
case.
5. y = sin x sin 2x sin 3x sin 4x
Solution: y = sin x sin 2x sin 3x sin 4x

log y = log x + log x + 1

1 + 3x
1 dy 1
2x

= +
=
y dx x x 2 + 1 x x 2 + 1

L 1+ 3x OP
LM 1+ 3x OP
dy

= y MM
=
x

x
+
1

e j M x x +1 P
dx
MN x e x + 1j PPQ
MN e j PQ
2

dy
2
= 1 + 3x
dx
Note: The above problem (2) can be differentiated in
a much simpler way by using directly the product
formulas but above method has been used only to
show the procedure of logarithmic differentiation is
also applicable.
3. y = x log x
Solution: y = x log x, x > 0

y = sin x sin 2 x sin 3x sin 4 x


log y = log sin x + log sin 2 x + log | sin
3x | + log | sin 4x |
1 dy cos x 2 cos 2 x 3 cos 3x 4 cos 4 x

=
+
+
+
y dx sin x sin 2 x
sin 3x
sin 4 x

dy
= y (cot x + 2 cot 2x + 3 cot 3x + 4 cot 4x)
dx
dy

= (sin x sin 2x sin 3x sin 4x) (cot x +


dx
2 cot 2x + 3 cot 3x + 4 cot 4x)
6. y = x | x |
Solution: y = x | x |

y = x log x = x log x

y = x x

log y = log x + log log x

log y = log x + log x

log x + 1
1 dy 1
1

= +
=
y dx x x log x
x log x

log x + 1
log x + 1
dy
= y
= x log x
= log x + 1
dx
x log x
x log x

LM
N

OP
Q

LM
N

OP
Q

4. y = x log y
Solution: y = x log y
y = x log y
log y = log x + log log y

1 dy 1
1
dy

= +

y dx x
y log y dx

FG 1 1 IJ dy = 1
H y y log y K dx x
log y
dy 1 F x log y log y I alog y f

= G
=
=
J
dx x H log y 1 K log y 1 log y 1

555

Note: The problems (1) and (4) belong to problems


of implicit function which can be done by using the
rule of implicit differentiation but above method has

1 dy 2

=
y dx
x

dy
y 2x x
=2 =
=2 x
dx
x
x

Precaution:
1. Whenever we have to find the differential
coefficient of mod of a function by logarithmic
differentiation, the operation of taking the modulus
on both sides of the equation defining y as a function
of x is not performed but directly the operation of
taking logarithm on both sides of the equation defining
y as a function of x is performed just before the
differentiation.
2. Whenever we have to find the differential
coefficient of a function using the mod of a function,
the operation of taking the modulus and logarithm on
both sides of the equation just defining y as a function
of x must be performed respectively before the
differentiation.

556

How to Learn Calculus of One Variable

3. Whenever we have to find the differential


coefficient of a logarithmic function (i.e. y = log f (x) or
log log log f (x)), we have not to perform both the
operation of taking the modulus and logarithmic
function of x but simply, we have to use the formula:

af af
af

af

Solved Examples
Find the differential coefficient of the following.
sin 3 x

1. y =

e4x x 3 + 5

f x
d
, f x > 0.
log f x =
dx
f x
Examples
Find the differential coefficient of the following.
1. y = | x |
Solution: y = | x |

x2 + 1

Solution: y =

1
3

sin 3 x

x2 + 1

e4x x 3 + 5

log y = log x

log y = 3 log sin x +

1 dy 1

=
y dx
x

1
3
log x + 5
3

x
dy
y
= =
,x 0
dx
x
x

2. y =

y =

dy
x
x
= y 3 cot x + 2
4 3
dx
x +1
x +5

,x 0

x
= 1 ( 3 mod of a mod of a function
x

= mod of a function)

log y = log 1 = 0 3 log 1 = 0

1 dy

=0
y dx

2. y =

4x

x +1
2

Form 3: Problems based on quotient of two or more


than two differentiable functions.

1
3

3 cot x +

x
x +1
2

x
x

I
JK

I
J
+ 5K
2

2 x 1 sin 1 x

ex

log y
2

F
GH
e x + 5j

sin x
3

F
GH

+2

Solution: y =

dy

=0
dx
3. y = log (x2 + 1)
Solution: y = log (x2 + 1)

dy F 1 I

=G
J 2 x = 1 +2 xx
dx H x + 1K

cos x
1 dy
2x
3x

=3
+
4
2
3
y dx
sin x 2 x + 1
3 x +5

x
x

1
2
log x + 1 4 x
2

Solution: y =

1
3

3
2

3 tan 1 2 x
2 x 1 sin 1 x

ex

,x>

j tan 2 x
1
= log a2 x 1f + log sin
2
e

+2

3
2

1
3
1
2
log x + 2 log tan 2 x
3
2

1
2
x

557

Logarithmic Differentiation

1 dy
2
1

=
+

1
y dx 2 2 x 1
sin x 1 x 2

g e

3 2x

2 x +2

F
GG
H

3 tan 2 x 1 + 4 x

3
4

4.

3 tan
=

I
2
JJ
2 x e1 + 4 x j K

x2 + 2

3
2

log y

x +1
2

Solution: y =

1 x

3x

x +2
2

5
3

+ 3 x3 + 7

4
3

1
4
log x log
2
3

f e

x +2

x 2x + 1

F
GG
Ha f e

3
4

dx

a f

j e

j ex
x e x + 2j
2

+3

5
3

F 3
GG 4 b x + 1g + 10x + 21x
e3x + 9j e x + 7j
H
I
1
2
J

2 x 3 x e x + 2j J
K
a x + 1f e sin aax f
y=
a x 1f a x + 2f
mx

3
4

5.

+7

4
3

7
3

5
7

I
J
j JK

FG
IJ
a fK
H
a2 x + 1f F 2
4
1 I
G +

J
2
1
2
+
+ 1f K
x
x
x
a
H
x +1

dy
3
10x
21x
= y
+
+ 3

2
dx
4 x +1
3x + 9
x +7

dy a x + 1f e x

j e

x +2 2 x

1
2

2x 3 x x + 2

dy
2
4
1

= y
+

dx
x 2x + 1 2 x + 1
2

2x 3

dy x
=
dx

1
log y = 2 log x + 2 log 2 x + 1 log x + 1
2

x +1

5 2 x
7 3x
1 dy
3

=
+
+ 3

2
y dx 4 x + 1
x +7
3 x +3

+7

j e j ,x>0
x e x + 2j
3
5
= log a x + 1f + log e x + 3j +
4
3

x 2x + 1

4
3

7 log x 3 + 7

F 1
1
where k = G
GH 2 x 1 + esin xj
I
2
J
3 tan 2 x e1 + 4 x j JK
3. y =

5
3

3 tan 1 2 x

+3

3
4

2 x 1 sin 1 x

j ex
x e x + 2j
2

b x + 1g e x
Solution: y =

dy
1
1
3x

=y
+
2

2
1
dx
2x 1
x
+2
sin x 1 x

ax + 1f e x
y=

558

How to Learn Calculus of One Variable

a x + 1f e sin aax f
y=
a x 1f a x + 2f

Solution:

7
3

log y =
log sin

mx

3
4

3
log x + 1 + mx +
4
7
5
ax log x 1
x+2
3
7

a f

b gj

7
5

3 x 1
7 x+2

a f a f
F
GG
Hb g

b gj

IJ
a f a fK
dy a x + 1f e sin aax f

dx
a x 1f a x + 2f
7
5

3 x +1 7 x + 2

mx

7
3

2 2

I
a f a fJJK

7
5

3 x 1 7 x + 2

Form 4: Miscellaneous problems.


Solved Examples

If log | xy | = x2 + y2, show that

e
e

log

bx

g=x

+ y
2

log y = log x + log sin y

1 dy 1
1
dy

= +
cos y
y dx x sin y
dx

1 dy cos y dy 1

=
y dx sin y dx x

FG 1 cos y IJ dy = 1
H y sin y K dx x
F sin y y cos y IJ dy = 1
G
H y sin y K dx x

j
j

dy
y sin y
y sin y
=
=
dx x sin y y cos y x sin y x sin y cos y

f a

y sin y
x sin y 1 x cos y

y
x 1 x cos y

Problems belonging to type (1)


(i): Problems based on irrational functions

log x + log y = x + y

y 2x 1
dy
=
.
dx x 1 2 y 2

Solution: log | xy | = x2 + y2
2

5
7

F 3
a
GG 4 ax + 1f + m +
sin aax f 1 a x
H

1.

y = x sin y = x sin y

dy
y
2. If y = x sin y, show that dx = x 1 x cos y .
Solution: y = x sin y

dy
3
a

=y
+m+

1
dx
4 x +1
sin ax 1 a 2 x 2

3
4

1 dy
3
1 a

=
+ m+

1
y dx 4 x + 1
sin ax 1 a 2 x 2

b g

FG 1 2 yIJ dy = 2 x 1
H y K dx
x
F 1 2 y I dy = 2 x 1
G
H y JK dx x
y e2 x 1j
dy

=
dx x 1 2 y
e
j

5
7

1
1 dy
dy
+
= 2x + 2 y
x
y dx
dx

Exercise 13.1.1
Find the differential coefficient of each of the
following.

Logarithmic Differentiation

FG 2 x 1IJ , x > 1
H 2 x + 1K 2
Answers
y a2 x + 1f
1.
2 x a x + 1f
3
2
2x O
L
3y M 2 x
2
5
5 P
+

2.
P
M
2 M x + 1 3x + 4 2 x 3 x 4 P
Q
N

2x 1
1
, | x| >
2x + 1
2

1. y =

4. y = x

2. y = x + 1 2 , x > 1
3. y =

x+1
, | x| 1
x 1

fa f ,x >2
e x + 1j a2 x + 3f
x x +1 x 2

4. y =

Answers
1.

e4 x

j
4.

3
y

2.
2 x +1

LM 1 + x 2 OP
MN x x + 1 3a x + 1f PQ
L
4 O
P
y M1 + log x +
MN
4 x 1 PQ

3. y
1

2y

2 y

(iii) Problems based on exponential composite


functions

3. 3 x 2 1

4.

Exercise 13.1.3

LM
MN

1
1
2x
2
y 1
+
+

3 x
x + 1 x 2 x2 + 1 2x + 3

OP
PQ

(ii) Problems based on product and quotient of


functions
Exercise 13.1.2
Find the differential coefficient of each of the
following.
1. y2 = x (x + 1) , x > 0

e x + 1j a3x + 4f , x > 2
a2 x 3f e x 4j
2

3
2. y =

1
2

x x +1

a x + 1f

2
3

Find the differential coefficient of each of the


following.
1. y = xsin x, x > 0
2. y = (sin x)tan x, sin x > 0
3. y = xlog x, x > 0
4. y = xx, x > 0
5. y = (ax)bx, x > 0
6. xy = e y x, x > 0
7. y = etan x
8. y = e

,x>0
x

9. y = e
10. y = elog sin x, sin x > 0
11. xy = ex y, x > 0
1

12. y = e

3. y =

559

,x>0

Answers
1. y

xx

,x>0

LM sin x + cos x log xOP


Nx
Q

560

How to Learn Calculus of One Variable

a f

7. y = sin x
Answers

2
2. y 1 + sec x log sin x

3.

2y log x
x

a f

a f

5. b ax
6.

bx

2 log x

x 1 + log x

9. e e
10. cos x

11.

y log x
x 1 + log x

12.

F e I F x I a1 log x f
GH JK GH JK

1
x

1
x

cot x

tan x

7. Find
8. e

tan x

1 + log ax , a > 0

a1 log xf

+ x sin x

FG IJ b1 + log xg + x FG 1 log x IJ
HK
H x K
tan x
F
I
G
2. x
H x + sec x log xJK + atan xf
cosec x b1 log tan x g
e1 + sec x log sin x j + a tan x f

3 . asin x f
bsec x + cos x log tan xg
4. 2 x b1 + log x g + b2 x g b1 + log 2 x g
F log x IJ + blog xg FG 1 + log log xIJ
G
5. 2 x
H x K
H log x
K
F sin x + cos x log xIJ + 3 cos x sin x
6. x
G
H x
K 2
x
+ sin x ,
7. asin x f a x cot x + log sin x f +
1. 1
x

2 log x
x
log x
4.
x

sin x

2x

log x

sin x

1 2 x
x

(iv) Problems based on the sum of two differentiable


functions

1 x

0 < x < 1.

(v) Miscellaneous problems.


Exercise 13.1.5

Exercise 13.1.4
x

Find the differential coefficient of each of the


following.
1.

F 1I
y=G J
H xK

3.
4.
5.
6.

tan x

1
x

+ x ,x>0
+ tan x

cot x

tan x

2x

log x

sin x

x y
dy
=
.
dx x log x

2. If xp yq = (x + y)p + q, x > 0, y > 0, show that

a f , 0 < x < 4

+ a tan x f
y = asin x f
,0< x <
2
y = x + a2 x f , x > 0
+ alog x f , x > 1
y= x

+ asin x f , 0 < x <


y= x
2

2. y = x

1. If x = e y , show that

dy
x
= , qx py .
dx
y

3. If ex + y = ex + ey, show that

sin x

3
2

4. If xy = ex y, show that

1 ey
dy
= eyx x
.
dx
e 1

log x
dy
=
dx
1 + log x

,x >0.

Logarithmic Differentiation

Problems belonging to type (2)


(i) Problems based on square root of functions
Exercise 13.2.1
Find the differential coefficient of each of the
following.
1. y =

a x + 1fa x + 2f

2. y =

x+a
xa

4. y =

a + x
3

2ax

6. y = 5 x

dy
of each the following.
dx
1. y = cos x cos 2x cos 3x cos 4x

1
2

x2 9

9. y =

10. y =

11. y =

sin x

12. y =

sin 3x

13. y =

tan 3x

a f FGH

4. Find
5. Find

2x + 3

2.

b x 3g

FG
H

x +1

IJ sin x
K

IJ
K

x+2

1
x2 +1
2

a f

1
x+2
2

12

32

LM
N
LM
yM
MM FG
NH

3. y 2 + log x +

4.

x+2
2

x +1

2
3
+
sin x 2 x log x

1
1 x

IJ sin
K

b g

2x + x 3

and simplify.

1+ x

Answers
1. cos x cos 2x cos 3x cos 4x (tan x + 2 tan 2x +
3 tan 3x + 4 tan 4x)

b x + 1gb x + 2g
F a I F x +aI
GH a x JK GH x a JK
F a x I
2a x
G
J
x a GH a + x JK
4

6. y = xx log x sin x
7. y = sin x sin 2x sin 3x sin 4x

Answers (with proper restrictions on x)

3.

4. y = (sin1 x) (cos x)x


5. y = sin x

FG x 1IJ
H
K
log e x x j

x+2

8. y =

3. y = tan x x e

x 7x + 4

2.

2. y = x 3

7. y =

1.

(ii) Problems based on the product of two or more


than two differentiable functions

Find

5. y = 5 x

6. Find
7. Find
8. Find
9. Find
10. Find
11. Find
12. Find
13. Find

Exercise 13.2.2

a x

3. y =

561

OP
Q

a f

2 x 3

OP
+ log cos x x tan x P
PP
x
Q

562

How to Learn Calculus of One Variable

LM
MN

5. y log sin x + x cot x +

x
+ cot x
1+ x 2

OP
PQ

LMFG1 + log x + cot x + 1 IJ x log x sin xOP


x log x K
Q
NH
7. sin x sin 2x sin 3x sin 4x (cot x + 2 cot 2x + 2 cot

3. y

6.

4. y

3x + 4 cot 4x)
(iii) Problems based on quotient of two or more than
two differentiable functions
Exercise 13.2.3

1. y =

+5

3x

3
2

5
2

10 x cot x 2 x 3
1
log e 10 4 x cosec 2 x 2 +
2 cot 2 x
sin 2 x
3x

(iv) Problems based on exponential composite


functions

3
2

Exercise 13.2.4

a x + 1f x 1
a x + 4f e
a2 x + 3f e x + 5j
y=
e3x + 1j
3x + 4 a2 3x f e x + 4j
y=
a2 3xf a4 x + 1f
a5 2 xf
y=
a5 + 3xf a5 4 x f
2

1
2

1
3

3.

1
2

4.

dy
of each of the following.
dx
1. y = (sin x)sin x
2. y = (log x)x
3. y = (tan1 x)sin x
Find

1
3

4. y = x
1
3

2
3

3
2

5.

5
2

a2 x 3x cos x + sin xf ax sin xf


x x

ax + 1f
a x + 4f

x1

2.

5. y = (log x)log x
6. xy = ysin x
7. y = xx
1

sin x
8. y = x
9. y = (1 + x)log x
10. y = (sin x)tan 3x

10 cot x x 3
6. y =
sin 2 x
Answers (with proper restrictions on x)
1.

2. y =

LM 5x 3 x + 6 OP
MM 2 e x 1j x + 4 PP
Q
N
2

OP
+ 1 PQ

12 x

OP
LM 3
3
2x
1
8

+
+

MM2 a3x + 4f 2 a2 3xf 3e x + 4j a2 3xf 3a4 x + 1f PP


Q
N

dy
of each of the following.
Find
dx

a5 2 xf LM 3 15 + 8 OP
5.
a5 + 3x f a5 4 xf N 5 2 x 2 a5 + 3xf 5 4 x Q
6.

2 x sin x

LM 2 +
MN 3a2 x + 3f x

11.
12.
13.
14.
15.

y= x

y = (tan x)sin x
y = xy
y = (cos x)log x
ysin x = xsin y

16. y = cot

1
x

17. ytan x = xtan y


18. (sec x)y = (tan y)x

Logarithmic Differentiation

19. y = (x log x)log log x


20. y = xtan x
y

a f LMN log1 x + log log x OPQ


L
etan xj MM tan sinx b1x+ xg + cos x log tan
N

3.

4. x

x 1 + 2 log x

OP
PQ

LM
MN

8. x

9. 1 + x

log x
1 x

log x

sin x
+
x

OP
PQ

LM log a1 + x f + log x OP
1 + xQ
N x

x 1 + log x

12. Find
2

y
13.
1

x
y log x

14.

15.

f
acos xf LMN log xcos x log x tan xOPQ
LM x log y cos x sin y OP y
N y log x cos y sin x Q x
log x

x 1+ x

1
2

j cot

OP
P
xP
Q

log log x

LM log b x log xg + b1+ log xg log log x


x log x
MN x log x

tan x O
LM
x PQ
N
F y I FG log y IJ FG 1 + x log x log y IJ
H x K H log x K H 1 x log y K

20. x

21.

tan x

sec x log x +

(v) Problems based on the sum of two or more than


two exponential composite (or, other) functions
Exercise 13.2.5

dy
of each of the following.
dx
y = x1 + x + log x
y = xcos x + sin log x
y = xx + etan x
y = log log x + 2sin x
y = xx + xsin x

Find

10. (sin x)tan 3x [3 sec2 3x log sin x + tan 3x cot x]


11. x

log tan y y tan x


log sec x x cosec y sec y

18.

log x

sin x

log cot x

1
tan y
x
17.
2
1
sec y log x tan x
y

19. x log x

7. xx (1 + log x)

a f 1x a1 + log log xf
y L y cos x log y O

x MN sin x y log x PQ

5. log x

6.

sin x

LM
j M
MN
1
x

sec x log y

2. log x

16.

21. y = x
Answers
1. (sin x)sin x [cos x (1 + log sin x)]

cot x

563

1.
2.
3.
4.
5.

x
6. y = x + x
7. y = xsin x + (sin x)x
8. y = xtan x + (sin x)cos x
9. y = xx + (log x)x
10. y = xsin x = 107

11.
12.
13.
14.

y = e x sin x

y = xsin x + 5x2
y = (sin x)cos x + (cos x)sin x
y = (log x)tan x + (tan x)log x

564

How to Learn Calculus of One Variable

15. y = xn log x + x (log x)n


16. y = x3 + (log x)x

b g LMN xtanlogxx + blog log xg sec xOPQ + btan xg

14. log x

Answers

1 + xO 1
LM
+
x PQ x
N
LMsin x log x + cos x OP + coslog x
x Q
x
N
a1 + log xf + e sec x

3. x
4.

n 1

cos x

tan x

5. x 1 + log x + x

sin x

x
6. x 1 + log x + x x

LM
N

7. x sin x cos x log x +

8. x

tan x

1. If sin y = x sin (a + y), show that


2

OP b g b
Q

sin x
x
+ sin x logsin x + x cot x
x

LMsec x log x + tan x OP + asin xf


x Q
N

10. x

11. e

sin x

cos x

gOPQ

tan x + x cot x sec2 x]


12. x

sin x

LM
N

cos x log x +

2. If y ey = x, show that

dy
y
=
.
dx x 1 + y

3. If y = x ey, show that

dy
y
=
.
dx x 1 y

4. If y =

dy sin a + y
=
dx
sin a

f a f FGH log log x + log1 x IJK


LM sin x + cos x log xOP
Nx
Q
esin x + 3x cos x j + (tan x) [log

x sin x

Exercise 13.2.6

(cos x cot x sin x log sin x)


x

a1 xfa1 + xf , show that

5. If y =

1 x
, show that
1+ x

e1 x j dydx + y = 0
2

OP
Q

sin x
+ 10x
x

6. If y = x x , show that

dy
vanishes when x = e.
dx

e1 x j dydx + xy = 0

F
cos x I
13. asin x f G sin x log sin x +
J + acos xf
sin x K
H
F cos x log cos x sin x I
GH
J
cos x K
cos x

(vi) Miscellaneous problems

LM sin x + cos x log xOP


N x
Q
b1 log x g

9. x 1 + log x + log x

log x

n 1

1
sin x
+ log 2 cos x 2
x log x
x

LMsec x log x cosec x + log tan x OP


x
N
Q
15. x
an log x + 1f + alog x f
alog x + n f
L 1 + log alog xfOP
16. 3 x + alog x f M
N log x
Q

1. x 1+ x log x +

2. x

tan x

sin x

dy
7. If x = cos (xy), show that
=
dx

for 0 < xy < .

FG
H

1+ y 1 x
x 1 x

IJ
K;

565

Logarithmic Differentiation

8. If y = xexy, show that

9. If ex = xy, show that

a
a

f
f

a f

y 1 + xy
dy
=
.
dx
x 1 xy

0 = c1 2c2 + 3c3 ... + cn n 1

a f

Problems on Binomial Coefficients


One can find many equalities with the help of a given
binomial expansion in terms of binomial coefficients.
Working rule: It consists of following steps:
Step 1: Differentiate both sides of the given condition
(1 + x)n = c0 + c1 x + c2 x2 + + cn xn w.r.t. x
Step 2: Either put x = 1, 2, 3, etc if each term is
positive in the required result or put x = 1, 2, 3,
etc. if term are for each term being alternatively positive
and negative in the required result.
Solved Examples
1. (1 + x)n = c0 + c1 x + c2 x2 + + cn xn, show that c1
+ 2c2 + 3c3 + + n cn = n 2n 1
Solution: Given condition is (1 + x)n = c0 + c1 x + c2 x2
+ + cn xn differentiating both sides of the given
condition w.r.t. x, we have n (1 + x)n 1 (1 + 0)=0 + c1

1 + c2 2x + + n xn1 cn 3 c0 = 1
Now, putting x = 1 in (i), we have

a f

n 1+1

n 1

n2

n 1

(i)

= c1 + 2c2 + 3c3 + ... + n cn

= c1 + 2c2 + 3c3 + ... + ncn which is


the required result.

2. If (1 + x)n = c0 + c1 x + c2 x2 + + cn xn, show that

a f

c1 2c2 + 3c3 + ... + 1 1 n cn = 0


Solution: Given condition is (1 + x)n = c0 + c1 x + c2 x2
+ + cn xn differentiating both sides of the given
condition w.r.t. x, we have

n 1+ x

n 1

= 0 + c1 1 + c2 2 x + ... + nx

n 1

cn (i)

Now putting x = 1 in (i) ( 3 terms in the required


result are alternatively +ve and ve), we get

a f

n 11

n 1

which

is the required result. (Note: Positive and negative


are shortly written +ve and ve respectively.

dy log x 1
=
2 .
dx
log x

n 1

a f

= c1 2c2 + 3c3 + ... + cn n 1

n1

Notes: (A): When the last term of the given equality


to be proved contains kn + r, where k, r and n are
positive integers, then one should note that (1) x is to
replaced by xk on both sides of the given condition, k
being the coefficient of n. (2) the expression obtained
after x being replaced by xk should be multiplied by
xr, where r is the addend in kn + r (3) the expression
obtained in step (2) should be differentiated w.r.t.
x and lastly either the substitution x = 1, 2, 3, etc or
x = 1, 2, 3, etc is made accordingly as whether
terms in the required equality to be proved are positive
or alternatively positive and negative.
(B): When the last term in the equality to be proved
has n2, one should multiply both sides of the equality
(obtained after differentiating the given condition)
by x.
3. If (1 + x)n = c0 + c1 x + c2 x2 + + cn xn, show that
c1 + 22 c2 + 32 c3 + + n2 cn = n (n + 1) 2n 2
Solution: Given condition is (1 + x)n = c0 + c1 x +
c2 x2 + + cn xn whose last term is n2 cn.
Now, differentiating both sides of the given
condition w.r.t. x, we get
n (1 + x)n1 = c1 x + 2x c2 + 3 x2c3 + + nxn1 cn (i)
Multiplying both sides of (i) by x, we get
n (1 + x)n1 x = c1 x + 2x2 c2 + 3 x3c3 + + nxn cn (ii)
Again differentiating both sides of (ii), w.r.t. x, we
get
n [1 (1 + x)n1 + n (n 1) (1 + x)n 2 1] = c1 +
22x c2 + 32 x2c3 + + n2 xn1 cn
(iii)
Lastly, putting x = 1 (iii), we get
2

c1 + 2 c2 + 3 c3 + ... + n cn
=n 2

n 1

+ n12

n2

=n2

n2

a2 + n 1f

n2

=n n+1 2
which is the require result.
Remark: On putting x = 1 in (iii), we get
2

a f

c1 2 c2 + 3 c3 4 c4 + ... + 1

n 1 2

a f

n cn = n 0 + 0 = 0

4. If (1 + x)n = c0 + c1 x + c2 x2 + c3 x3 + + cn xn,
show that c0 + 2c1 + 3c2 + + (n + 1) cn = 2n 1 (2 + n).

566

How to Learn Calculus of One Variable

Solution: Given condition is (1 + x)n = c0 + c1 x +


c2 x2 + + cn xn whose last term has (nk + r) = n + 1
for k and r = 1 multiplying both sides of the given
condition by x, we get
x (1 + x)n = c0 x + c1 x2 + c2 x3 + + cn xn + 1 (i)
Now, differentiating both sides of (i) w.r.t. x, we get
1 (1 + x)n x + n (1 + x)n 1 = c0 + 2x c1 + 3x2 c2 +
+ cn (n + 1) xn
(ii)
Putting x = 1 in (ii), we get the required result
2n + 1 n + 2n 1 = c0 + 2c1 + 3c2 + + (n + 1) cn =
n

2 1 (2 + n)

Now, differentiating both sides of (ii) w.r.t. x, we


get
1 (1 + x2)n + x n (1 + x2)n 1 2x
= c0 + c1 3 x2 + c2 5 x4 + ... + cn (2n + 1) x2n
(iii)
Lastly, on putting x = 1 in (iii), we get the required
result c0 + 3c1 + 5 c2 + + (2n +1) cn
= 2n + n 2n 1 2 = 2n (1 + n)

5. If (1 + x)n = c0 + c1 x + c2 x2 + + cn xn, show that


c0 + 3c1 + 5c2 + + (2n + 1) cn = 2n (n + 1).
Solution: Given condition is (1 + x)n = c0 + c1 x +
c2 x2 + + cn xn whose last term has (nk + r) = 2n + 1
for k = 2 and r = 1 on replacing x in each term of the
given condition by x2, we get
(1 + x2)n = c0 + c1 x2 + c2 x4 + + cn x2n
(i)
on multiplying both sides of (i) by x (since additive
constant in 2n + 1 is 1), we get
x (1 + x2)n = c0 x + c1 x3 + c2 x5 + + cn x2n + 1 ...(ii)

1. If (1 + x)n = 1 + c1 x + c2 x2 + c3 x3 + + cn xn, show


that
(i) c0 + 2c1 + 2c2 + 4c3 + + (n + 1) cn = (n + 2) 2n 1
(ii) c0 2c1 + 3c2 4c3 + + (1)n (n + 1) cn = 0
(iii) c1 2c2 + 3c3 + (1)n 1 n cn = 0
(iv) c1 22c2 + 32c2 42c4 + + (1)n 1 n2 cn = 0

Exercise 13.3

2. If (1 + x)n = 1 + c1 x + c2 x2 + c3 x3 + + cn xn, find


the values of
(i) c0 + 2c1 x + 3c2 x2 + + (n + 1) cn xn
(ii) c1 + 22c2 + 32c3 + 42c4 + n2 cn

Successive Differentiation

567

14
Successive Differentiation

Question: What do you mean by successive


differentiation?
Answer: The process of finding derivatives of the
derivatives in succession is called successive
differentiation.
Explanation: A function may be differentiated more
than once in the following way.
We know that differential coefficient of a function
f (x) in general is itself a function of x known as derived
function of x or first derivative of the function f (x)
symbolised as f x indicating f (x) has been
differentiated the first time, the first derivative f x
can be differentiated to obtain the second derivative
of the function f (x) symbolised as f x indicating f
(x) has been differentiated two times. After the derived
function f x having been differentiated second
can be
time, the second derivative f x
differentiated third time providing us again a function
of x knwon as third derivative symbolised as f x
indicating f (x) has been differentiated three times.
This process of getting a derived function may go on

af

af

af

af

af

af

bg

indefinitely and after the derived function f b n 2 g x

n 1
f b gx

is differentiated n 1 th derivative
is
obtained which again can be differentiated providing
us again a function of x known as nth derivative symbolised as f n x indicating f (x) has been differentiated
n-times. This process of differentiation of a function
f (x) repeated more han one successively or the process
of finding derivatives one after the other from a given
function f (x) is known as successive differentiation.

af

Remember:
1. If y be a function of x, the derived function f x
will be in general itself a differentiable function of x
except perhaps at those points at which derived
function f x becomes undefined.
2. Points at which f (x) or f x are undefined are
knwon as points of discontinuities of f (x) or f x
respectively.
3. Points of discontinuities of the function f (x) are
also the points of discontinuities of the given
function f x .

af

af

af

af

af

Question: How would you differentiated


successively y = x 6 ?
6
Answer: Q y = x

First derivative = y1 =

dy
= 6 x b 6 1g = 6 x 5
dx

Second derivative

F I
H K

d dy
d2y
51
=
= 6 5 x b g = 30 x 4
dx dx
dx 2
Third derivative
= y2 =

F d yI = d y
GH dx JK dx
= 30 4 a x f a f = 120 x
d
= y3 =
dx

41

Fourth derivative
= y4 =

d
dx

F d yI = d y
GH dx JK dx
3

568

How to Learn Calculus of One Variable

a fa

= 120 3 x
Fifth derivative

f = 360 x 2

F d yI = d y
GH dx JK dx
= 360 2 a x fa f = 720 x
= y5 =

d
dx

fourth, fifth, and higher orders can be denoted by


any one of the following notations.

31

2 1

= 720 x

F d yI = d y
GH dx JK dx
= 720 1 a x f a f = 720 x
5

d
dx

11

= 720 1 = 720

Seventh derivative
= y7 =

d
dx

F d yI = d y = 0
GH dx JK dx
6

Eighth, 9th , . . . . . . . nth derivative = 0


Question: How would you differentiate successively
y = sin x ?
Answer: Q y = sin x

F + xI
H2 K
F I F I
y = cos G + xJ = sin G 2 + x J
H2 K H 2 K
F I F I
y = cos 2 + x = sin 3 + x
H 2 K H 2 K

y1 = cos x = sin
y2 = sin x

y3 = cos x
y4
y5
y6
y7

= sin x
= cos x
= sin x
= cos x

:
:
:
:

F
H

I
K

+x
2
N.B.: We inspect (1) y1 , y3 ....., y2n + 1 = cos x ,
alternatively +ve and ve (2) y2 , y4 ,.....,
y2 n = sin x , alternatively + ve and ve provided
y = sin x .
Notation: If y = f x = a function of independent
variable x, then the differential coefficient (or,
derivatives or derived function) of first, second, third,
y8 = sin x

yn = sin n

af

af

af

Sixth derivative
= y6 =

dn y
dy d 2 y d 3 y d 4 y
,
,
,
,
denoting that a
2
3
4 ...,
dx dx
dx
dx
dx n
function y = f x has been differentiated one time,
two times, three times, four times, ..., n times as well as
first, 2nd, 3rd, 4th derivative, ..., n th derivative of
some function y = f x . Hence to differentiate a
function y = f x n-times means to find n th
derivative of the function y = f x
1.

af

af

2. Dy, D y , D y , D y , ..., D y , (n being any + ve


integer) which is known as capital D - notion..

af

af

af

af

3. f x , f x , f x ...... , f n x (n being any +


ve integer)
4. y , y , y , ...... , y n (n being + ve integer)
5. y1 , y2 , y3 , ....... , y n (n being + ve integer)

af

af

af

af

6. f a1f x , f a 2 f x , f a3f x , ...... f a n f x , (n being +


ve integer)
7. Dx y , Dx2 y , Dx3 y , ...... Dxn y (n being any + ve
integer)
Nomenclature: Read as (Nomenclature)
Notation
dy
dee wy over dee eks
dx
or, dee wy by dee eks
d2 y
dee two wy over dee eks two
dx 2
or, dee two wy by dee eks squared
3
d y
,
dee three wy over dee eks three
dx 3
or, dee three wy by dee eks cubed
M

dny
dx n

dee en wy over dee eks en


or, dee en wy by dee eks en

Note: 1. The first notation


is in common use.

dn y
dy d 2 y d 3 y
,
,
,
2
3 ...,
dx dx
dx
dx n

Successive Differentiation

2. The capital D-notation or dash notation

ei.e; dy, d

y , d y ,...,d y or y , y , y ,..., y

is used when the independent variable w.r.t which we


differentiate is understood. The capital D or dash
notation has the disadvantage of not indicating the
variable with respect to which the differentiation is
carried out. This is why it is convenient to use the
dy
to mean the operation of finding the
symbol
dx
derivative with respect to x.
3.

d2 y
d
=
2
dx
dx
...
...

F dI
H dx K

F dy I = F d I a yf = d y
H dx K H dx K
dx
2

F I
H K
af

dn
d d
= n
(... upto n times)
dx dx
dx
which requires the operand y = a function of an
independent variable x = f x put in the last of the
operator.
4. The derivative of a function of an independent
variable = First derivative of the given function of an
independent variable or simply first derivative.
Derivative of first derivative = second derivative
of the original function or simply second derivative.
Derivative of second derivative = third derivative
of the original (or, given) function or simply third
derivative.
Derivative of third derivative = Fourthond
derivative of the original function or simply fourth
derivative.
...
...

Derivative of n 1 derivative = nth derivative


of the original function or simply nth derivative.
5. Second and higher order derivatives of a function
are called higher derivatives and the process of finding
them is called successive differentiation.

d2y
6. Care must be taken to distinguish between
dx n

F dy I
and
H dx K

dny
n means the nth differential coefficient
dx

of function of x whereas

F dy I
H dx K

569

means the nth power

of the first differential coefficient of function of x.


Problems based on finding higher derivatives
Examples worked out:
1. Differentiate y = x 3 + 5 x 2 7 x + 2 four times.
Solution: Q y = x 3 + 5 x 2 7 x + 2

y1 = 3x 2 + 10 x 7
y2 = 6 x + 10
y3 = 6
y4 = 0
x

2. Differentiate y = e + log x three times.


x

Solution: Q y = e + log x , x > 0

y1 = e x +
y2 = e x

1
x2

y3 = e x +

1
x

x3

,x>0

3. If y = log x find y 4 .
Solution: Q y = log x , x > 0

1
x
1
y2 = 2
x

y1 =

y3 =

2
x3

y4 =

2.3
6
= 4 ,x>0
4
x
x

4. If y = e mx , find y 2 .
mx
Solution: Q y = e

y1 = m e mx
y2 = m m e mx = m 2 e mx

570

How to Learn Calculus of One Variable

Working rule to find second derivative from the


equations of a curve given in parametric form:
x= x t ,y=y t

af

af

dy
dy
= dt
1. Find
dx
dx
dt
2.

d2y
dx 2

d y
provided
dx 2

aa cos f =
d

LM a
N

aa sin f =

d
d
a cos
d d

LM a
N

d
d
a sin
d d

d
a cos = a sin
d

d a cos
dx
=
= a sin
d
d

Now, using the formula,

d2y
=
dx 2

d 2 y dx
d 2 x dy

2
d d 2 d
d

LM dx OP
Nd Q

a sin + a cos
3

a sin

1
dx
d

1
a sin

= cosec

= cosec

fOPQ

d
a sin = a cos
d

d y

a sin

1
3

a sin

=
...(1)

fOPQ

...(1)

LM OP
N Q

y = a sin
Solution:
d x

g , n

d
d dy
d
=
cot
dx dx
d
dx

x = a cos

gb

dy
d
a cos
dy
=
=
= cot
dx
a sin
dx
d

Examples worked out on finding second derivative


of parametric equations of a curve
Question: 1. Find

g b

a sin

Or, alternatively,

should be used

dt
1
=
3. Write
and simplify.
dx
dx
dt

a sin a sin a cos a cos

LM dy OP dt
N dx Q dx

d
dt

1
a sin 3

, n

2. If x = 2 cos cos 2 , y = 2 sin sin 2 ,


find

d2y
.
dx 2

Solution: x = 2 cos cos 2


...(2)
...(3)

y = 2 sin sin 2
dx
= 2 sin sin 2 2 = 2 sin 2 sin ...(1)
d
dy
= 2 cos cos 2 ( 2 ) = 2 cos cos 2
d
...(2)

dy
2 cos cos 2
d
dy
=
=
[ using (1) and (2)]
dx
2 sin 2 sin
dx
d

a
a

f
f

571

Successive Differentiation

cos cos 2
sin 2 sin

dy
tan
3 a tan 2 sec 2
dy d
=
=
=
= sin
dx dx 3 a sec 2 sec tan sec
d

sin
2
2 = tan 3
=
3

2
sin
2 cos
2
2

FG IJ
H K

2 sin

d y
2

d
dx

4 2 cos 3 sin
2
2

...(3)

...(3)

F dy I = d F dy I d
H dx K d H dx K dx
dx
3 I d
d F
=
tan
G
J
dH
2 K dx
3
F 3 I 1
= sec G J
H 2 K F dx I
2
GH d JK
3
1
F 3 I
= sec G J
H
K
2
2
2 asin 2 sin f [ using (1)]
F 3 I
sec
H 2K
3
=
2 2 asin 2 sin f
F 3 I
sec
H 2K = 3
3
1
,
=
2

8 sin cos2 3
2
2

g 3

d y
dx

1
d sin d

= cos
dx
d
dx
d
cos

a 3 sec 2 sec tan

y = a 1 cos

dy
2
2
= a 3 tan sec
d

...(1)

...(1)
...(2)

a sin
sin
dy
dy d
=

=
=
dx d dx
a 1 cos
1 cos

d y
2

LM
N

f a

OP
Q

sin
d
d

1
cos

d
dx

a1 cos f cos sin sin 1


a a1 cos f
a1 cos f
2

cos 1

a 1 cos

a 1 cos

, 2n

5. If x = a cos + sin y = a sin cos ,

where 0 < <


...(2)

d2y
dx 2

(1) and (2)

Solution: x = a sec

dy
= a sin
d

dx
= a 3 sec 2 sec tan
d

2n

cos 4 cot
n
,
3a
n

d
dx
1
= a 1 cos
=
d
dx
a 1 cos ,

dx

y = a tan

Solution: x = a sin

d y
.
dx 2

4. If x = a sin y = a 1 cos , Find

for 2n , 2n + 1

3. If x = a sec 3 y = a tan find

d y sec

prove that 2 =
.
a
dx
2
2

572

How to Learn Calculus of One Variable

Solution: If x = a cos + sin

y = a sin cos

Now,

dx
= a sin + cos + sin = a cos
d
...(1)

dy
= a cos sin + cos
d

= a sin
...(2)

dy
d
a sin
dy
=
=
= tan
(1) and (2)
dx
a cos
dx
d

FG dy IJ d = d b tan g d
H dx K dx d
dx

d2y
d
=
2
d
dx
2

= sec

= sec

1
dx
d

d2y
d
=
2
dt
dx

LM dy OP dt = d LM1 OP dt
N dx Q dx dt N t Q dx

1
1

2
at
2
t
1

2at3

,t 0

Problems based on showing that a given function


y = f x satisfies a differential equation

af

Question: What is a differential equation?


Answer: An equation containing one (or, more
derived functions) is called a differential equation. or,
in more explicit form, an equation containing the
independent variable x, the function y and its
derivatives or differentials is called a differential
equation.
Notation: A differential equation is symbolised as
follows.

F dy I = 0
H dx K
F d yI = 0
F b x , y , y g = 0 or F G x , y ,
H dx JK
F dy , d y I = 0
F b x , y , y , y g = 0 or F G x , y ,
H dx dx JK
F d x , y , y , y , y , ...... y i = 0 . . . . . . etc
b

1. F x , y , y = 0 or F x , y ,

1
a cos

2.

sec
=
a

3.
4.

d2y
6. If x = at , y = 2 a t find 2 .
dx
2

Solution: x = at
y = 2a t

dx
= 2 at
dt
dy
= 2a
dt

dy
=5
dx
d2y
dy
5=0
2. 4 2 + 7
dx
dx
dy
dy
+ 2x
y=0
3. y
dx
dx
Note: 1. The general type of differential equation of
Examples: 1.

...(1)
...(2)

dy
2a
dy
1
(1) and (2)
= dt =
= , t 0 ...(3)
dx
dx
2a t t
dt

F I
H K

dy
+ p y = Q where P and Q are
dx
given functions of x.
2. The general type of differential equation of the
the first order is

second order is

af

d2 y
dy
+a
+ by = f x
2
dx
dx

Successive Differentiation

Now we come to our main problem.


If y = f x , then we are required to show that
1.
2.
3.
4.
5.

af
F b x , y , y g = 0
F b x , y , y g = 0
F b x , y , y g = 0
F b x , y , y , y g = 0
F b x , y , y , y , y g = 0

af

b
g
2. F b x , y , y g = 0
3. F b x , y , y g = 0
4. F b x , y , y , y g = 0
5. F b x , y , y , y , y g = 0
1. F x , y , y = 0

b
g
(ii) F b x , y , y g = 0
(iii) F b x , y , y , y g = 0
(iv) F b x , y , y , y , y g = 0
(i) F x , y , y = 0

Then firstly we find first derivative and then in


most cases using the rule of cross multiplication, we
change the quotient into product form so that the
rule of d.c. of product of two functions

F i.e; d l f a xf f a xfq = f a xf f a xf + f a xf f a xfI


H dx
K
1

or we show L.H.S = R..H.S

About Mathematical Manipulation


1. If the differential equations are

(i) F x , y , y = 0

b
g
(iii) F b x , y , y g = 0
(iv) F b x , y , y g = 0 i.e; only one derivative, then
(ii) F x , y , y = 0

we find that derivative only by successive


differentiation and putting its value in the given
differential equation, we show that

2. If the differentail equation is F x , y , y , y = 0 ,


then we find y and y by successive differentiation

Remember: 1.

(ii) F x , y , y = 0

should be applied. But this method is not always


fruitful.
5. If the given differential equation
F x , y , y , y = 0 or, F x , y , y , y = f x
does not contain radical sign and the first derivative
of the given function contains the radical sign, then
both sides of the equation containing the first
derivative may be raised to the same*.
*Power in any stage to remove the radical symbol
or sometimes rationalization is also fruitful device
provided the given function is irrational or the first
derivative of the given function is irrational.

(i) F x , y , y = 0

b
g
(iii) F b x , y , y g = 0
(iv) F b x , y , y g = 0

af

af

Working rule: We proceed from the given function


y = f x in general finding those all derivatives
appearing in the given differential equation by using
successive differentiation and using various
mathematical manipulations, we show that

and then putting the expressions obtained for y and


y in the left hand side of differential equation, we
show that F x , y , y , y = 0
3. If
the
differential
equation
is
F x , y , y , y = f x , then we find y and y by
successive differentiation and then using various
techniques, we show that L.H.S. = R.H.S.
4. If y = f x = a rational function, inverse circular
function, a function containing inverse circular
function or logarithm of a function (i.e; a function
whose first derivative is a fractional expression in x
rational or irrational) and we are required to show

We adopt the following working rule:

573

d
dx

|RSF dy I
|TH dx K

af

|UV = 2 F dy I d F dy I
|W H dx K dx H dx K

dy d y
dy
=2
2 in which
is regarded as a symbol
dx dx
dx
z which is differentiated w.r.t. x which

b g

d
y1
dx

= y1 y2 which is generally used when

574

How to Learn Calculus of One Variable

af

f x = a rational function of x, inverse circular


function of x, a function involving inverse circular
function of x, logarithm of a function of x.
2. Whenever we have y f x or y f x
we are required to square both sides provided given
differential equation to be proved does not contain
the square root symbol.
3. Sometimes a modification in the form of a given
function is also fruitful device before finding the first
derivative to get the required differential equation.

af

af

y2 = y =

Ld y O
5. M dx P
N Q
F d yI
GH dx JK

d2y
dx 2

x = a,

x = a,

= sin x

dx

= cos x

d y
x = a,

by g

n x=a

yn

a,

af

b g

or yn

dx

fn a ,

etc. denotes the

value of the nth derivative ( or, nth differential


coefficient) of the given function y = f x at (or,
for) x = a .

af

F
H

d2y
dy
6. In general 2 = f x , y ,
dx
dx
d3y
= f
dx 3
...
...

dy
= cos x
dx

d y

dny
dx n

yn

d4y
= y
dx 4

Proof: y = sin x

dx

1. If y = sin x , show that

d y

...
yn = y n =

Examples worked out:

dy
dx

4. y1 = y =

Type 1: To show that a given explicit function satisfies


a given differential equation.

F x , y , dy , d y I
GH dx dx JK

I
K

= sin x

d4y
= y
dx 4
y = a cos n x + b sin n x , show that

2. If

d2 y
+ n2 y = 0
dx 2
Proof: y = a cos n x + b sin n x

dy
= n a sin n x + nb cos n x
dx

d y

dx

F dy , d y , ... d y I
d y
= f Gx, y,
H dx dx dx JK
dx
n

n1

n 1

are second, third, fourth . . . and nth differential


equation.
7. A given function y = f x is said to satisfy a
differential equation if we can find (or, derive) that
differential equation by differentiating the given
function y = f x and using various mathematical
manipulations.

af

af

= n a cos n x n b sin n x
2

= n a cos n x + b sin n x

= n2y

d2y
+ n2 y = n2 y + n2 y = 0
dx 2

d2y
+ n2y = 0
dx 2

3. If y = a e mx + b e mx , show that

d2y
m2 y = 0
dx 2

Successive Differentiation

Proof: y = a e mx + b e mx

a f

mx
mx
mx
mx
dy
= a e m + b e
m = m a e b e
dx

d y
2

dx

=m ae
2

=m ae

mx

mx

m b e
+be

mx

mx

a f
2

d2y
m 2 y = m2 y m 2 y = 0
dx 2

d2y
m2 y = 0
dx 2

FG
H

1 + x2

Proof:

IJ ,
K

show

x + 1 + x2

1
x + 1 + x2

d y

x
1+ x

FH

x
2

x +1

1+ x

=0

1 + x2

f
d y
dy
2a
+ da + b i y = 0
dx
dx
Proof: y = e sin ab x + cf
dy

= a e sin bb x + cg + e b cos bb x + cg
dx
= a y + b e cos ab x + cf
5. If y = e

ax

sin b x + c show that


2

IJ
K

ax

LM
N

1+

1
2 1 + x2

1 + x2 + x
1 + x2

OP
Q
O
2 xP
PQ

OP
Q

ax

ax

ax

d y
dx

=a

ax
2 ax
dy
+ b e cos bx + c b e sin bx + c
dx

=a
Q y=e

F
H

I
K

dy
dy
+a
a y b2 y
dx
dx
ax

sin bx + c

= given function

dy
a2 y b2 y
dx
dy
= 2a
a2 + b 2 y
dx

1 + x2
1

j
j

d y
=
dx 2

LM
MN
LM
N

that

e
e

dy x + 1
+
=

x +1
x
2
dx x 2 + 1
dx
2

1+ x

dy
1
d

x + 1 + x2
2
dx
dx
x + 1+ x
=

d x + 1i

d2y
dy
+ x
= 0.
dx
dx 2

F
y = log G x +
H

1 + x2

Putting the values of

=m y

y = log x + 1 + x

d2y
dy
and 2 in the L.H.S of
dx
dx
the differential equation,

4. If

575

= 2a

2x
1 + x2

1 + x2

IK

LM d F 1 I
f axf
=
Q
G
J
dx
f
x
a
f
MN H K l f a xfq

Putting the value of

OP
PQ

equation,

d2y
in L.H.S of the differential
dx 2

576

How to Learn Calculus of One Variable

d y
dx

dy
dy
2
2
2
2
= 2a
a + b y + a + b y 2a
dx
dx
=0

6. If a + bx e x = x , show that x

d y
dx

F
H

= x

dy
y
dx

I
K

Proof: We have a + bx e x = x

...(1)

F ax IJ
= x
=
=G
x b a + bx g
ba + bx g H a + bx K
L dy yOP
R.H.S. = M x
N dx Q

log a + bx

log a + bx

x log

a + bx

ba + bx g

L R| F x I a U|
F x I OP
= M x Slog G
+
x log G
V
J
MN |T H a + bx K a + bx |W
H a + bx JK PQ

FG ax IJ
H a + bx K

= x log x log

+ y 1

= log x

...(2)

Again differentiating both sides of (3) w.r.t. x,

x a + bx

gd

nx

= n y + be

nx

...(1)

Q y = a + bx e nx

Now, differentiating (1) again w.r.t. x,


2

d y
dx

+ 2n

dy
dy
n x
dy
2
2
+n y=n
bn e
+ 2n + n y
dx
dx
dx

dy
bn e nx + n 2 y
dx
= n n y + b e nx bn e nx + n 2 y
=n

=0

LM
N

2
8. If y = x + 1 + x
2

xb a + bx x ab
2

= n a + bx e nx + b e nx

b g + x LMN 1x a +bbx OPQ


L a + bx bx OP
dy

= log x log b a + bx g + x M
dx
MN x ba + bx g PQ
dy
a

= log x log ba + bx g +
...(3)
dx
a + bx
g

g
i + be

dy
= a + bx n e nx
dx

ba + bxg
ba + bxg

Proof: we are given y = a + bx e nx

+ y = x log x

d2y 1
b
ab
=

2
+
x
a
bx
dx
a + bx

d2y
dy
+ 2n
+ n2 y = 0
dx
dx 2

dy
= 1 log x log a + bx
dx

ba + bxg

Now, differentiating both sides of (2) w.r.t. x,

7. If y = a + bx e nx , show that

y = x log x x log

Hence, L.H.S. = R.H.S

y
+ log e = log x
x

x log

= log x

x 2 a2

a2

Now taking log of both sides of (1),


y
ex

d2 y
dx 2

3
Now L.H.S. = x

2
2
dy
2a
+ a +b y
dx

e1 + x j FGH dydx IJK


Proof: y = LM x +
N
2

x a + bx

OP , show that
Q
n

= n2 y2 .
1 + x2

OP
Q

577

Successive Differentiation

LM
N

LM
N

= n Lx +
MN

OPb
Q

LM
N

2
d
x + 1+ x
dx

n1

1+

1+ x

i ddx y + x dxdy m y = 0
10. If y = A LM x + x 1 OP + B LM x x 1 OP
N
Q N
Q
show that d x 1i y + xy n y = 0 .
Proof: y = A Lx + x 1 O + B L x x 1O ...(1)
NM
QP NM
QP
LM
N

dy
=n A x+
dx

LM
N

...(1)

1+ x

LM
MN

. 1+

x
1 + x2

OP
PQ

LM
N

nA x +

LM
N

x 1

x2 1

x2 1

nA x +

1 + x2

d1 + x i FH dxdy IK
2

1 + x2

x 1

LM
N

...(2)

x2 1

dy
dx
2

= n A x + x 1

= m2 y2

x 1

OP
Q

...(3)

Now, differentiating both sides of (3) again w.r.t x

OP
Q

OPa
Q

n 1

OP LM1
Q MN

OP
Q

my

dy
= my
dx
Now, squaring both sides of (2), we get

+ nB x

Now, differentiating the given function (1) w.r.t x

Now, differentiating both sides of (1), we get

m 1

OPb
Q
1+ x O
PQ

LM
N
m Lx +
MN
=

dy
= m x + 1 + x2
dx

1 + x2

2 2

d1 + x i ddx y + x dydx = m y
2

dy
we get
dx

Now, dividing the equation (4) by 2

dy
1 + x2
= ny
dx
Now, squaring both sides,
2

dy
dx
...(4)

2 x = 2 m2 y

d1 + x i FH dxdy IK = n y
9. If y = LM x + 1 + x OP , show that
N
Q
d1 + x i ddx y + x dxdy m y = 0 .
Proof: Q y = LM x + 1 + x OP
N
Q

1+ x 2

d1 + x i 2 dydx ddx y + FH dxdy IK

OP
Q

n 1

ny

n 1

LM
OP
2x
MN 2 1 + x PQ
b g L
1+ x O
PQ MNx + 1 + x OPQ
LMQ y = F x + 1 + x I OP
KQ
N H

= n x + 1 + x2

OPa
Q

2
dy
= n x + 1+ x
dx

LM
MN

x
x

LM
N

OP
P
1Q

nB x

LM
N

LM
N

d
dx

F dy I +
H dx K

x
2

x 1

OP
Q

x2 1

x2 1

n B x x 1

Again differentiating (2) w.r.t x to get

x 1

OP
P
1Q

x2 1

nB x

x2 1

1+

dy
dx

OP
Q

OP
Q

...(2)

578

How to Learn Calculus of One Variable

LM
N

n A x+
=

x 1
2

OP
Q

LM
N

x 1
2

x 1

dx

LM
N

x 1

OP
Q

n A x+

x 1
2

OP
Q

x 1

dy
dx
2

+n B x
2

x 1

OP
Q

x 1

x 1

d y
dx

L F
= n MA Gx +
MN H
d x 1i y
d x 1i y
2

x 1

+ x y1 n 2 y = 0

1
m

= 2x

Proof: Let us suppose that

IO
1J P = n y
K PQ
n

1
=
z
Now, adding (1) and (2)

Z=x

show

that

FH

y= x

=Z

...(1)

4x 2 4
2
...(3)

IK

x2 1

...(5)

i FH dxdy IK

= m2 y2

x2 1

...(6)

=2m y

F I
H K

dy d y
dy
2 +
dx dx
dx

d
dx

2x

...(7)

dy
,
dx

i ddx y + x dydx m y = 0
1i y + x y m y = 0
2

Note: In some cases the form of the function suggests


simplification. For these functions, we modify the form
of the function using mathematical manipulation
before differentiation to save labour and time.
Now, we shall do harder and tricky problems on
the explicit functions having rational, trigonometric,
inverse trigonometric and logarithmic functions which
satisfy the given differential equation.
Examples worked out:

...(4)

dy
dx

Now, Dividing (6) by 2

...(2)

x2 1

Now, squaring both sides of (5)

1
= 2x
Z
2x

m1

x2 1

x 1 2

Z 2 + 1 2x Z = 0

Z=

x2 1

x2 1

1
ym

Z+

x2

m 1

my

x 2 1 y2 + xy1 m 2 y = 0
1
ym

LM
N

OPb
Q

on differentiating (6) w.r.t. x

ym + y

FG
H

+B x+ x

+ x y1 = n 2 y

IJ
K

x2

LM
MN

OP
x 1 PQ
b g Lx x 1O
1O
QP MMN x 1 PPQ
1O
PQ
x2 1

LM
N

m x

dy
dx

11. If

+ x

LM
N

dy
=m x
dx

= m x

LM
N

Now, differentiating both sides of (4) w.r.t x

x 1

d y

n B x

1. If y =

ax + b
, show that 2 y1 y3 = 3 y2 2
cx + d

Successive Differentiation

Solution: y =
y1 =

ax + b
d
,x
cx + d
c

g b

bcx + d g
2 c b ad bc g
=
bcx + d g
6 c b ad bcg
=
bcx + d g

y2

cx + d y3 + y 2 c + 2 y 2 c = 0

a cx + d ax + b c
2

a f
af
b cx + d g y + 3 y c = 0
Now, (1) b cx + d g y = 2 y c
(2) b cx + d g y = 3 y c
3

ad bc

bcx + dg

bcx + d g y
bcx + d g y

Now, L.H.S = 2 y1 y3
2

bcx + d g
12 c b ad bc g
=
bcx + d g

, show that y2 at x = 1 is equal to zero.


x

log y = x 0 log x = x log x


Now differentiating (1) w.r.t. x, we get

g b

ax + b
y=
cx + d y = ax + b
cx + d

acx + d f y = dxd aax + bf


acx + d f y + y c = a
d
d

cx + d f y + y c =
a
aa f
dx
dx
a cx + d f y + y c + y c = 0
b cx + d g y + 2 y c = 0
d
acx + d f y + 2 y c = 0

dx

2 x =1

dy
= y log x + 1
dx
1

a
f
y = y alog x + 1f
1
y = y alog x + 1f y
x
1O
L
y
= M y a1 + log x f y P
xQ
N
1O
L
But M y a1 + log x f y P
xQ
N

d
dx

...(1)

1 dy
1

= log x x = log x 1
y dx
x

Hence, 2 y1 y3 = 3 y2 2
Second method:

Solution: y =

F 1I
H xK

F 1I , x > 0
H xK
F 1I
F 1I
log y = log G J = x log G J = x log 1 log x
H xK
H xK

4 c ad bc

2 y1
2 y1 c

3 y2
3 y2 c

y1
2 y
= 1
3 y2
y2

2. If y =

= 3

3 y2 2 = 2 y1 y3

b ad bcg 6 c bad bcg


=2
bcx + d g bcx + d g
12 c b ad bc g
=
bcx + d g
2

...(4)

Dividing (3) by (4),

R.H.S. = 3 y2 2

...(3)

...(2)

y3

579

...(1)

= y1

x =1

x =1

x =1

1 + log x

x =1

LM y OP
NxQ

x =1

...(2)

580

How to Learn Calculus of One Variable

b
g
= b y g blog x + 1g
= 1 b 0 + 1g = 1
a1+ log xf = a1+ log1f = a1+ 0f = 1
LM y OP = b yg = 1
NxQ
y1

x=1

= y log x + 1

x =1

x =1

x =1

x =1

x =1

z x =1 =

...(b)
...(c)

F x , y , dy I and we are required


H dx K

d2y
at a point x = a, we are
dx 2
dy
required to find the value of (i)
at x = 1 (ii) The
dx
value of y at x = 1 (iii) The value of function of x at

to find out the value of

d y
at x = a
dx 2

x = 1 i.e; the value of each term of

LM FG
N H

y = log x + 1 + x

3. If

d1 + x i y
2

+ x y1 = 2

LM FG
N H

Proof: y = log x + 1 + x

IJ OP
KQ

IJ OP
KQ

11 1= 0

d2y
= f
Note: If
dx 2

...(a)

Putting the values of (a), (b) and (c) in (2) , we


have

by g

LM FG
IJ OP
KQ
N H
LM
FG x + 1 + x IJ OP
H
KP
1
MM F

I
PP
1+ x
MN GH x + x + 1JK
Q
L F
IO 1
= 2 Mlog G x + 1 + x J P
H
KQ x + 1
N
= 2 log x + 1 + x

x =1

show that ,

Now we square both sides to get

LM F
N H

x2 + 1

1+ x

1 + x 2 y1 2 = 4 y

...(1)

Now differentiating again to get the second


derivative from (1), we have

b g

d
d
1 + x 2 y12 =
4y
dx
dx

d i dydx + y dxd d1 + x i = 4 dydx


dy
dy
d1 + x i 2 y
+ y 2x = 4
dx
dx
d1 + x i 2 y y + y 2 x = 4 y
1 + x2

dy

= y1
dx

4y

IK OP 1
Q e x + 1j
2

y12 = 4 log x +

Now, dividing both sides of (2) by 2 y1 we have

L F
I O d LMlog FG x + 1+ x IJ OP
= 2 Mlog G x + 1 + x J P
H
K Q dx N H
KQ
N
1
L F
IO
= 2 Mlog G x + 1 + x J P
H
K
N
Q FG x + x + 1IJ
H
K
LM
O
1
1+
2 xP
MN 2 x + 1 PQ
2

d1 + x i y
2

+ x y1 = 2

FH

2
2
4. If y 1 + x = log x + 1 + x

e1 + x j y + x y = 1

IK

show that

FH

2
2
Proof: y 1 + x = log x + 1 + x

IK

...(1)

Now differentiating both sides of (1) w.r.t. x , we


have

Successive Differentiation

FG
H

Now, differentiating both sides of (1) w.r.t x, we


have

I d 1+ x y
1+ x J +
K
dx
L
OP
1
2x
=
M1 +
FG x + 1 + x IJ MN 2 1 + x PQ
H
K
dy

dx

dy

dx

FH

1+ x

IK +

xy
1+ x

1+ x

...(2)

Now multiplying both sides of (2) by 1 + x 2 we


have

e1 + x j dydx + x y = 1
e1 + x j y + x y = 1
2

5. If y = a sin log x , show that x 2 y2 + x y1 + y = 0

a f
dy
1

= a cos a log x f
dx
x

Proof: y = a sin log x

d y
dx

...(1)

dy a sin log x
=
dx
x

6. If x = cos log y , show that 1 x

log y = cos

...(4)

2 y2 1 x 2 2 x y1 = 2 y

a f

7. If y = sin sin x , show that


2

d y
2

+ tan x

2
dy
+ y cos x = 0 .
dx

a f

x y2 + x y1 + y = 0

Proof: x = cos log y

dy
dx

= 2y

LM OP d1 x i + F dy I b 2 xg = 2 y dy
H dx K
dx
N Q
dy d F dy I
F dy I
2

e1 x j 2 x
H dx K
dx dx H dx K
d dy
dx dx

...(1)
b g
dy

= cos a sin x f cos x = cos x cos asin x f ...(2)


dx

...(3)

Proof: y = sin sin x

= y2

Again differentiating (3) w.r.t. x ( to get second


derivative)

dx

d y
dy
+ x
= y
2
dx
dx
Q a sin log x = y

x2

1 x 2 y2 x y1 = y

F dy I + dy dx = a asin log xf 1
H dx K dx dx
x

...(2)

Now, dividing both sides of (4) by y1 , we have

a f

d
dx

i FH dydx IK

1 x2

2 y1 y2 1 x 2 2 x y1 2 = 2 y y1

dy
= a cos log x
dx
Now, differentiating (1) w.r.t. x, we get
x

1 x2

1 x2

dy
=y
dx
Now squaring both sides of (2), we have

1
2

1 dy

=
y dx

581

jy

d y
dx

x y1 = y

a f

a f

= cos x sin sin x sin x cos sin x


2

L. H.S =

...(1)

a fa

= cos x sin sin x cos x + cos sin x sin x

d y
dx

+ tan x

dy
2
+ y cos x
dx

...(3)

582

How to Learn Calculus of One Variable

a f

a f

= cos x sin sin x sin x cos sin x +

Proof: y = sin m sin

a f

sin x
2
cos x cos sin n + y cos x
cos x

a f
a f
sin x cos bsin x g + y cos x
= cos x sin a sin x f + y cos x
bQ y = sin asin xfg
2

= cos m sin

= 0 = R.H.S
8. If

a f

1 x

a f

a f

...(1)

a f

a f

...(2)

a f

dy
= a sin log x + b cos log x
...(3)
dx
Again differentiating the equation (3) ( to get
second derivative), we have,
x

F dy I + dy dx
H dx K dx dx
1
1
= a cos alog x f + b b sin a log x fg
x
x
2

d y
dx

+x

a f

a f

x 2 y2 + x y1 + y = 0

9. If y = sin m sin

e1 x j y
2

x prove that
2

x y1 + m y = 0 .

1 x

1 x

dy
dx

= m cos
2

em sin xj
1

LM
N

= m 1 sin m sin x

= m2 1 y2

jOPQ

...(1)

Now again differentiating both sides of equation


(1) to get second derivative,

e1 x j 2 dydx dxd FH dydx IK + FH dydx IK a 2 xf


F dy I
= m 2y
H dx K
d1 x i 2 y y 2 x y = m 2 y y
2 d1 x i y 2 x y = m 2 y
d1 x i y x y = m y
2

a f

a f

x y2 + x y1 = a cos log x + b sin log x


x 2 y2 + x y1 = y

dy
= a cos log x b sin log x
dx

d
x
dx

jOPQ

x m

dy
1
1
= a sin log x + b cos log x
dx
x
x

1
dy
= m cos m sin x
dx

Proof: Q y = a cos log x + b sin log x


Now, differentiating (1) w.r.t x, we have

a f

LM
N

OP
Q
F dy I
e1 x j
H dx K
F dy I
d1 x i
H dx K

y = a cos log x + b sin log x , show that

x 2 y2 + x y1 + y = 0

LM e
N

1
dy
d
sin sin x
=
dx dx

= cos x sin sin x sin x cos sin x +

d1 + x i y
2

10. If y = tan

x prove that

+ 2 x y1 = 0 .

1
Proof: Q y = tan x
...(1)
Now differentiating both sides of (1) w.r.t. x, we
have,

dy
1
=
dx 1 + x 2

1 + x2

i dydx = 1

Successive Differentiation

Again differentiating (2) w.r.t.x to get the second


derivative,

d i FH dydx IK + dxdy dxd d1 + x i = 0


d y dy
d1 + x i
+
2x = 0
dx
dx
d1 + x i y + 2 x y = 0
11. If y = e tan x j show that
d
dx

1 + x2

d1 + x i

2 2

d2y
dy
+ 2 x 1 + x2
=2
2
dx
dx

Proof: y = tan

d1 x i y
2

d tan x 2 tan x
=
2
dx
1+ x

j dydx = 2 tan x
F dy I = 4 e tan x j
e1 + x j
H dx K
F dy I = 4 y
d1 + x i
H dx K
1

1+ x

2 2

2 2

L
e1 + x j MMN dxd FH dydx IK
d a4 y f
=

OP + F dy I L d 1 + x O
PQ H dx K MN dx e j PQ
2

dx

L dy d F dy I OP +
e1 + x j M2
N dx dx H dx K Q
F dy I L2 e1 + x j 2 x O = 4 dy
PQ dx
H dx K MN
2

2 2

, prove that

m1

1 x

my
1 x2

dy
= my
...(1)
dx
(We square both sides of (1) so that square root
symbol may be removed)

...(2)

x y1 = m 2 y

= e m sin

...(1)

dy d e m sin

=
dx
dx

Now again differentiating both sides of (2) w.r.t. x,


we have
2 2

m sin
Proof: y = e

dy
we have
dx

d 2y
dy
+ 2x 1 + x 2
=2
dx
dx 2

m sin
12. If y = e

...(3)

d 2y
dy
+ 2x 1 + x2
=2
2
dx
dx

2 2

e1 + x j
1 + x2

1
dy
d

=
tan x
dx dx

= 2 tan

Now dividing equation (3) by 2

O
L
j MM2 dydx ddx y PP +
Q
N
F dy I L4 x e1 + x jO = 4 dy
PQ dx
H dx K MN
2 2

1+ x

583

1 x2

d1 x i FH dxdy IK

= m2 y2

...(2)

Again differentiating both sides of (2) to get


second derivative,

L
e1 x j dxd MMFGH dydx IJK
N
d
=
em y j
dx
2

1 x

OP + F dy I
PQ GH dx JK

d
1 x2
dx

j 2 dydx dxd FH dydx IK + FH dydx IK a 2 xf


2

584

How to Learn Calculus of One Variable

j 2 dydx ddxy 2 x FH dydx IK


2

1 x

1+ x

f
Q y = x log x + x sin a log x f from (1)
y x log x = x sin a log x f
2

x y2 = x log x y + x y1 y

x y1 = m y
2

x y2 = x log x + x y1 2 y

13. If y = x sin log x + x log x , Show that


2

d y
dx

LM a f
a fOPQ +
N
LMx 1 + log xOP
N x
Q
y = cos alog x f + sin a log x f + 1 + log x

...(2)
1
Now, multiplying both sides of (2) by x, we have

a f

x y1 = x cos log x + x sin log x + x + x log x


...(3)

a f

x y1 = x cos log x + y + x , (by using (1) in


(3) )
...(4)
Now, differentiating again both sides of (4) w.r.t x
to get y 2 ,

bg

b g

1
+ cos log x + y1 + 1
x

a f

...(5)
x y2 = sin log x + cos log x + 1
Now multiplying both sides of (5) by x, are get

a f
a f
= x sin alog x f + xy y x

x y2 = x sin log x + x cos log x + x


2

x y2

dy
1
= x cos log x + sin log x
dx
x

x y 2 + y1 = x sin log x

FG x IJ , prove that
H a + bx K
x y =ay x y f .
F x IJ , defined for x > 0
Proof: y = log G
a + bx
H a + bx K
F x I
y = x log G
H a + bx JK
L 1 b OP
y = log | x | log b a + bx g + x M
N x a + bx Q
L a + bx bx OP
x
y = log
+xM
a + bx
N x aa + bxf Q
14. If y = log

Proof: We are given y = x sin log x + x log x ...(1)


Now differentiating each terms of both sides w.r.t.
x of (1), we have

x y2 x y1 + 2 y = x log x

dy
+ 2 y = x log x
dx

a f

iy

x y 2 = y x log x + x y1 y

2 1 x 2 y1 y 2 2 x y1 2 = 2 m 2 y
2

x y1 y x = x cos log x
2

dy
dx

= m 2y

Q x y1 = x cos log x + y + x from (4)

dy
=m
dx
2

...(6)
+ x

= log
=

x
a
+
a + bx
a + bx

y
a
+
x
a + bx

x y1 = y +

ax
a + bx

x y1 y =

ax
a + bx

...(1)

Successive Differentiation

Again differentiating (1) to get y2

a2

x y2 =

b a + bxg

15. If y = e

Solution: y = e

j dydx = e

tan x

tan 1

x= y

...(1)

d i ddx y + b2 x 1g dxdy = 0
d1 + x i y + b 2 x 1g y = 0
2

dx

1+ x

1+ x

tan x

Type 3: Problem based on finding a differential


equation when the given function is in the implicit
form f x , y = c (where c = any constant and c = 0
in particular) or givin a function having the form
x = f1 f2 y

b g
af

b g

F 1 I e
GH 1 + x JK
e1 + x j

tan x

x y1 = y

b g

Proof: Q x = sin log y

We may straight way (or, directly), find

d y

a1 2 x f a1 2 xf e

d1 x i y

dy
e
=
dx 1 + x 2

1+ x

1. If x = sin log y show that

tan x

e1 + x j e

tan x

tan x

Example worked out:

Second method after having

a1 2 x f + a2 x 1f e

Working rule: Proceed as usual i.e finding the


derivatives involved in the given differential equation
to be satisfeid by the given function .

tan x

=0
= R.H.S
N.B.: The first method is more convenient in
comparision to the second method.

d 2 y dy
dy
+
2x =
dx
dx
dx 2

1 + x2

tan x

1+ x

Differentiating both sides of (1) w.r.t x, we have

1 + x2

tan x

1+ x

1
e
tan x d
1
dy
tan x =

=e

2
dx
dx
1+ x

show that 1 + x y2 + 2 x 1 y1 = 0

tan x

2 2

tan x

2 2

x 3 y2
proved.

tan x

g = bax+ bxa g = x x y [from (2)]


= b x y yg Which was required to be

...(2)

Now squareing (1), we get

bx y

a1 2 xf
e1 + x j
L.H.S. = d1 + x i y + b2 x 1g y
e1 + x j e a1 2 xf + a2 x 1f e
=
1+ x
e1 + x j
1

ba + bx g a ax b
ba + bx g

x y2 + y1 y1 =

585

tan x

log y = sin 1 x
2x

2 2

w.r.t. x)

1 dy

=
y dx

1
1 x2

...(1)
(on differentiating (1)

586

How to Learn Calculus of One Variable

y1
=
y

1
1 x

y1 1 x 2 = y

...(2)

b g d1 x i = y

y1

(on squaring both sides

...(1)

...(2)
...(3)

af

+ 2 y1 = 0

b
g b

(3) )

=
=

g b

b x + 4g

6 x 6 x 24

...(1)

sin x

= sin x

...(2)
[N.B.: Product rule of derivative is easier than the
quotient rule. This is why we cross multiply]
Now differentiating (2) w.r.t x to find y1,
2

...(3)

e x y2 + 2 x e x y1 + 2 x e x y1 + 2 x 2 e x y + e x y

g
g

LM FG 3x IJ 6OP
b x + 4g b x + 4g N H x + 4 K Q
2y 6

ye

3 y
2 y1
2
y2 =
=

(by using
x+4
x+4
x+4

e x y1 + 2 x e x y = cos x
Again differentiating (3) w.r.t x to find y 2

d 2 y dy
dy
x+4
+
1 =
2
dx
dx
dx

LM sin x OP
MN e PQ

g
g

+ y1 + y1 = 0

log y = log

Now, taking anti log of both sides of (1) y =

3 y
dy
y1 =
=
dx
x+4
Now considering (2),

d y
dx 2

Proof: x y + 4 y = 3 x
Now, differentiating (1) w.r.t x
dy
dy
+ y 1 + 4
=3
x
dx
dx
dy
x+4
=3 y
dx

b g
b x + 4g y
b x + 4g y

Q log e e x = x 2 log e e = x 2 1 = x 2

b
b

a f
d y
dy
+ 4y
+ d 4 x + 3i y = 0
dx
dx
Proof: log y = log asin x f x
log y = log a sin x f log e

12
24
y2 =
x+4
x+4

Note: y = 3

2. If x y + 4 y = 3 x find

b x + 4g

24

b g

24 =

b y g d1 x i = dydx
y b 2 x g + 2 y y d1 x i = 2 y y
x y + d1 x i y = y
d1 x i y x y = y
d
dx

b x + 4g

3. If log y = log sin x x , show that

of (2) )

= sin x = e y (from (2) )

...(4)
2

Now dividing the equation (4) by e x and then


transposing,
y2 + 2 x y1 + 2 x y1 + 4 x 2 y + 2 y + y = 0

y2 + 4 x y1 + 4 x 2 + 3 y = 0

4. If cos

F y I = log F x I
H b K H nK

, show that

587

Successive Differentiation

x 2 y2 + x y1 + n 2 y = 0 .

FG y IJ = log FG x IJ since x > 0


H bK H n K
n
FG y IJ = n log FG x IJ = n clog x log n h
H bK
HnK
n

Proof: cos 1
cos1

...(1)

1
1

F yI
H bK

b b2 y 2

y1 n
=
b
x

Proof: y = sin pt

n
x

...(2)

Now on cross multiplying the equation (2) and


then squaring, we get

x 2 y12 = n 2 b 2 y 2

...(3)

Now, again differentiating the equation (3) w.r.t x

d
d
n2 b2 y2
x 2 y12 =
dx
dx

x 2 y1 y2 + 2 x y1 = 2 n y y1
Now dividing (4) by 2 y1 , we get
2

dy
+ p2 y = 0 .
dx

x = sin t

y1 b

Examples worked out:


1. If y = sin pt , x = sin t , show that

d1 x i ddx y x

Now differentiating the equation (1) w.r.t. x

dy
according to the need of the problem
dx
of differential equation. Lastly by simplification,
cancellation, transposition and using axioms of an
equation, we arrive at our target (i.e., we find the
required differential equation)
directly from

...(4)

x 2 y2 + x y1 + n 2 y = 0
Type 4: Problems based on finding a differential
equation when the given function is in parametric

R
form S
T y = f at f
x = f1 (t )
2

Working rule: We proceed directly for finding the

dy
dy
first derivative using the formula
= dt and then
dx
dx
dt
we find other derivatives (2nd , 3rd , .... etc) involved
in the given differential equation using various
dy
mathematical manipulation performed on
or
dx

dy
dy
p cos pt

= dt =
dx
cos t
dx
dt

F dy I

H dx K
=

p cos pt

cos t

p2 1 y 2
1 x

j for

i FH dxdy IK

p 1 sin pt

e1 sin t j

x 1

d i
d L
F dy I OP = d p d1 y i

1 x i
M
d
H dx K PQ dx
dx MN
dy d y
F dy I
e1 x j 2

+ b 2xg G J
H dx K
dx dx
F dy I
= p G 2 y J
H dx K
1 x2

= p2 1 y2
2

Dividing by 2

dy
, we have for x 1 , y 1;
dx

d1 x i ddxy x dxdy + p
2

y = 0 proved.

588

How to Learn Calculus of One Variable

2. If x = a cos + sin and y = a sin cos

b g e

1 + y12

1
where 0 < < prove that tan y1 =
a y2
2

3
2

dx
Proof: x = a cos + sin
d

= a sin + 1 sin + cos = a cos

y = log t

1
dy
1
dy

= dt = t =
dx
t sin t
dx
sin t
dt

...(1)

dy

= a cos 1 cos sin = a sin


d

y = a sin cos

gh

...(2)
dy
d
a sin
dy
Now,
=
=
= tan
dx
a cos
dx
d

...(3)

F dy I = d atan f
H dx K dx
2

d y
dx

= sec

d
1
2
= sec
dx
a cos

d y
dx

e1 + tan j
=
2

3
2

e1 + y j hence the result.


=
a tan b y g
2
1

3
2

F I
H K

=0

Solution: x = cos t

at t =

.
2

dx
= sin t
dt

sin t + t cos t

dt
dx

...(ii)

Now, squaring both sides of (1) , we get


2

=+

1
2

...(iii)

t sin t

Adding (ii) and (iii), we get

F I
H K

d y
dy
+
dx
dx

asin t + t cos t f +
2

LM d y + F dy I OP
MN dx H dx K PQ
L asin t + t cos t f +
=M
MN t sin t
t
LM
O
1 P
1
=M
+
MN 42 42 PPQ

1
2

1
2

sin

t sin t

=0

t sin t

t=

3. If x = cos t , y = log t , show that

d2 y
dy
+
2
dx
dx

sec
a

b g b
sin t + t cos t F 1 I
=
G
H sin t JK
t sin t
a sin t + t cos t f
=
1
d2y
=
2
dx
t sin t

t sin t

...(i)

F dy I = d FG 1 IJ
H dx K dx H t sin t K

F dy I
H dx K

FGQ dx = a cos IJ
H d
K
2

d
dx

Differentiating both sides of (3) w.r.t. x, we get

d
dx

dy 1
=
dt
t

OP
t PQ

t=

Successive Differentiation

Type 1: Problems based on finding second derivative


(A) Algebraic functions

6.

Exercise 14.1
9.
2

Find

d y
of the following functions.
dx 2

1. y =

2. y = x 2 x + 3

1
3. y = 3x 4

4x

dx

+4

4x

8
2x + 3
9

7. y =

12

Find

sec 2 x

3. y = sin 3x cos x

sin x + 2
2 cos x + 3
Answers:
5. y =

x2 9

e
j
L
O
2. 2 y M2 x cot e x 3j + 2 x y cot e x 3jP
N
Q
3. b8 sin 4 x + 2 sin 2 x g
1
4. b cos 2 x + 8 cos 4 x 9 cos 6 x g
8

2
3

2
2
1. y 2 sec 2 x + tan 2 x

12. y = x 3 7 x + 1

3x + 4
x

Answers:

b2 x 3g
18

b3 x 4 g

4. y = sin x cos x

11. y = 2 x + 3

3.

1.

+ 84 x 2 7 x + 1 + 98 x 3

2. y = cosec x 3

1
9. y = 2
x +4

13. y =

4
3

d2y
of the following functions.
dx 2

1. y =

ax
8. y =
a+x

3
2

Exercise 14.2

10. y =

(B) Trigonometric function:

b g
y = b3 2 x g

6. y =

Type 1: (continued)

1
4. y = a x
5.

10. 9 x 2 9

12. 6 x 7 x + 1
13. Find

2
3

4a
8. a + x
x

7.

6 x3 8

11.

2x 3
2

589

3
2

4.

2.

i
9 d x 2 x + 3i
4 x2 2x + 7

4
3

ba xg

5. 3 3 2 x

5.

5
2

8 cos2 x + 16 sin 2 x + 12 sin x cos x + 12 cos x 10 sin x

b2 cos x + 3g

590

How to Learn Calculus of One Variable

Type1: (continued)
(C) Inverse Trigonometric functions:
Exercise 14.3
d2y
of the following functions
Find
dx 2
1
1. y = sin x
1

2. y = tan 2 x
3. y = sec

Answers:
1. 4 y

2. y cos 2 x sin x
3.

y2 =

(E) Logarithmic functions:

Find

d2y
of the following functions
dx 2

1. y = log 2 x + 3

2. y = log sin x

d1 i

3
x2 2

3. y = log cos x

16 x

la2 x + 3f a3 5xfq
F a + x IJ
y = log G
H a xK

d1 + 4 x i
e2 x 1j x
x e x 1j

4. y = log

x 1 x 2 sin 1 x

Answers:

2 2

4.

Exercise 14.5

5.

3.

d1 x i

Type 1: (continued)

Answers:

2.

m y + x y1

4. y = 1 x 2 sin 1 x

1.

3
2

6. y = log

e1 x j

3
2 2

Type 1: (continued)
(D) Exponential functions:
Exercise 14.4

1.

FG ax + b IJ
H cx + d K

b2 x + 3g

2. cosec x
2

3. sec x
Type 1: (continued)
(F) Implicit functions:

d2y
of the following functions.
Find
dx 2

Exercise 14.6

1. y = e 2 x + 3
2. y = e

sin x

3. y = e

m sin x

Find
1

d2y
of the following functions.
dx 2

1. x 2 + x y + y 2 = a 2

Successive Differentiation

2. x 2 + y 2 + 3xy 7 = 0
x + y = a
4. cos x cos y = c

2. x =

3t
1 + t3

y=

3t 2
1 + t3

3.

5. y = tan x + y

2
3. x = a t
y = 2 at

6. y + 3 x = 25
2

x2
y2
+
=1
a2
b2
Answers:

4. x = a cos

7.

6a

1.

y = b sin

b x + 2 yg
10 e x + 3x y + y j
b 2 y + 3x g
3

2.

3.

a
f
y = a a1 cosf

5. x = a + sin

6. x = a sec 2

1
2

4.

y = a tan
3

a
x3

7. x = 3 cos cos
3

tan x + tan y + 2 tan x tan y


2

tan 3 y

a x + yf
sin a x + y f
9
6. b 25 3 x g
4
5. 2

cot

3
2

b 4
7. 2 3
a y

y = 3 sin sin

8. x = t 2

b g

y = t 1
9. x =

y=

1
t
1
t +1

10. x =

Type 1: (continued)
(G) Parametric functions:
Exercise 14.7
d2y
Find
of the following functions.
dx 2
3

1. x = a cos
3

y = b sin

y=

a 1 t2
1+ t

2 at
1 + t2

11. x = a cos t

y = a sin t

12. x = a sin 2t + 3

y = a cos 2 t + 3

13. x = 2 1 sin t

y = 4 cos t

591

592

How to Learn Calculus of One Variable

y = a sin

15. x = a cos3

2.

y = a sin

16. x = a tan

3. x = 2 cos t cos 2 t

1
2
y = a sin
2
17. x = a cosec

y = 2 sin t sin 2 t at =

3.
6.

y = t 3 at t = 1

sec 4 cosec 2.

3a 2

j
3 e1 2 t j
2 1 + t3

5. If x =

3 3

FG IJ
H K

1
1
b
4
3
4. cosec 5. 4 a sec 2
2a t 3
a

3
cot 7. cot 2 cosec 5
4a

11.

b1 + t g

e1 + t j

2 3

2t3

1
8.
9.
2 t3

10.

8a t

sec 4 cos ec
3
13. sec t 14. 0 15.
3a

16.

cos 4 4 cos 2 3
a

tan 3
17.
a

Type 2: Problems based on finding the value of at


the indicated points
Exercise 14.8
2

d y
at the indicated points for the following
dx 2
functions.

Find

1
t3

y = t 2 at t =

1
2

Answers:
1.

1
3
1
3
5
2.
3.
4.
5.
a
2
a
4
1152

Type 3: To show that a given function satisfies a


differential equation
(A) Problems based on showing that an algebraic
function satisfies a differential equation

sec 3 2 t + 3
cos ec 3 t
12.
a
a

4. If x = t 2

y = a cot
Answers:

1.

a
f

y = a a1 cos f at =
2
x = a a1 cos 2 f

y = a a 2 sin 2 f at =
4

1. x = a + sin

14. x = a cos

Exercise 14.9
1. If y 2 = 2 x 2 + 3 x + 5 , show that

d2 y
=
dx 2

31

4 2 x 2 + 3x + 5

3
2

2. If x 3 + y 3 = 3x 2 , show that

d 2 y 2x 2
+ 5 =0
dx 2
y

3. If x 2 xy + 2 y 2 5 = 0 , show that

2
2
d 2 y 14 x xy + 2 y
=
3
dx 2
x 4y

Successive Differentiation

4. If y = x 2 3x + 5 , show that

FG dy IJ
b g H dx K
5. If y = FH x 1 + x IK , show that
2
d2y
=
2
dx
2x 3

1 + x2

d2 y
dy
+x
9y = 0
dx
dx 2

6. If 2 y = x + 1 +

x 1 , Show that

i ddx y + 4 x dxdy y = 0
2

7. If y m + y m = 2 x , show that

dx

i ddx y + x dxdy my
2

d2y
h 2 ab
=
3
dx 2
hx + by

2
16. If y = 1 x

d1 x i y

22. If y =

ax 2 + bx + c
, prove that
1 x

b1 xg y

1
y5

d x 1i y

+ xy1 m 2 y = 0

+y

13. If Z =

1
5

= 2 x , show that

+ xy1 25 y = 0
2
1
d2y
4 d y
3 dy
x
2
+
=
, show that x
dx dZ 2
x
dx 2

x+ y +

2
y x = c , show that y2 = 2
c

23. If y =

= 3 y2
ax + b
, show that 2 y1 y3 = 3y2 2
a + bx

IJ
K

b
, show that x 2 y2 + x y1 y = 0
x

18. If y = ax +

ax b
, show that 2 y1 y3 = 3y 2 2
a bx

a xy

x 1

21. If y =

d x 1i y

FG
H

+B x

ax + b
, show that 2 y1 y3 = 3y 2 2
cx + d

20. If y =

14. If

IJ
K

2 x 1 , show that

dy axi
10. If y = FH x + 1 + x IK , show that
d1 + x i y + xy n y = 0
11. If y = FH x + x 1 IK show that
12. If

2
2
show that x 1 y2 + xy1 n y = 0

9. If x 3 + y 3 = 3 a x y , show that

FG
H

17. If y = A x + x 1

ax 2 + b , show that y2 y 3 = ab

, show that

+ xy1 + 3 y = 0

y2 = 2

3
2

19. If y =

i ddx y + 4 x dydx y = 0

=0

8. If y = 2 x + 1 +

d4 x

15. If ax 2 + 2h xy + b y 2 = 1, show that

4 x2 1

593

24. If y = 2ax +
25. If y =

3b
, show that x 2 y2 + x y1 y = 0
x

ax 2 + 2b , show that y2 y 3 = 2ab

ax b
26. If y = cx d , show that 2 y1 y 3 = 3 y2 2
27. If y =

ax b
, show that 2 y1 y 3 = 3 y2 2
2a bx

28. If

y=

b1 xg y

ax 2 + bx + c
,
1 x

= 3 y2

prove

that

594

How to Learn Calculus of One Variable

4ax + b
, show that 2 y1 y3 = 3y2 2
2a + bx

29. If y =

30. If x = sin log y , show that

e1 x j y

x2

d2y
dy
+x
+y=0
dx
dx 2

11. If y = tan

xy1 = y

10. If y = 2 cos log x + 3 sin log x , show that

x , show that 1 + x 2 y 2 + 2 xy1 = 0

Type 3: (continued)
(B) Problems based on showing that a trigonometric
function satisfies a differentiatial equation

12. If y = 4 cos x + 9 sin x , show that

Exercise 14.10

13. If y = A cos mx + B sin mx , show that

dx

f a

sec ax + b tan ax + b

14. If y = sin

2. If y = 2 sin 3x 5 cos 3x, show that

d y
dx

3. If y = sin (m cos1 x), show that


2

e1 + x j dx

d y
2

15. If y = sin
2

d 2 y 36
= 3
dx 2
y

y2 + m 2 y = 0

1. If y = sin (ax + b) show that


d y

y+

dy
2
+m y=0
dx

4. If y = a sin (log x), show that


2

x y1 + xy1 + y = 0
5. If y = a cos (log x), show that

+ 9y = 0

d1 x i y

x y1 + xy1 + y = 0
2

y2 + y1 tan x + y cos x = 0

7. If y = cos (m sin1 x), show that

e1 x j y
2

xy1 + m y = 0

x , show that

j
d x + 1i y + 2 x d x + 1i y = 2
17. If y = e cos x j , show that
d1 x i y = xy + 2
16. If y = tan

x , show that

6. If y = sin (sin x), show that

x y1 = 2

2
x , show that 1 x y2 xy1 = 0

18. If p = a cos + b sin , show that

p+

d 2 p a2 b 2
=
d 2
p3

19. If y = P sin 2 x Q cos 2 x , where P and Q are


d2y
+ 4y = 0
dx 2

8. If x = sin (log y), show that

constant, show that

e1 + x j y

20. If y = a sin x + b cos x show that

xy1 = y

9. If x = cos log y , show that

d1 x i y
2

xy1 = y

21. If y = a cos log x + b sin log x

x 2 y2 + xy1 + y = 0

d2y
+ y=0
dx 2

show that

Successive Differentiation

sin

22. If y =

d1 x i y

1 x

23. If y = sin m sin

show that

xy1 + m 2 y = 0

IJ
K

x + 1 , show that

d1 + x i ddx y + x dydx = 0
L F
9. If y = Mlog G x + x
N H
d1 + x i ddx y + x dydx = 2

3 xy1 + m y = 0

d1 x i y

FG
H

8. If y = log x +

show that

595

+1

IJ OP
KQ

show

that

Type 3: (continued)
(C) Problems based on showing that an exponential
or a logarithmic function satisfies a differential
equation
Exercise 14.11

10. If y = e

a sin x

, show that

d1 x i ddx y x dxdy = a y
11. If y = log a sin x f ,
show that y + b y g + 1 = 0
F
I
y = log G x + a + x J ,
12. If
H
K
da + x i y + xy = 0
2

1. If y = A e

nx

2. If y = 2 e

3x

+ Be

nx ,

+ 3e

show that y2 = n y
2

2 x , Show that

d2y
dy
+ 5
+ 6y = 0
2
dx
dx

3. If y = e

2 sin

Show that

Type 3: (continued)

show that

(D) Problems based on showing that parametric


functions satisfy differential equations

e1 x j ddx y x dydx 4 y = 0
2

g
g

4 x+ y
d y
4. If x + y = e x y , show that dx 2 =
x + y+1
2

5. If y = e

m tan x

show that

d1 + x i ddx y + b2 x mg dxdy = 0
2

6. If y = e

dx

show that

+ 1 y2 + 2 x 1 y1 = 0

7. If y = e

tan x

1. If x = a + sin y = a 1 + cos , show that

d2y
a
= 2.
2
dx
y
2. If x = 2 cos + 3 sin y = 2 sin 3 sin show

13
d2y
= 3.
that
2
dx
y

tan x

Exercise 14.12

a
f y = a asin sin f ,
F 0 < < I show that d y = sec
H
2K
a
dx

3. If x = a cos + sin

, show that

ecos xj y a1 + sin 2 xf y = 0
2

596

How to Learn Calculus of One Variable

4. If x = cos
that

F1 t I
GH 1 + t JK
2

y = sin

d2y
is independent of t.
dx 2

Hint: If x = cos
y = sin

F 2t
GH 1 + t

F 1 t I = 2 tan
GH 1 + t JK
I = 2 tan t
JK
2

F 2t I
GH 1 + t JK show
2

3. If
1

5. If x = cos t y = log t show that

F I
H K

d2 y
dy
= 0 at t =
+
2
2
dx
dx
Type 3: (continued)

(E) Problems based on showing that product of any


two functions satisfy a differential equation
Exercise 14.13

2. If y = x tan

that

y=e

F
H

x
2

2
d2 y
=
2
dx
1 + x2

x show that

4. If x = a + bx e x show that

x3

F
H

I
K

d2 y
dy
= x
y
dx
dx 2

ax

5. If y = e cos bx show that

i
6. If y = A e cos abx + cf
y + 2 a y + da + b i y = 0
y2 2 a y1 + a 2 + b 2 y = 0
ax

show that

n 1

1. If y = x sin x Show that

7. If y = x

x 2 y2 2 xy1 + x 2 + 2 y = 0

x 2 y2 + 3 2n x y1 + n 1 y = 0

log x show that

v3
v3
x + b sin
x , show
2
2

a cos

d 2 y dy
+
+ y = 0.
dx
dx 2

I
K

LHospitals Rule

597

15
LHospitals Rule

Question: What is LHospitals rule?


Answers: LHospitals rule is a rule permitting the
evaluation of an indeterminate quotient of functions
as the quotient of the limits of their derivatives.
sin x
Example: 1. lim
is an indeterminate of the
x0
x
cos x
0
=1
form
but it can be evaluated as lim
x0
1
0
Indeterminate Forms
There are eight indeterminate forms involving
difference, product, quotient or power of 0 and ( in
connection with finding limit) which we face in practice

0
which are (1)
(2)
(3) 0

0
(4) (5) 0 (6) (7) 00 (8) 1 which we face
in practice are the expressions known as indeterminate
forms of the given functions for the limit.
0

= 0 or
= is the
Remember: 1 0 = 1,

0
determinate form having a meaning for the given
expression for the limit.
Statement of LHospitals Rule

bg

bg

af
af

f x
, where
xa
xa
F x
f and F are the derivatives of f and F, approaches
f x
a limit as x approaches a, then
approaches the
F x
If lim f x = lim F x = 0 and

af
af

same limit.

Proof: lim

bg
F bxg
f b x g f ba g
xa
F b x g F ba g
f x

xa

= lim

xa

xa
[Since f (a) = F (a) = 0]

b g , F bag 0
F ba g
e.g. If f a x f = e x 1j
=

fa

F (x) = (x 1)
a=1
then

af
af

f a
0
=
F a
0 and

e x 1j .
which is lim
ax 1f

lim

x 1

af
af

2x
f x
= lim
=2
x

1
1
F x

x 1

Method of Evaluating Limits in


Indeterminate Forms
1. Evaluation of
If lim

xa

af
af

0
and
forms:

af
af

f x
f x

0
= or, lim
=
x
a

0
g x
g x

then we adopt the following working rule:

598

How to Learn Calculus of One Variable

Working rule: 1. Go on differentiating numerator and


denominator separately till we get a definite value at
x = a. This working rule may be expressed in the
symbolic form in the following way

lim

xa

af
af

af
af

af
af

f x
f x
f x
= lim
= lim
x a g x
x a g x
g x

= lim

xa

af
af

The above types of indeterminate forms and the


working rule for these indeterminate forms may be
put in the chart form in the following way.
Types of indeterminate
froms at x = a

f x
= ... = a definite value
g x

L f axf OP , LM f axf OP ,
N.B.: Any one of the M
N g axf Q N g axf Q
LM f axf OP , ...etc which first provide us a
N g axf Q
x=a

x=a

f 0
=
g 0

lim

2. lim

f
=
g

f
= ... then any one of
g
these which is not meaningless
at x = a will give the required
answer.
= lim

3. lim f g = 0

lim

2. Working rule to evaluate 0 at x = a

FG lim f b xg g b xg = 0 IJ
H
K

form, we are required to change f g =

f
1g

lim f g should be changed into lim

afaf

xa

g x

1g

which

FG g IJ
1 log f K
, then we apply
eH

LHospitals rule.
4.

5. lim f

=1

lim

use
=0

6. lim f

LHospitals

rule.

If

'b' = a finite value for the limit,


then answer to the original
problem in exponential form is eb.

b f a xf g a xfg = form:
Working rule to evaluate a f .
For this indeterminate form, we write
FG 1 1 IJ
Hg f K
f g=
FG 1 1 IJ and then we use LHospitals
H f gK

g
7. lim f g = lim
1

1
f
,
1
f

then

use

LHospitals rule which will


directly give us the answer.

lim

xa

rule which provide us the required answer.

log f
g
then
or lim
1
1
g
log f

LHospitals rule. If LHospitals

or

and then use

rule applied to one of these give

provides us 1 , 0 , at x = a .
For these indeterminate forms, we are required to

FG log f IJ
H 1g K
g
change f into e

or

g
to lim
, then apply LHospitals rule.
1 f
3. Working rule to evaluate lim f x

4. lim f

which

f
g
or
1
1
g
f

LHospitals rule

For this indeterminate

xa

f
f
f
= lim = lim
g
g
g

1. lim

x=a

definite value (i.e.; not meaningless form) will give us


the answer.

working rules

Remember: 1. Some times LHospitals rule may fail


to lead to any fruitful result. In such situation, we use
the usual method of finding limit of the given
function.
Example: Let

af
af

f x
=
g x

x
1+ x

as x

LHospitals Rule

Here , lim

af
af

f x
= lim
x
g x

x
1+

a f

0
which can be further reduced to F I or F I form.
H 0K H K
f a xf
as a
7. The students must not differentiate
g axf
can be reduced to the indeterminate form 0

=1

1
2

But LHospitals rule gives us

bg
bg

f x
=
g x

af
af

x
f x
=
g x

1
x
1+ x

1 + x2

1+ x
1

x
1+ x

af
af

f x
g x

= original function which limiting value can


not be obtained by LHospitals rule.
2. Generally, functions involving surds should be
avoided for application of LHospitals rule.
For example, to evaluate lim

x0

xa +

x a

3x + a 2 x + a

we first go for rationalization instead of LHospitals

0
rule though it has the form .
0
3. L-Hospital rule is applicable only when the

0
indeterminate expression has the form
or
in

0
the limit.

fraction. The numerator and the denominator have to


be differentiated separately.
8. Differentiation of numerator and denominator is
performed w.r.t. the variable which converges
9. LHospitals rule involves differentiating the
functions. This is why it is useful only when the
functions are easily differentiable.
10. LHospitals rule may be applied repeatedly till we
get a definite value value at x = a or when x a
(i.e.; lim x = a).
N.B.: 1. To evaluate limit of a function whose value
0

at x a is 00 , or 1 (in exponential form), we


are required to take the logarithm first and then
proceed.

F 0 I or FH IK ,
H 0K
F 0 I or
we are required first to change that form into
H 0K
F I by simplification or using any mathematical
H K

x0

F 0I or F I .
to the form
H 0K H K

6. The three indeterminate exponential forms 00,

0
can easily be evaluated by first taking
1 ,
logarithm. By taking logarithm, all these three forms

3 y = x log y = x log x =

log x
1
x

1
log x
lim log y = lim
= lim x
1
1
x0
x0
x0
2
x
x

functions as x a do not assume

5. The two indeterminate forms 0 and


can be easily evaluated by reducing them

e.g. lim x x = 00 form

4. If the indeterminate expressions of the given

manipulation.

599

= lim x = 0 which lim x = e = 1


x0

2.

x0
x

lim + log x = , lim e = +


x +

x0

log 1 = 0 , lim e = 0
x

lim log x =

x +

3. log a m = log b m log a b

= logb m

1
logb a

600

How to Learn Calculus of One Variable

log b m
logb a 3 log b a log a b = 1

log m
( 3 when the base is same in numerator
log a

and in denominator, the base may be omitted for


easiness of computation)
This formula may be remembered by the equality:

m
m
= b
a
a
b

= lim

xa

F 0I .
H 0K

Examples worked out:


1. Evaluate lim sin x
x0
x
Solution: 3 lim

x0

I
K

bg

xa

tan x tan y
x y

3. Evaluate lim

xy

F
H

x y

sec x 0
2
= lim sec x
xy
1

sec x

xy

x=y

= sec y

tan x tan y
= sec 2 y .
x y
-1

4. Evaluate lim

-1

sin x tan x

x0

F
H

I
K

0
x sin a a sin x
Solution: 3 lim
=
form
xa
0
xa

= lim

lim

x sin a a sin x
.
xa

x sin a a sin x
lim
xa
xa

I
K

tan x tan y
0
=
form
0
x y

Solution: 3 lim

= cos 0 = 1

sin x
= lim cos x = 1
x0
x

2. Evaluate lim

= sin a a cos

0
sin x
=
form
0
x

x=0

= sin a a cos

d
tan x tan y
tan x tan y
dx
lim
= lim
d
xy
xy
x y
x y
dx

d
sin x
sin x
cos x
dx
lim
= lim
= lim
d
x0
x0
x0
1
x
x
dx

= cos x

x=a

x sin a a sin x
xa

xy

F
H

sin a a cos x
1

= sin a a cos x

xa

Type 1: Problems based on indeterminate form

x0

lim

(This formula is known as changing into the same


base formula or base changing formula)

lim

d
x sin a a sin x
dx
lim
=
d
xa
xa
dx

-1

Solution: 3 lim

-1

sin x tan x

x0

-1

lim

x0

-1

sin x tan x
x

F 0 formI
H0 K

LHospitals Rule

-1
-1
d
sin x tan x
dx
= lim
3
d
x0
x
dx

1
1 x

= lim

x0

3x

d
dx
= lim

x0

= lim

LM
MN

OP
PQ

2
1+ x
1 x
2
d
3x
dx

e j

2x

2 1 x

2 32

2x

e1 + x j

2 2

6x

1
= lim
x0 6

LM
MM 1
N e1 x j
2 32

OP
+
P
e1 + x j PQ
2

2 32

L
1 M
1
= M
6 M
N e1 x j

2 2

OP
2
+
P
e1 + x j PQ

sin -1 x tan -1 x

x0

x3

1
2

LM sin x x + x OP
6 P
Evaluate lim M
MM x
PP
N
Q
3

x0

x0

x0

= lim

cos x 1 +

x0

5x

x=0

1 2
x
0
2
=
form
0

F
H

F
H

1 2
d
cos x 1 + x
2
dx
= lim
4
d
x0
5x
dx

I
K

I
K

e j
F - sin x + x I = F 0 formI
= lim G
H 20 x JK H 0 K
d
a- sin x + x f
dx
= lim
d
e20 x j
dx
cos x + 1 F 0
I
= lim
=
form
H
K
0
60 x
3

x0

2 2

3 1
1
1+ 2 = =
6 2
6

which lim

5.

F 0 formI
H0 K

x0

1+ x

x0

LM sin x x + x OP
6 P = F 0 formI
Solution: 3 lim M
MM x
PP H 0 K
N
Q
F sin x x + x I
G
J
6 J
lim G
GG x
JJ
H
K
d F
x I
sin x x +
G
J
6K
dx H
= lim
d
ex j
dx
3

e j

601

x0

x0

d
- cos x + x
dx
= lim
2
d
x0
60 x
dx

= lim

x0

F
H

I
K

sin x
0
=
form
120 x
0

602

How to Learn Calculus of One Variable

d
sin x
cos x
cos x
dx
lim
=
= lim
=
x0 d
x 0 120
120
120 x
dx

LM
N

OP
Q

= lim

x0

x=0

cos x
2 cos x + 1 cos x x sin x

LM cos x OP = 1
3
N 3 cos x x sin x Q
L x cos x sin x OP = 1
which lim M
MN x sin x PQ 3
=

x=0

cos 0
1
=
=
120
120

F sin x x + x I
G
6 J = 1
which lim G
JJ 120
GH x
K
F x cos x sin x I
6. Evaluate lim G
H x sin x JK
F x cos x sin x I = F 0 formI
Solution: 3 lim G
H x sin x JK H 0 K
F x cos x sin x I
lim G
H x sin x JK
a

d
x cos x sin x
dx
= lim
2
d
x0
x sin x
dx

1 cos x x sin x cos x


2 x sin x + x cos x

= lim

sin x
2 sin x + x cos x

= lim

cos x
2 cos x + cos x x sin x

= lim

cos x
1
=
3 cos x x sin x
3

x0

2 x sin x + x cos x

a x sin x f

x0

x0

x 2 sin x + x cos x

x0

F0
H0

F
I
H
K
a
f
d
a sin xf
dx
d
b2 sin x + acos xf a xfg
dx

sin x
0
= lim
=
form
x 0 2 sin x + x cos x
0

x0

= lim

1 cos x x sin x cos x

x0

= lim

x0

x0

x0

F x cos x sin x I
GH x sin x JK

x0

= lim

lim

x0

= lim

N.B.: In practice, differentiation of numerator and


denominator is done separately mentally as below
just for saving time.

x0

x0

I
K

form

Type 2: Problems based on indeterminate form

F I .
H K

Examples worked out:


1. Find lim

log x
cot x

Solution: y =

log x

=
form

cot x

x0

F
H

I
K

1
log x
x
lim y = lim
= lim
2
x0
x 0 cot x
x0
cosec x

e sin xj = F 0 formI
= lim
H0 K
x
2

x0

LHospitals Rule

= lim

x0

2. Find lim

x0

2 sin x cos x 0
= =0
1
1

log x
cot x

Solution: y =

xa

xa

log x

F formI
H K

log x

lim y = lim

x0

x0

cot x

= lim

x0

e 2 sin x

= lim

cos x 2 x

3. Find lim

log x a

j = a 2f a0f a1f = 0

j
log a x a f
F formI
=
Solution: y =
K
log ee e j H
log b x a g
lim y = lim
log ee e j
FG 1 IJ
H x aK F I
= lim
F 1 I e = H formK
GH e e JK
xa

log e e

xa

xa

xa

= lim

e +0

=1

IJ
K = FG formIJ
H K

IJ
K

tan x

F x I
H 2 K = F formI
H K
sec x
2

F
I
K
I H
K
2 cos x a sin xf
a sin 2 xf = 0
= lim
2

= lim

x
2

tan x

FG
H

I
K

FG
H

log x
lim

tan x

Solution: lim

2x

log x

= lim 2 sin x cos x


x0

F
H

j = F 0 formI
H0 K

x0

e 1 + x a e

4. Find lim

sin x

log x

FG 1 IJ b2 xg
Hx K
e cosec x j b2 xg
2

= lim

= lim

cot x

x0

F 0 formI
ax af H 0 K

e e

= lim

603

= lim
x

F
H

cos x
0
=
form

0
x
2

5. Find lim sin x log x


x0

Solution: y = sin x log x =

lim y = lim
x 0

= lim

x 0

x 0

log x
cosec x

FG
H

IJ
K

log x

=
form

cosec x

F 1I
H xK

a cosec x cot xf

(by L-Hospital rule)

604

How to Learn Calculus of One Variable

LM sin x OP
MN x cos x PQ
F sin x IJ atan xf
= lim a 1f G
H x K
= a 1f a1f a0f = 0
6. Find lim log
atan 2 xf

x0

= lim

x0

x 0

x0

log tan 2 x
log tan x

x0

x0

F
H

I
K

log tan 2 x

=
form
log tan x

FG 1 sec 2 x 2IJ
H tan 2 x
K [by L- Hospital rule]
= lim
FG 1 sec xIJ
H tan x
K
FG 2 IJ
H sin 2 x cos 2 x K = lim 2 sin x cos x
= lim
sin 2 x cos 2 x
FG 1 IJ
H sin x cos x K
2 sin 2 x F 0
I
= lim
=
H 0 formK
sin 4 x
2

x0

x0

x0

x0

2 cos 2 x 2 2 1 2
= lim
=
=1
x 0 cos 4 x 4
1 4
Type 3: Problems based on the indeterminate form

a0 f .

F 1 I FG 1 IJ
H xK H x K
= lim
FG 1 IJ
H xK
F 1I = 1
= lim sec
H xK
x
3. Determine lim a1 x f tan FG IJ
H2K
F x IJ
Solution: y = b1 x g tan G
H2K
F x IJ = b0 formg
lim y = lim b1 x g tan G
H2K
FG sec x IJ FG IJ
F x IJ
tan G
H 2 K H 2K
H2K
sec

x 1

x1

Examples worked out:

x0

LMby base changing formulaOP


m
MN log m = log
PQ
log a
lim y = lim

1
x
1

x1

1. Determine lim x log x


x0

Solution: y = x log x

FG IJ = lim a xf = 0
H xK
F 1I
2. Determine lim x tan
H xK
F 1I
Solution: y = x tan G J
H xK
F 1I
lim y = lim x tan G J = b0 formg
H xK
F 1I
tan
H x K = F 0 formI
= lim
H0 K
1
= lim

tan x

Solution: y = log tan x tan 2 x =

g b
log x F
I
=
form
H
K
1

F I
H xK
x0

x 0

x 0

lim y = lim x log x = 0 form

= lim

= lim

x 1

1
1 x

= lim

x 1

b1 xg

LHospitals Rule

a1 - xf FH 2 IK F 0 I
form
= lim
=
H
K
xI
0
F
cos
H2K
F I
2 b1 - x g b 1g G J
H 2K
= lim
FG 2 cos x IJ FG sin x IJ FG IJ
H 2 K H 2 K H 2K
2 a1 - x f
a 2f
= lim
= lim
sin x
af acos x f
x 1

F
H
asin x + x cos x f

x0

a x sin x + 2 cos x f
F cot x 1 I
G
xJ
3. Evaluate lim G
JJ
x
GH
K
= lim

x0

x 1

x 1

Solution:

a fa f

Type 4: Problem based on the indeterminate form

lim y = lim

Examples worked out:

= lim

a f .

1. Evaluate lim cosec x cot x

g b
a1 cos x f = F 0 formI
H0 K
sin x

lim y = lim cosec x cot x = form


= lim

x0

x0

x0

x0

x0

x0

x0

x0

x0

x0

= lim

sin x 0
= lim
= =0
x 0 cos x
1
1
2. Evaluate lim F cot xI
Hx
K
F1
I
Solution: y = G cot xJ
Hx
K
F1
I
lim y = lim G cot xJ = b formg
Hx
K
F sin x x cos x IJ = F 0 formI
= lim G
H x sin x K H 0 K

F cot x 1 I
H
xK

FG x cos x sin x IJ FG x IJ
H x
K H sin x K
F x cos x sin x IJ 1 = F 0 formI
= lim G
H x
K H0 K
a x sin xf = lim a sin xf = F 0 formI
= lim
H0 K
3x
3x

Solution: y = (cosec x cot x)


x0

0+0 0
= =0
0+2 2

FG cot x 1 IJ
H
xK
y=

2
2
=
=
-1

x0

x0

x1

x0

I
K

0
x sin x
=
form
0
x cos x + sin x

= lim

605

x0

x0

cos x
1
=
3
3

Type 5: Problems based on indeterminate form


0

, 1 , 0 etc.

Examples worked out:

1. Find lim sec x

x
2

cot x

FG
H

Solution: Let y = (sec x)cot x = 0 form as x

log y = cot x log sec x


lim log y = lim cot x log sec x
x

IJ
K

606

How to Learn Calculus of One Variable

log sec x
sec x tan x sec x
= lim
2

tan x
x
sec x

= lim

x
2

log sin x log x

lim log y = lim


x0

x2

x0

(by LHospitals rule)

tan x

= lim

x
2

x
2

sec x

lim log y = lim

sin x
2
cos x
cos x

= lim

x0

log

F
GG lim
H
x

I
JJ
K

x0

cot x

x0

x0

cos x
1 + sin x

f =1

Solution:

x0

x0

FG sin x IJ
H x K

F sin x IJ
y=G
H x K

x0

I
K

F sin x IJ
log y = log G
H x K

FG 1 IJ FG tan x IJ
H 6K H x K

1
6

cos

y=e

Solution: y = cos

= 1 form as x 0
1
x2

6x

x 0

cos

lim log y =

FG
H

= 00 from as

cos

lim log y = lim cos log cos

sin x
1
= 2 log
x
x

IJ
K

f = cos log cos


lim acos log cos f

log y = log cos

1 sec x

1
lim y = e 6
x 0
6

1
x2

x
tan x

1
x

x 0

1
6

F sin x IJ = e
lim G
H x K
4. Find lim acos f

sec x

F lim yI = log e lim


H K

x0

which lim log y =

F
H

x0

3. Find lim

= lim

log lim y =

lim log y = log e


log

x0

x 0

lim log y = lim


x0

6x

1 = lim

tan 2 x

= lim

Solution: Let y = (1 + sin x)cot x


log y = log (1 + sin x)cot x = cot x log (1 + sin x)
log 1 + sin x
tan x

2x

2x

x tan x

x0

x tan x

x0

2 x tan x

= lim

lim y = 1

2. Find lim 1 + sin x

2x

= lim

x0

as log x a is continuous function for x > 0.


x

x tan x

= lim

y = 0 = log 1

x0

1
x

1
1

tan x
x
lim log y = lim
x0
x0
2x

= lim sin x cos x = 0


x

cot x

1
6

607

LHospitals Rule

= lim

= lim

F
H

I
K

log cos
=
form

sec

LM
N

tan
1
= lim
sec tan
sec
2

UV = 1 log x
W b1 x g
log x
F0 I
lim log y = lim
a1 xf = H 0 formK
RS
T

OP
Q

x 1

log lim y = 0 lim y = e = 1

5. Find lim cos x


x0

x 1

F
H

a f

x 1

7. Find lim x

Solution: y = cos x

1
x2

log y = log cos x


=

= 1 form as x 0

1
x

log cos x

x2

FG 1 log cos xIJ = b0 formg


Hx
K
log cos x F 0
I
= lim
=
form
H
K
0
x
F tan x IJ = F 0 formI
= lim G
H 2x K H0 K

lim log y = lim


x0

x0

x0

= lim

x0

log

sec x
1
=
2
2

F lim yI = 1
H K 2
x0

lim y = e
x0

6. Find lim x 1 x

x 1

1
2

I a f
K

x 1

1
e

x 0

Solution: y = xx = (00 form in the limit)

log y = log xx = x log x = ( 0 form )

f
log x F
I
= lim
F 1 I = H formK
H xK

lim log y = lim x log x


x0

x0

x0

x0

F I
H K

log lim y = 1 which lim y = e =

1
x

1
1
lim x = lim
= 1
x 1 1
x 1
x

log y = log x b1 x g

= lim cos = 0

Solution: y = x b1 x g = 1 form, as x 1

1
x
1

FG IJ (by L-Hospital rule)


H xK
= lim a x f = 0
a
8. Find lim F1 + I
H xK
F aI
Solution: y = G1 + J = e1 form in the limit j
H xK
= lim

x0

x0

FG a IJ
H xK

log y = log 1 +

FG a IJ = b0 formg
H xK

= x log 1 +

LM
N

F
H

lim log y = lim x log 1 +


x

a
x

I OP
KQ

608

How to Learn Calculus of One Variable

F aI
H x K = F 0 formI
F 1I H 0 K
H xK

log 1 +
= lim

F a I
G J
a
F1 + I H x K
H xK
FG 1 IJ
H xK

Solve the simultaneous equations involving


constants.
5. That the given limit is finite may give a relation
between the constants.
Examples worked out:
1. Find the values of a and b such that

= lim

F1 + a I
H xK

= lim

lim

x0

lim y = e

limit = lim

Working rule: To find the values of the constants


from the given finite limit, we adopt the following
working rule:
1. find the limit using LHospitals rule for the given
function in case of indeterminate form.
2. Equate the limit of the given function determined
by using LHospitals rule to the given value of the
limit of the given function and form an equation
involving constants only.
3. Observe whether limit should assume
indeterminate forms

0
0
0
0
, , 0 , , 0 , , 0 , 1 etc or not.
0
0
4. If the limit is , , ... etc., we have a relation
0
between the constants we again use LHospitals rule
and find the limit which is equatied to given limit of
the given function and form another equation
involving constants.

3x

1 + a cos x b cos x ax sin x

x0

x0

Type 6: Problems based on finding the values of


constants occuring in f (x) from the known limiting
value.

1 + a cos x + x a sin x b cos x

[by L-Hospital rule]


This limit = if 1 + a b 0
But limit = 1 (given)
1 + a b = 0

= lim

=1

x 0

Solution: Limit = lim

=a

F I
log lim y = a
H K

x 1 + a cos x b sin x

3x

a f

(i)
=

F 0 formI
H0 K

1 + a b cos x ax sin x

a f

3x

0 a b sin x a sin x + x cos x


6x
using L-Hospital rule]
= lim

x0

[by

LM ab a f sin x a sin x a cos x OP


N 6 x 6 x 6 Q
ab a f a a = ab 3a f
=
= lim

x0

which limit =

ab 3a f

6
But this limit is given to be equal to 1
Equation (ii) and (iii), we get

ab 3a f = 1 ab 3af = 6
6

solving (i) and (iv) 1 + a b = 0


b 3a = 6

1 2a = 6
2a = 6 1

(ii)
(iii)

(iv)

LHospitals Rule

a=

5
2

...(v)
3.

Now, putting a =

5
in (1),
2

x3

5.

1 + a cos x + x a sin x b cos x


3x

[using L Hospital rule]

1+ a b
=
0
but this limit is given to be equal to 1
equating (i) and (ii), we get
1+ a b
=1
0

1+ a b = 01
1 + a b = 0 (inaccurate reasoning)

0
Type 1: Form: and

Exercise 16.1
Evaluate (using LHospitals rule)

x 2

3x + 5x 2

x x2
2

x + x6
a + 2x

7. lim

(i)
(ii)

1.

2
5
3
6.
7.
3 3
3
5
Type: continued
5.

Problems based on a combination of algebraic and


trigonometric functions
Exercise 16.2
Evaluate (using L-Hospital rule)
x 0

1 cos x
x

2. lim

sin x cos x

x
4

3. lim

tan 2 x x
3x sin x

x 4

3n + 4n + 5
2

4n + 6n 7

1
3
3
2.
3. 4. 6
3
7
4

; a>0

Answers:

x 0

2. lim

3x

3a + x 2 x

xa

3x + 4 x + 5
7 x 5x + 7

x 3x + 4

6. lim

1. lim

x 9
x3

x0

Problems based on algebraic functions

1. lim

5
3
,b =
2
2
Note: The following argument to have 1 + a b = 0 is
wrong.

b1 + xg 1
lim

Thus, a =

x0

je

5x + 6 x 3 x + 2

x2

4. lim

5
3
=
2
2

Limit = lim

5
1 b = 0
2
b =1

ex
lim

609

4. lim

x 0

cos x 1
x

610

How to Learn Calculus of One Variable

5. lim

tan x x

x 0

x 0

20. lim

9. lim

x sin sin x
x

10. lim

22. lim

cos y
y1

23. lim

sec 2 x 2 tan x
x
1 + cos 4 x

24. lim

x + sin x
x sin x

2 x sin

11. lim

x 0

y 1

tan x sin x

x 0

2 x + tan

1
1

25. lim

x 0

26. lim

tan x
x

x sin x

x 0

16. lim

x 0

tan x x

x 0

1
1
1
2 3. 2 4. 2 5. 3

6.

1
1
7.
8. 2 9. sin cos
6
2

10.

1
1
1
11.
12. 1 13. 1 14.
2
3
2

15.

1
1
1
16.
17.
18.
4 2
6
3

x tan x

19. 2 a sin a + a cos a 20. 2

2 cos sin
2

18.

sin x

1
2.
2

b4 g
x b1 + a cos x g b sin x
lim
, ba > b 1g

17. lim

1.

sec x 2 tan x
14. lim
x 4
1 + cos 4 x

15. lim

sin x

Answers:

x 0

x 4

x 0

sin x
12. lim
x 0
x

13. lim

x
1 cos x

x 0

cos x
2x

8. lim

x 0

21. lim
x 2

sin x

b g

sin a + h a 2 sin a

1 cos 2 x

x 0

tan x sin x

x 0

h0

x sin x

6. lim

7. lim

19.

ba + hg
lim

x3

21.

1
22. 23.
8
2

24.

1+
25. 0 26. 0
1

LHospitals Rule

Type 1: Continued.
Problems based on a combination of algebraic,
exponential, logarithmic, trigonometric or inverse
trigonometric functions
Exercise 16.3
Evaluate (using LHospitals rule)

b g

11. xlim
0

x log 1 + x
1 cos x

x2

x 0

x 0

13. xlim
0

a f

3. lim

x 0

e 2 cos x + e
x sin x
x

Hint: lim

x 0

e 2 cos x + e

x 0

log x + h log x
4. lim
h0
h

x+4 5
x 1

5. lim

h 1

log x
x 1

6. lim
x 1

7. lim

a 1
x

8. lim

x 0

a b
x

x 0

x 0

16. lim

e e
sin x

x 1e
17. lim

x 0

cos x 1

Answers:
1.

1
3
1
1
2.
3. 2 4.
5.
6. 1
2 5
2
2
x

7. log a 8. log

F aI
H bK

9. 2 k 10. 1 11. 2

1
1
2
14.
15. 2 16. 2 17.
2
120

Problems based on only on the form:

1 cos x
x

10. lim

1 3
x
6

e + e 2 cos x
x sin x

x 0

Type 1: Continued

log 1 + k x
9. lim

F x IJ
G
H sin x K

15. lim

x 0

12. 2 13.

x 0

x 0

e 2 cos x + e
x sin x

lim

14. lim

x cos x log 1 + x

sin x x +

x cos x log 1 + x

2. xlim
0

x e log 1 + x

12. lim

x e x log 1 + x

1. lim

e e
x

x
2

Exercise 16.4
Evaluate
1. lim

x a

log x a

log e e

611

612

How to Learn Calculus of One Variable

F
H

log x

2. lim
x 2

3. lim

x 0

I
K

Type 2: Continued

tan x
log x
cot x

Exercise 16.6

Evaluate

x 2

x 0

3x + 1

2.
x

5. lim

x
Answers:

3.

1. 1 2. 0 3. 0 4.

1
5.
3

Type 2: Form: 0 and

Problems based on the form: 0


Exercise 16.5
Evaluate

1. lim 1 x tan
x 1

FG x IJ
H2K

2 x

6. lim x e

x 0

x 1

5. lim sec tan


2

6. lim cosec cot x


x 0

LM a cot x OP
Nx aQ
L 1 x OP FG x 2 IJ
8. lim M
N log x x 1Q H x K
L 1 2 x OP
9. lim M
N log 2 x 2 x 1 Q
L 2 1 OP
10. lim M
MN x 1 x 1 PQ
L
O
11. lim M x x 1P
N
Q
L
F 1 I OP
12. lim M x x log 1 +
H xKQ
N
x 21

5. lim x e

x 1

3. lim x tan

x 1

x 0

x 0

F 1I
H xK
lim a1 + cos x f cot

4.

x 0

7. lim

2. lim x log x

4.

LM 1 1 OP
MN x sin x PQ
L 1 1 OP
lim M
MN sin x x PQ
L1
O
lim M cot x P
Nx
Q
L x 1 OP
lim M
N x 1 log x Q

1. lim

4. lim

Problems based on the form:

x 1

Answers:

2
1
1.
2. 0 3. 1 4.
5. 0 6. 0

Answers:
1.

1
1
1
2.
3. 0 4.
5. 0 6. 0
3
2
3

LHospitals Rule

7. 0 8.

1
1
1
1
9.
10.
11. 0 12.
2
2
2
2

Problems based on the form : 0 , 0 , 0 0


(exponential form of 0 and )

7.
8.

Exercise 16.7
9.
Evaluate

tan x

2.
3.
4.
5.

11.

x 0

1
x

12.

1
log 1 x

13.

x 1

1
x

cot x

x 2

cos x

x 2

af
lim a1 + sin x f
lim acosec x f
lim acos x + a sin b x f
F xI
lim cos
H mK
F tan x IJ
lim G
H xK
a f
L
3x 4 O
lim M
N 3x + 2 PQ
L x 2 x + 1 OP
lim M
MN x 4 x + 2 PQ
lim acos x f
1
x 1

x 1

1
log x

1
x

x 0

10. lim x

1. xlim
0

cot x

x 0

Type 3: Problems based on the form: 0 , , 0 , 1

F 1I
H xK
lim acosec x f
lim a1 + x f
a
lim e1 x j
lim F a 1I
H K

a f
F tan x IJ
lim G
H x K
lim asec x f
lim a tan x f

6. lim cos x

1
x

x 0

sin x

x 0

x 0

14.

Answers:

1
2

1
1. 1 2.
3. 1 4. e 5. log a
e
Type 3: Continued

15.

Problems based on the form: 1 ,

x 0

x +1
3

16.

Exercise 16.8

17.

Evaluate

a f
lim acot x f
F 2 x + 1IJ
lim G
H x +1K
F 1I
lim cos
H xK
F sin x IJ
lim G
H xK

1. lim sin x

tan x

x 2

2.

3.

18.

1
log x

x 0

x 0

1
x

1
x

5.

x 0

cot x

x 2

a f
lim acot x f
lim acos x f
F sin x IJ
lim G
H xK

19. lim log x

sin x

x 1

20.

4.

21.
22.

sin x

x 0

cot x

x 0

x 0

1
2

1
x

613

614

How to Learn Calculus of One Variable

a
f
F1I
lim G J
Hx K
lim alog x f

23. lim 1 + sin x

1
x

2. If lim

x 0

x 0

tan x

24.
25.

3. If lim

x 0

26. lim a + x
x 0

x 0

Answers:
1
1. 1 2.
3. e 4. 1 5. 1 6. e 12 7. 1 8. 1 9. 1
e
1
ab
3
10. e 11. e 12. 1 13. e 14. 1 15. e
23

16. e
22. e

16

17. e2 18. 1 19. 1 20. 1 21. 1


23. e 24. 1 25. 1 26. a e

Type 4: Problems based on finding the values of the


constants from the given limit of the function of
independent variable.

= lim

x0

Evaluate
1. If lim

x 0

tan x a sin x
x

be finite, find a.

= 1 , find a relation

1
1+ a +b
2
But this limit = 1 (given)
=

4. If lim

F
H

I
K

x 1 + a cos x + b sin x
0
=
form
2x
0

1 + a cos x ax sin x + b cos x


2

sin 2 x + a sin x

x 0

...(i)
...(ii)

be finite, find the value of

a and the limit.


5. Find the values of a, b and c so that
x

Exercise 16.9

= 8 , find a and b.

2x

Hint: Limit = lim

1
x

between a and b.

1
1 log x

2x

x 1 + a cos x + b sin x

x 0

x 0

ae + 3e
x

lim

x 0

ae b cos x + ce
x sin x

=2

Answers:
1. a = 1 2. a = 2 and b = 5 3. a + b = 1
4. a = 2 and limit = 1 5. a = 1, b = 2 and c = 1

615

Evaluation of Derivatives for Particular Arguments

16
Evaluation of Derivatives for
Particular Arguments
Evaluation means to find the value of or to fix the
value of a quantity, e.g.
1. to evaluate 8 + 3 4 means to reduce it to 7.
2. to evaluate x 2 + 2 x + 2 for x = 3 means to
replace x by 3 and collect the result which is 17.
2

3. to evaluate lim x means to find the limit of x 2


x2

as x 2 which is 4.
4. to evaluate an integral
out integration.

z f axf dx means to carry


z f a x f dx means to

5. to evaluate a definite integral

carry out integration and then substitute the limits of


integration a and b. Now we shall learn the process
of finding the value of a derived function at a given
number or point x = a
Defination of a derived function of an independent
variable x: The derivative of a function f (x) is a
function of x represented as f x which is derived
from the original function f (x) through a limiting
process or various techniques. In this sense, the
derivative f x is called the derived function and
the graph representing it is called the derived graph.
Notation for the value of the derived function
f x at x = a.

af

af

af

af

Just as the value of f x for (or, at) x = a is


symbolised as f x x = a = f a , a D f
The value of a derivative (or, derived function)
f x for ( or, at) x = a is symbolised as

af

af

af

af

af

f x

x=a

LM dy OP
N dx Q

x=a

F dy I
H dx K

x=a

af

Question: How to find f x

af

a f

= f a , a D f

x=a

Answer: To find the derivative values ( or, the values


of the derivative) at x = a (i.e. f a ), we must first
differentiate the given function f x by using the
formulas to get the derived function (or, derivative)
f x and then put x = a in f x if 'a' belongs to the
domain of f x obtained by using general formula
or, in other words, to find f a , we adopt the
following working rule.
1. Find the derived function f x by using the
formulas
2. Put x = a in f x provided f a is not undefined

af
af

af

af
af
af
af

af

F i.e; a ,
H 0

af
0
I
, etc. and lastly simplify to get the
K
0

required answer.

af
af

3. If f a cannot be obtained thus then we should


find f a directly from the defination [ provided
f a is defined i.e. a D f ]

af

bg

f a = lim

h0

af
f ba + hg f ba g
bif existsg.

h
Note: The process of finding f a by differenting a
function y = f x with the help of known formulas
and then putting x = a in f x is fruitful and gives
right answer when f x is continuous at x = a.

af

af

af

af

616

Let

How to Learn Calculus of One Variable

R|
f a xf = S
|T

x sin

F 1I , x 0
H xK

, x=0

F 1 I FG 1 IJ + sin F 1 I a2 xf
H xK H x K H xK
F 1 I + 2 x sin F 1 I and lim f a xf
= cos
H xK
H xK
F F 1 I + 2 x sin F 1 I IJ does not exist
= lim G cos
H xKK
H H xK
F 1 I does not exist ) , so f a x f is
(because lim cos
H xK
af

Now f x = x cos

x0

x0

x0

not continuous at x = 0. It is not possible to find


f 0 by putting x = 0 in f x
However

af

af

bg

b g b g = lim

f 0 lim =

f h f 0

h0

h 2 sin

h0

FG 1 IJ 0
H hK

af

af

Where N x means a function of x in Nr and


D x means a function of x in Dr

af
L d f a xf OP
6. M
N dx Q

= The value of the derivative at a


x=x

a f

general point x D f
7.

LM d f a x f OP
N dx Q

= The value of the derivative, not at


x=a

a general point x, but at some definite point namely

a f

a D f

a f gf aaxxff where F axf and g a xf are

8. Letting F x =

any two differentiable functions, the derivative of


such a function is another function of x as

a f a f a f a f and this derivative exists


lg a x fq

g x f x f x g x

af

a f a f has no derivative at the


a f point where g a x f = 0 , e.g.
(i) f a x f = x 1 f a1f does not exist
(ii) f a x f = 1 + sin x f a f does not exist
10. If F b x g = f b x g g b x g then F ba g may exist
af
even if f ba g or g ba g does not exist.
LM d b f a xf f a xfgOP = LM d f a xf OP
x
N dx
Q N dx Q
cos x
e.g.: f b x g = x
2
L d f a xf OP = f aaf f aaf
M
F + hI cos F h + I
N dx Q

F +I = lim + H 2 K 2 H 2 K
f
LM d b f a xf f a xfgOP
H2 K
h
N dx
Q

F I=0
= lim cos h +
H 2K
= f ba g f ba g + f ba g f ba g
F I
h cos h +
LM d f b x g OP = LM d N b xg OP
H 2K = 0
I
F
and f
MN dx f b xg PQ MN dx D b xg PQ
H 2 K = lim
h

Remember:
1. To substitute specific values ( or, particular values)
for x = 1, 2, 3, . . . a etc. into the original function f x
before differentiation is not permissible.
2. Original (or, given function) is also termed as
primitive function denoted as f x without dash.

9. In general f x = g x

x=a

x=a

5.

af af

at x = 0 provided that denominator is not zero at x = 0


i.e., g 0 0

h 0

4.

af

D a

F 1I = 0
lim h sin
H hK

3.

af

D a N a N a D a

h0

x=a

h0

1
2

x=a

x=a

h 0

617

Evaluation of Derivatives for Particular Arguments

F + I = f F I = 0 f F I = 0
H2 K H2 K
H 2K
11. lim f b x g = f b x g is not differentiable at
x = a, where a is the root of f b x g = 0 f ba g

dy
= 5cos x
dx

dy

= 5 cos = 5 1 = 5
2
dx x =

Hence, f

xa

does not exist.

bg b

e.g.: f x = 2 x 1

bg

1
2

lim + f x = lim +
x

di
1
2

di

2x 1

1
2

= +

1
f (x) is not differentiable at x =
2
1
f
does not exist.
2
Type 1: To find the value of the derived function
(obtained by using the power, sum, difference,
product or quatient rule of d.c.) at the indicated point.

FG IJ
HK

Examples worked out:

1
dy
1. Find
at x =
if (or, when or, for)
2
dx
y = 3x + 2 x
Solution: 3 y = 3 x 2 + 2 x
dy

= 6x + 2
dx
2

LM dy OP
N dx Q

1
x=
2

= 6x + 2

x=

1
2

1
+2= 3+ 2=5
2
dy
2. Find
at x = 2 when y = x 3 + 2 x + 3
dx
3
Solution: 3 y = x + 2 x + 3
dy

= 3x 2 + 2
dx
dy

= 3x 2 + 2
= 3 4 + 2 = 14
x=2
dx x = 2
=6

dy
at x = when y = 5sin x
2
dx
Solution: 3 y = 5sin x

dy
4. Find
at x = t when y = 2 + tan x
dx
Solution: 3 y = 2 + tan x
dy
2

= 0 + sec x
dx
dy
2

= sec t
dx x = t

dy
2
5. Find
at x =
when y = x + cos x +
2
dx
1
log x
2
1
2
Solution: 3 y = x + cos x + log x
2
dy
1 1

= 2 x sin x + ; x > 0
dx
2 x

LM OP
N Q

LM dy OP = 2 sin + 1 2
2
2 2
N dx Q
L dy O = 1 + 1
M P

N dx Q
F 3I
6. If f a x f = 5 sin x 3 cos x , find f G 2 J
H K
Solution: 3 f a x f = 5 sin x 3 cos x
d f b xg
5
3
8

=
+
=
; x < 1.

x=

x=

dx

LM OP
N Q

3. Find

LM OP
N Q

LM d f a xf OP
N dx Q
8
43
4

1 x

v3
x=
2

8
1
4

1 x

3
4

8 8 2
= = 16
1 1 1
2

a f LMNi.e. d dxf a xf OPQ

7. Find the derivative of f x

1 x

1
x 2 at x = 1 and x = 3.

618

How to Learn Calculus of One Variable

af

Solution: 3 f x =

af b
= 1 b x 2g

1
x2

f x = 1 x 2
2

11

af

d x2
dx

, x 2.

af

d
n
f x =
dx
derivative of a power function rule to a radical or to
the reciprocal of a function, it is useful to put the
radical or reciprocal function into a power function
whose index is negative or fraction as we require. e.g.
1
5

1.
5 should be put into the form x 2
x
Note: While applying the result

1
should be put into the form x + a
x+a
then apply the power function
2.

OP
PQ = n f a x f

n 1

af

Solution: 3 f x =

af
2

1
a2 2

a f 12 d x + a i d d x dx+ a i
1
f a xf =
2x
f x =

x
x + a2
2

1
2 2 1

x2 + a2

af

10. Find

2x 3

2 x 3 at x = 2

1
2

F x 3I
2K
2x 3 H
1

2=

1
=
223

1
if a > 0
v2

2x 3 = 2x 3
1

f 2 =

1
1
= =1
43 1

x + cos x
dy
at x = 0 when y =
1 + cos x
dx

Solution: 3 N = x + cos x

N = 1 sin x
and D = 1 + cos x
D = 0 sin x = sin x

dy DN ND
=
2
dx
D
=

rule

x 2 + a 2 at the

af

f x =

and

af

x +a = x +
2

df x

dx

8. Find the derivative of f x =


point x = a

af

Solution: 3 f x =

a +a

af

L d b f a x fg
M
MN dx

9. Find the derivative of f x =

b x 2g
1
1
1
= = 1
=
Now, f a1f =
b+ 1 2g b 1g 1
1
1
f b3g =
=
= 1
b3 2g 1
2

Now, f a =

a1+ cos xf a1 sin xf a x + cos xf a0 sin xf


a1+ cos xf
2

dy 1 + cos x sin x + x sin x


=
2
dx
1 + cos x

LM dy OP
N dx Q

=
x=0

1+1

b1 + 1g

1
2

e j
x

log x e
dy
11. Find
at x = 1 when y =
x
dx
x+e
Solution: y =

e j

log x e x
x + ex

e j = log x + log e

Now, N = log x e

= log x + x log e e = log x + x

Evaluation of Derivatives for Particular Arguments

N =

1
+1
x

Again, D = x + e x

D = 1 + e x

dy
DN ND
=
dx
D2

F 1 + 1I log e x e j e1+ e j
x +e j
e
Hx K
dy

=
dx
e1+ e j
x

dy

=
dx

1+

e1+ e j

2+
dy
at x = 1 when y =
dx
2
2+

af

Solution: 3 f x =

FG
H

IJ
xK

, x 0.

bg
f a0 + hf f a0f
f a0f = lim
h

3x 3

we find f 0 using definition of derivative at x = 0.

h0

h 0

2
3x 3

a0 + hf
= lim

x + log e x log 2

1
x3

ex

1 dy
1
1
1

+ x ex
y dx 2 + x
e
2 x
1

1
3

Since 0 does not belong to the domain of

ex

log y = log 2 +

x =1

bg

Taking log ,

e 2j
L1 e2 x j + 2 x b4 x g + e2 + x j OP
dy

= yM
MMN 2 x e2 + x j e2 x j PPQ
dx
L 4 + 2 x b4 xg OP
dy L 2 + x O

=M
e M
P
dx MN 2 x PQ
MN 2vx b4 xg PQ
L dy O = 2 + 1 e LM 10 OP
Hence, M P
N dx Q 2 1 MN 2 1 b4 1g PQ
L 10 OP = 5e
3
= eM
1
N 2 3Q
13. If f ( x ) = x , find f a0 f

x =1

Solution: 3 y =

f x =

L dy O = 1+ e + 1+ e 0 1 e 0 e
Hence, M P
N dx Q
a1+ ef
b1 + eg = 1
=
b1 + eg b1 + eg
12. Find

x
x
x
e
+ x + e log x x e log x e x
x
x 2

1 dy
1
1

=
+ 1+
y dx 2 x 2 + x
2 x 2

x 2

x
x
e
1+
+ x + e log x + x 1 + e
dy
x

=
x 2
dx
1+ e

619

h 0

1
3

lim

1
3

h
= lim h 3 = lim
h 0
h h0

1
3

Remember: The use and meaning of the expression


a
must become clear in the mind of ours.
0

620

How to Learn Calculus of One Variable

af

1. If a function f x is an infinitesimal as x a

F i.e; if
H

a f IK

lim f x = 0

x a

1
then the function f x is

af

infinitely large which is symbolically represented as


1
and we say that f x has no limit or infinite limit

af

as x a i.e., lim

xa

af

1
= (no limit or infinite
f x

af

limit) if lim f x = 0 .
xa

a
is undefined since division by
0
a
zero is not allowed in mathematics. The expression
0
a
always should be written as
= undefined for the
0
a
value of the expression for x = 0 which provides
x
us the following sense.
If a function f x is zero for any value of x, then
1
the function f x is undefined or meaningless for
that value x at which f x = 0

af

a
Thus = undefined should be used in the sense
0
of the actual value (or, simply value) of the function
f x at a point at which f x = 0 and

af

lim

xa

af

af

a
a
= = if lim f x = 0 in the sense
xa
0
f x

af

1
of limiting value of f x as x a , e.g.,

af

1. The symbols , lim tan x = +


x

lim tan x =
x

+
2

= undefined.
2
Type 2: To find the value of the derivative of a
composite of two or more than two functions at the
indicated point.
tan x

x=
2

af

af
af

af

af

Examples worked out:

2. The expression

af
af

Working rule:
1. Find the differential coefficient (i.e. d.c.) of a
composite of two or more than two functions by using
the chain rule.
2. Put x = a in the derived function and see whether
f a is defined or undefined.
3. If f a is defined, then simplify to get the required
answer and if f a is undefined, then we should
find f a directly from the defination provided f a
is defined.

af
a f
af
log log x
; b x > 1g
Solution: f b x g = log blog x g =
log
1. If f x = log x log x , find f x at x = e
e

af

f x =

log log x
log x

for x >1 )
Now,

af

af

(3 f x is defined only

af

df x
dy
=
= f x
dx
dx

a f

1
1
1
log x log log x
log x x
x

alog xf

1 1
log log x
x
= x
2
log x

a f

1 log log x

a f
L dy O = LM1 log log x OP
Hence, M P
N dx Q MN x alog xf PQ
2

x log x

x =e

= tan

1 log log e

a f

e log e
1 0

bg

e 1

1
e

x =e

Evaluation of Derivatives for Particular Arguments

af
a f g a x f into

Note: Derivative of a function of the form


log f a x f g x is alwasy obtained by first changing

log f

af
af

log e g x
log e f x

and then we use the

quotient rule to find the d.c.


2. If y = a +
x

1+ x
, find d.c. at x = 0
1 x

Solution: 3 y = a x +

Now, differentiating both sides w.r.t. x, we obtain

dy d a x
d
=
+
dx
dx
dx

Now,

F
GH

1+ x
1 x

I
JK

L
1 0 M
MN 2

af

af

LM
N

OP
LM
PQ
MN
e 1 x j
L
1 O
1+ 0 M
P
1 + 0 PQ
MN 2
e 1 0 j

1+ x
1 x

OP
Q

a fOPP
Q

1
1
1+ x
1
1+ x
2 1 x
2

a fOPP
Q;

1
1
1 0

af

Remember:

af

af

x=0

x=0

bg

bg

f x =

afb af g

F 3I
H 4K
b

x =0

+1

cos x
d
cos x =
sin x , cos x 0
cos x
dx

FG IJ
H K

3
f
=
4

F 3 I
H 4 K FG sin 3 IJ
H 4K
3
cos

cos

1
1
v2
=

1
v2

v2

FG
H

IJ
K

1
v2

af

af
af

af

f x
d
f x ; f x 0
f x =
dx
f x

2. If f x = cos x sin x , find f

LM dy OP = a log a
N dx Q
L dy O = a log a + 1
M P
N dx Q
L dy O = log a + 1
M P
N dx Q
x =0

af

Solution: f x = cos x

1 1
+
1
= 2 22 = = 1
1
1

bg

af

1. If f x = cos x , find f

for x = 0

Hence,

af

Examples worked out:

d
d ax
= a x log a and
dx
dx

L
1 x M
MN 2

Type 3: To find the value of d.c of mod of a function


at x = a working rule:
1. Find d.c of f x
2. Put x = a in the d.c of f x and see whether
f a is defined or undefined.
3. If f a is defined, then put x = a in f x to get
the required answer and if f a is not obtained thus,
then we should find f a directly from the defination.
Provided f a is defined.

af

1+ x
1 x

621

af

F I
H 2K

Solution: 3 f x = cos x sin x


x sin x
d

a f acos
acos x sin x f
cos x sin x f dx
cos x sin x
f b x g =
bcos x sing b sin x cos xg

f x =

for x n +

622

How to Learn Calculus of One Variable

sin

2
2
f
=
sin cos

2
2
2
cos sin
2
2

F I
H K F
H

LM
I N
K

cos

b g b
1
=
b 1g b 1g

01
1 0
0 1

Solution: 3 y = x

dy
d
=
dx dx
=

LM dy OP
N dx Q

x=3

d x

dx

LM OP
N Q

af
=x

4 3
3

bg

sin x

Examples worked out:

b g b

1. Find d.c. of y from xy + 4 = 0 , at x , y = 2 , 2

dx
dy
+x
=0
dx
dx
dy
y+x
=0
dx
dy
y

=
dx
x

cos x for x n

This formula can not give us the value of

bg

f x at x = n and so at x = 0.
Hence we find this value by definition:

bg bg

f h f 0
f 0 = lim
h 0
h

sin x

b g

Solution: xy + 4 = 0

,x0

bg

x = a
y = b

b g

d 2
dx

in

dy
dy
to find dx
dx

Remember: In the derived function of y in


f x , y = c , x - coordinate and y - coordinate both
may be provided at which we require the value of
derived function obtained from the given implicit
function f x , y = c

4. f x = sin 3 x , find f x at x = 0

bg

x=a
y=b

4 x +2

=64
=2

bg

b g

dy
from f x , y = c
dx

=23

Solution: f x =

1. Find
2. Put

x
dx
=
4
+0 3 x
dx
x
= 2x 4

sin h
h

Working rule:

4 x +2

dx

h0

d x

= lim sin 2 h

b g

dy
at x = 3
4 x + 2 , find
dx

h 0

sin 3h
h

= 0.1
=0
Type 4: To determine the value of d.c of implicit
function defined by f x , y = c at x = a ; y = b

=1
3. If y = x

OP
Q

= lim

LM dy OP
N dx Qb

2. Find

g b

x , y = 2, 2

dy
for
dx

Solution: y +

y=

LM y OP
N x Qb

2, 2

x = y + a at (a, 0)
a =

d a
dy d x
=

dx
dx
x

b g =1

2
2

Evaluation of Derivatives for Particular Arguments

dy
1
=
, x > 0.
dx 2 x

LM dy OP
N dx Qb

g b g

x , y = a, 0

LM 1 OP
MN 2 x PQb

Solution: Given function is x 2 + xy y 2 = 1


Now, differentiating both sides of the equation
w.r.t x, we get,

a,0

b g

dy
for x 2 = y at the point (1, 1)
dx
Solution: y = x 2
3. Find

dy

= 2x
dx

LM dy OP
N dx Qb

= 2x

1, 1

b 1, 1g

= 2 1= 2

dy
when x 2 xy + y 2 = 3 and find its
dx
value at 1 , 1
4. Find

Solution: x xy + y 2 = 3
Now, differentiating both sides of the equation
w.r.t x, we get,
2

dy
dy
2x y x
+ 2y
=0
dx
dx

dy
2y x = y 2x
dx
dy y 2 x

=
dx 2 y x

Now, the value of

dy
at
dx

1 21
=
b1 , 1g = LMN dydx OPQ
b g b g 2 1 1
L dy O = 3 = 1
M P
N dx Qb g 3
x , y = 1 , 1

1, 1

dy
5. Find
when x 2 + xy y 2 = 1 at (2, 3) and at
dx
(1, 2)

dy
dy
2y
=0
dx
dx

2x + y + x

2 a

N.B.: Since y does not appear in the derived function


of f x , y = c , there is no question of putting
y = 0 in the derived function.

623

dy
x 2y = 2 x y
dx
dy 2 x y
2x + y

=
=
dx
x 2y
2y x

LM dy OP
N dx Q b

2, 3

22+3 7
=
232 4

(1, 2) does not satisfy the given equation and so


dy
the question of finding
at (1, 2) does not arise.
dx
Type 5: To find the value of differential coefficient
of a given function w.r.t. another given function at
x=a
Working rule: To find the value of differential
coefficient of a given function w.r.t. another given
function at a point x = a, we adopt the following
working rule:

af
af

af
af

af
af

d
f1 x
d f1 x
f x
= dx
= 1
1.
where f1 x =
d
d f2 x
f2 x
f2 x
dx
the function whose differential coefficient is required
and f2 x = the function with respect to which
differential coefficient is required.

af

af

2. Put x = a in

af
af

f1 x
f 2 x

Examples worked out:


1. Find d.c. of sec

1
2x 1

af

Solution: Let f 1 x = sec

af

and f 2 x = 1 x 2

w.r.t 1 x at x =

F 1 I
GH 2 x 1JK
2

1
2

624

How to Learn Calculus of One Variable

F
GH

af

d f1 x
1
1
d
sec
=
2
dx
dx
2x 1

F 1 I F 1 I
GH 2 x 1 JK GH 2 x 1JK
2

e2 x 1j
2

4x2 4 x4

e2 x

F
GH

1
d
dx 2 x 2 1

I
JK

4x

4x 2 4 x 4

af

b g

af
af

af
af

2 1 x2
2
x

LM d f a xf OP
N d f axf Q

LM 2 OP
NxQ

parameter t or , = a .
Working rule:
1. Let x = x t and y = y t

af

2. Find

dx
dy
and
dt
dt

af

y = a 1 cos , Find

dy
dx

...(1)
...(2)

(1)

dy
= a 0 sin = a sin
d

...(3)

(2)

dx
= a 1 cos
d

...(4)

dy
sin
d
a sin
dy

=
=
=
dx
1 cos
a 1 cos
dx
d

L dy O
Hence, M P
N dx Q

1 x2

2
=4
1
1
1
2
x=
x=
2
2
2
Type 6: To find the value of differential coefficient of
parametric equations at a particular value of the given

.
2

when =

Examples worked out:

x = a sin

d
f1 x
4 1 x2
d f1 x
= dx
=
Hence,
d
d f2 x
4x 2 4x 4
f x
dx 2
=

x=a

Solution: y = a 1 cos

4x

1 x

LM OP
N Q

dy
4. Find dx

1 2x
d f2 x
=
dx
2 1 x2
=

dy
dy
dx
dy
= dt
3. Divide
by
to have
dx
dt
dt
dx
dt

1. If x = a sin

I
JK

2. If

=
2

g b

sin

1 cos

x = a 1 cos t

dy
at t =
dx
2

Solution: y = a t + sin t

f
dy
= a a1 + cos t f
(1)
dt

1
=1
1

y = a t + sin t

x = a 1 cos t

(2)

dx
= a 0 + sin t
dt

Find

...(1)
...(2)
...(3)

Evaluation of Derivatives for Particular Arguments

= a sin t

dy a 1 + cos t

=
dx
a sin t

L dy O
Hence, M P
N dx Q

f
FG
H

a sin
2

IJ
K , t n .

a 1+ 0 a
=
= =1
a 1
a

Solution: y = a sin

...(1)

x = a cos

...(2)

dy
2
= 3 a sin cos
d

dx
= 3 a cos2 sin
(2)
d

...(3)

= 3a sin cos

...(4)

dy
3a sin 2 cos
n
=
= tan ,
dx 3a sin cos2
2 .

Hence,

LM dy OP
N dx Q

x=
4

LM
N

= tan

dx
2
= 3 sin 1 2 cos
d

OP
Q

dx
= 3 sin cos 2
d

when sin 0 , cos2 0 .


Hence we cannot put

LM dy OP
N dx Q

= cot
=

x = 3 cos 2 cos

dy
2
= 3 cos 6 sin cos
d

to have

= 1 (an erroneous process since


4

for = , cos2 0 )
4
However applying LHospitals rule we have

FG dy IJ
H dx K

= lim

dy
when = .
4
dx

Solution: y = 3 sin 2 sin

= lim

y
x

b g FGH 4 IJK
F I
x bg x G J
H 4K
y y

...(4)

dy 3 cos cos 2
=
= cot
dx 3 sin cos 2

= 1
4

(1)

dx
2
= 3 sin 6 cos sin
d

4. If x = 3 cos 2 cos , y = 3 sin 2 sin


Find

...(3)

= 3 sin 2 cos 1

dy
3
3
3. If x = a cos y = a sin Find
when
dx

=
4

(1)

dy
= 3 cos cos 2
d

(2)

a f

= 3 cos 1 2 sin

a 1 + cos

t=
2

...(4)

625

= lim

...(1)
...(2)

b g = lim cot = 1
x b g
y

Type 7: To determine the value of d.c of a function


found by taking first logarithm at a point x = a

626

How to Learn Calculus of One Variable

LM
MN

Working rule:
1. APPLY the rule GLAD to find derived function.
2. Put x = a in the derived function .

Remember: Derivative of a function raised to the


g x
power another function as y = f x a f is found
only with the help of logarithmic differentiation.

Examples worked out:

3 y=u+v

af

L x OP
+ M tan
N 4Q
a fO
Solution: Let u = LM2
N QP

L OP
1. If y = M2
N Q

log 2 x 2 x

u = x2x 3 a

dy
Find
at x = 1
dx
...(1)

= N

OP a
Q

1 du
1

= 2 x + 1 log x = 2 1 + log x
u dx
x

du
2x
2x
= 2 x 1 + log x 3 u = x
dx

LM du OP
N dx Q

L x OP
Again, let v = M tan
N 4Q

4
x

LM
N

x
4
log tan
x
4

FG
H

IJ
K

=
x =1

=2+2
=4

LM du OP
N dx Q

+
x =1

LM dv OP
N dx Q

x =1

af

af
f axf = 2x 4
f axf = 2 2x 4 = 2

Solution: f x = x 2 4 x + 3

2x = 2 + 4
x=

OP
Q

af

6
=3
2

2
2. If f x = x 3x + 2 Find the value of x for
which the derivative is zero.

af

2
Solution: 3 f x = x 3x + 4
f x = 2x 3

1 dv

v dx
4

LM dy OP
N dx Q

af

Now, differentiating both sides w.r.t. x , we get

dy du dv
=
+
dx dx dx

2
1. If f x = x 4 x + 3 , Find the value of x for
which the derivative is 2.

Now, taking log of both sides, we get

log v =

Examples worked out:

OP LM1 cot sec


Q N1 4

af
af

= 2 1 1 + log 1 = 2 1 1 + 0 = 2
x =1

Working rule:
1. Find d.c of the given function f x
2. Write f x , i.e. d.c. of the given function = The
given constant
3. Solve f x = the given constant. This provides
us the required value or values of x.

2x

LM
N

LM
N

= tan

Type 8: To find the value of x for which d.c of a


function is a constant:

log u = 2 x log x
Now, differentiating w.r.t. x, we get

x =1

Hence,

Now taking the log of both sides, we get


log u = log x

LM dv OP
N dx Q

2x

log2 x

log a N

4
x

FG IJ OP
H K PQ
4
O
log tan P
4
4Q

2 x
dv
x
x
1
4
=v
cot
sec

log tan
dx
x
4
4 x2
4

FG
H

x 4
x
1
sec 2
+ log tan
x
4 4
4
tan
4

IJ FG 1 IJ
KH x K
2

af
f axf = 0

2x 3 = 0
2x = 3

Evaluation of Derivatives for Particular Arguments

3
2
Type 9: Problems based on existance of a derived
function at a point x = a
Whenever we say that something exists, we mean
that it (something) has a finite value in the world of
mathematics which implies that whenever we say that
f x or f x at x = a exists, we mean that f a or
x=

af af
af
or
f aa f has a finite value and whenever f a x f
LM dy OP has an infinite value or meaningless or
N dx Q
F a 0 I
undetermined value H i.e; , , etcK we say that
0 0
x
=
a
x = a does not exist.
at
or
at
f axf
f axf
x=a

x=a

Question: How to guess the existance of a derived


function at a point x = a ?
Answer: A simple rule to guess the existance of f x
at x = a is the following :
1. Differentiate the given function f x w.r.t the
independent variable x by using the formulas of d.c.
of a power function, sum, difference, product, quotient,
function of a function etc.

af

af

af

2. Find f x

af

x=a

af

3. If f a is finite, we expect that f x exists at


x = a and if f a is undefined, we expect that f x
does not exist at x = a

af

af

Question: How to test that existance of a derived


function f x at a point x = a ?
Answer: Whenever we are required to test (or,
examine) whether f x exists at x = a or f x does
not exist at x = a , we adopt the following working
rule.
1. Find the left hand derivative = f a = L1 = a
where a is a fininte number and the right hand
derivative = f + a = L2 = a where a is a finite
number.
2. If L1 = L2 , then f a is said to exist (or, we say
that f x exists at x = a ) and if L1 L2 , then f a
is said not to exist ( or, f x is said not to exist at
x = a or we say that f x does not exists at x = a )

af

af

af

af

af

af

af

af

af

af

627

Remember:
1. f a exists f a = f+ a = a a finite
number.
2. f a does not exists f a f + a a a
finite number.
or f a does not exist

af
af
af

af

af

af
af

af

af

f a = f + a = , ;

af

or f a does not exists

af

af

f a = + and f+ a =
Examples worked out:
1. If y =

x=

2x 3
dy
, examine the existance of
at
3x 4
dx

4
3

af

Solution: Let f x =

2x 3
3x 4

F 4I 3
F 4 I = H 3K
f
H 3K 3 F 4 I 4
H 3K
2

8
3 8 9
17
= 3
=
=
4 4
8 3 24

F 4 + hI 3
2
H3 K
4
F
I
f +h =
H 3 K 3 F 4 + hI 4
H3 K
8
17
+ 2h 3
+ 2h
3
3
=
=
4
4 + 3 h h
+h 4
3
3

F
H

I
K

17
+ 2h
= 3
8 + 3h

F 4 I F 4I =
f +h f
H 3 K H 3K

17
+ 2h
17
3

8 + 3h
24

628

How to Learn Calculus of One Variable

17 + 6 h
17

3 8 + 3h
24

17 + 6 h 17

24 + 9 h 24

24 17 + 6h 17 24 + 9h
24 + 9h 24

408 + 144 h + 408 153 h


24 24 + 9 24 h

F 4 I = lim
H 3K

h0

F
H

af a

h 0
h

= lim

h0

h
h

= lim

h0

af

af

af

Hence, f x = x is not differentiable at x = 0 i.e;

dy
does not exist at x = 0
dx

I
K

af

To find f a or some special types of functions:

F I
H K

4
4
f h f
3
3
h

Type 1: When a function is defined in the following


way y = f1 x , x a ; = c , x = a then for finding
f a we have to calculate the derivative at
x = a directly from its defination.

af

af

Examples worked out:

R|x sin F 1 I for x 0


S| H x K
T0 for x = 0
2

1. If y =

1
=
64

F 4I = f F 4I = 1
H 3 K H 3 K 64
4
dy
Hence, f a x f , i.e;
exists at x =
3
dx

f+

dy
2. If y = x examine the existance of
at x = 0
dx
Solution: 3 f x = x

af

af

h0

f 0h f 0
; h>0
h

= 1
f + 0 = f 0

F I
H K

af

f 0 = lim

9 h
= lim
h 0 h 24 24 + 9 24 h

4
f
= lim
h 0
3

h0

F 4 + hI f F 4 I
H 3 K H 3 K ; a h > 0f

9 h
24 24 + 9 24 h
= lim
h 0
h

9
1
=
=
24 24 64
Similarly,

h
h

= lim

=1

9 h
144 h 153 h
=
=
24 24 + 9 24 h 24 24 + 9 24 h

f +

h0

h 0
h

= lim

f + 0 = lim

h0

af

f 0+h f 0
;h >0
h

Solution: For x 0 the derivative may be calculated


by the formulas and the rules of differentiation (as y
is the product of two differentiable functions)

af

f x =

F
H

1
d
2
x sin
dx
x

I
K

FG
H
F 1 I cos F 1 I
= 2 x sin
H xK H xK
=

1
1
dx 2
d
sin
sin + x 2
dx
x
dx
x

IJ
K

for x 0

629

Evaluation of Derivatives for Particular Arguments

[we can not use the expression for x = 0. At the


point x = 0, we can calculate the derivative using the
defination of the derivative]

af

af a

f 0+h f 0
, h>0
h

f + 0 = lim

h 0

= lim h sin
h 0

1
=0
h

[ 3 The product of an infinitesimal function and


a bounded function is an infinitesimal]

af

af a

f 0h f 0
, h>0
h

f 0 = lim

h0

1
=0
h

= lim h sin
h 0

af
Hence f b0g = 0.
F 1 IJ , x 1 , = 0 , when
2. If y = a x 1f sin G
H x 1K
x = 1 Find f a1f .
f a1 + hf f a1f
; ah > 0f
Solution: f a1f = lim
h
f 0 = 0

= lim

h0

a1 + h 1f

h 0

sin

FG 1 IJ 0
H 1 + h 1K

h
2

h sin
= lim

h0

F 1I
H h K = lim h sin 1 = 0
h0

[ 3 The product of an infinitesimal function and


a bounded function is an infinitesimal]
Similarly, f 1 = 0

bg

af

f1 =0
Type 2: When a function is defined on both sides of
x = a by different formulas in various intervals of its
domains of defination and we are required to find
f x and f ( a) , e.g.,

af

af af
af af
af

1. f x = f1 x , when x < a
f x = f2 x , when x a and we are required to
find f x and f ( a)
Working rule:
1. Find f1 x , f2 x , ... etc. by using the formulas
of d.c. of a power function, sum, difference, product,
quotient etc and retain the same intervals (or,
restrictions or, conditions) against f1 x , f2 x ...

af af

af af

af af
af
af
af a f

etc. This provides us f1 x , f2 x , ... etc. in various


given intervals.
2. Find left hand derivative = f a = L1 and right
hand derivative = f+ a = L2 by using the
defination.
3. If L1 = L2 , then f a = L1 or, L2 3 L1 = L2
Examples worked out:
1. If f x = 2 x 2 + 3 x when x < 0 , = 3 x x 2 ,
when 0 x 1 = x + 1 , when x > 1 Find
f x , f 0 and f 1 .
Solution: (1) To find f x
When x < 0 ,

af
af af

bg

af

f x =

af

...(i)

bg

...(ii)

d
2
2 x + 3 x = 2 2 x + 3 1 = 4 x + 3
dx
When, 0 < x < 1
d
3 x x 2 = 3 1 2 x = 3 2 x
dx
When, x > 1 ,
f x =

af

d
x +1 =1
dx
(2) It remains to find f 0 and f 1
f x =

af

f 0 = lim

h 0

af
af
e2h
f a hf f a0f
= lim
h

h0

...(iii)

3h 0
h

= lim 2 h + 3 = 3
h0

af af

3h h 0
f h f 0
= lim
f + 0 = lim+
h0
h
h
h 0

af

630

How to Learn Calculus of One Variable

a f

= lim 3 h = 3
h 0

af

f 0 = 3

af

f 1 = lim

...(iv)

f a

f a3 1f

3 1 h 1 h

Note:
1. We should remember that evaluating indeterminate
form means finding its limits or showing that its limit
does not exist.
2. The derivative is not defined at the points of
discontinuities of the function.
3. When the function y = f x is not defined for
(or, at or when) x = a , this function f x can not
be said to take any value at x = a . Therefore its
derivative f x also can not be said to take any
value at x = a

3 3h 1 h + 2h 2

= lim

h0

3 h 1 h

= lim

2hh

= lim

h0

h 0

h 0

af

undefined at x = 0
Examples worked out:

h 0

af

f 1+ h f 1
h

a1 + hf + 1

af

31

1. If f x =

Solution: 3 f x =

h 0

=1
f 1 + f + 1 = 1

af af
f a1f = 1
Hence f a0 f = 3 and f a1f = 1

...(v)

Type 3: To examine the existance of a derived function


f x at a point at which the given function f x
is undefined.

af

af

af

Working rule: Regarding derivative f x at a point


x = a at which the given function f x is

F i.e; a , 0 , , etc.I
H 0 0 K

following facts:

F I
H K

1 cos x

, does f
exist ?
1 sin x
2

af

h
h

= lim 1

undefined

F 1 I , cos F 1 I and tan F 1 I are


H xK H xK
H xK

4. The function sin

af

af

h 1 + h
= lim 1 + h = 1
h0
h

h 0

= lim

af

Again f + 1 = lim+

= lim

h0

= lim

af

af
af

h 0

af
af

A function f x is undefined at a point x = a


The function f x is discontinuous at x = a
The function f x is not differentiable at
the same point x = a
f x does not exist at the same point x = a
f x can not be obtained at x = a finitely.

af

we use the

1 cos x
1 sin x

F I
1 cos
H 2K 1 0 1
I
F
f
H 2 K = 1 sin F I = 1 1 = 0 =
H 2K
undefined

a x f is undefined at x = 2

f a x f is discontinuous at x =
2

f a x f does not exist at x =


2
f

2. If y =

x4
2 x

, does

dy
at x = 0 exist ?
dx

Evaluation of Derivatives for Particular Arguments

af

Solution: Let f x =

x4
2 vx

a f 02 04 = 04 = undefined.
f b x g is undefined at x = 0
f a x f is discontinuous at x = 0
f a x f does not exist at x = 0

13. y = 2 + tan x at x = t

f 0 =

14. y = cos 1 x cot 1 x at x =


15. y =

Type (1) , (2) , (3) and (4)


Problems based on evalution of

dy
at x = a
dx

dy
at the indicated points if
dx
1
at x = 2
1. y =
x+5
Find

18. y =

e j at x = 1

log x e
x+e

at x = 1

F I at x =
H 2K
3

y = cos esin x j at x =
2

7. y = cos 2 x +
8.

9. y =

21. y =

2x

2
x+4
+
at x = 2
x
4 x

22. y =

4x

4
4+
+
x
4

10. y = cosec x + tan x + cot x


4

11. y = sec x tan x at x =

24. y = sin

2
at x =

at x =

20. y =

23. y = tan

sin 4 x 2 + cos4 x 2 at x = 0,

sin x

3
4

5. y = x 2 x y + 4 xy 8 xy + 6 x 3 at y = 2
6. y = x 3 y + xy 3 x at x = 1

e x at x = 1

b4 x + 1g
2 + 5 x b2 x 3g

4. y = cos x at x = c

2+

19. y = tan x

1
at x = 1
x +3

2
4 at x = 4
x3

2x3 +

3. y = x 3 + x + 1

3
2

x + cos x
16. y = 1 + cos x at x = 0

17. y =

Exercise 15.1

2. y =

12. y = 5 sin x at x =

25. y = tan
26.

x +

at x = 12

5
3

x
x

at x = 4

cos x
at x = 0
1 + sin x

RSx
T

1 x

3x x
1 3x

UV at x = 1
2
W

at x = 1

y = 3 at x = 1 , y = 4

b g

27. x 2 + y 2 = 10 at 1 , 3

631

632

How to Learn Calculus of One Variable

3a
3a
,y=
2
2

3
3
28. x 3a xy + y = 0 at x =

29. Find

dy
at a =
if y =
cos x + x cos a
2
da

dy
at x =
30. If y = cos x , find
dx
dy
at x = 0
31. If y = sin x , find
dx

af

32. If f x = 3 tan

af
F 2I
x , find f G
H 2 JK
F 1I
x , find f G J
H 2K
f e 3j

x 2 cot

bg

x , find f 2

1
1
33. f x = 2 sin x + cos

bg
35. If f b x g = tan

1
1
34. f x = 4 sin x + cos
1

x , find

y = x + 16 w.r. t

2. Find differentiation of
y = sec

1
1
2
2. 3. 20 4. 3 cos c sin c
8
49
5. Find 6. 1 7. 1 8. 0 9. 0, 0 10. 114688
10
2
11. 16 3 12. 0 13. sec t 14.
7
1
189
1
2 16.
15.
17.
18. 5e
1+e
64
2
19.

1.

24.

1
3
4
25.
26. 2 27. 28.
39
3
2
5

at x =

1
.
2

12
2. 4
5

dy
for the parametric equations at the indicated
dx
points.
Find

1.

2.

3.

F
H

x = a cos + log tan


y = a sin

y = 2 sin sin 2

1
4
Type 5: To find the value of d.c of a given function
w.r.t another given function at x = a

5.

Exercise 15.2

6.

I U|
K V at = 3
|W

UV at =
2
W
x = a acos + sin fU

at =
V
y = a asin cos fW
4
3 at U
x=
| 1
1+ t |
V at = 2
3 at |
y=
1+t |
W
UV at =
x = 2 cos 2 cos |
4
y = 2 sin 2 sin |
W
|V at =
x = a cos U
4
y = a sin |
W
x = 2 cos cos 2

4.

2 34. 2 3 35.

1. Find differentiation of

1 x

w.r. t

Exercise 15.3

29. x sec 2 x 30. 0 31. does not exist 32. 1


33.

F 1 I
GH 2 x 1JK

Type 6: Evalution of d.c. of parametric equations (or,


functions) at a point (or, parameter t , or etc)

77
1
11
9
2 20. 16 21. 4 22. 8 23. 2
4

Answers:

Answers:
1.

x
at x = 3
x 1

633

Evaluation of Derivatives for Particular Arguments

a
fUV at t =
a
fW
3
x = a at + sin t f U

at t =
V
8.
y = a a1 cos t fW
2
x = a a sin fU

at =
9.
V
y = a a1 cos fW
2
x = a a1 cos t fU
10.
V at t = 2
y = a at + sin t f W
|V at t = 2
x = at U
11.
|
y = 2at W
7.

bg

x = a cos t + t sin t
y = a sin t t cos t

8. If f x =

af

, evaluate f 1

af d
af
i
L x 1OP , evaluate f e 3j
10. If y = M
N x + 1Q
3x
11. If y =
b3x 1g , evaluate f e 3j
12. If f a x f = d x 1i x + 1 , evaluate f e 3 j
13. If f auf = 2 + 2u , compute f a2 f
14. If f a x f = 5 x + 2 x + 1 , compute f b 1g
3

9. If f t = t m + t n , evaluate f 1
2

Answers:

1 + ax
, compute f
1 ax

1.

5
3 2. 1 3. 1 4. 4 5. Find 6. 1

15. If y =

7.

1
3 8. 1 9. 1 10. 1 11. 2

16. If f z =

af

Miscellaneous problems on evaluation:

af

17. If f x =

Exercise 15.4
1. If y =

4+

x +1
, evaluate f 1 if possible.
x 1

af

4 + z2
, compute f
z
x3

af

af

x 1

af

18. If f x =

x 1

af

, evaluate f 4

x3 + 1
, evaluate f 1
x

=
19. If f x

af

x3 + 1
, evaluate f 0
x2 + 1

2
8
20. If f x = x x +

af

3. If f x =

bg

4. If f x =

af
af

af
,evaluate f a 4f

5. If s = t + 3 t ,evaluate f 4
6. If y =

bg

2
x

7. If f x =

3
3

x +1
x

af

, evaluate f 4

x + 1 + x2

bg

1
, evaluate f 0
x 2 + 3x + 2

af
21. If f a x f = d x

e 5j

, compute f 1

8 + x3

af

2. If f x =

e 5j

, evaluate f
6
3

e 3j

+ 2 x + 6x 2

x ,

find f 1

x 2 + 1 , find f

e 3j

a f 9Z , find f e2 2 j
Z +1
1
3
4
+

+ 3x 2 x
23. If f a x f =
x
x
2 x
find f a1f
22. If f Z =

x,

634

How to Learn Calculus of One Variable

af d
25. If f a x f =

24. If f u = u 2 + 3 u 2 1 , find f

af
f axf =

26. If f x = e
27. If

x2 + 1

2x

, find f

e 2j

e 3j

a f sin FH 1x IK , when x 0 , f a0f = 0 ;


find f a x f at x = 0 and x 0 .
F 1 I , x 0 , f a0f = 0 find f a x f at
2. If f a x f = sin
H xK

1. If f x = e

af
, find f a1f

log x , find f 1
x

e log x

3
1. Does not exist 2. 1 3. 0 4.
4

LMF 1 I t
5. MGH 2 JK
N
LM x x
6.
MN

1
2

4
3

+
3
2

FG 1IJ t
H 3K
OP
PQ

OP
PQ

2
3

af a f

3. If f x = x 1

af
af

x=

1
12. 18 3 13.
14. 2
8

LM a b1 axg
15. M
NM b1 + axg

1
2

3
2

OP
PP
Q

x = v5

af

4
16.
5

v3
17
7 4 v3
18.
19.
20. 15
36
18
2
9 v3
1
21.
22.
23. 3 24. 7 v 2
2
3
1
25.
26. 2 e 2 27. 2 e
2
17.

6. If

af

Type 1: Problems based on finding the derivative of


function having the form:
y = f1 x , when x a
= c , when x = a

f x =

af

x 0 , f 0 = 0 ; find

af

af

af

1 e

1
x

af

, x 0 , f 0 = 0 ; find

af

f x at x = 0 and f x at x 0

af

7. If f x =

af

e x

af

, when x 0 , f 0 = 0 ; find

af
F 1 I , for x 0 , f a0f = 0 ; find
8. If f a x f = x cos
H xK
f a x f at x = 0 and f a x f when x 0
f x at x = 0 and f x at x 0

Type 2: Problems based on the function defined on


both sides of x = a by various formulas in different
intervals of its domains of definition and we are
required to find f x as well as f a . Exercise set:

af

Special types of functions:

af

F 1I ,
H xK

f x at x = 0 if derivative of f x exists and f x


at x 0

af

5. If f x = x sin

LM 9 x OP
MN b3x 1g PQ

11.
x=

af

4. If f x =

FG 1 IJ , when x 1, = 0
H x 1K

sin

1
sin 2 x for x 0 , = 1 for x = 0 find
x
f x at x = 0 and at x 0

t=4

x=4

LM 4 b x 1g OP
MN b x + 1g PQ

when x = 1 ; find f x at x = 1 and at x 1

1
4
7.
8.
9. 12 (m + n)
16
9
10.

x = 0 and x 0

Answer:

Exercise 15.5

af

Exercise 15.6

af
af b

1. If f x = 3 x 4 , when x 2
f x = 2 2 x 3 , when x > 2 , Find f 2

af

Evaluation of Derivatives for Particular Arguments

af
af
af
af
af
af

2. If f x = 2 x 1, 0 x < 1
f x = 2 x , 1 x 3 Find f 1
x
, when x 0
3. If f x =
1+ x
x
, when x < 0 Find f 0
f x =
1 x
4. If f x = 2 x 2 + 7 x , when x < 0
f x = x 3 4, when x > 0 Find f 1 .
Answers:
1. Does not exist
2. Does not exist
3. 1 4. 4

af

af

b g

Type 3: Problems based on finding the value of d.c of


the function which is undefined at a point x = a
x4

1. If y =

2 x

, can the value of

dy
be obtained
dx

at x = 0 ?
2. If y =

1 x
dy
, can the value of
be obtained at
1+ x
dx

x=1?
3. If y =

2x 3
dy
can the value of
be obtained at
3x + 4
dx

4
?
3

af

4. If f x =

x
x

, can the value of

dy
be obtained
dx

at x = 0 ?

af

5. Prove that the function f x =


derivative at x = 0

sin x
has no
1 cos x

af

1
tan x has
cos x

af

x 2 3x + 5
has
2 x 2 + 5x 3

6. Prove that the function f x =


no derivative at x =

635

.
2

7. Prove that the function f x =

no derivative at x = 3 .
8. Prove that the following functions have no
derivative at the indicated points.

af

(i) f x = sin

1
at x = 0
x

af

(ii) f x = cos

af

(iii) f x = tan

1
at x = 0
x
1
at x = 0
x

Answers:
1. No, as the function is not defined at the indicated
point.
2. No, as the function is not defined at the indicated
point.
3. No, as the function is not defined at the indicated
point.
4. No, as the function is not defined at the indicated
point.
5. No, as the function is not defined at the indicated
point.
6. No, as the function is not defined at the indicated
point.

636

How to Learn Calculus of One Variable

17
Derivative as Rate Measurer

General definition: If a variable z is a function of


another variable y, then the rate of change of z with
dz
respect y is
.
dy

af

dz d f y
which is known
=
dy
dy
as rate of change of z with respect to y or rate at which
z changes with y.
A case of great practical importance occurs when
the independent variable represents time.
That is, we may have y = f (t) and we wish to find
the rate of change of y with respect to time t. It is
dy
= f t directly.
sufficient to calculate
dt
Again, we may have y = f (x) and x = g (t). This is
often the case in problems of physics. If we have
dx
been given
, th time rate of change of x, we can
dt
dy
calculate
from the formula.
dx
That is, z = f (y)

af

af

dx
dy dy dx
=

= f x
dt
dt
dx dt
Moreover, we may have y = g (t), x = f (t).

bg bg
bg

dy
g t
dy
Then
,f t 0
= dt =
dx
dx
f t
dt
Thus, the rate of change of one variable can be
calculated if the rate of change of the other variable is
known.

Notes:

dy
is positive, then the rate of change of y with
dx
respect to x is positive. This means that if x increase,
then y also increases and if x decreases, then y also
decreases.
1. If

dy
is negative, then the rate of change of y with
dx
respect x is negative. This means that if x increases,
then y decreases and if x decreases, then y increases.
2. If

Remember:
1. The pharase rate of change or rate of variation or
rate of increase of a variable quantity is often used
in reference to time and the words with respect to
t are omitted. This is why when no special mention
is made of the variable with respect to which the rate
is calculated, it is assumed that the rate is taken w.r.t
time t.
2. By the rate of change or rate of variation or rate of
increase of a variable quantity is meant the change in
the value of a quantity per unit of time.
3. Whatever be the quantity Q its derivative

dQ
dt

gives how fast Q is changing with t.


4. If the quantity Q increases with time t, its

dQ
is positive.
dt
5. One important point to be remembered is to use
the same units of measurements for the same variable.
derivative

Derivative as Rate Measurer

If in a problem, some distances are given in feet and


others in inches, we may convert them into inches. If
the rates are given as feet per minute as well as feet
per second, then we may convert them all into feet
per second.
Examples

dy
=4
dx
4
2
dy

=
=
dx 2 y
y
Now, from y2 = 4x, when x = 4, y2 = 16 or y = 4
2y

which the rate = value of

dQ
Units of rate
= units of Q / units of t.
dt
Where Q = volume, area, distance, etc.
T = time

637

dy
(at the point x = 4),
dx

1
1
; (The negative sign shown
2
2
y is decreasing with increase in x at (4, 4).
(ii) The radius of a spherical soap bubble is uniformly
increasing. Find the rate at which the volume of the
bubble is increasing with radius, when its radius r is
8 cm.
Solution: Volume v of a sphere of a radius r is
4
3
given by v = r .
3
Rate of increase with respect to r is given by
i.e., (4, 4); (4, 4) =

dr
(= rate increase of radius w.r.t time t
dt
where r = radius) is cm/sec, (r is in cm and t is in secs).
Units of

dS
(=rate of increase of S w.r.t time t
dt
where S = surface) is cm2/sec, (if distance is in cm and
Units of

dv
(= rate of increase of
dt
volume w.r.t time t where v = volume) is cm3/sec.
6. When one quantity changes or grows or varies,
we like to know the rate of change or rate of growth or
rate of variation of another related quantity. This is
why it is some times called related rates. We are
concerned with the rate of changes of one quantity
relative to other with which it is connected by some
given relation.
7. If y is a function of x and x is varying with time t,
time is in secs) and Units of

dy dy dx
=

then
which rate of change of y =
dt
dx dt
rate of change of x times d.c of y w.r.t x. which in turn
dy

rate of chage of y
dy
dt
can be written as
=
=
dx rate of change of x dy
dt
dy
which means that
compares the rate of change of
dx
both y and x.
8. If the rate of change of a quantity is not w.r.t time,
it must be mentioned in the problem (question). e.g.,
(i) If y2 = 4x, find the rate at which y is changing with
respect to x when x = 4.
Solution: y2 = 4x

dv
= 4r2 .
dr
dv
2
3
= 4 8 = 256 cm / cm
dr r =8
Note:
1. Increases with, varies with or changes with means
varies increase or changes w.r.t.
2. A quantity Q increase with uniform rate

LM OP
N Q

dq
= k = constant.
dt
3. If x and y are two variables, then the rate of change
of y with respect to x at (or, when) x = a means the

value of

LM OP
N Q

dy
dy
at x = a =
dx
dx

a f = f aaf .

= f x
x =a

4. Change in a quantity, a variable or a variable


quantity = final value of the quantity (or, variable)
initial value of the quantity (or, variable).

dy dy dx
=

, then we can find


dt
dx dt
dx
and
are known or we can find
dt
dy
and
are known.
dx

5. If

dy
dt
dy
dt

dy
provided
dx
dx
provided
dt

638

How to Learn Calculus of One Variable

On Language of Mathematics
1. Language of rate problems:

dr
= a cm/sec means rate of increase of radius is
dt
a cm/sec where r = radius.
dv
= a cm3/sec means rate of increase (or change
dt
or variation or growth etc) of volume is a cm3/sec
where v = volume.
dS
= a cm2/sec means rate of increase (or change
dt
or variation or growth etc) of surface area or simply
area is a cm2/sec where S = surface.
... and so on
2. Language of variation:
(i) Direct variation: When the ratio of two variables
equals a constant, the variables are said to vary
directly.
If we let y and x represent any two variables, then
we may state that they vary directly by writing

y
=k
x

where k represents a constant.

y
= k is also symbolised as
x
y x y = kx.
(ii) Inverse variation: When the product of two
variables equals a constant, then the variables are
said to vary inversely.
If we let x and y represent any two variables, we
may state that they vary inversely by writing x y = k
where k represents a constant.
k
The relationship may also be expressed as y =
x
k
or x =
or alternatively we may say
y
y is inversely proportional to x i.e;
Thus,

1
k
y = or x is inversely proportional
x
x

to y i.e: x

1
k
x= .
y
y

N.B.: We can always replace the sign ' ' by the sign
of equality = provided we introduce a multiplying
constant on one side of the equation. This constant
is often termed as constant or proportionality. The
symbol ' ' is termed as sign of variation, moreover,
instead of is proportional to or varies as the symbol
' ' if often used.
3. Joint variation: When a variable varies directly as
the product of two or more than two variables, it is
said to vary jointly as these variables.
If x varies jointly as y and z, then x y z or x = k y
z where k is any constant.

dQ
is constant = Q increases or decreases with a
dt
constant rate respectively where Q = any quantity
and t = time.
4.

dQ
tells us at what rate a physical quantity Q
dt
changes or increases or decreases if Q be a certain
quantity varying with time.
Or, alternatively, if Q be a certain quantity varying

5.

dQ
represents the rate at which that
dt
quantity Q is changing.
with time

Type 1: Problems based on finding the rate of physical


quantities like volume, area, perimeter etc.
Working rule: In such problems where time rate of
change (rate of change w.r.t. time) of certain variable
(variables) is (are) given and time rate of change of
some other variable (variables) is (are) to be found
out, we use the rule described in following four steps:
1. Find the relation by mensuration formulas (between
the two variables) between that quantity whose rate
of change is required and whose rate of change w.r.t
time is given. The following relations are very helpful.
A = area of a square = x2, its perimeter = 4x
A = area of rectangle = x y, its perimeter = 2 (x + y)
Area of trapezium =

1
(sum of parallel sides).
2

distance between them.


2

A = area of a circle = r , its perimeter = 2 r .

Derivative as Rate Measurer

1 2
r h , its total
3
surface = r r + l whereas its curved surface is
r l only.
2
V = volume of a cylinder = r h , its total surface
= 2 r r + h . Whereas its curved surface is
V = volume of right cone =

Worked out
Problems based on type (1)
1. At what rate is the area increasing when the side
of an equilateral triangle is 10 ft, if the side of an
equilateral triangle increases uniformly at the rate of 3
ft/sec.
Solution: Let A = area of an equilateral triangle.
2

Now, differentiating both sides of (1) w.r.t t, we


get
dA
3
dx
=
2x
dt
4
dt

= 2r h .
V = volume of a box = x y z and its surface or
surface area = 2 (xy + yz + zx)
V = volume of a cube = x3.
2. Differentiate the equation (between the quantities
whose rate of change is required and whose rate of
change w.r.t. time is given which exists at any instant
or time during which the condition of the problem
holds) w.r.t time t. Generally we differentiate any
one of the formula of area, volume or perimeter etc of
a substance having a geometrical shape like square,
rectangle, circle cylinder or cone etc with respect to
time t provided that time rate of change of a certain
variable is given in the problem.
3. Substitute the known quantities in the differentiated result.
4. Solve for the required unknown.
N.B.: 1. We may draw the figure for convenience.
2. Formulas for volumes, areas, perimeters of a
substances having geometrical regular shape like
square, rectangle, trapezium, circle, cone, cylinder or
cube etc must be remembered.
3. While working out problems of rate measurer, we
must note the two variables, one whose rate of change
is given and the other one whose rate of change is to
be found. Then we express either variable in terms of
the other by means of an equation. (or known formula)
and lastly we differentiate through out with respect
to time t.

3x
, (x = length of a side)
4

...(1)

639

x = 10

x = 10

F
H

x = 10

I
K

3
dx
2x 3 3
= 3 is given
4
dt

LM dA OP
N dt Q

=
x = 10

3
2 10 3
4

= 15 3 sq. ft/sec.
2. A spherical balloon is inflated and the radius is
1
increasing at inches/minutes. At what rate would
3
the volume be increasing at the instant when its radius
is 2 inches.
Solution: let V = volume of a spherical balloon
4
3
= r , radius = r).
...(1)
3

Now, differentiating (1) w.r.t t we get

dv 4
2 dr
= 3r
3
dt
dt
=

FG
H

IJ
K

dr 1
4
1
3
3 r2
= is given
dt
3
3
3

LM dv OP
N dt Q

=
r=2

LM 4 r OP
N3 Q

r =2

16
inch3 /minute.
3

640

How to Learn Calculus of One Variable

3. A spherical balloon is pumped at the rate of 10


cubic inches per minute. Find the rate of increase of
its radius when its radius is 15 inches.

4 3
Solution: v = r (when r and v represent radius
3
and volume respectively)

F
H

I
K
dr F dr
I
3

= 10J
G
H
K
dt
dt

dv
d 4 3
4
dr
2

=
r = 3 r
dt
dt 3
3
dt
10 = 4 r 2

dr
10

=
2
dt
4r

LM dr OP
N dt Q

a f

4 15

10
1
=
900 90 inch/

minute.
4. The circular waves in a tank expand so that the
circumference increases at the rate of a feet per
second. Show that the radius of the circle increases
a
at the rate of
feet per second.
2
Solution: let p = perimeter of the circular wave at
time t from the start.
= 2 r (where r is the radius of circular wave at
time t from the start)

a f

I
K

dr
a
=
ft/sec.
dt
2
5. A cylindrical vessel is held with its axis vertical.
Water is poured into it at the rate of one unit per
second. Given that one unit is equal to 34.66 cu. Inch.
Find the rate at which the surface of water is rising in
the vessel when the depth is x inches.
Solution: let A = are of cross section of cylindrical
vessel.
x = height of cylindrical vessel.
V = volume = A x

dv
d
=
A x
dt
dt

dx
dv
= A
dt
dt

dx 34.66
=
dt
A

dv
= 34.66 cu. inch/sec)
dt

dx 34.66
=
inch/sec.
dt
A

FG dx = rate of change of x = rate of rising of the


H dt
I
surface of water in the vessel J .
K
6. A balloon which always remains spherical has a
variable radius. Find the rate at which its volume is
increasing with radius, when the latter is 7 cm.
Solution: Let the radius of the balloon = r
and the volume of the balloon = v
4 3
v = r
3
rate of increase in volume with radius r

=
dP d 2 r

=
dt
dt

F
H

dr
dP
3
=a
dt
dt

(3 Given

10
r = 15

a = 2

dv 4
= 3r 2
dr 3

LM dv OP
N dr Q

bg

=4 7
r=7

= 196 cm 3/cm.

Derivative as Rate Measurer

7. The volume v and the pressure p of a gas under


constant temperature are connected by pv = c, where
c is a constant, show that

dp c
=
.
dv v 2

p 1 + v

(proved)
v2
8. A sphere of metal is expanding under the action of
heat. Compare the rate of increase of its volume with
that of its radius. At what rate is the volume increasing
when the radius is 2 inch and increasing at the rate of

1
inches per minute.
3
Solution: let v = volume of the metalic sphere at time
t
r = radius at time t
v =

4 3
r
3

dv
2 dr
= 4 r
dt
dt

given that

4 3
r
3

(1)

s = 4 r 2
Differentiating (1) w.r.t t, we get

dp
=0
dv

FG c IJ
H vK = c
dp
p

= =
dv

Solution: Let v, x and r denote the volume, surface


and radius of the spherical soap bubble respectively
at time t from the start.

3v =

Solution: 3 pv = c

...(1)

dr 1
= inches/minute where r = 2 inches
dt
3

F dv I = FG 4 2 IJ cube inch/minute

H dt K H 3 K
F 16 I cube inch/minute
=
H3K
2

dv
2 dr
= 4 r
dt
dt
Again differentiating (2) w.r.t t, we get
dr
ds
= 8 r
dt
dt

F
H

2
2 dr
4 r
r
dt

2 dv

[from (3)]
r dt

dv
ds 2 dv
2 dr
= 4 r
=
(2)
dt
dt
dt 3 dt

(3)

Problems based on cylinder, cone, cube, etc.


1. A cone is 10 inches in diameter and 10 inches deep
water is poured into it at 4 cubic inches per minute. At
what rate is water level rising at the instant when the
depth is 6 inches.
Solution: Let A B C = a cone
A D = depth of the cone = 10
B C = diameter = 10
= CAD

tan =

CD
5
1
=
=
AD 10 2
D

P
x

9. The volume of spherical soap bubble is denoted


by v, its surface by s, the radius being r, show that
(1)

(2)

I
K

r =2

dv
2
Comparison: dt = 4 r
dr
dt

641

642

How to Learn Calculus of One Variable

Let A P = depth of water = x and


v = volume of water at time t

v =

1
x tan
3

1 3
2
x tan
...(1)
3
Now, differentiating both sides of (1) w.r.t t,
v=

dv 1
dx
2
2
= 3 x tan
dt
3
dt

FG IJ FG dx IJ
H K H dt K

1
1
3 62
3
2

4=

F dx I
H dt K

=
x =6

x=6

44
4
inch/min.
=
36 9

3. Water runs into a circular conical tank at the


constant rate of 2 cubic ft per minute. How fast is the
water level rising when the water is 6 ft deep? It being
given that the radius of the circular base of the cone
= 5 ft and height of the cone = 10 ft.
Solution: let at any time t
O P = h = height of water level
P R = r = radius of water surface
v = volume of water at any time t.
Given that O N = 10 ft
N M = 5 ft
dv
=2
dt
dh
then to find
when h = 6
dt

v=

2. Water is being poured at the rate of one cubic foot


per minute into a cylindrical tube. If the tube has a
circular base of radius a ft, find the rate at which
water is rising in tube.
Solution: Let at any time t,
the height of water level = x ft and volume = v

1 2
r h
3

FI
HK

v=

1
h

3
2

v=

1
1
h
3

=
h
3
4
12

v = a x

2 dx
dv
= a
dt
dt
2

1 = a

F
H

I
K

dx
dv
3 Given
=1
dt
dt

dx
1
=
ft/minute.
dt a 2

PR
OP
r
h
5h h
=
=
r=
=
NM ON
5 10
10 2
s

From is

dv 3 2 dh
=
h
dt
12
dt

Derivative as Rate Measurer

2=

FG IJ
H K

2 dh
6
dt
4

F dh I
G J
H dt K

h=6

which
h=6

r=

r
x

x
v3

...(1)
N

LM dx OP
N dt Q

=
x =3

LM 9
MN x

OP
PQ

x =3

9
1
=
ft/min. = the rate at which depth of
9
water is increasing when the depth of water = 3 ft.
5. An inverted cone has a depth of 10 cm and a base
of radius 5 cm. Water is poured into it at the rate of
2.5 c.c per minute. Find the rate at which the level of
the water in the cone is rising when the depth of the
water is 4 cm.
Solution: Let the depth of the water at time t minutes
from the start in x cm. If the radius of the surface of
water at this time is r, then from the similar triangles
COQ and COB, we have
r
x
=
5 10
x
r=
(1)
2
=

1 2 dx
x
3
dt
9
dx

=
2
dt
x

30 =

1 2 dx
dv 1
2 dx
= 3x
= x
3
9
dt
dt
dt

3=

8
2
=
=
= 0.071 ft/min.
36 9

4. A hallow cone whose semi vertical angle is 30 is


held with its vertex downwards and axis vertical and
water is poured into it at the steady rate of 3 c. ft/min.
Find the rate at which the depth (measured along the
axis) of the water is increasing when the depth of the
water is 3 ft.
Solution: Let v = volume of water at any time t
= 30 = semi vertical angel (given)
r
Now, tan =
(from rt d OPR )
x
r
tan 30 =
x

643

Q
10 cm

Again 3 v =

1 2
r x = volume of water at any
3
C

time t

v =
r =

x
3

1
x
1

x = x 3 (putting from (1)


3
3
9

Again 3 v =
time t

1 2
r = volume of water at any
3

644

How to Learn Calculus of One Variable

F I
H K

1 x
3 2

v=

x=

v = x

1
3
x
12

which

x
(3 r = from (1))
2
dv
1
2 dx

=
3x
dt 12
dt

b g

= .03x cubic inch/hour


2

...(2)

dv 5
= c.c. per minute.
dt
2
Putting this value in (2), we get
dx
45
=
dt
x2 2

but we are given

LM dx OP = LM 4 5 1 OP
N dt Q MN 4 2 PQ
L5O
= M P cm/minute
N8 Q
2

x =4

dv
2 dx
= 3x
dt
dt

= 3x 2 .01 cube inch/hour

dv 1 2 dx
= x
dt
4
dt

LM dv OP
N dt Q

= .03 4 = .12 cubic inch per hour


x =2

7. A right circular cylinder has a constant height h


but the radius r of its base varies. If v be the volume
and S the curved surface of the cylinder, prove that

dv
=S.
dr
Solution: let r = radius of right circular cylinder and v
is its volume at time t from the start, then
2

v = r h

= rate of rising water when the depth of water is


4 cm.
6. The temperature of metal cube is being raised
steadily so that each edge expands at the rate of .01
inch per hour. At what rate is the volume increasing
when the edge is 2 inches.
Solution: let x = length of the edge of the metal cube
A B C D E F G H at the time t
Given that

dv
dr
= 2rh
dt
dt

=S

dr
3 S = 2rh
dt

...(1)

dv dv dt
=

dr
dt dr
dv dt
=
dr dt

dx
= .01 inch/hour
dt
Let v = volume of the metal cube when the edge is
x inches at any time t
E

G
h
C

D
A

= S (from (1))
(proved)

Derivative as Rate Measurer

Problems based on proportionality


1. If the area of the circle increases at a uniform rate,
then the rate of increase of the perimeter varies
inversely as the radius.
2

Solution: let the area of the circle A = r


(1)
Now, differentiating both sides of (1) w.r.t t, we
get

dr
dA
= 2r
dt
dt
k = 2r
(3

dr
dt

dA
= uniform rate = k (say) = given)
dt

2 d r k

=
dt
r

dP
dr
= 2
dt
dt
Putting (2) in (4), we get

v is proportional to time t
Again

dv
= 2k
dt

dv
= a constant
dt
rate of increase in velocity = constant (proved).
3. A spherical ball of salt is dissolving in water in
such a manner that the rate of decrease in volume is
proportional to the surface. Prove that the radius is
decreasing at a constant rate.
Solution: let v and S denote the volume and surface
of the spherical ball of salt respectively when the
radius is r at time t.

We know that v =
...(2)

Again perimeter of the circle P = 2 r


(3)
Differentiating both sides of (3) w.r.t t, we get

645

S = 4r

4 3
r
3

(1)

(2)

(4)
r

dP k
=
dt
r
dP 1
(proved)
dt
r
2. Prove that if a particle moves so that the space
described is proportional to the square of the time of
description, the velocity will be proportional to the
time and rate of increase of the velocity will be
constant.
Solution: letting S = the space described by the
particle in time t.

dv
S (given)
dt

dv
= u S [ 3 v decreases with increase in
dt
t], u > 0 (constant)
(4)

Now,

S = k t , where k = a constant

dS

= 2k t
dt

v = 2 k t (3
vt

dv
dr
2 dr
= 4r
=S
dt
dt
dt

uS = S

S t (given)

dS
= v = velocity of the particle)
dt
(1)

(3)

dr
(using (4))
dt

dr
= u (constant)
dt
radius is decreasing at a constant rate (ve
sign signifies the decreasing of r)
4. The diameter of an expanding smoke ring at time t
is proportional to t2. If the diameter is 6 cm after 6 sec,
find at what rate it is then changing.

646

How to Learn Calculus of One Variable

Solution: let D denote the diameter of an expanding


smoking then according to question, D t 2
2

D = k t , (where k is a constant)

dD
= 2kt
dt

(1)
(2)

4. Find the other rate by solving the equation


dx
dy
and
.
involving
dt
dt
Notes:
1. Figures must be drawn for convenience, for doing
rate problems based on right angled triangle as well
as formulas for volume, area, perimeter of regular
geometrical figures (as triangle, rectangle, square,
cylinder, cone, sphere or cube etc) must be
remembered.
2. We substitute the given quantities and rates in
the differentiated results to get the required rate.
3.

Now, since, it is given D = 6, when t = 6

6
1
=
(3)
36 6
Putting the value of k from (3) in (2), we have
from (1), 6 = k 36 k =

1
1
dD
=2 t= t
6
3
dt
lastly, when t = 6, the rate of change of
D=

LM dD OP
N dt Q

LM t OP
N3Q

=
t=6

t =6

6
= 2 cm/sec.
3

Type 2: Problems based on right angled triangle


In such types of problems, we adopt the following
working rule.
1. An question given in x and y stated in words should
be translated into symbolic equation in x and y or we
form an equation in x and y by using the properties
s
of .
2. Then differentiate both sides of the equation
determined w.r.t time t which provides us a relation
(or equation) between

dy
dx
and
.
dt
dt

3. Use the given value for


given.

dy
dx
or
which ever is
dt
dt

LM dQ OP
N dt Q

x =a

= the rate of Q when (or, at) x = a where

Q = any quantity.
4. Rate at which two bodies are nearing (or being
separated) is the rate of distance r between them
dr
denoted
.
dt
Worked out
Problems based on type (2)
1. A man 6 ft tall walks away along a straight line
from the foot of a light post 24 ft high at the rate of 3.
m. p. h. How fast does the end of his shadow move?
Find the rate at which the length of his shadow
increases.
Solution: LP = height of the lamp post = 24 ft
MN = height of the man = 6 ft
PN = x = the distance between the foot of light
post and the position N obtained after t second, when
the man is x feet away from the foot P of light post.
PR = y
NR = PR PN = y x
Now from similar LPR and MNR , we have

PL
PR
=
MN
NR
24
y
=
which
6
yx
y
4=
yx

4 y x = y

3y = 4x

(1)

Derivative as Rate Measurer


L

24 ft

6 ft

90
P x

90
N

ay = bx + by
a b y = bx
( y x)

Now, differentiating both sides of (1) w.r.t t, we


have
dy
dx
=4
3
dt
dt
dy 4 dx

=
dt
3 dt
dy 4
dx

= 3 3
= 3 given
dt
3
dt

LM
N

OP
Q

dy

= 4 miles/hour
dt
The end of the shadow moves at the rate 3 miles/
hour.
The rate at which the shadow lengthens
=

y
b
=
x+ y a

b g = d b y xg

d NR
dt

dt

dy dx

= 4 3 m.p.h. = 1 m.p.h.
dt
dt
2. A point source of light is hung a meter directly
above a straight horizontal path on which a bot b
meter in height is walking. How fast is the boys
shadow lengthening when he is walking away from
the light at the rate of c meter per minute.
Solution: let y = length of the shadow in meter and x
= the distance of the boy after time t
=

y=

bx
a b

dy
b
dx
=

dt
a b dt

dx
= c = velocity of the boy
dt
rate of lengthening in the boys shadow
=

dy dy dx
=

dt
dx dt

FG b IJ c LM3 dy = b and dx = cOP


H a b K N dx a b dt Q
F bc IJ meter/minute.
=G
H a bK
=

3. A lamp is at a height 376 cm from the ground. A


man 188 cm tall is walking on the ground steadily at
the rate of 10 cm/sec, in a straight line passing through
the lamp post. Find the rate at which the end of his
shadow is moving.
Solution: let A be the lamp
BC, the man
D, the point where AC produced meets OB
produced,
A
C

90
O x

90
B x

90
C

ECD is EBA

...(1)

But we are given

376
x

647

188
(y x)
90
B

yx

then we are required to find out the rate at which D is


moving.
Again let us suppose that
OB = x
OD = y

648

then

How to Learn Calculus of One Variable

dx
= 10 (given)
dt

AB
BC
=
MN
NC

dy
.
dt
From similar triangles AOD and CBD, we have
BD BC
=
OD OA

and we have to find

y x 188 1
=
=
y
376 2

which

y = 2x
dy
dx
=2
= 2 10 = 20 cm/sec.
dt
dt
(3

dx
= 10 is given in the question)
dt

1
ft
2
high at the rate of 3 miles per hour. Find how fast is
his shadow lengthening.
4. A man 5 ft tall walks away from a lamp post 12

Solution: let AB = lamp post = 12

1
ft
2

MN = height of the man = 5 ft


BN = x ft = distance of a man at any time t from
the lamp post.
NC = y = length of the shadow

dx
=3
dt
dy
then to find
dt
given

25 2 x + y
=
5
y

5y = 2 x + 2 y

3y = 2x
dy 2 dx
=
dt
3 dt
2
= 3
3
= 2 miles/hour.
5. A ladder 25 ft long reclines against a wall. A man
begins to pull the lower extremity which is 7 ft distant
from the bottom of the wall along the ground outwards
at the rate of 3 ft/sec. At what rate does the other end
begin to descend along the wall?
Solution: let AB = wall
AC = ladder
BC = 7 = 7 feet (given)
Let after t second, the foot c be at Q
and let CQ = x
BP = y
AC = 25 feet
From PBQ

2y 2x = y

U|
V|
W

PB = PQ BQ
2

a f a7 + xf

y = 25

...(1)
Now differentiating both sides of (1) w.r.t t, we
have,
A
s

Now from the similar ABC and MNC, we have


25

ft

5
90
B x

90
N

90

7 ft

Derivative as Rate Measurer

2y

dx
dy
= 2 7 + x
dt
dt

dy 7 + x
dx
=

dt
y
dt

...(2)

Now, we know that when x = 0, y = 24 feet (when


t = 0)
Again from (2) , we have

t=0

a f

d
2
d 2
400
x + y =
dt
dt

2x

b7 + xg 3 FG3 dx = 3 is givenIJ
H dt
K
y

FG dy IJ
H dt K

649

dx
dy
+2y
=0
dt
dt

dx
dy
+ y
=0
...(1)
dt
dt
Now, again from the relation x2 + y2 = 400, when
x = 12, we have
(12)2 + y2 = 400
x

144 + y = 400

21
7
=
= feet/sec.
24
8

y = 400 144 = 256

(ve sign shows that y decreases with t i.e. the


other end descends and the rate at which it descends

7
ft/sec)
8
6. The top of ladder, 20 feet long, is resting against a
vertical wall and its foot on a level pavement, when
the ladder begins to slide outwards. At the moment
when the foot of the ladder is 12 feet from the wall, it
is sliding away from the wall at the rate of 2 feet per
second. How fast is the top sliding downwards at
this instant? How far is the foot from the wall when it
and the top are moving at the same rate.
Solution: let at any time t
AB be the position of the ladder having the length
= 20 feet where
OA = x
OB = y where O represents the foot of the wall.
Now, from the right angled triangle, we have,
=

y=

256

y = 16
dx
= 2 ft/sec.
dt
Substituting these values in (1), we have
Also we are given x = 12, as well as

12 2 + 16

dy
=0
dx

dy
24
3
=
= ft/sec.
dx
16
2
Hence B is sliding downwards (as the negative
3
ft/sec at the instant
sign shows) as the rate of
2
under consideration.
If, at a particular instant, A and B are sliding at the
same rate, then

dx
dy
=
and then (2) provides us x = y
dt
dt
and for the reason, x2 + y2 = 400

20

fe e

2 x = 400
t
2

x =
90

OA2 + OB2 = AB2


2

a f a3 AB = 20 feet f

x + y = 20

400
= 200
2

x = 10 2 feet.
7. A ladder is inclined to a wall making an angle of
30 with it. A man is ascending the ladder at the rate
of 3 ft/sec. How fast is he approaching the wall.

650

How to Learn Calculus of One Variable

Solution: AB is the position of the ladder inclined to


a wall OB having the length L feet.
x = distance moved along the ladder and y =
distance from the wall at time t

Given that

x = 10 ft y =

to find:

30

From (1), 2 x

x
A

Now, sin 30 =

b
a

CD
y
=
BD
Lx

g
f

y = L x sin 30
1
y= Lx
2
dy
1 dx

=
( 3 L is a constant)
dt
2 dt

262 102 = 24 ft,

dy
when x = 10 ft.
dt

dx
= 4 ft/sec
dt

dx
dy
+ 2y
=0
dt
dt

dy
x dx
=
dt
y dt

FG dx IJ
H dt K

=
x = 10

10
5
4 = ft/sec.
24
3

which the top of the ladder is sliding down at the

5
ft/sec.
3
9. A kite is 100 ft high and a length of 260 ft of the
string is out. If the kite is moving horizontally at the
1
rate of 6 m.p.h directly away from the person who
2
is flying it, how fast the string (or, cord) is being paid
out.
Solution: let k be kite which is flying in the horizontal
direction in the height of 100 ft.
M is the point where the man is standing and is
flying the kite. Let at time t,
MK = the length of the string (or, cord) which is
out = y (say)
MA = the horizontal distance of the kite from M at
time t = x (say)
Now, from AKM we have
rate of

dy
1
dx
= 3 (3
= 3 feet/sec is given)
dt
2
dt

dy
3
= ft/sec
dt
2
Hence, y decreases (as the ve sign shows) at the

3
ft/sec.
2
8. A ladder 26 ft long leans against a vertical wall.
The foot of the ladder is drawn away from the wall at
the rate of 4 ft per second. How fast is the top of the
ladder sliding down the wall when the foot of the
ladder is 10 ft away from the wall.
Solution: let AB = ladder = 26 ft
OA = x and
OB = y at time t
Now from the right angled triangle, we have
x2 + y2 = (26)2
(1)

rate of

100 feet

B
90
26

fee

y2

90

1002

= x2

(1)
since, the height of the kite is same always = 100 ft
Now, differentiating (1), we get

Derivative as Rate Measurer

Initially, when t = 0, Z = OA = 260

dx
dy
= 2x
dt
dt

2y

Hence,

dy
dx
=x
dt
dt

...(2)
2

y = 260 x =

260 100 = 240

Substituting these values of x and y in (2), we


have

260

dy
13
= 240
dt
2

dy
= 6 m.p.h = the required rate.
dt
Second method:

x = MA =
=

MK AK

a260f a100f
2

dZ 13 240
=
dt
2 260

= 6 m. p. h
= the require rate
10. A kite is moving horizontally at a height of 151.5
meters. If the kite is moving horizontally directly away
from the boy who is flying it at the rate of 10 meter/
sec, how fast is the string being let out when the kite
is 250 meters from the boy who is flying the kite, the
height of the boy being 1.5 meters.
Solution: Let AB = x and
AC = y at time t.
AE = 1.5
DC = 151.5 m
BC = 150 m
C

= 240

y=

26

e
0 fe

90

1
S
distance
KN = 6 t (3 velocity =
i.e, v = )
2
t
time
13
= t = AO
2
13
MO = MA + AO = 240 +
t
2

2Z

F
H

F
H

I
K

I + a100f
K

90

F
H

13
13 240 + t
dZ
2

=
dt
2Z

I
K

Also

Ground

dx
= 10 m/sec (given)
dt

dy
, when y = 250 ft.
dt
Clearly, y2 = x2 + 1502

Then we find

2y

dZ
13
13
= 2 240 +
t
+0
dt
2
2

90 B

1.5 m

90

and Z = MN = MO + ON = 240 + 13 t

MN = Z

150

t
= 100 feet

651

dx
dy
= 2x
dt
dt

dy
x dx
=
dt
y dt

When y = 250 ft, x = 200 ft, from (i)

FG dy IJ
H dt K

=
y = 250

= 8 ft/sec.

200
10 ft/sec
250

...(i)

652

How to Learn Calculus of One Variable

11. If the side of an equilateral triangle increases at


the rate of 3 ft/sec and its area at the rate of 12 sq.
ft/sec find the side of the triangle.
Solution: let x = each side of the equilateral at
time t and A = area of PQR , then

AC = r, BC = y, CAB = at time t.

dy
= 140 ft/min
dt
AB = 500 ft

Given that

Then we have to find (i)

Now, tan =
y

sec
60

90
L

A=

d
1 dy
=

dt
500 dt

d
1
=

dt
500

...(1)
=

dA
3
dx

=
2x
dt
4
dt

...(2)

dy

F1 + y I dt
GH a500f JK
2

3 which when

3
2x
4

r
y

90
500 feet

F1 + y I 140
GH a500f JK
2

FG d IJ
H dt K
F d IJ
G
H dt K

which x = 8 ft (Ans.)
12. A balloon rising from the ground at 140 ft/minute
is tracked by range finder at a point A located 500 ft
from the point of lift off. Find the rate at which the
angle at A and the range r are changing when the
balloon is 500 ft above the ground.
Solution: let the range finder be at A.

500

dA
dx
= 12 and
=
dt
dt

substituted in (2), we have 12 =

y
500

3 2
x
4

Given that

d
dr
(ii)
when y = 500 ft
dt
dt

7
1
1
140
500 2

.7
= 0.14 radian/minute
5

y = 500

y = 500

Also, r2 = y2 + (500)2

2r

dr
dy
= 2y
dt
dt

dr
y dy

=
dt
r dt

500
500 2

FG dr IJ
H dt K

y = 500

140 = 70. 2 ft/min.

12. Two bodies start from O, one traveling along OX


at the rate of 3 miles per hour and the other along OY
(which is perpendicular to OX) at the rate of 4 mile per
hour. Find the rate at which the distance between
them is increasing at time t.
Solution: let r be the distance between two bodies
after t hours.

Derivative as Rate Measurer

distance
S OA I
F
i . e. v = =
H
time
t
t K
distance
S OB I
F
i . e. v = =
OB = 4t H3 velocity =
time
t
t K

OA = 3t 3 velocity =

Now, from the right angled triangle, we have


r2 = (3t)2 + (4t)2
2

r = 9 t + 16t

dy
dx
= 2
=
dt
dt
dy

= 2v
dt
Second method:

F
H

2v 3

dx
=v
dt

653

I
K

F S = OP I
H t tK
F S PQ I
PQ = vt 3 v = =
H t tK

OP = vt 3 v =

r = 5t

y2 = v2 t2 + v2 t2

2 2

y = 2v t

y=

4t

90

3t

dr
= 5 miles/hour which is the required rate at
dt
which the distance AB increasing.
13. Two cars started from a place, one going due
north and the other due west with equal uniform speed
v, find the rate at which they were being separated
from each other.
Solution: let P and Q denote the position (place) of
the cars after time t
OP = OQ = x ( 3 speed is same)
PQ = y

dy
=
dt

a f

2 vt
2 v

90

Now from the right angled triangle, we have


y2 = x2 + x2
2

y = 2x

y=x 2

vt

vt

14. Obtain the rate at which the distance between


two cyclists is widening out after an hour given that
they start simultaneously from the junction of two
roads inclined at 60, one in each road and that they
cycle with the same speed v miles per hour.
Solution: Let the distance travelled after t-second
from the junction = x now, according to the question,
x
x = vt 3v =
(1)
t
It is clear from the data that joining the junction
and the position of two cyclist, we get an equilateral
triangle the distance between two cyclist = x
Now, differentiating (1) w.r.t, we have
dx
= v which is the rate at which distance between
dt
two cyclists is winding.

LM
N

OP
Q

654

How to Learn Calculus of One Variable


B

Proof: In the right angled P1 0 P2 ,

aa sf + ab sf
2

r=

dr
=
dt 2

60

15. Two cars start with the same velocity v miles/


hour from distances a miles and b miles from the
junction of two roads inclined at 90 and travel towards
the junction. Prove that after 2 hours, they are nearing
each other at the rate of
2

4v a + b v
2

OP1 = a s
OP2 = b s

=
t =2

4v 2 a + b v

b g

a + b 4v a + b + 8v 2
2

FG dr IJ
H dt K

=
t=2

b g
2 ba + b g 2v + 2 b2 v g

2 2v a + b v
a 2 + b2

t =2

vt

t =2

a fj

= 2 v from 1

b g
4 ba + bg v + 8v

4v a + b
a 2 + b2

miles/hour

Note: They are nearing each other at the rate of


means the rate of distance between them is.
Problems based on velocity and acceleration as a rate
measure
To investigate generally the motion (velocity,
acceleration or the position of a particle at any
time t)of a particle (or, body) moving in a straight
line according to a law of motion give by y = f (t)
Where y = distance or velocity of the moving body
represented by S or v at any time t.
The procedure to be followed may be summarised
as follows.

dy
dv
and a =
by
dt
dt

differentiation.
2. We perform the operations on the expression

dy
dv
and a =
to arrive at out
dt
dt
target accordingly as the question says.
obtained for v =

(b s)

1. Find the expression for v =

90

ba sg + bb sg
F ds I
2 ba sg + 2 bb sg G J
H dt K
2s aa + bf v
+ b 2 aa + bf s + 2s

e3 S

P2

Solution: let us suppose that A and B be the initial


positions of two cars and P1 and P2 be their positions
at time t.
Given: OA = a, OB = b, AOB = 90 velocities
S
of two cars are same v =
which vt = S
t
for both cars
(1)
CD = r, AC = s, BD = s

FG dr IJ
H dt K

miles per hour.

a + b 4v a + b + 8v

To prove:

(a s)

P1
a

Remember:
1. If we are required to find out the time when the
velocity vanishes (or becomes, zero) or we are required

Derivative as Rate Measurer

to find out the acceleration at a point at which the


velocity of the body becomes zero (or vanish) and
y = f (t), a given law of motion, then we put v = 0 in the
dy
expression for
.
dt
2. Initial given conditions determine the values of
the constants appearing in the law of motion given
by y = f (t).
3. If we are given a law of motion y = f (t) as well as
initial values of velocity k1 and acceleration k2 and
we have to find out the velocity and acceleration at
dv
dS
= k2
= 0 to
time t, then we put t = 0,
dt
dt
determine the constants appearing the give law of
motion y = f (t).
4. Initial values of velocity and acceleration means
the values of velocity and acceleration when t = 0 or
alternatively, the initial position, initial velocity and
initial acceleration correspond to time t = 0.
5. Uniform velocity (acceleration) means that the
velocity (acceleration) is constant.
6.

ds dt
ds
1

= 1 which
=
dt ds
dt dt ds

7.

dv ds

= a = acceleration.
ds dt
2

8. a =

9.

d s
dt

F ds I
H dt K

t=c

F I
H K

d ds
dv ds
dv
=

=v
dt dt
ds dt
ds

means the value of velocity at time t = c

or after time t = c or at the end of time t = c or when


time is t = c.

F d sI
GH dt JK

10.

12. If a point move in a straight line, the velocity v at


a given instant of time t = t0 (called the instantaneous

ds
of the path
dt
s with respect to time t evaluated for t = t0.
13. The acceleration a at a given instant of time t = t0
velocity) is defined as the derivative

dv
of the velocity v with respect to
dt
time t calculated for t = t0.
14. The particle or body comes to rest at a point
where v = 0.
is the derivative

15. It should be noted that

means the value of acceleration when


t=c

the time is t = c or after time t = c or at the end of time


t = c or at time t = c.
11. In the problems considered if the path s is
expressed in meters (m), time t in seconds (s), velocity
is v in metres per second (m/s) and acceleration a in
metres per second per second (m/s2) which is read
meter per second squared.

ds
is positive when s is
dt

ds
is negative when s is decreasing.
dt
dv
16. It should be noted that
is positive when the
dt
velocity is increasing and negative the velocity is
decreasing.
increasing and

Worked out problems


Based on velocity and acceleration as a rate measurer
1. A stone projected vertically upwards with initial
velocity 112 ft/sec moves according to the law s =
112 t 16 t2, where s is the distance from the starting
point. Find the velocity y and acceleration when t = 3
and when t = 4.
Solution: s = (112 t 16 t2) ft

ds
= 112 32t ft/sec
dt

d s

(1)

dt

655

= 32 ft/sec2

LM OP
N Q
L d s OP =
and M
MN dt PQ
L ds O
Again M P
N dt Q
ds
Now, dt

t =3

(2)

= 112 96 = 16

32

t=3

= 32

t=3

t =4

= [112 32 4] = 112 128 = 16

656

How to Learn Calculus of One Variable

L d s OP
and M
MN dt PQ
2

= 32

t=4

= 32

t =4

2. The position of a particle in motion is given by s =


180 t 10 t2. What is its velocity at the end of t
second? At what instant would its velocity be zero.
Solution: s = 180 t 10 t2
Velocity at the end of t second,

ds
= 180 20t
dt

ds

= 0 180 2t = 0
dt
180
t=
= 9 sec
20
The volocity is zero when t = 9 sec.
3. A body moves in a straight line according to the
law of motion s = t3 4t2 3t. Find its acceleration at
the instant (time) when the velocity is zero.
Solution: s = t3 4t2 3t
ds
= 3t2 8t 3
dt

(1)

d s
(2)

= 6t 8
dt
Now, we are required to find out the value of t.

when

ds
= v = 0 . Now v = 0
dt
2

3t 8t 3 = 0

3t 2 9t + t 3 = 0
3t + 1 t 3 = 0

gb g

t = 3secs
The acceleration when the velocity is zero
=

F d sI
GH dt JK

d 2s
dt 2
4. A point moves in a plane according to the law
dy
when t = 0, t = 2
x = t2 + 2t and y = 2t3 6t. Find
dx
(ii) Put the value of t in the expression in t for

and t = 5.
Solution:

(2)
2

dy dy dt
6t 6
=
=
Now,
dx
dx dt
2t + 2

e j = 3 et 1j = 3 bt 1g
=
2 bt + 1g
bt + 1g
F dy I = 3b0 1g
G J
H dx K
6 t2 1

t =0

= 3 units

FG dy IJ
H dx K

=3 21
t=2

= 3 units

FG dy IJ
H dx K

b g

=3 51
t =5

= 12 units.
5. Find the velocity and acceleration of a moving
point after 10 sec if its position is given by s = 5t2 + 5t
3 if s is measured in centimeters.
Solution: s = 5t2 + 5t 3 cm

ds
= 10t + 5 cm/sec
dt

d s

t =3

= 6.3 8
= 10 units.
Note: To find the acceleration when velocity is zero
.
ds
= 0 and solve the
(i) First find t by putting
dt
equation for t.

(1)

dy
2
= 6t 6
dt

dx
= 2t + 2
dt

(1)

dt

Now,

LM ds OP
N dt Q

= 10 cm/sec2
= 10t + 5
t = 10

(2)

t = 10

= 105 cm/sec

Derivative as Rate Measurer

LM d s OP
MN dt PQ

5
secs points that mea4
surement of time is started from the position where

= 10

t = 10

Note: Here negative time

= 10 cm/sec2.

t = 10

Remember: To find the velocity and acceleration at


time t = c or after time t =c or at the end of time t = c.
We find

LM ds OP
N dt Q

LM d s OP
and M
N dt PQ
2

t =c

t=c

6. If s represents the distance which a body moves in


time t, determine its acceleration if s = 250 40t
16t2. Determine the acceleration and time when the
velocity vanishes and the value of s then.
Solution: s = 250 40t 16t2

ds
= 40 32t units
dt

d s

(1)

= 32 units
(2)
2
dt
Now, we are required to find out the time and
acceleration when velocity = 0

LM ds OP = 0
N dt Q

40 32t = 0
t=

LM d s OP
MN dt PQ

5
sec
4

...(3)

Now,

= 32
5
t=
4

t=

Lastly, the value of S at t =

= s

t =

5
4

= 32 units

5
4

5
sec
4

FG 5 IJ 16 25
H 4K
16

= 250 40

s=0

s = 250
t=0

5
secs earlier.
4
7. An aeroplane moves a distance of (3t2 + 2t) ft in
t-seconds. Find its velocity when it has flown for
5 minutes.
Solution: let the distance moved by the aeroplane in
t-seconds be s.
s = 250 and the motion started

s = 3t 2 + 2t
ds
= 3 2 t + 2 = 6t + 2
dt
velocity of the aeroplane when it has flown for
ds
5 minutes i.e. 300 seconds = the value of
when
dt
t = 300.

LM ds OP
N dt Q

= 6t + 2
t = 300

t = 300

= 1802 ft/sec.

8. A point moves in accordance with the law v = a +


bt + ct2 and the initial values of the velocity and
acceleration are 3 ft/sec and 2 ft/sec2 respectively
and at the end of the first second the acceleration is
12 ft/sec2.
Find: (i) the velocity at the end of 3 seconds:
(ii) the acceleration at the end of 4 seconds.
Solution: we are given v = a + bt + ct2
(1)
Differentiating both sides of (1) w.r.t t we have the

dv
= b + 2ct
(2)
dt
Now we will find the values of constants a, b and
c from the given initial conditions of the problem.
acceleration =

Initial velocity = 3 ft/sec2 which v

t =0

Initial acceleration = 2 ft/sec2 which

= 250 + 50 25 = 275 units


(v = 0)
s = 275

t = 5
4

657

= 3 (3)

LM dv OP
N dt Q

=2
t =0

(4)
(3) v

t =0

= a + bt + ct

a=3

2
t =0

=3

658

How to Learn Calculus of One Variable

(4)

LM dv OP
N dt Q

= b + 2 ct
t =0

=b=2
Again, acceleration = 12 ft/sec2 at the end of first
second
acceleration = 12 ft/sec2 when t = 1 which

LM dv OP = 12
N dt Q
dv
(5) L O = b + 2 ct
MN dt PQ

expression obtained for

(5)

t =1

t =1

t =1

= b + 2c = 12

(6)

Now putting the value of b in (6), we have


2 + 2c = 12 which 2c = 12 2 = 10 c = 5
Required velocity when t = 3 =[v]t = 3
= a + bt +

a =3
2 b=2
ct c = 5
t =3

Examples worked out


1. If a body moves according to the law s = a + bt +
ct2, show that its acceleration is constant.
2. If s2 = at2 + 2bt + c, show that acceleration varies

1
as

3 .
s
Solution: 1. s = a + bt + ct2

ds
= b + 2ct
dt
d ds

= 2c
dt dt

F I
H K

= [3 + 6 + 45] = 54 ft/sec.

L dv O
Required acceleration when t = 4 = M P
N dt Q

2
dy
d y
and
.
2
dx
dx

= [3 + 2t + 5t2]t = 3
= [3 + 2 3 + 5 9]

= b + 2 ct

2
dy
d y
and
, we arrive at our
2
dx
dx
target (required law or formula) using various
mathematical manipulations (like simplification,
cancellation or substitution etc) performed upon the

2. After obtaining

t=0

t =4

b=2
c=5
t =4

d s

dt
(proved).

= acceleration = 2c = k (say) = constant

2. Given that s2 = at2 + 2bt + c

ds
= 2at + b
dt
ds
s
= at + b
dt
2s

=2 + 2 5 4
= 42 ft/sec2.
Conditional Problems

(1)

When a law of motion of a particle (or, body) is given


by a formula y = f (x) (or the law of motion stated in
words be written in the symbolic form) and we have
to show that distance, velocity or acceleration of the
particle (or, body) obeys a differential equation of
motion we adopt the following working rule.

ds ds
d s

+ s 2 = a
dt dt
dt
(differentiating (1) w.r.t t)

F ds I
H dt K

+s

d s
dt

dy
d y
and
by differentiating the given
2
dx
dx
formula given in words translated into symbolic from
or the given law y = f (x) with respect to the given
independent variable.
1. Find

d s
dt

=a

=a

F ds I
H dt K

=a

aat + bf

FG3 ds = at + b from a1fIJ


K
H dt s

659

Derivative as Rate Measurer

dt

j e

ac b

2 2

s
2

d s
2

1
line be ks = log , prove that acceleration f is given
v
2
by f = kv ; s and v represent distance and velocity
respectively.
1
= log 1 log v = log v ( 3 log
v

1 = 0)
(1)
v=e
ks
dv

= ke
= kv (from (1)
(2)
ds
dv
2
v
= kv (multiplying both sides of (2)
ds
by v)

FG3 f = v dv IJ
H
ds K

4. If the velocity of a point moving in a straight line is


given by v 2 = ses, prove that the acceleration is

F
H

I
K

1
1
2
v , where x is a constant.
1+
s
2
Solution: v2 = ses

F I a fa
H K

dv d ds 1
=
= 1 as + b
dt dt dt
2

1
dv d s
=
= as + b
2
dt dt 2

(1)

f FH a dsdt IK
2

1
as + b
2

F3 ds = 1 aas + bf from a2fI


H dt 2
K
1
a f 1
a f af
= a aas + bf = a {aas + bf
}
4
4
1
af
= a a2v f
4
1
= a a2v f (from (2))
4
1

ks

dt
x
3. If the law of motion of a point moving in a straight

f = kv

5. If the law of motion is t = as2 + 2bs + c, show that


the acceleration varies as v3 and has a sign opposite
to that of a.
Solution: t = as2 + 2bs
(1)
dt

= 2as + 2ab
ds
ds
1
1
1
v=
=
=
as + b
(2)
dt 2as + 2b 2

Solution: ks = log

(proved).

(where k = ac b2 (say))

= at + 2bt + c

ac b

d s
dt

e j= d

d v

d
ds
s e j = s e + e = se + e
e
dt
ds
ds
ds
dv
v F
v I
2v
= as + 1f
3 from a1f e =
G
J
ds
s H
sK
dv 1 F
1I
v
=
1+
v
ds 2 H
sK

d s
dt

a at + 2bt + c a t + 2abt + b

as at + b

e3 s
=

d s

1 3

1
3
a 8v
4
= 2av3
=

= kv3 (where k = 2a)

dv
varies as v3
dt
Also, it is clear that if a is +ve, the acc. is ve and
if a is ve, the ecc. is +ve, i.e., the acc. is opposite to
the sign of a. (proved).
which

660

How to Learn Calculus of One Variable

6. If the equation of a rectilinear motion be s = t +1


where s is the displacement and t the time, show that
the acceleration is negative and proportional to the
cube of velocity.
Solution: s = t +1

a f
dv d F ds I F 1 I F 1 I
a= =
=

at + 1f
dt dt H dt K H 2 K H 2 K
1
= a2v f [from (1)
4
v=

ds 1
1
= t +1 2
dt 2

(1)
32

= 2v3 which is negative


= kv3 (where k = 2)
which acceleration = a varies as cube of velocity
and is negative.
7. Prove that if a particle moves so that the space
described is proportional to the square of the time of
description, the velocity will be proportional to the
time and the rate of increase of the velocity will be
constant.
Solution: let s = the space described by the particle
in time t.
st

s = kt where k = a constant
ds

= 2 kt
dt
ds
= v = velocity of the particle)
v = 2 kt (
dt
(1)
vt

dv
= 2 k (differentiating (1) w.r.t t)
dt

dv
a constant
dt
rate of increase in velocity = constant.
8. Prove that if a particle moves so that the space
described is proportional to the cube of the time of
description, the velocity will be proportional to the
square of the time and acceleration will be
proportional to the time.

s t3
s = kt 3 (where k = constant)

ds
= 3kt 2
dt

ds
t2
dt

(1)
(2)

d 2s
d 2s
= 6 kt 2 t .
2
dt
dt
9. A moving body describes a distance x which is
proportional to sin at in time t. prove that the
acceleration will be proportional to the distance
travelled by the body.
Solution: let x = distance described in time t
Given that x sin at
and

x = k sin at

dx
= ak cos at
dt

d x

( 1)

dt

= a k sin at = a x (from (1))

Again,

Solution: According the given law

d x

x (proved).
dt

N.B.: The motion is simple harmonic.


Type 1: Problems based on area, perimeter and
volume
Exercises 17.1
(A) Problems based on triangle
1. If the side of an equilateral triangle increases
uniformly at the rate of 3 ft/sec, at what rate is the area
increasing when the side is 10 ft?
(Ans. 15 3 sq. ft/sec)
2. If the side of an equilateral triangle increases at
the rate of 3 ft per second and its area at the rate of
12 sq. ft per second, find the side of the triangle.
(Ans. x = side of the = 8 ft)

Derivative as Rate Measurer

3. The area of an equilateral triangle is expanding.


How many time, as fast as each of its sides is the area
increasing at any instant? What is the rate of increase
of the area when each of the equal sides is 15 inches
long and increasing at the rate of 10 inches a second.
(Ans.

3
times as fast as each of its sides;
2

75 3 square inches per sec)


4. A triangle whose sides are varying with time is
always equilateral. The rates at which the area and
the height increase simultaneously at an instant at an
instant are 6 m2/sec and 3 m/sec respectively. Find
the rate of increase of the side at that instant.
(Ans. 2 m/sec)
(B) Problems based on square and rectangle
1. The side of a square sheet of metal is increasing at
3 cm/minute. At what rate is the area increasing when
the side is 10 cm long.
(Ans. 60 cm per minute)
2. The sides of a square plate of metal are expanding
uniformly at the rate of 0.3 cm per second. Find the
rate at which its area is (i) 30 cm (ii) 50 cm.
(Ans. (i) 18 sq cm/sec (ii) 30 sq cm/sec)
3. A square plate of metal is expanding and each of
its sides is increasing at a uniform rate of 2 inches per
minute. At what rate is the area of the plate increasing
when the side is 20 inches long?
(Ans. 60 sq. cm per second)
4. A rectangle is of given perimeter p. Find the rate of
change of the area at the instant when the length
equals the breadth.
(Ans. 0 = zero)
5. The breadth of a rectangle is increasing at the rate
of 2 cm per second and its length is always 3 times its
breadth. When the breadth is 5 cm, at what rate is the
area of the rectangle increasing.
(Ans. 60 sq. cm per second)
(C) Problems based on circle
1. A balloon which always remains spherical has a

3
(2x + 3). Determine the rate of
2
change of its volume with respect to x.

variable diameter

661

2. The radius of a circle is increasing uniform by at


the rate of 3 cm/sec. At what rate is the area increasing
when the radius is 10 cm.
(Ans. 60 cm/sec2)
3. If the radius of a circle increase at a uniform rate of
6 cm per second, find the rate of increase of its area
when the radius is 50 cm.
(Ans. 600 cm2/sec)
4. If the radius of a circle is increasing at the constant
rate of 2 ft per second, find the rate of increase of its
area when the radius is 20 ft.
(Ans. 80 ft2/sec)
5. If the circular waves in a tank expand so that the
circumference increases at the rate of a ft/sec, show
that the radius of the circle is increasing at the rate of

a
ft/sec.
2
6. The area of a circle is increasing at the uniform rate
of 5 sq. cm per minute. Find the rate in cm per minute
at which the radius is increasing when the
circumference of the circle is 40 cm.
1
cm/minute)
8
7. A spherical balloon is inflated and the radius is
(Ans.

1
inches/minute. At what rate would
3
the volume be increasing at the instant when its radius
is 2 inches.
16
inch3/minute)
(Ans.
3
8. A spherical balloon is pumped at the rate of 10
cubic inches per minute. Find the rate of increase of
its radius when its radius is 15 inches.
1
(Ans. 90 inch/minute)
9. If the area of a circle increase at a uniform rate,
show that rate of increase of the perimeter varies
inversely as the radius.
10. A spherical ball of salt is dissolving in water in
such a manner that the rate of decrease in volume is
proportional to the surface. Prove that the radius is
decreasing at a constant rate.
increasing at

662

How to Learn Calculus of One Variable

11. The volume of a spherical soap bubble is denoted


by v, its surface by S, the radius being r. Prove that

dv
ds 2 dv
2 dr
= 4 r
=
(ii)
.
dt
dt
dt
r dt
12. A sphere of metal is expanding under the action
of heat. Compare the rate of increase of its volume
with that of its radius. At what rate is the volume
increasing when the radius is 2 inches and increasing

(i)

at the rate of

1
inches per minute.
3
(Ans. 8 cubic inch/sec)

(D) Problems based on cube


1. An edge of a variable cube is increasing at the rat
of 3 cm/sec. how fast is the volume of the cube
increasing when the edge is 10 cm long?
(Ans. 900 cm3/sec)
2. When a cubical block of metal is heated, each edge
increases .1 percent per degree of rise in temperature.
Show that the surface increases .2 percent and the
volume .3 percent per degree.
3. A metal cube is heated so that its edge increases
at the rate of 2 cm/minute. At what rate the volume of
the cube increases when the edge is 10 cm long?
(Ans. 2400 cubic cm/minute)
4. The volume of a cube increases at a constant rate.
Prove that the increase in its surface varies inversely
as the length of the side.
5. The temperature of a metal cube is being raised
stead by so that each edge expands at the rate of .01
inch per hour. At what rate is the volume increasing
when the edge is 2 inches.
(Ans. 12 cu. in per hour)
(E) Problems based on cylinder
1. A cylindrical vessel is held with its axis vertical.
Water is poured into it at the rate of one point per
second. Given that one point is equal to 34.66 cubic
inches, find the rate at which the surface of water is
rising in the vessel when the depth is x inches.

34.66
inch per second where A is
A
the area of cross-section)
2. Water is running out of cistern in the form of an
inverted right circular cone of semi vertical angle 45
with its axis vertical. Find the rate at which the water
(Ans.

is flowing out at the instant when the depth of water


is 2 ft; given that at that instant, the level of the water
is diminishing at the rate of 3 inches per minute.
(Ans. ft3 /minute)
3. The volume of a right circular cone is constant. If
the height decreases at a constant rate of 8 cm/minure,
how fast is the radius of the base changing at the
instance when the height is 8 cm and the radius of the
base is 4 cm.
(Ans. 2 cm/min)
Type 2: Problems based on right angled triangle
Exercises 17.2
(A) Problems based on the height of a man
1. A man of height 6 ft walks directly away from a
lamp post of height 15 ft at the rate of 3 miles per hour.
At what rate does his shadow lengthen?
(Ans. 2 miles/hour)
2. A man 1.6 m high walks at the rate of 50 meters per
minute away from a lamp which is 4m above the
ground. How fast is the mans shadow lenghtning?

1
ft approaches directly towards
2
a lamp-post along a horizontal road. If the light is 8 ft
above the level of the road, show that the length of

3. A man of height 5

11
times the rate
25
at which be approaches the lamp-post.
4. A man 6ft tall walking away along a straight line
from the foot of the light post, 30 ft high at the rate of
4 ft per second; find how fast be is approaching the
wall.
this shadow decreases at a rate of

(Ans.
5. A man of height 5

5
miles/hour)
4

1
ft walks directly away from a
2

1
miles per
2
hour; find how fast is his shadow lenthening.
lamp-post of height 120 at the rate of 4

(Ans.

96
miles/hour)
58

Derivative as Rate Measurer

6. A man 160 cm tall walks from a lamp-post 4m high


at the rate of 3 km/hr. Find the rate at which the shadow
of his head moving on the pavement and the rate at
the which his shadow is lengthening.
(Ans. (i) 5 km/hr (ii) 2 km/hr)
7. A man 6 ft tall walking away along a straight line
from the foot of light-post, 30 ft high at the rate of 5
miles per hour. How fast does the end of this shadow
move?

5
miles/hour)
4
8. A man 6 ft tall walks at the rate of 5 ft/sec towards
street light 16 ft above the ground. At what rate is the
tip of his shadow moving? At what rate is the length
of his shadow changing when ne is 10 ft from the foot
of the light?
(Ans. 8 ft/sec and decreasing at 3 ft/sec)
(Ans.

(B) Ladder problems


1. A ladder 13 ft long slides down from a vertical wall
remaining in a vertical plane all the time. What is the
velocity of the upper end when the lower end is at a
distance of 5 ft from the wall and has a velocity of 2 ft
per second?
(Ans. 5 ft/sec)
2. A ladder is inclined to a wall making an angle of
30 with it. A man is ascending the ladder at the rate
of 3 ft/sec. how fast is he approaching the wall.

2
ft/sec)
3
3. A ladder is inclined to a wall making an angle of
45 with it. If a man is ascending the ladder at the rate
of 4 ft/sec; find how fast he is approaching the wall.
(Ans.

(Ans. 2 2 ft/sec)
4. A ladder 26 ft in length is resting on a horizontal
plane inclined against a vertical wall. It slips away
from the wall at the rate of 5 ft/sec. Find the velocity
of the top of the ladder down the wall when it is at a
height of 24 ft.

25
ft/sec)
12
5. A ladder 5 meter long standing on a horizontal
floor leans against a vertical wall. If the top of the
ladder slides downwards at the rate of 10 cm/sec.
(Ans.

663

Find the rate at which the angle between the floor and
the ladder is decreasing when the lower end of the
ladder is 2 meter from the wall.

1
radius/sec)
20
6. A ladder 25 ft long slides down from a vertical wall
remaining in a vertical plane all the time. What is the
velocity of the upper end when the lower end is at a
distance of 15 ft from the wall and has a velocity of 3
ft/sec.
(Ans.

(Ans.

9
ft/sec)
4

(C) Kite problems


1. A kite is 45 ft high and there is 117 ft cord out. If
the kite is moving horizontally at the rate of 13 m.p.h
directly away form the position who is flying it, find
how fast the cord is being paid out.
(Ans. 19 m.p.h)
2. A kite is 100 ft high and there is 260 ft of cord out.

1
4
miles per hour directly away from the person who is
flying it; how fast is the cord being paid out.
(Ans. 3 miles/hour)
3. A kite is moving horizontally at a height of 151.5
meters. If the speed of the kite is 10 meters/sec, how
fast is the string being paid out when the kite is 250
meters from the boy who is flying the kite, the height
of the boy being 1.5 meters.
(Ans. 8 m/sec)
4. A girl flies a kite at a height of 300 ft; the wind
carrying the kite horizontally away from her at a rate
of 25 ft/sec. How fast must she let out the string when
the kite is 500 ft away from her.
(Ans. 20 ft/sec)
if the kite is moving horizontally at the rate of 3

(D) Rod Problems


1. A rod 13 ft long moves with its ends A, B on two
perpendicular lines OX and OY respectively. If the
end A is 12 ft from O and is slipping away at 2

1
ft/
2

sec; find how fast end B is moving?


(Ans. 6 ft/sec)

664

How to Learn Calculus of One Variable

2. Two rods AB and AC are inclined to each other at


an angle of 120. A car starts from B towards A with a
velocity of 40 miles an hour and an other can starts
from C towards the same place with an equal velocity
at the same. If AB = AC, find the rate at which each is
approaching the other.
(Ans. 40 3 m/h)
3. Two straight rods OA and OB cross each other at
O at right angles; If OA = 10 miles and OB = 8 miles, a
man starts at the rate of 3 miles per hour from O to B.
Find the rate at which his distance from A is altering.
(Ans.

3x
10 x + x

4. A rod AB of length 10 ft slides with ends A and B


on two perpendicular OX and OY respectively. If the
end A on OX moves with a constant velocity 2 ft/
minute, find the velocity of its med-point at the time
the rod makes an angle of 30 with OX.
(Ans. 2 ft/minute)
Problems based on velocity and acceleration
Exercise 17.2.1
1. A point moves in a straight line according to the
law:
S = 2t3 + t2 4
Find its velocity and acceleration at the instant of
time t = 4.
(Ans. (i) v (4) 104 m/s (ii) a (4) = 50 m/s2)
2. A point moves in a straight line as given by the
equation:
S = 6t t2
At what instant of time will the velocity of the
point be equal to zero?
(Ans. t = 3 second)
3. Find the velocity and acceleration at the indicated
instants of time for a point moving in a straight line if
its motion is described by the following equations.
(i) S = t3 + 5t2 + 4, t = 2
(ii) S = t , t = 1
(iii) S = t2 + 11t + 30, t = 3
(Ans. (i) 32 m/s; 22 m/s2 (ii) 0.5 m/s; 0.25 m/s2
(iii) 17 m/s; 2 m/s2)

4. At time t, the distance x of a particle moving in a


straight line is given by x = 4t2 + 2t. Find the
1
velocity and acceleration when t = .
2
(Ans. 6, 8)
5. The distance S, at the time t, of a particle moving in
a straight line is given by the equation S = t4 18t2.
Find its speed at t = 10 seconds.
(Ans. 340 units/sec)
6. A particle is moving in a straight line in such a way
that its distance in cm from a fixed point on the line
after t seconds is given by 4t3 + 2t + 5. Find the
distance, velocity and acceleration at the end of 3
seconds.
(Ans. (i) 119 cm (ii) 100 cm/sec
(iii) 72 cm/sec2)
7. The distance S meters moved by a particle
travelling in a straight line in t seconds is given by S
= 45t + 11t2 t3. Find the time when the particle comes
to rest.
ds
[Hint: solve
= 0]
dt
(Ans. 9 seconds)
8. A particle is moving on a line where its position S
in meters is a function of time t in seconds given by S
= t3 + at2 + bt + c, where a, b, c are constants. It is
known that t = 1 second, the position of the particle is
given by S = 7 meters, velocity is 7 m/sec and the
acceleration is 12 m/sec2. Find the values of a, b and
c.
(Ans. a = 3, b = 2, c = 5)
9. Find the acceleration of a moving point at the
indicated instants of time if the velocity of the point
moving in a straight line is given by the following
equation:
(i) v = t2 + t 1, t = 3
(ii) v = t2 + 5t + 1, t = 3
(Ans. (i) 7 m/s2 (ii) 11 m/s2)
10. A point moves in a straight line according to the
law S = t2 8t + 4. At what instant of time will the
velocity of the point turn out to be equal to zero?
(Ans. t = 4s)
11. A point moves in a straight line according to the
law:
S = sin2 t

Derivative as Rate Measurer

665

Find the instant of time at which its acceleration is


equal to 1.

More problems on physical application of derivatives

+ k , k z )
6
12. A point moves in a straight line according to the
law:
S = sin2 t
Find the instant t at which its acceleration is equal
to zero.

Exercise 17.2.3

(Ans. t =

(Ans. t =

k
+
, k z )
4
2

1 3
t 16t , find the acceleration at the
3
time when the velocity vanishes.
(Ans. 8 units/sec2)

13. If S =

1. The law of change of temperature T of a body with


time is given by T = 0.2t3. At what rate does this body
get warm at the instant of time t = 10?
2. A body of mass 10 kg moves in a straight line
according to the law:
S = 3t2 + t + 4
2
mv
Find the kinetic energy of the body
four
2
seconds after it started.

F I
GH JK

a f LMN dsdt OPQ

[Hint: (i) v 4 =

= 6t + 1
t =4

t =4

= 25 m/sec

(ii) Determine the kinetic energy of the body at t = 4


Conditional problems

Exercise 17.2.2
1. If a particle vibrates according to the law:
y = a sin (Pt e), show that the velocity and
acceleration at any instant are aP cos (Pt e) and
P2 y respectively.
2. The motion of a particle moving in a straight line is
given by x = 3 cos 2t with usual symbols. Show that
its acceleration is proportional to the distance
travelled by the particle and determine the distance x
when the speed is zero.
3. If a particle moves so that the space described
varies as the square of the time of description, prove
that the velocity varies as the time and the acceleration
is constant.
4. If the law of motion is t = s2 + s 1, show that
acceleration varies as v3.
5. If the law of motion is t = as2 + bs + c, show that
the rate of change of velocity is proportional to the
cube of the velocity and has a sign opposite to that
of a.
6. If t = 2s2 + 3s + 1, show that acceleration is
proportional to the cube of the velocity.

af

25
mv
= 10
= 3125 J ]
2
2
3. The strength of current I changes with time t
according to the law:
I = 0.4t2, (I is expressed in amperes, t in seconds)
Find the rate of change in strength of current after
expiry of the 8th second.
which is

[Hint:

F dI I
H dt K

a f

= 0.8t
t =8

t =8

= 0.8.8 = 6.4 A/m]

4. The temperature T of a body changes with time t


obeying the law:
T = 0.5t2 2t
At what rate does this body get warm at the instant
of time t = 5?
(Ans. 3 deg/s)
5. A body of mass 100 kg moves in a straight line
according to the law:
S = 5t2 2
Find the kinetic energy of the body in two seconds
after the beginning of motion.
(Ans. 20,000 J)
6. The change in the strength of current I with time t
is given by the equation I = 2t2 5t (I is amperes, t in
seconds). Find the rate of change in the strength of
current after the expiry of 10th second.
(Ans. 25 A/S)

666

How to Learn Calculus of One Variable

18
Approximations

Mathematics is a language.
J. Willard Gibbs

af

Let y = f (x)
(A) We should recall that

y
dy
= lim
dx x 0 x

(1)

when x is very small , we write,

dy y
=
(nearly)
dx x

af

(2)

dy
(3)
x (when x is small)
dx
y
dy
= lim
(B) Again we should recall
(1)
dx x 0 x
y
when x is not sufficiently small, then
will in
x
y =

af

general differ from f x . If is the difference


y
dy
between
and
, we have
x
dx
y dy

= which implies
x dx

y dy

+
x dx
y

= f x +
x

af

(where 0 as x 0 )

(3)
y = f x x + x
where the second term on the r.h.s of the equation (3)
of (B) is very small and can be neglected (since 0
and x 0 x 0 on using the product
theorems on limits) and the first term f x x is
the larger part of y (in equation (3) of (B)) which is
known as the principal part. The principal part y is
called the differential of y and is denoted by dy i.e.

(2)

af

But if we let dy = f x x

(4)

x = dx , using this result in (4) for y = x, we


have, therefore, dy = f x dx which tells the
differential of y is obtained by differentiating the given
function f (x) w.r.t its independent variable x and then
multiplying d.c of that function f x by the
differential of x (i.e. dx).

af

af

(C) We know that

y
dy
= lim
dx x 0 x

(1)

and when x is very small,


dy y
=
(approximately)
dx x
dy
y =
x (nearly, approximately, approx)
dx
dy
f x + x f x =
x
dx

f af

667

Approximations

f af

dy
x which prodx
vides us an approximate formula and can be written in
following way:
f x + x = f x +

af

af

f a +h = f a + f a h

where x = a and x = h are given numerical values.

af

af

aa hf , tan aa hf , cot aa hf , sec aa hf ,


cosec aa h f , sin aa h f , cos aa h f , tan
aa hf , cot aa hf , sec aa hf , cosec
aa hf , log aa hf , e a f , etc. some examples.
1

Remember:
1. If the increament of the variable on which y depends
(i.e. x ) is small enough then y ~ dy .

af

4. Use the formula f a h = f a h f a


when f (a), f a and h are known. h is a small number
which is positive or negative and a h = given
number for the independent variable. f a h
n
= n a h , a h , log a h , sin a h , cos
1

ah

2. The fact that differential dy is the approximation of


y when dx is small may be used to approximate
errors.

Which provides us the required approximate value


of the function for the given value of the independent
variable.

y dy
(nearly) if the increament of the variable
=
x dx

Second working rule:


1. In finding the value of f (c) approximately we
express the given number c as a n to choose
x = a s.t f (a) can be determined as a rule as a whole
number b where a = any number whose nth root,
power n, t-ratio, inverse t-ratio, log, e, etc are known
to us and n = x = h = any number +ve or ve. After
this step, we

3.

on which y depends is very small (or small enough).

af

4. y = f x + x f x dy in general.
5. Relative error in y; let y = f (x), then relative error in
y at x = a is

bg
bg

f a
h
f a

2. Use the formula y =

af

derivative of the function f x at x = a


times the increament
=
value of the function at x = a
Now we shall explain different types of problems
on approximations and errors besides their techniques
to solve them.
Type A
To find the approximate value of a function of an
independent variable x when the independent variable
x is replaced by a number, we adopt the following
working rule:
First working rule:
1. For finding f (c) we choose a and h such that f (c)
= f (a + h) where f (a) is easily obtainable and h is
small

af

2. Find f x by differentiating f (x)

af

3. Find f (a) and f a

LM dy OP
N dx Q

af

x = f a h.
x =a

3. Lastly are find the required approximate value using


the formula:

af

af

y + y = f a + f a h
where

y + y = f (a + h)

Note:
1. There are two types f notations for the approximate
value of a function of an independent variable.

(i) y + y (ii) f x + x or f (a + h) where x = h


is written for easiness and y =

af

LM dy OP
N dx Q

x
x=a

= f a h.
2. Generally h = a decimal fraction +ve or ve or an
integer +ve or ve like 0.001, 0.002, 0.003,
0.009 incase of given number c is a decimal fraction
and 1, 2, etc in case of given whole number c.
3. If h = ve integer or ve decimal fraction, we use
the formula:

668

How to Learn Calculus of One Variable

af

af

af

af

f ah = f a h f a
and if h = +ve integer or +ve decimal fraction, we use
the formula:
f a+h = f a h f a
4. Usually incase of decimal fraction c the value of a
is taken as the nearest integer. a = 2 is to be considered
so that a + h = 1.999 gives us h = 0.0001 (3 2 0.001
= 1.999), if c = 1.999.
5. Usually incase of whole number a to be
considered, we express the given number c as a n
where x = a is s.t. f (a) is known to us already where

f n

, ( )n, sin, cos, sin1, cos1, log, e or any

other operator.
6. When y = f (x), then x = dx and y = dy
provided that x is small enough (or, provided that
x is nearly equal to zero which means x 0 ).

y dy
nearly x is small enough.
=
y
y
8. Notation = , ~ , = or means approximately
equal to.
7.

Some useful hints to find the values of a and h in


some problems
1. If a given number c = b +

or b +

where
n
10
k = digits from 1 to 9, b = any whole number, n = +ve
integer, then b = a = x.

x =

1
10

or

10

10n

and given number = x + x

where x = + ve
e.g.: 3.003 = 3 +

= x + x or a + h if x = a = 3

10 n

and x = h = 0.003
Given number = 3.003 = 3 + 0.003
.

2. If a given number c = b + 9 ... where dots after


.

recurring decimal 9 denote any one or more than


one digit from 1 to 9, then x = a = b + 1 and x = h
= given decimal fraction (b + 1)

FG
H

IJ b g
K
given number = bb + 1g + x where x = ve
.

= b + 9 ... b + 1

e.g.: 31.98 + 31 + .98 x = a = b + 1 = 31 + 1 = 32


(where b = 31) and x = 31.98 32 = 0.002 = ve
decimal fraction given number 32 + x = 32 0.002
3. If a given number =

9 k l m ...

10
= digits from 1 to 9 then x = 1
x = h =

where k, l, m,

9 k l m ...

1
n
10
given number = 1 + x where x = ve
e.g.: x = a = 1 and x = 0.998 1 = 0.002
given number = 1 + x = 1 0.002
N.B.: Type (A) has two types of problems which are
explained below
Type 1: To find the approximate value of a function
when the independent variable x is replaced by a
whole number, decimal fraction or a whole number +
decimal fraction.
Type 2: Conditional problems.
Problems based on finding the approximate value of
logarithmic, trigonometric, inverse trigonometric
function of an independent variable replaced by a
number.
Examples worked out:
1. Given loge 2 = 0.6931, find the approximate value of
loge 2.01.
Solution: 2.01 = 2 + 0.001
(1)
x = 2 (= a)
x = 2.01 2 = 0.01
(2)
Now on letting y = loge x, we have
dy d log x 1
=
=
dx
dx
x

a f LMN dydx OPQ


L dy O
Now, y = M P
N dx Q
f a =

=
x=a

LM 1 OP
NxQ

=
x=2

1
2

bg

x = f a x
x =2

669

Approximations

af

1
0.01 (from (2))
2

f a =

hence, required approximate value = y + y

1
= log 2 + 0.01
2
= 0.6931 + 0.005
= 0.6981
2. Given 10 = 0.0175C, find the approximate value of
cos 610.
Solution: letting, f (x) = cos x where given number
= 60 + 1 = x + 1 which x = 10 = 0.0175 ,

b g
Further, f a x f = sin x
f ba g = cos60
f ba g = sin 60
now, y = f aa f x
x=

=a
3

3
0.0175
2
Hence, required approximate value
x =a

F
GH

I
JK

3
0.0175
2

1
3

0.0175
2
2
1
=
1 3 0.0175
2
3. Find the approximate value of tan1 (0.99).
Solution: y = f (x) = tan1 x
=

af

f x =

1
1+ x

now expressing 0.99 as 1 0.01, we have


a = 1, and h = x = 0.01

a f af

1
1
1
=
=
1+ 1 1+ 1 2

af

af

Hence, f (a + h) = tan1 (0.99) = f a + f a h


1
= + 0.01
4 2

= 0.005 .
4
Derivation of a formula for approximate calculation
of powers
To compute the approximate value of the function
f (a + h) = (a + h)n
On applying the formula f (a + h) = f (a) + h f a ,
we have

af

af

Working rule: For finding f (c)


1. Express the given number c as a + h. i.e., given
number c = a + h (proper a)
2. Put f (x) = xn and find f (a). Moreover differentiate
the function f (x) = xn and find f a .
3. Find h from the formula:
h = given number a
4. Lastly, we use the formula

af

a f af
af
i.e.; aa + hf = a + n a
to get the required
f a+h = f a +h f a
n

n 1

approximate value of the power of the given number


used as base of the power.

f a = f 1 = tan

af

f 1=

a f a f af

1+ a

1
n
f a +h = a +h ; f x = x , f a =na ,
Hence, (a + h)n = an + nan1 h (approx)
Where x = a = a whole number which is nearly
equal to (or, approximately equal to) given number
which means a may be slightly (or, alittle) greater than
or less than the given number.
and h = given number a.
Hence, in the light of above explanation, we provide
the following working rule for approximate calculation
of powers.

= sin 60 0.0175

= y + y = cos x

af

1 =
4

Problems based on approximate calculations of


powers
Examples worked out:
1. Find the approximate value of (0.998)8
Solution: 0.998 = 1 0.002 = a + h

670

How to Learn Calculus of One Variable

Let f (x) = x8

af

f x = 8x
taking x = a = 1 and h = 0.002

af

f (1) = (1)8 and f 1 = 8 (1)7 = 8 1 = 8


(0.998)8 = (1 0.002)8 = f (1 0.002)
on using the formula

af

af

f a h = f a h f a i.e. (a + h)n = an + n
an 1, we have

b g bg

bg

f .998 = f 1 .002 f 1 (approx)


2
=1
8
1000
= 1 0.016
= 0.9840 (approx).
2. Find the approx value of (0.9999)6
Solution: 3 0.9999 = 1 0.0001
(0.9999)6 = (1 0.0001)6
Let f (x) = x6

af

f x = 6x
taking x = a = 1 and h = 0.0001
f (a) = f (1) = 16 = 1

af

af

af

and f a = f 1 = 6 1 5 = 5
now, using the formula

af

af

f a h = f a h f a i.e. (a + h)n = an + n
an1, we have
f (1 0.0001) = (1 0.0001)6
= (1)6 0.0001 6
= 1- 0.0006
= 0.9994 (approx).
Note: 1 is the integer nearest to .999.

Find the approximate value of (1.999)6.

3.
Solution: First method:
3 1.999 = 2 0.001
(1.999)6 = (2 0.001)6
5
let f (x) = x6 f x = 6 x
Taking a = 2 and h = 0.001
f (a) = f (2) = 26 = 64
f a = f 2 = 6 25 = 6 32 = 192

af

af

af

now, using the formula


f (a h) = f (a) h f a i.e. (a h)n = an n an1
h, we have
(2 0.0001)6 = (2)6 0.001 192
= 64 0.192
= 63.808
Second approach
Let y = x6

af

dy
5
= 6x
dx

af

af

f a = f 2 = 6 2 = 192
again a + x = 1.999

2 + x = 1.999
x = 1.999 2 = 0.001
now, on using the formula

af

y = f a x
= 192 (0.001)
= 0.192

Required approximate value = y + y


= 26 0.192
= 192 0.192
= 63.808
4. Find the approximate value of (4.012)2
Solution: 3 4.012 = 4 + 0.012
(4.012)2 = (4 + 0.012)2

af

let f (x) = x2 f x = 2 x

af

taking x = a = 4 and h = 0.012, f (4) = 42 = 16 and f 4


= 2 4 = 8. Hence approximate value of f (a + h) = f (a)

af

+ h f a i.e.; (a + h)2 = a2 + 2ah


(4.012)2 = (4 + 0.012)2 = 42 + 2 4 0.012
= 16.096 (approx).
Deriving formula for approximate solution of
equation
Let a root of the equation f (x) = 0 be approximately
equal to a.
We are able to obtain a better value of the root by
using the concept of derivative and differential.

671

Approximations

Let ' a + a ' be the exact root,

Then f a + a = 0

af

(1)

af b
af g
(2)
f aa + a f f aa f = f aa f a

Again, f x = f x x or y = f x x
[since the increament in the function is

af

Second method:
By the use of the formula:

af

f (a + h) = f (a) + h f a also, we can find


approximate nth root of the given number where

af

f a + a f a

f n and hence f (a + h) = f (a) + h f a can be


written as

as x changes from a to a + a ]

N.B.: The above method is practically more easier


than any other method to find the nth (approximate
value of the) root of a given number.

now putting f a + a = 0 , from (1), into (2), we


have

af af
f aa f = f aa f a
f aa f
a =
f aa f

(given number) n = a + a

1
n

0 f a = f a a

(3)

on adding a to both sides of (3), we have the required formula a + a = a

af
af

f a
f a

which a

better approximation of the root ' a + a ' is

af
af

f a
f a

This formula a + a = a

(4)

b g is fruitful for
f ba g
f a

approximate computation of roots.


Hence, in the light of above explanation, we can
provide a rule to find the nth approximate root of a
given number.
Working rule:
1
1. Let x = (given number) n and x = a, a whole number
whose nth power is approximately equal to the given
number.
2. Solve the equation xn = given number
3. Put f (x) = xn and differentiating it (i.e. f (x)),find

af

af

f x and f a . Moreover we find f (a) from f (x).


4. Last we use the formula

af
af

f a
which provides us the required
f a
approximate value of the root of the given number.
a

a+

n 1

where h = given number a = increament and a = a


whole number approximately equal to the given number which means a whole number which is a little (or,
slightly) greater than or less than the given number
and which can be expressed as nth power (or, perfect
square incase of square root)
Remember: In the formula f (a + h) = f (a) + h f a

af

af

1. h f a denotes the differential of a function at


the value a of the independent variable x. Thus to
obtain the value of the differential of a function, it is
necessary to know two numbers: the value of the
independent variable x and its increament h.
e.g.: Calculate the differential of the function y = x2
for a change in x from 3 to 3.1.

af

af

Solution: dy = f x x = h f a

= 2x

x=a

= 2x

x =3

(the independent variable = a = 3 and h

= final value initial value = 3.1 3 = 0.1)


dy = 2 3 0.1 = 6 0.1 = 0.6
2. a always denotes the approximate value of an
independent variable x obtained as a result of
measurement and (a + h) denotes its true or given
value. then a determines the approximate value of
the function f (x) and (a + h) gives the value of the
function f (x + h).
3. Given value indicates the changed value or final
value of the independent variable so it must be
expressed as (a + h) so that we may have the initial
value as well as the increament of independent
variable.

672

How to Learn Calculus of One Variable

4. Increament may be positive or negative. If the final


value is greater than initial value, increament is
positive. If the final value is smaller than initial value,
increament is negative. Increament is always
determined by the formula h = final value initial
value = given number a.
Problems based on approximate calculation of roots
Examples worked out:

.
1. Find the approximate value of 3198
help of calculus.
Solution: First method:

1
5

with the

x 31.98 = 0
again let f (x) = x5 31.98
f x = 5x
f (2) = 32 31.98 = 0.02
4

f 2 = 5 2 = 5 16 = 80
Required approximate value of the root

af
af

=a

f a
f a

=2

0.02
80

160 0.02
80

159.98
80

Now, taking x = a = 32 and h = 0.02


f (a) = f (32) = 2

af

f a =

af

= f (a h) = f (a) h f a

= 2 0.02
2
8000
1
=2
4000
= 2 0.00025
= 1.99975

1
4

Let x = 80.999 4 x = 3 (approximately) = a


(say)
Now we have to solve the equation
x4 = 80.999
4

again let f (x) = = x4 80.999

af

f x = 4x
f (3) = 34 80.999 = 81 80.999 = .001

af

f 3 = 4 33 = 4 27 = 108
Required approximate value of the root

f = a32 0.02f
1
5

Now on letting f (x) = x

x 80.999 = 0

15.998
8
= 1.99975
Second method:
31.98 = (32 0.02)

1
5 16

=2

1
5

af
af

=a

f a
f a

=3

0.001
108

31.98

1
1
=
5 16 80

2. Find the approximate value of 80.999


Solution: First method:

af

1 45
1
x =
4
5
5 x5

Required approximate value of the root

Let x = 3198
. 5 x = 2 (approximately) = a
(say)
Now we have to solve the equation
x5 = 31.98

af

af

f x =

324 0.001
108

323.999
= 2.9999
108

1
5

Second method:
80.999 = (81 0.001)

af
1
f axf = x
4

43

4 x4

af

a f

a f

1
1

1000 108
= 3 0.00009259
= 2.99990741
=3

3. Find the approximate value of 145 .


Solution: First method:
1

Let x = 145 2 x = 12 (approximately) = a


(say)
Now we have to solve the equation
x2 = 145
2

x 145 = 0
again let f (x) = x2 145

af

f x = 2x
f (12) = 144 145 = 1
f 12 = 2 12 = 24
Required approximate value of the root

af
af
a1f
= 12
=a

f a
f a

24
1
= 12 +
24
=

288 + 1
24

289
24
= 12.04
=

1
2 x

Now using the formula,

80.999 = f 81 0.001 f 81

a f

x ,a

af

f (a h) = f (a) h f a , we have

a f

145 = 144 + 1 = f (a + h) where f (x) =


= 144, h =1
f x =

Now putting a = 81, h = .001 in


4

673

Second method:

1
4

Let f x = x

Approximations

af

f (a + h) = f (a) + h f a

a
f
2
1
f a145f = 12 +
2 12
f 144 + 1 = 12 + 1

1
24
= 12 + 0.04
= 12.04.

1
144

= 12 +

a f

4. Find the approximate value of 33 5 .


Solution: First method:

a f

Let x = 33 5 x = 2 (approximately) = a (say)


Now we have to solve the equation:
x5 = 33
5

x 33 = 0
Again let f (x) = x5 33

af

f a = 5x
f (2) = 32 33 = 1

af

f 2 = 5 2 4 = 80
Required
approximate value of the root

af
af
a1f
=2
f a 2f
F 1I
=2
H 80 K
=a

f a
f a

=2+

1
80

674

How to Learn Calculus of One Variable

Now we have to solve the equation


x3 = 215

160 + 1
80

x 215 = 0
Again we let f (x) = x3 215

161
=
80
= 2.0125
Second method:

af

f x = 3x

a33f = a32 + 1f
1
5

1
5

a f

and 32

f (6) = 23 215 = 216 215 = 1


1
5

af

=2

1
5

Hence, if we write f (x) = x , then 33 5 = f (a + h) for


a = 32, h = 1

af
1
f a xf = x
5

f 6 = 3 36 = 108
Required approximately value of the root

=a

f 2 = 25 = 2

af

f 2 =

45

=6

5x 5

=6

4
5

a f

5 32

Now, using the formula

af

bg
f ba g
f a6f
f a6f
f a

1
108

648 1
108

647
108
= 5. 9907
Second method:
=

f (a + h) = f (a) + h f a

f 32 + 1 = 2 +

1
1

5
32

b g

4
5

af
1
f axf = x
3
1

f x = x3

1
=2+
5 24
1
=2+
5 16
1
=2+
80
160 + 1
=
80

23

3x 3

Now, we write

a f

f (a + h) = 215
where f (x) =

1
3

x , a = 216, h = 1

e j

f (a) = f (216) = f (63) = 6

161
=
80

bg

f a =

161
.
8
= 2.0125.
=

a f

5. Find the approximate value of 215


Solution: First method:

a f

Let x = 215

1
3

1
3

x = 6 (approximately) = a (say)

e j

3 63

2
3

1
1
=
3 36 108
Now, using the formula,
=

af

f (a + h) = f (a) + h f a

1
3

=6

1
3 62

Approximations

f 216 1 = 6

648 1 647
1
=
=
1 108
108
108

= 5.9907 (approx)

a f

a f

x = 127
x = 5 (approximately) = a (say)
Now we have to solve the equation
x3 = 127
3

x 127 = 0
Again we let f (x) = x3 127

af

f x = 3x
f (5) = 125 127 = 2

af

f 5 = 3 25 = 75
Required approximate value of the root

bg
bg
f b5g
=5
f b5g
a 2f
=5
f a

f a

75
2
=5+
75

1 1
1
1
= 2 =
3 3 5
75

af

f 125 + 2 = 5 + 2

1
2
=5+
= 5.0266
75
75

Third method:

af

e j

3
Let y = f x = x = 5

1
3

=5

bg

dy
1
x = f a x =
2
dx
75
Required approximate value
= y + y = 5 + .0266 = 5.0266
y =

7. Find approximately 4 627 .


Solution: First method:

a f

Let x = 627 4 x = 5 (approximately) = a


(say)
Now we have to solve the equation
x4 = 627
4

x 627 = 0
Again on setting f (x) = x4 627
3

bg

bg

f a = f 5 = 4 53 = 4 125 = 500

377
75
= 5.0266
Second method:
Expressing given number = 127 = 125 + 2 = 53 + 2
On letting x = 53 = 125 = a (say)

Required approximate value


=a
=5

f x = x3

af

e5 j

2
3

Now, using the formula,

af

f x =

=5

f x = 4x
f (a) = f (5) = 625 627 = 2

375 + 2
75

af

b g

1
3

f (a + h) = f (a) + h f a , we have

1
3

=a

bg

1
3

f a = f 125 =

6. Find the approximate value of 127 3 .


Solution: First method:
Let x = 127

a f a f = e5 j

f a = 125

675

c 13 1h 1 23
1
x
= x
3
3

af
af
f a5f
f a5f
f a
f a

2
1
=5+
500
250
= 5.004
=5+

676

How to Learn Calculus of One Variable

Second method:
We observe that 627 is close to 625 of which fourth
root is 5.
3 627 = 625 + 2
on setting x = a = 625, h = 2

af af
1 c h
we have, f a x f = x
4
f aa f = f a625f = a625f = e5 j
1
f aa f = f a625f = a625fc h
4
1
4

f x = x

1
4

af

a f

af

8. Find the approximate value of 1006


.
.
Solution: First method:

0.006
2

af

f x =

We have: f (625 + 2) = f (625) + 2 f 625


= 5 + 2 0.002 = 5 + 0.004 = 5.004

(1.006) = 1 + 0.006 = f (a + h) where f (x) =


h = 0.006

=5

1
1
4 c 4 h
3
= 5
= 5
4
4
1
1
1
=
=
=
= 0.002
3
4 125 500
45
Now, using the formula: f (a + h) = f (a) + h f a

e j

2.006
= 1003
.
2
Second method:

34

=1+

0.006

43

1
4

=1

2
Let x = 1006
.
x = 1 (approximately) = a
(say)
Now, we have to solve the equation
x2 = 1.006

x , a = 1,

1
2 x

f (a) = f (1) = 1

af

af

1
2
Now, using the formula,
f a = f 1 =

af

f (a + h) = f (a) + h f a

= 1 + 0.006

1
2

2 + 0.006
= 1.003
2
Remember: The formula:
=

a + a = a

af
af

f a
f a

x 1.006
Again, let f (x) = x2 1.006

is also applicable to find the approximate root of the


equation f (x) = 0 which is nearly equal to a given root
x = a.

f x = 2x
f (1) = 1 1.006 = .006

Examples based on approximate solution of the


equation

af

af

f 1 = 21= 2

If is small, then we can use the formula:

1+ 1+

.
2

Required approximate value of the root

=a

af
af

f a
f a

1. Find the root of the equation x4 12x + 7 = 0 which


is near to 2.
Solution: 3 f (x) = x4 12x + 7 and a = 2

af

f x = 4 x 12

f (2) = 24 12 2 + 7 = 1

af

f 2 = 4 2 12 = 20
Required root = a

af
af

f a
(approx)
f a

Approximations

af
af
a1f
=2
=2

af

f 2
f 2

f x =

20
40 + 1 41
1
=2+
=
=
= 2.05
20
20
20
2. Find the root of the equation x4 12x2 12x 3 = 0
which is approximately 4.
Solution: Let f (x) = x4 12x2 12x 3 and a = 4

af
f a4f = 4 4 24 4 12

= 148
f (4) = 44 12 42 12 4 3

af

x
x

(3)

af

f x + x = f x + f x x , we have

1
1 x
2
x + x x
x

af

af

1
.
1004
.

Solution: 1.004 = 1 + 0.004


On setting x = a = 1, h = x = 0.004

af
af
f a4f
f a4f

af

f a
f a

f x =

1
x

af

f x =

1
2

x
f (a) = f (1) = 1

13
148

af

af
f (a + h) = f aa f = h f aa f , we have

f a = f 1 = 1
Now, using the formula,

579
148
= 3.91
=

Derivation of a formula for approximate calculation


of reciprocal quantities
Let us consider the function:
1
f x =
(1)
x
We let the argument x receive a small increament
x , then

af

Now using the formula:

1. Find the approximate value of

592 13
=
148

Examples worked out:

= 256 192 48 3
= 256 243 = 13
Required
approximate root

1
f x + x =
x + x

1 x

Problems based on approximate computation of


reciprocal quantities

= 256 108

=4

af

f x x =

[i.e.; f a + h f a + h f a ]

=4

f x = 4 x 24 x 12

=a

1
x

677

1
= 1 0.004 1 = 0.996
a+h
1
2. Find the approximate value of 4 when x = 2.04.
x
1
Solution: Let f (x) = 4
x
x = 2.04 = 2 + 0.04
on setting, x = a = 2, h = x = 0.004

af

3f x =
(2)

1
x

678

How to Learn Calculus of One Variable

af

N.B.: Sometimes x = h is provided in the given


problems to be approximated which means there is no
need of finding x .

f x =

a f 161
4
f b 2g =
2
f 2 =

Remember:
1. Approximate change in

1
8
Now, using the formula:
5

af

af

f a+h = f a +h f a

af

LM dy OP
N dx Q
= f aa f h
y=

af
1
F 1I
=
+ 0.04
H 8K
16

x = approx value of the given value for x

= f 2 +h f 2

2. Approximate value of y = y

1
0.005
16
1 0.005 16
=
16

dy, when x = given value


3. Given number (or value of x) may be whole number
or simply pure decimal fraction like 26, 65, 0.9993 etc,
then to find a and h we should consult the hints
given earlier in the topic on finding the approximate
value of a function of an independent variable replaced
by a number (i.e. in type (A)).
4. Given value of x always requires a result of
applying increament or changed value or final value
of the argument x in the problems of approximation.

1 0.080
16

0.920
16
= 0.0575

1. Find the approximate value of (1.001)5

a f

Conditional Problems

k
10

is provided where a = an integer

= x1 (say) and h = x =

k
n

= a a decimal fraction,

10
and the expression in x is to be approximated, we
adopt the following working rule.
Working rule:
1. Let y = given expression in x
2. Suppose a = x1 = a given integer before decimal
and h = x1 =

+ dy = f (a) +

Worked out example on conditional problems

When x = a +

x =a

k
n

the decimal fraction (the number

10
after the decimal).
3. Use the formula:

a
f a f a f
or, f aa + hf = f aa f + f aa f h

f x1 + x1 = f x1 + f x1 x1

2 1.001 3 + 3 by considering y = x 2 x 3 + 3 .
Solution: Since, x = 1.001 which can be expressed as
1 + 0.001 on setting x1 = a = 1

h = x1 = 0.001

af

f x = x 2x 3 + 3

We have, f (1) =
f (a) = 1 2 + 3 = 2

af

af

af

f x = 5x

8 13
x
3

f 1 = f a =5

8 7
=
3 3

af

Hence, f (1 + 0.001) = f (a + h) = f (a) + f a h

7
0.001
3
= 2.0023
=2+

Approximations

2. Find the approximate value of y + y .


When y = 2x2 3x + 5, x = 3 and x = 0.1
Solution: y = 2x2 3x + 5, a = 3

dy
= 4x 3
dx

F dy I
H dx K

af a

= f a = 4x 3
x=a

x =3

LM dy OP
N dx Q

x
x=3

= 9 0.1 = 0.9
Hence, the required approximate value

bg

bg

= f a + y = y

x =3

+ y

af

= 14 + .9
= 14.9 (approx)
3. Evaluate 3x2 7x + 5 when x = 3.02
Solution: f (x) = 3x2 7x + 5

af

f (a + h) = f (a) + h f a
we find

h = x = 0.02

af

we have f (a + h) = f (a) + h f a which

af

f ( 3.02) = f (3 + 0.02) = f (3) + 0.02 f 3

af

f (3) = 11 and f 3 = 11
f (3.02) = 11 + 0.02 11
= 11.22 (approx)
4. Find the approximate value of a function by using
differential when f (x) = 5x3 2x + 3 and x = 2.01.
Solution: f (x) = 5x3 2x + 3
2

f x = 15x 2

on setting x1 = a = 2 and x1 = h = 0.01 , we have


f (a) = f (2) = 5 23 2 2 + 3 = 39

af

af

f a = f 2
Now using the formula:

= 15 22 2 = 60 2 = 58

af

af

f a +h = f a + f a h
we find

a f af

af

af

f x = 6x 7
on setting x1 = a = 3

af

= 39 + 58 0.01
= 39 + 0.58
= 39.58
N.B.: Exact value of the function for x = 2.01
= 5 (2.01)3 2 2.01 + 3
= 39.583005
5. Find the approximate value of y = x3 3x2 + 2x 1
when x = 1.998.
Solution: f (x) = x3 3x2 + 2x 1
f x = 3x2 6x + 2
Now on setting 1.998 = 2 0.002
Where x1 = a = 2 and x1 = h = 0.002
( 3 x = given value of x a)
f (2) = f (a) = 23 3 22 + 2 2 1
= 8 12 + 4 1
= 1
f 2 = f a = 3 22 6 2 + 2
= 12 12 + 2
=2
Now, using the formula:

af

= 4 3 3 = 12 3 = 9
and y =

af

f 2.01 = f 2 + f 2 0.01

679

af

f (1.998) = f (2 0.002) = f (2) + (0.002) f 2


= 1 0.002 2
= 1 0.004
= 1.004
N.B.: To check the closeness of this method, we
substitute 1.998 in the given (original) function to get
the exact value for this point and hence we get

y + y = f a + h = 1.003988008
Which shows that the method of approximation
by differentials holds very closely (or nearly).
6. Find the approx value of x3 + 5x2 3x + 2 when
x = 3.003.
Solution: f (x) = x3 + 5x2 3x + 2

af

f x = 3x + 10 x 3
on setting x = a = 3 and h = 0.003, we have
f (3) = 33 + 5 (3)2 3 (3) + 2 = 65

af

f x = 3 (3)2 + 10 (3) 3 = 54
Now, using the formula of approximation

af

af

f (a + h) = f a + h f a

680

How to Learn Calculus of One Variable

we find

af

f (3.003) = f (3) + 0.003 f 3


3
= 65 +
54
1000
162
= 65 +
1000
= 65.162
7. Find the approximate value of the increament in
the function y = 2x3 + 5 for x = 2 and x = 0.001.
Solution: We have y = dy = 6x2 dx = 6x2 x
= 6 22 0.001
= .024
The exact value of the increament:
3
3
y = 2 x + x + 5 2x 5
2
2
3
= 6x x + 6 x x + 2 x
= 6 4 0.001 + 6 2 0.000001 + 2 0.000000001
= 0.024012002
( 3 y = f x + x f x = f x dx = dy)
8. Given that 45 = 1024, find the approximate value of
fifth root of 1028.
1
Solution: let y = x 5 = f x

f
a f

a f

af

af

af

Since 45 = 1024, we set x = a = 1024 and

x = final

value a
= 1028 1024
=4=h

af
1
f axf = x
5
f b1024g = f ba g = b1024g
1
f a1024f = f aa f = a1024f
5
3f x = x

=4+

af

1
4
5

FG3 1024 = 4
H
=4+

54

45

a f

=4

IJ
K

Verbal problems on approximation

bg

The formula Q = f q q is practically fruitful


for calculating approximate change or simply change
in the function Q (i.e. dependent variable Q) due to
small change in the independent variable q, i.e. if
Q = f (q) be a functional relation between q and Q and
q is the small change in q, then the consequent
change in Q is given by the formula:

bg

Q = f q q ,
Where, Q = any dependent quantity, dependent
variable or dependent physical quantity like volume,
area, perimeter, etc.
q = independent quantity, independent variable or
independent physical quantity like radius, length,
height, thickness, etc.

q = q2

Working rule:
dQ
1. Find
and q = q f qi
dq
2. Compute

f 1024 + 4 = f 1024 + h f 1024

1
5/

OP
b g LMN dQ
dq Q

af

45

5/

= 4 + 0.0031
= 4.0031

Now, using the formula:

1
5

1
5

f qi =

1
5

a
f a f
1
= a1024f + 4 a1024f
5

a f = oa4f t

1024

qf = final value of independent variable


qi = initial value of independent variable

45

af

q = q f qi

1
5

f (a + h) = f a + f a h
we find

LM dQ OP
N dq Q

q = qi

3. Lastly use the formula:

= Q =

LM dQ OP
N dq Q

q
q = qi

Approximations

which gives us the required change or approximate


change in the dependent physical quantity Q.
Note:
1. Approximate change in a quantity Q or change in
a quantity Q due to a small change in q.

= Q =

LM dQ OP
N dq Q

LM dV OP
N dr Q

r
r = ri = initial value of r

(ii) Approximate change in area A, change in area A

= A=

LM dA OP
N dr Q

bg

= Q = f q q
2. Whenever, we have a formula of a physical
quantity like volume, area, perimeter etc, differentiation
is performed w.r.t the variable whose increament in
the problems of approximate is given to us.
Verbal problems on approximate change of a quantity

q = qi

Thus, (i) approximate change in volume V, change


in volume V due to small change in r

= V =

r
r = ri = initial value of radius r

2. In example 3 thickness suggests change (or,


increament) in the argument (or, independent variable) q. Hence, q = thickness provided q is the
independent variable.
3. If we have
2
volume of a circular cylinder V = r h
surface area of a right circular cone
A = r r 2 + h2
i.e. a function of two variables, then any one of the
two variables whose increament is given or can be
determined should be regarded as a variable w.r.t
which differentiation is performed. Moreover in the
function of two variables, the variable which is a constant is always mentioned by stating that it remains
fixed or by giving its numerical value.
e.g. (1) What is the approximate volume of a thin circular cylinder with fixed height h?

Examples worked out:


1. If the radius of the sphere changes from 3 cm to
3.01 cm, find the change in the volume.
4 3
Solution: Volume of the sphere = V = r
(1)
3
Change in the radius = increament in the radius
= 3.01 3 = 0 01 cm = r
Now, differentiating (1) w.r.t the variable involved
in it (i.e. r)

dv 4
2
= 3r
dr 3
dv
2

=4r
dr

LM dv OP
N dr Q

af

af

= f a +h = f a + f a h
whereas approximate change in a dependent physical quantity

(2)

= 4 3 = 4 9 = 36
r=3

N.B.: r1 = r
r2 = r + r
lastly, using the formula:
V = change in V =

LM dV OP
N dr Q

r
r=3

we have
V = 36 0.01
V = 0.36 cm

Explanation: The problems says h remains fixed


which means h is a constant.
Remark:
1. The approximate value of a dependent physical
quantity

681

r1 = r
r1 + r
rr

682

How to Learn Calculus of One Variable

2. Find the approximate increase in the area of circular


ring of inner radius 4 cm and outer radius 4.04 cm.

= V =

Solution: 3 A = r
dA

= 2r
dr
dA
= 2 r
and
dr r = 4

LM OP
N Q

a f

r =4

= 225 cm

= 2 4 = 8

A = 8 0.04 = 0.32 cm
3. Find approximately the volume of a metal in a hallow
cylindrical pipe 60 cm in length, 7.5 cm inside radius
and 0.25 cm thick.
2

Solution: Volume of the hallow sphere = V = r h


...(1)
Where h = height = 60 cm (given)

(2)

dV
= 60 2r = 120 r
dr

F dV I
H dr K

r
r = 7 .5

On small errors

LM OP
N Q

FG dV IJ
H dr K

= 0.25 900

Now change in radius = change in r = r


= 4.04 4 = 0.04
lastly approximate increase in the area
dA
= A=
r
dr r = 4

V = 60 r

Required approximate volume

= 120 7.5 = 12 75 = 900


r = 7 .5

Question: What do you mean by error?


Answer: Errors are increaments or changes in the
values of x and y and are taken as x and y
respectively,
Where y = dependent quantity, dependent variable
or dependent physical quantity.
And x = independent quantity, independent
variable or independent physical quantity.
Derivation of formula for calculation of small errors
The result:

af

y = f x x + x
obtained in the beginning of this chapter shows that

af

y = f x x approximately
this fact is symbolically expressed by writing

af

y = f x dx
which is useful for finding small errors in dependent
variable.
Use of the formula:

a f

Q = f qi q
The formula:

and r = thickness = 0.25 cm

a f

cm
0.25

Q = f qi q
is practically fruitful for calculating small errors in the
dependent physical quantity Q due to small errors in
the independent quantity q, i.e. if Q = f (q) be a functional relation between q and Q and q is the small
errors in q, then the consequent small error in Q in
given by the formula:

a f

15 cm

Q = f qi q
where, Q = any dependent quantity like volume, area,
perimeter, temperature, etc.
q = any independent quantity like radius, length,
height, thickness, etc.

683

Approximations

q = qf qi
qf = final value of independent quantity
qi = initial value of independent quantity

L dQ O
f aq f = M P
N dq Q

Solution: Volume of the cube = (a side)3 = s3 (1)


V = s

q = qi

Q = approximate error in Q or simply error in Q.


Remember:
1. In the problems of small errors, the approximate
error (or, value) of a dependent physical quantity like
volume, area, etc is subjected to an error in the
independent physical quantity like radius, length,
thickness, height, etc.
The error in independent quantity like radius and
length = r and L respectively are generally given
in the problem and we are required to find dV, dA,
etc.
2. The approximate error (or, change) of a dependent
physical quantity Q subjected to an error independent
physical quantity q
Q =

FG dQ IJ
H dq K

q
q = qi

3. Error in Q = Q
4. Approx error in Q = dQ = q
5. The error to which a variable q is subject (or,
subjected) means the error in q which is symbolised
as q = dq.
6. Max error, possible error or greatest error in
dependent quantity is to be determined means we are
required to find the error in dependent physical
quantity, i.e. Q .
7. q is also called absolute error or total error in
independent variable q.
8. q is not measured correctly to the extent or q is
measured with uncertainty, etc means the
increament in q, where q = independent quantity or
which a physical quantity Q depends.
Verbal problems on errors
Examples worked out:
1. A box in the form of a cube has an edge of length
= 4cm with a possible error of 0.05. what is the possible
error in volume V of the box.

dV
2
= 3s
ds

FG dV IJ
H ds K

3 4 2 = 3 16 = 48cm3/cm
x=4

(2)
(3)

and we are given s = 0.05 cm


Putting (2) and (3) in the formula:
dV =

F dV I
H ds K

s
x =4

We have V = 48 0.05 = 2.40 cm3 = possible


error in the volume.
2. The volume of a cone is found by measuring its
height and the diameter of a base as 7" and 5"
respectively. It is found that diameter is not correctly
measured to the extent 0.03. Find the consequent error
in the volume approximately.
Solution: The volume of a cone = V =

1 2
r h
3

(where h is a constant)
=

F I
H K

1
R

3
2

h (where R = diameter = 2r)

2
hR
12
Now, differentiating (1) w.r.t R, we have
dV
1
= Rh
dR 6
=

F dV I
H dR K

=
R =5

1
5h
6

and R = 0.03" as well as h = 7"


Putting these values of R , h and
the formula:

V =

FG dV IJ
H dR K

R
R =5

...(1)

F dV I
H dR K

R =5

in

684

How to Learn Calculus of One Variable

we have

h = 7

FG dA IJ
H dr K
L
= M
N

=
r=2

LM
MN

4
4 + 36

4 + 36

OP
40 Q

40 +

OP
PQ
(2)

(3)
and we are given r = 0.02 cm
Putting the values of (2) and (3) in the formula:
d = 5

A=

V = 5 7 0.03 inch 3
6
=

22 1
3
57
7
6
100

22 5 3

6
100

22
5

2
100

110
55
=
= .55 inch
2 100 100

We have:

r = 2 cm

= error in the volume.


3. The altitude of a right circular cone is 6 cm. The
measurement of the radius of the base is 2 cm with an
uncertainly of 0.02 cm. Find approximately the greatest
possible error in the computed lateral surface area.
Solution: The lateral surface area of a right circular
cone with base radius r and height
h= A= r r +h

which

r + 36
r 2 + 36

(1)

dA
=
dr

r + 36
2

+ r 2 + 36

OP b0.02g
40 Q
L 40 + 4 OP b0.02g
A=M
N 40 Q
=

44
40

b g

0.02 cm 2

If Q = f (q) be a functional relation between two


quantities Q and q and q is a small error in q, then

1 2r
dA
= r
+
dr
2 r 2 + 36

LM
N

A = 40 +

Verbal problems on relative errors

Here h = 6 cm (a constant)
A = r

r
r=2

h = 6 cm

a f

LM dA OP
N dr Q

dQ
q
is called the relative error in Q and
is called
Q
q
the relative error in q. But since we use dy for an
Q
approximate value of y , for this reason
Q
suggests the convenience of finding first log Q and

then calculating d (log Q) =

dQ
.
Q

Approximations

Hence, in the light of above explanation, we can


provide the following working rule to find the relative
error in Q.
First working rule:
Let Q = f (q) be a functional relation between two
quantities Q and q
1. Take first log of both sides of Q = f (q).
2. Differentiate both sides of log Q = log f (q).
3. Multiply both sides of the simplified form of the
equation:

af

d
d
log Q =
log f q by dq.
dq
dq

Which provides us the required relative rate in Q.


4. Put the given values of q and q = dq in the
expression obtained for the relative error in Q.
Second working rule:
To find the relative error in Q = f (q) at q = qi = initial
value of a quantity q, we proceed in the following
way:

a f

1. Find f qi and then multiply it by

h = q = q f qi

a f

2. Divide the product f qi q by the value of


the function at q = qi (i.e. by f (qi)) which provides us
the required relative rate, i.e.

a f
a f

f qi q
dQ
=
Q
f qi

Remember:

y
if y is the
y
dy
y
error in y and
may by approximated by
if the
y
y
1. Relative error in y is defined by

increament of the variable (the quantity being


measured) on which y depends is small enough.
2. Method (1) (or first method) is convenient.
Whenever we have a formula for volume, area or
perimeter etc in the form of power of an independent
variable or in the form of product, whereas method (2)
or, second method is applicable in all cases.

685

Verbal problems on relative errors


Examples worked out:
1. The radius of a sphere is found to be 10 cm with a
possible error 0.02 cm, what is the relative error in the
computed volume?
Solution: First method:
4 3
We have V = r
(1)
3
Taking log of both sides of (1), we have

4
3
log V = log + log r
3
4
(2)
= log + 3 log r
3
Now, differentiating both sides of (2) w.r.t r, we get
1 dV
1
(3)

= 3
V dr
r
Multiply both sides of (3) by dr, we obtain
3 dr
dV
(4)
=
V
r
Putting the given values r = 10 cm and r = dr =
0.02 in (4), we have the required relative rate in V.
dV
0.02 0.06
=3
=
= 0.006
V
10
10
Second method:
V =

af

4 3
r V
3

r = 10

F4 r I
H3
K
3

r = 10

4
(1)
= 1000
3
2
2
2
dV
4
4
= 3 r = 3 r = 4 r (2)
dr
3
3

F dV I
H dr K

= 4r
r = 10

r = 10

= 4 100

(3)

and we are given dr = 0.02


(4)
Putting the values of (1), (3) and (4) in the formula:

a f
a f

f qi
dQ
=
q where Q = V and q = r} in
Q
f qi

this problem

686

i.e.

How to Learn Calculus of One Variable

bg
bg

f ri
4 100 .02 3
dV
=
r =
V
f ri
4 1000

Solution: A = r A =

FG3 d = 2r = r d = rIJ
H
K
2

0.06
=
10
= 0.006
2. Show that the relative error in xn is n times the
relative error in x.
Solution: First method:
y = xn

1 2
x (where x = diameter)
4
Taking log of both sides of (1), we have
=

log A = log

log y = log x = n log x

dy n dx

=
y
x
Second method:
Let y = xn
n 1
dy

= nx
dx
Now, we are required to show

d
d
log A =
2 log x
dx
dx

1 dA
1 2

=2 =
A dx
x x

dA
dx
=2
A
x

f
(2)

Now, putting the given values: x = 10 and x =


dx = 0.01 inch in (2), we have,

0.01 0.02
dA
=2
=
= 0.002
10
10
A

(2)
x

(3)

(applying for small x and using (1))


n 1

(1)

y
x
=n
y
x
Now, using the definition
n 1

F x I
H4 K

2
+ log x
4

= log + 2 log x
4

d
d
log y = n
log x
dx
dx
1 dy
1

=n
y dx
x

af

(1)

= log

y = f x x = nx

1
2
d
4

y nx
x
x
= n 1
=n
y
x
x
x

Verbal problems on percentage errors


Definition: Percentage error in a quantity means 100
times relative error in that quantity, i.e.

dx
= percentage error in x
x
= k1 (say)
= a number written before the percentage symbol

1. 100

N.B.: If y is a function of x, then y = f (x) and

af

af
Moreover, y = f a x f x
(approximately)
And dy = f a x f dx (accurately)

dy d f x
or f x .
=
dx
dx

3. What is the relative error in the area of a circle if


the diameter is found by measurement to be 10 inches,
with a maximum error of 0.01 inch?

%
x k1
100
= k1 % of x (or k1 % in the value of x)
where x = an independent variable

which dx = error in x =

2. 100

dy
= percentage error in y
y

Approximations

= k2 (say)
= a number to be determined

3. Apply the percentage error formula

y k2
which dy = error in y =
100
= k2 % of y (or k2 % in the value of y)
where y = dependent variable (i.e. a quantity being
measured).
Note:
1. If y = f (x) be a functional relation between x and y,
then percentage error in dependent variable y is
caused by the percentage error in independent
variable x or in other words;
Let y = f (x) be a functional relation between two
quantities x and y.
If x is k1 per cent (or, k1 %) error in x (or, in the
value of x), then y will be k2 per cent (or, k2 %)
error in y (or, in the value of x).
Now in the light of above explanation, we can
provide the following working rule to find the
percentage error or percentage increase, etc in a
function (i.e. dependent quantity) = y.
To find the percentage error in a dependent
quantity Q implies we are required to use the formula
dQ
which is =
100 where Q = f (q) = y = a
Q

dependent quantity, a dependent physical quantity


like volume, area, etc.
And q = independent quantity on which Q
depends like length, radius, height or thickness etc.
First working rule:
dQ dy
=
1. Find
by differentiating the given
dq
dx

function, the formula for volume, formula for area,


etc obtained by mensuration formula, etc w.r.t the
independent variable involved in the formula or given
function.
dQ
2. Use the formula Q =
q
dq

which dQ =

dQ
q 3 Q = dQ
dq

687

dQ
q
dQ
dq
which
100 =
100
Q
Q
Second working rule:
1. Find

dQ
by differentiating w.r.t the independent
dq

variable q.
2. Use

dQ Q
=
dq
q

LM Q = dQ q OP
N dq Q
divided by Q)

3. Divide

by Q (both sides be

dQ
q
Q
dq
which
=
Q
Q
where q = error in q = k % of q (or, k % in the value
of q)
Remember:
1. Error in Q = Q and error in q = q
Q
2. Relative error in Q =
and relative error in
Q
dq
q=
.
q
3. Percentage error in Q =

Q
100 and percentage
Q

q
100 .
q
4. Q = dQ and q = dq
error in q is =

Verbal problems on percentage


Examples worked out:
1. If the radius of a spherical balloon increases by 0.1
% find approximately the percentage increase in the
volume.
Solution: The volume of a sphere of radius r is

V =

4
r3
3

(1)

688

But

How to Learn Calculus of One Variable

4
dV
2
= r 3
3
dr

V
dV
~
= 4 r 2
dr
r

V ~ 4r 2 r
There is a percentage increase in r of
0.1% =

(2)

r 0.1
= r
100
2

100 V 100 4 r
=
r
3
4
V
r
3

g . Find the approximate error in T

corresponding to an error of .5 % in the value of L, g


is a constant.
Solution: The time for one comple oscillation is T
T
T
units of time i.e.
second for
(Which means)
2
2
T
tick and
second for tock)
2
And T = 2

dL
100 = .5 = k (say)
L
(given in the problem)

(1)
(2)

since (1) consists of product, so taking logarithm of


both sides

1
1
log L log g
(3)
2
2
Now differentiating both sides of (3) w.r.t L noting
that g is constant
log T = log 2 +

1 dT
1 1
=0+ 0
2 L
T dL

dL
100
2L

.5
= 0.25 (from (2))
2
= percentage error in T.
3. If p is a small percentage error in measuring the
radius of a sphere, find the percentage error in the
calculated value of volume and surface.
=

Solution: V =

400 3
r
=

= 0.3
4r
1000
2. The time period for one complete oscillation
of a simple pendulum of length L is given by
T = 2

dT
dL
dT
=

100
T
2L
T

4 2
r
3

(1)

dr
100 = p (given)
(2)
r
Now differentiating both sides of (1) w.r.t r after
taking the logarithm of both sides, i.e.
logV = log 4 log 3 + log + 3 log r

1 dV
1
=3
V dr
r

dV
dr
=3
V
r

dV
dr
100 = 3
100
V
r
= 3p (on putting (2) in (3))

(3)

2nd part:
surface area = S = 4 r

(1)

dr
100 = p
(2)
r
Taking logarithm of both sides of (1),
log S = log 4 + 2 log r
(3)
Differentiating both sides of (3) w.r.t r we get
1 dS
1
=0+2
S dr
r
dS
dr

=2
S
r
dS
dr

100 = 2
100
S
r

(4)

Approximations

= 2p (on putting (2) in (4))


= percentage error in calculated value of required
surface area.
4. Find the percentage error in x3 corresponding to a
small error of r % in the value of x.
Solution: Let y = x3
(1)
error of r % in the value of x

dx
100 = r
x

(2)

y = x3
log y = 3 log x

= cot 36

100
60 180

=0.12 %
error A = dA = 3 =

(A) Problems based on finding the approx values of


numbers.

1 dy
3
=
y dx
x

dy
dx
=3
y
x

1. Find approximately
(i) 4 627

dy
dx
100 = 3
100
y
x

(ii)

dx
100 = r )
x
Verbal problems on percentage errors continued:
5. If b and c are measured correctly. A = 36 with a
possible error of 3, find the possible % error in .
1
Solution: = bc sin A
2
= area of triangle has been given as a product,
we use logarithmic differentiation.

60 180

Type 1: Problems based on finding the approximate


values of numbers:

= 3r (from (2),

689

Exercise 18.1

(iii)
(iv)
(v)

66

3
4
4

122
252

82

a f
(vii) a63f
(viii) a126f
1
3

(vi) 28

1
3

(ix)

1
3

401

a f
(xi) a235f
1
4

1
+ log b + log c + log sin A
2
Now differentiating w.r.t A, we have

(x) 15

d log d
d log sin A d sin A

= 0+ 0+ 0+

d
dA
d sin A
dA

2. Find the approximate values of the following


number

log = log

1 d
1

=
cos A
dA
sin A

d cos A
=
dA = cot A dA = relative error

sin A

in A.

d
100 = cot A dA 100

(i)

1
4

0.0037

(ii) 3 0.009
(iii) (0.998)8

(iv) 3198
.
(v)

80.999
1

(vi)

1
2

100.5

690

How to Learn Calculus of One Variable

(vii)

4.08

(viii)

65

a f
a3.998f

(ix) 1000.1
(x)

1
3

3
2

(xi) (02.97)3
(xii)

.82

(xiii)
(xiv)
(xv)

6.23
3

376
75
(ix) 20.025
(x) 1.96875
(xi) 3.9961
(viii)

8.01

a2.001f

3. By use of differentials, calculate approximately the


values of the following:
(i) 792
(ii) 1032
(iii) (9.06)2
(iv) (1.012)3
(v) (9.95)3
(vi) (1.005)10
(vii) (0.975)4
4. Calculate approximately the reciprocal of 997 and
102.
5. Find the approximate values of the following
quantities:

1
0.99
1
(ii)
9.93
(i)

2. (i) 0.060833
(ii) 0.208
(iii) 0.9840
(iv) 1.99975
(v) 2.99990741
(vi) 0.09975
(vii) 2.02
(viii) 4.02083
(ix) 10.0003
(x) 7.994
(xi) 26.19
(xii) 0.956
(xiii) 2.496
(xiv) 2.00083
(xv) 0.24975
3. (i) 6240
(iii) 82.08
(iv) 1.036
(v) 985
(vi) 1.05
(vii) 0.9
4. Approximate value of reciprocal of 102 = 0.0098.
5. (i) 1.01
(ii) 0.1007
(iii) 0.992
(B) Conditional problems

(iii)

a1.004f

Answers
1. (i) 5.004
(iii) 4.96
(v) 3.009
(vi)

82
27

(vii)

191
48

Exercise 18.2
1. Find the approximate values of the following
quantities:
(i) sin 31 when 1 = 0.175 radians
(ii) cos 29 when 1 = 0.175 radians
(iii) sin 60 2' given sin 60 = 0.86603 and 1' = 0.00029
radians
(iv) tan 44 given 1 = 0.0175 radians
(v) tan 45 30' when 1 = 0.0175 radians
(vi) cos 30 1' when 1 = 0.01745 radians

Approximations

691

(vii) tan when 1 = 0.01745 radians


(viii) loge 10.01 when loge 10 = 2.3026
(ix) loge (3.0001) when loge3 = 1.0986
(x) loge (3.001) when loge3 = 1.0986
2. Find the approximate value of the function
f (x) = x3 2x2 + 1 when x = 1.0001
3. Find the approximate value of the function
f (x) = x3 + 5x2 3x + 1 when x = 3.003
4. Find the approximate value of the function
f (x) = sin x + 2 cos x when x = 46
5. Find the approximate value of the function
f (x) = 3x2 8x + 11 when x = 8.007
6. Find the approximate value of the function
f (x) = x4 + 2x2 + 5 when x = 1.998

2. What is approximate error in the volume and


surface of a cube of edge 8 inches, if an error of 0.03
inch is made in measuring the edge?
3. Find the approximate error in the curved surface of
a cylinder of diameter one foot and height 4 feet, if
there is a possible error of 1 inch in the height.
4. The radius of a sphere is found by measurement

Answers:
1. (i) 0.5152

length L is given by T = 2

1
of an inch.
10
Find the consequent errors possible in (i) the surface
area and (ii) the volume as calculated from the
measurement.
5. The time of oscillation T of a simple pendulum of
to be 10 inches with a possible error of

. If L is creased

by 1 % (one per cent), show that the percentage error


in T is 0.5.
6. The angle of elevation of the top of a tower at a
point 200 ft away from its base is 45. Find the
approximate error in the height of the power due to an
error of 2 % in the angle of elevation.
7. Find the percentage error in calculating the area of
a triangle when one of its angles has been measured
as 45 with an error of 1'.
8. A metal cube of side 3 cms is heated. Find the
approximate increase in its volume if its side becomes
3.0001 cms.
9. Find the percentage error in x3 corresponding to
small error of r % in the value of x.

3
(ii)
2
(iii) 0.86632
(iv) 0.965
(v) 1.0175
(vi) Find
(vii) Find
(viii) 2.3036
(ix) 1.098633
(x) 1.0989
2. 0.0001
3. 65.162
4. Find
5. 139.28
6. 28.92
Verbal problems on approximations and errors
Exercise 18.3
1. The radius of a circle is 10 inches and there is an
error of 0.1 inch in measuring it. Find the consequent
error in the area.

Answers
1. 2 sq inch
2. 5.76 cu in and 2.88 sq in
3. in
4. (i) 8 sq inch (ii) 40 cu. in.
5. 2 ft
6. 0.029
8. 0.002700
9. 3r

692

How to Learn Calculus of One Variable

19
Tangent and Normal to a Curve
Give me a place to stand on and I will move the earth.
Archemedes (287212 BC)

The concepts of tangent and normal are based on the


following concepts.
1. Inclination: The inclination of a line is the angle
between the line (or its extension) and the positive
direction of the x-axis (the direction from a point on
the left (the point of intersection of the line and the
x-axis) to an other point on the right situated on the
x-axis) measured by convention in the anticlockwise
direction from the x-axis to the part of the line
(intersecting the x-axis) above the x-axis.
Notation: An angle of inclination is generally denoted
by the symbol i or .
C

il

il

il

Explanation:
X

N
O

O
N

2. Slope of a line. The slope of a line is the tangent


(or trigonometric tangent) of its inclination.
Notation: The slope of a line is represented by
y
m=
= tan i , where y = y 2 y1 difference of
x
y-coordinates of two points x1 , y1 and x2 , y2
on the line whose slope is sought.

il

(i) The inclination of the line CD is the angle XNC.


(ii) The inclination of the line AB is the angle XMA.

Tangent and Normal to a Curve

x = x2 x1 difference of x-coordinates of two

points x1 , y1 and x2 , y2 on the line whose slope

693

(b) The slope of the line AB is negative.


Y

is sought.
Nomenclature: y = delta y

x = delta x
Explanation: The slope of the line FG = tan i
Y

il

F
R

T
O

il

SR
TS
=m

Now, let us procede to have a clear idea of the


concepts of tangent and normal to a point on the
curve. Ordinarily, it is known that a line l cuts a circle
in two points, but a tangent line cuts it in one point
only.
From this special case, we are led to describe a
tangent as a line which cuts the curve at only one
point.

To remember:
(a) When the inclination of a line is an acute angle,
the slope of a line is positive.
(b) When the inclination of a line is an obtuse angle,
the slope of a line is negative.
(c) The inclination is acute (0 < < 90) or obtuse
(90 < < 180) accordingly as the line leans to the
right or to the left of the point of intersection of the xaxis and the line. Further, the inclination of a line
parallel to the axis is 0 and the inclination of a line
parallel to the y-axis is 90.

O
90

Now this above description is all right in case of a


circle, but it is not applicable to a tangent to a curve in
general.
Y

Explanation:
(a) The slope of the line CD is positive.
Y

C
O

N
O

il

For example, in the figure given below, the tangent


line to the curve at a point namely P intersects the
curve at another point namely Q. This is why in
Calculus a tangent to a curve is a straight line which
cuts the curve at only one point is not applicable as
a definition. But it is endeavored to arrive a suitable
definition of the tangent line at a point on the graph
of the function f defined by y = f (x).

694

How to Learn Calculus of One Variable

Definitions of Tangent and


Normal to a Curve
It is common to define a tangent line (or simply a
tangent) at a point P in two ways:
Definition I: (In terms of the limit of a secant line of a
curve): The limit or the limiting position of a secant
line (or simply a secant) of a curve y = f (x) through a
fixed point P and a variable point Q on the curve
when the variable point Q moves from either side of P
arbitrarily close to P, but never coincident with the
fixed point P (i.e., the distance between the fixed point
P and the variable point Q is non-zero and less than
any given small positive number ) is called the tangent
to a curve y = f (x) at a fixed point P on the curve.

t
Tangen

Q
y = f (x)
Q

Let P be a fixed point and Q be a variable point on


the same curve y = f (x). Then the line PQ is called a
secant line of the curve y = f (x). Now, when the variable
point Q is made to move along the curve y = f (x),
towards the fixed point P, the positions of the secant
line PQ changes. The different positions of the secant
line are shown in the figure by dotted lines when the
variable point Q is made to move to the point Q, Q,
etc. The limiting position of the secant line PQ when
the variable point Q moves from either side of P
indefinitely close to but never coincident with the
fixed point P is called the tangent at the fixed point P
on the curve y = f (x).
Definition II: (In terms of slope of a curve): A tangent
line to a curve y = f (x) at a point P (x, y) on the curve
dy
is a line through P (x, y) with a slope
at P (x, y).
dx
i.e., the tangent to a curve y = f (x) at a point P (x, y) on
the curve is the line which passes through P (x, y)
dy
and which has the same slope
as the curve at
dx
P (x, y).

Slope of P
y f x f xx
=
=
x
x

bg b

(1)

P Tangent at P (x, y)
y =f x ( )
(x x,
y y) Q

ant
sec

y = f (x) f (x

x)

(x

x)

In accordence with earlier discussion, its limit as


x 0 x > 0 or x < 0 , i.e.,
y
lim
(if exists)
x0 x
dy
=
,
(2)
dx
which is said to be the slope of the tangent line at
(x, y) and consequently the tangent is defined to be
the line through P (x, y) with the slope given by (2).
Length of the tangent to a curve: It is the segment (or
part or portion) of the tangent joining the point of
tangency (the point where the tangent touches the
curve) and the point of intersection of the tangent
with the x-axis.

y
y = f (x)

P
(x, y)

Thus, PT is the length of the tangent.


Normal to a curve at a given point: The normal at
any given point P on (to or of) a curve defined by
y = f(x) is a line which passes through P and is
perpendicular to the tangent at P on the curve
y = f(x). The point P is called the foot of the normal.
Length of normal to a curve: It is the segment (or
part or portion) of the normal (or normal line) joining
the point of tangency on the curve and the point of

695

Tangent and Normal to a Curve

= QM LP = y
= QM LP
= y1 + y y1
= y

intersection of the normal with the x-axis. Thus PN is


the length of the normal. Thus PN is the length of the
normal.
y = f (x)

y = f() x

subnormal

,y
1

(x

Ta
ng
en
t

al

subtangent

il
N

Subtangent: The projection on the x-axis, of the part


(or portion or the segment) of the tangent joining the
point of tangency on the curve and the point of
intersection of the tangent with the x-axis is called
the subtangent or length of the subtangent, i.e. the
projection of the length of the tangent is called the
length of subtangent. Thus, TG is the subtangent.
Subnormal: The projection on the x-axis, of the
segment (or part or portion of the normal joining the
point of tangency on the curve and the point of
intersection of the normal with the x-axis is called the
subnormal or length of subnormal, i.e. the projection
of the length of the normal is called the length of the
subnormal or simply the subnormal. Thus, GN is the
subnormal or the length of subnormal.
Remark: The slope of the tangent line to a curve at a
dy
point is equal to the slope of the curve, i.e. m =
dx
at (x, y) = slope of the tangent at (x, y) on the curve
= slope of the curve defined by y = f (x) at a given
point (x, y) where x = a point on the domain of f.

dy
: Supposing that P (x, y)
dx
is a point on the continuous curve C whose equation
is y = f (x). Again supposing that Q is a point on the
given curve y = f (x), whose co-ordinates are
x + x, y + x .
Construction: We draw perpendiculars PL, QM upon
OX and PN QM . We join PQ and produce it to
meet OX at R.
Let PQ make (or, QR) make an angle with OX.
Now, NQ = QM MN
Geometrical meaning of

x1

m
or

x,

il
O

P (x, y)

Perpendicular

90

T
N

= i

= i

x M

Again, PN = LM = OM OL = x
3 OM = x + x , OL = x
= x1 + x x1
Q (x1 + x,
y1 + y )

P (x1, y1)

x
N

Since, PN is parallel to OX NPQ =


NQ
tan = tan NPQ =
PN
QM LP
=
LM OL
y
=
(1)
x
Let the tangent T PT at P intersect the x-axis at T
and makes an angle with the positive direction of
the x-axis.
XTP =
We observe that as Q moves along the curve
C towards P, then the chord QP tends to the tangent
PT If we suppose that x 0 , then M L
(i.e., Q P along the curve C) The chord QP
tends to the tangent at P PQ PT . Hence
x 0 , y 0 tan = tan

696

How to Learn Calculus of One Variable

Hence, lim tan = lim


tan =

x0

LM dy OP
N dx Qb

x1 , y1

LM OP
N Q

y
dy
=
x
dx

= value of

P x1 , y1

dy
at the point
dx

(x1, y1).
But tan represents the slope of the tangent PT.
dy
Hence,
at P (x, y) represents the gradient of the
dx
dy
tangent at that point i.e.,
gives the value of the
dx
slope of the tangent at P (x, y) or the slope of the
curve at (x, y).

Note: 1. In general, if the function f (x) has a finite


derivative at every point x X , then we can write
the derivative f x as a function of x which is also
defined on x.

af

a f LMN dydx OPQ

2. f x1 =

x = x1

af

3. We also use the symbol y = f x , the dash


indicating that it is the derived function.
To find the equation of the tangent and normal at
(x1, y1) of the curve C = y = f (x).
y = f (x)
Y

Facts to know:
dy
= 0 at a point tan = 0 The tangent
1.
dx
at that point is parallel to the x-axis.

Tangent

A (x1, y1)
Normal at A

2. If

dx
= 0 then = 90 the tangent is parallel
dy

to the y-axis.

Refresh your memory:


Definition of normal to a curve at a point P (x1, y1) on
the curve:
The normal to a curve at the point A (x1, y1) on it is the
line through the point A perpendicular to the tangent
at the point.
Remember:
1. If the slope of the tangent is m, then the slope of
1
the normal is .
m
2. Equation of a line passing through (x1, y1) and

3. Notation: The slope of the curve at a point


P (x1, y1) is denoted by

F dy I
or H dx K
a f
f
or for the purpose of simplicity, we write f a x f .
p

F dy I
or H K
dx

L dy O
or M P
N dx Qa

x1 , y1

x1 , y1
1

y y1
= m y y1 = m x x1
x x1
which is the point slope form of the equation of a
straight line.
3. The normal is a line passing through a point
(x 1, y 1) lying on the curve and having the slope
having the slope m is

LM dy OP
N dx Q

LM OP
N Q
b g

y y1
1
1
dy

=
(where m = tan =
m
x x1
m
dx p
= derivative of a function at a general point x , y ) is
=

Tangent and Normal to a Curve

the equation of the normal (a line) passing through


(x, y).
4. Two lines are perpendicular to each other if the
product of their slopes is
1 if m1 m2 = 1, then lines having the slopes
m1 and m2 respectively are perpendicular to each other
Slope of one line is equal to the negative reciprocal of the slope of the other line means two lines
are perpendicular to each other.
N.B.: Normal and tangent to a curve are
perpendiculars to each other. Derivation of the
equation of the tangent and normal at (x1, y1) lying on
the curve C = y = f (x).
TAT ' and NAN ' are respectively tangent and normal
at A (x1, y1) to the curve C, y = f (x).

By the geometrical meaning of

LM dy OP
N dx Qa

1
f x

f a xf

af

x x1

x = x1
y = y1

x x1
x = x1

697

Alternative proof based on definition of tangent to a


curve at a point.
Let us suppose that the equation of the curve is
y = f (x)
(1)
Let P (x1, y1) be a point lying on the curve C given
by (1) and Q x1 + x , y1 + y be any point
adjacent to P on C.

C
y = f (x )

dy
,
dx

S
Q
y

P (x1, y1)

x1 , y1

= The slope m of the tangent TAT '

Now we know that the equation of the straight


line passing through (x1, y1) and having a slope m is
y y1 = m (x x1)
(1)
A tangent is a line passing through (x1, y1) and

L dy O
having a slope m = M P
N dx Qa

x1 , y1

(2)

Putting (2) in (1), we get the required equation

of the tangent, y y1 =

LM dy OP
N dx Qb

x1 , y1

F dy I
H dx K a

(4)

x1 , y1

LM dy OP
N dx Qb

x1 , y1

x x1

Then by the co-ordinates geometry, the equation


y
x x1 (2)
of the secant line PQ is y y1 =
x
Now, the slope of the tangent is the limit of the
slope of the secant the equation of the tangent is

g FGH dydx IJK b x x g

x 0

y
x x1
x

y y1 =

which is the required equation of tangent where

LM dy OP
N dx Q b

p x1 , y1

Therefore, the normal has the equation

y y1 =

i1

given by y y1 = lim

x1 x1 .

Again since the normal is perpendicular to the


tangent, slope m of the normal

x M

= value of

dy
at (x1, y1).
dx

Angle between two curves: The angel between two


intersecting curves is the angle between the tangents
at their common point of intersection.

698

How to Learn Calculus of One Variable

= Value of d.c. of f2 (x) at the point of intersection


P (x1, y1).

C 1 = f 1 ( x)
(x1,
y1)

C 2 = f 2 ( x)

C1

C2

Orthogonal Curves: Two curves are said to be


orthogonal if they intersect each other at right angles.
Thus, for the curves C1 = y = f1 (x) and C2 = y
= f2 (x) to be orthogonal, they must make at an angle

i1

i2
T2

T1

Derivation of the formula: Supposing that


y = f1 (x)
y = f2 (x) are the two equations of the two curves
C1 and C2 respectively intersecting at P (x1, y1). Let
the tangents PT1 and PT2 to the curves C1 and C2
make angles i1 and i2 with OX.
Let the angle < T1PT2 between the tangents be .
Now, tan i1 =

LM dy OP
N dx Q

LM dy OP
N dx Q

C1

= tan i1 = m1

and tan i2 =

LM d f b x g OP
N dx Q

LM d f a xf OP
N dx Q

x = x1
y = y1

C2

x = x1
y = y1

= tan i2 = m2

at their point of intersection


2

i2 = + i1
2
tan i2 = cot i1
tan i1 tan i2 = 1
m1 m2 = 1
derivative of f1 (x) at (x1, y1) derivative of
f2 (x) at (x1, y1) = 1.
Remember: 1. Angle between two curves or angle
between two intersecting curves at a point of
intersection or angle of intersection of two curves are
synonyms by which we mean the angle between their
tangents at the point of intersection of two curves.
2. If the angle between the tangents to the two curves

= ,
at the point of intersection of two curves is
2
then the two curves are said to be orthogonal or to
cut orthogonally.
3. Ortho means right and gonal means angular.

Now, since = i2 i1

tan = tan i2 i1 =
m2 m1
=
1 + m1 m2

tan i2 tan i1
1 + tan i1 tan i2

L m m OP ,
= tan M
N1 + m m Q
L d f a x f OP
where m = M
N dx Q
1

x = x1
y = y1

= Value of d.c of f2 (x) at the point of intersection


P (x1, y1) and m2

L df b xg OP
=M
N dx Q
2

x = x1
y = y1

C 2 f 2 ( x)

T2
tangent to y = f 2 (x)

C 1 y = f 1 (x)
T 1 tangent to y = f 1 ( x)

4. Tangent of an angle between two curves

Difference of slopes at their


common point of intersection
=
1+ product of slopes at their
common points of intersection
5. When the two curves touch at (x1, y1), = 0
tan = 0 m1 = m2 when the slopes of
the tangents (or curves) at their common point of
intersection are same, the curves touch each other.

699

Tangent and Normal to a Curve

6. The equations of the curves may be explicit or


implicit. When the given equation is explicit function
of x only, the derived function contains only x and
when the given equation is implicit function of x and
y, the derived function contains both x and y. This is
why we put only x-coordinate of point of intersection
in the derived function of explicit function while
finding the slopes at the point of intersection of two
given curves while we put x co-ordinate and y coordinate both of point of intersection in the derived
function of implicit function while finding the slopes
at the point of intersection of two given curves.
Types of the problem:
1. Finding the inclination or slopes when x1 and /y1
is given.
2. Finding the equation of the tangent and normal
when x1 and /y1 is given.
3. Finding intercepts and proving the result based
on equation of tangent and normal.
4. Finding (x1, y1) = co-ordinates of the point where
the tangent and finding the length of perpendicular.
5. Finding the angle of intersection or proving
orthogonal or touch etc.
6. Proving the equation of tangent and normal to the
curve y = f (x) at any point (x1, y1) to a straight line ax
+ bx + c = 0.
7. Finding the angle between two tangents to a curve
at two given points.
8. Finding the area of a triangle.
9. Finding the length of subtangent and subnormal.
Refresh your memory: In every type of problem, the
equation of the curves in planes may be given in
three forms:
(i) Explicit form: y = f x .
(ii) Implicit form: f x , y = 0 or constant
(iii) Parametric form: x = f 1 t
y = f2 t

bg
b g

bg

3. Find the angle of inclination using tan i = tan


which i = .
Note: 1. Actually (x1, y1) is taken to be the point of
contact of the tangent and the curve which is given
in the problem.
2.

b g FG

f x

x= x I
H y = y JK
1

= tan i = tan

dy
3. If
= an expression in x only, we put only the
dx
dy
value of x coordinate of the given point and if
=
dx
f (x, y), we put x coordinate and y coordinate both of
the given point in f (x, y).
4.

b g FG

f x

I
H y = y JK
x = x1

= value of f (x) at x = x1 and y = y1

which are the coordinates of the point of contact of


the curve and the tangent.
Examples worked out:
Question 1: Find the inclination of the tangent at the
3

2
x
x +2.
3
Solution: Given equation of the curve is

point where x = 1 to the curve y =

x3
x2 + 2
(1)
3
Now, differentiating both sides of (1) w.r.t x, we get
y=

dy
x
2
=3
2x + 0 = x 2x
3
dx

af

Now, f x

x =1

= x 2x

a f FH

x = x1
y = y1

tangent line.
= tan i

IK

= 1 2 = 1 2 = 1

bg

P (x1, y1)

Working rule:
1. Find y1 i.e. find d.c of the given equation of the
line.

f x

x =1

tan i = 1 = tan 135 i = 135

Problems based on finding inclination:

2. Find

(2)

which is the slope of the

af

Note: If f x

x = x1
y = y1

is an obtuse angle.

= ve Angle of inclination

700

How to Learn Calculus of One Variable

Question 2: Find the inclination of the tangent at


(1, 1) lying on the curve y = x2 x + 1.
Solution: Given equation is y = x2 x + 1
(1)
Now, differentiating both sides of (1) w.r.t x
dy

= 2x 1 + 0 = 2 x 1
dx

bg

Now, f x

x =1

= 2x 1

x =1

= 2 1 1 = 2 1 = 1

tan i = 1 tan i = tan 45


i = 45

i = 45
(3)
Hence, the inclinations of the two tangents to the
curve at the points (1, 0) and (2, 0) are 135 and 45
respectively.
Problems based on finding the slope of a curve at a
given point
Working rule:

dy
by differentiating both sides of the given
dx
equation w.r.t x.

1. Find

a f FH

P (x1, y1)

2. Find f x

Question 3: Find the inclination of the tangents at the


points (1, 0) and (2, 0) to the curve y = (x 1) (x 2).
Solution: Given equation of the curve is y = (x 1)
(x 2)
(1)
Now, differentiating both sides of (1) w.r.t x, we get
dy
= ( 2x 3)
dx
dy
at (1, 0)
Now, f x x =1 = The value of
dx
y =0
=[2x 3]x = 1 = 2 3 = 1
tani = 1
tan i = tan 135
i = 135
...(2)

F I
H K

af

af

f x

x=2
y=0

x = x1
y = y1

IK

= Value of

dy
at the point
dx

(x1, y1) which is given in the problem.


The slope of the curve = The slope of the tangent.

Again,

tan i = tan 45

= value of

F I
H K
dy
dx

at (2, 0)

Remember:
1. Slope of the given curve = the slope of the tangent
at the same point.
2. If x = a y = b and tangents are required to
find out at x = a then (x1, y1) = (a, b) and (x2, y2)
= (a, b) are the two points on the curve where the
tangents are to be found out.
3. If y = c x = a , b , then (x1, y1) = (a, c), (x2, y2)
= (b, c) are two points on the curve where tangents
are required to find out.
Hence, we see that common x-coordinate or y coordinate is included in both points.
Note:

af

1. If f x = a function containing only x terms


and no y terms, then we put only the x coordinate of
the given point in derived function

=[2x 3]x = 2 = 2 2 3 = 1
y

LM dy OP
N dx QFH

x =a
y =b

IK

LM dy OP .
N dx Q
f a x f = a function containing only y terms

x =a

135

(1, 0)

2. If
45
(2, 0)

and no x terms, then we put only the y coordinate of


the given point in the derived function

Tangent and Normal to a Curve

LM dy OP = LM dy OP .
N dx QFH IK N dx Q
f a x f = derived function = a function con-

3. If

x =a
y =b

y =b

taining both x terms and y terms, then we put xcoordinate and y-coordinate of the given point in the
derived function.

LM dy OP
N dx QFH
4. If f a x f

x =a
y =b

IK

a f FH

= f x

x =a
x =b

N.B.: In such type of problems, we must find the


missing coordinate of the point by putting its value
in the given equation of the curve provided that
derived function does not contain the variable whose
coordinate is given.
Question: In the curve y2 = 4x, obtain the slope of
the curve at the point where y = 2.
Solution: Given equation is y2 = 4x
(1)
Now, differentiating both sides of (1) w.r.t x, we get

IK

= derived function = a function


containing only constant but no x and y terms, then
we do not put x-coordinate and y-coordinates of the
given point in the derived functions.
Question: How to find the x-coordinate or ycoordinate of the point lying on the curve whose
equation is given and only one coordinate of the xcoordinate and y-coordinate is given?
Solution: Since the point lies on the curve, equation
of the curve will satisfy the given equation, i.e. we
put x-coordinate or y-coordinate in the given equation
of the curve and solve for x or solve for y which will
provide us the value of x or value of y representing xcoordinate or y-coordinate of the points lying on the
curve.
Example: In the curve y2 = 4x, obtain the slope of
the curve at the point where y = 2.
Solution: Before finding the slope of the curve at a
point whose y-coordinate is provided in the question
as y = b (Use y = 2) x-coordinate should be determined
first.
Hence, y = 4 x y = 4 x 2 or 4x 4
2

x=1
Therefore, required point of the curve y2 = 4x is (x,
y) = (1, 2)
Notation:

LM dy OP
N dx QFH

x = x1
y = y1

IK

701

= value of derived function at

x = x1 and y = y1.
Examples worked out:
Type 1: Finding the slope of a curve at a given point
whose x and /y-coordinate is produced.

P(

1,

2)

2y

LM dy OP = 4
N dx Q

dy
4
2
=
= ; y0
dx 2 y
y

LM dy OP
N dx Q

y =2

LM 2 OP
NyQ

=
y=2

2
=1
2

which is the required slope.


Question: Find the slope of the curve y2 = 4x at the
point where x = 1.
Solution: Given equation is y2 = 4x
(1)

x = 1 y2 = 4 1 y = 4 = 2
Now, differentiating both sides of (1) w.r.t x, we get
dy
=4
dx
dy
4
2

=
=
dx 2 y
y
2y

LM 2 OP
N y QFGH

LM OP L dy O
N Q MN dx PQFGH

I=
J
y = 2K

x =1

2
=1
2

x =1
y =2

IJ
K
...(2)

702

How to Learn Calculus of One Variable

L dy O
and M P F
N dx QGH

x =1
y = 2

L2O
I = MN y PQ F
JK
GH

I
J
y = 2K

x =1

3 Given coordinates are (a, 0) and (0, b)

2
=
= 1
2

...(3)

y
(0, b)
(a , 0)

y
O

Now, differentiating both sides of (1) w.r.t x, we get

2x
a

Hence (2) and (3) are the required slopes.


Question: What is the slope of the curve y = 2x2
6x + 3 at the point in the curve where x = 2?

or

FG x
Ha

Solution: Given equation of the curve is


(1)
y = 2x2 6x + 3
Now, differentiating both sides of (1) w.r.t x, we get
dy
= 4x 6
dx

F
GH

dy
b2 x
= 2
dx
a y

I L dx O
JK MN dy PQFG
H

I
y = 0JK

x =a

=0

cot i = 0
i=

Similarly,

i
T

= 4x 6
x=2

x =2

I=0
J
y = bK

x=0

i = 0 which

Question: Find the slope of the tangent to the curve


y = 2x2 + 3 sin x at x = 0.
What is the slope of the normal at x = 0?
Solution: Given equation is y = 2x2 + 3 sin x (1)

= 4 2 6=8 6= 2

Question: Find the slope of the ellipse

LM dy OPF
N dx QGH

Tangent at (0, b) is parallel to x-axis.

Now, the slope of the curve at the point x = 2

LM dy OP
N dx Q

IJ
K

y dy
=0
b 2 dx

.
2
tangent at (a, 0) is perpendicular to x-axis.

2 y dy
=0
2
b dx

2
2

a
b
at the point where (i) x = a and (ii) y = b.
Solution: 3 Given equation of the ellipse is

2
2

=1

x2
y2
+
=1
(1)
a2
b2
When x = a, y = 0 [from the given equation]
When y = b, x = 0 [from the given equation]

i
T

Tangent and Normal to a Curve

dy
= 4 x + 3 cos x
dx
dy

= 4 x + 3 cos x
dx x = 0

Now, differentiating both sides of (1) w.r.t x, we get

LM OP
N Q

af

dy
2
= 3x + 2 x 4 = f x
dx

af

x =0

f x

= 4 0 + 3 cos 0 = 0 + 3 1 = 3

LM dy OP
N dx Q

x=0

LM dy OP
N dx Q

x =1
y = 2

=3+24=1

tan = 1
tan = tan 45

= 3 = Slope of the tangent to the

= 45 inclination = 45.

curve at x = 0.
Now again, we know that slope of the normal is
negative reciprocal of the slope of tangent at the
same point which means slope of the normal

1
.
3

Question: Find the slope of the curve y2 = 4x at the


point (1, 2).
Solution: Given equation of the curve is
y2 = 4x
(1)
Now, differentiating both sides of (1) w.r.t x
y

x =0

Question: Find the slope of the curve y2 = x at the


point x = 1.
Solution: (1) Given equation is y2 = x
When x =1, y = 1 Points are (1, 1) and(1, 1).
(2) Differentiating the given equation w.r.t x, we get
2y

1
dy
dy
= 1
=
dx
dx 2 y

dy
= 41
dx
4
2
dy

=
=
dx 2 y
y
2y

i
T

LM dy OP
N dx Q
dy
Slope at (1, 1) = L O
MN dx PQ

(3) Slope at (1, 1) =

i
T

703

x =1
y =1

x =1
y = 1

1
1
=
2 1 2
=

LM dy OP
N dx Q

x =1
y=2

LM 2 OP
N yQ

x =1
y =2

LM 2 OP
NyQ

=
y =2

2
=1
2

Question: Given the curve y = 6x x2, find the slope


of the curve at (x1, y1) and (0, 0).
Solution: Given equation of the curve is y = 6x x2
y

1
1
=
2 1
2

a f

Question: Find the slope and inclination of the curve


y = x3 + x2 4x at the point (1, 2) lying on the curve.
Solution: Given equation of the curve is
y = x3 + x2 4x
(1)

i
T

704

How to Learn Calculus of One Variable

Solution: Given equation of the curve is x = y2 4y


(1)
Now, differentiating both sides of (1) w.r.t x, we get

dy
= 6 2x
dx

LM dy OP
N dx Q

x = x1
y = y1

= 6 2x

x = x1
y = y1

= 6 2 x1

The slope of the tangent at (x1, y1)= 6 2x1


The slope of the given curve at (x1, y1)
= 6 2x1
Now, the slope of the tangent at (0, 0)
=

LM dy OP
N dx Q

x =0
y =0

= 6 2x

x=0
y=0

dy
dy
4
dx
dx
dy
1

=
(2)
dx 2 y 4
2
Since the curve x = y 4y cuts the y-axis where
x=0
1 = 2y

=6

(Slope of the curve at P is the slope of the tangent


at P).
x
Question: Find the slope of y = 2
at the origin.
x 1
Solution: Given equation of the curve is
x
y= 2
(1)
x 1
Now, differentiating both sides of (1) w.r.t x, we get

a f

x = 0 y 4 y = 0 y y 4 = 0 y = 0 ,4

The two points are (0, 0)and (0, 4) where the


curve crosses the y-axis.
Now, the slope of the curve at (0, 0)
=

LM dy OP
N dx Q

1
4

x=0
y=0

LM 1 OP = 1
N2y 4Q 2 0 4

...(3)
y
(0, 4)

y
V

(4, 2)

Again the slope of the curve at (0, 4)


i
T

O
2

x 1 2 x
dy
=
2
2
dx
x 1

1+ x

e x 1j
2

LM e1+ x j OP
=M
MN e x 1j PPQ
2

x =0
y =0

b g = 1 = 1
=
b0 1g 1
1+ 0

x =0

LM dy OP
N dx Q

x =0
y =4

LM 1 OP
N2 y 4 Q

x =0
y =4

1
244
1
1
=
=
84 4
=

The slope of the curve at the origin (0, 0)

L dy O
=M P
N dx Q

y=0

Question: Find the slopes of the curve x = y2 4y at


the point where it crosses the y-axis.

...(4)

Important results to be committed to memory:


1. The geometrical meaning of differential coefficient

dy
at the point (x, y) of a curve is the tangent of the
dx
angle which the tangent line to the curve at (x, y)
makes with the positive direction of the x-axis.

Tangent and Normal to a Curve

2. (i) The equation of the tangent to the curve y = f (x)


at the point (x1, y1) is y y1 =

where

LM dy OP
N dx Q a

p x1 , y1

LM dy OP
N dx Q a

p x1 , y1

stands for the value of

ax x f ,

Step 1: Find y1 if it is not given in the following way:


Put the given x-coordinate of the point P (x1, y1)
on the curve y = f (x) in the given equation of the
curve y = f (x) i.e., y1 = f (x1).

dy
at the
dx

Step 2: Find

point P whose x-coordinate = x1 and y-coordinate


= y1.
(ii) If the tangent line to the curve y = f (x) at the point

L dy O
(x , y ) is parallel to the x-axis, then M P
N dx Q a
1

p x1 , y1

p x1 , y1

dy
by differentiating the equation of
dx
the given curve w.r.t x using the rule of explicit and
implicit function provided the given equation is explicit
or implicit.
dy
at the given point
dx
P (x1, y1). This gives us the slope m of the tangent
line at the given point.
Step 4: Now put the value m in the equation of the
tangent line y y1 = m (x x1) where x1, y1 are the
given coordinates of the point where tangent line is
drawn to the curve.
Step 3: Find the value of

= 0.

(iii) If the tangent line to the curve y = f (x) at the


point (x1, y1) is perpendicular to the x-axis, then

LM dx OP
N dy Q b

705

= 0.

3. The angle of intersection of two curves is defined


to be the angle between the tangent lines to the curves
at their points of intersection.
4. We know from coordinates geometry that:
(i) If two lines are parallel, their slopes are equal.
(ii) If two lines are perpendicular, the product of their
slopes = 1 m1 m2 = 1 where m1 and m2 are
slopes of two lines perpendicular to each other if m1 =
slope of the tangent line at (x1, y1) to C1
m2 = Slope of the tangent line at (x2, y2) to C2.
(iii) Given equation y = f (x) = an expression containing only x terms Explicit function.
(iv) Given equation f (x, y) = 0 or c (c = any constant)
= an expression containing both x and y terms
Implicit function.
N.B.: Generally tangent means trigonometrical
tangent which is written in short tan. Whereas tangent
line means geometrical tangent which represents a
line to a curve at a point. Moreover, whenever we say
the tangent to a curve y = f (x) at the point P (x, y), we
understand geometrical tangent or the tangent line to
a curve at a point.
Type 2: Working rule to find the equation of the
tangent to any curve y = f (x) at the point (x1, y1) of
the curve whose x-coordinate x1 = a is given and ycoordinate is absent.

Note: 1. The equation of the tangent line must be


simplified as much as possible.
2. If x-coordinates and y-coordinate both of a point
P are given which is generally represented as P (x1, y1)

dy
by differentiating
dx
the equation of the given curve w.r.t x and we follow
the steps (3) to (4).
3. How to find the slope or the gradient of a given
curve y = f (x) at a point P (x1, y1) of the curve where
x-coordinate = x1 = a is given.
Method: (i) Find y1 if it is not given.
dy
(ii) Find
dx
or (x1, y1), then we directly find

(iii) Find

LM dy OP
N dx Q

x = x1 = a
y = y1 = f a

af

=m

4. Remember:
(i) If

LM dy OP
N dx Q

= 0 , then tan = 0 = 0 at P.

This means that the tangent line at P is parallel to the


x-axis.

LM dx OP = 0 then the tangent line at P is


N dy Q
perpendicular to the x-axis and parallel to y-axis.
(ii) If

706

How to Learn Calculus of One Variable

Notation: 1. Slope of the tangent line at P of the


curve y = f (x) is

LM dy OP
N dx Q

.
p

2. The slope of the normal at P of the curve y = f (x)


is

1
dy
dx

LM OP
N Q

.
p

3. Equation of the tangent to the curve y = f (x) at the


dy
point (x, y) is Y y =
(X x) since (x, y) is given
dx
point, so we use X and Y for current coordinates.
N.B.: 1. In question generally (x1, y1) notation are
used instead of (x, y) so we have

y y1 =

LM dy OP
N dx Q

x = x1
y = y1

a x x f for the equation of the


1

tangent. This notation in tangent line equation is


common in use.
dy
dy
2. The value of
at P (x, y) is denoted by
dx
dx
dy
dy
itself,
.
=
dx
x
x
=
dx

LM OP
N Q

y= y

3. The value of

LM dy OP
N dx Q

x = x1
y = y1

dy
at P (x1, y1) is given by
dx

dy
at the given point P (x1, y1).
dx
This gives the slope m of the tangent line at the
given point.
4. Now put the value m in the equation of the normal
1
y y1 =
x x1 .
m
Note: 1. If x-coordinate and y-coordinate both of a
dy
point P are given, then we directly find
by
dx
differentiating the equation of the curve w.r.t x and
we follow the steps (3) and (4).
2. The derivative of the function y = f (x) at the point
x0 is equal to the slope of the tangent line to the
graph of the function y = f (x) at the point M (x0, f (x0)).
3. Find the value of

Question: What is the slope of the normal to the curve


y = 2x2 6x 3 at the point in the curve where x = 2.
Solution: Given equation of the curve is
y = 2x2 6x 3
(i)
Now, differentiating both sides of (i) w.r.t x, we get
dy
= 4x 6
dx
dy
= slope of the tangent line to the
Now,
dx x = 2

LM OP
N Q

graph of the function y = f (x) at the point (2, 7)


= (x0 , f (x0)).
y

=m

About the figure


Draw the figure provided it is easily possible to do
so even though drawing a rough sketch of the figure
is not necessary.
Working rule to find the equation of the normal at
the point P (x1, y1) whose x-coordinate x1 = a is given
( P lies on the curves y = f (x)).
1. Find y1 by putting x1 in the given equation of the
curve y = f (x) i.e., y1 = f (x1).
dy
2. Find
by differentiating the equation of the
dx
given curve w.r.t x using the rule of explicit and implicit
function provided the given equation is explicit or
implicit function.

P (2, 1)
= (x 0, f (x 0))

[3 y = 2x2 6x 3
[y]2 = 2 4 6 2 3
= 8 12 3
= 7]
= [4x 6]x = 2
=426
=86
=2

1
slope of the normal at (2, 7) is .
2

Tangent and Normal to a Curve

a f a fa f
y 1 = 1 a xf

Examples worked out:

y 1 = 1 x 0

Question: Find the equation of the tangent to the


curve y = 10 x

at the point whose x-coordinate

is 1.
Solution: (1) Given equation of the curve is
y = 10 x

x = 1 y = 10 1 =

f a f

9 =3

x1 , y1 = 1 , 3
(2) Now, differentiating both sides of the given

equation y = 10 x

dy
=
dx

L dy O
(3) m = M P
N dx Q

w.r.t x

1
2 10 x
=

x =1

a f.

2 x

1
=
3
10 1

(4) Equation of the tangent is (y y1) = m (x x1)

a y 3f = 13 a x 1f

i.e.,

Question: Find the equation of the normal to the curve


y = ex at the point where x = 0.
Solution: (1) Given equation of the curve is y = ex
3 x = 0 y = e = 1

x1 = 0 y1 = 1
x

(2) y = e
(3) m =

LM dy OP
N dx Q

= e

x =0
y =1

= e = 1

1
1
=
= 1
dy
m
dx x = 0
y =1

(4) Equation of the normal at (0, 1) is ( y y1 )

1
x x1
m

3x =
x1 =

1
y = sin =
6
6 2
1

y1 =
6
2

f FH 6 , 21 IK

x1 , y1 =
Now

dy
= cos x
dx

LM dy OP
N dx Q

x= 6
y =1 2

= cos

=
6
2

The equation of the tangent at (x1, y1) is


y y1 =

F
H

LM OP
N Q

y 1 = x
y + x 1= 0
Question: Find the equation of the tangent to the

curve y = sin x at x = = 30 .
6
Solution: Given equation of the curve is y = sin x
(1)

x
dy
=e
dx

x =0
y =1

707

LM dy OP
N dx Q
I
K

x = x1

bx x g
1

y = y1

1
3
=
2
2

F x I
H 6K

3
1
3
3
1
3
x=
y
x=
2
2 2 6
2
2 12

Question: Find the equation of the normal to the curve


y = 2x + 3x3 at the point where x = 3.
Solution: Given equation is y = 2x + 3x3
(1)
3

3 x = 3 y = 2 3 + 3 3 = 87
x1 = 3 , y1 = 87
(x1, y1) = (3, 87)
Now, we are required to find out the equation of
the normal at (3, 87).
Differentiating (1) w.r.t x, we get

708

How to Learn Calculus of One Variable

Now, slope of the tangent to the curve at the point


(1, 1).

dy
2
2
= 2 + 3 3x = 2 + 9 x
dx

LM dy OP
N dx Q

Now, m =

x = x1

= 2 + 9x2

x =3

= [2 + 9 32] = [2 + 9 9] = 83

1
1
Slope of the normal = =
m
83
Equation of the normal is

ay y f
1

1
=
x x1
m

y 87 =

a f

1
x3
83

83 y 7221 = x + 3
x + 83y = 7224

2 2x 3 + 2 2 y

dy
dy
x
y 1 = 0
dx
dx

dy
4 y x = 4x + y + 3
dx
dy 4 x + y + 3

=
4y x
dx

x =1

=0

2 1 3 1 + 2 y 1 y = 0

F1 , 1 I .
H 2K
L dy O
=M P
N dx QFH

fa f

2 y +1 y 1 = 0
1
y = 1 or y =
2
x = 1 corresponds to two points whose

F
H

coordinates are (1, 1) and 1 ,

I
K

1
.
2

x =1
y =1

(2)

x =1
y = 1 2

IK

LM 3 4 x + y OP
N 4y x Q

x =1
y = 1 2

3 4 12 1
=
2
2 1

(3)

(2) The tangent at (1, 1) is parallel to x-axis


since

LM dy OP
N dx Q

x = x1
y = y1

= 0 tangent is parallel to x-axis

through (1, 1).


the normal is parallel to y-axis through (1, 1)
The equation of this normal is x = 1.

F
H

I
K

1
1
=
(3) The slope of the tangent at 1 ,
2
2

The slope of the normal is negative reciprocal


of

1
= 2.
2

F
H

Equation of the normal at 1 ,

LM dy OP b x x g
N dx Q
F 1 I = 2 a x 1f
y
H 2K
1
y + = 2 a x 1f
2
y y1 =

2y y 1 = 0

3 4 +1 0
= =0
41
3

IK

x =1
y =1

Slope of the tangent to the curve at the point

Now putting x =1 in the equation of the curve, we


get

2 x 2 3x + 2 y 2 xy

Question: Find the equation of the normal to the curve


2x2 3x + 2y2 xy = 0 at the point where x = 1.
Solution: Given equation of the curve is
2x2 3x + 2y2 xy = 0
(1)
Now, differentiating (1) w.r.t x, we get

LM dy OP
N dx QFH

y = 87

y = y1

LM 3 4 x + y OP
N 4y x Q

x =1

y = 1 2

1
2

I is
K

Tangent and Normal to a Curve

1
= 2x + 2
2
1
y + + 2 x 1 = 0
2
3
y + 2x = .
2
Question: Find the equation of the tangent and
normal to the curve y2 (2a x) = x3 at points on it
where x = a.
Solution: Given equation of the curve is
y2 (2a x) = x3
(1)
Put x = a in (1), we get y2 (2a a) = a3
y +

y a = a y = a y = a

Two points (a, +a) and (a, a) correspond to x


=a
Now, differentiating the equation of the curve (1)

dy
2
2
2a x y = 3 x
dx
2
2
dy 3x + y
2y
=
2a x
dx

w.r.t x, we get 2 y

1 3x + y
dy

2a x
dx 2 y

LM dy OP
N dx Q

x =a

a f a f

dy
= 3x 2 4 x
dx
The slope of the tangent at (2, 4)
=

LM dy OP
N dx Q

x =2

= 3x 2 4 x

bg bg
2

x =2

=3 2 4 2 =4

y =4

Question: Find the equation of the tangent to the

F 1I .
H 3K

curve xy = 1 at the point 3 ,

1 4a 2

=2
2a
a

Solution: xy = 1 is the given equation of the curve.


(1)
Differentiating both sides of this equation (1) w.r.t

The slope of the tangent = 2 at (a, a)

dy
=0
dx
dy
y

=
dx
x

x, we get y + x

1
at (a, a)
2

Similarly,
The slope of the tangent = 2 at (a, a)

F 1I = 1
H 2K 2

The slope of the normal =

(a, a).
Now, the equation of the tangent at (a, a) is
(y a) = +2 (x a) y 2x + a = 0
Equation of the normal at (a, a) is
1
ya =
x a
2
2 y + x 3a = 0

a f

y + a = 2 x a y + 2x a = 0
and the equation of the normal at (a, a) is
1
y+a =
x a 2 y x + 3a = 0
2
Question: Find the equation of the tangent to the
curve y = x3 2x2 + 4 at (2, 4).
Solution: y = x3 2x2 + 4 is the equation of the curve.
(1)
Differentiating both sides of (1) w.r.t x, we get

y 4 = 4 x 2 y 4x + 4 = 0

y =a

a f

Similarly, the equation of the tangent at (a, a) is

Now, the equation of the tangent to the curve at


(2, 4) is

The slope of the normal =

709

F 1I is
H 3K

The slope of the tangent at 3 ,


at

LM dy OP
N dx Q

x=3
y =1 3

LM y OP
NxQ

x =3
y =1 3

13
3

F 1I is
H 3K

The equation of the tangent at 3 ,

F y 1I = 1 a x 3f
H 3K 9
x + 9y 6 = 0

1
9

710

How to Learn Calculus of One Variable

Question: Find the equations of the tangent and


normal at (1, 4) to the curve y = 2x2 3x + 5.
Solution: y = 2x2 3x + 5 is the given equation of the
curve.
(1)
Now, differentiating both sides of the equation (1)

dy
= 4x 3
dx
Now, the slope of the tangent at (1, 4)

w.r.t x, we get

=m=

LM dy OP
N dx Q

x =1
y=4

= 4x 3

= 43 = 1
x =1

Solution: Given equation is y = (x3 1) (x 2) (1)


The curve cuts the x-axis where y = 0 and y = 0
(x3 1) (x 2) = 0
either (x 2) = 0 or (x3 1) = 0
x = 2 or x3 = 1
x = 2 or x = 1 x = 1, 2
Thus, the points are (1, 0) and (2, 0).
Now, differentiating both sides of (1) w.r.t x, we get

a f e j
L dy O = 4 x 6 x
M P
N dx Q

dy
2
3
3
2
= 3x x 2 + x 1 1 = 4 x 6 x 1
dx
3

Negative reciprocal of the slope of the tangent

1
= = 1
m
Equation of the tangent to the curve at (1, 4) is

b y y g = m bx x g y 4 = 1bx 1g
1

y x 3=0
Equation of the normal to the curve at (1, 4) is
y y1 =

g b g

b g

1
1
x x1 y 4 = x 1
1
m

a f

y 4 = x 1

y 4 = x + 1 x + y 5 = 0
Question: Obtain the equation of the tangent to the
curve 3y = x3 3x2 + 6x at the point (3, 6) on it.
Solution: Given equation is 3y = x3 3x2 + 6x (1)
Now, differentiating the equation (1) w.r.t x we get

dy
dy
2
2
3
= 3x 6 x + 6
= x 2x + 2
dx
dx
The slope of the tangent at (3, 6) on the curve
=

LM dy OP
N dx Q

x =1
y =0

x =1

=47=3
and

LM dy OP
N dx Q

x=2
y =0

= 4 x 3 6x 2 1

x=2

=48641=7
Now, the equation of the tangents at (1, 0) is

a y 0f = LMN dydx OPQ a x 1f


a f

x =1
y =0

y0 = 3 x 1

y = 3x + 3

y + 3x 3 = 0
The equation of the tangents at (2, 0) is

a y 0f = LMN dydx OPQ a x 2f


a y 0f = 7 a x 2 f
x=2
y =0

y = 7 x 14

x =3
y =6

= [x2 2x + 2]x = 3 = 32 2 3 + 2 = 9 6 + 2 = 5
Equation of the tangent is y 6 = 5 (x 3)
y 6 = 5x 15 y 5x = 15 + 6
y 5x = 9 y 5x + 9 = 0 5x y 9 = 0
Question: Find the equation of tangents to the curve
y = (x3 1) (x 2) at the points where the curve cuts
the x-axis.

y 7 x + 14 = 0
Question: Find the equation of the tangent to the
curve x2 + 4y2 4x = 0 at the point (4, 0).
Solution: Given equation is x2 + 4y2 4x = 0 (1)
Now, differentiating both sides of (1) w.r.t x, we get

dy 2 x
=
dx
4y

711

Tangent and Normal to a Curve

F dx I
G J
H dy K

Solution: Given equation of the curve is y2 = x


x=4
y=0

=0

(1)
and (x1, y1) = (1, 1)
Differentiating both sides of (1) w.r.t x, we get

The tangent is perpendicular to x-axis or


parallel to y-axis.
In this case the tangent line is the line through
the point (4, 0) parallel to y-axis.
Its equation is x = 4.

2y

1
dy
dy
= 1
=
dx
dx 2 y

Now, the slope of the tangent at (1, 1)


=

LM dy OP
N dx Q

x =1
y =1

LM 1 OP
N2 y Q

x =1
y =1

1
2

The slope of the normal at (1, 1) = negative


reciprocal of slope of tangent = 2
Equation of the tangent is

LM dy OP a x x f
N dx Q
1
a y 1f = a x 1f x 2 y + 1 = 0
2

90

(y y1) =

(4, 0)

Question: Find the equation of the tangent at the


point (1, 1) on the curve x3 xy2 4x2 xy + 5x + 3y
+ 1 = 0.
Solution: Given equation of the curve is x3 xy2
4x2 xy + 5x + 3y + 1 = 0
(1)
Now, differentiating both sides of (1) w.r.t x, we get
2

3x 2 xy

dy
dy
dy
2
y 8x x y + 5 + 3
=0
dx
dx
dx

dy
2
2
3 x 2 xy + 3x y 8x y + 5 = 0
dx
2

y 3x + 8 x + y 5
dy
=
3 x 2 xy
dx

The slope of the tangent to the curve at (1, 1)

LM dy OP
N dx Q

x =1
y = 1

1 3 + 8 1 5 0
= =0
3 1+ 2
4

Hence, the required equation of the tangent to the

b a fg LMN dydx OPQ

curve at (1, 1) is y 1 =

x =1
y = 1

x1

y + 1 = 0 (x 1)
y+1=0
Question: Find the equations of the tangent and
normal to the curve y2 = x at the point (1, 1).

x =1
y =1

Again equation, of the normal to the curve at (1, 1)


is

1
bx x g
b y y g = Slope of the
tangent
1

b g

y 1 =

b g b g

b g

1
x 1 y 1 = 2 x 1
1
2

(y 1) = 2 (x 1)
(y 1) = 2x + 2
2x + y 3 = 0
Question: Find the equations of tangents and normals
to the curve yx2 + x2 5x + 6 = 0 where it cuts the axis
of x.
(1)
Solution: The curve yx2 + x2 5x + 6 = 0
Cuts the axis of x at the points where y = 0
y = 0 0 x2 + x2 5x + 6 = 0 x2 5x + 6 = 0
(x 2) (x 3) = 0 x = 2, 3
Thus, the given curve cuts the x-axis at the points
(2, 0) and (3, 0).
Now, differentiating both sides of (1) w.r.t x, we get
2 dy
y 2x + x
+ 2x 5 = 0
dx

712

How to Learn Calculus of One Variable

dy 5 2 x 2 xy
=
2
dx
x
Now, the slope of the tangent at (2, 0)

LM dy OP
N dx Q

x =2
y =0

LM5 2 x 2 xy OP
N x Q
2

52 2 2 2 0
2

5 2 3 2 3 0
2

x=2
y =0

L dy O
=M P
N dx Q

5 4 1
=
4
4

1
9

a y 0f = LMN dydx OPQ a x 2f


f

1
x2
4
4y = x 2
Again the tangent to the curve at (3, 0) is

Question: Find the equation of the tangent to the


curve y =

x
2

x +1
x

x +1

LM
1 x
=M
MN e1 + x j
2

x=0
y=0

2 2

a y 0f = LMN dydx OPQ

OP
PP
Q

=
x=0

LM 1 0
MN a0 + 1f

OP = 1
PQ

x =0
y =0

a f

x0

y 0 = 1 (x 0)
y=x

Now, m =

= e

LM dy OP
N dx Q

x
x =0
y =1

(1)

Now differentiating both sides of the equation (1)


w.r.t x, we get

x =0
y =1

= the value of

dy
at (0, 1)
dx

= e = 1

Slope of the tangent passing through (0, 1) = 1


Hence, the equation of the normal to the curve at
(0, 1) is
1
y y1 =
x x1
m
1
y 1 =
x0
m
(y 1) = 1 (x 0)
y+x1=0

a f

at (0, 0).

Solution: Given equation of the curve is

y=

x
dy
=e
dx

1
x3
9
9y = x + 3
x + 9y = 3
Now, the slopes of the normal at (2, 0) and (3, 0) are
4 and 9 respectively.
Normal at (2, 0) is y 0 = 4 (x 2) y + 4x = 8
And normal at (3, 0) is y 0 = 9 (x 3)
y = 9x 27
y 0 =

Question: Find the equation of the normal to the curve


y = ex at the point (0, 1).
Solution: Given equation is y = ex
(1)
Now, differentiating both sides of (1) w.r.t x, we get

x=2
y =0

Now, the equation of the tangent at (0, 0) is

3
The
equation
of the tangent to the curve at

(2, 0) is

y0=

Now, the slope of the tangent to the curve at (0, 0)

2
again, the slope of the tangent at (3, 0)
=

1 x
dy x + 1 2 x
=
=
2
2
2
dx
1+ x
x +1

a f

Question: Find the equation of the normal to the curve


9x2 4y2 = 108 at the point (4, 3).
Solution: Given equation of the curve is 9x2 4y2
= 108
(1)

Tangent and Normal to a Curve

Now, differentiating both sides of (1) w.r.t x, we get

9 2x 4 2 y

dy
=0
dx

dy 9 x
=
dx 4 y

Now, m =

L9x O
=M P
N4 y Q

LM dy OP
N dx Q

= The value of

x =4
y =3

dy
at (4, 3)
dx

94
=
=3
x =4
43

1
1
=
m
3
Hence, the required equation of the normal is

The slope of the normal at (4, 3) =

a y 3f = 13 a x 4f
3y 9 = x + 4
Question: Find the equations of the tangent and
normal to the circle x2 + y2 = a2 at the point (x1, y1).
Solution: The point (x1, y1) lies on the curve
2

(1)

x = x1
y = y1

point (x1, y1)

L xO
= M P
N yQ

x = x1
y = y1

+ 2 = 1 at the point (x1, y1).


2
a
b
Solution: Equation of the curve is
(1)
b2 x2 + a2 y2 = a2 b2
Differentiating both sides of (1) w.r.t x, we get

dy
=0
dx
dy
b2 x

= 2
dx
a y
The value of
=

LM dy OP
N dx Q

x = x1

dy
at the point (x1, y1)
dx

y = y1

b 2 x1
a 2 y1

=m

a 2 y y1 a 2 y12 = b 2 x x1 + b 2 x12
dy
at the
dx

2 2

2 2

b x x1 + a y y1 = b x1 + a y1
Now, dividing both sides by a2b2, we get
x x1

x
= 1
y1

to the ellipse

dy
dy
x
2x + 2y
=0
=
dx
dx
y

= The value of

Required equation of the tangent is


b2 x
y y1 = 2 1 x x1
a y1

d
d
2
2
2
x +y =
a
dx
dx

LM dy OP
N dx Q

(2)
x1 + y1 = a
Now, differentiating both sides of the given
equation of the circle (1) w.r.t x, we get

Now m =

2b 2 x + 2a 2 y

x + 3 y = 13

Required equation of the tangent to the curve


x1
x x1
at the point (x 1 , y 1 ) is y y1 =
y1
y y1 y12 = x x1 + x12 x x1 + y y1 = x12 + y12
= a2 from (2)
Again the slope of the normal at (x1, y1) negative
reciprocal of the slope of tangent
Required equation of the normal to the curve at
y1
x x1 x 1 y x 1 y 1
(x 1, y 1 ) is y y1 =
x1
= xy1 x1 y1 xy1 x1 y = 0
Question: Find the equation of the tangent and normal

y =3

x 2 + y 2 = a2

713

y y1

x1

y1

=1
2
2
2
2
a
b
a
b
[ 3 x1 , y1 lies on the ellipse]

714

How to Learn Calculus of One Variable

Now, the slope of the normal = Negative reciprocal


of the slope of the tangent

a 2 y12
b 2 x12

b y y g = ab xy b x x g

b y y g = ab xy b x x g
2

a 2 x x1
x1

g = b by y g
2

y1

a x
b y
a2 =
b2
x1
y1

a2 x b2 y

= a 2 b2
x1
y1

Question: Find the equation of the normal to the


x

= 1 at the point (x , y ).
2
2
1 1
a
b
Solution: Given equation of the curve is
x

(i)

2x
a

=1
(1)
2
a
b
Now, differentiating both sides of the equation of
the curve (1) w.r.t x, we get,
2

2 y dy
=0
2
b dx

dy b 2 x
=
dx a 2 y

LM dy OP
N dx Q

x = x1
y = y1

b 2 y y1
y1
x x1

g = a b x x g
2

x1

y y1

FG x IJ FG y IJ
H a K H b K
=

1
2

1
2

Question: Find the equation of the tangent and


normal to the curve y (2a x) = x2 at the point (a, a).
Solution: The equation of the curve is
y (2a x) = x2
(1)
Now, differentiating both sides of the equation (1)
w.r.t x, we get,

a f a

y 0 1 + 2a x

f dydx = 2 x

dy 2 x + y
=
dx 2a x

LM dy OP
N dx Q

x =a
y =a

2a + a
=3
2a a

1
3
Hence, the equation of the tangent at (a, a) is
y a = 3 (x a)
y = 3x 2a and the equation of the normal is
The slope of the normal at (a, a) =

dy
at the point (x1, y1)
The value of
dx
=

curve

b 2 x1

Hence, the equation of the normal at (x1, y1) is

Required equation of the normal to the curve at


(x1, y1) is

a 2 y1

b 2 x1
a 2 y1

The slope of the normal at (x1, y1)

1
(x a)
3
3y 3a = x + a
x + 3y = 4a

ya=

Question: Prove that the normal to the curve 9x2


4y2 = 128 at the point (4, 2) lying on it intersects the xaxis at a distance 13 from the origin.

Tangent and Normal to a Curve

Solution: Given equation of the curve is


9x2 4y2 = 128
(1)
Differentiating both sides of the equation (1) w.r.t
x, we get,

9 2x 4 2 y

dy
=0
dx

dy 9 x
=
dx 4 y

Examples worked out:


Question: The equation of the tangent at the point
(2, 3) on the curve y2 = ax2 + b is y = 4x 5, find the
values of a and b.
Solution: Given equation of the curve is
y2 = ax2 + b
(1)
On differentiating both sides of (1) w.r.t x, we get

2y

dy
= 2ax
dx

dy ax
x
=
=a
dx
y
y

LM dy OP
N dx Q

Now, the slope of the tangent at (4, 2)

L dy O
=M P
N dx Q

x =4
y =3

L 9x O
=M P
N4 y Q

x =4
y =2

94 9
=
=
42 2
and the slope of the normal at (4, 2)
=

1
2
=
slope of tangent
9

Required equation of the normal at (4, 2) to the


curve 9x2 4y = 128 is
2
y y1 =
x x1
9
2
y2 =
x4
9
9y 18 = 2x + 8
2x + 9y = 26
x = 13 when y = 0

b g

Problems based on finding the constants present in


the equation of the curve.
Working rule:
1. Obtain the equation of the tangent to the curve.
2. Write the given equation of the tangent to the
curve.
3. Equate the coefficients of x in both the equations
of the curve which will given us the value of one
constant.
4. Put this value of one constant in the equation of
the curve which contains another constant passing
through the given point (x1, y1).
N.B.: Given conditions provide us the value of
required constants.

x=2
y=3

LM
N

= a

x
y

OP
Q

x =2
y =3

2
a = slope of the tangent at (2, 3)
3
Now, the equation of the tangent is y = 4x 5
which tells us the slope of the tangent = 4
(3)
Equating
equations,
(2)
and
(3),
we
get

2
a=4
3

b g

715

3
=6
2
Again equation of the curve is y2 = ax2 + b which
passes through (2, 3).
a=4

32 = 6 2 2 + b

9=64+b
9 24 = b
b = 15
Hence, a = 6, b = 15
Note:

b y y g = LMN dydx OPQ


1

x = x1
y = y1

bx x g

a f 23 a a x 2f
2
4a
b y 3g = ax
3
3
y3 =

716

How to Learn Calculus of One Variable

Note:

2
4a
ax
+3
3
3
2
4a
y = ax
3
3
3
y=

FG
H

Putting this value of a =

IJ
K

(1)

Comparing this equation (1) with the given


equation of the tangent y = 4x 5 and equating the
coefficients of x in both equations, we get

y=

4.6
4a
3 = 5 , as it should
3 =
3
3

Question: If the equation of the normal to the curve


y = x3 + x 2 at the point (1, 0) on it is y = ax + b, then
find the value of a and b.
Solution: Given equation of the curve is
y = x3 + x 2
(1)
Differentiating (1) w.r.t. x, we get

1
=b
4
1
b =
4

Question: If there are two values of a such that the


tangents at x = 1 and x = 3 to the curve
y = ax2 2x 1
are perpendicular, find the two values of a.
Solution: Given equation of the curve is
y = ax2 2x 1
...(1)
Differentiating (1) w.r.t. x, we get

dy
= 2ax 2
dx

dy
= 3x 2 + 1
dx
Slope of the tangent at (1, 0)
2

x =1
y=0

= 3x + 1

x =1
y =0

= 3 +1 = 4
and

1
Slope of the normal at (1, 0) =
4
The equation of the normal to the curve at (1, 0) is

b y y g = Slope of1tangent b x x g
1
a y 0f = a x 1f
4
1

1
1
x+
(2)
4
4
Also the equation of the normal is y = ax + b
Comparing eqn (2) with the given equation of the
normal y = ax + b and equating the coefficients of x,
1
1
and b =
4
4

LM dy OP
N dx Q

LM dy OP
N dx Q
x =3

x =1

= 2a 2 = m1 (say)

(2)

= 2 3a 2 = 6a 2 = m2 (say) (3)

The tangents at x = 1 and x = 3 are perpendicular.


m1 m2 = 1

(2a 2) (6a 2) = 1
12a2 16a + 5 = 0

y=

a=

1
+b
4

0+

be.

L dy O
=M P
N dx Q

1
+b
4

and this line passes through (1, 0) 0 =

43
2
a=4a =
= 6.
3
2

We see that

1
in y = ax + b, we get,
4

(3)

(2a 1) (6a 5) = 0
1
5
a = or a =
2
6
Problems based on perpendicularity and touching.
Working rule to show that two curves intersect at
right angles:
1. Find the point of intersection of two curves by
solving the given equations simultaneously provided
the point of intersection is not given.

Tangent and Normal to a Curve

2. Find the differential coefficient

dy
from each given
dx

equation.

bg

bg

d
d
f x and
g x at the
dx
dx
point of intersection which will represent m1 and m2
respectively.
4. If m1 m2 = 1 , then two curves cut orthogonally
or they intersect at right angles at the point of
intersection.
3. Find the value of

Note: 1. To show that two curves are perpendicular


to each other at a given point (x1, y1) means (x1, y1) is
the point of intersection of two given curves whose
equations are given. This is why there is no need to
find the point of intersection of two curves.
2. To show that two tangents to a curve y = f (x) at a
given point x0 are perpendicular to each other means
we have to show m1 m2 = 1 , where

LM dy OP
N dx Q
L dy O
=M P
N dx Q

m1 =

m2

x = x0
y = f x0

a f

x = x0

b g

y = f x0

x0 = given x-coordinate of the point where the

b g

bg

tangents are perpendicular and f x0 = f x

x = x0

is required to find out.


Working rule to show that two curves touch each
other
1. Find the point of intersection of two curves by
solving simultaneously by the given equations of
the curves (or, the curve and a line) provided the
point of intersection is not given.
dy
from each given
2. Find the differential coefficient
dx
equation.
d
d
f x and
g x at the
3. Find the values of
dx
dx
point of intersection which will represent tan 1 and

bg

tan 2 respectively.

bg

717

4. If tan 1 = tan 2 i.e. 1 = 2 then two


curves touch each other.
Note: 1. To show that two curves touch each other
at a given point (x1, y1) means (x1, y1) is the point of
intersection. This is why there is no need to find the
point of intersection of two curves.
Question: What is the criteria to show that two curves
touch each other?
Solution: The two curves touch each other provided

LM d f b xgOP = LM d gbxgOP
N dx Q N dx Q
p

3 tan 1 = tan 2 1 = 2
Where P is the point of intersection determined
by solving the given equation if it is not given.
Examples worked out:
Question: Show that the curves 2y = 3x + x2 and
y2 = 2x + 3y intersect at right angles at the origin (0, 0).
N.B.: Here the point of intersection (0, 0) origin is
given.
There is no need to determine the point of
intersection.
Solution: The curves are 2y = 3x + x2
(1)
and y2 = 2x + 3y
(2)
Now, differentiating (1) w.r.t x, we get

dy
= 3 + 2x
dx

dy 3
= +x
dx 2
Again differentiating (2) w.r.t x, we get

2+3

dy
dy
= 2y
dx
dx

dy
2y 3 = 2
dx
dy
2

=
dx 2 y 3

(3)

Now, from (3), m1 =


Again, from (4),

LM dy OP
N dx Q

(4)

x =0
y =0

LM 3 + xOP
N2 Q

x =0
y =0

3
2

718

How to Learn Calculus of One Variable

m2 =

LM dy OP
N dx Q

x =0
y =0

LM 2 OP
N2y 3Q

x =0
y =0

2
3

2 3
= 1
3 2
Two curves are perpendicular to each other at
the origin.
Now, m1 m2 =

Question: Show that the curves y = x3 and 6y = 7 x2


intersect orthogonally.
Solution: Given equations of the curve are
(1)
y = x3
and 6y = 7 x2
(2)
Now, from (1) and (2), we get
6x3 = 7 x2

6x 3 + x 2 7 = 0

a fe
j
a x 1f = 0 or e6 x + 7 x + 7 j = 0
2

x 1 6x + 7 x + 7 = 0

x=1
[ 3 The quadratic equation 6x2 + 7x + 7 = 0 has
imaginary roots, therefore the only root under
consideration is x = 1]
Again, differentiating both sides of (1) w.r.t x, we
get
dy
2
= 3x
(3)
dx
and differentiating both sides of (2) w.r.t x, we get
dy
dy
2x
dy
x
= 2x
=

=
dx
dx
6
dx
3

LM dy OP = 3x
N dx Q
L dy O L x O
= M P = M P
N dx Q N 3 Q

Now, m1 =

m2

(4)

x =1

x =1

x =1

= 3 from (3).

=
x =1

1
from (4).
3

1
= 1
3
The curves intersect orthogonally.

Hence, m1 m2 = 3

p
in (1), we get
2
1
y2 = 2 p p = p2 y = p
(2)
2
1
1
p , p and
p, p
Points are
2
2

Putting x =

F
H

I
K

F
H

I
K

1
p
2
Now, differentiating both sides of (1) w.r.t x, we get

corresponding to x =

dy
= 2 p1
dx
dy
p

=
dx
y
2y

Question: Show that the tangents to the curves


1
y2 = 2px at the points where x = p are at right
2
angles.
Solution: Given equation of the curve is
(1)
y2 = 2px

LM dy OP
N dx Q
L dy O
=M P
N dx Q

Now, m1 =

m2

x = 21 p
y= p

x = 12 p
y=p

p
=1
p
p
= 1
p

b g

Hence, m1 m2 = 1 1 = 1

Two tangents are perpendicular to the curve


1
1
p , p and
p, p .
y2 = 2px at the points
2
2

F
H

I
K

F
H

I
K

Question: Prove that the normals to the curve


y2 = 4ax at the point where x = a are perpendicular to
each other.
Solution: Given equation of the curve is
y2 = 4ax
(1)
2
2
Putting x = 1 in (1), we get y = 4a y = 2a
Points are (a, 2a) and (a, 2a).
Now, we have to show that tangents at (a, 2a) and
(a, 2a) are perpendicular.

719

Tangent and Normal to a Curve

Now, differentiating given equation (1) w.r.t x, we


get

2y

2a
dy
dy 4a
= 4a
=
=
dx
dx 2 y
y

LM dy OP
N dx Q
LM dy OP
N dx Q

and

x=a
y = 2a

x =a
y = 2a

2a
= 1 = m1
2a

L 2a O
=M P
NyQ

Hence, m1 m2 = 1
Tangents are perpendicular to each other.
Normals are perpendicular to each other.
Question: Show that the normals to the curve
3
y2 = 3x at x = are at right angles to each other.
4
Solution: Given equation of the curve is y2 = 3x (1)

y=

3
2

and

LM dy OP
N dx Q

=
y = 32

Again,

LM dy OP
N dx Q

x = 34
y = 32

= tan 2 = 1 = tan 135

tan 2 = tan 135 2 = 135

Now = 2 1
= 135 45 = 90 the two tangents are
perpendicular to each other.
Hence the normals are also perpendicular to each
other.
y

GH 43 , 23 JK
45

F 3 , 3 I and F 3 , 3I
H 4 2K H 4 2K

3
2

3
2

= 1 = m1

FG 3 IJ = 1 = m
H 2K
F 1 IF 1 I
Hence G J G J = 1.
H m KH m K

The normals are at right angles.

...(2)

y = 23

= tan 1 = 1 = tan 45 tan 1

3
3 9
y2 = 3 =
4
4 4

Now, differentiating both sides of eqn (1) w.r.t x,


we get
3
dy
dy
2y
= 31
=
dx
dx 2 y

LM dy OP
N dx Q

x = 34
y = 32

135

Given points are

Now,

LM dy OP
N dx Q

= tan 45 1 = 45

2a
=
= 1 = m2
2a
y = 2 a

From (1), when x =

Alternative Method:

(3)

GH 43 , 23 JK
Question: Show that the curves y2 = 4x and x2 + y2
6x + 1 = 0 touch each other at the point (1, 2).
Solution: Here the point of intersection of the curves
= (1, 2)
y2 = 4x
(1)
and x2 + y2 6x + 1 = 0
(2)
Now, differentiating both sides of eqn (1) w.r.t x,
we get
dy
=4
2y
dx
4
2
dy

=
=
(3)
dx 2 y
y
Again, differentiating both sides of eqn (2) w.r.t x,
we get
dy
2x + 2 y
6=0
dx

720

How to Learn Calculus of One Variable

dy 6 2 x 3 x
=
=
dx
2y
y

(4)

Now the slope of the tangent at (1, 2) from eqn (3)


to the curve (1),
m1 =

LM 2 OP
NyQ

x =1
y=2

2
=1
2

(5)

And the slope of the tangent at (1, 2) from (4) to


the curve (2),

m2 =

LM 3 x OP
N yQ

x =1
y =2

3 1 2
= =1
2
2

(6)

m1 = m2
The two curves touch each other at (1, 2).
Question: Show that the curves xy = 4 and x2 + y2 = 8
touch each other.
Solution: The equations of the given curves are
xy = 4
(1)
x2 + y2 = 8
(2)
Differentiating (1) w.r.t x, we get
dy
1 y + x
=0
dx
dy
y

=
...(3)
dx
x
Differentiating eqn (2) w.r.t. x, we get
dy
=0
2x + 2 y
dx
dy
x

=
...(3)
dx
y
Now, we are required to find the point of
intersection.
4
3 xy = 4 y =
x
4
Putting y =
in x2 + y2 = 8, we get
x
16
x 2 + 2 = 8 x 4 + 16 = 8 x 2
x

x 4 8 x 2 + 16 = 0 x 2 4
2

x 4 = 0 x = 4 x = 2

=0

From (1), when x = 2 , y =

4
=2
2

4
= 2
2
Hence, the point of intersection of the two curves
are (2, 2) and (2, 2).
The slope of the tangent to the curve xy = 4 at
(2, 2)
When x = 2 , y =

LM dy OP
N dx Q

= m1 =

x=2
y =2

LM y OP
N xQ

x=2
y =2

2
= 1
2

(5)

The slope of the tangent to the curve x2 + y2 = 8 at


(2, 2)

LM dy OP
N dx Q

= m2 =

x=2

LM x OP
N yQ

y=2

x=2

2
= 1 (6)
2

y=2

Thus, we get m1 = m2 = 1 The two curves


touch each other at (2, 2). Similarly, we can show that
the two curves touch each other at (2, 2).
Hence, the two curves xy = 4 and x2 + y2 = 8 touch
each other.
n
n
x
y
+
=2
Question: Show that the curve
a
b
x
y
+ = 2 at the point
touches the straight line
a b
(a, b) whatever be the value of n.
Solution: Given equations of the curves are
n
n
x
y
+
=2
(1)
a
b
x
y
+ =2
..(2)
a b
Now, differentiating the equation (1) w.r.t x, we get

F I F I
H K H K

F I F I
H K H K

n 1

+n

b
n

n 1
n

n 1

dy
b
x
= n n 1
dx
a
y

m1 =

LM dy OP
N dx Q

x =a
y =b

dy
=0
dx

Tangent and Normal to a Curve

n 1

b
(3)
a
a
b
Again, differentiating the equation (2), w.r.t x, we
get
1 1 dy
+
=0
a b dx
dy
b

= (constant)
(4)
dx
a
Thus, we see from (3) and (4), m1 = m2
The curve touches the line at (a, b) for any n.
=

n 1

Question: Show that the curve y2 = 2x touches the


straight line 2y x = 2. Also find the point of
intersection.
Solution: Given equation of the curves are
y2 = 2x
(1)
2y x = 2
(2)
Now, differentiating eqn (1) w.r.t x, we get
2y

2
1
dy
dy
=2
=
=
dx
dx 2 y
y

(3)

Again, differentiating eqn (2) w.r.t x, we get

dy
dy 1
1= 0
=
(4)
dx
dx 2
Now, we are required to find the point of
intersection where, they intersect by solving the
equations of the given curves.
2

2y = x + 2 [from eqn (2)] y =


Putting (5) in (1), we get

LM x + 2 OP
N2 Q

a f

= 2 x x + 2 = 8x

x2 4x + 4 = 0

x2

=0

x=2
x+2 2+2
=
=2
2
2
Thus, we get P (x, y) = (2, 2)
Now, for the curve (1),
y =

x+2
2

(5)

m1 =

LM dy OP
N dx Q

x=2
y=2

LM 1 OP
NyQ

x =2
y =2

And for the curve (2), m2 =

1
2

LM dy OP
N dx Q

721

(6)

x=2
y=2

1
2

(7)

1
2
The given curves touch each other at (2, 2).

From (6) and (7), we see that m1 = m2 =

Question: Show that the curve y = be a touches


x
y
the straight line + = 1 at the point (0, b).
a b
Solution: Differentiating y = be

FG IJ
H K

w.r.t x, we get

x
dy
1
b x
= be a
= e a
(1)
dx
a
a
x
y
Again differentiating + = 1 w.r.t x, we get
a b
1 1 dy
dy
1
b
+
=0
= b=
(2)
a b dx
dx
a
a

Now, from (1), m1 =

LM dy OP
N dx Q

x=0
y =b

b
b
b 0a
e = 1=
a
a
a

And from eqn (2), m2 =

LM b OP
N aQ

x=0
y =b

LM dy OP
N dx Q

b
a

(3)

x =0
y =b

(4)

Eqns (3) and (4) m1 = m2


Both the curves touch each other at (0, b).
Question: Show that the curve

2x 2x2
y2
+ 2 + 2 =1
a
b
b
x
touches the curve y = be a at the point where the
curve crosses the axis of y. Find the equation of the
common tangent.

722

How to Learn Calculus of One Variable

Solution: The point where the curve cuts (or crosses)


the y-axis x = 0
Now, we are required to find out y
x

y = b e a = b e a = b e = b1= b
Hence, required where the curve crosses the axis
of y = (0, b)
Again given equations of the curves are
2x 2x2
y2
+ 2 + 2 =1
a
b
b

(1)

(2)
y = be a
Differentiating the equation (2) w.r.t x, we get

LM dy OP
N dx Q

x=0
y =b

m1

LF 1
= Mb G e
NH a

IJ OP
KQ

x=0

bg

af
af

dy
f t
dy dy dt
=

= dt = 2
dx
f 1 t
dx dt dx
dt
2. The equation of the tangent at the given point t
is

af

y f2 t

(3)

where the slope of the normal = negative reciprocal of


the slope of the tangent

dy
dx

2b 2 + 4ax
dy
=
dx
2ay
clearly (0, b) is a point on (1) and from (1),

LM dy OP
N dx Q

bg
bg

f t
1
= 2
Slope of the tangent
f 1 t

Examples worked out:


Question: Find the equation of the tangent and
normal to the ellipse x = a cos , y = b sin at the
given point .
Solution: Here

dx
dy
= a sin and
= b cos
d
d

dy
b cos
dy

= d =
dx
a sin
dx
d

x =0
y =b

b
= m2
a
m1 = m2
Hence the curves touch at (0, b). The equation of
the common tangent is
b
yb = x 0
a
x
y
+ = 1.
a b
=

b g

bg

x f1 t

=0

af

b g = ff bbttgg

2 4 x 2 y dy
+
+ 2
a b2
b dx

=0

af
af

f 2 t
x f1 t
f 1 t

y f2 t

Differentiating the equation (1) w.r.t x, we get

2b 2 + 4ax + 2ay

The equation of the normal at the given point t is


0

bg

Working Rule:
dy
1. Find
using the formula.
dx

y =b

L F 1 I e OP = b LM 1 e OP = b
= Mb
N H aK Q N a Q a
0

To find the equation of the tangent and normal to the


curve whose equation is given in parametric form
x = f1 t , y = f 2 t at a point t.

(1)

Now, the equation of the tangent at is


b cos
y b sin =
x a cos
a sin

ay sin ab sin 2 = bx cos + ab cos2

bx cos + ay sin = ab sin 2 + cos2 = ab


x
y
cos + sin = 1
a
b

Tangent and Normal to a Curve

Again, the slope of the normal at the given point

is

and the equation of the normal is

LM
N

a sin
.
b cos

(y a) = 1 x a

The equation of the normal is

=
2

by cos b sin cos = ax sin a sin cos

ax sin by cos = a b

sin cos

ax
by
2
2

=a b
cos sin

Question: Find the equation of the tangent and


normal at =

to the cycloid x = a sin ,


2

y = a 1 cos .

a
fUV
y = a a1 cos fW

Solution: Given equations x = a sin

dy
= a sin = 2a sin cos
2
2
d

(3)

dy

dy
d = cot
=
2
dx dx
d

LM OP
N Q

= 2

F
H

= cot = 1
4

I and y = 1.
K
L F 1I , aOP
Equation of the tangent at the point Ma
N H2 K Q
L F 1I OP x y = a 2a
is a y a f = 1 M x a
2
N H 2 KQ
Also for =

1
,x =a
2
2

1
= 1]
Slope of the tangent

Question: Find the equation of the tangent and


normal to the parallel x = at2, y = 2at at the point t.

b gU| dy dy dt 2a 1
V = = =
b g |W dx dx dt 2at t

dx
= 2at ... 1
Solution: dt
dy
= 2a ... 2
dt

Equation of the tangent at any point t is

y 2at =

1
2
x at
t

j
2

yt 2at = x at ty = x + at
Slope of the normal at the given point t
=

(2)

dy
dy
at =

dx
dx
2

1
a [Slope of the normal
2

(1)

dx
2

= a 1 cos = 2a sin
2
d

Eqns (2) and (3)

F 1I OP
H 2 KQ

which implies x + y =

sin
a x a cos f
a y b sin f = ba cos

723

1
Slope of the tangent

Hence, the required equation of the normal is

y 2 at = t x at

y 2at = tx + at 3
y + tx = 2 at + at

Question: Find the equation of the tangent to the


curve x =

t,y=t

1
t

at the point t = 4.

Solution:

b gU|| dy dy

= dt which implies
V
dx
dx
b g||
dt
bg
W

1
dx
... 1
=
dt 2 t
1
dy
... 2
= 1+
3
dx
2 t 2

724

How to Learn Calculus of One Variable

1+
dy
=
dx

LM2 bt g + 1OP 2
2 bt g
N
Q
=
1
2 bt g
1

3
2

3
2

3
2

2 t

Slope of the tangent at t = 4 is

bg

2 t

3
2

+1

bg

2 4

3
2

+1

4
The equation of the tangent at t = 4 is

f LMN dydx OPQ a X xf with the given

3. Compare Y y =

17
4

LM y 7 OP = 17 a x 2f
N 2Q 4

line ax + by + c = 0 and then eliminate x and y.


On intercepts:
The equation of the tangent of the curve y = f (x) at
the point P = (x1, y1) is

a y y f = LMN dydx OPQ a x x f


1

The intercept OA made by the tangent on the axis


of x is obtained by putting y = 0 in (1) and solving
for x.

17 x 4 y 20 = 0

Problems based on proving the equation of the tangent


to a curve at any point (x1, y1) to be a linear equation
y = ax + b or finding the condition for a given line to
touch a given curve.

B
L
y = f ( x)

Working rule to show the equation of the tangent


line to a curve at any point (x1, y1) to be a linear
equation y = ax + b.

dy
1. First of all find
. This gives the slope of the
dx
curve or the slope of the tangent at the general point
(x, y).

L dy O
2. Find M P
N dx Q

(1)

P(
x1 ,

y1 )

y1 =

LM dy OP a x x f
N dx Q
1

a x x f = L dyy O
MN dx PQ
1

x = x1
y = y1

3. Afterwards apply the slope form equation

a y y f = LMN dydx OPQ a x x f


1

x = x1
y = y1

OA = x = x1

which on simplification gives the required equation


of the tangent or the straight line which touches the
curve.
Question: How would you find the condition for a
given line to touch a given curve?
Solution: 1. Let the line be tangent to the given curve
at (x, y).
2. Write the equation of the tangent at (x, y) as

aY yf = LMN dydx OPQ a X xf

equation of the curve.

where

dy
= d.c of the
dx

y1
dy
dx

LM OP
N Q

Similarly, to obtain the intercept on y-axis, we put


x = 0 in eqn (1) to have y y1 =

y = y1 x1

LM dy OP
N dx Q

LM dy OP b x g
N dx Q
1

= OB
p

The portion of the tangent intercepted between


the axis
= AB =

OA + OB

bIntercept on x-axisg + bIntercept on y-axisg


2

Tangent and Normal to a Curve

Substituting the values of OA and OB, we can find


the length AB.

= PN

1 + cot

Facts to know:
1. Length of the perpendicular from the origin upon
the tangent to the curve y = f (x) at any point (x1, y1) is

= PN

1+

OL = y - intercept of the tangent cos

B
L
P(

x1 ,

y1 )

LM
MN

y1 x1
=

= y1 x1

F dy I
H dx K
F I
H K
dy
dx

L dy
1 + MF I
MNH dx K

= y

y = f (x)

x = x1
y = y1

OP
PQ

1+

LMF dy I
MNH dx K

x = x1
y = y1

OP
PQ

x = x1
y = y1

x = x1
y = y1

OP
PQ

+1

x = x1
y = y1

LMF dy I OP
NH dx K Q
F dy I
H dx K

+1

LMF dy I OP
MNH dx K PQ
LM3 y = PN and F dy I
H dx K
MN
2

(
=

LMF dy I
MNH dx K
F dy I
H dx K

= y 1+

P (x1, y1)

tan

= PN sec = y 1 + tan

2. Length of tangent:
Y

3. Length of the normal: The portion of the normal


at any point on the curve intercepted between the
curve and the axis of x is called the length of normal.
In NGP
Length of the normal = PG

x = x1
y = y1

OP
PQ

= y

725

The portion of the tangent intercepted between


the point of contact and the axis of x is called the
length of the tangent.
In PTN ,
Length of tangent
= PT = PN cosec

= tan 1 = m1
p

OP
PQ

4. Angle of intersection between two curves: The


angle of intersection of two curves is the angle
between the tangents drawn to the two curves at the
common point of intersection of two curves.
Examples worked out:
Question: Show that the equation of the tangent
to the curve y = 2x3 + 2x2 8x + 7 at the point (1, 3) is
2x y + 1 = 0.
Solution: Equation of the curve is
y = 2x3 + 2x2 8x + 7
(1)

726

How to Learn Calculus of One Variable

Now differentiating (1) w.r.t x, we get

Solution: Differentiating the given equation of the

dy
2
2
= 2 3x + 2 2 x 8 1 + 0 = 6 x + 4 x 8
dx

curve

LM dy OP
N dx Q

x =1
y=3

= 6x + 4 x 8

f LMN OPQ

y =3

= 6 1 + 4 1 8 = 2

2x
a

dy
2
= 3x + 2ax
dx
2
dy 3x + 2ax

=
dx
6ay
6ay

x = 2a
y = 2a

LM 3x + 2ax OP
MN 6ay PQ

= slope of the tangent at (2a, 2a)

=
x = 2a
y = 2a

3 4a + 2a 2a 16a
4
=
=
2
6a 2a
3
12a

x = 2a
y = 2a

x x1

point a cos , b sin on the curve


is

x
y
cos + sin = 1 .
a
b

2 y dy
dy
b x
=0
= 2
2
dx
b dx
a y

LM dy OP
N dx Q

x = a cos
y = b sin

b
a

b cos
a sin

2
2

(1)

a cos
b sin

(2)

is y b sin =

b cos
x a cos
a sin

x cos
y
sin sin 2 =
+ cos2
b
a

x
b
2
2
cos + sin = cos + sin = 1
a
y

x
y
cos + sin = 1
a
b
which is the required equation of the tangent to the
curve

2
2

y
b

2
2

= 1 at a cos , b sin .
2

= 1 w.r.t x, we get

Question: If the normal to the curve x 3 + y 3 = a 3

4
y 2a =
x 2a
3
4x = 3y + 2a
4x 2a = 3y.
Question: Show that the equation of the tangent at a

The equation of the tangent at a cos , b sin

Now, the equation of the tangent at (2a, 2a) is

a y y f = LMN dydx OPQ

y
b

Question: Show that the equation of the tangent to


the curve 3ay2 = x2 (x + a) at (2a, 2a) is 3y = 4x 2a.
Solution: We have 3ay2 = x2 (x + a)
(1)
Now, differentiating (1) w.r.t x, we get

LM dy OP
N dx Q

Now,

y 3 = 2 (x 1)
y 3 = 2x 2
2x y + 1 = 0.

x =1

Hence, the equation of the tangent to the curve at


(1, 3) is
dy
x x1
y y1 =
dx x =1

x
a

2
2

y
b

2
2

=1

makes an angle with the axis of x, show that its


equation is y cos x sin = a cos 2 .
Solution: Given equation of the curve is
2

(1)
x3 + y3 = a3
Now, differentiating the equation (1) w.r.t x, we
have
1

2 13 2 13 dy
dy
y3
=0
= 1
x + y
3
3
dx
dx
x3

727

Tangent and Normal to a Curve

b g
Lx O
1
=
=M P
Slope of the tangent N y Q

y cos x sin = a cos 2

Now, slope of the normal at x1 , y1


1
3

(2)

Again, as the normal makes an angle with xaxis,


(3)
tan = Slope of the normal
1

Equating (2) and (3) tan =

x
y
+
=1
a
b
at any point on it makes the intercepts p and q along
Question: If a tangent to the curve

p q
+ = 1.
a
b
Solution: The point (x1, y1) is on the curve
the axes, then show that

x
y
+
=1
a
b
(x1, y1) must satisfy the equation (1)

x13
1
3

y1

(1)

x3
sin

= 11
cos
y13

1
3

ex j + ey j
1
3

1
3

x1
y
= 1 =
sin cos

= a

2
3

=a

1
3

21
3 2

=a

1
3

1
1
2

2a x

1
2

2b y
1

x1 = a sin

x1 = a sin 3

dy
=0
dx

1
2

The value of

f LMN OPQ

dy
dy
at x1 , y1 =
dx
dx

1
3

3
and y1 = a1 cos y = a cos
Now, the equation of the normal at

y a cos =

y1 )

y = f (x)

sin
3
x a sin
cos
4

O (0, 0)

j
4

y cos x sin = a cos sin


2

je

Now, the tangent at (x1, y1) is

y cos a cos = x sin a sin

B
P(
x1 ,

1
3

x = x1
y = y1

ea sin , a cos j is a y y f = FGH xy IJK a x x f


y a cos = a tan f e x a sin j
3

1
2

(2)

b2 y2
dy

= 1
1
dx
a2 x2

1
3

1
3

1
3

y12

+ 1 =1
1
a2
b2
Now, differentiating (1) w.r.t x, we have

sin + cos
2

x12

= a cos sin cos + sin

b y y g = LMN dydx OPQ


1

y y1 =

x = x1 .

bx x g
1

y = y1
1

b 2 y12
1

a 2 x12

x x1

b 2 y12
a 2 x12

728

How to Learn Calculus of One Variable


1

y
1

b 2 y12

1
2

a x1
x
1
2

1
2

a x1

1
2

b y1
+

x12

y12

y
1
2

a 2 x12

y
1
2

b 2 y12

x
1
2

y12

1
2

a 2 x12

The value of

1
2

1
2

axis are a x1 and b y1 .


1

2
2
2
According to question, a x1 = p x1 =

p
a

2
2
2
and b y1 = q y1 =

1
2

LM dy OP
N dx Q

FG
H

x2

sin

y12
1

=1

p
1
2

q
1
2

1
2

we get

=1

a a
b b
p q
+ =1
a b
Question: Prove that the points on the curve

RS
T

y = 4a x + a sin

F x I UV at which the tangents are


H aKW

parallel to the axis of x lie on the parabola y2 = 4ax.


Solution: The point (x1, y1) is on the curve

RS
T

y = 4a x + a sin

F x I UV
H aKW

(1)

(x1, y1) must satisfy the equation (1)


2

RS
T

y1 = 4a x1 + a sin

FG x IJ UV
H a KW
1

(2)

Now, differentiating (1) w.r.t x, we get

2y

IJ
K

x1
2 x
= 1 1 sin 1 = 0
a
a

x1
x
= 0 sin 1 = 0
a
a

Now, substituting the value of sin

b2
1
2

IJ
K

x
x
2a
1 + cos 1 = 0 cos 1 = 1
y1
a
a

cos

But from (2),


1

=0

x = x1
y = y1

b2

x12

If the tangent at (x1, y1) is parallel to x-axis, then

b y12

1
2

UV
W

dy
at x1 , y1
dx

x
2a
1 + cos 1
y1
a

The intercepts of the above tangent along the


1
2

FG
H

1
2

= 1 [from (2)]

RS
T

dy 2a
x
=
1 + cos
dx
y
a

x1

RS
T

F
H

dy
x
= 4a 1 + a cos
dx
a

I 1 UV
K aW

x1
= 0 in (2),
a

y1 = 4 a x1 + a 0
2

y1 = 4 ax1

x1 , y1 lies on y2 = 4ax.

Question: Tangents are drawn from the origin to the


curve y = sin x show that their points of contact lie on
x2 y2 = x2 y2.
Solution: There is a point (x1, y1) on the curve
y = sin x
(1)
(x1, y1) satisfies the equation y = sin x
(2)
y1 = sin x1
Now, differentiating the equation (1) w.r.t x, we get
dy
= cos x
dx
dy
Again the value of
at
dx
dy
= cos x x = x = cos x1
x1 , y1 =
1
dx x = x1

f LMN OPQ

y = y1

729

Tangent and Normal to a Curve

Equation of the tangent at any point (x1, y1) on


the curve is given by

b y y g = LMN dydx OPQ


1

g b

x = x1

. x x1

y = y1

(x1, y1) satisfies the equation of the curve since


it lies on the curve.
1

x12 + y12 = a 2

(3) [from (1)]

y y1 = cos x1

g bx x g

y = f (x)

If this line passes through (0, 0), then

b0 y g = bcos x g b0 x g
y = bcos x g b x g
1

a y y f = ax x f
y12

y1 = x1 1 sin x1

y12 = x12

2
1

= sin x1

LM3 x
N

x1 , y1 lies on the curve

x2

y2

x2

y2.

tangent to the parabola x 2 + y 2 = a 2 on the coordinate axis is constant and equal to a.


Solution: Given equation of the curve is
1

x2 + y2 = a2
Now, differentiating (1) w.r.t x, we get

1
2

Question: Show that the sum of the intercepts of the

1
x
2

LM dy OP
N dx Q

1
y
2

12

FH

IK

12

(1)
1
2

dy
dy
x
y
=0
= 1 = 1

dx
dx
y 2
x2

+ y12 = a 2

OP
Q

(4)
x y1 + y x1 = x1 y1 a
Now, to find the intercepts on the axis, we put x = 0
and y = 0 respectively in the equation of the tangent
(4)
For intercepts on x-axis, we put y = 0.

LM
N

y = 0 x y12 = x12 y12 a 2


1

21

x y1 + y x1 = x12 y12 x12 + y12 = x12 y12 a 2

x1 y1 = x1 y1

y1 x1 = x1 y1

x y12 + y x12 = x1 y12 + x12 y1 = x12 x12 y 2 + x12 y12 y 2

y x12 y1 x12 = y12 x + x1 y12

y1 = x1 x1 y1
2

x12

Now, the equation of the tangent at P (x1, y1) is

e
j
e1 y j b3 y

y1 )

y1 = x1 cos x1
2

x1 ,

y1 = x1 cos x1
2

P(

OP
Q

x = x12 a 2
(5)
x = x1 a
Again for the intercept on y-axis, we put x = 0.

P x1 , y1

y12
1

(2)

x12

Again since (x1, y1) is a point where the tangent


touches the curve.

x = 0 x12 y = y12 x12 a 2


1

y = y12 a 2

y=

y1 a

...(6)

730

How to Learn Calculus of One Variable

LM x OP + LM y OP
Na Q N b Q

Now, adding (5) and (6) the sum of the


intercepts on the axes

F
I = a a = a
H
K
xI F yI
F
Question: Show that
H a K + H b K = 1 touches the
1

1
2

x12 a 2 + y12 a 2 = a 2 x12 + y12

curve y = b e

ax

at the point where the curve crosses

ax

(1)

And the curve y = b e

ax

ba cos g

1
2

y-axis.
Solution: Given equation of the curve is

y = be

meets y-axis at a point

where x = 0 (Since x = 0 is the equation of y-axis).


Now, to find the point of intersection, we put x = 0
in the equation of the curve (1), we get

m
m 1

y = be

F I
H K

LM dy OP
N dx Q

x =0

dy
at (0, b)
dx

b 0a
b
b
e = e0 =
a
a
a

y =b

Hence, the tangent at (0, b)

y y1 =

a f

LM dy OP
N dx Q

y b =

x=0

x x1

y =b

a f

b
y
x
x a 1 =
a
b
a

x
y
+ =1
a b
Question: Show that the condition that the line
x cos + y sin = p touches the curves

m
m1

= p m1 .

my

=1
(1)
m
m
a
b
Now, differentiating both sides of (1) w.r.t x, we get
mx
a

m 1
m

m 1

dy
=0
dx

m m1

dy
b x
= m m1
dx
a y

Equation of the tangent at (x, y) is


m

m1
m1

a X xf

X x m 1 Y y m 1
xm
ym
+
=
+
=1
(2)
am
bm
am
bm
Now, since the equation of a straight line can be
written as
X cos + Y sin = p
(3)
Thus, the equation (2) and (3) represents the same
straight line

+ m = 1.
m
a
b
Solution: Given equation of the curve is

a y

x
dy
1
b x
= be a
= e a
dx
x
a

Now, the value of

+ b sin

aY yf = b

ax

=1 is

Or, find the condition that the line


x cos + y sin = p should touch the curve

(2)
y = be a = b
Thus, (0, b) = The point of intersection of the curve
with y-axis.
Now, differentiating the given equation of the curve

m1

a cos

La
x=M
MN

m 1

b sin

cos
p

OP
PQ

1
m1

1
p

Lb
,y=M
MN

sin
p

OP
PQ

1
m1

Since the point (x, y) lies on the curve

LM
MN

1 a m cos
m
p
a

OP
PQ

m
m1

LM
MN

1 b m sin
+ m
p
b

OP
PQ

m
m1

=1

Tangent and Normal to a Curve

a cos

m
m1

+ b sin

m
m1

= p

m
m1

Hence proved.

LM x OP + LM y OP = 1 ,
Na Q Nb Q
aa cos f + ab sin f = p .

curve

Solution: Given equation of the curve is

LM x OP
Na Q

n
n 1

LM y OP
Nb Q

n
n 1

(1)

Now, differentiating the equation of the curve, we


get

FG n IJ FG x IJ 1 + n FG y IJ
H n 1K H a K a n 1 H b K
F xI F bI
dy

= G J G J
dx
H yK H aK
1
n 1

1
n 1

1 dy

=0
b dx

aY y f = FGH xy IJK FH ba IK a X xf
Yy

LM
N

= x

1
n 1

n
n 1

n
n 1

n
n 1

n
n1

Xx

n
n 1

OP
Q

(2)
n

Now, dividing both sides of (2) by a n 1 b n 1 , we


get
x

1
n 1

n
n 1

1
n 1

n
n 1

F xI
H aK

1
n 1

1
n 1

b sin

= p

1
n 1

n
n 1

n
n 1

= p

[Raising both sides to the nth power]

aa cos f + ab sin f = p
F xI + F yI
H aK H bK
aa cos f + ab sin f = p
LMQ F x I
MN H a K
n
n 1

n
n 1

n
n
n 1

F yI
H bK

which is the required condition.

n
n1

n
n 1

a cos

Hence, the equation of the tangent at any point


(x, y) is

1
n 1

n
n 1

F xI F yI
H aK H bK
aa cos f = ab sin f

F xI
F yI
H aK
H bK

n
n 1

1
n 1

from (3)
n
n 1
a
Coefficient of X = cos from (4)

=1

1
n 1

p
= p
1

Coefficient of X =

then prove that

n
n 1

1
n 1

n
n 1

sin

1
n 1

Question: If X cos + Y sin = p touches the


n
n 1

cos

n
n 1

F yI
H bK

n
n 1

=1

(3)

a
b
Also, we are given cos X + sin Y = p is
the equation of the tangent.
(4)
Thus, eqns (3) and (4) represent the same straight
line. Comparing eqns (3) and (4), we have the
coefficients of x and y in (3) and (4).

731

n
n 1

OP
PQ

=1

Note: We should note while solving the problem


involving x and y that touching point has been
supposed to be (x, y) instead of (x1, y1) here. This is
why the necessity of substituting the coordinates of
dy
the point in
does not arise.
dx
Question: Show that the normal to the curve

a
f
y = a asin cos f at any point

x = a cos + sin

constant distance from the origin.

dy
dy
d
=
Solution:
dx dx
d

is at a

732

How to Learn Calculus of One Variable

mb

dy a cos sin + 1 cos


=
= tan
dx
a sin + 1 sin + cos

The equation of the normal at any point on


the curve is given by
cos
y a sin cos =
x a cos + sin
sin

y sin a sin cos sin


= x cos + a cos cos + sin

(1)

y
B

x1 ,

0 cos + 0 sin a
2

cos + sin

= a = constant.

Note: Length of the perpendicular from the origin


upon the tangent to the curve at (x1, y1).
y

y = f ( x)

y1 )

Since the equation of the tangent at (x1, y1) is

b y y g = LMN dydx OPQ


1

x = x1
y = y1

y = y1

x = x1
y = y1

OP
PP
Q

x x1

Question: Find the equation of the tangent


and normal at the point t on the curve x = a cos3 t,
y = a sin 3 t. Show that portion of the tangent
intercepted between the axes is of constant length a.
Solution:

dx
2
= 3a cos t sin t
dt

(1)

dy
2
= 3a sin t cos t
dx

(2)

dy
2
sin t
3a sin t cos t
dy
dt
=
=
=
Now,
2
dx
dx
cos t
3a cos t sin t
dt
Slope of the tangent at any point t = tan t

B
x1 ,

x = x1

a 2 + b2

Now, the length of the perpendicular from the origin


to the normal (1)

P(

x = x1
y y1

I
JJ = 0
K

ah + bk + c

y1 )

LM dy OP
N dx Q

LM dy
F I
1 + MG J
MNH dx K

LM dy OP
N dx Q

Q The length of perpendicular from


(h, k) to any line ax + by + c = 0 is

y = f (x)

P(

x = x1
y = y1

F
GG
H

y + y1 x1

The length of the perpendicular upon this line


from the origin whose coordinates are (0, 0) is

y1 x1

x cos + y sin = a sin + cos = a

x cos + y sin a = 0

LM dy OP
N dx Q

Slope of the normal at any point t = cot


Equation of the tangent at t is
3

y a sin t =

sin t
3
x a cos t
cos t

y cos t a sin t cos t = x sin t + a cos t sin t


3

x sin t + y cos t a cos t sin t a sin t cos t = 0

Tangent and Normal to a Curve

x sin t + y cos t a cos t sin t cos t + sin t = 0

x sin t + y cos t = a sin t cos t


(1)
Again, since the slope of the normal at the given
point 't' =

cos t
sin t

The equation of the normal at t is


cos t
3
3
y a sin t =
x a cos t
sin t

= a cos t + sin t

je

cos t sin t

a 2 cos2 t + a 2 sin 2 t

= a = a = Constant which implies the


required result.

Question: Find the equation of the tangent to the


2

curve x 3 + y 3 = a 3 and show that the portion of


the tangent intercepted between the coordinate axes
is constant and is equal to a.
Solution: Equation of the given curve is
2
3

x + y =a

2
3

a y y f = ax x f

(1)

x13

y x13 y1 x13 = y13 x + x1 y13


1

y x13 + y13 x = x1 y13 + y1 x13


1

= x13 y13

Fx
H

2
3

+ y13

I
K

F I
H K
2

y x13 + x y13 = x13 y13 a 3


2

(2)

[ Q x13 + y13 = a 3 from the given equation since


(x1, y1) lies on the curve It must satisfy given
equation of the curve]
To find the intercept on the x-axis, we put y = 0,
1

to have 0 + x y13 = x13 y13 a 3


1

Note: This question may be asked as follows.

2
3

bIntercept on x-axisg + bIntercept on y-axisg


2

y13

= The slope of the tangent at

y13

x cos t y sin t = a cos2t


Now, since the tangent is x sin t + y cos t = a sin t
cos t (From eqn (1))
(2)
To get the intercept on x-axis, we put y = 0 in the
equation (2) to have
x sin t = a sin t cos t x = a cos t
To get the intercept on y-axis, we put x = 0 in the
equation (2), then y = a sin t
The portion of the tangent intercepted between
the axes.

x = x1
y = y1

x13
Equation of the tangent at (x1, y1) on the given
curve is

y sin t a sin t = x cos t a cos t

LM dy OP
N dx Q

P (x1, y1) =

x cos t y sin t = a cos t sin t

2 13 2 13 dy
dy
y3
=0
= 1
x + y
3
3
dx
dx
x3
Now, let the tangent touch the given curve at a
point P (x1, y1).

y a sin t sin t = x a cos t cos t


4

Now, differentiating both sides of the equation (1)


w.r.t x, we get

733

x = x13 a 3
and to find the intercept on y-axis, we put x = 0 in (2)
to have
2

y = a 3 y13 [On putting x = 0 in (2)]


Hence, the length of intercept between the axes
=

x + y

734

How to Learn Calculus of One Variable

bIntercept on x-axisg + bIntercept on y-axisg


a Fx + y I = a = a
H
K
2

4
3

2
3

2
3

Question: Find the coordinates of the point on the


curve xy = 16, the normal at which intersects at the
origin of the coordinates.
Prove also that the portion of any tangent to the
curve intercepted between the coordinate axes is
bisected at the point of contact.
Solution: If (x 1, y 1) be any point on the curve
xy = 16
(1)
Then (x1, y1) must satisfy the equation (1).
x1 y1 = 16
(2)
Now differentiating both sides of the given
equation (1) w.r.t x, we get
dy
dy
y
x
+ y 1= 0
= the value of
dx
dx
x
dy
y
at x1 , y1 = 1
dx
x1

From eqns (2) and (3) y1 =

LMQ x
N

1 y1

16
y1

= 16 x1 =

OP
Q

y1 = 16 16 y1 = 4

Again from eqn (2), x1 y1 = 16

x1 =

16 16 16
=
=
= 4
y1
y1 4

Hence, the required points are (4, 4) and (4, 4).


Now the equation of the tangent at any point
(x1, y1) is (y y1) =

y1
x x1
x1

(4)

Since the tangent meets the axis of x at the point


where y = 0, so to find x-intercept, we put y = 0 in the
equation (4) to have

a0 y f = xy a x x f
1

y1 x1 = y1 x + y1 x1
2 y1 x1 + y1 x = 0

y = f (x)
P(
x1 ,

a
a
f
bQ y 0g x = 2 x

y1
The slope of the tangent at x1 , y1 =
x1
x1
The slope of the normal at x1 , y1 =
y1

Now, the equation of the normal at (x 1, y 1) is

x
= 1 x x1
y1

y1 = x1

x
= 1 0 x1
y1

OA = 2 x1

Q x = OA
Thus, the tangent at (x1, y1) meets the x-axis at the
point (2x1, 0)
Again, the tangent at (x1, y1) meets y-axis at the
point where x = 0 To find y-intercept we put x = 0

in eqn (4) to have y y1 =

y1
0 x1
x1

y y1 = y1 y = 2 y1 = OB Q y = OB

The normal at (x1, y1) passes through (0, 0)

0 y1

y1 2 x1 + x = 0 2 x1 + x = 0
y1 )

y1

ay y f

16 16

The tangent meets y-axis at the point (0, 2y1)


Thus, we see that the tangent at (x1, y1) meets xaxis at (2x1, 0) and the tangent at (x1, y1) meets y-axis

f
...(3)

at (0, 2y1).

735

Tangent and Normal to a Curve

The mid-point of the portion of the tangent to


the curve xy = 16 intercepted between the coordinate

FG
H

2 x1 + 0 0 + 2 y1
axes is
,
2
2

IJ
K

Question: A normal is drawn to the curve y = x2 at


the point (1, 1) on it. Find the length of the part of the
normal intercepted between the coordinate axes. Also
find subtangent and subnormal.
Solution: We suppose that the tangent and the
normal at the point (1, 1) on the curve y = x2 (i) meet
the x-axis in T and G.
Now, differentiating the given equation y = x2
dy
= 2x
w.r.t x, we get
dx
dy
The slope of the tangent at (1, 1) =
dx x = 1

LM OP
N Q

x =1 =

3
unit.
2
Length of the normal intercepted between the
coordinate axes is
normal on the axes are 3 and

Which is the point of contact.

= 2x

Thus, the lengths of the intercepts made by the

b x - interceptg + b y - interceptg
F 3I = 9 + 9
= 3 +
H 2K
4
2

36 + 9
4

45 3 5
=
4
2

. =2
TN = Subtangent = PN tan = 12
NG = Subnormal = PN cot = 1. =

1
.
2

F
H

y =1

Question: In the curve x = a cos t + log tan

2 1= 2

Similarly, the slope of the normal at (1, 1)

t
2

I
K

1
1
=
=
Slope of the tangent
2

y = a sin t
Show that the portion of the tangent between the
point of contact and the x-axis is of constant length.

The equation of the normal at (1, 1) is


1
y 1 =
x 1
2
2y + 2 = x 1

1
1
dx
2 t
= a sin t +
sec
Solution:
t 2
2
dt
tan
2

b g

b g

x + 2y = 3

(2)

R|
S|
T

2 t
sec
dx
1
2

= a sin t +
dt
2 tan t
2

P (x1, y1)

U|
V|
W

dy
= a cos t
dt
Now on simplifying the equation (1),

R|
S|
T

Now, the intercept x on x-axis is found by putting


y = 0 in eqn (2) i.e., x + 0 = 3 x = 3
Again the intercept y on y-axis is found by putting
x = 0 in eqn (2) i.e., 0 + 2 y = 3 y =

3
2

R|
S|
T

2 t
sec
1
dx
2
= a sin t +
2 tan t
dt
2

RS
T

= a sin t +

U|
V|
W

1
t
2 t
sec
cot
2
2
2

UV
W

U|
V|
W

(1)

(2)

736

How to Learn Calculus of One Variable

tU
R|
cos |
1
1
2
= a Ssin t +

t
t V
2
|T
cos
sin |
2
2W
U|
R|
1
= a Ssin t +
t
tV
|T
2 sin cos |
2
2W
R| sin t + 1U|
R
1 U
= a S sin t +
=aS
V
V
sin t W
T
T| sin t W|
2

a cos t
sin t
Now the slope of the tangent at (x, y) is
=

(3)

dy dy dx
=

dx dt
dt

m1

sin t
2

a cos t

PT
= cosec = cosec t
NP

(x
1,

y1 )

1
= a which is a constant.
sin t

y1
O

N
T
Y
(y = a sin t is given in the problem)

LM dy OP
N dx Q

x1
x = x1

LM
N

y = y1

tangent at x1 , y1

m y
n x

OP
Q

x = x1
y = y1

dy
at (x1, y1) = Slope of the
dx
m y1
=
n x1

The value of
X

y = f ( x)

y = f (x )

P (x 1, y 1)

PT = NP cosec t = y cosec t
PT = a sin t cosec t
[Q y = a sin t is given in the problem]

(2)

m 1
dy
m m 1 m n n + 1
y
= x
= x y
n
dx
n
dy
m y

= = Slope of the tangent at (x, y)


dx
n x
dy
Now the value of
at x1 , y1
dx

sin t
= tan t
cos t

= t
Now from the figure

PT = a sin t

dy m x
y

=
m
n 1
dx
nx y

a cos t
tan = a cos t
sin t

= a cos t

Question: (a) Show that the condition that the line


x cos + y sin = p touches the curve x m y m =
am + n is pm + n mm nn = (m + n)m + n am + n
m
n
n cos sin .
Or, find the condition that the line x cos +
y sin = p may be the tangent to the curve xm yn =
am + n.
(b) In the curve xm yn = am + n, prove that the portion
of the tangent intercepted between the axes is divided
at its points of contact into segments which are in a
constant ratio.
Solution: Given equation is xm yn = am + n
(1)
Now, differentiating both sides of eqn (1) w.r.t x,
we get
m
n 1 dy
n
m1
+ y m x
=0
x ny
dx

737

Tangent and Normal to a Curve

Equation of the tangent at (x1, y1) is

LM OP ax x f
N Q
m y
ay y f =
ax x f
n x
dy
(y y1) =
dx

ma

x = x1
y = y1

na
n

m x1m 1 y1n

y y1 =

y1n 1

n x1m

x x1

LMQ y
MN x

m1

yn

y1

n 1

x1 y1

x1m y1n 1

x1 cos =

x1

OP
PQ

y1 nx1m y1n 1

m+ n

Q x1 y1 = a

.(3)

y1 sin

m+ n

x1 cos

(7)

pn
m+n

and y1 cos =

(8)
m

m
x1

Eqn (7)

cos =

m x x1m 1 yn + n y x1m y1n 1

=
Q (x1, y1) lies on the curve
(x1, y1) satisfies the equation xm yn = am + n
x1m y1n = am + n
am + n

Equation of the tangent is m x x1m 1 yn + n y x1m

y1n 1 = (m + n) am + n

(4)

and the line x cos + y sin = p


(5)
Thus, eqns (4) and (5) represent the same straight
line
Coefficients of x, y and constant terms are in
proportion

x1 cos
m
m x1

y1

n
y1

n 1

nx1 y1
y1 sin
m
n x1

n
y1

x1 y1 cos sin =

= (m + n) am + n

n 1

p m

am + n f a

m+n

am + n f ea j
m+ n

am + n f
p n

(9)

am + nf

(10)

Now, multiplying these two results of eqns (9) and


(10) together, we get

= mx1m y1n + nx1m y1n = (m + n) x1m y1n

xm yn

(6)

y n x1m y1n 1 + mx1m 1 y1n x

mx1

m+ n

pm
m+ n

Eqn (8) y1 sin =

= mx1m 1 y1n x1 + ynx1m y1n 1

am + nf ea j

y nx1m y1n 1 + mx1m 1 y1n x

sin

m+n

= mx1m 1 y1n x + m x1m1 y1n x1

cos

m+n

am + n f

m+n

m+n

m+ n

m n

am + nf

m+n

cos sin

= p
m n which is the required condition.
Now, again letting that the tangent at (x1, y1) cuts
the axis in A and B.
To find the intercept x on x-axis of the tangent, we
put y = 0 in the equation of tangent eqn (4).
Equation of tangent is
m1

n 1

y1 + ny x1 y1

mx x1

m1

mx x1

m 1
m x1

ma

m+ n

m+n

m+ n

m+n

f (when y = 0)
am + nf a x
=

y1 = m + n a

am + nf a
x=
am + nf a
=

= m+n a
m+ n

m+ n

n
y1

x1

m
m x1

am + nf x

n
y1

= OA

738

How to Learn Calculus of One Variable

N.B.: 1. Imposed condition on tangent to the curve


may be: tangential line is parallel to x-axis or parallel
to a line / tangential line passes through origin /
Tangential line passes through the point , , etc.

(x
1,

y1 )

y = f (x)

2.

y1
A

Now, OT = x1 and OT + TA = OA
TA = OA OT

am + nf x x
m
m + nf x m x
a
TA =
TA =

found from the given condition on the

tangential line is generally constant or zero

3.

n x1
TA =
m
In OAB ,

n x1
AP
AT
n
which
PT OB
=
= m =
PB OT
x1
m
proves the required.
Problems based on finding the coordinates of a point
P (x1, y1) where the tangent touches the given curve
y = f (x) / f (x, y) = 0 or constant.
Working rule: 1. We suppose that there is a required
point (x1, y1) on the given curve where the tangent
touches the curve y = f (x).
dy
by differentiating the given equation
2. Find
dx
w.r.t. x.
dy
3. Find
dx x = x1

LM OP
N Q
L dy O
4. Find M P
N dx Q

x = x1
y = y1

LM dy OP
N dx Q

x = x1
y = y1

, an expression in x1 and y1 is equated

to zero or constant according to the given condition


imposed on the tangential line (i.e tangent to the curve
or slope of the curve).

LM dy OP
N dx Q

a f

LM dy OP
N dx Q

x = x1
y = y1

= 0 (Imposed condition) provided the

condition imposed on tangent tells that tangent is


parallel to x-axis.
4. We shall consider the problems on tangent at one
point of the curve/two or more tangents of the curve
making an angle with x-axis/y-axis/ parallel to x-axis/
y-axis.
Examples worked out:
Question: Find the points on the graph of the function
defined by y = x2 6x + 9 at which the tangents are
parallel to x-axis.
Solution: Let the required point on the curve y =
x2 6x + 9 be (x1, y1).
(x1, y1) must satisfy y = x2 6x + 9
y1 = x12 6x1 + 9

(1)
(2)

y = y1

x = x1
y = y1

from the given condition imposed

on tangential line (i.e., tangent to the given curve).


5. Solve the equation satisfied by (x1, y1) and the
equation in x1, and y1 obtained after imposing the

L dy O
given condition on M P
N dx Q

x = x1
y = y1

X
(3, 0)

Now, differentiating the equation (1) w.r.t x, we get

dy
= 2x 6
dx

739

Tangent and Normal to a Curve

Now, the value of

dy
at (x1, y1) = The slope of the
dx

tangent at (x1, y1)

LM dy OP
N dx Q

x = x1
y = y1

Now, since the tangent is parallel to x-axis


Slope of the tangent at (x1, y1) is zero.

= 2x 6

x = x1
y = y1

= 2 x1 6

LM dy OP
N dx Q

x = x1
y = y1

= 0 2 x1 6 = 0 x1 =

(2)

Now, on differentiating the equation (1) w.r.t x, we


get

2x + 2 y
2y

dy
dy
4
= 2 2x
dx
dx

a f
a f

dy
Now, the value of
at x1 , y1
dx

LM dy OP
N dx Q

x = x1
y = y1

LM 1 x OP
N y 2Q

x = x1

1 + y12 2 1 4 y1 + 1 = 0
2

y1 4 y1 = 0

y1 y1 4 = 0
y1 = 0 or 4

(4)

Equations (3) and (4) (x1, y1) = (1, 0) and (1, 4)


where the tangent is parallel to x-axis.
(b) Since the tangent is parallel to y-axis
x = x1

=0

y = y1

y1 2
= 0 y1 2 = 0 y1 = 2
1 x1

(5)

Putting y1 = 2 from eqn (5) in eqn (2), we get,


x12 + 4 2x1 4 2 + 1 = 0
x12 + 4 2x1 8 + 1 = 0

2 1 x
1 x
dy

=
=
dx 2 y 2
y2

On putting x1 = 1 from eqn (3) in eqn (2), we get

dy

2 y 4 = 2 2x
dx

(3)

LM dx OP
N dy Q

dy
dy
24
=0
dx
dx

1 x1
=0
y1 2

6
=3
2

Question: At what points on the curve x2 + y2 2x


4y + 1 = 0, the tangent is parallel to (a) x-axis (b) y-axis.
Solutions: (a) Equation of the given curve is:
x2 + y2 2x 4y + 1 = 0
(1)
Letting the required point to be (x1, y1) on the curve
(x1, y1) must satisfy the given equation of the
curve.
2

=0

Substituting this value of x1 in eqn (2), we get


y1 = 9 18 + 9 = 0
Hence, the required point = (x1, y1) = (3, 0).

x1 + y1 2 x1 4 y1 + 1 = 0

x = x1
y = y1

1 x1 = 0 x1 = 1

Now, since the tangent at (x1, y1) is parallel to


x-axis

LM dy OP
N dx Q

1 x1
y1 2

y = y1

= Slope of the tangent at (x1, y1)

x12 2x1 3 = 0
x12 3x1 + x1 3 = 0
x1 (x1 3) + (x1 3) = 0
(x1 3) (x1 + 1) = 0
Hence, required points where the tangent is parallel
are (x1, y1) = (1, 2) and (3, 2).

740

How to Learn Calculus of One Variable

Question: At what points on the following curve are


the tangents parallel to x-axis.

xU
R
= 4a S x + a sin V
aW
T

(a) y = sin x (b) y 2

and

x1 , y1 must satisfy y = sin x

LM dy OP
N dx Q

x = x1

dy
at x1 , y1
dx

= cos x1 = Slope of the tangent at

Since the tangent at (x1, y1) is parallel to x-axis.

LM dy OP
N dx Q

x = x1
y = y1

RS
T

= 0 cos x1 = 0

f 2

x1 = 2n + 1

(3)

x
a

UV
W

y1 = 4a x1 + a sin

x1
a

(1)

UV
W

(2)

Now, differentiating eqn (1) w.r.t x, we get

RS F I UV
T H K W
dy 2a F
xI

=
1 + cos
H
dx
y
aK
FG
H

x
2a
1 + cos 1
y1
a

F
H

dy
x
1
x

= 4a 1 + cos
= 4a 1 + a cos
dx
a a
a

The value of

y = y1

(x1, y1).

RS
T

y = 4a x + a sin

2y

Now, the value of

LMa2n + 1f , 1OP when n = 1, 3, 5


2
N
Q

(1)

y1 = sin x1
Now, differentiating the equation (1) w.r.t x, we get

LM dy OP = cos x
N dx Q

n = 0, 2, 4

(b) Let (x1, y1) be the required point on the curve.

Solution: (a) Let the required points on the curve


y = sin x be (x1, y1).

When

I
K

dy
at (x1, y1)
dx

IJ
K

= Slope of the tangent at

(x1, y1)
Now, since the tangent at (x1, y1) is parallel to xaxis.
2a
dy
x
1 + cos 1 = 0

=
x
x
dx = 1 y1
a

IJ
LM OP
FG
K
H
N Q
x
Fx I
cos G J = 1 = cos
= 2n
HaK
a
x

= a2n 1f where n is zero, positive or


a
y = y1

where n is zero, positive or negative integer.

negative integer.
X

Substituting this value of x1 in eqn (1), we get

y1 = sin 2n + 1
2
= 1 when n is even.
= 1 when is odd.

LMa
N

f 2 , 1OPQ

Hence, the required points (x1, y1) = 2n + 1

x1 = 2n + 1 a
Now, substituting this value of x1 in eqn (2), we get

la2n + 1f a + a sin a2n + 1f q


a2n + 1f + 0 = 4a a2n + 1f
= 2 a a2 n + 1f

y1 = 4a
= 4a

y1

Hence, the required points

f {a2n + 1f a , 2a a2n + 1f }

= x1 , y1 =

where n is zero, positive or negative integer.

Tangent and Normal to a Curve

Question: Find the coordinates of the point of contact


of tangent on the curve xy + 4 = 0 at which the tangent
makes an angle of 45 with x-axis. (Or, find the
equations of the tangents on the curve xy + 4 = 0
which are inclined at an angle of 45 with the axis
of x). Also find the coordinates of the points of
contact.
Solution: Let (x1, y1) be the required point on the
curve xy + 4 = 0
(1)
Now, since (x1, y1) lies on the curve xy + 4 = 0

x1 , y1 satisfies the equation (1)

x1 y1 + 4 = 0
Now, differentiating (1) w.r.t x, we have
x

(2)

dy
dy
y
+ y =0
=
dx
dx
x

(3)

Hence, the required points are (2, 2) and (2, 2).


The tangent at (2, 2) is y + 2 = tan 45
(x 2) y + 2 = 1 (x 2) x y = 4 and the
tangent at (2, 2) is y 2 = tan 45 (x + 2) y 2
= x + 2 x y + 4 = 0.
Question: At what points on the curve

2 3 1 2
x + x are the tangents equally inclined
3
2
to the axis?
Solution: Let (x1, y1) be the required points on the
y=

curve y =

2 3 1 2
x + x
3
2

(1)

y1 =

2 3 1 2
x1 + x1
3
2

(2)

B
45

P(

y=

x1 ,

f (x

y1 )

(x

y = f (x)
,y

45o

Now, the value of

x = x1
y = y1

dy
at x1 , y1
dx

L yO
= M P
N xQ

x = x1
y = y1

1
dy 2
2
2
= 3x + 2 x = 2 x + x
2
dx 3
dy
at x1 , y1
The value of
dx

(4)

Now substituting y1 = x1 from eqn (4) in eqn (2),


we get
2

13
5

Now, differentiating (1) w.r.t x, we get

y
= 1
x1

x1
= 1 y1 = x1
y1

Now, the tangents at (x1, y1) are inclined at an


angle of 45 with the axis of x.
y
1 = tan 45
x1

)
45

L dy O
=M P
N dx Q

741

x1 + 4 = 0 x1 = 2

(5)

Again from (4), y1 = m 2

(6)

(3)
= 2 x12 + x1
The tangents at (x1, y1) are equally inclined to xaxis and y-axis (i.e., axis)
The tangent at (x1, y1) makes an angle of 45 or
135 with x-axis.
Now, considering angle 45
tan 45 = 2x12 + x1 2x12 + x1 = 1
2x12 + x1 1 = 0 2x12 + 2x1 x1 1 = 0
2x1 (x1 + 1) (x1 + 1) = 0

(x1 + 1) (2x1 1) = 0 x1 = 1,

1
2

(4)

742

How to Learn Calculus of One Variable

On putting the values of x1 = 1 ,

= Slope of the tangent at (x1, y1)

1
from eqn (4)
2

(3)

in eqn (2), we get


y1 =

b g

b g

2
1
2 1 4+3
1
3
2
1 + 1 = + =
=
3
2
3 2
6
6

when x1 = 1 and y1 =

FI
HK

2 1
3 2

FI
HK

1 1
2 2

f FH 21 , 245 IK

Again considering the angle of 135, from (3), we


get 2x12 + x1 = tan 135 = 1

1 1 8
which
4
are imaginary, not to be included as the required point.
2

2 x1 + x1 + 1 = 0 x1 =

f FH 21 , 245 IK

Hence, the required points are x1 , y1 =

F
H

and 1 ,

I
K

1
.
6

a f

dy
= x 2 1 + x 3 1 = 2x 5
dx
The value of
=

LM dy OP
N dx Q

x = x1
y = y1

Now, since the tangent at (x1, y1) is parallel to the


line 2y = 10x + 5

LM dy OP
N dx Q

x = x1
y = y1

must be equal to the slope of the line

which is equal to the coefficient of x in y = mx + c


10
5
5
y=
x + y = 5x +
2
2
2

LM
N

LM dy OP
N dx Qa

OP
Q

x1 , y1

=5

2 x1 5 = 5

10
=5
(4)
2
Now, putting x1 = 5 from eqn (4) in eqn (2), we get
x1 =

y1 = (x1 2) (x1 3) = (5 2) (5 3) = 6

Question: At what points on the curve y = (x 2)


(x 3) is the tangent parallel to the line 2y = 10x + 5?
Solution: Let (x1, y1) be the required point on the
curve whose equation is
y = (x 2) (x 3)
(1)
(2)
y1 = (x1 2) (x1 3)
[ Q Since (x1, y1) satisfies the equation (1) because
of lying on the curve y = (x 2) (x 3)]
Now, differentiating eqn (1) w.r.t x, we get

a f

y = f (x)
,y

2 1
+
3 8

Thus, the required points are x1 , y1 =

F 1I
and 1 ,
H 6K

(x

1
1 1 2 1 2+3 5
= + =
=
when x1 =
2
4 2 24 8 24 24

x = x1

= 2 x1 5

dy
at x1 , y1
dx

= 2x 5

Required point (x1, y1) = (5, 6)


Question: Find the points on the curves y = x3 at
which the tangent makes an angle of 60 with x-axis.
Solution: Let (x 1, y 1) be the coordinates of the
required point on the curve y = x3
(1)
Then (x1, y1) satisfies the equation (1)
3

y1 = x1
Now, differentiating (1) w.r.t x, we get
dy
2
= 3x
dx
dy
at x1 , y1
The value of
dx
dy
=
dx x = x1

LM OP
N Q

= 3x12

(2)

y = y1

...(3)

743

Tangent and Normal to a Curve

Now, differentiating both sides of (1) w.r.t. x, we get


4x
dy
dy
(3)
8 x + 18 y
=0
=
9y
dx
dx
dy
Now, the value of
at x1 , y1
dx

(x

,y

y=

P2

f (x

(x
2,

y2 )

60o

Now since, the tangent at (x1, y1) makes an angle


of 60 with x-axis.

LM OP
N Q
dy
dx

x = x1
y = y1

x12 =

= tan 60

2
3x1

bg

3
1
=
x12 = 3
3
3

af

eqn (2), we get y1 = 3

21

LM 4 x OP
N 9y Q

x = x1
y = y1

34

F
H

and 3 4 , 3

34

I.
K

14

,3

43

I
K

(4)

LM dy OP
N dx Q

x = x1
y = y1

2
(Given in
9
(5)

4 x1
2
=
9 y1
9

4 x1
2
=
9 y1
9
y
=
2x
1
1
Now, substituting (6) in (2), we get

(6)

4x12 + 9 (2x1)2 = 40

Question: Determine the coordinates of the points


on the ellipse 4x2 + 9y2 = 40 at which the slope of the

2
.
9
Solution: Let the required coordinates of the points
be (x1, y1) on the curve
4x2 + 9y2 = 40
(1)
curve is

2
+ 9 y1

Eqns (4) and (5)

4 x1
9 y1

the problem)

Hence, the required points are (x1, y1) = 3

2
4 x1

Again, the slope of the tangent =

x = x1
y = y1

The slope of the tangent at x1 , y1

x1 = 3 4
Now, on substituting x1 = 31/4 from (4) in

F
H

LM dy OP
N dx Q

= 40

(2)

4x12 + 36x12 = 40
40x12 = 40
x1 = 1
y1 = 2 x1 = 2

f a

Required points are x1 , y1 = 1 , 2


Question: Find the point on the curve
which the slope of the curve is 2.

x2

y2

= 2 at

Solution: Let the required point be (x1, y1).


Y

Given equation of the curve is x2 y2 = 2 (1)


2

(2)
x1 y1 = 2
[Q Since (x1, y1) lies on the curve].
O

Now, differentiating both sides of the equation (1)


w.r.t x, we get
dy
2x 2 y
=0
dx
dy
x

=
(3)
dx
y

744

How to Learn Calculus of One Variable


Y

Now, differentiating (1) w.r.t x, we get

y=

f(

x)

,y
(x 1

Now, the value of


=

dy
x1
at x1 , y1 =
Slope of
dx
y1
the tangent
(4)
Now, according to question, the slope of the curve
at (x1, y1) = 2
(5)
Q The slope of the curve = The slope of the tangent
at (x1, y1).
Now, the value of

x1
= 2 x1 = 2 y1
y1

(6)

Now, putting (6) in (2), we get

4 y12

y12

=2

3 y12

LM dy OP
N dx Q

x = x1
y = y1

dy
2
= 3x
dx

dy
at (x1, y1)
dx

= 3x12

(3)

The equation of the tangent at (x1, y1) is

(y y1) = 3x12 (x x1)


Since this line passes through (0, 54)
(0, 54) will satisfy the equation of tangent

54 y1 = 3x12 (0 x1)

y1 54 = 3x12 x1
y1 = 3x13 + 54

(4)

Putting the value of y1 from eqns (4) in (2), we get

2
= 2 y1 =
3

(7)

3x13 + 54 = x13 2x13 = 54

54
= 27 x1 = 3
2
Required point is (x1, y1) = (3, 27)
3

2
Now, from eqn (6), x1 = 2 y1 x1 = 2
3

F
GH

2
2
Required points are (x1, y1) = 2
,
3
3

F
GH

= 2

2
2
,
3
3

I and FG 2
JK H

2
2
,
3
3

I
JK

x1 =

I
JK

Question: Find the coordinates of the point where


the tangent to the curve y = x2 + 3x + 4 passes through
the origin.
Solution: Let (x 1, y 1) be the coordinates of the
required point on the curve y = x2 + 3x + 4
(1)
Y

y=

Question: Find the coordinates of the point on


y = x3, where the tangents through the point (0, 54)
meet the curve.
Solution: Let (x1 , y1 ) be the coordinates of the
required point on the curve y = x3
(1)
3

y1 = x1

(2)

y = f (x)
P(
x1 ,

y1 )

x 1,
P(

f (x

y 1)

Now, differentiating (1) w.r.t x, we get


dy
= 2x + 3
dx
dy
Now, the value of
at (x1, y1)
dx
dy
=
= 2 x + 3 x = x = 2 x1 + 3
1
dx x = x1

LM OP
N Q

y = y1

Tangent and Normal to a Curve

x = x1
y = y1

(2)
Now, the tangent (2) passes through the origin
(0, 0)
(0 y1) = (2x1 + 3) (0 x1)
y1 = (2x1 + 3) (x1)
y1 = (2x1 + 3) x1
(3)
y1 = (2x1 + 3) x1 = 2x12 + 3x1
2
Again (x1, y1) lies on the curve y = x + 3x + 4
(4)
y1 = x12 + 3x1 + 4
Eqns (3) and (4) 2x12 + 3x1 = x12 + 3x1 + 4
2x12 x12 = 3x1 3x1 + 4
x12 = 4 x1 = 2
Again, substituting the values of x1 in (4), we get
y1 = 14 for x = 2 and 2 for x = 2
Hence, the required points are (2, 14) and (2, 2).
1

Question: Find the coordinates of the points at which


the tangents to the curve y = x2 + 2x pass through
origin.
Solution: Letting that there is a point (x1, y1) on the
curve y = x2 + 2x
(1)
We have, y1 = x12 + 2x1
(2)

Now eqns (2) and (5) 2x12 + 2x1 = x12 + 2x1


2x12 x12 = 0 x12 = 0 x1 = 0
Now putting x1 = 0 in (2), we get,
y1 = 2x12 + 2x1 = 0

Required point = (x1, y1) = (x1, y1) = (0, 0)


Question: Find the coordinates of the points on the
curve y = 5 log (3 + x2) at which the slope is 2.
Solution: We are given y = 5 log (3 + x2)
(1)
(1) y1 = 5 log (3 + x12) provided (x1, y1) lies on the
curve.
Now, differentiating (1) w.r.t x, we get

10 x
1
dy
= 5
0 + 2x =
2
2
dx
3+ x
3+ x
dy
= tan = 2 (given)
Again since
dx

(2)
(3)

y=

)
f (x

x 1,
P(

P(

x1 ,

y1 )

y=

(4)

y1 = 2 x1 + 2 x1

Y
Y

x = x1
y = y1

a y y f LMN OPQ ax x f
a y y f = a2 x + 3f a x x f
dy
=
dx

a y y f = LMN dydx OPQ ax x f


a y y f = a2 x + 2f a x x f
Now, the tangent passes through (0, 0)
a0 y f = a2 x + 2f a0 x f

f (x

= Slope of the tangent


Again the equation of the tangent at (x1, y1) is

745

y 1)

Now, differentiating both sides of eqn (1) w.r.t x,


we get
dy
= 2x + 2
dx
dy

= 2 x1 + 2 = The slope of the tangent


dx x = x1

LM OP
N Q

y = y1

at (x1, y1)
(3)
Now, the equation of the tangent at (x1, y1) is

Now let the required point be (x1, y1)

LM dy OP
N dx Qb
L 10 x OP
and M
MN 3 + x PQ

x1 , y1

=2

10 x1

3 + x12
10 x1
= 2 10 x1 = 6 + 2 x12
(3) and (4)
2
3 + x1
2

x = x1

(4)

746

How to Learn Calculus of One Variable

5x1 = 3 + x12

N.B.: Since the length of perpendicular from


(x 1, y 1) to any line ax + by + c = 0 is

x12 5x1 + 3 = 0
x1 =

5 25 12 5 13
=
2
2

ax1 + by1 + c

g
R|5 + 13 F 5 + 13 I U|
=S
, log G
H 2 JK VW|
|T 2
R|5 13 R| F 5 13 I
, 5 log S3 + G
and S
|T 2
|T H 2 JK

a +b

Required points are x1 , y1

since length is always

positive.

Examples worked out:

U|U|
V|V|
WW

tangent PB.

Problems based on length of the perpendicular from


the origin upon the tangent at (x1, y1).
Q The equation of the tangent is
1

a y y f = LMN dydx OPQ ax x f


L dy O y + FG y x LM dy OP
xM P
GH N dx Q
N dx Q

Question: Find the length of perpendicular from the


foot of the ordinate upon the tangent to the curve
y = f (x).
Solution: Let us draw PM to x-axis MK to the

x = x1
y = y1

x = x1
y = y1

I
JJ = 0
K

K
M

,y
x1

y = f (x)

yx

The length of the perpendicular upon this


tangential line from the origin whose coordinates are
(0, 0) is

y1 x1
=

L dy
1 + MF I
MNH dx K

x = x1
y = y1

OP
PQ

LM dy OP
N dx Q

x = x1
y = y1

x = x1
y = y1

x x1

y1 + x1

LM dy OP
N dx Q

x = x1
y = y1

=0

MK = The length of the from M (x, 0) on


the tangent at P

0 x1
x = x1
y = y1

The equation of the tangent at P (x1, y1) is


1

LM dy OP
N dx Q

y = f ( x)

a y y f = LMN dydx OPQ

x = x1
y = y1

,y
x1

LM dy OP
N dx Q

x = x1
y = y1

y1 + x1

LF dy I
1+ M
MNH dx K

x = x1
y = y1

OP
PQ

LM dy OP
N dx Q
2

x = x1
y = y1

747

Tangent and Normal to a Curve

LM dy OP
N dx Q
y1

=
1+

LMF dy I
MNH dx K

x = x1
y = y1

OP
PQ

y1

1+

LMF dy I
MNH dx K

x = x1
y = y1

OP
PQ

(1)

(2)

= a cos a 1 cos + sin

= a cos a cos + a sin = a sin

y sin 1 a sin 1 + a 1 sin 1 cos 1


2

= x cos 1 + a cos 1 + a 1 sin 1 cos 1


2

...(3)

dx
d
=
a cos + sin
d d

= a sin + a sin + a cos


= a cos

a sin
dy dy dx
=

=
= tan
dx d d a cos

(7)

Now, the length of from (0, 0) to the normal (7)


is =

0 cos 1 + 0 sin 1 a
cos2 1 + sin 2 1

= | a | which is independent of the value 1 of


and hence a constant for all values of .

a
f
R d cos UV + a RS d a sin f UV
=aS
T d W T d W
= a sin + a ksin + cos p

x cos 1 + y sin 1 = a cos 1 + sin 1 = a

d cos
dy d a sin cos
d sin
=
=a
a
d
d
d
d

Again

gU|V
cos g|
W

x1 = a cos 1 1 sin 1

fUV
fW

for some value of = 1

x = a cos + sin
y = a sin sin

(6)

at any point is at a constant distance from the


origin.
Solution: Let (x1, y1) be any point on the given curve
where the parametric equation is

= tan 1

OP bx x g
1
b y y g = LMNSlope of the
tangent Q
y a asin cos f
cos
=
x a acos + sin f
sin
sin y a asin cos f
= cos x a acos + sin f
1

a
f
y = a asin cos f

y1

= 1

x = a cos + sin

b
= a bsin

= tan

Now, the equation of the normal at any point on


the given curve is given by

Question: Show that the normal to the curve

a
a

x = x1
y = y1

Working rule to find the coordinates of a point on


the curve whose parametric equations are given and
the tangential line passes through that point such
that some condition is imposed on that tangent.
Working rule:
1. Find
dy

(4)
(5)

dy
dy dy dt
=

using the formula


dx
dx dx dx

af
af

dt = f 1 t
f 2 t
dt
2. Let (x1, y1) be a point on the curve corresponding
to the parameter [or t, u, v etc. whichever is given
=

dx

748

How to Learn Calculus of One Variable

af

af

in the problem x = f 1 , y = f 2 ] = 1 [or, t1,


u1, v1, etc.].
3. Find

LM dy OP
N dx Q

= 1

= The slope of the tangent at 1

f LMN

dy

at P x1 , y1 = tan
dx
2

Now, the value of

1
where 1 is the parameter at P.
2

= tan

OP
Q

= 1

(4)

[or, u1, t1, v1 etc.] from the given condition and solve
for 1 [or, u1, t1, v1 etc.].

The slope of the tangent at x1 , y1 = tan

4. Find the values of x1 and y1 from the equations


x1 = f1 ( 1 ), y1 = f2 ( 2 )

Let

Examples worked out:


Question: Find the coordinates of the point on the
curve
x = a ( + sin ), y = a ( 1 cos )

where the tangent is inclined at an angle of


to the
4
x-axis.
Solution: Equation of the curve is

[At (x1, y1), the tangential line makes

Eqns (4) and (5) tan

fUV
fW

(2)

Now, differentiating (1) w.r.t , we get

dx
= a 1 + cos
d
dy
= a sin
d

(6)

f (x

From (2), for 1 =

F + sin I = a F + 1I
H 2 2K H 2 K
I
F
=a
y = a 1 cos
H
2K
L F I O
Required point is b x , y g = Ma G + 1J , a P
N H2 K Q
x1 = a

fU|
V|
W

a sin
dy
=
=
dx a 1 + cos

= tan
2
4

= 1 =
2
4
2

(1)

x1 = a 1 + sin 1
y1 = a 1 cos 1

angle
4

with x-axis]

Let (x1, y1) be a point on the curve corresponding


to = 1

(5)

fUV
fW

=1
4

y=

a
a

x = a + sin
y = a 1 cos

= tan

1
2

y1 )

Problems based on finding the coordinates of a point


on the curve whose parametric equations are given.

LM dy OP
N dx Q

P(
x1 ,

by putting the values of parameter 1 [or, u1, t1, v1


etc.].

f F
H

a sin

I
K

a 1 + 2 cos 1
2

cos
2
2 = tan
=

2
a 2 cos cos
2
2

Question: Find the co-ordinates of the points where


the tangents to the curve given in parametric form
x = 2u4 + u

a 2 sin

(3)

y = u4 2u2 +1 are parallel to x-axis.

Tangent and Normal to a Curve

Solution: Let (x 1 , y 1 ) be a point on the curve


corresponding to u = u1
(1)
y = u4 2u2 + 1, x = 2u4 + u
4
2
4
(2)
y1 = u1 2u1 + 1, x1 = 2u1 + u1
Now, differentiating the equation (1) w.r.t u, we get

3
3

OP
Q

j OP
+1 P
Q

4 u13 u1

8u13 + 1

u = u1

(4)

Again since the tangents are parallel to x-axis


dy

=0
(5)
dx x = x1
y = y1

Equating (4) and (5)

ta ng

en t

2 x 2 y dy
dy
b2 x
+ 2
=0
= 2
2
dx
a
b dx
a y

j =0u

= 0 or u1 = 1 or u1 = 1

u14 2u12

LM dy OP
N dx Q

y = y1

b 2 x1
(3)

a 2 y1

The slope of the normal at (x1, y1)


=

a 2 y1

b 2 x1
The equation of the normal is

ay y f = a

y1

b x1

b2 x1 y b2 x1 y1 = a2 y1 x a2 x1 y1

b2 x1 y + a2 x1 y1 = a2 y1 x + b2 x1 y1

b y y g = ab xy bx x g
1

Thus, the required points are (0, 1), (3, 0) and (1, 0).
2

+ 2 = 1,
2
a
b
the length of the normal varies inversely as the
perpendicular from the origin.

(4)

ax x f

u1 = 1 x1 = 1, y1 = 0 (x1, y1) = (1, 0)

Question: Show that in the ellipse

Now, the equation of the tangent is

u1 = 1 x1 = 3, y1 = 0 (x1, y1) = (3, 0)

x = x1

x1 (b2 y + a2 y1) = y1 (a2 x + b2 x1)

+ 1 = 1 (x1, y1) = (0, 1)

The slope at x1 , y1

+1
Now u1 = 0 x1 = 2u14 + u1 = 0,

y1 =

(1)

Now, differentiating (1) w.r.t x, we get

8 u13

=1

x = x1
y = y1

u1

P (x1, y1) P2 (x2, y2) P3 (x3, y3)

y = f (x)

u13

(3)

LM OP
N Q

LM
N
L 4 eu
=M
MN 8u

dy
Now, dx

equation is

ma

Solution: Let (x1, y1) be a point of the curve whose

no r

dy
dx
3
3
= 4u 4u ,
= 8u + 1
du
du
3
dy dy dx 4 u u

=
dx du du
8u 3 + 1

749

a 2 y y1 a 2 y12 = b 2 x x1 + b 2 x12
2

2 2

2 2

b x x1 + a y y1 = b x1 + a y1

[Dividing both sides by a2 b2]

(5)

750

How to Learn Calculus of One Variable

x x1
2

y y1

x1

y1

=1

...(6)

a
b
b
a
[from (2)]
Thus, the equation of the tangent is

x x1
a

y y1
b

1= 0

Length of from the origin (0, 0) upon the


tangent

constant term of equation of tangent

acoefficient of xf + acoefficient of yf
2

l=

b
p

1
p

Problems based on showing that two curves cut each


other orthogonally whose implicit equations are
f1 (x, y) = 0 or constant and f2 (x, y) = 0 or constant
Working rule:
1. Mark the given equations f1 (x, y) = 0 or constant
as first curve and f2 (x, y) = 0 or constant, second
curve.

dy
for the first curve and the second curve.
dx
3. Let (x1, y1) be the point of intersection of two given
curves.
2. Find

p=

FG x IJ + FG y IJ
Ha K Hb K
2

1
2

2
x1

2
+ y1
4 4

2
x1

a b
2

b + y1 a
2

a b

(7)

x1 b + a y1

Now, length of the normal

= y1

LF dy I
1+ M
MNH dx K
4

l = y1

l=

x = x1
y = y1

OP
PQ

= y1 1 +

a y1 + b x1
a

y1

2
y1

b x1

a y1
4

a y1

a y1 + b x1

1
a 4 y12 + b 4 x12
a2
2

Hence, l p =

a b
2
x1

which is a constant.

2
4

b + a y1

1
a

dy
at (x1, y1) for each given
dx
curves (i.e., for the first curve and second curve) which
will provide us m1 and m2 for the first and second
curve.
5. Check whether m1 m2 = 1 or not.
Thus, m1 m2 = 1 Two curves cut each other
orthogonally and m1 m2 1 Two curves do
not cut each other orthogonally.
4. Find the values of

a b
=

x12
y12
+
a4
b4

1
2

N.B.: 1. When the given two equations can be solved


simultaneously by using simultaneous equations
methods (i.e., by elimination/comparison of
coefficients of x and y/equating two equations/
additions and subtraction method/etc), we should
first of all find the point of intersection of two given
dy
for the first
curves at which are find the values of
dx
curve and second curve and then we should check
whether m1 m2 = 1 or not.
2. Whenever it is not possible to find the point of
intersection by using simultaneous equations method,

dy
for each given equation
dx
at (x1 , y1) which is supposed to be the point of
intersection of given curves (i.e., the first curve and
second curve).
then we find the values of

a y1 + b x1

Tangent and Normal to a Curve

Examples worked out:


x3

y2

Question: Show that the curves 3x = a and


3x2 y y3 = b cut each other orthogonally where a
and b are constants.
Solution: Let (x1, y1) be the point of intersection of
two curves whose equations are
x3 3x y2 = a
(1)
3x2 y y3 = b
(2)
Now, differentiating both sides of equation (1) w.r.t
x, we get

F
H

3x 3 1 y + x 2 y
2

dy x y
=
2 xy
dx

L dy O
M P
N dx Q

x = x1
y = y1

x y1
= 1
= m1
2 x1 y1

(3)

Again differentiating both sides of equation (2)


w.r.t x, we have

dy I
dy
F
3y
=0
3 2 xy + x
H
dx K
dx
dy
ex y j
+ 2 xy = 0
dx
2

LM dy OP
N dx Qb

x1 , y1

Now m1 m2 =

LM dy OP
N dx Q

b g

from the first curve

P 1,1

LM dx OP
MN dx PQ b g = 3x = 3
L dy O from the second curve
m =M P
N dx Q b g
LM d F 7 x I OP
GH 6 6 JK P
M
1
L 2x O
=M
= M P =
P
3
N 6Q
MM dx PP
MN
PQ a f
F 1I = 1 which means two
m m = 3
H 3K
3

x =1

P 1, 1

2 x1 y1
x12 y12

2 x1 y1
2
x1

Now, m1 =

b b 4ac
7 49 4 6 7
=
2a
26

P 1,1

dy
2 xy

= 2
2
dx
x y

x=

7 49 168
which are imaginary.
12
The above explanation implies only root under
consideration is x = 1 since imaginary root is not
considered y = 1 ( Q y = x3 y = 13 = 1)
Required point of intersection = P (x1, y1)
= P (1, 1)

Solving the equations (1) and (2) simultaneously


by eliminating y, we get 6x3 = 7 x2
6x3 + x2 7 = 0
(x 1) (6x2 + 7x + 7) = 0
Either (x 1) = 0 or (6x2 + 7x + 7) = 0
(x 1) = 0 x = 1
(6x2 + 7x + 7) = 0

dy
=0
dx

x y 2 xy

I
K

dy
=0
dx

751

2
y1

x =1
y =1

= m2

(4)

P 1, 1

2
x1

2
y1

= 1 which
2 x1 y1

means given curves cut each other orthgonally.

given curves cut orthogonally.

Question: Show that the curves y = x3 and 6y = 7 x2


intersect orthogonally.
Solution: Given equations of the curves are
y = x3
(1)
6y = 7 x2
(2)

Question: Show that that curves x2 = 2y and


6y = 5 2x3 intersect orthogonally.
Solution: Letting (x1, y1) to be the point of intersection
of the given curves

752

How to Learn Calculus of One Variable

x2 = 2y
(1)
6y = 5 2x3
(2)
and solving the equations of the given curves by

x12 (a a1) = y12 (b1 b)


2

x
= 5 2x3
eliminating y from (1), (2) we find 6
2
3x2 + 2x3 5 = 0 2x3 + 3x2 5 = 0
2x3 + 3x2 5 = 0
2x3 2x2 + 5x2 5x + 5x 5 = 0
(x 1) (2x2 + 5x + 5) = 0

x = 1 or x =

ax12 a1 x12 = b1 y12 by12

5 24 40
4

LM x OP = 1
MN 2 PQ 2
F 1I
Required point = a x , y f = 1 ,
H 2K
2

x =1
y = 12

x
dy 2 x

=
= x
2
2
dx

=1

2ax + 2by

dy
1
2
2

= 0 3x = x
dx
3
2
x =1
y = 21

(5)

Now, differentiating the equation (1) w.r.t x, we get

5 2x
5 1 3
= x
Again y =
6
6 3

m2 = x

U|
V|
|W

x1 =

m1 = x

1
x1
y1
=
=
b1 b a a1 ab1 a1 b

b1 b
2
x
b b
ab1 a1 b
12 = 1
a a1
2
a1
a
y1
and y1 =
ab1 a1 b

x =1

b1 b
a a1

Hence x = 1 and y =

Now, 2 y = x y =

2
y1

Alternatively,
(x1, y1) lies on the curves
(x1, y1) satisfies the equations (1) and (2)
(1) a x12 + b y12 1 = 0
(3)
(2) a1 x12 + b1 y12 1 = 0
(4)
Now, solving eqns (3) and (4) simultaneously by
cross-multiplication rule:

which is imaginary.

x1

dy
=0
dx

LM OP
N Q

ax
dy ax
=
m1 =
dx
by
by

x = x1
y = y1

ax1
by1

Similarly, differentiating the equation (2) w.r.t x, we


get

= 1

dy
=0
dx
a x
dy

= 1
dx
b1 x
2a1 x + 2b1 y

m1 m2 = 1 1 = 1 which means the given


curves cut each other orthogonally.
Question: Find the condition that the curves
ax2 + by2 = 1 and a1 x2 + b1 y2 = 1 may intersect
orthogonally.
Solution: Letting (x1, y1) to be the point of intersection
of the curves
ax2 + by2 = 1

(1)

a1 x2 + b1 y2 = 1

(2)

we have, ax12 + by12 = 1 = a1 x12 + b1 y12


[Q lies on the curve]

m2 =

LM a x OP
Nb yQ
1

x = x1
y = y1

a1 x1
b1 y1

The two curves intersect orthogonally at (x1, y1)


m1 m2 = 1

FG a x IJ FG ax IJ = 1
H b y K H by K
1

1 1

a a1 x1 = bb1 y1

x1

2
y1

b b1
a a1

(6)

Tangent and Normal to a Curve

Eqns (5) and (6)

LM dy OP
N dx Q

LM dy OP
N dx Q

b b
bb
a a1
aa
1
= 1
= 1
a a1
a a1
b1 b
bb1

a a1 b1 b
=
bb1
a a1
1
1
1
1

a1 a b1 b
which is the required condition.

Again the slope of the tangent to the curve y = f2

Question: Find the angle between the curves y = f1 (x)


and y = f2 (x) at (x1, y1), the point of intersection of the
curves.
Or, find the angle between the curves f1 (x, y) = 0
constant and f2 (x, y) = 0 at (x1, y1), the point of
intersection of the curves.
Solution: Supposing that y = f1 (x) or f1 (x, y) = 0 and
y = f2 (x) or f2 (x, y) = 0 are the two equations of the
curves C1 and C 2 respectively intersection at
P (x1, y1). Let the tangents PT1 and PT2 to the to the
curves C1 and C2 make angles of inclination i1 and i2
respectively with x-axis.
(x 1, y 1)

C2
C

C1
C

i1
O

T1

= tan i1 = m1
C1

(x) or f2 (x, y) = 0 at (x1, y1)

Remember: 1. The algebraic form of a condition is


always a relation (or, an equation) among the given
constant. Thus when we are asked to find the
condition, we mean to find the algebraic form of a
condition.
2. The student should know that by the conditions
under which a straight line touches a given curve is
meant the relation that must exist among the constants
occurring in the equations of the straight line and the
curve in order that they may touch each other.

x = x1
y = y1

753

i2
T2

Again let the angle between the two tangents =


Now, the slope of the tangent to the curve y = f1 (x)
or f1 (x, y) = 0 at (x1, y1)

LM dy OP
N dx Q

x = x1
y = y1

LM dy OP
N dx Q

= tan i2 = m2
C2

Now, angle of intersection of the curves

= = i2 ~ i1

Q i2 = + i1

tan = tan i2 ~ i1 =

m2 ~ m1
1 + m1 m2

LM m ~ m OP when m m 1
N1 + m m Q
L m ~ m OP
Also tan a f = tan = M
N 1+ m m Q
= tan

Hence, the angles between the curves


= = tan

LM m ~ m OP
N 1+ m m Q
1

Remember:
1. When the curves touch, the angle of intersection
= 0 when the slopes of the tangents are same, the
curves touch each other.
2. When the angle between the tangents is a right
angle, the curves are said to cut each other
orthogonally If m1 m2 = 1, then the curves are
said to cut each other orthogonally.
3. If either m1 or m2 (or both of them) is infinity, then
it is advisable to express m1 and m2 in the form of
tan . In other words,
If m1 = tan i1 and m2 = tan i2, then the angle between
the two tangents to the two intersecting curves at
(x1, y1) is
= i1 i2 = absolute value of difference of i1
and i2 provided m1 or m2 is infinity or both of them are
infinity.

754

How to Learn Calculus of One Variable

4. In general

LM dy OP = LM dy OP
N dx Q N dx Qa
C1

1 = 0 for the curve C1.

and

LM dx OP
N dy Q

=
C2

LM dx OP
N dy Qb

x1 , y1

x1 , y1

= 0 tan 1 = 0
(i)

=0

2 = 90 for the curve C2.


(ii)
Hence from (i) and (ii), we get the angle of
intersection between the two curves is
= 2 1 = 90 .
In particular (a) If the slope of the tangent line
drawn to any curve y = f1 (x) at the point (0, 0) is zero,
then the tangent to the curve y = f1 (x) at the point
(0, 0) is x-axis (i.e., m1 = 0 The tangent to the
curve is x-axis).
(b) If the slope of the tangent line drawn to the

curve y = f2 (x) at the origin (0, 0) is infinity (i.e.,

dx
=0
dy

at origin) then the tangent to the curve y = f2 (x) at the


point (0, 0) is y-axis.
(a) and (b) The curves y = f1 (x) and y = f2 (x) at
(0, 0) intersect orthogonally. Since m1 = 0 tan 1
= 0 1 = 0 and

FG dx IJ
H dy K b

0, 0

are always inclined to each other at some angle called


the angle of intersection of two curves. When the
angle between tangents is a right angle, the
intersection is called orthogonal.
Question: How to find the angle of intersection
between two curves y = f1 (x) or f1 (x, y) = 0 and
y = f2 (x) or f2 (x, y) = 0
Working Rule:
1. Let (x1, y1) be the point or intersection of two given
curves y = f1 (x) or f1 (x, y) = 0 and y = f2 (x) or
f2 (x, y) = 0
2. Find the coordinates of the point of intersection
of two given curves by putting (x1, y1) in the given
two equations of the curves and solving these two
equations to find (x1, y1).
dy
3. Find
of both curves (differentiate both
dx
equations of the curves w.r.t x).
4. Put the numerical values of the coordinates of the
dy
for
point of intersection (x1, y1) of the curves in
dx
both equations of the curve and call these values as
m1 and m2.
Where m1 = The value of

=0

LM dy OP = LM dy OP
N dx Q N dx Q
C1

2 = 90
= 2 1 = 90 .

m2 = The value of

y = f1 (x)

y = f2 (x)
X

3. Angle between two curves/angle of intersection


of two curves implies the angle between their tangent
lines drawn to the curves at a common point known
as point of intersection of two curves.
4. When two curves intersect each other, at the point
of intersection, each curve has a tangent separate
and distinct (unless they touch there). The tangents

for the first curve

dy
at (x1, y1)
dx

LM dy OP = LM dy OP
N dx Q N dx Q
C2

x = x1
y = y1

dy
at (x1, y1)
dx

x = x1
y = y1

for the second curve

5. Use the formula = Angle of intersection of two


intersecting curves at (x1, y1)
= tan

LM m ~ m OP
N 1+m m Q
LM dy OP = LM dy OP
N dx Q N dx Qb

Note: 1.

x1 , y1

, provided (x1, y1) is

supposed to be the point of intersection of the curves.

Tangent and Normal to a Curve

2. If (x, y) is supposed to be the point of intersection


of the two intersecting curves, then (x, y) is found
directly by solving simultaneously the two given
equations of the curves y = f1 (x) and y = f2 (x).
3. The slopes m1 and m2 at the point of intersection
is always found by the derivatives calculated from
the two given equations at the known coordinates of
the point of intersection of the two given equations
for the curves.
4. Remember that the slopes m1 and m2 are always
calculated at a given point whose coordinates are
known represented as (x1, y1).
5. m1 and m2 are slopes of two tangents drawn to the
two curves at the same common point of intersection
of two curves.
Examples worked out:
Question: Find the angle of intersection of the curves
x2 + y2 = 8 and y2 = 2x.
Solution: (a) Let (x1, y1) be the point of intersection
of the given curves
(1)
x12 + y12 = 8
and y12 = 2x1
(2)
Now, solving eqns (1) and (2) by eliminating y1
from (1), we have
x12 + 2 x1 = 8 (x1 + 4) (x1 2) = 0 x1 = 2, 4
when x1 = 2 , y1 = 2
when x1 = 4, y1 = imaginary for y12 = 8
Required points of intersection are (x1, y 1)
= (2, 2) and (2, 2)
(3)
(b) The equations of the curves are
x2 + y2 = 8
(4)
y2 = 2x
(5)
Differentiating eqn (4) w.r.t x, we get
dy
x
= = Slope of the tangent to the curve
dx
y

LM dy OP = LM dy OP
N dx Q N dx Q
C1

LM x OP
N yQ

x = x1
y = y1

1 x1 = 2
y1 = 2

755

2
= 1
2

= m1 = tan i1
(6)
Similarly, the slope of the tangent to the curve
y2 = 2x at any particular point (x1, y1)
=

LM dy OP
N dx Q

=
C2

LM dy OP
N dx Q

x = x1
y = y1

LM 1 OP
Ny Q

1 x1 = 2
y1 = 2

1
2

= m2 = tan i2
(7)
Thus, at (2, 2) we get the slopes which are 1
1
= tan i1 and = tan i2
2
Again, we find the slopes of the tangents at (2, 2)
is

LM dy OP = LM dy OP
N dx Q N dx Q
L dy O L dy O
and M P = M P
N dx Q N dx Q

x = x1

C1

y = y1

C2

LM x OP
N yQ
L1O
=M P
Ny Q

x = x1
y = y1

x1 = 2

2
= 1 (8)
2

y1 = 2
x1 = 2

1
2

(9)

y1 = 2

Thus, at (2, 2), we get the slopes which are

1
= tan i2
2
(d) Hence, the angle of intersection at (2, 2) is

1 = tan i1 and

LM
N

= tan 1
= tan

= tan 1

x2 + y2 = 8 at any point (x, y).


Again differentiating eqn (5) w.r.t x, we get
1
dy
= = Slope of the tangent to the curve
dx
y

= tan

y2 + 2x at any point (x, y).


(c) Now, the slope of the tangent to the curve
x2 + y2 = 8 at any particular point (x1, y1)

= tan

m1 ~ m2
1 + m1 m2

OP
Q

LM difference of slopes OP
N 1 + product of slopes Q
LM 1 b 1g OP
MM 2 1 PP
N 1 + b1g 2 Q
LM 1 + 1OP
LM 3 OP
2
MM 1 PP = tan MM 21 PP
N 1 2Q
N 2Q
1

(10)

756

How to Learn Calculus of One Variable

for y 0
Again differentiating (2) w.r.t x, we get

Again, the angle of intersection at (2, 2)

= tan

= tan

LM 1 F 1 I OP
MM HF 21KI PP = tan
MN 1 + 1H 2 K PQ
LM 3 OP
MM 21 PP = tan 3
N 2Q

LM 1 + 1 OP
MM 21 PP
N 1 2Q

dy 2 x
=
= Slope of the tangent at any point
dx
a
(x, y)
(4)
(c) Now the slope of the tangent to the curve
y2 = ax at any particular point (x1, y1) is

(11)

C1

Thus, at (2, 2) and (2, 2), we get the angle of


intersection = tan

Question: Find the angle of intersection between the


parabolas y2 = ax and x2 = ay.
Solution: Letting (x1, y1) to be the point of intersection
of both given curves
(3)
y2 = ax ...(1) y12 = ax1
2

x
x
...(2) y1 = 1
(4)
a
a
Now, solving eqns (3) and (4) by eliminating y1
first, we find that
y=

LMF x I OP
MNGH a JK PQ
2
1

x4
= a x1 12 = ax1 x14 = a 3 x1
a

x14 a3 x1 = 0 x1 (x13 a3) = 0 x1 = 0


or a
x1
0
= =0
a
a
2

x1
a
=
=a
a
a
Required points are (0, 0) and (a, a).
(b) Now considering eqns (1) and (2) differentiating
(1) first, we have

and when x1 = a , y1 =

dy
dy
a
=a
=
dx
dx 2 y

= Slope of the tangent at point (x, y)

Q y = ax y =

ax

b g LMN dydx OPQ = lim f b0 + hhg f b0g


b g
a b0 + h g
(5)
= lim
= i = 90

f 0 =

h 0

0, 0

h0

h
Again, the slope of the tangent to the curve
2

x
at any particular point (x1, y1) is
a

y=

LM dy OP = LM dy OP
N dx Q N dx Q
L 2 x O = LM 2 x OP
=M P
NaQ N a Q
x = x1
y = y1

C2

x = x1
y = y1

LM dy OP = LM dy OP
N dx Q N dx Q
C1

x1 = 0
y1 = 0

20
a

(3)

x = x1
y = y1

1
= m1
2
dy
dy
=
dx C2
dx

LM a OP
N2y Q

1 x1 = a
y1 = a

LM OP
N Q

d
d 2
ax
y =
dx
dx
2y

x = x1
y = y1

= 0 i2 = 0
(6)
Similarly, we find the slopes of the tangents at
(a, a)

when x1 = 0 , y1 =

LM dy OP = LM dy OP
N dx Q N dx Q

LM OP
N Q

2a
= 2 = m2
a

a
2a
(7)

x = x1
y = y1

LM 2 x OP
NaQ
1

x1 = a
y1 = a

(8)

757

Tangent and Normal to a Curve

LM dy OP = LM dy OP
N dx Q N dx Q
L x O = L 2 O = 1 = m
= M P
N y Q MN 2 PQ

(d) Hence, the angle of intersection at (0, 0)

= = i1 i2 = 90
Again, the angle of intersection at (a, a)

LM m ~ m OP
N 1+m m Q
LM 2 1 OP
MM 2 1 PP = tan LMN 43 OPQ
N 1+ 2 2Q
1

= = tan

= tan

16
x12

(x12 4)2 = 0 x1 = 2 y1 = 2 x1 = 2

x = x1
y = y1

C2

x1 = 2
y1 = 2

tan i1 = 1 and tan i2 = 1 i1 = i2


Again, the slope of the tangent to the curve
x2 + y2 = 8 at any particular point (x1, y1)

g LMN dydx OPQ = LMN dydx OPQ


L x O = L 2 O = 1 = m
= M P
MN 2 PQ
N yQ
b

= 2 , 2 is

C1

x1 = 2
y1 = 2

1 x1 = 2
y1 = 2

Required points of intersection are (x1, y1) = (2, 2)


and (2, 2)(5)
(b) The equations of the curves are
x2 + y2 = 8

(1)

xy = 4

(2)

Now, differentiating (1) w.r.t x, we get


2x
dy
dy
x
2x + 2y
=0
=
=
2y
dx
dx
y

Again, differentiating (2) w.r.t x, we get


dy
dy
y
4
x
+ y 1= 0
= = 2
dx
dx
x
x
4
Q xy = 4 y =
x

OP
Q

(10)

and the slope of the tangent to the curve xy = 4 at


any particular point (x1, y1)

f LMN dydx OPQ = LMN dydx OPQ

= 2 , 2 =

LM
MN

(6)

(9)

Thus, at (2, 2), we get the slopes which are

[from x1 y1 = 4 ...(4)]

LM
N

LM dy OP = LM dy OP
N dx Q N dx Q
L 4 O = L 4 O = 1= m = tan i
= M P
N x Q MN 4 PQ
=

= 8 x14 + 16 = 8x12

(8)

Similarly, the slope of the tangent to the curve


xy = 4 at any particular point (x1, y1)

= 2 x1 = 2

= tan i1

x1 = 2
y1 = 2

Question: Find the angle of intersection of the curves


x2 + y2 = 8 and xy = 4.
Solution: (a) Letting (x1, y 1) to be the point of
intersection of the given curves
x2 + y2 = 8 (1) x12 + y12 = 8
(3)
(4)
and xy = 4
(2) x1 y1 = 4
Now, solving (3) and (4) simultaneous by
eliminating y1 first from (4), we get

x12 +

x = x1
y = y1

C1

C2

4
2
x1

OP
PQ

LM 4 OP = 1 = m
N 4Q

=
x1 = 2

x1 = 2
y1 = 2

(11)

i1 = i2

(7)

(c) Now, the slope of the tangent to the curve


x2 + y2 = 8 at any particular point (x1, y1) = (2, 2) is

(d) Hence, the angle of intersection at (2, 2) and at


(2, 2) is i1 i2 = 0, as i1 = i2 at both the points.
Question: Find the angle at which the curves x2 y2
= a2 and x2 + y2 = 2a2 intersect.
Solution: (a) Letting (x1 , y1) to be the point of
intersection of the given cuves

758

How to Learn Calculus of One Variable

x2 y2 = a2 (1) x12 y12 = a2 (3)


and x2 + y2 = 2a2 (2) x12 + y12 = 2a2 (4)
Now, solving the above two equations (3) and (4)
x12 y12 = a2
x12 + y12 = 2a2
2

2x1 =

3a2

3a
x1 =
2

Q x1 y1 = a

3
2

y1 =

3
2

a
2

is

I , (four points).
J
2K

(b) The given equations of the curves are


x2 y2 = a2
x2 + y2 = 2a2
Now, differentiating (1) w.r.t x, we get

(1)
(2)

dy
dy
x
2x 2 y
=0
=
dx
dx
y

(5)

dy
dy
x
=0
=
dx
dx
y

(6)

(c) Now, the slope of the tangent to the curve


x2 y2 = a2 at any particular point (x1, y1)

I
J
2
2K
dy
dy
is LM OP = LM OP
N dx Q N dx Qb
=

F
GH

3a

C1

3a
2

C1

2
=
a

x1 , y1

LM x OP
N yQ

1 x1 =
y1 =

3a
2
a
2

OP
Q

2
= 3 = m2
a

(8)

LM x OP
Ny Q

3 = m1

3a
2
a
2

LM dy OP
N dx Q

=
C2

LM F
MN GH

1
1

x1 =
y1 =

3a
2
a
2

LM dy OP
N dx Qa

3a
2

x1 , y1

2
a

LM x OP
N yQ

1
1

I OP =
JK PQ

x1 = 3a
2
y1 = a

3 = m2

(10)

(d) Hence, the angle of intersection at

F
GH

a,

= tan

(7)

LM x OP
Ny Q

= 3 = m1
(9)
a
2
Similarly, the slope of the tangent to the curve
x2 + y2 = 2a2 at any particular point (x1, y1)

I is = tan L m ~ m O
J
MN 1 + m m PQ
2K
LM 3 e 3j OP
L 3 + 3 OP

= tan M
MN 1+ 3 e 3j PQ
N 1 3 Q
LM 2 3 OP = tan m 3
N 2 Q
a

1 x1 =
y1 =

x1 , y1

3a

= tan

x1 , y1

F 3a , aI
GH 2 2 JK
LM dy OP = LM dy OP
N dx Q N dx Qb

Again, differentiating (2) w.r.t x, we get


2x 2y

LM dy OP
N dx Qb

=
2

a,

C2

LM
N

=
2
2
Required points of intersection are (x1, y1)

F
= G
H

Again, the slope of the tangent to the curve x2 y2


= a2 at any particular point (x1, y1)

LM dy OP
N dx Q

= 1

3a 2a
3a
2
a =
2
2

3 2
a y12 = a 2
2

y1 =

Similarly, the slope of the tangent to the curve


x2 + y2 = 2a2 at any particular point (x1, y1)

= 120 and 60

Tangent and Normal to a Curve

Again, the angle of intersection at

F
GH

3
2

= tan

I is = tan L m ~ m O
MN 1 + m m PQ
J
2K
LM 3 e 3 j OP
L 2 3 OP

= tan M
MN 1+ 3 e 3j PQ
N 1 3 Q
LM 2 3 OP = tan 3
N 2 Q

a,

= tan

= tan

m 3 = 120 and 60. Similarly the angles

of intersection at the points

F 3a , a I
GH 2 2 JK are the same.

F
GH

3 a aI
,
J
2
2K

and

U|
V 8x = 4x
y = 8x |
W
2
3

8x3 4x2 = 0

F 1 , 1I .
H2 K

dy
= 8x
dx

LM dy OP
N dx Q

= 8x
C1

b0, 0g = 0 = m1

(5)

Similarly, the slope of the tangent to the curve


y = 8x3 at the point of intersection (x, y)
= (0, 0) is

LM dy OP
N dx Q

= 24 x 2

b0 , 0 g =

C2

0 = m2

(6)

Again, the slope of the tangent to the curve y2 =


4x at the point of intersection (x, y)

FG 1 , 1IJ is LM dy OP
H 2 K N dx Q

= 8x
C1

d , 1i = 8
1
2

1
2

= 4 = m1
(7)
Similarly, the slope of the tangent to the curve
y = 8x3 at the point of intersection (x, y)

F 1 , 1I = LM dy OP
H 2 K N dx Q

= 24 x

c , 1h
1
2

=
C2

= 24

LM dy OP
N dx Qc

1 ,1
2

1
= 6 = m2
4

(8)

(d) Hence, the angle of intersection of the curves


at the point of intersection (x, y)

b g

1
2
Required
points
of
intersection
are
(x, y) = (0, 0)

(b) The given equations of the curves are


y = 4x2
and y = 8x3
Now, differentiating (1) w.r.t x, we get

= (0, 0) is

LM
N

= 0 , 0 is = tan 1

4x2 (2x 1) = 0 x = 0 or x =

and

dy
2
= 24 x
(4)
dx
(c) The slope of the tangent to the curve y = 4x2 at
the point of intersection (x, y)

Question: Find the angle between the curves y = 4x2


and y = 8x3.
Solution: (a) If (x, y) be the point of intersection of
the given curves
y = 4x2
(1)
y = 8x3,
(2)
then let us find the point of intersection of the given
curves by solving the given equations (1) and (2),

y = 4x

Again, differentiating (2) w.r.t x, we get

759

= tan

OP
Q

m1 ~ m2
1 + m1 m2

LM 0 0 OP = tan
N 1 + 0 0Q

0=0

Again, the angle of intersection of the curves at


the point of intersection (x, y)
(1)
(2)

FG 1 , 1IJ is = tan LM m ~ m OP
H2 K
N 1+ m m Q
L 6 4 OP = tan L 2 O
= tan M
MN 25 PQ
N 1 + 24 Q
1

(3)

760

How to Learn Calculus of One Variable

Question: Find the angle of intersection of the curves


2y2 = x3 and y2 = 32x.
Solution: (a) If (x, y) be the point of intersection of
the given curves
2y2 = x3 and
(1)
y2 = 32x,
(2)
then let us find the point of intersection of the
given curves by solving the above equations (1) and
(2), x3 = 64x [Eliminating y form (1)]
x3 64x = 0 x (x2 64) = 0
x (x 8) (x + 8) = 0 x = 0, 8, 8
Now, from (2), we have y2 = 32 0, 32 8, 32 (8)

y = 0, 16 , 16 1
Required points of intersection of the given
curves are (x, y) = (0, 0), (8, 16) and (8, 16).
(b) The given equations of the curves are
2y2 = x3
(1)
and y2 = 32x
(2)
Now, differentiating (1) w.r.t x, we get
dy
dy
2
2
= 3x 4 y
= 3x
2 2y
dx
dx
2

dy 3x
=
dx
4y

...(3)

dy
dy 16
= 32 1
=
dx
dx
y

LM OP LM OP
N Q N Q
dy
=
dx
C1

(4)

x
2

Q 2y = x y =

b g LMN dydx OPQ


b

f 0 =

bg

f 0 = lim

h 0

=
C2

LM dy OP
N dx Q b

0, 0

b0 + hg
h

= lim

h 0

0, 0

LM dy OP = LM dy OP
N dx Q N dx Qb

= = m2

8 , 16

C1

LM 3x OP
MN 4 y PQb
2

8 , 16

388
= 3 = m1
4 16
Similarly, the slope of the tangent to the curve
y2 = 32x at the point of intersection (8, 16)
=

LM dy OP
N dx Q

=
C2

LM dy OP
N dx Qa

8 , 16

LM16 OP
N y Qa

8 , 16

16
= 1 = m2
16
Lastly the slope of the tangent to the curve 2y2 = x3
at the point of intersection (8, 16)
=

LM dy OP = LM dy OP
N dx Q N dx Qb

8 , 16

3 64
= 3 = m1
4 16

a f

and the slope of the tangent to the curve y2 = 32x at


the point of intersection (8, 16)

LM dy OP = LM16 OP
N dx Q N y Qa
L 16 O
= M P = 1 = m
N 16 Q
=

C2

x1 = 0
y1 = 0

8 , 16

LM dy OP
N dx Q

Again the slope of the tangent to the curve


2y2 = x3 at the point of intersection (8, 16)

(c) The slope of the tangent to the curve 2y2 = x3 at


the point of intersection (0, 0)

dy
=
dx

C1

for y 0
Again, differentiating (2) w.r.t x, we get
2y

Similarly, the slope of the tangent to the curve


y2 = 32x at the point of intersection (0, 0)

b g bg

f 0+ h f 0

= 0 = m1

(d) Now, we consider the angle of intersection at


(0, 0)

m1 = 0 i1 = 0

= i1 i2 = .
2
m2 = i2 =

761

Tangent and Normal to a Curve

Similarly, the angle between the curves at (8, 16)

LM m ~ m OP
N 1+ m m Q
LM a3 1f OP
N a1 + 3 1f Q
LM 2 OP = tan LM 1 OP
N 4Q
N 2Q

= = tan

= tan

= tan

= tan
= tan

= tan 1

x2 + y2 4x 1 = 0

(3)

Now, putting the value of y from (3) into (1), we get

dy
dy 2 x
4=0
=
,y0
dx
dx
y

4)2 4x 1 = 0

5x2 20x + 15 = 0

x2 4x + 3 = 0
(x 1) (x 3) = 0

x = 1, 3
Form (3), putting x = 1, 3 in (1), we get y = 2, 2
Hence, the required points of intersection are
(x, y) = (1, 2), (3, 2)

dy
dy
2
=0
dx
dx

g dydx = 0 dydx = 1 x y , y 1.

x + y 1

(c) The slope of the tangent to the curve x2 + y2


4x 1 = 0 at the point of intersection (x, y)

g LMN dydx OPQ = LMN dydx OPQ


b g
L 2 x OP = 2 1 = 1 = m
=M
N y Q a f 2 2
b

= 1 , 2 is

1, 2

C1

1 , 2

Similarly, the slope of the tangent to the curve


x2 + y2 2y 9 = 0 at the point of intersection (x, y)

g LMN dydx OPQ

= 1 , 2 is

(1)

x2 + y2 2y 9 = 0
(2)
and solving these equations (1) and (2) simultaneously, we get
4x + 2y + 8 = 0 y = 2x 4

2y 9 = 0

2x + 2 y

Question: Find the angle of intersection of the curves


x2 + y2 4x 1 = 0 and x2 + y2 2y 9 = 0.
Solution: (a) Letting (x, y) to be the point of
intersection of the given curves

x2 + (2x

(2)

+ y2

Again differentiating (2) w.r.t x, we get

LM m ~ m OP
N 1+ m m Q
LM 3 a1f OP = tan LM 3 + 1OP
N 1+ 3 Q
N 1 + a3f a1f Q
LM 2 OP = tan LM F 1 I OP
N 4Q
N H 2K Q
LM 1 OP
N 2Q
1

(1)

x2

2x + 2 y

Lastly, the angle between the curves at (8, 16)


1

x2 + y2 4x 1 = 0
Now, differentiating (1) w.r.t x, we get

= = tan

(b) The given equations of the curves are

=
C2

LM dy OP
N dx Q b

1, 2

LM x OP
N1 y Q b

1, 2

1
1
= = m2
1+ 2 3

Again the slope of the tangent to the curve


x2 + y2 4x 1 = 0 at the point of intersection (x, y)

b g LMN dydx OPQ = LMN dydx OPQ


b g
L 2 x OP = 2 3 = 1 = m
=M
2
N y Qa f 2
= 3 , 2 is

C1

3, 2

3, 2

Similarly, the slope of the tangent to the curve


x2 + y2 2y 9 = 0 at the point of intersection (x, y)

b g LMN dydx OPQ

= 3 , 2 is

=
C2

LM dy OP
N dx Qb

3, 2

762

How to Learn Calculus of One Variable

L x OP
=M
N1 y Q a

C 1 = f 1 (x, y ) = 0

3
3
=
=
= 3 = m2
1 2 1
3 , 2f

Q (x2, y 2)

(d) Hence, the angle of intersection of the curves


at the point of intersection (x, y)

LM
N

= 1 , 2 is = tan 1

= tan

m1 ~ m2
1 + m1 m2

LM 1 F 1 I OP
MM 3 1HF 21KI PP = tan
MN 1 + 3 H 2 K PQ

OP
Q

b g

LM
N

= tan 1

m1 ~ m2
1 + m1 m2

i1

LM 1 + 3 OP
MM 2 3 PP = tan b 1g
N 1+ 2 Q

i2
T2

T1

Now, differentiating the equation f1 (x, y) = 0 (1)

LM dy OP = tan i = m (say)
N dx Q
L dy O
Similarly, M P = tan i = m (say)
N dx Q
w.r.t x, we get

OP
Q

(2)

(3)

Now, required angle between the two tangents to


a curve at two given points = = i2 ~ i1

tan = tan i2 ~ i1

= 45 , 135
Type: To find the angle between two tangents to a
curve at two given points:
Supposing that there are two points P and Q lying
on the curve c1 represented by the equation
f1 (x, y) = 0

C1

P (x1, y1)

= 45 or 135
Again, the angle of intersection of the curves at
the point of intersection (x, y)
= 3 , 2 is = tan 1

(1)

PT1 = tangent at P
QT2 = tangent at Q
R = The point of intersection of the two tangents
PT1 and QT2.
Now, we are required to find out the angle between
these two tangents.
Now, we suppose that = Angle between two
tangents to the single curve at two given points P
and Q.
P = (x1, y1)
Q = (x2, y2)

tan i2 ~ tan i1
m2 ~ m1
=
1 + tan i2 tan i1 1 + m1 m2

LM m ~ m OP
N1 + m m Q
m ~m
Also, tan a f = tan =
1+ m m
= tan

= Angle between the tangents


= tan

LM m ~ m OP
N 1+m m Q
2

Working Rule:

dy
by differentiating both sides of the given
dx
equation w.r.t x.
1. Find

LM dy OP
N dx Qb
L dy O
3. Find M P
N dx Qb
2. Find

x1 , y1

= The value of

dy
at (x1, y1) = m1
dx

= The value of

dy
at (x2, y2) = m2
dx

x2 , y2

763

Tangent and Normal to a Curve

4. Use = tan

LM m ~ m OP or = i
N 1+m m Q
2

Note: 1. = tan 1

i1 .

acute angle between the tangents to a single curve at


two given points.

m2 ~ m1
is used to find the obtuse
1 + m1 m2
angle between the tangents to a single curve at two
given points.
3. If ml = tan i1 and m2 = tan i2, then the angle between
2. = tan 1

the two tangents is = i1 i2 = Absolute value

)
2

Examples worked out:


Question: Find the angle between the tangents to the

F
H

I
K

F I
H K

5
3
and 4 ,
.
4
4
Solution: (a) Given equation of the curve is
x2 = 8y + 6 8y = x2 6
(1)
Now, differentiating the equation (1) w.r.t x, we get
curve x2 = 8y + 6 at the points 0 ,

dy
dy
= 2x 0 8
= 2x
dx
dx
dy 2 x x
=
=
dx
8
4
3
dy
(b) The value of
at 0 ,
4
dx
8

LM dy OP
N dx QFGH

LxO

3
0,
4

= 0 = m1
and the value of
=

LM dy OP
N dx QFGH

= 1 = m2

IJ = MN 4 PQ FG 4 , 5 IJ =
K
H 4K

and m2 = 1 tan i2 = tan

i2 = = 45 (6)
4
4

Angle between the tangents = = i2 i1

= 45 0 = 45 =
4
Question: Find the angle between the tangents to
1 5
and
the curve y = x 2 + 1 at the points
,
2 4

F
H

F 3 , 7I .
GH 2 4 JK

...(3)

I
K

Solution: (a) Given equation of the curve


y = x2 + 1
(1)
Now, differentiating equation (1) w.r.t x, we get

dy
= 2x + 0 = 2x
dx
(b)

LM dy OP
N dx Q

x = 12
y = 54

= 2x

(2)
x = 21
y = 45

1
= 1 = m1
2
dy
= 2x
and
dx x = 7 23

LM OP
N Q
F 3I =
= 2 G
H 2 JK
y= 4

5
dy
at 4 ,
4
dx

LxO

5
4,
4

(2)

0
4

FG IJ
H K

(Acute angle between the tangents)


4
Alternatively,
(5)
Q m1 = 0 tan i1 = tan 0 i1 = 0

=2

FG IJ
H K

IJ = MN 4 PQ FG 0 , 3 IJ =
K
H 4K

01
= 1
1+0

m2 ~ m1
is used to find the
1 + m1 m2

of difference of i1 and i2. (If i1 or i2 =

tan =

(3)
x=
y = 74

3
2

3 = m2

Now, m1 = tan i1 = 1 = tan

(4)

i1 = = 45
4
4

m2 = tan i2 = 3 = tan 120 i2 = 120


The acute angle between the tangents to the

curve is given by = i2 i1 = 120 45 = 75

4
4

(4)

Alternatively, = Acute angle between the


tangents

764

How to Learn Calculus of One Variable

= tan

LM
N

OP
3 + 1Q
3 1

Question: Find the angle between the tangents to


x

+ 2 = 1 at the points (a, 0) and (0, b).


2
a
b
Solution: (a) Given equation of the curve is

the curve

2
2

dy
b
x
= 2
dx
y
a
for y 0

...(2)

bg

(b) y = f x = b 1

b g LMN dydx OPQ


b

f a =

a,0

x
, x2 a2 .
2
a

= lim

b g b g,

f a+h f a

h0

h<0

b h h + 2a

h 0

and

LM dy OP
N dx Qa

g = i

ah

0, b

LM dy OP
N dx Q

(1)

a
b
Now, differentiating equation (1) w.r.t x, we get

= lim

Working Rule:
1. Let (x1, y1) be the point of contact of the tangent
to the given curve.
2. Put (x1, y1) in the given equation of the curve.
3. Find

=1

Problems based on finding the area of a triangle


formed by the portion included between the axis and
the tangent to a curve.

b
a

2
2

LM x OP
NyQ

= 90

(3)

x = x1
y = y1

4. Find the equation of the tangent, using the formula


at (x1, y1)

a y y f = LMN dydx OPQa


1

b
a

x x1

f a

f a

1
x1 y2 y 3 + x2 y 3 y 2 + x 3 y1 y2
2

Examples worked out:


Question: Find the area of the triangle formed by the
portion included between the axis and the tangent to
2

Solution: (a) Given equation of the curve is


2

x3 + y3 = a3

= 0 = tan 0 i2 = 0

(4)

= i1 i2 = 90 0 = 90

(1)

Letting (x1, y1) be the point of contact of tangent


to the given curve.
2

x1 3 + y1 3 = a 3

(2)

Y
(0, b )

90
B

(a , 0)

the curve x 3 + y 3 = a 3 .

2
2

5. Find x-intercept of the tangent by putting y = 0 in


the equation of the tangent.
6. Find the y-intercept of the tangent by putting x = 0
in the equation of the tangent.
7. Use the formula of area of a triangle.

x =0
y =b

x1 , y1

P(

x1 ,

y1 )
X
A

765

Tangent and Normal to a Curve

Now, differentiating (1) w.r.t x, we get

2
x
3

2 1
3

13

x
y

2
y
3

+ y

13

2
3

13

= (x1, y1)

dy
=0
dx

1
dy
= x 3
dx

dy
y3

= 1
dx
x3

L dy O
=M P
N dx Q

(3)

dy
at (x1, y1)
dx

(b) Now, the equation of the tangent at (x1, y1) is

ay y f =
1

x x1

x13

1
3

1
3

1
3

1
3

x1 y + y1 x = x1 y1 + x1 y1
1

x13 y + y13 x = x13 y13


1

LM OP
N Q

Fx
H

2
3

+ y13

I
K

x1 y1

[from (2)]

(5)

(c) x-intercept on x-axis is determined by putting y = 0


in (5)
1
3

1
3

2
3

1
3

2
3

1
3

2
3

1
3

1
3

2
3

4
3

Question: Find the area of the triangle formed by the


x-axis the tangent and normal to the curve y (2a x)
= x2 at the point (a, a).
Solution: (a) Given equation of the curve is
y (2a x) = x2
(1)
Now, differentiating equation (1) w.r.t x, we get
dy 2 x + y
dy
=
y 0 1 + 2a x
= 2x
dx 2a x
dx
dy
Now, the value of
at (x1, y1) = (a, a) is
dx

x1 , y1

LM 2 x + y OP
N 2a x Q b

x1 , y1

1
3

2
3

1
3

of the normal at (a, a) =


Y

2
3

2a + a 3a
= =3
2a a a

= y-coordinate of B.

al
rm
No

FH

O
2

IK

Thus, the coordinates of A = x13 a a3 , 0

(x 2 , y 2)

nt

)
, y1
(x 1

ge

x13 y = x13 y13 a a3 y = y13 a a3

1
3

n
Ta

of point A. And y-intercept on y-axis is determined by


putting x = 0 in (5)
1

1
3

1
2
1
1 1
OA OB = x13 y13 a 3 .
2
2

y1 x = x1 y1 a a x = x1 a a = x-coordinate

2
3

Slope of the tangent at (a, a) = 3 and the slope

1
3

x1 y + y1 x =

1
3

1 13
x1
2

LM dy OP
N dx Q b

= x13 y13 a13


1
3

LM
N
1
= Lx
2 MN
=

a f

x13 y x13 y1 = y13 x + x1 y13

f a f a f
a FH y a 0IK + 0 b0 0g + 0 FH 0 y a IK O
PQ
1
a y a O= x y a
PQ 2

(4)

IK b

Note: Area =

y13
x13

y13

1
x1 y2 y3 + x2 y3 y1 + x 3 y1 y2
2

x = x1
y = y1

Coordinates of origin O = (0, 0) = (x3, y3)


(d) Area of OAB having the vertices (x1, y1),
(x2, y2) and (x3, y3)

dy
=0
dx

Now, the value of

FH

Coordinates of B = 0 , y13 a 3 = x2 , y 2

(x 1 , y 1)

Now, the equation of the tangent at (a, a) is


(y y1) = slope of the tangent (x x1)

How to Learn Calculus of One Variable

(y a) = 3 (x a)
(y a) = 3x 3a
y 3x = 3a + a
(2)
y = 3x 2a
Now, the equation of the normal at (a, a) is
1
(y y1) = x x1
3
1
ya = xa
3
3y 3a = x + a
(3)
x + 3y = 4a
(b) Now, for the point of intersection of the tangent
and x-axis.
2a
y = 0 in (2) 3x 2a = 0 3x = 2a x =
3
2a

, 0 = x1 , y1 = point of intersection of
3
tangent and x-axis
(4)
Again, for the point of intersection of normal and
x-axis.
y = 0 in (3) x = 4a
(4a, 0) = (x2, y2) = point of intersection of normal
and x-axis
(5)
Also we are given the point of intersection of the
tangent and normal = (a, a) = (x3, y3)
(6)
(c) Now, we are required to find out the area of the
triangle having the vertices (x 1, y1 ), (x2, y 2) and
(x3, y3).

a f

F
H

a f

I a
K

Refresh Your Memory:


Definitions: 1. Subtangent: The projection of the
tangent on x-axis is called the subtangent the
subtangent to a curve at the point of tangency of a
tangent to a curve is the portion of the x-axis
intercepted between the tangent at the point and the
ordinate through the point.
In the figure, the tangent at P to the curve show
intersects x-axis at T and F is the foot of the
perpendicular from P to the x-axis. FT is then
subtangent to the curve at P.
Y

Ta
n

Y
B
P (x1, y 1)

A
n
Ta

n
ge

i
t

1 10a

2
3

5a
3

al

a fOPQ

1
2a
a 00 +
0 a + 4a a 0
2
3

rm
No

LM a f a f
N
F 2a I + 4a OP
1 L
= M0 + G
J
2 M
N H 3 K PQ
1 L 2 a + 12 a O
PP
= M
2 M
3
Q
N

2. Subnormal: The projection of the normal on


x-axis is known as the subnormal Subnormal to
the curve at any point is the portion of the x-axis
intercepted between the normal and the ordinate
through that point.

1
Area of the = [x1 (y2 y3) + x2 (y3 y1) +
2
x3 (y1 y3)]
=

ge
n

Sub tangent

Ordinate

766

Sub tangent

Question: Find the expressions for the subtangent


and subnormal.
Solution: Let us suppose that P (x1, y1) be any point
on the curve APB whose equation is y = f (x) or f (x, y)
= 0.

Tangent and Normal to a Curve

The tangent and the normal at P meet the x-axis in


T and G respectively. Draw the ordinate from P meeting
the x-axis in N.
Let i be the angle which the tangent at P makes
with the x-axis.
Then NPG = NTP = i
and tan i =

LM dy OP
N dx Q

x = x1
y = y1

= Value of

dy
at (x1, y1)
dx

Now, from the right angled TPN subtangent


NT = NP cot i
[ Q NP = y1 and tan i =

F dy I
H dx K a

x1 , y1

y1
NT
=
dy
tan i
dx a x1 , y1 f

LM OP
N Q

LM OP
N Q

dy
and length of subnormal = y1 dx

x = x1
y = y1

= Ordinate of the given point times the value of

dy
at (x1, y1).
dx
Note: 1. When the point of contact of tangent and
curve is not provided, it is supposed (x1, y1).
2. Negative length is not considered. This is why we
take the absolute value as the length of subtangent
and subnormal to a curve at any point (if negative).
3. When the given equation of the curve is in
parametric form, it is better to use the following
notational form of the formula.

(i) Length of subtangent =

Again, from the rt right angled NPG


Subnormal = NG = NP tan i

LM dy OP
N dx Q

y
at or any parameter
given in the equation

(ii) Length of subnormal

= y1 tan i
= y1

767

LM dy OP
N dx Q

= y

x = x1
y = y1

LM dy OP
N dx Q

at or any parameter
given in the equation

Working rule to find the length of subtantgent and


subnormal for a given curve at any point (x1, y1):

(iii) At any point of the curve whose parametric


equation is given, we may suppose / t / ... , the

Steps: 1. Differentiate the given equation w.r.t x.

parameter given in the equation of the curve as


representing the given point.

LM OP
N Q

dy
2. Find dx

(iv) When numerical value of is not provided,

x = x1
y = y1

3. Use the formula for the subtangent and subnormal


Length of subtangent

y1

LM OP
N Q
dy
dx

serves as the value of

dy
dx

dy
at any point .
dx

Problems based on subtangent and subnormal.


Examples worked out:

x = x1
y = y1

Question: Find the lengths of subtangents and

Ordinate of the given point


dy
Value of
at the given point x1 , y1
dx

x
y
+
= 1 at (2, 3)
8
18
Solution: Given equation of the curve is

subnormal for the curve

768

How to Learn Calculus of One Variable


2

x
y
(1)
+
=1
8
18
Now, differentiating the equation (1) w.r.t x, we
have

x
y dy
+
=0
4 9 dx

9x
dy
=
4y
dx

LM dy OP
N dx Q

LM
N

9x
4y

OP
Q

x = x1 = 2
y = y1 = 3

18 3
=
12 2

= y1

x = x1
y = y1

= 3

3
=2
2

x = x1
y = y1

where C = curve

m
or
al

Sub tangent

Now, differentiating the equation (1) w.r.t x, we


have

y+ x

dy
dy
+2
=0
dx
dx

1
1
=
3
3

LM dy OP
N dx Q

x = x1
y = y1

a f

= 1 3 = 3

F , 3 2I
GH 2 2 JK .

x
at the point
2

Solution: Given equation of the curve is

y = 3 sin

x
2

(1)

x1
(2)
2
Now, differentiating the given equation (1) w.r.t x,
we have
dy 3
x
= cos
dx 2
2
3
3
3
dy
x

cos x = = cos =

=
x
2
2
2
2
4 2 2
dx = 2
y1 = 3 sin

LM OP
N Q

Ordinate

P (2, 1)
t

x = x1
y = y1

subnormal for the curve y = 3 sin

C x y + 2 x y 5 = 0

en

LM dy OP
N dx Q

Question: Find the lengths of subtangent and

3
9
= 3
=
2
2

= y1

= y1

Question: Find the lengths of subtangent and


subnormal for the curve xy + 2x y 5 = 0 at the point
(2, 1).
Solution: Given equation of the curves is
xy + 2x y 5 = 0
(1)

ng
Ta

2 ,1

1+2
= 3
12

and length of subnormal at (2, 1)

and length of subnormal at (2, 3)

L dy O
M P
N dx Q

Now, the length of subtangent at (2, 1)

x = x1 = 2
y = y1 = 3

LM dy OP
N dx Q

dy
x1 =0
dx

2,1

Length of subtangent at (2, 3)


= y1

f
a y + 2f = a y + 2f
dy

=
dx
a x 1f a1 xf
L dy O = LM y + 2 OP
Again, M P
N dx Q a f N 1 x Qa
y+2+

y = 3 22

LM
N

OP
Q

y = 3 22

Now, the length of subtangent at the point

F , 3 2I
GH 2 2 JK

769

Tangent and Normal to a Curve

= y1

L dy O
M P
N dx Q

x = x1
y = y1

F , 3 2I
GH 2 2 JK
= y1

= a

x = x1
y = y1

Question: Find the lengths of subtangent and


subnormal for the curve

a
f

y = a a1 cosf at the point = , a > 0.


3

x = a + sin

(2)

Now, differentiating (1) and (2) w.r.t , we have

RS d + d sin UV = a a1 + cos f
T d d W
d a1 cos f
dy
=a
and
=a

RS d a1f d cos UV = a sin


T d d W

LM dy OP
N dx Q

= 3

sin
dy dy dx
=

=
dx d d 1 + cos

LM sin OP
N1 + cos Q

= 3

3
3 2
1
2
=
=
=
=

1
2
3
3
1 + cos
1+
3
2
sin

(3)

F
H

= a 1 cos

1
3

= a

I
K

3
3

FG 2 1IJ
H2K

1
3

a
2 3

a
f
y = a a2 sin + sin 2 f

x = a 2 cos + cos 2

Solution: Given equation of the curve is

a
f
y = a a2 sin + sin 2 f

(1)
(2)

Now, differentiating (1) and (2) w.r.t , we have

a
a

f
f

dy
a 2 cos + 2 cos 2
dy
= d =
dx
dx
a 2 sin 2 sin 2
d

(3) and (4)

a 3
a 3
=
2
2

x = a 2 cos + cos 2

=a

IJ
K

3
3

Question: Find the lengths of subtangent and


subnormal at any point of the curve
(1)

dx da + sin
=
d
d

LM dy OP
N dx Q
F 1I
a 1
H 2K

= y1

Solution: Given equation of the curve is

a
f
y = a a1 cosf

FG
H

= a 1 cos

and the length of subnormal at =

3 2
3
9
=

=
2
4
2 2

x = a + sin

LM dy OP
N dx Q
F 1I
3 G1 J
H 2K

= y

and the length of the subnormal at the point

L dy O
M P
N dx Q

Now, required length of subtangent at =

3 2
3
=

=2
2
2 2

(4)

cos 2 + cos
sin 2 + sin

cos
2
2 = cot 3
=
3

2
cos
2 sin
2
2
2 cos

The length of subtangent at (x, y)


= y

F I
H K

dy
3
3
= y cot
= y tan
dx
2
2

and the length of subnormal at (x, y)

770

How to Learn Calculus of One Variable

= y

dy
3
3
= y cot
= y cot
dx
2
2

where y = a 2 sin + sin 2 which is given in the


equation of the curve.
Question: Show that subtangent at any point of the
curve xm yn = am + n varies as the abscissa.
Solution: Given equation is xm yn = am + n
(1)
Now, differentiating the given equation (1) w.r.t x,
we have m x

m 1

m 1

y + nx y

n 1

dy
=0
dx

dy
mx
y
my
= m n 1 =
dx
nx
nx y

LM dy OP
N dx Q
L my OP
= M
N nx Q

dy
= Value of
at (x1, y1)
dx

x = x1
y = y1

x = x1
y = y1

my
= 1
nx1

= y1

LM dy OP
N dx Q b

P x1 , y1

c x1

3
x1

= cx14 4cx13

x1
4

4c
The subtangent varies as the abscissa of the
point (x1, y1).
Question: Show that for the curve y2 = 4ax, the length
of the subtangent varies as the abscissa of the point
of contact.
Solution: Let (x1, y1) be the point on the curve
...(1)
y2 = 4ax
2
(2)
y1 = 4a x1
Now, differentiating (1) w.r.t. x, we have
2y

2a
dy
dy 4a
= 4a
=
=
dx
dx 2 y
y

LM dy OP
N dx Q

P x1 , y1

LM 2a OP
NyQ

P x1 , y1

2a
y1

(3)

LM dy OP
N Q

Hence, the subtangent at any point P (x1, y1)

Length of the subtangent = y1 dx

dy
= y1
dx

= y1

= y1

LM OP
N Q b g=y
L nx OP = L nx O
M
N my Q MN m PQ
1

P x1 , y1

L my OP
M
N nx Q
1

= kx1 which implies

Question: Show that for the curve y = cx4, the


subtangent varies as its abscissa.
Solution: Given equation is y = cx4
(1)

dy
3
= 4c x
dx
4

y
2a
= y1 1
2a
y1

that the length of subtangent varies as the abscissa


of the point (x1, y1) [ Q x y x = ay where a =
constant].

P x1 , y1

Again, y = cx y1 = cx1
(2)
4
[ Q (x1, y1) lies on the curve y = cx ]
The subtangent for the point (x1, y1) on the
given curve

4 a x1
y1
=
= 2 x1 [from (2)]
2a
2a
Thus, the length of subtangent at (x1, y1) on the
given curve = 2x1
Subtangent is twice the abscissa.
The subtangent varies as the abscissa.
[Q Subtangent varies as the abscissa Subtangent
= a constant abscissa of the point]
=

Question: Show that in the curve y = b e a the


subtangent at any point is of constant length. Also
show that the subnormal varies as the square or ordinate.
Solution: Let P (x1, y1) be the point of contact of the
tangent and normal to the given curve whose equation
x

is y = b e a

(1)

Tangent and Normal to a Curve

771

(2)

Summary of important facts of working rule of


different types of problems on tangent and normal:

Now, differentiating both sides of equation (1) w.r.t


x, we get

1. Equation of the tangent to the curve y = f (x) at any


point P (x1, y1) of the curve is given by

y1 = b e

x1
a

dy b a
y
= e =
dx a
a

LM dy OP
N dx Q

P x1 , y1

(y y1) =

LM y OP
Na Q

P x1 , y1

y1
a

LM dy OP
N dx Q

= y1

P x1 , y1

a
= a
y1

= y1

(3)

y1
a

LM dy OP
N dx Q

(constant)

P x1 , y1

= y1

y1
y2
= 1
a
a

which implies that subnormal varies as the square of


the ordinate.
Question: Show that for the curve y = 3x + 2 , the
subnormal is of constant length.
Solution: Let (x1, y1) be the point of contact of the
normal and the curve whose equation is

y=

3x + 2

y1 =

(1)

3x1 + 2

y = f (x) is y y1

(2)

LM dy OP
N dx Q

P x1 , y1

3
(3)

The length of subnormal


= y1

LM dy OP
N dx Qa

x1 , y1

P x1 , y1

dy
= 0.
dx
If the tangent is perpendicular to x-axis or normal
is parallel to x-axis

dx
dy
= 0 and
= op or, its reciprocal,
dy
dx

b g

we write m =
4. Angle of intersection of the two curves:
By angle of intersection of two curves, we mean the
angle between the tangents to the curves at their
common point of intersection.
Hence, if be the acute angle between the
tangents, then tan =

m1 m2
1 + m1 m2

LM dy OP = LM dy OP
N dx Q N dx Q
C1

2 3x1 + 2

x x1 .

Where m1 =

dy
3
=
dx 2 3x + 2

LM dy OP
N dx Q

x x1 .

Now, differentiating (1) w.r.t x, we get

3. Let us consider the tangent to be parallel or


perpendicular to the x-axis.
If the tangent is parallel to x-axis or normal is
perpendicular to the x-axis, then m = 0, so that

and the subnormal at the point P (x1, y1) on the curve

= y1

P x1 , y1

2. Equation of the normal at P (x1, y1) to the curve

Now the subtangent at the point P (x1, y1) on the


curve

= y1

LM dy OP
N dx Q

3
which is a constant.
2

x = x1
y = y1

= Value of

dy
dx

at the common point of intersection (x1, y1) for the


first curve.

LM dy OP
N dx Q

LM dy OP
N dx Q

dy
at the
dx
common point of intersection (x1, y1) for the second
curve.
m2 =

C2

x = x1
y = y1

= Value of

772

How to Learn Calculus of One Variable

5. Condition for orthogonal intersection:


Two curves are said to cut orthogonally if the angle
between them is a right angle i.e. = 90

i2 i1 =
2
tan i2 = cot i1

m2 =

LM dy OP
N dx Q

LM OP LM dy OP
N Q N dx Q
1

= Value of
1

LM dy OP
N dx Q

= Value of
2

(iv) y =

dy
at the common
dx

dy
at the common point
dx

of intersection for the second curve.


6. Condition for the two curves to touch:
If the two curves touch, then the angle between
them is zero, i.e., = 0 tan = 0

m1 m2 = 0 m1 = m2

LM dy OP = LM dy OP
N dx Q N dx Q
1

7. Intercepts of tangent on axis:


Find the equation of the tangent. Put y = 0 find the
value of x which will be the intercept on axis of x
known as x-intercept. Then put x = 0 and find the
value of y which will be the intercept on y-axis known
as y-intercept.
8. Condition for a given line to touch a given curve:
Let the line be a tangent to the given curve at (x, y),
then write the equation of the tangent as

aY yf = LMN dydx OPQ a X xf

4
at the point x = 0
x+2

1
x
(vi) y2 = 4x at the point x = 1
(vii) y = x2 + 2x + 3 at the point (0, 3)
(viii) y2 = 52 x2 at x = 3
1
(ix) y = x at the point where x = 1
x
(x) 3y = x2 at the point (3, 3)
(xi) y = x2 x + 1 at the point (1, 1)
(xii) 2x2 + 2y = 7 at the point where x = 2
(xiii) y2 = 9 x2 at the point where x = 2
(xiv) y = x4 4x at the point x = 0.
(v) y =

2. Find the inclination of the tangent to the curve


y = x3 x2 + 1 at the point (1, 1) on it.
3. Find the inclination to the x-axis of the tangent to

a f

the parabola y 2 = 4ax at the point , and


determine the point at which the tangent makes an
angle of 45 with the axis.
4. Find the inclination of the tangents at the point
(1, 0) and (2, 0) to the curve y = (x 1) (x 2).
Answers:
1. (i) 45 (ii) 135 (iii) [45, 135] (iv) 135
(v) 135 (vi) [45, 135] (vii) tan1 2
1

LM 3 OP
N 4Q

compare this equation of tangent with given line


ax + by + c = 0 and then eliminate x and y which will be
a relation involving the given constants only
representing the required condition for a given line to
touch a given curve.

(viii) tan

Type 1: Problems based on inclination and slopes


when x1 and / y1 is given.

(xiv) tan1 ( 4)

(A) Problems based on finding inclinations:

F 3a , 3a I
H 2 2K

(iii) y2 = 4ax at the point x = a

= 1

point of intersection for the first curve.

m2 =

1. Find the inclination of the following curves:


(i) y = x2 x + 1 at (1, 1)
(ii) x3 + y3 = 3axy at the point

1
dy

m1
dx

Where m1 =

Exercise 19.1

(ix) tan1 2 (x) tan1 2

(xi) 45 (xii) tan1 ( 4) (xiii) tan

2.

5
3. [a, 2a] 4.
2
12

FG 2 IJ
H 5K

Tangent and Normal to a Curve

Type 1: (B) Problems based on finding slopes:

773

Exercise 19.3

Exercise 19.2
1. Find the slope of the normal to the curve y = 3x2 at
the point whose x co-ordinate is 2.
2. Find the slopes of the tangents to the following
curves:

1. Find the equation of the tangent to the curve


y = x3 2x2 + x + 2 at the point (1, 2).
2. Find the equation of the tangent of the curve y = 2

(i) y = 3x2 at x = 1

.
2
3. Find the equations of the tangents to the curve

(ii) y = 2x2 1 at x = 1

y = sin x at x =

.
4
4. Find the equation of the tangent to y = 4 + cos2 x at

(iii) y = x3 + 4x at x = 1
(iv) y = x3 x at x = 2
(v) y = 2x3 + 3 sin x at x = 0
(vi) y2 = 4x at the point x = 1

.
4

2x + 5
2

x 6

F 1 I and F 2 , 1 I are parallel


at the points 3 ,
H 2K H 2K
to x-axis.
5. Find the gradients of the following curves at the
given points on them.
(i) y = x2 + 2x + 5 at the points (0, 5), (1, 8) and (2, 13)
(ii) y = 3x2 + 7x + 5 at the point where it cuts the
y-axis.
(iii) y = x2 5x + 8 at the points where it cuts the
straight line y = x.
Answers:

1
at x = 2
12
2. (i) 6 (ii) 4 (iii) 7 (iv) 11 (v) 3 (vi) Find
1.

3.

.
4
5. Find the equation of the tangent to
x=

3. Find the slope of the curve y = (1 + x) sin x at x =


4. Show that the tangents to the curve y =

sin x + sin 2x at x =

F2 + I
4K
2 H

5. (i) 2, 4, 6 (ii) 7 (iii) 1, 3


Type 2: Problems based on finding the equation of
the tangent and normal when x1 and / y1 is given:
(A) Problems based on finding the equation of the
tangent:

9
.
4
6. Find the equation of the tangent to the curve

y = sin2 x + cot2 x + 3 at x =

y = x sin x cos x at x =

.
2

7. Find the equation of the tangent at x =

to the
4

curve y = cot2 x 2 cot x + 2.


8. Find the equation of the tangent to the curve

.
3
9. Find the equation of the tangent to the curve y = ex
at (0, 1).
10. Find the equation of the tangent to the curve
y = sec4 x tan4 x at x =

.
2
11. Find the equation of the tangent of the curve:
x = a cos3 t
y = a sin3 t at the point t.
12. Find the equation of the tangent of the curve:
x = a cos
y2 sin x = 9 at x =

.
4
13. Find the equations of the tangents drawn to the
curve y2 = 2x3 4y + 8 = 0 from (1, 2).
y = a sin at =

774

How to Learn Calculus of One Variable

14. Find the equation of the tangent to the parabola


y2 = 4ax at (at2, 2at).

y=

Answers:
1. x + 2y + 1

tan ax + sin 2 x
1+ x

at x = .

7. Find the equation of the normal to the curve

2. 2 y = 3 3

3. 4 x 2 y + 1 = , 4

2x y =

2 3 2

4. y 4 = 0
5. 4 y + 12 x 27 18 = 0
6. 2 y 4 x + = 0
7. y = 1
8. 3 y 48 3x + 16 3 21 = 0
9. y = x + 1
10. y + 3 = 0
11. y a sin3 t = tan t (x a cos3 t)

x
y
+ =
a b

12.

6. Find the equation of the normal to the curve

b g

13. y 2 = 2 3 x 1
2

14. ty = x + at
Type 2: (B) Problems based on finding the equation
of the normal:
Exercise 19.4

1 + sin x

x= .
cos x at
4
8. Find the equation of the normal to the curve
y=

1
.
2
9. Find the equation of the normal to the curve
x2 = 4y which passes through the point (1, 2).
10. Find the equation of the normal at the point
(am2, am3) for the curve ay2 = x3.
11. Find the equation of the normal to the curve
y = x3 2x2 + 4 at the point whose x-coordinate is 2.
12. Find the equation of the normal to
4x2 + y2 = 2 at the point where x =

.
2
13. Find the equation of the normal to y = cos (5x + 4)
y = (sin 2x + cot x + 2)2 at x =

at x =

FG 8IJ .
H 6K

Answers:
1. 8 y 4 x + 8 = 0

2
3. x + y 1= 0
2. x =

1. Find the equation of the normal to y = cot x at

.
4
2. Find the equation of the normal to the curve
x=

y=
.
2
3. Find the normal to the curve y = sin x + cos x at
x = 0.
4. Find the equation of the normal to the curve
sin2

x at x =

y = 2 sin x at x = .
3
6
5. Find the equation of the normal to the curve

y = x + sin x cos x at x = .
2

4. x =

5. 2 x = 0

j x = e1 + j

2j y 4 3 2 = 0
4

6. 3 y + 1 +

7. x + 2 +
8. x 2 y +

3
=0
2

9. x + y = 3
10. 2 x + 2my = 23am2 + 3am4
11. 4y + 8x = 18

775

Tangent and Normal to a Curve

12. y 4 =

F
H

I
K

24 y 2 x + 26 = 0
x
12
2

13. 18 y 6x + 8 = 0
Type 2: (C) Problems based on finding the equation
of the tangent and normal simultaneously:
Exercise 19.5
1. Find the equations of the tangent and normal to
the curve y = x3 + 2x + 6 at (2, 18).
2. Find the equations of the tangent and normal to
the curve 16x2 + 4y2 = 144 at (x1, y1) where x1 = 2 and
y1 > 0.
3. Find the equations of the tangent and normal to
the curve y = x2 4x 5 at x = 2.
4. Find the equations of the tangent and normal to
the curve y = x2 4x + 2 at (4, 2).
5. Find the equations of the tangent and normal to
the curve:
x = at2
y = 2at at the point t.
6. Find the equations of the tangent and normal at

2x + 1
.
3 x

the point (2, 5) of the curve y =

Type 3: Problems based on intercepts of tangents


on the axis:
Exercise 19.6
1. Show that the sum of the intercepts of the tangent
1

at any point to the curve x 2 + y 2 = a 2 on the axes is


constant and is equal to a.
2. Prove that the length intercepted by the coordinate
2

x 3 + y 3 = a 3 which is intercepted between the axes


is of constant length.
7. Find the equation of the tangent to the curve
x 3 + y 3 = a 3 and show that the portion of the
tangent intercepted between the coordinate axes is
constant.
8. Find the normal to the curve xy = a + x which
makes equal intercepts upon coordinate axes.
9. The tangent at any point on the curve x3 + y3 = 2a3
cuts off lengths p and q on the coordinate axes, show
2

7. Find the equations of the tangent and normal to


the curve y = x2 3x + 4 at the point where it cuts the
y-axis.
8. Find the equations of the tangent and normal to
the curve 3y = x2 6x + 17 at (4, 3).

3. If the tangent to the curve x + y = a at any


point on it cuts the x and y-axes respectively at A and
B, prove that OA + OB = a.
4. In the curve xm yn = am + n, prove that the portion of
the tangent intercepted between the axes is divided
at its point of contact into segments which are in a
constant ratio.
5. Find the intercepts made upon the axes by the
tangent at (x1, y1) to the curve x + y = a and
show that their sum is constant.
6. Show that the portion of the tangent to the curve

axes on any tangent to the curve x 3 + y 3 = c 3 is


constant.

that p 3 + a 2 = 2 2 a 2 .
10. If p and q be the intercepts on the coordinate

F xI + F yI
H aK H bK
n

axes by the tangent to

FaI
that G J
H pK

n
n 1

FbI
+G J
HqK

n
n 1

= 1 , then show

= 1.

LM
N

OP
Q

x
C 2x
e + e 2 the
2
length of the perpendicular, from the foot of the
ordinate of a point p on the curve, upon the tangent
at p is constant.
12. Find the equation of the tangent at the point
determined by on the ellipse x = acos ,
y = b sin . Also find the length of the portion of the
tangent intercepted between the coordinate axes.
13. Prove that the portion of the tangent of the curve
x = a cos3 , y = a sin3 at the point , intercepted
between the coordinate axes is of constant length.

11. Prove that, in the curve y =

776

How to Learn Calculus of One Variable

Answers:
8. Normal at the point

FG a , 3a + 2aIJ if
H 2 2 K

x y + a 2+ 2 = 0
12.

x cos y sin
2
2
2
2
+
= 1 , a sec + b cosec
a
b

Type 4: Finding the points where the tangent


Exercise 19.7
1. Find the points on the curve x2 y2 = 2 at which
the slope of the tangent is 2.
2. Find at what points on the circle x2 + y2 = 13, the
tangent is parallel to the line 2x + 3y = 7.
3. At what points on the curve x2 + y2 2x 4y + 1
= 0, the tangent is parallel to (i) x-axis (ii) y-axis.
4. Find the point on the curve y = x3 + 2x2 3x +1, the
tangent at which is parallel to the line 4x y = 3.
5. Find the coordinates of the points on the curve

7
.
9
6. At what point does the curve y = x2 4x have the
slope 2?
7. Find the coordinates of the points on the curve
5 = log (x2 + 3) at which the slope is 2.
8. Find the point on the curve y = 4 + x2 at which the
tangent is horizontal.
9. Find the point at which the tangent to the curve x2
+ y2 + 2x 4y = 20 is parallel to the x-axis.
10. Find the point at which the tangent to the curve
y = x3 12x + 10 is parallel to x-axis.
11. Find the point on the curve y = x3 x2 x + 3
where the tangent is perpendicular to the y-axis.
2x2 + 3xy + 4y2 = 9 at which the slope is

+ 2 = 1 is the
2
a
b
tangent perpendicular to the x-axis.
13. At what points on the curve y = x2 + 2x is the
tangent
(i) parallel to the x-axis
(ii) equally inclined to the axes
(iii) inclined at 30 to the x-axis.

12. At what point on the curve

14. Find the points at which the tangent is parallel to


the axis of x for the curve y = x3 6x2 15x + 5.
15. Find the coordinates of the point at which the
tangent to the curve xy + 4 = 0 make an angle of 45
with the axis of x.
16. Find the points on the curve x2 y2 = 2 at which
the slope of the tangent is 2.
17. At what point on the curve y = 2x2 x + 1 is the
tangent parallel to the line y = 3x + 9.
18. (i) Find at what points on the circle x2 + y2 = 13,
the tangent is parallel to the line 2x + 3y = 0.
(ii) Find the points on the curve y = cos (x + y),
2 x 2 at which the tangents are parallel to
the line x + 2y = x + 2y = 0.
3

x
y
+
= xy ,
a
b
the tangent is parallel to one of the coordinate axes.
20. Find the points on the curve y = x3 2x2 + x 2,
where the gradient is zero.
21. (i) Find the point on the curve y = 4 + x2 at which
the tangent is horizontal.
(ii) Find the points on the curve y = x3 x2 x + 3
where the tangent is perpendicular to the y-axis.
22. (i) Find the points at which the tangent is parallel
to the axis of x for the curve y = x3 6x2 15x + 5.
(ii) Find the points on the curve y = x3, the tangents
at which cut the x-axis at an angle of 60.
23. (i) Find the point on the curve y2 = 4ax, the
tangent at which is inclined at 45 to the x-axis.
(ii) At what point on the curve y = 2x2 x + 1 is the
tangent parallel to the line y = 3x + 9.
(iii) Find at what points on the circle x2 + y2 = 13, the
tangent is parallel to the line 2x + 3y = 0.
(iv) Find the points on the curve 4x2 + 9y2 = 1 where
the tangents are perpendicular to the line 2y + x = 0.

19. Find at what points on the curve

Answers:
1.

F 2 2 , 2 I , F 2 2 , 2 I
GH 3 3 JK GH 3 3 JK

2. (2, 3) and (2, 3)


3. (3, 2) and (1, 2)
4.

F 7 , 247 I
H 3 27 K

Tangent and Normal to a Curve

5. (1, 1) and (1, 1)

23. (i) (a, 2a) (ii) (1, 2)

6. (3, 3)
7.

RS5 + 3 , 5 log 5 e5 + 13 jUV ,


2
T 2
W
RS5 3 , 5 log 5 e5 13 jUV
2
T 2
W

8. (0, 4)
9. (1, 7) and (1, 3)
10. (2, 6)and (2, 26)
11. (1, 2) and

F 1 , 2 16 I
H 3 27 K

14. (1, 13) and (5, 95)


15. (2, 2) and (2, 2)

F 2 3 , 2 I and F 2 3 , 2 I
GH 3 3 JK GH 3 3 JK

17. (1, 2)
18. (i) (2, 3) and (2, 3)
3

(ii)
, 0 and
,0
2
2

F
I F I
H
K H K
R1
U
1
19. S e2a bj , e2a bj V and
3
3
T
W
RS 1 e2ab j , 1 e2ab j UV
3
T3b
W
1

50
F , I
20. a1 , 2f and
H 3 27 K
1
3

2
3

Type 5: (A) Problems based on finding angles of


intersection or angle between two curves:
Exercise 19.8

3. Find the angle between the curves:


(i) y2 = x and x2 = y

F 1 3I
(ii) ,
H 2 4K
F 2 3 1 , 16 3 9 I
(iii) G
12 JK
H 2 3

F 2 , 1 I
GH 2 10 3 10 JK

1. Find the angle of intersection of the curves y = x2


and y = x3.
2. Find the angle of intersection of the curves
y = 4 x2 and y = x2.

12. (a, 0) and ( a, 0)


13. (i) (1, 1)

16.

(iii) (2, 3) and (2, 3) (iv)

777

2
3

2
3

(ii) y = x2 and y = 4 x2
(iii) xy = 4 and x2 + y2 = 8
(iv) x2 y2 = 2a2 and x2 + y2 = 4a2
(v) y = 6 x2 and x2 = 4y at the point (2, 2).
4. (i) At what angle the parabolas y2 = x and x2 = 8y
cut each other.
(ii) Find the angle at which the curves y = sin x and
y = cos x intersect.
5. Find the angle of intersection of the parabolas
y2 = 2x and the circle x2 + y2 = 8.
6. Find the angle between the line y = x and the curve
2y = 7x 5x2.
7. Find the angle between the curve y = x3 and the
straight line y = 9x at each of their point of intersection.
8. Find the angle between the curves:
(i) x2 + y2 = 5 and y2 = 4x + 8
(ii) xy = 2a2 and y2 = 4ax
9. Show that the curves y =

x+3
2

x +1

and

21. (i) (0, 4) (ii) (1, 2)


22. (i) (1, 13) and (5, 95)

x 7 x + 11
cut each other at the point (2, 1)
x 1
at an angle 45.
10. Show that the curves y = 2 sin2 x and y = cos 2x

(ii)

intersect at x =

F3 1 , 3 2I
H 4 3K

y=

. Find the angle of intersection.


6

778

How to Learn Calculus of One Variable

Answers:
1. tan
2. tan

F 1 I and 0
H 7K
F4 2I
GH 7 JK

3. (i) 90 and tan

5. Find the angle between the tangents to the curve


x

F 3I
H 4K

(iii) 0 (iv) 60 (v) tan

F 3I and 90
4. (i) tan
H 5K
(ii) tan e2 2 j

(ii) tan

F 7I
H 11K

F4 2I
GH 7 JK

= 1 at the points (a, 0) and (0, b).


2
2
a
b
6. Show that the tangents to the curve y2 = 2ax at the
1
points where x = are at right angles.
2
Type 6: (A) Problems based on finding the condition
for two curves for orthogonal intersection.

(B) Problems based on showing for two curves to


cut orthogonally.
(A) Firstly we set the problems on finding the
conditions for two curves for orthogonal intersection.

Exercise 19.10

5. tan1 (3) at both common points (2, 2) and (2, 2)

FG 5 IJ and at (1, 1),


H 9K

6. at (0, 0), = tan 1


= tan
7. Find

8. (i) tan

a5f
1

F 1I
H 3K

1. Find the condition that the curves


2

10. Angle of intersection = 60.


Type 5: (B) Problems based on finding the angle
between two tangents to a curve at two given points:

x
y
+
= 1 cut orthogonally.

2. Find the condition in order that the curves


and

(ii) tan1 (3)

x
y
+
=1
a
b

= 1 and

= 1 should intersect at
2
2
2
2
a
b
a1 b1
right angles.
3. Find the conditions that ax2 + by2 = 1 and
a1 x2 + b1 y2 = 1 may cut right angles.

Type 6: (B) Problems based on showing for two


curves to cut orthogonally.

Exercise 19.9
Exercise 19.11
Prove that the tangents to the curve y2 = 4ax

1.
at the
points where x = a are perpendicular to each other.
2. Prove that the tangents to the curve y2 = 2x at the
1
points where x = are at right angles.
2
3. Find the angle between the tangents to the curve

F
H

I
K

F I
H K

3
5
and 4 ,
.
4
4
4. Prove that the tangents to the curve y2 = x at the
x2 = 8y + 6 at the points 0 ,

points

F 1 , 1 I and F 1 , 1 I are at right angles.


H 4 2K H 4 2K

1. Do the curves x2 + y2 = 2a2 and 2y2 x2 = a2 cut


each other orthogonally?
2. Show that the curves x3 3xy2 + 2 = 0 and
3x2 y y2 = 2 cut orthogonally.
[Hint: Show that m1 m2 = 1 at any point (x1 y1)]
3. Show that the curves x2 y2 = 16 and xy = 25 cut
each other at right angles.
4. Prove that the curves 2y2 = x3 and y2 = 32x cut each
other at right angle at the origin.
5. Show that the curves x2 + 4y2 = 8 and x2 2y2 = 4
intersect orthogonally.

779

Tangent and Normal to a Curve

y
+
= 1 and y3 = 16x intersect at
2
4
a
4
2
right angles. Show that a = .
3
3
7. Show that the curves x 3xy2 = a and 3x2 y y2
= b cut orthogonally.
8. Prove that the curves y2 = 4x and x2 + y2 6x + 1
= 0 touch each other at the point (1, 2).
x y
9. Prove that for all values of n, the line + = 2
a
b
n
n
x
y
+
= 2 at any point
touches the curve
a
b
(a, b).

6. If the curves

F I F I
H K H K

Type 6: (C) Problems based on condition for two


curves to touch.

2. Prove that the condition that x cos + y sin = p


should touch the curve:

b g

a cos

m
m 1

+ b sin

x
a

m
m1

m
m

y
b

= p

= 1 is

m
m1

3. Prove that the condition that


x cos + y sin = p
should touch the curve xm yn
= am+n is
p

m+ n

m =n m+n

m+n

m+ n

cos sin .

4. If x cos + y sin = p touches the curve

F xI
H aK

n
n 1

F yI
+
H bK

n
n 1

f a
n

= 1 , show that a cos + b sin

Exercise 19.12

= p .

F xI + F yI
H aK H bK
n

1. Prove that the curves y = 6 + x x2 and y (x 2) =


x + 2 touch each other at (2, 4). Also find the equation
of the common tangent.
2. Prove that the curves xy = 4 and x2 + y2 = 8 touch
each other.
x y
3. Show that + = 1 touches the curves
a b
x

y = b e a at the point where the curve crosses the


axis of y.
4. Prove that the curves y2 = 4x and
x2 + y2 6x + 1 = 0 touch each other at the point (1, 2).
5. Prove that the curves y = eax and y = eax touch at
the points for which bx = 2n +

.
2

5. Show that the curve


the straight line

= 2 touches

x y
+ = 2 at the point (a, b) for all
a b

values of n.
6. Prove that the straight line y = 2x 1 touches the
3

curve y = x x + 1 .
7. Prove that the straight line y = 2x 1 touches the
4

curve y = x + 2 x 3x 2 x + 3 at two distinct


points.
n
n
x
y
+
= 2 touches
8. Prove that the curve
a
b
x y
the straight line + = 2 at the point (a, b) for all
a b
value of n.

F I F I
H K H K

Type 7: Problems based on condition for a given line


to touch a given curve.

Type 8: (A) Problems based on length of


perpendicular:

Exercise 19.13

Exercise 19.14

x y
x
+ = 1 touches the curve y = b e a
a b
at the point where the curve crosses the y-axis.
[Hint: Show that equation of the tangent at
x y
0 , b = + = 1 ].
a b
1. Prove that

1. Find the length of the perpendicular from the origin


(0, 0) on the tangent of the following curve:

a f

(i)

x
a

2
2

y
b

2
2

= 1 at the point (x1, y1)

780

How to Learn Calculus of One Variable

(ii) y2 = 4ax at (am2, 2am)


(iii) x =

at2, y =

2. Find the lengths of subtangents and subnormals

2
2. Show that the normal at any point on the curve
x = a cos + a sin

(iv) x = a sin , y = a 1 cos at =

y = a sin a cos is at a constant distance


from the origin.
3. Prove that the perpendicular drawn from the foot
of the ordinate to the tangent of a curve is
y

F I
H K

dy
1+
dx

a2 and

y = a 1 cos

(ii)

4. Prove that in the curve y = b e

[Hint: Prove that

ax

, the subtangent

subtangent

aabscissaf

= constant]

5. Prove that the subtangent is of constant length in


the curves:
(i) log y = x log a (ii) y = ax.
6. Show that the subtangents and subnormals of the
2

y
.
nx
7. Find the subtangent and subnormal to the curve
y = 2x2 + 3x at the point (2, 14).
8. Find the lengths of subtangents, subnormal,
tangent and normal to the curve y = x3 at the point
(1, 1).

curve y n = a n1 x are nx and

Answers:
2

a b

(iii)

y2

varies as the square of the abscissa.

dy
dy
X
Y + y x
= 0 , find the length of the perdx
dx
pendicular from the foot of the ordinate (x, 0) to the
tangent]

am
4

b x1 + a y1
at

=1
2
2
a
b
3. Find the length of subtangent, subnormal, tangent
and normal at the point t of the cycloid:
x = a (t + sin t)

(i)

x2

[Hint: The equation of the tangent at P (x, y) is

1. (i)

at the point x , y of the curves:

2at at 't'

(ii)

1+ t

(iv)

1+ m

a 4a
2 2

Type 8: (B) Problems based on finding the area


Exercise 19.15
1. Show that the area of the triangle formed by a
tangent to the curve 2xy = a2 and the coordinate axes
is constant.
Type 9: Problems based on length of subtangent or
subnormal

Answers:
1. 3 ,

16
3

2. (i)

y
, x
x

(ii)

x2 a 2
b2 x
; 2
x
a

t
t
2a sin 2
t
2 , 2a sin ,
2
3. a sin t ,
t
t
2
cos
cos
2
2
2a sin

Exercise 19.16

7.

14
, 154
11

1. Find the lengths of the subtangent and subnormal


at the point (3, 4) of the rectangle hyperbola xy = 12.

8.

1
10
and 3;
and 10
3
3

Rolle's Theorem and Lagrange's Mean Value Theorem

781

20
Rolle's Theorem and
Lagrange's Mean Value Theorem

Rolle's Theorem
Statement: If a function y = f (x) defined over [a, b] is
such that
(i) It is continuous over [a, b]
(ii) It is differentiable over (a, b)
(iii) f (a) = f (b)
then there exists at least one point x = c a , b
such that f c = 0 .
Proof: Given:
y = f (x) is continuous in the closed interval [a, b]
graph of y = f (x) is a continuous curve without
any break from the point x = a to the point x = b.
Again, y = f (x) is differentiable in the open interval
(a, b). graph of y = f (x) has unique tangent at
each point in open interval (a, b).
Further given f (a) = f (b)
(ordinary at x = a) = (ordinate at x = b)
Now two possibilities arise.

a f

af

Case 1: When y = f (x) is constant.


Let us suppose that f (x) = k, and c a , b

a f

FG f ac hf f acf IJ , h > 0
H h K
F k k IJ = 0
= lim G
H h K
F f ac + hf f acf IJ
R f a cf = lim G
H
K
h
af

FG k k IJ = 0
H h K
L f a cf = R f a cf = 0 f a cf = 0
c b a , bg
= lim

h 0

y
(a , f (a))
A

(b, f (b))
B

f ( a)

f ( b)

Case 2: Let f (x) be not a constant function. Since in


[a, b] f (x) is continuous it attains its bounds in [a, b].
At least one of the bounds is different from f (a) =
f (b). For definiteness let the upper bound f a .

bg

y
N

L f c = lim

h0

f (a)

M
f ( c)

f ( b)

h 0

h 0

for all

A
0

x=a

C
x=c

B
x=b x

782

How to Learn Calculus of One Variable

y
L

M
N
f ( c)

f ( a)
A

x=a

f ( b)

x=c

x=b x

af

af
Now, f bc + hg f bcg f bc + hg f bcg 0
f bc + hg f bcg

0 b3 h > 0g
f x f c , x a , b and a < c < b

lim

g b g0

f c+h f c

h0

af

af

Let the function f (x) attains its upper bound


(maximum) at x = c.

2. The three conditions of Rolles theorem are


sufficient but not necessary for f x = 0 for some
x in (a, b).
3. If a function y = f (x) defined over [a, b] does not
satisfy even one of the three conditions, then Rolles
theorem fails, i.e. there may or may not exist point
where f x = 0 .

(i)
bg
Again, f bc hg f bcg f bc hg f bcg 0
f bc hg f bcg

0 b3 h > 0g
h
F f bc hg f bcg I 0
lim G
H h JK
a3 h > 0 h < 0f
L f b cg 0
(ii)

Rfc 0

Geometrical Meaning of Rolles Theorem


If the graph of a function y = f (x) defined over [a, b] is
such that
1. It is a continuous curve without any break from a
point A (a, f (a)) to another point B (b, f (b)).
2. It has a unique tangent at each point in between
the two points A (a, f (a)) and B (b, f (b)) (i.e. f (x) is
differentiable in the open interval (a, b)).
3. It has equal ordinates f (a) and f (b) at two points A
(a, f (a)) and B (b, f (b)) (i.e. f (x) assumes equal values
at the end points of the closed interval [a, b ]), then
Rolles theorem provides that there is at least one
point C (c, f (c)) between A (a, f (a) and B (b, f (b)) on
the graph of the function y = f (x) defined over [a, b]
such that the tangent to the graph at C is parallel to
the x-axis.
y
C (c, f (c))
(a , f (a))

h 0

As f (x) is differentiable over (a, b), hence f ' (c)


must exist.
Lf c = Rf c = f c
From (i) and (ii), we get
f c < 0 and f c > 0 .

af

af af
af
af
This means that f acf = 0 . A similar argument
can be used if the lower bound f ba g .
Remarks: 1. Converse of Rolles theorem is not true,
i.e. f a x f may vanish (zero) at a point within (a, b)
without satisfying all the three conditions of Rolles
theorem.

f ( a)

B (b, f (b))

f ( c)

f (b )

A1 (a, 0) C1 (c, 0)

f ( a) = f ( b)

B1 (b, 0)

Note: It is Rolles theorem which helps us to prove


Lagranges mean value theorem.
Lagranges Mean Value Theorem
Statement: If a function y = f (x) defined over a closed
interval [a, b] is such that
1. It is continuous in the closed interval [a, b].
2. It is differentiable in the open interval (a, b) then
there exists at least one point x = c a , b such

af

that f c =

af

af

f b f a
ba

a f

Rolle's Theorem and Lagrange's Mean Value Theorem

Proof: Given: y = f (x) is a continuous function over


[a, b] and differentiable over (a, b).

a f

To prove: There is at least one point x = c a , b

af

such that f c =

af

af

f b f a
ba

Main Proof: Let us consider a function defined as


F (x) = f (x) + Ax,
(1)
Where A is a constant whose value is to be
determined from the condition that is F (a) = F (b)
(2) imposed on (1)
Now from (1),
F (a) = f (a) + Aa
F (b) = f (b) + Ab
f
(a) + Aa = f (b) + Ab (on using (2))
f (b) f (a) = A (b a)

A=

af

af

f b f a
ba

(3)

Again, it is given that y = f (x) is continuous over


[a, b] and differentiable in (a, b) and Ax being a
polynomial is continuous over [a, b] and differentiable
over (a, b) since every polynomial in x is always
continuous as well as differentiable in ,
F (x) = f (x) + Ax is continuous in [a, b] and
differentiable in (a, b).
Also, from (2), F (a) = F (b)
Therefore F (x) = f (x) + Ax is
(i) Continuous is [a, b]
(ii) Differentiable in (a, b) and also
(iii) F (a) = F (b)
Thus, F (x) satisfies all the three conditions of
Rolles theorem there exists at least one value

af

a f
f acf + A = 0
A = f a cf

x = c a , b such that F c = 0 which

(4)

From (3) and (4), it is concluded that

af

f c =

af

af

f b f a
ba

Hence, the required is proved.


Remarks: 1. The statement there exists at least one
point x = c a , b means that the point c is not

a f

783

unique, i.e., there may exist more than one point c as


c1 and c2.
2. If a function y = f (x) defined over [a, b] does not
satisfy even one of the two conditions, then
lagranges mean value theorem fails for y = f (x), i.e.
there may or may not exists points where

af

f c =

af

af

f b f a
.
ba

On another form of Lagranges mean value theorem


On taking b = a + h, the closed interval [a, b] becomes
equal to [a, a +h] and the number c which lies in
between a and a + h can be written as c = a + h ,
where is some proper fraction lying in (0, 1).
Thus, the result of Lagranges mean value theorem
becomes

f a +h =

a f

af

a f

af

f a +h f a
f a +h f a
=
,
a +ha
h

where 0 < < 1 .


Therefore, the Lagranges mean value theorem can
be stated as under also:
If a function y = f (x) defined over a closed interval
[a, a + h] is such that
1. It is continuous over [a, a + h]
2. It is differentiable in (a, a + h),
then there exists at least one number 0 , 1 such
that f a + h f a = h f a + h

a f

af

a f

On geometrical meaning of Lagranges mean value


theorem
The hypothesis of Lagranges mean value theorem
provides that the graph of a function y = f (x) defined
over [a, b] is
1. A continuous curve without any break, gap or
jump from the point A (a, f (a)) to an other point B (b,
f (b)) and
2. Has a unique tangent at each point in between the
two points A (a, f (a)) and B (b, f (b)) (i.e f (x) is
differentiable in the open interval (a, b)).
The result of the Lagranges mean value theorem
provides that there is at least one point C (c, f (c)) on
the curve (i.e on the graph of the function y = f (x)
defined over [a, b]) where the tangent is parallel to

784

How to Learn Calculus of One Variable

the chord through the points A (a, f (a)) and B (b, f (b))
because the slope of the chord AB =

af

af

C3

t
Tangen
C1

f b f a
ba

B
C hord

C2

Tangent

difference of ordinates
and the slope of the
difference of abscissas

bg

f ( a)

f ( b)

tangent at any point C (c, f (c)) is f c


y

y = f (x)
A
(a , f (a))

at C
ent

A1=
(a, 0)

C1=
(c, 0)

af

B1= x
(b, 0)

af

f b f a
BD
=
AD
ba

tan =

af

On continuity and differentiability of a function


y = f (x)
Readers should remember the following facts to
ensure the continuity and differentiability of a function
y = f (x) defined over an interval open or closed.

[Let y = f (x) be the curve being continuous from A


(a, f (a)) to B (b, f (b)) and also possessing tangents to
the curve between A and B.
Let AA1 and BB1 be the perpendicular drawn to
the x-axis. Let us join the chord AB which makes an
angle with the positive direction of the x-axis.
Again from A, a perpendicular AD on BB1 is drawn.
Then, BD = BB1 DB1 = f (b) f (a)
and AD = A1 B1 = OB1 OA1 = b a

(b , f (b))

(c , f (c))

g
Ta n

af

f b f a
, where = BAD =
ba

slope of the chord AB.

bg

But f c = slope of the tangent at c.


That is, there is a point C (c, f (c)) where the
derivative has f c = tan , i.e. the tangent at C (c,
f (c)) is parallel to the chord AB.]

af

Note: There may be more than one point namely C1,


C2 and C3 on the curve between A and B where the
tangents are parallel to the chord AB.

af

1. The domain of a derived function f x is a subset


of the domain of the function f (x), because it contains
all the points x in the domain of f (x) such that the limit

lim

h 0

f af

f x+h f x
exists, but does not contain
h

those points where the limit lim

h 0

bg

f x+h f x
h

does not exist.


2. A function y = f (x) which has a derivative is called
differentiable. The function y = f (x) is differentiable at
a point x = a, if a lies in the domain of f x i.e. if
f a exists, i.e.,

af

af

af

L f a = lim

af

h 0

af

f ah f a
h

f af

f a+h f a
, h > 0.
h
3. A function y = f (x) is continuous in an open interval
(a, b) it is continuous at any point c a , b .
4. A function y = f (x) is continuous is a closed interval
[a, b] it is continuous at any point c a , b and
is continuous at a from the right and continuous at
b from the left.
5. A function y = f (x) is differentiable in an open
interval (a, b) it is differentiable at any point
c a , b .
= R f a = lim

h0

a f
a f

a f

Rolle's Theorem and Lagrange's Mean Value Theorem

6. A function y = f (x) is differentiable in a closed


interval [a, b] it is differentiable at any point
c a , b and has a right derivative at x = a and a left
derivative at x = b.
7. All the discontinuities of a function y = f (x) are

a f

af

also the discontinuities of the derived function f x .


8. All standard functions (i.e. as simple form in which
a function is commonly written, also termed as
elementary functions) can be discontinuous only at
points where they are not defined, i.e. all LIATE
(logarithmic, inverse, trigonometric, algebraic and
exponential) standard functions are continuous and
differentiable in whole of its domain.
9. A function y = f (x) is discontinuous at a point
x = c the function y = f (x) is not differentiable
(non differentiable) at the point x = c.
10. A function y = f (x) is differentiable at a point
x = c the function y = f (x) is continuous at the
point x = c.

af

11. f x is continuous f (x) is continuous.


12. Every positive power function y = x n is
continuous and differentiable in any interval open or
closed since it is continuous and differentiable for all
values of x in R.
13. Any polynomial function is continuous and
differentiable in any interval open or closed since it is
continuous and differentiable for all values of x in R.
14. Any rational algebraic or non algebraic function
is continuous and differentiable for all values of the
independent variable x excepting those point where
its denominator is zero, i.e. any rational function is
continuous and differentiable in any interval open or
closed excluding the points where its denominator is
zero.
Where to check the continuity and differentiability
of a function y = f (x) defined in a given closed interval
[a, b].
In order to check the continuity and differentiability
of a given function y = f (x) defined in a closed interval
[a, b] one must check them at the following points.
1. The points where the given function is undefined
or imaginary.

af

2. The point where the derived function f x is


undefined or imaginary.

785

3. The common points of adjacent intervals where


different forms of a given function are defined.
4. The end points of a given closed interval.
Notes: 1. In order to show that a function is
discontinuous in a given interval open or closed, it is
sufficient to show that it is discontinuous at atleast
at one point belonging to the given interval.
2. If f x becomes undefined on putting x = c,
then it is wrong to conclude that f (x) is not
differentiable at x = c. In that case we find L f c
and R f c by first principles and test the
differentiability at c.

bg

bg

bg

Illustrations: (Erroneous approach)


1. f (x) = | x |

af

f x =

af
f a 0f

f 0 =

x
x

af

0
, i.e. f 0 is undefined
0

does not exist, i.e. f (x) is not


differentiable at x = 0.

af

2. f x = x 3

af

f x =

2 c 23 1h 2 13 2 1
= x =
x
3
3
3
x

a f 20 , i.e. f a0f is not a finite number


f a 0f does not exist, i.e. f (x) is not differen f 0 =

tiable at x = 0.
Type 1: To establish the validity of Rolles theorem
when interval is given.
To verify Rolles theorem, we have to show
following conditions are satisfied.
1. Find f (a) and f (b) and show that f (a) = f (b).
2. Show the continuity of the given function in the
closed interval by using the facts that LIATE functions
are continuous at points where they have finite values
and theorems on continuity.
3. Show that differentiability of the given function in
the given interval by using theorems on
differentiability and the facts that LIATE functions
have finite derivatives at points where they are

786

How to Learn Calculus of One Variable

defined.
4. Put f x = 0 and find the value of x or choose
the value of x from among roots of f x = 0 which
is in the given interval [a, b].

af

af

N.B.:
1. LIATE L = log function
I = Inverse circular function
A = Algebraic function
T = Trigonometric function
E = Exponential function
2. If a function is discontinuous in an interval, it must
have atleast one point of discontinuity (in the given
interval) where one or other condition for continuity
fails to satisfy. A function is not continuous if it
exhibits at least one point of discontinuity in the given
interval.
Note: That this (2) is practically fruitful to examine
the validity or applicability of Rolle;s theorem or
Lagranges mean value theorem for a given function
in a given interval.
Examples worked out:
1. Verify Rolles theorem for f (x) = x3 4x in the
interval 2 x 2 .
Solution: (1) 3 f (x) = x3 4x = a polynomial
f (x) is differentiable in [2, 2] and so continuous
in [2, 2] as f (x) is a polynomial function.

af
af
a f a f a f
3

(2)

U|
V|
W

af

a f

f 2 = 2 4 2 = 0
f 2 = f 2
3
f 2 = 2 4 2 = 0

(1) and (2) all the conditions of Rolles theorem


are satisfied.

af

3x = 4
4
2
=
=c
3
3
Both the values of c lies in the open interval (2, 2)
= ]2, 2[ hence, the fact that f x = 0 for atleast one
c 2 , 2 has been verified.
Rolles theorem is verified.
2. Verify Rolles theorem for the function
x
f x = x 1 x 4 e in the interval (1, 4).
x=

af a fa f

= 0 e

ex

x 5x + 4 e

7x + 9 = 0

x 7x + 9 = 0

7 13
2
x = 5.3, 1.7 approximately = c1, c2.

x=

b g

Since c2 1 , 4 ,
Rolles theorem is verified.

af e

3. Verify Rolles theorem for f x = x 4 x + 3 e

2x

in [1, 3].

af e

Solution: (1) 3 f x = x 4 x + 3 e

2x

f (x) is differentiable in [1, 3] as it is the product


of differentiable functions (x2 4x + 3) and e2x.
It is differentiable in (1, 3) which it is
continuous in (1, 3).

af a f
(2)
f a 3f = a9 12 + 3f e

U| f a1f = f a3f
V|
= 0W

f 1 = 1 4 + 3 e = 0

3x 4 = 0

af

Now, f x = 0 2 x 5 e

Now f x = 0

af a fa f

Solution: (1) 3 f x = x 1 x 4 e
f (x) is differentiable in [1, 4] as it is the product
of differentiable functions (x 1), (x 4) and ex.
It is differentiable in (1, 4) which it is
continuous in (1, 4)
(2) f (1) = 0 = f (4)
(1) and (2) all the conditions of Rolles theorem
are satisfied.

af

(1) and (2) all the condition of Rolles theorem


are satisfied.

bg

Now, f x = 0

x 2 4x + 3 2 e2 x + 2x 4 e2 x

= 0 2 x 3x + 1 e
2

x 3x + 1 = 0

2x

=0

Rolle's Theorem and Lagrange's Mean Value Theorem

x=

94
3 5
=
2
2

and x =

3+ 5
Since the value of x =
open interval
2
(1, 3)
Rolles theorem is verified.
4. Show that the function f (x) = ex cos x satisfies

LM , OP
N 2 2Q
Solution: (1) 3 f a x f = e cos x
L O
f (x) is differentiable in M , P as it is the
N 2 2Q
Rolles theorem in

product of differentiable functions ex and cos x which

LM
N

it is continuous in

F
H

It is differentiable in

F
H

I
K

I
K


,
which it is
2 2

F I = e cos F I = e
H 2K
H 2K

F I = e cos F I = 0
f
H 2K
H 2K
2

F I = f F I
H 2K H 2K

cos

F I = 0U|
H 2K V
||
W

(1) and (2) all the conditions of Rolles theorem


are satisfied.

bg

Now, f x = 0 e x cos x sin x = 0 cos x

sin x = 0 3 e x 0

cos x = sin x tan x = 1


tan x = tan

x = n + ;
4
4

an = 0 , 1 , 2 , ...f

IJ
K

Rolles theorem is verified.

5. Verify Rolles theorem for


(a) x2 in [1, 1]
(b) x2 x 6 in [2, 3]
Solution: (a) (1) f (x) = x2 = a polynomial in x
f (x) is differentiable in [2, 3] and so continuous
in [2, 3] as f (x) is a polynomial function.
(2) f (1) = 1 = f (1)
(1) and (2) all the conditions of Rolles theorem
are satisfied.

af

Now f x = 0 2 x = 0 x = 0 which is a
point in (1, 1)
Rolles theorem is verified.
(b) (1) f (x) = x2 x 6
f (x) is differentiable in [2, 3] and so continuous
in [2, 3] as f (x) is a polynomial function.
(2) f (2) = f (3) = 0
(1) and (2) all the conditions of Rolles theorem
are satisfied.

af

Now f x = 0 2 x 1 = 0 x =


,
.
2 2

continuous in

(2)

OP
Q


,
.
2 2

FG
H


,
4
2 2

787

1
2 , 3
2

Rolles theorem is verified.


6. Verify Rolles theorem for the following functions:

b g b g b x bg

(i) f x = x a

on [a, b] where m, n

are positive integers.

af

(ii) f x = e sin x on 0 ,

b g b g b x bg

Solution: (i) 3 f x = x a

where m

and n are +ve integers.


= a polynomial in x
f (x) is differentiable in [a, b] and so continuous
in [a, b] as f (x) is a polynomial function.
(2) f (a) = f (b) = 0
All conditions of Rolles theorem are satisfied
Now f x

af
= m a x af
a x bf + n a x a f a x b f
= a x af
a x bf
m a x bf + n a x a f
m1

m1

n 1

n 1

788

How to Learn Calculus of One Variable

af

f x =0

a f a x bf

x a

m1

n 1

a f a f

m x b + n x a = 0

Now equating each factor to zero.

a f = 0 x = a (if m > 1)
or a x bf
= 0 x = b (if n > 1)
mb + na
or, m a x bf + n a x a f = 0 x =
m+n
xa

m 1

n 1

bg

mb + na
Thus, f x = 0 for x =
and we see
m+n

(ii) (1) 3 f x = e sin x

continuous in 0 ,

af af

(2) f 0 = f = 0
All conditions of Rolles theorem are satisfied.
x

F + xI
H4 K

one value is x =

(ii) log

{e x

+ ab

j aa + bf x} in [a, b]; ab > 0

af

sin x

Solution: (i) (1) f x =

= sin x e

a f sin 0 = 01 = 0 U|
e
(2)
V f a0f = f af
sin
0
=
= 0|
f a f =
|W
e
e
0

All conditions of Rolles theorem are satisfied.

af

Now f x =

F + xI = 0
af
H4 K
F + xI = 0 3 2 0 , e 0
cos
H4 K
F + xI = cos
cos
H4 K 2

= 2n
4
2

in 0 ,

e cos x sin x e
e

2x

n = 0 , 1 , 2 , ...

2

x=
=
2
4
4
4

acos x sin xf = 0
e

cos x sin x = 0 3 e 0

cos x = sin x
1 = tan x
tan

= tan x
4

x = n +

n = 0 , 1 , 2 , ...
4

a f

0,
4
Rolles theorem is verified.

and x =

acos x sin xf
e

f x =0

f x = 0 2e cos

x+

af

Now, f x = e sin x + e cos x


x

a f

sin x

f 0 =

f (x) is differentiable in 0 , as it is the product


of differentiable functions ex and sin x which it is

= 2 e cos

(i)

continuous in 0 , .

Rolles theorem is verified.

af

7. Verify Rolles theorem for

f (x) is differentiable in 0 , as it is the product


of differentiable functions sin x and ex which it is

mb + na
that x =
lies in (a, b).
m+n

af

a f

0,
4
Rolles theorem is verified.

Now since

789

Rolle's Theorem and Lagrange's Mean Value Theorem

a f

R| e x + abj U|
(ii) (1) f a x f = log S
|T aa + bf x V|W

Clearly both 1 , 3
Hence, the fact that f x = 0 for at least one
c 1 , 3 has been verified.
Rolles theorem is verified.

a f

f (x) is differentiable on [a, b] as it is the log


function which it is continuous on [a, b].

(2)

a f UV
af W

af

af

f a =0
f a = f b
and f b = 0

af

2x
2

x + ab

af

x ab
1
=
x x x 2 + ab

f x =0
2

x ab

x x + ab

9. Verify Rolles theorem for the function log (x2 + 2)


log 3 on [1, 1]
Solution: (1) f (x) = log (x2 + 2) log 3

U|
V
f a1f = log a1f + 2 log 3 = log 3 log 3 = 0|
W
f a 1f = f a1f
a f

a f

f 1 = log 1 + 2 log 3 = log 3 = 0

All conditions of Rolles theorem are satisfied.

Now f x =

=0

(2) f (x) is differentiable on [1, 1] as it is the


difference of two differentiable functions log (x2 + 1)
and log 3 (a constant function) which it is
continuous in [1, 1].
All conditions of Rolles theorem are satisfied.

af

af

f x =0

x ab = 0
x = ab

x = ab
Now c = ab lies in the open interval (a, b) being
geometric mean of a and b.
Rolles theorem is verified.
8. Verify Rolles theorem for the function f (x) = (x 1)
(x 2) (x 3) on [1, 3].
Solution: (1) f (x) = (x 1) (x 2) (x 3)
f (1) = f (3) = 0
(2) f (x) is differentiable on [1, 3] as it is a polynomial
function of x which it is continuous in [1, 3].
All conditions of Rolles theorem are satisfied.
Now f ' (x) = (x 2) (x 3) + (x 1) (x 3) + (x 1)
(x 2)
= x2 5x + 6 + x2 4x + 3 + x2 3x + 2
= 3x2 12x + 11

2x
2

x +2

1
2

x +2

2x 0 =

3x 12 x + 11 = 0
1
3

,2 +

1
3

2x
2

x +2

=0

2x = 0
x = 0 and x = 0 1 , 1
Rolles theorem is verified.
10. Verify Rolles theorem for the function

a f

LM OP
N 2Q

af

f x = sin x + cos x 1 on 0 ,
Solution: (1) f (x) = sin x + cos x 1

af

U|
V|
W

f 0 = sin 0 + cos 0 1 = 0 + 1 1 = 0

f
= sin
+ cos
1 = 1+ 0 1= 0
2
2
2

F I
H K

af

f 0 = f

F I
H 2K

F I
H K

F I
H K

LM OP as it is the sum
N 2Q
of differentiable functions sin x, cos x and a constant
L O
function it is continuous in M0 , P .
N 2Q
(2) f (x) is differentiable in 0 ,

f x =0

x=2

Now, f x =

af

af

790

How to Learn Calculus of One Variable

F
H

All conditions of Rolles theorem are satisfied.


Now, f x = cos x sin x

and

cos x sin x = 0
cos x = sin x

LM 1 , 2 OP .
N2 Q

af
f a xf = 0

x=

bg

x = 1 and x =

x = n + , n Z and
4
4

F I
H K

and

Type 2: Verification of Rolles theorem when interval


is not given.
Working rule: Find the interval or intervals equating
the given function f (x) to c i.e. f (x) = c provides us
interval (where c = 0 in particular). We then proceed
as usual.
Examples worked out:
1. Verify Rolles theorem for the function f (x) = 2x3 +
x2 4x 2.
Solution: To obtain some interval or intervals, we
put f (x) = 0

ja

x 2 2x + 1 = 0

x = 2,

2
3

F
H

And we see that 1 2 ,

Rolles theorem is verified.

2,

OP
Q

1
and
2

2
Now, f x = 6 x + 2 x 4 = 2 3x 2 (x + 1) = 0

0,
4
2

I and continuous on L
MN
K

All conditions of Rolles theorem are satisfied.

cos x

=1
sin x
cot x = cot

1
, 2
2

1
, 2
2

FG
H

2
1
, 2
3
2

IJ
K

LM
N

1
2

OP
Q

LM 1 , 2 OP such that the functional values at the


N2 Q
end points of each f these two intervals are equal and
and

is zero.
Now, f (x) is a polynomial it is differentiable in

F
H

any interval it is differentiable on 2 ,

1
2

I
K

I
K

af
F 1 I
c , 2
H 2 K

Hence, the fact that f x = 0 for at least one

F
H

c 2,

1
2

I
K

and

has been

verified.
Rolles theorem is verified.

af a f
Solution: (1) 3 f a x f = x a x + 3f e

2. Verify Rolles theorem f x = x x + 3 e

2x

2x

Here it is not given in which interval Rolles theorem


is to be verified, so to obtain the interval we put
f (x) = 0

af

Now, f x = 0 x x + 3 e

Thus, we obtain two closed intervals 2 ,

1
2

LM
N

x x + 3 = 0 3e

2x

2x

0 x

OP
Q

=0

x = 0, 3
Hence, we verify Rolles theorem in [3, 0].
(2) f (x) is differentiable in [3, 0] as it is the product
x

of differentiable functions x, (x + 3) and e 2 which


it is continuous in [3, 0]
(3) f (3) = f (0) = 0
All conditions of Rolles theorem are satisfied.

Rolle's Theorem and Lagrange's Mean Value Theorem

Now,

b g mb g r

f x = 1 x+3 + x e
=e

2x

af

b g

2x

+ x x+3 e

1
2

x + 6 x2

2x

FG 1 IJ
H 2K

x x6=0

a f a1f

a f

4 1 6

2 1

1 25 1 5
=
= 3 or 2
2
2

Thus we get one c = 2 3 , 0


Rolles theorem is verified.
N.B.: e

af

f x

Type 3: Verification of Rolles theorem when a


function is defined by various equations
f (x) = f1 (x), when x a
= f2 (x), when x a
or, f (x) = f1 (x), when x a
= f2 (x), when x = a
or, f (x) = f1 (x), when x > a
f2 (x), when x < a
f3 (x), when x = a
Examples worked out:
1. Verify that Rolles theorem applies to the function

F 1 I , when x 0 and = 0,
H xK
L 1O
when x = 0 on the interval M0 , P .
N Q
F 1 I , when x 0 and
Solution: (1) f a x f = x sin
H xK
af

given by f x = x sin

= 0, when x = 0

LM 1 OP .
N Q

continuous in the closed interval 0 ,

F0 , 1 I .
H K

x + x + 6= 0

f (x) is continuous in any interval and hence it is

Again f (x) is differentiable every where except x =


0 and hence it is differentiable in the open interval

f x =0

x=

791

af a
F I
H K

f U|
V|
W

F I
H K

f 0 = 0 given
1
1
1
f 0 = f
(2)
= sin = 0
f

af

All conditions of Rolles theorem are satisfied.


Hence, Rolles theorem is applicable for the
function.

af

f x = x sin

F 1 I , when x 0
H xK

LM 1 OP .
N Q

= 0, when x = 0 on the interval 0 ,

Problems based on examining the truth of the


statement of Rolles theorem:
Refresh your memory:
1. If f (x) is not differentiable at the end points of
closed interval [a, b], then continuity of the function
f (x) is to be tested at the end point a and b of the
closed interval [a, b].
2. Rolles theorem is not applicable in

af
af

f1 x
a rational
f2 x

function if x = c I (where I = given interval) makes


f2 (x) = denominator = 0
3. Rolles theorem does not hold good if one or more
of the following hold.
(i) f (x) is discontinuous at some point in the closed
interval [a, b].
(ii) f ' (x) does not exist at some point in the open
interval (a, b).

af

af

(iii) f a f b
4. All PILET-RC functions are continuous and
differentiable at points belonging to the domain of
definition of the function.

792

How to Learn Calculus of One Variable

Where P = Power function / polynomial function


I = Inverse trigonometric functions / Identity
function
T = Trigonometric functions
L = Linear function / Logarithmic function
E = Exponential function
R = Rational function
C = Constant function
(A) Highlight on discontinuity of trigonometric
functions:

f 2

(i) tan x is discontinuous at x = 2n + 1

multiple of .
2

= odd

(B) Highlight on discontinuous of inverse


trigonometric function:
(i) tan1 x and / cot1 x have no discontinuity.
(ii) sin1 x and cos1 x are undefined x 1 , 1 =
closed interval.

(iii) sec1 x and cosec1 x are undefined x 1 , 1


= open interval.

(C) Differentiable and non differentiable functions:

af

F 1 I , x 0 , f a0f = 0 is continuous
H xK

(ii) f x = x sin

F 1 I , x 0 , f a0f = 0 is differenH xK

tiable everywhere except at x = 0. Similarly,

af

(iii) f x = x cos

F 1 I , x 0 , f a0f = 0 is continuH xK

ous in any interval.

af

tive at n-points x = a1, a2, a3, ..., an.

a f a

(vi) f x =

(iv) f x = x cos

F 1 I , x 0 , f a0f = 0 is differH xK

entiable everywhere except at x = 0.

cos b x where 0 < a < 1 and

b is an odd positive integer subject to the condition

ab > 1 +

3
, for example, the functions,
2

a f FH 23 IK
F 5I
f bxg = G J
H 6K

multiple of .
2
(iii) cosec x and / cot x are discontinuous at
x = n = multiple of .

in any interval.

r =1

cos 15 x

af

f1 x =

(ii) sec x is discontinuous at x = 2n + 1


= odd
2

(i) f x = x sin

a f a x a f sin FGH x =1a IJK has no deriva-

(v) f x =

cos 7 n x

are continuous for all x R but have no derivative


for any value of x.
Remember: 1. Rolles theorem or Lagranges mean
value theorem is not applicable to the above functions
if points of non-differentiability I (where I = given
interval).

af

2. f x = x sin

F 1 I , x 0 , f (0) = 0 is continuous
H xK

for all values of x if p is positive (i.e. if p > 0).

Question: How to show Rolles theorem is not


applicable for a given function defined in a given
interval [a, b]?
Answer: One or more of the following is to be shown.

af

af

1. Show that f a f b
2. Show that f ' (x) does not exist at least at one point
of (a, b) which means that derivative of f (x) does not
exist for some value of x between a and b.
3. Show that f (x) is not continuous at some point
lying in the closed interval [a, b].
N.B.: To examine that the statement of the theorem
consisting of some particular conditions is untrue,
we are required to show by constructing requisite
examples that one or more of the conditions of the
theorem is (or, are) not fulfilled. e.g.,
(i) f (x) = x, x 0 , 1 satisfied the conditions (1)
and (2) of Rolles theorem but does not satisfy the

793

Rolle's Theorem and Lagrange's Mean Value Theorem

condition (3) f (a) = f (b) and there is no point c for it


such that f ' (c) = 0.
(ii) f (x) = | x |, x 1 , 1 satisfied the condition (1)
and (3) but does not satisfy the conditions (2) [i.e.; f
(x) is differentiable on (a, b)]. Hence, there is no point
c for this function at which its derivative would vanish.
Type 1: When non differentiable points are the end
points of the given closed interval (one of the two
end points or both end points of the given closed
interval).
Examples worked out:
Question 1: Examine the validity of Rolles theorem

af

af

f x =0

x = c
Thus, we obtain a closed interval [c, c] such that
the functional values at the end points are equal and
is zero, i.e.
f (c) = f (c)
(ii) f (x) is differentiable for c < x < c which it is
continuous for c < x < c.
Now,
c x = lim c x
bg
= 0 = f a cf f (x) is continuous at x = c
lim f a x f = lim c x = lim c x =
0 = f a cf f (x) is continuous at x = c
lim

f x = lim

x c +

x c

x c

x c

xc

Hence, f (x) is continuous in (c, c)and at x = c, c


which means f (x) is continuous in the closed interval
[c, c].
All conditions of Rolles theorem are satisfied.

bg

Now, f x =

x a +

af af
f a x f = f abf

lim

x b

af

f x and lim are


N.B.: It may be noted that xlim
x b+
a

af

not defined and this is why lim = lim f x and

c x =0

x c +

a f

lim f x = f a

(iii)

=0

af

lim f x = f a 0 for a < a < b


0

x a0

Solution: (i) In this problem, a closed interval in


which the given function is defined is not given, so
to obtain the interval or intervals, we put f (x) = 0.
c x

is [c, c]. If the domain of the definition of a function


is a closed interval a x b , then such functions
are called continuous on [a, b] provided
(i) f (x) is continuous on the open interval (a, b) i.e.

(ii)

Note: (a) The domain of the definition of c x

for the function f x = c x .

x = 0 and 0 c , c
Rolles theorem holds good for the given
function in the given.

2 x
2 c x
2

x
c x
2

, x c

af

xa+

af

x a

lim f x = lim f x .

xb

xb

(b) In the above problem,

b g

f c = lim

bg

U|
=
|V f ( x )
h
|
bc + hg
= |
h
W
b

c2 c + h

h0

c2

f c = lim

h 0

is not differentiable at x = c and x = c which


continuity of the function must be tested at the end
points c and c of the closed interval.

Question 2: Are all conditions of Rolles theorem


satisfied for the function f x = x 1 on [1, 3].
Answer: (i) f x = x 1
f (x) is differentiable in (1, 3] which f (x) is
differentiable at every value of x excepting x = 1.
Hence, continuity at x = 1 must be tested.

af

af

af

Now, r . h. l = lim f x = lim


x 1+

x 1+

x 1 = 1 1

= 0 = f (1) which continuity of the function

af

f x =

x 1 at x = 1.

794

How to Learn Calculus of One Variable

af

N.B.: lim is not required since f x = x 1


x 1
becomes imaginary when x < 1, i.e.; lim is not

af

x 1

required since f x = x 1 is not defined when


x < 1.

af

Again lim f x = lim


x3

x3

x 1 = 31 = 2 = f

(3) which continuity of the function f (x) at x = 3.


In the light of above explanation, we can say f (x)
is continuous on [1, 3] since it is continuous at the
end points x = 1 and x = 3 as well as in between 1 and
3 (i.e. in the open interval (1, 3) = (a, b)).

af
(ii)
f a 3f =

UV f a1f f a3f
3 1 = 2W

f 1 = 1 1 = 0

Hence, all above explanations provide us light to


say two conditions of Rolles theorem are satisfied
and one condition is not satisfied.
Type 2: When non differentiable point given open
interval:
Examples worked out:

f (x) is not defined at x = 1 which it is not


continuous at x = 1 which in turn it is not
differentiable at x = 1
f (x) is not continuous in [0, 2] and f (x) is not
differentiable in (0, 2).

af

(ii) f 0 =

a f 2 a2212f = 2 1 0 = 0
f a 0f f a2 f

No conditions of Rolles theorem is satisfied.


Rolles theorem is not applicable for the given
function in the given interval. In fact not satisfying of
only one condition leads to the conclusion.

Question 3: Does Rolles theorem apply to


on [0, 2].

af

af

Solution: (i) f x = 1 x
3 f (x) is not defined for x > 1 which it is not
continuous for x > 1 which in turn it is not
differentiable for x > 1. Hence, f (x) is not continuous
in [2. 2] and hence, f (x) is not differentiable in [2, 2].
f (x) is not differentiable in (2, 2).

af

(ii) f 2 f 2
No condition of Rolles theorem is satisfied and
any one.
Rolles theorem is not applicable for the given

af

a f

x x2
Question 2: Does Rolles theorem apply to
x 1
on [0, 2].

a f x axx12f

Solution: (i) f x =

a f

x x2
x +1

a f x axx+12f

Solution: (i) f x =

not hold for the function defined by f x = 1 x


on [2, 2].

function f x = 1 x on [2, 2].

f 2 =

Question 1: Give a reason why Rolles theorem does

a f

0 0 2
=2
01

f (x) is differentiable in [0, 2] since it is the


quotient of two differentiable functions (under the
conditions x + 1 0 ) which it is continuous in
[0, 2].

af

(ii) f 0 =

0
=0
1

a f 2 a22+12f = 2 2 0 = 0

f 2 =

f (0) = f (2)
All conditions of Rolles theorem are satisfied.
This is why Rolles theorem is applicable for
the given function in the given interval [0, 2].
Question 4: Does Rolles theorem apply to

af

f x =

1
on [1, 2].
x

af

1
x
3 f (x) is not defined at x = 0 f (x) is not
continuous at x = 0 which in turn f (x) is not
differentiable at x = 0.

Solution: (i) f x =

795

Rolle's Theorem and Lagrange's Mean Value Theorem

Hence, f (x) is not continuous in [0, 2] and f (x) is


not differentiable in (0, 2).
1
f 1 =
= 1
1
f 1 f 2
1 1
(ii)
f 2 = =
2 2
No condition of Rolles theorem is satisfied.
This is why Rolles theorem is not applicable for f (x)

U|
V|
W

a f
af

a f af

1
= in [0, 2].
x
Question 5: Discuss the applicability of Rolles
theorem on the function f (x) = | x | on [1, 1].
Solution: (i) f (x) = | x |
f (x) is continuous for every value of x and
hence it is continuous in [1, 1]

af

At x = 0, R.H.D = R f 0 = lim

= lim

h 0

af

h0

a f af

f 0+h f 0
h

a f
f a0 hf f a0f

h 0
h
= lim = 1 h > 0
h 0 h
h

L.H.D = L f 0 = lim

= lim

h0

h 0

h 0
h
= lim
= 1 h > 0
h
0 h
h

L. H . D R . H . D
Hence, the given function is not differentiable at

a f

x = 0 1 , 1 this is why given function f (x) is not


differentiable in (1, 1).
(ii)

a f
af

UV
W

a f

af

f 1 = 1 = 1
f 1 = f 1
f 1 = 1 =1

Thus we see that two conditions of Rolles theorem


are satisfied and one condition is not satisfied.
Rolles theorem is not applicable for the given
function f (x) in the given interval [1, 1].
Question 6: Is Rolles theorem applicable to the
function f (x) = | x 1 | on [0, 2].
Solution: (i) f (x) = | x 1 |
f (x) is continuous for every value of x since
such a mod function is continuous for every value of
x which it is continuous in [0, 2].

Again at x = 1

af

a f

= lim

h0

a f
f a1 hf f a1f

1+ h 1 0

h 0

= lim

h 0

af

L.H.D = L f 1 = lim

h
=1 h>0
h

h 0

= lim

1 h 1 0
| h|
= lim
h 0 h
h

= lim

h
=1 h>0
h

h 0

h 0

af

f 1+ h f 1
h

R.H.D = R f 1 = lim

L. H. D R.H.D
Hence, the given function is not differentiable at

a f

x = 1 0 , 2 this is why given function is not


differentiable in (0, 2).
(ii) f (0) = | 0 1 | = 1
f (2) = | 2 1 | = 1
Thus we see that two conditions of Rolles theorem
are satisfied and one condition is not satisfied.
Rolles theorem is not applicable for the given
function f (x) in the given interval [0, 2].
Question 7: Discuss the applicability of Rolles
theorem to the function f (x) = | x |3 on [1, 1].
Solution: (i) f (x) = | x |3
f (x) is continuous on [1, 1].

af

d x
x
x
2
=3 x
=3
dx
x
x
f (x) is differentiable for all values of x except
per haps at x = 0.
(ii) f x = 3 x
2

af

R f 1 = lim
= lim

h 0

0+h

h 0

h
= lim
h0 h

af

L f 0 = lim

h 0

a f

af

f 0+h f 0
h
3

for h > 0
3

2
h
= lim
= lim h = 0
h 0 h
h 0

a f

af

f 0h f 0
h

796

How to Learn Calculus of One Variable

= lim

0h

h 0

h 0

= lim

h
h

= lim

h 0

Rolles theorem is applicable to the function f


(x) = x2 in [1,1].

for h > 0

e j

2
h
= lim h = 0
h h 0

af
af
f a x f exists at x = 0 and f a0f = 0
f a x f exists for all values of x a 1 , 1f
Rf 0 = Lf 0

f (x) is differentiable in (1, 1).


(iii) f (1) = | 1 |3 = 1
f (1) = | 1 |3 = 1
f (1) = f (1)
All conditions of Rolles theorem are satisfied.
Roll theorem is applicable in the given function
f (x) on [1, 1].

Question 2: Are all the conditions of Rolles theorem


verified for the function f (x) = x2 in 2 x 3 .
Solution: (i) f (x) = x2
f (x) is continuous and differentiable in any
finite interval as it is a power function which it is
continuous and differentiable in [2, 3].
(ii)

a f UV f a2f f a3f
f a3f = 9 W

f 2 =4

Thus we see that two conditions of Rolles are


satisfied and one condition is not satisfied.
Question 3: Can Rolles theorem be applied to the
functions
(i) f (x) = sec x in 0 , 2

Type 3: When x = c given open interval


Where c = critical point

(ii) f (x) = tan x in 0 ,


Solution: (i) f (x) = sec x

Question: What do you mean by a critical point?


Answer: A critical point is a point at which a function
has the first derivative that is zero, infinite or
undefined.
Or in other words,
A critical point is a point c at which f ' (c) = 0 or
f ' (c) = infinite or f ' (c) does not exist.

3
and x =
2
2
both of which belong to 0 , 2 since it is

Examples worked out:


Question 1: Discuss the applicability of Rolles
theorem for the function f (x) = x2 in [1, 1].
Solution: Since y = x2 is a power function it is
continuous and differentiable in any given finite
interval it is continuous and differentiable in the
given finite interval [1, 1] two conditions of
Rolles theorem namely continuity in the closed
interval and differentiability in the open interval hold
good.
Again f (1) = f (1) = 1 f (a) = f (b) showing that
third condition of Rolles theorem is also satisfied.
Thus, all conditions of Rolles theorem are
satisfied.
f x = 2x = 0
2x = 0
x = 0 and 0 1 , 1 which is true.

af

a f

f (x) is discontinuous at x =

f 2 an = 0 , 1 , ...f

discontinuous at x = 2n + 1

f (x) is not continuous in 0 , 2 which it is


not differentiable in 0 , 2 .
(ii)

af
a f

f 0 = sec 0 = 1

UV f a0f = f a2 f
W

f 2 = sec 2 = 1

Thus we see that two conditions of Rolles theorem


are not satisfied and one condition is satisfied.
Rolles theorem can not be applied to the given
function in the given closed interval.
(ii) f (x) = tan x

as f (x) is not
f (x) is differentiable for all x
2

defined at x =
and 0 , .
2
2
Hence, f (x) is neither continuous nor differentiable

a f

at x =

0 , which it is not continuous in


2

a f

0 , and it is not differentiable in 0 , .

Rolle's Theorem and Lagrange's Mean Value Theorem

af
UV f a0f = f af
f a f = tan = 0W
f 0 = tan 0 = 0

Again

Thus, we see that two conditions of Rolles theorem


are not satisfied and one condition is satisfied.
Rolles theorem can not be applied.
Question 4: Is Rolles theorem applicable to the
function.

797

Rolles theorem is not applicable.


Question 5: Is Rolles theorem applicable on the
function f (x) = sin x in 0 , .
Solution: (i) f (x) = sin x
f (x) is continuous and differentiable for all
values of x which it is continuous and differen-

af

tiable in 0 ,
Thus, two conditions of Rolles theorem are
satisfied.

Solution:

Question 6: Is Rolles theorem applicable on the

F 1 I in the closed interval LM 1 , 1 OP


H xK
N Q
F 1 I in the closed interval [1, 1].
f a x f = cos
H xK

(i) f x = sin

(ii)

af

(i) f x = sin

F 1I
H xK

L 1 1O
3 f (x) is not defined at x = 0 and 0 M , P
N Q

f (x) is neither continuous nor differentiable at x

LM
N

=0

1 1
,

OP
Q

L 1 1O
f (x) is neither continuous in M , P nor
N Q
F 1 1I .
differentiable in the open interval ,
H K
Two conditions of Rolles theorem are not
satisfied.
Rolles theorem is not applicable.

af

(ii) f x = cos

F 1I
H xK

3 f (x) is not defined at x = 0


f (x) is not continuous or differentiable at x = 0
1 , 1 .
f (x) is neither continuous nor differentiable in
[1, 1]
f (x) is not differentiable in (1, 1)
Two conditions of Rolles theorem are not
satisfied.

af

(ii) f (0) = 0 = f
Thus, we observe in the light of above explanation
that Rolles theorem is applicable.

af

function f x = 1 x 5 in [1, 1].

af

Solution: 3 f x = 1 x 5
f (x) is differentiable for x 0 and

af

f x =

4 1
; x0

5 x 15

af

Now, f 0 = lim

h0

a f

af

f 0+h f 0
h

LM OP = ; if h > 0
N Q

1 h5 1
1
= lim
= lim 1
h 0
h 0
h
h5

and = ; if h < 0.

f (x) is not differentiable at x = 0 1 , 1


f (x) is not differentiable in [1,1]
Rolles theorem is not applicable.
Question 7: Are all conditions of Rolles theorem

a f FH 1x IK in [1, 1].
1
Solution: (i) 3 f a x f = cos F I
H xK

satisfied for the function f x = cos

f (x) is neither continuous nor differentiable in


[1, 1] since f (x) is not defined at x = 0 1 , 1
f (x) is not differentiable in (1, 1)

798

(ii)

How to Learn Calculus of One Variable

a f a f UV f a1f = f a1f
f a1f = cos a1f = cos 1 W
f 1 = cos 1 = cos1

Thus, we observe in the light of (i) and (ii) that the


first two conditions of Rolles theorem are not
satisfied and the third condition is satisfied.
All conditions of Rolles theorem are not
satisified.
N.B.: 1. When we are asked whether all conditions
of Rolles theorem are satisfied for the given function,
we are required to examine all the three conditions.
i.e; we examine.
(i) Continuity of the given function f (x) in the given
closed interval [a, b].
(ii) Differentiability of the given function f (x) in the
open interval (a, b).
(iii) Equality of value of the given function f (x) at
the end points a and b of the closed interval. i.e.; f (a)
= f (b).
2. When we are asked to show that Rolles theorem
is not applicable to the given function f (x) on [a, b],
then sometimes we are not required to examine all the
three conditions. i.e.
If any one condition is not satisfied, then that
shows non applicability of Rolles theorem for the
given function defined on the given closed interval
[a, b].
Question 8: Is Rolles theorem applicable to the

af

a f on [0,2].
Solution: (i) 3 f a x f = 1 a x 1f
f a0f = 1 a0 1f = 1 a 1f = undefined

function f x = 1 x 1

3
2

3
2

3
2

3
2

f (x) is discontinuous at x = 0 and 0 0 , 2


Rolles theorem is not applicable.

Question 9: Discuss the applicability of Rolles

af

theorem for the function f x = 2 + x 1


interval [0, 2].

af

2
3

on the

(ii) f (x) is differentiable for x 1 and

af

f x =

bg

a f

3 x 1

1
3

bg

f + 1 = , f 1 =
f (x) is not differentiable at x = 1 and 1 0 , 2
(iii) f (0) = f (2) = 3
All conditions of Rolles theorem are not
satisfied.
Rolles theorem is not applicable to the given
function defined in a given closed interval.
Type 4: When we are provided a function f (x) defined
by various or (different) formulas (or, expression in x)
against which an interval is mentioned s.t union of
those intervals provides us a given closed interval
[a, b] on which Rolles theorem is to be verified.
Working rule: In such types of problems mentioned
above, we adopt the following working rule:
1. We test for continuity and differentiability at the
point a if the function is defined by different formula
on the left and right of a. e.g.,
(i) f (x) = x2, when x 0
= 1, when 0 < x < 1

1
when x 1
x
In the above function, we should test the continuity
and differentiability at x = 0 and x = 1.
(ii) f (x) = x2 + 1, when 0 x 1
=

= 3 x, when 1 < x 2
In the above problem, we should test the continuity
and differentiability at x = 1.
2. Use the theorems and facts for the continuity,
discontinuity differentiability or non differentiability
for the given function y = f (x) i.e.;
(a) Discontinuity at a point (or, number) x = a
non-differentiability at the same point (or, number) x
= a etc.

Solution: (i) f x = 2 + x 1 3 is an irrational


function.
f (x) is continuous function on [0, 2] for being
an algebraic function of x.

N.B.: (i) Continuity at x = a does not guarantee


differentiability at x = a. This is why the test for
differentiability is required if continuity at x = a is
examined.

Rolle's Theorem and Lagrange's Mean Value Theorem

(ii) For testing the differentiability at x = a, we should


find l.h.d and r.h.d using the definition. i.e.

af

L f a = lim

h 0

a f

af

f ah f a
, for (h > 0)
h

a f

af

f a +h f a
, for (h > 0)
h 0
h
3. Find f (a) and f (b) and see whether f (a) = f (b).
4. In the light of (1), (2), (3), we conclude whether
Rolles theorem is applicable or not.
= lim

a f
f a1 hf f a1f
L f a1f = lim
h
{a1 hf + 1} 2 , (h > 0)
= lim
= lim 1 = 1
h0

h 0

= f2 (x), when c2 < x b is provided, then the


domain of f (x) is

f a

= a , c2 c2 , b = a , b

Examples worked out:


Question 1: Discuss the applicability of Rolles
theorem on the function f (x) = x2 + 1, when 0 x 1 ,
= 3 x, when 1 < x 2 , on [0, 2]
Solution: (i) Continuity and differentiability test at
x=1

af

x 1

af

x 1

a f a f

lim f x = lim 3 x = 3 1 = 2

x 1+

j af
= x +1
= a1f + 1 = 1 + 1 = 2
f a1f = f a x f
lim f a x f = lim f a x f = f a1f
e

2
lim f x = lim x + 1 = 1 + 1 = 1 + 1 = 2

x 1

x =1

x 1+

x =1

x 1

the function f (x) is continuous at x = 1 which


it is continuous in the closed interval [0, 2].
Now, for the differentiability at x = 1,

af

R f 1 = lim

h 0

a f

af

f 1+ h f 1
, (h > 0)
h

l3 a1 + hfq 2
= lim
h 0

= lim

h 0

2h2
h

h0

a f

= lim

h0

Remember:
1. When f (x) = f1 (x), when a x c2

799

2h h
= lim 2 h = 2
h 0
h

af

af

af

R f 1 L f 1 which f 1 does not


exist.
The given function has no derive at x = 1

a f

0, 2
The given function is not differentiable in the
open interval (0, 2).

af
af

U|
V|
W

af

af

f 0 = 0 +1 = 1
f 0 = f 2
(ii)
f 2 = 3 2 = 1
Thus we observe that two conditions of Rolles
theorem namely continuity in the closed interval and
equality of the values of the function f (x) at the end
points 0 and 2 of the closed interval [0, 2] are satisfied
but one conditions namely differentiability of the
function f (x) in the open interval (0, 2) is not satisfied.
Rolles theorem is not applicable to the function
f (x) in [0, 2].
Question 2: A function f (x) is defined on [0, 2] s.t f
(x) = 1, when 0 x < 1
= 2, when 1 x 2 then discuss the applicability
of Rolles theorem.
Solution: (i) Continuity and differentiability test at
x = 1.

af
lim f a x f = lim 1 = 1
f a1f = f a x f
= 2
=2
lim f a x f = 2 = f a1f lim f a x f = 1
lim f x = lim 2 = 2

x 1+

x 1

x 1

x 1

x =1

x 1+

x =1

x 1

800

How to Learn Calculus of One Variable

f (x) is discontinuous at x = 1 and 1 0 , 2


f (x) is non differentiable at x = 1 and 1 0 , 2
f (x) is discontinuous and non differentiable in
[0, 2].
f (x) is not differentiable in (0, 2).

af U
a f VW

af

af

f 0 =1
f 0 f 2
f 2 =2

(ii)

No condition of Rolles theorem is satisfied.


Rolles theorem is not applicable for the given
function f (x) in the given interval [0, 2].

Question 3: The function f is defined in [0, 1] as


follows

1
= 2, when x 1 show that f (x) satisfied none
2
of the conditions of Rolles theorem.
Solution: (i) Continuity and differentiability test at
1
.
2

F 1 0I =
H2 K
F 1 + 0I =
H2 K

af

af

lim f x = lim1 1 = 1

x 12

af

x 2
x < 12

F 1 0I f F 1 + 0I
f
H2 K H2 K

af

3 lim f x = lim x = 1
x 1
x <1

x 2
x > 21

and f (1) = 0 (given)

af

f (x) is not continuous at one of the end point of


the given closed interval [0, 1] namely x = 1.
f (x) is not continuous in [0, 1].
Rolles theorem is not applicable to the given
function f (x) defined in the given interval [0, 1].

Note: In the above problem


(i) f (0) = 0
f (1) = 0 (given)
f (0) = f (1)
(ii) Now to show the differentiability in the open

af

a f

a f

Let x = c 0 , 1

bg

R f c = lim

which f (x) is

1
0, 1
2
f (x) is discontinuous and non-differentiable in
[0, 2].
f (x) is not differentiable in (0, 2)

a f a fUV f a0f f a1f


f a1f = 2 a givenfW

g bg

f c+h f c

h 0

= lim

c/ + h c/
h

= lim

h
h

h 0

h 0

f (x) is non-differentiable at x =

f 0 = 1 given

af

lim f x f 1

1
1
discontinuous at x = and 0 , 1 .
2
2

(ii)

x 1

interval (0, 1) we take any c 0 , 1 i.e.;

lim1 f x = lim1 2 = 2

x 2 +

Question 4: A function f (x) is defined on [0, 1] s.t


f (x) = x, when 0 x < 1
f (x) = 0, when x = 1 then discuss the applicability
of Rolles theorem.
Solution: (i) Continuity and differentiability test at
x = 1.

x 1
x <1

1
f (x) = 1, when 0 < x <
2

x=

Hence, each of the three conditions of Rolles


theorem are not satisfied by the given function f (x)
defined in the closed interval [0, 1].

= lim 1 = 1
h 0

bg

L f c = lim

h 0

= lim

h 0

b g bg

f ch f c
h

chc
h

Rolle's Theorem and Lagrange's Mean Value Theorem

= lim

h 0

a f

h
h

a f

= lim +1
h 0

=1
Lf c = Rf c
f c exists and c 0 , 1
f (x) is differentiable in (0, 1)
Thus we see that two conditions namely equality
of values of the function at the end points 0 and 1 of
the closed interval [0, 1] and the differentiability of
the given function f (x) defined by different parts of
the closed interval [0, 1] in the open interval (0, 1) are
satisfied but one condition namely continuity in the
closed interval [0, 1] is not satisfied which has been
shown in the above question 4.

af
af

af

a f

Question 5: A function f (x) is defined on [0, 1] s.t


f (x) = 1, when x = 0
= x, when 0 < x 1 is Rolles theorem applicable?
Solution: (i) Continuity and differentiability test at
x=0

af

af

3 lim f x = lim x = 0
x0
x >0

x0

and f (0) = 1 (given)

af

3. Check whether c a , b given open interval if


c lies in between a and b, them c should be accepted
as the required value and if c does not lie in between
a and b, then c should be rejected i.e.; in the
notational form,
If a < c < b c a , b should be accepted as
the required value and if c a , b it should be
rejected.

a f

b g

Examples worked out:


1. If f (x) = x3 27x on 0 , 3 3 find the value of c in
Rolles theorem.

bg

bg e j

Solution: 3 f x = x 3 27 x , f 0 = f 3 3

bg
f bcg = 3c

and f x = 3x 2 27
2

27 = 0

3c 27 = 0
2

c =

27
=9
3

c = 3
Now accepting + 3 as the required value since

af

3 0 , 3 3 and rejecting 3 0 , 3 3 , we see


that c = + 3 is the required value.

lim f x = f 0
x0
x >0

801

f (x) is discontinuous at one of the end points


of the given closed interval [0, 1] namely x = 0
f (x) is not continuous in [0, 1]
Rolles theorem is not applicable to the given
function f (x) defined in various parts of the closed
interval [0, 1].

Type 5: Problems based on finding the value of c


using Rolles theorem.
Working rule:
1. Differentiate the given function f (x).
2. Put x = c in the differentiated result (or, derived
function or, differentiated function) on both sides of
the sign of equality which we put x = c in the
derived function and in the notation f ' (x) and equate

af

it to zero which f x
be solved for c.

x =c

af

= f c = 0 should

2. Find c of Rolles theorem when f (x) = x2 + 3x + 2 is


defined in [2, 1].

bg

b g b g

Solution: 3 f x = x 2 + 3x + 2 , f 2 = f 1 = 0

bg
f b cg = 2 c + 3 = 0

and f x = 2 x + 3

2c + 3 = 0

2c = 3
3
= 15
.
2
Now c = 1.5 is s.t 2 < 1.5 < 1 which 1.5
(2, 1) should be accepted as the required value.
c = 1.5 is the required value.
c=

802

How to Learn Calculus of One Variable

LM
N

3. If f (x) = cos x be defined on


Rolles theorem.
Solution: 3 f (x) = cos x, f

bg
f bcg = sin c = 0
sin a cf = sin 0

OP
Q


,
, find c of
2 2

FG IJ = f FG IJ = 0
H 2 K H 2K

f x = sin x

FG
H

c = 0 is one value and 0


,
2 2

IJ
K

c = 0 is the required value.

af

4. Find c of Rolles theorem if f x =

Geometrical meaning of Rolles theorem


If the graph of the function
1. f (x) is continuous from a point A (a, f (a)) to another
point B (b, f (b)) [i.e.; f (x) is continuous on the closed
interval [a, b]].
2. f (x) is differentiable in between the two points A
(a, f (a)) and B (b, f (b)) [i.e.; f (x) is differentiable in the
open interval (a, b)].
3. f (x) has the equal ordinates f (a) and f (b) at the
two points A (a, f (a)) and B (b, f (b)) [i.e.; f (x) assumes
equal values at the end points of the closed interval
[a, b]]; then the graph of the function f (x) has at least
one point C (c, f (c)) in between A (a, f (a)) and B (b, f
(b)) (i.e., a < c > b) at which the tangent is parallel to xaxis (i.e. f ' (c) = 0)
y

sin x
e

C (c, f (c))

is
(a , f (a))

f ( a) = f ( b)

B (b, f (b))

defined on 0 , .

bg

Solution: 3 f x =

sin x
ex

bg

bg

bg

bg

f c =

cos c sin c
ec

=0

cos c sin c = 0
cos c = sin c
cos c = cos (90 c)
c = 90 c (a particular solution)
c + c = 90
2c = 90

c=

90
2

c = 45 and 45 =
c =

f (c)

f ( b)

, f 0 = f =0

e x cos x sin x e x
cos x sin x
f x =
=
2x
e
ex

f ( a)

a f

0,
4

is the required value.


4

A1 (a, 0) C1 (c, 0)

B1 (b, 0)

Refresh your memory:


1. Geometrical meaning of Rolles theorem tells that
if the graph of the function (i) which is continuous on
the interval [a, b] and differentiable in (a, b) (ii) which
assumes equal values at the end points a and b of [a,
b] (i.e. f (a) = f (b)), then the graph of the function has
a point (c, f (c) at which the tangent is parallel to xaxis.
Or alternatively,
Under the given conditions
(a) Continuity of the function f (x) in [a, b]
(b) Differentiability of the function f (x) in (a, b)
(c) Equality of functional values at the end points a
and b of [a, b], there is at least one point c which is
not the end point for the closed interval [a, b] such
that the tangent at that point is parallel to x-axis.
2.

LM dy OP
N dx Q

= 0 tan = 0 = 0 (where p is a
p

point on the curve, the tangent at p is parallel to


x-axis.

Rolle's Theorem and Lagrange's Mean Value Theorem

3.

LM dy OP
N dx Q

= the tangent at p is perpendicular


p

to y-axis.
Problems based on geometrical meaning of Rolles
theorem
Problems based on geometrical meaning of Rolles
theorem consists of
1. A function f (x) = an expression in x.
2. Two point (a, f (a)) and (b, f (b))
3. A third point (c, f (c)) between (a, f (a)) and (b, f (b))
is required to find out at which there is a tangent
parallel to x-axis.
Working rule to find (c, f (c)) = a third point in
between two given points (a, f (a)) and (b, f (b)) on the
graph of y = f (x) on using Rolles theorem.
We adopt the following procedure to find a third
point (c, f (c)).
1. Show that f (x) is continuous at all points lying
from (a, f (a)) to (b, f (b)).
Or, alternatively, show that f (x) is continuous on
[a, b].
2. Show that f (x) is differentiable at all points lying
between (a, f (a)) and (b, f (b)).
Or, alternatively; show that f (x) is differentiable
on (a, b).
3. Check whether f (a) = f (b)
4. If all conditions of Rolles theorem are satisfied,
then use the geometrical meaning of Rolle s theorem
which tells about the existance of a point (c, f (c)), or
(z, f (z)) in between two given point (a, f (a)) and (b, f
(b)) at which the tangent line is parallel to x-axis. i.e.;

a f

f ' (z) = 0 or f ' (z) = 0 where z or a , b


5. Find c and f (c).

Question A: How to find c.


Answer: (a1) find f ' (x) and put f ' (x) = 0
(a2) substitute x = c (or, z) in f ' (x) and solve for c (or,
solve for z).
Question B: How to find f (c).
Answer: (b1) Put x = c (or, z) (i.e., the roots of the
equation f ' (c) = 0 or f ' (z) = 0) in f (x) which put x
= c or z in the given function y = f (x) of the question.

803

Remember:
1. x-coordinates of the required third point is
obtained is obtained from f ' (x) = 0.
2. y-coordinates of the required point is obtained by
putting the root of f ' (x) = 0 which belongs to (a, b) in
the given function y = f (x).
3. Given point (a, f (a)) and (b, f (b)) form the closed
interval [a, b] and the open interval (a, b).
4. f ' (c) or f ' (z) = 0 for some c (a < c < b or, a < z < b)
or for some z the tangent to the curve (the graph
of the given function y = f (x)) at x = c (or, z) is parallel
to x-axis.
Worked out examples on geometrical meaning of
Rolles theorem
Question 1: Using Rolles theorem, show that on the
graph of y = x2 4x + 3, there is a point between (1, 0)
and (3, 0) where the tangent is parallel to x-axis. Also
find the point.
Solution: (i) Since given function y = f (x) = x2 4x +
3 is a polynomial function and hence it is continuous
and differentiable in [1, 3].
y = f (x) is differentiable in (1, 3).
(ii) f (1) = f (3) = 0
All conditions of Rolles theorem are satisfied.

a point (z, f (z)) between (1, 0) and (3, 0) at


which tangent is parallel to x-axis. i.e.; f ' (z) = 0
y

(1, 0)

(3, 0)

(2, 1)

bg

f z =0

2z 4 = 0
z=

af

and f z

4
=2
2

(i)
2

z =2

= z 4z + 3

z =2

= 22 4 2 + 3 = 4 8 + 3 = 7 8 = 1
(ii)
Thus, we get (z, f (z)) = (2, 1) which is the required
point.

804

How to Learn Calculus of One Variable

Question 2: Using Rolles theorem, prove that there


is a point between the points (1, 2) and (2, 2) on the
graph of y = x2 3x where the tangent is parallel to xaxis. Also find the point.
Solution: (i) Given function y = f (x) = x2 3x is a
polynomial function of x and this is why it is
continuous and differentiable in [1, 2].
y
3/2

x
9/4

(1, 2)

(2, 2)

af

Now, f c = 0 2c 3 = 0 c =

af

F 3 I = x 3x
H 2K
F 3I 3 3
=
H 2K
2

3
2

x = 23

9 9
9
=
4 2
4

b a fg FH 23 , 94 IK

Required point = c , f c =

Problems based on Rolles theorem


Type 1: Problems based on verification of Rolles
theorem
Exercise 20.1
Verify Rolles theorem for the following functions:

af

af a

4. f x = x x + 3 e

2x

in [3, 0]

5. f (x) = sin x in 0 ,

2
1. f x = x 5x + 4 in 1 x 4

af

7. f x = x sin

F 1 I , x 0 and f (0) = 0 in LM0 , 1 OP


H xK
N Q

1
x 2
2
9. f (x) = 2 (x + 1) (x 2) defined in [1, 2]
10. f (x) = x (x 1) in [0, 1]
11. f (x) (x 1)3 (x 2)2 in [1, 2]
12. f (x) = (x 1) (x 2) (x 3) when 0 x 4
13. f (x) = | x | in [1, 1]
8. f (x) 2x3 + x2 4x 2 when

f (x) is differentiable in (1, 2).


(ii) f (1) = f (2)
All conditions of Rolles theorem are satisfied.
According to geometrical meaning of Rolles
theorem, there is a point (c, f (c)) in between (1, 2)
and (2, 2) where the tangent is parallel to x-axis.

f c = f

3. f (x) = 4 sin x in 0 ,

6. f (x) = x3 (x 1)2 in the interval 0 x 1

0
2

LM OP
N 2Q

2. f (x) = sin 2x in 0 ,

14. f (x) = 3x x3 in 0 , 3
15. f (x) = x3 3x in 3 , 0

LM
N

1 1
16. f (x) = x 1 in ,
2 2

af

17. f x =

sin x
e

OP
Q

in 0 ,

af a
f LMN 4 , 54 OPQ
f a x f = a x a f a x bf , where m and n are
x

18. f x = e sin x cos x in


m

19.
integers, in [a, b]
20. f (x) = x2 4x + 3 in [1, 3]
Answers
1. Rolles theorem is true
2. Rolles theorem is true
3. Rolles theorem is true
4. Rolles theorem is true
5. Rolles theorem is true
6. Rolles theorem is true
7. Rolles theorem is true

Rolle's Theorem and Lagrange's Mean Value Theorem

8. Rolles theorem is true


9. Rolles theorem is true
10. Rolles theorem is true
11. Rolles theorem is true
12. Rolles theorem is true
13. Rolles theorem is true
14. Rolles theorem is true
15. Rolles theorem is true
16. Rolles theorem is true
17. Rolles theorem is true
18. Rolles theorem is true
19. Rolles theorem is true
20. Rolles theorem is true
Type 2: Problems based on verification of Rolles
theorem when interval is not given.
Exercise 20.2
Verify Rolles theorem for the following functions:
1. f (x) = 16x x2
2. f (x) = x3 x2 4x + 4
3. f (x) = sin x

af

4. f x = sin

F xI
H 2K

Type 3: Problems based on examining of Rolles


theorem whether Rolles theorem is applicable or
not is required to test.
Exercise 20.3.1
Discuss the applicability of Rolles theorem for the
following functions:
1. f (x) = | x | in [1, 1]
2. f (x) = | x 1 | in [0, 2]

F 1 I in L 1 , 1 O
H x K MN PQ
L O
4. f (x) = sin x cos x in M0 , P
N 2Q
5. f a x f = 1 a x 1f in [0, 2]
L x + ab OP
6. f a x f = log M
MN aa + bf x PQ in [a, b]
af

3. f x = sin

3
2

805

7. f (x) = ex sin x in 0 ,

a f x 1 in [1, 3]
1
L1 O
f a x f = 2 x + in M , 2P
N4 Q
x

8. f x =
9.

10. f (x) = tan x in the interval [1, 2]


1
11. f x =
in the interval [1, 2]
x
x x2
in the interval [0, 2]
12. f x =
x 1
2
13. f (x) = 8x x in the interval [0, 8]
14. f (x) = x3 6x2 + 11x 6 in the interval [1, 3]
15. f (x) = (x + 1) (x 2) in the interval [1, 2]

af
a f aa f f

Answers:
1. Not applicable since f (x) is not differentiable at
x=0
2. Not applicable since f (x) is not differentiable at
x=1
3. Not applicable as f (0) is not defined
4. Rolles theorem is applicable
5. Not applicable as f (0) is not defined
6. Rolles theorem is applicable
7. Rolles theorem is applicable
8. Not applicable since f ' (x) is does not exist
9. Applicable
10. Not applicable since f (x) is neither continuous

2
11. Not applicable as f (x) is neither continuous nor
differentiable at x = 0
12. Not applicable as f (x) is neither continuous nor
differentiable at x = 1
13. Applicable
14. Applicable
15. Applicable
nor differentiable at x =

Exercise 20.3.2
Rolles theorem can not be applied in the following
functions in the interval specified. Explain why.
1. f (x) = x in 1 x 1

806

2.

How to Learn Calculus of One Variable

RS f a xf = 2 x for x 1 in 0 x 2
T f a x f = 4 2 x for x > 1
f a x f = 1 x in 1 x 1
2

3
3.
4. f (x) = tan x in 0 x

Answers:

a f

af

1. f 1 f 1
2. f (x) is not differentiable at x = 1

a f

4. f (x) is discontinuous at x = a0 , f
2
3. f (x) is not differentiable at x = 0 1 , 1

Type 4: Problems based on verification of Rolles


theorem and finding the value of c.
Exercise 20.4
Verify Rolles theorem for the following functions and
find the value of c provided Rolles theorem is true.
in 0 , 3
af
f a x f = x 3 x in 3 , 0
f a x f = sin x in 0 ,
f a x f = x + 3x + 2 in 2 , 1
f a x f = 4 sin x in 0 ,
L O
f a x f = sin 2 x in M0 , P
N 2Q

1. f x = 3x x
2.
3.
4.
5.
6.

a f a f in 3 , 0
8. f a x f = x a x in 0 x a
9. f a x f = x a x 2f a x + 1f in 0 x 2
L 5 O
10. f a x f = e asin x cos x f in M , P
N4 4 Q
11. f a x f = a x af a x bf in a x b , m and
7. f x = x x + 3 e
2

2x

n being positive integer.

af

a f

12. f x = x x 1 in 0 x 1

af

13. f x = 2 x + x 4 x in 2 x 2
Answers:
1. c = 1
2. c = 1

3. c =

4. c =

6. c =
4
7. c = 2

3
2

5. c =

8. c =

9. c = 3 1
10. c =
11. c = a , b ,

mb + na
m+n

1
2
1
13. c =
3
12. c =

Lagranges Mean Value Theorem


Statement of Lagranges mean value theorem
Let y = f (x) a real function (or, real valued function)
defined on a closed interval [a, b]. If
(i) f (x) is continuous in (or, on or, over) the closed
interval [a, b].
(ii) f (x) is differentiable in (or, on or, over) the open
interval (a, b) then there is at least one point

a f
f abf f aa f
f a cf =
ba
value of f a x f at x = b value of f a x f at x = a
=

x = c a , b at which the first derivative.

difference of end points of the given interval

Rolle's Theorem and Lagrange's Mean Value Theorem

N.B.: 1. Rolles theorem has three conditions


whereas Lagranges mean value theorem (or,
sometimes called simply mean value theorem) has
only two conditions.
2. f (x) satisfies two conditions of Rolles theorem
except the condition f (a) = f (b) in mean value theorem
which is also termed as Lagranges mean value
theorem.
Type 1: Verification of Lagranges mean value
theorem.
Working rule: To verify Lagranges mean value
theorem, we have to show that following conditions
are satisfied.
1. Show that a given function is continuous in the
closed interval and differentiable in the open interval
which is provided by using the facts that all PILETRC functions are continuous and differentiable at
points where they have finite value and also
remembering that sin x, cos x, sin1 x, cos1 x, log x,
ex, polynomial, power, constant, identity functions
are continuous and differentiable in any give finite
interval.
2. Find f ' (x) as well as f (a) and f (b)

af

af

af

f b f a
3. Then use the result f c =
ba

and

solve it which will provide us the value of c such that

a f

F 1I
H 2K

(ii) f (x) is differentiable in the open interval 0 ,

Thus the two conditions of Lagranges mean value


theorem are satisfied.

af
f acf = 3c

Now (i) f x = 3x 6 x + 2
2

6c + 2

(ii) f (a) = f (0) = 0

a f FH 21 IK = 81 43 + 22

(iii) f b = f

16+8 3
=
8
8

1
1
0=
2
2
Now, using the result of Lagranges mean value
theorem, at least one c such that
(iv) b a =

af

f c =

af

af

f b f a
ba

3
0 3
3
8
3c 6c + 2 =
= 2=
1
4
8
2
2

a < c , b i.e. c a , b .
Note: In short Lagranges mean value theorem is
written as L.M.V.T.

3c 6c + 2 =

Examples worked out:

3c 6c + 2

F
H

Question 1: Verify Lagranges mean value theorem

L 1O
for the function f (x) = x (x 1) (x 2) in M0 , P .
N 2Q
Solution: 3 f a x f = x a x 1f a x 2 f
f a x f = x 3x + 2 x
3

3 f (x) is a polynomial in x
f (x) is continuous and differentiable in any
given interval
(i) f (x) is continuous on the closed interval
1
0,
2

LM OP
N Q

807

3c 6c +

c=
=

a f

3
4

I
K

3
=0
4

5
=0
4

36 4 3

5
4

23
6 36 15
6

1
1
21 or , 1 21
6
6
= 1.76 or, 0.24
=1+

808

How to Learn Calculus of One Variable

F 1 I and for
H 2K
f abf f aa f
which the equation f acf =
is true.
ba
Thus, we get a value of c = 0.24 0 ,

Hence, Lagranges mean value theorem is verified.


Note: (i) 1 +

FG IJ
H K

21
1
This is why
= 1.76 0 ,
6
2

it is rejected.

FG IJ
H K

21
1
This is why its is
= 0.24 0 ,
6
2

(ii) 1
accepted.

Question 2: Verify Lagranges mean value theorem


for the function f (x) = (x 1) (x 2) (x 3) on [0, 4]
Solution: 3 f (x) = (x 1) (x 2) (x 3)
= x3 6x2 + 11x 6
f (x) is a polynomial in x
f (x) is continuous and differentiable in any
given finite interval.
(i) f (x) is continuous in the closed interval [0, 4]
(ii) f (x) is differentiable in the open interval (0, 4)
The two conditions of Lagranges mean value
theorem are satisfied.

af

bg

f c = 3 x 2 12 x + 11

x =c

af
a4 1f a4 2f a4 3f a6f

bg

bg

f 4 f 0
40

3 2 1+ 6
=
4

4 3c 12c + 11 = 12
2

12c 48c + 44 32 = 0
2

3c 12c + 8 = 0

c=

12 144 96 12 48
=
6
6

3 = 2 1155
.

Question 3: Verify Lagranges mean value theorem


2
for the function f x = x + 3x + 2 , x 0 , 1 .

af
Solution: 3 f a x f = x

+ 3x + 2
f (x) is a polynomial in x
f (x) is continuous and differentiable in any
given finite interval.
(i) f (x) is continuous on the closed interval [0, 1]
(ii) f (x) is differentiable in the open interval (0, 1)
Two conditions of Lagranges mean value
theorem are satisfied.

af
f acf = 2c + 3

Now, (i) f x = 2 x + 3
(ii) f (a) = f (0) = 2
(iii) f (b) = f (1) = 1 + 3 + 2 = 4
(iv) b a = 1 0 = 1
Now, using the result of Lagranges mean value
theorem,

af

f c =

f c = 3c 12 c + 11

2
3

c = 3155
.
or , 0.845
But these values of c lie in the open interval (0, 4)
Hence, the theorem is verified.

Now, f x = 3 x 12 x + 11

=2

af

af

f b f a
42 2
=
= =2
ba
1
1

2c + 3 = 2
2c + 3 2 = 0
2c + 1 = 0
2c = 1
1
which belongs to (0, 1)
2
1
c = 0, 1
2
Thus, we observe that we get a value of
c=

a f

a f and for which the equation


f abf f aa f
f a cf =
holds good hence,

c=

1
0, 1
2

ba
Lagranges mean value theorem is verified.

809

Rolle's Theorem and Lagrange's Mean Value Theorem

Question 4: Verify Lagranges mean value theorem


for the function f (x) = x3 in [a, b].
Solution: (i) f (x) = x3
f (x) is a power function in x
f (x) is continuous and differentiable in any
given finite interval
(i) f (x) is continuous in the closed interval [a, b].
(ii) f (x) is differentiable on the open interval (a, b)
Two conditions of Lagranges mean value
theorem are satisfied.

af

Now, (i) f x = 3x 2

af

f c = 3c
(ii) f (b) = b3
(iii) f (a) = a3
(iv) b a = b a
Now, using the result of Lagranges mean value
theorem,

a f f abbf af aaf 3c

f c =
2

3c = b + ab + a
c=

2
2
b a
= b + ab + a
ba

F
GH

I
JK

b 2 + ab + a 2
a 2 + ab + b 2
<b
, a<
3
3

Hence verified.
Question 5: Verify Lagranges mean value theorem
for the function f (x) = lx2 + mx + n on [a, b]

af

Solution: 3 f x = lx + mx + n
f (x) is a polynomial in x
f (x) is continuous and differentiable in any
given finite interval.
(i) f (x) is continuous on the closed interval [a, b]
(ii) f (x) is differentiable on the open interval (a, b).
Two conditions of Lagranges mean value
theorem are satisfied.

af
f acf = 2 lc + m

af af e

j a f a f

f b f a l b a +m ba
(v)
=
=l ba +m
ba
ba

Now, using the result of Lagranges mean value


theorem,

af

af

af

f b f a
ba

f c =

a f
b+a
c=
aa , bf
2

2 lc + m = l b + a + m

Lagranges mean value theorem is verified.


Question 6: Verify Lagranges mean value theorem

af

for the function f x =

af

x 4 on [2, 4].

af

Solution: 3 f x =

g x =

x 4

Now, g (x) = x2 4, (g (x) > 0) is continuous and


differentiable on a given finite interval g x is
also continuous and differentiable on the same given
finite interval using the continuity and differentiability
theorem for a function of a function.
(i) f (x) is continuous on the closed interval [2, 4]
(ii) f (x) is differentiable on the open interval (2, 4)
Two conditions of Lagranges mean value
theorem are satisfied.

af

af

Now, (i) f x =

2x
2

2 x 4

af

f c =

2x

2 x 4

x 4
c
2

c 4
44 =0

Now, (i) f x = 2 lx + m

(ii) f (a) = f (2) =

(ii) f (a) = la2 + ma + n


(iii) f (b) = lb2 + mb + n
(iv) b a = b a

(iii) f (b) = f (4) = 16 4 = 12 = 2 3


(iv) b a = 4 2 = 2
Now, using the result of Lagranges mean value
theorem,

810

How to Learn Calculus of One Variable

af

af

c 4

2
2

c 4

b g

c=

= 3 c = 3c 12 c 3c = 12

2c 2 = 12 c 2 = 6 c =

6 and

6 2,4

b g

Thus, c = 6 2 , 4 verifies the Lagranges


mean value theorem.
Question 7: Verify Lagranges mean value theorem
for the function f (x) = log x in [1, e]
Solution: 3 f (x) = log (x)
f (x) is continuous and differentiable in a given
finite interval for being a log function.
log x is continuous in [1, e] and differentiable
in (1, e).
Two conditions of Lagranges mean value
theorem are satisfied.
1
Now, (i) f x = , (x > 0)
x
1
f c = , (c > 0)
c
(ii) f (b) = f (e) = log e = 1
(iii) f (a) = f (1) = log 1 = 0
(iv) b a = e 1
Now, using the result of Lagranges mean value
theorem

af

af

af

af

Question 8: Verify the hypothesis of mean value


theorem for the function f x = x + 2 on the
interval [4, 6] and find a suitable value for c that
satisfied the conclusion of the theorem.
Solution: Having the fact that g (x) = x + 2 is a
continuous and differentiable on a given finite interval
and using the theorem for continuity and differentiability for a function of a function, we conclude that
f x = x + 2 is continuous and differentiable on
a given finite interval (where f (x) > 0)
is continuous and
f x = x+2
differentiable in the closed interval [4, 6].
(i) f (x) is continuous in the closed interval [4, 6]
(ii) f (x) is differentiable in the open interval (4, 6).
Two conditions of Lagranges mean value
theorem are satisfied.
Now, using the result of Lagranges mean value
theorem, we get

af

2 30
= 3 , (c > 0)
2

af

f 4 f 2

42

f c =

af

f b f a
f c =
ba

af

af

2 c+2

1
2 c+2
1
c+2

1 1 0
=
c e1

1
1
=
c e1

6
1

6+2 4+2
64

8 6
2

af

f b f a
ba

c = e 1 and 1 < e 1 < e. Hence verified.

af

af

f c =

c+2

=c+2

1
8 + 6 2 48
1
14 2 48
1
14 8 3

=c+2

=c+2

=c+2

811

Rolle's Theorem and Lagrange's Mean Value Theorem

1
14 8 3

2=c

1 28 + 16 3
14 8 3
27 + 16 3
14 8 3

4.

x =c

= c and 4 <

16 3 27
14 8 3

<6

bg

= f c = or undetermined by

using the defination.

af

f af

f c+h f c
R f c = lim
h 0
h

af

L f c = lim

h0

= 2n + 1

af

bg

= 2n + 1 x

dx

=c

Question 9: How to test the non-applicability of


Lagranges mean value theorem in a given interval I =
[a, b].
Solution: At least one of the following is to be shown
for the non-applicability of Lagranges mean value
theorem.
1. Show that f (x) is not continuous at some point
lying in the given closed interval [a, b].
2. Show that f x does not exist at least at one
point c of the open interval (a, b) for which we are
required to show l .h. d r . h. d or, we show

f x

a2n +1f

d x

a f

af

f ch f c
h

Remember:
1. In some of the cases Lagranges mean value
theorem is not applicable for f x if c I s.t
f (c) = 0 as f ' (c) may not exist.
2. Lagranges mean value theorem is not applicable

af

af
af

f1 x
in a rational function
if f2 (c) = 0 for some
f2 x
c I .
3. The point at which given function is nondifferentiable may be the end points of the closed
interval or may be the mid point of the closed interval
or may belong to the open interval different from the
given closed interval.

5.

d x
dx

= 2n x
x

x
x

for x 0

2n

= 2n

a2n +1f

2n

b2n 1g

x
x

2n

for x 0

Type 2: To test whether Lagranges mean value


theorem is applicable or not in a given interval I =
[a, b].
Examples worked out:
Question 1: Discuss the applicability of Lagranges

af

mean value theorem to the function f x =

1
x

in

[1, 1].

af

Solution: (i) Given f x =

1
3

x
f (x) is continuous and differentiable for all
values of x 1 , 1 except where x3 = 0 for being a
rational function in x which f (x) is discontinuous
at x = 0 1 , 1 since f (x) is undefined at x = 0.
Hence, Lagranges mean value theorem is not
applicable to f (x) on [1, 1].
Question 2: Are all the conditions of Lagranges
mean value theorem satisfied for the function
f x = x 1 in [1, 3]. If so, find c of the mean
value theorem.

af

af

Solution: (i) Given function f x = x 1


f (x) is continuous and differentiable for all
values of x belonging to the given interval [1, 3].
f (x) is differentiable in (1, 3)
All conditions of Lagranges mean value
theorem are satisfied.

bg

Now, f c =

bg

b g = f b3g f b1g

f b f a
ba

31

812

How to Learn Calculus of One Variable

2 0
1
=
2
2

1
2 c1

1
2

1=

a f

2 c 1 =1 c 1=

ac 1f

1
1
c =1+
2
2

f (x) is continuous for all values of x 2 , 1


for being modulus of a function which is continuous
for all values.

af

(ii) f x =

which is differentiable for all values


x
of x except perhaps at x = 0

af

3
3
and 1 , 3
2
2
Question 3: Discuss the applicable of mean value
theorem to the function f (x) = | x | in [1, 1].
Solution: 3 Given f (x) = | x |

h0

af

Lf 0 =

f (x) is continuous for all values of x 1 , 1


for being a modulus function.

af

af

f af

f af

f 0+h f 0
R f 0 = lim
h 0
h
h 0
h
= lim
= lim = 1
h0
h 0 h
h

af

L f 0 = lim
= lim

h 0

af

h0

f 0h f 0
h

h 0
h
= lim
= 1 (for h > 0)
h 0 h
h

af

R f 0 Lf 0
The given function f (x) is not differentiable at

a f

x = 0 and 0 1 , 1 .
One condition of Lagranges mean value
theorem is not satisfied.
The mean value theorem is not applicable to
f (x) in [1, 1].
Question 4: Examine the validity of the hypothesis
and the conclusion of Lagranges mean value theorem
for the function.

bg

= lim

h0

af

R f 0

which is differentiable for all


x
value of x except perhaps at x = 0.

f x = x in 2 , 1
Solution: (i) 3 f (x) = | x |

h 0

= lim

f af

f af

f 0+h f 0
h
h 0
h
= lim = 1
h 0 h
h
f 0h f 0
lim
h 0
h
h 0
h
= lim
= 1
h 0 h
h
Lf 0

R f 0 = lim

c=

Now, f x =

af

The given function f (x) is not differentiable at


x = 0 and 2 , 1
f (x) is not differentiable in (2, 1).
One conditions namely differentiability of the
given function in the given open interval (2, 1) is not
satisfied which the hypothesis is not valid.
Now, we examine the result:
1. f (1) = | 1 | = 1 = f (b)
2. f (2) = | 2 | = 2 = f (a)
3. b a = 1 (2) = 1 + 2 = 3

b g cc , c 0
f abf f aaf 1 2
1
=
=

4. f c =
5.

1+ 2

1
is not true.
3
c
Thus, we observe neither the hypothesis nor the
conclusion is valid.

But

Question 5: Discuss the applicability of Lagranges


mean value theorem to the function f (x) = | x |3 on
[1, 2].
Solution: (i) f (x) = | x |3
f (x) is continuous in [1, 2] as a mod function is
continuous in a given finite interval.

Rolle's Theorem and Lagrange's Mean Value Theorem

af

d x
x
3 x
2
=3 x
=
dx
x
x
f (x) is differentiable for all values of x except
perhaps at x = 0.
(ii) f x = 3 x

af

f af

f 0+h f 0
h

R f 0 = lim

h 0
3

0+h 0
= lim
for h > 0
h 0
h
= lim

h 0

af

h
h

= lim

h 0

h 0

0h

h 0

h
= lim
h 0
h

f af

f 0h f 0
h

L f 0 = lim
= lim

2
h
= lim h = 0
h

0
h

af

f x =

sin x
, x 0.
x

af

for h > 0

e j

2
h
= lim
= lim h = 0
h0 h
h 0

af
af
f a x f exists a x = 0 and f a0f = 0
f a x f exists for all values of x a 1 , 2 f

f (x) is differentiable in (1, 2).


All conditions of Lagranges mean values
theorem are satisfied.
Lagranges mean values theorem is applicable
to the given function f (x) on [1, 2].
Note: The above example shows that the statement
Rolles theorem or Lagranges mean value theorem
is not applicable to mod of a function = | f (x) | if x = c
given interval is one of the roots of f (x) = 0 is not
true.
Question 6: Is L.M.V.T applicable to the function
1
defined as f x = x + on [1, 2]. Give reason.
x
1
Solution: (i) f x = x +
x
f (x) is continuous on [1, 2] as it is the sum of
1
two continuous function x and
for all values of
x
x0.

af

af

= 1, x = 0 for 1 x 1 1 , 1

R f 0 = Lf 0

af

(ii) f (x) is differentiable in (1, 2) as it is the sum of two


1
differentiable functions x and for all values of x 0 .
x
All conditions of Lagranges mean value
theorem are satisfied.
Lagranges mean value theorem is applicable to
1
the function f x = x + on [1, 2].
x
Question 7: Give reason whether Lagranges mean
value theorem is applicable to the function

Solution: f x =

813

af

f x =

sin x
,x0
x

x cos x sin x

,x0
2
x
f (x) is differentiable for all values of x except
perhaps at x = 0.

bg

g bg

f 0+h f 0

R f 0 = lim

h 0

sin 0 + h
1
0+h
[ 3 f (0) = 1 is given]
= lim
h0
h

sin h
1
= lim h
h 0
h
= lim

h 0

= lim

h 0

sin h h 1

h
h

sin h h
h

= lim

cos h 1
2h

= lim

sin h
(by L-Hospital's rule)
2

h 0

h0

=0

814

How to Learn Calculus of One Variable

bg

f 0 =0
f (x) is differentiable in [1, 1] and so Lagrange's
mean value theorem is applicable and c = 0.
Question 8: Given reason whether Lagranges M.V.T

F I
H K

1
,x 0 =
is applicable to function f (x) = x cos
x
0, x = 0 for 1 x 1 1 , 1
Solution: x cos

F 1I , x 0
H xK

f (0) = 0
f (x) is continuous in any interval but it is
differentiable every where except at x = 0 and
0 1 , 1 . This is why f (x) is not differentiable in
(1, 1).
One of the two conditions of M.V theorem is
not satisfied. This is why L.M.V.T is not applicable to
f (x) on [1, 1].

Question 9: Discuss the applicability of Lagranges

af

mean value theorem to the function f x =

1
in the
x

closed interval [1, 1].

af

Solution: 3 f x =

Question 10: Discuss the applicability of Lagranges


1
mean value theorem for the function f x =
in
x
[1, 2].

af

1
x

1
x

which is finite for every value of

x 1, 2
f (x) is differentiable in [1, 2]
All conditions of Lagranges mean value
theorem are satisfied.
Lagranges mean value theorem is applicable.

Question 11: A function f (x) in [1, 2] is defined by


f (x) = 2, if x = 1
= x2 if 1 < x < 2
= 4, if x = 2
are all the conditions of Lagranges mean value
theorem satisfied in this case?
Solution: (i) Continuity and differentiability test at
x = 1 and 2 (i.e. at the end points).

af

3 lim f x = lim x = 4
x 2

f (2) = 4

x2

bg bg
lim f a x f = lim x

lim f x = f 2

Again,

f (x) is not defined at x = 0 while 0 1 , 1


f (x) is not continuous at x = 0.
f (x) is discontinuous in [1, 1].
f (x) is non-differentiable in [1,1].
f (x) is non-differentiable in (1, 1)
No conditions of Lagranges mean value
theorem is satisfied.
Lagranges mean value theorem is not applicable
to the given function on the given closed interval.

af

af

(ii) f x =

x 2

1
x

Solution: (i) 3 f x =

f (x) is finite for every value of x 1 , 2 which


it is continuous in [1, 2] for being a rational
function.

f (1) = 2

x 1+

af

x 1

=1

af

lim f x f 1
x 1+

f (x) is not continuous at the end points x = 1 of


the closed interval [1, 2].
f (x) is not continuous in the closed interval
[1, 2].
One condition of Lagranges mean value
theorem is not satisfied and this is why Lagranges
mean value theorem is not applicable to the given
function f (x) defined in the closed interval [1, 2].

Question 12: Discuss the applicability of Lagranges

af

mean value theorem to the function f x = x 3 in


[1, 1].

af

Solution: f x = 3 x
f (x) is not defined for x < 0

815

Rolle's Theorem and Lagrange's Mean Value Theorem

f (x) is not continuous in the closed interval


[1, 1]
No condition of Lagranges mean value theorem
is satisfied.
Lagranges mean value theorem is not applicable
to the given function f (x) in the given interval [1, 1].
Question 13: A function f (x) is defined in [1, 1] by

af

f x = x sin

F 1 I , x 0 f (0) = 0, x = 0 are all the


H xK

conditions of first mean value theorem of differential


calculus satisfied in this case?

af

Solution: 3 f x = x sin

F 1I , x 0
H xK

f (0) = 0, x = 0
We know that the above function is continuous in
any given finite interval but it is differentiable every

a f

where except x = 0 while 0 1 , 1 .


f (x) is continuous in [1, 1] and f (x) is not
differentiable in (1, 1).
One condition namely differentiability of the
given function on the open interval (1, 1) for
Lagranges mean value theorem is not satisfied which
M.V.T can not be applied to f (x) in the given
interval [1, 1].
Question 14: Give a reason why the mean value
theorem does not hold in the function defined by

af

f x = 1 x on [2, 2]

bg

Solution: f x = 1 x is not defined for x > 1

bg

f x is not continuous in [2, 2].


Hence Lagranges mean value theorem does not
hold.
Type 3: Problems based on finding the value of c
using Lagranges mean value theorem.
Working rule:
1. Show the differentiability of the given function in
the given open interval and the continuity of the
given function in the given closed interval or, show
only the differentiability for the given function in the
given closed interval.

2. Find f (a), f (b) and (b a) where a and b are the left


end point and right end point of the given closed
interval [a, b].

af
f abf f aa f
4. Find f acf and equate it to
.
ba
f abf f aa f
5. Solve the equation f acf =
which
ba
3. Find f x .

will provide us one or more than one root of the

af

af

af

f b f a
.
ba
6. The value of c s.t a < c < b should be accepted and
the value of c which does not satisfy a < c < b should
be rejected. In other words, c a , b should be
accepted and c a , b should be rejected.
equation f c =

a f

a f

Solved Examples
Question 1: Find c of Lagranges mean value
theorem when the given function f (x) = x2 3x 1 is
defined on [1, 3].
Solution: (1) 3 f (x) = x2 3x 1
f (x) is differentiable for all values of x for being
a polynomial.
f (x) is differentiable in [1, 3].
f (x) is continuous in [1, 3] and differentiable in
(1, 3).
All conditions of Lagranges mean value
theorem are satisfied.

af

There is a point c s.t f c =

af

af

f b f a
ba

(2) 3 f (x) = x2 3x 1
f (a) = f (1) = 1 3 1 = 1 4 = 3
f (b) = f (3) = 9 9 1 = 1
f (b) f (a) = 1 (3) = 1 + 3 = 2
ba=31=2
Thus, in the light of above determined quantities,
we get

af

af

f b f a
2
= =1
2
ba

(i)

Now, f ' (x) = 2x 3 [from the given equation]

af

f c = 2c 3
Lastly equating (i) and (ii), we get

(ii)

816

How to Learn Calculus of One Variable

af

af

af

f b f a
ba
2c 3 = 1
2c = 1 + 3 = 4
4
c = = 2 and 1 < 2 < 3
2
c = 2 1, 3
Question 2: Find the value of c using L.M.V theorem
on the function f (x) = x2 + 2x 1on [0. 1].
Solution: (1) 3 f (x) = x2 + 2x 1
f (x) is differentiable for all values of x for being
a polynomial.
f (x) is differentiable in [0, 1].
f (x) is continuous in [0, 1] and differentiable in
(0, 1)
All conditions of Lagranges mean value
theorem are satisfied.
f c =

a f

af

There is a point c s.t f c =

af

af

f b f a
ba

(2) 3
f (a) = f (0) = 1
f (b) = f (1) = 1 + 2 1 1 = 2
f (b) f (a) = f (1) f (0) = 2 (1) = 3
Thus, in the light of above determined value of f
(x) at x = 1 and x = b as well as the difference of the
end points of the given closed interval, we find that
f (x) = x2 + 2x 1

af

af

f b f a
3
= =3
1
ba

(i)

Question 3: Find c of Lagranges mean value theorem


when the given function f (x) = (x 1) (x 2) (x 3) is
defined on [0, 4].
Solution: (1) f (x) = (x 1) (x 2) (x 3) which is a
polynomial in x.
f (x) is differentiable in [0, 4].
f (x) is continuous in [0, 4] and differentiable in
(0, 4).
All conditions of Lagranges mean value
theorem are satisfied.
(2) 3 f (x) = x2 + 2x 1
f (a) = f (0) = (1) (2) (3) = 6
f (b) = f (4) = 3 2 1 = 6
ba=40=4

af

af
f acf = 3c

af

af

f c =

af

af

f b f a
ba

2c + 2 = 3
2c = 3 2 = 1
c=

1
1
and 0 < < 1
2
2

c=

1
0, 1
2

a f

(ii)

(i)

Now, f x = 3x 12 x + 11
2

12c + 11

(ii)

lastly, we consider the equation formed by equating


(1) and (2),

af

f c =

af

af

f b f a
ba

3c 12c + 11 = 3
2

3c 12c + 11 3 = 0
2

3c 12c + 8 = 0

c=

62 3
62 3
and 0 <
<4
2
3

c=

62 3
0, 4
3

Now, f ' (x) = 2x + 2 [from the given equation]

f c = 2c + 2
lastly equating (i) and (ii), we get

af

f b f a
12
=
=3
ba
4

a f

Question 4: Find c of Lagranges mean value

af

theorem. When the function f x = x +

LM 1 , 3OP .
N2 Q
1
Solution: (1) 3 f a x f = x +
x

defined on

1
is
x

Rolle's Theorem and Lagrange's Mean Value Theorem

f (x) is differentiable in

LM 1 , 3OP since it is the


N2 Q

sum of two differentiable functions represented by x

1
and , for x 0 .
x
f (x) is continuous in

LM 1 , 3OP
N2 Q

as it is

1
differentiable in L , 3O .
MN 2 PQ
All conditions of Lagranges mean value
theorem are satisfied.

af

af

af

af

f b f a
ba

There is a point c s.t f c =

1
x

(2) f x = x +

a f a f 13 = 103
F 1I = 1 + 2 = 5
f aa f = f
H 2K 2
2
10 5 20 15 5
=
=
f abf = f aa f =
3
2
6
6

f b = f 3 =3+

af

af

1
x

af

f c = 1

1
3

3c 3 = c
2

3c c = 3
2

2c = 3

1
3
. <3
= 15
. = 122
. and 2 < 122
2

c = 122
.

F 1 , 3I
H2 K

Question 5: Find c of Lagranges mean value


theorem when the function f (x) = log x is defined in
[1, e].
Solution: (1) 3 f (x) = log x which is differentiable in
[1, e]
f (x) is continuous in [1, e] and differentiable in
(1, e)
(2) f (x) = log x
f (b) = f (e) = log e = 1
f (a) = f (1) = log 1 = 0
f (b) f (a) = 1 0 = 1
ba=e1

af

af

(i)

af

1
1
f c =
x
c
Lastly, equating (i) and (ii), we get
Now, f x =

(i)

(ii)

1
1
=
c = e 1 = 2.73 1 = 173
. (3 e =
c e 1
2,73 approx)

a f

c = 1.73 and 173


. 1, e
1

Lastly, equating (1) and (2), we get


c 1

c=

af

af

Now, f x = 1

3
2

c =

f b f a
1
=
ba
e 1

5
f b f a
5 2 1
= 6 = =
5 6 5 3
ba
2

817

c 1
c

(ii)

a f

c = 1.73 1 , e
Question 6: Find c of mean value theorem for the

af

function defined as f x = x x 0 , 1 .
Solution: (1) x3 being a polynomial function is
continuous on [0, 1] and differentiable on (0, 1) all
the conditions of L.M.V.T are satisfied a number

af

c s.t f c =

af

af

f b f a
ba

(2) Now, f (x) = x3


f (a) = f (0) = 0

818

How to Learn Calculus of One Variable

f (b) = f (1) = 1
ba=10=1

c=

af af

f b f a
1 0 1
=
= =1
ba
1
1

af

Again, f x = 3x

af

f c = 3c

(i)

a f

c = 15
. 1, 2

Problems based on Lagranges mean value theorem

3
. and 1 < 15
. <2
= 15
2

...(ii)

Type 1: Problems based on verification of Lagranges


mean value theorem (i.e., L.M.V.T)

Lastly, equating (1) and (2), we get

Exercise 20.5

3c = 1

Question: Verify Lagranges mean value theorem (or,


simply mean value theorem) for the following functions
in the interval specified.
1. f (x) = x (x 2) (x 4) in [1, 3]

1
c =
3
2

1
1
=
3
3

c=

Since c has to be in (0, 1) the acceptable value


1
1
of c is
which c =
is the required value.
3
3
Question 7: Find c of mean value theorem when the
function f (x) = x2 + 3x + 2 is defined on [1, 2].
Solution: (1) Since f (x) = x 2 + 3x + 2 being a
polynomial in x is continuous on [1, 2] and
differentiable on (0, 1) all conditions of L.M.V.T
are satisfied which a number c s.t

af

f c =

af

af

f b f a
ba

af

Now, f x = x + 3 x + 2
f (a) = f (1) = 1 + 3 + 2 = 6
f (b) = f (2) = 4 + 6 + 2 = 12
f (b) f (a) = 12 6 = 6
ba=21=1

af

af

f b f a
6
= =6
1
ba

af

2c = 3

1
x

in 1 , 3

a f x 4 in 2 , 4
1
in 1 , 2
4. f a x f = x + 2 +
x3
5. f a x f = x in 1 , 2
6. f a x f = log x in 1, e
7. f a x f = a x 1f a x 2 f a x 3f in 0 , 4
1
in 1 , 2
8. f a x f = x +
x
9. f a x f = x 1 in 1 , 3
10. f a x f = x in 1 , 2
11. f a x f = x 2 x + 4 in 1 , 2
12. f a x f = x 3 x + 2 in 2 , 3
13. f a x f = e in 1 , 2
L1 O
14. f a x f = log x in M , 2P
N2 Q
15. f b x g = sin x in 30 , 60
3. f x =

(i)

Again, f ' (x) = 2x + 3

f c = 2c + 3
Lastly, equating (i) and (ii), we get
2c + 3 = 6
2c = 6 3

af

2. f x =

(ii)

Rolle's Theorem and Lagrange's Mean Value Theorem

Type 2: Problems based on examination of Lagranges


mean value theorem whether L.M.V.T is applicable
or not is required to test.
Exercise 20.6.1
1. Discuss the validity of the mean value theorem for

af

1
in [1, 2].
x
2. Discuss the validity of the mean value theorem for
the function f x =

af a

the function f defined by f x = x 1 3 in the


interval [1, 2].
3. State whether the mean value theorem is applicable
to the function f defined by

af

sin x
; x 0 f (0) = 1 in [1, 1]
(i) f x =
x
(ii)

F 1 I , x 0 = 0, x = 0 in [1, 1]
f a x f = x cos
H xK

(iii) f (x) = | x |3 in [1, 2]

af x
f a xf = 1

(iv) f x =
(v)

1 in [1, 3]
x

in [2, 1]

(vi) f (x) = log sin x in

LM , 5 OP
N6 6 Q

af

(viii) f x =

R|1 + x , x 0
S|1 x , x < 0 in
T

2
3

(iv) f (x) = 2 if x = 1
= x2 if 1 < x < 2 in [1, 2]
= 1 if x = 2
Answers:
(i) Not differentiable at x = 0
(ii) Not differentiable at x = 0
(iii) Derivative does not exist at x = 1
(iv) f (x) is continuous in the open interval (1, 2) and
not in the closed interval [1, 2].
Type 3: Problems based on finding the value of c of
Lagranges mean value theorem.
Exercise 20.7.1
1. Find the value of c of mean value theorem for the
function f (x) = x2 in the interval [1, 4].
2. Find c of the mean value theorem for the function
f (x) = x3 in [1, 2].
3. Find c of L.M.V.T for the function f (x) = (x 1)
(x 2) (x 3) in 0 < x < 4.
4. Find c of the mean value theorem for the function

af

af
L 1 1O
(ii) f a x f = x in M , P
N 8 8Q
(iii) f a x f = 1 x in 2 , 2
(i) f x = x in 1 , 3

1
in the interval [1, 9].
x
5. Find c of L.M.V.T for the function f (x) = log x
defined in the interval [1. 2].
6. Find c of the mean value theorem for the function
f (x) = ex in [0, 1].
7. Find c of the mean value theorem for the function
f (x) = 2x x2 in [0, 1].
8. Using Lagranges mean value theorem, find the
value of c in the following cases.
f x =

(vii) f (x) = | x | in [1, 1].

1 , 1

Answers:

1
2
2. Not valid as f (x) is not differentiable at x = 0
1. Valid and c =

Exercise 20.6.2
Question: Given a reason in each of the following
why the mean value theorem does not hold in each of
the functions defined by

819

af
(ii) f a x f = x 2 x , x 0 , 2
(iii) f a x f = x in [1, 9]
x
(iv) f a x f =
a3 xf in [0, 2]
2

(i) f x = x + 3x + 2 in [0, 1]
2

820

How to Learn Calculus of One Variable

af

(v) f x =

x1
in [1, 4]
x

Answers:
5
1. c =
2

1.

7
3
3. c = 3.15 and 0.84
4. c = 3

6. c = log (e 1)

1
2
1
2

8. (i) c =

5
3.
2

5 5. 5 5 7. 4 9. Both

Geometrical meaning of Lagranges mean value


theorem

1
5. c = log 2 e =
log 2

(ii) c =

Answers:

2. c =

7. c =

af

8. f x = x 3x 6 x + 8 , a = 2, b = 1
9. Is problem (8) an illustration of Rolles theorem or
of the mean value theorem.

If the graph of the function


1. f (x) is continuous from a point A (a, f (a)) to another
point B (b, f (b)) [i.e. f (x) is continuous on the closed
interval [a, b]].
2. f (x) is differentiable in between the two points A
(a, f (a)) and B (b, f (b)) [i.e. f (x) is differentiable in the
open interval (a, b)] then there is a point c such that
the tangent to the graph at the point (c, f (c)) is parallel
to the secant line (or, the chord) passing through (or,
joining) the end points (a, f (a)) and (b, f (b)) of the
curve.

(iii) c = 4
( c,

(iv) 3 3
(v) c = 2
Exercise 20.7.2

f ( a)

))
f (c

f (c)

f ( b)

Find a point x = c such that the mean value theorem is


satisfied. If no such point exists, state what condition
of the mean value theorem is violated.

af
f a x f = x 5x + 7 , a = 2, b = 5
1
f a x f = , a = 1, b = 5
x
f a x f = x = 3 x , a = 0, b = 4
5x
f axf =
, a = 0, b = 4
5 x
f a x f = x , a = 2, b = 2
f a x f = 6 x x , a = 0, b = 6
2

1. f x = x , a = 1, b = 4
2.
3.
4.
5.
6.
7.

2
3

Remember:
(i) The quantity

af

af

f b f a
ba

is the slope of the

secant which passes through the points A (a, f (a))


and B (b, f (b)) of the graph of the function y = f (x).
(ii) The quantity f ' (c) is the slope of the tangent to
the graph of the function y = f (x) at the point (c, f (c)).
(iii) Slope of the chord = slope of the tangent
The tangent line is parallel to the secant line (or, the
chord) i.e.
(iv) The co-ordinates of the extremities (or, the end
points) A and B of the curve f (x) are (a, f (a)) and (b,
f (b)).

Rolle's Theorem and Lagrange's Mean Value Theorem

821

Problems based on geometrical meaning of


Lagranges mean value theorem

Worked out examples on geometrical meaning of


Lagranges mean value theorem

Problems based on geometrical meaning of Lagranges


mean value theorem consists of
1. A function y = f (x)
2. Two points (a, f (a)) and (b, f (b))
3. A third point (c, f (c)) in between (a, f (a)) and (b, f
(b)) is required to find out at which the tangent line to
the curve is parallel to the chord joining the end points
(a, f (a)) and (b, f (b)) of the curve.
Working rule to find (c, f (c)) = a third point in
between two given points (a, f (a)) and (b, f (b)) on the
graph of the function y = f (x) by using Lagranges
mean value theorem.
We adopt the following procedure to find a third
point (c, f (c)).
1. Show that all conditions of Lagranges mean value
theorem are satisfied for which we are required to
show
(a) Show that f (x) is continuous at all points lying
from (a, f (a)) to (b, f (b))
or, alternatively, show that f (x) is continuous on
[a, b].
(b) Show that f (x) is differentiable at all points lying
between (a, f (a)) and (b, f (b)).
Or, alternatively, show that f (x) is differentiable on
(a, b).

Question 1: Use the mean value theorem to find the


point at which the tangent to the curve y = 4 x2 is
parallel to the chord joining the point A (2, 0) and B
(1, 3).
Solution: (1) f (x) = 4 x2
Now, f (x) being a polynomial in x is continuous in
[2, 1] and differentiable in (2, 1).
All conditions of L.M.V.T are satisfied.
(2) f (x) = 4 x2

af

af

af

f b f a
2. Use f c =
to find the value of c.
ba
3. Find the value of f (c) on putting the obtained
value of c in

af

f x

x =c

af

= f c

from the given

af

f c = 2c
(a, f (a)) = (2, 0) = A
(b, f (b)) = (1, 3) = B
a = 2
b=1
f (a) = 0
f (b) = 3
Now, using the result

af

f c =

af

a f

2c = 1
c=

af

1 15
1
and f c = 4 =
4
4
2

b a fg FH
A a x , y f and B a x

(b, f (b))

Question 2: If

x3 =

(a, f (a))

af

x2 + x1
2

af

Solution: (1) f x = ax + bx + c

I
K
, y f be two

1 15
Required point c , f c = ,
2 4

points on the curve y = ax + bx + c , then using


Lagranges mean value theorem, show that there will
be at least one point (x3, y3) where the tangent will be
parallel to the chord AB. Also show that

af

f b f a
30
3
=
= =1
ba
1 2
3

function.
4. Required point will be (c, f (c)).

(c, f (c ))

af

f x = 2x

(i)

f x = 2ax + b
...(ii)
Now, f (x) being a polynomial in x is continuous in
[x1, x2] and differentiable in (x1, x2).

822

How to Learn Calculus of One Variable

All conditions of L.M.V.T are satisfied.


By geometrical meaning of Lagranges mean
value theorem, at least one point P (x3, y3) between
(x1, y1) and (x2, y2) where the tangent will be parallel
to the chord AB.

a f

f x3

a f

a f

f x 2 f x1
y y1
=
= 2
x2 x1
x2 x1

LM3 f axf = ax + bx + cOP


MM= axf ax +f bx= y + c ... etc PP
N
Q
a e x x j + b ax
2

2ax3 + b =

2ax3

x1

x1

f la ax

f q

a f

f b f a

(1)

Now, b a = h b = a + h
(2)
and the interval (a, b) becomes equal to (a, a + h)

a3 b = a + hf as well as c aa , a + hf a < c < a


+h
Again, a < c < b 0 < c a < c b

0<

ca
<1
cb

a f = f aa + hf is valid.

Types of the problems:

Lagranges mean value theorem

b g = f bcg ; a < c < b

f a+h f a

-form of Lagranges mean value theorem

ba

0 < < 1 ) in the open interval (a, a + h) for which

x + x1
x3 = 2
2

bg

...(6)

Let y = f (x) = a real function defined on [a, a + h]


If (1) f (x) is continuous on [a, a + h]
(2) f (x) is differentiable on (a, a + h) then there
exists at least one point c = a + h (where

2ax3 + b/ = a x 2 + x1 + b/
2ax
/ 3 = a/ x2 + x1

b = a + h for = 1
using (4), (5) and (6) in (1), we get,

Statement of L.M.V.T in -form

+ x1 + b

x2 x1

(5)

x2 x1

ax
+b=

Also, a = a + h for = 0

f
f ba + hg f ba g
(1)
ba + hg a = f ba + hg ; 0 < < 1
f ba + hg f ba g

= f ba + hg ; 0 < < 1

2
1

(4)

2
2

c = a + b a = a + h , 0< <1

f (a) = f (a) 3 = 0

a f

ca
=
ba

f (b) = f (a + h) 3 = 1

ax2 + bx 2 + c ax12 bx1 c


x2 x1

2ax3 + b =

0 < < 1 , where

(3)

We consider three types of the problems in finding


from L.M.V.T.
1. When any two of the constants namely a, b and h
as well as a function y = f (x) are given.
2. When only a function y = f (x) is given and no
constant namely a, b and h is provided.
3. Finding the limiting value of t of trigonometrical
ratios by using Lagranges mean value theorem.
Type 1: Problems based on finding ' ' of Lagranges
mean value theorem.
Working rule:
1. Find f (a) and f (b) as well as h using b = a + h.
2. Find f ' (x) and then f ' (a + h) on replacing x in f
' (x) by (a + h)

823

Rolle's Theorem and Lagrange's Mean Value Theorem

a f

af

f a+h f a
= f a + h to find .
h
Remember:
1. a, b, h are constants.
2. Any two of three constants namely a, b, h are given.
3. a and b are the finite values of the end points of
the closed interval [a, b] = [a, a + h] where b = a + h.
4. Sometimes b is not given. b can be found from b =
a + h when a and h are given.

3. Use

Solved Examples

af

Question 1: If a = 1 , h = 3 and f x =
' ' by mean value theorem.
Solution: Since we are given

x , then find

af

af a f
1
Now, f a x f =
,x>0
2 x

f b = f a+h = 4 =2

1
2 a +h

af

Question 2: If f x = x , a = 1, b = 4, find by
L.M.V.T.
Solution: 3 we are given
a=1
b=4
b = a + h 4 = 1+ h 4 1= h 3 = h
f (a) = f (1) = 1

a f a f 4 =2
f aa + hf = 4 = 2
Now, f a x f = x
1
f a xf =
2 4
f b = f 4 =

a = 1, h = 3 and f x = x
f (a) = f (1) = 1
3b = a + h b = 1 + 3 = 4

f a + h =

94 5
9
1=
=
4
4
4
5
=
12
3 =

f a +h =

1
2 1+ 3

1
2 a +h

2 1 + 3h

3h = 3

Now, using the result

a f a f = f aa + hf ; 0 < < 1,

f a+h f a

21
1
=
3
2 1 + 3
1

21
1
=
3
2 1 + 3

1
1
=
3 2 1 + 3

2
=
3

1
1 + 3

1
=
3 2 1 + 3

4
1
=
9 3 + 1

2
=
3

4
1
=
9 1 + 3

9
= 3 + 1
4

1
1 + 3

1 + 3 =

9
4

2 1+ 3

a f a f = f aa + hf we get

f a+h f a

We get,

; 0 < <1

Now, putting the above value


h

3 =

94
9
1=
4
4

in

824

How to Learn Calculus of One Variable

3 =

5
4

5
12
N.B.: We should mark that above two questions are
same except the following difference.
We are provided in question (1) a and h whereas
we are provided in question (2) a and b.
=

Type 2: To find ' ' of Lagranges mean value theorem


provided that only the function is given.
Working rule:

af a

af
a f
f aa + h f f aa f
= f aa + h f

1. Find f a , f a + h , f x and f a + h .
2. Use the formula

Solved Examples
Question 1: Given f (x) =
L.M.V.T.

mx2

af
f aa + hf = 2m aa + h f + n
f x = 2mx + n

Now, using the - form of Lagranges mean value


theorem, we get,

a f a f = f aa + hf; 0 < < 1

f a+h f a

m a + h + 2 ah + na + nh + p ma + ma + p

= 2ma + 2m h + n
2
2
2
ma
/ + mh + 2amh
/ + na
/ + nh
/ + p/ ma
/ na
/ p/

2
= 2amh
/ + 2m h + nh
2

1
2

af
f a x + hf = a x + h f = x + 2 hx + h
(1)
= f a x f + h a2 x + h f
(2)
f a xf = 2 x
(2) f a x + h f = 2 a x + h f
(3)
f aa + h f f aa f
Now,
(4)
= f aa + h f
h
f a x + hf f a x f

= f a x + hf [on replacing

Solution: 3 f x = x

x/ 2/ + 2hx + h 2 x/ 2/
= 2 x + h
h
2hx + h
h

2hx + h

= 2 x + h
= 2 x + h

g = 2 b x + hg

h/ 2 x + h
h/

2 x + h = 2 x + 2 h

h = 2 h

mh = 2 m h

Question 2: Find the value of ' ' by using Lagranges


mean value theorem for the function f (x) = x2.

+ nx + p, find of

Solution: 3 f (x) = mx2 + nx + p


f (a + h) = m (a + h) + n (a + h) + p
f (a) = ma2 + na + p

a by x in (4)]

where 0 < < 1 .

2 mh

5 1
=
4 3

mh

h
1
=
2h 2

Question 3: Find the value of ' ' by using Lagranges


mean value theorem for the function f (x) = log x
(x > 0).
N.B.: Here an interval has not been mentioned. This
is why we may take (or consider) the interval [a, a + h]
arbitrarily where a > 0.

Rolle's Theorem and Lagrange's Mean Value Theorem

af

af

Solution: 3 f x = log x

af

he

f a = log a
f (a + h) = log (a + h)

af

a f = f aa + hf [using ' '

f a+h f a
h

form of L.M.V.T]

log a + h log a
1
=
1 + h
h

log a + h log a =

h =

h
a + h

when h 0 using Lagranges mean value theorem,

a f af

IJ
K

a
=
h

FG
H

h
log 1 +
a

IJ
K

a
h

af

fa

theorem f x + h = f x + h f x + h 0 < < 1

af

when f x = e

af

Solution: 3 f x = e

af
Now, f a x + hf = f a x f + h f a x + h f
f x =e

af

f x = cos x
f (x + h) = sin (x + h)

Question 4: Find the value of ' ' using mean value

a f

f x + h = f x + h f x + h (where 0 < < 1).


Solution: (1) sin x is continuous and differentiable
for all finite values of x All conditions of L.M.V.T
are satisfied.
(2) f (x) = sin x (given)

Question: If f (x) = sin x, find the limiting value of

a
hI
F
log 1 +
H aK

h
h log 1 +
a

Solved Examples

F hI
log 1 +
H aK

L e 1OP
log e = log M
MN h PQ
L e 1OP
h = log M
MN h PQ
L e 1OP
1
= log M
h
MN h PQ

Type 3:

FG
H

F a + h IJ = h
log G
H a K a + h
a + h =

1
f a + h =
a + h
Now,

=e 1

e 1
h
Now, taking log of both sides, we get
e

1
x

f x =

825

Hence, using (2), we get , e

x +h

= e + he

(1)
(2)
x +h

f x + h = cos x + h
Now, using Lagranges mean value theorem, we
get

a f af
a f
sin a x + h f = sin x + h cos a x + h f
sin a x + hf sin x = h cos a x + hf
F x + h + x IJ sin FG x + h x IJ = h cos ax + hf
2 cos G
H 2 K H 2 K
F h I sin F h I = h cos ax + hf
2 cos x +
H 2K H 2 K
f x +h = f x + h f x +h

826

How to Learn Calculus of One Variable

FG
H

2 cos x +

IJ
K

sin

FG h IJ
H 2 K = cos b x + hg

F hI
sin
H 2K
hI
F
cos x +
H 2 K h = cos a x + hf

LM
N

cos x cos

h
h
sin x sin
2
2

OP
Q

R| 1 F h I + ...U|
S| N2 GH 2 JK V|
W
T
R| h 1 F h I U|O
sin x S G J + ...VP
T| 2 N3 H 2 K W|PQ
L 1 F h I + ...OP
M1
MN N3 GH 2 JK PQ
R| bhg U| R| bhg
= cos x S1
+ ... sin x h
|T N2 V|W S|T N3
F 1I
lim bsin x g G J = 0
H 2K
3

U|
V|
W

+ ...

h0

1
for sin x 0
2
Question 2: If f (x) = cos x, find the limiting value of
+
when h 0 using Lagranges mean value

af

I
K

sin

F hI
H 2 K = sin a x + hf

= sin x cos h + cos x sin h

FG h IJ
F h I H 2 K = sin b x + hg
2 sin G x + J
H 2 K FG h IJ
H 2K
R hU
h
h O | sin 2 |
L
Msin x cos + cos x sin P S
V
2
2Q | h |
N
T2 W

h 0

sin

cos x 1

lim =

F x + h + x I sin F x + h x I
H 2 K H 2 K = sin a x + hf

F
H

= cos x cos h sin x sin h

LM
MN

= sin x + h

2 sin x +

h
2

a f

h
2

af

cos x + h cos x

2 sin

2
sin

a f

f x+h f x
= f x+h
h

theorem, f x + h = f x + h f x + h (where

0 < < 1 ).
Solution: (1) 3 f (x) = cos x which is continuous and
differentiable in any finite interval all conditions
of L.M.V.T are satisfied.
(2) 3 f (x) = cos x

LM
MN

|RS 1 FG h IJ + ...|UV +
|T N2 H 2 K |W
R| h 1 F h I U|O
cos x S G J + ...VP
|T 2 N3 H 2 K |WPQ
LM1 1 F h I + ...OP
MN N3 GH 2 JK PQ
2

sin x 1

RS 1 bhg
T N2

= sin x 1

UV
W

b g FGH 12 IJK = 0
1
lim = b for cos x 0g
2
lim cos x
0

RS 1 bhg + ...UV
T N3
W

+ ... + cos x h

Rolle's Theorem and Lagrange's Mean Value Theorem

Question: In the mean value theorem f (h) = f (0) + h

a f

f h , 0 < < 1 show that the limiting value of


1
1
or
according as f (x) =
' ' as h 0 + 0 is
3
2
cos x or sin x.
(a) Solution: Let f (x) = cos x

af
f a h f = sin a h f

f x = sin x

1
lim
2 h0+

1
1 = 1 lim+
2
h 0

af

h0

a f

f 0 + h = f h = cos 0 + h = cos h
Now, from the mean value theorem, f (h) = f (0) + h

a f
f ah f f a0f
= f a h f

a f
cos h 1

= sin a hf
h

h
2 sin
h

b g

sin h sin o
= cos h
h

2 h2
h3
1
h
+ ... = 1
+ ...
h
3
2

h0

lim =
h0

h0

a f

sin h
lim+
h
h 0

1
2

(b) If f (x) = sin x then

h
cos h cos 0

= f h
h

= lim+

F
I
GH N JK
F1 I=0
lim G
H N3 N2 JK

f h , 0 < < 1 we get,

1 2 sin

lim+ =

f (0) = cos 0 = 1

a f

LM sin h OP
MM h 2 PP
N 2 Q

h
1
2
= sin h

a f

1
3

Problems based on finding the value of ' ' using


L.M.V.T.

h
2 = sin h

Exercise 20.8

a f

1. Find ' ' in the mean value theorem

a f

af

h
2 = sin h

h
h
h
2

f a + h = f a + h f a + h where
(i) f (x) = log x, a = 1, h = e 1
(ii) f (x) = 2x2 7x + 10 in [2, 5].
(iii) f (x) = 3x2 5x + 12 in [0, 1].

L sin h OP
1 M
2
M
P
2 M h P
N 2 Q

(iv) f x = x in [1, 4]
2. In Lagranges mean value theorem

b g

sin2

lim+
h 0

af

a f

sin h

L sin h OP
1 M
2
M
P
2 M h P
N 2 Q

a f

af

f a + h = f a + h f a + h show that

1
where f (x) = cos x or sin x.
2
3. In the mean value theorem
lim =

= lim+
h 0

827

a f

sin h

h0 +0

a f

af

(i) If a = 1, h = 1 and f (x) =

x2, find

f a + h = f a + h f a + h .

828

How to Learn Calculus of One Variable

af

(ii) If a = 0, h = 3 and f x =

1 3 3 2
x x + 2x ,
3
2

find .

af

1
(iii) If a = 2, h = 1 and f x = , find the value
x
of .
(iv) If f (x) = ex, express the value of in terms of a
and h.
(v) If f (x) = sin x, find the limiting value of when

h 0.
Answers:

5. cos x 1 or sin x 1
6. x < y and y < z x < z
7. x < y and y z x < z
8. x < y and y x x = y
Key point to prove inequality by L.M.V.T.
1. Application of L.M.V.T on continuous and
differentiable functions in a finite given interval [a, b]
or [0, x] as the case may be.
2. a < c < b if the interval is [a, b].
3. 0 < c < x if the interval is [0, x].
Examples Worked Out:

1
e2
1. (i) =
(ii) =
2
e1
5
(iii) = Find (iv) =
12
1
3 3
3. (i)
(ii)
(iii) 2
2
6
h
1
e 1
1
(iv)
(v)
log
2
h
h

Problems based on proving inequality by using


Lagranges mean value theorem
Working Rule:

af

af

af

f b f a
1. Use
= f c by L.M.V.T on [a, b]
ba

a f af
a f

1. Show that

f a+h f a
= f a + h where 0 < <
or use
a+h h
1 by L.M.V.T on [a, a + h].
2. Prove Left hand expression < f ' (c) < right hand
expression of the required inequality with in the help
of a < c < b or 0 < < 1 and using mathematical
manupulations.
Note: 1. If we use the inequality 0 < < 1, firstly we
find .
2. When the restriction x 0 is given with the
required inequality, the interval [0, x] has been
considered.
3. When the restriction a < b is given with the
required inequality, the interval [a, b] has been
considered.
4. x y x > y or y < x

1+ x

< tan

x < x , x > 0 by using

L.M.V.T.
Solution: Let f (x) = tan1 x and x > 0
tan1 x is continuous and differentiable for all
values of x.
tan1 x is continuous and differentiable in any
finite interval.
L.M.V.T is applicable to the function tan1 x
defined on [0, x].

There is a point c s.t

b g

af af

af

f x f 0
= f c
x0

where c 0 , x

LM
N

tan

x tan
x0

bg

Since f x =

1
1+ c

, 0 < c < x;

bg

1
1
f c =
2
1+ x
1 + c2

OP
PQ

tan x
1

=
2
x
1+ c

tan

x=

x
1+ c

Now, x > c > 0


2

x >c >0

(1)

Rolle's Theorem and Lagrange's Mean Value Theorem


2

a f

x
< log 1 + x < x which is the required
1+ x

x +1> c +1>1

x +1 c +1 1
3x > 0
>
>
x
x
x

x
x
<
<x
2
1+ x
1+ c2

inequality.
Question 3: Find ' ' in Lagranges mean value
theorem for the function f (x) = ex over [a, a + h], then

x
x +1

< tan

x < x which is the required

a f

x
< log 1 + x < x for all
1+ x
x > 0 by using L.M.V.T to the function f (x) = log
(1 + x).
Solution: Let f (x) = log (1 + x)
log (1 + x) is continuous and differentiable for all
values of x > 0.
L.M.V.T is applicable to the function log
(1 + x) defined on [0, x].
Question 2: Show that

af af

af

f x f 0
= f c
There is a point c s.t
x0
where 0 < c < x

a f

Putting (1) in (2), we get

that

a f a f = f aa + hf

f a+h f a
h

a+h

e
a +h
=e
h

a +h

e = he

a +h

= he

a +h

a +h

+e

e 1
h
=e
h

F e 1I = log e
log G
H h JK
F e 1I =
1
log G
h
H h JK
h

(1)

= h log e e = h

1+ x 1+ c 1
>
>
x
x
x
x
x
<
<x
1+ x 1+ c

a number ' ' lying between 0 and 1 such

e = 1 + he

Now, x > c > 0


1+ x >1+ c >1

Solution: Let f (x) = ex (given)


ex is continuous and differentiable in any finite
interval
L.M.V.T is applicable to f (x) = ex defined on [a,
a + h]

ea eh = ea + hea eh

log 1 + x log 1
1
=
1+ c
x 0

LMSince, f bxg = 1 f bcg = 1 OP


1+ x
1+ c Q
N
x
log a1 + x f =
a0 < c < xf
1+ c

I
JK

1
e 1
log
< 1 when x > 0.
x
x

show that 0 <

inequality.

F
GH

(2)

Now putting (1) in (2), we get


2

829

Now, using the inequality 0 < < 1


Putting (1) in (2), we get
(2)
0<

F
GH

I
JK

1
e 1
log
< 1, h > 0
h
h

(1)
(2)

(3)

830

How to Learn Calculus of One Variable

Now, replacing h by x in (3), we get

Fe
1
0 < log G
x
H

1I
J < 1 for x > 0 which is the
x K

required inequality.
Question 4: Prove by using Lagranges mean value
theorem that log (1 + x) < x when x > 0.
Solution: Let f (x) = log (1 + x) x which is defined on
[0, x]

af
f a x f = log a1 + x f x
1 1 x
x
1
f a xf =
1=
=
1+ x
1+ x
1+ x
f 0 = log 1 0 = log 1 = 0

Now f (x) = log (1 + x) x is continuous and


differentiable on [0, x].
L.M.V.T is applicable on [0, x]

af

af

a f

f x f 0
= f x 0 < < 1
x0

log 1 + x x 0
h
=
1+ h
x 0

a f

1+ x
x
=
Now,
x
log 1 + x x

a f

x
1
=
1
x
log 1 + x x

a f

x
1
+1=
x
log 1 + x x

a f
x/ + log b1 + x g x/
1

=
x
log b1 + x g x

a f
a f
0 < x log a1 + x f b3 x log a1 + x f > 0g
0<

x log 1 + x
x log 1 + x

log (1 + x) < x which is the required inequality.


Question 5: Apply Lagranges mean value theorem
to the function f (x) = log (1 + x) to show that
0<

1
1
< 1, x > 0 .
log 1 + x
x

a f

Solution: f (x) = log (1 + x)

af
1
f a xf =
1+ x

f 0 = log 1 = 0

a f

log 1 + x x
x

=
1+ x
x

log 1 + x
1
=
x
log 1 + x x

Now since, 0 < < 1

LM3h=h b= x a = x 0OP
N
Q
x
f aa + hf = f a0 + hf = f a x f =
1+ x

a f
a f
x log a1 + x f
1

log a1 + x f x
log a1 + xf x

=
x log a1 + x f
x log a1 + x f

=
x log a1 + x f

af

1
1+ c

g b

f c =
(1)

g b g b g

f a + h = f 0+h = f h = f x =

1
1+ x

Now, using L.M.V.T, we get

af

af

af

f b f a
= f c ;a<c<b
ba

a f

log 1 + x log 1
1
1
; for x > 0
=
=
1+ c 1+ x
x 0

b0 < < 1g

831

Rolle's Theorem and Lagrange's Mean Value Theorem

b g

log 1 + x 0
x

b g=

log 1 + x
x

b g

log x + 1 =

af

1
1+ x

f c =

1+ c

Now, tan1 x being continuous and differentiable


on any finite interval implies that L.M.V.T is applicable
to tan1 x on [a, b].

1
1+ x
x
1+ x

af

af

af

f b f a
= f c ,a<c<b
ba
1

1
1 + x
=
log 1 + x
x

tan b tan

ba

x
= 1+ x
log 1 + x

Now, since, a < c < b

a f

1
1+ c

(1)

c> a c > a 1+ c >1+ a

x
1= x
log 1 + x

x
1
=
x log 1 + x
x

Again, c < b c < b c + 1 < b + 1

1
1
= (1)
log 1 + x
x

a f

0<

a f

(2)

inequality.

ba

ba
2

< tan b tan a

2 if a < b by using Langranges mean value


1+ a
theorem.
Proof: Let f (x) = tan1 x

af
a
f ab f = tan b
1
f axf =

f a = tan

1
1+ a

(1)

1
1+ c

>

1
2

(2)
(3)

1
1+ b

<

tan b tan
ba

<

1
2

a +1

...(4)

Now, multiplying both sides of (4) by (b a)

b3 ab af > 0g , we have
<

ba
1+ b

< tan b tan a

ba
1+ a

which is the required inequality.

1+ x

<

Putting (1) in (3)

a f

1+ b

1+ c

b +1
1
1
1
<
< 2
(1) and (2)
2
2
1+ b
1+ c
a +1

1
1
< 1 which is the required
log 1 + x
x

Question 6: Prove that

Now, using the inequality 0 < < 1


Putting (2) in (1), we get

<

Note: In establishing elementary functional


inequalities, a slight adjustment in the choice of f (x)
save many times much labour while using L.M.V.T.
1. Existance of t-function in the required
inequality
f (x) = t-function of x which is continuous and
differentiable in any finite interval.

832

How to Learn Calculus of One Variable

2. Existance of inverse t-function in the required


inequality
f (x) = t 1function of x which is continuous and
differentiable in any finite interval.
3. Existance of log-function in the required inequality
f (x) = log x which is continuous and differentiable
in any finite interval.
4. Existance of exponential function in the required
inequality
f (x) = ex which is continuous and differentiable in
any finite interval.

f (x) is continuous and differentiable in [a, b],


where 0 < a < b.
L.M.V.T is applicable on f (x) defined in the
closed interval [a, b].
There is a point c s.t. a < c < b satisfying the
equality

af

af

log b log a 1
=
ba
c

Question 7: Prove by L.M.V.T

Now, a < c < b

(b a) sec2 a < tan b tan a < (b 1) sec2 b if

1 1 1
> >
a c b
Putting (1) in (2), we get

0<a <b< .
2
Solution: Let f (x) = tan x which is continuous and
differentiable on any closed interval [a, b] where

0<a <b< .
2
f b f a
= f c ;
By applying L.M.V.T,
ba

af

af

af

tan b tan a
2
= sec c
ba

(1)

Now, a < c < b


2

sec a < sec c < sec b

(2)

LM OP
N 2Q

3 sec x is increasing in 0 ,

Now, putting (1) in (2), we get

a < tan b tan a < (b a)


(b a)
which is the required inequality.

(2)

1 log b log a 1
>
>
a
ba
a

ab af > log b log a ab af > ab af


a
b
ab af
bb ag > log b log a > bb ag which is the

Question 9: Show that sin x < x for x > 0 by using


L.M.V.T.
Solution: Let f (x) = sin x
sin x is continuous and differentiable in any finite
interval.
sin x is continuous and differentiable in [0, x].
Lagranges mean value theorem is applicable
to f (x) = sin x on [0, x].
a number c s.t. 0 < c < x for which

af

tan b tan a
2
sec a <
< sec b
ba
2

sec2

(1)

a
required inequality.

(a < c < b)

af

f b f a
= f c
ba

sec2

Question 8: Prove by L.M.V.T

ba
b ba
< log <
where 0 < a < b.
b
a
a
Solution: Let f (x) = log x which is continuous and
differentiable in any finite interval.

af

af

f b f a
= f c
ba
3 f (x) = sin x

f (0) = sin 0 = 0 = f (a)


f (x) = sin x = f (b)

af
f acf = cos x

f x = cos x

Rolle's Theorem and Lagrange's Mean Value Theorem

af

af

af

(Since, 1 + c2 > 1)
Putting (1) in (2), we get

f b f a
Hence, f c =
ba

tan

sin x sin 0
cos c =
x0
cos c =

(1)

sin x
<1
x

[from (1)]

2
Question 10: Using Lagranges mean value theorem,
1

x x,x 0.

af af

af

f b f a
= f c , for some c, a < c < b.
ba
1

x x

x x 3 x = x , for x 0 which

Solution: Let f (x) = sin x

b g = f bcg , for some c, x < c < y

x tan
x0

sin y sin x
= cos c
y x

(1)

cosc < 1
Putting (1) in (2), we get

(2)

sin y sin x
<1
yx

sin y sin x < y x


x sin x < y sin y which is the required
inequality.

af

Question 12: If f x =

1
1+ c

[ 3 sin x is continuous and differentiable in any


finite interval]

Since 0 < c

Solution: Let f (x) = tan1 x


tan1 x is continuous and differentiable in any
finite interval.
tan1 x is continuous and differentiable in
[a, b] = [0, x]
L.M.V.T is applicable to f (x) = tan1 x

tan

y x

sin x < x is obvious as sin x 1 <

f y f x

, the result.
2

show that tan

tan

bg

sin x < x
For x

xx

is the required result.


Question 11: If 0 < x < y , then show that
x sin x < y sin y by using L.M.V.T in [x, y].

0 < c cos c < 1

tan

sin x
for x <
2
x

833

1
1+ x

for all x and f (0) =

0, show that 0.4 < f (2) < 2.


1

af

tan x
1
=
2
x
1+ c

tan

Now,

x=

x
1+ c

x
< x , x 0
1+ c2

Solution: 3 f x =

(1)

(2)

1
1+ x

f (x) is continuous and differentiable in any


finite interval.
f (x) is continuous and differentiable in [0, 2].
L.M.V.T is applicable to f (x) defined in [0, 2].

834

How to Learn Calculus of One Variable

af

af

af

af

for some c, 0 < c < 2

af

f 2 0
1
=
2
2
1+ c

af

f 2 =

2
1+ c

0 <c <2

(1)

sin a sin b

1< c +1< 4 +1

ab

1
1>
>
2
5
1+ c
2
2
2
>
>
2
1 1+ c
5

(1)
(2)

(3)
sin a sin b a b
If a > b, then we can consider the interval [b, a] i.e.,
...(2)

Putting (1) in (2), we get

af

sin a sin b
= cos c
ab

Again, since, cosc 1


Putting (1) in (2), we get

2> f 2 >

sin b sin a
= cos c which can be further
ba

sin a sin b
= cos c
ab

0<c <4

af

written as

Now, 0 < c < 2


2

af

f b f a
= f c for some c, a < c < b
ba

f 2 f 0
1
= f c =
2
20
1+ c

2
5

af
0.4 < f a2f < 2
2 > f 2 > 0.4

which is the required inequality.


Question 13: Prove by the mean value theorem
sin a sin b a b
Solution: Let b > a
f (x) = sin x
f (x) is continuous and differentiable in any finite
interval.
L.M.V.T is applicable to sin x in any interval
[a, b].
3 f (x) = sin x
f (a) = sin a
f (b) = sin b
Again, f ' (x) = cos x
f ' (c) = cos c
Now, using L.M.V.T, we have

a f
f aa f f abf
f a cf =
ab

if a > b, a number c a , b s.t.

sin a sin b
= cos c and we prove as above
ab

that
sin a sin b a b
For a = b

(4)

(5)
sin a sin b = a b = 0 = 0
Hence, the result follows for any a, b.
Question 14: Using Lagranges mean value theorem,
show that cos a cos b a b .
Solution: 3 f (x) = cos x
f (a) = cos a
f (b) = cos b

af
f acf = sin c

f x = sin x
f (x) = cos x = a continuous and differentiable
function in any finite interval.

Rolle's Theorem and Lagrange's Mean Value Theorem

L.M.V.T is applicable to cos x in [a, b] if b > a.


Now, using L.M.V.T,

af af

af af

af

f a f b
f b f a
=
= f c , for some
ab
ba
cos a cos b

= sin c
ab

Again, since sin c 1


Putting (1) in (2), we get

(1)
(2)

cos a cos b
1
ab

af

af

cos a cos b
and we prove as above
ab

that
cos a cos b a b
The result is obvious for a = b.
Hence, the result follows for any a, b.

(4)

Question 15: Prove that


1

x tan

y < x y ,x y

Solution: Let f (x) = tan1 x

bg
f a xf =

f y = tan 1 y

af

1
1+ x

f c =

1
1+ c

tan

x y

x tan
x y

1+ c

Now, taking the mod of both sides, we get


tan

x tan
x y

1
1+c

Now, f (x) = tan1 x which is continuous and


differentiable in any finite interval.

1
1+c

1
1+ c

(1)

< 1 (2)

From (1) and (2),


tan

f a f b
[a, b], we have
= f c
ab

tan

b g = f b xg f b y g = f bcg

Again, c > 0 1 + c 2 > 1

(3)
cos a cos b a b
Similarly, if a > b, then considering the interval

sin c =

bg

f y f x

cos a cos b

= sin c = sin c
ab

af

L.M.V.T is applicable to f (x) = tan1 x in [x, y]


if y > x.
a number c; x < c < y such that
y x

c; a < c < b

835

tan

x tan
x y
1

x tan

<1
y < x y

If x > y, then considering L.M.V.T in [y, x], we have


the same result (3).
Hence, (3) is true x y .
Problems based on, Cauchy's mean value theorem,
Lagranges mean value theorem and Rolles theorem
Statement of Cauchy's Mean Value Theorem:
If two functions f (x) and g (x) defined on [a, b] are
(i) continuous in the closed interval [a, b]
(ii) differentiable in the open interval (a, b).
(iii) g x 0 for any x a , b then there exists
at least one real number c between a and b [i.e.,
c a , b ] such that

af
a f
a f
f abf f aa f
f a cf
=
g abf g aa f
g acf

Note: 1. Cauchy's mean value theorem cannot be


deduced by applying Lagranges mean value theorem
separately to the two functions f (x) and g (x) and
then dividing them since then we get

836

How to Learn Calculus of One Variable

af af
af af

a f
a f

f c1
f b f a
=
g b g a
g c2

where c1 and c2 may not be equal.


2. Cauchy's mean value theorem is also known as
second mean value theorem whereas Lagranges mean
value theorem is known as first mean value theorem.
Examples Worked Out
Question 1: If f (x) and g(x) are differentiable on
0 x 1 such that f (0) = 2, g (0) = 0, f (1) = 6, g (1) = 2
and g x 0 in 0 , 1 then show that there exists
c satisfying 0 < c < 1 and f c = 2 g c .
Solution: It is a question on Cauchy's mean value
theorem since it satisfies all the conditions of Cauchy's
mean value theorem.
Now, according to Cauchy's mean value theorem,
we have

af

a f

af

af

af af af
af af af
f a1f f a0f
f acf

=
for some c, 0 < c < 1
g a1f g a0f
g acf
f acf
62

=
[3 f (1) = 6, g (1) = 2, etc are
2 0 g a cf
f b f a
f c
=
for some c, a < c < b
g b g a
g c

given]

af
af

af
af

af

af

f c
f c
4
=
2=
f c = 2 g c
2
g c
g c

Alternative Method:
We suppose that h (x) = f (x) 2g (x)
f (x) and g (x) are given differentiable on the closed
interval [0, 1].
f (x) and g (x) are continuous on the closed
interval [0, 1] and differentiable in the open interval
(0, 1).
h (x) being the difference of two differentiable
functions in [0, 1] is also differentiable in [0, 1].
h (x) is continuous in [0, 1] and differentiable
in (0, 1).
Now h (x) = f (x) 2g (x)
h (0) = f (0) 2g (0) = 2 2 0 = 2
[3 f (0) = 2, g (0) = 0]

h (1) = f (1) 2g (1) = 6 2 2 = 6 4 = 2


[3 f (1) = 6, g (1) = 2]
Hence, we observe that h (0) = h (1)
All conditions of Rolles theorem are satisfied.
Therefore, there is at least one point x = c where
h ' (x) = 0, i.e.,

af

h x

x=c

af

= 0 h x

x=c

af

af

= f x 2 g x

x=c

=0

a f a f =0
f acf 2 g acf = 0
f acf = 2 g acf which is the required result.
f x 2 g x

x=c

Question 2: If f (x) and g (x) are continuous on [a, b]


and differentiable in (a, b), then show that
f aa f f ac f
a f f abf
a f g abf = ab a f g aa f g acf

f a
g a

af

Solution: (1) Let F x =

where a < c < b.

a f f a xf
a f g axf

f a
g a

= f (a) g (x) g (a) f (x)


(i)
where f (a) and g (a) are constants.
f (x) and g (x) are continuous on the closed interval
[a, b] and differentiable on the open interval (a, b).
F (x) being the difference of two continuous
function in [a, b] and differentiable in the open interval
(a, b) is also continuous in [a, b] and differentiable in
(a, b). Which implies that all conditions of Lagranges
mean value theorem are satisfied by the function f (x)
defined on [a, b]. Therefore, by mean value theorem,
at least one point c, a < c < b, such that

af

F c =

af

af

F b F a
ba

(ii)

af

af af af af
F aa f = f aa f g aa f g aa f f aa f = 0
F abf = f aa f g abf g aa f f ab f
F a x f = f aa f g a x f g aa f f a x f
F acf = f aa f g acf g aa f f acf

(2) 3 F x = f a g x g a f x (form (1))


...(iii)
...(iv)

(v)

Now, putting (iii), (iv) and (v) in (ii), we get

af af af af

f a g c g a f c =

af af af af

f a g b g a f b
ba

837

Rolle's Theorem and Lagrange's Mean Value Theorem

f f aaf g acf g aaf f acf


= f aa f g abf g aa f f ab f
f aa f f ab f
f aa f f ac f

= ab a f
g aa f g ab f
g aa f g acf

a f a x cf = f a x f

f d

ba

af

af

Solution: Given f x exists for all values of x in


[0, 1]
(1)
And also, f (0) = f (1)
(2)

af

f x 1
To show: | f ' (x) | < 1

(3)

bg

Proof: f x exists for all values of x in [0, 1].

f (x) and f ' (x) are differentiable in [0, 1] (4)


and we are given f (0) = f (1)
(5)
(4), (5) f (x) satisfy all conditions of Rolles theorem on [0, 1] at least one value of x = c in the
open interval (0, 1) s.t. f c = 0
(6)
Now we consider three cases, x < c, x > c, x = c
Case (i) When x = c
3 f c = 0 (from (6))

bg

af
f axf = 0
f a xf = 0 = 0 < 1
f a x f < 1 when x = c

...(7)

Case (ii) When x < c


Further, f' (x) is continuous on [0, 1] and differentiable on (0, 1)
Lagranges mean value theorem is applicable
on f ' (x) in [x, c] if 0 < x < c.

bg

f d =

af

b g for some d,

af

af

c x

0 x <d <c <1


f d =

bg

f c f x

f x f c
xc

a f a x cf = f a x f
f bd g x c = f b x g
f a xf
f ad f =
xc

f d

Question 3: For all x in the interval [0, 1], let the second
derivative f x of a function f (x) exists and satisfy
f x 1 .
If f (0) = f (1), show that | f ' (x) | < 1 for all x in the
interval [0, 1].

af

af

[3 f c = 0 from (7)]

(8)

Now, 0 x < 1 x c < 1

af

(9)

a f 1 (on

f x 1 given f d

Again

replacing x by d)
Now, using (8) and (9) in (10), we get

af

LM
N

af

f x
1 3 f d
xc

af

f x

(10)

a f OP
Q

f x
xc

xc

(11)

and | x c | < 1

a f < 1 Which is the required result.

f x

Case (iii): When x > c

af

af

af

f x f c
= f d , 0 < c < d x < 1
xc

af

a f 3 f acf = 0
f a x f = a x cf f ad f
f b xg
f bd g =

f x 0
= f d
xc

xc

Now, 0 c < x < 1 x c < 1

af

Again, f x

bg

f d

(a)
(b)

1 (given)

1 (replacing x by d)

On using (a) in (c), we get


f x
1
xc

...(c)

af

a f xc
(d)
and | x c | < 1 (from (b))
f a x f < 1 which is the required result.
f x

838

How to Learn Calculus of One Variable

An important type of problem


When the values of one function at the end points of
the closed interval [a, b] are given as well as f ' (c) is
required to show to be a constant d where a < c < b.
Note: 1. Values of a function at the end points are
not equal (i.e., f a f b ) and the given function
is continuous in the closed interval and differentiable
in open interval or simply the given function is
differentiable in the closed interval means L.M.V.T is
applicable.
2. In this type of problem functions are not given as
an expression in x like x2, log x, ex etc but in notational
form y = f (x) or g (x) etc are given.
3. When the values of two functions f (x) and g (x) at
the end points of the closed interval [a, b] are given
as well as f ' (c) and g ' (c) are given besides f (x) and
g (x) are differentiable in the closed interval [a, b],
then we are required to Cauchy mean value theorem.
(where a < c < b).

af af

Examples Worked Out


Question 1: If a function f (x) is differentiable in the
closed interval [2, 5] and f (2) = 5, f (5) = 11, then show
that there will be at least one c where 2 < c < 5 such
that f ' (c) = 2.
Solution: 3 f (x) is differentiable in [2, 5]
f(x) is continuous in [2, 5] as well as differentiable in (2, 5).
All conditions of L.M.V.T are satisfied.
According to L.M.V.T, at least one c,
2 < c < 5 s.t.

af

f c =

af

a f L3 f a2f = 5
MN f a5f = 11 are givenOPQ

f 5 f 2
52

11 5
= 2 which was required to show.
52
Problems based on proving inequalities with the help
of Rolles theorem and Lagranges mean value
theorem
=

Exercise 20.9

, show that
2
(a) sin b sin a < b a
1. If 0 < a < b <

(b) a sin a < b sin b


2. Show that log (1 + x) < x, where x > 0.
3. Using the function f (x) = tan1 x, show that

ba
1+ b

< tan 1b tan 1a <

ba
1+ a 2

where 0 < a < b.

, show that
2
(a) | cos b cos a | < | b a |
(b) tan1 b tan1 a < b a
tan b a
>
(c)
tan a b

5. If 0 x < , show that


2
(a) sin x < x
(b) tan x > x
2
x
(c) cos x > 1
2
4. If 0 < a < b <

FG
H

6. Show that 1 + x log x + 1 + x


if x > 0.
7. Show that

IJ
K

1+ x

(a) b a < log b < b a if 0 < a < b


b
a
a

ba
ba
< tan b tan a <
if 0 < a < b < .
(b)
2
2
2
cos a
cos b
Problems based on an important type
Exercise 20.10
1. If f (x) is differentiable in the closed interval [1, 2],
where f (1) = 3, f (2) = 6, show that there exists a
number c, 1 < c < 2, for which f ' (c) = 1.
2. If a function f (x) is differentiable in the closed
interval [0, 3] and f (0) = 10, f (3) = 25, then show that
there exists at least one c, where 0 < c < 3 such that
f ' (c) = 5.
3. If f (x) is differentiable in [1, 2] where f (1) = 3,
f (2) = 3 then show that there exists a point c,
1 < c < 2, for which f ' (c) = 2.
4. If f (x) is continuous in [2, 2] and differentiable in
(2, 2) where f (2) = 3, f (2) = 1, then show that there
1
exists a number c, 2 < c < 2 such that f c = .
2

af

Rolle's Theorem and Lagrange's Mean Value Theorem

5. If the function f (x) and g (x) are differentiable in


[1, 1], then show that there exists a point c,
1 < c < 1 for which

a f f a1f
g a1f g a1f
f 1

=2

a f f acf .
g a 1f g acf
f 1

839

6. If the function f (x) and g (x) be differentiable in the


closed interval [1, 3], show that there exists at least
one point c, where 1 < c < 3, such that

a f f a3f
g a1f g a3f
f 1

=2

a f f acf .
g a1f g acf
f 1

840

How to Learn Calculus of One Variable

21
Monotonocity of a Function

af

On Monotonocity of a Function

af

Definition 1: A function y = f x defined on its


domain D is called an increasing function in D if, for
any two different values of the independent variable
x in D, to the greater value of x, there is always a
greater value of the function ( i.e., value of the
dependent variable y).
That is, a function y = f x defined in an interval
D is said to be increasing or rising in D if y, i.e., f x
increases as x increase in the interval D. That is, if x1
and x 2 are two values of x in the interval D where the
given continuous function f is defined by the formula
y = f x such that x1 < x 2 f x1 < f x 2 then
y = f x is said to be increasing in the interval D.
Geometrically, it means that as one moves from left to
right, values of the function f, i.e., values of the
dependent variable y increase.

af

af

b g b g

af
af

Definition 2: A function y = f x defined on its


domain D is called a decreasing function in D if, for
any two different values of the independent variable
x in D, to the greater value of x, there is a always a
smaller value of the function (i.e., value of the
dependent variable y).
That is, a continuous function y = f x defined
on an interval D is said to be decreasing or falling in D
if y, i.e., f (x) decreases as x increases in the interval D.
That is, if x1 and x 2 are any two values of x in the
interval D where a given continuous function f is
defined by the formula y = f x such that
x1 < x 2 f x1 > f x 2 then y = f x is said to
be decreasing in the interval D.Geometrically, it means
that as one moves from left to right, values of the
function f , i.e. values of the dependent variable y
decrease.

af

b g b g

af
af

y = f (x)

af

x2

Note: A function y = f x is an increasing function


in an interval D

f (x + h) > f (x)
for all x in the interval D, where h is any positive
number such that x + h D .

f (x 2)

f (x 1)

f (x 2)

f (x 1)

y = f (x)

x2

af

Note: A function y = f x is a decreasing function


in an interval D.

f (x + h) < f (x)
for all values of x in the interval D, where h is any
positive number such that x + h D .

841

Monotonocity of a Function

af
af af
af

Remarks: 1. A function y = f x is said to be


increasing at a point a if there is a h-neighbourhood
of the point a in which f x > f a for x > a ,
f x < f a for x < a .
That is, a function y = f x is said to be increasing
at a point x = a , if there exists an open interval
a h , a + h containing a i.e.,
a h , a a , a + h such that f x is
increasing in the open interval a h , a + h .
2. A function y = f x is said to be decreasing at a
point a if there is a h - neighbourhood of the point a
in which f x < f a for x > a , f x > f a for
x < a.
That is, a function y = f x is said to be
decreasing at a point x = a if there exists an open
interval a h , a + h containing a i.e.,
a h , a a , a + h such that f x is
decreasing in the open interval a h , a + h .
3. The statement a function is increasing or
decreasing is not precise unless it is clearly
mentioned in the problem the point or the interval
where it is increasing or decreasing.
4. One should take notice carefully in the language
of definition where x is always supposed to be
increasing while the function may be increasing or
decreasing.
5. It is not necessary that a function must be either
increasing or decreasing on its domain, i.e., the same
function may be increasing in some interval and
decreasing in other interval.
Hence, such a function which is increasing in
certain interval and decreasing in another interval is
termed as a mixed function.
In the adjoining figure, f x is increasing in a , b
and decreasing in b , c .

bg bg

af
af af

af
g

af af

af

af
g

af

On the Use of Terminology

af

According to some authors, a function y = f x is


monotonically increasing, non decreasing or simply
increasing in an interval, if, for any x1 , x 2 belonging
to the interval x1 < x 2 f x1 f x2 and strictly
increasing in the interval if x1 < x 2 f x1 < f x2 .
That is, a nondecreasing or increasing function
differs from a strictly increasing function, i.e., in a
nondecreasing function, two values of the function
(at different values of the independent variable) may
be equal while this is not possible in the case of a
strictly increasing function. Likewise, they define
monotonically decreasing, nonincreasing or simply
decreasing functions in the intervals, i.e., a function
y = f x is monotonically decreasing, nonincreasing
or simply decreasing in an interval if, for any x1 , x2
belonging to the interval x1 < x 2 f x1 f x 2
and strictly decreasing in the interval if x1 < x 2

a f a f
a f a f

af

b g b g

b g b g

f x1 > f x2 .
That is, a monotonically decreasing, nonincreasing
or simply decreasing differs from a strictly increasing
function, i.e., in a nonincreasing function, two values
of the function (at different values of the independent
variable) may be equal while this is not possible in
the case of a strictly decreasing function.
Moreover, a function which is either nondecreasing or nonincreasing is termed as monotone or
monotonic function and a function which is either
strictly increasing or strictly decreasing is termed as
strictly monotone or monotonic function.
On first derivative for increasing and
decreasing functions

y
A

6. The symbol stands for increasing or increasing


functions whereas the symbol stands for decreasing
or decreasing functions.

x=a x=b x=c

af
bg

Theorem 1: A differentiable function y = f x is


increasing on an open interval a , b f x > 0
for all x in (a, b).
Proof: A function y = f x is increasing on (a, b)
f x + h > f x for all x in (a, b), where h is

g af

af

b g

b g

small positive number such that x + h a , b

842

How to Learn Calculus of One Variable

g af
g af
b g af

b
b

f x + h f x > 0 for all x in (a, b)


f x+h f x

> 0 for all x in (a, b)


h
f x+h f x
lim
> 0 for all x in (a, b)
h0
h

bg

b g b g = f bcg

f x 2 f x1
x2 x1

f x > 0 for all x in (a, b).

af
bg

Theorem 2: A differentiable function y = f x is


decreasing on an open interval a , b f x < 0
for all x in (a, b).
Proof: A function y = f x is decreasing on (a, b).

b g

af

b g af
f b x + hg f a x f < 0 for all x in (a, b)
f b x + hg f a x f

< 0 for all x in (a, b)

f x + h < f x for all x in (a, b), where h is a


sufficiently small positive number.

Proof: Let x1 , x 2 a, b such that a x1 < x2 b.


As y = f (x) satisfies both the conditions of Lagranges
mean value theorem a real number c in x1 , x 2
such that

b g b g b
g bg
since x > x and f bc g < 0.
f b x g f b x g = b+ veg b veg = b veg < 0
f bx g f bx g < 0
f bx g < f bx g
f b x g is decreasing in [a, b]
f x 2 f x1 = x 2 x1 f c < 0
2

Note: in the right hand side of the equality

Theorem 3: If y = f x is a continuous function on


[a, b] and differentiable on (a, b) then f x > 0 for
all x in (a, b) y = f x increases on [a, b].

b g b g b g bx x g ,
the sign of b x x g is always positive whereas the
sign of f bcg is positive or negative according to the
hypothesis. This is why the right hand of the equality
f b x g f b x g = f bcg b x x g has the same sign
as the sign f bcg because b x x g is always

Proof: Let x1 , x 2 a, b such that a x1 < x 2 b.


As y = f x satisfies both the conditions of
Lagranges mean value theorem in x1 , x 2 , this is
why a real number c in x1 , x 2 such that

How to know that a given monotonic function


y = f x defined in an interval satisfies the
>
condition x1 < x 2 f x1 < f x 2

lim

h0

g af

f x+h f x
< 0 for all x in (a, b)
h

bg

f x < 0 for all x in (a, b).

af
bg

af

b g

bg

b g = f acf which further

f x 2 f x1
x 2 x1

b g b g b
g af
since x > x and f bcg > 0.
f bx g f bx g > 0
f b x g > f b x g for x > x
y = f a x f is increasing in [a, b].
Theorem 4: If y = f a x f is a continuous function in
[a, b] and differentiable in (a, b), then f b x g < 0 for
all x in (a, b) y = f b x g decreases in [a, b].
f x 2 f x1 = x 2 x1 f c > 0 ,
2

f x2 f x1 = f c
2

positive.

bg

b g b g

1. Take any two particular numbers namely c1 , c2


given interval such that c1 < c2 .
2. Substitute c1 and c2 in the monotonic functions
and see whether f c1 > f c2 or f c1 < f c2 .

bg b g

bg b g
How to know that a derived function f b x g 0 in an
interval where a given function y = f b x g is defined
>
<

1. Take any particular number namely c given


interval.
2. Substitute c in the derived function f x .
3. See whether f c > 0 or f c < 0 for all c.
f c > 0 f x > 0 , x given interval
and f c < 0 f x < 0 , x given interval.

bg
bg

bg
bg
bg

bg

bg

Monotonocity of a Function

On the methods of examining monotonocity in a


given interval

bg

There are two methods of examining or showing a


function y = f x to be increasing or decreasing in
an interval.
1. Method of definition.
2. Method of first derivative test.
On method of definition
It consists of following steps:
1. Let x1 and x 2 be any two real values of x in the
given interval such that x1 < x 2 or x1 > x 2
2. Find the difference
f x1 f x2 or f(x2) f(x1).
3. f x1 f x 2 > 0 for x1 > x2 f x
is increasing in the given interval
Also, f x1 f x2 < 0 for x1 > x2
f x is decreasing in the given interval
is decreasing in the given interval.)

b g b g
b g b g
b g b g
bg

bg

Note: All that is necessary in the method of definition


is to examine the sign of the difference f x1 f x2
if x1 > x 2 where x1 , x 2 given interval.

b g b g

On the method of first derivative test

bg

Note: 1. All that is necessay in the method of first


derivative test is to examine the sign of the first
derivative f x in the given interval.
2. When no method is mentioned in the problem,
there may be the use of first derivative test.
3. Show that the given function y = f x is
increasing or decreasing in the whole of its domain
when the interval ( where f x is defined) is not given
in the problem.
Now problems are divided in different categories
to explain their method of procedure.

af

af

(ix) x
Examples worked out:

af

x
is monotone increasing ,
1+ x
x > 0 , without the use of derivative.
Solution: Let x1 and x 2 be any two values of x s.t.
x1 > x 2 > 0
1. Show that f x =

b g

Now, f x1 =

b g

f x2 =

x1
1 + x1

x2
1 + x2

b g b g 1 +x x
x b x 1g x b x + 1g
=
b1 + x g b1 + x g

f x1 f x2 =

af

x2
1 + x2

1. Find f x
2. See whether f x >< 0 , x given interval.
3. f x > 0 , x given interval.
f x is increasing in the given interval and
f x < 0 , x given interval.
f x is decreasing in the given interval.

bg
af
bg
af

af

Problems based on showing a function y = f x to


be increasing or decreasing when ever any one of the
following restrictions is imposed on it ( by using
definition of increasing and decreasing function )
(i) x a (ii) x < a (iii) x a
(iv) x > a (v) x < 0 (vi) x 0
(vii) x > 0 (viii) x a

It consists of the following steps:

af

Category A:

843

x1 x 2
1 + x1 1 + x 2

f a f > 0 since x > x > 0


which f b x g f b x g > 0
f b x g > f b x g when x > x which means
f b x g is an increasing function for x > 0
Hence, proved.
2. Show that f b x g = x for x 0 is an increasing
=

function without the use of derivative.


Solution: Let x1 and x2 be any two values of x, such
that x1 > x 2 0

b g

Now, f x1 = x12

b g
f bx g f bx g = x
f x2 = x2 2
1

x1 > x 2 0

2
1

x 2 2 > 0 when

844

How to Learn Calculus of One Variable

b g b g
f b x g > f b x g when x > x which means
f b x g is an decreasing function for x 0 .
Hence, proved.
3. Show that f b x g = x for x 0 is a decreasing
f x1 f x2 > 0
1

function.

b g

Now f x1 = x12

b g
f bx g f bx g = x
f x2 = x12
1

x2 2 < 0 when

x2 < x1 0

b g b g
f b x g < f b x g when x < x < 0
Hence , f b x g is a decreasing function for x 0
4. Show that f b x g = x + 3x + 3x 100
which f x1 f x2 0
1

in increasing for all x R .


Solution: Let x1 and x 2 be any two values of x R
s.t. x1 > x 2

b g

b g
f bx g f bx g = ex
e x + 3x + 3x

f x2 = x2 3 + 3x2 2 + 3x2 100


1

x13 3 x12

= x1 x 2
3

3 x12 + 3x1 100

100

+ 3x1 100 x 2 + 3x 2 2 3x 2 + 100


3

i+3d

x12

x2

i + 3 ax

x2 > 0

when x1 > x 2

b g b g
f b x g > f b x g when x > x
f
b
Hence, x g is an increasing function for all values
of x R .
5. Show that f b x g = 3x + 1 is an increasing funcWhich

f x1 f x 2 > 0 when x1 > x 2 .


1

tion on R without using derivative.


Solution: Let x1 and x 2 be any two values of x R
(i.e. x1 , x 2 R ) s.t x1 > x 2

constants and a > 0 is an increasing function of x


(without using the derivative) x
Solution: Let x1 and x 2 be any two values of x R
(i.e. x1 , x 2 R ) s.t x1 > x2

b g

Now, f x1 = a x1 + b

b g
f ax f f ax f = a x + b a x b
= a x a x = a ax x f
f a x f f a x f = a a x x f > 0 when x > x
f b x g f b x g > 0 when x > x
f b x g > f b x g when x > x
Hence, f a x f is an increasing function x .
f x 2 = a x2 + b
1

Now, f x1 = x13 + 3x12 + 3x1 100

Solution: Let x1 and x 2 be any two values of x s.t


x2 < x1 0

b g
f b x g = 3x + 1
f b x g f b x g = 3x + 1 3x 1
= 3 a x x f > 0 when x > x .
Which f b x g f b x g > 0 when x > x
f b x g > f b x g when x > x .
Hence, f b x g is an increasing function on R.
6. Show that f b x g = ax + b where a and b are
Now f x1 = 3x1 + 1

Category B:
Type 1: Problems based on showing a function
y = f x to be increasing or decreasing at a point
x = a by using derivative test.

af

Working rule:

af

dy
= f x
dx
dy
= f x
= f a
2. Find
x=a
dx x = a
3. If f a = + ve number, then f x is an increasing
function at the point x = a
4. If f a = ve number, then f x is decreasing
said function at point x = a .
1. Find

LM OP
N Q
af
af

bg

af
af

bg

Monotonocity of a Function

af

af

Note : f a = 0 , then f x is said to be stationary


at the point x = a
Examples worked out:
1. Show that the function y = 2 x 3 3x + 1 is
increasing at x = 1 and decreasing at x = 0
Solution : y = 2 x 3 3x + 1

dy
= 6x 2 3
dx

L dy O
Now , M dx P
N Q

= 6x 3
x=1

x =1

= 6x 3
2

x=0

log x
x>0
x
1
x log x 1
1 log x
x
f x =
=
2
2
x
x
Now, we are required to determine the sign of

1 log x
x

x=0

= 6 0 3 = 3 = ve number

Hence, y = 2 x 3 3x + 1 is a decreasing function at x = 0

with the help of given restriction :

Q x > 3 and 3 > e Q e = 2 718281


x>e
log x > log e e
log x > 1

Type 2: Problems based on showing that a given


function y = f x is increasing or decreasing when
any one of the following restrictions are imposed on
it.
(i) x > a (ii) x a (iii) x < a
(iv) x a (v) x > 0 (vi) x 0
(vii) x < 0 (viii) x 0 (ix) x a

af

(x) x (where a is any + ve number) by using


the derivative
Examples worked out:
1. Show that the function y = x 4 4 x + 1 is a
decreasing function when x < 1 and is an increasing
function when x > 1

bg

Solution: y = f x = x 4 4 x + 1
3

f x = 4x 4 = 4 x 1

log x 1 > 0
1 log x < 0

1 log x
2

x
1 log x
x

< 0 which means

= ve

bg

af

af

f x = ve , i.e; f x < 0 which f x


is decreasing function when x > 3 .

f a2 2+x xf

3. Show that y = log 1 + x

is an

increasing function of x for all values of x > 1

bg

g b2 2+x xg
R| 2 b2 + xg 2 x U|
1
S
f b xg =
V|
1 + x | b2 + x g
W
T

Solution: y = f x = log 1 + x

3
Now we have to determine the sign of x 1
with the help of given restriction:

Qx <1

af

af

Hence, y = 2 x 3 3x + 1 is an increasing
function at x = 1

af

bg
af
that f a x f is a decreasing function when x < 1
log x
2. Show that f a x f =
is a decreasing function
x

f x = ve , i.e; f x < 0 which means

Solution: f x =

= 6 1 3 = 6 3 = 3 = + ve number

L dy O
Next, M P
N dx Q

x3 < 1
3
x 3 1 < 0 which x 1 = ve

for x > 3 .
2

845

846

How to Learn Calculus of One Variable

=
=

1
4

1+ x
2+x

b2 + xg 4 b1 + xg
g b1 + xg b2 + xg
2

4 + x 2 + 4x 4 4x

b1 + x g b2 + x g

x2

b1 + x g b2 + x g

Now we have to determine the sign of

2x 9 0

9 2 x 0 which means that 9 2 x = ve


...(i)
9
which f x is decreasing function for x
2

af

bg

5. Show that f x = 7 x 2 + 11x 9 is decreasing for x > 1 .

a1+ xfa2 + xf whose sign depends only on the Solution: f b xg = 7 x + 11x 9


factor a1+ x f in denominator because x and
f b x g = 14 x + 11
a2 + xf are always + ve for being perfect squares.
Now we are required to determine the sign of
Now we determine the sign of b1 + x g with the help
b 14 x + 11g with the help of given restriction:
of given restriction:
2

Q x >1

Q x >1

1 + x > 0 which means 1 + x = + ve


Hence, each term in the numerator and
x2
2 is + ve.
denominator of
1+ x 2+ x

gb

b g b1 + xgbx 2 + xg = + ve which means


that f a x f > 0
Hence, f a x f is an increasing function for x > 1 .
Note : Whenever the derived function f a x f contain
2

f x =

a linear factor along with factors which are perfect


squares, we examine the sign of the linear factor only
with the help of given restriction.

af

4. Show that f x = 6 + 9 x x is decreasing for

9
x .
2

9
Q x
2

2x 9

x > 1 which
x > 1.

af
af
f a x f is decreasing function for
3

6. Show that y = 2 x + 3x 12 x + 7 is increasing


and positive for x > 1 .

bg

3
2
Solution: y = f x = 2 x + 3x 12 x + 7

af

f x = 6 x 2 + 6 x 12 = 6 x 2 + x 2

af
f a xf = 9 2x
Now, we are required to determine the sign of
b9 2 x g with the help of given restriction :

Solution: f x = 6 + 9 x x 2

14 x < 14
14 x + 11 < 14 + 11
14 x + 11 < 3 < 0
14 x + 11 < 0 which means that (14x + 11)
= ve
This is why f x = ve , i.e; f x < 0 for

Now we are required to determine the sign of


x + x 2 with the help of given restriction:
2

Q x >1

x2 > 1
x2 + x > 1 + x
x2 + x 2 > 1 + x 2

x 2 + x 2 > x 1 > 0 Q x > 1 which


x 2 + x 2 = + ve

bg

af

f x = + ve , i.e; f x > 0 which means that


f x is an increasing function when x > 1

af

Monotonocity of a Function

af

Next we are required to show f x to be positive.

af

f 1 =0

af

af

af

and x > 1 f x > f 1 f x > 0 (Q f (1) = 0)

af

x>0

1+ x >1> 0

Hence, each term in the numerator and denomi-

x2

gd

which means that f x is positive.


Hence, f x is increasing and positive for x > 1

nator of 1 + x 2 + x 2

Note: The following is a sufficient test for positivity


of a function for x > 0 . If
(i) The function f x is continuous in 0 x < b

f x =

af

af

af
af

(ii) f 0 is non negative.


(iii) f x > 0 , when 0 < x < b then f x > 0 in
0 < x < b.
This test remains valid even when b is replaced
by . Hence these conditions are sufficient for f x
to be positive for x > 0 .

af

af

7. Show that the function y = log 1 + x


is an increasing function for x > 0 .

2x
2+x

bg

b g
2 b2 + x g 1 a 2 x f
1
f axf =

1+ x
b2 + x g

x2

b1 + xg b2 + xg

af

2x

f x =

d1 + x i

2 2

with the help of given restriction:

bg

Now we have to determine the sign of

af

f x =

b1 + xg b2 + xg

af

2x 0

b2 + xg 4 b1 + xg = 4 + 4 x + x 4 4 x
=
b1 + xg b2 + xg
b1 + x g b2 + xg
=

8. Examine the monotonocity of the function


1
f x =
for x 0 .
1 + x2
1
Solution: f x =
1 + x2

x0

2 2

d1 + x i

1
4 + 2 x 2x

2
1+ x
2+ x
1
4
=

2
1+ x
2+x

is +ve. This is why

a f b1 + xgx 2 + x = + ve which means that


d i
f a xf > 0.
Hence, f a x f is an increasing function for x > 0

2x

Now we are required to determine the sign of

2x
Solution: y = f x = log 1 + x
2+ x

847

whose sign depends only on the

factor (1 + x) in denominator because x2 and (2 + x)2


are always positive for being perfect squares.

bg
and f a0 f = 0

2x

e1 + x j

2 2

af

f x = + ve , i.e; f x > 0

af

...(1)
...(2)

Hence, (1) and (2) f x in an increasing


function when x 0 .

af

9. Show that f x = x 2 is a decreasing function for


x 0.

af
af

Solution: f x = x 2
f x = 2x
Now, we are required to determine the sign of 2x
with the help of given restriction:
Q x0

848

How to Learn Calculus of One Variable

2 x 0 which 2 x = ve

bg
and f a0 f = 0

12. Show that y = 6 x + 3 x 2 + x 3 is an increasing

af

f x = ve , i.e; f x < 0

af

...(1)

function of x, x

...(2)

Solution: y = f x = 6 x + 3 x + x

af

af

Hence, (1) and (2) f x is a decreasing


function when x 0 .

f x = 6 + 6 x + 3x = 3 x + 2 x + 1

10. Show that f x = x 2 is an increasing function


for x 0 .

f x = + ve x , i.e; f x > 0 , x

af

af

af

Solution: f x = x 2 f x = 2 x
Now we have to determine the sign of 2x with the
help of given restriction:
Q x0

2 x 0 which 2 x = + ve

bg
and f a0 f = 0

af

f x = + ve , i.e; f x > 0

...(1)
...(2)

af

Hence, (1) and (2) f x in an increasing


function when x 0 .

af

x
11. Show that f x =
is an increasing function
1+ x
for x 1 .

af

Solution: f x =

x
1+ x

a f a x + 1fa1 1+xfx a1 + 0f

f x =

>0x

af

af

Hence, f x is an increasing function x , i.e;

af

f x is increasing in , + .

bg

1
, Examine the monotonocity.
x
1
Solution: f x = , x 0
x
1
f x = 2 , x 0 < 0 for x 0.
x
f x < 0 , x 0

13. f x =

af

af

a
b

bg

af

f
g

Hence, f x is a decreasing function, x excluding x = 0 which means f x is a decreasing function


in , 0 and in 0, .

af
b g

14. Examine the monotonocity of the function


f x = 2x + 7, x .
Solution: f x = 2 x + 7 f x = 2 which is +ve

af

af

af

af

af

15. Show that f x = 5 x + 1 is a decreasing


function, x .

af

af

Solution: f x = 5 x + 1 f x = 5 which is

> 0 for

x 1
b1 + xg b1 + xg
f b x g = + ve , i.e; f a x f > 0 for x 1 .
Moreover, f a x f is undefined at x = 1
Hence, f a x f is an increasing function for every
value of x excluding x = 1 , i.e; f a x f is an increasing function for every value of x 1 , i.e. in
b , 1g b 1, g .
2

bg

Hence, f x is an increasing function x .

x +1 x

=3 x +1

a x + 1f 1 x a1 + 0f
=
a1 + xf
=

af

ve which means f x < 0 x .

af

Hence, f x is an decreasing function x .


16. Show that the function y = x 3 + x increases
every where.

af

Solution: y = f x = x 3 + x

af
bg

f x = 3x 2 + 1 which is > 0 , x
f x = + ve
Hence, f x increases every where.

af

Monotonocity of a Function
1

17. Show that the function y = tan x x decreases


every where.
Solution : y = f x = tan 1 x x

af

af

f x =

1
1 1 x2

=
1
1 + x2
1 + x2

x2
< 0, x
1 + x2
f x = ve
Hence, f x decreases every where.
=

bg

af

af

Type 3: Problems based on showing y = f x to be


increasing or decreasing in an open interval (a, b) by
using derivative.

af

Definations: 1. A function f x is called increasing


in an open interval (a, b) if it is increasing at every
point within this interval (a, b).
2. A function f x is called decreasing in an open
interval of it is decreasing at every point within this
interval (a, b).

af

Test for monotonocity in an open interval:

af

An increase and decrease of a function y = f x is


tested by the sign of its derivative f x . If in some
interval (a, b) , f x > 0 , then the function
y = f x increase in this open interval (a, b) and if
f x < 0 , then the function decrease in this interval
(a, b).

af

af

bg

af

Working rule:
1. Find f x
2. Determine the sign of f x with the help of given
interval (a, b).
3. If f x = + ve in (a, b) then y = f x increases
in (a, b) and if f x = ve in (a, b) then y = f x
decreases in (a, b)

af

af

af
Note: If f a x f = T l f a x fq

af

af

af

Where T = any trigonometric function sin, cos tan,


cot, sec and cosec.
And f x = an expression in x for the angle of
trigonometric function which is generally linear in x
(i.e ax + b ).

af

849

Then from the given interval, we derive the angle


f x by using various mathematical manipulations
s.t. it determine in which quadrant f x lies from which
we observe the + ve or ve sign of trigonometric
derived function using the rule of

af

af

sin

all

tan

cos

Remember: 1. Whenever we need to know the sign


of a trigonometric function of any angle, we consider
the quadrant in which its terminal (moving) side lies.
e.g.: (i) Since the terminal side of 120 o is in the first
quadrant, therefore, sin 120 o = + ve
(ii) Since the terminal side of 220 o lies in the third
quadrant, therefore cos 220 = ve
o
(iii) The terminal side of 60 lies in the fourth

quadrant, therfore, sin 60 = ve and cos ( 60 )


= + ve
2. Let f x = angle of any trigonometric function
sin, cos etc. then,

means the angle f x lies in the


(i) 0 < f x <
2
Ist quadrant.
(ii) 0 < f x < means the angle f x lies in the
Ist and 2nd quadrant.
3
(iii) < f x <
means the angle f x lies in
2
the 3rd quadrant.
(iv) < f x < 2 means the angle f x lies in the
3rd or 4th quadrant.

(v) < f x < means the angle f x lies in the


2
2
Ist or 4th quadrant.

af

af

af

af

af

af

af

af

af

af

af

af

Type 3: Problems based on showing y = f x to be


increasing or decreasing in an open interval.
Examples worked out:

af

1. Show that: f x =

4x 2 + 1
x

850

How to Learn Calculus of One Variable

F 1 , 1 I and
H 4 2K
F1 I
an increasing function in the interval H , 1K .
2
4x + 1
1
= 4x +
Solution: f a x f =
x
x
1
4x 1
f a xf = 4
=
x
x
is a decreasing function in the interval

Now we have to determine the sign of

4x2 1
x2

with the help of given intervals.


1
1
1
< x < x2 < 4x2 < 1
4
2
4
2

4 x 1 < 0 4 x 1 = ve

...(1)

and x 2 being a perfect square, it is always + ve, i.e;

x = + ve
2

...(2)

4x 1
= ve
x2
f x = ve which means that f x is a
2

From (1) and (2), we conclude,

af

decreasing function in the given interval


Next, we consider the interval

x>

F 1 , 1I
H 4 2K

F 1 , 1I with the help


H2 K

of which we determine the sign of

4x2 1
x2

...(1)

and x 2 being a perfect square, it is always + ve, i.e;


...(2)

4x 2 1
= + ve
From (1) and (2), we conclude,
x2

bg

gb

af
F1 I
increasing function in the given interval H , 1K
2

f x = + ve which means that f x is a

3x + 1 3x 1
1
9x2 1
=
=
2
2
3x
3x
3x 2

1
1
1
< x < x2 < 9x2 < 1
9
3
9
2

9 x 1 < 0 9 x 1 = ve

bg

...(1)

af

f x = ve which means that f x is a


decreasing function in the given interval
Next, we consider the interval

x>

4 x 1 > 0 4 x 1 = + ve

x 2 = + ve

F 1 , 1 I and an increasing
H 9 3K
F1 I
function in the interval H , 1K
3
1
Solution: f a x f = 3x +
3x
1F 1I
f a xf = 3 + G
J = 3 3x1
3H x K
function in the interval

F 1 , 1I
H 9 3K

F 1 , 1I with the help


H3 K
9x2 1
3x 2

of which we consider the sign of

1
1
2
2
x > 4x > 1
2
4
2

1
is a decreasing
3x

Now we have to determine the sine of

af

af

2. Show that f x = 3x +

1
1
2
2
x > 9x > 1
3
9

9x2 1 > 0

bg

af

f x = + ve which means that f x is an


increasing function in the given interval

F 1 , 1I .
H3 K

3. Show that the function y = 2 x 3 + 3 x 2 12 x + 1

decreases in the interval 2 , 1

af

Solution: y = f x = 2 x + 3 x 12 x + 1
3

Monotonocity of a Function

af

f x = 6 x + 6 x 12 = 6 x + x 2

{
}
= 6 mx b x + 2g b x + 2gr
= 6 b x 1g b x + 2g
2

= 6 x + 2x x 2

Now we have to determine the sign of

gb

6 x 1 x + 2 with the help of given interval.


2< x <1

x < 1 and x > 2


x > 2
x + 2 > 0 x + 2 = + ve

...(i)

x < 1 x 1 < 0 x 1 = ve

...(ii)

and 6 = a + ve number
From (i) , (ii) and (iii), we conclude that

...(iii)

gb g
f b x g = ve
Hence, f a x f decreases in the interval b 2 , 1 g
6 x1 x +2 <0

4. Show that the function y =

2 x x 2 increases

b g

in the interval 0, 1 and decreases in the interval

b1, 2 g.

bg

Solution: y = f x =

af
2
2 b1 x g

f x =
=

2x x2 , 0 x 2

1
2x x 2

g
b1 xg
x e 2 xj

b1 xg

=
2 2x x
2x x2
Now, we have to determine the sign of
2

2 2x

b1 x g with the help of given interval.


x e 2 xj
f b x g = ve for 0 < x < 1 , since b1 x g > 0
Hence, f a x f increases in interval b0, 1g
For 1 < x < 2,

851

b1 xg < 0 since b1 xg = ve
x e 2 xj
f b x g = ve
Hence, f a x f decreases in b1, 2 g .
5. Show that f a x f = cos x is a decreasing function
F I
on H 0 , K .
2
Solution: f a x f = cos x
f a x f = sin x
Now, we have to determine the sign of sin x with
the help of given interval.

0 < x < which means x lies in the first quadrant


2
where sin x > 0 which further sin x < 0 .

af

F I which f a x f is
H 2K

Thus f x = ve on 0 ,

F I .
H 2K

decreasing on 0 ,

F I .
H 2K

6. Show that cos 2 x is decreasing on 0 ,

af

af

Solution: f x = cos 2 x f x = 2 sin 2 x

Now, we have to determine the sign of 2 sin 2 x


with the help of given interval.

0<x<
which 0 < 2x < which further
2
means 2x lies in the Ist or 2nd quadrant where
sin 2 x > 0 which sin 2 x < 0 sin 2 x < 0

a
f
F I
Hence, f a x f = ve on H 0 , K which f a x f
2
F I
is a decreasing function on 0 ,
H 2K
which means 2 sin 2 x = ve

7. Show that f (x) = tan x is an increasing function

F I .
H 2K

on 0 ,

852

How to Learn Calculus of One Variable

af

af

Solution: f x = tan x f x = sec x

< 2x +
2

F
H

< 2 which means 2 x +


4
4

I
K

F I is
H 4K
F I
F I
ve, ie; sin 2 x + < 0 which 2 sin 2 x + > 0
H 4K
H 4K
where sec x > 0 which further sec x > 0 i.e;
F 3 7 IJ which
Hence, f a x f = + ve on G ,
H8 8K
sec x = + ve
F I
F 3 7 IJ
Hence, f a x f = + ve on H 0 , K which f a x f
f a x f is an increasing function on G ,
2
H8 8K
I
F
is increasing on H 0 , K .

2
9. Show that f a x f = + sin x is an increasing
2
F I is an increasing
8. Show that f a x f = cos 2 x +
H 4K
F I
function on H , 0K .
3
3
7
<x<
function for
x
8
8
Solution: f a x f = + sin x
2
F I
Solution: f a x f = cos 2 x +
H 4K
1
f a x f = + cos x
2
F I
f a x f = 2 sin 2 x +
H 4K
Now we have to determine the sign of
Now we have to determine the sign of
F 1 + cos xI for x in the given interval.
F I with the help of given interval. H 2 K
2 sin 2 x +
H 4K

F I < cos x < cos 0


< x < 0 cos
H 3K
3
F I is ve which
f ' (x) will be +ve when sin 2 x +
H 4K
1
< cos x < 1
I
F
2
is only possible when 2 x +
H 4 K lies in the 3rd or 4th
1
1 1
1
Now, we have to determine the sign of sec x
with the help of given interval.

0 < x < which menas x lies in the first quadrant


2

lies in the 3rd or 4th quadrant where sin 2 x +

quadrant.
Now consider the given interval.
3
7
<x<
8
8
3
7

< 2x <
4
4
7
3

+ < 2x + <
+
4
4
4
4
4
8
< 2x + <
= 2
4
4

0<

<

+ cos x < 1

1
1
+ cos x <
2
2

Which means that

F cos x 1 I = + ve
H 2K

F cos x 1 I > 0
H
2K

i.e;

Monotonocity of a Function

F , 0I f a xf in
af
H 3 K
F I
increasing on , 0 .
H 3 K
F I is an increas10. Show that f a x f = cos 2 x +
H 4K
F 3 , 5 I .
ing function on the interval
H8 8K
F I
Solution: f a x f = cos 2 x +
H 4K
F I
f a x f = 2 sin 2 x +
H 4K
Hence, f x = + ve on

Now we have to determine the sign of

F I for x in the given interval.


2 sin 2 x +
H 4K
F I is ve.
f a x f will be +ve when sin 2 x +
H 4K
F I
Which is possible only when H 2 x + K lies in the
4
F I lies on the interval
3rd or 4th quadrant , i.e; 2 x +
H 4K
3

b , 2g or FH , 2 IK .

F 3 , 5 I which means
H8 8K
F 3 5 I
f a x f is an increasing function on H ,
8 8 K
af

Hence, f x = + ve on

5
3
+ < 2x + <
+
4
4
4
4
4

af

f =
=

cos

sin

a cos sin f

a tan f

af

on letting Z = tan = g

af

g =

dz
2
= 1 sec < 0
d

FQ sec
H

> 1 for 0 < <

I
K

Which Z is a decreasing function for

bg bg

g < g 0 = 0, 0 < <


2
2
cos

> 0 for 0 < <

...(1)
...(2)

af

F
H

<
which means 2 x +
4
2
4

IJ
K

>0
4

2
Hence, f is a decreasing function of for

0<<
2
0<<

F
H

lies in the 3rd quadrant where sin 2 x +

FG
H

af

Solution: y = f =

From (1) and (2) we conclude that f < 0 for

3
5
< 2x <
4
4

which sin 2 x +

.
2

for 0 < <

Also

< 2x +

sin
is a decreasing function of

11. Show that

Now we consider the given interval.

3
5
<x<
8
8

853

I
K

I
K

<0
4

af

x
12. Show that sin x is an increasing function of x in
the interval 0 < x <

854

How to Learn Calculus of One Variable

af

af

sin x x cos x
x
f x =
2
sin x
sin x

Solution: f x =
2

Now, sin x > 0 for 0 < x <

...(i)

and tan x > x for 0 < x <


Now since tan x > x

sin x
> x sin x > x
cos x

FQ cos x > 0 for 0 < x < I


...(ii)
H
2K
From (1) and (2) we conclude that f a x f = + ve in
af

Thus f x =

af

f x =

cos x sin x

x
is an increasing function in
sin x

4 sin

a2 + cos f is an increasing
F I
function of in the interval H 0 , K .
2
Solution: y =

4 sin

2 + cos

cos x sin x
2 + sin 2 x
Now we have to determine the sign of
=

cos x sin x
for x in the given interval.
2 + sin 2 x

0 < 2 x < which 2x lies in


4
2
sin
2x
the first quadrant where
is positive and for
this reason 2 + sin 2 x > 0

F
H

dy cos 4 cos
=
> 0 for 0 ,
2
2
d
2 + cos

Hence, y =

I
K

4 sin
is an increasing func2 + cos

F I
H K

tion of in 0 ,
.
2

tan x < tan


4
4

tan x < 1
sin x

<1
cos x
sin x < cos x , 0 < x <

sin x cos x < 0

Now differentiating y w.r.t and then simplifying,

1 + sin x + cos x + 2 sin x cos x

Now again x <

13. Prove that y =

0<x<

F0 , I .
H 2K

we get

asin x + cos xf , x > 0


F I .
is an increasing function in 0 ,
H 4K
Solution: f a x f = tan a sin x + cos x f
1
f a xf =
acos x sin x f
1 + asin x + cos x f
1

cos x
sin x x cos x > 0

the given interval 0 < x <

af

14. Show that f x = tan

cos x sin x > 0


Thus, we see that Nr and Dr both > 0 seperately
of the derived function f x which means

af

f a x f = + ve in the interval F 0 , I
H 4K
Hence, f a x f is an increasing function of x in the
F I
interval H 0 , K .
4

855

Monotonocity of a Function

af

Type 4: Problems based on showing y = f x to be


increasing or decreasing in a closed interval a , b
by using derivative.
Question: How to test monotonocity of a function

af

y = f x in a closed interval a , b .
Answer: we have the following rule to test for

monotonocity in a closed interval a , b .

af

af
and if f a x f 0 in closed

1. If f x 0 in a , b , then f x is said to be
non-decreasing on a , b

af

interval a , b then f x is said to be non-increasing

b g
2. If f a x f > 0 in a , b , then f a x f is said to be
increasing in a , b and if f a x f < 0 in the interval
a , b , then f a x f is said to be decreasing in the
interval a , b for all x b a , bg .
in closed interval a , b for all x a , b .

Working Rule:

af

1. Find f x .
2. Determine the sign of f x for x in the given

af

closed interval a , b .

af

af
f a x f = ve , then

3. If f x = + ve , then y = f x increases in the


closed interval a , b and if

af

y = f x decreases in the closed interval a , b .

af

<x<
which x lies in the 4th or Ist
2
2
quadrant where cos x is + ve

F I
bg
b af g
H 2K

F I = 0 , f F I = cos F I = 0
= cos
H 2K
H 2K H 2K
Hence, f a x f is increasing in the closed interval
LM , OP.
N 2 2Q
2. Show that f a x f = cos x is decreasing for
f x = + ve i.e; f x > 0 and f

0 x .

af
f a x f = sin x

Solution: f x = cos x

Now, we have to determine the sign of sin x


with the help of given interval .
0 < x < which x lies in the first and
second quadrant whose sin x is + ve , i.e; sin x > 0
sin x < 0
f x = ve and f 0 = sin 0 = 0 , f
= sin = 0

bg

af

af

af

Hence, f x is decreasing on 0 , .

FG
H
0 , + g .

3. If y = 2 x tan x log x + 1 + x

IJ
K

Show

Type 4: Problems based on showing y = f x to be

that y is increasing in

increasing or decreasing in a closed interval


by using derivative.

Solution: y = 2 x tan x log x + 1 + x

a, b

Examples worked out:

af

1. Show that f x = sin x is increasing for

x .
2
2
Solution: f x = sin x

af
f a x f = cos x

Now, we have to determine the sign of cos x with


the help of given interval:

1
dy
=2

dx
1 + x2

=2

and

F dy I
H dx K

F 1
GG
H 1+ x

1 + x2

IJ
K

b g

I
JJ > 0 for 0 < x < +
K

=2 1+1 =0
x=0

1
1+ x

FG
H

Hence, y is increasing on 0 , + .

856

How to Learn Calculus of One Variable

b a , ag

i.e;

Question: What do you mean by interval of


increasing and decreasing of the function y = f x .

4.

Answer: (i) The interval in which f x is positive

means x lies outside the interval a , b

i.e;

Type 5: Problems based on finding the interval of


increasing and decreasing function y = f x .

which means x lies outside

af

af

af

bi.e; f a xf > 0g is called the interval of increasing of


the function y = f a x f .
(ii) The interval in which f a x f is negative
bi.e; f a x f < 0g is called the interval of decreasing
function y = f a x f .
Working rule: To find the interval of increasing or
decreasing function or the values of x for which the
given function y = f x is increasing or decreasing,
we adopt the following working rule :
1. Find f x

af

af
2. Put f a x f > 0 to find the interval of an increasing
function and solve f a x f > 0 for x whose values
determine the required interval of increasing of
y = f a x f.
3. Put f a x f < 0 to find the interval of decreasing
and solve f a x f < 0 for x whose values determine
the required interval of decreasing of y = f a x f .
Remember: If f a x f = a quadratic equation in x
(i.e algebraic quadratic in x) then the following hints
to solve the quadratic inequality f x > 0 or

af

af

f x < 0
helpful.

af

af

or f x 0 or f x 0 are very

1. x 2 a 2 < 0 a < x < a x < a


or x 2 a 2 0 a x a x a

b g b x b g < 0 a < x < b bb > a g


or b x ag b x bg 0 a x b bb > ag
3. x a > 0 x < a or x > a x > a

g b

a, a

or x a a x a or x a x a

g b g
or a x a fa x bf 0 x a or x b , (b > a) which
means x lies outside the interval ba , bg i.e;
x b , a b , + g
Note: 1. If a function is increasing ( or, decreasing)
in an open interval, then it is also increaing (or,
decreasing) in the corresponding closed interval i.e;
if f x is an increasing (or decreasing) in the open
interval a < x < b then it is also increasing (or
decreasing) in the closed interval a x b
respeectively.

af

Type 5: Problems based on finding interval in which


a given function y = f x increases or decreases.

af

Examples worked out:

af
f a xf = 2 x

2
1. Find the interval in which f x = x increases or
decreases.

af
Now, for f a x f to be increasing , f a x f > 0
2 x > 0 x > 0 b0 , g
Hence, the required interval for f a x f = x to be
increasing is x 0 = 0 , g
Next, for f a x f to be decreasing, f a x f < 0
2 x < 0 x < 0 b , 0g
Hence, the required interval for f a x f = x to be
decreasing is x 0 b , 0 .
2. Find the interval in which f a x f = x 2 x
increases or decreases.
Solution: f a x f = x 2 x f a x f = 2 x 2
Now, for f a x f to be increasing , f a x f > 0
2 x 2 > 0 a x 1f > 0 x > 1 = a1 , f
Solution: f x = x 2

which means x lies outside


x , a a, +

x , a b, +

2. x a

b
g
a x a fa x bf > 0 x < a or x > b , ab > a f which

x , a a , +

i.e;

Monotonocity of a Function

af

Hence, the required interval for f x = x 2 2 x


to be increasing is x 1 = 1,

af
af
b x 1g < 0 x < 1 = b , 1g
Hence, the required interval for f a x f = x
to be decreasing is x 1 = b , 1 .

x 2 x 3x + 3 > 0

2x

bg

af
f a x f = 6 x 30 x + 36
= 6 d x 5 x + 6i
= 6 b x 2g b x 3g
Now, for f a x f to be increasing , f a x f > 0
6 b x 2 g b x 3g > 0
b x 2 g b x 3g > 0

Solution: f x = 2 x 15x + 36 x 57
2

x < 2 or x > 3 which means x lies outside

g b g

the interval [2, 3] i.e; x , 2 3 ,


Hence, the required interval for

af

f x = 2 x 15x + 36 x 57
3

to be increasing is , 2 3 ,

af
b x 2g b x 3g < 0
2 < x < 3 = b2 , 3g

af

Hence, the required interval for

af

f x = 2 x 3 15x 2 + 36 x 57
to be decreasing is 2 x 3 = 2 , 3
4. Find the interval in which the function

af

f x = x 3 6 x 2 + 9 x + 1 is increasing or decreasing.

af
f a x f = 3 x 12 x + 9
Now, f a x f will be increasing provided f a x f > 0

Solution: f x = x 6 x + 9 x + 1
2

x < 1 or x > 3
which means x lies outside the interval [1, 3] i.e;

g b g

x , 1 3,
Hence, the required interval for

af

f x = x 3 6x2 + 9x + 1

to be increasing is , 1 3 ,

af

af

Next, f x will be decreasing provided f x < 0

gb

x 1 x 3 <0

b g

1 < x < 3 = 1, 3
Hence, the required interval for

af

f x = x 3 6x2 + 9x + 1
to be decreasing is 1 < x < 3 = 1 , 3

bg

5. Find the interval in which the function f x


= tan x 4 x 1 is increasing and decreasing,

<x< .
2
2

af
a f
f a x f = sec x 4
Now for f a x f to be increasing , f a x f > 0

Solution: f x = tan x 4 x 1

Next, for f x to be decreasing, f x < 0

g b g
b x 1g b x 3g > 0

x x 1 3 x 3 > 0

3. Find the interval in which the function f x


= 2 x 3 15x 2 + 36 x 57 is an increasing function
of x and a decreasing function of x.

3 x 2 12 x + > 0

x2 4x + 3 > 0

Next, for f x to be decreasing, f x < 0

857

i.e; sec 2 x 4 > 0


2

sec x > 4

sec x > 2 as
x<

<x< .
2
2

or x >
which means x lies outside
3
3

LM
N

OP
Q

the interval


,
i.e;
3 3

FG
H

IJ FG
K H

,
,
2
3
3 2

IJ
K

858

How to Learn Calculus of One Variable

Hence, the required interval for

af

g
FG , IJ FG , IJ
H 2 3K H 3 2 K

f x = tan x 4 x 1
to be increasing is

af

af

Next, f x will be decreasing provided f x < 0


2

sec x 4 < 0
2

F
H

I
K

af

LM
N

OP
Q


.
,
3 3

bg
af
af
af
b , 0 g b0 , g
2. If f a x f > 0 , x , x 0 then f a x f increases in
b , 0 g b0 , g
3. If f a x f is undefined for x 0 , then f a x f is also
undefined for x 0 and hence only test for f a x f > 0
is required to find the interval in which f a x f increases
af
af

af
g
af
g

4. If f x > 0 , x , then f x increases in the


entire number line ,
5. If f x < 0 , x , then f x decreases in the
entire number line , .
These facts are illustrated by the following
examples:

b
b

af

Solution: f x =

Type 6: Important facts: The following facts based


on the rule of testing of monotonocity of a function
are useful to find the interval in which a given function
y = f x increases or decreases (If f 0 does not
exist).
1. If f x < 0 , x , x 0 then f x decreases in

for x 0 .

Examples worked out:

af

g b
af

Hence, f x decreases in , 0 0 , +
x2
,
2. Find the interval in which f x =
x +1
x 1 increases.

f x = tan x 4 x 1

to be decreasing is

1
< 0 which is true (because x 2 = + ve
2x 2
for all +ve and ve values of x) except x = 0 where the
function is undefined.

i.e;

sec x < 4
sec x < 2


<x<
= ,
3
3
3 3
Hence, the required interval for

af

a f 21x
1
f axf =
2x
Now f a x f will be decreasing provided f a x f < 0

Solution: f x =

1
, x0
1. Find the interval in which f x =
2x
decreases.

x2
x +1

a f b x + 1g bx1 + 1bgx 2g 1 = b x +3 1g
Now, f a x f will be increasing provided f a x f > 0
f x =

i.e;

b x + 1g

> 0 which is always true for all +ve and

b g

ve values of x 1 .

af
b , 1g b 1, g .

Hence, the function f x increases in

3. Find the interval in which the function

af

a f
1
Solution: f a x f = log x f a x f = , b x > 0g
x
Now, f a x f will be increasing provided f a x f > 0
f x = log x , x > 0 increases.

1
> 0 x > 0 which is true since we are given
x
x > 0.

i.e;

af
f b x g is undefined for

Hence, f x increases in the interval 0 , + .


Note:

af

x 0 because

f x = log x is undefined for x 0 .

859

Monotonocity of a Function

bg

4. Find the interval in which the function f x


= e ax , a > 0 increases.

g
Solution: f a x f = e f b x g = a e
Now, f a x f will be increasing provided f a x f > 0
ax

ax

i.e; a e > 0 which is always true for all +ve and ve


values of x and a > 0 .
ax

af

Now ,

x2
> 0 for every value of x > 0
1+ x

af

which f x > 0 for 0 < x < 1

af

a f

f x = log 1 + x x +

x
is an increasing
2

function in the interval 0 x 1

bg bg
x
5. Find the interval in which the function f b x g
log a1 + x f x +
>0
2
=e
, ba > 0g decreases.
cQ f b0g = log b1 + 0g 0 + 0 = 0h
Solution: f a x f = e
f a xf = a e
Now, f a x f will be decreasing provided f a x f < 0
x
for all x in ( 0 , 1 ).
log a1 + x f > x
2
i.e; a e
is
< 0 which is true because e
always + ve for all +ve and ve values of x and
R
U
2. Show that 1 + x log S x + 1 + x V 1 + x for
a > 0.
T
W
x0 .
Hence, the function f a x f decreases in the entire
number line b , + g .
R
U
Solution: Let f a x f = 1 1 + x + x log Sx + 1 + x V
T
W
Category C:
Hence, the function f x increases in the entire

number line , .

f x > f 0 for 0 < x < 1


2

ax

ax

ax

ax

ax

Problems based on proving inequality:

af

af

af

af

Type 1: To show f1 x > f2 x or f1 x < f2 x in


a given interval.
Working Rule:
1. Let f x = f1 x f2 x
2. Examine whether f x > 0 or f x < 0
3. Use the following facts:
(a) If f x > 0 , then f x > f a for x > a and
f x < f a for x < a

af

af

af
af
af
af af

af
af af
(b) If f a x f < 0 , then f a x f < f a a f for x > a and
f a x f > f a a f for x < a
Examples worked out:

x
1. Show that log 1 + x > x
if 0 < x < 1
2

af

x
Solution: Let f x = log 1 + x x +
2

af

1
x2
f x =
1+ x =
1+ x
1+ x

bg

2 x

f x =

2 1+ x 2

+x

1
x + 1+ x 2

log x + 1 + x 2
=

x
1+ x

x
1+ x

FH

R|
S|1+ 2
T

U|
V| +
W

2x
1+ x 2

}
{

+ log x + 1 + x 2

IK > 0 for all x > 0

= log x + 1 + x 2

bg bg
R
1 + x log S x +
T
R
1 + x log S x +
T

f x f 0 =0
1+ x

1+ x

UV
W
UV
W

1+ x 0
1+ x

x 0

3. Show that 2 sin x + tan x 3x when 0 x <

.
2

860

How to Learn Calculus of One Variable

af

bg

= cos x cos x + 1 cos x 1

f x = 2 cos x + sec x 3
2

2 cos x 3 cos x + 1
2

cos x
Now, we have to determine the sign of
3

2 cos x 3 cos x + 1 because cos x being a


perfect square it is always +ve.

Q 2 cos3 x 3 cos2 x + 1 = 1 cos x

g b2 cos x + 1g
2

which is +ve for all values of x lying in the given


interval 0 < x <

bg

f x =

2 cos x 3 cos x + 1
3

cos x

0 for 0 x <

af

Which f x 0 when 0 x <

bg

f x 0 for 0 x <

bg bg

2 sin x + tan x 3x 0

bQ f a0f = 2 sin 0 + tan 0 3 0 = 0g

4. Show that sin x + tan x > 2 x for 0 < x <

af

cos2 x

bg bg

sin x + tan x 2 x > 0


Q f 0 = sin 0 + tan 0 2 0 = 0
sin x + tan x > 2 x .

c bg

5. Show that tan x > x whenever 0 < x <

af

Solution : f x = tan x x

bg

f x = sec 2 x 1 = tan 2 x > 0 for


0<x<

af
f a x f = tan x x is an increasing function for

2
f x >0

0<x<

bQ f a0f = tan 0 0 = 0g

af

2
Type 2: To show f1 x > f2 x > f3 x in a given
interval
tan x > x for 0 < x <

bg

cos3 x 2 cos2 x + 1

.
2
f x > f 0 for 0 < x

function in 0 x <

tan x x > 0

f x = cos x + sec 2 x 2
=

f x > 0

Solution: Let f x = sin x + tan x 2 x when

0 x<

Type 1:

f x f 0
2

2 sin x + tan x 3x for 0 x <

f
ja
+ cos x} a cos x 1f
and cos x < 1 cos x 1 > 0
aQ 1 < cos x < 1f
Thus each factor {acos x 1f + cos x} and
acos x 1f > 0 for 0 x < 2
f b x g > 0 which f a x f is an increasing
e
= {a cos x 1f

Solution: Let f x = 2 sin x + tan x 3x

>0

2
when 0 < x <
because cos x being a perfect
2
3
2
square it is always +ve and cos x 2 cos x + 1

af

af

af

Working rule: It consists of two parts:


First Part:
1. Let f x = f1 x f2 x
2. Examine whether f x > 0 for the values of x in
the given interval.

af

af

af
af

Monotonocity of a Function

af

af

3. If f x > 0 , f x is increasing in the given


interval and then use the following fact:
f x > f a for x > a and put f (x) = f1 (x) f2
(x) in f x > f a which will provide us the inequality

af af
af af
f a x f > f a x f provided that f ba g = 0
1

bg bg
From first and second part, we have
f a x f > f a x f > f a x f in the given interval
f

x
< log 1 + x < x for x > 0
1+ x

af

Solution: Let f x = log 1 + x

af

f x =

1
1

1+ x
1+ x

for x > 0

x
1+ x
x

g = b1 + xg
2

af
x
f a x f = log a1 + x f
a1+ xf

>0

f x is + ve for all x > 0

is an increasing

function for all x 0

af af
x
log a1 + x f
>0
1+ x

f x > f 0 = log 1 = 0 for x > 0

x
< log 1 + x
1+ x

af

...(1)

af

g x = 1
=

1
1+ x 1
=
1+ x
1+ x

x
> 0 for all x > 0
1+ x

af

F I
af
H 2K
f a x f = tan x x is an increasing function in
LM0 , IJ
N 2K

f a x f > f a 0f for 0 < x <


2
I
F
f a x f is +ve in H 0 , K
2
bQ f a0f = tan 0 0 = 0g
2

f x = sec x 1 = tan x > 0 x 0 ,

...(1)

af

g a x f = 1 cos x > 0 for 0 < x <


2
aQ 1< cos x < 1 cos x < 1 1 cos x > 0f
g a xf > 0
g a x f = x sin x is an increasing function in
L I
the interval M0 , J
N 2K

g a x f > g a0f for 0 < x <


2
g a x f > 0 bQ g a0f = 0 sin 0 = 0g
Again g x = x sin x

Again let g x = x log 1 + x

, then tan x > x > sin x.


2
Solution : f x = tan x x

tan x x > 0 for 0 < x <

x
1+ x

log 1 + x >

2. Show that if 0 < x <

...(2)

inequality.

Examples worked out:


1. Show that

af af
bQ g a0f = 0 log 1 = 0g
g a xf > 0
x log a1 + x f > 0 x > log a1 + x f

x
< log 1 + x < x which is the required
1+ x

Similarly we show f 2 x > f 3 x


2

af

Which g x is an increasing function for all


x 0 g x > g 0 for x > 0

From (1) and (2) we declear

Second part:

861

862

How to Learn Calculus of One Variable

F I
H 2K

x sin x > 0 x 0 ,

x > sin x
From (1) and (2), we have,

...(2)

tan x > x > sin x for 0 < x <


which is the
2
required inequality.
3

1
x
< tan x < x .
3. If x > 0 show that x
3

af

Solution: f x = x tan

x0

af af
f a x f > 0 eQ f a0f = 0 tan

f x > f 0 for x > 0

x tan

x>0

x > tan

x,x > 0

af

Again g x = tan

af

g x =

af

1
1+ x

g x = tan

0=0

...(1)
3

x
x x+
3

1+ x =

1+ x

> 0 for x > 0

x x+

x
3

is an increasing

function for x 0

af af
FQ g a0f = tan
g a xf > 0
H
g x >g 0 ,x>0

tan

x x+

x > tan

...(2)

x> x

x
3

1
x
< tan x < x for all x > 0 which is the
3
required inequality.

af

Solution: f x = x

1 2
x < log 1 + x < x
2

1 2
x log 1 + x
2

1
1 x 1 x
=
=
af
1+ x
1+ x
1+ x
f a x f < 0 if x > 0
f a x f is a decreasing function for x 0
f a x f < f a 0f x > 0
f a x f < 0 bQ f a0f = 0 0 log 1 = 0g
1
x x < log a1 + x f
...(1)
2
Again, g a x f = log a1 + x f x
x
1
11 x
g a xf =
1=
=
1+ x
1+ x
1+ x
g a x f < 0 if x > 0
g a x f is a decreasing function for x > 0
g a x f < g a0 f for x > 0
g axf < 0
bQ g a0f = log a1 + 0f 0 = log1 = 0g
log a1 + x f x < 0
...(2)
log a1 + x f < x
2

f x = 1 x

x
tan x > x
3
From (1) and (2), we have

4. Show that for x > 0 , x

a f 1 +1x = 1 +x x
f a x f > 0 for every value of x > 0
f a x f = x tan x is an increasing function
f x =1

x
>0
3

00+

I
K

0
=0
3

From (1) and (2), we have

1 2
x < log 1 + x < x which is the required
2
inequality.
x

Monotonocity of a Function

Type 3: Problems based on choosing a specified


function to prove a given inequality in a given interval
When in the given inequality to be proved in a given
interval, an independent variable is not x but the
independent variable is , or ect. we adopt the
following working rule:
Working rule:
1. Choose a specified function y = f x where x is
in a given interval.
2. Find f x .
3. Examine whether f x is an increasing function
or a decreasing function in the given interval.
4. If f x is an increasing function, then the same
function of given independent varibale , or
etc is also an increasing function i.e; f , f
or f ( ) is also increasing corresponding to increasing
function f x in the given interval ) and if f x is a
decreasing function, then the same function of given
independent variable , or etc. is also a
decreasing function i.e; f , f or f is
also decreasing corresponding to decreasing function
f x in the given interval.)
5. Lastly using various mathematical manipulations,
we prove the required inequality in the given interval.

af

af

af

af

af

af af

af af
af
af

af

Examples worked out:


1. If 0 < < <

, show that sin < sin .


2

F I
H 2K

af
f a x f = 1 cos x

We know that if 0 < x <

af

...(1)

0 < cos x < 1


2

Thus, f x is an increasing function in the interval

F0, I .
H 2K

FG IJ and < f af < f af


H 2K

, B 0,

2. Prove that if 0 < <

, then
2

a f a f
F I
Solution: f a x f = x sin x , x 0 ,
H 2K
f a x f = 1 cos x
cos sin > sin cos

We know that 0 < x <

0 < cos x < 1


2

af

1 cos x > 0 f x > 0

LM OP
N 2Q

bg

f x is an increasing function in 0 ,

bg
2
f bg > f b0g for > 0
f bg > 0 bQ f a0f = 0 sin 0 = 0 0 = 0g
f is also increasing for 0

sin > 0

b g

sin < cos sin > cos


cos sin < cos

b g

FG
H

F I IJ
H 2 KK

...(2)

again f (x) > f (0) = 0 for x > 0 or, x 0,

FG IJ
H 2K

f cos > 0 for 0,

1 cos x > 0 f x > 0

af

sin < sin which is the required


inequality.

b g

Solution: f x = x sin x , x 0,

863

a f

cos sin cos > 0

a f

sin cos < cos

...(3)

a f

(3) (2) = sin cos cos sin < cos cos

a f
sin a cos f < cos a sin f
f

= sin cos cos sin < 0 which

3. Prove that

2 sin x
<
< 1 whenever 0 < x < .
2
x
x

af

Solution: Let a continuous function f x be defined


in the following way.

864

How to Learn Calculus of One Variable

(iii) f (x) = x3, at x = 2

af

sin x
f x =
, if x 0
x
= 1, if x = 0

(iv) f (x) = x3, at x = 2

af

x cos x sin x

When x 0 , we get f x =

= x tan x cos x

a f a x tan x f cos x
x

.
2
is a decreasing

which means x tan x is negative in 0 < x <

af

af

Hence, f x < 0 f x
function in 0 < x <

bg bg

FG IJ for 0 < x <


H 2K
2
RSQ f a0f = 1 f F I = 2 UV
H 2K W
T

f 0 > f x > f
1>

sin x 2
>
x
x

2 sin
<
< 1 which is the required inequality.

Type 1: Problems based on examining or showing a


function of an independent variable to be increasing
or decreasing at a point x = a. (Category B)
Exercise 21.1
1. Examine whether the following functions is
increasing or decreasing at the points indicated.
(i) f (x) = 2x + 3, at x = 0
(ii) f (x) = 5 3x, at x = 1

1 2
x + cos x is increasing at
2
x = 1 while decreasing at x = 1.
Answers:
1. (i) Increasing (ii) Decreasing (iii) Increasing
(iv) Increasing (v) Decreasing (vi) Decreasing

x 2 being a perfect square, it is always + ve and


cos x is + ve in the first quadrant. Moreover, if

0<x< ,
2
tan x > x tan x x > 0 x tan x < 0

(vi)

2. Prove that y =

x
Now, we have to examine the sign of
f x =

a f 1x , at x = 1
1
1
f a x f = x + , at x =
x
2

(v) f x =

Type 2: Problems based on showing the function


y = f (x) to be increasing or decreasing in the infinite
interval
(i) x > a (ii) x < a (iii) x a
(iv) x a (v) x > 0 (vi) x < 0 (vii) x 0
(viii) x 0 (ix) x a (x) x
Exercise 21.2
1. Show that following functions are increasing in
the indicated intervals:

a f 1 ,x0
1+ x
(ii) f b x g = a x + b , a > 0 , x R
9
(iii) f a x f 6 + 9 x x , x
2
(i) f x =

2. Show that following functions are decreasing in


the indicated intervals:
(i) f (x) = 7x2 + 11x 9, x > 1

af

1 3
x + 1, x > 0
2
log x
3. Show that y =
decreases for x > e.
x
(ii) f x = x

4. Show that the function y = log 1 + x

2x
2+x

is an increasing function of x for all values of x > 0.

865

Monotonocity of a Function

5. Prove that the function y = 2x3 15x2 + 36x + 10 is


an increasing function of x for x > 3 and a decreasing
function of x for x < 2.

6. Show that the function y =

1 x

log 1 + x

increases for all values of x > 0.


7. Prove that y = 2x3 + 3x2 12x 7 is an increasing
fucntion when x > 1.
x
8. Show that f x =
increases for all x 1 .
1+ x

af

9. Show that

ex
f a xf =

1
x

increases for all

Exercise 21.4
1. Show that

x .
2
2
(ii) f (x) = cos x is decreasing for 0 x .
(i) f (x) = sin x is increasing for

af

2. Show that the function f x = x +

1
x

is

decreasing in 0 < x < 1.


3. Prove that the function f (x) = log sin x is increasing

F I and decreasing on F , I .
H 2K
H2 K

on 0 ,

x 0.

4. Prove that the function f (x) = sin x is increasing in

Type 3: Problems based on determination of values


of x for which the given function y = f (x) is increasing
or decreasing.

the interval

Exercise 21.3
1. Determine the values of x for which the function

x2
y=
, x 1 is increasing or decreasing.
x+ 1
2. Determine the values of x for which the function

af

F , I .
H2 K

F0 , I
H 2K

and decreasing in the interval

af

decreasing function in the interval

af

1
,x0
x
4. Find the values of x for which the following
functions are
(i) increasing (ii) dDecreasing (iii) stationary
(a) 3 x + x3 (b) x3 3x + 2 (c) x4 2x2 + 1
(iv) f x = x +

Type 4: Problems based on showing (or, examining)


the given function to be increasing or decreasing in
the interval (a, b) or [a, b].

F 1 , 1I .
H 4 2K

6. Show that f (x) = tan1 (sin x + cos x), x > 0 is

F I .
H 4K

f x = 5x 2 3x 2 , x > 0 is increasing or decreasing.


3. Determine the values of x for which the following
functions are increasing and for which they are
decreasing.
(i) f (x) = 6x2 2x + 1
(ii) f (x) = 3x2 + 12x + 8
(iii) f (x) = x8 + 6x2

4x + 1
is a
5. Show that the function f x =
x

always increasing in the interval 0 ,

af

x
is an increasing function
sin x

in the interval 0 < x < .


2
8. Find the least value of a such that the function
f (x) = x2 + ax + 1 is increasing on [1, 2].
9. Determine whether the following functions are
always increasing or decreasing in the indicated
interval.
7. Show that f x =

af

(i) f x =

af

x
+ sin x in < x <
2
3
3

F
H

(ii) f x = cos 2 x +

I
K

3
5
in
<x<
4
8
8

866

How to Learn Calculus of One Variable

af

(iii) f x = tan x 4 x in

(b) f (x) is an increasing function in (0, 1).

<x<0
3

F
H

af

10. Determine whether f x = cos 2 x +


increasing or decreasing for

3
7
<x<
.
8
8

I
K

is

18. The function in parts (a) and (b) are decreasing.

a f

11. Prove that y = log x tan1 x increases in 0 , .

Type 5: Problems based on finding the intervals in


which the given functions increase or decrease.

sin x
is decreasing function from
x

12. Show that y =


0 to

F , I .
H2 K
F I .
(d) f (x) is increasing on 0 ,
H 2K
(c) f (x) is increasing in

Exercise 21.5
1. Find the intervals on which the following functions
are increasing and those in which the functions are
decreasing.

.
2

13. Show that y =

tan x
is an increasing function
x

in the range 0 < x < .


2
14. Prove that the function f (x) = x2 x + 1 is neither
increasing nor decreasing in the interval (0, 1).
15. Show that the function f (x) = log x is increasing
for 0 < x < .

af

16. Show that f x = x +

is decreasing in

0 < x < 1.
17. One of which the following intervals is the
function f (x) = x100 + sin x 1 in creasing?
(a) (1, 1) (b) (0, 1) (c)

F
H

I
K

,
2

(d)

F I
H K

0,
.
2

18. Which of the following functions are increasing

F I .
H 2K

on 0 ,

(a) cos x (b) cos 2x (c) cos 3x (d) tan x


Answers:
8. Least value of a = 4 for f (x) = x2 + ax + 1 to be
increasing on [1, 2].
17. (a) f (x) is neither increasing nor decreasing in
(1, 1).

af
f a xf = x

(i) f x = x + 3x + 4
(ii)

af

+ 2x

F
H

1
3

3
(iii) f x = x x +

I
K

1
3

bg
(v) f a x f = x 3 x + 2
1
(vi) f a x f = x +
x
(vii) f a x f = a x 2f a x + 3f
(viii) f a x f = x 18 x
x2
, x 2
(ix) f a x f =
x+2
(x) f a x f = x a x + 1f a x + 2 f
x
(xi) f a x f =
1

(iv) f x = 3x 2 x 2
3

1+ x

when x 2
a f RST2xx ++ 91 ,, when
x > 2
f a x f = sin x , x a 0 , f

(xii) f x =
(xiii)

Monotonocity of a Function

af

F I
H 2K

(xiv) f x = sin x + cos x , x 0 ,

af

(xv) f x = x 1 + x

2. Find the intervals in which the function


f (x) = x4 2x2 decreases.
3. Find the intervals in which the function
f (x) = x3 + 2x2 1 decreases.
4. Find the intervals in which the function
f (x) = x3 3x increases and decreases.
5. Determine the intervals in which the function
f (x) = 2x3 24x + 7 increases or decreases.
6. Determine the intervals where f (x) = sin x cos x,
0 x 2 is increasing or decreasing.
7. Find the intervals in which f (x) 6x2 2x + 1
increases or decreases.
8. Determine the intervals in which the function
f (x) 5x2 + 7x 13 is increasing or decreasing.
9. Find the intervals in which the function

F
H

f (x) = cos 2 x +

I
K

,
is increasing or
4 0 x

decreasing. Find also the points on the graph of the


function at which the tangents are parallel to x-axis.
10. Determine the intervals in which the function
f (x) = (x 1) (x + 2)2 is increasing or decreasing. At
what points are the tangents to the graph of the
function, parallel to x-axis?
11. Find the intervals in which the function

bg

x3
x2
+
2 x + 1 is increasing or decreas3
2
ing. At what points are the tangents to the graph of
the function parallel to x-axis.
12. Find the interval in which the function f (x) =
2x3 9x2 + 12x + 30 is increasing or decreasing.
f x =

Answers:
1. (i)

F , 3 I increasing, LM 3 , I decreasing.
H 2K
N2 K

867

F , 3 I decreasing, LM 3 , I increasing.
H 2K
N2 K
F 2 I decreasing, LM 2 , I increasing.
(iii) ,
H 9K
N9 K
(iv) , 1 increasing, 1, f decreasing.
(ii)

(v) Increasing when x < 1 and x > 1, decreasing in


1 < x < 1.
(vi) Increasing when x < 1 and x > 1, decreasing in
1 < x < 1.
(vii) Increasing when x <

4
and x > 2, decreasing
3

4
< x < 2.
3
(viii) Increasing when 3 < x < 0 and x > 3, decreasing
in

when x < 3 and 0 < x < 3.


(ix) Increasing in the domain of definition.
(x) Increasing in
x+1 <

x+1 >

1
3

, decreasing for

.
3
(xi) Increasing in 1 < x < 1, decreasing in x < 1 or
x > 1.
(xii) Increasing for x 2 and x 0 , decreasing
in 2 < x < 0.

F I , decreasing in F , I .
H 2K
H2 K
F I , decreasing in F , I .
(xiv) Increasing in 0 ,
H 4K
H 4 2K
(xiii) Increasing in 0 ,

(xv) Increasing when x >

x<

3
, decreasing when
4

3
4

2. f (x) is increasing for 1 < x < 0, x < 1 and decreasing


for x < 1, 0 < x < 1.

868

How to Learn Calculus of One Variable

3. f (x) is increasing for x <

F
H

for

I
K

4
, x > 0 and decreasing
3

10
< x < 0 as well as f (x) is decreasing in
3

F 4 , 0I .
H 3 K

4. f (x) is increasing for x . 1 or x < 1; Decreasing for


1 < x < 1.
5. f (x) is increasing for x 2 and for x 2 ; f (x)
is decreasing for 2 x 2 .
6. f (x) is increasing when 0 x
decreases when

3
7
.
x
4
4

7. Increasing for x

1
1
; Decreasing for x .
6
6

8. Increasing for x

3
7
or
; f (x)
4
4

7
; Decreasing for
10

7
.
10

9. Increasing for

3
7
; Decreasing for
x
8
8

3
as well as the required points at
x
8
8
x=

3
7
and x =
.
8
8

10. Increasing for x 0 or x 2 ; Decreasing for

2 x 0 as well as the required points at (0, 4)


and (2, 0).
11. Increasing for x 2 or x 1 ; Decreasing for
2 x 1 .
12. Increasing for x 1 or x 2 ; Decreasing for
1 x 2.
Type 6: Problems based on showing the function
y = f (x) to be increasing or decreasing for all values
of x (or, in every interval).

Exercise 21.6
1. Show that the function f defined on R by f (x)
= x3 + 3x2 + 3x 8 is increasing in every interval.
2. Prove that y = 2x3 + 4x is increasing for all values
of x.
3. Show that the exponential function y = e x is
increasing for all x.
4. Show that y = tan1 x is an increasing function of x
for all x.
5. Determine whether the following functions are
increasing or decreasing for stated values of x.
(i) f (x) = x cos x, for all x.
(ii) f (x) = x + sin x, for all x.
6. Show that f (x) = 3x + 1 is an increasing function
on R.
7. Prove that f (x) = ax + b is an increasing function
for all real values of x, where a and b are constant and
a > 0.
8. Prove that the function f (x) = x3 3x2 + 3x 100 is
increasing on R.
9. Show that the function y = x3 3x2 + 6x 8 increases
while the function y = 3 x3 decreases for all x.
10. If y = 3x 3x2 + x3, show that y always increases
whatever the value of x.
3

4
x
+ x
11. Show that the function y =
is an
3
3
increasing function for all values of x.

12. If f (x) = (x 1) ex + 1, show that f (x) is positive for


all positive values of x.
Type 7: Problems based on proving an inequality a
given interval. (Category C)
Exercise 21.7

af

1 2
x cos x , show that when x is
2
positve , f x in negative. Hence, deduce that for a
1 2
positive values of x, 1 x < cos x < 1
2
1. If f x = 1

af

Monotonocity of a Function

2. Show that when x is positive, x

1 2
x < sin x < x.
6

LM Hint: Take f a xf = x 1 x sin x and OP


6
MMprove that f a xf is negative
; note that P
PP
NM f a0f = 0
Q
2

8. If 0 < x < , show that cos x > 1


2
2
b
2
9. If ax + c , for all positive x, where a > 0
x

and b > 0 show that 27 ab 2 4c 3 .

x
3. Prove that cos x 1 +
> 0 for x > 0 . Is this
2
true for x < 0 ?

10. Prove the following :

(ii) e x x > 1 , x 0

OP
LMHint : f a xf = x 2 x + 3
MN f a xf = 3x 2 > 0 for every value of xPQ
3

5. If x > 0 Show that

(i) tan x if x 0

j if x > 0

(x) 0 < < <

(iii) x 1 e x + 1 > 0 for all x

tan x
, if x > 0
1+ x

7. Show that 1 + x log x +


for all x 0

(xi) ax +

IJ
K

x +1 1+ x

tan

>
2
tan

b
c2
c , x > 0 ab
x
4
b, c are constants.

FG
H

I
K

6. Show that

(iv) log e 1 + x >

F
H

a
f
b g
2 a x 1f
(vi) log x >
a x > 1f
x +1
I
x
F
(vii) x
< sin x < x 0 < x
H
3
2K
I
x F
(viii) tan x > x +
0< x<
3 H
2K
(ix) x > a1 + x f log a1 + x f a x > 0f

LMHint :(a) f a xf = x log a1x+ xf , x > 0 OP


MMabf f a xf = log a1 + xf 1 + x , x > 0 PP
MMthen find f a xf for aaf and a bf which willPP
Q
Nbe positive for x > 0

x
1 + cot x 0 < x <
2
1
(v) 2 vx + > 3 x > 1
x

a f
a1 + xf > 1 +x x

(ii) 2 x tan x > log e 1 + x

(iv) cot

(a) log e 1 + x < x

(iii) sin x + tan x > 2 x 0 < x <

tan x
, x>0
1+ x

(i) log 1 + x >

4. Show that for all x , x R , x 3 + 3 > 2 x

(b) log e

869

where a,

870

How to Learn Calculus of One Variable

22
Maxima and Minima

Maxima and Minima of a Function


We shall define:
1. Local (or relative) maxima and local (or relative)
minima.
2. Absolute (or global) maxima and absolute (or
global) minima.
Each one is defined in various ways:
1. Local (or regional or relative) maxima of a function:
Definition (i): (In terms of neighbourhood): A
function f defined by y = f (x) on its domain D is said
to have local (or relative) maximum value (or simply a
local (or relative) maximum) at an interior point x = c of

af

the domain of the function f namely D N c :

af kp

af af

x N c c f c > f x

That is, there is a -neighbourhood of the interior


point x =c of the domain of the function f denoted by
N c such that the value of the function f at the
interior point x = c denoted by f (c) is greater than the
values of the function f at all values of the independent
variable x which lie in the -deleted neighbourhood
of the interior point x = c of the domain of the function

af

af kp

denoted by N c c .
Definition (ii): (In terms of distance): A function f
defined by y = f (x) on its domain D is said to have a
local (or relative) maximum value (or simply a local (or
relative) maximum) at an interior point x = c of the
domain of the function f namely D a > 0 :

bg bg

0< xc < f c > f x

That is, there is a positive number such that the


value of the function f at an interior point x = c denoted
by f (c) is greater than the values of the function f at
every value of the independent variable x whose
distance (or difference) from the interior point x = c of
the domain of the function f is non zero and less than
the positive number .
2. Local (or regional or relative) minima of a function:
Definition (i): (In terms of neighbourhood): A
function f defined by y = f (x) on its domain D is said
to have a local (or relative) minimum value (or simply
a local (or relative) minimum) at an interior point x = c
of the domain of the function f namely

af
acf kcp f acf < f a xf

D a N c :
x N

That is, there is a -neighbourhood of the interior


point x = c of the domain of the function f denoted by
N c such that the value of the function f at the
interior point x = c denoted by f (c) is less than the
values of the function f at all the values of the
independent variable x which lie in the -deleted
neighbourhood of the interior point x = c of the
domain of the function f denoted by N c c .
Definition (ii): (In terms of distance): A function f
defined by y = f (x) on its domain D is said to have a
local (or relative) minimum value (or simply a local (or
relative) minimum) at an interior point x = c of the

af

a f kp

domain of the function f namely D a > 0 :

af

af

0< xc < f c < f x

Maxima and Minima

That is, there is a positive number such that the


value of the function f at an interior point x = c denoted
by f (c) is less than the values of the function f at
every value of the independent variable x whose
distance (or difference) from the interior point x = c of
the domain of the function f is non zero and less than
the positive number .
Now the definitions of absolute (or global) maxima
and absolute (or global) minima of a function are
presented.
1. Absolute (or universal or global) maxima of a
function:
Definition: A function f defined by y = f (x) on its
domain D is said to have an absolute (or global)
maximum value (or simply a maximum) at a point x = c
D x C f c f x , x D f
That is, the value of the function f at the point
x = c in its domain D denoted by f (c) is not less than
the values of the function f at any value of the
independent variable x which is not c and is in its
domain D.

bg bg

bg

2. Absolute (or universal or global) minima of a


function:
Definition: A function f defined by y = f (x) on its
domain D is said to have an absolute (or global)
minimum value (or simply a minimum) at a point x = c

bg bg

bg

D x D f c f x , x D f

That is, the value of the function f at the point


x = c in its domain D denoted by f (c) is not greater
than the value of the function f at any value of the
independent variable x which is not c and is in its
domain D.

continuous function, i.e. the maximum and /minimum


values occur alternatively in a continuous function.
e.g. y = sin x and y = cos x have the maximum and the
minimum points alternatively.
But in a discontinuous function, the local maximum
and the local minimum points may or may not occur
alternatively.
(iii) A function y = f (x) may have only one maximum
value.
e.g.: y = 60x x2
(iv) A function y = f (x) may have only one minimum
value.
72
e.g.: y = 2 x +
x
(v) A function y = f (x) may have both the maximum
and the minimum values.
e.g.: y = x3 x2 8x + 2
On Points of Local Exterma
1. The point of local maximum: A point x = c in the
domain of the function y = f (x) at which the value of
the function is a local maximum value of the function
is called the point of local maximum (or simply the
point of maximum or maximum point) of the function
y = f (x).
2. The point of local minimum: A point x = c in the
domain of the function y = f (x) at which the value of
the function is a local minimum value of the function
is called the point of local minimum (or, simply the
point of minimum or minimum point) of the function
y = f (x).
y

Notes: A: (i) Maximum and / minimum values are


often termed as extrema (plural of extermum).
(ii) Plural of maximum is either maxima or maximums.
(iii) Plural of minimum is either minima or minimums.
(iv) Plural of extremum is only extreme.
(B): (i) It is not necessary that the given function y
= f (x) should always have the maxima and / minima.
e.g. y = x3, y = cot x, y = ax and y = ax + b do not
have either a maximum and / a minimum.
(ii) There may be several local maxima and local
minima which occur alternatively in case of a

871

f (c)

P1

x=c

872

How to Learn Calculus of One Variable

2. Points at which the first derivative is zero: If x = c


is a critical point at which f c exists and f c = 0
and x = c is a point of local maximum, then f is
increasing in the left -neighbourhood of c and f is
decreasing in the right -neighrbourhood of c, i.e.
f x changes sign from positive to negative as x
passes through c.

af

P2

af

f (c)
o

af

x=c

Geometrically, a point x = c in the domain of a function y = f (x) is a point of local maxima or local minima
according as the graph of the function f has a peak
(crest) or cavity (trough) at the point x = c.
Here P1 is a min. point and P2 is a max. point.
Local Extreme Values at Critical Points
In fact, to find out local extreme values of a function
defined in its domain, critical points are considered
which are of two kinds:
1. Points at which the first derivative does not exist:
If x = c is a critical point where f c is undefined
and x = c is a point of local maximum, then f is
increasing in the left -neighbourhood of c (i.e. f is
increasing x c , c] and f is decreasing in
the right -neighbourhood of c (i.e. is decreasing
x c , c + , i.e., changes sign from positive
to negative as x passes through c.

af

fg

= B

= B

In the same fashion, if x = c is a point of local


minimum, then it is decreasing in the left neighbourhood of c and f is increasing in the right neighbourhood of c, i.e. f ' (x) changes sign from
negative to positive as x passes through c.
Notes: 1. ymax = max f (x) = maximum value of f (x)
= [y]x =c , where c = a point of maximum
2. ymin = min f (x) = minimum value of f (x)
= [y]x =c , where c = a point of minimum.
Necessary condition for the maximum or the
minimum value
Theorem: If y = f (x) defined on its domain has a
maximum value at x = c and f c exists, then
f c = 0 where c is an interior point of the domain
of the function f.
Proof: Let y = f (x) be a real valued function of the
independent variable x whose domain is the interval
D.
Hypothesis: 1. f (x) has the maximum value at the
interior point x = c in the domain D, i.e.,
a : x N c c f x < f c
a : c < x < c f x < f c
(i)

af

af

=c Bx = c =c +B

In the same fashion, if x = c is a point of local


minimum, then f is decreasing in the left neighbourhood of c (i.e. f is decreasing
x c , c] and f is increasing in the right neighbourhood of c (i.e. f is increasing
x c , c + , i.e. f x changes sign from
negative to positive as x passes through c.

f
fg

af

o c x = c c +

a f kp

af af
af af
and a : c < x < c + f a x f < f acf
(ii)
2. f acf exists L f acf = R f acf = f acf
To prove: f acf = 0

Maxima and Minima

Main proof: From (i), it is seen that

Notes: 1. The above theorem is also true when


y = f (x) has a minimum value at x = c in the domain of

f (x) < f (c) for c < x < c

af
af

f and f c exists.

i.e., f (x) f (c) < 0 for c < x < c

2. f c exists l.h.d = r.h.d a common finite


value, i.e. f c exists L f c = R f c = f c
3. x y and x > y x >/ y and x </ y x = y

af af
a f

f x f c
>0
xc

[Since c < x < c < x c < 0 ]

LM f a xf f acf OP 0
N a x cf Q
L f acf 0
xc

(A)

From (ii), it is seen that


f (x) < f (c) for c < x < c +
i.e., f (x) f (c) < 0 for c < x < c +

af af
a f

f x f c
<0
xc

[Since c < x < c + 0 < x c < ]

LM f a xf f acf OP 0
N a x cf Q
(B)
R f acf 0
On putting L f acf = R f acf = f acf [from the
lim

xc

hypothesis (2)] in (A) and (B), it is found that

af
f acf 0

f c 0 (A1)
and
(B1)
Hence, from (A1) and (B1), it is concluded that and

a f UV
af W

af

f c 0
f c = 0 which was required to be
f c 0

proved.
y

=B

f c 0

= B

f c

x = c

=B

f c =0

f (c )

x=c

=B

af af

bg
bg

bg
bg
af af

af
af

bg
bg

bg
bg
af af

af

c < x < c f x > 0 / f x > 0 / f x > 0 and


c < x < c + f x < 0 / f x < 0 / f x < 0 then
it is said that f (x) / f x / f x changes sign from
positive to negative at x = c as (when or while) x
passes through c from left to right.
Similarly, if there exists an > 0 such that

af

c < x < c f x < 0 / f x < 0 / f x < 0 and


c < x < c + f x > 0 / f x > 0 / f x > 0 then
it is said that f (x) / f x / f x changes sign from
negative to positive at x = c as (when or while) x
passes through c from left to right.
Sufficient criteria for the maxima and minima
Theorem (first derivative test or rule of change of
sign of first derivative): If a function y = f (x) defined
in its domain is differentiable in a -deleted

a f kp

neighbourhood of the point x = c (i.e. in N c c

af
af kp
af

af

af kp

af

and f c = 0 , then
(a) x N c c , x < c f x > 0 and x > c
f x < 0 i.e. f (x) changes sign from positive to
negative (i.e. from plus to minus) f (x) has a
maximum at x = c.
(b) x N c c , x < c f x < 0 and x > c
f x > 0 i.e. f (x) changes sign from negative to
positive (i.e. from minus to plus) f (x) has a
minimum at x = c.
Proof: Verse part:

af

= B

f c +
x = c +

af

Let y = f (x) be a function of the independent variable


x whose domain contains a point x = c.
If there exists an > 0 such that

af

f c 0

(c, f (c ))

af

On the Language of Calculus

lim

873

It is given that f c = 0 , then of course c is a


critical point of the function.

874

How to Learn Calculus of One Variable

af

Also, in (a), it is given that x < c f x > 0


which further
f (x) is increasing on the left of c
f (x) is increasing in a left -deleted
neighbourhood of c.
f (x) is increasing in (c , c).

af

af

f x f c , x c, c

(i)

af

Again, in (a), it is given that x > c f x < 0


which further
f (x) is decreasing on the right of c.
f (x) is decreasing in a right -deleted
neighbourhood of c.
f (x) is decreasing in (c, c + )

af af

(ii)
f x f c , x c, c+
Hence, from (i) and (ii), it is concluded that

af af

f a

f x f c , x c, c c, c+
i.e. f (c) is greater than every value of the function at

f a f
f (c) is maximum in x ac , c f ac , c + f

every value of x c , c c , c + which

f (x) has a maximum at x = c.


Next, accordingly as in (b), it is given that

af

x < c f x < 0 which f (x) is decreasing on


the left of c.
f (x) is decreasing on the left -deleted
neighbourhood of c.
f (x) is decreasing in (c , c)

af

af

f x f c , x c , c

(iii)

af

Also, in (b), it is given that x > c f x > 0


which
f (x) is increasing on the right of c.
f (x) is increasing on the right -deleted
neighbourhood of c.
f (x) in increasing in (c, c + )

af af

(iv)
f x f c , x c, c+
Thus, from (iii) and (iv), it is concluded that

af af

f a

f x f c , c , c c, c+

i.e. f (c) is less than every value of the function at

f a

every value of x c , c c , c +

f a

f (c) is minimum in c , c c , c +
f (x) has a minimum at x = c.

Converse part:
Hypothesis: f (x) has a local extrema at x = c.

af

To prove: f x changes sign at x = c.

af

Main proof: The claim that f x does not change

af

sign at x = c a > 0 such that f x has the


same sign in c , c c , c + , i.e. f x is
either positive or negative x c , c
c , c + . On supposing that f x x
c , c c , c + f (x) increasing in c , c

f a

a
af
a

af

f a f
f
ac , c + f x = c is not an extreme point of y = f

(x) which is absurd because it is given that x = c is


extreme point, i.e. f (x) has an extrema at x = c.
Hence, the required is proved.
Theorem: (second derivative test of maxima and
minima): If a function y = f (x) defined on its domain is
twice differentiable in a -neighbourhood of the point
x = c (i.e. in N c ) such that
(i) f c = 0 and f c > 0, then f (x) has a local
maximum at x = c.
(ii) f c = 0 and f c < 0, then f (x) has a local
maximum at x = c.
Proof: (i) Hypothesis: y = f (x) is twice differentiable

af
af

af

af
af

af
f acf = 0 and f acf > 0
To prove: f (x) has local minimum at x = c.
Main proof: f acf = 0 c is a critical point of
y = f (x). y = f (x) is twice differentiable in
N acf f a x f exists in ac , c + f , for some
> 0.
Further f acf > 0
f a x f is increasing in ac , c + f
f a x f < f acf for c < x < c
(1)
and f a x f > f acf for c < x < c +
(2)

in N c .

Maxima and Minima

af

But it is given that f c = 0.


Hence, from (1) and (2), it is concluded that

(3)
af
and f acf > 0 for c < x < c +
(4)
(3) and (4) f a x f changes sign from ve to

f x < 0 for c < x < c

+ve as one moves from left to right in the neighbourhood of f (x) has a local minima at x = c.
(ii) Hypothesis: y = f (x) is twice differentiable in

af
f acf = 0 and f acf < 0
To prove: f (x) has a local maxima at x = c.
Main proof: f acf = 0 c is a critical point of f (x)
f (x) is twice differentiable in a N acf .
f a x f exists in ac , c + f
Further, f acf < 0
f a x f is decreasing in ac , c + f
f a x f > f acf for c < x < c
(1)
and f a x f < f acf for c < x < c +
(2)
But it is given that f acf = 0
Hence, from (1) and (2) it is concluded that
f a x f > 0 for c < x < c
(3)
and f acf < 0 for c < x < c +
(4)
(3) and (4) f a x f changes sign from +ve to
N c

ve as one moves from left to right in the neighbourhood of f (x) has a maximum at x = c.

af

f c is local minimum in (c , c + ).
Hence, the required is proved.

a xf > 0 for all x ac , c + f .


a f a xf
f
is increasing for all x ac , c + f .
f a x f > 0 for all x ac , c + f
f a x f is increasing for all x ac , c + f
f a x f > 0 for all x ac , c + f

Note: f

n 1

af

875

f x is increasing for all x c , c +

Similarly, f

an 1f a xf

x c, c+

a xf < 0 for all x ac , c + f


is

decreasing

for

f
all

af
a
f
f a x f is decreasing for all x ac , c + f
f a x f < 0 for all x ac , c + f
f a x f is decreasing for all x ac , c + f

f x < 0 for all x c , c +

On methods of finding the extrema of a function


y = f (x) whose domain is an open interval or not
mentioned.
There are three methods to find out the extreme
value (values) of a continuous function y = f (x) whose
domain is an open interval (a, b) or not mentioned.
1. Method of definition.
2. Method of first derivative test.
3. Method of second derivative test.
Now each method will be explained seperately.
1. On method of definition: It consists of following
steps:
Step 1: To locate the critical points, i.e. the points
where f x = 0 or f x is undefined.
Step 2: To consider one of the critical points say c
and to find f (c), f (c h) and f (c + h).
Step 3: f (c) > f (c h) and f (c + h) both for small
values of h > 0 f x has the maximum value at
x = c a , b .
Similarly, f (c) < f (c h) and f (c + h) both for small
values of h > 0 f x has the minimum value at
x = c a , b .

af

af

af

a f

af

a f

Examples worked out:


1. Find the turning points on the curve f (x) = 4x3
3x2 18x + 6. Discriminate the maximum and minimum
points by definition.
Solution: f (x) = 4x3 3x2 18x + 6

af
Now f a x f = 0 12 x
2

f x = 12 x 6 x 18
2

6 x 18 = 0

876

How to Learn Calculus of One Variable

fa

2x x 3 = 0 2x 3 x + 1 = 0

3
or x = 1
2
f (x) = 4x3 3x2 18x + 6

x=

F 3I = 4 27 3 9 18 3 + 6
H 2K
8
4
2

= 13.50 6.75 27 + 6 = 14.25


and f (1) = 4 3 + 18 + 6 = 17
Now for h > 0, f

FG 3 + hIJ f FG 3 IJ
H 2 K H 2K

F 3 I F 3I
f G hJ f G J
H 2 K H 2K

= 15h2 4h3 > 0 for sufficiently small h.


Hence, we observe that

F 3 I < f F 3 + hI and
f
H 2K H 2 K

F 3 hI , for sufficiently small h > 0


H2 K
x =

3
is a minimum point and f
2

F 3I = 14.25
H 2K

is a minimum value.
Again f (x) = 4x3 3x2 18x + 6
f (1) = 4 3 + 18 + 6 = 17
Now for sufficiently small h > 0,
f (1 + h) f (1) = 4h3 9h2 < 0,
f (1 h) f (1) = 4h3 9h2 < 0
x = 1 is a maximum point and f (1) is the
maximum value of f (x) at x = 1.
2. On method of first derivative test: It consists of
following steps:
Step 1: To locate the critical points, i.e. the points
where f x = 0 or f x is undefined.
Step 2: To examine whether f x changes sign at a
critical point, say, x = c.
Step 3: (i) f c h > 0 and f c + h < 0 (i.e. from
plus to minus) f (x) has the maximum value at
x = c.
(ii) f c h < 0 and f c + h > 0 (i.e. from minus
to plus) f (x) has the minimum value at x = c.

af

af

a f

a f

af
a f

a f

Little < c At c

af
f a xf
f x

Little > c

Nature of the
point c

+ve

ve

Maximum

ve

+ve

Minimum

af

Notes: (i) To find the critical points where f x = 0


one should solve f x = 0.
(ii) If f x is a rational functions, one should put
numerator = 0 to see where f x = 0 since
denominator of a rational function cannot be zero
and one should put denominator = 0 to see where
f x is undefined.
2. On method of second derivative test: Instead of
examining f x for change of sign at a critical point
(critical points), one can use the second derivative
test to determine quickly the presence of a local
extreme value (local extreme values). It consists of
following steps (if f " (c) exists).
Step 1: To locate the critical points, where f ' (x)= 0.

af

af

= 15h2 + 4h3 > 0

and

Similarly, the sign of each critical point c1, c2, is


examined provided that there are more than one critical
point besides x = c.
The above method of procedure can be put in a
tabular form as given below (if f ' (c) exists)

af

af

af

af

Step 2: To find f x .
Step 3: To examines the positivity and the negativity

af

of f x at all the critical points located. Let x = c be


any one of the located critical points. Then

af

af

af

af

f c = 0 and f c < 0 y = f (x) has a local


maximum at x = c.
f c = 0 and f c > 0 y = f (x) has a local
minimum at x = c.
Similarly, each critical point c1, c2, c3, is examined
provided that there are more than one critical points
besides x = c.
The above method of procedure can be put in a
tabular form as given below (if f " (c) exists).

f c

bg

f c

bg

Nature of the critical point c

ve

Maximum

+ve

Minimum

Maxima and Minima

sign of f (n) (c)

Nature of the critical


point c

odd

+ve or ve

even
even

ve
+ve

Neither maximum
nor minimum
Maximum
Minimum

where to apply which method to determine the extreme


value (or, values) of a given continuous function y =
f (x) on a given open interval (a, b) or whose domain
is not mentioned.
1. Method of definition: This method is practically
of no use because this method is lengthy and involves
tedious calculation.
2. Method of first derivative test: This method is
practically convenient in the following cases:
(i) When the given continuous function y = f (x) can
be factorised, the first derivative test is always used.
(ii) When it is difficult to find out the second
derivative a given continuous function y = f (x), the
first derivative test is used.
(iii) When the answer is given, the first derivative
test is used to save time.
(iv) When f x = 0 or f x does not exist at a
critical point x = c, the first derivative test is used.

af

bg

af

How the know that there is no maximum or minimum


points:

af

1. When f x = 0 provides us an impossible or


imaginary result, then the given continuous function
y = f (x) on an open interval (a, b) or whose domain is
not mentioned, has neither the maximum nor the
minimum and one is required not to proceed further.
3

af

e.g. f x = 3 x + 4 x + 7 f x = 9 x + 4

4
x
9

4
which are imaginary and hence f (x) has no
9
maximum or minimum points.
2. When f x is undefined at a critical point x = c,
there is neither the maximum nor the minimum value
for a given continuous function y = f (x) at x = c.
3. When f x is found to be positive (or negative)
for all real values of x, then it cannot change sign and
consequently, there is neither the maximum nor the
minimum values for the given continuous function
y = f (x).
=

af
af

dy
(slope of the curve) at the
dx
maximum or minimum f (x)?
dy
= 0 the
Answer: At maximum or minimum x
dx
slope is zero the tangent line is parallel to the
x-axis.
Question: What is

y
f ( c) = 0
B
f (b) = 0
A

af

3. Method of second derivative test: This method is


practically convenient for any given continuous
function y = f (x) which has a second derivative such
that f x exists at a critical point x = c and f c 0 .

af

af

f x = 0 9x + 4 = 0 x =

x=b

f (c)

af

f (b)

af
af

Note: If c is a critical point, such that f c exists


and f c = 0 , and supposing that n 2 is the
anf c 0 then
smallest positive integer such that f
method of procedure of the second derivative test is
put in the following tabular form:

877

x=c

Remarks: 1. If the question says, find the max. and


/ min. values of a continuous function y = f (x) defined
on its domain D, then it is required to be found out
firstly where these values occur (i.e. the points of
maxima and / the points of minima) and then secondly
what are these values (i.e. the values of a continuous
function at the maximum and / the minimum points).
2. If the question says, examine for max. and / min.,
then it is required to be found out the points only
where the max. and / min. value (values) occur, i.e. the
points of maxima and / the points of minima are
required to be determined.

878

How to Learn Calculus of One Variable

3. The essential conditions for the existance of the


maximum and / the minimum value (values) of a
differentiable functions y = f (x) for x = c are (i) f x
must be zero for x = c and (ii) f x must change
sign as x passes through the critical point x = c.

Now, f x = 0 2ax + b = 0 x =

Problems Based on Algebraic


and Mod Function

On putting x =

af

af

Examples worked out on algebraic functions


1. Examine the max / min for the function y = x + 2.
Solution: y = x + 2

dy
= 1 (constant)
dx

Now,

dy
= 0 for extreme value of the function y =
dx

x+2
0 = 1 which is absurd / impossible

dy
= 0 has no solution
dx
y has no point of maximum or minimum.

f x = 3x + 2 x + 1
2

4 12
2 2 i
=
which are
6
6

imaginary

af

f x = 0 provides us imaginary values

f (x) has no point of maximum or minimum


3. Where is the minimum / maximum of the function y
= ax2 + bx + c.
Solution: y = f (x) = ax2 + bx + c

af

dy
= f x = 2ax + b
dx

= f x = 2a

(2)

af

b
2a

b
in (2), we have
2a

a f = 2a
F b I = 2a which is positive if a > 0.
f
H 2a K
f x

x = 2ba

x = 2ba

Hence, if a > 0, then at x =

b
, the function f (x)
2a

F b I = a b
H 2a K 4a
Fb
b
b
=

+c=c+G
4a 2a
H
F bI
= Gc
H 4a JK

f (x) = f

b
, if a < 0)
2a

b
+c
2a

2b
4a

I
JK

4. Where is the minimum / maximum of the function y


= 2x2 8x + 6.
Solution: y = 2x2 8x + 6 = f (x) (say)

+ 2x + 1 = 0

3x + 2 x + 1 = 0

x=

dx

2. Examine the max/min for the function y = x3 + x2 +


x + 1.
Solution: Let y = x3 + x2 + x + 1 = f (x)
2

d y

has a minimum and min. (maximum at x =

dy
= 0 provides us an impossible result
dx

af
f a xf = 3x

af

(1)

dy
= 4x 8
dx

d y

(1)

=4
2
dx
dy
= 0 in (1)
Now,
dx

4x 8 = 0
8
=2
4
Putting x = 2 in (2)
x=

(2)

Maxima and Minima

L d y OP
M
MN dx PQ
2

= 4

x=2

= 4 which is positive

x=2

At x = 2, the function y = f (x) has a minimum


and min. f (x) = f (2) = 2 22 8 2 + 6 = 2.
5. Where is the maximum / minimum of y = 2x2 3x.
Solution: y = 2x2 3x

dy
= 4x 3
dx

3
4

dx

=4

a fa f
a fa f

d y
dx

= 4 which is positive
x = 43

(3)

= 2x 5

x=2

= 4 5 = 1 (ve)

x =2

3
, the function has a minimum, and
4

min.
3
4

= 2x 5

At x =

LM d y OP
MN dx PQ
L d y OP
and M
MN dx PQ

=2

F 3I
H 4K

= 3

F 3I = 9
H 4K 8

6.
6x2 + 9x + 1.
Solution: f (x) = x3 6x2 + 9x + 1

af e
j
Now, f a x f = 0
3 e x 4 x + 3j = 0 x = 1 , 3
f a x f = 6 a x 2 f
f a1f = 6 which is ve
and f a3f = 6 which is +ve
2

f x = 3 x 4x + 3

= 2x 5

x =3

= 6 5 = 1 (+ve)

x=3

At x = 2, y has a maximum and

a f a f LMM x3 5x2
N
3

max. f x = f 2 =

Find the extreme values of the function f (x) = x3

x 5x

+ 6x + 4
Solution: y =
3
2

Now,

d y

x=

L d y OP
M
MN dx PQ

y= y

5x
x

+ 6x + 4 ?
3
2

7. Where is the max / min. of y =

dy
2
= x 5x + 6 = x 2 x 3
(1)
dx
dy
= 0 x 2 x 3 = 0 x = 2 , 3 (2)
Now,
dx

Again,

Hence, at x = 1, the function has a maximum and


max. f (x) = f (1) = 13 6 12 + 9 1 + 1 = 5, where as at
x = 3, the function has a minimum and min. f (x) = f (3)
= 33 6 32 + 9 3 + 1 = 1

dy
= 0 4x 3 = 0
Now,
dx
x=

879

52
2
=

3
2

+ 6x + 4

OP
PQ

x =2

+62+4

8
26
10 + 12 + 4 =
3
3
and at x = 3, y has a minimum and
=

a f a f LMM x3 5x2
N
3

min. f x = f 3 =

=9

45
17
+ 18 + 4 =
2
2

+ 6x + 4

OP
PQ

x=3

880

How to Learn Calculus of One Variable

8. Where is the maximum or minimum of y =

x
+
3 x

3 x
?
x

af

min y = y

Solution: y =

x = 23

FG x + 3 x IJ
H 3 x x K

a f a f a fa f
a f

Solution: y =

a f

a3 xf

a3 xf
a3 x f

1
2

3
x

y=

2+ x +

=0

=0

= x

2x.
2+ x +

dy
1
1
=

,|x|<2
dx 2 2 + x
2 2 x

1
2 2+ x

2 2 x

2 2+ x = 2 2 x

2+ x =
2

2 x

x 6x + 9 x = 0

2+ x=2 x

6x + 9 = 0

2x = 0

6x 9 = 0

x=0

9 3
=
6 2

d y
d L 3
=
M
Again,
dx MN a3 x f
dx
2

a3 xf

Now,

3 O

P
x PQ

positive

LM
MN

1
1
d

dx 2 2 + x
2 2 x

LM a f 1OP
N
Q
1 L 1
O
M a2 x f a1fP
2 N 2
Q
1
1
= a2 + x f a2 x f
4
4
1
1
2+ x
2
2

OP
PQ

32

23

L
O
F d y I = 6 MM 1 + 1 PP
GH dx JK
MM F 3 3I F 3I PP
NH 2K H 2K Q
3

x = 23

dx

f x =

32

d2y

af

6
x

2 x = f (x) (say)

dy
1
1
=0

=0
dx
2 2+ x
2 2 x

x=

=2
x = 23

9. Where is the maximum / minimum of the function

dy
3
=0
Now,
dx
3 x

3
, the function y has a minimum and
2

3 x
x
+
3 x
x

3 x x 1
x 1 3 x 1
dy

=
+
2
2
dx
3 x
x

At x =

which is

=
4

a2 + x f

af

f 0 =

quantity.

32

4 2

a2 x f
1

4 2

which is negative

Maxima and Minima

Hence, at x = 0, the function f (x) has a maximum


and

af

max. f x =

2 +

2 =2 2

(1)

a12f

12

12 + 2 3 12 2 3
,
6
6
Now, differentiating (1) again w.r.t x, we get

dx

F
f G 2 +
H
F
f G 2 +
H
(Positive)

1
3

, the function y has a maximum

x = 2

1
3

2
3 3

5
dy
= 0 2x 5 = 0 x =
2
dx
d y
dx

=2

LM d y OP
MN dx PQ
2

6
3

F 1 IJ = 6 FG 2 1 IJ + 12
Again, f G 2
H
H
3K
3K
F 1 IJ = 12 6 + 12 = 6
f G 2
H
3K
3
3

(negative)

3 3

dy
= 2x 5
dx

Now,

= 6 x + 12

IJ = 6 FG 2 + 1 IJ + 12
H
3K
3K
1 I
6
= 6 2 +
+ 12 =
J
K
3
3

d y

1
3

11. Find the max / min. values of y on the curve y =


(x 2) (x 3).
Solution: y = (x 2) (x 3)
= x2 2x 3x + 6
= x2 5x + 6

x=

af

, the function y has a

and max.

4 3 11
23

x = 2 +

At x = 2

12 4 3 12 2 3
=
6
6

f x =

af

12 144 132
6

x=

af

y= y

y= y

dy
2
= 0 3x + 12 x + 11 = 0
dx

x=

minimum and min.

10. Find the extreme value of the function y = (x + 1)


(x + 2) (x + 3).
Solution: y = (x + 1) (x + 2) (x + 3)
= (x2 + 2x + x + 2) (x + 3)
= x3 + 2x2 + x2 + 2x + 3x2 + 6x + 3x + 6
= x3 + 6x2 + 11x + 6

dy
2

= 3x + 12 x + 11
dx

At x = 2 +

881

= 2

x = 25

At x =

min.

x = 25

= 2 (positive)

5
, the function y has a minimum and
2

a f = ax 2fax 3f
F 5 2I F 5 3I = 1 F 1 I = 1
=
H 2 K H 2 K 2 H 2K 4

y= y

x = 25

x = 25

12. Find the extreme value of the function y = (x 1)


(x 2) (x 3).

882

How to Learn Calculus of One Variable

Solution: y = (x 1) (x 2) (x 3)
= x3 6x2 + 11x 6
dy
2

= 3x 12 x + 11
dx
dy

=0
dx

af

12 144 132
1
=2
6
3

dx

= 6 x 12

FG
H
F
f G 2 +
H

IJ = 6x 12
3K
O
1 I L F
1 I
= M6 G 2 +
12 P =
J
J
K
H
K
3
3
N
Q
1

f 2 +

x=2+

(positive)

FG
H

and f 2

F
f G 2 +
H
= 12

1
3

x=2

6
3

12 =

Hence, at x = 2 +

1
3

(negative)

af

x=2+

the function y has a

1
3

x =2

3 3

To find the max / min values of the function with the


help of first derivative only
Examples worked out:
1. Find the max / min values of the function y = (x
3)5 (x + 1)4.
Solution: y = (x 3)5 (x + 1)4
(1)

a f a f a f a f
= a x 3f 4 a x + 1f + a x + 1f 5 a x 3f
= b x 3g b x + 1g b x 3g 4 + b x + 1g 5
(2)
= b x 3g b x + 1g b9 x 7g
dy
5 d x +1
4 d x3
= x 3
+ x +1
dx
dx
dx
3

1
3

1
3

x=2+

1
3

a x 1fa x 2fa x 3f
LF 1 IJ 1OP LMFG 2 + 1 IJ 2OP LMFG 2 + 1 IJ 3OP
= MG 2 +
NH 3 K Q NH 3 K Q NH 3 K Q
LF 1 IJ FG 1 IJ FG 1 1IJ OP
= MG1 +
NH 3 K H 3 K H 3 K Q
=

x =2

minimum and min.

y= y

, the function y has a maximum

6
3

IJ = 6x 12
K
O
1 I L F
1 I
= M6 G 2
12 P
J
J
3K N H
3K
Q

a x 1fa x 2fa x 3f
LF 1 IJ 1OP LMFG 2 1 IJ 2OP LMFG 2 1 IJ 3OP
= MG 2
NH 3 K Q NH 3 K Q NH 3 K Q
LF 1 IJ FG 1 IJ FG 1 1 IJ OP
= MG1
3 KQ
NH 3 K H 3 K H
F 1 IJ FG 1 IJ FG1 + 1 IJ
= G1
H 3K H 3K H 3K
y= y

Now,

and max.

3 x 12 x + 11 = 0

d y

3 3

and at x = 2

x=

dy
=0
dx
4
3
x 3 x +1 9x 7 = 0

Now,

a fa fa
U
x = 3|
|
x = 1 V
7 |
x=
W
9 |

Maxima and Minima

we now investigate the sign of

dy
in the neighdx

bourhood of these points.


(1) for h > 0,

LM dy OP
N dx Q

= 3 h 3

3 h +1

a f

9 3 h 7

a4 hf a20 9hf > 0 for sufficiently small


L dy O
values of h and M P
N dx Q
= 3 + h 3 3 + h + 1 9 a3 + h f 7
= h a4 + h f a20 + 9h f > 0 for small values
=h

x =3+h

of h

a x 3f a x + 1f
5

4
x = 1

= (1 3)5 (1 + 1)4 = 0
(3) For h > 0

LM dy OP
N dx Q
= a x 3f a x + 1f a9 x 7f
LF 7 I F 7 h + 1I OP L9 F 7 hI 7O
= M h3
MNH 9 K H 9 K PQ MN H 9 K PQ
LF 20 hI F 16 hI OP b7 qh 7g
= MG
MNH 9 JK GH 9 JK PQ
LF 20 I F 16 hI ahfOP < 0 for sufficeintly
= M h
MNH 9 K H 9 K PQ
x = 79 h
4

x = 79 h

dy
does not change sign in
dx
moving from left to right through the point x = 3.
Hence, x = 3 is a point known as a point of inflection
(2) for h > 0,
Thus, we observe

LM dy OP
N dx Q
= a1 h 3f a1 h + 1f 9 a 1 h f 7
= a4 hf a h f a 9 9 h 7f
= a4 hf a h f a16 9 hf > 0 for small values
x = 1 h

of h

LM OP
N Q
= a1 + h 3f a1 + h + 1f 9 a1 + hf 7
= a4 + hf ah f a 9 + 9 h 7f
= a4 + hf ah f a16 + 9h f < 0 for suffi-

dy
and dx

dy
changes sign from plus to
dx
minus in moving from left to right through the point
x = 1. Hence, x = 1 is a point of maximum.
Thus, we observe

max. y =
x = 3 h

883

x = 1 + h

ciently small values of h.

LM dy OP
N dx Q
= a x 3f a x + 1f a9 x 7f
LF 7 I F 7 + h + 1I OP L9 F 7 + hI 7O
= M +h3
MNH 9 K H 9 K PQ MN H 9 K PQ
LF 20 + hI F 16 + hI b7 + 9h 7gOP
= MG
MNH 9 JK GH 9 JK
PQ
F 20 + hIJ FG 16 + hIJ b9hg > 0 for small
= G
H 9 K H9 K

small values of h and

x = 79 + h

x = 79 + h

values of h

dy
changes sign from minus
dx
to plus in moving from left to right through the point
7
x= .
9
Thus, we observe

884

How to Learn Calculus of One Variable

Hence, x =

Note: Whenever it is difficult to investigate (or,


dy
, one can calculate
determine) the sign of
dx
arithmetically by taking for h a sufficiently small
positive number like h = 0.0001 for example, in the

7
is a point of minimum.
9

a f ax + 1f

min. y = x 3

4
x = 79

Footnote: Whenever it is not specially asked to find


out the point of inflection but the point of inflection
occurs, it must be mentioned as in this question.
2. Find the extreme values of the function f (x) = (x
1) (x + 2)2.
Solution: f (x) = (x 1) (x + 2)2

a f a fa f

a f

f x = 2 x 1 x + 2 + 1 x + 2
= 2x2 + 2x 4 + x2 + 4x + 4
=3x2 + 6x = 3x (x + 2)

af
3x a x + 2 f = 0

x = 0, 2

dy
in the
dx
neighbourhood of these points x = 0 and x = 2.
Now, we investigate the sign of

af

a f af a f
f a0 hf = f a hf = 3h a2 h f < 0 for suffi-

f 0 + h = f h = 3h h + 2 > 0 for small h

ciently small h

af

f x changes sign from minus to plus in


moving from left to right through the point x = 0.
At x = 0, the function f (x) has a minimum and
min. (y) = f (0) = 4.
(2) for h > 0

a f a f ahf > 0 for small h


f a2 + hf = 3 a2 + hf h < 0 for sufficiently
f 2 h = 3 2 h

small h

af

f x changes sign from plus to minus in


moving from left to right through x = 2.
At x = 2, the function f (x) has a maximum and
max. (y) = f (2) = 0

a f

af

a f

f 0 + h = f h = 3h h + 2
= 3 0.0001 (0.0001 + 2) = 0.0003 0.0002
= 0.000006 =

a f

and f 0 h = f h
= 3h (2 h) = 3 0.0001 (2 0.0001)

= 0.0003 19999
=
.

f x =0

3 f x = 3x x + 2
(1) for h > 0,

af

above problem, we have f x = 3 x x + 2


(1) for h > 0,

(2) f 2 h = 3 (2 h) (h)
= 3 (2 0.0001) (0.0001)
= 3 (2.0001) (0.0001)
= (6.0003) (0.0001) =

and f 2 + h = 3 (2 + h) (+h)
= 3(2 + 0.0001) (+0.0001)
= 3 (2 + 0.0001) (+0.0001)
= 3 (1.9999) (0.0001) =
Problems on Mod. Functions
While finding maxima and / minima of the given mod.
functions, we should remember the following facts.
1. A function f (x) may have a maximum / a minimum
at a point x = c without being differentiable at that
point x = c.
2. If f c does not exist but f x exists in the
neighbourhood of x = c, then f c h is positive
and f + c + h is negative f x changes sign
from plus to minus at x = c while passing through x = c
from left to right (i.e.; f x changes sign from
positive to negative at x = c as we move from left to
right in the neigh-bourhood of x = c) f x has a
maximum at x = c.
3. If f c does not exist but f x exist in the
neighbourhood of x = c, then f c h is negative
and f c + h is positive f x changes sign
from minus to plus at x = c while passing through

af

b g

af
b g
af

af

af

af

b g

af
b g
af

Maxima and Minima

af

x = c from left to right (i.e.; f x changes sign from


negative to positive at x = c as we move from left to
right in the neigh-bourhood of x = c) f (x) has
minimum at x = c.
The above facts mentioned in (1), (2) and (3) may
be summarised in the tabular form which tells the
behaviour (or, nature) of the function f (x).
x

Slightly
<c

Slightly at x = c Nature of the point


>c
x = c / Nature of the
function f (x)

f ' (x) +ve

ve

f ' (c)
does
not
exist

Maxima

f ' (x) ve

+ve

f ' (c)
does
not
exist

Minima

Examples worked out in mod functions


1. Find the max / min value of the function f (x) = | x |.
Solution: f (x) is a continuous function

af

x
d f x
=
,x 0
Also,
dx
x

and

af

d f x
is ve for x < 0
dx

af

d f x
is +ve for x > 0
dx

af

x=0

=0

2. Find the max / min value of the function


f (x) = | x3 | + 1.
Solution: f (x) = | x3 | + 1

b g=

d f x

x3

dx

x3

af

,x 0

d f x
3 x
2
=
= 3x when x < 0
dx
x

af

3x 2 + 0 , x 0

(1)

d f x
3x
2
and
=
= 3 x , when x > 0
dx
x

(2)

af

d f x
changes sign from ve to
dx
+ve in passing through x = 0 from left to right f (x)
has the minimum at x = 0.

(1) and (2)

a f LMN

min. f x =

OP
Q

+1

x =0

=1

3. Find the max / min value of the function


f (x) = | x + 1 | + 3.
Solution: f (x) = | x + 1 | + 3

af

x +1
d f x
=
, x +10
dx
x +1

af

d f x
= +ve for x < 1
dx
= ve for x > 1

af

d f x
changes sign from plus to minus while
dx
passing through x = 1 from left to right.
At x = 1, f (x) has the maximum

af

Hence, max. f x = x + 1 + 3

af

d f x

changes sign from minus to plus while


dx
passing through x = 0 from left to right
At x = 0, f (x) has the minimum
min. f x =

af

3 x
d f x

=
dx
x

885

x = 1

=3

4. Find the max / min value of the function


f (x) = | sin 4x + 3 |.
Solution: f (x) = | sin 4x + 3 |
We know that 1 sin 4 1

1 + 3 sin 4 x + 3 1 + 3

2 sin 4 x + 3 4
(sin 4x + 3) lies between 2 and 4
sin 4x + 3 is +ve
f (x) = | sin 4x + 3 |

= sin 4x + 3 (period being )


2

...(i)

886

How to Learn Calculus of One Variable

af

there is no need to consider the general value of the


angle for stationary points to identify the maximum
and / minimum value of the function. But only the
given particular value of the angle is to be considered
as a stationary point obtained from the equation
f x = 0 to identify the maximum and / minimum
value of the function.

d f x
= 4 cos 4 x
dx

af

d f x
=0
dx
3
x= ,
8 8

Now,

af

a f = 16 sin 4 x

Remember:

d f x
dx

d f x
dx

and

a f = + ve for x = 3

d f x
dx

Solutions

( = smallest +ve or ve angle having the given


sin, cos and tan
= any other angle having the same sin, cos and
tan
n = an integer.)

a f = ve for x =

Equations

1. sin = 0

= n

2. cos = 0

= 2n + 1

3. tan = 0

= n

4. cos = 1

= 2n

5. cos = 1

= 2n + 1

6. sin = k , 1 k 1

= n + 1

Problems based on finding the maxima and / minima


of a function when the interval in which the given
function is defined is not mentioned:

7. cos = k , 1 k 1

= 2n

While doing problems on finding the maxima and/


minima of a function when the interval in which the
given function is defined is not mentioned, we should
keep in mind the following facts.

9. a cos + b sin = c

af

Hence, max. f x = sin 4 x + 3

x=

= sin + 3 = 1 + 3 = 4
2

af

and min. f x = sin 4 x + 3

x=

3
8

3
+ 3 = 1 + 3 = 2
2
Note: From (i), max. f (x) = 4, min f (x) = 2.
= sin

1. If the interval in which a given function is defined


is not given, we should study throughout the domain
of definition of the given function in which it is defined.
2. Whenever the interval in which given trigonometric
function is defined is not mentioned, we consider the
general value of the angle for stationary points to
identify the maximum and / minimum value of the
function.
3. Whenever a particular value of the independent
variable is given at which we are required to
investigate the maxima and / minima of the function,

a
a

a f

8. tan = k , < k < = n +

a +b c

= 2n +

tan =
cos =

b
and
a
c
a + b2
2

Note: The following results are also worth noting.


1. sin n = 0
2. cos n = 1 n
3. sin n + = 1 n sin
4. cos n + = 1 n cos
5. tan n = tan

a f
a f a f
a f a f
a f a f
b g

Maxima and Minima

a
a
a
a

f
f

F
H

6. sin 2n + = sin
7. cos 2n + = cos
8. tan n + = tan
9. cot n + = cot

f
f

f n +

Worked out examples on trigonometric functions


1. Find the maximum and / minimum values of the
function y = sin x.
Solution: y = sin x

dy

= cos x
dx
Now, for the extreme values of y,

dy
=0
dx

dy
= 0 cos x = 0 x = n +
dx
2

af

Again, f x =
For even-n,

F
H

f n +

L d y OP
=M
MN dx PQ

d y
dx

d
cos x = sin x
dx

= sin x

x = n + 2

F I
H 2K

= a1f a1f sin


2
= a1fa1fa1f = 1 = which indicates minimum
= sin n +
n

value of y = f (x)
= sin x at x = n +

for odd-n
2

Hence, y has maximum at x = n +

(n being
2

even integer) where y max = 1

(n being odd
2

2. Find the maximum and / minimum values of the


function y = cos x.
Solution: y = cos x
dy

= sin x
dx
dy
=0
Now for the extreme values of y ,
dx
dy
= 0 sin x = 0 sin x = 0 x = n
dx

x = n + 2

F I
H 2K

= a1f sin
2
= sin n +
n

= (1) (1) (1) = 1 =


value of y = f (x).
= sin x at x = n +
and for odd-n

x=n+

integer) where y min = 1

x =n +

And y has minima at x = n +

I
K

= sin x

2. cos 2n = cos

LM d y OP
MN dx PQ

I
K

10. sec and cosec can never be less than 1.


N.B.: 1. sin 2n = sin

887

which indicates maximum

af

Again, f x =

d y
dx

d
sin x = cos x
dx

For even-n,

for even-n.
2

L d y OP
f an f = M
MN dx PQ
2

= cos x

x =n

x =n

= cos n

888

How to Learn Calculus of One Variable

a f

a f

n
which indicates
= 1 = + 1 = 1 =
maximum value of y = f (x) at x = n for even-n and

a f LMM d y OPP = cos x


N dx Q
= cosn = 1 a1f
= a1fa1f = 1 = which indicates minimum

Fd yI
GH dx JK
2

for odd-n, f n =

x=n

= sec n

value of y = f (x) at x = n for odd-n.


Hence, y has maxima at x = n (n being an even
integer) where y max = + 1
And y has minima at x = n (n being odd integer)
where y min = 1
3. Show that the function f (x) = tan x has neither
maxima nor minima.

af

af

Solution: f x = tan x f x = sec x ,

x n +

.
2

af
1

f x = 0 sec x = 0

= 0 which

cos x

)
2
Hence f (x) = tan x has neither maxima nor minima.
4. Discuss the extreme values of the function, y =
sec x.

is not possible. (f (x) is undefined for x = n +

Solution: y = sec x, x n +

dy
= sec x tan x
dx

dy
= 0 (for extreme values)
dx
sec x tan x = 0
sec x = 0 or tan x = 0
But sec x 0 always and
tan x = 0 x = n
Now,

j}

x=n

{atan n f + asec n f }
2

= 1 if n is even and = 1 if n is odd, which indicates

a f

y has minima at x = n (n, even) and y min = f n ,


n even
= sec n
= 1 and y has maxima at x = n (n odd) and y (max)
= 1.
5. Find the maximum values and / minimum values of
the function y = f (x) = a sec x + b cosec x (0 < a < b),

FG
H

in 0 ,

IJ
K

n
.
2

Solution: y = a sec x + b cosec x (defined for x

Now, for extreme values of y, f x = 0

af

= sec x tan x + sec x

x =n

x=n

n
)
2

n dy
,
= a sec x tan x b cosec x cot x
2 dx
dy
=0
Now, for the extremum values of y ,
dx
For, x

a sec tan x b cosec x cot x = 0

sec x tan x
b
=
cosec x cot x a
3

tan x =

F I
H K

b
b
tan x =
a
a

1
3

(only real root is

considered)
dy
= a sec x tan x b cosec x cot x
dx
2

d y

2
3
2
2 = a sec x tan x + a sec x + b cosec x cot
dx
x + b cosec3 x

d2y

is + ve if 0 < x <
2
2
dx

Maxima and Minima

y min = a sec x + b cosec x

FG
H

tan x =

ch

1
b 3
a

y min = a 1 + tan x + b 1 + cot x

= a 1+

2
3

+ b 1+

a3
2

= a3

F
H

I
K

IJ
K

tan x =

ch

1
b 3
a

2
3

a3 + b3 + b3

= a3 + b3

F aI
H bK

= 4 cos cos 2

b3 + a3

6. Find the maximum and / minimum values of the


function y = sin 2 + sin 2 where + = .
2

Solution: y = sin + sin

fq as + =

For the extreme values of y ,

b
g
sin 2 = sin a2 2f

dy
=0
dx

n
+
2
2

dy
= sin 2 sin 2 2
d
2

dx

n
+
2 2

n
+
2
2
for odd n and cos > 0, and ymax = 1 + cos
Similarly if cos < 0 then y has minima for
n
n
+
+
(n odd) and maxima for =
2
2
2
2
(n even),
ymin = 1 + cos
ymax = 1 cos
=

n
)
2
Now for the extreme values of the function y,
dy
=0
dx
dy

= 0 sec x tan x cosec x cot x = 0


dx
sec x tan x = cosec x cot x

2 = 2 2 + 2n

d y

= 1 cos if n is even
= 1 + cos if n is odd

and

7. Find the maximum and / minimum values of the


function y = sec x + cosec x.
Solution: y = sec x + cosec x (which is defined for

sin 2 sin 2 2 = 0

Now,

FG + n IJ < 0 if n is odd and cos > 0.


H2 2 K
1
cos 2 + cos b2 2g
Now y = 1
2
= 1 cos cos b2 g
F n IJ = 1 cos cos n
fG +
H2 2 K

for n = even integer and cos > 0, and

dy
= sin 2 sin 2 2
d

and cos > 0.

ymin = 1 cos y has manima for =

1
cos 2 + cos 2 2
2

=1

FG + n IJ = 4 cos cos n > 0 if n is even


H2 2 K

Thus y has the minimum values for =

1
1
1 cos 2 +
1 cos 2
2
2

1
cos 2 + cos 2
=1
2

And f

3
2

889

= 2 cos 2 + 2 cos 2 2

sin x = cos x

890

How to Learn Calculus of One Variable


3

tan x = 1

tan x = 1
x = n +

sin x
cos x
dy
= 0 2a 2
2b 2
= 0 [for
3
dx
cos x
sin 3 x

max or min value of y]

tan x =

Now,

d y
dx

= sec x sec x + tan x sec x tan x

cosec x cosec 2 x cot x cosec x cot x

dx

cosec x cot x
= sec x (sec2 x + tan2 x) + cosec x (cosec2 x + cot2 x)

L d y OP
M
MN dx PQ
L d y OP
and M
MN dx PQ

= ve when n is odd

(1)

= +ve when n is even

(2)

x = m +
4

x = m +
4

and given function y = sec x


4

+ cosec x is minimum when n is even in x = n + .


4

Hence, y has maxima at x = n +


for n-odd
4

for n-even.
and y has minima at x = n +
4
n is odd in x = n +

bg

max

dy
= 2a 2 1 + tan 2 x tan x 2b 2 1 + cot 2 x
dx

cot x

dy
2
2
3
2
= 2a tan x + 2a tan x 2b cot x
dx
2b2 cot3 x

d y

d y

2
2
2
2
2
2
2 = 2a sec x + 2a 3 tan x sec x + 2b
dx
cosec2 x + 6b2 cosec2 x cot2 x which being a sum of
squares is positive.
2

dx

(1) and (2) y = sec x + cosec x is maximum when

b
2
b
[ 3 tan x since a square
a
a
cannot be negative]

= sec x + sec x tan x + cosec x +

tan x =

Now,

d y

bg

= 2 2 and y

min

= 2 2 .

8. Find the minimum values of the function y =


sec2 x + b2 cosec2 x; (a > 0, b > 0).
Solution: y = a2 sec2 x + b2 cosec2 x which is defined
n
for x
2
dy
2
2
2
2

= 2a sec x tan x 2b cosec x cot x


dx

b
a

minimum value when tan x =

has

a f = a yf = a sec x + b cosec
= a e1 + tan x j + b e1 + cot x j
F b I + b FG1 + 1 IJ
= a 1+
H a K H tan x K
F b I + b F1 + a I
= a 1+
H aK H bK

f x
2

min

min

a2

= + ve at tan x =

= a2 + ab + b2 + ab
= (a + b)2
9. Find the maximum and / minimum values of the
function of y = a cos x + b sin x; (a > 0, b > 0).
Solution: y = a cos x + b sin x

891

Maxima and Minima

dy
= a sin x + b cos x
dx

b
, then sin x =
a

a
2

a +b

tan x > 0)

b
2

a +b

cos x = sin x tan x = 1

dx

= a cos x b sin x

dy
= cos x sin x
dx

d y

The positive values of sin x and cos x make

dx
negative and the negative values of sin x and cos x
2

dx

a +b

aa

a +b
b

x = n + 4

a +b

a +b

a 2 + b2
a +b
2

a f

n+

= 1 1 2
Now, for even-n,

F d yI
GH dx JK
2

= a 2 + b2

and the min. value =

bb

x = n +
4

= 2=

has maximum for x = n +

a
2

a +b

F d yI
R F I F IU
= Ssin G n + J + cos G n + J V
GH dx JK
T H 4K H 4KW

U
R
= Sb 1g sin + b1g cos V
4
4W
T
F I
= a1f a1f sin + cos
H 4 4K

= sin x cos x = sin x + cos x

positive

The max. value =

d y
dx

make

= (a cos x + b sin x)

d y

n = 0 , 1 , 2 , ...
4

x = n +
Again

d y

dy
= cos x sin x
dx

Again,

a +b

dy
= 0 cos x sin x = 0
dx

(both +ve or both ve, as

10. Examines the function y = sin x + cos x for extreme


values.
Solution: y = sin x + cos x

b
tan x =
a

and cos x =

a +b

dy
= 0 , we get
Now on putting
dx
a sin x + b cos x = 0
b cos x = a sin x

Now, when tan x =

a +b

b
2

a +b

which indicates that y

when n is even integer.


4

Again for odd-n,


2

Fd yI
GH dx JK
2

x = n +

2 = which indicates that y

892

How to Learn Calculus of One Variable

has minimum for x = n +


Hence, at x = n +

when n is odd integer.


4

when n is even integer


4

F n + I
H 4K
F I + cos F n + I
= sin n +
H 4K H 4K
= a1f sin n + a1f sin n
= a1f 2 = 2 when n is even integer
F I
and y = f H n + K
4
F I + cos F n + I
= sin n +
H 4K H 4K
= a1f sin n + a1f sin n
= a1f 2 = 2 when n is odd integer.
y max = f

min

11. Find the maximum and / minimum values of the


function y = sin x (1 + cos x).
Solution: y = sin x (1 + cos x)

dy
d
=
sin x + sin x cos x
dx dx

af

and cos x = 1 = cos x = 2n + , n I


Further,

dy
= cos x cos 2x
dx

d y
dx

= sin x 2 sin 2x = (sin x + 2 sin 2x)

Fd yI
G
H dx JK
R F I + 2 sin F 2n I cos F 2n I UV
= Ssin 2n
T H 3K H 3 K H 3 K W
R F I + 2 sin F I cos F I UV
= Ssin
H 3K H 3KW
T H 3K

U
R
= Ssin + 2 sin cos V < 0
3
3W
T 3
R U
and Ssin + sin cos V > 0
T 3 3 3W
2

x = 2 n 3

Hence, y has maxima at x = 2n +


minima at x = 2n

and y has
3

F I
H
3K
d L
1
d L
1
O
O
=
+
=
+
sin
x
sin
x
cos
x
sin
sin
2
x
x
2
PQ dx MN 2 PQ
dx MN
2
F I RS1 + cos F 2n + I UV
= sin 2n +
H
H
3K T
3KW
1
= cos x + 2 cos 2 x = cos x + cos 2 x
2
F
I
3 F 1I
3 3 3 3
= sin G1 + cos J =
1+ J =
=
G
dy
3H
3K 2 H 2K 2 2
4
= 0 cos x + cos 2 x = 0 (for extreme
Now,
dx
I
F
value)
and y = f G 2n J
H
3K
2 cos x + cos x 1 = 0
F I RS1 + cos F 2n I UV
= sin 2n
1 1 + 8
1 3 1
H
H
3K T
3KW
cos x =
=
= or 1
4
4
2
F I RS1+ cos F I UV = sin RS1+ cos UV
1

= sin
Again cos x = = cos x = 2n , n I
H 3K T H 3KW
3T
3W
2
3
3

Where y max = f 2n +

min

Maxima and Minima

F I
H K
F d yI
Lastly, G
H dx JK a f
= lsin a2n + f + 2 sin a2n + f cos a2n + fq
= ksin + 2 sin cos p
= l0 + 2 0 a1fq = 0
F d yI
= a cos x 4 cos 2 x f a
and G
f
H dx JK
1
3
3 3 3
3
1+
=
=
2
2
2
2
4

12. Show that the function y = sin x +


a maximum value at x =

x = 2 n +

a fq
l a f
a
fq
l
= kcos + 4 cos 2 p
= oa1f + 4 a 1f t
= k1 + 4p = a3f 0
Therefore y has a point of inflection at
x = a2 n + f because at x = a2 n + f ,
= cos 2n + + 4 cos 2 2n +
= cos + 4 cos 4n + 2
2

d y

= 0 and

af

af

a t f n x + bt f n + c = 0 where a 0 and t f n x
represents a trigonometric function (sin x, cos x, tan
x, cot x, sec x, cosec x), we have two general values of
the angle x (if the interval in which a given
trigonometric function is defined is not mentioned in
the problem).
(ii) Whenever we have a linear trigonometric

af

a f

equations of the form: at at f n x + b = 0 , a 0 we


have one general value of the angle x of the
trigonometric function of x (sin x, cos x, tan x, etc) for
b
the equation t f n x =
if the interval in which
a
the given trigonometric function is defined is not
mentioned in the problem.

af

and find the

1
sin 2 x
2

dy
= cos x + cos2 x
dx

Now,

dy
= 0 (for extreme value)
dx

cos x + cos 2 x = 0
2

2 cos x + cos x 1 = 0

cos x =

1 1 + 8
1 3 1
=
= or 1
4
4
2

Again, cos x =
Further,

d x

0
3
dx
dx
Note: (i) Whenever we have a quadratic
trigonometric equations of the form:
2

Solution: y = sin x +

x = 2 n +

1
sin 2 x has
2

corresponding maximum value.

x = 2n + 1

893

dy
= cos x + cos2 x
dx

d y
dx

Fd yI
GH dx JK

d
cos x + cos 2 x = sin x 2 sin 2 x
dx

x=
3

= sin
=
=

F
GH

1
= cos x =
2
3
3

= sin x 2 sin 2 x

x = 3

2
2 sin
3
3

3
3
2
2
2

I
JK

3+2 3
3 3
=
=
2
2

indicates y has a maximum at x =

which

and at x =
3
3

894

How to Learn Calculus of One Variable

LM
N

y max = sin x +

= sin

1
sin 2 x
2

OP
Q

af

y min = f 0 = sec 0 = 1

x=
3

F
H

+ sin 2
3 2
3

14. Discuss the extreme values of the function y = x


sin x at the origin.
Solution: y = x sin x

I
K

3 1
3 2 3+ 3 3 3
=
+
=
=
2
2
2
4
4

Now,

Note: When x = ,

d y
2

dx

F d yI
GH dx JK

=0

= cos x 4 cos 2 x

x=

d y

F d yI
GH dx JK
F d yI
and G
H dx JK
2

a f a f

d x

0 which at x = ,

dy
= sec x tan x
dx

dy
= 0 (for extreme values)
dx
sec x tan x = 0
sec x = 0 or tan x = 0

LM
N

= sec x tan x + sec x


x =0

= cos 0 = 1 0

dy
d y
d y
= 0 , 2 = 0 and
0 which y has
3
dx
dx
dx
an inflection point at the origin (i.e. at x = 0).

Problems based on logarithmic and exponential


functions.

dy 1
=
dx x

Now, for the extreme values of y ,

jOPQ

x =0

dy
=0
dx

1
= 0 which is not possible which f (x)
x
does not have max and / min values.

tan x = 0 which x = 0 is an extremum

x =0

Hence, we observe at x = 0,

But sec x 0

= cos x

1. Find the max and / min values of y = log x.


Solution: y = log x, x > 0

Now,

F d yI
GH dx JK

= sin 0 = 0

x=0

13. Discuss the extreme values of the function y =


sec x at the origin.
Solution: y = sec x

x =0

x =0

dx
dx
we get an inflection point

= sin x

= 0 and

dy
= 0 (for extreme values)
dx

x=

= cos 4 cos = 1 4 1

=1+ 4= 5 0
Hence, at x =

1 cos x = 0 cos x = 1 = cos 0 x = 0 is


an extremum

dy
d y
= 1 cos x and
= 0 sin x = sin x
2
dx
dx

, i.e.,

1 (0 + 1)
= 1 = which indicates y has a minimum at x = 0
i.e., origin and at x = 0 (i.e. at origin)

2. Find the max and / min values of y = ex.


Solution: y = ex

x
dy
=e
dx

Maxima and Minima

dy
=0
dx
x
x
e = 0 which is not possible (since e > 0
always) which f (x) does not have max and / min
values.
Now, for the extreme values of y ,

bg

f e =

1
x

Solution: y = x , x > 0
(1)
Taking log of both sides of the equation (1)

bg

e j

1
log x
(2)
x
Now, differentiating both sides of (2) w.r.t x
1

log f x = log x x =

UV 1
We

b1 1g f beg + f beg mbeg b1 log eg b2egr

y=x .

f e +f e

(1 log e) (2e)

3. Examine the following function for max and / min


1
x

b g b g RST e FGH 1e IJK

1 log e
e

895

b3 log

e=1 =

bg

1
e4

f e
e3

= ve (since e and e e > 0 ) which f (x) has


maximum at x = e.
1

1
x log x
1
1 log x
x

f x =
=
2
2
f x
x
x

af

af

af a

af

f x 1 log x

f x =

x x 1 log x
x

Hence, y max = e e

Problems based on combination of transcendental


functions
1. Find the max and / min values of the functions

(3)

f [3 f b x g = x

y = e + 2 cos x + e

Solution: y = e + 2 cos x + e
is given] (4)

af
a1 log xf = 0
x

bg

af

af

F 1 I a1 log xf a2 xf
x
H xK
2

=0 x =0

af

f x + f x

dy
=0
dx

e 2 sin x e = 0 which is only possible


when x = 0 which means
e 2 sin x e

log x = 1 x = e1 x = e
Now, f x =

x
x
dy
= e 2 sin x e
dx

1 log x = 0 3 f x = x x 0

1 log x

Now, for extreme values of y,

1
x

Now, putting f x = 0 in (4) for extreme values

xx

Now,

d y
dx

LM d y OP
MN dx PQ

= e 2 cos x + e

af

= e 2 cos 0

=1 2

x =0

1
=22=0
1

1
e

896

How to Learn Calculus of One Variable

LM d y OP = e + 2 sin x e
MN dx PQ
L d y OP = e + 2 sin x e
M
MN dx PQ
3

x =0

x=0

= 0 (i.e.

x =0

x=0

= 4 > 0 = +ve
Hence, y is minimum at x = 0.

dy
=0
dx

1 log x
=0
x

d y
dx
=

x 2 x 1 log x
x

which

af

a f

x x+2 e = 0

af

Now, f x = x e + 2 x e + e 2 + 2 x e
2

= x e + 4 x e + 2e

af

f 0 = 2 = + ve which f (x) has a minimum


at x = 0, and
x
x =0

af

= 0
2

e =01=0

a f

+ 4 2 e

+ 2e

e
= ve which again f (x) has a maximum at
x = 2 and

a f

F 1 I 2 x a1 log xf
H xK
a

Again, f 2 = 4e

dy 1 log x
=
2
dx
x

af
f a xf = x a x + 2f e

log x = 1 x = e

<0

f x = x e + e 2x

y min = x e

1 log x = 0

x = 0 and x = 2

Now, for the extreme values of y,

x
x = 0 or (x + 2) = 0 3 e 0

log x
,x>0
x

1
x 1 log x
dy
1 log x
x

=
=
2
2
dx
x
x

Again,

x=e

Now for max and / min values of f (x), f x = 0

log x
.
2. Find the max and / min for y =
x

Solution: y =

3 + 2 log e

1
e
3. Find the max and / min values of the function f (x)
= x2 ex.
Solution: f (x) = x2 ex

y max =

y has max. for x = e.

again zero when x = 0)

L d y OP = e + 2 cos x + e
M
MN dx PQ
L d y OP = e + 2 cos x + e
M
MN dx PQ

Fd yI
GH dx JK

3 + 2 log x

ymax = y min = x e

x
x = 2

a f

= 2 e
2

4
2

e
4. Find the max or min value of the function y = sin 2x
x.
Solution: y = sin 2x x

dy
= 2 cos 2 x 1
dx

Maxima and Minima

Now, for the extrema of y,

dy
= 1 cos x
dx
dy

= 0 1 cos x = 0 (for extrema of y)


dx

dy
=0
dx

2 cos 2 x 1 = 0
1
2

cos2 x =

1 cos x = 0 cos x = 1 x = 2n , n = 0,
1 , 2 , ...

2 x = 2n
3
x = n

Now,

n = 0 , 1 , 2 , ...
6

Again

dx

F d yI
G
H dx JK

= sin x

F d yI
GH dx JK

a f

x = n

a f RST FH

= 4 sin 2 n
6

I UV
KW

F I
= b4g sin G J
H 3K

x = 2n

x = 2n

= sin 2n = 0

(or,
6

L F IO F I
= Msin 2n + P n +
N H 3KQ H 6K

= sin n =
n
3
6
2
6

d y
dx

= sin x

x = n ) respectively,
6

LM F
N H

= sin x

Again,

or 4 sin < 0 > 0


3
3

and y min = sin 2n

y has maxima (or, minima) at x = n +

where y max

d y
dx

= 4 sin 2 x

= 4 sin

dy
= 1 cos x
dx
2

d y

897

I OP F n I
KQ H 6K

= sin n + =
n +
3
6
2
6
5. Find the maximum and minimum values of the
function y = x sin x.
Solution: y = x sin x

d y
dx

= cos x

F d yI
GH dx JK
3

= cos 2n = 1

x = 2n

Hence, at x = 2n
2

d y
dx

= 0 and

d3y
0 y has neither a
dx 3

maximum nor minimum at x = 2n .


6. Find the maxima and minima of the function y = sec
x + log cos2 x.
Solution: y = sec x + log cos2 x, x n +

dy
1
= sec x tan x +
2 cos x sin x
2
dx
cos x

= sec x tan x 2 tan x


= tan x (sec x 2)

898

How to Learn Calculus of One Variable

F I LMRSsec F I UV 2sec F I +
H 3 K MNT H 3 K W
H 3K
RStan F I UV OP
T H 3 K W PQ
L F I + tan F I 2 sec F I OP
= 2 Msec
H 3K
H 3K Q
N H 3K
b3 cos = cos afg
= 2 L2 + e 3 j 2 a2 fO = 2 [4 + 3 4] = 6 =
NM
QP

Now,

= sec

dy
= 0 (for extrema)
dx
tan x (sec x 2) = 0
tan x = 0 or (sec x 2) = 0

tan x = 0 x = n

and sec x 2 = 0 sec x = 2 = sec

x = 2n
Again,

dx

dy
= tan x sec x 2
dx

d y

= sec x sec 2 x 2 sec x + tan 2 x

F d yI
GH dx JK

LM
N

x = n

F
H

y min = f 2n

= sec x sec x 2 sec x + tan x

= sec n sec n 2 sec n + tan n

jOQP

x = n

= 1 (n even)

F d yI
Further, GH dx JK
= Lsec x esec x 2 sec x + tan
MN

= sec

I LMRSsec F 2n I UV
K MNT H 3 K W
RStan F 2n I UV OP
T H 3 K W PQ

2
+ log cos
3
3

= 2 + log

F
H

F I + log RScos F 2n I UV
H
3K
3K W
T H
F I + log RScos F I UV
= sec
H 3K T H 3KW

R F I UV b3 cos af = cos g
= sec + log Scos
3
T H 3KW
= sec 2n

= 1 (n odd)

= sec 2n

I
K

, we have
3

at x = n and y max = f n

x = 2n

= negative for all n, which indicates y has maxima

b g
= sec n + log acos nf

which indicates y has minima at x = 2n


Hence, at x = 2n

= sec x 2 sec x + sec x tan x

= sec x sec x 2 + tan x sec x tan x


3

jOPQ

F
H

1
2
= 2 2 log 2 = 2 (1 log 2)
= 2 + 2 log

x = 2 n 3

2sec 2n

F 1I
H 2K

I
K

+
3

Type 2: To find the maximum and / minimum values


of the function: (Trigonometric method)
y = a cos + b sin and y = a sin + b cos

Maxima and Minima

Working rule: Express a cos + b sin and


a sin + b cos as a single cosine and / a single sin
of an angle with the help of following rule:

1. Multiply and divide the given expression (both


2

a +b

sides) by
=

a +b

where

d y

2. Use A B formulas as the case may require


which transforms the given function into
2

a + b sin + .

y = a + b cos and /

How to find the maximum and / minimum values of the


function: a cos + b sin and / a sin + b sin .

a f
But the maximum value of cos a f = 1
2

3. Remember that maximum value of sin (or, cos )


is 1 and the minimum value of sin (or, cos ) is 1.
4. If the question is asked as: prove that the function
y = a cos + b sin and / y = a sin + b cos has
a max. and / min. value at some point, we use may also
2

acoefficient of cos f + acoefficient of sin f

3 a cos + b sin = a + b cos

2 method to find the extrem a at the indicated


dx
point. (See example 2 to follow)

Worked out examples on the max and / min values of


t-functions whose form is y = a cos + b sin and /
a cos + b sin .
1. Find the max. and / min. values of the function y =
3 sin + 4 cos .
Solution: y = 3 sin + 4 cos

(1)

3 1 cos 1

a cos + b sin

max

a +b 1 =

a +b

(1) becomes
2

(2)

And the minimum value of cos 1 = 1

a cos + b sin

min
2

= min a cos + b sin = a + b

Hence, remember:

R|Maximum value of a cos + b sin U|


|= a + b
|
S|Minimum value of a sin + b cos V|
||= a + b
||
T
W
R|Maximum value of a sin + b cos U|
|= a + b
|
S|Minimum value of a sin + b cos V|
||= a + b
||
T
W
2

1.

2.

(1)

3
4
and sin x =
5
5

y
= cos x sin + sin x cos which
5

a f

a f

y
= cos x + y = 5 cos x +
5
y max = 3 sin + 4 cos

max

=5

y min = 3 sin + 4 cos

min

= 5

(3)
2. Find the max and / min value of y =

3 sin x + 3

cos x or, prove that max. value of y is at x =

y 3
4
= sin + cos
5 5
5

Now, on supposing that cos x =

= max a cos + b sin


=

899

min at

7
.
6

Solution: y =
a2 +b2 =

= 12 =

and
6

y
2 3

3 sin x + 3 cos x = a sin x + b cos x

e 3 j + b3g
2

3+9

43=2 3
=

3
2 3

sin x +

3
2 3

cos x

900

How to Learn Calculus of One Variable

3
2 3
y

2 3

3 3

sin x +

on putting x =

cos x

2 3

1
3
sin x +
cos x
2
2

(i)

3
and
Now, we know that cos 30 = cos =
6
2

F I = sin 30 = 1 .
H 6K
2

sin

Hence, (1) becomes equal to

F I cos x
H 6K

cos

I
K

y = 2 3 cos x

2 3

F
H

= cos x

F
H

F
H

I
K

and

dy
=0 2
dx

F
H

(ii)

I
K
F I = 0 (for extreme
3 sin x
H 6K

dy

2 3 sin x
dx
6

sin x

I
K

= 0 = sin 0 = sin
6

F
H

Therefore, on considering sin x

F I sin x +
H 6K

(iii)

I
K

= sin 0
6

F
H

I
K

= 0 x = and considering sin x


=
6
6
6

sin x
2

d y
dx

3 cos

F I
H 6 6K

= 2 3 cos 0 = 2 3

F I = 2 3 = indicating max. at
H 6K

F 7 I = 2 3 cos F 7 I
x=
and f
H6K
H 6 6K
6
F 6 I = 2 3 cos
= 2 3 cos
H 6K
f

y max =

value)

2 3

= sin

F I = 2
H 6K

= 2 3 = indicating min. at x =

7
in (iv)
and
6
6

7
=x=+ =
6
6
6

F
H

= 2 3 cos x

I
K

(iv)

3 sin x + 3 cos x

7
and
6

x = 6

3 3
+
6
2

3 sin

3 3 3 4 3
+
=
=2 3
2
2
2

y min =

3 sin x + 3 cos x

= 3 sin

7
7
+ 3 cos = 3 sin 30 + 3 cos 30
6
6

x = 76

f a

4 3
3 3 3

=
= 2 3
2
2
2

Problems based on finding the extrema when a given


function is defined in an open interval.
To find the maximum and / minimum value of a given
function defined in an open interval, we should
remember the following facts.
1. A function defined in an interval (open) can have
maximum and / minimum values only for those value
of x which lie within this interval which means the
roots of f x = 0 must lie within the interval.

af

Maxima and Minima

af

2. If f x = 0 provides us the values of x which do


not lie within the given interval, we have not to
consider maximum and / minimum values at those value
of x (which do not lie within the given interval).
3. Let a function y = f (x) be differentiable in an open
interval (a, b). In order to find out all its maxima and /
minima in that open interval, we proceed as follows:
(a) Solve the equation f x = 0 . The real values of
x (or, roots) of the equation f x = 0 lying within
the interval (a < x < b) are considerable points. Among
these real values of x, we have to seek those values
of x which give us extrema (i.e. maxima and / minima)

af

af

of the function f (x) by using

d y
dx

method (i.e., second

dy
method (i.e.; first
dx

derivative test) or by using

derivative test).
4. In fact, f (x) may have several maxima and minima
in an open interval (a, b) or in an closed interval.
1. Find the maximum and / minimum value of the
function y = sin x + cos 2x in 0 , 2 = 0 < x < 2 .
Solution: y = sin x + cos 2x

dy
= cos x + sin 2 x 2
dx

dy
= 0 cos x 2 sin 2 x = 0 (for extrema)
dx
cos x 4 sin x cos x = 0
cos x (1 4 sin x) = 0
cos x = 0 or (1 4 sin x) = 0

Now, cos x = 0 = cos


Putting n = 0, x =

3
2
3 0 x 2

x = 2n + 1
2
2

3
or
2
2

and 1 4 sin x = 0 sin x =

Further,

1
4

dy
= cos x 2 sin 2 x
dx

d y
dx

= sin x 2 cos 2 x 2 = sin x + 4 cos 2 x

1
1 1
x = sin
4
4

F d y I = L+sin + 4 cos F 2 I O
G
MN 2
H 2 K PQ
H dx JK
L
O
= Msin + 4 cos P
2
N
Q
2

x = 2

= 3 = which indicates given function has a

.
2

minimum at x =
Again,

F d y I = sin x 4 cos 2 x
GH dx JK
L 3 4 cos F 3 2I OP
= Msin
H 2 KQ
N 2
= a+1f 4 a1f = 5 = which indicates given
2

x = 32

x = 32

function has a minimum at x =

3
2

Fd yI
Now, G
H dx JK
L
1O
L
F 1 IJ OP
= Msin sin
4 M1 2 sin G sin
H 4KQ
4 PQ
N
N
1
1O
L
= 4 M1 2 P
4
16 Q
N
1
F 1I = which indicates given
= 4 1
H 8K
4
2

x = sin

n = 1, x =

x =

and sin

901

1 1
4

function f (x) has a maximum at x = sin

1
4

902

How to Learn Calculus of One Variable

F d yI
J
Similarly, GH
dx K

= sin x + 1 2 sin x
1
x = sin 41

= sin x + 4 cos 2 x

x = sin

1I O F
1II
LM F
F
+ 4 G1 2 sin sin
J
P
H
K
H
H
4
4KK
N
Q
L
F F
1
1I I O
= Msin sin
+ 4 G1 2 G sin sin
J JP
4
4K K P
MN
H H
Q
1 IO
L1 F
L 1 F 1I O
= M + 4 1 2
= M + 4 1 P
P
H
K
16 Q
N4
N 4 H 8K Q
L 1 F 7 I O L 1 7 O L1 + 14 O
= M + 4 G JP = M + P = M
N 4 H 8 K Q N 4 2 Q N 4 PQ
= sin sin

15
=
4

which indicates given function has

maximum at x = sin

1
4

3
and
2
2

and given function has maxima at x = sin


1

1
.
4

1
At x = sin

x = sin

1 1
4

sin x = 14

1
At x = sin

At x =

= sin

1
and
4

y min = sin x + cos 2 x

x = 2

+ cos 2
2
2

= 1 + cos

ymin = sin x + cos 2 x

3
2

x = 32

3
3
+ cos 2
= 1 1 = 2
2
2
2. Find the maximum and / minimum values of the
= sin

= sin x + 1 2 sin x

FG IJ
HK

Solution: y = x + sin 2x

1
,
4

1
1
+1 2
4
4

I
K

1
+1 2
4

function y = x + sin 2x in 0 , 2 .

y max = sin x + cos 2 x

= 1 + (1) = 0 and at x =

given function has minima at x =

x = sin

1 1
4

RSF sin 1 I UV
4K W
TH
R F
UV
1 |
1I |
= sin sin
+ S1 2 sin sin
H
K
4 |
4 |
T
W
1 F
2I 9
= + G1 J =
4 H 16 K 8
F
H

= sin sin

1 1
4

x = sin

sin x = 41

1
2
9
+1
=
4
16 8

1
4

ymax = sin x + cos 2 x

dy
= 0 1 + 2 cos 2 x = 0 (for extrema)
dx
cos 2 x =

1 1
4

1
2
= cos 120 = cos
which
2
3

least value of 2x =

2n
x = sin

dy
= 1 + 2 cos 2 x
dx

2
and general value of 2x =
3

2
x = n
3
3

Maxima and Minima

b g

Putting n = 0 in the general value of x, we get

x = ,
3
3
Putting n = 1 and 2 in the general value of x, we get
4 2
5 7
,
,
, and
3 3
3 3
3 0 x 2

x=

dy
= 1 + 2 cos 2 x
dx

d2y
= 4 sin 2 x
dx 2

F d yI
GH dx JK
2

F d yI
GH dx JK

IJ
K

which indicates given function has

FG
H

x = 3

maximum at x =

x = 23

2
4
= 4 sin 2
= 4 sin
3
3

given function has minimum at x =

F d yI
GH dx JK
2

x=

4
3

= 4 sin 2 x

FG
H

IJ
K

= 4 sin

10
3

x = 43

b g

4
= 4 sin 2
= 2 3 ve
3

F d yI
GH dx JK
2

x = 53

4
5
and mimum at
.
3
3

2
.
3

4
and
3
3

Whereas y has a minima at x =

LM + sin 2 OP
3Q
N3

3
+
3
2

and at x =

2
5
and
3
3

,
3
x = 3

4
,
3

y max = x + sin 2 x

= 2 3 = + ve ,

and

x=

ymax = x + sin 2 x

= 4 sin 2

= 2 3 =

which indicates given function has a maximum at

Therefore, at x =

values for x 0 , 2
Now,

= 2 3 + ve

Hence, y has maxima at x =

2 4
5
x= ,
,
and
are considerable
3 3 3
3

903

F
H

x = 43

I
K

F I
H K

4
4
4
8
+ sin 2
=
+ sin
3
3
3
3

2
4
3 8 + 3 3
+
=
at x =
3
3
2
6

y min = x + sin 2 x

F
H

x = 23

F
H

2
2
+ sin 2
3
3

I
K

2
3
+ sin
=

3
3
3
2

4 3 3
5
and at x =
6
3

I
K

5
3

3
2
3. Find the maximum and / minimum value of the
y min =

function y = x + cos 2x in 0 , 2 = 0 < x < 2 .

904

How to Learn Calculus of One Variable

Solution: y = x + cos 2x
dy

= 1 + sin 2 x 2 = 1 2 sin 2 x
dx
dy
= 0 1 2 sin 2 x = 0 (for extrema)
dx

F I
H K

sin 2 x = = sin
2
6

a f

2 x = n + 1

Putting n = 1, x =

5
12

=
Now,

F
H

x= +

Putting n = 2, x = +
Putting n = 3, x =

F d yI
GH dx JK
2

12

18
3

=
2
12
12

= 2 cos 2 x 2 = 4 cos 2 x

F d yI
GH dx JK
2

= 4 cos

FG
H

= 4 cos 2

x = 12

12

IJ
K

= 4 cos
x = 512

FG
H

12

IJ
K

FG
H

= 4 cos 2 +
12
6

IJ
K

I and
K

x = + 512

FG
H

= 4 cos 2 +

5
12

IJ
K

= 2 3 = + ve which indicates given function

F
H

has minimum at x = +

I
K

5
.
12

.
12

5
= 2 3 which indicates
6

,
12

F + I and given function has minima at


H 12 K
5 F
5 I
x=
, +
.
12 H
12 K
Lastly, at x =

3
= 4
= ve which indicates
6
2

given function has a maximum at x =

F d yI
GH dx JK

Hence, given function has maxima at x =

dy
= 1 2 sin 2 x
dx

dx 2

x = + 12

17
5
=+
12
12

d2y

3
= 4
= 2 3 =
6
2
which indicates given function has a maximum at

b g

12

= 4 cos

n
x=
+ 1
2
12
Putting n = 0, x =

F d yI
GH dx JK

= 4 cos 2 +

5
.
12

given function has a minimum at x =

y max =

12

F
H

+ cos 2
12
12

I
K

+ cos
12
6

3
+
,
at x = +
12
12
2

F
H

y max = +

I
K

F
H

F
H

I
K

+ cos 2 +
12
12

I
K

905

Maxima and Minima

F I + cos F 2 + I
H 12 K H 6 K
F I + cos
= +
H 12 K 6
F I + 3
= +
H 12 K 2

at x =

5
,
12

y min =
=

F I
H K

5
5
+ cos 2
12
12

1
1
sin 2 x + sin 3x .
2
3

af

af

Putting n = 0, x =

Putting n = 1, x =

3
4

Putting n = 1, x = 2

1
1
sin 2 x + sin 3x
3
2

af
cos 2 x a1 + 2 cos x f = 0
cos 2 x = 0 or a1 + 2 cos x f = 0

FG IJ
H K

2 x = 2n + 1
or x = 2n
2
3

3
2

which indicates f (x) as a

.
4

F 3 I = sin 3 2 sin 3 3 sin 9


H4K
4
2
4

1
2
or cos x = = cos
2
2
3

maximum at x =

Now, f x (for an extreme values) = 0

2
3

af
f a x f = sin x 2 sin 2 x 3 sin 3x
F I = sin 2 sin 3 sin 3
f
H 4K
4
2
4

f x = cos 2x + 2 cos 2x cos x (on using C


+ D formula)
= cos 2x (1 + 2 cos x)

cos 2 x = cos

2
3

Again f x = cos x + cos 2 x + cos 3x

3
.
2
4. Investigate the maximum and / minimum values in
the interval 0 < x < of the function

Solution: f x = sin x +

Putting n = 0, x =

3
2
,
and
are only considerable
4 4
3
values of x.

I which is + 5
K
12

f (x) = sin x +

2
3

x=

5
3

12
2
Similarly, we can find the minimum value of y at
5
12

x = 2n

30x

F
H

f 4

x = 2n + 1

5
5
+ cos
12
6

x= +

f 4 or x = 2n 23

x = 2n + 1

= +

1
2

+ 21 3

1
2

1
2

+2

3
2

1 + 2 2 3
2
4 + 2 2
2

maximum at x =

which indicates f (x) has a

3
and
4

F 2 I = asin x 2 sin 2 x 3sin 3xf


H 3K

= sin

2
4
2 sin
3 sin 2
3
3

x = 23

906

How to Learn Calculus of One Variable

1
3
1
+2
= +
2
2
2

= sin

= which indicates f (x) has a minimum at


x=

2
3

Hence, at x =

, we have
4

F
H

a f FH 4 IK

=
at x =

1
2

1
1
sin 2 x + sin 3x
2
3

I
K

x = 4

LM3
N

3 2 +3+ 2
4 2 +3
=
6
6

y
3
1
=
cos x + sin x
2
2
2
2

e 3j

a +b =

3 2 3+ 2 4 2 3
=
=
6
6
2
and lastly, at x =
, we have
3
1
1
min. f x = sin x + sin 2 x + sin 3x
2
3

= cos x
2
6

F
H

F
H

I
K

F
H

I
K

dy

= 2 sin x
dx
6

4 =2

OP
Q

I
K

dy
= 0 (for an extreme value)
dx

F
H

2 sin x

FG
H

sin x
x

IJ
K

I
K

=0
6

=0
6

= n
6

x = n +
x = 23

3+1 =

Now,

I
K

+ 12 =

= cos cos x + sin sin x


2
6
6

x = 34

a f

(1)

1
1
1
1
1
1
+
1 +
=
+
3
2 3 2
2 2
2
2

a f FH

3 cos x + sin x

y = 2 cos x

3 1
3 1
9
= sin
+ sin
+ sin
4
2
2
3
4
=

3
4

3
, we have
4

Solution: y =

1
1
+
2 3 2

F 3 I
max. f a x f = f
H4K
1
1
F
I
= sin x + sin 2 x + sin 3x
H
K
3
2

3 3

2
4

a0 , 2 f = 0 < x < 2 .

1
1
3
= sin + sin + sin
4 2
2 3
4
=

5. Find the maximum and / minimum value of the


function y = 3 cos x + sin x in the interval

max. f x = f
= sin x +

2 1
4 1
+ sin
+ sin 2
3
2
3
3

(2)

Maxima and Minima

Putting, n = 0 in (2), we have x =

x=

F
H

, +
6
6

3 1 4
=
2 2
2
= 2
or, alternatively,

Putting, n = 1 in (2), we have x = +

3 0 x 2

I are only considerable values


K

F
H

y
= cos x
2
6

F I
H K
d y
F I

= cos x
H 6K
dx
F I
F I
f G J = 2 cos G J
H 6K
H 6 6K

6. Find the maximum and / minimum values of the

function y = sin 2x x when < x < .


2
2
Solution: y = sin 2x x

F
H

indicating max. at x =

I
K

F
H

I
K

and
6

a f


= 2 cos +
= 2 cos = 2 1
6
6 6

F I
= 2 1 = 2 = indicating min at x = +
H 6K
Hence, y max =

=
=
y min =

3 cos x + sin x

+ sin
6
6
3 1 3 1 4
3
+ = + = =2
2
2 2 2 2

F
H

= 3 cos +

F
H

F
H

+ sin +
6
6

I F
K H
F 3I 1
3 G
H 2 JK 2

= 3 cos
=

I
K

x = + 6

+ sin
6
6

I
K

dy
= 2 cos 2 x 1
dx

Now,

dy
= 0 (for extrema)
dx

2 cos 2 x 1 = 0
cos2 x =

I
K

1
2

2 x = 2n

x = 6

3 cos

3 cos x + sin x

I
K

and y min = 2

f +

y max = 2

dy
= 2 sin x
6
dx

= 2 =

F
H

I
K

y = 2 cos x

of x

907

x = n

Putting n = 0, in (1), we get x =

(1)

Putting n = 1, in (1), we get x =

<x<
2
2

x = and
are only two considerable
6
6
values of x
3

Again,

dy
= 2 cos 2 x 1
dx

908

How to Learn Calculus of One Variable

d y
dx

F d yI
G
H dx JK
2

= 4

F d yI
GH dx JK
2

x = 6

F I
= 4 sin G 2 J
H 6K

3
=
2

FG
H

= 4 sin

IJ
K

and
6

FG IJ
H K

2
2
= 4 sin
6
6

= sin

F
H

= sin

I
K

is only one considerable value of x


4

x =

dy
= cos x sin x
dx

again,

d y
2

= sin x cos x = sin x + cos x

F d y I = asin x + cos xf
GH dx JK
F I
= sin + cos
H 4 4K
F 1 + 1 IJ
= G
H 2 2K

x=

= sin

6 6


3
=

3 6
2
6

and ymin = sin 2 x x

30x

dx

Hence, y max = sin 2 x x

Putting, n = 1 in (1), we get, x = +

4 3
= 2 3 = which indicates y has
2

minimum value at x =

(1)

Putting, n = 0 in (1), we get, x =

indicates max. at x =

x = 6

x = n +

= 4 sin 2 x

x = 6

+
6
6


3
= sin + =
+
3 6
2
6
7. Find the maximum and / minimum values of the
function y = sin x + cos x in the interval 0< x < .
Solution: y = sin x + cos x
dy

= sin x + cos x
dx
dy
= 0 (for an extreme value)
Now,
dx
sin x + cos x = 0
sin x = cos x
tan x = 1

x = 4

2
2

indicating max. at x =

= 2 =

ymax = sin x + cos x

1
2

, we have
4

Hence, at x =

= sin

x = 4

x = 4

+ cos
4
4
+

1
2

2
2

8. Find the maximum value of the function y =


in 0 < x < .

log x
x

Maxima and Minima

log x
,x>0
x

Solution: y =

Now,

1
1 log x
1 log x
x
=
2
2
x
x

dy

=
dx

Solution: y =

dy
= 0 (for an extreme value)
dx

1 log x
x

dx
=

x = 2

F 1 I 2 x a1 log xf

H xK
x

3 + 2 log e e
e

x =e

3 + 2

4
4
d2y
=0+ 3 = 3
2
dx
x
x

Fd yI
GH dx JK
2

<0=

y max =

indicating maximum at x = e.

log e e 1
=
e
e

9. Find the maximum and / minimum values of the


function y =

x 2
+ , x > 0.
2 x

x =2

FG 4 IJ
Hx K
2

=
x =2

4 1
= =
8 2

which

indicates y has a minimum value at x = 2.


Therefore. at x = 2, we have

y min =

e
Hence, at x = e, y has maximum value and

e
1

a f

1
2
dy
= 2
dx 2
x

Again,

a f

But x = 2 0 , and x = 2 0 ,
Hence, only considerable stationary point is x = 2
at which we have to examine extrema.

x
3 + 2 log x
2

1
2

F d yI
G
H dx JK

x =4

x 2 x 1 log x

dy 1 log x
=
2
dx
x
x

x = 4

x=e =e

1
2
2 =0
2 x

d y

dy
= 0 (for an extreme value)
dx

log x = 1

x 2
+ ; 0 < x < ,
2 x

dy 1
2
=
dx 2 x 2

Now,

=0

1 log x = 0

Again,

909

LM x + 2 OP
N2 x Q

x =2

2 2
+ =1+1
2 2
=2
=

Verbal problems on maximum and / minimum values


of a function
The problems have been divided into four types:
1. Problems on numbers.
2. Problems on perimeter and area.

910

How to Learn Calculus of One Variable

3. Problems on volume.
4. General problems

sin

Working rule:
1. Find the function (if it is not given) of the quantity
whose maximum or minimum is required by expressing
the given conditions in symbols.
N.B.: It frequently appears (we observe / we see) as
a function of more than one variable, f (x, y) = c.
2. Then our next step is to express the quantity whose
max. and / min. is required in terms of a single variable
(we consider only such problems).
N.B.: By means of geometrical or other given relations
between the variables, all but one of these variables
must be eliminated.
3. The quantity (i.e., function) having thus been
expressed as a function of single variable, we put
dy
= 0 and solve for the independent variable x
dx
which will provide us roots x = a, b, c, etc.
2

4. Find

d y
2

L d y OP
and M
MN dx PQ
2

dx
x = a , b , c , ... etc
max and / min y exist (exists).

to test where

Remember: Given or specified means fixed i.e. the


quantity (volume, area, length, angle, height, etc)
which is given is a constant.
Explanation with the help of examples
Ex. 1: Given the length of an arc of a circle, find the
radius when the corresponding segment has maximum
or minimum area.
(Here, length of the arc is constant)
Ex. 2: Show that right circular cylinder of the given
surface and maximum volume is such that its height is
equal to the diameter of the base.
(Here, surface (area) is constant)
Ex. 3: Show that semi-vertical angle of the cone of
maximum volume and of given slant height is
tan

e 2j .

(Here, slant height is constant)


Ex. 4: Show that semi vertical angle of the right circular
cone of a given surface and maximum volume is

F 1I .
H 3K

(Here, surface area is constant)


N.B.: (i) Care should be taken to distinguish between
constants and variables.
(ii) To find the values of the independent variable at
which a differentiable function z = f 0 x , y can have
an extremum value, we must equate the derivative

a f

a f

dy d f 0 x , y
=
to zero [where y is expressed as a
dx
dx
function of a single variable x (i.e. y = f (x))] which
provides us the values of x and to find the values of
dz
= 0 ) in the
y, we put x = a (one of the roots of
dx
equation y = f (x).

Refresh your memory:


1. To be confirmed / to show the determined value of

a f

the function z = f 0 x , y , y = f (x) is max., show that

LM d z OP
MN dx PQ
2

= ve .

x=a

2. To be confirmed / to show the determined value of

a f
L d z OP
required to show that M
MN dx PQ

the function z = f 0 x , y , y = f (x) is min., we are


2

= + ve when z is

x =a

expressed as a function of single variable x.


3. We are provided a single equation of condition
f1 (x, y) = c which is expressed as y = f (x) = a function
of single variable x.
4. We are required to remember the following
formulae.
(i) Sphere:

OP
PQ

4 3
r
where r = radius
3
2
surface = 4 r

volume =

(ii) Cylinder:

Maxima and Minima


2

Volume: r h

dS
= 4 x 20
dx

d S

Curve surface: 2 r h

911

Total surface: 2 rh + r 2 ( 3 area of each

dx

=4

plane surface = r )
Where, h = height
r = radius of circular surface
(iii) Cone: (Right circular cone)

dS
= 0 4 x 20 = 0 x = 5
dx

1 2
r h
3
2

Curved surface: rl = r r + h
Semi vertical angle = = tan

Where h = height
l = slant height
r = radius of circular base.
(iv) Cube:
Volume: a3
Surface (area) = 6a2,
Where a = a side of a cube.

FrI
H hK

2. The sum of two numbers is given. show that their


product will be maximum if each number is equal to
half of the sum.
Solution: Let the sum of two numbers x and y = x + y
=a

dp
=0
dx

a 2x = 0

1. Divide 10 into two parts such that the sum of thier


squares is minimum.
Solution: x and y be two numbers such that x + y = 10
y = 10 x
Again on letting S = sum of squares of x and y, we
have

S = x + y = x + 10 x
2

extremum (minimum) in 0 , .
Hence, at x = 5, the function S attains the least (the
minimum) value.
S is the minimum when x = 5, y = 5.
N.B.: We recall that a quantity reaches a minimum
means it may not be the least value whereas a quantity
reaches (or, attains) the minimum means it is the least
value necessarily.

Type 1: Problems on numbers

= 4 = + ve

y= a x
p = xy = product of x and y = x (a x)

Examples worked out:

x =5

x =5

= 4

S reaches a minimum at x = 5. This is the only

Volume:

LM d S OP
MN dx PQ
2

and

= 2 x + 20 x + 100 , x 0

x=

a
2

Again,

d2 p
= 0 2 = 2
dx 2

LM d p OP
MN dx PQ
2

= 2

x = a2

x = a2

= 2 = ve

p reaches a maximum at x =

a
.
2

912

How to Learn Calculus of One Variable

This is the only extremum (maximum).


Hence, the maximum value of p is attained at
x=

a
a
a
, i.e. when x = , y = .
2
2
2

3. Divide the number 14 into two parts such that the


product of the two parts may be maximum or minimum.
Also find the maximum product.
Solution: Let x + y = 10
y = 14 x
and (x y) = x (14 x) = p (say)

dp
d
=
14 x x 2
dx dx

Now,

= 14 2x

dp
14
= 0 14 2 x = 0 x =
=7
dx
2

A
x
Putting (3) in (2), we have
Now, (1) y =

FG
H

dx

dp
A
= 2 1 2
dx
x

dp
=0
dx

FG
H

A
x

x=
= 2

=1

x=7

= 2 = ve

x =7

p reaches a maximum at x = 7.
Hence, maximum value of p is attained at x = 7
because this is the only extremum (maximum).

max. p = x 14 x

x =7

= 7 (14 7) = 7 7 = 49
Type 2: Problems on perimeter
1. Show that among rectangles of the given area, the
square has the least perimeter.
Solution: Let x = length of the rectangle
y = breadth of the rectangle
y

A 3x >0

IJ
K

IJ = 0
K

2 1

x = A

= 0 2 = 2

L d p OP
and M
MN dx PQ

FG
H

IJ
K

d p

A
x

p=2 x+

(3)

(4)

Again, on putting x =
condition which is (1)

A=xy=
A

y=

A in the equation of

Ay
=

(5)

(4) and (5) x = y = A


Now, to be confirmed whether p is maximum or
minimum, we need to know the sign of

FG 2 A IJ = 4 A when x =
Hx K x
L d p OP = L 4 A O
Now M
MN x PQ
MN dx PQ

x= A

Since, area is given, A = area = x y = constant


(1)
Perimeter = p = 2x + 2y = 2 (x + y)
(2)

4
A

= + ve

dx 2

d2p

=
x= A

4 A

e Aj

913

Maxima and Minima

Hence, p reaches a minimum at x = A . This


being the only extremum (minimum), the maximum
value of p is attained at , i.e., when the rectangle is
a square.
(*Note: An equation of condition (in this chapter) is
an equation between two variables x and y satisfying
a certain condition.)
2. Show that the perimeter of a right angled triangle
of a given hypotenuse is maximum when the triangle
is isosceles.
Solution: Let ABC = a right angled triangle
b = base = BC
p = perpendicular = AC
h = hypotenuse = AB
P = perimeter of ABC
A

h b
b=
2

p=

(1)

and perimeter = P = h + b + p

(2)

Putting (1) in (2), we have p = h + b +

af

h b

p = f b a function of single variable b

dp
= 0+1+
db
=1
dp
=0
db

2 h b

h
2

(5)

2
h

p=b=

(4) and (5)

ABC

d p
db

for b =

LM
OP
b
MN h b PQ
db
L
O
1
M h b eh b j a2bf bP
2
Q
= N
FG h b IJ
H
K
d F uI
(using the quotient rule
)
dx H v K
L
OP
b
M h b +
MN
P
h b Q
=
FG h b IJ
H
K
eh b j + b
=
FH h b IK FH h b IK
2

LM OP
N Q

d dp
d
1
=
db db
db
2

1
2 2

a f

2b

b
2

h b

a3 p > 0f

...(4)

2
2

p2 = h2 b 2
2

Putting (4) in (1), we have, p = h

d p

h b

=1

Now, h = b + p (by Pythogoras theorem)

p=

Given hypotenuse given h h = constant


2

is

isosceles
Now, to be confirmed whether p is max., we need

90

h b

h = 2b b =

h b b =h b

to know the sign of

=0

(3)

914

How to Learn Calculus of One Variable

FG
H

h b

L d p OP
M
MN db PQ
2

b=

h
2

IJ
K

e 2j

3
2

OP
PP
h
h I P

P
2 JK PQ
2

h
2

3
2

fa

d
d
4 c1 4c
=
dc
dc

d
4 + 5c c 2
dc
= 4 (5 2c)
= 20 8c
=4

Fh I
GH 2 JK

d
=0
dc
20 8c = 0

3
2

Now,

c=

= ve

20 5
=
8
2
2

Hence, P reaches a maximum at b =

h
2

= p

P = perimeter of ABC is max. when is


isosceles.
Type 3: Problems on perimeter and area
1. The perimeter of a triangle is 8 inches. If one of the
sides is 3 inches, what are other two sides for maximum
area of a triangle.
Solution: We know that
2

a fa fa f

= 4 43 4b 4c
= 4 1 (4 b) (4 c)
= 4 1 (4 5 + c) (4 c)
= 4 1 (c 1) (4 c)

LM
M
= M
MM F h
NM GH

F 2h h I
GH 2 JK
2

b=5c

a fa fa f

Again,

dc

= 8

LM d OP
MN dc PQ
2

= 8

c = 25

c = 25

= 8 = ve

2 and so is maximum when c =


b = 5 c

= s s a sb s c

Thus,

c = 52

LM
N

= 5

OP
Q

5
2

5
5
=
2
2

UV The triangle must be isosceles


W

c = 2 .5
b = 2.5

for the area to be maximum.


c

where S is the perimeter and a, b, c, the sides of the .


2s = P = a + b + c
8=3+b+c

2. The sum of a perimeter of a circle and a square is l.


Show that when the sum of the area is least, the side
of the square is double the radius of the circle.
Solution: Let r = radius of the circle
a = a side of the square

Maxima and Minima

Also,

dA
= 2 r 4a
dr
4

F
H

= 2 r a = 2 r
a

UV 4a + 2r = l
Perimeter of the square = 4a W
l 2 r = 4 a

Perimeter of the circle = 2 r

From the question,


Sum of the area of a circle and a square,
2

(1)
A = r + a
[In this expression, we observe there are two
variables which suggest us there should be (n 1) =
(2 1) = 1 relation which must be provided.]
Here, given relation is 2 r + 4a = l from which,
l 2 r
4
Putting (2) in (1), we get

we obtain a =

(2)

FG l 2 r IJ
H 4 K
dA
1

= 2r +
2 al 2 r f a 2 f
dr
16
2

A = area = r + a = r +

l 2 r
4
Now, for maxima and / minima,
= 2 r

R|
S|
T

a
2

915

I
K

a
ve when r <
dA
2

is
a
dr
+ ve when r >
2
dA
changes sign from minus to plus in moving
dr
a
from left to right through r = .
2
a
A has the minimum (least) value for r = , i.e.
2
when the side of the square is double the radius of
the circle.

3. Let ABC be a right angle triangle right angled at C.


Find the position of a point P on BC such that AP2 +
PB2 may be minimum.
Solution: Let P be a point on BC such that CP = x

(3)

a
x

dA
=0
dr

2 r
l 2 r = 0
4
2r =

l 2 r
4

2 r =

4a
4

2/ r = / a

F
H

Let BC = a and AC = b and also letting, y = AP2 +


PB2,

j b

y = b2 + x2 + a x
2

= a + b 2 ax + 2 x

dy
= 2a + 4 x
dx
dy
=0
Now,
dx

2r = a r =

a
2

I
K

916

How to Learn Calculus of One Variable

2a + 4 x = 0

Now,

2a a
=
4
2

x=

and

d y
dx

=4

LM d y OP
MN dx PQ

d l
d

= b tan2 sec + b sec3 + a cot2

cosec + a cosec3 which is positive for being


acute.

dl
= a cot cosec + b tan sec
d

= 4

x=

= 4 = + ve

x = a2

l has the minimum value for 1 , where

a
2

y is min. at x =

tan 1 =

a
2

F aI
H bK

1
3

and the minimum value of l


P

a
2
4. A point in the hypotenuse of a right angled triangle
is distant a and b from the two sides. show that the
Hence, AP2 + PB2 is min. when x =

F
H

3
3
length of the hypotenuse is at least a + b

Solution: Let ABC = = MPC

PB = a cosec

I
K

PC = b sec

F
H

Length of the hypotenuse = l = PC + PB


= a cosec + b sec

a cos b sin
dl
=
+
2
2
d
sin
cos

=
= 1

a3 + b3 a 3 + b3
2

= a 3 + b3

I
K

Area = A =

1
a b sin
2

FI
HK

a
a
a
tan = tan =
b
b
b

1
3

5. Two sides of a triangle are given. Find the angle


between them such that area shall be as great as
possible.
Solution: Let a and b be the sides (or a triangle) and
= angle between them

sin
dl
a
=0
=
3
d
cos b
3

a
b
+
sin 1 cos 1

3
2

b3

FG a + b IJ
H sin cos K

a3

3
2

=
C

a 3 +b 3

Maxima and Minima

Area = A =

1
a b sin
2

1
dA
= 0 a b cos = 0
2
d

d2 A
Now,

cos = 0 =
2

1
= a b sin
Now,
2
2
d
d A

LM d A OP
MN d PQ
2

=
2

dA
2
= 4a cos 2

dA
2
= 0 4a cos 2 = 0
d

cos2 = 0 2 = =
2
4

dA 1

= a b cos
d 2

LM d A OP
MN d PQ

= 8a 2 sin 2

= 8a 2 sin

F I
H K

1
1
= a b = ve
a b sin
2
2
2

FG IJ = 8a
H 2K

PS = 2a sin = 2 a sin

FG a co s , a sin IJ
H x
y K

= ve

2a

=
4
2

(Second method S = f (x))


Letting ABCD = a rectangle inscribed in a circle
whose radius = a
AB = 2x
Now, on drawing from o, a perpendicular oL to AB,
it is evident from the figure

D
o
2

a x

Area of PQRS = A = 2a cos 2a sin


2

PQ = PS PQRS = a square.
Thus, the required result.
Note: This problem also can be done by the
expression S = f (x) in the following way.

IJ
K

, P = 2a cos
4
2a

= 2a cos =
4
2

6. Show that maximum rectangle inscribed in a circle


is a square.
Solution: Let the equation of the circle be x2 + y2 = a2
Let PQRS be the rectangle inscribed in the circle
P, a point on the circle, = (x, y) = (a cos , a sin )
PQ = 2a cos
PS = 2a sin

Area is max at =

Hence, if =

FG
H

= 8a 2 sin 2

Area is max. at =
2
Angle between them is a right angle for the area
to be greatest.

917

a
x

A = 4 a cos sin = 2a sin 2

OL =

a2 x2

(1)

918

How to Learn Calculus of One Variable

BC = 2 OL = 2 a 2 x 2

Area of the rectangle = S = AB BC


= 2x 2
S = 4x

(3)

a2 x2

L 1 b 2 x g
ds
= 4 Mx
MN 2 a x
dx
L x
=4M
MN a x + a
L x + a x OP
=4M
MN a x PQ
L a 2 x OP
=4M
MN a x PQ
2

a2 x2

x2

OP
PQ

a2 x2

=0

a 2x = 0 a = 2x
2

a= x 2 x=
AB = 2 x = 2

a
2

2 2a 2 a 2

LM d s OP
MN dx PQ
2

2
2a

...(5)

16

a2
2

x=

2 a2
2

3
2

2a

(6)

(5) and (6) AB = BC = 2 a ABCD = square

, we have

a
2

FG a IJ |RSa FG a IJ |UV + FG a 2 a IJ 4a
2K
H 2 K |T H 2 K |W H
2
R|a F a I U|
S| GH 2 JK V|
W
T
2

3
2

BC = 2 a 2 x 2 = 2 a 2

Now, putting in this expression, x =

3
2

a2 2x2

ge

ds
=0
dx

j 2 ba2 xgx OPP


PP
FH a x IK
PQ
LM
OP
a
x
x
a
x
x

2
4
2
2
b
g
b
g
e j
e
j P
=4M
MM
PP
2 FH a x IK
N
Q
LM ea x j b 4 xg + ea 2 x j b x g OP
= 4 2/ M
PP
2/ ea x j
MN
Q
LM b4 xg ea x j + ea 2 x j x OP
=4M
PP
MN
ea x j
Q
b

a 2 x 2 2 2 x a 2 2 x 2

LM
MM
MM
N

OP
PQ

d 2s
=4
dx 2

(4)

Now, differentiating (4) w.r.t x, we have

d 2s
a
at x =
dx 2
2

Note: To test the sign of

a2 x2

Thus the require result.

(2)

3
2

Maxima and Minima

R 2a a UV 16 F a I
16 S
T| 2 W| = GH 2 JK
F 2a a I FG a IJ
GH 2 JK H 2 K
2

a
2

3
2

= ve

7. Show that among rectangles of given perimeter


the square has greatest area.
Solution: Let x and y be the sides of the rectangle
Perimeter = P = 2x + 2y = 2c (constant)
x

(i) Central section of a sphere = a circle


(ii) Central section of cone = a triangle
(iii) Central section of a cylinder = a rectangle
Examples worked out:
1. Prove that the height and the diameter of the base
of a right circular cylinder of given surface area and
maximum volume are equal.
Solution: Let r = radius of the base of the cylinder =
CB
h = height of the cylinder
v = volume of the cylinder
S = constant (given) = The surface area.

x+y=c
y=cx
Area = A = x y = x (c x) = cx x2

919

(1)
h

dA
= x 2x
dx

r
c

dA
=0
dx

x=

1
c
2

LM d A OP
MN dx PQ
2

Now,

= 2

x=

c
2

x = 2c

= 2 = ve

c
A is greatest for x =
2
Now, from (1), y

x = c2

= cx

The sum of the areas of the base (circle) and top


(circle) = 2 r 2 area of the curves surface = 2 r h
Total surface of the cylinder
= sum of the areas of the base and top + area of the
curved surface

S = 2 r 2 + 2 r h = (constant)

(1)

V = r2 h

(2)

Now, from 2 r h = S 2 r ,
2

x = 2c

A is the greatest when x = y =

= c

c c
=
2 2

c
2

i.e. when the rectangle is a square.


Type 4: Problems on volume
Note the following key point while working out the
problems on volume.
Whenever a figure is to be inscribed in another
solid figure, we are required to consider the central
section.

h=

S 2 r 2
2 r

(3)

F S 2 r I = 1 r eS 2 r j
GH 2 r JK 2
dV
1

= {r b 2 2r g + eS 2 r j 1}
dr
2

From (2), V = r 2

1
2

{e4 r j + eS 2 r j}
2

920

How to Learn Calculus of One Variable

1
S 6 r 2
2

Now, l 2 = h 2 + r 2 h 2 = l 2 r 2

F A r I r
GH r JK
F A rIJ r
h =G
H r K
2

h2 =

6 r 2 = 2 r 2 + 2 r h

4 r 2 = 2 r h

2r 2 = r h

h = b (base = b = 2r)

d 2v 1
0 6 2r = 6 r < 0.
=
2
dr 2

s
, v = volume is
6
maximum and, when the volume of the cylinder is
maximum, h = 2r = b. Hence, the result.
2. Show that the semi vertical angle of a right circular
cone of a given surface and maximum volume is
Thus, we observe when r =

sin 1

FG 1IJ .
H 3K

2A
A2

r + r2 r2
2
r
r

2A
A2

b 2g V

(5)

1 2 4 2
r h
9
Again on putting (5) in (6), we get
2

1 2 4
r
9

Solution: Let r = the radius of the base


h = OC = height
= AOC = semi vertical angle
surface of the cone which is given in the problem
2
= r + rl

=
l

2V

( 3 total surface of a cone = surface of the base +


area of the curve surface)
= A (constant), say
(1)

1
and volume = V = r 2 h
3

(2)

(7)

bg

dV
1
= f r = A A 2r 2 4 r 3
dr
9

1
A 2 Ar 8 r 3
9

2V
C

Now, differentiating (7) w.r.t r, we have

(6)

F A 2 AI
GH r JK
F A 2 A r I
1
= r G
9
H r JK
1
= A e Ar 2 r j = f br g
9

V2 =

(4)

Now, on putting (4) into (3), we have

S = 6 r 2
putting this value of S in (1), we have

Now, from (4)

A r2
r

and A = r 2 + rl l =

dV
1
=0
S 6 r 2 = 0
dr
2

(3)

{ e

1
dV
= A 2r r 2 + rl 8 r 3
9
dr

{ e

1
A 2r r 2 + rl 8 r 3
9

1
A 2 r 2 l 6 r 3
9

Maxima and Minima

g
b

1
A 2 r 2 l 3r
9
dV
1
= 0 A 2 r 2 l 3r = 0
dr
9
l
l 3r = 0 3r = l r =
3
=

FG
H

(8)

tan 2 = 0
tan =

IJ
K

AOC , sin =

AC r
r
= =
OA l
3r

FG IJ
HK

e j

2 .

Solution: v = volume of the cone =

(1)
(2)

1 2
r h
(3) (say)
3
Now, on putting (1) and (2) in (3), we get
3V =

l3 2
1 2 2
l sin l cos =
sin cos
3
3
dV l 3
sin 2 sin + cos 2sin cos

=
d 3
(4)
dV
=0
d

V =

l3
sin 3 + 2 sin cos2 = 0
3
sin 3 + 2 sin cos2 = 0
2 sin cos2 = sin 3
2 cos2 = sin 2 3 0

l
dV
sin cos2 2 tan 2
=
3
d
dV

> 0 for < tan 1 2


d
< 0 for > tan 1 2
V has the greatest value for = tan 1 2

Note:

l
and from the
3

1 2
r h
3
Where r = radius of the base circle
h = height of the cone
Now, if l = slant height of the cone then
r = l sin
h = l cos
where = BOC semi vertical angle

IJ
K

1
1
sin = = sin 1
3
3
3. Show that semi vertical angle of the cone of
maximum volume and the given slant height is
tan 1

= tan

l
dV
l
> 0 for r < and < 0 for r >
Also,
3
dr
3
V has the max. value for r =

FG
H

2 = tan 2 3

921

Local extreme values of a function in a closed interval


[a, b]:
Definitions: A function y = f (x) defined on a closed
interval [a, b] is said to have
A: 1. Local maximum at x = a (left end point) if f (a) >
f (a + h), h > 0.
2. Local minimum at x = a (left end point) if f (a) < f (a
+ h), h > 0.
B: 1. Local maximum at x = b (right end point) if f (b)
< f (b h), h > 0.
2. Local minimum at x = b (right end point) if f (b) < f
(b h), h > 0.
To find the local extreme values of a function f defined
by y = f (x) in a closed interval [a, b], one should
know:
1. How to find the local extreme values of a
continuous function f at the interior points of the
domain of the function f where the derivatives of the
function f denoted by f ' do not exist.
Rule: One should use the rule of the first derivative
test, i.e., one should find f c h and f ' (c + h)
where x = c is an interior point where f does not
exist and h is a small positive number and then use:
(i) f c h > 0 and f c + h < 0 local
maximum value at the interior point x = c of the domain
[a, b] of the function f.
(ii) f c h < 0 and f c + h > 0 local
minimum value at the interior point x = c of the domain
[a, b] of the function f.

922

How to Learn Calculus of One Variable

2. How to find the local extreme values of the


function f at the interior points of the domain of the
function f where the derivative of the function f
denoted by f is zero.
Rule: One should use the rule of the first derivative
test or the rule of the second derivative test.
3. How to find the local extreme values of a function
f at the left and the right end points of a closed interval
[a, b].
Rule (a): One should find f a + h , where h is a
small positive number, x = a is the left end point of the
given closed interval [a, b], where the given function
f is defined and f a + h = the value of the first
derivative of the function f for value of x little (just)
more than a and then use the rule:
(i) f a + h > 0 local minimum value at the
point x = a.
(ii) f a + h < 0 local maximum value at the
point x = a.
Rule (b): One should find f b h , where h is a
small positive number, x = b is the right end point of a
given closed interval [a, b] and f b h = the value
of the first derivative of the function f for a value of x
little (just) less than b and then use the rule:
(i) f b h > 0 local maximum value at the
point x = b.
(ii) f b h < 0 local minimum value at the
point x = b.
Notes: (A): The only possible points where a given
function f defined by y = f (x) in a closed interval [a, b]
can have local extreme values are

(B) While finding the local extreme values at the end


points of a closed interval [a, b], one is required to
find out f a + h at the left end point namely x = a
and f b h at the right end point namely x = b.

b g
b g

b g
b g

1. The critical points (also called stationary points or


turning points), i.e.,
(i) The interior points of the domain of the function f
where the derivative of the function f denoted by
f ' does not exist.
or (ii) The interior points of the domain of the function
f where the derivative of the function f denoted by f '
is zero.
2. The end points of a given closed interval [a, b],
where a given function f is defined.

This is why while finding the local extreme or global


extreme values of a function f defined by y = f (x) in a
closed interval [a, b], one should firstly locate the
critical points of the given function f.

ah

x=a

a+h

bh

x=b

b+h

The rule to find out the local extrema at the end


points can be put in tabular form:
At the left end pint x = a:
x

bg
f b xg
f x

Little > a

Nature of the point

+ve

Minima

ve

Maxima

At the right end point x = b:


x

bg
f b xg
f x

Little < b

Nature of the point

+ve

Maxima

ve

Minima

(C) One should note that there may be more than


one local extrema of a function in a closed interval.
This is why the question says to find out:
(i) The local extreme values (or simply the extreme
values)
(ii) The local maxima and / the local minima (or simply
the maxima and / the minima).
i.e., the words the extrema, the maxima and the
minima are used in plural to signify the local extrema
of a function defined in a closed interval.
Examples worked out:
1. Find the point of local maxima and minima of a
function f defined by f (x) = 3x4 4x3 + 5 in [1, 2].
Solution: f (x) = 3x4 4x3 + 5, x 1 , 2

bg

f x = 12 x 3 12 x 2

923

Maxima and Minima

bg
bg
b g

f x = 36x 2 24 x
Now, f x = 0
12 x 3 12 x 2 = 0
12 x 2 x 1 = 0
x = 0 and x = 1 which are the possible critical
points.
Hence, the only possible points where the extreme
values of the given function f defined in the closed
interval [1, 2] may occur, are
(i) x = 0 and x = 1, where f x = 0
(ii) x = 1 = the left end point and x = 2 = the right end
point of the given closed interval [1, 2].

bg

1. Local extrema at critical points namely x = 0, 1.


At x = 0, using the rule of the second derivative

bg

= +ve > 0

right end point x = 2.


Hence, at x = 0, f has neither a maximum or a
minimum.
At x = 1, f has a local minimum and at x = 1 and
x = 2, f has a local maxima.
2. Find the local maxima and minima of the function f

bg

f 0 = 36 24 0 = 0 which does give us


inference to calculate f 0 .
Now, f x = 72 x 24

bg

bg
e3 f b xg = 36x 24 xj
f b 0g = 72 0 24 = 24
2

Solution:

f has a local minimum at x = 1


2. Local extrema at the end points of the closed
interval:
(i) At the left end point x = 1
3
2
f 1 + h = 12 1 + h 12 1 + h
= 12 ve 12 + ve = ve < 0
f 1 + h < 0 f has a local maximum at the
left end point x = 1
(ii) At the right end point x = 2.
3
2
f 2 h = 12 2 h 12 2 h
Now, for convenience h = 0.1 can be put in

b g
g

b g

b g b g b g
f b2 hg , to know the sign of f ' (2 h) for small h.
. g = 12 b2 01
. g 12 b2 01
.g
f b2 01
3

= 12 (1.9)3 12 ( 1.9)2
= 12 (6.859) 12 (3.61)

b g

2 x ,

(i)

f x = 2 x , x 2 , 2

(ii)

bg
b g
f b x g = 2 x , x b 3 , 2g b2 , 3g
bg

bg

e4 x j

x 2 , x 3 , 3

3f x =

f 1 = 36 12 24 1 = 36 24 = 12 > 0

4 x 2 , x 3 , 3

4 x2

bg

f x =

and

f has no extremum value at x = 0.


Again, at x = 1,

bg
f b xg =

defined by f x = 4 x 2 , x 3 , 3

test, it is seen that f x at x = 0, i.e.,


02

f 2 h > 0 f has a local maxima at the

4 x2

4 x2

(iii)

b g

2 x , x 2 4 0 ,

i.e.,

x2 4 , x 2

bg

f x = 2 x , for x 2 but x 2 and


x 3 , 3

bg

f x = 2 x , x 2 , 2 , i.e., x < 2
Also,

bg

f x =

4 x2
4 x2

x2 4 , x 2

b g

2 x , x 2 4 0 ,

i.e.,

= 2x, for | x | > 2, i.e., x < 2 and x > 2 but x 3 , 3

bg

g b g

f x = 2 x , for x 3 , 2 2 , 3

bg

Now, f x = 0

924

How to Learn Calculus of One Variable

4 x2

e4 x j
2

b g

x=0
Hence, the possible points where the extreme
values of the given function f defined in the closed
interval [3, 3] may occur, are

bg

(i) x = 0 where f x = 0

bg

(ii) x = 2 and x = 2 where f x do not exist.


(iii) x = 3, the left end point x = 3, the right end point
of the given closed interval [3, 3].
1. Local extrema at critical points:
At x = 0, using the rule of first derivative test,

g b

b g

f has a local maximum at x = 3


local maximum value of the function f (x) at
x = 3 is f (3) = | 4 (3)2 | | 4 9 | = | 5 | = 5
(ii) At the right end point x = 3

b g b g

b g b g
b
g
f b x g changes sign from +ve to ve in N (0).
f 0 h = 2 0 h = 2h > 0 3 h > 0

x = 0 is a point of local maximum.


local maximum value of the function f (x) at x =
0, is f (0) = | 4 02 |= | 4 | = 4
At x = 2, using the rule of the first derivative test,

b g
f b 2 + hg = 2 b2 + hg = 4 + 2h > 0
f b x g changes sign from ve to +ve in N (2).

f 2 h = 2 2 h = 4 + 2h < 0

f has a local minimum at x = 2


Local minimum value of the function f (x) at
x = 2 is f (2) = | 4 x2 | = | 4 4 | = | 0 | = 0
At x = 2, using the rule of first derivative test,
f 2 h = 2 2 h = 4 + 2h < 0

g b g b g
f b 2 + hg = 2 b 2 + hg = 4 2h > 0 for h > 0
f b x g changes sign from ve to +ve in N (2).

f has a local maximum at x = 3


Local maximum value of the function f (x) at
x = 3 is f (3) = | 4 32 | = | 4 9 | = | 5 | = 5
3. Find the local maximum and local minimum values
of the function f defined by f (x) = sin 2x x,

x .
2
2

bg
f b x g = 4 sin 2 x
Now, f b x g = 0

2. Local extreme at the end points of the closed


interval:
(i) At the left end point x = 3.

x
2
2

f x = 2 cos 2 x 1

cos2 x =
2x =

1
2

or =
3
3

or since x
6
6
2
2
Hence, the possible points where the extreme
values of the given function f defined in the closed
x=

LM
N

interval

OP
Q


,
may occur, are
2 2

bg

(i) x =

and x =
where f x = 0
6
6

(ii) x =

= the left end point and x = = the


2
2

f has a local minimum at x = 2


Local minimum value of the function f (x) at
x = 2 is f (2) = | 4 (2)2 | = | 0 | = 0

b g

f 3 h = 2 3 h = 6 2h > 0 h > 0

Solution: f (x) = sin 2x x,

f 0 + h = 2 0 + h = 2h > 0

f 3 + h = 2 3 + h = 6 + h < 0 h > 0

2 x = 0 , x 2 , x 3 , 3

LM
N

OP
Q


right end point of the given closed interval 2 , 2 .

Maxima and Minima

1. Local extreme at critical points x =

x=

and
6

:
6

, using the rule of second derivative


6
test, it is seen that

FG IJ = 4sin FG IJ = 4
H 6K
H 3K

3
=2 3<0
2

and the local


6

maximum value of the function f (x) at x =


is
6

f has a local maximum at x =

FG IJ = sin FG IJ = =
H 6K H 3K 6

Also, at x =

2 6

and the
2

local maximum value of the function f (x) at x =


2

(ii) At the right end point x =

:
2

FG hIJ = 2 cos FG 2 FG hIJ IJ 1


H2 K
H H 2 KK
= 2 cos b 2hg 1
f

f has a local minimum at x =

3>0

and the local


6

minimum value of the function f (x) at x =


is
6

f has a local minimum at x =

FG IJ = sin FG IJ =
H 6K H 3K 6

2 6

2. Local extrema at the end points of the closed


interval:
(i) At the left end point x =

IJ
K

FG IJ = sin bg FG IJ = 0 + = .
H 2K
H 2K 2 2

is f

2
Local minimum value of the function f (x) at

FG IJ = 4sin FG IJ = 4sin FG IJ = 2
H 6K
H 3K H 3K

FG
H

= ve < 0 for small h > 0

= 2 cos 2h 1 < 0

= 2 cos 2h 1

f has a local maximum at x =

At x =

925

FG FG
HH

IJ IJ 1
KK

+ h = 2 cos 2 + h
2
2

b
g
= 2 cos c b 2h gh 1
= 2 cos b 2hg 1
= 2 cos + 2h 1

x=

FG IJ
H K

bg

= sin =
is f
2
2
2
2

4. Find the local maxima and minima of the function f

bg

defined f x = 4 sin x + cos 2 x , x 0 , 2 . Also


find the absolute maximum and minimum values in

0 , 2 .

bg

Solution: f x = 4 sin x + cos 2 x , x 0 , 2

bg
f b x g = 4 (sin x + cos 2x)
f b xg = 0
4 cos x 2 sin 2 x = 0
4 cos x b1 sin x g = 0
f x = 4 cos x 2 sin 2 x

cos x = 0 or sin x = 1 but x 0 , 2


x=

3
or x =
2
2

926

How to Learn Calculus of One Variable

Hence, the possible points where the local extreme


values of the given function f defined in the closed

bg

3
and x =
where f x = 0
2
2
2. x = 0 = the left end point and x = 2 = the right
end point of the given closed interval 0 , 2

1. Local extrema at critical point x = and x =


:
2
2

At x = , using the rule of second derivative


2
test, it is seen that
f

FG IJ = 4 b1 1g = 0 which does not given


H 2K

any inference about extremum.

bg b
g
and f b x g = 4 bsin x + 4 cos 2 x g
F I F
I
f G J = 4 G cos 2 sin J = b0 2 0g = 0
H 2K H 2
K
F I F
I
and f G J = 4 G sin + 4 cos J = 4 (1 4)
H 2K H 2
K
f x = 4 cos x 2 sin 2 x

= 12 < 0

f has a local maximum at x =

and the local


2

maximum value of the function f (x) at x =

FG IJ = 4 sin FG IJ + cos = 4 1 1 = 3
H 2K
H 2K
3
Also, at x =
:
2
f

FG IJ
H K

FG 3 IJ = 4 FG sin FG 3 IJ + cos 3IJ


H2K H H2K
K

= 4 (1 1) = 8 > 0

f has a local minimum at x =

3
2

is
2

FG IJ
H K

3
3
3
= 4 sin
+ cos 3 = 4 (1) +
is f
2
2
2
(1) = 5
x=

interval 0 , 2 may occur, are


1. x =

Local minimum value of the function f (x) at

2. Local extrema at the end points of the closed


interval:
(i) At the left end point x = 0:

b g

b g

b g

f 0 + h = 4 cos 0 + h 2 sin 2 0 + h

= 4 cos h 2 sin 2h = 4 cos h (1 sin h)


= + ve, for small h > 0.

f 0 + h > 0 f has a local minimum at the


left end point x = 0 and the local minimum value of the
function f (x) at x = 0 is f (0) = 4 sin 0 + cos 0 = 4 0 + 1
=1
(ii) At the right end point x = 2 :

f 2 h = 4 cos 2 h 2 sin 2 2 h

= 4 cos h 2 sin 4 2h = 4 cos h + 2 sin 2h =


+ve
f 2 h > 0 f has a local maximum at
the right end point x = 2 and the local maximum
value of the function f (x) at x = 2 is
f 2 = 4 sin 2 + cos 4 = 4 0 + 1 = 1

Hence x =
and x = 2 are points of local
2
3
are the points of local
maxima and x = 0 and x =
2
minima

b g

3f

FG IJ = 4 sin FG IJ + cos FG 2 IJ = 4 1+ cos


H 2K H 2K H 2K

=41=3

FG 3 IJ = 4 sin FG 3 IJ + cos 3 = 4 b1g + b1g = 5


H2K
H2K
f b 0g = 4 sin 0 + cos 0 = 4 0 + 1 = 1
f b 2g = 4 sin 2 + cos 4 = 4 0 + 1 = 1
f

Absolute maximum value of the function


f (x) = 3 and absolute minimum value of the function
f (x) = 5.

Maxima and Minima

The absolute extreme values (or simply the absolute


extrema) of a continuous function in a closed interval:
There is a method to find out:
The absolute extreme values (or simply the absolute
extrema), i.e.,
the maximum and / minimum values (value) (or simply
the maximum and / minimum) of a continuous function
f in a given closed interval [a, b].
The method is to:
1. To find out the values of the given function defined
by y = f (x) at all critical points and the end points of
the given closed interval [a, b].
2. To take out:
(i) The greatest of the numbers of the set {values of
the given function at all critical points and end points
of the given closed interval} which is the required
absolute maximum value, the maximum value or simply
the maximum of the given continuous function in a
given closed interval.
That is, the absolute maximum value, the maximum
value or simply the maximum of a given continuous
function in a given closed interval [a, b] = the greatest
of the numbers of the set {f (a), f (b), f (c1), f (c2), f
(c3)}, where c1, c2 and c3 etc. are all critical points and
a and b are the end points of the closed interval where
the given function y = f (x) is defined.
(ii) the least of the numbers of the set {values of the
given function at all critical points and end points of
the given closed interval} which is the required
absolute minimum value, the minimum value or simply
the minimum of the given continuous function in a
given closed interval.
That is, the absolute minimum value, the minimum
value or simply the minimum of a given continuous
function in a given closed interval [a, b] = the least of
the numbers of the set {f (a), f (b), f (c1), f (c2), f (c3)},
where c1, c2 and c3 etc are all the critical points and a
and b are the end points of the closed interval [a, b]
where the given function y = f (x) is defined.
Notes: 1. One should note that there is only one
absolute maximum and / minimum value (value) of a
function in a closed interval. This is why the words
the maximum and / minimum (values) are used always
in singular to signify the absolute extrema of a
continuous function y = f (x) in closed interval [a, b]

927

whereas the words the maxima and minima are used


in plural always to signify the local extrema of a
continuous function f in a closed interval [a, b] since
there are several local maximums and / minimums of a
continuous function in an open or in a closed interval.
2. The absolute extrema of a function is always
determined only in a given closed interval whereas
local extrema of a function are determined in both
open and closed interval.
3. If a function is continuous in an open interval (a,
b) or in closed interval [a, b] and it has only one
extreme point in (a, b), then it is a point of absolute
maxima or a point of absolute minima accordingly it is
a point of local maxima or a point of local minima.
4. One should note the difference between the
absolute extrema and the local extrema of a function
which is presented in the following way:
If x = c is a point of an interval D such that f (c)
f x (or f (c) f (x) for all x in the interval D, then
f (x) is said to have an absolute maximum (absolute
minimum) value f (c) in the interval D at x = c.
That is, a value of the function f at point x = c in the
domain of its definition D represented by f (c) is an
absolute maximum (or an absolute) minimum) value
of the function f f (c) is the greatest (or the least)
value of the function in its domain of definition D if
the values of the function f at x = c and other values
of the independent variable x which belong to its
domain of definition D are considered.
If x = c is a point of an interval D such that f (c) > f
(x) (or f (c) < f (x)) is true only for x in some deleted neighbourhood of the point x = c, where > 0 (i.e. for
all x such that 0 < | x c | < ), then f (x) is said to have
a local maximum (or local minimum) value f (c) at x = c.
That is, a value of the function f at a point x = c in
the domain of its definition D represented by f (c) is a
local maximum (or local minimum) value of the function
f f (c) is the greatest (or the least) value in the deleted neighbourhood of the point x = c if the values
of the function f at x = c and other values of the
independent variable x which belong to the -deleted
neighbourhood of the point x = c are considered.
In short, an absolute extrema of a function at a
point of its domain of definition is the greatest or the
least value of the function in its domain of definition

bg

928

How to Learn Calculus of One Variable

while a local extrema of a function is the greatest or


the least value of the function in a deleted
neighbourhood of a point in its domain of definition.
On points of absolute extrema
1. The point of absolute maximum: A point x = c in
the domain of the function y = f (x) at which the value
of the function f is the largest value of the function is
called the point of absolute maximum of the function
y = f (x).
2. The point of absolute minimum: A point x = c in
the domain of the function y = f (x) at which the value
of the function f is the smallest value of the function
is called the point of absolute minimum of the function
y = f (x).
3. The point of attainment of absolute extrema is either
(a) a point where f ' (x) = 0,
(b) a point at one end of the closed interval,
or (c) a point where y = f (x) is not differentiable. This
situation may occur also, but this will be more rare in
common practice.

bg b g
x+1
,0 x 2
(iii) f b x g =
x +1
Solution: (i) f b x g = b x + 1g , 0 x 8
2
2
f b x g = b x + 1g =
, x 1
3
3 b x + 1g
f b xg 0 , x 0 , 8
1

(ii) f x = x x 1 + 1 3 , 0 x 1

2
3

13

1
3

Hence, this is why it is required to find out the


values of the given function only at the end points
namely 0 and 8 appearing in the given closed interval
[0, 8].

b g b g = b1g = 1
f b8g = b8 + 1g = b 9g = e3 j = b 3g
= 3b 3g = 3 3
Therefore, max. f b x g = 3 3
f 0 = 0+1

2
3

2
3

2
3

2
3

1
3

(Problems on algebraic functions)

(ii): f x = x x 1 + 1

bg

6) (x 2)

bg

f x = 0 3 (x 6) (x 2) = 0 x = 2 and
x=6
Now, f (1) = (1)3 12 (1)2 + 36 (1) + 17 = 1 12 + 36
+ 17 = 42
f (2) = (2)3 12 (2)2 + 36 (2) + 17 = 8 48 + 72 + 17
= 49
f (6) = (6)3 12 (6)2 + 36 (6) + 17 = 216 432 + 216 +
17 = 17
f (10) = (10)3 12 (10)2 + 36 (10) + 17 = 1000 1200
+ 360 + 17 = 177
max. f (x) = 177
min. f (x) = 17
2. Determine the maximum and / minimum values
(value) of each of the following functions in stated
domains.
2
(i) f x = x + 1 3 , 0 x 8

bg b

4
3

Min. f (x) = 1

f x = 3x2 24x + 36 = 3 (x2 8x + 12) = 3 (x

2
3

Examples worked out:


1. Find the maximum and / minimum of the function
f (x) = x3 12x2 + 36x + 17 in [1, 10].
Solution: f (x) = x3 12x2 + 36x + 17

bg b g
1
f b x g = x b x 1g + 1
3
b2 x 1g
3 x b x 1g + 1
f b xg = 0

1
3

23

b g

d
x x 1 +1
dx

2
3

(2x 1) = 0
1
x=
2
Now, it is required to find out the values of the
1
given function at the points x = 0, x = and x = 1.
2
f 0 =1

bg
F 1I F 3I
fG J =G J
H 2K H 4K

1
3

, f (1) = 1

Hence, max. f (x) = 1

F 3I
min. f b x g = G J
H 4K

1
3

Maxima and Minima

bg

x+1

(iii) f x =

bg

f x =0

,0 x 2

x +1
2

bg b g e
FH
IK
x b x + 1g
+1

cos x sin x = 0

1
1
x + 1 1 x +1 x2 + 1 2 2 x
2
f x =
2
2
x +1
2

bg
x

3
2

bg

bg

bg

2
2

bg

bg

bg

Solution: f x = 5 sin x + cos 2 x

=0

.
2 = 1414

(Problems on trigonometric functions)


1. Examine the maximum and minimum of the function
f (x) = sin x + cos x, 0 x .

bg b
f bxg = 0

f x = 5 cos x 2 sin 2 x

cos 2 sin 2x = 0
cos x 4 sin x cos x = 0
cos x (1 4 sin x) = 0
cos x = 0 or sin x =

x=

FG IJ
HK

1
4

FG IJ
H K

1 1
or x = sin 1
;
<
2
4 0 < sin
4 2

bg b
g
= 5 b0 + 1g = 5
1
F 1I
and cos 2x = 1 2
x = sin G J sin x =
H 4K
4

Now, f 0 = 5 sin 0 + cos 0

sin2 x

Solution: f (x) = sin x + cos x

bg

2. Find the maximum and minimum of the function

f x = 5 sin x + cos 2 x , 0 x .
4

3 5 3 2.236 6.708
=
=
=
= 1.34
5
5
5
5

f x = cos x sin x

min. f (x) = 1

Therefor, max. f (x) =


min. f (x) = 1

Hence, max. f x = 2

(1 x) = 0
x=1
Now, it is required to find out the values of the
function at x = 0, x = 1 and x = 2
f 0 = 1, f 1 =

FG IJ = cos FG IJ + sin FG IJ
H 4K H 4K H 4K

f = cos + sin = 1 + 0 = 1

3
2

f 2 =

f
=

3
2

bg

4
Now, f (0) = cos 0 + sin 0 = 1 + 0 =1
x=

x2 + 1

tan x = 1

x2 + 1

e x + 1j x bx + 1g
=
e x + 1j
b1 xg
=
e x + 1j
b1 xg
f b xg = 0
e x + 1j

929

=1 2

FG 1 IJ
H 4K

=1

1 7
=
8 8

930

How to Learn Calculus of One Variable

FG 5 IJ = 2 cos FG 5 IJ cos FG 10 IJ
H 6K
H 3K H 3 K
1 F 1I
1 3
= 2 G J = 1 + =
2 H 2K
2 2
f bg = 2 cos b2g cos b4 g

FG 1 IJ = 5 FG 1 + 7 IJ = 9 5 = 9 5
H 4K H 4 8K 8
8
F I F F I F II
f G J = 5 G sin G J + cos G J J
H 2 K H H 2 K H 2 KK
= 5 b1 1g = 5 + 0 = 0

f sin 1

and

bg

9 5
Hence, max . f x =
8
min. f (x) = 0

= 2 (1) 1 = 2 1 = 1

max. f (x) =

3
2

min. f (x) = 3

3. Determine the maximum and minimum values of


the function in the stated domain. f (x) = 2 cos 2x
cos 4x in 0 x .

Problems on mod functions and / other functions


whose derivatives have points of discontinuity as
critical points:

Solution: f (x) = 2 cos 2x cos 4x

1. Find the maximum and / minimum values (value) of


the function f (x) = 2 + | x 1 | in [3, 2].

bg
f bxg = 0

f x = 4 sin 2x + 4 sin 4x = 4 (sin 4x sin 2x)

4 (sin 4x sin 2x) = 0


4 (2 sin 2x cos 2x sin 2x) = 0
4 sin 2x (2 cos 2x 1) = 0
sin 2x = 0 or 2 cos 2x 1 = 0
Now sin 2x = 0 and 0 x
2 x = 0 , , 2 x = 0 ,
and

bg

Solution: f x = 2 + x 1 , x 3 , 2

,
2

1
5
2 cos 2 x 1 = 0 cos 2 x = 2 x = ,
2
3 3

5
x= ,
as 0 x
6 6
Now, f (0) = 2 cos 0 cos 0 = 2 1 1 = 2 1 = 1

FG IJ = 2 cos FG IJ cos FG 2 IJ
H 6K
H 3K H 3 K
F 1I F 1I 1 3
= 2G J G J = 1 + =
H 2K H 2K 2 2
F I
f G J = 2 cos cos 2
H 2K
f

= 2 (1) 1 = 2 1 = 3

b g bxx 11g , x 1
Also f b x g is not differentiable at x = 1

f x =

x = 1 is a critical point

bg

Now, f x = 0 , x 1
i.e.,

x 1

bx 1g = 0 , x 1 ; has no solution

Hence, f (x) has only one critical point namely 1.


Now f (1) = 2 + | 1 1 | = 2 + | 0 | = 2
f (3) = 2 + | 3 1 | = 2 + | 4 | = 2 + 4 = 6
f (2) = 2 + | 2 1 | = 2 + | 1 | = 2 + 1 = 3
Therefore, max. f (x) = 6
min. f (x) = 2
2. Find the absolute maximum and / minimum values
of the function f defined by f (x) = 3 + | x + 1 | in [2, 3].

bg

Solution: f x = 3 + x + 1 , 2 x 3

bg

f x =0+
=

x +1

x +1

bx + 1g , x 1

b x + 1g , x 1

Maxima and Minima

bg

Now, f x = 0 , x 1 ; i.e., x 1 and

x +1

bx + 1g = 0 has no solution.

Hence, the given function has only one critical


point x = 1 where the derivative of the function does
not exist.
Now, f (1) = 3 + | 1 + 1 | = 3 + 0 = 3
f (2) = 3 + | 2 + 1 | = 3 + | 1 | = 3 + 1 = 4
f (3) = 3 + | 4 | = 3 + 4 = 7
Therefore, max. f (x) = 7
min. f (x) = 3
3. Determine the maximum and minimum values
(value) if any, for the function f defined by f (x) = 1
4

x 5 , x 1 , 1 .
4

Solution: f (x) = 1 x 5 , x 1 , 1

bg

4 15
4
x = 1 ,x 0
5
5x 5
Also f (x) is not differentiable at x = 0
f x =

bg

Now, f x = 0 , i.e.,

4 15
x = 0 has no solution.
5
Hence, the given function has only one critical
point namely 0,

bg
f b 1g = 1 b 1g = 1 1 = 0
f b1g = 1 b1g = 1 1 = 0

And f 0 = 1 0 = 1
4
5

4
5

Therefore, max. f (x) = 1


min. f (x) = 0
The greatest and the least values of a continuous
function in a closed interval (recapitulation)
The rule to find out the greast and / the least values
(value) of a given continuous functions in a given
closed interval [a, b] is to use the following facts:
1. The greatest value of a given continuous functions
f in a given closed interval [a, b] = the greatest of the
numbers of the set {f (a), f (b), f (c1), f (c2), f (c3)} where

931

c1, c2 and c3 etc are critical points and a and b are end
points of the given closed interval [a, b] where the
given function y = f (x) is defined.
2. The least value of a given continuous function f in
a given closed interval [a, b] = the least of the numbers
of the set {f (a), f (b), f (c1), f (c2), f (c3)} where c1, c2
and c3 etc are critical points and a and b are end
points of the given closed interval [a, b] where the
given function y = f (x) is defined.
Notes: 1. A maximum means not necessarily the
greatest value whereas one should note that the
maximum means necessarily the greatest value.
2. A minimum means not necessarily the least
value whereas one should note that the minimum
means necessarily the least.
3. A continuous function f (x) has a single extremum
in its domain the maximum (the minimum) is also
the greates (the least) value of the continuous
function f in its domain.
4. The largest and the smalles values (value) are
also in use instead of the terms the greatest and the
least values (value)
5. The greatest value of a function f on (in or over)

bg

max. f x
and
x a , b
the least value of a function f on (in or ove) the interval
the interval [a, b] is designated as:

bg

min. f x
x a , b
6. (i) Maximum and greatest: A continuous
functions y = f (x) on a closed interval [a, b] has a
greatest value at an interior point or at an end point of
its domain which is the given closed interval. Further
the greatest value of a continuous function f in a
closed interval [a, b] is unique.
On the other hand, a continuous function y = f (x)
in a closed interval [a, b] may have local maxima (i) at
an interior point or more interior points (ii) at end
points of the closed interval but not at the end points
of the open interval used as the domain of the
continuous function. Moreover, a local maxima of a
continuous function in an open or in a closed interval
may not be unique, i.e., a continuous function may
have more than one local maxima in an open in a closed
interval.
[a, b] is designated as:

932

How to Learn Calculus of One Variable

(ii) Minimum and least: A continuous function y =


f (x) in a closed interval [a, b] has a least value at an
interior point or at end point of the domain which is
the given closed interval. Further the least value of
continuous function f in a closed interval [a, b] is
unique. On the other hand, a continuous function y =
f (x) in a closed interval [a, b] has a local minima (i) at
an inter point or more interior points as well as (ii) at
end points of the closed interval but not at the end
points of the open interval used as the domain of the
continuous function. Moreover, a local minima of a
continuous function in an open or in a closed interval
may not be unique, i.e., a continuous function ,may
have more than one local minima in an open or in a
closed interval.
Lastly, one must note that the greatest value of a
function in a closed interval, when it occurs at an
interior point is also a local maximum and similarly,
the least value when it occurs at an interior point is
also a minimum where as the converse may not always
be true.
7. If a continuous function y = f (x) has single
extremum in its domain of definition, then if it is a
maximum (minimum), then it is also the greatest (the
least) value of the function y = f (x).
8. If a function is defined and continuous in some
interval, and if the interval is not a closed one, then it
can have neither the greatest nor the least value.

max. f (x) = 357


x 0, 3
min. f (x) = 10

x 0, 3
2. Find the greatest and the least values (value) of

bg

the function f x =

bg

Solution: f x =

Solution: f (x) = 12x5 45x4 + 40x3 + 6 x 0 , 3

bg
f bxg = 0

f x = 60x 4 180 x 3 + 120x 2

60x4 (x 1) (x 2) = 0
x = 0, 1, 2 which belong to [0, 3]
x = 0, 1, 2 are all the critical points
Now,
f (0) = 6
f (3) = 357
f (1) = 13
f (2) = 10

x 2
+ , x 1, 6
8 x

b g 81 x2
f b xg = 0
f x =

x 2 16
8x 2

x 2 16
=0
8x 2
x2 16 = 0

x = 4 but only x = 4 1 , 6
x = 4 is the only critical point

bg

bg

bg

1
1
Now, f 4 = 1 , f 1 = 2 , f 6 = 1
12
8

b g bg

max. f x = f 1 = 2

Examples worked out: (some more)


1. Find the greatest and the least values (value) of
the function f (x) = 12x5 45x4 + 40x3 + 6 on the
interval [0, 3].

x 2
+ on the interval [1, 6].
8 x

1
8

x 1, 6
min. f (x) = f (4) = 1

x 1, 6
3. Find the greatest and least values (value) of the
function f (x) = x3 3x2 + 6x 2 in the interval [1, 1].
Solution: f (x) = x3 3x2 + 6x 2, x 1 , 1

bg
f b xg = 0

f x = 3x 2 6 x + 6

3x 2 6 x + 6 = 0
x=

b g

36 4 3 6
26

Maxima and Minima

36 72
12

36
12

which

are

imaginary
the given function f (x) = x3 3x2 + 6x 2, has
no critical point.
it is required to find out only the values of the
function f (x) = x3 3x2 + 6x 2, x 1 , 1 only at
the end points namely x = 1 and x = 1 of the closed
interval [1, 1].

bg bg
f b 1g = b1g

bg
3 b1g + 6 b 1g 2 = 12

bg

min. f (x) = 12

3
2

3
1 x 2
2

e1 x j

1
2

x 2x

2 3

bg

e1 x j
2

3
2

=0 x =0

bg

Again, f 0 =

x 1 , 1

b g
b g

3
1 0
2
3
1 0 2

11 0 2

1
2

10
=1> 0
1
The given function f has a minimum value at
x = 0.
Further, the function f is defined in an open interval
(1, 1), this is why there is no need to look at the end
points namely 1 and 1.
Hence, max. f (x) = 1 since f (0) = 1
=

x 1 , 1
4. Find the greatest and the least value of the function
on the curve f (x) = 4x x2, x R .
Solution: f (x) = 4x x2

bg
f b xg = 0 4 2 x = 0 x = 2
Also, f b x g = 2
f b2g = 2 < 0
f x = 4 2x

As f (x) has only one extremum (maximum) at x = 2


max. f (x) = greatest value of f (x) (at x = 2) = 8
4=4
N.B.: One should note that the given function f (x) =
4 x2 has not been defined only in a closed interval,
i.e. the domain of the given function is not closed
interval.
Example:
Question: Find the maximum and minimum of
1
f x =
, 1 < x < 1.
1 x 2

bg

bg

Hence, max. f (x) = 2

3
2

Now, f x = 0

Solution: f x =

e1 x j
1 e1 x j
=
2

bg

f 1 1 3 1 + 6 1 2 = 2
3

f x =

f x

933

1
1 x

, 1 < x < 1

On the method to find the range of y = f (x) in [a, b]


Whenever the range of a given continuous function
y = f (x) defined in a closed interval [a, b] is required to
be found out, one must find its absolute maxima and
absolute minima in the closed interval [a, b].
Remarks: 1. When the domain of a function y = f (x)
is not given, its domain must be found out before
finding its range to examine whether its domain is a
closed interval or not.
2. The domain of composite functions defined by y =
g (f (x)) is the domain of the inner function namely f
provided that the range of the inner function f is a
subset of the domain of the outer function namely g.
e.g.:

y = tan

F
b g GG
H

2
x2 D y = D
9

2
x2
9

I
JJ
K

LM OP is b , g where LM0 , OP = range


N 3Q
N 3Q

because 0 ,

934

of

How to Learn Calculus of One Variable

2
x 2 and , = domain of tan x.
9

Examples worked out:


1. Find the range

y = 1+ x +

of

the

function

2 x .

Solution: Let y = y1 + y2 where

2
2
x2 0
x2 0
16
16
x2 =

y1 = 1 + x and y2 = 2 x

b g

1 + x 0 x 1 D y1 = 1 ,
Also,

2 x is defined for

b gb

2 x 0 x 2 x 2 D y2 = , 2

bg

b g

0 x
16
4
4

b g LMN

D y =

Now, 1 + x is defined for

b g

Hence, D y = D y1 D y 2
= [1, 2]


,
4 4

LM
N

bg

2 x

2
x2
16
2

Again to get the range, it is required to be found


out the absolute extrema of the function

y = 1+ x +

bg

f x =

1
2

2 x in [1, 2].

LM
MN

1
1+ x

2 x = 1+ x x =

OP
2 x PQ
1

bg

2
x2
16

2
x2
16

2
x2
16

and f x = 0

bg

1
when f x = 0
2

FG 1 IJ = 3 + 3 = 2
H 2K 2 2
f b 1g = f b2g = 3
Therefore, R b y g = 3 , 6
Lastly, f

3x cos

3
=
2

2. Find the range of the function y = 3 sin

2
x2 .
16

3x cos

2
x2 = 0
16

F
I
GG 16 x > 0 and < 2 JJ
H
K
F I 3
lastly, f b0g = 3 sin G J =
H 4K 2
F I
f G J = 3 sin 0 = 0
H 4K
x=0 3

OP
Q


,
.
4 4

2
x2
16

f x = 3 cos

2
x 2 , it is
16

required to find out its absolute extrema in

bg

OP
Q

Now to get the range of y = 3 sin

3 f x = 3 sin

2 1

2
x 2 is defined for
16

Solution: y = 3 sin

Maxima and Minima

FG IJ = 3sin 0 = 0
H 4K
L 3 OP .
Therefore, R b y g = M0 ,
N 2Q

935

(c, f (c))

Concavity, Convexity and Inflection


Points of a Curve
Before the definition of each term namely concavity,
convexity and inflection points of a curve y = f (x)
defined on its domain, one should know the following
facts.
1. If there are two points P1 and P2 such that P1 and
P2 have the same abscissa but the ordinate of P1 is
larger (smaller) than the ordinate of P2, then it is said
that P1 lies (is situated or simply is) above (below) P2.
2. It is said that the curve y = f (x) lies above (below)
the curve y = g (x) in the interval (a, b) if for every
point in this interval the point on the first curve lies
above (below) its corresponding point on the second
curve, i.e. if
f (x) > g (x) [or f (x) < g (x)]
On concavity of a curve: It is defined in various ways:
Definition (i): (In terms of functional values): A curve
y = f (x) is said to be concave upwards over the interval
(a, b) if at every point on this interval (a, b), the curve
y = f (x) lies above the tangent to the curve at that
point, i.e.,
(In terms of first derivative): A curve y = f (x) is said to

x=a x=b x=c

On convexity of the curve: It is defined in various


ways:
Definition (i): (In terms of functional values): A curve
y = f (x) is said to be convex upwards over the interval
(a, b), if at every point of this interval (a, b), the curve
y = f (x) if below the tangent to the curve at that point,
i.e.,
(In terms of first derivative): The curve y = f (x) is said
to be convex upwards at the point (c, f (c)), if f c
exists and there is an open interval (a, b) containing
c, such that for all x c , in (a, b), the point (x, f (x))
of the curve y = f (x) is below the tangent to the curve
at (c, f (c)).
Definition (ii): (In terms of second derivative): If f "
(x) < 0 on an interval (a, b), then the curve y = f (x) is
convex upwards on the interval (a, b), i.e., the curve
y = f (x) is situated below any of its tangent lines
drawn at any point of the interval (a, b).

bg

bg

be concave upwards at the point (c, f (c), if f c

exists, and there is an open interval (a, b) containing


c such that for all x c , in (a, b), the point (x, f (x) of
the curve y = f (x) is above the tangent to the curve at
(c, f (c)).
Definition (ii): (In terms of second derivative): If

bg

P
A

B
(c, f (c))

x=a x=b x=c

f x > 0 , then the curve y = f (x) is concave


upwards on the interval (a, b), i.e., the curve y = f (x) is
situated above any of its tangent lines drawn at any
point of this interval (a, b).

Definition (i): Points of inflection: (In terms of


concavity and convexity): A point P (c, f (c)) where
the curve y = f (x) has a tangent line and the curve
y = f (x) changes from being concave to convex or
vice versa as a point moving along the curve passes
through it, is called an inflection point on (or, of) the
curve.

936

How to Learn Calculus of One Variable

That is, a point P (c, f (c)) on the curve y = f (x) is a


point of inflection on one side of P, the curve lies
below the tangent at P and on the other side of P, the
curve lies above the tangent at P, i.e. a point where
the curve crosses the tangent is a point of inflection
of the curve, i.e. a point in whose neigbourhood, the
graph of the function y = f (x) geometrically passes
from one side of the tangent line to the other and
bends or twists over it making a shape of English
alphabet S.
y

bg

bg

bg

(c ) )

Tangent

bg

bg

Tangent

(c, f

2. The curve y = sin x, y = cos x and y = tan x have


inflection points (or, flex points or points of inflection)
where these curves cut the x-axis.
Definition (ii): (In terms of extrema): A point c where
f ' (c) = 0 and has neither a maximum nor a minimum is
called an inflection point on the curve y = f (x).
Definition (iv): (In terms of first derivative): If for
any value of the independent variable namely x = c,
f c = 0 and f x does not change sign on (to,
or at) the left side of (i.e. just before) the point x = c or
on the right side of (i.e. just after) the point x = c, i.e.
f x does not change sign for the values of the
independent variable x lying in a neighbourhood of
the point x = c, i.e. f x does not change sign at x =
c, i.e. f x does not change sign while passing
through the point x = c, then the function y = f (x) is
said to have an inflection point (c, f (c)) at the abscissa
x = c or it is said that (c, f (c)) is an inflection point on
the curve (the graph of the function) y = f (x).
This definition can be put in a tabular form as below:

P
(c, f (c ))

On the left
of the point
x=c

At the
point
x=c

On the right
of the point
x=c

Nature of the
critical point
x=c

f c = +ve

f c

f c = +ve

There is a point

bg
f bcg = ve

bg
f bcg

bg
f bcg = ve

of inflection
namely (c, f (c)).

Illustrations:
1. A simple illustration of the curve with a point of
inflection is a road with an S bend (a shape of English
alphabet S) whose midpoint may be supposed to be
a point of inflection of the curve where there is a twist
or bending of the road.
y
(c, f (c )) = an inflection point

y=

bg

bg

f (c )

x=c

Definition (iv): (In terms of slope of a function at a


point): The points where the derivative f of a
function f) is (or, has) the maximum or the minimum
are called inflexion points (or points of inflection, or
simply flex points).
Definition (v): (In terms of second derivative): If
f c = 0 or does not exist but f c does exists
and f x changes sign while passing through the
point x = c, i.e. f x changes sign at x = c, i.e.,
f x changes sign for the values of the
independent variable x which belong to the
neighbourhood of the point x = c, i.e., f x has
different signs for the values of the independent
variable x which are little (just or slightly) less and
little greater than the value of the independent

bg

bg

bg

bg

Maxima and Minima

variable x = c, then the point (c, f (c)) is the point of


inflection (or, the inflection point or simply the flex
point) of the curve y = f (x) or it is common to say that
the curve y = f (x) has an inflection point (c, f (c)) at the
abscissa x = c on (or, of) the curve (the graph of the
function) y = f (x).
y
f c = ve
Point of inflexion

f c = 0

f c = + ve

Point of inflexion

f c = 0
f c =+ ve

x=c

Notes: (i) Inflection points exist at points belonging


to the domain of the function y = f (x) where either the
second derivative f x is zero or f x is
undefined (i.e., f x does not exist).
(ii) If f x is continuous, an inflection point of
the curve y = f (x) exists between every pair of
consecutive maxima and minima of the function y = f
(x) as in the graphs of the function y = sin x, y = cos x
and y = tan x.
(iii) The general condition for a flex point:
If x = c is a critical point such that i.e., f c exists
and f c = 0, x = c belonging to the domain of the
function and supposing that n > 2 is the smallest
f c =
positive
integer
such
that
n 1
n
n
f c = ... = f b g c = 0 and f c 0 , f c
being continuous at x = c, then the curve y = f (x) has
an inflection point namely (c, f (c)) at x = c n is
odd.

bg

bg

bg

bg

bg

bg

bg

bcg 0 , i.e. , f bcg is


bcg > 0 or f bcg < 0 .

One should note that f


non zero f

bg
bg

bg

bg

bg

bg

bg

To find the point of inflection of the given function


y = f (x), one should:
1. Find the second derivative f x .
2. Set f x = 0 and find its real roots.
3. Find also those values of x (if any) for which f " (x)
is undefined but f ' (x) exist.
4. Test f x for values in the neighbourhood of
the real roots f x = 0 and those values of x where
f x is undefined.
i.e., to find f c h if x = c is a root of

bg

bg

f c = ve

Question: What is the criterion for a flex point?


Answer: A point P is a flex point of the curve y = f (x)
The curve y = f (x) has a point P at which f x
= 0 or does not exist and f x is positive on one
side and f x is negative on the other side of the
point P.
To remember: A definition is always a criterion for a
mathematical quantity or entity. This is why a term to
be used in mathematics is firstly defined. Further,
whenever one is required to show whether a quantity
is a mathematical quantity or entity considered or
not, one must go by its definition.
How to find the inflection points of a given function
y = f (x):

x=c

937

bg

bg

bg

bg

bg

f x = 0 or x = c is the value of x where f x is


undefined.
i.e., to be sure that x = c is a point of inflection, one
must see whether the second derivative f x
changes sign at x = c or the third derivative f x
exists and is non zero at x = a (i.e. f a 0 )
i.e., f c + h is positive and f c h is
negative x = c is an inflection point
or, f c + h is negative and f c h is
positive x = c is an inflection point.

bg

b
b

bg
bg
g
g

Examples worked out:


1. Find the point of inflection of the curve, if any,
y = x3 7x 6.
Solution: y = x3 7x 6

dy
= 3x 2 7
dx

938

How to Learn Calculus of One Variable

d y
=0
dx 2
6x = 0
x=0
Now, on putting x = 0 h and x = 0 + h, (h > 0), in
d2y
, it is observed that
dx 2

LM d y OP
MN dx PQ
LM d y OP
and M dx P
N Q
2

b g

x =0h

b g

= 6 0 + h = 6h = + ve

x =0+ h

d y
changes sign at x = 0
dx 2
the point (0, 6) is point of inflection of the
curve y = x3 7x 6 (since, x = 0 y = 6)

2. Find the point of inflection of the curve, if any, y =


2x3 6x2 18x + 19.
Solution: y = 2x3 6x2 18x + 19

dy
= 6x 2 12 x 18 = 6 x 2 2 x 3
dx

d2y
2 = 6 2x 2
dx

d2y
, it is seen that
dx 2
2

= 12 x 1

x =1 h

LM d y OP
MN dx PQ

x =1 h

= 12 x 1

x = 1+ h

x =1+ h

bg

= 12 1 + h 1 = 12 h = + ve
d2y
changes sign at x = 1.
dx 2
the point (1, 3) is a point of inflection of the
curve y = 2x3 6x2 18x + 19 (since x = 1 y = 3)

dy
= 5x 4
dx

d2y
= 20 x 3
dx 2

d2y
=0
dx 2

20x 3 = 0
x3 = 0 x = 0
Now, on putting x = 0 h and x = 0 h, (h > 0), it is
seen that

LM d y OP
MN dx PQ

b g

= 20 0 h

d2y
=0
dx 2
6 2x 2 = 0
x =1
Now, on putting x = 1 h and x = 1 + h (h > 0) in

LM d y OP
MN dx PQ

and

b g

3. Find the point of inflection of the curve, if any,


y = x5.
Solution: y = x5

= 6 0 h = 6h = ve

= 12 1 h 1 = 12 h = ve

d2y
= 6x
dx 2

x = 0 h

LM d y OP
MN dx PQ
2

b g

= 20 0 + h

x =0+ h

b g

= 20 h = 20h 3 = ve

bg

= 20 h = 20h 3 = + ve

d2y
changes sign at x = 0
dx 2
(0, 0) is the point of inflection of the curve y = x5
(since x = 0 y = 0)

4. Find the point of inflection of the curve, if any, y =


x sin x.
Solution: y = x sin x

939

Maxima and Minima

LMe
N
f b xg =

dy
= 1 cos x
dx

d2y
2 = sin x
dx

Now on putting x = n h and x = n + h


(h > 0), it is seen that

= sin n h

x =n h

b g

= 1

LM d y OP
MN dx PQ
2

n 1

x = n +h

b g

bg
2 b0 hg b0 h g + 12
f b0 hg =
b0 hg 4

small and > 0) in f x , it is seen that:


2

x
x 4

1 x2 4 x 2x

ex

ve

bg

3 f (x) is undefined at x = 2
domain of f (x) is the set of all real numbers
excepting x = 2 .

bg

Solution: f x =

f x =

On putting x = 0 h and x = 0 + h (h sufficiently

x
.
x 4

f x = 0

bg

x = 0 is the only real value which makes

5. Find the points of inflection of the curve, if any,

bg

x=0

d2y
2 changes sign at x = n for each n
dx
for each n, n , n is a point of inflection of
the curve y = x sin x 3 x = n y = n

f x =

= 1 sin h

sin h

= sin n + h

ex

x = n, n I

j OQP

j e

LM2 x e x 4j 2 e x + 4jOP 2 x x + 12
Q= e j
=N
e x 4j
e x 4j
f b x g is undefined at x = 2 . But x = 2
does not belong to the domain of the given function
f which means f b x g exists for each value of x in the
domain of the function f.
Now, f b x g = 0
2 x e x + 12j

=0

4
x
e j
2 x e x + 12 j = 0
2

d2y
2 = 0 sin x = 0
dx

LM d y OP
MN dx PQ

x 2 4 2 x x 2 + 4 2 x2 4 2 x

x2 4

ex

= + ve

bveg
2 b0 + h g b0 + h g + 12
f b0 + h g =
b0 + hg 4
3

and

bg

+ ve

bveg

= ve

f x changes sign at x = 0

940

How to Learn Calculus of One Variable

(0, 0) is the required inflection point of the

bg

bg h

x
3 x=0 f 0 =0
x 4
6. Find the points of inflection of the curve, if any,
given curve f x =

bg b

g
f b x g = b x 3g
1

f x = x 3 7.
Solution:

1
7

b g 71 bx 3g and f b xg = 496 bx 3g
Again f b x g 0 for any finite real value of x but
f b x g is undefined at x = 3 which means x = 3 is the
76

13
7

probable point of inflection.


Now, on putting x = 3 h and x = 3 + h (h > 0) in

bg
6

f b3 hg =
49

f x , it is seen that:

6
1

49 h

b g

13
7

b3 h g 3

6
1

49 ve

6
1

49
3+h 3

13
7

bg

f x changes sign at x = 3.
f (x) has an inflection point at x = 3
(3, 0) is the required point of inflection of the

b g b g c3 x = 3 f b3g = 0h

given curve f x = x 3

1
7

7. Find the points of inflection of the graph of the

bg

function f x = x +

4
.
x

is undefined at x = 0 which is not in the domain of the

bg

given function f. This why, f x is defined at each


point of the domain of f.

bg

Also, f x 0 , for any value of x

bg

function f x =

bg

x 1
.
x2
3

g b x 2g

x 1
= x 1
x2

1
3

13

the domain of the given function f is the set of


all real numbers excepting x = 2

b g 13 bx 1g bx 2g + FGH 13IJK bx 2g bx 1g
23

f x =

6
1
=

13 = ve
49 h 7

bg

bg
b g x8
f b x g 0 for any finite value of x and f b x g

and f x = 0 4 2 x 3 =

Solution: f x =

4
x2

8. Find the points of inflection of the graph of the

13
7

= +ve
and f 3 + h =

bg

Hence, the graph of the given function f has no


inflection point.

4 x2 + 4
=
x
x
domain of the given function f is the set of all
real numbers excepting x = 0
domain of f is the set of all non-zero real
numbers.
Now, f x = 1

The domain of the given function f is the set of all


real numbers.

f x =

bg

Solution: f x = x +

13

g b g bx 2g bx 1g
b g b g

1
2
x1 3 x2
3
1
2
= x 1 3 x 2
3
=

43

bg
1L 2
M b x 1g b x 2g
3N 3
2
= Lb x 1g b x 2g
9 MN
f x =

53

53

43

43

43

43

b g bx 1g OPQ
2 b x 2g b x 1g O
PQ

4
x2
3

73

73

23

23

1
3

941

Maxima and Minima

bg

b g b x 2g b3x 4g

2
x 1
9

3x 4
2

9 x 1 53 x 2

53

f x changes sign at x = 1

73

g b

bg

Now f x = 0

f (x) has an inflection point at x = 1


Also f (1) = 0
the inflection point of the given curve f at x = 1
is (1, 0)

7
3

3x 4

b x 1g b x 2g
5
3

7
3

=0

3x 4 = 0
4
3
Again, f x is undefined at x = 1 and x = 2 but
x = 2 does not belong to the domain of the given
function f.
This is why x = 2 is rejected while considering the
possible inflection points.
Hence, the possible points of inflection are at x = 1
x=

bg

4
3
At x = 1: for h > 0 (small)

and x =

b g
b g b
g
F 2 I L b1 + 3hg OP
=G J M
H 9 K MN bhg b1 + hg PQ
F 2 I L b veg OP = ve
=G J M
H 9 K MN bveg b veg PQ
OP
3b1 + hg 4
2 L
f b1 + hg = M
9 M b1 + h 1g b1 + h 2g P
N
Q
F 2 I L b3h 1g OP
=G J M
H 9 K MN bhg bh 1g PQ
F 2 I L b veg OP = + ve
=G J M
H 9 K MN b+veg b veg PQ
b g

f 1 h =

3 1 h 4
2

5
9 1 h 1 3 1 h 2

5
3

7
3

5
3

5
3

7
3

7
3

7
3

At x =

4
, for small h > 0,
3

LM 3F 4 hI 4
GH 3 JK
4 I 2 M
F
f G hJ =
H 3 K 9 MM F 4 I F 4 I
MN GH 3 h 1JK GH 3 h 2JK
5
3

OP
LM
b4 3h 4g P
2
= M
9 MF
MM GH h 13IJK FGH h 23 IJK PPP
Q
N
OP
LM
3h
2
PP
= M
9 MF
MM GH h + 13IJK FGH h 23IJK PP
Q
N
bveg = + ve
=
b+veg b veg
LM 3 F 4 + hI 4
GH 3 JK
F4 I 2
f G + hJ = M
H 3 K 9 MM F 4 I F 4 I
MN GH 3 + h 1JK GH 3 + h 2JK
OP
LM
b4 + 3h 4g P
2
= M
9 MF
MM GH h + 13IJK FGH h 23 IJK PPP
Q
N
b+veg
=
b+veg b veg = ve
5
3

7
3

5
3

7
3

5
3

5
3

7
3

7
3

7
3

OP
PP
PP
Q

OP
PP
PP
Q

942

How to Learn Calculus of One Variable

bg

f x changes sign at x =

f (x) has an inflection point at x =

L FG 4 1IJ OP
K
4I M H 3
F
Also f G J = M
H 3 K M FG 4 2IJ PP
MN H 3 K PQ
F 1 I 1
= G J =
H 2K 2

(i) x3 2x2 + x + 6
(ii) (x 1) ( x 2)2
(iii) 2x3 15x2 + 36x + 10

4
3
4
3

x3
x2

6x + 8
3
2
(v) 2x3 15x2 + 36x + 10
(iv)

1
3

(vi)

1
3

is

FG 4 , 1 IJ
H3 2K

4
3

Type 1: Problems based on finding the maximum and


/ minimum point as well as maximum and / minimum
values of a function f (x).
(A) Problems based on algebraic functions
Exercise 22.1
1. Investigate the value of x for which the functions
have the maximum and / minimum values.
(i) 2x x2 + 10
(ii) x2 + 2x + 11
(iii)

1 3 1 2
x + x + 17
3
2

(iv) 9 x 3 1 3x 2

(v) 2 x 3 12 x 2 + 18 x + 3
(vi) x3 3x + 10
(vii) x5 5x4 + 5x3 10
(viii) 3x4 + 16x3 + 16x2 72x + 13
1

1
x
(viii) xy where x + y = a
3. What, if any, is the maximum and / minimum values
(vii) x +

1
3

the inflection point of the given curve f at x =

x2 7x + 6
x 10

(ix) 3x 3 x 3 1

(x) 12 (x + 2) (x2 4)2


2. Find the maximum and / minimum values (value) of
the following functions.

250
.
x
4. What is the maximum slope of the curve y = x3 +
3x2 9x 27 and what point is it.
2
(value) of y, where y = x +

[Hint: Slope of the curve =

dy
= 3x 2 + 6 x + 9 .
dx

Find the max. value of y]


Answers:
1. (i) Max at x = 1
(ii) Min at x = 1
(iii) Max at x = 1, Min at x = 0

5
5
, Min at x =
5
5
(v) Max at x = 1, Min at x = 3
(vi) max at x = 1, Min at x = 1
(vii) Max at x = 1, Min at x = 3, Max or Min at x = 0
(viii) Min at x = 3 and at x = 3, Max at x = 2
(iv) Max at x =

(ix) Min at x =

1
8

2
, Min at x = 2
5
2. (v) Max at x = 2, Min at x = 3
(vi) Max at x = 4, Min at x = 16
(vii) Max at x = 1, Min at x = 1
(x) Max at x =

(viii)

a2
4

943

Maxima and Minima

Type 1 continued
(B) Problems based on mod. of a function, i.e. | f (x) |.
Exercise 22.2

(xii) y = a sec x + b cosec x (0 < a < b)


(xiii) y = a2 cosec2 x + b2 sec2 x
(xiv) y = sin nx sinn x
(xv) y = sin 2 + sin 2 , where + =

1. Find the maximum and / minimum values (values)


of the following functions f (x) given below.
(i) | x + 2 |
(ii) | x + 1 | + 3
(iii) | sin 4x + 3 |
(iv) |x3 | + 1
Answers:
1. (i) The minimum value of f (x) = 0 and is obtained
when x + 2 = 0 and there is no maximum value of f (x)
when x = 2.
(ii) The maximum value of f (x) = 3 and is obtained
when x + 1 = 0 and there is no minimum value of f (x).
(iii) The minimum value of f (x) is 2 and is obtained

b g

, n I .
4
And the maximum value of f (x) is 4and is obtained
n

when sin 4x = 1; i.e. when x = n 1

b g

n
when sin 4x = 1; i.e. when x = n 1 ,n I .
8
(iv) The minimum value of f (x) = 1 and is obtained
when x3 = 0, i.e. when x = 0. There is no maximum
value of f (x).
Type 1 continued
(C) Problems based on trigonometric functions

Exercise 22.3
1. Investigate the value of x for which the following
functions have maximum or minimum value of y.
(i) y = sin x
(ii) y = cos x
(iii) y = sin x + cos x
(iv) y = sin x (1 + cos x)
(v) y = a sin x + b cos x
(vi) y = cos2 x
(vii) y = a sin2 x + b cos2 x
(viii) y = a tan x + b cot x
(ix) y = sin 2x x
(x) y = 3 sin x + 4 cos x
(xi) y = sin x x cos x

b g

(xvi) y = sin x

sin x

x
x
(xvii) y = e + 2 cos x + e
(xviii) y = sin x + cos 2x
(xix) y = x + 2 sin x , 0 x 2
(xx) y = 3 cos x + 4 sin x
(xxi) y = cos 2x sin 2x
(xxii) y = 5 cos x + 12 sin x + 3

2. Prove that y = sin x +

3 cos x has maximum

.
6
3. Find the maximum or minimum value of the function
value at x =

y = x + sin 2x 0 < x < 2 .


4. Does the function y = sin x (1 + cos x) has maximum
value at x = 0?
5. Does the function y = x sin x have a maximum or
minimum?
Answers:

3
, Min at x =
2
2
(ii) Max at x = 0, Min at x =

1. (i) Max at x =

FG
H

IJ
K

(iv) Max at x =

FG
H

IJ
K

5
1

, Min at x = 2n +
4
4

(iii) Max at x = 2n +

5
, Min at x =
3
3

(v) Max at sin x =

a
a +b

well as Min at sin x =


(vi) Max at x = n

and cos x =

a
a +b
2

and cos x =

b
a +b
2

as

b
a + b2
2

944

How to Learn Calculus of One Variable

FG 1 IJ and Min at
H 2K
F 1I
x = n if a < b, Min at x = G n + J and Max at
H 2K

(vii) If a > b, Max at x = n +

5. Neither the max or min.


Type 3: Problems based on finding the absolute
maximum or minimum values of a function in a given
closed interval.
Exercise 22.4

x = n .

(viii) If a and b are both positive: Min at tan x =

a
b

Find the points of maximum of minimum of each of the


following functions.

b g b g
f b x g = x in 2 , 2
f b x g = b x 1g + 3 in 3 , 1
F1 I
f b x g = G xJ + x on 2 , 2.5
H2 K
1
f b x g = 4 x x in 2 , 4.5
2
f b x g = x + 2 sin x in 0 x 2
f b x g = x 6 x + 9 x + 15 in 0 x 6
1

1. y = x 1 3 x 2 , 1 x 9

b
a

Max at tan x =

2.

If a and b are both negative: Max at tan x =

b
a

3.

b
Min at tan x =
a

4.

3
3 5
; Min =

2
6
2
6

(ix) Max =

(x) Max at x = tan

FG 3IJ
H 4K

6.
7.

(xi) Max at x = ; Min at x = 2

F bI
(xii) Min at tan x = G J
H aK
F bI
(xiii) Min at tan x = G J
H aK
(xiv) x =

1
3

(xv) Max and min values respectively = 1 cos

1
; Min at x =
2
e
(xvii) Min for x = 0
(xvi) Max at x =

(xx) y max = 5 , y min = 5


(xxi) y max =

Answer:
1. y max at x = 9 , y min at x =

5
4

2. y max at x = 2 is 8 , y min at x = 2 is 8
1
2

r
gives max or min
n+1

5.

2 , y min = 2

(xxii) y max = 16 , y min = 10

3. y max at x = 3 is 19 , y min at x = 1 is 3
4. y max at x = 6.29 is 19.625
5. y max at x = 4 is 8 , y min at x = 2 is 10
Type 4: Problems based on finding the greatest and
least value of the function in a given closed interval;.
Exercise 22.5
1. Find the largest and smallest values of the function
f (x) = 3x4 2x3 6x2 + 6x + 1 in the interval [0, 2].
[Hint: Reject the value x = 1 since it does not belong
to the interval [0, 2]]
2. Find the greatest and the least values of the
following functions on the given intervals.

Maxima and Minima

bg
(ii) f b x g = x 5x + 5x + 1 in 1 , 2
(iii) f b x g = 100 x in 6 , 8

(v) 6, 0

(i) f x = x + 2 x in 0 , 4
5

(vi)

bg

1 x + x2

in 0 , 1

1 + x x2

x in 0 , 4
bg
(vi) f b x g = b1 x g e1 + 2 x j in 1 , 1
F 1 x IJ in 0 , 1
(vii) f b x g = tan G
H1 + xK
L 1 , 1 OP
(viii) f b x g = cos e x j in M
N 2 2Q
(ix) f b x g = sin x sin 2 x in ,
(x) f b x g = x e in 0 ,
(xi) f b x g = x log x in 1 , e
(xii) f b x g = x in 01
. ,
(xiii) f b x g = x 2 log x in 1 , e
L 1 , 3OP
1
(xiv) f b x g = tan x log x in M
2
N3 Q
L 3 O
(xv) f b x g = 2 sin x + sin 2 x in M0 , P
N 2Q
1

3 3
,2
2

(ix)

4
3 3

4
3 3

1
,0
e
(xi) e2, 0
(x)

(xii) Least value =

(xiii) 1 , 2 1 log e
(xiv)

,0

(viii)

,0
4

(v) f x = x +

(vii)

(iv) f x =

945

(xv)

FG 1IJ
H eK
2g

1
e

; no greatest value

+ 0.25 log 3e , 0.25 log 3e


6
3
3 3
,2
2

3. 0 , 3

3. Find the rang of the function: y = tan


Answers:
1. Greatest value = 2, smallest value = 1
(i) 8, 0
(ii) 2, 10
(iii) 10, 6
(iv) 1 ,

3
5

2
x2
9

Type 5: Verbal problems on maxima and / minima.


(A) Problems on numbers
Exercise 22.6
1. Find the two numbers whose
(i) Sum is 12 and whose product is a maximum.
(ii) Product is 16 and whose sum is a minimum.
(iii) Sum is k and the product of one by the cube of
the other is a maximum.
(iv) Sum is k and the product of one raised to the
power m by the other raised to the power n is a
maximum.
2. Find the number whose sum with its reciprocal is a
minimum.
3. The sum of two number is 10. Find the numbers so
that the sum of their squares is minimum.

946

How to Learn Calculus of One Variable

4. Divide 64 into two parts such that the sum of the


cubes of two parts is minimum.
5. Split 15 into two numbers so that the product of
the square of the first and the second is minimum.
6. Divided 80 into two parts such that the product of
the cube of one and the fifth power of the other shall
be as great as possible.
7. Divide a number into two parts such that the square
of one part multiplied by the cube of the other shall
given the greatest possible product.
8. Find two positive numbers whose product is 64,
having minimum sum.
9. Determine two positive numbers whose sum is 15
and the sum of those squares is minimum.
10. Divided the number 4 into two positive numbers
such that the sum of the square of one and the cube
of the other is minimum.
11. Divide 50 into two parts such that their product
is maximum.
12. Divided 50 into two parts such that the product
of the square of one part and the cube of the other is
maximum.
13. Divide 20 into two parts such that the product of
the cube of one and the square of the other will be
maximum. Also find the greatest product.
14. Divide 20 into two parts so that the sum of the
squares of two parts may be maximum.
15. Divide 40 into two parts so that the product of
cube of one part and fifth power of the other may be
maximum.
16. Find the maximum value of the product of two
numbers if there is 12.
17. Prove that if the product of two numbers is
constant, their sum will be minimum when the two
numbers are equal.
18. Find the two positive numbers x and y such that
x + y = 60 and xy3 is maximum.
19. Find the two positive numbers x and y such that
their sum is 35 and the product x2 y5 is a maximum.
20. Find two positive numbers whose sum is 16 and
the sum of whose cubes is maximum.
21. Determine two positive numbers whose sum is
15 and the sum of squares is minimum.
22. Divide a number 15 into two parts such that the
square of one multiplied with cube of the other is
maximum.

23. Divide the number 4 into two positive numbers


such that the sum of the square of one and the cube
of the other is a maximum.
Answers:
1. (i) 6, 6 (ii) 4, 4
(iii)
2.
3.
4.
5.
6.

k 3k
km
kn
,
,
(iv)
4 4
m+n m+n

1
5, 5
32, 32
10, 6
30, 50

3a 2a
,
5 5
8. 8, 8
9. Find
10. Find
11. Find
12. 20, 30
13. 12, 8
14. 10, 10
15. 25, 15
16. Find
17. Find
18. xy2 is maximum when x = 15 and y = 45.
19. x2 y5 is maximum when x = 10 and y = 25.
20. The required numbers are 8 and 8.
7.

15
15
and the other part =
.
2
2
22. One part is x = 6 and the other part y = 9.
21. One part is

23. One part is x =

y=4

8
and the other part
3

8 4
= .
3 3

Type 5: continued
(B) Problems based on perimeter and area.
Exercise 22.7
1. The perimeter of a rectangle is given
(i) What shape makes the area a minimum?

Maxima and Minima

(ii) What shape makes the diagonal a minimum?


2. The area of a rectangle is given. what shape makes
the perimeter a minimum?
3. Prove that the rectangle with maximum perimeter
inscribed in a circle is a square.
4. A circle with radius r is to be closed in a rhombus.
What shape of the rhombus makes (i) the perimeter a
minimum (ii) the area a minimum.
5. A sheet of paper is to contain 18 square inches of
printed matter. The margins at top and bottom are 2
inch each and the sides 1 inch each. Find the
dimensions of the sheet of smallest area.
6. The three sides of a trapezium are equal, each being
6 long. Find the area of the trapezium when it is
maximum.
7. Show that among all rectangles with given area,
square has least perimeter.
8. Prove that the area of the right angled triangle
drawn on a given side as hypotenuse is maximum
when the triangle is isosceles.
9. The sum of the perimeters of a square and a circle
is constant. If the sum of their areas is minimum, find
the ratio of the side of the square and the radius of
the circle.
10. Find the greatest area of a rectangle whose
perimeter is given (= 2p).
11. Show that the rectangle of least perimeter for a
given area is a square.
[Hint: Let the sides of the rectangle be x and y, p
its perimeter and A is its area, then
A
A
dp
A
A = xy y = p = x +
= 1 2
x
x
dx
x
dp x 2 A
=
= 0 x = A . Then and the
dx
x2
rectangle is a square.]
12. Find the length of the diagonal of rectangle with
minimum perimeter having area 100 square meter s.
13. Prove that the triangle of greatest area inscribed
in a circle is equilateral.
14. If in a right angled triangle, the sum of its
hypotenuse and one of the other sides is given, prove
that the area of the triangle will be maximum if the
angle between the two sides is

.
3

947

15. Find the area of the greatest rectangle that can

x2
y2
+
= 1.
a2
b2
16. Prove that among all rectangles with given
perimeter k, the square is the one with maximum area.
Find the maximum area when k = 16.
17. The lengths of three sides of a trapezium are
equal, each being 6 cm. Find the maximum area of
such a trapezium.
18. An isosceles triangle is drawn with its vertex at
the origin, its base parallel to and above the x-axis
and the vertices of its base on the curve 12y = 36 x2.
19. Prove that the least perimeter of an isosceles
triangle in which a circle of radius r can be inscribed
be inscribed in the ellipse

is 6 3r .
20. Show that for a given perimeter, the area of a
triangle is maximum when it is equilateral.
21. A sheet of poster has its area 18 square meter.
The margin at the top and bottom are 75 cm and at
sides 50 cm. what are the dimensions of the area of
the printed space is maximum.
22. A wire of length 20 cm is cut into two parts. One
part is bent in to a circle and the other into a square.
Prove that the sum of the area of the circle and the
square is least if the radius of the circle is half the side
of the square.
23. A flower bed is to be in the shape of a circular
sector of radius r and central angle . Find r and
if the area is fixed and the perimeter is minimum.
24. A window is of the shape of a rectangle
surmounted by a semicircle. What should the
proportions be for a given area and minimum perimeter.

25. A picture is in the shape of a rectangle


surmounted by a semicircle. If the perimeter is 20 cm,
then find the dimensions of the picture for the greatest
area.

948

How to Learn Calculus of One Variable

26. A window is in the form of a rectangle


surmounted by a semicircle. If the perimeter in 30
meters, find the dimensions so that the greatest
possible amount of light may be admitted.
27. A rectangle is inscribed in a semicircle so that a
side lies along the bounding diameter. If the area of
the rectangle is maximum, show that this area is to the
area of the semicircle as 1 : .

|UV
|W

R|
S|
T

6. Show that the curve surface of a right cone of a


given volume is least when its semi-vertical angle is

FG 1 IJ .
H 2K

e j

b g

A +4
2

U|
b g V|W ,

A
2 +4

where A = area
25. x =

FG 1IJ .
H 3K

7. Prove that semi vertical angle of a cone with given

6. 27 3
15. 2 ab
16. 16
24. The length and the breadth are in the ratio:

2A
1
:
2
+4

sin 1

tan 1

Answers:
1. (i) Square (ii) Square
2. Square
4. Square
5. 10 inch by 5 inch

|RS2
|T

2. Find the altitude of the cylinder of maximum volume


that can be inscribed in a given sphere.
3. Show that the height and the radius of the base of
an open cylinder of given surface area and maximum
volume are equal.
4. A box of maximum volume with top open is to be
made out of square tin sheet of sides 6 ft length by
cutting out small equal squares from four corners of
the sheet. Find the height of the box.
5. Show that semi vertical angle of a right circular
cone of given surface and maximum volume is

20
4+

30
m
+4
Type 5: continued
26. x = y =

(C) Problems based on volume


Exercise 22.8
1. Show that the height of a closed cylinder of a given
volume and least surface is equal to its diameter.

slant height whose volume is maximum is tan 1 2 .


8. The sum of the surfaces of a cube and a sphere is
given. Show that when the sum of their volumes is
least, the diameter of the sphere is equal to the edge
of the cube.
9. The sum of the volumes of sphere and a cube is
given. show that when the sum of the surfaces is
greatest, the diameter of the sphere is equal to the
side of the cube.
10. Show that the height of an open cylinder of a
given surface and greatest volume is equal to the
radius of its base.
11. Show that the right circular cylinder of a given
surface and the maximum volume is such that its
height is equal to the diameter of the base.
12. Show that the height of the closed cylinder of
given volume and least surface is equal to its diameter.
13. Prove that the height and diameter of the base of
a right circular cylinder of maximum volume are equal
when the total surface area is given.
14. Show that a right circular conical tent of given
volume will require the least amount of convas of its
height is 2 times the radius of its base.

Bibliography

949

Bibliography

1.
2.
3.
4.
5.
6.
7.
8.
9.
10.
11.
12.
13.
14.
15.
16.
17.
18.
19.
20.

Bers, Lipman. Calculus. Publishing Company, Bombay, 400005.


Bugrov, Ya. S. and Nikolsky, S.M. Differential and Integral Calculus. Mir Publishers, Moscow.
Bugomolov, N.V. Mathematics for Technical Schools. Mir Publishers, Moscow.
Butzov, B.F. Mathematical Analysis in Questions and Problems. Mir Publishers, Moscow.
Bacon, M. Harold. Differential and Integral Calculus. McGraw-Hill Book Company, Inc, 1955.
Fanasyeva, O.N.A., Brodsky, Ya. S., Gukkin, I.I and Pavlov, A.L. Problem Book in Mathematics for
Technical Colleges. Mir Publishers, Moscow.
Graves, M. The Theory of Functions of Real Variable. McGraw-Hill Book Company, Inc, 1946.
Ilyin, V.A. and Poznyak, E.G. Fundamentals of Mathematical Analysis. Mir Publishers, Moscow.
Johnson, R.E. and Kioke, Meister, F.L. Calculus with Analytic Geometry. Prentice-Hall of India, Pvt,
New Delhi, 1975.
Klaf, A.A. Calculus Refresher for Technical Men. Dover Publications, New York, Inc, 1944.
Lamb, Horace. An Elementary Course of Infinitesimal Calculus. The English Language Book Society
and Cambridge University Press-1962.
Leithold, Louise. The Calculus with Analytic Geometry. Harper and Row Publishers, New York, Inc.
Maron, I.A. Problems in Calculus of One Variable. Mir Publishers, Moscow.
Natason, I.P. Theory of Functions of a Real Variable. Fredrick, Unogar Publishing Company, New
York.
Piskonov, N. Differential and Integral Calculus. Mir Publishers, Moscow.
Panchishkin, A. and Shavgulidge, E. Trigonometric Functions. (Problem solving approach) Mir
Publishers, Moscow.
Shipachev, V.S. Higher Mathematics. Mir Publishers, Moscow.
Smith, F. Percy and Longley, R. William. Elements of the Differential and Integral Calculus. Ginn and
Company, New York, 1941.
Thomas, G.B. Calculus. AddisonWesley Publishing Company, Inc, 1964.
Taylor, E. Angus. Advanced Calculus. Blaisdell Publishing Company, New York.

950

Index

Index

Absolute value 12
Algebraic function 9, 23, 161
Approximation 667
Binomial coefficients 565
Boundary point 124
Bounded set 128
Chain rule for the derivative 383
Codomain 6
Composite function 58, 68
Constant 2
Constant function 11
Continuity 271, 351
Continuous function 151, 158
Dense set 126
Dependent variable 7
Differentiability 351
Domain of a function 8, 21
Evaluation 331
Even function 69
Explicit function 10
Exponential function 227
Exponential functions 402
Function

1, 9

Graph of a function 99
Greatest integer function 14, 45, 256
Identity function 11
Implicit differentiation 499
Implicit differentiation 499

Implicit function 10
Increament 2, 3
Independent variable 7
Intervals 17, 18
Closed interval 17
Open interval 17
Inverse circular function 318, 424
Inverse function 111
Inverse trigonometric functions 424
Lagranges mean value theorem 781
Left hand limit 149
L-Hospital rule 597
Limit of a function 131, 136, 148
Limit point of a set 124
Limits of a continuous function 154
Limits of a sequence 134
Linear function 11
Logarithmic differentiation 543
Logarithmic functions 30, 403
Maxima and minima 870
Method of rationalization 302
Mod functions 478
Odd function 69
One sided derivative 322
One sided limits 143
One-one and onto function 76, 78
Parametric differentiation 519
Periodic function
Periodic function 71
Piecewise function 15, 254
Point by point method 99

Index
Real variables 15
Right hand limit 149
Rolles theorem 781

Tangent to a curve 692


Transcendental function 10
Trigonometric functions 35

Sequence 127

Unbounded function 130

951

You might also like